You are on page 1of 756

t.

me/desidentist
Quick Review Series for
BDS 3rd Year

t.me/desidentist
This page intentionally left blank
Quick Review Series for
BDS 3rd Year
Second Edition

J Jyotsna Rao
bds, mds (Osm), pgcoi (mahe), fisoi
Director, SRS Dental Exams Academy, Bengaluru
Ex-Professor, Department of Oral and Maxillofacial Surgery
The Oxford Dental College, Hospital and Research Centre
Bengaluru, INDIA
RELX India Pvt. Ltd.
Registered Office: 818, 8th Floor, Indraprakash Building, 21, Barakhamba Road, New Delhi 110001
Corporate Office: 14th Floor, Building No. 10B, DLF Cyber City, Phase II, Gurgaon-122 002, Haryana, India

Quick Review Series for BDS 3rd Year, 2e, Rao J Jyotsna
Copyright © 2016, 2012 by RELX India. Pvt. Ltd.
All rights reserved.
ISBN: 978-81-312-4441-8
e-Book ISBN: 978-81-312-4644-3

No part of this publication may be reproduced or transmitted in any form or by any means, electronic or
mechanical, including photocopying, recording, or any information storage and retrieval system, without
permission in writing from the publisher. Details on how to seek permission, further information about the
Publisher’s permissions policies and our arrangements with organizations such as the Copyright Clearance
Center and the Copyright Licensing Agency, can be found at our website: www.elsevier.com/permissions.

This book and the individual contributions contained in it are protected under copyright by the Publisher (other
than as may be noted herein).

Notice
Knowledge and best practice in this fi eld are constantly changing. As new research and experience broaden
our understanding, changes in research methods, professional practices, or medical treatment may become
necessary.
Practitioners and researchers must always rely on their own experience and knowledge in evaluating and
using any information, methods, compounds, or experiments described herein. In using such information or
methods they should be mindful of their own safety and the safety of others, including parties for whom they
have a professional responsibility.
With respect to any drug or pharmaceutical products identifi ed, readers are advised to check the most
current information provided (i) on procedures featured or (ii) by the manufacturer of each product to be
administered, to verify the recommended dose or formula, the method and duration of administration, and
contraindications. It is the responsibility of practitioners, relying on their own experience and knowledge of
their patients, to make diagnoses, to determine dosages and the best treatment for each individual patient, and
to take all appropriate safety precautions.
To the fullest extent of the law, neither the Publisher nor the authors, contributors, or editors, assume any
liability for any injury and/or damage to persons or property as a matter of product liability, negligence
or otherwise, or from any use or operation of any methods, products, instructions, or ideas contained in the
material herein.
Although all advertising material is expected to conform to ethical (medical) standards, inclusion in this
publication does not constitute a guarantee or endorsement of the quality or value of such product or of the
claims made of it by its manufacturer.
Please consult full prescribing information before issuing prescription for any product mentioned in this
publication.

Content Strategist: Nimisha Goswami


Sr Project Manager—Education Solutions: Shabina Nasim
Managing Editor: Anand K Jha
Sr Project Manager: Nayagi Athmanathan
Sr Operations Manager: Sunil Kumar
Sr Production Executive: Ravinder Sharma
Sr Cover Designer: Milind Majgaonkar

Typeset by GW India
Printed in India by
Dedicated
to
My son Master K Ragha Sai
&
My husband Mr K Vinayak Rao
for their continuous support and encouragement
This page intentionally left blank
Govt. Dental College & Hospital
(Affiliated to KNR University of Health Sciences)
Afzalgunj, Hyerabad 500 012
Dr. P. Bal Reddy MDS
Principal, Professor & Head of the Department College. 246019360
Deparment of Oral & Maxillofacial Surgery Mobile: 9440058399
Govt. Dental College & Hospital
Afzalgunj, Hyderabad

FOREWORD
I am extremely happy to pen a few words about this conscientiously written book. It is common knowledge that books
play a major complementary and contributing role in any educational process, where they are envisioned to facilitate
self-learning beyond classroom exercises.
This book, Quick Review Series for BDS 3rd Year authored by Dr J Jyotsna Rao is presented with such a systematic
approach that it demonstrates her consummate skill in preparing students for examinations. It is good to see that she has
shared her vast experience in academics with the students through this book.
While going through the pages of this book, I found that the author has made a sincere attempt to present a succinct
panorama of various subjects taught in 3rd year BDS as per the new regulations of DCI to fulfill the long-term need of
students. A concise quick review book with best content, simple language and required depth of explanation of every sub-
ject, through questions and answers of various university examinations.
Designing such a book is a challenging task, especially if it is to be concise and comprehensive. Such a version
demands wise sifting, prudent pruning and meaningful condensing of the enormous and variegated knowledge base of
all 3rd year BDS subjects.
A well written concise book as this, serves as both preparatory tool at the start of study course providing road map of
the subject to be learnt and at the end of the course it helps in rapid review and recapitulation of what was learnt.
I am confident that this book is undeniably appropriate for 3rd year BDS exam going students who are craving for a
thorough review of entire subjects in a short period.

(DR. P. BAL REDDY)

vii
This page intentionally left blank
Preface

Encouraged by the overwhelming response from student community and remarkable success of the first edition of this
book, second edition is definitely a great improvement over the first edition.
The outstanding feature of this book is incorporation of Index for easy reference to the answers which is first of its kind
among question bank books. This edition also contains a unique addition of Key Points to Remember in each subject for
easy recollection.
This edition continues to retain the primary objective of the previous edition, being a concise, informative and reader-
friendly compilation of all subjects of third year BDS with updation of latest question papers. It bridges the gap between
what students know and how much they have to present while answering in theory exams.
Refer the cover of the book to explore the online MCQs and solved papers, which we have painstakingly developed,
especially for you. The symbols “MCQ symbol” and “Solved paper symbol” in the table of contents of this book will help
you identify these online resources. Besides these, you will get access to the complimentary e-book also.
I feel all the improved features of this book will definitely go long way in helping candidates gain necessary knowledge
and confidence to tackle the examinations successfully.
I wish my readers good luck and great fortune.
J Jyotsna Rao
drjjrao@gmail.com

  

ix
This page intentionally left blank
Acknowledgements

First of all I thank the almighty for his blessings without which this work would not have been possible.
I would like to thank at the outset, my father Mr J Sudharshan Rao who is the key person behind all my successful
endevours. I am thankful to my mother Mrs S Sujatha Laxmi for her unforgettable sacrifices and choicest blessings.
My warmest regards to my husband Mr K Vinayak Rao for his constant support to enhance my software skills in op-
erating computers. My thanks and love to my son Master Ragha Sai without whose cooperation this work would not have
been possible.
I am thankful to my brother Mr J Jayakrishna for his valuable constructive suggestions.
I wish to thank Dr BK Reddy, Ex-Principal, Government Dental College Hyderabad and Meghana Dental College,
Nizamabad and Dr P Bal Reddy, Professor and HOD of OMFS, Govt Dental College, Hyderabad, for their support, bless-
ings and advice.
My sincere thanks to Dr P Chidambar, Dr Omkar and Mr Kiran (Librarian, Oxford Dental College, Bengaluru) and
Narayan Swami for their invaluable support in collecting previous years’ question papers from various universities.
I sincerely thank Dr Anuradha Mahadevappa, Senior Lecturer, Department of Oral and Maxillofacial Surgery, CODS,
Davangere for her contribution of a topic in General Medicine section. I would like to specially thank Dr Abhijit Mahantha
for his valuable contribution in preparing General Surgery section.
I would like to extend my regards to Dr Keerthi Yamini, Sampath Kumar, my postgraduate students, Dr Gaurav and
Dr Naveen, for their help in correction of manuscripts.
I acknowledge Dr JP Shetty, Professor and HOD, Dept of Oral and Maxillofacial Surgery, Dr Sanjay Mohan Chandra,
Professor and HOD, Dept of Oral Implantology, The Oxford Dental College and Hospital for their support and appreciation
of the work.
Thanks to Elsevier India, especially Dr Lalit Singh, Ms Nimisha Goswami, Mr Anand K Jha along with other team
members, for their active contribution in publishing this book.
I would like to take this opportunity to thank all those people who, directly or indirectly were instrumental in success-
fully bringing out this book.
Last but not the least, I acknowledge all my friends and colleagues for their best wishes that boost my morale.

xi
This page intentionally left blank
t.me/desidentist

Contents

Foreword vii Topic 3 Burns, Skin Grafting and Flaps 245


Preface ix Topic 4 Haemorrhage and Shock 254
Acknowledgements xi
Topic 5 Bacterial Infections and Transmissible 
Viral Infections 269
Section I Topic 6 Tumours, Cysts and Neck Swellings 287
General Medicine Topic 7 Diseases of the Oral Cavity 
Topic 1 Aims of Medicine and Clinical Methods 3 (Mouth, Tongue and Lips) 302
Topic 2 Diseases of the Gastrointestinal System 4 Topic 8 Diseases of Salivary Glands 316
Topic 3 Diseases of Liver and Biliary System 22 Topic 9 Infections and Diseases of the 
Topic 4 Haematology 35 Larynx and Nasopharynx 327
Topic 5 Diseases of the Cardiovascular System51 Topic 10 Diseases of Arteries, Veins 
Topic 6 Diseases of the Respiratory System 83 and the Lymphatic System 335
Topic 7 Diseases of Nervous System 120 Topic 11 Nervous System 353
Topic 8 Diseases of the Kidneys  Topic 12 Fractures—General Principles 364
and Genitourinary System 138 Topic 13 Anomalies of Development of Face 
Topic 9 Disturbances in Water, Electrolyte  (Cleft Lip and Cleft Palate) 376
and Acid-Base Balance 145 Topic 14 Thyroid and Parathyroid Glands 382
Topic 10 Endocrine and Metabolic Diseases 149 Topic 15 Local Anaesthesia, Biopsy and 
Topic 11 Nutritional Factors in Diseases 169 Miscellaneous 405
Topic 12 Infectious Diseases  181 Key Points to Remember 411
Topic 13 Immunological Factors in Disease 
(Anaphylaxis and Drug Allergy) 195
Section III
Topic 14 Diseases of Connective Tissues, 
Oral Pathology
Bones and Joints 200
Topic 1 Developmental Disturbances of Oral 
Topic 15 Acute Poisoning and Environmental 
and Paraoral Structures 425
Emergencies and Miscellaneous 204
Topic 2 Benign and Malignant Tumours 
Key Points to Remember 213
of the Oral Cavity 439
Topic 3 Tumours of Salivary Glands 456
Section II Topic 4 Cysts and Tumours of Odotogenic 
General Surgery Origin 468
Topic 1 General Principles of Operative Surgery, Topic 5 Bacterial Infections of the Oral Cavity 484
Sterilization/Asepsis, Diathermy,  Topic 6 Viral Infections of the Oral Cavity 495
Cryosurgery and Lasers 225 Topic 7 Mycotic Infections of the Oral Cavity 503
Topic 2 Wounds, Sinus and Fistulae 232 Topic 8 Diseases of the Periodontium 508

xiii
xiv Contents

Topic 9 Dental Caries 516 Section IV


Topic 10 Diseases of the Pulp and  Multiple Choice Questions
Periapical Tissues 534 Section I General Medicine 629
Topic 11 Spread of Oral Infections 543 Section II General Surgery 644
Topic 12 Physical and Chemical Injuries of  Section III Oral Pathology 659
the Oral Cavity 545
Topic 13 Regressive Alterations of the Teeth 551
Topic 14 Healing of Oral Wounds 560
Section V
Topic 15 Oral Aspects of Metabolic Disease 567
Previous Years’ Question Bank
Section I General Medicine 677
Topic 16 Allergic and Immunologic Diseases 
Section II General Surgery  694
of the Oral Cavity 574
Section III Oral Pathology 706
Topic 17 Diseases of Bone and Joints 578
Index 723
Topic 18 Diseases of Blood and Blood 
Forming Organs 592
Topic 19 Diseases of the Skin 603 Online university exam
Topic 20 Diseases of the Nerves and Muscles 613 patterned MCQs and solved
Topic 21 Forensic Odontology 617 papers
Key Points to Remember 619
Colour Plates

PLATE 1 Verrucous carcinoma H/P PLATE 2 Leukoplakia H/P


(refer Topic 2, LEQ 1, page 439, for details) (refer Topic 2, LEQ 3, Page 440, for details)

PLATE 3 Oral submucous fibrosis PLATE 4 Central giant cell granuloma H/P
(refer Topic 2, LEQ 5, page 442 for details) (refer Topic 2, LEQ6, page 443 for details)
Colour Plates

PLATE 5 Capillary haemangioma


PLATE 6 Warthin’s tumour
(refer Topic 2, SEQ 8, Page 447 for details)
(refer Topic SEQ 9, Page 463 for details)

PLATE 7 Mucoepidermoid carcinoma H/P PLATE 8 Dentigerous cyst


(refer Topic 3, LEQ 1, Page 456 for details) (refer Topic 4, LEQ 1, Page 468 for details)
Colour Plates

PLATE 9 Odontogenic keratocyst PLATE 10 Pindborg’s tumour


(refer Topic 4, LEQ 3 Page 471 for details) (refer Topic 4, LEQ 6, Page 475 for details)

PLATE 11 Adenomatoid odontogenic tumour PLATE 12 Pyogenic granuloma


(refer Topic 4, LEQ 8, Page 476 for details) (refer Topic 5, SN 21, Page 495 for details)
Colour Plates

PLATE 13 Radicular cyst PLATE 14 Mucous extravasation cyst


(refer Topic 10, LEQ 5, Page 537 for details) (refer Topic 12, SNQ 1, Page 549 for details)

PLATE 15 Fibrous dysplasia PLATE 16 Pemphigus


(refer Topic 17, LEQ 2, Page 579 for details) (refer Topic 19, LEQ 1, Page 603 for details)

t.me/desidentist
Colour Plates

PLATE 17 Lichen planus


(refer Topic 19, LEQ 2, Page 605 for details)
Section I

General Medicine

Topic 1Aims of Medicine and Clinical Methods 3


Topic 2Diseases of the Gastrointestinal System 4
Topic 3Diseases of Liver and Biliary System 22
Topic 4Haematology 35
Topic 5Diseases of the Cardiovascular System 51
Topic 6Diseases of the Respiratory System 83
Topic 7Diseases of Nervous System 120
Topic 8Diseases of the Kidneys and Genitourinary System 138
Topic 9Disturbances in Water, Electrolyte and Acid-Base Balance 145
Topic 10
Endocrine and Metabolic Diseases 149
Topic 11
Nutritional Factors in Diseases 169
Topic 12
Infectious Diseases  181
Topic 13
Immunological Factors in Disease
(Anaphylaxis and Drug Allergy) 195
Topic 14 Diseases of Connective Tissues, Bones and Joints 200
Topic 15 Acute Poisoning and Environmental Emergencies and
Miscellaneous 204
This page intentionally left blank
Section I

General Medicine

Topic 1

Aims of Medicine and Clinical Methods

SHORT NOTES
Q. 1. Jugular venous pulse. v. The “a” wave is due to atrial contraction. This is fol-
lowed by “x” descent due to descent of tricuspid valve
Ans.
ring which is interrupted by a small “c” wave.
i. Jugular venous pulse may be examined from movement vi. The “v” wave is due to passive filling of blood from
of the skin overlying the internal jugular vein with veins into the right atrium during ventricular systole.
patient reclining against pillow at 45° and with neck This is followed by “y” descent due to rapid flow of
muscles relaxed. blood from the right atrium to the right ventricle when
ii. Venous pulsations are better seen while arterial pulsa- the tricuspid valve is open.
tions are better palpable. The upper level of venous
pulsation varies with the change in posture and phases The “a” wave is absent in atrial fibrillation while it
of respiration. Jugular veins are distended and pulsatile is prominent in tricuspid stenosis (TS). Prominent “y”
in congestive heart failure and pericardial effusion. descent is seen in tricuspid regurgitation.
iii. Normally, there is fall in the jugular venous pressure
(JVP) during inspiration. There may be a paradoxical Q. 2. Clubbing.
rise in the JVP during inspiration in constrictive peri- Or
carditis and cardiac tamponade (Kussmaul’s sign).
iv. The venous pulse has three positive waves, “a”, “c”, and Finger clubbing.
“v”, and two negative waves or descents, “x” and “y”. Ans.
i. The enlargement of the distal portion of the fingers
Ventricular and toes, due to proliferation of connective tissues, is
systole diastole known as clubbing.
a ii. The shape of the nail becomes convex in both horizon-
c
a tal and vertical directions. The angle between nail base
c
v
and the adjacent finger is obliterated. In severe cases,
v there is bulbous enlargement of the distal segment of
the fingers giving drumstick appearance.
y
iii. Schamroth’s window test is positive: There is reduction
x x y in the space between thumb nails when placed in
FIGURE 1.1 Jugular pulse apposition.

3
4 Quick Review Series for BDS 3rd Year

iv. The exact mechanism of clubbing is unknown. How- ii. Bronchogenic carcinoma
ever, it is thought to be due to some humoral substances iii. Pulmonary tuberculosis
leading to increased vascularity in the nailbed. iv. Fibrosing alveolitis
v. Clubbing may be present since birth (congenital), or B. Cardiac diseases:
acquired. i. Congenital cyanotic heart diseases, e.g. Fallot’s
tetralogy
Acquired causes of clubbing are as follows: ii. Subacute bacterial endocarditis
A. Respiratory diseases: C. Gastrointestinal diseases
i. Chronic suppurative lung diseases, e.g. i. Inflammatory bowel diseases, e.g. ulcerative colitis,
l Lung abscess Crohn’s disease
l Bronchiectasis ii. Hepatic cirrhosis
l Empyema D. Idiopathic

Topic 2

Diseases of the Gastrointestinal System

LONG ESSAYS
Q. 1. Describe pathophysiology, clinical picture, diagno- b. Loss of the reinforcing effect of intra-abdominal
sis and management of reflux oesophagitis. pressure on the lower oesophageal sphincter.
These two factors of hiatus hernia facilitate gastro-
Q. What is gastro-oesophageal reflux disease (GERD;
oesophageal reflux but do not directly cause it.
reflux oesophagitis? What are the normal mechanisms
ii. Cardiomyotomy and vagotomy: They reduce the
preventing reflux? Give the causes, clinical features,
efficiency of the lower oesophageal sphincter.
investigations, complications and treatment of reflux
iii. Increased intra-abdominal pressure: Pregnancy, obe-
oesophagitis.
sity, ascites, weight-lifting and straining act by increas-
Ans. ing the intra-abdominal pressure.
iv. Reduced tone of lower oesophageal sphincter: Ciga-
l Reflux of gastric contents into the lower oesophagus is
rette smoking, alcohol, fatty foods and caffeine act by
called gastro-oesophagial reflux disease ( GERD).
reducing the lower oesophageal sphincter tone.
l Several mechanisms operate to prevent the reflux of gas-
v. Impaired gastric emptying: Impaired gastric emptying
tric contents into the oesophagus. They are as follows:
due to obstruction of gastric outlet or use of anticholin-
a. Lower oesophageal sphincter (LES).
ergic drugs, fatty foods and large volume meals act by
b. Striated muscles of the diaphragmatic crura.
increasing the gastric content available for reflux.
c. Intra-abdominal pressure reinforces the LES tone.
vi. Systemic sclerosis.
d. Oblique entry of oesophagus to the stomach.
vii. Drugs which reduce the tone of lower oesophageal
e. Swallowed saliva neutralises the refluxed acid, etc.
sphincter, e.g. aminophylline, beta-agonists, nitrates,
When these mechanisms fail, reflux oesophagitis occurs. calcium channel blockers, etc.

Aetiology Clinical Features


i. Sliding hiatus hernia: Where the oesophagogastric i. Heart burn: A deeply placed burning pain behind the
junction slides up through the diaphragm resulting in: sternum radiating to the throat usually occurs after meals,
a. Loss of the obliquity of entry of oesophagus into brought on by bending, lifting weight and straining. It
stomach and occurs on lying down and is then relieved by sitting up.
Section | I General Medicine 5

ii. Regurgitation of gastric contents into the mouth (acid l In severe cases, proton pump inhibitors (PPIs) are
eructation). used: These include omeprazole (20-40 mg/day), lanso-
iii. Tracheal aspiration with coughing or laryngismus or prazole (15-30 mg/day), pantoprazole (40 mg/ day),
aspiration pneumonia. esomeprazole (40 mg/day) and rabeprazole (10-20 mg/
iv. Odynophagia (i.e. painful swallowing). day). These are useful in severe cases and are for
v. Transient dysphagia to solids due to oesophageal spasm. 6-8 weeks in higher doses. Maintenance doses may be
vii. Strictures leading to persistent dysphagia to solids. required, for 6-8 months.
viii. Iron deficiency anaemia due to blood loss. l Metoclopramide or domperidone 10 mg t.i.d increases
lower oesophageal sphincter tone and promotes gastric
emptying.
Complications of Gastro-oesophageal Reflux l Oesophageal stricture is treated by repeated dilatations.
l Oesophagitis l Oral iron or blood transfusion for anaemia.
l Oesophageal strictures
l Oesophageal ulcers iii. Surgical treatment:
l Aspiration pneumonia l Surgical resection of strictures.
l Iron deficiency anaemia l Surgical return of lower oesophageal sphincter to the
l Barrett’s oesophagus abdomen: In case of sliding hiatus hernia, construction
l Carcinoma of oesophagus of an additional valve mechanism (fundoplication) is
carried out.
Investigations Q. 2. Describe the aetiopathogenesis, clinical features
and treatment of peptic ulcers.
i. Endoscopy: Enables visualization of oesophagitis,
strictures and Barrett’s mucosa which all can be con- Or
firmed by biopsy.
ii. Barium meal can reveal a hiatus hernia. Describe clinical features, diagnosis and management
iii. Bernstein test is done in patients with high clinical sus- of peptic ulcer. Enumerate its complications.
picion but negative endoscopy. Or
iv. Resting ECG and stress ECG to rule out ischaemic
heart disease. Write the clinical features of gastric duodenal ulcers.
v. Oesophageal motility studies. What are the complications?
Or
Treatment Write in detail about acid peptic disease.
The treatment includes: Or
i. General measures,
ii. Medical treatment, and Describe the aetiology, clinical features, diagnosis and
iii. Surgical treatment. medical management of peptic ulcers.

i. General measures: Or
l Weight reduction. Q. Helicobacter pylori.
l Small volume, frequent feeds.
Ans.
l Avoid smoking, alcohol, fatty food, caffeine, mint,
orange juice, some medications. l Peptic ulcer is defined as the presence of ulcer in the
l Avoid late night meals. lower oesophagus, stomach or duodenum, in the jeju-
l Avoid weight-lifting, stooping and bending at waist. num after surgical anastomosis to the stomach or rarely,
l Elevation of head end of the bed should be maintained in the ileum adjacent to a Meckel’s diverticulum.
at 15°. l The peptic ulcer is a breach in the mucosa due to
an imbalance between mucosal defensive factors and
ii. Medical treatment: luminal factors such as acid and pepsin. A break in the
l In mild cases, use of liquid antacid 10–15 mL, one and mucosal surface of more than 5 mm and depth to the
three hours after meal relieves heart burn. submucosa are characteristics of a peptic ulcer.
l In moderate cases, H2 receptor antagonists like cimeti- l Duodenal ulcer is the more common than gastric ulcer
dine 400 mg or ranitidine 150 mg, b.d or q.i.d with and occurs at younger age group (30-55 years) as com-
meals and before bedtime, for at least 6 weeks. pared to gastric ulcer (55-70 years).
6 Quick Review Series for BDS 3rd Year

Aetiology l The most common presentation is that of recurrent


abdominal pain which has three notable characters:
a. Hereditary: Strong family history with gastric ulcers,
a. Localisation to the epigastrium.
but less strong family history with duodenal ulcers.
b. Relationship to food.
b. Helicobacter pylori infection:
c. Periodicity.
l The gram-negative bacteria are supposed to be main
l Epigastric pain: Pain is referred to epigastrium, and is
cause accounting for 70% of gastric ulcer.
so sharply localised that the patient will localise the site
l H. pylori is a gram-negative bacillus which produces
with one finger (pointing sign). It is usually burning in
mucosal damage.
character.
l Key factors secreted by the bacillus are urease, catalase,
l Hunger pain: Pain occurs on empty stomach (hunger
lipase, adhesions, platelet-activating factor and others.
pain) and is relieved by food or antacids.
l Urease converts urea into ammonia, thus alkalinising
l Night pain: Typically, the pain wakes the patient from
the surrounding acidic medium for its survival but
sleep around 3 AM and is relieved by food, milk or
simultaneously-producing ammonia-induced muco-
antacids.
sal damage.
l Pain relief: Pain is usually relieved by food, milk, antac-
l In a developing country, nearly 80% of persons are
ids, belching or vomiting.
colonised with it by the age of 20 years.
l Periodicity (episodic pain):
c. NSAIDs: NSAIDs are an important aetiological factors in
l Pain occurs in episodes, lasting 1-3 weeks every
30% of gastric ulcers. By depleting mucosal prostaglan-
time, 3-4 times a year. Between episodes, patient is
din levels, aspirin and NSAIDs impair the cytoprotection
asymptomatic.
resulting in mucosal injury, erosions and ulceration.
l In the initial stages, the episodes are short in duration
These drugs also reduce the integrity of duodenal mucosa.
and less frequent. As the natural history evolves,
d. Smoking: Smoking confers an increased risk of gastric
the episodes become longer in duration and more
ulcer and to a lesser extent duodenal ulcers. It does not
frequent.
cause ulcer but more likely to cause complication and is
l Patients are more symptomatic during winter and
responsible for non-healing or delayed healing.
spring.
e. Corticosteroids: They are responsible for silent perfora-
l Relapses are more common in smokers than in
tion of ulcer.
non-smokers.
f. Acid-pepsin versus mucosal resistance:
l Other symptoms:
l Cause of peptic ulceration is digestion of the mucosa
l Water-brash (excessive salivation), heart burns, loss
with acid and pepsin of gastric juice. Normal stomach is
of appetite and vomiting.
capable of resisting this digestion. So, the concept of
l Anorexia, nausea and dyspepsia.
peptic ulceration is acid plus pepsin versus mucosal re-
l Rarely, the patient might present for the first time with
sistance. Factors which tilt this balance produce ulcers.
anaemia of chronic blood loss, abrupt haematemesis,
l Gastric hypersecretion.
acute perforation or gastric outlet obstruction.
l Severe ulceration occurs in Zollinger-Ellison syndrome,
l Haematemesis is more common.
which is characterised by very high acid secretion.
l There is consequent weight loss.
l Acid secretion is more important in the aetiology of

duodenal ulcer than in gastric ulcer.


g. Factors reducing mucosal resistance:
l Several drugs, particularly those used in rheumatoid
Complication
arthritis. Investigations
l Aspirin is an important aetiological factor in gastric
i. Double contrast barium meal may show the ulcer as a
ulcer. crater or deformed duodenal cap.
l The organism Helicobacter pylori.
ii. Endoscopy can visualise the ulcer. A biopsy can be
l Reflux of bile and intestinal contents into stomach
taken from a ulcer to rule out malignancy and H. pylori
due to poorly functioning pyloric sphincter. infection.
h. Other risk factors include smoking and alcohol con- iii. Serum gastrin and gastric acid analysis in patients
sumption. suspected to Zollinger-Ellison syndrome.
iv. Tests for H. pylori: They include invasive and non-
invasive tests as follows.
Clinical Features
On endoscopic biopsy material:
l Peptic ulcer is a chronic condition with a natural history l Rapid urease test
of spontaneous relapses and remissions lasting for de- l Histology
cades or even life. l Culture
Section | I General Medicine 7

Non-invasive: l PPIs act by crossing the parietal cell membrane and


l Serology entering the acidic parietal cell canaliculus.
l Urea breath test l In the acidic environment, the PPI becomes protonated,
producing the activated form of the drug that binds cova-
Treatment lently with the H1/ K1 ATPase enzyme that results in ir-
reversible inhibition of acid secretion by the proton pump.
The treatment includes: l The parietal cell must then produce new proton pumps
I. Short-term management or activate resting pumps to resume its acid secretion.
II. Long-term management: l PPIs have superior healing rates, shorter healing time
i. Intermittent treatment and fast symptom relief compared to H2 blockers.
ii. Maintenance treatment l All PPIs appear to have similar efficacy in the treatment
iii. Surgical treatment of various acid-peptic disorders.
General measures l Recommended dosages are:
Omeprazole and rabeprazole: 20 mg daily for
l Avoid smoking 4-8 weeks.
l Avoid aspirin and non-steroidal anti-inflammatory drugs Lansoprazole: 15-30 mg daily for 4-8 weeks.
l Alcohol to be moderated Esomeprazole: 20-40 mg daily for 4-8 weeks.
l No special dietary advice Pantoprazole: 40 mg daily for 4-8 weeks.
I. Short-term management l Omeprazole, lansoprazole and esomeprazole should be
taken 30 minutes before a meal.
This includes use of many different drugs for management
l Rabeprazole and pantoprazole may be taken without
of acid peptic symptoms.
regard to meals.
a. Antacids and alginates: l They are indicated in: Reflux oesophagitis, GERD
l Commonly used antacids are a combination of alu- l Peptic ulcer unresponsive to other medical measures.
minium and magnesium compounds like aluminium l As an adjunct to anti-H. pylori treatment.
hydroxide, magnesium trisilicate, and alginic acid. l Zollinger- Ellison syndrome.
l The antacids and alginates act by forming a protective d. Prostaglandin analogues:
mucosal ‘raft’.
l They are mainly prescribed for symptomatic relief only. l Eg: Misoprostol
l Minor pain is treated with tablet preparations and severe l It acts by enhancing mucosal blood flow, stimulating
pain with liquid preparations. mucus and bicarbonate secretion and as well as stimu-
l Sodium bicarbonate is the quickest acting antacid. lating epithelial proliferation.
Usual dose is 15-30 ml liquid antacid 1 and 3 hours after l Misoprostol 200 micrograms 4 times daily. Useful in
food and at bedtime, for 4-6 weeks. preventing NSAID-induced mucosal injury.

b. Histamine H2-receptor antagonists: e. Colloidal bismuth compounds:


l For example, cimetidine, ranitidine, famotidine, and l Eg: Bismuth subsalicylate and colloidal bismuth
nizatidine. subcitrate.
l The mechanism of action is competitive inhibition of H2 l Mechanisms of action are:
receptors on parietal and ECL cells. i. These precipitate in acid conditions, binding with
l They can be prescribed as twice daily doses or as a proteins in the ulcer base to form a coat which pro-
single large dose at bedtime. tects against further acid-pepsin digestion.
l Eg: Cimetidine — 400 mg BD or 800 mg at night. ii. Powerful antimicrobial effect against Helicobacter
l Ranitidine and nizatidine — 150 mg BD or 300 mg pylori.
at night.
f. Complex salts:
l Famotidine — 20 mg BD or 40 mg at night.

l Symptomatic relief occurs within days and ulcer heal- l Eg: Sucralfate
ing within weeks. l Forms a protective coating for the ulcers.
l Duration of treatment of duodenal ulcer patients is 4 weeks l Aluminium salt of sucrose octasulphate little effect on
while gastric ulcer patients are treated for 6 weeks, fol- acid and may protect ulcer base from peptic activity and
lowed by endoscopy and further treatment, if necessary. enhance epithelial cell turnover.

c. Proton Pump Inhibitors: H. Pylori Eradication Therapy


l Eg: Substituted benzimidazoles like omeprazole, lanso- l Triple drug therapy is employed to eradicate H. pylori
prazole, pantoprazole, esomeprazole and rabeprazole. action.
8 Quick Review Series for BDS 3rd Year

l The regimen includes two antibiotics and PPI. Q. Discuss the aetiology, clinical features, investigations
Single antibiotic is not given to avoid the problem of management of malabsorption syndrome.
resistance.
Ans.
l Commonly employed regimen includes amoxycillin
1 g twice daily plus clarithromycin 500 mg twice daily Disorders of digestion, and diminished absorption of
plus twice a day PPI (omeprazole or rabeprazole 20 mg, dietary nutrients are referred as malabsorption syndromes.
lansoprazole 30 mg, pantoprazole 40 mg) for 14 days. Various diseases with varied aetiologies can lead to
Metronidazole may be used in place of amoxicillin in malabsorption as follows:
cases of penicillin allergy.
l Eradication of H. pylori is achieved in 85-90% cases. Aetiology
Successful eradication reduces the recurrence of ulcer. Disorders of intraluminal digestion
l If infection persists after giving triple therapy, quadru-
ple therapy (PPI, bismuth, tetracycline, metronidazole) A. Pancreatic enzyme deficiency:
is given. l Chronic pancreatitis
l Many clinicians administer drugs against H. pylori to all l Cystic fibrosis
patients with peptic ulcer disease even without docu- l Pancreatic carcinoma
menting the presence of bacteria.
l It reduces the risk of recurrence of ulcer formation. B. Disturbances of gastric function:
l Gastroenterostomy
II. Long-term management l Partial gastrectomy
i. Intermittent treatment:
C. Deficiency of bile acids:
l For symptomatic relapses less than 4 times a year.
l 4 weeks course of one of the ulcer healing agents. l Crohn’s disease
l Resection of terminal ileum
ii. Maintenance treatment l Stagnant loop syndrome or blind loop syndrome.
l Continuous maintenance treatment should not be neces-
sary after successful H. pylori eradication. Disorders of transport in the intestinal mucosal cell
l In minority who do require it, the lowest effective dose A. With histologically abnormal mucosa (infiltration,
should be used. inflammation or infection of mucosa):
l Long-term maintenance is with H2-receptor antagonists
l Coeliac disease
(cimetidine 400 mg at night, ranitidine 150 mg at night,
l Tropical sprue
famotidine 20 mg at night or nizatidine 150 mg at night).
l Lymphoma
iii. Surgical treatment l Whipple’s disease
l For gastric ulcer, the procedure of choice is partial gas- l Giardiasis
trectomy with a Billroth I anastomosis, in which ulcer l Radiation enteritis
itself and ulcer bearing area of the stomach are resected. B. With histologically normal mucosa
l Duodenal ulcer treatment could be truncal vagotomy (genetic diseases):
plus pyloroplasty or gastroenterostomy.
l Selective vagotomy with pyloroplasty or highly selec- l Lactase deficiency
tive vagotomy is no longer indicated for duodenal ulcer l Pernicious anaemia
disease. C. Disorders of transport from mucosal cell:
l In the emergency situation, ‘under-running’ the ulcer for
bleeding or ‘over sewing’, i.e. patch repair for perfora- l Abdominal lymphoma
tion is all that is required, in addition to taking a biopsy. l Tuberculosis
l In the presence of giant duodenal ulcers, partial gastrec- l Telangiectasia of mesenteric lymphatics
tomy using a ‘Polya’ or Billroth II reconstruction may l Abetalipoproteinaemia
be required. D. Impaired nutrient uptake:
l Elective surgery is done in gastric outflow obstruction
and recurrent ulcer despite medical treatment. l Lymphatic obstruction
l CHF, pericarditis
Q. 3. Discuss aetiology, clinical features and manage-
ment of malabsorption syndrome? E. Miscellaneous:
Or l Diabetes mellitus
Section | I General Medicine 9

l Hyperthyroidism l Sudan III stain may show an increase in the stool fat.
l Hypoparathyroidism l Quantitative estimation of fat in the stool is more
reliable and sensitive.
A 72-hour stool collection while the patient is on a de-
Clinical Features l

fined diet is used for fat estimation. Excretion of more


The onset of malabsorption syndrome is insidious and than 10 g fat per day suggests fat malabsorption.
progression is gradual.
ii. Schilling test:
General features l This is useful in the diagnosis of cobalamin (B12) mal-
Diarrhoea, abdominal pain, distension, loss of weight, absorption and its causes like pernicious anaemia,
anaemia and vague ill health. chronic pancreatitis, achlorhydria and bacterial over-
growth.
Specific features l Schilling test: Radio-labelled cobalamin (1mg 58Co) is
Specific features due to defective absorption of different given orally and its excretion in urine is measured.
constituents are as follows: One mg cobalamin is administered intramuscularly
i. Carbohydrates: Abdominal distension, belching, bloated to saturate hepatic binding sites so that all radio-
feeling in abdomen. labelled cobalamin is excreted in the urine. The test is
ii. Protein: Progressive emaciation, pitting pedal oedema. abnormal, if less than 10 % of the radio-labelled co-
iii. Fat: Loss of weight, diarrhoea, and loose, pale, bulky balamin is excreted in the urine in 24 hours. If the test
offensive stool which floats on water, i.e. steatorrhoea. is abnormal, it is repeated by giving radio-labelled
iv. Vitamins: cobalamin in combination with intrinsic factor or pan-
a. Vitamin A: Follicular keratosis, night blindness, xe- creatic enymes. It may also be repeated after a 5-day
rophthalmia, keratomalacia. course of antibiotics. This will help in differentiating
b. Vitamin D: Muscular irritability, tetany, features of the various defects responsible for malabsorption of
osteomalacia. cobalamin.
c. Vitamin K: Haemorrhagic tendencies.
iii. D-xylose test:
d. Vitamin Bl and B2: Angular stomatitis, cheilosis,
glossitis, neuropathy. It is performed to detect carbohydrate malabsorption. 25 g
e. Folic acid: Macrocytic anaemia, glossitis. D-xylose is given orally and its excretion is measured in
v. Minerals and electrolytes: urine. Excretion of less than 4.5 g in 5 h is indicative of
a. Sodium: Muscle cramps, weakness, hypotension. malabsorption.
b. Potassium: Weakness, areflexia, intestinal disten-
iv. Upper GI endoscopy and biopsy of small
sion, cardiac arrhythmias.
intestinal mucosa:
c. Calcium: Muscular irritability, tetany, features of
rickets, features of osteomalacia. It is essential for the diagnosis of conditions like tropical
d. Magnesium: Weakness, tingling sensation, tetany. sprue, celiac sprue, Whipple’s diasease, and Crohn’s disease.
e. Zinc: Anorexia, weakness, tingling, impaired
v. Barium meal contrast radiography:
taste.
f. Iron: Hypochromic microcytic anaemia, glossitis, Radiological assessment of the small intestine with barium
koilonychia. contrast is helpful in evaluation of structural abnormalities
g. Water: Dehydration, low blood volume. in Crohn’s disease, diverticulae and strictures.

vi. Pancreatic exocrine functions:


Investigations They are assessed in patients with steatorrhoea.
Routine laboratory studies
vii. Serological studies:
Tests are performed to detect any nutrient deficiency. These
Autoantibodies are detected in some conditions such as
tests reveal malabsorption of particular nutrient but do not
celiac sprue and pernicious anaemia.
establish the cause.
viii. Breath tests:
Specific Tests
l Cholyl-14C-glycine for bacterial overgrowth
i.Faecal fat estimation: l Lactose H2 for bacterial overgrowth
l This is a reliable test to confirm steatorrhoea and fat
malabsorption. ix. Small intestinal biopsy (duodenal or jejunal):
10 Quick Review Series for BDS 3rd Year

Treatment E. Neuromuscular disorders


l Bulbar paralysis
l Deficient nutrients are replaced. l Myasthenia gravis
l Gluten-free diet in coeliac disease. l Polymyositis
l Pancreatic supplements in pancreatic insufficiency.
l Low fat diet and cholestyramine for bile acid deficiency. F. Miscellaneous
l Replacement therapy for anaemia, bone disease and l Sjogren’s syndrome
coagulation defects. l Rabies.
l Oral folic acid, oral iron and intramuscular B12
l Vitamin D and calcium supplements.
l Vitamin B complex Investigations
l Treat dehydration and electrolyte deficiency by intrave- i. Endoscopy and biopsy: Endoscopy is the investigation
nous infusion. of choice as it facilitates the biopsy and dilatation of
Q. 4. How would you investigate a case of dysphagia. strictures.
Name some of the conditions causing dysphagia? ii. Barium swallow, with videofluoroscopic swallowing
assessment will detect most motility disorders.
Or iii. Oesophagoscopy allows removal of foreign body, visu-
What are the causes of dysphagia? How will you inves- alisation and biopsy of tumours, ulcers, strictures, etc.
tigate a case of dysphagia? iv. CT scan of thorax.
v. Oesophageal motility studies.
Ans.
Q. 5. Describe the pathogenesis, clinical features and
Dysphagia is defined as difficulty in swallowing. It has management of intestinal amoebiasis.
oropharyngeal and oesophageal causes as follows:
Ans.
I. Congenital
The disease is caused by infection by protozoan parasite
l Congenital stenosis of oesophagus. Entamoeba histolytica which gets lodged in the large intes-
l Tracheo-oesophageal fistula. tine causing dysentery and may spread to the liver and other
l Congenital web. organs.
II. Acquired
A. Causes within the oesophageal lumen Pathogenesis
l Foreign body. l It results from the infection of the large intestine by
Entamoeba histolytica.
B. Causes in the oesophageal wall l After ingestion, the cyst undergoes further nuclear divi-
l Strictures sion and trophozoites are released. The trophozoites
l Carcinoma oesophagus are carried to the large intestine where they produce
l Diverticulum the characteristic flask-shaped amoebic ulcerations.
l Reflux oesophagitis Incubation period is 2-6 weeks.
l Achalasia cardia l A localized granuloma, presenting as a papable mass in
l Plummer-Vinson syndrome the rectum or causing a filling defect in the colon on
l Oesophagitis (cytomegalovirus, candida) radiography, is a common complication.
l Amoebae may enter a portal venous system and be car-
C. Causes outside the oesophageal wall ried to the liver where they multiply and produce an
l Thyroid swelling amoebic liver abscess.
l Secondaries in the neck
l Mediastinal nodes
l Mediastinal abscess
Clinical Features
l Aortic aneurysm l In amoebic dysentery, there is intermittent diarrhoea
consisting of one to four, foul smelling, loose, watery
D. Painful diseases of mouth and pharynx stools which may contain mucus and blood.
l Stomatitis
l Tonsillitis
l Pharyngitis
Complications
l Retropharyngeal abscess l Massive haemorrhage
Section | I General Medicine 11

Perforation and peritonitis


l
Treatment
l Toxic megacolon in fulminant cases
l Post dysenteric colitis l Invasive intestinal amoebiasis is treated with metronida-
l Amoebic liver abscess zole 800 mg thrice daily for 5 days or tinidazole 2 g
l Pleuropulmonary amoebiasis daily as a single dose for three days.
l Amoebic pericarditis l Nitroimidazole classes of antimicrobial agents used
l Cutaneous amoebiasis are as follows:
a. Metronidazole 400 to 800 mg TDS 5 to 7 days.
b. Tinidazole 2 gm as single or divided three days dose.
Diagnosis c. Secnidazole 2 gm as single dose.
l Stool examination: Microscopic examination of a fresh d. Diloxamide furoate: It should be added because
sample of stool may demonstrate the motile trophozo- it has effect on cyst. It is given 500 mg TDS for
ites. They are about 30 microns in diameter, with a clear 10 days. Metronidazole has no effect on cyst.
ectoplasm and a granular endoplasm, and usually con- l General supportive measures, fluid and electrolyte
tain red blood cells. imbalance correction.
l Sigmoidoscopy: It may reveal the characteristic flask- l Diet given should be soft, liquid or semi-liquid.
shaped ulcers with normal surrounding mucosa, the as- l After treatment, furamide should be given at a dose of
pirated material, or scrapings from the ulcer, or biopsy 500 mg thrice daily for 10 days to eliminate luminal
of the ulcer may demonstrate the tropozoites. cysts, alternative agents include iodoquinol and paro-
l Serological test: Amoebic serology can detect antibodies. momycin.

SHORT ESSAYS
Q. 1. Glossitis. l Syphilis, and other disorders
l Occasionally, glossitis can be inherited.
Ans.
i. Glossitis is an abnormality of the tongue that results
Clinical Features
from an acute or chronic inflammation of the tongue.
ii. It causes the tongue to swell and change colour. Papillae l Tongue is usually swollen and attains a smooth
are lost, causing the tongue to appear smooth. appearance.
l Tongue colour usually dark “beefy” red and pale, if
caused by pernicious anaemia. Fiery red, if caused by
Etiology deficiency of B vitamins.
The causes of glossitis may be local or systemic. l Sore and tender tongue
l The nodules on the surface of the tongue (papillae) may
A. Local causes of glossitis: be absent.
l Bacterial or viral infections.
Mechanical irritation or injury from burns, rough
Investigations
l

edges of teeth or dental and oral appliances, or other


trauma. Appropriate laboratory tests may confirm systemic causes
l Exposure to irritants such as tobacco, alcohol, hot of the disorder.
foods, or spices.
Sensitization (allergic reaction) from toothpaste, mouth-
Treatment
l

wash, breath fresheners, dyes in candy, plastic in den-


tures or retainers, etc. l The goal of treatment is to reduce inflammation. Treat-
ment usually does not require hospitalization unless
B. Systemic causes of glossitis: tongue swelling is severe.
l Iron deficiency and pernicious anaemias and other B l Good oral hygiene is necessary, including thorough
vitamins deficiencies. tooth brushing at least twice a day, and flossing at least
l Oral lichen planus daily.
l Erythema multiforme l Corticosteroids, such as prednisone, may be given to
l Aphthous ulcers reduce the inflammation of glossitis. For mild cases,
l Pemphigus vulgaris topical applications such as prednisone suspension that is
12 Quick Review Series for BDS 3rd Year

used as a mouthrinse, may be recommended to avoid the b. Histamine H2-receptor antagonists:


side effects of systemic corticosteroids. l Eg: cimetidine, ranitidine, famotidine, and nizatidine.
l Antibiotics, antifungal medications, or other antimicro- l The mechanism of action is competitive inhibition of
bials may be prescribed, if the cause of glossitis is an H2 receptors on parietal and ECL cells.
infection. l Symptomatic relief occurs within days and ulcer heal-
l Anaemia and nutritional deficiencies must be treated, ing within weeks.
often by dietary changes or other supplements.
l Avoid irritants (such as hot or spicy foods, alcohol, and c. Proton Pump Inhibitors:
tobacco) to minimize the discomfort. l Eg: Substituted benzimidazoles like omeprazole, lanso-
prazole, pantoprazole, esomeprazole and rabeprazole.
Complications l In the acidic environment, the PPI becomes protonated,
producing the activated form of the drug that binds co-
l Discomfort valently with the H1/ K1 ATPase enzyme that results in
l Airway blockage irreversible inhibition of acid secretion by the proton
l Difficulties with speaking, chewing, or swallowing. pump.
l The parietal cell must then produce new proton pumps
Q. 2. Describe the treatment of peptic ulcer.
or activate resting pumps to resume its acid secretion.
Ans. l PPIs have superior healing rates, shorter healing time
and fast symptom relief compared to H2 blockers.
l Peptic ulcer is defined as mucosal ulceration near the
acid bearing regions of gastrointestinal tract. It is the d. Prostaglandin analogues:
ulcer in duodenum and stomach. l Eg: Misoprostol
l The treatment of peptic ulcer includes: l It acts by enhancing mucosal blood flow, stimulating
I. Short-term management mucus and bicarbonate secretion and as well as stimu-
II. Long-term management lating epithelial proliferation.
i. Intermittent treatment l Misoprostol 200 micrograms 4 times daily. Useful in
ii. Maintenance treatment preventing NSAID-induced mucosal injury.
iii. Surgical treatment
e. Colloidal bismuth compounds:
General measures l Eg: Bismuth subsalicylate and colloidal bismuth
l Avoid smoking subcitrate.
l Avoid aspirin and non-steroidal anti-inflammatory l Mechanisms of action are:
drugs i. These precipitate in acid conditions, binding with
l Alcohol to be moderated proteins in the ulcer base to form a coat which pro-
l No special dietary advice tects against further acid-pepsin digestion.
ii. Powerful antimicrobial effect against Helicobacter
I. Short-term management pylori.
This includes use of many different drugs for management
of acid peptic symptoms. f. Complex salts:
l Eg: Sucralfate
a. Antacids and alginates: l Forms a protective coating for the ulcers.
l Commonly used antacids are a combination of alu- l Aluminium salt of sucrose octasulphate little effect on
minium and magnesium compounds like aluminium acid and may protect ulcer base from peptic activity and
hydroxide, magnesium trisilicate, alginic acid. enhance epithelial cell turnover.
l The antacids and alginates act by forming a protective
mucosal ‘raft’. H. Pylori Eradication Therapy
l They are mainly prescribed for symptomatic relief l Triple drug therapy is employed to eradicate H. pylori
only. action.
l Minor pain is treated with tablet preparations and severe l The regimen includes two antibiotics and a PPI. Single
pain with liquid preparations. antibiotic is not given to avoid the problem of resis-
l Sodium bicarbonate is the quickest acting antacid. tance.
Usual dose is 15-30 mL liquid antacid 1 and 3 hours l Eradication of H. pylori is achieved in 85-90% cases.
after food and at bedtime, for 4-6 weeks. Successful eradication reduces the recurrence of ulcer.
Section | I General Medicine 13

II. Long-term management Diagnosis


i. Intermittent treatment: l Endoscopy
l For symptomatic relapses less than 4 times a year. l Biopsy
l 4 weeks course of one of the ulcer healing agents.

ii. Maintenance treatment: Treatment of Acute Gastritis


l Continuous maintenance treatment should not be neces- l The consumption of alcohol and drugs (NSAIDs) should
sary after successful H. pylori eradication. be avoided.
l In minority who do require it, the lowest effective dose l H2-receptor antagonists (ranitidine 10 mg twice daily,
should be used. famotidine 20 mg twice daily) started as it improves
l Long-term maintenance is with H2 receptor antagonists healing.
(cimetidine 400 mg at night, ranitidine 150 mg at night, l More potent agent omeprazole or other proton pump
famotidine 20 mg at night or nizatidine 150 mg at night). inhibitors can be given for more rapid relief.
l In mild cases, antacids 30 mL 4 - 6 hourly can improve
the condition.
iii. Surgical treatment: l Advice the patient not to take the drugs on empty
l For gastric ulcer, the procedure of choice is partial gas- stomach.
trectomy with a Billroth I anastomosis. l Switch over to selective inhibitors of COX-2, if analge-
l Duodenal ulcer treatment could be truncal vagotomy sics are really needed, e.g. valdecoxib or rofecoxib can
plus pyloroplasty or gastroenterostomy. be prescribed.
l In the presence of giant duodenal ulcers, partial gastrec-
tomy using a ‘Polya’ or Billroth II reconstruction may II. CHRONIC GASTRITIS
be required.
l Elective surgery is done in gastric outflow obstruction When the acute gastritis remains for the longer time and is
and recurrent ulcer despite medical treatment. not treated, it becomes chronic and is known as chronic
gastritis.
Q. 3. Gastritis.
Or Aetiology
Outline the clinical features and treatment of acute l Repeated injury to gastric mucosa by tea, coffee, alco-
gastritis. hol, spices
Ans. l Infection from throat, teeth, gums and sinuses
l NSAIDs
Gastritis is a histological diagnosis characterized by l Autoimmune pathology
inflammation of gastric mucosa, although it can be recog- l Very hot beverage
nized by endoscopy. l Gastrectomy.
There are mainly two types of chronic gastritis:
I. ACUTE GASTRITIS A. Type A gastritis
Acute gastritis is usually erosive and haemorrhagic. B. Type B gastritis.

A. Type A Gastritis (less common)


Aetiology l It involves body of stomach and spars antrum.
Drugs (aspirin, NSAIDs), alcohol, stress due to severe l It is caused during autoimmune disorders like type I
illness, and congestive gastropathy due to portal hyper- diabetes mellitus, Sjogren’s syndrome, Graves’ disease,
tension. Hashimoto disease, myasthenia gravis, etc.
l It is caused due to autoimmune activity against parietal
cells and parietal cell antibodies can be detected in
Clinical Features
serum.
l Erosive gastritis is usually asymptomatic. l In severe cases, parietal cell atrophy leads to deficiency
l It presents with anorexia, nausea, vomiting and ab- of intrinsic factor which leads to pernicious anaemia.
dominal pain. Haematemesis and melena are important l The disease is asymptomatic and long-term complica-
manifestations of erosive gastritis. tion is gastric carcinoma.
14 Quick Review Series for BDS 3rd Year

Treatment Investigations
In severe cases, corticosteroids are administered. In mild l Endoscopy
cases, parenteral iron should be administered. l Oesophagal pH monitoring
l Evaluation for H. pylori
B. Type B Gastritis l Gastric biopsy with, histology or urease test, urea
l This is more common form of gastritis and involves breath test
antrum of stomach.
l The usual cause is gram-negative bacteria H. pylori. The
condition is precursor of peptic ulcer. Treatment
l There is possibility of gastric carcinoma. l Avoid smoking, alcohol, spicy food
l Stress management
Diagnosis l Eradication of H. pylori: Amoxycillin 1 Clarithromycin 1
Omeprazole for two weeks
l Gastric acid study, i.e. achlorhydria l Antacids
l Haemoglobin decreases l H2 blockers: ranitidine
l Serum gastrin increases l Proton pump inhibitors: Omeprazole, pantoprazole
l Sucralfate
Management l Misoprostol

l Anti- H. pylori treatment: Q. 5. Causes of malabsorption syndrome?


a. Triple drug therapy: Proton pump inhibitor or raniti-
Or
dine 400 mg BD 1 Bismuth subcitrate 1 Amoxicil-
lin 1 gm or clarithromycin 500 mg or metronidazole Write short note on malabsorption syndrome.
500 mg BD.
b. Quadruple therapy: Omeprazole 10 mg BD1 Ans.
Tetracycline 500 mg QID 1 Bismuth subcitrate The malabsorption syndrome comprises a large number
QID 1 Metronidazole 500 mg TDS. of pathological conditions in which there is disturbance of
c. Parenteral vitamin B12 is administered. processes by which nutrients are transferred from lumen
Q. 4. Acid peptic disease. of intestine in to circulation.
Ans.
l Peptic ulcer is defined as the presence of ulcer in the
Aetiology
lower oesophagus, stomach or duodenum, in the jeju- i. Disorders of intraluminal digestion: Pancreatic enzyme
num after surgical anastomosis to the stomach or rarely, deficiency:
in the ileum adjacent to a Meckel’s diverticulum. l Chronic pancreatitis

l Pancreatic carcinoma

ii. Disturbances of gastric function:


Causes
l Gastroenterostomy
l Helicobactor pylori l Partial gastrectomy
l NSAIDs iii. Deficiency of bile acids:
l Smoking l Crohn’s disease
l Stress l Resection of terminal ileum
l Zollinger-Ellison syndrome l Stagnant loop syndrome or blind loop syndrome

iv. Disorders of transport in the intestinal mucosal cell:


l Coeliac disease
Clinical Features
l Tropical sprue
l Epigastric burning pain l Lymphoma
l Heart burn l Whipple’s disease
l Postprandial fullness l Lactase deficiency
l Nausea and vomiting l Pernicious anaemia
l Haematemesis v. Disorders of transport from mucosal cell:
l Anorexia and weight loss l Abdominal lymphoma
l Epigastric tenderness on palpation l Tuberculosis
Section | I General Medicine 15

vi. Impaired nutrient uptake: Describe the management of acute diarrhoea.


l Lymphatic obstruction
Ans.
l CHF

l Pericarditis Acute diarrhoea is caused mainly by infections (90%).


vii. Miscellaneous: It may also be caused by drugs, ischaemia, toxins and
l Diabetes mellitus other conditions.
l Hyperthyroidism

l Hypoparathyroidism
Aetiology
Causes of acute diarrhoea are as follows:
Clinical Features
A. Infectious:
a. The steatorrhoea is the presenting symptom.
b. Diarrhoea or abdominal discomfort. i. Viral
c. Nutritional deficiencies, i.e. deficiency of vitamin A, D, l Rotavirus
B12 and K. l Norwalk agents
d. General features like anaemia, sore mouth, loss of l Cytomegalovirus
weight, fatigue and lethargy.
e. Bone pain may be present. ii. Bacterial
f. Skin changes like pellagra are present. a. Preformed toxin
g. The patient also suffers from peripheral neuropathy, l S. aureus
irritability and lack of confidence. l B. cereus
l Clostridium perfringens
Investigations b. Enterotoxin induced
i. Routine laboratory studies: Tests are performed to l Enterotoxigenic E. coli (ETEC)
detect any nutrient deficiency. l Vibrio cholerae
ii. Specific tests:
c. Cytotoxin production
l Faecal fat estimation
l Enterohaemorrhagic E. coli (EHEC)
l Schilling test
l Clostridium difficile
l D-xylose test

l Upper GI endoscopy and biopsy of small intestinal


d. Mucosal invasion
mucosa l Sheigella
l Barium meal contrast radiography
l Campylobacter
l Pancreatic exocrine functions
l Salmonella jejuni
l Serological studies
l Enteroinvasive E. coli (EIEC)
l Breath tests
l Yersinia enterocolitica
l Small intestinal biopsy (duodenal or jejunal)

iii. Protozoal
Treatment l Entamoeba histolytica
l Giardia lamblia
l Deficient nutrients are replaced. l Cryptosporidium
l Gluten-free diet in coeliac disease.
l Pancreatic supplements in pancreatic insufficiency. B. Non-infectious:
l Low fat diet and cholestyramine for bile acid deficiency. l Diverticulitis
l Replacement therapy for anaemia, bone disease and l Inflammatory bowel disease (ulcerative colitis, Crohn’s
coagulation defects. disease)
l Oral folic acid, oral iron and intramuscular B12. l Metabolic (DKA, carcinoid)
l Vitamin D and calcium supplements. l Sepsis
l Vitamin B complex. l Drugs (NSAIDs, antibiotics)
l Treat dehydration and electrolyte deficiency by intrave- l Ischaemic colitis
nous infusion.
Q. 6. Diarrhoea. Diagnosis
Or Stool culture for bacteria or virus. Stool study for parasite.
16 Quick Review Series for BDS 3rd Year

General Management l Dysphagia to liquids: Neuromuscular


l Duration and course of dysphagia
i. Rest, maintenance of fluid and electrolyte balance.
l Associated symptoms like nasal regurgitation
ii. ORS should be given in all children as early as
l Bulbar palsy
possible.
l Severe weight loss
iii. Patient with constant vomiting or moderate to severe
l Malignancy
dehydration require IV fluid.
l Hoarseness of voice
iv. Ringer lactate is ideal, normal saline may be given.
l Malignancy
Anti-microbial / Anti-diarrhoeal Treatment l Laryngitis
l Chest pain
l Ciprofloxacin 500 mg BD for three days or
l Nalidixic acid 1gm 6 hourly for 5 - 7 days.
l It may be combined with the tinidazole 300 mg BD. Investigations
Anti-motility Agent l Barium swallow
l Upper GI endoscopy
l This should be used in children below 5 years of age.
l Oeophageal motility studies
l Loperamide or diphenoxylate atropine. Codeine could
be used.
l Sodium and water conserving agent, i.e. racecadrolit is Treatment
the newer drug safely given in the children and adults.
It reduces the loss of sodium and water in the stool l Treatment of cause
l Dietary modification
Q. 7. Dysphagia. l Nasogastric tube feeding
Ans. l Gastrostomy/jejunostomy.

Difficulty or a sense of obstruction while swallowing food Q. 8. Constipation.


is known as dysphagia.
Or

Causes Define constipation. What are the common causes of


a. Mechanical: constipation?
l Luminal: Large bolus, foreign body
l Intrinsic narrowing: Inflammation, webs and rings, be- Ans.
nign strictures, malignancy l Patients having bowel movements less frequently than
l Extrinsic compression: Cervical spondylosis, retropha- 3 times a week or if stool is hard and difficult to pass it
ryngeal abscess, enlarged thyroid, aortic aneurysm is known as constipation.
l Whatever may be the frequency the causes are as follows:
b. Neuromuscular
l Difficulty in initiation: Sjogren’s syndrome, paralysis of Acute:
tongue, oral anaesthesia i. Dehydration
l Disorders of striated muscle: Bulbar or pseudobulbar ii. Acute intestinal obstruction
paralysis, myasthenia gravis, motor neuron disease, iii. Acute appendicitis.
polymyositis, myopathies, rabies, tetanus.
Chronic:
c. Disorders of smooth muscle I. Functional:
l Scleroderma A. Rectal stasis.
l Myotonic dystrophy
l Oesophageal spasm l Faulty habits.
l Achalasia cardia l Impaired consciousness.
l Chagas disease l Painful anal area

B. Colonic stasis.
Clinical Features l Decreased food intake.
Approach to a patient with dysphagia l Decreased fibre residue.
l Dysphagia to solids: Mechanical dysphagia l Endocrine dysfunction.
Section | I General Medicine 17

l Drugs: Cocaine, morphine, antidepressants, calcium vi. Systemic sclerosis.


channel blockers, etc. vii. Drugs which reduce the tone of lower oesophageal
sphincter, e.g. aminophylline, beta-agonists, nitrates,
C. Irritable bowel syndrome. calcium channel blockers, etc.
II. Organic:
l Myxoedema. Clinical Features
l Systemic sclerosis
l Depression. a. Heartburn
l Diabetes mellitus. b. Regurgitation
l Hypercalcaemia c. Transient dysphagia to solids due to oesophageal spasm
l Diverticulitis. d. Hoarseness
l Megacolon. e. Odynophagia (i.e. painful swallowing).
l Pressure on rectum from tumours or gravid uterus. f. Strictures leading to persistent dysphagia to solids.
l Neurological: Parkinsonism, spinal cord injury. g. Iron deficiency anaemia.

Q. 9. Differences between gastric ulcers and duodenal


Investigations
ulcers.
i. Endoscopy
Ans.
ii. Barium meal.
Differences between gastric ulcers and duodenal ulcers are iii. Bernstein test.
as follows: iv. Oesophageal motility studies.

Parameter Gastric ulcer Duodenal ulcer Treatment


Age More than 40 years 20-50 years The treatment includes:
Sex Equal in both sexes More in males i. General measures
Course of the illness Less remittent More remittent ii. Medical treatment
iii. Surgical treatment.
Duration of episodes Relatively longer in Relatively shorter
of pain duration in duration i. General measures:
Use of antacids Relief of pain not Relief of pain l Weight reduction.
consistent promptly
l Small volume, frequent feeds.
Heartburn Less common More common l Avoid smoking, alcohol, fatty food, caffeine, mint,
Anorexia and nausea More common Less common orange juice, some medications.
l Avoid late night meals.
l Avoid weight lifting, stooping and bending at waist.
Q. 10. Reflux oesophagitis. l Elevation of head end of the bed should be maintained
Ans. at 15°.

l Reflux of gastric contents into the lower oesophagus is ii. Medical treatment:
called reflux oesophagitis or gastro-oesophageal reflux l In mild cases, use of liquid antacid after meal, it relieves
disease (GERD) heart burn.
l Chronic condition in which the lower oesophageal l In moderate cases, H2 receptor antagonists like cimeti-
sphincter allows gastric acids to reflux into the oesopha- dine 400 mg or ranitidine 150 mg, BD, etc.
gus, causing heartburn, acid indigestion, and possible l In severe cases, proton pump inhibitors (PPls) are used,
injury to the oesophageal lining. e.g. omeprazole (20-40 mg/day), lansoprazole (15-30
mg/day), pantoprazole (40 mg/ day), etc.
Metoclopramide or domperidone 10 mg t.i.d increases
Aetiology: l

lower oesophageal sphincter tone and promotes gastric


i. Sliding hiatus hernia emptying.
ii. Cardiomyotomy and vagotomy l Oral iron or blood transfusion for anaemia.
iii. Increased intra-abdominal pressure, e.g. pregnancy,
obesity, ascites, etc. iii. Surgical treatment:
iv. Reduced tone of lower oesophageal sphincter l Oesophageal stricture is treated by repeated dilatations
v. Impaired gastric emptying or surgical resection.
18 Quick Review Series for BDS 3rd Year

l Surgical return of lower oesophageal sphincter to the l Fungal, e.g. candidiasis and actinomycosis
abdomen. l Recurrent aphthous stomatitis
l Fundoplication. l Mucocutaneous diseases, e.g. lichenplanus, pemphigus
vulgaris, lupus erythematous, etc.
Q. 11. Stomatitis.
l Miscellaneous, e.g. diabetes, uremia and drug toxicity.
Ans.
Stomatitis is the inflammation of mouth and is caused by Clinical Features
bacterial, viral and fungal infections in persons with poor
l Ulcerative stomatitis due to infection by spirochaetes
oral hygiene or in blood dyscrasias.
and fusiform bacteria is characterised by halitosis and
oral ulcers.
Aetiology l Aphthous stomatitis is a recurrent painful superficial
ulceration of oral cavity precipitated by stress or men-
i. Local causes: Poor oral hygiene, excessive use of
struation.
tobacco, alcohol and spices, use of broad-spectrum
antibiotics and drugs such as iodine or gold.
ii. General causes: The main general causes are the infec- Treatment
tious diseases.
The general treatment for stomatitis consists of:
l Elimination of cause.
There are various types of infective stomatitis:
l Alkaline mouthwash.
l Bacterial, e.g. streptococcal stomatitis and Vincent’s
l Vitamin B complex.
stomatitis
l Treatment with topical steroids and vitamins in case of
l Viral, e.g. herpes simplex and herpes zoster
aphthous stomatitis.

SHORT NOTES
Q. 1. Constipation. ii. Upper GI bleeding (bleeding from oesophagus, stomach
and duodenum) commonly presents with heamatemesis
Ans.
or melaena and even haemorrhage from right side of
i. Patients having bowel movements less frequently than colon is occasionally responsible.
3 times a week or if stool is hard and difficult to pass it iii. The characteristic appearance of stool is the result of action
is known as constipation. of digestive enzymes and of bacteria up on haemoglobin.
ii. The causes of constipation are as follows: iv. Severe acute upper gastrointestinal bleeding sometimes
Acute: Dehydration, acute intestinal obstruction and cause maroon or bright red stool.
acute appendicitis.
Q. 3. High fibre diet.
Chronic: Rectal stasis, colonic stasis, irritable bowel syn-
drome and endocrine and metabolic diseases, myopathic Ans.
diseases, neurologic diseases and structural diseases
iii. Investigations: Complete blood count, serum glucose, i. Dietary fibre can be defined as those parts of food
TSH, stool examination and colonoscopy. which are not digested by human enzymes.
iv. Treatment: ii. Most dietary fibre is made up of non-starch polysac-
a) Eliminate offending medication, if any. charides (NSP), which is the natural packing of plant
b) Treat underlying medical condition causing consti- foods, e.g. fruits, milk, vegetables, beet or cane sugar,
pation. cereals like wheat, rice, root vegetables like potato,
c) If no secondary cause of constipation identified, legumes like lentils, beans, peas, etc.
empiric treatment for functional constipation. iii. Some types of NSP, mainly the hemicellulose of wheat,
d) Nonpharmcologic methods to improve bowel increase the water holding capacity of colonic contents
regularity. and the bulk of faeces. They relieve simple constipation
e) Laxatives and appears to prevent diverticulosis and reduce the risk
of cancer of colon.
Q. 2. Melaena. iv. Other viscous, indigestible polysaccharides like pectin
and guar gum have main effect in the upper gastrointes-
Ans.
tinal tract and they slow gastric emptying, contribute to
i. Melaena is the term used to describe passage of foul satiety and reduce the absorption of bile salt and hence
smelling, black tarry stools containing altered blood. plasma cholesterol concentration.
Section | I General Medicine 19

Q. 4. Ulcerative colitis. ii. Causes may be local or systemic: B-complex defi-


ciency, megaloblastic anaemia, cirrhosis, pernicious
Ans.
anaemia, iron deficiency anaemia, pellagra, scarlet fe-
i. Ulcerative colitis is an inflammatory disease affecting ver, syphilis, gonorrhoea, and tuberculosis.
mainly the large intestine, characterised clinically by iii. It causes the tongue to swell and change colour. Papil-
recurrent attacks of bloody diarrhoea, and pathologi- lae are lost, causing the tongue to appear smooth.
cally by diffuse inflammation of colonic mucosa. iv. The goal of treatment is to reduce inflammation. Corti-
ii. Aetiology: Familial or genetic, infectious, dietary fac- costeroids such as prednisone may be given to reduce
tors, smoking, psychological and defective immune the inflammation of glossitis.
regulation. v. Good oral hygiene and avoid irritants (such as hot
iii. Primarily involves the colonic mucosa uniformly. or spicy foods, alcohol, and tobacco) to minimize the
iv. General symptoms include bloody diarrhoea with mu- discomfort.
cus and pus, abdominal pain, especially lower abdomi- vi. Antibiotics, antifungal medications, or other antimicro-
nal, fever, weight loss, loss of appetite, symptoms and bials may be prescribed, if the cause of glossitis is an
signs of dehydration and anaemia, tenderness on palpa- infection.
tion over the colon, especially in the left iliac fossa.
Q. 7. Aetiological factors of acute diarrhoea.
Exacerbations and remissions are characteristic.
v. Treatment: Ans.
a. General measures: Parenteral nutrition in seriously
ill-patients, high protein, low residue diet, blood and The causes of acute diarrhoea are as follows:
plasma infusion, correction of dehydration and elec- i. Bacterial: The bacteria which causes acute diarrhoea
trolyte imbalance. are Shigella, Salmonella, E. coli, etc.
b. Medical management: Parenteral broad-spectrum ii. Viral: The viruses which results in diarrhoea are parvo-
antibiotics in septicaemia, codeine phosphate and virus, Hawaii, etc.
loperamide for diarrhoea, sulphasalazine is useful iii. Protozoal: The protozoans are E. histolytica, Giardia, etc.
in mild or moderately severe attacks (2-4 g/day) and iv. Toxic and systemic causes.
to prevent relapses, as a maintenance (0.5 g qid for v. Some drugs result in acute diarrhoea, i.e. broad-
1-2 years). spectrum antibiotics.
c. Surgical management: Q. 8. Dysphagia?
l Emergency surgical procedure is colectomy with

ileostomy. Ans.
l Elective surgical procedure is total proctocolec-
i. Difficulty or a sense of obstruction while swallowing
tomy with ileostomy or ileorectal anastomosis or food is known as dysphagia.
ileo-anal anastomosis. ii. Causes:
Q. 5. Recurrent Aphthous ulcers. a. Mechanical: Like large bolus, foreign body, inflam-
mation, webs and rings, benign strictures, malig-
Ans. nancy, etc.
l Recurrent aphthous ulcers occur in two forms major and b. Neuromuscular: Sjogren’s syndrome, paralysis
minor. The recurrent aphthous major is simply a more of tongue, oral anaesthesia, bulbar or pseudobulbar
severe form of recurrent aphthous minor. paralysis, myasthenia gravis, polymyositis, teta-
l Large-sized painful ulcers occur mostly on keratinized nus, etc.
mucosa ranging from 1 to 10 in number. iii. Clinical features: Nasal regurgitation, bulbar palsy,
l The lesions may exceed 1 cm in diameter persist up to severe weight loss, malignancy, hoarseness of voice,
6 weeks and healing occurs with scarring. malignancy, laryngitis and chest pain.
l The ulcers recur in waves over a long period of time iv. Investigations: Barium swallow, upper GI endoscopy
Their incidence is more in patients with HIV infection. and oesophageal motility studies.
l Histological features: Anitskow cells are also found v. Treatment: Dietary modification, nasogastric tube feed-
with elongated central chromatid in nucleus ing, gastrostomy/ jejunostomy.
l Treatment: Symptomatic treatment Q. 9. Acute gastritis.
Q. 6. Glossitis.
Ans.
Ans.
i. Gastritis is a histological diagnosis characterized by
i. Glossitis is an abnormality of the tongue that results inflammation of gastric mucosa, although it can be
from an acute or chronic inflammation of the tongue. recognized by endoscopy.
20 Quick Review Series for BDS 3rd Year

ii. Acute gastritis is usually erosive and haemorrhagic. ii. Aetiology: Pancreatic enzyme deficiency, chronic pan-
iii. Aetiology: Drugs (aspirin, NSAIDs), alcohol, stress creatitis, gastroenterostomy, Crohn’s disease, coeliac dis-
due to severe illness, and congestive gastropathy due to ease, tropical sprue, lymphoma, Whipple’s disease, etc.
portal hypertension. iii. The steatorrhoea is the presenting symptom.
iv. Clinical features: iv. Other features are diarrhoea or abdominal discomfort,
l Erosive gastritis is usually asymptomatic. nutritional deficiencies, general features like anaemia,
l It presents with anorexia, nausea, vomiting abdomi- sore mouth, loss of weight, fatigue and lethargy.
nal pain. Haematemesis and melaena are important v. Bone pain may be present and skin changes like pella-
manifestations of erosive gastritis. gra are also present.
v. Investigation: Endoscopy and biopsy. vi. Treatment consists of replacement of deficient nutri-
vi. Treatment: ents, gluten-free diet in coeliac disease, pancreatic
Short-term therapy – antacids (aluminium hydro- supplements, low fat diet, correction of dehydration and
chloride) electrolyte deficiency by intravenous infusion.
Acid suppression – proton pump inhibitors (omepra- Q. 13. Causes of chronic diarrhoea.
zole, lansoprazole)
Antiemetics—metoclopramide Ans.
Q. 10. Complications of peptic ulcer. Causes of chronic diarrhoea are as follows:
i. Chronic enteric infections: Salmonella, Streptococ-
Ans.
cus, fungi and viruses.
The complications of peptic ulcer are as follows: ii. Parasytic causes: Amoebic, giardiasis, leishmaniadon-
i. Gastrointestinal bleeding. ovani, Microsporidium and trichuriasis.
ii. Perforations. iii. Malabsorption syndrome.
iii. Gastric outlet obstruction (with fluid and electrolyte iv. Postoperative enterocolostomy.
imbalance). v. Intestinal, bilary, gastric fistulae.
iv. Gastric malignancy. vi. Inflammatory bowel diseases like ulcerative colitis
v. Pancreatitis (due to posterior penetration of ulcer) and Crohn’s disease.
vii. Diverticulitis, neoplasms
Q. 11. Lactose intolerance.
viii. Colitis due to drugs, i.e. mercury and arsenic.
Ans. ix. Miscellaneous: Hyperthyroidism, radition injury and
i. Lactose intolerance occurs due to deficiency of the carcinoid.
enzyme lactase due to which lactose cannot be hydro- Q. 14. H2 antagonists.
lysed, and hence it goes to colon where it is fermented
Ans.
by bacteria, causing symptoms.
ii. Primary lactase deficiency is racial and characterised i. H2 antagonists include cimetidine, ranitidine, famoti-
by normal intestinal biopsy. dine and nizatidine.
iii. Secondary lactase deficiency is associated with abnor- ii. The mechanism of action is inhibition of acid and pep-
mal intestinal biopsy. sin secretion by blocking H2 receptors.
iv. Seen in coeliac disease, tropical sprue, Crohn’s disease, iii. They can be prescribed as twice daily doses or as a
giardiasis and viral gastroenteritis. single large dose at bedtime symptomatic relief occurs
v. Clinical features: within days and ulcer healing within weeks.
Abdominal colic, abdominal distension, increased iv. Cimetidine: 400 mg BD or 800 mg at night. Side effects
flatus and diarrhoea after ingesting milk or milk include gynaecomastia in males, confusion in elderly,
products. oligospermia, and delay in elimination of warfarin,
vi. Improvement of symptoms on withdrawal of milk or phenytoin, and theophylline.
milk products. v. Ranitidine and nizatidine 150 mg BD or 300 mg at
vii. Treatment consists of lactose free or lactose restricted night. Side effects are confusion and liver dysfunction.
diet. vi. Famotidine: 20 mg BD or 40 mg at night. Side effects
Q. 12. Malabsorpion syndrome. are headache, dizziness and dry mouth.
Q. 15. Stomatitis.
Ans.
Ans.
i. The malabsorption syndrome comprises a large number
of pathological conditions in which there is disturbance i. Stomatitis is the inflammation of mouth and is caused
of processes by which nutrients are transferred from by bacterial, viral and fungal infections in persons with
lumen of intestine into circulation. poor oral hygiene or in blood dyscrasias.
Section | I General Medicine 21

ii. There are various types of infective stomatitis like l In most of the cases, these lesions are asymptomatic,
bacterial, viral, fungal, mucocutaneous diseases and however, in some cases, they may cause pain, feeling of
miscellaneous. thickness and burning sensation
iii. Ulcerative stomatitis due to infection by spirochaetes
and fusiform bacteria is characterised by halitosis and
Treatment Modalities
oral ulcers.
iv. Aphthous stomatitis is a recurrent painful superficial l Surgical excision, cryosurgery, CO2 – laser surgery,
ulceration of oral cavity precipitated by stress or men- retinoids and other drugs
struation. l Photodynamic therapy
v. The general treatment for stomatitis consists of elimi-
Q. 18. Chronic gastritis.
nation of cause, alkaline mouthwash, vitamin B com-
plex, topical steroids and vitamins in case of aphthous Ans.
stomatitis.
i. Gastritis is a histological diagnosis characterized by
Q. 16. Mention the upper gastrointestinal bleeding. inflammation of gastric mucosa. When the acute gastri-
tis remains for the longer time and is not treated, it
Ans. becomes chronic and is known as chronic gastritis.
The causes of upper GI haemorrhage are as follows: ii. Aetiology: Repeated injury to gastric mucosa by tea,
i. Oesophageal causes: coffee, alcohol, spices, infection from throat, teeth,
a. Oeophageal varices gums and sinuses, NSAIDs, autoimmune pathology
b. Oeophagitis and gastrectomy, etc.
c. Oeophageal ulcers iii. There are mainly two types of chronic gastritis:
d. Oeophageal cancers C. Type A gastritis
ii. Gastric causes: D. Type B gastritis.
a. Gastric ulcer iv. Type A gastritis is less common and asymptomatic,
b. Gastric cancer involves body of stomach and spars antrum, caused
c. Gastric erosions due to NSAIDs or alcohol during autoimmune disorders like type I diabetes mel-
iii. Duodenal causes litus, Sjogren’s syndrome, Graves’ disease, Hashimoto
a. Duodenal ulcer disease, myasthenia gravis, etc.
iv. Vascular malformations v. Treatment of type A gastritis: In severe cases, cortico-
v. Retching steroids are administered and in mild cases parenteral
a. Mallory-Weiss tear iron should be administered.
vi. Type B gastritis is more common form of gastritis and
Q. 17. Leukoplakia. involves antrum of stomach. The usual cause is gram-
negative bacteria H. pylori.
Ans. vii. Type B gastritis is treated by Anti-H. pylori treatment
Leukoplakia is a keratotic white lesion on oral mucosa that and parenteral vitamin B12 is administered.
cannot be characterized clinically or histologically as any Q. 19. Amoebiasis—clinical features and treatment.
other disease.
Ans.

Aetiology Ameobiasisis a gastrointestinal protozoal infection caused


by Entamoeba histolytica which is spread between humans
Tobacco, alcohol, candidiasis, dietary deficiency, syphilis, by its cysts.
trauma, galvanism, idiopathic, UV. radiation, etc.
Clinical Features
Clinical Features
i. In acute cases, very frequent motions with much blood
l It occurs in 4th, 5th, 6th and 7th decades of life. and mucus simulating bacillary dysentery or ulcerative
l Buccal mucosa and commissural areas are most fre- colitis occur in old people.
quently affected sites followed by alveolar ridge, tongue, ii. In chronic cases, incubation period is 2 weeks to many
lip, hard and soft palate, etc. years, abdominal pain and 2 or more unformed stools
l It presents as solitary or multiple white patches. a day, diarrhoea alternating with constipation, stools
l The surface of lesion may be smooth or finely wrinkled with blood and offensive odour, tenderness along
or even rough on palpation and lesion cannot be re- the line of colon especially over caecum and pelvic
moved by scrapping. colon.
22 Quick Review Series for BDS 3rd Year

iii. Investigation: Microscopic examination, sigmoidos- iv. The incubation period is short in case of non-infective
copy, examination of stools for cysts of E. histolytica, causes or due to ingestion of food with preformed
immunofluorescence test for antibodies. toxins.
iv. Management: v. Usually, there is a history of simultaneous occurrence
Oral metronidazole- 800 mg 8 hourly for 5 days Or of symptoms in more than one member of a household
Tinidazole- 2 g daily for 3 day or institution.
Diloxanide furoate-500 mg 8 hourly for 10 days vi. Diagnosis is based on the identification of the specific
v. Prevention: Avoid eating fresh uncooked vegetables or pathogen by culture of the patient’s stool, vomitus or
drinking unboiled water. blood, or the contaminated food.
Q. 20. Food poisoning.
Management
Ans.
l Non-specific therapy includes, oral or intravenous
i. Food poisoning is gastroenteritis of infective or non-
replacement of fluid and electrolyte deficits, and
infective origin.
codeine phosphate or loperamide for controlling
ii. The important infective causes are S. aureus, Salmo-
diarrhoea young children and elderly persons.
nella, B. cereus, and E. coli. Non-infective causes are
l Antibiotics should not be given routinely, as they are
allergy to sea foods, fish or fungal toxins.
usually ineffective and frequently exacerbate symptoms.
iii. The presentation is in the form of vomiting, diarrhoea
However, antibiotic therapy is indicated in selected cases
or both which usually occurs within 1-48 hours of
where the specific pathogen is isolated.
consumption of contaminated drinks or food.

t.me/desidentist
Topic 3

Diseases of Liver and Biliary System

LONG ESSAYS
Q. 1. Describe aetiopathogenesis, investigations and Clinical Features:
management of hepatic carcinoma.
l Usually occurs in patients with underlying cirrhosis.
Ans. l Non-specific symptoms like weakness, anorexia, weight
loss and fever.
l Hepatic carcinoma or hepatoma is the most common l More than 50% of patients present with hepatomegaly.
primary malignancy of liver. Pain or tenderness is the most common presentation.
l Frictional rub or bruit over the liver.
Blood-tinged ascites.
Aeiology l

l Metabolic disturbances include polycythaemia, hypo-


l Chronic hepatitis B and C virus infection glycaemia, acquired porphyria and hypercalcaemia.
l Contamination of food by aflatoxin (a fungal toxin) l A common feature among these patients is intravascular
l Alcoholic cirrhosis invasion and growth into portal vein or inferior vena cava.
l Primary biliary cirrhosis
Haemochromatosis
l
Investigations
l Wilson’s disease
l Thorotrast and arsenic l Serum alkaline phosphatase levels are very high.
l Oestrogens and androgens l A markedly increased or rising levels of alpha-fetoprotein.
l Anabolic steroids l Ultrasonography.
Section | I General Medicine 23

l CT scan of abdomen (triple-phase). l Blood culture may be positive.


l Liver scintigraphic scans. l Chest radiograph shows raised right dome of dia-
l Hepatic artery angiography shows ‘tumour blushes’. phragm, right basilar atelectasis and pneumonia or
l Liver aspiration or biopsy confirms the diagnosis. effusion.
l Ultrasonography confirms the diagnosis.
Management / Treatment l The pus is aspirated for culture sensitivity.
l If only the tumour is confined to one lobe, in the absence
of cirrhosis then surgical removal is performed. Treatment
l Palliative therapy. l Commence treatment with a combination of ampicillin,
l Percutaneous ethanol injection. gentamicin and metronidazole. Later, change the
l Transcatheter arterial chemoembolization (TACE). antibiotic according to the sensitivity reports.
Q. 2. What are the causes of liver abscess? Describe the l Ultrasound-guided aspiration of the abscess.
clinical features of liver abscess and complications? l Surgical drainage for the patients who fail to respond to
above therapy.
Ans.
Q. 3. Describe clinical features, complications, diagnosis
Bacteria reach the liver and cause abscess by one of the
and management of serum hepatitis B. Add a note on its
following 5 mechanisms:
prevention.
Aetiology Or
l The mechanisms by which bacteria causes the liver ab- Describe aetiology, clinical features, complications,
scess formation are as follows: diagnosis and management of serum hepatitis B. Add
i. Portal vein bacteraemia from appendicitis, diverticulitis a note on its prevention.
and perforated bowel.
ii. Systemic bacteraemia reaching liver via hepatic artery. Or
iii. Ascending cholangitis.
Enumerate the viruses causing acute hepatitis A. De-
iv. Direct extension from a contiguous focus of infection
scribe the clinical features and complications of viral
like subphrenic abscess.
hepatitis B. Add notes on the dental significance.
v. Penetrating trauma introducing the bacteria into liver,
or blunt trauma resulting in a hepatic haematoma Or
which gets secondarily infected.
l Common organisms involved are E. coli, anaerobic Describe the aetiology, clinical features and manage-
there is no response to salt. ment of viral hepatitis.
Ans.
Clinical Features
Various viruses causing acute viral hepatitis are as follows:
l Most have a subacute onset.
l Fever with chills and rigors. Specific hepatitis viruses are:
l Weight loss, anorexia, nausea and vomiting. l Hepatitis A virus (HAV)
l Right upper quadrant pain radiating to right shoulder. l Hepatitis B virus (HBV)
l Pleuritic chest pain. l Hepatitis C virus (HeV)
l Tender hepatomegaly. l Hepatitis D virus (HDV)
l Mild jaundice. l Hepatitis E virus (HEV)
l Single abscesses are more common in the right lobe of
the liver. Other viruses include:
l Multiple abscesses are seen in elderly patients, usually l Cytomegalovirus
due to ascending cholangitis. l Epstein-Barr virus
l Herpes simplex virus, etc.
Investigations
l Leucocytosis and increased ESR.
HEPATITIS B
l Mildly elevated serum bilirubin. l Hepatitis B is a DNA virus. It consists of a core contain-
l Markedly elevated serum alkaline phosphatase. ing DNA and a DNA polymerase enzyme needed for
l Low serum albumin. virus replication.
24 Quick Review Series for BDS 3rd Year

l Humans are the only source of infection. It is transmit- Signs of acute hepatitis are as follows:
ted through infected blood and blood products or by l The yellow sclera.
sexual contact. Infected mother can transmit infection l The skin may show scratch marks due to pruritus.
to the child at delivery. Tattooing and acupuncture can l In more than 50% cases, tender hepatomegaly is present
also spread the disease. while splenomegaly and lymphadenopathy may occur
in 10-20% cases.
Route of Transmission Complications
Horizontal transmission The most feared complications of acute hepatitis are as follows:
l Injection drug use l Acute fulminant hepatitis
l Infected unscreened blood products l Relapsing hepatitis
l Tattooing and acupuncture needles. l Cholestatic hepatitis
l Sexual (homosexual and heterosexual) l Chronic hepatitis and cirrhosis
l Aplastic anaemia
l Papular acrodermatitis
Vertical transmission l Myelitis and neuropathy, etc.
l HbsAg-positive mother.
l High-risk groups for HBV infection are patients of Investigations
haemodialysis, physicians, surgeons, dentists, para-
medical staff and persons working in laboratory and Serology
blood bank. l HBV has a number of antigens. The three important
l Patients are infectious during the incubation period and antigens are hepatitis B surface antigen (HBsAg), core
the illness and as long as they are positive for HBsAg. antigen (HBcAg) and hepatitis e antigen (HBeAg).
l Acute infection can progress to the chronic phase in l Appearance of hepatitis B surface antigen (HBsAg) in
about 1-10% adults and 90% children who have serum is the first evidence of infection. It normally per-
acquired the infection from the mother at birth. sists for 3-4 weeks but can persist up to 6 months. After
disappearance of HBsAg, antibody against HBsAg
(Anti-HBs) appears and persists for years and confers
Clinical Features of Acute Hepatitis immunity. Presence of Anti-HBs antibody means either
The clinical features of acute hepatitis according to various previous infection or vaccination.
stages are as follows: l The HBcAg is not found in the blood. However, anti-
body to it (anti-HBc) appears early during the illness.
I. Prodromal phase: Presence of IgM anti-HBc indicates acute infection and
l Development of jaundice is usually preceded by a pro- IgG anti-HBc suggests chronic infection (when HBsAg
dromal phase during which nonspecific systemic symp- positive) or recovery (when anti-HBs positive).
toms like anorexia, nausea, vomiting, headache, fatigue, l The presence of HBeAg indicates active viral replica-
malaise, myalgia, and arthalgia may occur. tion and high degree of infectivity. Anti-HBe appears as
l Low grade fever. HBeAg disappears and its presence suggests low level
of viral replication and decreased infectivity.
II. Icteric phase: l Serological tests are performed to identify the cause of
l Prodromal symptoms usually diminish with the onset of the hepatitis.
clinical jaundice. Patients may notice dark urine and yel-
lowish discolouration of eyes and skin. Clay-coloured Viral blood
stool and pruritus suggest cholestasis. l HBV-DNA can be measured by polymerase chain reac-
l Many patients with acute hepatitis may never become tion (PCR) in the blood. Viral loads are in excess of
icteric (anicteric hepatitis). 105 copies/ml in the presence of active viral replication.

III. Recovery phase: Other investigations


l The icteric phase is followed by an improvement in l During the early phase of hepatitis, there is significant
general symptoms and a diminution of jaundice. (.400 units/L) increase in the plasma ALT and AST.
l Complete clinical and biochemical recovery occurs l High alkaline phosphatase level suggests cholestasis.
within 3-4 months in the majority of patients with l Prolongation in prothrombin time (PT) is a reliable
hepatitis B. indicator of severe liver damage.
Section | I General Medicine 25

Management l Autoimmune hepatitis


l Drug induced
A. Acute hepatitis l Biliary cirrhosis
Treatment is supportive with monitoring for acute liver failure. l Hemochromatosis
a. Physical activity is restricted, bed rest is recommended l Wilson’s disease
only in severe cases. l Cardiac cirrhosis
b. A high calorie diet is recommended. A good protein in- l Alpha-l antitrypsin deficiency
take should be encouraged. Hospitalization and intrave- l Idiopathic
nous fluid (10% glucose) are indicated, if oral intake is
not adequate or there is marked nausea and vomiting.
c. Drugs which are hepatotoxic or those that are metabo- Pathogenesis
lized in the liver should be avoided. l Irrespective of the cause, the activation of stellate cells
d. Bile salt sequestering agent (cholestyramine) reduces is the central event in the development of cirrhosis.
pruritus in cases with cholestasis. l The activated stellate cells transform into multifunc-
e. Patients with features of severe hepatic failure such as tional cells upon interaction with hepatocytes, Kupffer
alteration in mental status (hepatic encephalopathy) and cells, and cytokines.
prolonged PT or bleeding time should be hospitalized. l The transformed cells form type I collagen leading to
f. No specific therapy is recommended for acute viral fibrosis.
hepatitis except in acute HCV infection. Subcutaneous l The cirrhosis can be micronodular typically in alcoholics
interferon alpha has been shown to reduce the rate of where the regenerating nodules are small usually (,1 mm).
chronicity in acute HCV hepatitis. The macronodular form is characterized by larger nodules
g. Liver transplantation may be required in cases with and is seen in posthepatitic or postnecrotic cirrhosis.
acute fulminant hepatic failure.
Clinical Features
Prevention of Viral Hepatitis
l Cirrhotic patients may be asymptomatic.
The prophylaxes for HAV and HBV are available: l Mostly symptoms such as weakness, fatigue, weight loss,
Active immunization (pre-exposure prophylaxis): anorexia, nausea, vomiting and abdominal discomfort
Recombinant hepatitis B vaccine containing HBsAg is occur insidiously.
available and is capable of producing active immunization l They may be only diagnosed incidentally.
in 95% of normal individuals. l Fibrosis and distorted vasculature may lead to portal
Q. 4. Classify cirrhosis of liver and discuss the clinical hypertension and complications associated with it.
features, diagnosis and complications of cirrhosis of liver. l Patient may present with features of portal hypertension
like abdominal distension due to ascites and spleno-
Or megaly, haematemesis and melaena due to variceal rup-
Discuss aetiology, clinical features, complications and ture or hepatic encephalopathy.
treatment of cirrhosis of the liver. l The liver is firm, nontender, and nodular and is enlarged
initially. As the disease progresses, the liver size reduces
Or
due to fibrosis.
Discuss the aetiopathogenesis, clinical features and l Hepatocellular dysfunction leads to jaundice, oedema,
complications of cirrhosis of liver. coagulopathy and metabolic abnormalities.
l Jaundice is generally absent or mild initially. It may
Ans.
become severe at later stages.
Cirrhosis of liver is the end result of the hepatocellular in- l The signs of chronic hepatic dysfunction such as spider
jury characterized by the presence of extensive fibrosis, nevi, palmar erythema, gynaecomastia, testicular atro-
regenerative nodules and loss of liver architecture. phy and loss of hair may occur due to disturbances in
hormonal metabolism.
Females may have loss of libido, menstrual abnormalities
Aetiology l

and breast atrophy.


i. Two of the commonest causes of cirrhosis are viral hepa- l Spider nevi are dilated central arterioles with radiating
titis (B, C and D) and prolonged excessive use of alcohol. small vessels looking like spider are found mainly on
ii. Other important causes of cirrhosis are as follows: the upper part of the body.
l Viral hepatitis l Ascites and hepatic encephalopathy can result from
l Chronic alcoholism (Laennec’s cirrhosis) both mechanisms.
26 Quick Review Series for BDS 3rd Year

l Patients may also have haemorrhagic manifestations l Vitamin supplementation.


such as epistaxis and increased menstrual flow. l Salt restriction is required in case of ascites.
l Other features include enlargement of parotid gland and l Medications which are hepatotoxic or metabolized

lacrimal glands, digital clubbing, Dupuytren’s contracture, in liver should be given with caution.
and skin pigmentation. ii. Treatment of specific cause:
l Clinical features of cirrhosis may be summarized as follows: l Alcohol abstinence is mandatory in alcoholic

i. Features due to hepatocellular dysfunction include: cirrhosis.


Jaundice, ascites, hepatomegaly, spider nevi, pal- l Specific therapy is needed in hemochromatosis and

mar erythema, gynaecomastia, testicular atrophy, Wilson’s disease.


menstrual abnormalities, breast atrophy, bleeding iii. Management of specific complications
tendency and hepatic encephalopathy. iv. Liver transplantation: The irreversible progressive
ii. Features due to portal hypertension are: Ascites, sple- chronic liver failure due to cirrhosis is the most common
nomegaly, variceal bleeding, and hepatic encepha- indication for liver transplantation.
lopathy.
iii. Other miscellaneous features are: Parotid and lacri-
mal gland enlargement, clubbing, opaque nails Prognosis
(leukonychia), Dupuytren’s contracture and skin Overall the prognosis of cirrhosis is poor. The prognosis is
pigmentation. favourable if the cause can be corrected.
Q. 5. Classify jaundice, describe the aetiology, clinical
Complications
features, diagnosis, complications and its management.
Complications of cirrhosis of liver are as follows:
l Portal hypertension Ans.
l Ascites i. Jaundice is defined as yellowish pigmentation of skin,
l Upper GI bleeding mucous membrane and sclera, resulting from elevated
l Spontaneous bacterial peritonitis (SBP) levels of bilirubin in the blood. Scleral pigmentation is
l Hepatic encephalopathy due to the richness of this tissue in elastin which has a
l Hepatorenal syndrome special affinity for bilirubin.
l Hepatocellular carcinoma ii. Clinical jaundice occurs when serum bilirubin is more
than 3 mg% (normal is 0.3 - 1.0 mg%).
Laboratory Findings iii. The term latent jaundice is applied when serum biliru-
i. Blood examination: bin is more than 1 mg% but less than 3 mg%.
l Anaemia can occur due to bleeding, folate defi- iv. The clinical detection of jaundice is difficult in artificial
ciency, marrow suppression or hypesplenism. Leu- light. Hence, it should be examined preferably in day
kopenia and thrombocytopenia. light.
l Aminotransferases (ALT, AST) are frequently ele- v. Yellow discolouration of the skin can also occur in
vated whereas a rise in the serum bilirubin and ALP carotenaemia (carotenoderma) and exposure to quina-
may occur later. Serum albumin is low and PT is crine or phenols. Sclera is typically not involved in
frequently prolonged. carotenaemia.
ii. Imaging: Ultrasonography to assess the liver size and
texture, ascites, portal hypertension and splenomegaly. Classification of Jaundice
iii. Endoscopy: Upper gastrointestinal endoscopy to detect
oesophageal varices and to exclude other causes of upper Jaundice can be classified in two ways:
gastrointestinal bleeding in the stomach and duodenum. A. Based upon the underlying derangement of bilirubin
iv. Liver biopsy: Helps in the assessment of severity of the cir- metabolism.
rhotic changes and confirms the specific cause of the cir- B. Based upon the pathological mechanism giving rise to
rhosis. jaundice.

Management A. Classification based upon the underlying


derangement of bilirubin metabolism
It includes general management, treatment of specific cause,
i. Predominantly unconjugated
management of the complications and liver transplantation.
a. Overproduction
i. General management:
b. Decreased hepatic uptakes
l The diet should contain an adequate amount of pro-
c. Decreased conjugation
tein and calories.
Section | I General Medicine 27

ii. Predominantly conjugated hyperbilirubinaemia. b. Hepatocellular diseases:


a. Intrahepatic l The hepatocellular jaundice results from inability of
b. Extrahepatic. the liver to transport bilirubin into the bile as a conse-
l Jaundice is classically divided into two broad quence of parenchymal liver disease.
types, unconjugated hyperbilirubinaemia and con- l Bilirubin uptake, conjugation and its transport to bile
jugated hyperbilirubinaemia. canaliculi can all be affected due to parenchymal liver
disease.
i. Unconjugated hyperbilirubinaemia: l Causes of hepatocellular jaundice are:
Unconjugated hyperbilirubinaemia may result from either a. Viral hepatitis: A,B,C,D,E, EBV, CMV
over production of bilirubin or from impaired hepatic b. Alcohol
uptake or conjugation of bilirubin. This is characterized by c. Drugs: Isoniazid, paracetamol
predominantly high levels of serum unconjugated bilirubin d. Toxins:.Vinyl chloride, amanita phalloides.
The colour of urine is normal. e. Metabolic: Wilson’s disease
a. Over production of bilirubin (prehepatic): This may oc- f. Immune: Autoimmune hepatitis
cur in haemolytic disorders such as hereditary spherocy-
tosis, sickle cell anaemia, G6PD deficiency, paroxysmal c. Cholestatic jaundice (obstructive jaundice):
nocturnal haemoglobinuria and autoimmune haemolytic l This type of jaundice results from the obstruction in the
anaemia. Other causes include ineffective erythropoie- blood flow which may be intrahepatic or extrahepatic.
sis, haemolytic reaction and resolution of haematoma. l Causes of obstructive jaundice are as follows:
b. Impaired hepatic metabolism: The hepatic uptake of a. Intrahepatic: Hepatitis, primary biliary cirrhosis and
bilirubin is impaired because of some drugs like rifampi- drugs like chlorpromazine, anabolic steroids, contra-
cin and probenecid and in Gilbert’s syndrome. The con- ceptives.
jugation of bilirubin is impaired in Gilbert’s syndrome b. Extrahepatic: Stone in bile duct, sclerosing cholangi-
and Crigler-Najjar syndromes because of decreased ac- tis, neoplasms of gallbladder and pancreas, compres-
tivity of the enzyme, glucuronyl transferase. sion by enlarged lymph nodes.
l Conjugated bilirubin enters into blood and appears in
Most neonates develop mild rise in unconjugated biliru-
urine. The stool becomes pale due to reduce, formation
bin because of incompletely developed hepatic functions
of stercobilinogen in the intestine.
(physiologic neonatal jaundice). Marked elevation of
l Retention of bile salts leads to its deposition in the skin
unconjugated bilirubin in premature infants can lead to
and severe pruritus (itching). Patients usually have
kernicterus.
scratch marks on skin.
Gilbert’s syndrome, an autosomal dominant disorder, is
l Reduced availability of bile in the intestine may lead to
a common disorder with mild jaundice. Conjugation of
malabsorption of fat and fat-soluble vitamins (vitamin
bilirubin is also impaired due to decreased activity of gluc-
K and vitamin D). This may result in bleeding diasthesis
uronyl transferase enzyme. Serum bilirubin is generally
and osteomalacia.
less than 6 mg/dL. No treatment is required.
l Serum alkaline phosphatase raised out of proportion as
Crigler-Najjar syndromes are characterized by complete
compared to aminotransferases.
absence of activity of glucuronyl transferase (type I) or re-
duced activity of the enzyme (type II). The jaundice is se- Q. 6. Define and describe the clinical features of ascites.
vere and death occurs early in type I Crigler-Najjar syn- Describe the pathogenesis and treatment of cirrhotic
drome. Most patients with type II Crigler-Najjar syndrome ascites.
survive up to adulthood.
Ans.
ii. Conjugated Hyperbilirubinaemia:
Ascites is defined as an accumulation of excess fluid within
Serum conjugated bilirubin is raised in inherited conditions the peritoneal cavity.
such as Dubin-Johnson syndrome and Rotor syndrome,
hepatocellular diseases and cholestatic conditions.
Aetiology
a. Dubin-Johnson and Rotor syndromes: Causes of ascites are as follows:
l These are rare inherited disorders characterized by asymp-
tomatic jaundice due to altered excretion of bilirubin in the i. Normal peritoneum (transudates):
bile ducts. l Cirrhosis with portal hypertension
l The liver transaminases and alkaline phosphatase levels l Congestive heart failure (CHF)
are normal. l Hypoproteinaemia (nephrotic syndrome, malnutrition)
28 Quick Review Series for BDS 3rd Year

l Pancreatitis l The epigastrium and umbilical regions remain resonant


l Biliary ascites due to floating intestines.
l In massive ascites, the whole of the abdomen is dull
ii. Diseased peritoneum (exudates): except for a small area over the umbilical region.
l Tuberculous peritonitis
Bacterial peritonitis
Secondary Effects of Ascites
l

l Malignancy (peritoneal, hepatic)


l Scrotal oedema.
iii. Miscellaneous: l Pleural effusion, especially right-sided. Pleural effusion
l Chylous ascites (filariasis, trauma, tumour) develops due to defects in the diaphragm allowing the
l Meig’s syndrome ascitic fluid to pass into the pleural space.
l Budd-Chiari syndrome l Oedema results from hypoproteinaemia and a func-
tional block of inferior vena cava due to tense ascites.
Mechanisms of ascites formation are as follows: l Cardiac apex is shifted upwards due to raised dia-
i. Inflammation of peritoneum leads to increased capil- phragm.
lary permeability and transudation of fluid into the l Neck veins may be distended secondary to an increase
peritoneal cavity. The same mechanism is responsible in right atrial pressure, which follows tense ascites and
for diminished reabsorption of fluid. Both these factors raised diaphragm.
favour the development of ascites in bacterial peritoni- l Meralgia paresthetica can occur due to compression of
tis and tuberculous peritonitis. lateral cutaneous nerve of thigh.
ii. Venous obstruction can lead to transudation of fluid into
peritoneal cavity, e.g. inferior vena caval obstruction.
iii. Lymphatic obstruction can lead to chylous ascites. Investigations
Involvement of mesenteric lymph nodes, thoracic duct i. Ultrasonography is very sensitive in confirming ascites,
and abdominal lymphatic ducts can result in leakage of detecting small amounts of fluid and in identifying the
chyle into peritoneal cavity. cause.
iv. Rupture of a viscus can result in outpouring of blood, ii. Diagnostic paracentesis.
cystic fluid or contaminated material, favouring ascites, iii. Laparoscopy and peritoneal biopsy.
e.g. pancreatitis. iv. Examination of ascitic fluid

Clinical Features Management


l The main symptoms are distension of abdomen with or Treatment of ascites in cirrhosis consists of the following
without discomfort or pain. steps:
l Dyspnoea and orthopnoea due to elevation of diaphragm. l Salt in the diet is restricted to 2-3 gm/day. Diuretics are
l Indigestion and heart burns due to gastro-oesophageal given if there is no response to salt ristriction. Spirono-
reflux resulting from increased intra-abdominal pressure. lactone is initially used in a dose of 100 mg/day which
l Other symptoms occur according to the cause of ascites. is increased to 400 mg/day. Frusemide (40-160 mg/day)
There may be fever in infective pathology while marked is added in case the response is inadequate. The goal is
weight loss suggests malignancy. to reduce the weight at a rate of not more than 1 kg/day
l Patients should be questioned to ascertain, if any risk in patients with anasarca and not more than 0.5 kg/day
factors of chronic liver disease such as viral hepatitis, in patients with ascites alone. Vigorous k 1diuresis may
alcohol use, transfusions, and IV drug abuse exist. precipitate hepatic encephalopathy and renal failure.
l Inspection of abdomen may reveal distension with full- l Therapeutic paracentesis is needed in massive ascites
ness in the flanks, everted and horizontal umbilicus, ab- with respiratory compromise. A volume of 3-5 liters is
dominal striae, prominent and tortuous veins and hernia. removed. Intravenous colloids such as albumin is given
l Fluid thrill is present in massive ascites and shifting dull- concomitantly to maintain intravascular volume.
ness is demonstrable on percussion in moderate ascites. l Portacaval shunt surgery or implantation of peritoneo-
Shifting dullness elicitation requires a minimum of venous shunt (between abdominal cavity and superior
1,000 ml of fluid. Fluid thrill is elicitable in tense ascites. vena cava) may be indicated in refractory ascites.
l Other findings will be present depending on the under- l TIPSS (transjugular intrahepatic portosystemic stent
lying cause of ascites. For example, spider nevi, palmer shunt): It can relieve resistant ascites. It can be used
erythema, and gynaecomastia suggest chronic liver dis- where liver function is reasonable or in patients await-
ease. Splenomegaly is present in portal hypertension. ing liver transplantation.
Section | I General Medicine 29

SHORT ESSAYS
Q. 1. Classify jaundice. Discuss the aetiology and clini- l Bilirubin uptake, conjugation and its transport to
cal features of jaundice. bile canaliculi can all be affected due to parenchy-
mal liver disease.
Ans. l Causes of hepatocellular jaundice are:

i. Jaundice is defined as yellowish pigmentation of skin, a. Viral hepatitis: A,B,C,D,E, EBV, CMV
mucous membrane and sclera, resulting from elevated b. Alcohol
levels of bilirubin in the blood. Scleral pigmentation is c. Drugs: Isoniazid, paracetamol
due to the richness of this tissue in elastin which has a d. Toxins:.Vinyl chloride, amanita phalloides.
special affinity for bilirubin. e. Metabolic: Wilson’s disease
ii. Clinical jaundice occurs when serum bilirubin is more f. Immune: Autoimmune hepatitis
than 3 mg% (normal is 0.3 - 1.0 mg%). c. Cholestatic jaundice (obstructive jaundice):
iii. The term latent jaundice is applied when serum biliru- l This type of jaundice results from the obstruction

bin is more than 1 mg% but less than 3 mg%. in the blood flow which may be intrahepatic or
extrahepatic.
l Causes of obstructive jaundice are as follows:

Classification of Jaundice a. Intrahepatic: Hepatitis, primary biliary cirrho-


sis and drugs like chlorpromazine, anabolic
Jaundice is classically divided into two broad types, un-
steroids, and contraceptives.
conjugated hyperbilirubinaemia and conjugated hyperbili-
b. Extrahepatic: Stone in bile duct, sclerosing
rubinaemia.
cholangitis, neoplasms of gallbladder and pan-
i. Unconjugated hyperbilirubinaemia: Unconjugated hy-
creas, compression by enlarged lymph nodes.
perbilirubinaemia may result from either over produc-
l Conjugated bilirubin enters into blood and appears
tion of bilirubin or from impaired hepatic uptake or
in urine. The stool becomes pale due to reduced
conjugation of bilirubin. This is characterized by pre-
formation of stercobilinogen in the intestine.
dominantly high levels of serum unconjugated bilirubin.
l Retention of bile salts leads to its deposition in the
The colour of urine is normal.
skin and severe pruritus (itching). Patients usually
a. Over production of bilirubin (prehepatic): This may
have scratch marks on skin.
occur in haemolytic disorders such as hereditary sphe-
l Reduced availability of bile in the intestine may
rocytosis, sickle cell anaemia, and G6PD deficiency.
lead to malabsorption of fat and fat-soluble vita-
Other causes include ineffective erythropoiesis, hae-
mins (vitamin K and vitamin D). This may result
molytic reaction and resolution of haematoma.
in bleeding diasthesis and osteomalacia.
b. Impaired hepatic metabolism:The hepatic uptake of
bilirubin is impaired because of some drugs like ri- Q. 2. Discuss in detail, aetiology, clinical features, inves-
fampicin and probenecid and in Gilbert’s syndrome. tigations and management of viral hepatitis-E.
Most neonates develop mild rise in unconjugated
Ans.
bilirubin because of incompletely developed hepatic
functions (physiologic neonatal jaundice). Marked l Hepatitis E virus (HEV) is an RNA virus spreads by
elevation of unconjugated bilirubin in premature in- faeco-oral route. Previously called epidemic or enteri-
fants can lead to kernicterus. cally-transmitted non-A, non-B hepatitis.
ii. Conjugated hyperbilirubinaemia: Serum conjugated l Accounts for epidemic, water-borne hepatitis, common
bilirubin is raised in inherited conditions such as Dubin- in India.
Johnson syndrome and Rotor syndrome, hepatocellular l Commonly occurs after contamination of water supplies
diseases and cholestatic conditions. as after monsoon flooding.
a. Dubin-Johnson and Rotor syndromes: l The clinical features are similar to HAV infection. It
l These are rare inherited disorders characterized by does not cause chronic infection.
asymptomatic jaundice due to altered excretion of l Acute fulminant hepatic failure occurs at high fre-
bilirubin in the bile ducts. quency if the hepatitis occurs during pregnancy and is
l The liver transaminases and alkaline phosphatase associated with high mortality.
levels are normal. l The diagnosis is made by detecting anti-HEV antibod-
b. Hepatocellular diseases: ies, IgM type during early phase and IgG type after
l The hepatocellular jaundice results from inability recovery.
of the liver to transport bilirubin into the bile as a l Incubation period is 40 days (15-60 days).
consequence of parenchymal liver disease. l Acute onset and occurs usually in young adults.
30 Quick Review Series for BDS 3rd Year

l Prevention is similar to hepatitis A, i.e. no vaccine l Hypoproteinaemia (nephrotic syndrome, malnutrition)


available. l Budd-Chiari syndrome
l Good prognosis. l Pancreatitis
l Biliary ascites
Q. 3. How will you proceed to investigate a case of jaun-
l Chylous ascites
dice in a patient about 50 years age?
l Tuberculous peritonitis
Ans. l Bacterial peritonitis
l Malignancy
Investigation of a case of jaundice proceeds as follows:
l First step is:
i. History and clinical examination Clinical Features
ii. Urine for bilirubin
l The main symptoms are distension of abdomen with or
iii. Serum for biochemical liver tests.
without discomfort or pain.
l Based on the results of above tests, if only there is
l There may be fever in infective pathology while marked
weight loss suggests malignancy.
l Patients should be questioned to ascertain if any risk
factors of chronic liver disease such as viral hepatitis,
Isolated bilirubin rise with other liver Conjugated bilirubin rise and
alcohol use, transfusions, and IV drug abuse exist.
biochemistry tests being normal abnormal liver function tests l Inspection of abdomen reveals distension with fullness
in the flanks, everted and horizontal umbilicus, abdomi-
Ultrasonography nal striae, prominent and tortuous veins and hernia.
l Fluid thrill is present in massive ascites and shifting
Unconjugated bilirubin rise Conjugated bilirubin rise
dullness is demonstrable on percussion in moderate
ascites.
Blood film or reticulocyte count Dubin–Johnson/ l Depending on the underlying cause of ascites, other
Rotor’s syndrome
(very rare) findings will be present like, spider nevi, palmer
erythema, gynaecomastia and splenomegaly.
+ve Coombs test + −ve Gilbert’s
Haemolysis work-up syndrome
Biliary obstruction No evidence of
Investigations
(i.e. dilated bile ducts) biliary disease
l Ultrasonography: Ultrasonography of abdomen detect
presence of minimal amount of fluid and helpful when
Cholangiography Hepatocellular clinical signs are not present. It is used for guiding para-
(MRCP or ERCP) jaundice centesis.
l Paracentesis:
Clotting l Abdominal paracentesis is a routine investigation to
Hepatitis serology determine the cause.
Immunoglobulins
l In cirrhosis, the appearance of ascitic fluid is clear,
Autoantibodies
Iron and copper straw-coloured. The fluid is milky white in chyle asci-
studies tes while it is cloudy in infections. Haemorrhagic fluid
may be seen due to trauma, tumour or tuberculosis.
Q. 4. Discuss aetiology, clinical features and manage- l The presence of more than 500 leucocytes/mL sug-
ment of ascites? gests inflammatory conditions.
l Based on the specific gravity and total protein con-
Ans.
centration, ascitic fluid has traditionally been classi-
l Accumulation of excess fluid in the peritoneal cavity is fied as transudative and exudative.
called ascites. l Serum-ascites albumin gradient (SAAG) is the indi-

cator to classifying ascites into portal hypertensive


and non-portal hypertensive causes.
Aetiology
The most common cause of ascites is cirrhosis with portal
hypertension. The other causes of ascites are as follows:
Management
l Cirrhosis with portal hypertension The treatment of ascites consists of:
l Congestive heart failure (CHF) l Salt in the diet is restricted to 2-3 gm/day.
Section | I General Medicine 31

l Diuretics are given, if there is no response to salt restric- l Fibrosis and distorted vasculature may lead to
tion. Spironolactone is initially used in a dose of 100 mg/ portal hypertension and complications associated
day which is increased to 400 mg/day. Frusemide (40- with it.
160 mg/day) is added in case the response is inadequate. l Patient may present with features of portal hypertension
l Therapeutic paracentesis is needed in massive ascites like abdominal distension due to ascites and spleno-
with respiratory compromise. megaly, haematemesis and melaena due to variceal rup-
l Portacaval shunt surgery or implantation of peritoneo- ture or hepatic encephalopathy.
venous shunt may be indicated in refractory ascites. l The liver is firm, nontender, and nodular and is enlarged
l TIPSS is an alternative to surgical shunting in cases initially. As the disease progresses, the liver size reduces
with refractory ascites. due to fibrosis.
l Hepatocellular dysfunction leads to jaundice, oedema,
Specific treatment is given to patients according to the
coagulopathy and metabolic abnormalities.
cause. Antitubercular therapy is indicated in tuberculous
l The signs of chronic hepatic dysfunction such as spider
ascites.
nevi, palmar erythema, gynaecomastia, testicular atro-
Q. 5. Hepatitis-A, its clinical features and brief phy and loss of hair may occur due to disturbances in
management. hormonal metabolism.
l Other features include enlargement of parotid gland and
Ans. lacrimal glands, digital clubbing, Dupuytren’s contrac-
l Hepatitis is an RNA virus of the picornavirus family. ture, and skin pigmentation.
The transmission is by the faeco-oral route.
l An epidemic can occur due to contamination of food Complications
and drinking water by HAV.
l Poor sanitation and over crowding facilitate its spread. Complications of cirrhosis of liver are as follows:
l Infected persons excrete the virus in their faeces about l Portal hypertension
two weeks before the onset of and during the first two l Ascites
weeks of illness. l Upper GI bleeding
l Rarely the HAV can also be transmitted by blood and l Spontaneous bacterial peritonitis (SBP)
via the sexual route. l Hepatic encephalopathy
l The incubation period of HAV infection is about 4 weeks. l Hepatorenal syndrome
l Infection and subsequent immunity occurs usually in l Hepatocellular carcinoma
childhood.
l Mortality is low and fulminant hepatitis is uncommon. Management
l There is no carrier state and chronic hepatitis does not occur.
l Anti-HAV of IgM type is useful for the diagnosis of It includes general management, treatment of specific
acute hepatitis A. cause, management of the complications and liver trans-
l Anti-HAV of IgG type appears later and indicates previ- plantation.
ous infection and immunity.
i. General management:
Q. 6. Cirrhosis of liver—Clinical features, complications l The diet should contain an adequate amount of protein
and management. and calories.
Ans. l Vitamin supplementation.
l Salt restriction is required in case of ascites.
l Cirrhosis of liver is the end result of the hepatocellular l Medications which are hepatotoxic or metabolized in
injury characterized by the presence of extensive fibro- liver should be given with caution.
sis, regenerative nodules and loss of liver architecture.
l Two of the commonest causes of cirrhosis are viral hepa- ii. Treatment of specific cause:
titis (B, C and D) and prolonged excessive use of alcohol. l Alcohol abstinence is mandatory in alcoholic
cirrhosis.
Specific therapy is needed in haemochromatosis and
Clinical Features l

Wilson’s disease.
l Cirrhotic patients may be asymptomatic. i. Management of specific complications
l Mostly symptoms such as weakness, fatigue, weight ii. Liver transplantation: The irreversible progressive
loss, anorexia, nausea, vomiting and abdominal discom- chronic liver failure due to cirrhosis is the most com-
fort occur insidiously. mon indication for liver transplantation.
32 Quick Review Series for BDS 3rd Year

Prognosis iii. Delta hepatitis:


Delta antigen - occasionally detectable
Overall the prognosis of cirrhosis is poor. The prognosis is
Anti-delta - initially of IgM type and later IgG type.
favourable, if the cause can be corrected.
Q. 7. Viral hepatitis—investigations? iv. Hepatitis C:
Ans. l Anti-HCV - appears after infection; disappears after
Investigations in cases of viral hepatitis are as follows: recovery; persists in chronic hepatitis C.
l During the early phase of hepatitis, there is significant l HCV RNA - remains detectable in most, continuously
(.400 units/L) increase in the plasma ALT and AST. or intermittently.
This is followed by the rise in serum bilirubin level.
Q. 8. Hepatitis-B prophylaxis.
l However, in anicteric hepatitis, the rise in ALT and AST
is not associated with any rise in bilirubin. Ans.
l High alkaline phosphatase level suggests cholestasis.
l Serum protein levels or serum albumin concentration is Hepatitis B prophylaxis is as follows:
normal. l Individuals are most infectious when markers of
l Prolongation in prothrombin time (PT) is a reliable in- continuing viral replication, such as HBeAg, and high
dicator of severe liver damage and correlates with the levels of HBV-DNA are present in the blood; they are
prognosis. least infectious when only anti-HBe is present with low
l The total leucocyte count is normal or low. There may levels of virus.
be relative lymphocytosis. l HBV-DNA can be found in saliva, urine, semen and
l Blood glucose level may be low. vaginal secretions.
l Anti-smooth muscle antibody, rheumatoid factor, l The virus is about ten times more infectious than hepa-
antinuclear antibody and heterophil antibody may be titis C, which in turn is about ten times more infectious
present in low titres during prodromal phase. than HIV.
l A recombinant hepatitis B vaccine containing HBsAg is
Serological tests for viral hepatitis: available (Engerix) and is capable of producing active
Serological tests (as mentioned above) are performed to immunisation in 95% of normal individuals.
identify the cause of the hepatitis. l The vaccine gives a high degree of protection and
should be offered to those at special risk of infection
i. Hepatitis A: who are not already immune, as evidenced by anti-HBs
l Anti-HAV of the IgM type early in the course in the blood.
l Anti-HAV of the IgG type later in the course l The vaccine is ineffective in those already infected by
HBV. Infection can also be prevented or minimised by
ii. Hepatitis B: the intramuscular injection of hyperimmune serum
l HBsAg - a reliable marker of infection globulin prepared from blood containing anti-HBs. This
l Anti-HBs - appears after disappearance of HBsAg and should be given within 24 hours, or at most a week, of
persists lifelong exposure to infected blood in circumstances likely to
l HBcAg - not found in the blood cause infection.
l Anti-HBc - persists lifelong. Initially it is of IgM type l Vaccine can be given together with hyperimmune
and later IgG type globulin. Neonates born to hepatitis B-infected mother
l HBeAg - detected transiently, early in the course. Its should be immunised at birth and given immunoglobu-
persistence is correlated with ongoing viral replication lin. Hepatitis B serology should then be checked at
l Anti-HBe - detected later during the course hepatitis 12 months of age.

SHORT NOTES
Q. 1. Hepatitis C. vertical spread is less common than in hepatitis B
infection.
Ans.
iii. Acute HCV infection is usually subclinical. Chronic
i. Hepatitis C virus (HCV) is an RNA virus. infection occurs in 70-80% of cases.
ii. The route of transmission is parenteral (transfusion of iv. Cirrhosis and hepatocellular carcinoma can also
blood and blood products, IV drug abuse). Sexual and occur. Extrahepatic manifestations like vasculitis,
Section | I General Medicine 33

arthritis, glomerulonephritis and cryoglobulinaemia mothers and in individuals who had sexual contact with
may occur. persons with HBV infection.
v. The most commonly used screening tests detect anti- l A dose of 0.06 ml/kg hepatitis B immune globulin
HCV. These antibodies generally appear late and thus (HBIG) is given intramuscularly as soon as possible.
identify chronic infection. Active immunization with vaccine is also initiated
vi. PCR test can detect HCV-RNA in the serum 1-2 weeks simultaneously, if the person is unvaccinated.
after infection and is used for the confirmation of diag-
Q. 4. Investigations in obstructive jaundice.
nosis and for the monitoring of therapy, prevention is
not possible. Ans.
Q. 2. Liver abscess. l The obstructive jaundice results from the obstruction in
the blood flow which may be intrahepatic or extrahepatic.
Ans.
i. Bacteria reach the liver either through portal vein bac- Investigations in obstructive jaundice are as follows:
teraemia, systemic bacteraemia, ascending cholangitis l Bilirubin present in urine while urobilinogen is absent
or penetrating trauma and cause liver abscess. in it.
ii. Common organisms involved are E. coli, anaerobic l Serum bilirubin markedly raised (predominantly
streptococci and bacteroides. conjugated hyperbilirubinaemia).
iii. Most have a subacute onset. l Serum alkaline phosphatase markedly raised (.3-4
iv. Clinical symptoms include fever with chills and rig- times normal)
ors, weight loss, anorexia, nausea and vomiting. l Minimal biochemical changes of parenchymal damage.
v. Pleuritic chest pain, tender hepatomegaly and mild l Ultrasonography (for underlying cause).
jaundice. l Antimitochondrial antibody (primary biliary cirrhosis).
vi. Single abscesses are more common in the right lobe of l Endoscopic retrograde cholangiopancreatography (ERCP).
the liver and multiple abscesses are seen in elderly l Percutaneous transhepatic cholangiography (PTC).
patients. l Liver biopsy (only if there is evidence of liver cell disease).
vii. Commence treatment with a combination of ampicil-
Q. 5. Ascites – name four causes.
lin, gentamicin and metronidazole. Later, change the
antibiotic according to the sensitivity reports. Ans.
viii. Ultrasound-guided aspiration of the abscess or surgi-
Ascites is defined as an accumulation of excess fluid within
cal drainage.
the peritoneal cavity.
Q. 3. Prevention of viral hepatitis B.
Causes of ascites are as follows:
Ans.
i. Normal peritoneum (transudates):
Viral hepatitis B can be prevented by:
l Cirrhosis with portal hypertension
i. Active immunization (pre-exposure prophylaxis): l Congestive heart failure (CHF)
l Hypoproteinaemia (nephrotic syndrome, malnutrition)
l Recombinant vaccines containing HBsAg are available
l Pancreatitis
l The standard regimen:
a. In adults is 20 mg IM in the deltoid region at 0, 1, ii. Diseased peritoneum (exudates):
and 6 months. For rapid immunity a schedule of 0, 1,
l Tuberculous peritonitis
2, and 12 months is followed.
l Bacterial peritonitis
b. Children need half the dose (10 mg) while
l Malignancy (peritoneal, hepatic)
c. Immunocompromized patients need 40 mg.
l The active immunization is particularly indicated in iii. Miscellaneous:
those at high risk of getting HBV infection. Many coun-
l Chylous ascites (filariasis, trauma, tumour)
tries have included HBV vaccination in their infant im-
l Budd-Chiari syndrome
munization programme.
Q. 6. Complications of cirrhosis of liver.
ii. Passive immunization (post-exposure prophylaxis):
Ans.
l This is given to nonvaccinated persons who have expo-
sure to HBV through mucous membrane and breaks in Complications of cirrhosis of liver are as follows:
the skin such as accidental needle stick injury. This is l Portal hypertension
also indicated for newborn infants of HBsAg positive l Ascites
34 Quick Review Series for BDS 3rd Year

l Upper GI bleeding Q. 10. Prevention of hepatitis A infection.


l Spontaneous bacterial peritonitis (SBP)
Ans.
l Hepatic encephalopathy
l Hepatorenal syndrome l The prophylaxes for HAV and HBV are available.
l Hepatocellular carcinoma l Improvement in sanitation and provision for safe
drinking water are helpful in preventing water-borne
Q. 7. Spider navei. infections like hepatitis A and E.
Ans. l Active immunization is available against HAV and
HBV.
l Synonyms of spider navei are spider telangiectasia, vas- l Hepatitis A vaccine (formalin-inactivated vaccine) is
cular spiders, spider angiomas or arterial spiders. given intramuscularly followed by a booster dose at
l They are thought to be due to arteriolar changes induced 6-12 months. The dose is 1440 ELU for adults and half
hyperoestrogenism. the dose for the children.
Usually seen in the territory drained by the superior l Passive immunization: Immune serum globulin is
vena cava, i.e. head, neck, upper limbs, front and back administered intramuscularly in the dosage of 0.02 ml/kg
of upper chest. to the contacts soon after the exposure.
l Vary in size from 1-2 mm to 1-2 cm in diameter.
l Spider nevi are dilated central arterioles with radiating Q. 11. Hepatomegaly – any three causes.
small vessels looking like spider and are found mainly Ans.
on the upper part of the body.
l Spider navei is one of the clinical features seen in cir- Important causes of hepatomegaly are as follows:
rhosis of liver and is one of the signs of chronic hepatic
Vascular causes:
dysfunction.
l CHF
Q. 8. Hepatocellular jaundice – three /four causes. l Hepatic vein thrombosis
Ans. Inflammatory causes:
The causes of hepatocellular jaundice are as follows: l Hepatitis (viral, drugs, alcohol)
l Viral hepatitis: A, B, C, D, E, EBV and CMV. l Cirrhosis (early stage)
l Alcoholic hepatitis l Liver abscess
l Chronic hepatitis
l Cirrhosis Neoplasm:
l Drug-induced hepatitis:Isoniazid, paracetamol, chlor- l Hepatoma
promazine, imipramine, INH, rifampicin, erythromycin, l Metastatic
amitriptyline, halothane, etc. l Lymphoma, leukaemia
l Metabolic: Wilson’s disease.
l Immune: Autoimmune hepatitis. Granulomatous causes:
l Tuberculosis
Q. 9. Obstructive jaundice.
l Sarcoidosis
Ans.
Infiltrative causes:
l Obstructive jaundice or cholestatic jaundice results
l Fatty liver (alcohol, diabetes, toxin)
from the obstruction in the bile flow which may be in-
l Non-alcoholic steatohepatitis (NASH)
trahepatic or extrahepatic.
l Amyloidosis
l Conjugated bilirubin enters into blood and appears in
l Hemochromatosis
urine. The stool becomes pale due to reduce, formation
l Storage disorders
of stercobilinogen in the intestine.
l Retention of bile salts leads to its deposition in the skin Q. 12. Tender hepatomegaly – two common causes.
and severe pruritus (itching). Patients usually have
Ans.
scratch marks on skin.
l Reduced availability of bile in the intestine may lead to l Tender hepatomegaly may occur in CHF, acute hepatitis,
malabsorption of fat and fat-soluble vitamins (vitamin and liver abscess.
K and vitamin D) which results in bleeding diasthesis l The liver is soft, smooth and tender in CHF, and firm,
and osteomalacia. regular in cirrhosis.
Section | I General Medicine 35

l The size of the liver is usually expressed in centimetres l Fluctuating neurologic signs.
below the right costal margin. The consistency of the liver l Asterixis or “flapping tremor”.
should be noted as soft, firm or hard. The surface may be l Distinctive electroencephalographic changes.
smooth or nodular and edges may be sharp, regular or l Hepatic encephalopathy is due to a biochemical distur-
irregular. Liver may be non-tender or tender to touch. bance of brain function resulting from various toxic
substances reaching the brain like ammonia, gamma-
Q. 13. Four causes of splenomegaly.
aminobutyric acid (GABA), short-chain fatty acids, etc.
Ans.
Q. 15. Portal hypertension – clinical features.
Classification and causes of splenomegaly are as follows:
Ans.
A. Mild splenomegaly: Portal hypertension is a condition characterised by pro-
i. Acute infection – enteric fever, infection hepatitis, longed elevation of portal venous pressure. Normal portal
infectious mononucleosis, brucellosis, and septicaemia. venous pressure is 10-15 cm saline or 7-10 mmHg.
ii. Subacute and chronic infections – tuberculosis, infec-
Clinical features of portal hypertension are as follows:
tive endocarditis, syphilis, brucellosis, and chronic bac-
l Caput medusae–a number of prominent collateral ves-
teraemia.
sels radiate from the umbilicus. Usually only or two
iii. Parasitic infestations – malaria, and kala-azar.
veins are seen, especially in the epigastrium. The flow
B. Moderate splenomegaly: of blood is always away from the umbilicus.
l In extrahepatic portal hypertension, prominent dilated
Lymphomas, portal hypertension, acute leukaemias, chronic
veins are in the left flank.
lymphocytic leukaemia, chronic myeloid leukaemia,
l A venous hum may be audible in the region of xiphoid
chronic haemolytic anaemias, malaria, and kala-azar.
process.
C. Massive splenomegaly: l Splenomegaly is the single most important diagnostic
sign of portal hypertension.
Chronic myeloid leukaemia, myelofibrosis, hairy cell leu-
l Liver size may be enlarged or shrunken.
kaemia, tropical splenomegaly, kala-azar, portal hyperten-
l Small, contracted, fibrotic liver is associated with very
sion, Gaucher’s disease, thalassaemia major, lymphomas,
high portal venous pressure.
cysts and tumours of spleen.
l Soft liver suggests extrahepatic portal vein obstruction.
Q. 14. Hepatic encephalopathy. l Firm liver suggests cirrhosis and hence intrahepatic
portal hypertension.
Ans.
l Ascites occurs partly due to portal hypertension and
Hepatic encephalopathy is a neuropsychiatric syndrome mainly due to liver cell failure.
characterised by the following: l Complications of portal hypertension are variceal
l Disturbances in consciousness and behaviour. bleeding, hepatic encephalopathy, ascites, renal failure,
l Personality changes. hypersplenism, etc.

Topic 4

Haematology

LONG ESSAYS
Q. 1. Classify anaemias. Discuss the clinical features, Describe the aetiological factors, clinical features and
diagnosis and treatment of iron deficiency anaemia. management of iron deficiency anaemia.
Or Ans.
36 Quick Review Series for BDS 3rd Year

Anaemia may be defined as a state in which the blood Diagnosis


haemoglobin level is below normal range for the person’s
Every case of anaemia should have the following investiga-
age and sex.
tions to detect degree and cause of anaemia:
l RBC count, packed cell volume, mean corpuscular
Classification of Anaemias volume and mean corpuscular haemoglobin concentra-
tion (MCHC). Total leucocyte count and differential
I. According to cause:
leucocyte count.
a. Blood loss: Acute or chronic post-haemorrhagic l Peripheral blood film for type of anaemia and shape of
anaemia. RBCs and presence of any abnormal cells.
b. Inadequate production of normal red cells or impaired l Blood platelets, clotting time and bleeding time are
red cell formation. decreased in haemolytic anaemia.
l Genetic disorders of haemoglobin synthesis, i.e. thal-
l Bone marrow examination is done when cause of anae-
assemic syndrome. mia requires further investigation especially to detect
l Acquired deficiency of substances essential for he-
type of erythropoiesis.
mopoiesis, i.e. drug-induced disorders or endocrine
deficiencies.
c. Excessive destruction of red cells (haemolytic anaemias) Treatment
l Intrinsic l Physical and mental rest.
l Extrinsic l Good nourishing diet with supplementation of foods
rich in iron.
II. According to morphology (size) of red cells
l Oral iron therapy: Ferrous sulfate 200 mg thrice a day
and their haemoglobin content :
in between meals. If any abdominal pain, nausea,
a. Normocytic vomiting or constipation, the salt is changed to ferrous
b. Microcytic gluconate or ferrous fumarate.
l Hypochromic microcytic l Parenteral iron therapy: Iron sorbitol citric acid complex
l Normochromic microcytic 1.5 mg/kg body weight is given as IM or iron dextran in
c. Macrocytic. 5% glucose is given by IV.
l Blood transfusion: Transfusion of packed red cells is
IRON DEFICIENCY ANAEMIA given.

The iron deficiency anaemia is commonest cause of anae- Q. 2. Describe the aetiological factors, clinical features
mia and is a microcytic hypochromic anaemia. and management of megaloblastic anaemia. Describe
the differential diagnosis of megaloblastic anaemia.
Ans.
Clinical Features
l Megaloblastic anaemia results from deficiency of
Symptoms
vitamin B12 or folic acid, or from disturbances in folic
l Tiredness acid metabolism. Megaloblasts are RBCs which are
l Weakness abnormally large in size and nucleated and are well
l Lethargy haemoglobinated.
l Loss of appetite l Macrocytes are erythrocytes with increased mean
l Breathlessness carpuscular volume and are derived from megaloblasts.
l Epigastric discomfort. Hence the term megaloblastic macrocytic anaemia
describes the outstanding feature of both the bone mar-
Signs row and the peripheral blood.
l Pallor
Angular stomatitis
l
Aetiology
l Flattening or spoon-shaped nails
l Paleness of tongue l Most megaloblastic macrocytic anaemias are due to a
l Glossitis deficiency of vitamin B12, folate or both.
l Hepatosplenomegaly l Both vitamin B12 and folate are essential for DNA
l Pulmmer-Vinson syndrome, i.e. dysphagia and cricoid synthesis. The deficiency of one or both results in a
webs. failure of DNA synthesis. This results in abnormal cell
Section | I General Medicine 37

proliferation. The abnormality in cell proliferation l Serum levels of both methylmalonic acid and homocys-
affects mainly haemopoietic tissues and gastrointesti- teine are increased in cobalamin deficiency while only
nal epithelial cells. homocysteine level is increased in folate deficiency.
l The morphological changes in the erythrocyte series in l Schilling test is performed to detect the malabsorption
bone marrow are described as megaloblastic. When of cobalamin. Autoantibodies against intrinsic factor
DNA synthesis is impaired, cell division is delayed, the and parietal cells are found in pernicious anaemia.
time between divisions increases, more cell growth oc-
curs and the cell becomes larger – megaloblasts. The Management
synthesis of haemoglobin is unimpaired. The resultant
mature erythrocytes derived from these megaloblasts Management includes:
are abnormally large and abnormal in shape but well a. Supportive therapy and
haemoglobinated. b. Specific therapy
l But, a large number of these megaloblastic erythoid
a. Supportive therapy:
cells are destroyed in the bone marrow. These result in
liberation of large amounts of lactate dehydrogenase l Blood transfusion should be given to significantly
which rises to higher levels in blood. symptomatic and severely anaemic patients. Adequate
l In the bone marrow, abnormal proliferation effecting precautions are to be taken to avoid circulatory over-
the granulocyte series results in gaint metamyelocytes, loading, especially in elderlly patients. Intravenous
and megakaryocyte series results in dysplastic mega- frusemide 20-40 mg may be given prior to transfusion.
karyocytes. l Treatment of infections
l Treatment of cardiac failure

Clinical Features b. Specific therapy:


l The clinical manifestations are mainly due to involve- l Treatment of underlying cause of vitamin B12 or folate
ment of haematological, gastrointestinal and nervous deficiency.
systems. l Packed red cell transfusion is needed in case of severe
l Patients are anaemic and they may also have mild jaun- anaemia with cardiac symptoms.
dice due to raised plasma unconjugated bilirubin. l Cobalamin deficiency: Parenteral therapy with intra-
l Significant gastrointestinal manifestations are spleno- muscular cyanocobalamin is preferred since deficiency
megaly, anorexia, weight loss, diarrhoea, and smooth is mostly due to malabsorption. The treatment begins
and beefy red tongue. Neurological manifestations with a dose of 1000 mg (1 mg) per week for 8 weeks
such as paraesthesia, ataxia, sensory-motor paraparesis, followed by 1000 mg each month. The treatment is life-
forgetfulness and psychosis are found in cobalamin long in case of pernicious anaemia.
deficiency. l Folate deficiency: The usual oral dose of folic acid is
1 mg per day. In cases of malabsorption, higher dosage
up to 5 mg daily may be needed.
Investigations
Q. 3. Mention causes of aplastic anaemia. Describe
l RBC count, reticulocyte count, total leucocyte count, its clinical features, diagnosis, complications and
haemoglobin are reduced. management.
l Platelet count is reduced (thrombocytopenia).
Ans.
l Indirect bilirubin is midly eleveted.
l Serum iron is elevated. Aplastic anaemia is defined as condition in which an
l Iron binding capacity is reduced. acelluar or markedly hypocelluar bone marrow results
l Serum ferritin level is increased. in pancytopenia, i.e. anaemia, granulocytopenia, and
l Plasma LHD is markedly increased. thrombocytopenia.
l Specific tests for B12/folate deficiency.
Peripheral blood smear reveals macrocytosis (MCV
Aetiology
l

.110 fL).
l Lactate dehydrogenase (LDH 1) and unconjugated i. Drug-induced aplasia
bilirubin are raised due to ineffective erythropoiesis. l Cytotoxic drugs – alkylating agents, antimetabolites,

l Bone marrow is hypercellular and reveals megaloblastic etc.


changes. Megaloblasts are abnormally large RBC pre- l Antibacterial drugs – chloramphenicol, sulphon-

cursors with nuclei less mature. amides, isoniazid, arsenicals.


38 Quick Review Series for BDS 3rd Year

lTranquilizers – chlorpromazine, meprobamate, l Severe anaemia is managed with packed red cell
chlordiazepoxide, promazine, thioridazine. transfusion while platelet concentrates are used to
l Antirheumatic drugs – oxyphenbutazone, phenylbu- maintain platelet count at or more than10, 000/ mL.
tazone, indometacin, gold salts, diclofenac, D-peni- l Aspirin and NSAIDs which inhibit platelet function

cillamine. should be avoided. Infections should aggressively


l Antidiadetic drugs – tolbutamide, chlorpropamide. dealt with broad-spectrum antibiotics and antifungal
l Miscellaneous drugs – chlorthiazide, mepacrine, hy- agents.
dralazine, acetozolamide, potassium perchlorate, l Granulocyte transfusion has also been used in over-

carbamazepine, carbimazole, etc. whelming infections. Aseptic precautions must be


ii. Chemicals: observed to prevent infections.
l Benzene, lindane, DDT. ii. Bone marrow transplantation:
iii. Viral infections: l Allogenic bone marrow transplantation (BMT) from

l Viral hepatitis, infectious mononucleosis, HIV, etc. HLA matched siblings.


iv. Miscellaneous: l It is curative and the preferred mode of therapy in

l Pancreatitis. Paroxysmal nocturnal haemoglobin- young patients (,40 years).


uria, radiation, etc. l This is limited by high cost, non-availability of

matched donors, significant morbidity and mortality


Clinical Features and availablility of facility is limited to few centres.
iii. Immunosuppressive therapy:
The common presentations are bleeding and symptoms of l Immunosuppressive agents like anti-thymocyte glob-
anaemia. ulin (ATG) or anti-lymphocyte globulin (ALG) along
l The excessive tendency to bleed is due to thrombocyto- with cyclosporine is the standard medical treatment
penia which may present as easy bruising, epistaxis, of choice for patients who cannot be given BMT.
gum bleeding, heavy menstrual flow and petechiae. l Anti-lymphocyte globulin(ALG) or anti-thymocyte
Intracranial and retinal haemorrhages may also occur. globuline(ATG) produces haemotological recovery
l Anaemia results from weekness, fatiguability, lassitude, in about 50% of cases.
dyspnoea, on exertion and pallor. iv. Other drugs:
l Granulocytopenia or neutropenia may predispose pa- l The role of anabolic steroids is not clear though
tients to develop infections like sore throat, oral and some patients may respond to them.
pharyngeal ulcers, fever with chills and sweating,
chronic skin infections, recurrent respiratory infections, Q. 4. Describe the aetiological factors, clinical features
pneumonia and septicaemia. and management of polycythemia vera.
l Jaundice, splenomegaly and lymphadenopathy are absent.
Ans.
Investigations Polycythemia vera is a clonal stem cell disorder characterised
by an increased number of red blood cells in the circulating
l Peripheral blood smear shows normocytic anaemia,
blood. It is one of the chronic myeloproliferative disorders.
decreased granulocyte and platelet count resulting in
leucopenia and thrombocytopenia. Haemoglobin is re-
duced, reticulocyte count is low. Aetiology
l Other investigations include viral markers, chromo-
l Unkown
somal studies, and tests for paroxysmal nocturnal hae-
l Any thing from hypersensitivity to interleukin-3 may
moglobinuria (PNH).
play a role.
l Bone marrow study reveals:
i. Markedly hypocellular or acellular marrow.
ii. Increase in fat cells. Clinical Features
iii. Bone marrow iron stores are usually increased.
l Ferrokinetic studies show delayed clearance of radioac- l It is more common in males and over the age of
tive iron from the blood and increased uptake by the liver. 40 years.
l Complaints related to increased and/or decreased cere-
bral perfussion include headache, dizziness, vertigo, a
Management sense of fullness in the head, rushing in the ear, visual
i. Supportive therapy: disturbances, tinnitus, syncope and even chorea.
l It includes prevention and treatment of infections and l Pruritus, particularly after bathing, is frequent and may
haemorrhage and red blood cell transfusion for anaemia. be disabling.
Section | I General Medicine 39

l The patients often have a high colour, suffused con- Classification


junctive, deep red palate, dusky red hands and retinal
l Acute lymphoblastic leukaemia
venous engorgement.
l Acute myeloid leukaemia
l Splenomegaly is very common, and symptoms related
l Chronic leukaemia
to it may be present, hepatomegaly occurs in 30% cases.
l Chronic lymphatic leukaemia
l Symptoms of peripheral vascular insufficiency, and
l Chronic myeloid leukaemia
thrombotic and haemorragic complications are seen.
l Incidence of peptic ulcer is five times higher in these
patients. Aetiology
l Bleeding manifestations like epistaxis, bleeding from
l Genetic factors
peptic ulcer intramuscular haemorrages and bruising.
l Environmental factors, i.e. atomic radiation and pollution
l Hyperuricaemia may result in formation of urate stones
l Ionizing radiations
and nephropathy.
l Retroviruses
l Chemical agents, i.e. alkylating agents, cytotoxic drugs
Management
l The aim of therapy is to maintain the haemoglobin Clinical Features
,14 g/dl in men and,12 g/dl in women. l Peak incidence in fourth and fifth decades.
l Repeated venesection is the treatment of choice. l The common presenting features are weakness, tired-
l Alkylating agents and phosphorus should be avioded as ness, weight loss and abdominal fullness.
they are leukaemogenic. l Anaemia and fever.
l If a cytotoxic agent has to be used, hydroxyurea is the l Some patients are asymptomatic at the time of diagnosis.
agent of choice. l Bone and joint pains and tenderness over the bones.
l Aspirin should not be used to prevent thrombosis. l Combination of pallor, petechiae or purpura is present.
l Asymptomatic hyperuricaemia does not require treat- l Hepatomegaly and splenomegaly are present.
ment unless chemotherapy is planned. l In blast phase, patient may exhibit bleeding, severe
l Generalised pruritus should initially be treated with infection and lymphadenopathy.
antihistamines. Psoralens with UV light in A range
(PUVA) may be tried.
Investigations
Q. 5. Classify leukaemias. Describe the aetiology, clini- l Normocytic normochromic anaemia
cal features, diagnostic tests and management of chronic l Leucocytosis: Total leucocyte count is markedly raised
myeloid leukaemia. . 20,000 /mL often exceeding 2,00,000/ mL.
Ans. l Thrombocytosis.
l Bone marrow studies: Hypercellular bone marrow with
l Leukaemia is defined as clone of malignant cells de- marked proliferation of all granulocyte elements.
rived from myeloid or lymphoid stem cells. l Cytogenetic studies: Diagnosis of CML is confirmed
l Chronic myeloid leukaemia (CML) is a type of myelo- by the demonstration of Philadelphia chromosome on
proliferative disorder. It occurs as a result of malignant cytogenetic analysis or the presence of bcr-abl fusion
transformation of pluripotent stem cells leading to ac- gene by molecular techniques.
cumulation of large number of immature leukocytes in
the blood.
Treatment
l The underlying chromosomal abnormality in CML is
the Philadelphia chromosome (short 22) which results l Allogeneic bone marrow transplantation: At present,
due to the reciprocal translocation between chromo- the only known curative method of treatment of CML is
somes 9 and 22. allogeneic bone marrow transplantation (Allo-BMT).
l Typically, the course of CML consists of three phases:
a. The chronic stable phase Chemotherapy
b. The accelerated phase l The drugs used are imatinib, busulphan, hydroxyurea.
c. The blast crisis phase l Imatinib mesylate is a recent drug recommended in
l The initial phase is the chronic phase which may last for chronic stable disease and produces complete cytoge-
2-3 years without treatment. This evolves into acceler- netic response in 50% of cases.
ated phase which finally transforms into a terminal blast l Other drugs used in chronic phase are hydroxyurea,
phase. The blast phase is like acute leukaemia. melphalan and busulphan.
40 Quick Review Series for BDS 3rd Year

l Alpha interferon: interferon alpha 3 to 9 million units ii. Haematological diseases:


SC 3 times a week. It can induce remission and main- l Aplastic anaemia, acute leukaemia, myelofibrosis

tain control in chronic stable phase. and megaloblastic anaemia


l Splenectomy: It is done to relieve symptoms resulting iii. Infections:
from massive size of spleen and in repeated splenic l Tuberculosis, typhoid, kala-azar, malaria, infectious

infarctions. mononucleosis, and HIV


v. Autoimmune:
Q. 6. Describe the aetiological factors of eosinophilia.
l Systemic lupus erythematosus, and Felty’s syndrome

Ans. vi. Congenital:


l Cyclic neutropenia
Eosinophilia is an absolute eosinophil count exceeding
500/ml. Clinical Features
The common causes of eosinophilia are as follows: l The manifestations depend on the severity and duration
l Helminthic infestations of neutropenia.
l Loeffler’s syndrome l Oral ulcers are usual in agranlocytosis.
l Tropical eosinophilia l Patient develops fever and infections mainly orophar-
l Allergic conditions ynx, perirectal area, sinuses, lungs, and skin.
Hay fever l A careful history of exposure to drugs, toxins duration
Asthma of illness should be asked.
Serum sickness l The presence of lymphadenopathy, and hepatospleno-
l Drugs megaly.
Sulphonamides
Aspirin Management
Nitrofurantoin
Pencillins l First the cause should be identified and removed.
Cephalosporins l Secondly, the infection is controlled and patient is put
l Collagen vascular diseases on isolation ward and barrier nursing is done.
Rheumatoid arthritis l Anti-bacterial drugs like penicillin 5 mega units IM or
Churg-Strauss syndrome IV 4 hourly or ciprofloxacin 500 mg IV 4 hourly is
l Malignancies started immediately. In addition, metrogyl 500 mg every
Hodgkin’s diseases 6 hourly to take care of infection.
Chronic myeloid leukaemia l In cases where toxaemia is severe, corticosteroids are
Solid organ cancers employed.
l Idiopathic hyper eosinophilic syndrome. l Granulocytes transfusion is given for about 5-7 days to
tide over crisis.
Q. 7. Describe in detail oral manifestations of haemato-
logical disorders. How would you treat a case of agranu- Q. 8. How will you investigate a case of bleeding diathe-
locytosis? ses. Mention implications of the bleeding diseases and
dental considerations.
Ans.
Ans.
l Agranulocytosis term is used to describe a state of severe l Bleeding disorders are due to one of the following
neutropenia or absence of circulating neutrophils. Usu- abnormalities:
ally neutrophil count is ,500/ml. a. Coagulation defects
b. Platelet disorders
Aetiology c. Vessel wall abnormalities
l Evaluation of bleeding disorders include:
Neutropenia can occur in conditions with: A. A detailed history,
a. Decreased production B. physical examination and
b. Increased destruction C. The initial screening tests.
c. Excessive peripheral pooling of neutrophils.
Based on the results of these, some additional complex
Important causes of neutropenia are as follows: investigations may be necessary in some cases.
i. Drug induced:
l Anti-cancer drugs, antibiotics (sulphonamides, A. Detailed History
chloramphenicol), anticonvulsants (carbamazapine), I. Coagulation defects
and anti-thyroid drugs l Bleeding into viscera, muscles and joints.
Section | I General Medicine 41

l Bleeding starts several hours after surgery or trauma Implications of bleeding disorders on dental practice:
l Local pressure ineffective l Elective oral surgical and periodontal procedures should
be avoided in patients with severe anaemia. Increased
i. Congenital defects bleeding and impaired wound healing may occur in
l Lifelong history presence of anaemia.
l Family history l Oral ulcers, advanced periodontal disease, pericoronitis
and pulpal infections can lead to life-threatening septi-
ii. Acquired defects caemia in patients with severe neutropenia. The patients
l Short duration should be given appropriate antibiotics and mouthrinses.
l Evidence of liver disease, renal failure, disseminated l Oral bleeding, oral ulcers, gingival infiltrates, oral in-
intravascular coagulation fections and cervical lymphadenopathy may be the
presenting features of leukaemias. Hence, the dentist
II. Platelet Disorders may be first clinician to suspect the disease.
l Spontaneous gingival bleeding is common when plate-
i. Bleeding into skin (purpura, petechiae), mucous mem- let count is below 20,000/mm3.
brane, GI system and central nervous system; epistaxis; l A history of any bleeding manifestations must be asked
prolonged bleeding from superficial cuts in patients undergoing dental procedures. In case of
ii. Bleeding immediately after surgery or trauma such history, investigations should be performed to
iii. Local pressure effective diagnose or exclude haemostatic disorders.
iv. History of drug intake, especially non-steroidal anti- l A history of anticoagulant therapy must be noted.
inflammatory drugs like aspirin Non-surgical treatment can be carried out provided
the PTINR is not grossly above the therapeutic range
III. Vessel Wall Abnormalities and trauma is minimal.
l Surgical treatment is not recommended for those who
i. Recurrent bleeding at a single site. have INR .3.5. At an INR ,3.5 where bleeding is
ii. Bleeding occurs immediately after trauma expected, local measures should be used along with
iii. Local pressure effective the reduction of INR to 2-3. Extensive flap surgery or
multiple bony extraction may require an INR of ,1.5.
B. Physical Examination
l The transfusion of blood and blood components should be
Physical examination should include a diligent search for done if absolutely necessary to avoid the risk of transfusion
the following: associated infections such as HIV, hepatitis B, hepatitis C,
l Purpura, bruises, ecchymoses. syphilis, malaria, cytomegalovirus, and parvovirus.
l Telangiectasis in hereditary haemorrhagic telangiec-
tasias. Q. 9. Write in detail clinical features, diagnosis, compli-
l Scars over elbows and knees in factor XIII deficiency. cations and management of haemophilia A.
l Examination of joints, particularly knees, ankles and Ans.
elbows for haemarthrosis.
Signs of liver cell disease.
l
Aetiology
l Hepatosplenomegaly and lymphadenopathy.
l Neurological signs. l Haemophilia A is an X-linked genetic coagulation disorder.
It results from a reduction of factor VIII (antihaemophilic
C. Screening Tests factor).
l Peripheral smear – platelets, leucocytes, red blood l Incidence of haemophilia A is 1 in 10,000 males.
cells l It affects males while females are carriers. A carrier
l Platelet count – number of platelets woman has a 50% chance of producing a haemophilic
l Bleeding time (N - ,9 minutes) – platelet function, von male or a female carrier.
Willebrand factor
Prothrombin time (N - 12 seconds) – extrinsic pathway,
l
Classification
factors V, VII, X
l Activated partial thromboplastin time (N - 33-45 l Normal factor VIII level in the blood is 0.50-1.50 IU/mL.
seconds) – intrinsic pathway, factors V, VIII, IX, X, XI, XII l Haemophilia A can be classified based on the factor
l Thrombin time (N - 3-5 seconds . control) – common VIII activity in blood.
pathway, factors I, II a. Severe–less than 1% of nomal activity
l Clot retraction – platelets b. Moderate–1-5% of nomal activity
l Fibrinogen concentration – fibrinogen c. Mild–more than 5% of normal activity
42 Quick Review Series for BDS 3rd Year

Clinical Features Replacement therapy


l Haemophilia A is characterized by excessive bleeding. l Replacement therapy is aimed at rapid correction of
Bleeding tendency may range from mild to severe. deficiency of factor VIII.
l When bleeding follows trauma, it is characteristically l Agents currently used for replacement therapy are cryo-
‘delayed’. precipitate and factor VIII concentrate intravenously.
l Bleeding episodes are treated with factor VIII concentrates
Bleeding into joints (haemarthroses) l IV desmopressin.
l Recurrent bleeding into large joints, especially knees,
Desmopressin
elbows, ankles, wrists and hips. Bleeding is usually
l Desmopressin (DDAVP; 1-desamino-8-D-arginine
spontaneous or may follow minor trauma.
vasopressin) can transiently raise factor VIII activities
l In the acute stage, the affected joint is swollen, hot,
by 3-5 times. It is given at a dose of 0.3 mcg/kg intrave-
tender, and movements severely restricted. All these
nously over 15 minutes.
gradually subside over a period of days.
l Indications are minor bleeding and minor surgery.
l Repeated haemarthroses eventually result in deformity,
crippling and disuse atrophy of muscles around the joint. Epsilon aminocaproic acid (EACA) and tranexamic acid
l When used concurrently with replacement therapy,
Bleeding into muscles these agents can reduce the factor VIII requirements.
l Muscle haematomas are common in calf and psoas l Commonly used for dental extractions. Regimen is
muscles. EACA taken orally in a dose of 5 g four times daily for
l Calf haematoma can result in contraction and shortening 7 days, starting on the day of extraction
of the Achilles tendon.
Indications of replacement therapy
Other bleeding manifestations l Early treatment of spontaneous bleeding episodes.
l Bleeding from wounds, bleeding from sockets after l Established severe or prolonged wound and tissue
dental extraction, easy bruising, retroperitoneal, mesen- bleeding.
teric and intra-abdominal bleeding. l Control of bleeding during and after surgery and trauma.
l Intracranial haemorrhage. l One unit of factor VIII is that amount present in 1 ml of
l Haematuria and ureteric colic due to passage of blood clots. normal plasma. Each unit of cryoprecipitate contains
about 100 units of factor VIII.
Investigations Complications of replacement therapy
l Bleeding time, prothrombin time and platelet counts l Hepatitis (hepatitis B virus, delta virus and hepatitis C
are normal. virus infection).
l Activated partial thromboplastin time (aPTT) is l Infection with human immunodeficiency virus (HIV).
typically prolonged. l Development of anti-factor VIII antibodies.
l Specific factor VIII assay can confirm the diagnosis. Q. 10. What is thrombocytopenia? Enumerate its causes
l Antenatal diagnosis: Chorion villus sampling (CVS) and complications. Write management of idiopathic
at 8-9 weeks gestation, sexing the fetus and using thrombocytopenic purpura.
informative factor VIII probes. Sexing the fetus at
16 weeks gestation by amniocentesis. Ans.
l Immune (idiopathic) thrombocytopenic purpura (ITP) is
Management defined as thrombocytopenia not associated with any
recognized cause.
Local treatment l It is an autoimmune disorder due to antibodies directed
l In case of wounds and mucous membrane bleeding, against platelet membrane glycoprotein resulting in
local pressure may be applied digitally with pressure the premature clearance of these cells by the immune
bandages or sutures. Topical haemostatics like adrena- system.
line and thrombin and immobilization of wounds by
bandages, splinting, etc.
Etiopathogenesis
l In case of haematomas and haemarthrosis, in the acute
stage, elevation of the affected part and immobilization l About 85-95% of cases of idiopathic thrombocytope-
by splinting and bandages should be used. nic purpura result from the presence of an anti-platelet
l Once the acute stage is over, patient should be mobilized antibody of IgG type. These antibody-bound platelets
and receive physiotherapy. are removed and prematurely destroyed by the spleen.
Section | I General Medicine 43

Clinical Features l Adults – initial dose is 1-2 mg/kg of prednisolone per


day.
l There are two forms of idiopathic thrombocytopenic pur-
l Children – initial dose is 0.5-1 mg/kg of prednisolone
pura, an acute form which is more common in children,
per day.
and a chronic form which is more common in adults.
l This initial dose is continued for at least two weeks, or
l Characteristically, the patient has no physical signs
if necessary 3-4 weeks, and then reduced slowly and
other than those due to anaemia and bleeding.
stopped.
l Bleeding from skin or mucosal membranes.
l Purpura, bruises, epistaxis and nasal or oral bleeds. Splenectomy
l Splenomegaly is characteristically absent in most patients,
Indications of splenectomy are as follows:
in about 10% of cases tip of spleen may be palpable.
l Chronic cases, especially in adults who have not
responded to corticosteroids.
Diagnosis l Unacceptably high doses of corticosteroids in maintain-
ing remission.
l Positive tourniquet test (Hess test).
l Emergency measure in children and adults, when there
l Thrombocytopenia.
is severe bleeding or threatening cerebral haemorrhage
l Prolonged bleeding time.
(despite adequate corticosteroid therapy).
l Bone marrow examination characteristically shows nor-
l In the first 4-5 months of pregnancy, if steroids have
mal or increased number and size of megakaryocytes.
failed to induce full remission.
l Antiplatelet antibodies may be demonstrated in blood.
l Splenectomy brings down the concentration of circulating
l Normal coagulation profile.
anti-platelet antibodies.
l Abnormal long clot retraction time.
l Immunosuppressive therapy:
l Agents used are vincristine, vinblastine, azathioprine
Treatment and cyclophosphamide.
l Immunosuppressive therapy is indicated in refractory
l In acute and mild cases, no specific therapy will be necessary.
cases.
l The main therapeutic measures are corticosteroids and
splenectomy. Anti-RhD globulin
l Immunosuppressive therapy, intravenous immunoglobu-
lin and anti-RhD globulin are indicated in selected cases. l Anti-RhD globulin is also effective in patients with ITP.
It should be given only if the patient is Rh-positive. It
Corticosteroids may be useful in patients who do not respond to steroids
and may be tried before splenectomy. The effect is usu-
Indications are as follows:
ally temporary and nearly 50-75% patients relapse over
l To induce remission. Postoperatively, in failed splenec-
variable period.
tomy cases.
l Pregnant women after the fifth month of pregnancy. Androgens
l To maintain remission in chronic idiopathic thrombocy-
topenic purpura. l Danazol, an androgen with low virilizing activity has
l Dosage of corticosteroids: been tried in idiopathic thrombocytopenic purpura.

SHORT ESSAYS
Q. 1. Eosinophilia – causes. l Loeffler’s syndrome
l Tropiocal eosinophilla
or l Allergic conditions:
Tropical eosinophilia. Hay fever
Asthma
Serum sickness
Ans.
l Drugs:
Eosinophilla is an absolute eosinophils count exceeding Sulphonamides
500/ml. Aspirin
The common causes of eosinophilia are as follows: Nitrofurantoin
l Helminthic infestations Pencillins
44 Quick Review Series for BDS 3rd Year

Cephalosporins Q. 4. Iron deficiency anaemia – investigations and treatment.


l Collagen vascular diseases:
Ans.
Rheumatoid arthritis
Churg-Strauss syndrome l The iron deficiency anaemia is the commonest cause of
l Malignancies: anaemia and is a microcytic hypochromic anaemia.
Hodgkin’s diseases
Chronic myeloid leukaemia
Investigations
Solid organ cancers
l Idiopathic hypereosinophilic syndrome l RBC count, packed cell volume, mean corpuscular vol-
ume and mean corpuscular haemoglobin concentration
Q. 2. Haemophilia A.
(MCHC). Total leucocyte count and differential leuco-
Ans. cyte count.
l Peripheral blood film for type of anaemia and shape of
l Haemopilia A is due to a deficiency of factor VIII.
RBCs and presence of any abnormal cells.
l It is an X-linked recessive disorder presenting exclu-
l Blood platelets, clotting time and bleeding time are
sively in males.
decreased in haemolytic anaemia.
l Bone marrow examination is done when cause of anae-
Clinical Features mia requires further investigation especially to detect
type of erythropoiesis.
l Recurrent bleeding episodes into the muscles, brain,
and more commonly the joints causing haemarthrosis.
l Severity varies depending on the factor levels, from Treatment
traumatic to spontaneous bleeds.
l Physical and mental rest.
l Good nourishing diet with supplementation of foods
Investigations rich in iron.
l Oral iron therapy: Ferrous sulfate 200 mg thrice a day
l Reveal a subnormal factor VIII level in the blood.
in between meals. If any abdominal pain, nausea,
vomiting or constipation, the salt is changed to ferrous
Treatment gluconate or ferrous fumarate.
l Parenteral iron therapy: Iron sorbitol citric acid complex
l Bleeding episodes are treated with factor VIII
1.5 mg/kg body weight is given as IM or iron dextran in
concentrates
5% glucose is given by IV.
l IV desmopressin.
l Blood transfusion: Transfusion of packed red cells is
l Avoid IM injections
given.
Q. 3. Christmas disease.
Q. 5. Mention the signs of iron deficiency anaemia.
Ans.
Ans.
l Christmas disease is an X-linked disorder resulting
from deficiency of factor-IX. The iron deficiency anaemia is commonest cause of anae-
l Clinical features are indistinguisable from haemo- mia and is a microcytic hypochromic anaemia.
pilia A.
l Patients with severe disease present with muscle haema- Clinical Features
tomas and haemarthroses progressing to crippling joint
deformities Symptoms
l Diagnosis is established by factor IX assay. l Tiredness
l Management is similar to that of haemopilia A. l Weakness
l Replacement therapy: l Lethargy
a. Fresh frozen plasma is used to treat mild to moderate l Loss of appetite
bleeding. l Breathlessness
b. Factor IX concentrate (prothrombin complex concen- l Epigastric discomfort
trates) is used to treat moderate to severe bleeding.
c. Presently available factor IX concentrates are a Signs
mixture of factors II, VII, IX and X, and protein l Pallor
C and S. l Angular stomatitis
Section | I General Medicine 45

l Flattening or spoon-shaped nails ii. Anaemia results from weakness, fatiguability, lassi-
l Paleness of tongue tude, dyspnoea, on exertion and pallor.
l Glossitis iii. Granulocytopenia [neutropenia] results in various infec-
l Hepatosplenomegaly tions, sore throat, oral and pharyngeal ulcer, fever with
l Pulmmer-Vinson syndrome, i.e. dysphagia and cricoid chills and sweating, chronic skin infections, recurrent
webs. respiratory infections, pneumonia and septicaemia.
iv. Thrombocytopenia results in various bleeding manifes-
Q. 6. Plummer-Vinson syndrome.
tations, when the platelet count is less than 20,000 per
Ans. microlitre. These include bleeding in skin [ecchymosis,
petechiae], epistaxis, menorrhagia, bleeding from
Plummer-Vinson syndrome [sideropenic dysphagia, Patter-
gums, gastrointestinal tract, retinal haemorrhage and
Kelly syndrome]
cerebral haemorrhage.
l Iron deficiency is most common cause of anaemia.
v. Physical examination characteristically shows that jaun-
dice, splenomegaly and lymphadenopathy are absent.
Aetiology [Causes of Iron Deficiency]
Q. 8. Megaloblastic anaemia – aetiology.
l Physiological – postnatal growth spurts, adolescent
Ans.
growth spurts, mensturation, pregnancy
l Iron loss due to bleeding – hookworm infestation, schis-
tosomiasis, menorrhagia, postpartum haemorrhage, Aetiology of megaloblastic anaemia:
pepetic ulcer, piles, neoplastic diseases, erosion from l Most megaloblastic macrocytic anaemias are due to a
anti-inflammatory drugs. deficiency of vitamin B12, folate or both.
l Inadequate diet l Both vitamin B12 and folate are essential for DNA
l Malabsorption – inflammatory bowel diseases, postgas- synthesis. The deficiency of one or both results in a
ectomy failure of DNA synthesis. This results in abnormal cell
proliferation. The abnormality in cell proliferation ef-
Clinical Features fects mainly haemopoietic tissues and gastrointestinal
epithelial cells.
l Symptoms: Tiredness, weakness, lethargy, loss of ap- l The morphological changes in the erythrocyte series in
petite, breathlessness and epigastric discomfort. bone marrow are described as megaloblastic. When
l Signs: Pallor, angular stomatitis, flattening or spoon- DNA synthesis is impaired, cell division is delayed, the
shaped nails, tongue is pale, glossitis, hepatosplenomegaly. time between divisions increases, more cell growth oc-
l Characteristic feature of iron deficiency includes angu- curs and the cell becomes larger – megaloblasts. The
lar stomatitis, glossitis, brittle finger nails, platynachia, synthesis of haemoglobin is unimpaired. The resultant
and kolionychia. mature erythrocytes derived from these megaloblasts
are abnormally large and abnormal in shape but well
Management haemoglobinated.
l But, a large number of these megaloblastic erythoid
l Treatment of underlying cause cells are destroyed in the bone marrow. These results in
l Treatment of iron deficiency anaemia by oral iron therapy liberation of large amounts of lactate dehydrogenase
or parentral iron therapy. which rises to higher levels in blood.
l In the bone marrow, abnormal proliferation affecting the
Q. 7. Aplastic anaemia – clinical features.
granulocyte series results in gaint metamyelocytes, and
Ans. megakaryocyte series results in dysplstic megakaryocytes.
Aplastic anaemia is defined as condition in which an acelluar Q. 10. Leukaemia – clinical features and investigations.
or markedly hypocelluar bone marrow results in pancytope-
Ans.
nia [anaemia, granulocytopenia, thrombocytopenia]. There
are no leukaemic or abnormal cells in marrow or blood. Leukaemia is defined as clone of malignant cells derived
from myeloid or lymphoid stem cells.
Clinical Features
Clinical Features
i. Aplastic anaemia results from the complications of
pancytopenia [anaemia, granulocytopenia, thrombocy- l Anaemia
topenia]. l Fatigue and lethargy
46 Quick Review Series for BDS 3rd Year

l Fever Clinical Features


l Bone and joint pain
l Combination of pallor, petechiae or purpura is present. l Affects adults between 25 and 60 yrs of age
l Mucous membrane is bleeding. l Tiredness
l Hepatomegaly, splenomegaly and renomegaly are present. l Lethargy
l The tenderness over other bones. l Weight loss
l Abdominal pain and fullness
l Massive splenomegaly
Investigations l Pallor
l Bone marrow picture: Hypercellular reaction is present
with premature and primitive cells. Treatment
l Biochemical changes and presence of Philadelphia l Allogenic bone marrow transplant
chromosome. l Chemotherapy with busulphan, hydroxyurea.
l Clinical findings like loss of appetite, low grade fever, l Interferons.
night sweat, breathlessness on exertion, discomfort in
abdomen, anaemia, pallor and body tenderness. Q. 13. Complications of blood transfusion.
l Peripheral blood film shows normocytic, normochromic Ans.
picture with abundance of neutrophills, myelocytes,
metamyelocytes; white cell counts are elevated. Complications of blood transfusion can be grouped into
two as follows:
Q. 11. Clinical features and management of acute l Immunologically mediated reactions
myeloid leukaemia. l Nonimmune trasfusion reactions.
Ans. i. Immunologically mediated reactions
Acute myeloid leukaemia is a malignant neoplasm of the l Intravascular haemolysis – e.g. ABO incompatibility
haemopoietic stem cell of the myeloid series, characterized l Extravascular haemolysis – e.g. caused by antibodies of
by the presence of immature cells called blasts. Rh system

ii. Non-immune transfusion reactions


Clinical Features l Circulatory overload especially in patients with renal
l Common in the young and middle aged. and cardiac failure.
l Abrupt onset of easy fatiguability, bleeding tendencies, l Adverse effects of massive trasfusion – e.g. hyperkalae-
proneness to repeated infections. mia, ammonia and citrate toxicity, and thrombocytopenia.
l Bony pain due to the infiltration of the marrow. l Infections – e.g. viral hepatitis, HIV, sypilis, malaria,
l Central nervous system symptoms due to malignant and CMV infections.
infiltration like headache, visual blurring. l Metabolic shock
l Hepatosplenomegaly may be present. l Air and fat embolism
l Thrombophlebitis
l Haemosiderosis
Treatment l Pyrogenic reactions
l Chemotherapy with drugs like daunorubicin; cytosine Q. 14. Idiopathic thrombocytopenic purpura (ITP)–
arabinoside, all trans retinoic acid investigations and treatment.
l Allogenic BMT
l Supportive therapy with blood transfusion, antibiotics. Ans.
l Immune (idiopathic) thrombocytopenic purpura is
Q. 12. Clinical features and management of chronic
an autoimmune disorder due to antibodies directed
myeloid leukaemia.
against platelet membrane glycoprotein resulting in
Ans. the premature clearance of these cells by the immune
system.
l Chronic myeloid leukaemia is a malignant neoplastic
transformation of mature WBCs with a relatively indolent
course.
Clinical Features
l 90% have a positive Philadelphia chromosome, i.e. a l Bleeding from skin or mucosal membranes.
9:22 translocation. l Purpura, bruises, epistaxis and nasal or oral bleeds.
Section | I General Medicine 47

Investigations Investigations
l Low platelet counts l Blood film – polychromasia, spherocytes, sickle cells.
l Prolonged bleeding times l Elevated reticulocyte count
l Normal coagulation profile l Coomb’s test – positive in autoimmune haemolytic
l Abnormal long clot retraction time anaemias.
l Liver function tests – showing elevated unconjugated
bilirubin levels.
Treatment
l Platelet transfusion Treatment
l Corticosteroids.
l Splenectomy
Q. 15. Clinical features of haemolytic anaemia. l Blood transfusion
l Bone marrow transplantation
Ans.
Q. 16. Clinical features and complications of thrombo-
l Haemolytic anaemia occurs due to decreased RBC
cytopenia.
life-span to less than 120 days.
Ans.
Causes l Thrombocytopenia is defined as the decrease in the
number of the circulating platelets below 1,00,000 per
i. Congenital microlitre.
l Hereditary spherocytosis
Thalassaemia
Clinical Features
l

l Sickle cell anaemia


l G6PD deficiency. l Bleeding from skin or mucosal membranes.
l Purpura, bruises, epistaxis and nasal or oral bleeds.
ii. Acquired
l Autoimmune Investigations
l Infections like malaria
l Low platelet counts
iii. Drugs l Prolonged bleeding times
l Normal coagulation profile
l Abnormal long clot retraction time
Clinical Features
l Features of anaemia like fatigue, anorexia, breathlessness Treatment
l Jaundice l Platelet transfusion
l Splenomegaly l Treatment of the underlying cause

SHORT NOTES
Q. 1. Bleeding time. Q. 2. Purpura.
Ans.
Ans.
The purpura is of following types:
i. The time taken for the arrest of bleeding is known as i. Non-thrombocytopenic purpura: Is due to defect of cap-
bleeding time. The arrest of bleeding is due to the illaries; e.g. increased permeability, fragility.
formation platelet plug. ii. Thrombocytopenic purpura: Is due to decreased number
ii. Bleeding time normally ranges from 2 to 5 minutes. of platelets;
iii. This can be determined by pricking the ear lobe or the l Primary thrombocytopenic purpura also known as

finger tip Werlhoff’s disease, is an autoimmune disease; anti-


iv. Bleeding time is prolonged in purpura due to platelet platelet globulin/antibodies to platelets in blood are
deficiency. present.
48 Quick Review Series for BDS 3rd Year

l Spleen is not palpable. l The common causes of eosinophilia are as follows:


l Severe and profuse gingival bleeding, so any oral l Metazoan parasite infections

surgery is contraindicated untill controlled. l Loeffler’s syndrome

l platelets are , 60,000 / mm3 l Atopy and allergic conditions like hay fever, asthma

l Increased bleeding time: 1ve tourniquet / Hess’ test and serum sickness.
l Cessation of bleeding depends on physical blockade l Drugs: Sulphonamides, aspirin, nitrofurantoin, pen-

of severed capillaries by platelets which depends on cillins, etc.


number of platelets. l Sarcoidosis

l Treatment is splenectomy. l Malignancies (lymphomas and leukaemias): Hodg-

kin’s disease, chronic myeloid leukaemia and solid


Q. 3. Haemophilia A.
organ cancers.
Ans.
Q. 6. Three causes of neutropenia.
l Haemophilia A is the most common type and is due to
Ans.
a deficiency of factor VIII.
l It is an X-linked recessive disorder presenting exclu- Neutropenia is defined as the absolute neutrophil count
sively in males. ,2, SpO/ml
Causes of neutropenia are as follows:
Clinical Features i. Viral infections – hepatitis, HIV, influenza
ii. Bacterial infection – fulminant sepsis, typhoid, miliary TB
l Recurrent bleeding episodes into the muscles, brain,
iii. Protozoal diseases – malaria, kala-azar
and more commonly the joints causing haemarthrosis.
iv. Drugs – antithyroid, antimalarials, antibioitc drugs
l Severity varies depending on the factor levels, from
v. Autoimmune – connective tissue disease
traumatic to spontaneous bleeds.
vi. Alcohol
vii. Congenital – Kostmann’s syndrome
Investigations viii. Bone marrow infiltration – leukaemia, myleodysplasia.
Reveal a subnormal factor VIII level in the blood. Q. 7. Aplastic anaemia – Aetiology.
Ans.
Treatment
i. Aplastic anaemia is defined as condition in which an
l Factor VIII concentrates acelluar or markedly hypocelluar bone marrow results
l IV desmopressin. in pancytopenia, i.e. anaemia, granulocytopenia, and
l Avoid IM injections thrombocytopenia.
Q. 4. Importance of haemophilia to dental surgeon.
Ans. Aetiology
l Dental procedures and surgeries can be safely per- i. Drug-induced aplasia
formed in haemophiliacs provided the patient does not l Cytotoxic drugs – alkylating agents, antimetabolites, etc.
have an inhibitor to factor VIII and receives appropriate l Antibacterial drugs – chloramphenicol, sulphonamides,
amount of concentrate. isoniazid, arsenicals.
l A single infusion of factor VIII is adequate for simple l Tranquilizers – chlorpromazine, meprobamate, chlordi-
dental extractions. In an individual with severe haemo- azepoxide, promazine, thioridazine.
philia along with this, a 10-day course of tranexamic l Antirheumatic drugs – oxyphenbutazone, phenylbutazone,
acid and an antibiotic is required. indomethacin, goldsalts, diclofenac, D-penicillamine.
l In major surgeries, twice daily therapy for 14 days or l Antidiadetic drugs – tolbutamide, chlorpropamide.
longer is required. l Miscellaneous drugs – chlorthiazide, mepacrine, hy-
Q. 5. Eosinophilia – causes. dralazine, acetozolamide, potassium perchlorate, carba-
mazepine, carbimazole, etc.
or
Tropical eosinophilia. ii. Chemicals
l Benzene, lindane, DDT.
Ans.
l Eosinophilia is an abnormal eosinophils count exceeding iii. Viral infections
500/ml. l Viral hepatitis, infectious mononucleosis, HIV, etc.
Section | I General Medicine 49

iv. Miscellaneous Q. 12. Thalassaemia.


l Pancreatitis, paroxysmal nocturnal haemoglobinuria,
Ans.
radiation, etc.
i. Thalassaemia is also known as Cooley’s anaemia or
Q. 8. Four causes of anaemia.
erythroblastic anaemia.
Ans. ii. The disease usually occurs below 2 yeas of age;
siblings are commonly affected.
Anaemia may be defined as a state in which the blood hemo-
a. Thalassaemia minor or thalassaemia trait: It is het-
globin level is below normal range for the person’s age and sex.
erozygous form of thalassaemia.
Causes of anaemia are as follows: b. Thalassaemia major: Homozygous form of thal-
l Decreased or ineffective marrow production assaemia, severe form.
l Inadequate iron, B12, folate, trace elements. ● C/F: Mongoloid face, due to prominent cheekbones,
l Hypoplasia of the marrow depression of the bridge of the nose, unusual promi-
l Peripheral causes (increased RBC destruction or nence of the premaxilla.
blood loss) ● O/F: Prominent premaxilla.
l Blood loss ● R/G: Crew-cut or hair-on end appearance of skull
l Haemolysis surface and IOPA reveals salt and pepper effect in
l Hypersplenism jaws due to mild osteoporosis; thinning of lamina
dura
Q. 9. Clinical features of iron deficiency anaemia.
iii. Lab findings: Hypochromic microcytic anaemia con-
Ans. tains HbF (fetal), presence of safety pin cells, target
cells and normoblasts, i.e. nucleated RBCs are charac-
The iron deficiency anaemia is commonest cause of anae-
teristically seen.
mia and is a microcytic hypochromic anaemia.
The serum bilirubin is elevated, indicating severe
The clinical features are as follows: haemosiderosis.
l Easy fatiguability Bone marrow aspiration reveals large immature RBC,
l Breathlessness indicating maturation arrest.
l Anorexia
Q. 13. Folic acid.
l Knuckle and oral cavity pigmentation
l Neurological symptoms – tingling and numbness of Ans.
both lower limbs
l Folic acid is a parent compound of a group of derivatives,
Q. 10. Iron deficiency anaemia – four common causes. referred to as folates. Folic acid does not exist as much in
nature, but is available as a medicinal compound.
Ans.
l Body obtains folates from the polyglutamates of food.
The common causes of iron deficiency anaemia are as Absorption is mainly from jejunum.
follows: l Folate is obtained from both vegetables and animals
l Increased iron utilization foodstuffs. Much of it is destroyed by cooking.
l Physiological conditions like mensturation, pregnancy l Folate is mainly stored from liver. The stores are ex-
l Pure vegans, alcoholics hausted in about 3 months and hence the manifestations
l Diseases of the terminal ileum of folate deficiency appear in about 3 months.
l Pernicious anaemia l Normal dietary requirement of folate is 100-200 mcg.
l Fish tapeworm infestation
Q. 14. Leukaemias – causes.
l Drugs – anticancer drugs
Ans.
Q. 11. Polycythemia.
l Leukaemia is defined as clone of malignant cells de-
Ans.
rived from myeloid or lymphoid stem cells.
i. Polycythemia is also known as Osler’s disease or l Aetiology:
Vaquez disease or poly rubra vera i. Genetic factors such as familial, identical twins,
ii. C/F: Skin blushing due to capillary engorgements, sple- ii. Congenital
nomegaly; cyanosis of digits/tongue/gingiva due to iii. Environmental factors, i.e. atomic radiation and
amount of reduced, Hb.5gm%. pollution
iii. Treatment: Phenylhydrazine for destruction of blood iv. Ionizing radiations Retroviruses
N2-mustard, P 32, X-rays to interfere with blood for- v. Chemical agents, i.e. alkylating agents, cytotoxic
mation drugs.
50 Quick Review Series for BDS 3rd Year

Q. 15. Chronic myeloid leukaemia. Aetiology


Ans. Important causes of neutropenia are as follows:
i. Drug-induced: Anti-cancer drugs, antibiotics (sulphon-
i. Chronic myeloid leukaemia is a clonal disorder of amides, chloramphenicol), anticonvulsants (carbam-
pleuripotent stem cell. azapine), and anti-thyroid drugs
ii. The most important characteristic feature of chronic ii. Haematological diseases: Aplastic anaemia, acute leu-
myeloid leukaemia is demonstration of Philadelphia kaemia, myelofibrosis and megaloblastic anaemia
chromosome in leukemic blast cells. iii. Infections: Tuberculosis, typhoid, kala-azar, malaria,
iii. Common features: Loss of weight, fatigue, malaise, infectious mononucleosis, and HIV
excessive perspiration, splenomegaly, excessive men- iv. Autoimmune: Systemic lupus erythematosus, and Felty’s
strual or other bleeding, anaemia, bone pain. syndrome
iv. Rare features: Splenic infarction, leucostasis, gout, reti- v. Congenital: Cyclic neutropenia
nal haemorrhage and fever.
v. Aetiology: Chronic myeloid leukaemia results from Q. 19. Causes of massive splenomegaly.
translocation of genetic material between chromosome Ans.
9 and 22. The translocation result in production of abnor-
i. The spleen is a reticuloendothelial organ that synthesizes
mal tyrosine kinase that makes affected cell immortal.
antibodies, and removes senescent RBCs and antibody
vi. Treatment: Interferon AHa 3 to 9 million units SC
coated bacteria and blood cells from the circulation.
3 times a week. Hydroxyurea 2 gm daily orally or
ii. Causes of splenomegaly are as follows:
busulphan 4 gm daily orally.
l Haemolytic anaemias, e.g. thalassaemia, spherocytosis.

Q. 16. Erythroblastosis fetalis. l Infections, e.g. malaria, kala-azar

l Collagen vascular diseases, e.g. rheumatoid arthritis,


Ans. SLE.
l Congestion, e.g. portal hypertension
i. Erythroblastosis fetalis is a congenital hemolytic anae-
l Malignant infiltrations, e.g. leukaemias, Hodgkin’s
mia due to Rh incompatibility
disease.
ii. Aetiology: Strongest Rh-antigen is D-antigen; occurs
when father is Rh 1ve and mother is Rh -ve, so kid is Q. 20. Name few haematological disorders.
Rh -ve, so anti-bodies are produced. First pregnancy is Ans.
uneventful.
iii. C/F : 1ve direct Coombs’ test on cord blood. l Immune thrombocytic purpura
iv. Oral findings: Green/ blue / brown discolour of teeth l Coagulation disorders like haemopilia A, B, C, etc
due to bilirubin; it affects only deciduous teeth; stains l Von Willebrand diseases
are intrinsic; l Thrombocytopenia
v. Enamel hypoplasia: A ring like defect and Rh-hump l Neutropenia and agranulocytosis
occurs. Q. 21. Hazards of blood transfusion.

Q. 17. Von Willebrand’s disease. Ans.


Hazards of incompatible transfusion are as follows:
Ans. l Agglutination
i. Von Willebrand’s disease is also known as pseudohe- l Haemolysis
mophilia. l Fever and chills
ii. Only BT is prolonged-excessive bleeding l Jaundice, due to increased bile pigments resulting form
iii. Autosomal dominant; more in females RBC destruction.
iv. Classic haemophilia is caused by a deficiendy of one of l Renal failure
the 3 components of factor VIII, but classic Von Wille- l Uraemia, coma and death
brand’s disease is due to decrease in all the 3 components Q. 22. ESR.
of factor VIII.
Ans.
Q. 18. Agranulocytosis – causes. i. If a sample of blood mixed with an anticoagulant is al-
Ans. lowed to stand in a vertical tube, the RBC settles down due
to gravity (denser, specific gravity –1.095) as compared to
l Agranulocytosis term is used to describe a state of severe plasma (specific gravity –1.032) with a clear supernatant
neutropenia or absence of circulating neutrophils. Usu- layer of plasma. The rate at which the cells settle down is
ally neutrophil count is ,500 /ml. called erythrocyte sedimentation rate (ESR).
Section | I General Medicine 51

ii. There are two standard methods of estimation of Q. 23. Peripheral blood smear in iron deficiency anaemia
ESR:
Ans.
a. Wintrobe’s and Landsberg’s method
b. Westergren’s method Peripheral blood picture in iron deficiency anaemia exhibits
iii. The conditions where ESR is markedly raised are as following features:
follows: i. Chromicity: Hypochromia of RBC central pallor
l Infective: Tuperculosis, kala-azar, in most of the increased. Anisochromia present
chronic infections. ii. Size: Microcytic anisocytotic
l Inflammation: Rheumatoid arthritis, rheumatic fever, iii. Shape: Poikilocytosis often present, pear-shaped tailed
other connective tissue disorders. variety of RBC, elliptical form common.
l Neoplastic: Multiple myeloma, lymphoma, parapro- iv. Target cell: Present
teinaemias. v. Reticulocytes: Present, either normal or reduced
l Miscellaneous: Aplastic anaemia, autoimmune dis- vi. Osmotic fragility: Slightly decreased
orders, mixed connective tissue disorders. vii. Absolute values of MCV, MCH, MCHC are reduced.

Topic 5

Diseases of the Cardiovascular System


LONG ESSAYS
Q. 1. Describe diagnosis, investigations and manage- Classification
ment of hypertension.
I. Primary or essential hypertension
Or II. Secondary hypertension
Describe clinical features, classifications, complications Classification of blood pressure for adults:
and management of systemic hypertension.
Systolic BP Diastolic BP
Or
Category (mmHg) (mmHg)
Enumerate the pathogenesis, complications and man- Normal ,120 ,80
agement of hypertension.
Prehypertension 120-139 80-89
Or Stage 1 hypertension 140-159 90-99
Describe briefly the aetiological factors of systemic Stage 2 hypertension .160 .100
hypertension and its treatment in a man of 50 years age
with complication in dental extraction. I. Primary or essential hypertension:
Or l Patients with elevated arterial blood pressure without any
identifiable cause are said to have essential or primary or
What are the causes of hypertension? Classify anti-
idiopathic hypertension.
hypertensive drugs. Describe briefly the complications
l Accounts for 85-90% of the cases and 70% of them
of hypertension.
have a positive family history.
l Pathogenesis of essential hypertension is not clearly
Ans.
understood, it is thought to be multifactorial.
l Hypertension is defined as a level of blood pressure at l Important environmental factors include high salt in-
which there is increased risk for target organ damage, take, heavy alcohol use, obesity, and sedentary lifestyle.
and the benefits of treatment outweighs the cost and
hazards. II. Secondary hypertension:
l 15% of the general population can be regarded as hyper- l In secondary hypertension, a specific cause is identified.
tensives. l Accounts for 15% of the cases.
52 Quick Review Series for BDS 3rd Year

Aaetiology l The signs of complications of hypertension such as


heaving apex, 4th heart sound, loud aortic second heart
Important causes of secondary hypertension are as follows:
sound, pulmonary crackles, and retinal changes may
Renal disorders: present.
l Risk factors are smoking, obesity, hyperlipidaemia, dia-
l Renovascular stenosis
betes mellitus, family history of premature CAD, all of
l Parenchymal renal disease, particularly glomerulonephritis
which interact with hypertension, especially in the gen-
l Polycystic kidney disease
esis of ischaemic heart disease.
Endocrinal disorders:
l Pheochromocytoma Complications of Hypertension
l Cushing’s syndrome
l Primary hyperaldosteronism i. Central nervous system complications
l Acromegaly l Transient ischaemic attacks.
l Hypo-and hyperthyroidism l Cerebrovascular accidents (strokes).
l Subarachnoid haemorrhage.
Drugs: l Hypertensive encephalopathy characterised by very
high blood pressure, neurological manifestations in-
l Oral contraceptives
cluding transient disturbances in speech and vision,
l Corticosteroids
paraesthesias, fits, disorientation, loss of consciousness
l Sympathomimetic drugs
and papilloedema. The neurological deficits are usually
l Cyclosporine
reversible with control of blood pressure.
l NSAIDs

Miscellaneous: ii. Ophthalmic complications


l Hypertensive retinopathy: It is characterised by thicken-
l Toxaemia of pregnancy
ing of the walls of the retinal arterioles, diffuse or seg-
l Coarctation of aorta
mental narrowing of blood columns, varying width of
l Raised intracranial pressure
the light reflex from vessel walls, arteriovenous nipping,
l Obstructive sleep apnoea.
retinal haemorrhages, soft and hard exudates and papill-
oedema. Severe retinopathy can cause visual field de-
Clinical Features fects and blindness.
l Grading of hypertensive retinopathy
l The majority of patients are asymptomatic and are diag-
Grade I: Arteriolar narrowing and increase in light
nosed on routine clinical examination.
reflex over the arterioles
l Symptoms due to raised blood pressure are occipital
Grade II: Marked arteriolar narrowing and arteriove-
headache, dizziness, palpitation and fatigue.
nous nipping
l Patient may also present with symptoms related to target
Grade III: Grade II plus flame-shaped haemorrhages
organ damage like epistaxis, hematuria, blurred vision,
and soft exudates
TIA, angina and breathlessness.
Grade IV: Grade III plus papilloedema.
l Symptoms pertaining to underlying cause may be pres-
ent, e.g. weight gain (Cushing’s syndrome), weight loss
(thyrotoxicosis), episodic headache, palpitation, and iii. Cardiovascular complications
sweating (pheochromocytoma). l Coronary artery disease (angina, myocardial infarction).
l History must include age, sex, occupation, lifestyle of l Left ventricular failure.
patient along with history of smoking, diabetes mellitus, l Aortic aneurysm.
hyperlipidaemia, alcohol and drug intake and presence l Aortic dissection.
of hypertension in family members.
l Physical examination: Presence of truncal obesity (Cush- iv. Renal complications
ing’s disease), palpable kidneys (polycystic kidneys), l Proteinuria.
radiofemoral delay (coarctation of aorta), recurrent back l Progressive renal failure.
pain, undiagnosed fever and recurrent urinary infections
suggest (chronic pyelonephritis) abdominal bruit (reno- v. Malignant hypertension
vascular), may help in identifying the secondary cause of l It is a clinical syndrome of markedly high blood pres-
hypertension. sure with retinal haemorrhages and exudates, and often
Section | I General Medicine 53

including confusion, headache, vomiting, visual distur- II. Antihypertensive drug therapy
bances and deterioration in renal functions. Various classes of drugs used in treating hypertension are as
follows:

Investigations A. Diuretics:
Basic investigations in all patients Thiazide diuretics:
l Chlorothalidone
l Urine analysis for protein, blood and glucose.
l Chlorothiazide
l Blood urea nitrogen and serum creatinine (to assess
l Hydrochlorothiazide
renal function).
l Metolazone
l Serum electrolytes, i.e. serum sodium and potassium.
l Fasting and post-prandial blood glucose (for hypergly- Loop diuretics:
l Furosemide
caemia).
l Bumetanide
l Serum lipid profile.
l Serum calcium and uric acid. Potassium-sparing diuretics:
l Amiloride
l Electrocardiogram (for left ventricular hypertrophy).
l Triamterene
l Chest radiograph (for cardiac size, evidences of cardiac
failure and aortic dilatation). Aldosterone receptor blockers:
l Spirolonolactone

Investigations in special group of patients B. Beta-blockers:


l Chest X-ray and echocardiography
l Propranolol
l Renal ultrasonography and angiography
l Metoprolol
l Serum calcium and phosphate
l Atenolol
l Thyroid-stimulating hormone (TSH)
l Bisoprolol
l Urinary cortisol and catecholamine
l Plasma renin activity and aldosterone. C. Combined alpha- and beta-blockers:
l Carvedilol
Treatment l Labetalol

l The management of hypertension includes general mea- D. Angiotensin-converting enzyme inhibitors:


sures and drug therapy. l Captopril
Treatment of hypertension can be discussed under three l Enalapril
headings: l Lisinopril
i. General measures. l Ramipril
ii. Antihypertensive drug therapy. l Perindopril
iii. Treatment of underlying cause (in secondary hypertension). E. Angiotensin II antagonists:
l Losartan
I. General measures (lifestyle modification)
l Candesartan
i. Relief of stress: Patients are advised to avoid unneces-
sary tension. Relaxation techniques also be practised. F. Calcium channel blockers:
ii. Salt restriction: Low sodium diet (,100 mEq sodium l Nifedipine
or ,6 g salt). Dietary restriction of sodium chloride up l Verapamil
to only 5 g per day helps to reduce blood pressure in l Diltiazem
some hypertensive patients. l Amlodipine
iii. Weight reduction: Caloric restriction in obese patients
and weight reduction leads to decreased blood pressure. G. Alpha blockers:
iv. Control of risk factors: Restriction of cholesterol satu- l Prazosin
rated fat in diet reduces the atherosclerosis complica- l Terazosin
tions in hypertensive patients. Oil intake should be l Doxazosin
reduced. Smoking is to be abandoned. Alcohol con-
sumption should be moderated. H. Direct vasodilators:
v. Regular exercises, relaxation classes, meditation and l Hydralazine
biofeedback. l Minoxidil
54 Quick Review Series for BDS 3rd Year

I. Central a2-blockers and other drugs that act pulmonary hypertension, congestive cardiac failure and
centrally: valvular diseases like MR and AR.
l Clonidine l Nifedipine and verapamil are commonly used drugs,
l Alpha-methyldopa they act on cell membrane selectively to block the ac-
l Reserpine. cess of calcium and exert negative inotropic action.
They increase myocardial oxygen supply, prevent coro-
Diuretics: nary artery spasm, increase coronary blood flow and
l Thiazides are the most frequently used diuretics to treat reduce peripheral vascular resistance.
hypertension.
l The side effects of thiazides can be minimized by re- Other groups of drug occasionally used in
stricting the dose of hydrochlorothiazide to 25 mg/day. hypertension are:
l A combination of thiazide and potassium sparing agents a. Alpha adrenergic receptor blockers – prazosin, doxazo-
(spironolactone, amiloride) may also be used. sin
l Loop diuretics are less extensively used. b. Centrally acting agents – clonidine, methyldopa
c. Vasodilators – hydralazine, minoxidil
Beta adrenoreceptor antagonists (beta-blockers): d. Agents with mixed alpha and beta adrenergic antagonist
l These agents lower heart rate and cardiac output. action – labetalol, carvedilol.
l The cardioselective agents which are preferred include
atenolol and metaprolol. Broad guidelines on selection of drugs:
l They are used with caution in patients with congestive l Diuretics especially thiazide group are often the first
heart failure, heart blocks, and asthma agents to be started.
l Uses of beta-blockers: l Beta-blockers are useful in young hypertensives and
Angina pectoris, acute myocardial infarction and post- those with ischaemic heart disease.
myocardial infarction period, cardiac arrhythmias, l Calcium channel blockers are useful in patients who
hypertension, thyrotoxicosis, fallot’s tetralogy, phaeo- have low renin levels, elderly patients who have a his-
chromocytoma, portal hypertension, migraine prophy- tory of bronchial asthma and in those with peripheral
laxis, etc. vascular disease.
l Contraindications of beta-blockers: l Angiotensin-converting enzyme (ACE) inhibitors would
COPD and asthma, cardiac failure, heart block, periph- be very beneficial in patients who have diabetes, conges-
eral vascular disease, diabetes mellitus, etc. tive cardiac failure and renovascular hypertension.

Angiotensin-converting enzyme ACE inhibitors and Guidelines on the use of antihypertensive drugs (after
angiotensin receptor blockers (ARB): lifestyle modification):
l angiotensin converting enzyme inhibitors (enalapril, Stage of Hypertension Drug Treatment
lisinopril, ramipril) and ARB (losartan, irbesartan, val-
Prehypertension without None
sartan) have good antihypertensive effect and cause associated risk factors as
significant reduction in death, acute MI and stroke. stated above
l These agents also reduce the progression of nephropa-
Prehypertension with Diuretics
thy in type II diabetes. risk factors Beta-blockers
l ARBs have lesser side effects like cough and angioedema ACE inhibitors
and are better tolerated compared to ACE inhibitors. Calcium channel blockers
l Mechanism of action includes powerful arteriolar and Angiotensin receptor blockers
venous dilatation and inhibition of aldosterone release. Stage 1 hypertension Thiazide group of diuretics
l Contraindicated in patients with impaired renal function May add:
(creatinine .3 mg/dL) and bilateral renal artery stenosis. Beta-blockers
ACE inhibitors
l Angiotensin receptor blockers are useful in the treat-
Calcium channel blockers
ment of hypertension, myocardial infarction and dia- Angiotensin receptor blockers
betic nephropathy.
Stage 2 hypertension Thiazide diuretics plus one or more
Calcium channel blockers (calcium antagonists): of the following:
Beta-blockers
l Calcium channel blockers act by arterial vasodilatation ACE inhibitors
and are used in angina pectoris/Prinzmetal’s angina, Calcium channel blockers
hypertension and hypertensive crisis, peripheral vaso- Angiotensin receptor blockers
Add other drugs if blood pressure
spastic conditions, e.g. Raynaud’s disease, migraine,
still not under control
achalasia cardia, biliary dyskinesia, bronchial asthma,
Section | I General Medicine 55

Q. 2. Describe clinical features, complications, diagnosis l Auscultation: Loud first heart sound, opening snap and
and management of chronic rheumatic mitral stenosis. mid-diastolic low-pitched rumbling murmur best heard
at apex.
Or
l Signs of pulmonary hypertension: RV heave, loud P2.
Describe the aetiology, clinical features, investigations l Signs of raised pulmonary capillary pressure: Crepitations,
and management of mitral stenosis. pulmonary oedema, effusions.
l Others
Ans.
l Basal crackles (crepts) in lungs due to pulmonary

l Mitral stenosis (MS) is almost always rheumatic in origin, congestion


although in elderly it may be caused by heavy calcifica- l Ascites and pleural effusion (in severe cases)

tion of the mitral valve apparatus. Rarely it may be con-


genital or degenerative.
Investigations
l It is more commonly found in females.
i. ECG reveals the evidence of LA enlargement, RV hy-
pertrophy, and atrial fibrillation.
Pathophysiology ii. Chest X-ray reveals LA enlargement and pulmonary
l The size of the orifice of normal mitral valve is 4-6 cm2. congestion.
The orifice is progressively narrowed by fibrosis and iii. Echocardiography is the most sensitive and specific
calcification of valve leaflets, fusion of commissures non-invasive methods to diagnose the valvular disease.
and shortening of chordae tendineae. It may reveal structural abnormalities of the valves, size
l Mitral valve orifice less than 2 cm2 is hemodynamically of cardiac chambers, pulmonary artery pressure, ven-
significant and becomes “critical” at ,1 cm2. tricular dysfunction and presence of thrombi.
l The obstruction to the flow of blood through the ste- iv. Cardiac catheterization is used to assess associated
nosed mitral valve during diastole causes a rise in left valvular lesions and to detect coronary artery disease.
atrial pressure. This leads to the hypertrophy and dilata-
tion of the left atrium, pulmonary venous congestion
Treatment
and pulmonary arterial hypertension.
l Right ventricular failure may occur due to pulmonary A. Medical management of mitral stenosis
hypertension. Reduced lung compliance causes breath- i. Restriction of sodium
lessness while decreased left ventricular filling leads to ii. Diuretics are used to reduce pulmonary congestion.
low cardiac output and fatigue. iii. In patients with atrial fibrillation, digoxin is given to
l Atrial fibrillation (AF) commonly occurs because of left control ventricular rate.
atrial dilatation. Patients, particularly with AF, are more iv. Beta blockers and calcium antagonists (verapamil, dil-
susceptible to develop left atrial thrombus and systemic tiazem) can also be used.
thromboembolism. v. Oral anticoagulants (warfarin) are given to patient with
a history of thromboembolic events or to those with AF.
vi. Prophylaxis should be given to all patients to prevent
Clinical Manifestations rheumatic fever.
l Breathlessness on exertion and fatigue are the early
presentations of mitral stenosis. B. Surgical management
l As the stenosis progresses, patients are dyspnoeic on When patient remains symptomatic despite of medical
rest and even have orthopnoea and paroxysmal noctur- treatment or when mitral stenosis is severe, surgical inter-
nal dyspnoea (PND). vention is needed:
l Acute pulmonary oedema may also occur. i. Mitral valvotomy
l There may be hemoptysis due to rupture of pulmonary- a. Percutaneous balloon valvotomy is indicated when mi-
congestion and pulmonary embolism and cough due to tral valve is non-calcified and without regurgitation. The
pulmonary congestion. procedure involves the passing of catheter across the
l Chest pain due to pulmonary venous hypertension. valve and inflation of the balloon to dilate the orifice.
l Symptoms of thromboembolic complications like b. Open valvotomy is carried out in patients where
stroke, limb ischaemia, etc. balloon valvotomy is not possible or in cases with
restenosis. In this procedure, the fusion of the valve
is loosened and calcium deposit and thrombi are
Signs of mitral stenosis are as follows:
removed.
l Atrial fibrillation ii. Mitral valve replacement: The mitral valve is replaced
l Mitral facies when there is critical mitral stenosis and/or there is
56 Quick Review Series for BDS 3rd Year

associated significant mitral regurgitation. Replace- iv. Signs of tissue damage:


ment is also done when the mitral valve is severely l Mild fever

distorted and calcified. l Arrhythmias

v. Signs of complications of myocardial infarction:


Q. 3. Describe, aetiology, clinical profile, investigation l Mitral regurgitation, pericarditis, ventricular ectopic
and management of acute myocardial infarction. beats, ventricular tachycardia diffuse apical impulse,
Or heart blocks, etc.

A 60-year-old patient had myocardial infarction (isch-


aemic heart disease) one year ago, he has to undergo Complications of MI
total extraction of teeth, describe the line of manage- i. Cardiogenic shock may be caused by an arrhythmia,
ment and precautions to be taken. or excessive diuretic therapy leading to hypovolaemia
Or is another cause. Otherwise cardiogenic shock usually
reflects extensive myocardial damage and indicates a
What are the risk factors for coronary artery disease? bad prognosis.
Describe the clinical features investigations and man- ii. Cardiac failure, most commonly manifested as pulmo-
agement of acute myocardial infarction. nary oedema.
iii. Infarction of the mitral papillary muscle leading to
Ans.
mitral regurgitation and pulmonary oedema.
l Myocardial infarction is myocardial necrosis occurring iv. Rupture of interventricular septum leading to a mur-
as a result of a critical imbalance between coronary mur of VSD and severe hypotension.
blood supply and myocardial demand. v. Cardiac tamponade due to rupture of ventricle into
l Myocardial infarction is almost always due to the for- pericardial sac.
mation of occlusive thrombus at the site of rupture or vi. Cerebral and peripheral embolism resulting from the
erosion of an atheromatous plaque in a coronary artery. detachment of a ventricular mural thrombus.
l Subendocardial infarct is an infarct confined to suben- vii. Deep vein thrombosis and pulmonary embolism in
docardial region. patients on prolonged bed rest.
l Transmural infarct is an infarct involving the full thickness viii. Dressler’s syndrome is otherwise known as post-
of myocardium. myocardial infarction syndrome. It is an autoimmune
l Silent infarct is characterised by no symptoms, but only reaction to necrotic muscle.
ECG changes.
Investigations
Symptoms
i. Electrocardiography
l Cardinal symptom is chest pain, but breathlessness, l ECG is usually useful in confirming the diagnosis.
syncope, vomiting and extreme tiredness are common. l The typical changes are seen in leads facing the infarcted
l Pain is at the same site as for angina, but is more severe area. These changes include:
and prolonged. It is severe, with pallor and a peculiar l ST segment elevation
facial expression. l Appearance of pathologic Q-waves.
l Pain is described as tightness, heaviness or constriction. l In very early MI, the T waves may become tall and
l Anxiety and fear of impending death. peaked.
l Nausea and vomiting. l These are transient and last for a few hours only.
l Collapse and syncope. l In contrast to transmural lesions, partial thickness or
subendothelial infarction causes ST/T wave changes
without Q waves or prominent ST elevation.
Physical Signs
l The ECG changes evolve in predictable fashion over
i. Signs of sympathetic activation: next few days to weeks.
l Pallor, sweating, tachycardia.

ii. Signs of vagal activation: Plasma biochemical markers:


l Vomiting, bradycardia MI causes detectable rise in the plasma concentration of
iii. Signs of impaired myocardial function: enzymes and proteins that are normally concentrated within
l Hypotension, oliguria, cold peripherals, narrow the cardiac cells.
pulse pressure, raised JVP (jugular venous pres-
sure), third heart sound, diffuse apical impulse, lung Plasma enzymes (cardiac injury enzymes) are as follows:
crepitations. i. Creatine kinase (CK).
Section | I General Medicine 57

ii. Aspartate aminotransferase (AST). l 12-lead ECG


iii. Lactate dehydrogenase (LDH). l IV analgesia (opiates) and antiemetic
iv. Myoglobin l Aspirin 300 mg.

v. Troponins (troponin I and troponin T) iii. Reperfusion


l Creatine kinase (CK) starts to rise at 4-6 hours, peaks l Primary PCI or thrombolysis

by about 12 hours and falls to normal in 48-72 hours. iv. Detect and manage acute complications
Measurement of the myocardial isoenzymes of CK l Arrhythmias

(CK-MB) is more specific for myocardial infarction. l Ischaemia

l Aspartate aminotransferase (AST) starts to rise by l Heart failure

about 12 hours and reaches a peak on the first or


second day. Specific therapy
l Lactate dehydrogenase (LDH) starts to rise after i. Thrombolysis
12 hours, reaches a peak after 2-3 days and may ii. Intravenous beta-blockers unless contraindicated
remain elevated for a week. A rise in the level of iii. Treat complications (arrhythmias, congestive failure and
LDHI (an isoenzyme of LDH) is a more sensitive shock)
indicator of myocardial infarction than total LDH. iv. Admit in intensive coronary unit.
l Cardiac troponins include cardiac troponin-T (cTn-T)

and cardiac troponin-I (cTn-I). The sensitivity of tro- Oxygen


ponins is similar to that of CK-MB. However, cTn-T l Hypoxaemia in patients with uncomplicated MI is usu-
remains elevated for 100-200 hours after acute MI ally caused by ventilation-perfusion abnormalities and
and, therefore, this assay may have particular utility may be exacerbated by CHF. Therefore, oxygen is ad-
in the evaluation of patients who present sufficiently ministered to patients suspected of having an acute
long after their episode of chest pain. ischaemic syndrome.

Other investigations Nitrates


l Leucocytosis with a peak on first day. l Nitrates reduce oxygen demand and myocardial wall
l Raised ESR which may remain so for days. stress by reducing both preload and afterload.
l Elevated C-reactive protein. l If patient is stable, sublingual nitrate should be given.
l Chest radiography. l Intravenous nitroglycerin (NTG) is recommended in
l Heart size is usually normal. Enlargement of cardiac patients with pump failure, pulmonary oedema, acceler-
shadow may indicate previous myocardial damage or ated hypertension and continuing chest pain.
pericardial effusion.
l Evidences of pulmonary oedema. Control of pain
l Radionuclide scanning shows the site of necrosis and l Morphine in a dose of 3-5 mg is administered intrave-
the extent of impairment of ventricular function. nously every 10-15 minutes along with an antiemetic to
l Echocardiography for regional wall motion abnormality control chest pain.
and ejection fraction. l The beneficial effects of morphine are mainly due to
sedation and analgesia produced by it which results in
reduction in myocardial oxygen demand.
Management l It also reduces preload to some extent which may be
l Patients with suspected acute MI require immediate beneficial in patients with MI.
access to medical/paramedical care and defibrillation l Beta-blockers, nitroglycerin and thrombolysis may also
facilities. Patients are usually managed in dedicated help in relieving pain.
cardiac units.
l In the first 24-48 hours when the risk of fatal arrhythmia Antiplatelet agents
is higher, the patients are best treated in an intensive l Early administration of aspirin to patients with acute MI
coronary care unit. has been shown to be of significant benefit in acute MI.
The patient is asked to chew 150 mg aspirin.
Early management of acute myocardial infarction: l If patient is intolerant to aspirin, administer clopidogrel
Initial treatment: (300 mg stat)
i. Provide facilities for defibrillation.
ii. Immediate measures: Beta-blockers
l High-flow oxygen l These agents decrease oxygen demand by lowering
l Secure an intravenous line the heart rate and blood pressure. They also counter
l ECG monitoring the direct adverse effects of catecholamines and have
58 Quick Review Series for BDS 3rd Year

antiarrhythmic properties. These agents are also use- Signs of reperfusion:


ful to control tachycardia, hypertension and continued l Sudden relief of chest pain
angina. l Rapid resolution of ST-segment elevation
l Intravenous beta-blockers given within first few hours l Onset of reperfusion arrhythmias.
of infarction are useful in reducing mortality. These are l Early peaking of CK-MB enzyme.
followed by oral beta-blockers. The commonly used
beta-blockers include metoprolol and esmolol. Complications:
l Contraindications to the use of beta-blockers include brady- l The major complication of thrombolytic therapy is
cardia (rate ,60/min), systolic blood pressure of less than haemorrhage which occurs in about 30% of cases.
100 mmHg, moderate to severe congestive heart failure, AV l The most common site of haemorrhage is from puncture
conduction defects and severe obstructive lung disease. sites and genitourinary system.
l Intracranial haemorrhage occurs in about 0.5% of cases.
Angiotensin-converting enzyme inhibitors l Use of STK and APSAC may be associated with aller-
l Angiotensin-converting enzyme (ACE) inhibitors im- gic reactions.
prove the myocardial function by reducing myocardial l Hypotension may occur, if STK is infused rapidly.
remodelling.
l ACE-inhibitors are recommended within 24 hours in Contraindications:
patients with acute MI with or without CHF. Relative contraindications to thrombolytic therapy:
l Initiate the treatment with short-acting ACE inhibitors, l Active internal bleeding
captopril, in a dose of 12.5 mg. If the systolic pressure l Uncontrolled hypertension
remains above 100 mmHg, increase the dose to 25 mg l Previous subarachnoid or intracebral haemorrhage
8 hourly with a constant watch on blood pressure. l Recent intracranial/spinal surgery (within last 4- 6 weeks)
l Recent trauma (including traumatic resuscitation)
Thrombolytic therapy l Pregnancy
l Thrombolytic agents include: Streptokinase (STK), uro- l High probability of active peptic ulcer.
kinase (UK); human tissue plasminogen activator (t-PA)
alteplase, reteplase(rPA), tenecteplase, anisoylated plas- Antithrombin therapy:
minogen streptokinase activator complex (APSAC, an- l Antithrombin therapy in the form of heparin is required
tistreplase) and single-chain urokinase plasminogen ac- before the completion of infusion of rt-PA. Its role along
tivator (scu-PA). All these agents act as direct or indirect with other thrombolytics is controversial.
plasminogen activators leading to generation of plasmin l Direct thrombin inhibitors (hirudin and bivalirudin)
which lyzes the clot. rt-PA, reteplase, tenecteplase and may be better than the unfractionated heparin and com-
scu-PA do this only at the site of clot whereas others parative trials are underway.
generate plasmin in the systemic circulation leading l Low-molecular weight heparin (LMWH) as an adjunct
to a systemic lytic state which leads to a reduction in to SK or other thrombolytics may result in higher reper-
blood viscosity and exerts strong anticoagulant and anti- fusion rate and lower reocclusion rate compared to un-
platelet effects. fractionated heparin.
l STK is used in a dose of 1.5 million units intravenously
as an infusion over a period of one hour. Prophylactic anticoagulants:
l Use of low-dose heparin, i.e. 5, 000 units twice a day
Indications for thrombolysis in acute myocardial subcutaneously to prevent deep vein thrombosis and
infarction pulmonary embolism.
i. Thrombosis shown to be of definite value:
l ST segment elevation of greater than 0.1 mV in two or more Right ventricular infarction
contiguous leads, with time to therapy 12 hours or less. l In this type of infarction, there is reduced right ventricu-
l Bundle-branch block and history suggestive of acute MI lar stroke volume. As a result, left ventricular filling is
for less than 12 hours impaired and systemic hypotension may develop.
l Volume expansion is the initial treatment of choice
ii. Thrombolysis may be of some benefit: for haemodynamically significant right ventricular
l ST segment elevation with time to therapy 12-24 hours. infarction.
l Avoid diuretics.
III. Thrombolysis not indicated/may be harmful: l Inotropics and vasodilators may also be required espe-
l ST segment depression only. cially when a significant component of associated left
l Time to therapy .24 hours. ventricular dysfunction is also present.
Section | I General Medicine 59

Percutaneous coronary interventions (PCI): intravascular volume) cannot maintain an adequate cardiac
l These interventions include angioplasty (percutaneous output to meet the metabolic needs of peripheral tissues or
transluminal coronary angioplasty–PTCA) or stent place- can do so only with an elevated filling pressure.
ment in the coronary artery.
l Angioplasty may be used solely in acute MI (primary Pathophysiology
PTCA) or in combination with thrombolytic therapy
when thrombolysis fails (rescue PTCA). Concept of preload, afterload and myocardial
l Primary PTCA performed within 90 minutes of onset of contractility
symptoms has been shown to give better results com- Preload:
pared to thrombolytic therapy.
Preload refers to the pressure that fills the left ventricle
l Primary PTCA is indicated in cardiogenic shock, and in
during diastole. It is measured either directly as the
presence of contraindications to thrombolytic therapy.
left-ventricular end-diastolic pressure, or indirectly and
l Glycoprotein IIb/IlIa inhibitors (abciximab, tirofiban)
more commonly as the pulmonary artery wedge pressure
are often recommended in patients undergoing percuta-
(PAWP).
neous interventions.
Coronary arteries bypass grafting Afterload:
l It is probably of little use in the setting of uncompli- Afterload refers to the pressure against which the left ven-
cated acute myocardial infarction. tricle contracts and is measured as the mean aortic pressure.
l It offers benefit in patients with acute myocardial infarc- The main determinants of after load are total peripheral
tion with deteriorating haemodynamic status, and pa- resistance and left ventricle size.
tients who develop coronary occlusion during coronary
Myocardial contractility (inotropic state):
angiography and PTCA.
l This mainly depends on the adrenergic nervous activity
Aftercare and rehabilitation and the levels of circulating catecholamines.
l Restrict physical activities for 4-6 weeks since infarct l Heart failure is characterised by a decrease in cardiac
takes 4-6 weeks to become replaced with fibrous tissue. output. Cardiac output in turn is a function of preload,
l Gradual mobilisation and return to work over 6 weeks. afterload and myocardial contractility. In heart failure,
l When there are complications, the regimen has to be preload is increased, afterload is increased and myocar-
modified accordingly. dial contractility is decreased.
l Exercise within the limits set by angina and tiredness l In the initial stages of heart failure with the reduction
will do no harm but much good. Same limits apply to in cardiac output, certain compensatory mechanisms
sexual activity. come into operation. These compensatory mechanisms
l Control of obesity, regular exercises, cessation of smok- are initially beneficial, but later become counterproduc-
ing, adoption of a less frenetic way of life and control of tive and account for most of the manifestations of heart
plasma lipids by diets and drugs. failure. These compensatory mechanisms are mediated
l Complications should be managed. Pain relief, reassur- through renin–angiotensin system and autonomic ner-
ance, rest and correction of hypokalaemia play a major vous system.
role in prevention of arrhythmias. These mechanisms are discussed below.
Q. 4. What are the symptoms and signs of cardiac fail- Increased myocardial contractility:
ure. How will you treat this condition.
l As the ventricle dilates (resulting in increased preload),
Or the ventricular contractility increases resulting in rela-
Describe the aetiology, clinical features and treatment of tively increased volume of blood ejected (Frank-Star-
acute left ventricular failure. ling law). However, there is a limit to which the myocar-
dial cells can be stretched so as to enhance their
Or contractility and beyond this limit, their contractility
Define heart failure. Discuss the types, common causes, diminishes.
pathophysiology, clinical features and management of l The increased left ventricular filling pressures are trans-
heart failure. mitted to the pulmonary veins which result in alveolar
trasudation of fluid and pulmonary congestion. Finally,
Ans. oxygen demand is enhanced due to increased contractility.
Heart failure or cardiac failure is defined as a state, in which l Myocardial hypertrophy occurs due to volume and pres-
the ventricles at normal filling pressures (i.e. adequate sure overload in the ventricle and helps in enhanced
60 Quick Review Series for BDS 3rd Year

contractility and hence increased cardiac output. How- ii. Low output heart failure is associated with a low cardiac
ever, it produces increased oxygen demand and reduced output. Eg: Heart failure associated with ischaemic
compliance of the ventricle that results in increased heart disease, hypertension, cardiomyopathy, valvular
preload which is again transmitted to the pulmonary diseases and pericardial disease.
vasculature.
l Sympathetic stimulation results in peripheral vaso- C. Left-sided, right-sided and biventricular
constriction which in turn leads to salt and water heart failure
retention due to direct and indirect effects on the re- Left-sided (left ventricular) heart failure:
nin-angiotensin system. Sympathetic stimulation also
l ‘Left-side’ is a term for the functional unit of left atrium,
produces increased heart rate and enhances contractil-
left ventricle, mitral valve and aortic valve.
ity. Increased afterload may be deleterious as it tends
l Excess fluid accumulates upstream behind the failing left
to reduce cardiac output and increases oxygen de-
ventricle. There is reduction in left ventricular output,
mand of the heart.
increase in left atrial pressure and increase in pulmonary
venous pressure.
Myocardial remodelling:
l Acute increase in left atrial pressure causes pulmonary
l The fundamental mechanism that underlies the progres- congestion and pulmonary oedema, e.g. myocardial in-
sive nature of myocardial dysfunction has been termed farction.
remodelling. l Gradual increase in left atrial pressure causes reflex
l Important changes in structure and function of the pulmonary hypertension, but no pulmonary oedema,
myocardium include hypertrophy and apoptosis (pro- e.g. aortic stenosis.
grammed cell death) of myocytes, and alterations in the
quantity and composition of extracellular matrix. Right-sided (right ventricular) heart failure:
l Remodelling involves not only the ischaemic area, but l ‘Right-side’ is a term for the functional unit of right
also the viable myocardium resulting in gradual loss of atrium, right ventricle, tricuspid valve and pulmonary
contractility over a period of time. valve.
l Excess fluid accumulates upstream behind the failing
Types of Heart Failure right ventricle.
l There is reduction in right ventricular output which
A. Acute and chronic heart failure results in systemic venous congestion.
B. High output and low output heart failure l Best Eg: cor pulmonale, pulmonary valvular stenosis
C. Left-sided, right-sided and biventricular heart failure and multiple pulmonary emboli.
D. Forward and backward heart failure
E. Systolic and diastolic heart failure. Biventricular heart failure:
l There is failure of both left and right ventricles. Eg:
A. Acute and chronic heart failure l Disease processes affecting both ventricles like di-
i. Acute heart failure develops suddenly. The sudden re- lated cardiomyopathy and ischaemic heart disease.
duction in cardiac output results in systemic hypoten- l Diseases of left heart leading to chronic elevation
sion without peripheral oedema. Eg:Acute myocardial of left atrial pressure, pulmonary hypertension and
infarction and rupture of a cardiac valve. subsequent right ventricular failure.
ii. Chronic heart failure develops gradually. Here, sys-
temic arterial pressure is well-maintained, but oedema D. Forward and backward heart failure
accumulates. Eg: Dilated cardiomyopathy and multi- l Forward heart failure is characterised by decreased car-
valvular disease. diac output and inadequate perfusion of organs.
iii. Compensated heart failure implies that the compensa- l Backward heart failure is characterised by a normal
tory changes have prevented the development of overt cardiac output, but marked salt and water retention and
heart failure. Eg: A minor insult like an infection may pulmonary and systemic venous congestion.
precipitate severe heart failure.
E. Systolic and diastolic heart failure
B. High output and low output heart failure l Systolic heart failure is characterised by an abnormality
i. High output heart failure is associated with an increased of ventricular contraction. The ejection fraction is below
cardiac output. Eg: Cardiac failure associated with hy- 40%.
perthyroidism, anaemia, pregnancy, arteriovenous fistu- l Diastolic heart failure is characterised by an abnormal-
lae, beriberi and Paget’s disease. ity of ventricular relaxation.
Section | I General Medicine 61

Aetiology l Clinically, it is characterised by severe breathless-


ness, cough with copious, pinkish frothy expecto-
Common causes of heart failure are as follows:
ration and bilateral crepitations.
i. Conditions associated with pressure overload of ven-
vi. Cheyne-Stokes respiration:
tricle like
l A type of periodic breathing with alternate periods
l Systemic hypertension
of apnoea and hyperventilation, seen in advanced
l Pulmonary hypertension
heart failure.
l Aortic stenosis
vii. Nocturia:
l Pulmonary stenosis
l A feature of early heart failure.
ii. Volume overload of ventricle conditions associated
l Underlying mechanism is a better renal perfusion
with increased metabolic demand like
and diuresis at night when the patient is supine.
l Mitral regurgitation
viii. Cerebral symptoms:
l Aortic regurgitation
l Confusion, difficulty in concentration, memory
l Ventricular septal defect
impairment, headache, insomnia and anxiety.
l Persistent ductus arteriosus
l All these symptoms are due to arterial hypoxae-
l Atrial septal defect
mia and reduced cerebral perfusion.
iii. Inflow obstruction of ventricle
ix. Nonspecific symptoms:
l Mitral stenosis
l Fatigue and weakness from reduced perfusion of
l Tricuspid stenosis
skeletal muscles.
l Endomyocardial fibrosis
l Low-grade fever from reduction of cutaneous flow.
iv. Impaired ventricular function in conditions like
l Anorexia, nausea, abdominal pain and fullness
l Diffuse myocardial disease like myocarditis, cardio-
from congestion of liver and portal venous system.
myopathy
x. Cardiac oedema:
l Myocardial infarction
l Due to gravity, cardiac oedema accumulates over

In practice, the most commonly encountered causes of dependent parts. In ambulant patients, it occurs
heart failure are ischaemic heart disease, hypertensive heart symmetrically in the legs, particularly in the pre-
disease and valvular heart diseases. tibial region and around the ankles. It is less in the
morning and more towards the evening. Oedema
is sacral in bedridden patients.
Clinical Manifestations of Heart Failure
l In advanced stages, there is anasarca with
i. Dyspnoea: oedema fluid accumulating throughout the body.
l Dyspnoea is initially exertional, but progres- Face and arms are typically spared until the ter-
sively worsens to a stage of breathlessness even minal stages.
at rest. xi. Cyanosis:
ii. Orthopnoea: l Mainly affects lips and nail beds. Extremities are
l Dyspnoea occurring soon after lying flat and re- cold and pale due to reduced blood flow, and also
lieved by sitting up is known as orthopnoea. cyanosed.
iii. Paroxysmal nocturnal dyspnoea (PND): xii. Pulse:
l This is an attack of severe shortness of breath l Pulsus alternans is a sign of severe heart failure.
and coughing usually occurring at night, awak- xiii. Blood pressure:
ening the patient from sleep. Paroxysmal noctur- l Diminished pulse pressure due to reduced stroke
nal dyspnoea persists even after sitting upright volume.
whereas simple orthopnoea is relieved by sitting l Diastolic blood pressure may be slightly raised
upright. occasionally, due to generalised vasoconstriction.
iv. Cardiac asthma: l Hypotension is prominent in acute heart failure.
l This is closely related to paroxysmal nocturnal l Jugular venous pressure is raised as a conse-
dyspnoea and nocturnal cough. It is characterised quence of elevated systemic venous pressure.
by wheezing secondary to bronchospasm, most xiv. Third and fourth heart sounds:
prominent at night. Acute pulmonary oedema is a l The presence of a third heart sound in an adult is
severe form of cardiac asthma. highly suggestive of heart failure.
v. Acute pulmonary oedema: l The presence of either of the two (S3 or S4) is
l It is a severe form of cardiac asthma resulting from known as triple rhythm.
marked elevation of pulmonary capillary pressure l The presence of both of them (S3 and S4) is
leading to alveolar oedema. known as quadruple rhythm.
62 Quick Review Series for BDS 3rd Year

l In some cases, S3 and S4 merge to become a congestive heart failure. Control of HF can be achieved by
single sound which sounds like a triple rhythm, general measures and drug therapy.
but known as summation gallop. l The management of heart failure consists of:
xv. Respiratory system: A. General measures
l Dull percussion notes over the lung bases. Inspi- B. Drug therapy
ratory crepitations over lung bases.
l In patients with pulmonary oedema, crepitations A. General measures
are coarse and heard widely over both lung fields i. Physical and emotional rest:
associated with expiratory rhonchi. l Reduced physical exertion lowers myocardial oxy-
xvi. Liver (congestive hepatomegaly): gen demand and thus controls failure. Absolute bed
l Right upper quadrant pain from stretching of the rest may be needed in severe heart failure.
capsule of the liver. l Small doses of tranquillizers may be used.
l Liver is enlarged and tender due to systemic l In stable patients, regular isotonic exercises are
venous hypertension. advised.
l Cardiac cirrhosis occurs in prolonged heart failure l The salt intake is reduced to half of the normal or
which results in atrophy and centrilobular necro- even less than 2 g per day in some cases.
sis of liver cells, leading later to extensive fibrosis. ii. Correction of obesity by restriction of caloric intake.
l Jaundice is a late feature characterised by hyper- Weight loss in obese patients is also helpful in control-
bilirubinaemia of both conjugated and unconju- ling heart failure.
gated types.
xvii. Pleural effusion, ascites and pericardial effusion: B. Drug therapy
l Pleural effusion is more common on the right side
Drugs used in heart failure are as follows:
and results from elevation of pleural capillary pres- i. Diuretics
sure and transudation of fluid into pleural space. l Thiazide diuretics–chlorthiazide, metolazone
l Pericardial effusion can occur rarely.
l Loop diuretics–frusemide, bumetanide, torsemide
xviii. Kidney: l Potassium sparing diuretics–spironolactone, amiloride,
l Oliguria, urinary sodium is low, specific gravity is
triamterene.
high and it contains proteins (less than 1 g/day). ii. Vasodilators
l Prerenal azotaemia is common. Blood urea is typi-
a. Oral:
cally elevated out of proportion to serum creatinine. l ACE inhibitors–ramipril, enalapril, lisinopril
xix. Cardiac cachexia: l ARBs–losartan, valsartan
l Advanced heart failure is associated with severe
l Isosorbide dinitrate
anorexia, weight loss and malnutrition. l Hydralazine

b. Parenteral:
Common diagnostic studies
l Sodium nitroprusside
l Chest radiograph may show cardiomegaly, prominence l Nitroglycerine
of upper lobe veins, Kerley A and B lines, and other l Nesiritide (rBNP)
features of pulmonary oedema. iii. Ionotopic agents–Digitalis
l Electrocardiography. l Sympathomimetic amines–dopamine, dobutamine
l Assessment of ejection fraction, valvular functions, and l Phosphodiesterase inhibitors–amrinone, milrinone.
chamber size and shape by echocardiography. iv. Beta blockers
l Assessment of ejection fraction by radionuclide ven- Metoprolol, carvedilol, bisoprolol.
triculography.
l Exercise stress testing and cardiac catheterization, if Drugs help in heart failure by reducing preload and afterload
indicated. and improving myocardial contractility. This can be achieved
l Ambulatory Holter monitoring, if arrhythmias are suspected. generally with the help of more than one class of drugs as above.
l Other studies include serum levels of brain natriuretic Q. 5. Describe the clinical features, investigations and
peptide (BNP) which are elevated, and renal and liver treatment of rheumatic fever.
functions as baseline.
Or
Management of Heart Failure Acute rheumatic fever. Write about aetiology, pathology,
clinical features, complications and dental considerations.
The treatment of HF includes correction of the underlying
cause, removal of the precipitating factor, and control of Or
Section | I General Medicine 63

Discuss in detail, clinical features, diagnosis, manage- Presence of two or more major criteria or one major and
ment and prophylaxis of acute rheumatic fever. at least two minor criteria plus evidence of previous strep-
tococcal infection is required for the diagnosis of ARF.
Ans.
l Acute rheumatic fever (ARF) is a multisystem disorder Sore throat
which follows pharyngeal streptococcal infection. This l Only two-thirds of patients remember having any upper
is the commonest cause of acquired heart disease in respiratory symptoms in the past 1-5 weeks.
childhood and adolescence.
l Rheumatic fever is an inflammatory disease occurring Polyarthritis
as a delayed, sequel to pharyngeal infection with group l Arthritis is an early feature of ARF and occurs in about
A streptococci. It primarily, involves the heart, joints, 75% of cases. There is painful inflammatory involve-
central nervous system, skin and subcutaneous tissues. ment of the large joints like ankles, knees, elbows which
are red, swollen and tender.
Classical presentation is acute migratory polyarthritis. The
Pathogenesis l

pain and swelling in the involved joints subside or disap-


l Acute rheumatic fever affects children, most commonly pear as newer joints are affected (migratory polyarthritis).
between 5 and 15 years of age. It occurs in about 3%
individuals who develop pharyngeal infection with Carditis
certain serotypes of group A streptococci. It is relatively l Features of carditis develop early, i.e. within 3 weeks of
rare in infants and young children below the age of onset and occur in 40–50% of cases.
5 years. l It is more common in younger children and may be
l Streptococcal antigens have cross-reactivity with car- asymptomatic and picked up on echocardiography
diac tissue; hence anti-streptococcal antibodies mediate only.
inflammatory reaction in the myocardium, endocardium l Rheumatic fever may involve pericardium, endocar-
and pericardium. Joint and skin tissues are also affected dium and myocardium resulting in pancarditis occur-
due to tissue cross-reactivity. ring in 40 to 60% of patients with ARF.
l Streptococcus-induced autoimmunity is believed to be l The symptoms are chest pain, palpitation and breath-
the mechanism resulting in rheumatic process. Several lessness.
streptococcal antigens have demonstrated cross-reactiv- l Examination may reveal tachycardia, third heart sound,
ity with cardiac and other tissues. pericardial rub, murmur of mitral regurgitation and
cardiomegaly.
Clinical Manifestations l Apical mid-diastolic murmur (Carey Coombs’ murmur)
may be present due to mitral valvulitis. Fibrosis and
l The manifestations of ARF are fever, lethargy, anorexia adhesion of the valve may develop following healing of
and symptoms caused due to involvement of heart, the valvulitis which may lead to stenosis and/or regurgi-
joints, skin and central nervous system. tation (rheumatic valvular heart disease).
l The diagnosis of acute rheumatic fever (ARF) is based l The mitral valve is most commonly involved and the
on updated Jone’s criteria as described below. aortic valve is next most affected. The valvular involve-
ment increases the risk of infective endocarditis.
Major criteria
l Polyarthritis Diognosis of carditis requires the presence of one or more
l Carditis of the following:
l Chorea l Appearance of, or change in the character of organic
l Subcutaneous nodules murmurs
l Erythema marginatum l Cardiomegaly
l Pericarditis or pericardial effusion
Minor manifestations l Congestive heart failure.
l Fever
l Arthralgia Subcutaneous nodules
l Raised ESR l This rare manifestation occurs in less than 10% of cases.
l Positive CRP l Subcutaneous nodules are usually associated with severe
l Prolonged PR interval. carditis and tend to occur several weeks after its onset.
l And positive throat culture for streptococci or elevated l Description: Small, pea-sized (0.5 to 2 cm), painless
or increasing streptococcal antibody titer. nodules over bony prominences.
64 Quick Review Series for BDS 3rd Year

l Common sites: Extensor tendons of hands and feet, el- Other tests confirming an inflammatory reaction:
bows, margins of patellae, scalp, occiput, over scapulae l Polymorphonuclear leucocytosis.
and over spinous processes of vertebrae. l Increase in serum complements.
l They usually persist for 1-2 weeks. l Increase in serum mucoproteins, alpha2 and gamma
globulin levels.
Erythema marginatum
l Anaemia due to suppression of erythropoiesis.
l It occurs in less than 10% cases of acute rheumatic
fever. Electrocardiogram:
l These are evanescent erythematous macules with a clear l The most consistent abnormality is a prolongation of
centre and round or serpiginous margins. These rashes the PR interval.
are transient, migrating from place to place, non-pruritic, l Less common findings are second degree AV block and
not indurated and blanch on pressure. They are brought some other non-specific changes.
on by application of heat.
l They are most commonly seen on the trunk and proxi-
mal parts of extremities, but never on face.
DIagnosis of Acute Rheumatic Fever
l For diagnosis, modified T. Duckett-Jones criteria are
Chorea (Sydenham’s chorea; Chorea minor; applied (Revised Jones criteria).
Saint Vitus’ dance)
l Chorea usually appears after a long latent period (up to Management of Acute Rheumatic Fever
6 months) after the initial streptococcal infection.
l Chorea is a manifestation of central nervous system in- i. Bed rest:
volvement in ARF. This is also known as Sydenham’s l Patients who have not had carditis should be advised

chorea or St. Vitus dance. bed rests until temperature and erythrocyte sedi-
l It is characterized by involuntary purposeless movement mentation rate (ESR) are normal.
of hands, feet or face. Chorea is a late manifestation in l Patients who have had carditis should continue to

the course of ARF and is more common in females. have bed rest for 2-6 weeks after the ESR and tem-
perature have returned to normal.
Other clinical manifestations of rheumatic fever ii. Antistreptococcal therapy
l Other clinical manifestations include fever, arthralgia, l A course of antibiotic should be given to eradicate

abdominal pain and epistaxis. the streptococci, even if the throat culture is negative.
l The average duration of an untreated attack of acute l One of the following regimens may be used:

rheumatic fever is approximately 3 months. a. Single injection of benzathine penicillin 1.2 million
l Chronic rheumatic fever, generally defined as disease per- intramuscularly.
sisting for longer than 6 months, occurs in less than 5% of b. Daily injection of procaine penicillin 6, 00, 000
cases and is a cause of persisting congestive heart failure. units intramuscularly for 10 days.
c. Oral erythromycin 20-40 mg/kg/day in three di-
vided doses, in patients who are sensitive to
Laboratory Features penicillin.
l The blood examination reveals leucocytosis, raised ESR iii. Salicylates:
and CRP. l Aspirin is effective in providing symptomatic relief.

l Throat swab culture may be positive for group A strep- l Aspirin is given in 6 divided dosages of 60 to 120

tococci. mg/kg per day to relieve the arthritis.


l Anti-streptolysin O (ASO) titer is raised to .200 units l Aspirin at this dose should be continued until the

in adults or .300 unit in children. Other antibodies like ESR is normal, and then gradually tapered over
anti-deoxyribonuclease and anti-hyaluronidase may 4-6 weeks.
also be helpful in the diagnosis. iv. Corticosteroids:
l Antistreptozyme (ASTZ) test is a very sensitive indica- l Prednisolone (1-2 mg/kg/ day) is given to, patients

tor of recent streptococcal infection. Titres more than with severe carditis with CHF or severe arthritis
200 units/mL are considered positive. The real value of alone. As the patients improve, steroid should be
ASTZ test is in ruling out rheumatic fever. tapered and salicylates added.
l These four tests when combined together give an accu- l To prevent a ‘post-steroid rebound’ an ‘overlap’

racy rate of more than 95%. course of aspirin may be added when the steroid is
l The chest radiograph, ECG, and echocardiography are being tapered off. Aspirin is then continued for an
helpful in the diagnosis of carditis. additional 2-3 weeks.
Section | I General Medicine 65

v. Supportive therapy: Aaetiology


l Includes treatment of congestive heart failure, val-
i. Diseases of lung
vular lesions, heart blocks and chorea.
l Chronic obstructive pulmonary disease (chronic bronchitis
and emphysema)
Prevention of Rheumatic Fever l Chronic bronchial asthma
Prevention of rheumatic fever includes: l Pulmonary tuberculosis
l Primary prevention–prevention of initial rheumatic l Interstitial lung disease
attacks. l Diffuse bronchiectasis
l Secondary prevention–prevention of recurrence of rheu- l High altitude dwelling
matic fever. l Cystic fibrosis
l Pleural fibrosis
Primary prevention ii. Diseases of pulmonary circulation
l Primary prevention can be summarised as ‘accurate di- l Recurrent pulmonary thromboembolism
agnosis and treatment of group A streptococcal pharyn- l Primary pulmonary hypertension
geal infection’. l Collagen vascular diseases
l An outbreak of rheumatic fever in a closed population is l Chronic liver disease
best treated by mass penicillin prophylaxis.
l Established streptococcal pharyngitis can be treated by iii. Diseases of thorax
benzathine penicillin or oral penicillin or erythromycin. l Kyphoscoliosis
l Neuromuscular diseases
Secondary prevention (rheumatic fever l Sleep apnoea syndrome
prophylaxis) l Obesity
l Rheumatic fever prophylaxis should be given to all
patients who have experienced a documented attack of iv. Chronic obstructive pulmonary disease (COPD)
rheumatic fever. l COPD including chronic bronchitis and emphysema
l Duration of prophylaxis is controversial. Broad outlines are: causes more than 50% cases of chronic cor pulmonale.
i. Those under the age of 18 years should receive con- l Patients with chronic bronchitis develop erythrocytosis,
tinuous prophylaxis. oedema and early onset of cor pulmonale (blue bloaters).
ii. Those who are over 18 years who develop rheumatic l Patients with emphysema develop cor pulmonale later
fever without carditis should receive prophylaxis for (pink pffers).
a minimum period of 5 years. l Chronic bronchial asthma rarely leads to chronic cor
l Decision to continue prophylaxis beyond 5 years in pulmonale.
the second group depends on many variables like age
of the patient, relative risk of acquiring infection, Pathogenesis
socioeconomic state, presence of rheumatic heart
disease, etc. l Increased pulmonary vascular resistance and pulmonary
l Regimens: One of the following regimens may be used: hypertension are the central mechanisms in all cases of
a. Intramuscular injection of 1.2 million units of benza­ chronic cor pulmonale.
thine penicillin G every 3 weeks (most efficient regimen).
b. Oral penicillin V 250 mg twice a day. Clinical Features
c. Sulphadiazine 1 g/day orally as a single dose (in Symptoms
those allergic to penicillins).
d. Erythromycin 250 mg twice a day orally (in those There is no specific symptom for chronic cor pulmonale.
allergic to penicillins and sulpha). Following are few symptoms:
i. Dyspnoea due to pulmonary hypertension, not relieved
Q. 6. Describe the aetiopathogenesis and clinical profile by sitting up.
of chronic cor pulmonale. ii. Dry cough
Ans. iii. Atypical anterior chest pain.
iv. Excercise-induced peripheral cyanosis.
l Chronic cor pulmonale is defined as a combination of
hypertrophy and dilatation of the right ventricle second- Signs
ary to pulmonary hypertension, which results from dis- i. Tachypnoea
eases of lung, pulmonary circulation or thorax. ii. Ankle oedema
66 Quick Review Series for BDS 3rd Year

iii. Prominent a waves on JVP. l Character of the pain is squeezing, crushing or aching.
iv. Right ventyricular heave in left parasternal region and Pain commonly radiates to left arm, or less commonly
epigastrium. to right arm, throat, back, chin and epigastrium. Often,
v. Pulmonary ejection sound the pain comes on while walking uphill after a heavy
vi. Ejection systolic murmur at pulmonary area. meal, on a cold winter day.
vii. Pansystolic murmur of tricuspid regurgitation, with a l Angina decubitus is pain while lying flat.
high pulmonary arterial pressure. l Nocturnal angina is an unusual form of angina occur-
viii. Early diatolic murmur of pulmonary regurgitation. ring in aortic regurgitation. It is characterised by parox-
ysmal, nocturnal anginal pain associated with night-
Additional clinical features in chronic bronchitis mares, dyspnoea, palpitations, skin flushing, profuse
(blue bloaters) sweating and wide pulse pressure. It does not respond to
l Chronic cough with sputum production sublingual nitroglycerin.
l Frequent mucopurulent exacerbations. l Prinzmetal’s angina or variant angina is pain which
l Secondary erythrocytosis. comes capriciously due to coronary arterial spasm and
l Repeated episodes of right heart failure and respiratory is accompanied by transient ST segment elevation on
failure. ECG.

Additional clinical features in emphysema (pink puffers) On examination


l Severe exertional dyspnoea. l Physical examination is usually negative, but may find
l Cough and sputum production are considerably less tendon xanthomas, thickening of Achilles tendons, ar-
prominent. cus lipidis in a young patient, aortic valve disease, dia-
l Erythrocytosis is uncommon betes, peripheral vascular disease, thyroid disease or
l Right heart failure and respiratory failure occur as a obesity.
terminal event.
The following physical signs are those of ‘myocardial
Q. 7. Describe signs and symptoms of angina and how ischaemia’. The presence of one or more of them during an
would you manage the case. attack of pain may be suggestive:
Or l Rise in blood pressure and heart rate
l Fourth heart sound
Discuss in detail the clinical features, investigations, l Murmur of mitral regurgitation due to papillary muscle
management and prevention of angina pectoris. dysfunction
Ans. l Dyskinetic segment around the apex
l Paradoxical splitting of second heart sound
l Angina pectoris is a clinical syndrome of discomfort l Relief of pain by carotid sinus massage (Levine test).
due to transient myocardial ischaemia.
l Transient myocardial ischaemia is due to one or both of
the following: Investigations
i. Obstruction of coronary flow by atheroma. i. Electrocardiography: ECG is normal in most patients at
ii. Coronary arterial spasm. rest and in between attacks. Most convincing evidence
l Angina is worsened by factors which increase myocardial is demonstration of reversible ST segment depression or
oxygen requirement or reduce supply. elevation, with or without T wave inversion during an
l Exercise attack of pain. Patient may require exercise testing, e.g.
l Anaemia treadmill testing or bicycle ergometry.
l Tachycardia ii. Myocardial perfusion scanning using radioactive thallium.
l Hypertension iii. Echocardiography and radionuclide blood-pool scan-
l Hyperthyroidism ning provide information about ventricular function.
l Aortic stenosis iv. Coronary arteriography provides detailed information
l Aortic regurgitation about the extent and site of coronary artery stenosis.
l Arrhythmias.

Management
Clinical Features
Management of angina pectoris involves three phases:
History A. Assessment of severity of symptoms and extent of the
l Pain is usually retrosternal in location and brought on disease.
by exertion. It is relieved by rest and sublingual nitrates. B. Measures to control symptoms.
Pain seldom lasts more than 20 minutes. C. Measures to improve life expectancy.
Section | I General Medicine 67

The management of angina pectoris can be dealt under l Cardioselective beta-blockers like metoprolol and aten-
three headings: olol have fewer peripheral side effects.
I. General measures. l Beta-blockers should not be withdrawn abruptly, as
II. Drug treatment. there is the risk of dangerous arrhythmias and myocar-
III. Surgical treatment. dial infarction.

I. General measures c. Calcium antagonists:


l Proper explanation about the disease. l They inhibit the slow inward current caused by the entry
l Avoid walking after meals, particularly in cold, against of extracellular calcium through cell membrane of ex-
a wind. cited cells, particularly arteriolar smooth muscle and
l Avoid unaccustomed strenuous exercises, stop smoking cardiac atrial cells.
and reduce weight. l In arterioles, it results in vasodilatation and hence
l Hyperlipidaemia treated with diet and drugs (with the reduces blood pressure. In cardiac muscle, it reduces
goal of reducing LDL ,100 mg/dl). excitability and conductivity. Eg:
l Control of hypertension and diabetes. l Nifedipine is a powerful coronary and systemic arte-

l Correction of precipitating conditions such as anaemia, riolar dilator.


valvular disease and arrhythmias. l Verapamil is given at a dose of 40-80 mg thrice daily.

l It has a negative inotropic effect and should be avoided


II. Drug treatment in patients with impaired ventricular function.
l Four groups of drugs are used in the management of l Nicardipine and diltiazem are other calcium antagonists.

angina pectoris: Calcium blockers are indicated in the following situations:


a. Nitrates. l Response to beta-blockers inadequate.
b. Beta adrenoceptor antagonists (beta-blockers). l History of asthma or chronic obstructive airway dis-
c. Calcium antagonists. ease, or peripheral vascular disease where beta-
d. Platelet inhibitors. blockers should be avoided.
l Sick-sinus syndrome or significant atrioventricular
a. Nitrates: blocks.
l Fresh glyceryl trinitrate (500 mg) sublingually relieves l Prinzmetal’s angina.
pain in 2-3 minutes, and produces mild headache. once l Adverse effects to beta-blockers.
pain subsides, patient is instructed to spit out or swallow
the tablet. d. Antiplatelet agents:
l Isosorbide dinitrate has prolonged action (10-20 mg 3-6 i. Aspirin:
times day) and is given orally. Tolerance develops and l Unlike the other anti-anginal drugs discussed above
dose needs to be increased. It is important to have a which have been shown to ameliorate only symptoms
10- to 12-hour nitrate-free period to avoid tolerance. but may not reduce mortality, aspirin and lipid-lowering
Hence, doses are given in the morning and afternoon. agents have been shown to improve survival.
l Isosorbide mononitrate needs to be given once or twice l Dose is 75-150 mg/day.
a day. l Aspirin inhibits the synthesis of prostaglandins, notably
l Best use of GTN is prophylactically before exercise thromboxane A2, a potent vasoconstrictor and platelet
known as liable to produce pain. GTN acts by venous and activator.
arteriolar dilatation, which lowers blood pressure, re-
duces venous return to heart and dilates coronary vessels. ii. Clopidogrel:
l Not more than two tablets per hour should be used. Use l It is a platelet inhibitor which is useful in patients who
of GTN should be encouraged because physical activity cannot tolerate aspirin.
promotes the formation of collateral vessels. l It may have a synergistic effect when combined with
l GTN can be given sublingually, percutaneously as paste aspirin in patients with CAD.
or plaster, and slow-release buccal tablet. GTN is virtu-
III. Surgical treatment
ally ineffective when swallowed.
A. Coronary artery bypass grafting (CABG):
b. Beta-blockers: l Length of patient’s saphenous vein is anastomosed to
l They reduce myocardial oxygen demand by reducing the aorta at one end and to a coronary vessel distal to a
heart rate for a given level of exercise, reducing heart stenosis at the other.
rate response to anxiety and reducing myocardial l Ideal for left main coronary artery stenosis and signifi-
contractility. cant stenosis in all three major coronary vessels.
l Propranolol is started in a small initial dose (20 mg l Alternatively, intemal mammary artery can be used for
thrice daily) and gradually increased. grafting.
68 Quick Review Series for BDS 3rd Year

B. Percutaneous coronary interventions–Percutaneous l With the development of pulmonary hypertension, the


transluminal coronary angioplasty (PTCA): signs of pulmonary hypertension appear. Later, with
l It is the dilatation of coronary artery stenosis by a small reversal of the shunt (Eisenmenger’s syndrome), the
balloon introduced percutaneously via an arterial catheter. murmur of VSD disappears, and central cyanosis and
l Ideal for single vessel coronary disease. Dilatation can clubbing appear.
be repeated, if there is recurrence of symptoms.
l This is the treatment of choice for unstable angina when Investigations
rest pain recurs despite full medical treatment.
l An advancement is the placement of a stent at the site of l Chest radiography, electrocardiography, echocardiogra-
block which reduces the risk of re-occlusion. Recently phy, and if necessary, cardiac catheterisation.
stents coated with drugs (e.g. sirolimus) have been ad-
vocated to reduce the risk of re-occlusion. Complications
Q. 8. Discuss briefly the haemodynamics, clinical fea- l Congestive heart failure.
tures, investigations, complications and management of l Pulmonary hypertension.
congenital ventricular septal defect (VSD). l Eisenmenger’s syndrome.
Ans. l Right ventricular outflow tract obstruction.
l Aortic regurgitation.
l The interventricular septum has a membranous and a l Infective endocarditis.
muscular portion. Ventricular septal defect implies a
defect in the interventricular septum. Most of the ven-
tricular septal defects are ‘perimembranous’, i.e. at the Management
junction of membranous and muscular portions. l Small ventricular septal defects with small shunts require
l Maladie de Roger: Small ventricular septal defects pre- no treatment as they are likely to close spontaneously.
senting in older children as a loud pansystolic murmur l Operative correction is indicated in moderate to large
without other haemodynamic changes. Such defects defects with significant left to right shunt (pulmonary-
usually close spontaneously. to-systemic blood flow ratio .1.5: 1.0).
l In VSD, there is a moderate risk of developing carditis.
Haemodynamics Hence, endocarditis prophylaxis is mandatory before
any procedure.
l The blood flows from the left ventricle to the right ven-
tricle and then to the pulmonary artery. This ultimately Q. 9. Discuss the aetiology and clinical features of sub-
results in a volume overload of the ventricles, if the acute bacterial endocarditis. Add a note on the princi-
shunt is large. Pulmonary hypertension and reversal of ples of management of the same.
shunt occur at a later stage. Or
Describe the aetiology, clinical features, diagnosis and
Clinical Features treatment of infective endocarditis.
l It is the commonest congenital heart disease in paediatric Or
age group.
Describe the aetiology, pathogenesis and management
l Clinical features depend on the size of the defect and mag-
of infective endocarditis. Add a note on prophylaxis.
nitude of the shunt. The small VSD is generally asymp-
tomatic while large VSD causes symptoms of heart failure. Describe the causes, clinical features and management
l Increased incidence of recurrent respiratory infections. of sub-acute bacterial endocarditis.
l Failure to thrive.
Ans.
l Hyperdynamic precordium.
l Apex beat is shifted down and out and is hyperdynamic l Infective endocarditis is microbial infection of endothe-
in character. lium of the heart. The infection most commonly occurs
l Systolic thrill at the third or fourth left intercostal space. at the site of pre-existing endocardial damage in heart
l A harsh pan systolic murmur, best heard at the third or fourth valves (native or prosthesis) and endocardium of the
left intercostal space, but radiating all over precordium. chambers.
l Left ventricular third heart sound (S3) and a mid-dia- l Similar process involving arterial endothelium in arte-
stolic rumbling murmur (flow murmur) at the apex due riovenous shunts, patent ductus arteriosus and coarcta-
to increased flow across the mitral valve. tion of aorta is known as infective endarteritis.
Section | I General Medicine 69

Pathology l Echocardiography: This can reveal the site and size of


the vegetations, abscess formation, evidence of underly-
l The endothelium may get damaged due to high pressure
ing heart disease and heart failure.
jet injury as occurs in valvular or congenital heart dis-
l Transoesophageal echocardiography is a more sensitive
ease. The damaged surface of endothelium invites plate-
method as compared to transthoracic echocardiography.
let adhesion and aggregation and fibrin deposition (non-
l Other findings: These include normocytic, normochro-
bacterial thrombotic endocarditis).
mic anaemia, leucocytosis, high ESR and CRP, micro-
l Organisms which enter into the bloodstream through
scopic haematuria, and proteinuria.
mucosa, skin or sites of focal infection may colonize the
l Chest X-ray may show evidence of cardiomegaly and
platelets-fibrin deposit and form vegetation.
heart failure.
l The most common organism causing subacute endocar-
l Conduction defects may be observed on ECG recording.
ditis is Viridans group of streptococci. These are com-
mensals in the upper respiratory tract and may enter the
blood following brushing, chewing or dental proce- Diagnostic Criteria
dures. Staphylococcus aureus is the most common Duke’s criteria which are based on clinical, laboratory and
cause of acute endocarditis. echocardiographic findings are used for the diagnosis of
l Prosthetic valve endocarditis is generally due to coagu- endocarditis.
lase negative Staphy. epidermidis, a normal skin com-
mensal. HACEK group of gram-negative bacteria (Hae- Diagnosis of infective endocarditis (Modified
mophilus, Actinobacillus, Cardiobacterium, Eikenella, Duke’s criteria)
and Kingella) can also cause native valve endocarditis. Major criteria:
l Other uncommon organisms which cause endocarditis are
fungi (Candida), rickettsia, chlamydia, and anaerobes. i. Positive blood culture
l Typical organism from two cultures.

l Persistent positive blood cultures taken.12 hours apart


Clinical Manifestations l Three or more positive cultures taken over more than

l Endocarditis depending on manifestations and their 1 hour.


clinical course is classified into following types: ii. Endocardial involvement
i. Acute and l Positive echocardiographic findings of vegetations

ii. Subacute. l New valvular regurgitation

l General manifestations: Fever, weight loss, night sweats,


and weakness. Minor criteria:
l Cardiac manifestations: New murmur, heart failure, and i. Predisposing valvular or cardiac abnormality
heart blocks. ii. Intravenous drug misuse
l Extracardiac manifestations: Anaemia, clubbing, sple- iii. Pyrexia 38°C
nomegaly, petechial haemorrhages, Osler nodes, Jane- iv. Embolic phenomenon
way’s lesions (macular lesions over palm and soles), v. Vasculitic phenomenon
Roth spots on fundus examination, and subconjunctival vi. Blood cultures suggestive-organism grown but not
haemorrhages are main extracardiac manifestations. achieving major criteria
l Meningitis, embolic infarcts and intracranial bleeding vii. Suggestive echocardiographic findings:
due to rupture of mycotic aneurysms are neurological Definite endocarditis: two major, or one major and
presentations. three minor, or five minor criteria.
l Septic emboli may disseminate infection to distant or- Possible endocarditis: one major and one minor, or
gans such as skin, spleen, kidneys, bone and meninges. three minor criteria.
l Embolic events may also be associated with infarction
at various sites. Treatment:
l Immune complex deposition can lead to glomerulone-
phritis and haematuria. The principles of treatment are:
l The antibiotics should preferably be bactericidal.
l The antibiotics should be administered parenterally to
Investigations achieve high serum concentration since the vegetation is
l Blood culture: The most important test is blood culture avascular.
which tells about the organism and guides in antibiotic l The therapy is generally of prolonged duration.
therapy. Serological tests may be useful, if blood culture l The selection of antibiotics should be based on culture re-
is negative. ports and minimum inhibitory concentration (MIC) values.
70 Quick Review Series for BDS 3rd Year

l Empirical therapy may be initiated in acute severe cases Prophylaxis


after drawing blood samples for culture. The antibiotics
Patients with valvular and congenital heart disease who are
are later changed based on sensitivity reports, if necessary.
at high or moderate risk of endocarditis should receive pro-
l The treatment of infective endocarditis should be
phylactic antibiotics before undergoing any procedure
prompt and adequate. The list of antibiotics commonly
which may cause bacteraemia.
used, their dosage and indications are given below:
Antibiotic regimen for prophylaxis of endocarditis in
i. Penicillin G: 2-4 million units IV 4 hrly
adults at moderate or high risk is as follows:
ii. Gentamicin 1 mg/kg IV or IM 8 hrly
I. Oral cavity, respiratory tract, or oesophageal proce-
iii. Ceftriaxone 2 g IV OD
dures (in patients at high risk, administer a half dose
iv. Ampicillin/amoxycillin 2 g IV 4 hrly
after the initial dose).
v. Cefazolin 2 g IV 8 hrly
A. Standard regimen: Amoxycillin 2.0 g oral 1 hr. be-
vi. Vancomycin 15 mg/kg IV 12 hrly
fore procedure.
B. Inability to take oral medication: Ampicillin or amox-
Emperical Therapy icillin 2.0 g IV or IM within 30 min of procedure.
l Emperical therapy is given when the culture is negative C. Penicillin allergy:
or before culture and sensitivity reports are available. i. Clarithromycin 500 mg or azithromycin 500 mg
l Ceftriaxone plus gentamicin is given in subacute native oral 1 hr. before procedure
valve endocarditis. Vancomycin is added to the above ii. Cephalexin or cefadroxil 2 g oral 1hr before
regime in case of prosthetic valve endocarditis. procedure
iii. Clindamycin 600 mg oral 1 hr before procedure
D. Inability to take oral medication
Surgery i. Clindamycin 600 mg IV 30 min before procedure.
l Surgery is often needed in patients with prosthetic valve ii. Cefazolin 1.0 g IV or IM 30 min before procedure.
endocarditis and fungal endocarditis who have heart II. Genitourinary and gastrointestinal tract procedures:
failure due to valve damage, no response to antibiotics, Ampicillin, amoxycillin, gentamicin, and vancomycin
large vegetations, and abscess formation. are used.

SHORT ESSAYS
Q. 1. Classify causes of cyanosis. Pulmonary causes:
Ans. l Chronic bronchitis
l Interstitial lung disease
l Cyanosis is defined as a bluish discolouration of the l Pulmonary arteriovenous fistula
skin and mucous membranes, resulting from an in- l High altitude
creased amount of reduced haemoglobin (.5 g/dl), or
l Cirrhosis of liver
of haemoglobin derivatives in the capillary blood.
l Cyanosis is most marked in the lips, nail beds, ears and Due to abnormal haemoglobin derivatives
malar eminences.
l Methaemoglobinaemia
Types of cyanosis:
l Sulphaemoglobinaemia
l Cyanosis can be subdivided into:
a. Central cyanosis and
b. Peripheral cyanosis. Causes of Peripheral Cyanosis
l Central cyanosis is due to decreased arterial oxygen satura-
tion or the presence of an abnormal haemoglobin derivative. Due to diminished peripheral blood flow resulting
from reduced cardiac out put:
Causes of Central Cyanosis l Mitral stenosis
l Shock
Decreased arterial oxygen saturation l Congestive cardiac failure.
Cardiac causes:
l Fallot’s tetralogy Due to local vasoconstriction:
l Eisenmenger’s syndrome l Cold exposure
l Congestive cardiac failure. l Peripheral vascular disease
Section | I General Medicine 71

Q. 2. Syncope–definition and causes. l Primary pulmonary hypertension


l Pulmonary embolism
Ans.
l Left atrial ball valve thrombus or myxoma
l Syncope (fainting) comprises generalised weakness of l Tetralogy of Fallot.
muscles, loss of postural tone, inability to stand upright
and a loss of consciousness. Orthostatic syncope (postural syncope)
l ‘Faintness’ reflects the prodromal phase of ‘fainting’ l This occurs when the person suddenly gets up from a
(syncope) with sensation of impending loss of con- lying down position or stands still for a long time.
sciousness. The sequences of symptoms include in- l Basic mechanism is postural hypotension resulting from
creasing light headedness, blurring of vision proceeding loss of vasoconstrictor reflexes in the lower limb vessels.
to blackout, heaviness in the lower limb progressing to Common causes of orthostatic hypotension are:
swaying and loss of consciousness. l Physiological.
l Idiopathic orthostatic hypotension.
Mechanism l Drugs including antihypertensive drugs.
l Diabetic neuropathy and tabes dorsalis.
l Fainting (syncope) is due to cerebral ischaemia; more
l Haemorrhage.
specifically ischaemia to the brainstem.
l Vasodilators/volume depletion
l Faintness is due to cerebral ischaemia to a degree insuf-
l Angiotensin-converting enzyme inhibitors
ficient to impair consciousness. Cerebral ischaemia may
l Beta-blockers
be due to sudden vasodilatation, to a sudden fall in car-
l Calcium channel blockers
diac output or to both simultaneously.
l Diuretics
l Nitrates
Types of Syncope
Syncope associated with cerebrovascular disease
Vasovagal syncope
l Caused by occlusion in the large arteries in the neck.
l Mechanisms include reflex slowing of heart mediated l Physical activity can further reduce the blood flow to the
through vagus, and marked fall in arterial pressure and brainstem resulting in loss of consciousness.
peripheral vascular resistance.
l Causes include emotional stress, warm overcrowded Miscellaneous causes of syncope
room, sudden pain, mild blood loss, anaemia, fever and l Carotid sinus syncope (hypersensitive carotid sinus) is
fasting. common in elderly patients. It follows some form of
l Clinical features include a ‘prodromal phase’ character- compression on the carotid sinus as in turning the head
ised by nausea, sweating, yawning, epigastric distress, to one side, tight collar or shaving over the region of
tachypnoea, weakness and confusion. This is followed carotid sinus.
by faintness, pallor, coldness of hand and feet, and even- l Vagal and glossopharyngeal neuralgia.
tually loss of consciousness. l Micturition syncope is seen in elderly patients during or
l Physical examination reveals tachycardia during pro- after micturition, particularly after arising from the bed.
dromal phase and bradycardia later with low blood pres- l Cough syncope follows paroxysms of cough in elderly
sure and weak pulse. Consciousness is regained rapidly. patients with chronic bronchitis.
l Vasovagal syncope is common in the young people. l Hypoglycaemia.
l Hyperventilation and hysterical fainting.
Cardiac syncope
l Cardiac syncope is due to a sudden reduction in cardiac Q. 3. Classify congenital heart diseases.
output. This can occur when the heart is beating too fast Ans.
or too slow or when it is not beating at all.
l Congenital heart disease (CHD) may present in early
Common causes of cardiac syncope are as follows: childhood or may remain asymptomatic till adult life.
l Complete heart block (Stokes-Adams-Morgagni syn- l Its incidence is around 1% of live birth.
drome) l Congenital abnormalities are due to multifactorial envi-
l Paroxysmal tachycardias (supraventricular or ventricular) ronmental and genetic causes.
l Sick sinus syndrome l Rubella infection in mother is associated with patent
l Acute massive myocardial infarction ductus arteriosus (PDA) and pulmonary artery steno-
l Aortic stenosis (exertional syncope) sis. Use of drugs or toxins (alcohol) during pregnancy
l Hypertrophic cardiomyopathy (exertional syncope) can also lead to CHD. Chromosomal abnormalities
72 Quick Review Series for BDS 3rd Year

such as Turner’s syndrome and Down’s syndrome are l Meningitis, embolic infarcts and intracranial bleeding
associated with coarctation of aorta and septal defects, due to rupture of mycotic aneurysms are neurological
respectively. presentations.
l Septic emboli may disseminate infection to distant or-
The classification of congenital heart diseases is as follows:
gans such as skin, spleen, kidneys, bone and meninges.
I. Acyanotic:
l Embolic events may also be associated with infarction
a. Left to right shunt
at various sites.
l Ventricular septal defect (VSD)
l Immune complex deposition can lead to glomerulone-
l Atrial septal defect (ASD)
phritis and haematuria.
l Patent ductus arteriosus (PDA)

b. Without shunt Q. 5. Prophylaxis against infective endocarditis.


l Coarctation of aorta

l Congenital aortic stenosis


Ans.
l pulmonary stenosis The principles of treatment are:
l Congenital mitral stenosis l The antibiotics should preferably be bactericidal.
II. Cyanotic: l The antibiotics should be administered parenterally to
a. Increased pulmonary blood flow achieve high serum concentration since the vegetation is
l Complete transposition of great arteries avascular.
l Total anomalous pulmonary venous connection l The therapy is generally of prolonged duration.
b. Normal or decreased blood flow: l The selection of antibiotics should be based on culture re-
l Tetralogy of Fallot ports and minimum inhibitory concentration (MIC) values.
l Tricuspid atresia l Empirical therapy may be initiated in acute severe cases
l Pulmonary atresia after drawing blood samples for culture. The antibiotics
III. Others: are later changed based on sensitivity reports, if necessary.
a. Dextrocardia l The treatment of infective endocarditis should be prompt
b. Congenital complete heart block and adequate. The list of antibiotics commonly used,
their dosage and indications are given below:
Q. 4. Infective endocarditis. i. Penicillin G: 2-4 million units IV 4 hrly
ii. Gentamicin 1 mg/kg IV or IM 8 hrly
Or
iii. Ceftriaxone 2 g IV OD
Clinical signs of subacute bacterial endocarditis. iv. Ampicillin/amoxycillin 2 g IV 4 hrly
v. Cefazolin 2 g IV 8 hrly
Ans. vi. Vancomycin 15 mg/kg IV 12 hrly
l Infective endocarditis is microbial infection of endothe-
lium of the heart. The infection most commonly occurs at Prophylaxis
the site of pre-existing endocardial damage in heart Patients with valvular and congenital heart disease who are
valves (native or prosthesis) and endocardium of the at high or moderate risk of endocarditis should receive pro-
chambers. phylactic antibiotics before undergoing any procedure
which may cause bacteraemia
Antibiotic regimen for prophylaxis of endocarditis in
Clinical Manifestations adults at moderate or high risk is as follows:
l Endocarditis depending on manifestations and their I. Oral cavity, respiratory tract, or oesophageal procedures
clinical course is classified into following types: (in patients at high risk, administer a half dose after the
i. Acute and initial dose).
ii. Subacute. A. Standard regimen: Amoxycillin 2.0 g oral 1 hr. be-
l General manifestations: Fever, weight loss, night sweats, fore procedure.
and weakness. B. Inability to take oral medication: Ampicillin or amox-
l Cardiac manifestations: New murmur, heart failure, and icillin 2.0 g IV or IM within 30 min of procedure.
heart blocks C. Penicillin allergy:
l Extracardiac manifestations: Anemia, clubbing, spleno- i. Clarithromycin 500 mg or azithromycin 500 mg
megaly, petechial haemorrhages, Osler nodes, Jane- oral 1 hr. before procedure
way’s lesions (macular lesions over palm and soles), ii. Cephalexin or cefadroxil 2 g oral 1hr before
Roth spots on fundus examination, and subconjunctival procedure
haemorrhages are main extracardiac manifestations. iii. Clindamycin 600 mg oral 1 hr before procedure
Section | I General Medicine 73

D. Inability to take oral medication l Thyrotoxicosis


i. Clindamycin 600 mg IV 30 min before procedure. l Hypertrophic cardiomyopathy
ii. Cefazolin 1.0 g IV or IM 30 min before procedure. l Fallot’s tetralogy (cyanotic spells)
II. Genitourinary and gastrointestinal tract procedures: l Phaeochromocytoma
Ampicillin, amoxycillin, gentamicin, and vancomycin l Chronic open-angle glaucoma
are used. l Anxiety with somatic symptoms
l Portal hypertension
Q. 10. Acute left ventricular failure–clinical features
l Migraine prophylaxis
and treatment.
l Essential tremor
Q. 11. Right ventricular failure–clinical features.
Contraindications of Beta-blockers
Q. 12. Aortic incompetence–causes and cardinal signs. l COPD and asthma
l Cardiac failure
Q. 13. Angina pectoris–clinical features and treatment.
l Heart block
Q. 14. Clinical features and complications of mitral l Peripheral vascular disease
stenosis. l Diabetes mellitus (masks sympathetic signs of hypogly-
caemia)
Q. 15. Myocardial infraction–clinical features and man-
agement.
Propranolol
Q. 16. Complications of hypertension.
l Mechanism of action includes sympatholytic effect,
Q. 17. Hypertensive encephalopathy. antihypertensive effect, and relief of anxiety, palpitation
Or and angina.
l A large portion of the drug is destroyed in its first passage
Treatment of hypertensive encephalopathy. through liver.
l Hypertensive encephalopathy is characterised by a very l Treatment is started with 40 mg twice a day and gradu-
high blood pressure and neurological disturbances in- ally increased to 160 mg 6 hourly. Slow-release forms
cluding transient abnormalities in speech or vision, are given as a single daily dose.
paraesthesiae, disorientation, fits, loss of consciousness l Side effects–gastric disturbances, bradycardia, cardiac
and papilloedema. failure, bronchospasm, tiredness, bad dreams, halluci-
l Neurological deficit is fully reversible, if the hyperten- nations, cold hands and muscle weakness.
sion is properly controlled.

Metoprolol and Atenolol


Treatment
1. Cardioselective beta-1 antagonists, with mechanism of
l A controlled reduction of blood pressure, over a period of action similar to that of propranolol. They have a greater
30-60 minutes to a level of 150/90 mmHg is adequate. Too effect on the cardiac beta-1 receptors than on the beta-2
rapid fall in blood pressure might cause cerebral ischaemia, receptors (beta-2 receptors subserve bronchodilatation
blindness, myocardial infarction or renal insufficiency. and vasodilatation).
l Intravenous sodium nitroprusside (0.3-1.0 mcg/kg/min) 2. Dose–metoprolol 50 mg twice daily to 100 mg thrice
is the most effective drug. It has to be used very care- daily and atenolol 50-100 mg once daily.
fully in an intensive care set-up. 3. They may be used in hypertensive patients who have mild
l Alternatively, parenteral labetalol, hydralazine or nitro- airway obstruction (COPD, asthma), peripheral vascular
glycerin may be used. Bed rest, sedation and diuretics. disease and type 1 diabetes; however, caution is required.
Q. 18 Treatment of primary pulmonary hypertensions. 4. Other indications–ischaemic heart disease and supra-
ventricular tachycardia.
Q. 19. Beta blockers and its indications. l Side effects–same as for propranolol, but addition-

Beta adrenoceptor antagonists (beta-blockers) ally cause hyperkalaemia.

Uses of Beta-blockers
Labetalol
l Angina pectoris
l Acute myocardial infarction and post-myocardial in- l Mechanism of action is the same as that of propranolol.
farction period (to prevent re-infarction) This has a combined alpha and beta adrenoceptor
l Cardiac arrhythmias antagonistic action. Dose100-200 mg twice daily. Side
l Hypertension effects–same as of propranolol.
74 Quick Review Series for BDS 3rd Year

Q. 20. Discuss the clinical features, complications and l Cardiac arrhythmias


management of essential hypertension. l Supervening heart diseases like myocarditis, infective
endocarditis, myocardial infarction
Or l Systemic hypertension
l Pulmonary embolism
Discuss the management of essential hypertension.
l Drugs like beta-blockers, disopyramide, corticosteroids
Precipitating and aggravating causes of heart failure. and NSAIDs
l Excess salt intake
l Infections l Physical and emotional stress
l Anaemia, pregnancy, thyrotoxicosis l Poor compliance with therapy.

SHORT NOTES
Q. 1. Dyspnoea. amount of reduced haemoglobin (.5 g/dl), or of haemo-
globin derivatives in the capillary blood.
Ans.
l Cyanosis is most marked in the lips, nail beds, ears and
i. Dyspnoea is an abnormally uncomfortable awareness malar eminences.
of breathing. Dyspnoea of cardiac origin may vary in l Types of cyanosis: Cyanosis can be subdivided into:
severity from an uncomfortable awareness of breathing a. Central cyanosis and
to a frightening sensation of ‘fighting for breath’. b. Peripheral cyanosis.
ii. The sensation of dyspnoea originates in the cerebral cortex, l Central cyanosis is due to decreased arterial oxygen
and although the precise pathways that mediate it remain saturation or the presence of an abnormal haemoglobin
poorly defined, they include stimuli arising from receptors derivative.
in the lungs, upper airways and respiratory muscles.
iii. There are several causes of cardiac dyspnoea: Acute Causes of central cyanosis:
left heart failure, chronic heart failure, arrhythmia and l Decreased arterial oxygen saturation.
angina equivalent.
Q. 2. Heart Failure–three causes. Cardiac causes:
l Fallot’s tetralogy
Ans. l Eisenmenger’s syndrome
l Heart failure is an imprecise term used to describe the l Congestive cardiac failure.
state that develops when the heart cannot maintain an
adequate cardiac output or can do so only at the expense Pulmonary causes:
of an elevated filling pressure. l Chronic bronchitis
l In the mildest forms of heart failure, cardiac output is l Interstitial lung disease
adequate at rest and becomes inadequate only when the l Pulmonary arteriovenous fistula
metabolic demand increases during exercise or some l High altitude
other form of stress. l Cirrhosis of liver

Due to abnormal haemoglobin derivatives:


Causes l Methaemoglobinaemia
i. Reduced ventricular contractility l Sulphaemoglobinaemia
ii. Ventricular outflow obstruction Q. 4. Atrial tachycardia.
iii. Ventricular inflow obstruction
iv. Ventricular volume overload Ans.
v. Arrhythmias l Atrial tachycardia may be a manifestation of increased
vi. Diastolic dysfunction atrial automaticity, sinoatrial disease or digoxin toxicity.
Q. 3. Causes of central cyanosis. l It produces a narrow complex tachycardia with abnor-
mal P-wave morphology, sometimes associated with
Ans. atrioventricular block, if the atrial rate is rapid.
l Cyanosis is defined as a bluish discolouration of the skin l It may respond to b-blockers, which reduce automatic-
and mucous membranes, resulting from an increased ity, or class I or III antiarrhythmic drugs.
Section | I General Medicine 75

l The ventricular response in rapid atrial tachycardias l The chosen drug regimen should be sufficient to kill the
may be controlled by AV node-blocking drugs. Catheter likely organism, and should be given shortly before the
ablation therapy can be offered to patients with recur- anticipated bacteraemia in order to reduce the risk of
rent or drug-resistant atrial tachycardia. resistance.
Q. 5. Management of postural hypotension. Q. 8. Chronic constrictive pericarditis.

Ans. Ans.

Management of postural hypotension consists: l Constrictive pericarditis is due to progressive thickening,


l Correction of dehydration fibrosis and calcification of the pericardium.
l Tilt up the head of the bed
l Support stockings (older patients may struggle to get Clinical features
these on) l Fatigue, rapid, low-volume pulse
l Non-steroidal anti-inflammatory drugs (increase circu- l Elevated jugular venous pulse (JVP) with a rapid descent
lating volume due to salt and water retention) l Kussmaul’s sign (a paradoxical rise in the JVP during
l Fludrocortisone (causes salt and water retention but inspiration)
poorly tolerated due to cardiac failure) l Loud early third heart sound or ‘pericardial knock’
l Hepatomegaly
Q. 6. Cardiac arrest–The Chain of Survival. l Ascites
Ans. l Peripheral oedema
l Pulsus paradoxus.
l Cardiac arrest describes the sudden and complete loss of l A chest X-ray shows pericardial calcification and echo-
cardiac output due to asystole, ventricular tachycardia or cardiography often help to establish the diagnosis.
fibrillation, or loss of mechanical cardiac contraction. l CT and MRI are also useful techniques for imaging the
l The term Chain of Survival refers to the sequence of pericardium. Final diagnosis may depend on complex
events that are necessary to maximise the chances of a echo-Doppler studies and cardiac catheterisation.
cardiac arrest victim surviving.
l A victim of cardiac arrest is most likely to survive, if all
links in the chain are strong, i.e. if the arrest is witnessed, Management
help is called immediately, basic life support is adminis- l Surgical resection of the diseased pericardium can lead
tered by a trained individual, the emergency medical ser- to a dramatic improvement but carries a high morbidity
vices respond promptly, and defibrillation is achieved produces disappointing results in up to 50% of patients.
within a few minutes. Good training in both basic and ad-
vanced life support is essential to the practice of medicine. Q. 9. Treatment of acute rheumatic fever.
l Basic life support (BLS): BLS encompasses manoeu- Ans.
vres that attempt to maintain a low level of circulation
until more definitive treatment with advanced life sup- Treatment of the acute rheumatic fever consists of:
port can be given. i. A single dose of benzyl penicillin 1.2 million U i.m. or oral
l Advanced life support (ALS): ALS aims to restore normal phenoxymethylpenicillin 250 mg 6-hourly for 10 days
cardiac rhythm by defibrillation when the cause of cardiac should be given on diagnosis to eliminate any residual
arrest is due to a tachyarrhythmia, or to restore cardiac out- streptococcal infection.
put by correcting other reversible causes of cardiac arrest. ii. If the patient is penicillin-allergic, erythromycin or a
cephalosporin can be used.
Q. 7. Prevention of infective endocarditis. iii. Treatment is then directed towards limiting cardiac
damage and relieving symptoms.
Ans.
iv. Bed rest and supportive therapy:
l Patients with valvular or congenital heart disease may l Bed rest is important as it lessens joint pain and

be susceptible to infective endocarditis. reduces cardiac workload. The duration of bed rest
l These individuals should be made aware of the risk of should be guided by symptoms and markers of in-
endocarditis, the need to avoid bacteraemia and the flammation (e.g. temperature, leucocyte count and
importance of maintaining good dental health. ESR) and should be continued until these have
l Any potential source of infection in susceptible indi- settled.
viduals should be treated promptly and invasive proce- l Patients can then return to normal physical activity,

dures that may cause transient bacteraemia, should be but strenuous-exercise should be avoided in those
accompanied by appropriate antibiotic prophylaxis. who have had carditis.
76 Quick Review Series for BDS 3rd Year

v. Aspirin: l Spiral CT of the chest with contrast will usually provide


l This will usually relieve the symptoms of arthritis a definitive diagnosis and is preferable to invasive pul-
rapidly and a prompt response within 24 hours helps monary angiography, which may be hazardous.
to confirm the diagnosis. l Treatment is with high-flow oxygen and anticoagulation
l A reasonable starting dose is 60 mg/kg body weight with low molecular weight heparin.
per day, divided into six doses. In adults, 100 mg/kg l Thrombolytic therapy is valuable in selected cases with
per day may be needed up to the limits of tolerance severe haemodynamic compromise, and surgical embo-
or a maximum of 8 g per day. lectomy may be needed on rare occasions.
vi. Corticosteroids:
Q. 12. Syncope.
l They produce more rapid symptomatic relief than

aspirin, and are indicated in cases with carditis or Ans.


severe arthritis.
l Syncope is defined as a transient loss of consciousness
Q. 10. Diagnosis of infective endocarditis. due to diminished cerebral perfusion.
Ans. l Syncope may be preceded by symptoms of ‘presyncope’
such as lightheadedness, visual blurring, dizziness,
Diagnosis of infective endocarditis (modified Duke criteria): sweating and nausea.
l Syncope is characterized by loss of postural control and
Major Criteria spontaneous recovery.

i. Positive blood culture


l Typical organism from two cultures. Aetiology
l Persistent positive blood cultures taken .12 hours
a. Cardiovascular disorders: Arrhythmias, sinus bradycar-
apart dia, heart blocks, ventricular and supraventricular tachy-
l Three or more positive cultures taken over more
cardia, aortic stenosis, hypertrophic obstructive cardio-
than 1 hour. myopathy, left ventricular dysfunction, etc.
ii. Endocardial involvement b. Disorders of vascular tone and blood volume: Vasova-
l Positive echocardiographic findings of vegetations
gal syncope, postural hypotension, carotid sinus sensi-
l New valvular regurgitation
tivity, cough or micturition syncope.
c. Cerebrovascular disease: Vertebrobasilar insufficiency,
Minor Criteria basilar artery migraine.

i.Predisposing valvular or cardiac abnormality


ii.Intravenous drug misuse Investigations
iii.Pyrexia 38°C l ECG, and electrophysiological studies may be needed
iv. Embolic phenomenon to diagnose the cause of syncope.
v. Vasculitic phenomenon l Upright tilt test is used to confirm the diagnosis of vaso-
vi. Blood cultures suggestive-organism grown but not vagal syncope.
achieving major criteria l Other tests like EEG, CT, MRI scan may be needed to
vii. Suggestive echocardiographic findings diagnose any neurological cause.
Definite endocarditis: Two major, or one major and
three minor, or five minor criteria.
Possible endocarditis: One major and one minor, or Treatment
three minor criteria. The treatment depends upon the underlying cause.
Q. 11. Pulmonary embolism. Immediate actions to be taken during syncope are as
follows:
Ans. l The patient should be placed in supine position with
l Pulmonary embolism may complicate leg or pelvic vein head tilted to the side to maximize cerebral blood flow
thrombosis and usually presents with sudden collapse. and to avoid aspiration.
l Bedside echocardiography may be very helpful and usu- l Peripheral stimulation like sprinkling cold water over
ally demonstrates a small under filled vigorous left the face may help.
ventricle with a dilated right ventricle. l Clothing should be loosened.
l It is sometimes possible to see thrombus in the right l The patient should not be allowed to rise again till
ventricular outflow tract or main pulmonary artery. weakness persists.
Section | I General Medicine 77

Q. 13. Angina pectoris–aggrevating factors. l The non-cardiac causes of pulmonary oedema are:
Acute respiratory distress syndrome (ARDS), high alti-
Ans.
tude and narcotic overdose.
l Angina pectoris is a clinical syndrome of discomfort
due to transient myocardial ischaemia.
Clinical Features
l Angina is worsened or aggrevated by factors which in-
crease myocardial oxygen requirement or reduce supply: l There is rapid onset of dyspnoea, cough, anxiety and
l Exercise restlessness.
l Anaemia l Additionally, patients may have excessive sweating and
l Tachycardia cyanosis.
l Hypertension l The sputum may be profuse, pink frothy or blood
l Hyperthyroidism stained.
l Aortic stenosis l The examination may reveal cold extremities, tachycar-
l Aortic regurgitation dia, and tachypnoea with prominent use of accessory
l Arrhythmias. muscles of respiration. There are extensive crackles
l Anginal pain is usually retrosternal in location, seldom (crepitations) and wheezes on lung auscultation.
lasts more than 20 minutes and brought on by exertion.
Q. 16. Acute pericarditis–management.
It is relieved by rest and sublingual nitrates.
Ans.
Q. 14. Sinus bradycardia.
l Acute pericarditis or pericardial inflammation may be
Ans.
due to infection, immunological reaction, trauma or
l Sinus bradycardia is a sinus rate of less than 60/min, it neoplasm and sometimes remains unexplained.
may occur in healthy people at rest and is a common l The characteristic pain of pericarditis is retrosternal,
finding in athletes. radiates to the shoulders and neck and is typically ag-
l Some pathological causes of sinus bradycardia are as gravated by deep breathing, movement, a change of
follows: position, exercise and swallowing.
i. Cholestatic jaundice
ii. Raised intracranial pressure
Management of Acute Pericarditis
iii. Drugs, e.g. blocker, digoxin, verapamil
iv. Myocardial infarction l The pain is usually relieved by aspirin (600 mg 4-hourly),
v. Sinus node disease (sick sinus syndrome) but a more potent anti-inflammatory agent such as indo-
vi. Hypothermia methacin (25 mg 8-hourly) may be required.
vii. Hypothyroidism l Corticosteroids may suppress symptoms but there is no
l Asymptomatic sinus bradycardia requires no treatment. evidence of cure.
l Symptomatic sinus bradycardia usually responds to in- l In viral pericarditis, recovery usually occurs within a
travenous atropine 200/min, during intense exercise. few days but there may be recurrences.
l Purulent pericarditis requires treatment with antimicro-
Q. 15. Acute pulmonary oedema–mention two causes
bial therapy, paracentesis and, if necessary, surgical
and clinical features.
drainage.
Ans.
Q. 17. Atrial fibrillation–causes.
l Pulmonary oedema is characterized by the accumula-
Ans.
tion of fluid in the pulmonary interstitium and alveoli
due to increase in the pulmonary venous and capillary l Atrial fibrillation (AF) is the most common sustained
hydrostatic pressure. cardiac arrhythmia.
l The lungs become less compliant, airways resistance l AF can be classified as paroxysmal (intermittent, self
increases, and the capillary-alveolar gas exchange is terminating episodes), persistent or permanent.
compromised. l Common causes of atrial fibrillation are as follows:
i. Coronary artery disease.
ii. Valvular heart disease, especially rheumatic mitral
Causes valve disease
l The important cardiac causes of acute pulmonary iii. Hypertension
oedema are left ventricular failure (acute MI, acute MR, iv. Sinoatrial disease
and acute AR) and mitral stenosis. v. Hyperthyroidism
78 Quick Review Series for BDS 3rd Year

vi. Cardiomyopathy ii. Chest X-ray


vii. Congenital heart disease l Enlarged left atrium

viii. Pulmonary embolism l Signs of pulmonary venous congestion

ix. Pericardial disease iii. Echo


x. Idiopathic (lone AF) l Thickened immobile cusps

l Reduced valve area


Q. 18. Atrial septal defect
l Reduced rate of diastolic filling of LV

Ans. iv. Doppler


l Pressure gradient across mitral valve
l Atrial septal defect (ASD) is a defect in the interatrial
l Pulmonary artery pressure
septum.
l Left ventricular function

v. Cardiac catheterisation
Clinical Features l Assessment of coexisting coronary artery disease

l It is the commonest congenital heart disease seen in and mitral regurgitation


adults. Q. 20. Bradycardia.
l More common in females.
l Recurrent respiratory infections. Ans.
l Retarded growth and development. l A heart rate of less than 60 / minute is called a bradycardia.
l Palpitations, dyspnoea, cardiac failure and arrhythmias l Bradycardia may be due to either reduced automaticity
like atrial fibrillation occur late in the course of illness. (e.g. sinus bradycardia) or blocked or abnormally slow
conduction (e.g. atrioventricular block).
Signs l Some pathological causes of sinus bradycardia are:
l Cholestatic jaundice
l Hyperdynamic precordium.
l Raised intracranial pressure
l Visible and palpable pulmonary artery pulsations in the
l Drugs, e.g. blocker, digoxin, verapamil
second left intercostal space.
l Myocardial infarction
l Sl is loud, S2 is widely split and fixed in relation to res-
l Sinus node disease (sick sinus syndrome)
piration.
l Hypothermia
l Systolic thrill, ejection systolic murmur (ESM) over the
l Hypothyroidism
pulmonary area due to increased flow across the pulmo-
nary valve. Q. 21. Collapsing pulse.
l Mid-diastolic trumbling murmur over the tricuspid area
Ans.
due to increased flow across the tricuspid valve.
l Signs of pulmonary hypertension develop. l Collapsing (water-hammer) pulse is a pulse characterised
by a rapid upstroke, rapid downstroke and a high volume.
Investigations l Mechanism: The rapid upstroke is due to the markedly
increased stroke volume. The rapid downstroke or the
l Chest radiography, electrocardiogram, echocardiography collapsing character is due to two factors:
and if necessary cardiac catheterisation. i. The diastolic leak back into left ventricle.
ii. Rapid run-off to the periphery because of low sys-
Management temic vascular resistance.
l Significance of collapsing pulse is that it seen typically
l Prompt treatment of respiratory infections.
in aortic regurgitation.
l Surgical closure of the defect in patients above 3 years
l Similar pulse may also be seen in other conditions
of age, provided there are no signs of pulmonary hyper-
like patent ductus arteriosus, ruptured sinus of Valsalva,
tension and the pulmonary flow is 50% more than the
large arteriovenous fistulae, aortic regurgitation, hyper-
systemic blood flow.
kinetic circulatory states (anaemia, fever, thyrotoxico-
Q. 19. Investigations in mitral stenosis sis, exercise).
Ans. Q. 22. Aortic regurgitation–five peripheral signs.
Investigations in mitral stenosis are as follows: Ans.
i. ECG
Peripheral signs of aortic regurgitation are as follows:
l Left atrial hypertrophy
i. Collapsing pulse (water-hammer pulse; Corrigam’s
l Right ventricular hypertrophy
pulse): Best appreciated on the radial artery when
Section | I General Medicine 79

the arm is elevated. This is characterised by a rapid B. Inability to take oral medication: Ampicillin or
upstroke, rapid downstroke and a high volume. amoxicillin 2.0 g IV or IM within 30 min of pro-
ii. Pulsus bisferiens: It is best felt in the carotids. This is a cedure.
pulse with double peak (two peaks), both being felt in C. Penicillin allergy:
systole. It is seen in severe aortic regurgitation. i. Clarithromycin 500 mg or azithromycin 500 mg
iii. Wide pulse pressure: Results from a low diastolic pres- oral 1 hr. before procedure
sure from aortic run-off and high systolic pressure from ii. Cephalexin or cefadroxil 2 g oral 1hr before
increased stroke volume. procedure
iv. Corrigan’s neck sign: Prominent carotid pulsations vis- iii. Clindamycin 600 mg oral 1 hr before procedure
ible in the neck. D. Inability to take oral medication
v. De-Musset’s sign: To-and-fro motion of the head syn- i. Clindamycin 600 mg IV 30 min before procedure.
chronous with cardiac cycle. ii. Cefazolin 1.0 g IV or IM 30 min before procedure.
vi. Muller’s sign: Pulsations of the uvula. II. Genitourinary and gastrointestinal tract procedures:
Ampicillin, amoxycillin, gentamicin, and vancomycin
Q. 23. Mid-diastolic murmur.
are used.
Ans.
Q. 26. Aspirin.
l Mid-diastolic murmur (Austin Flint murmur): A mid-
diastolic trumbling murmur is heard at the apex in Ans.
severe AR. i. Acetyl salicylic acid or aspirin is a derivative of non-
l This is a flow murmur, and is not due to organic mitral selective COX inhibitors.
stenosis. ii. Pharmacological actions of acetyl salicylic acid or As-
l A blowing, pansystolic murmur, often radiating to the pirin are:
axilla may be heard at the apex. It is due to marked dilata- l Analgesic, anti-inflammatory, anti-pyretic and
tion of left ventricle and functional mitral regurgitation. Antiplatelet effects.
l It also acts on gastrointestinal tract, respiration and
Q. 24. Digoxin.
CVS.
Ans. l It acts on urate excretion and helps in acid-base and

i. Digoxin is a cardiac glycoside obtained from the seeds electrolyte balance.


of Strophanthus gratus and is the most widely used iii. Therapeutic uses or indications of aspirin are:
l As analgesic: It is used in headache, toothache, joint
cardiac glycoside.
ii. Pharmacokinetics: pains, dysmenorrhoea and myalgia in dosage 0.3 to
a. It has a plasma half-life of 5-7 days. 0.6 gm per day.
l As anti-pyretic: It is used in fever of any origin.
b. Its onset of action is in about 30-120 min.
l It is first drug to be used in acute rheumatic fever.
iii. Uses:
l It is used in rheumatoid arthritis and osteoarthritis.
a. Cardiac arrhythmias: Atrial fibrillation, atrial flutter,
l In myocardial infarction and stroke.
paroxysmal supraventricular tachycardia.
b. Congestive cardiac failure iv. Side effects of acetyl salicylic acid or aspirin are as fol-
iv. Adverse effects: lows: Nausea, vomiting, hypersensitivity reaction, epi-
a. Cardiac effects: Arrhythmias like extrasystoles, bra- gastric distress, gastritis, peptic ulcer, precipitation of
dycardia, pulse bigeminy, AV block asthma, salicylism and in children with viral infections,
b. Extra-cardiac effects: Anorexia, nausea, vomiting, di- it causes Reye’s syndrome.
arrhoea, weakness, confusion, hallucinations, blurred Q. 27. Oral anticoagulants.
vision and gynaecomastia.
Ans.
Q. 25. Infective endocarditis–prophylaxis.
l Oral anticoagulants are drugs given orally used to re-
Ans.
duce the coagulability of blood.
Antibiotic regimen for prophylaxis of endocarditis in adults l They may be classified as:
at moderate or high risk is as follows: i. Coumarin derevatives–bishydroxycoumarin, warfarin
I. Oral cavity, respiratory tract, or oesophageal proce- sodium, acenocoumarol, ethylbiscoumacetate
dures (in patients at high risk, administer a half dose ii. Indanedione derivatives–phenindione
after the initial dose). l They are used in vivo and not in vitro. This is because
A. Standard regimen: Amoxycillin 2.0 g oral 1 hr. they act indirectly by interfering with the synthesis of
before procedure. vitamin K dependent clotting factors in the liver.
80 Quick Review Series for BDS 3rd Year

l They behave as competitive antagonists of vitamin K but also cause arteriolar dilatation and as a result reduce
and reduce the plasma levels of functional clotting fac- both preload and afterload.
tors in a dose dependent manner. l Since nitroglycerine tablet disintegrates in liver its bio-
availability is decreased but when it is kept under tongue
Q. 28. Beta-blockers and their uses.
it is well absorbed from buccal mucosa. So when nitro-
Ans. glycerine tablet is given sublingually it is more effective.

Cardioselective beta blockers: Q. 31. Anti-anginal medication.


l Cardioselective beta blockers selectively block b1 recep- Ans.
tors and weakly b2 receptors. Eg: Atenolol, metoprolol
Anti-anginal medications are the drugs used in the treat-
and esmelol.
ment of anginapectoris. They are classified as follows:
l They are safe to use in diabetics as their inhibition of
glycogenolysis is low and incidence of bronchospasm is a. Nitrates:
less or negligible.
i. Short-acting:
l There is reduced chances of peripheral vascular diseases.
l Glycerol trinitrate (GTN, nitroglycerine)
l The exercise performance impaired to a lesser degree.
ii. Long-acting:
l They are used in the treatment of cardiac arrhythmias,
l Isosorbide dinitrate (short-acting by sublingual
angina pectoris and hypertension.
route)
l Isosorbide mononitrate
Non-selective beta-receptor antagonists.
l Erythrityl tetranitrate
l They include drugs like timolol, nadolol, sotalol, pro-
l Pentaerythritol tetranitrate
pranolol.
l They are used in the treatment of glaucoma, anxiety, b. Beta-blockers:
pheochromocytoma, thyrotoxicosis and prophylaxis of
l Propranolol
migraine.
l Metoprolol
Q. 29. Antiplatelet drugs mention any two. l Atenolol
Ans. c. Calcium channel blockers:
l Antiplatelet agents like aspirin and clopidogrel are anti- i. Phenyl alkylamine:
anginal drugs. l Verapamil

l Aspirin (and lipid-lowering agents) has been shown to ii. Benzothiazepine:


improve survival. l Diltiazem

l Aspirin inhibits the synthesis of prostaglandins, nota- iii. Dihydropyridines:


bly thromboxane, a potent vasoconstrictor and platelet l Nifedipine

activator. l Felodipine

l Dose is 75-150 mg/day. l Amlodipine


l Clopidogrel is a platelet inhibitor which is useful in l Nitrendipine

patients who cannot tolerate aspirin. l Nimodipine

l It may have a synergistic effect when combined with l Lacidipine

aspirin in patients with CAD.


d. Potassium channel openers:
Q. 30. Nitrates.
Nicorandil
Ans.
e. Others:
Organic nitrates are prodrugs and they are classified as:
Dipyridamole, trimetazidine, oxyphedrine.
A. Short-acting
l Glycerol trinitrate (nitroglycerine) Q. 32. Calcium channel blockers.
B. Long-acting
Ans.
l Isosorbide dinitrate

l Isosorbide mononitrate l Calcium channel blocking agents are the drugs which
l Erythrityl tetranitrate blocks voltage-sensitive calcium channels.
l Pentaerythritol tetranitrate l They have wide application in the diseases of cardiovas-
l The only major action of nitrates is direct nonspecific cular system. They are used as antiarrhythmic agents,
smooth muscle relaxation. They are mainly venodilators, antianginal drugs and antihypertensives.
Section | I General Medicine 81

l Classification of calcium channel blockers: Q. 35. Hypertensive retinopathy.


i. Phenyl alkylamine:
Ans.
l Verapamil

ii. Benzothiazepine: l Hypertensive retinopathy is an ophthalmic complication


l Diltiazem of hypertension.
iii. Dihydropyridines: l It is characterised by thickening of the walls of the reti-
l Nifedipine nal arterioles, diffuse or segmental narrowing of blood
l Felodipine columns, varying width of the light reflex from vessel
l Amlodipine walls, arteriovenous nipping, retinal haemorrhages, soft
l Nitrendipine and hard exudates and papilloedema.
l Nimodipine l Severe retinopathy can cause visual field defects and
blindness.
Q. 33. Treatment of deep vein thrombosis. l Grading of hypertensive retinopathy is as follows:
Grade I:Arteriolar narrowing and increase in light reflex
Ans.
over the arterioles.
l Deep vein thrombosis (DVT) means thrombus forma- Grade II: Marked arteriolar narrowing and arteriove-
tion in the deep venous system, especially of the lower nous nipping.
extremities and pelvis. Grade III: Grade II plus flame-shaped haemorrhages
l Treatment of DVT consists of: and soft exudates.
i. Bed rest with legs elevated to 15° Grade IV: Grade III plus papilloedema.
ii. Physiotherapy to legs. Q. 36. Give list of complications of hypertension.
iii. Graduated elastic stockings
iv. Start treatment with heparin and continue with war- Ans.
farin for a minimum period of 3 months or even Complications of hypertension are as follows:
longer. i. Central nervous system complications:
v. Low molecular weight heparins may be used in l Transient ischaemic attacks.
the place of conventional heparin in the initial l Cerebrovascular accidents (strokes).
period. l Subarachnoid haemorrhage.
vi. Thrombolysis with streptokinase in early stages l Hypertensive encephalopathy.
though its efficacy in preventing embolism is not ii. Ophthalmic complications
better compared to heparin. l Hypertensive retinopathy
vii. Thrombectomy (rarely). iii. Cardiovascular complications
l Coronary artery disease (angina, myocardial infarction).

Q. 34. Clinical features of infective endocarditis. l Left ventricular failure.

l Aortic aneurysm.
Ans. l Aortic dissection.
iv. Renal complications
Infective endocarditis is suspected in any patient with car-
l Proteinuria.
diac disease who develops persistent fever. The clinical
l Progressive renal failure.
manifestations are as follows:
v. Malignant hypertension
i. General features: Fever, weight loss, night sweats, and
l It is a clinical syndrome of markedly high blood pres-
weakness.
sure with retinal haemorrhages and exudates, and
ii. Cardiac symptoms: New murmur, heart failure, and
often including confusion, headache, vomiting, visual
heart blocks.
disturbances and deterioration in renal functions.
iii. Extracardiac: Anaemia, digital clubbing, splenomegaly,
petechial haemorrhages, Osler nodes, Janeway’s le- Q. 37. Jones criteria.
sions (macular lesions over palm and soles), Roth spots
Or
on fundus examination, and subconjunctival haemor-
rhages are main extracardiac manifestations. Diagnosis of rheumatic fever.
iv. Meningitis, embolic infarcts and intracranial bleeding
Ans.
due to rupture of mycotic aneurysms are neurological
presentations. l The diagnosis of acute rheumatic fever (ARF) is based on
v. Immune complex deposition can lead to glomerulone- updated Jone’s criteria. Presence of two or more major
phritis and haematuria. criteria or one major and at least two minor criteria plus
82 Quick Review Series for BDS 3rd Year

evidence of previous streptococcal infection is required l Angina is worsened by factors which increase myocar-
for the diagnosis of ARF. dial oxygen requirement or reduce supply.
l Exercise
Major criteria l Anaemia

l Polyarthritis l Tachycardia

l Carditis l Hypertension

l Chorea l Hyperthyroidism

l Subcutaneous nodules l Aortic stenosis

l Erythema marginatum l Aortic regurgitation

l Arrhythmias.
Minor manifestations
Q. 40. Left ventricular failure-four cardinal causes.
l Fever
l Arthralgia Ans.
l Raised ESR l Heart failure or cardiac failure is defined as a state, in
l Positive CRP which the ventricles at normal filling pressures (i.e. ad-
l Prolonged PR interval. equate intravascular volume) cannot maintain an ade-
And positive throat culture for streptococci or elevated quate cardiac output to meet the metabolic needs of
or increasing streptococcal antibody titer. peripheral tissues or can do so only with an elevated
Presence of two or more major criteria or one major and filling pressure.
at least two minor criteria plus evidence of previous strep- l ‘Left-side’ is a term for the functional unit of left atrium,
tococcal infection is required for the diagnosis of ARF. left ventricle, mitral valve and aortic valve.
l In left ventricular heart failure excess fluid accumulates
Q. 38. Vasovagal syncope. upstream behind the failing left ventricle. There is re-
Ans. duction in left ventricular output, increase in left atrial
pressure and increase in pulmonary venous pressure.
l Syncope (fainting) comprises generalised weakness of l Acute increase in left atrial pressure causes pulmonary
muscles, loss of postural tone, inability to stand upright congestion and pulmonary oedema, e.g. myocardial in-
and a loss of consciousness. farction.
l Mechanism of vasovagal syncope includes reflex slow- l Gradual increase in left atrial pressure causes reflex
ing of heart mediated through vagus, and marked fall in pulmonary hypertension, but no pulmonary oedema,
arterial pressure and peripheral vascular resistance. e.g. aortic stenosis.
l Causes include emotional stress, warm overcrowded
room, sudden pain, mild blood loss, anaemia, fever and Q. 41. Treatment of left ventricular failure.
fasting. Ans.
l Clinical features include a ‘prodromal phase’ character-
ised by nausea, sweating, yawning, epigastric distress, The overall management of heart failure is summarised as
tachypnoea, weakness and confusion. This is followed follows:
by faintness, pallor, coldness of hand and feet, and even- i. Physical and emotional rest.
l Absolute bed rest is only rarely required.
tually loss of consciousness.
l Small doses of tranquillizers may be used.
l Physical examination reveals tachycardia during prodro-
mal phase and bradycardia later with low blood pressure ii. Correction of obesity by restriction of caloric intake.
and weak pulse. Consciousness is regained rapidly. iii. Vasodilators.
l Vasovagal syncope is common in the young people. iv. Beta-blockers and ACE inhibitors.
v. Digoxin.
Q. 39. Coronary heart disease or angina pectoris–name vi. Sympathomimetic amines:
four risk factors. l Sympathomimetic amines improve myocardial con-

tractility in heart failure. They are dopamine, dobu-


Ans.
tamine, adrenaline isoproterenol (isoprenaline). Of
l Angina pectoris is a clinical syndrome of discomfort these drugs, dopamine and dobutamine are the most
due to transient myocardial ischaemia. effective.
l Transient myocardial ischaemia is due to either one or l Dopamine is useful in heart failure associated with
both of the following: hypotension.
i. Obstruction of coronary flow by atheroma. l Dobutamine is useful in acute heart failure without
ii. Coronary arterial spasm. hypotension.
Section | I General Medicine 83

vii. Amrinone is a phosphodiesterase inhibitor with positive l Pulmonary stenosis


inotropic and vasodilator actions. It acts by increasing l Congenital mitral stenosis
contractility and dilating systemic vascular bed. II. Cyanotic:
viii. Restriction of sodium intake. a. Increased pulmonary blood flow
l In mild heart failure, mild sodium restriction. l Complete transposition of great arteries

l In more severe heart failure, more rigid control of l Total anomalous pulmonary venous connection

sodium intake (1-2 g/day). b. Normal or decreased blood flow:


l In severe heart failure, sodium chloride intake l Tetralogy of Fallot

should be less than 0.5-1 g/day. l Tricuspid atresia

l Restriction of water intake is advised only in most l Pulmonary atresia

severe cases. III. Others:


ix. Diuretics a. Dextrocardia
x. Mechanical removal of fluid by thoracentesis, paracen- b. Congenital complete heart block
tesis and dialysis are measures used in severe cases.
Q. 43. Components of Fallot’s tertralogy.
Q. 42. Classification of congenital heart disease.
Ans.
Ans.
The tetralogy of Fallot consists of following 4 compo-
The classification of congenital heart disease: nents:
I. Acyanotic: i. Ventricular septal defect
a. Left to right shunt ii. Pulmonary stenosis
l Ventricular septal defect (VSD) iii. Overriding of VSD by aorta
l Atrial septal defect (ASD) iv. Right ventricular hypertrophy.
l Patent ductus arteriosus (PDA)
The pulmonary flow is reduced due to RV outflow ob-
b. Without shunt
struction. This results into shunting of desaturated blood
l Coarctation of aorta
from RV to LV across the VSD leading to cyanosis, club-
l Congenital aortic stenosis
bing and polycythemia.

Topic 6

Diseases of the Respiratory System


LONG ESSAYS
Q. 1. Describe the causes and management of bronchitis. mucopurulent, more copious and in tracheitis often
blood-stained.
Ans.
l Acute bronchial infection may be associated with a py-
rexia of 38–39°C and a neutrophil leukocytosis. Sponta-
A. Acute Bronchitis neous recovery occurs over a few days.
Clinical features
l Acute bronchitis often follows acute coryza.
l Initially irritating unproductive cough accompanied by Complications
retrosternal discomfort of tracheitis. l Bronchopneumonia.
l Chest tightness, wheeze and breathlessness when bron- l Exacerbation of chronic bronchitis which often results
chi become involved. in type II respiratory failure in patients with severe
l Tracheitis causes pain on coughing. Sputum is initially COPD.
scanty or mucoid. After a day or so sputum becomes l Acute exacerbation of bronchial asthma.
84 Quick Review Series for BDS 3rd Year

Management culture showing that has pathogenic microorganisms


l Specific treatment rarely necessary in previously healthy such as Streptococcus species.
individuals. e. Blood test: It would indicate inflammation as indicated by
l Cough can be eased by pholcodine 5–10 mg 6-hourly. a raised white blood cell count and elevated C-reactive
l In patients with COPD, aggressive treatment of exacer- protein.
bations may be required. Amoxicillin 250 mg 8-hourly f. High resolution computed tomography (HRCT): This is
should given to previously healthy patients are thought a special type of CT scan that provides the doctor with
to be developing bronchopneumonia. high-resolution images of the lungs.

Treatment
B. Chronic Bronchitis l Smoking cessation is of benefit.
l Antibiotics: Only about 5–10% of bronchitis cases are
l Chronic bronchitis is a chronic inflammation of the caused by a bacterial infection. Most cases of bronchitis
bronchi (medium-size airways) in the lungs. are caused by a viral infection and are ‘self-limited’ and
resolve themselves in a few weeks. For acute exacerba-
Aetiology tions of chronic bronchitis, if antibiotics are used,
l Tobacco smoking is the most common cause. amoxicillin or doxycycline is recommended.
l Pneumoconiosis and long-term fume inhalation are l Bronchodilators: Ipratropium is an example of a bron-
other causes. chodilator that may be useful for people suffering from
l Allergies can also cause mucus hypersecretion, thus chronic obstructive pulmonary disease, such as chronic
leading to symptoms similar to asthma or bronchitis. bronchitis.
Q. 2. Describe clinical features and management of
Signs and symptoms bronchial asthma.
l Bronchitis may be indicated by an expectorating cough
(also known as a productive cough, i.e. one that pro- Or,
duces sputum), shortness of breath (dyspnoea) and Describe the aetiology, clinical features, investigations
wheezing. Occasionally, chest pains, fever and fatigue and management of bronchial asthma.
or malaise may also occur.
l Mucus is often green or yellowish green and also may Ans.
be orange or pink, depending on the pathogen causing
l Asthma is a disease of the airways in which there is a
the inflammation.
chronic inflammation and increased responsiveness to a
wide variety of stimuli, leading to reversible airway ob-
Diagnosis struction. Patients generally have paroxysms of cough,
a. Physical examination: wheeze and dyspnoea, interspersed with symptom-free
l It will often reveal diminished breath sounds, wheezing periods.
and prolonged expiration.
l Presence of a persistent dry or wet cough as evidence
Pathophysiology
of bronchitis.
b. A variety of tests may be performed in patients present- l Genetic and environmental factors play a role in the
ing with cough and shortness of breath: pathogenesis of bronchial asthma.
l Pulmonary function test (PFT) or spirometry must be l Early onset (extrinsic) asthma begins in childhood, and
performed in all patients presenting with chronic a family history of atopy and other allergic disorders
cough. such as allergic rhinitis and eczema are usually present.
l An FEV/FVC ratio below 0.7 that is not fully revers- l Skin test to antigens is positive and serum levels of IgE
ible after bronchodilator therapy indicates the pres- are raised.
ence of COPD. l Late onset (intrinsic) asthma starts in adulthood in non-
c. Chest radiograph: atopic individuals with no family history of allergic
l Reveals hyperinflation; collapse and consolidation of disorders. Skin test is negative and serum levels of IgE
lung areas would support a diagnosis of pneumonia. are normal.
l Some conditions that predispose to bronchitis may l A wide variety of stimuli can provoke bronchospasm in
be indicated by chest radiography. an asthmatic. Stimuli can be allergic or nonallergic.
d. Sputum sample: A sample of sputum showing neutro- l Atopic individuals are sensitized after an exposure to al-
phil granulocytes (inflammatory white blood cells) and lergen and develop IgE antibodies. Subsequent exposure
Section | I General Medicine 85

to the allergen causes a two-phase bronchoconstrictor c. Others


response. l Peripheral blood may show eosinophilia.
l Early reaction (type I) occurs because of rapid interac- l Sputum may also contain an increased number of
tion of allergen with mast cells via IgE-dependant eosinophils.
mechanism. It results in the release of preformed me-
diators like histamine and leukotrienes which cause
bronchoconstriction. Late reaction (type II) is character- Management
ized by T cell-mediated influx of inflammatory cells, l Avoidance of factors: The identification of allergens is
chiefly eosinophils. It leads to chronic inflammatory possible only in a few cases. Wherever possible, the
reaction in the bronchial wall. exposure to such agents must be avoided.
l Desensitization: Desensitization by repeated adminis-
Clinical Features tration of allergen has not been proven effective in most
studies.
l The symptoms of bronchial asthma may be episodic or
l Drug therapy: The drugs used in asthma can be grouped
persistent.
as: (a) quick relievers, which inhibit smooth muscle
l Episodic asthma is characterized by paroxysm of dys-
contraction and cause bronchodilatation and (b) long-
pnoea, cough and wheezing which occurs more commonly
term control medications, which prevent or reverse
in children and young adults.
inflammation.
l The attacks may be mild or severe and may last for
hours, days or rarely weeks. Between episodes, the pa- Management of chronic persistent asthma
tients are usually asymptomatic.
l In older nonatopic patients, the asthma is chronic and l The overall aim of treatment is
l to abolish or minimize symptoms,
persistent. Symptoms are worse in the early morning.
l to attain the best possible PEF,
This can be confused with the COPD.
l to prevent exacerbation and
l The examination may reveal tachypnoea, tachycardia
l to normalize exercise capacity.
and involvement of accessory respiratory muscles.
l The breath sound is harsh vesicular with prolonged ex- Depending on the symptoms, signs, frequency of exac-
piration. Prominent wheeze is audible in both phases of erbations and PEF rate, the asthma is classified mild, mod-
respiration. erate or severe.
l Acute severe asthma is a severe life-threatening attack l A stepwise approach is chosen according to the severity
of asthma, previously known as status asthmaticus. of disease. Once the disease is controlled, a stepdown
l The patient may additionally have tachycardia, pulsus therapy is attempted.
paradoxus, cyanosis and active accessory respiratory l Step 1: Inhaled short-acting adrenergic agents (sal-
muscles. The air entry is drastically reduced with silent butamol, terbutaline) are used as needed for minor
chest on auscultation. The patient may become con- symptoms.
fused or drowsy. l Step 2: If the symptoms are not controlled with oc-

casional inhaled adrenergic drugs as mentioned


Investigations above, low dose of inhaled steroid is added. Mast cell
stabilizers can be used prophylactically before expo-
a. Pulmonary function tests sure to allergens, exercise or cold air.
l Demonstration of reversible air flow obstruction is the l Step 3: The dose of inhaled steroid is escalated. An

hallmark of the diagnosis. inhaled long-acting adrenergic agent such as salme-


l In bronchial asthma, FEV1, FEV1/VC ratio and peak terol or formoterol can be added. Sustained released
expiratory flow rate (PEFR) are reduced. An improve- theophylline may also be used orally.
ment of at least 15% in FEV1 or PEFR following ad- l Step 4: In addition to drugs used in Step 3, leukotri-

ministration of bronchodilator is diagnostic of bronchial ene modifiers and inhaled ipratropium bromide are
asthma. tried.
l PEFR is generally used for long-term home monitoring. l Step 5: Regular oral steroid in the lowest effective dos-

In asymptomatic individuals, exercise, histamine or age is added to Step 4 regimen to control symptoms.
methacholine can be used to provoke bronchospasm.
Management of acute severe asthma
b. Chest radiograph l Hospitalization and oxygen therapy: The patient should
This is usually normal. However, it may appear hyperin- be hospitalized for urgent management. Arterial blood
flated. This is also helpful in ruling out other conditions. gas analysis, chest radiograph and ECG should be done
86 Quick Review Series for BDS 3rd Year

to assess the severity and to rule out other causes. A l If this early phase does not resolve with treatment of the
high concentration of oxygen inhalation is given to underlying cause, a fibroproliferative phase ensues and
maintain PaO2 .60 mmHg. causes progressive pulmonary fibrosis. It is frequently
l Repeated dosage of short-acting adrenergic drugs (sal- associated with other organ dysfunction like kidney,
butamol 2.5–5 mg or terbutaline 5–10 mg) is given heart, gut, liver as a part of multiple organ failure.
through a nebulizer at an interval of 20–30 minutes. l The term ARDS is often limited to patients requiring
l The PEFR should be recorded frequently to assess the ventilatory support on the ICU, but less severe forms,
response. Metered-dose inhaler with a spacer device can conventionally referred to as acute lung injury (ALI)
alternatively be used. Ipratropium bromide can be added and with similar pathology, occur on acute medical and
to get additional bronchodilator effect. surgical wards.
l Systemic steroid: This is essentially needed in all cases l The clinical symptoms and signs are not specific, shar-
of severe asthma. Prednisolone (30–60 mg daily) or ing many features with other pulmonary conditions.
methylprednisolone is given orally or intravenously. l The criteria defining ARDS are:
l Others: a. Hypoxaemia, defined as PaO2/FIo2,6.7 kPa (200
a. Mechanical ventilation is needed in patients with mmHg)
coma, respiratory arrest, exhaustion and deteriorat- b. Chest radiograph showing diffuse bilateral infiltrates
ing blood gases despite adequate treatment. c. Absence of a raised left atrial pressure: PA WP,15
b. IV fluid is administered to prevent dehydration. mmHg
c. Antibiotics are not used routinely. These are only d. Impaired lung compliance.
given, if infection is present.
d. Opiates, sedatives or tranquillizers are contraindi-
cated. Aetiology
Inhalation (direct)
Patient education and monitoring of therapy l Aspiration of gastric contents, water (near-drowning)
l The patients are educated about the nature of disease and l Toxic gases/burn injury
its treatment. They should also be trained to recognize l Pneumonia
the severity of their disease and monitor the response to l Blunt chest trauma
therapy with the use of peak flow meter.
l Patients should also be demonstrated the proper use of Blood-borne (indirect)
inhalation devices such as metered-dose inhalers (pres- l Sepsis
surized aerosol system), rotahaler (dry powder system) l Necrotic tissue (particularly bowel)
and nebulizers. l Multiple trauma
l Use of inhaler therapy should be encouraged as it is ef- l Pancreatitis
fective in lower dosage together with a rapid onset of l Cardiopulmonary bypass
action and has fewer side effects. l Severe burns
l Drugs (heroin, barbiturates, thiazides)
Complications l Major blood transfusion reaction
l The main complications of bronchial asthma include l Anaphylaxis (wasp, bee, snake venom)
pneumothorax, respiratory failure and cor pulmonale. l Fat embolism
l Carcinomatosis
Q. 3. Describe aetiology, clinical features and manage-
l Obstetric crises (amniotic fluid embolus, eclampsia)
ment of acute respiratory distress syndrome (ARDS).
Ans.
Treatment
l Acute respiratory distress syndrome (ARDS) describes
the acute, diffuse pulmonary inflammatory response to General
either direct or indirect blood-borne insults that originate l ARDS is usually treated with mechanical ventilation in the
from extrapulmonary pathology. intensive care unit. Ventilation is usually delivered through
l It is characterized by neutrophil sequestration in pulmo- orotracheal intubation, or tracheostomy whenever pro-
nary capillaries, increased capillary permeability, pro- longed ventilation (2 weeks) is deemed inevitable.
tein-rich pulmonary oedema with hyaline membrane l Appropriate antibiotic therapy must be administered as
formation, damage to type 2 pneumocytes leading to soon as microbiological culture results are available.
surfactant depletion, alveolar collapse and reduction in The origin of infection, when surgically treatable, must
lung compliance. be operated on.
Section | I General Medicine 87

Mechanical ventilation l Rupture of a subpleural tuberculous focus.


l The overall goal is to maintain acceptable gas exchange l Rupture of a lung abscess, especially staphylococcal
and to minimize adverse effects in its application. bronchial carcinoma.
l Three parameters are used: PEEP (positive end-expiratory l Pulmonary infarction.
pressure, to maintain maximal recruitment of alveolar l Bronchial asthma.
units), mean airway pressure (to promote recruitment and l Acute respiratory distress syndrome (ARDS).
predictor of hemodynamic effects) and plateau pressure l Rare causes include sarcoidosis, pneumocystis carinii
(best predictor of alveolar overdistention). pneumonia and cystic fibrosis.
l Conventional therapy aimed at tidal volumes (Vt) of
12–15 ml/kg. Tension (Valvular) Pneumothorax
The communication between pleura and lung persists. It
Prone position l

acts as a one-way valve allowing air to enter the pleural


Distribution of lung infiltrates in acute respiratory distress syn- space during inspiration, coughing, sneezing and strain-
drome is nonuniform. Repositioning into the prone position ing, but not allowing it to escape.
(face down) might improve oxygenation by relieving atelecta- l Large amount of air gets ‘trapped’ in the pleural space
sis and improving perfusion. However, although the hypoxae- and the intrapleural pressure becomes much higher than
mia is overcome, there seems to be no effect on overall survival. the atmospheric pressure.
l The high intrapleural pressure results in compression of
Fluid management
the underlying lung as well as gross shift of the medias-
Several studies have shown that pulmonary function and tinum to the opposite side with consequent compression
outcome are better in patients that lost weight or pulmonary of the opposite lung.
wedge pressure was lowered by diuresis or fluid restriction. l Clinically, these patients present with rapidly progres-
sive breathlessness, central cyanosis, rapid thready
Corticosteroids pulse and signs of peripheral circulatory failure.
The initial regimen consists of methylprednisolone 2 mg/ l Frank signs of pneumothorax are present. Death can
kg daily. After 3–5 days, a response must be apparent. In occur within few minutes from asphyxia.
1–2 weeks, the dose can be tapered to methylprednisolone
0.5–1.0 mg daily. Patients with ARDS do not benefit from
high-dose corticosteroids. Treatment
l Tension pneumothorax is an acute medical emergency.
Nitric oxide l Emergency treatment is the introduction of a wide-bore
Inhaled nitric oxide (NO) potentially acts as selective pul- plastic cannula, the other end of which is attached to a
monary vasodilator. Rapid binding to haemoglobin pre- long rubber tubing, the end of which is placed underwa-
vents systemic effects. It should increase perfusion of better ter in a bottle.
ventilated areas. Almitrine bismesylate stimulates chemore- l The better alternative is the introduction of an intercos-
ceptors in carotic and aortic bodies. It has been used to tal catheter connected to a water-seal drainage system.
potentiate the effect of NO, presumably by potentiating
Q. 5. Define and classify pneumonias. Describe the inves-
hypoxia-induced pulmonary vasoconstriction.
tigations, complications and treatment of community-
Q. 4. What is pneumothorax? What are the causes of pneu- acquired pneumonias.
mothorax? How do you manage tension pneumothorax?
Or,
Ans.
Describe the clinical features, complications and treat-
l The presence of air in the pleural cavity is known as ment of community-acquired pneumococcal pneumonia.
pneumothorax.
Ans.
l Primary (simple) spontaneous pneumothorax com-
monly affects tall, slender males between the ages of 20 Pneumonia is defined as the infection of the lung parenchyma
and 40 years. (alveoli and distal airways) and interstitium of the lung.

Aetiology Classification
l It is believed to be due to rupture of subpleural blebs at Pathologically, pneumonia is classified into four types:
the lung apices. i. Lobar pneumonia
l Rupture of emphysematous bullae. ii. Bronchopneumonia
88 Quick Review Series for BDS 3rd Year

iii. Interstitial pneumonia l The most likely alternatives depend on the age of the
iv. Miliary pneumonia patient and the clinical circumstances. For example,
Mycoplasma pneumoniae and Chlamydia pneumoniae
i. Lobar pneumonia are common in young adults but seldom reported in the
l Generally, the entire lobe of lung is involved. elderly, whereas Haemophilus influenzae should be con-
l The first stage is known congestion, which occurs sidered in elderly patients but is rarely reported in
within 24 hours. The parenchyma is highly vascular and young adults.
oedematous plenty of bacteria and scanty neutrophils. l In younger children, viral infections predominate. A his-
l Second stage (red hepatization) is characterized by the tory of foreign travel may suggest Legionella infection and
presence of numerous erythrocytes, neutrophils, desqua- recent influenza may predispose to Staphylococcus aureus.
mated epithelial cells and fibrin in alveoli resulting in a red
and airless lung, and a consistency similar to that of liver.
Clinical Features
l In the third stage of grey hepatization, lung parenchyma
becomes grey, dry and friable. l Pneumonia typically presents as an acute illness in
l Finally in the stage of resolution, the exudates are which systemic features such as fever, rigors, shivering
digested and removed by scavenger or coughed out. and vomiting often predominate. The appetite is usually
lost and headache frequently reported.
ii. Bronchopneumonia l Pulmonary symptoms include cough, which at first is
l There is neutrophilic exudate in bronchi and bronchiole characteristically short, painful and dry, but later accom-
with peripheral spread of infection to alveoli. panied by the expectoration of mucopurulent sputum.
l One or several lobes of the lung may be involved. l Rust-coloured sputum may be seen in patients with
Generally, lower and posterior segments of lobes are S. pneumoniae and the occasional patient may report
affected. haemoptysis.
l Pleuritic chest pain may be a presenting feature and on
iii. Interstitial pneumonia occasion may be referred to the shoulder or anterior
l There is predominant involvement of the interstitium, abdominal wall. Upper abdominal tenderness is some-
alveolar wall and connective tissue around the broncho- times apparent in patients with lower lobe pneumonia or
vascular tree. if there is associated hepatitis.
l The pattern may be patchy or diffuse. l Less typical presentations may be seen in the very
young and the elderly.
iv. Miliary pneumonia
l Haematogenous spread of pathogens to the lung may
result into diffuse and discrete 2–3 mm lesions resem- Investigations
bling millet seeds. Radiological examination
l A chest radiograph is not essential but a confident diag-
COMMUNITY-ACQUIRED nosis necessitates chest radiography.
PNEUMONIA (CAP) l In lobar pneumonia, a homogeneous opacity localized
to the affected lobe or segment usually appears within
l The incidence of CAP varies with age, being much
12–18 hours from the onset of the illness.
higher in the very young and the elderly.
l Radiological examination is also helpful, if a complica-
l Pneumonia accounts for almost one-fifth of childhood
tion such as parapneumonic effusion, intrapulmonary
deaths worldwide, with approximately 2 million chil-
abscess formation or empyema is suspected.
dren under 5 dying each year.
l Most patients may be safely managed at home, but Microbiological investigations
hospital admission is necessary in 20–40% of patients
l Many cases of CAP can be managed successfully with-
(5–10%) of whom require intensive care.
out identification of the organism, particularly if there
l CAP is usually spread by droplet infection and most
are no features that indicate severe disease. Thus, the
cases occur in previously healthy individuals.
extent of microbiological investigations should be
l The classical pathological response evolves through the
guided by the clinical circumstances.
phases of congestion, red and then grey hepatization,
l A full range of microbiological tests should be per-
and finally resolution with little or no scarring.
formed on patients with severe CAP. The identification
of Legionella pneumophila has important public health
Aetiology implications and requires notification.
l The majority of cases of CAP are due to infection with l In patients who do not respond to initial therapy, micro-
Streptococcus pneumoniae. biological investigations may lead to the appropriate
Section | I General Medicine 89

modification of therapy. Microbiology also provides The choice of antibiotic is guided by clinical context, se-
useful epidemiological information. verity assessment, local knowledge of antibiotic resistance
patterns, and at times epidemiological information.
Assessment of gas exchange l In most patients with uncomplicated pneumonia, a
l Pulse oximetry provides a simple noninvasive method 7–10 days course is adequate, although treatment is usu-
of measuring arterial oxygen saturation (SaO2) and as- ally required for longer in patients with Legionella,
sists in monitoring response to oxygen therapy. staphylococcal or Klebsiella pneumonia.
l An arterial blood gas should be sampled in those with l Oral antibiotics are usually adequate unless the patient
SaO2 ,92% or with features of severe pneumonia to has a severe illness, impaired consciousness, loss of
assess whether the patient has evidence of ventilatory swallowing reflex or functional or anatomical reasons
failure or acidosis. for malabsorption.

General blood tests Treatment of pleural pain


l The white cell count is often only marginally raised or l It is important to relieve pleural pain in order to allow
may even be normal in patients with pneumonia caused the patient to breathe normally and cough efficiently.
by atypical organisms, whereas a neutrophil leucocyto- l Mild analgesics, e.g. paracetamol, are rarely adequate.
sis of more than 15 3 109/l favours a bacterial aetiology. However, opiates must be used with extreme caution in
A very high (.20 3 109/1) or low (, 4 3 109/1) white patients with poor respiratory function.
cell count may be seen in severe pneumonia.
l The urea, electrolytes and liver function tests should Physiotherapy
also be checked. l Formal physiotherapy is not indicated in patients with
l The C-reactive protein (CRP) is typically elevated. CAP but assisted coughing is important in patients who
suppress cough because of pleural pain.
Management
l Patients with CAP should be advised to rest, and if Complications
appropriate, avoid smoking.
l Fever may persist for several days and the chest radio-
l The severity of the illness should be defined and regu-
graph often takes several weeks or even months to re-
larly re-evaluated by nursing observation.
solve especially in the elderly.
l Attention is then paid to oxygenation, fluid balance and
l Delayed recovery suggests either that a complication
antibiotic therapy.
has occurred (such as an empyema), or that the diagno-
l In certain circumstances, such as prolonged illness,
sis is incorrect.
nutritional support may be required.
l Alternatively, the pneumonia may be secondary to a
Oxygen proximal bronchial obstruction or recurrent aspiration.
l Oxygen should be administered to all patients with
tachypnoea, hypoxaemia, hypotension or acidosis with Prevention
the aim of maintaining the PaO2 5 8 kPa (60 mmHg) or
l Influenza vaccination is recommended to those at high
SaO2 5 92%. High concentrations (.35%), preferably
risk of mortality from influenza or pneumonia.
humidified, should be used in all patients who do not
l Pneumococcal vaccination is followed by a good anti-
have hypercapnia associated with COPD.
body response but the efficacy in preventing pneumonia
l Assisted ventilation should be considered at an early
in high-risk groups remains uncertain.
stage in those who remain hypoxaemic despite adequate
oxygen therapy. Q. 6. Describe the clinical features of pulmonary tuber-
culosis. Write about the short-term chemotherapy.
Fluid balance
l An adequate oral intake of fluid should be encouraged Ans.
but intravenous fluids should be considered in those l Tuberculosis (TB) is among the oldest infections in
with severe illness, elderly patients and those whose humans and is still a significant cause of morbidity and
systemic features include vomiting. mortality in the developing countries.
l Inotropic support may be required in patients with shock. l It is caused by bacteria belonging to Mycobacterium
tuberculosis complex. The most common agent of hu-
Antibiotic treatment man disease is M. tuberculosis.
l If possible, culture specimens should be sent prior to start- l Most commonly, the infection is transmitted from in-
ing antibiotics but treatment should not be unduly delayed. fected patients to other persons through droplet nuclei
90 Quick Review Series for BDS 3rd Year

released by coughing, sneezing or speaking. Other l In tuberculosis, the classical physical signs of consoli-
rare routes of transmission are ingestion, skin and dation, cavitation, collapse, fibrosis, bronchiectasis,
placenta. pleural effusion or pneumothorax may be present.
l The risk of acquiring infection is increased by factors
like poverty and overcrowding. Patients with diabetes,
Treatment
silicosis, alcoholism or immunocompromised states are
at a greater risk of acquiring tuberculosis. Antitubercular drugs
l The antitubercular drugs are bactericidal or bacterio-
Clinical Features static in nature.
l The bactericidal drugs are used to rapidly reduce the
l Many patients are symptom-free, and tuberculosis may
number of viable organisms and render patients nonin-
be detected on routine radiography.
fectious. These also have sterilizing activity (kill all
l A characteristic feature of tuberculosis is the gradual
bacilli); hence, they prevent relapse.
onset symptoms over weeks or months.
l Bacteriostatic drugs are used along with the bactericidal
General symptoms drugs to prevent emergence of resistance.
l The first-line drugs are rifampicin (R), isoniazid (H),
l Loss of weight and loss of appetite.
pyrazinamide (Z) and ethambutol (E). Isoniazid and ri-
l Fever, especially evening rise of temperature.
fampicin are active against all population of TB bacilli
l Night sweats.
while pyrizinamide and streptomycin are active against
l Tiredness, malaise.
certain population of TB bacilli. In addition, pyrazin-
l Mental symptoms and amenorrhoea.
amide is active against bacilli in acid environment inside
Respiratory symptoms macrophages and has good CSF penetration. Streptomy-
cin is particularly active against extracellular bacilli.
l Cough is the most consistent symptom. Any patient who l Second-line drugs are less efficacious and more toxic.
has had a cough for more than three weeks should be These are used when first-line drugs fail.
investigated for pulmonary tuberculosis.
l Sputum, which may be mucoid, purulent or blood- Treatment regimen
stained.
l The treatment consists of the initial phase and the con-
l Haemoptysis is a classical symptom.
tinuation phase. The schedule is daily or intermittent
l Chest pain resulting from pleurisy, intercostal myalgia
(twice or thrice weekly). The response to therapy is
or cough.
monitored through culture or smear examination.
l Breathlessness is a feature of advanced and extensive
l Initial phase: The aim is to rapidly kill the bacilli, resolve
disease.
symptoms and bring out sputum conversion (AFB nega-
l Localized wheeze resulting from local ulceration and
tive) so that the patient becomes noninfectious. Gener-
narrowing of a major bronchus.
ally, a combination of 3–4 drugs is used for 2–3 months.
l Recurrent cold is at times a presenting symptom.
l Continuation phase: The purpose is to eliminate the remain-
l Pneumonia which turns out to be tuberculous, is another
ing bacilli from the lesion (sterilizing effect) so that relapse
mode of presentation.
may not occur. This is generally given for 4–6 months.
Physical signs l The most commonly used regimen consists of a 2-month
initial phase of RHZE followed by a 4-month continua-
l Pallor and cachexia in advanced cases.
tion phase of HR (2RHZE/4HR). The drugs can also be
l Fever, tachycardia and tachypnoea.
given thrice weekly (2H3R3Z3E3/4H3R3).
l Finger clubbing is unusual, but it may be present in
chronic disease with purulent sputum. Q. 7. Describe aetiology, clinical features, investigations,
l Physical signs in the chest: The most common sign is complications and management of acute lobar pneumonia.
fine crepitations in the upper part (apices) of one or both
Ans.
lungs. These are heard particularly on taking a deep
breath after coughing (post-tussive crepitations). Pneumonia is defined as the infection of the lung parenchyma,
l Later, there may be dullness to percussion or even bron- i.e. alveoli and distal airways and interstitium of the lung.
chial breathing in the upper part. At times there is a lo-
calized wheeze due to local tuberculous bronchitis or
LOBAR PNEUMONIA
pressure by a lymph node on a bronchus.
l In chronic tuberculosis with fibrosis, evidence of vol- l Lobar pneumonia is a radiological and pathological
ume loss and mediastinal shift may be present. term referring to homogenous consolidation of one
Section | I General Medicine 91

or more lung lobes, often with associated pleural in- l Pneumonia due to certain organisms like Pseudomonas
flammation. aeruginosa, Klebsiella spp., Escherichia coli, Staphylo-
l Generally, the entire lobe of lung is involved. coccus aureus is associated with high mortality rate.
l The first stage is known congestion, which occurs
within 24 hours. The parenchyma is highly vascular and
oedematous plenty of bacteria and scanty neutrophils.
Complications
l Second stage (red hepatization) is characterized by the The complications of pneumonia are as follows:
presence of numerous erythrocytes, neutrophils, des- i. Pulmonary complications:
quamated epithelial cells and fibrin in alveoli resulting l Para-pneumonic pleural effusion
in a red and airless lung, and a consistency similar to l Emphysema
that of liver. l Suppurative pneumonia/lung abscess
l In the third stage of grey hepatization, lung parenchyma l ARDS
becomes grey, dry and friable. l Pneumothorax (in S. aureus pneumonia)
l Finally in the stage of resolution, the exudates are di- ii. Extrapulmonary complications:
gested and removed by scavenger or coughed out. l Hepatitis, pericarditis, meningoencephalitis

l Multiorgan failure

l Formation of ectopic abscess


Route of Infection
The most common route of infection is microaspiration
Investigations
l

of oropharyngeal secretions colonized with pathogenic


microorganisms. i. Radiography: Chest radiograph shows homogeneous
l Other routes are gross aspiration, aerosolization, hae- opacity localized to the lobe or segment (lobar consoli-
matogenous spread from a distant site, and spread from dation) or diffuse infiltrates in bronchopneumonia.
adjacent tissues. Other findings may include pleural effusion, lung ab-
scess and hilar lymphadenopathy. Pneumatoceles may
be seen in Staphylococcus aureus pneumonia.
Pathology
ii. Sputum or pulmonary secretion examination: Gram
l More than 100 organisms including viruses, bacteria, staining and culture are helpful in identifying the caus-
fungi, parasites are reported to cause pneumonia. ative organisms and finding out the sensitivity pattern.
iii. Blood tests: These include blood culture, arterial blood
gas analysis, and complete total and differential counts.
Clinical Manifestations
Neutrophilic leucocytosis is found in bacterial pneumo-
l The onset may be sudden or insidious, and the disease nia while leukopenia indicates viral aetiology.
may be mild or severe. iv. Serological tests: The detection of antibodies may be
l Fever with chills or rigors, cough productive, pleuritic helpful in the diagnosis of Chlamydia, Mycoplasma
chest pain, breathlessness are typical manifestations of and Legionella pneumoniae.
pneumonia. v. Other tests: Polymerase chain reaction (PCR)-based tests
l Other symptoms are headache, nausea, vomiting, diar- and specialized culture tests may be needed in some cases.
rhoea, altered sensorium, myalgia and arthralgia.

The signs of lobar pneumonia are: Treatment


l Tachypnoea i. General principles:
l Decreased movement of the chest on the side of disease. l The patients are managed as outpatient or in-patient
l No shift of trachea or apex beat (no mediastinal shift). depending upon the severity of the disease.
l Increased vocal fremitus or resonance. l The patients are classified into five risk classes. Pa-
l Bronchial breath sounds. tients in classes I and II are treated as out-patients,
l Aegophony, whispering pectoriloquy, crackles (crepts) class III may need short hospital stay while those in
and pleural rub. classes IV and V need hospital admission.
l The severity of pneumonia is indicated by respiratory ii. Supportive therapy:
rate more than 30 per minute, pulse rate more than 125 l Oxygen therapy is given to all hypoxaemic patients.
per minute and blood pressure less than 90 mmHg. Those who do not respond may need mechanical
l Other markers of severity are altered mentation, hy- ventilation.
poxia (PaO2 ,60 mmHg), hyponatraemia, acidosis and l Intravenous fluid may also be required in hospital-
azotaemia. ized patients.
92 Quick Review Series for BDS 3rd Year

iii. Antibiotic therapy: l Tumours of the stomach or oesophagus


l The antibiotics should be started as early as possible. l Radiation poisoning
If possible, the specimen should be sent for culture l Viral haemorrhagic fevers
and sensitivity before starting antibiotics. l Gastroenteritis
l The aetiology is commonly unknown. Hence, the ini- l Gastritis
tial choice of antibiotics is empirical. However, this l Peptic ulcer
may be modified once culture reports are available. l Chronic viral hepatitis
l The preferred empirical antibiotic options are mac- l Intestinal schistosomiasis (caused by the parasite Schis-
rolide or doxycycline or fluoroquinolones (with ac- tosoma mansoni)
tivity against S. pneumoniae). A 10–14 days therapy l History of smoking
is generally required.
l Emperical antibiotic options for hospitalized pa-

tients are followed. These are given intravenously. Management


The duration of treatment is 10–14 days. l Haematemesis is treated as a medical emergency.
l Beta lactum (ceftriaxone 1–2 g/day or cefotaxime

2 g 6-hourly) plus a macrolide or fluoroquinolones. Minimal blood loss


l Beta lactam or beta lactamase inhibitor (ampicillin/ l The patient is generally administered a proton pump in-
sulbactam or piperacillin/tazobactam) plus macro- hibitor (e.g. omeprazole), given blood transfusions (if the
lide or fluoroquinolones. level of haemoglobin is extremely low, i.e. ,8.0 g/dl or
l Patients with aspiration pneumonia should receive 4.5–5.0 mmol/l), and kept NPO (nothing by mouth, or no
fluoroquinolones plus clindamycin or metronidazole. eating or drinking) until endoscopy can be arranged.
l In pseudomonas infection (as in patients with under- l Adequate venous access (large-bore cannulas or a cen-
lying bronchiectasis), a combination of fluoroquino- tral venous catheter) is generally obtained in case the
lones with carbapenem or cefipime. patient suffers a further bleed and becomes unstable.
Q. 8. A 40-year-old man brought to the casuality with
history of haematemesis. Mention its differential diag- Significant blood loss
nosis, investigation and management. l In a ‘haemodynamically significant’ case of haemateme-
sis, i.e. hypovolaemic shock, resuscitation is an immedi-
Ans.
ate priority to prevent cardiac arrest.
Haematemesis is the vomiting of blood. The source is gen- l Fluids and/or blood is administered, preferably by cen-
erally the upper gastrointestinal tract. tral venous catheter, and the patient is prepared for
The signs of the onset of haematemesis may include the emergency endoscopy, which is typically done in the-
following: atres. Surgical opinion is usually sought in case the
l A history of excessive alcohol use or liver disease source of bleeding cannot be identified endoscopically,
l Any oesophagogastric symptoms, such as nausea or and laparotomy is necessary.
vomiting l Securing the airway is a top priority in haematemesis
l Brown or black vomit patients, especially those with a disturbed conscious level
l Vomit that looks like coffee grounds (hepatic encephalopathy in oesophageal varices patient).
l Dark-coloured, tar-like stools (a condition known as melaena) A cuffed endotracheal tube could be a life-saving choice.
Q. 9. Describe the clinical features, diagnosis and com-
Aetiology plications of post-primary tuberculosis.
l Prolonged and vigorous retching (may cause a tear in
Ans.
the small blood vessels of the throat or the oesophagus,
producing streaks of blood in the vomit, and is called l Post-primary pulmonary tuberculosis presents mainly
Mallory–Weiss syndrome). as parenchymal disease with only minimal lymph node
l Irritation or erosion of the lining of the oesophagus or enlargement.
stomach. l This describes the lung disease, the characteristic patho-
l Bleeding ulcer located in the stomach, duodenum or logical feature of which is the tuberculous cavity. The cavity
oesophagus. is formed when the caseated and liquefied centre of a tuber-
l Vomiting of ingested blood after haemorrhage in the culous pulmonary lesion is discharged into a bronchus.
oral cavity, nose or throat. l The common sites of involvement are the apical and
l Vascular malfunctions of the gastrointestinal tract, such posterior segments of the upper lobe or the apical seg-
as bleeding gastrointestinal varices. ment of the lower lobe.
Section | I General Medicine 93

l This predilection may be due to the good ventilation and Investigations


decreased blood supply of these regions in the erect
i. Blood examination
posture.
l Moderate anaemia.
l White cell count is usually normal or low normal.
Clinical Features l Erythrocyte sedimentation rate (ESR) is usually raised.
l Hyponatraemia and hypokalaemia may be present in
l Many patients are symptom-free, and tuberculosis may severe disease.
be detected on routine radiography. l Impaired liver function tests are occasionally seen.
l A characteristic feature of tuberculosis is the gradual
onset symptoms over weeks or months.
ii. Radiographic examination of chest
A normal chest radiograph for practical purposes ex-
General symptoms l

cludes tuberculosis.
l Loss of weight and loss of appetite. l The radiological features of tuberculosis are given else-
l Fever, especially evening rise of temperature, night where.
sweats.
l Tiredness, malaise. iii. Sputum examination
l Mental symptoms. l Microscopic examination of sputum smears remains the
l Amenorrhoea. most important investigation. Three specimens of spu-
tum should be examined. If two smears are positive, one
Respiratory symptoms can regard the diagnosis as certain.
l Cough is the most consistent symptom. Any patient who a. A first spot specimen when the patient presents himself.
has had a cough for more than three weeks should be b. An early morning specimen consisting of all sputum
investigated for pulmonary tuberculosis. raised in the first 1–2 hours.
l Sputum, which may be mucoid, purulent or blood-stained. c. A second spot specimen when the patient returns
l Haemoptysis is a classical symptom. with the early morning specimen.
l Pain in the chest resulting from pleurisy, intercostal l Sputum culture for tubercle bacillus may be done, but
myalgia or cough fracture. results become available only in 4–8 weeks. Drug resis-
l Breathlessness is a feature of advanced and extensive tance tests should be done in selected cases.
disease. l Animal (guinea pig) inoculation studies of the sputum is
l Localized wheeze resulting from local ulceration and more sensitive than culture.
narrowing of a major bronchus. l Newer methods for rapid identification of mycobacteria
l Recurrent cold is at times a presenting symptom. include:
l Pneumonia, which turns out to be tuberculous, is an- a. Radiometric growth detection using a BACTEC
other mode of presentation. system.
b. Identification of species by nucleic acid probes.
Physical signs c. Identification mycolic acids using high-pressure
l Pallor and cachexia in advanced cases. liquid chromatography.
l Fever, tachycardia and tachypnoea.
l Finger clubbing is unusual, but it may be present in iv. Laryngeal swabs
chronic disease with purulent sputum. l While taking a swab, the patient will cough and the
l Physical signs in the chest. Often, there are no abnormal swab will catch some mucus. This is sent for culture.
signs. The most common sign is fine crepitations in the
upper part (apices) of one or both lungs. These are heard v. Gastric aspiration (gastric lavage or gastric washings)
particularly on taking a deep breath after coughing (post- In those who do not have sputum, early morning fasting
tussive crepitations). Later, there may be dullness to gastric washings can show tubercle bacilli. For this, a sterile
percussion or even bronchial breathing in the upper part. nasogastric tube is introduced into the stomach and 20 ml
At times there is a localized wheeze due to local tuber- of sterile normal saline injected. After one minute, aspirate
culous bronchitis or pressure by a lymph node on a out as much as possible. The aspirate should be examined
bronchus. In chronic tuberculosis with fibrosis, evidence by smear and culture.
of volume loss and mediastinal shift may be present.
l In tuberculosis, the classical physical signs of consoli- vi. Bronchoalveolar lavage
dation, cavitation, collapse, fibrosis, bronchiectasis, Bronchoalveolar lavage fluid gives more positive results,
pleural effusion or pneumothorax may be present. but it is reserved for selected cases.
94 Quick Review Series for BDS 3rd Year

vii. Tuberculin test l Ciliary dysfunction syndromes: Primary ciliary dyski-


l Tuberculin testing as a tool for diagnosis is much less nesia (immotile cilia syndrome), Kartagener’s syndrome
valuable. This is partly because the test may be negative (sinusitis and transposition of viscera)
due to malnutrition or other diseases, even though the l Primary hypogammaglobulinaemia
patient has active tuberculosis. On the other hand, many
people without active tuberculosis have positive tests. ii. Acquired-children
l A strongly positive test is a point in favour of tubercu- l Pneumonia (complicating whooping cough or measles)
losis, but a negative test does not exclude tuberculosis. l Primary TB
l Inhaled foreign body
Complications of Post-primary Tuberculosis iii. Acquired-adults
l Haemoptysis l Suppurative pneumonia
l Pneumothorax l Pulmonary TB
l Secondary infection of cavity l Allergic bronchopulmonary aspergillosis complicating
l Pleural effusion asthma
l Empyema l Bronchial tumours
l Pulmonary fibrosis
l Bronchiectasis There may also be inflammatory changes in the deeper
l Persistence of cavities even after treatment layers of the bronchial wall and hypertrophy of the bron-
l Infection of cavities by Aspergillus chial arteries. Chronic inflammatory and fibrotic changes
l Scar carcinoma are usually found in the surrounding lung tissue.
l Spread of tuberculosis to other organs
l Respiratory failure and right heart failure Clinical Features
l Amyloidosis
l Physical signs in the chest may be unilateral or bilateral.
l Anaemia
If the bronchiectatic airways do not contain secretions
Q. 10. Discuss the aetiology, clinical features and treat- and there is no associated lobar collapse, there are no
ment of bronchial obstruction. abnormal physical signs.
l When there are large amounts of the sputum in the bron-
Ans.
chiectatic spaces, numerous coarse crackles may be
‘Bronchiectasis’ is the term used to describe abnormal dila- heard over the affected areas.
tation of the bronchi. It is usually acquired but may result l When collapse is present the character of the physical
from an underlying genetic or congenital defect of airway signs depends on whether or not the proximal bronchus
defences. supplying the collapsed lobe is patent (breath sounds
are diminished, if the airway is obstructed). Advanced
disease may lead to scarring with associated overlying
Aetiology and Pathogenesis bronchial breathing.
l Bronchiectasis is usually caused by chronic inflamma-
tion and infection in airways. Investigations
l Among the common causes, TB is the most common
worldwide. i. Bacteriological and mycological examination
l Localized bronchiectasis may be due to bronchial dis- of sputum
tension resulting from the accumulation of pus beyond l In addition to common respiratory pathogens, sputum
an obstructing bronchial lesion, such as enlarged tuber- culture may reveal Pseudomonas aeruginosa, fungi
culous hilar lymph nodes, a bronchial tumour or an in- such as Aspergillus and various Mycobacteria. Frequent
haled foreign body (e.g. an aspirated peanut). cultures are necessary to ensure appropriate treatment
l The bronchiectatic cavities may be lined by granula- of resistant organisms.
tion tissue, squamous epithelium or normal ciliated
epithelium. ii. Radiographic examination
l Bronchiectasis, unless very gross, is not usually appar-
Causes of Bronchiectasis ent on a chest radiograph. In advanced disease, thick-
ened airway walls, cystic bronchiectatic spaces and as-
i. Congenital sociated areas of pneumonic consolidation or collapse
l Cystic fibrosis may be visible.
Section | I General Medicine 95

l CT is much more sensitive, and shows thickened dilated Surgical treatment


airways. l Surgery is only indicated in a small proportion of cases.
These are usually young patients in whom the bronchi-
iii. Assessment of ciliary function ectasis is unilateral and confined to a single lobe or
l A screening test can be performed in patients suspected segment as demonstrated by CT.
of having a ciliary dysfunction syndrome by assessing l Unfortunately, many of the patients in whom medical
the time taken for a small pellet of saccharin placed in treatment proves unsuccessful are also unsuitable for
the anterior chamber of the nose to reach the pharynx, pulmonary resection because of either extensive bron-
when the patient can taste it. This time should not ex- chiectasis or coexisting chronic lung disease.
ceed 20 minutes and is greatly prolonged in patients l In progressive forms of bronchiectasis, resection of de-
with ciliary dysfunction. stroyed areas of lung which are acting as a reservoir of
l Ciliary beat frequency may also be assessed using infection should only be considered as a last resort.
biopsies taken from the nose.
l Structural abnormalities of cilia can be detected by
electron microscopy. Prevention
As bronchiectasis commonly starts in childhood following
measles, whooping cough or a primary tuberculous infection,
Management it is essential that these conditions receive adequate prophy-
In patients with airflow obstruction, inhaled bronchodila- laxis and treatment. The early recognition and treatment of
tors and corticosteroids should be used to enhance airway bronchial obstruction are also particularly important.
patency.
Q. 11. Describe the aetiology, clinical features and treat-
ment of chronic bronchitis.
Physiotherapy
l Patients should be instructed how to perform regular Or,
daily physiotherapy to keep the dilated bronchi empty Discuss the aetiology, clinical features, complications
of secretions. Efficiently performed, this is of great and management of chronic bronchitis.
value both in reducing the amount of cough and sputum
and in preventing recurrent episodes of bronchopulmo- Ans.
nary infection. Chronic bronchitis is a chronic inflammation of the bronchi
l Patients should adopt a position in which the lobe to be (medium-sized airways) in the lungs.
drained is uppermost. Deep breathing followed by forced
expiratory manoeuvres (the ‘active cycle of breathing’
technique) is of help in allowing secretions in the dilated Signs and Symptoms
bronchi to gravitate towards the trachea, from which l Bronchitis may be indicated by an expectorating cough (also
they can be cleared by vigorous coughing. known as a productive cough, i.e. one that produces sputum),
l ‘Percussion’ of chest wall with cupped hands may help shortness of breath (dyspnoea) and wheezing. Occasionally,
to dislodge sputum, and a number of mechanical de- chest pains, fever and fatigue or malaise may also occur.
vices are available which cause the chest wall to oscil- l Mucus is often green or yellowish green and also may
late, thus achieving the same effect. be orange or pink, depending on the pathogen causing
l The optimum duration and frequency of physiotherapy the inflammation.
depends on the amount of sputum but 5–10 minutes
once or twice daily is a minimum for most patients.
Aetiology
Antibiotic therapy l Tobacco smoking is the most common cause.
The policy governing the use of antibiotics in most patients l Pneumoconiosis and long-term fume inhalation are
with bronchiectasis is the same as that in COPD. Some other causes.
present difficult therapeutic problems because of secondary l Allergies can also cause mucus hypersecretion, thus
infection with bacteria such as staphylococci and gram- leading to symptoms similar to asthma or bronchitis.
negative bacilli, in particular Pseudomonas species. In
these circumstances, antibiotic therapy should be guided by
Diagnosis
the microbiological results but frequently requires the use
of oral ciprofloxacin (250–750 mg 12-hourly) or ceftazi- l Aphysical examination will often reveal diminished
dime by intravenous injection or infusion (1–2 g 8-hourly). breath sounds, wheezing and prolonged expiration.
96 Quick Review Series for BDS 3rd Year

Most doctors rely on the presence of a persistent dry or Write aetiopathogenesis, clinical features and treatment
wet cough as evidence of bronchitis. of lung abscess.
l A variety of tests may be performed in patients present-
Ans.
ing with cough and shortness of breath.
l Pulmonary function tests (PFT) or spirometry must be Lung abscess is defined as necrotic area of lung paren-
performed in all patients presenting with chronic cough. chyma containing purulent material.
An FEV1/FVC ratio below 0.7 that is not fully reversible
after bronchodilator therapy indicates the presence of
COPD that requires more aggressive therapy and carries Aetiology
a more severe prognosis than simple chronic bronchitis. l Aspiration of nasopharyngeal or oropharyngeal con-
l A chest radiograph that reveals hyperinflation, collapse tents (aspiratiOi abscess) is the most common cause.
and consolidation of lung areas would support a diagno- l Depression of cough reflex favours aspiration. This oc-
sis of pneumonia. Some conditions that predispose to curs during sleep, anaesthesia, alcohol intoxication, epi-
bronchitis may be indicated by chest radiography. lepsy and coma.
l A sputum sample showing neutrophil granulocytes (in- l Aspiration also occurs in achalasia cardia, carcinoma of
flammatory white blood cells) and culture showing that has oesophagus, hiatus hernia and gastroesophageal reflux
pathogenic microorganisms such as Streptococcus spp. disease.
l A blood test would indicate inflammation (as indicated l Pre-existing sources of infection for aspiration include si-
by a raised white blood cell count and elevated C-reactive nusitis, dental sepsis, gingivitis, periodontal infection, etc.
protein). l The dominant organisms in aspiration abscess include
l Neutrophils infiltrate the lung tissue, aided by damage anaerobic organisms, streptococci and Haemophilus
to the airways caused by irritation. influenzae.
l Damage caused by irritation of the airways leads to l Because of the relatively more vertical course of right
inflammation and leads to neutrophils being present. main bronchus, aspiration abscesses are more common
l Mucosal hypersecretion is promoted by a substance in the right lung.
released by neutrophils. l Aspiration abscess cavities are located in those broncho-
l Further obstruction to the airways is caused by more pulmonary segments that are most dependent at the time
goblet cells in the small airways. This is typical of of aspiration. Aspiration in supine position results in ab-
chronic bronchitis. scess in the posterior segment of the upper lobe or supe-
l Although infection is not the reason or cause of chronic rior segment of the lower lobe. Aspiration in the upright
bronchitis, it is seen to aid in sustaining the bronchitis. position results in abscess in the basilar segments.
l High resolution computed tomography (HRCT): This is a l Pulmonary tuberculosis is an important cause of lung
special type of CT scan that provides your doctor with high- abscess. Necrotizing pneumonias, especially due to
resolution images of your lungs. Having an HRCT is no Klebseilla pneumoniae, Staphylococcus aureus and
different than having a regular CT scan; they both are per- streptococci.
formed on an open-air table and take only a few minutes. l Bronchial obstruction by foreign body, tumour or bron-
chostenosis leads to abscess formation distally.
Treatment l Haematogenous spread of the organisms to the lung re-
sults from bacteraemia, right-sided endocarditis and
l Smoking cessation is of benefit. septic thrombophlebitis.
l Antibiotics: Only about 5–10% of bronchitis cases are l Secondary infection of cavitary malignancy.
caused by a bacterial infection. Most cases of bronchitis l Secondary infection of pulmonary infarct.
are caused by a viral infection and are ‘self-limited’ and l Rupture of amoebic liver abscess into lung.
resolve themselves in a few weeks. For acute exacerba- l Rare causes include pulmonary fungal infections, in-
tions of chronic bronchitis, if antibiotics are used, fected cysts and vasculitis.
amoxicillin or doxycycline is recommended.
l Bronchodilators: Ipratropium is an example of a bron-
chodilator that may be useful for people suffering from Clinical Features
chronic obstructive pulmonary disease, such as chronic
bronchitis. History
Lung abscess can have two modes of presentations:
Q. 12. Describe the pathogenesis, clinical features and
l In majority, the disease starts acutely with high-grade
diagnosis of lung abscess.
fever, chills and rigors, pleuritic chest pain and a dry
Or, cough.
Section | I General Medicine 97

l After a few days, when the abscess cavity ruptures into l Antibiotic therapy of aspiration abscess is aspiration
a patent bronchus, the patient suddenly starts expecto- pneumonia.
rating large quantities of sputum. l Majority of the patients with lung abscess will respond
l The sputum is large in volume, purulent, foul smelling to oral treatment with ampicillin 500 mg four times daily
and often blood tinged. The expectoration varies with or cotrimoxazole 960 mg twice daily or clindamycin.
posture. l In anaerobic bacterial infection (e.g. those with foul
l In a few patients, the lung abscess is more insidious in smelling sputum), oral metronidazole 400 mg 8-hourly
onset with low-grade fever, malaise, weight loss, an- should be added, should never be used as a single agent.
orexia and a deep-seated chest discomfort. l Seriously ill patients will require parenteral antibiotic
therapy in the form of penicillin or clindamycin plus
Physical findings metronidazole.
l General examination reveals anaemia, fever, finger l Duration of antibiotic therapy is variable. Some patients
clubbing, halitosis and oronasal sepsis. may require prolonged treatment for 4 to 6 weeks.
l Respiratory system examination may be normal in the l Resectional surgery is indicated only in selected situ-
early stages. Later, Frank signs of consolidation like ations.
dullness on percussion increased vocal fremitus and vo- l Massive haemoptysis.
cal resonance, bronchial breathing, crepitations and l Localized malignancy.
pleural rub appear. l Associated symptomatic bronchiectasis.
l Once the abscess cavity opens into a bronchus, signs of l Persistent abscess cavity.
cavitation like cavernous or amphoric bronchial breath- Q. 13. Describe clinical features, complications and
ing and coarse post-tussive crepitations appear. management of adult pulmonary tuberculosis.
Ans.
Investigations
l Anaemia, leukocytosis and raised erythrocyte sedimen- Clinical Features
tation rate (ESR).
l Sputum studies should include Gram stain, Ziehl- l Many patients are symptom-free, and tuberculosis may
Neelsen staining for acid-fast bacilli, aerobic and be detected on routine radiography.
anaerobic cultures and sensitivity, and cytological l A characteristic feature of tuberculosis is the gradual
examination for malignant cells. onset symptoms over weeks or months.
l Chest radiograph often shows a radiolucency in an
opaque area of consolidation. The wall or the border of General symptoms
the cavity completely surrounds the lucent area and an l Loss of weight and loss of appetite.
air-fluid level may be seen. l Fever, especially evening rise of temperature.
l Bronchoscopy is indicated to exclude malignancy, obtain l Night sweats.
specimens for studies, and for removal of secretions. l Tiredness, malaise.
l CT scan of thorax can detect lung abscess with certainty. l Mental symptoms and amenorrhoea.

Respiratory symptoms
Complications l Cough is the most consistent symptom. Any patient who
l Haemoptysis has had a cough for more than 3 weeks should be inves-
l Pleural effusion, empyema tigated for pulmonary tuberculosis.
l Pneumothorax, pyopneumothorax and bronchopleural l Sputum, which may be mucoid, purulent or blood-
fistula stained.
l Metastatic cerebral abscess l Haemoptysis is a classical symptom.
l Amyloidosis l Chest pain resulting from pleurisy, intercostal myalgia
l Aspergilloma or cough.
l Residual fibrosis and bronchiectasis l Breathlessness is a feature of advanced and extensive
disease.
Localized wheeze resulting from local ulceration and
Treatment l

narrowing of a major bronchus.


l Postural drainage and chest physiotherapy. l Recurrent cold is at times a presenting symptom.
l Antibiotic therapy should be based on sensitivity re- l Pneumonia, which turns out to be tuberculous, is an-
ports. However, certain broad guidelines may be given. other mode of presentation.
98 Quick Review Series for BDS 3rd Year

Physical signs l Bacteriostatic drugs are used along with the bactericidal
l Pallor and cachexia in advanced cases. drugs to prevent emergence of resistance.
l Fever, tachycardia and tachypnoea. l The first-line drugs are rifampicin (R), isoniazid (H),
l Finger clubbing is unusual, but it may be present in pyrazinamide (Z) and ethambutol (E). Isoniazid and
chronic disease with purulent sputum. rifampicin are active against all population of TB bacilli
l Physical signs in the chest: The most common sign is while pyrizinamide and streptomycin are active against
fine crepitations in the upper part (apices) of one or both certain population of TB bacilli. In addition pyrazin-
lungs. These are heard particularly on taking a deep amide is active against bacilli in acid environment
breath after coughing (post-tussive crepitations). inside macrophages and has good CSF penetration.
l Later, there may be dullness to percussion or even bron- Streptomycin is particularly active against extracellular
chial breathing in the upper part. At times, there is a bacilli.
localized wheeze due to local tuberculous bronchitis or l Second-line drugs are less efficacious and more toxic.
pressure by a lymph node on a bronchus. These are used when first-line drugs fail.
l In chronic tuberculosis with fibrosis, evidence of vol-
ume loss and mediastinal shift may be present. Treatment regimen
l In tuberculosis, the classical physical signs of consoli- l The treatment consists of the initial phase and the con-
dation, cavitation, collapse, fibrosis, bronchiectasis, tinuation phase. The schedule is daily or intermittent
pleural effusion or pneumothorax may be present. (twice or thrice weekly). The response to therapy is
monitored through culture or smear examination.
l Initial phase: The aim is to rapidly kill the bacilli,
Complications resolve symptoms and bring out sputum conversion
l Haemoptysis (AFB negative) so that the patient becomes noninfec-
l Pneumothorax tious. Generally, a combination of 3–4 drugs is used for
l Secondary infection of cavity 2–3 months.
l Pleural effusion l Continuation phase: The purpose is to eliminate the
l Empyema remaining bacilli from the lesion (sterilizing effect) so
l Pulmonary fibrosis that relapse may not occur. This is generally given for
l Bronchiectasis 4–6 months.
l Persistence of cavities even after treatment l The most commonly used regimen consists of a
l Infection of cavities by Aspergillus 2 months initial phase of RHZE followed by a 4 months
l Scar carcinoma continuation phase of HR (2RHZE/4HR). The drugs
l Spread of tuberculosis to other organs can also be given thrice weekly (2H3R3Z3E3/4H3R3).
l Respiratory failure and right heart failure
l Amyloidosis Other drugs/therapy
l Anaemia l The supplementation of pyridoxine (10–25 mg/day)
prevents isoniazid-related neuropathy. It is given in
those who are at high risk of pyridoxine deficiency, such
Treatment as alcoholics, elderly, malnourished, diabetics, pregnant
The main aims of treatment of tuberculosis are: ladies and HIV patients.
l To cure the patients of tuberculosis l Corticosteroids are useful, if given with antitubercular
l To decrease transmission of tuberculosis to others drugs in seriously ill patients (miliary TB), and menin-
l To prevent relapse geal, pericardial, pleural or ureteric diseases to prevent
l To prevent morbidity and mortality from active tubercu- adhesions.
losis l Surgery may be needed in patients with massive and re-
l To prevent late effects of tuberculosis current hemoptysis, constrictive pericarditis, lymph node
suppuration, empyema and spinal cord compression.
Antitubercular drugs
l The antitubercular drugs are bactericidal or bacterio- National guidelines
static in nature. l Patients with tuberculosis are grouped into four catego-
l The bactericidal drugs are used to rapidly reduce the ries and the treatment regimens are chosen accordingly.
number of viable organisms and render patients nonin- l Drug-resistant tuberculosis.
fectious. These also have sterilizing activity (kill all l Resistance to TB drugs develops because of spontane-
bacilli); hence, they prevent relapse. ous point mutation in the Mycobacterium genome.
Section | I General Medicine 99

l It may be primary (patients infected with resistant ba- Investigations


cilli and have not received TB drugs earlier) or second-
ary/acquired (resistance develops during treatment). A. Radiographic investigations
l Factors which favour the development of resistance are i. Conventional radiographs
inadequate regimen, incomplete dosage or duration. l The classical appearance of pleural fluid on the erect

l Multidrug-resistant tuberculosis (MDR-TB) is charac- PA chest film is of a curved shadow at the lung base,
terized by the resistance to more than one anti-TB drugs blunting the costophrenic angle and ascending to-
or resistance to both isoniazid and rifampicin. The treat- wards the axilla.
ment of MDR-TB needs second-line drugs for a longer l Around 200 ml of fluid is required to be detectable

duration. on a PA chest radiograph, but smaller effusions can


be identified by ultrasound or CT scanning.
Directly observed treatment, short course (DOTS) l Previous scarring or adhesions in the pleural space

l Patients swallow the drugs before a health personnel or can cause localized effusions. Pleural fluid localized
some other responsible person. This ensures that the TB below the lower lobe (subpulmonary effusion) simu-
patient takes the right drugs, in the right dosage at the lates an elevated hemidiaphragm. Fluid localized
right intervals. DOTS help to improve cure rate and to within an oblique fissure may produce a rounded
reduce the chance of drug resistance. opacity simulaling a tumour.
ii. Ultrasonography
Q. 14. Write about pleural effusion. Describe clinical l It is more accurate than plain chest radiography at
features and treatment of pleural effusion. determining the volume of pleural fluid and fre-
Ans. quently provides additional helpful information.
l Visualization of fluid facilitates safe needle aspira-
l The accumulation of serous fluid within the pleural tion and guides pleural biopsy increasing the diag-
space is termed pleural effusion. In general, pleural fluid nostic yields.
accumulates as a result of either increased hydrostatic l The presence of loculation may suggest an evolving
pressure or decreased osmotic pressure (transudative empyema or resolving haemothorax. The technique
effusion as seen in cardiac, liver or renal failure) or from may also distinguish pleural fluid from pleural
increased microvascular pressure due to disease of the thickening.
pleural surface itself, or injury in the adjacent lung (exu- iii. CT scanning
dative effusion). l This displays pleural abnormalities more readily
l The cause of the majority of pleural effusions can usu- than either plain radiography or ultrasound, and may
ally be identified through a thorough history, examina- distinguish benign from malignant pleural disease.
tion and relevant investigations.
l Particular attention should be paid to a recent history of B. Pleural aspiration and biopsy
respiratory infection, the presence of heart, liver or renal l In some clinical settings, it should be necessary to
disease, occupation (e.g. exposure to asbestos), contact sample fluid unless atypical features are present; appro-
with tuberculosis and risk factors for thromboembolism. priate treatment should be administered and the effusion
re-evaluated. However, in most other circumstances,
sampling is necessary to establish a diagnosis.
Clinical Assessment
l Simple aspirations provide information on the colour
l Symptoms and signs of pleurisy often precede the de- and texture of fluid and on appearance alone lately
velopment of an effusion, especially in patients with suggest an empyema or chylothorax. The blood is con-
underlying pneumonia, pulmonary infarction or connec- sistent with pulmonary infarction or may represent a
tive tissue disease. traumatic tap.
l The onset may be insidious. l Biochemical analysis allows classification into transu-
l Breathlessness is the only symptom related to the effu- date and exudates and Gram stain may suggest parap-
sion and its severity depends on the size and rate of ac- neumonic effusion.
cumulation. l The predominant cell type provides useful information
l Examination reveals diminished movements on the side and cytological examination is essential.
of the effusion. l A low pH suggests infection but may also be seen in
l Mediastinal shift to the opposite side and stony dull on rheumatoid arthritis, ruptured oesophagus or advanced
percussion. malignancy.
l Auscultation of the chest shows reduced or absent l Combining pleural aspiration with biopsy increases the
breath sounds. diagnostic yield.
100 Quick Review Series for BDS 3rd Year

l An Abrams needle is most frequently employed. In- l Finally, in the stage of resolution, the exudates are di-
creased yields are reported when either ultrasound or CT gested and removed by scavenger or coughed out.
is used to guide the operator. The best results are obtained
from video-assisted thoracoscopy, allowing the operator ii. Bronchopneumonia
to visualize the pleura and guide the biopsy directly. l There is neutrophilic exudate in bronchi and bronchiole
with peripheral spread of infection to alveoli.
l One or several lobes of the lung may be involved.
Management Generally, lower and posterior segments of lobes are
l Therapeutic aspiration may be required to palliate affected.
breathlessness but removing more than 1.5 L in one
episode is inadvisable as there is a small risk of re- iii. Interstitial pneumonia
expansion pulmonary oedema. l There is predominant involvement of the interstitium,
l An effusion should never be drained to dryness before alveolar wall and connective tissue around the broncho-
establishing a diagnosis as further biopsy may be pre- vascular tree.
cluded until further fluid accumulates. l The pattern may be patchy or diffuse.
l Treatment of the underlying cause, e.g. heart failure,
pneumonia, pulmonary embolism or subphrenic abscess iv. Miliary pneumonia
will often be followed by resolution of the effusion. l Haematogenous spread of pathogens to the lung may
Q. 15. Define pneumonias. Classification of pneumonias. result into diffuse and discrete 2–3 mm lesions resem-
Describe the aetiology, clinical features, investigations, bling millet seeds.
complications and management of bacterial pneumonia.
Or,
Aetiology
Pneumococcal pneumonia or bacterial pneumonia is the
Classify pneumonia. Describe the clinical features, com-
most common form of pneumonia, and the causative organ-
plications and treatment of pneumococcal pneumonia.
ism is Streptococcus pneumoniae (i.e. pneumococcus, a
Or, gram-positive, lancet-shaped diplococcus). The disease
spreads by droplet infection.
Discuss the aetiology, clinical features, diagnosis and
treatment of pneumococcal pneumonia.
Ans.
Pathology
l Pneumococcal pneumonia is characterized by homoge-
Pneumonia is defined as the infection of the lung parenchyma
neous consolidation of one or more lobes or segments.
(alveoli and distal airways) and interstitium of the lung.
l There are three stages in the natural course of the
illness:
Classification i. Stage of red hepatization
ii. Stage of grey hepatization
Pathologically, pneumonia is classified into four types: iii. Stage of resolution
i. Lobar pneumonia
ii. Bronchopneumonia
iii. Interstitial pneumonia Clinical Features
iv. Miliary pneumonia l Onset is often sudden with fever, chills and rigors, and
vitamin D and convulsion may occur in children. Fever
i. Lobar pneumonia is usually high grade (39–40°C).
l Generally, the entire lobe of lung is involved. l Nonspecific symptoms include loss of appetite, head-
l The first stage is known congestion, which occurs ache and aching pains in the body and limbs.
within 24 hours. The parenchyma is highly vascular and l Localized pleuritic chest pain develops at an early stage.
oedematous plenty of bacteria and scanty neutrophils. It may be referred to the shoulder or abdominal wall.
l Second stage (red hepatization) is characterized by the l Cough is initially short, painful and dry, but soon be-
presence of numerous erythrocytes, neutrophils, des- comes productive.
quamated epithelial cells and fibrin in alveoli resulting l The sputum is characteristically rust-coloured (‘rusty’
in a red and airless lung, and a consistency similar to sputum), but occasionally frankly blood-stained.
that of liver. l Breathing is rapid, and shallow due to pleuritic pain.
l In the third stage of grey hepatization, lung parenchyma l Other features are tachycardia, hot and dry skin, herpes
becomes grey, dry and friable. labialis, flushed face and occasionally central cyanosis.
Section | I General Medicine 101

Physical signs in the chest ii. Cardiovascular:


l Acute circulatory failure
l In the first two days, physical signs are minimal. These
l Acute pericarditis
include:
l Endocarditis (rare)
a. Diminished respiratory movements,
b. Slight impairment of percussion note and iii. Pleural:
l Sterile pleural effusion
c. Pleural rub.
l Empyema
l Later, Frank signs of consolidation appear. These include
the following: iv. Neurological:
l Mental confusion
a. No mediastinal shift.
l Meningism
b. Diminished respiratory movements.
l Meningitis (rare)
c. Dull percussion note.
d. Markedly increased VF and VR.
e. High-pitched tubular bronchial breathing. Treatment
f. Bronchophony, aegophony and whispering pectoril-
oquy may be present. i. General measures:
l Oxygen in high concentrations should be adminis-
g. In the early stages, numerous fine crepitations are au-
dible, but later (during resolution) they become coarse. tered to all hypoxaemic patients.
l Treatment of pleuritic pain with mild analgesics like
h. If a parapneumonic effusion develops, additional
signs of pleural effusion appear. paracetamol. However, some patients require pethi-
dine 50–100 mg or morphine 10–15 mg intramuscu-
larly or intravenously.
Investigations l Assisted coughing in patients who suppress cough

l Marked neutrophil leucocytosis. because of pleuritic pain.


l Blood culture may show Streptococcus pneumoniae. ii. Antibiotic therapy:
l If the patient is not seriously ill, the initial treatment
l Gram staining of the sputum may demonstrate pneumo-
cocci as gram-positive lancet-shaped diplococci. should consists of one of the following:
l Sputum culture may show Streptococcus pneumoniae. a. Ampicillin 500 mg 4 times daily orally.
l Chest radiograph shows a homogeneous opacity localized to b. Erythromycin 500 mg 4 times daily orally.
the affected lobe or segment, with air bronchograms. Associ- c. Cotrimoxazole 960 mg 2 times daily orally.
ated parapneumonic effusion or empyema can be detected. d. Doxycycline 100 g twice a day orally.
l In moderate to severely ill cases, the treatment
l Serological tests can detect pneumococcal antigen in
the serum. consists of the following:
l In rare cases, fibreoptic, bronchoscopic aspiration or a. Ceftriaxone 2 g IV once a day or fluoroquino-
transthoracic needle aspiration is required. lone.
b. Oevofloxacin 500 mg once a day or gatifloxacin
400 mg once day.
Complications c. Ampicillin-clauvulanic acid 1.2 g 8-hourly intra-
venously.
i. Pulmonary:
l Delayed/incomplete resolution Total duration of therapy in an uncomplicated case is
l Spread to other lobes (rare) 7–10 days.

SHORT ESSAYS
Q. 1. Dyspnoea. Causes of acute dyspnoea
Ans. a. Respiratory causes
l Asthma
l Dyspnoea is defined as an abnormally uncomfortable l Acute exacerbation of COPD
awareness of breathing. l Pneumonia
l The pathophysiology of dyspnoea is multifactorial and l Pneumothorax
mainly includes hypoxia, hypercapnia, altered lung l Pulmonary embolism
and chest wall compliance, increased respiratory effort l ARDS
and respiratory muscle weakness. l Foreign body inhalation
l Dyspnoea can be acute or chronic. l Laryngeal oedema
102 Quick Review Series for BDS 3rd Year

b. Cardiac causes l Rare causes include pulmonary fungal infections, in-


l Acute pulmonary oedema fected cysts and vasculitis.

c. Other causes
l Metabolic acidosis (uraemia, diabetic ketoacidosis) Clinical Features
l Psychogenic History
Causes of chronic dyspnoea Lung abscess can have two modes of presentations:
l In majority, the disease starts acutely with high-grade
a. Respiratory causes
fever, chills and rigors, pleuritic chest pain and a dry
l COPD
cough.
l Interstitial lung disease
l The sputum is large in volume, purulent, foul smelling
l Pleural effusion
and often blood-tinged. The expectoration varies with
posture.
b. Cardiac causes
l In a few patients, the lung abscess is more insidious
l Chronic heart failure
in onset with low-grade fever, malaise, weight loss, an-
c. Other causes orexia and a deep-seated chest discomfort.
l Anaemia
Physical findings
l Obesity
l Dyspnoea, according to the severity, can occur only dur- l General examination reveals anaemia, fever, finger
ing exertion or it may be present even at rest. clubbing, halitosis and oronasal sepsis.
l Dyspnoea which occurs mainly at the night, i.e. noctural l Respiratory system examination may be normal in the
dyspnoea, may be due to asthma, sleep apnoea, gastro­ early stages. Later, Frank signs of consolidation like
oesophageal reflux or left ventricular dysfunction. dullness on percussion increased vocal fremitus and
vocal resonance, bronchial breathing, crepitations and
Q. 2. Lung abscess. pleural rub appear.
Ans. l Once the abscess cavity opens into a bronchus, signs of
cavitation like cavernous or amphoric bronchial breath-
Lung abscess is defined as necrotic area of lung paren- ing and coarse post-tussive crepitations appear.
chyma containing purulent material.
Investigations
Aetiology
l Anaemia, leucocytosis and raised erythrocyte sedimen-
l Aspiration of nasopharyngeal or oropharyngeal con- tation rate (ESR).
tents is the most common cause. l Sputum studies for acid-fast bacilli, aerobic and anaero-
l Depression of cough reflex favours aspiration. This bic cultures and sensitivity, and cytological examination
occurs during sleep, anaesthesia, alcohol intoxication, for malignant cells.
epilepsy and coma. l Chest radiograph often shows a radiolucency in an
l Aspiration also occurs in achalasia cardia, carcinoma of opaque area of consolidation.
oesophagus, hiatus hernia and gastroesophageal reflux l Bronchoscopy is indicated to exclude malignancy.
disease. l CT scan of thorax can detect lung abscess with certainty.
l Pre-existing sources of infection for aspiration include si-
nusitis, dental sepsis, gingivitis, periodontal infection, etc.
l The dominant organisms in aspiration abscess include Treatment
anaerobic organisms, streptococci and Haemophilus l Postural drainage and chest physiotherapy.
influenzae. l Antibiotic therapy of aspiration abscess is aspiration
l Aspiration abscesses are more common in the right lung. pneumonia.
l Pulmonary tuberculosis is an important cause of lung l Majority of the patients with lung abscess will respond
abscess. to oral treatment with ampicillin 500 mg 4 times daily
l Bronchial obstruction by foreign body, tumour or bron- or cotrimoxazole 960 mg twice daily.
chostenosis leads to abscess formation distally. l In anaerobic bacterial infection (e.g. those with foul
l Haematogenous spread of the organisms to the lung re- smelling sputum), oral metronidazole 400 mg 8 hourly
sults from bacteraemia, right-sided endocarditis and should be added, but it should never be used as a single
septic thrombophlebitis. agent.
Section | I General Medicine 103

l Seriously ill patients will require parenteral antibiotic therapy ii. Muscle tremors due to stimulation of b2 receptors
in the form of penicillin or clindamycin plus metronidazole. of skeletal muscle. Tolerance develops to this effect
l Duration of antibiotic therapy is variable. Some patients on continued administration.
may require prolonged treatment for 4 to 6 weeks. iii. Restlessness, nervousness, throat irritation and ankle
l Resectional surgery is indicated only in selected situa- oedema can occur.
tions like massive haemoptysis, localized malignancy, iv. On parentral administration, hyperglycaemia may
persistent abscess cavity, etc. occur in diabetics.
v. Hypokalaemia is due to shift of K1 into the cell.
Q. 3. Enumerate bronchodialators and write about
salbutamol. Q. 4. Pneumococcal pneumonia.
Ans. Ans.
Various bronchodialators are: Pneumococcal pneumonia or bacterial pneumonia is the
i. Sympathomimetics: Adrenaline, ephedrine, isoprena- most common form of pneumonia.
line, salbutamol, terbutaline, salmeterol, and formoterol.
ii. Methylxanthines: Theophylline, aminophylline, and
etophylline. Aetiology
iii. Anticholinergics: Atropine, methonitrate, ipratropium The causative organism is Streptococcus pneumoniae. The
bromide, and tiotropium bromide. disease spreads by droplet infection.
Salbutamol
l Salbutamol is a sympathomimetic, selective b2 adrener- Pathology
gic agonist drug acting as bronchodilator. There are three stages in the natural course of the illness.
l It is the fastest acting bronchodilator, convenient to use i. Stage of red hepatization
and relatively safe. ii. Stage of grey hepatization
l On inhalation, salbutamol has rapid onset within 1–5 iii. Stage of resolution
minutes and short duration of action, preferred for acute
attack of asthma.
Clinical Features
l Pharmacological actions of salbutamol are as follows:
i. Bronchodilatation l Onset is often sudden with fever, chills and rigors, and
ii. Relaxation of pregnant uterus vitamin D and convulsion may occur in children.
iii. Dilatation of blood vessels supplying the skeletal l Fever is usually high grade (39–40°C).
muscles l Nonspecific symptoms include loss of appetite, head-
iv. Promote hepatic glycogenolysis and uptake of po- ache and aching pains in the body and limbs.
tassium (K1) into the cells. l Localized pleuritic chest pain develops at an early
l Therapeutic uses of salbutamol are as follows: stage.
i. In bronchial asthma, it is usually administered as l Cough is initially short, painful and dry, but soon
aerosol. It produces prompt bronchodilatation with becomes productive.
minimum systemic side effects. l The sputum is characteristically rust-coloured (‘rusty’
ii. Salbutamol is used to delay the premature labour on sputum), but occasionally frankly blood-stained.
oral or parenteral administration. It relaxes pregnant l Breathing is rapid, and shallow due to pleuritic pain.
uterus by interacting with b2 receptors.
iii. It is useful in hyperkalaemia as it increases the Physical signs in the chest
uptake of K1 into the cells. l In the first two days, physical signs are minimal. These
l Route and dose: include:
i. Inhalation salbutamol 100–200 mcg every 6 hours a. Diminished respiratory movements,
or as and when required through metered-dose b. Slight impairment of percussion note and
inhaler (MDI). c. Pleural rub.
ii. It is available as metered-dose inhalers, nebulizers, l Later, Frank signs of consolidation appear. These include
tablets for oral use. the following:
iii. Other routes of administration are oral, IM, and IV. a. No mediastinal shift.
l Side effects: b. Diminished respiratory movements.
i. Cardiac side effects like tachycardia and palpitation c. Dull percussion note.
are less prominent. d. High-pitched tubular bronchial breathing.
104 Quick Review Series for BDS 3rd Year

e. In the early stages, numerous fine crepitations are au- i. Pulmonary emphysema
dible, but later (during resolution) they become coarse. l A generalized bilateral pulmonary emphysema occurs.
Compensatory emphysema is a form of pulmonary em-
Investigations physema which is localized.
l Marked neutrophil leucocytosis. l Compensatory emphysema is a condition where the
l Blood culture may show Streptococcus pneumoniae. normal lung tissue undergoes hypertrophy as a compen-
l Gram staining of the sputum may demonstrate pneumococci. satory mechanism for an extensive damage to the other
l Sputum culture. lung or to part of the same lung.
l Chest radiographs. l Being a compensatory phenomenon, this is asymp-
l Serological tests can detect pneumococcal antigen in tomatic.
the serum. l Physical findings in compensatory emphysema include
l In rare cases, fibreoptic, bronchoscopic aspiration or increased respiratory movements, hyperresonant per-
transthoracic needle aspiration is required. cussion note and increased intensity of the breath
sounds.
Treatment ii. Mediastinal emphysema
i. General measures l Mediastinal emphysema occurs as a result of escape
l Oxygen in high concentrations should be adminis- of air rapidly into the mediastinum following rupture
tered to all hypoxaemic patients. of overdistended alveoli in severe bronchial asthma,
l Treatment of pleuritic pain with mild analgesics like rupture of emphysematous bulla and rupture of
paracetamol. However, some patients require pethidine oesophagus.
50–100 mg or morphine 10–15 mg intramuscularly or l If severe, mediastinal emphysema may produce cardiac
intravenously. tamponade.
l Assisted coughing in patients who suppress cough l Auscultation may reveal a crunching sound known as
because of pleuritic pain. mediastinal crunch.
ii. Antibiotic therapy
l If the patient is not seriously ill, the initial treatment
iii. Subcutaneous emphysema
should consist of one of the following: l Subcutaneous emphysema is due to the presence of air
a. Ampicillin 500 mg 4 times daily orally. in the subcutaneous tissues.
b. Erythromycin 500 mg 4 times daily orally. l Common causes are penetrating chest injuries, fracture
c. Cotrimoxazole 960 mg 2 times daily orally. of ribs and intercostal tube introduction.
d. Doxycycline 100 g twice a day orally. l Clinically, subcutaneous emphysema imparts a charac-
l In moderate to severely ill cases, the treatment con-
teristic crepitation or crackling sensation on palpation.
sists of the following: l No treatment is indicated as the air will get absorbed
a. Ceftriaxone 2 g IV once a day or fluoroquinolone. slowly.
b. Oevofloxacin 500 mg once a day or gatifloxacin Q. 6. Acute severe asthma.
400 mg once day.
c. Ampicillin–clauvulanic acid 1.2 g 8 hourly intra- Or,
venously. Management of chronic bronchial asthma.
l Total duration of therapy in an uncomplicated case is
Ans.
7–10 days.
l Acute asthma is a severe life-threatening attack of
Q. 5. Clinical signs of emphysema.
asthma, previously known as status asthmaticus.
Ans. l The patient may additionally have tachycardia, pulsus
paradoxus, cyanosis and active accessory respiratory
muscles.
Compensatory Emphysema
l The air entry is drastically reduced. The patient may
l The word ‘emphysema’ means inflation or distension become confused or drowsy.
with air.
Based on the anatomical site, emphysema can be classi-
General Management
l

fied as follows:
i. Pulmonary emphysema l Avoidance of allergic factors: The identification of al-
ii. Mediastinal emphysema lergens is possible only in a few cases. Wherever pos-
iii. Subcutaneous emphysema sible, the exposure to such agents must be avoided.
Section | I General Medicine 105

l Desensitization: Desensitization by repeated adminis- Complications


tration of allergen has not been proven effective in most
The main complications of bronchial asthma include pneu-
studies.
mothorax, respiratory failure and cor pulmonale.
l Drug therapy: The drugs used in asthma can be grouped
as: (a) quick relievers, which inhibit smooth muscle Q. 7. Haematemesis—causes and management.
contraction and cause bronchodilatation; (b) long-term
Ans.
control medications, which prevent or reverse inflam-
mation. Haematemesis is the vomiting of blood. The source is gen-
erally the upper gastrointestinal tract.
Management of acute severe asthma
i. Hospitalization and oxygen therapy: The patient should Causes
be hospitalized for urgent management. Arterial blood
l Prolonged and vigorous retching.
gas analysis, chest radiograph and ECG should be done
l Irritation or erosion of the lining of the oesophagus or
to assess the severity and to rule out other causes. A
stomach.
high concentration oxygen inhalation is given.
l Bleeding ulcer located in the stomach, duodenum or
ii. Repeated dosage of short-acting adrenergic drugs (sal-
oesophagus.
butamol 2.5–5 mg or terbulaline 5–10 mg) is given
l Vomiting of ingested blood after haemorrhage in the
through a nebulizer at an interval of 20–30 minutes.
oral cavity, nose or throat.
iii. Systemic steroid: This is essentially needed in all cases
l Vascular malfunctions of the gastrointestinal tract, such
of severe asthma. Prednisolone (30–60 mg daily) or
as bleeding gastrointestinal varices.
methylprednisolone is given orally or intravenously.
l Tumours of the stomach or oesophagus.
iv. Others:
l Radiation poisoning.
a. Mechanical ventilation is needed in patients with
l Viral haemorrhagic fevers.
coma, respiratory arrest, exhaustion and deteriorat-
l Gastroenteritis.
ing blood gases despite adequate treatment.
l Gastritis.
b. IV fluid is administered to prevent dehydration.
l Peptic ulcer.
c. Antibiotics are not used routinely. These are only
l Chronic viral hepatitis.
given if infection is present.
l Intestinal schistosomiasis (caused by the parasite Schis-
d. Opiates, sedatives or tranquillizers are contraindi-
tosoma mansoni).
cated.
l History of smoking.

Management of chronic persistent asthma


Management
Depending on the symptoms, signs, frequency of exacer-
bations and PEF rate, the asthma is classified as mild, Haematemesis is treated as a medical emergency.
moderate or severe. A stepwise approach is chosen accord-
ing the severity of disease. Once the disease is controlled, Minimal blood loss
a stepdown therapy is attempted. The patient is generally administered a proton pump in-
i. Step 1: Inhaled short-acting adrenergic agents (salbu- hibitor (e.g. omeprazole), given blood transfusions (if the
tamol, terbutaline) are used as needed for minor level of haemoglobin is extremely low, i.e. ,8 g/dL or
symptoms. 4.5–5 mmol/L), and kept NPO (‘nothing by mouth’, or no
ii. Step 2: If the symptoms are not controlled with eating or drinking) until endoscopy can be arranged.
occasional inhaled adrenergic drugs as mentioned
above, low dose of inhaled steroid is added. Mast cell Significant blood loss
stabilizers can be used prophylactically before expo- l In a ‘haemodynamically significant’ case of haemateme-
sure to allergens, exercise or cold air. sis, i.e. hypovolaemic shock, resuscitation is an immedi-
iii. Step 3: The dose of inhaled steroid is escalated. An in- ate priority to prevent cardiac arrest.
haled long-acting adrenergic agent such as salmeterol l Fluids and/or blood is administered, preferably by cen-
or formoterol can be added. Sustained released theoph- tral venous catheter, and the patient is prepared for
ylline may also be used orally. emergency endoscopy and laparotomy is necessary.
iv. Step 4: In addition to drugs used in Step 3, leukotriene l Securing the airway is a top priority in haematemesis
modifiers and inhaled ipratropium bromide are tried. patients, especially those with a disturbed conscious
v. Step 5: Regular oral steroid in the lowest effective dos- level. A cuffed endotracheal tube could be a life-saving
age is added to Step 4 regimen to control symptoms. choice.
106 Quick Review Series for BDS 3rd Year

Q. 8. Chronic bronchitis—causes, clinical features and resolve themselves in a few weeks. For acute exacerba-
complications. tions of chronic bronchitis, if antibiotics are used,
amoxicillin or doxycycline is recommended.
Ans.
l Bronchodilators, e.g. ipratropium, may be useful for
Chronic bronchitis is a chronic inflammation of the bronchi people suffering from chronic obstructive pulmonary
(medium-sized airways) in the lungs. disease, such as chronic bronchitis.
Q. 9. Mantoux test.
Signs and Symptoms
Or,
l Bronchitis may be indicated by an expectorating cough,
Tuberculin test.
i.e. a productive cough, shortness of breath (dyspnoea)
and wheezing. Ans.
l Occasionally, chest pains, fever and fatigue or malaise
l The first infection with the tubercle bacillus leads to the
may also occur.
development of sensitization to the protein tuberculin.
l Mucus is often green or yellowish green and also may
When tuberculin is injected into the skin of an infected
be orange or pink, depending on the pathogen causing
person, a delayed local reaction develops in 24–48
the inflammation.
hours. This is mainly due to previously sensitized
CD41 lymphocytes.
Causes l There are two tests currently in wide use in tuberculin
testing:
l Tobacco smoking is the most common cause.
i. Mantoux test
l Pneumoconiosis and long-term fume inhalation are
ii. Heaf test
other causes.
l Allergies can also cause mucus hypersecretion, thus
leading to symptoms similar to bronchitis. Mantoux Test
It is ideal to begin the test with 5 IU PPD-S or one or two
Diagnosis IV PPD RT23.
l A physical examination will often reveal diminished
breath sounds, wheezing and prolonged expiration. The Method
presence of a persistent dry or wet cough is evidence of l Choose an area of skin at the junction of the mid and
bronchitis. upper thirds the dorsal surface of the left forearm. Clean
l A variety of tests may be performed in patients present- the area with soap and water and allow it to dry. Using a
ing with cough and shortness of breath, e.g. pulmonary tuberculin syringe and an intradermal needle, inject
function tests (PFT) or spirometry. 0.1 mL of the tuberculin solution strictly intradermally. It
l A chest radiograph that reveals hyperinflation, collapse should produce a papule in the skin 5–6 mm in diameter.
and consolidation of lung areas would support a diagno- l Reading and interpreting the result:
sis of pneumonia. l The test is read after 48–72 hours. If a reaction has

l A sputum sample showing neutrophil granulocytes and taken place, there will be an area of erythema (red-
culture showing that has pathogenic microorganisms ness) and an area of induration (thickening) of the
such as Streptococcus species. skin. Measure the diameter of induration across the
l A blood test would indicate inflammation as indicated transverse axis of the arm. The amount of erythema,
by a raised white blood cell count and elevated C-reac- i.e. redness present, is not important.
tive protein. l A positive reaction is an area of induration of the

l High resolution computed tomography (HRCT): This is skin with a diameter of 10 mm or more. This is one
a special type of CT scan that provides us with high- point in favour of the diagnosis. But remember that
resolution images of the lungs. many normal people have a positive test and so also
after previous BCG vaccination. However, a strongly
positive test is particularly valuable in children, espe-
Treatment cially very young children.
l Smoking cessation is of benefit. l If the diameter is below 10 mm, the test is negative.

l Antibiotics: Only about 5–10% of bronchitis cases are But a negative test does not exclude tuberculosis. A
caused by a bacterial infection. Most cases of bronchitis patient with active tuberculosis may have a suppressed
are caused by a viral infection and are ‘self-limited’ and tuberculin test (false negative) due to a number of
Section | I General Medicine 107

factors, e.g. malnutrition, viral infections, HIV infection, l Antibiotics are often given though there is no evidence
measles, chickenpox, glandular fever, cancer, severe that bacteria are important in the acute events.
bacterial infections, corticosteroids and similar drugs.
Q. 11. Treatment of haemoptysis.
Q. 10. Aspiration pneumonia.
Ans.
Ans.
i. Minor haemoptysis: Minor haemoptysis which is scant
l Aspiration pneumonia occurs due to abnormal entry of will stop spontaneously without specific therapy. Treat-
fluid, particulate exogenous substances or endogenous ment is aimed at the underlying cause.
secretions into the lower airways. ii. Substantial haemoptysis: It should be treated by keep-
l It is of two types: ing the patient calm, instituting complete bed rest and
i. Chemical aspiration pneumonia suppressing the cough. However, intubation and suction
ii. Bacterial aspiration pneumonia equipment should be ready at the bedside.
iii. Potentially lethal or massive haemoptysis
l Position the patient so that the side of the chest from
Predisposing Factors
which bleeding is arising is lowermost. This is to
l Reduced consciousness. prevent asphyxiation due to aspiration of blood into
l Dysphagia from neurologic deficits. the normal lung.
l Disorders of the upper gastrointestinal tract including l Set up an intravenous infusion and collect blood for
oesophageal disease, surgery involving the upper air- grouping and cross-matching. Maintain a chart of
ways or oesophagus, and gastric reflux. vital signs including blood pressure, pulse rate, re-
l Mechanical disruption of the glottic closure or cardiac spiratory rate and urine output.
sphincter due to tracheostomy, endotracheal intubation, l Administer oxygen.
bronchoscopy, upper endoscopy and nasogastric feeding. l Blood transfusions are given according to the usual
l Miscellaneous conditions such as protracted vomiting, clinical guidelines of quantity of blood lost, haema-
large volume tube feedings and feeding gastrostomy. tocrit, blood pressure, pulse rate and urine output.
l Strong sedatives should be avoided, but mild seda-
Chemical aspiration pneumonia tives may be given to relieve anxiety.
l It occurs due to aspiration of substances that are toxic to l Distressing cough may be suppressed with linctus
the lower airways, independent of bacterial infection. codeine 15 mL thrice daily.
l It includes chemical pneumonitis associated with the l Consider endotracheal intubation, if the patient has
aspiration of gastric acid (Mendelson’s syndrome). poor gas exchange, has rapid ongoing haemoptysis,
is haemodynamically unstable.
l An alternative strategy is to place an endotracheal
Clinical Features tube into either the right or left mainstem bronchus.
l Abrupt onset of symptoms with low-grade fever and This is easier to achieve with bleeding from the left
prominent dyspnoea. lung, when selective intubation of the right main-
l Examination reveals cyanosis and diffuse crepitations. stem bronchus is required.
l Chest radiograph shows infiltrates involving dependent l Double-lumen endotracheal tube allows the two

pulmonary segments which usually develop within 2 hours lungs to be isolated and ventilated separately.
of aspiration. l Consider emergency bronchoscopy, if the bleeding

l Some patients have a fulminant course and die shortly is torrential.


after aspiration, presumably from ARDS. l The rigid bronchoscope is preferred as it enables
l Others have a rapid clinical improvement with clearing of the blood to be aspirated more easily.
chest radiograph. Another subset of patients have initial rapid l The fibreoptic bronchoscope may be used for cold

improvement but then develop new expanding infiltrates on saline lavage, which may arrest bleeding some-
chest radiograph which probably represent secondary bacte- times.
rial infection superimposed on the acid-injured lung. l A balloon catheter passed through the bronchoscope

can be inflated proximally in the bleeding bronchus.


This will isolate the source of bleeding from the rest
Treatment of the lung and the contralateral preventing asphyx-
l Tracheal suction to clear fluids and particulate matter. iation by blood flooding.
l Support of respiration including mechanical ventilation, l Bronchial arterial catheterization and embolization

if required. can arrest bleeding, at least temporarily.


108 Quick Review Series for BDS 3rd Year

l Surgical intervention is indicated in selected cases. l In patients with obstructive lung disease, both total lung
Emergency resection of the lobe or lung which is capacity and residual volume are increased while these
bleeding may be necessary. are reduced in restrictive lung diseases due to parenchy-
mal diseases.
Q. 12. Pulmonary function tests.
l In extra-parenchymal diseases like ankylosing spondy-
Ans. litis, kyphoscoliosis, etc, RV is increased while TLC is
reduced.
i. Ventilatory capacity (FEV1, FVC, PEF)
ii. Reversibility of airflow limitation (by administering Flow volume curves
inhaled bronchodilator) l These measure flow rates against volume which helps in
iii. Flow volume curves analysing the site of obstruction within the lung.
iv. Airway resistance
v. Gas transfer (diffusion) Arterial blood gas analysis
vi. Lung volumes (TLC, RV) l Arterial blood gas (ABG) studies are done with auto-
vii. Blood gases matic analysers.
viii. Pulse oximetry l They measure PO2, PCO2 and H1 concentration in the
ix. Exercise tests arterial blood. Oxygen saturation and bicarbonate con-
centration can be derived from the above values.
Ventilatory capacity
l Arterial blood gas (ABG) studies are extremely useful in
l Using a spirometer forced expiratory volume in one the management of respiratory failure, status asthmati-
second (FEV1), forced vital capacity (FVC) and vital cus and acute respiratory distress syndrome (ARDS).
capacity (VC) are recorded.
l Two patterns of abnormalities can be detected as follows: Gas transfer factor
i. Obstructive ventilatory defect: Where there is nar- l Gas transfer factor is estimated by measuring uptake of
rowing of airways during expiration (e.g. bronchial carbon monoxide from a single breath and it helps to
asthma and chronic bronchitis) FEVl is markedly estimate the ability of the lungs to exchange gases.
decreased, VC decreased or normal and FEV1/VC l It is used in the assessment of interstitial lung disease,
decreased. emphysema and sarcoidosis.
ii. Restrictive ventilatory defect: Where FEVl is de-
creased, VC is decreased and FEVl/VC normal or Exercise test
increased. l The measurement of distance the patient can walk in 6
l Peak expiratory flow rate (PEF) can be measured during minutes is a simple method of ‘everyday’ exercise test.
forced expiration by a gauge or meter which is simpler l It is useful in detecting exercise-induced asthma, exer-
and cheaper than spirometer. Reduced values indicate cise hypoxaemia, etc.
airflow obstruction. It is of no use in restrictive ventila-
tory defect. Q. 13. BCG vaccination.
l Forced expiratory time (FET) is simple to measure, with Ans.
just a watch and a stethoscope. Normal people can
empty their chests from inspiration in 4 seconds or less. l Bacille Calmette–Guerin (BCG) is a live-attenuated and
Prolonged forced expiratory time of more than 6 seconds freeze-dried vaccine.
indicates airflow obstruction. l All the BCG strains used today all over the world are
derived from the original BCG strain maintained at
Reversibility of airflow limitation Pastuer Institute in Paris.
l Reversibility of airflow limitation is seen in bronchial
Dose and administration
asthma and up to some extent in chronic bronchitis.
l BCG vaccine is available either as fresh liquid vaccine
Lung volumes or in the form of freeze-dried (lyophilized) vaccine
which is commonly used.
l The values of lung volume are obtained either by dilut-
l The dose of the vaccine is a single intradermal injection
ing helium into the gas in the lungs or in whole body
of 0.1 mL containing 0.05–0.1 mg of the constituent
plethysmograph.
vaccine containing about 1–2 million viable organisms.
l Lung volumes include total lung capacity (TLC) and
residual volume (RV). It should be administered soon after birth failing which
l These are helpful in differentiating obstructive from re- it should be given anytime during the first year of life but
strictive lung diseases. not after age of 2 years.
Section | I General Medicine 109

Immune response l Emphysema can be classified based on the anatomical


Injection of the BCG vaccine induces a self-limited infec- site as follows:
tion with multiplication and dissemination of the bacillus in i. Pulmonary emphysema
different organs and production of small tubercles. This ii. Mediastinal emphysema
gives rise to delayed hypersensitivity and immunity. The iii. Subcutaneous emphysema
immunity may last for 10–15 years and is similar to the im-
munity following natural infection. i. Pulmonary emphysema
l Compensatory emphysema is a form of pulmonary em-
Complications physema which is localized.
The recognized complications of the BCG vaccination are: l This is a condition where the normal lung tissue under-
l Local abscess, indolent ulcer, keloid, tuberculides, con- goes hypertrophy as a compensatory mechanism for an
fluent lesion, lupoid lesions and lupus vulgaris. extensive damage to the other lung or to part of the
l Regional enlargement and suppuration of draining lymph same lung.
nodes. l Being a compensatory phenomenon, this is asymptomatic.
l Systemic fever, mediastinal adenitis, erythema nodo- l Physical findings in compensatory emphysema include
sum and very rarely nonfatal meningitis. increased respiratory movements, hyperresonant per-
cussion note and increased intensity of the breath
Contraindications sounds.
l Infants and children with active HIV disease.
l Babies born to mothers with AFB positive sputum can ii. Mediastinal emphysema
be given BCG vaccination after a course of preventive l Mediastinal emphysema occurs as a result of escape of
chemotherapy. air rapidly into the mediastinum following rupture of
overdistended alveoli in severe bronchial asthma and
Role of BCG rupture of oesophagus.
l Vaccination does not give absolute protection against l If it is severe, it may produce cardiac tamponade.
tuberculosis. However, it makes the disease milder and l Auscultation may reveal a crunching sound (mediastinal
noninfectious in immunized children. crunch).
l Vaccine prevents serious forms of primary tuberculosis,
such as meningitis, skeletal tuberculosis and military iii. Subcutaneous emphysema
spread of the disease. l Subcutaneous emphysema is due to the presence of air
Q. 14. Bronchial breathing. in the subcutaneous tissues.
l It is usually caused by penetrating chest injuries, frac-
Ans. ture of ribs and intercostal tube introduction.
l Bronchial breath sounds are harsh and poor in nature. l Clinically, subcutaneous emphysema imparts a charac-
l Unlike normal vesicular breath sounds, there is an au- teristic crepitation or crackling sensation on palpation.
dible gap between the inspiratory and expiratory phase l No treatment is indicated as the air will get absorbed
sounds. If they are heard in the chest, they are abnormal slowly.
and suggest the presence of consolidation or fibrosis. Q. 16. Widal test.
l The sounds of bronchial breathing are generated by tur-
bulent air flow in large airways and similar sounds can be Ans.
heard in healthy patients by listening over the trachea. Widal test is an agglutination test which detects presence of
l In healthy patients, however, these sounds are not con- serum agglutinins H and O in the serum of patients suffer-
ducted to the chest wall since they are attenuated by ing from enteric fever.
air-filled alveoli and lung parenchyma. l In enteric fever, antibodies to Salmonella start appear-
l In the presence of consolidation or fibrosis, however, ing in serum at the end of the first week and rises
the sound of air flow in the bronchi is conducted more sharply during the third week.
effectively to the chest wall rather than vesicular breath l It is preferable to test two specimens of sera at an inter-
sounds. val of 7–10 days to demonstrate a rising antibody titre.
Q. 15. Emphysema.
Procedure
Ans. Two types of test tubes are used in Widal test as follows:
l The word ‘emphysema’ means inflation or distension i. A narrow tube with a conical bottom (Dreyer’s tube) for
with air. H agglutination.
110 Quick Review Series for BDS 3rd Year

ii. A short round bottomed tube (Felix tube) for O aggluti- l The most common disorder of this type in developed
nation. countries is allergic bronchopulmonary aspergillosis, in
l Equal volume (0.4 mL) serial dilutions of serum tropical countries the presence of microfilariae in the
from 1:10 to 1:640, and H and O antigens are mixed pulmonary capillaries has to be considered.
in Dreyer’s and Felix tubes, respectively.
l Five test tubes are set up. Four for the test and the Acute eosinophilic pneumonia
one for a nonserum control for checking autoaggluti- l Acute eosinophilic pneumonia is a rare febrile illness
nation. of less than 5 days duration characterized by infil-
l The test tubes are thoroughly mixed by shaking the trates on the chest radiograph and hypoxic respira-
rack and then they are incubated in a water bath at tory failure.
37°C overnight. l Bronchoalveolar lavage demonstrates more than 25%
eosinophils. The condition responds to corticosteroids.
Observation
l The agglutination leads to formation of loose cotton- Treatment of pulmonary eosinophilia
wool clumps and O agglutination produces disc-like l Antimicrobials (nitrofurantoin, penicillin, tetracyclines,
granular deposit at the bottom of the tube in both the sulphonamides, nalidixic acid)
supernatant fluid rendered clear. l Antirheumatic agents (gold, aspirin, penicillamine,
l The antigens used in Widal test are the O and H antigens naproxen)
of S. typhi and the H antigens of S. paratyphi A and B. l Cytotoxic drugs (bleomycin, methotrexate, procarbazine)
l Psychiatric drugs (chlorpromazine, dosulepin, imipramine)
Result l Anticonvulsants (carbamazepine, phellftoin)
The highest dilution of patient’s serum in respect to each l Others (sulfasalazine, nadolol)
Salmonella antigen is read as the titre.
Q. 18. Interpretation of sputum of AFB (acid-fact bacilli)
Interpretation stain.
The result of the Widal test should be interpreted taking Ans.
into account the following:
i. The agglutinin titre depends on the stage of the disease. l An acid-fast bacilli (AFB) smear is a relatively quick
Agglutinins usually start appearing by the end of the way to determine if an infection may be due to one of
first week with sharp rise in second and third week, and the acid-fast bacilli, the most common of which is
the titre remains steady till the fourth week after which Mycobacterium tuberculosis.
it declines gradually. l AFB testing is ordered when someone has symptoms
ii. Demonstration of rising titre of four folds or greater of that suggest pulmonary TB or other mycobacterial lung
both H and O agglutinins at an interval of 4–7 days is infection.
the most important diagnostic criterion. A rising titre of l Someone has symptoms associated with a TB or other
H or O antibodies between test done in the first week mycobacterial infection located outside of the lungs
and third week is highly significant. (extrapulmonary).
In a single test, titre of 1:160 or above for O and 1:200 AFB Smear
or more for H signifies active infection, but this has to be
l A negative AFB smear means that no infection is pres-
interpreted with caution taking into consideration various
ent, that symptoms are caused by something other than
factors like local titre, immunization, anamnestic reaction,
mycobacteria, or that the mycobacteria were not present
effect of antibiotic treatment, etc.
in sufficient numbers to be seen under the microscope.
Q. 17. Describe aetiology, clinical features, investiga- l Positive AFB smears indicate a probable mycobacterial
tions and management of pulmonary eosinophilia. infection. However, a culture must be performed to con-
firm a diagnosis.
Ans.
l For people with signs and symptoms of an active TB
l Pulmonary eosinophilia is the term applied to a group infection, AFB smear results are considered together
of disorders of different aetiology in which lesions in with results from nucleic acid amplification tests
the lungs produce a chest radiograph abnormality asso- (NAAT) for TB, as recommended by the Centers for
ciated with pulmonary eosinophilia, with or without a Disease Control and Prevention (CDC).
peripheral blood eosinophilia. l Interpretation of smear and NAAT results are summa-
l There is no satisfactory classification of this disparate rized in the Table 6.1. Again, all results must be con-
group. firmed by results from culture.
Section | I General Medicine 111

TABLE 6.1 Interpretation of AFB smear and NAAT results gastric lavage (in children), bronchoalveolar lavage or
transbronchial biopsy.
AFB smear NAAT
ii. Culture methods: The culture provides confirmation
result result Interpretation
of the diagnosis by the isolation and identification of
Positive Positive Presumptive diagnosis for TB
Mycobacterium tuberculosis from the specimen. Drug
Negative Positive NAAT is more sensitive than smear sensitivity can also be tested.
so this may occur in patients with iii. Molecular methods: Nucleic acid amplification method
true disease; may test additional
provides diagnosis in hours but sensitivity is lower
samples using NAAT. If more than
one sample is positive by NAAT, than culture and the cost is high. It is useful in cases
this is a presumptive diagnosis with AFB negative pulmonary and extrapulmonary
for TB. tuberculosis.
Positive Negative Questionable results for TB; an iv. Radiographic tests: The typical findings include infiltra-
inhibitor may be present or the tion of upper lobe with fibrosis and/or cavity. However,
AFB seen on the smear are not any type of radiographic pattern such as consolidation,
M. tuberculosis. A test for the collapse, pleural effusion or miliary can be seen in
inhibitor may be performed.
tuberculosis.
Negative Negative Symptoms probably not due to v. Skin test: Purified protein derivative (PPD) skin test
mycobacterial infection. (using Mantoux method or Heaf method) is positive in
persons infected with M. tuberculosis.
vi. Histopathological tests: The fine needle aspiration cy-
Q. 19. Investigations for pulmonary tuberculosis. tology (FNAC) or biopsy specimens from the involved
tissue may typically reveal caseous granuloma.
Ans.
vii. Other tests: The haematological findings include anae-
The investigations for pulmonary tuberculosis are as follows: mia, raised ESR and C-reactive protein. The fluid
i. Demonstration of AFB: The diagnosis of tuberculosis (pleural, pericardial and peritoneal) is exudative in na-
is based on the demonstration of AFB in the smear of ture. Liver biopsy and bone marrow biopsy specimens
the sputum or in other specimens such as tissue biopsy may also be examined for the evidence of granuloma
materials or body fluids. Samples can be obtained by and AFB.

SHORT NOTES
Q. 1. Clubbing. ii. Cardiac causes
l Infective endocarditis
Or, l Congenital cyanotic heart diseases
Four causes of clubbing. iii. Alimentary causes
Ans. Ulcerative colitis
l Crohn’s disease
Clubbing is defined as a selective bulbous enlargement of l Cirrhosis of liver
the distal segment of a digit, due to an increase in soft l Hepatoma
tissue.
Common causes of clubbing are as follows: iv. Miscellaneous causes
l Hereditary
i. Respiratory causes l Idiopathic
l Acromegaly
l Bronchogenic carcinoma
l Unilateral clubbing in Pancoast’s tumour and subclavian
l Bronchiectasis
artery aneurysm
l Lung abscess
l Empyema Q. 2. Enumerate the causes or aetiology of bronchiectasis
l Mesothelioma
Ans.
l Long-standing tuberculosis
l Pulmonary AV fistula l Bronchiectasis is the term used to describe abnormal
l Interstitial lung diseases dilatation of the bronchi.
112 Quick Review Series for BDS 3rd Year

l It is usually acquired but may result from an underlying l It is of two types:


genetic or congenital defect of airway defences. i. Chemical aspiration pneumonia
ii. Bacterial aspiration pneumonia
Aetiology
Clinical Features
Congenital
l Abrupt onset of symptoms with low-grade fever and
l Cystic fibrosis
prominent dyspnoea.
l Ciliary dysfunction syndromes, e.g. Kartagener’s syndrome
l Examination reveals cyanosis and diffuse crepitations.
l Primary hypogammaglobulinaemia
l Chest radiograph shows infiltrates involving dependent
Acquired children pulmonary segments.
l Some patients have a fulminant course and die shortly
l Pneumonia (complicating whooping cough or measles) after aspiration.
l Primary TB l Others have a rapid clinical improvement with clearing
l Inhaled foreign body of the chest radiograph. Another subset of patients have
initial rapid improvement but then develop new expand-
Acquired adults
ing infiltrates which probably represent secondary bac-
l Suppurative pneumonia terial infection superimposed on the acid-injured lung.
l Pulmonary TB
l Allergic bronchopulmonary aspergillosis complicating
asthma Treatment
l Bronchial tumours l Tracheal suction to clear fluids and particulate matter
l Support of respiration
Q. 3. Stages of lobar pneumonia
l Antibiotics
Ans.
Q. 5. Organisms causing pneumonia.
Pneumonia is defined as the infection of the lung paren-
Ans.
chyma, i.e. alveoli and distal airways and interstitium of
the lung. l The term atypical pneumonia is used to describe pneu-
monia caused by agents such as Mycoplasma, Legionella,
Chlamydia and Coxiella.
Lobar Pneumonia l Primary pneumonia is caused by a specific pathogenic
l Lobar pneumonia is a radiological and pathological organism. There is no pre-existing abnormality of the
term referring to homogenous consolidation of one or respiratory system. The organisms causing primary
more lung lobes, often associated with pleural inflam- pneumonia are given below.
mation.
l Generally, the entire lobe of lung is involved. Common Organisms
l The first stage is known as congestion, which occurs
within 24 hours. l Streptococcus pneumoniae (most common)
l Second stage, red hepatization is characterized by the l Haemophilus influenzae
presence of numerous erythrocytes, neutrophils, des- l Moraxella catarrhalis
quamated epithelial cells and fibrin in alveoli resulting l Staphylococcus aureus
in a red and airless lung, and a consistency similar to l Legionella pneumophila
that of liver. l Mycoplasma pneumoniae
l In the third stage of grey hepatization, lung parenchyma
becomes grey, dry and friable. Less Common Organisms
l Finally, in the stage of resolution, the exudates are di-
gested and removed by scavenger or coughed out. l Klebsiella pneumoniae
l Streptococcus pyogenes
Q. 4. Aspiration pneumonia. l Pseudomonas aeruginosa
l Coxiella burnetii
Ans.
l Chlamydia pneumoniae
l Aspiration pneumonia occurs due to abnormal entry of l Chlamydia psittaci
fluid, particulate exogenous substances, or endogenous l Actinomyces israeli
secretions into the lower airways. l Viruses
Section | I General Medicine 113

Q. 6. Chronic obstructive lung disease. l It may be primary (patients infected with resistant bacilli
and have not received TB drugs earlier) or secondary/
Ans.
acquired (resistance develops during treatment).
Chronic obstructive lung disease (COPD) is a heterogenous l Factors which favour the development of resistance are
condition embracing several overlapping pathological pro- inadequate regimen, incomplete dosage or duration.
cess including chronic bronchitis, chronic bronchiolitis and l Multidrug-resistant TB (MDR-TB) is characterized by
emphysema. the resistance to more than one anti-TB drugs or resis-
tance to both isoniazid and rifampicin. The treatment of
Clinical Features MDR-TB needs second-line drugs for a longer duration.

l COPD should be suspected in any patient over the age Q. 8. Complications of pulmonary tuberculosis.
of 40 years who presents with the symptoms of cough Ans.
and sputum production and/or breathlessness.
l Cough is usually the first symptom. It is characteristi- The complications of pulmonary tuberculosis are as follows:
cally accompanied by small amounts of mucoid sputum. l Haemoptysis
l Physical signs are nonspecific, correlate poorly with l Pneumothorax
lung function. l Secondary infection of cavity
l Presence of pitting oedema should be documented. l Pleural effusion
l Two classical phenotypes have been described: pink l Empyema
puffers and blue bloaters. The former are typically thin l Pulmonary fibrosis
and breathlessness, and maintain a normal PaCO2 until l Bronchiectasis
the late stage of disease. The later develop hypercapnia l Persistence of cavities after treatment
earlier and may develop odema and secondary polycy- l Infection of cavities by Aspergillus
themia. l Scar carcinoma
l Spread of tuberculosis to other organs
l Respiratory failure and right heart failure
Investigations l Amyloidosis
l Chest radiograph l Anaemia
l Blood count Q. 9. Status asthmaticus.
l Measurement of lung volumes
l Exercise test Or,
Status asthmaticus—management.
Management Ans.
l Smoking cessation
l Acute asthma is a severe life-threatening attack of asthma,
l Bronchodilators
previously known as status asthmaticus.
l Corticosteroids
l The patient may additionally have tachycardia, pulsus
l Pulmonary rehabilitation
paradoxus, cyanosis and active accessory respiratory
l Oxygen therapy
muscles.
l Surgical intervention: In selected patients with COPD
l The air entry is drastically reduced. The patient may
like young patient with minimal airflow limitation and a
become confused or drowsy.
lack of generalized emphysema but in whom large bul-
lae compress surrounding normal lung tissue may be
considered for bullectomy. General Management
l Other measures: Like annual influenza or pneumococ- l Avoidance of allergic factors
cal vaccination. Obesity, poor nutrition, depression and l Desensitization
social isolation should be identified and improved, mu- l Drug therapy
colytic therapy(acetylcysteine—200 mg orally 8-hourly
for 8 weeks in the first instance). Management of acute severe asthma
Q. 7. Drug resistance in tuberculosis. l Hospitalization and oxygen therapy: The patient should
be hospitalized for urgent management. Arterial blood
Ans.
gas analysis, chest radiograph and ECG should be done
l Resistance to TB drugs develops because of spontane- to assess the severity and to rule out other causes. A
ous point mutation in the Mycobacterium genome. high concentration oxygen inhalation is given.
114 Quick Review Series for BDS 3rd Year

l Repeated dosage of short-acting adrenergic drugs (sal- l In healthy patients, however, these sounds are not con-
butamol 2.5–5 mg or terbulaline 5–10 mg) is given ducted to the chest wall since they are attenuated by
through a nebulizer at an interval of 20–30 minutes. air-filled alveoli and lung parenchyma.
l Systemic steroid: This is essentially needed in all cases l In the presence of consolidation or fibrosis, however,
of severe asthma. Prednisolone (30–60 mg daily) or the sound of air flow in the bronchi is conducted more
methylprednisolone is given orally or intravenously. effectively to the chest wall and bronchial rather than
l Others: Mechanical ventilation, IV fluids, antibiotics, if vesicular breath sounds are heard.
infection is present.
Q. 12. Hoarseness of voice.
Q. 10. Tonsillitis.
Ans.
Ans.
l Abnormal changes in the voice are called ‘hoarseness’.
l Tonsillitis is a gag reflex inflammation of the tonsils l When hoarse, the voice may sound breathy, raspy,
most commonly caused by viral or bacterial infection. strained or show changes in volume or pitch.
l Aetiology: l Causes of hoarseness are as follows:
l The most common causes of tonsillitis are the com- i. Acute laryngitis: The most common cause is acute
mon cold viruses. It can also be caused by Epstein– laryngitis.
Barr virus, herpes simplex virus, cytomegalovirus ii. Voice misuse: Speaking in noisy situations, exces-
or HIV. sive use, telephone use with the handset cradled to
l The second most common causes are bacteria like the shoulder, not using amplification during public
group A-haemolytic Streptococcus (GABHS), speaking.
Staphylococcus aureus, Streptococcus pneumoniae, iii. Benign vocal cord lesions, e.g. nodules, polyps
Mycoplasma pneumoniae, etc. and cysts.
iii. Common symptoms of tonsillitis are: iv. Gastroesophageal reflux disease (GERD)
l Red and/or swollen tonsils v. Laryngopharyngeal reflux disease (LPRD)
l White or yellow patches on the tonsils vi. Smoking
l Tender, stiff and/or swollen neck vii. Neurological diseases or disorders like Parkinson’s
l Sore throat, painful or difficulty in swallowing disease or a stroke
l Cough, headache, sore eyes, body aches, fever, viii. Other causes include allergies, thyroid problems,
chills and nasal congestions trauma to the voice box and occasionally very seri-
iv. Treatment: ous conditions such as laryngeal cancer.
l Pain relief, anti-inflammatory, antipyretic (acetamino- l Treatment: Vocal nodules, polyps and cysts are typically
phen/paracetamol or ibuprofen) treated with a combination of microsurgery and voice
l Sore throat relief (salt water gargle, lozenges, warm therapy.
liquids)
Q. 13. Tracheal foreign body.
l If the tonsillitis is caused by group A Streptococcus,

then antibiotics are useful with penicillin or amoxi- Ans.


cillin being first-line. Macrolides such as erythro-
l Young children comprise the most common age group
mycin are used for patients allergic to penicillin.
for foreign body aspiration.
l When tonsillitis is caused by a virus, usually, a com-
l After foreign body aspiration occurs, the foreign body
plete recovery is made within one week.
can settle into three anatomic sites, the larynx, trachea
l Chronic cases may be treated with tonsillectomy as
or bronchus; 80–90% become lodged in the bronchi.
a choice for treatment.
l Clinical presentation depends on the location of the
Q. 11. Bronchial breathing. foreign body.
l A large foreign body lodged in the larynx or trachea can
Ans.
produce complete airway obstruction from either the
l Bronchial breath sounds are harsh and poor in nature. dimensions of the object or the resulting oedema.
Unlike normal vesicular breath sounds, there is an au- l Laryngeal foreign bodies present with airway obstruc-
dible gap between the inspiratory and expiratory phase tion and hoarseness or aphonia.
sounds. l Tracheal foreign bodies present similarly to laryngeal
l If heard in the chest, these are abnormal and suggest the foreign bodies but without hoarseness or aphonia.
presence of consolidation or fibrosis. l Bronchial foreign bodies typically present with cough,
l The sounds of bronchial breathing are generated by unilateral wheezing, and decreased breath sounds, but
turbulent air flow in large airways. only 65% of patients present with this classic triad.
Section | I General Medicine 115

l Investigation: radiograph is obtained, usually at a larger health centre,


i. Imaging studies like high-kilovolt anteroposterior and the patient is evaluated.
and lateral radiographs of the airway, posteroante- l For DOTS, the patients with tuberculosis are catego-
rior and lateral chest radiographs are an adjunct to rized into three categories and treatment is given as per
the history and physical examination. Lateral decu- recommendations.
bitus chest films may be helpful in children.
Q. 15. Sputum of AFB (acid-fact bacilli) stain.
ii. Biplane fluoroscopy.
iii. Diagnostic procedures: Chest auscultation. Ans.
l Medical therapy:
i. Patients with complete airway obstruction require l An acid-fast bacillus (AFB) smear is a relatively
immediate medical attention. quick way to determine if an infection may be due to
ii. Use of the Heimlich manoeuvre. one of the acid-fast bacilli, the most common of
iii. Surgical therapy: An operating room well-equipped which is Mycobacterium tuberculosis.
with proper endoscopic equipment of various sizes, l AFB testing is ordered when someone has symptoms
personnel familiar with the use of the instrumenta- that suggest pulmonary TB or other mycobacterial lung
tion and anaesthesiologists experienced in foreign infection.
body removal are critical for safe removal of airway l Someone has symptoms associated with a TB or other
foreign bodies. mycobacterial infection located outside of the lungs
(extrapulmonary).
Q. 14. Directly observed therapy (DOT) on tuberculosis.
Ans. AFB Smear
l A negative AFB smear means no infection is present,
that symptoms are caused by something other than
Directly Observed Treatment Short-course mycobacteria, or that the mycobacteria were not
(DOTS) present in sufficient numbers to be seen under the
l ‘DOTS’ means administering potent antimycobacterial microscope.
regimens in an intermittent manner to patient with l Positive AFB smears indicate a probable mycobacterial
tuberculosis under direct supervision. infection. However, a culture must be performed to con-
l Despite short-course therapy, tuberculosis remains the firm a diagnosis.
leading infectious cause of death in India, killing close For people with signs and symptoms of an active TB
to 5,00,000 people every year. infection, AFB smear results are considered together with
l DOTS is the backbone of revised National TB Control results from NAAT for TB, as recommended by the Centers
Programme (NTCP) in India. for Disease Control and Prevention (CDC).
l DOTS is a five-point programme which can ensure
effective TB control Q. 16. CT scan.
i. Political and administrative support. Ans.
ii. Diagnosis by sputum microscopy in patients attend-
ing health facilities. l CT scan provides detailed images of the pulmonary
iii. Good drugs for short-course chemotherapy. parenchyma, mediastinum, pleura and bony structures.
iv. Directly observed treatment which should be acces- l Sophisticated software facilitates 3D reconstruction of
sible, acceptable and accountable the thorax and virtual bronchoscopy.
v. Systematic monitoring and accountability. l CT scan is superior to chest radiography in determining
the position and size of a pulmonary lesion and whether
calcification or cavitation is present.
Implementation of DOTS l It is now routinely used in the assessment of patients
l Patients with cough for 3 weeks or more are encouraged with suspected lung cancer and facilitates guided percu-
to attend the nearest healthcare facility for sputum ex- taneous needle biopsy. Information on tumour stage
amination over a two-day period. may be gained by examining the mediastinum, liver and
l If two or three of the smears are positive for acid-fast adrenal glands.
bacilli, antituberculosis treatment is initiated. If all three l A high-resolution CT (HRCT) scanning use thin sec-
smears are negative, one to two weeks of broad- tions to provide a detailed assessment of the pulmonary
spectrum antibiotics are prescribed. If only one of the parenchyma and is particularly useful in assessing dif-
three smears is positive or if symptoms persist after the fuse parenchymal lung disease, identifying bronchiec-
administration of broad-spectrum antibiotics, a chest tasis and assessing the type and extent of emphysema.
116 Quick Review Series for BDS 3rd Year

Q. 17. Define haemoptysis Q. 19. Koilonychia.


Or, Ans.
Haemoptysis—four common causes. Koilonychia literally means ‘spoon nails’, which refers to
nails that become flat, lose their convex contour and re-
Ans.
placed by a central concave contour with raised edges, and
l Haemoptysis is defined as the expectoration of blood or lose their natural colour.
bloody sputum.
l ‘Potentially lethal’ or ‘massive’ haemoptysis is more Causes
than 600–800 mL blood in 24 hours.
l Iron-deficiency with or without anaemia.
l It can be linked to other causes, such as thyroid dys-
Causes of Haemoptysis function, renal disease, impaired peripheral circulation,
Common causes systemic lupus erythematosus, hypoplastic patella
musculoskeletal conditions, haemochromatosis, Raynaud’s
l Pulmonary tuberculosis
disease, nail–patella syndrome and hypoplastic patella.
l Bronchial carcinoma
l Trauma
l Chronic bronchitis
l Malnutrition
l Bronchiectasis
l Injury of nail plate
l Lung abscess
l Constant occupational exposure of hand nails to petro-
l Pneumonia (particularly Klebsiella)
leum, diesel and other petroleum-based products and
Uncommon causes solvents
l Excessive use of oils and soaps
l Pulmonary thromboembolism
l Left ventricular failure
l Pulmonary arteriovenous malformation Symptoms of Koilonychia
l Primary pulmonary hypertension l Thin and brittle nails
l Goodpasture’s syndrome l Concavity of nails
l Idiopathic pulmonary haemosiderosis l Detached nails
l Haemorrhagic diathesis l Nails that come off completely
Q. 18. Mention four causes of haematemesis. l The appearance of ridges developing along the nails
l The colour of the nail changes slowly to light yellow
Ans. colour
Haematemesis is the vomiting of blood. The source is
generally the upper gastrointestinal tract. Treatment
l Koilonychia should be treated as soon as it is recog-
Causes nized consulting a specialist.
l Since it is associated with iron deficiency, take iron-
l Prolonged and vigorous retching
based foods and essential vitamins.
l Irritation or erosion of the lining of the oesophagus or
l Using an emollient intended especially for the nails
stomach
right after a shower or washing hands will keep the nails
l Vomiting of ingested blood after haemorrhage in the
nourished and koilonychia at bay.
oral cavity, nose or throat
l Tumours of the stomach or oesophagus Q. 20. Respiratory causes of clubbing.
l Radiation poisoning
Ans.
l Viral haemorrhagic fevers
l Gastroenteritis The respiratory causes of clubbing are as follows:
l Gastritis l Bronchogenic carcinoma
l Peptic ulcer l Bronchiectasis
l Chronic viral hepatitis l Lung abscess
l Smoking l Empyema
Section | I General Medicine 117

l Mesothelioma l Emphysema can be classified based on the anatomical


l Long-standing tuberculosis site as follows:
l Pulmonary AV fistula i. Pulmonary emphysema
l Interstitial lung diseases ii. Mediastinal emphysema
iii. Subcutaneous emphysema
Q. 21. ARDS management.
Ans.
Subcutaneous Emphysema
Acute respiratory distress syndrome (ARDS) describes the
l Subcutaneous emphysema is due to the presence of air
acute, diffuse pulmonary inflammatory response to either
in the subcutaneous tissues.
direct or indirect blood-borne insults that originate from
l Common causes are penetrating chest injuries, fracture
extrapulmonary pathology.
of ribs and intercostal tube introduction.
l Clinically, subcutaneous emphysema imparts a charac-
Treatment of ARDS teristic crepitation or crackling sensation on palpation.
l No treatment is indicated as the air will get absorbed
General slowly.
l ARDS is usually treated with mechanical ventilation in
the intensive care unit. Ventilation is usually delivered Q. 23. Bronchiectasis—three causes and three complica-
through orotracheal intubation or tracheostomy. tions.
l Appropriate antibiotic therapy must be administered as Ans.
soon as microbiological culture results are available.
The causes of bronchiectasis are as follows:
Mechanical ventilation i. Congenital
l Cystic fibrosis
l The overall goal is to maintain acceptable gas exchange
l Ciliary dysfunction syndromes
and to minimize adverse effects in its application.
l Primary hypogammaglobulinaemia

Prone position ii. Acquired children


l Pneumonia (complicating whooping cough or measles)
l Distribution of lung infiltrates in acute respiratory dis-
l Primary TB
tress syndrome is nonuniform. Repositioning into the
l Inhaled foreign body
prone position (face down) might improve oxygenation
iii. Acquired adults
by relieving atelectasis and improving perfusion.
l Suppurative pneumonia

l Pulmonary TB
Fluid management
l Allergic bronchopulmonary aspergillosis
l Several studies have shown that pulmonary function and
outcome are better in patients that lost weight or pulmonary The complications of bronchiectasis are as follows:
wedge pressure was lowered by diuresis or fluid restriction. i. Collapsed lung
ii. Heart failure, if the disease advances to affect all parts
Corticosteroids of the airways
l Patients with ARDS do not benefit from high-dose cor- iii. Brain abscess
ticosteroids.
Q. 24. Respiratory failure
Nitric oxide Or,
l Inhaled nitric oxide (NO) potentially acts as selective
Enumerate causes of respiratory failure.
pulmonary vasodilator. Rapid binding to haemoglobin
prevents systemic effects. It should increase perfusion Ans.
of better ventilated areas.
l The term respiratory failure is used when pulmonary
Q. 22. Subcutaneous emphysema. gas exchange fails to maintain normal arterial oxygen
and carbon dioxide levels.
Ans.
l It is classified into type I and type II related to the pres-
l The word emphysema means inflation or distension with air. ence or absence of hypercapnia.
118 Quick Review Series for BDS 3rd Year

Causes More severe cases can include signs such as:


l Cyanosis,
i. Type I
l Collapse and
l Acute asthma
l Circulatory instability due to decreased blood flow
l Pulmonary odema
through the lungs.
l Emphysema

l ARDS

ii. Type II Treatment


l COPD l In most cases, anticoagulant therapy is the mainstay of
l Acute severe asthma treatment.
l Upper airway obstruction l Acutely, supportive treatments, such as oxygen or analgesia.
l Kyphoscloliosis l Anticoagulation: Heparin, low molecular weight heparins.
l Narcotic drugs l Warfarin therapy often requires frequent dose adjust-
ment and monitoring of the INR. INRs between 2.0 and
Management 3.0 are generally considered ideal.
Q. 27. Pulmonary eosinophilia.
Rapid reversal of the precipitating event, e.g. tracheostomy
for laryngeal obstruction, fixation of ribs in a flail chest Ans.
injury, reversal of narcotic poisons, nebulized bronchodila- Pulmonary eosinophilia is the term applied to a group of
tors in acute severe asthma or tube drainage of a tension disorders of different aetiology in which lesions in the
pneumothorax will restore good gas exchange. lungs produce a chest radiograph abnormality associated
Q. 25. Enumerate complications of pneumococcal with pulmonary eosinophilia, with or without a peripheral
pneumonia. blood eosinophilia.

Ans.
Acute Eosinophilic Pneumonia
Pneumonia is defined as the infection of the lung parenchyma
(alveoli and distal airways) and interstitium of the lung. l Acute eosinophilic pneumonia is a rare febrile illness of
The complications of pneumococcal pneumonia are as less than 5 days duration characterized by infiltrates on
follows: the chest radiograph and hypoxic respiratory failure.
i. Pulmonary: l Bronchoalveolar lavage demonstrates more than 25%
l Para-pneumonic pleural effusion
eosinophils. The condition responds to corticosteroids.
l Emphysema

l Suppurative pneumonia/lung abscess Treatment of Pulmonary Eosinophilia


l ARDS
i. Antimicrobials (nitrofurantoin, penicillin, tetracyclines,
l Pneumothorax
sulphonamides, nalidixic acid)
ii. Extrapulmonary: ii. Antirheumatic agents (gold, aspirin, penicillamine,
l Hepatitis, pericarditis, meningoencephalitis
naproxen)
l Multiorgan failure
iii. Cytotoxic drugs (bleomycin, methotrexate, procarbazine)
l Ectopic abscess formation
iv. Psychiatric drugs (chlorpromazine, dosulepin, imipramine)
Q. 26. Pulmonary embolism. v. Anticonvulsants (carbamazepine, phenytoin)
vi. Others (sulfasalazine, nadolol)
Ans.
Q. 28. Mantoux test
l Pulmonary embolism is a blockage of the main artery of
the lung or one of its branches by a substance that has Or,
travelled from elsewhere in the body through the blood- Tuberculin test.
stream (embolism).
Ans.
l Usually this is due to embolism of a thrombus from the
deep veins in the legs, a process termed venous throm- There are two tests currently in wide use in tuberculin testing:
boembolism. i. Mantoux test
ii. Heaf test
Symptoms
Mantoux Test
l Sudden-onset dyspnoea (shortness of breath)
l Tachypnoea Method
l Chest pain of a ‘pleuritic’ nature Choose an area of skin at the junction of the mid and upper
l Cough and haemoptysis thirds of the dorsal surface of the left forearm. Clean the area
Section | I General Medicine 119

using a tuberculin syringe and an intradermal needle, inject Q. 31. Two causes of dull note on percussion of the chest
0.1 mL of the tuberculin solution strictly intradermally. It
Ans.
should produce a papule in the skin 5–6 mm in diameter.
l Percussion over the solid structures such as liver or a
Reading and interpreting the result consolidated area over the lung produces dull note.
l The test is read after 48–72 hours. l Percussion over fluid filled area such as pleural effusion
l A positive reaction is an area of induration of the skin produces extremely dull, i.e. stony dull note.
with a diameter of 10 mm or more. l Dull or thud-like sounds are normally heard over dense
l If the diameter is below 10 mm, the test is negative. But areas such as the heart or liver. Dullness replaces reso-
a negative test does not exclude tuberculosis. A patient nance when fluid or solid tissue replaces air-containing
with active tuberculosis may have a suppressed tubercu- lung tissues, such as when it occurs with pneumonia,
lin test (false negative) due to a number of factors. pleural effusions or tumours.
Q. 29. Heaf test. Q. 32. Lung cancer—three extrapulmonary manifesta-
Ans. tions

Heaf test is performed with a Heaf gun. It consists of six Ans.


spring-loaded needles which when fired pierce the skin The extrapulmonary manifestations of lung cancer are as
through a drop of undiluted PPD. follows:
l Dyspnoea
Method l Haemoptysis
l Using a dropper, place a drop of undiluted PPD on the l Chronic coughing or change in regular coughing pattern
clean dry skin at the junction of the mid and upper third l Wheezing
of the anterior surface of the left forearm. The sterilized l Chest pain or pain in the abdomen
Heaf gun is placed over the drop of tuberculin and the l Cachexia (weight loss), fatigue and loss of appetite
gun fired, allowing the needles to pierce the skin. l Dysphonia (hoarse voice)
l Dysphagia (difficulty swallowing)

Reading the Test Q. 33. Investigations for pulmonary tuberculosis.

l The test is read after 48–72 hours. The result is recorded Ans.
as one of the four grades.
The investigations for pulmonary tuberculosis are as follows:
l Grade 0: No reaction.
i. Demonstration of AFB
l Grade I: Palpable indurations (papule) around at
ii. Culture methods
least four puncture points.
iii. Molecular methods (nucleic acid amplification method)
l Grade II: Papules have formed a ring.
iv. Radiographic tests
l Grade III: A solid area of induration has been formed.
a. Skin test: Mantoux method or Heaf method.
l Grade IV: There are shiny vesicles over the solid area.
b. Histopathologic tests: FNAC (fine needle aspiration
There may be surrounding erythema.
cytology) or biopsy.
l Grade II, III and IV indicate a positive reaction.
vii. Other tests: The haematological findings include anae-
Q. 30. Name few antituberculosis drugs. mia, raised ESR and C-reactive protein. The fluid (pleu-
ral, pericardial, peritoneal) is exudative in nature. Liver
Ans.
biopsy and bone marrow biopsy specimens may also be
Antitubercular drugs are bactericidal or bacteriostatic in na- examined for the evidence of granuloma and AFB.
ture. The bactericidal drugs are used to rapidly reduce the
Q. 34. Clubbing—grading.
number of viable organisms and render patients noninfectious;
they prevent relapse. Bacteriostatic drugs are used along with Ans.
the bactericidal drugs to prevent emergence of resistance.
Clubbing is defined as a selective bulbous enlargement of
The first-line drugs are:
the distal segment of a digit, due to an increase in soft tissue.
l Rifampicin (R)
Clubbing is graded as follows:
l Isoniazid (H)
i. Grade I: Increased fluctuation of the nail bed.
l Pyrazinamide (Z)
ii. Grade II: Obliteration of the angle between nail and
l Ethambutol (E)
nail bed (Lovibond angle; normal 160°).
Isoniazid and rifampicin are active against all popula- iii. Grade III: Increased curvature of the nail resulting in a
tion of TB bacilli while pyrizinamide and streptomycin are parrot beak or drumstick appearance.
active against certain population of TB bacilli. iv. Grade IV: Hypertrophic osteoarthropathy.
120 Quick Review Series for BDS 3rd Year

Topic 7

Diseases of Nervous System


LONG ESSAYS
Q. 1. Describe aetiology, clinical features and localiza- l Geniculate ganglion - reduced lacrimation and sali-
tion of infranuclear facial nerve palsy. vation, involvement of eigth nerve.
l Pons - Associated sixth nerve paralysis with contra-
Or
lateral pyramidal signs (Millard-Gubler syndrome).
Describe the aetiology, clinical features and manage- l Supranuclear paralysis - Relative sparing of upper
ment of facial palsy. half of face, ipsilateral pyramidal signs.
Ans.
l Infranuclear facial nerve palsy means involvement of 7th Investigations
nerve in pons or along its path from pons to its exit from l There are no specific confirmatory diagnostic procedures.
stylomastoid foramen. l Electromyography (EMG)
l There is a complete paralysis of all the muscles of ex-
pression of face, hence furrows are lost on the forehead
on affected side while looking upwards. Treatment
l The patient is asked to wear a pad over the affected eye
to protect it from irritants.
FACIAL PALSY
l If patient is seen within 1 week, a short of predniso-
Aetiology lone:30-40 mg daily in divided doses for 5-7 days and
then tapering it over next 5-7 days has been found effec-
l Supra nuclear: Cerebrovascular disease, neoplasms, de- tive by reducing inflammation and oedema around the
myelinating diseases. facial nerve and speeds up the recovery.
l Nuclear: Brainstem infarction, neoplasm, poliomyelitis l Acyclovir at a dosage of 800 mg 5 times a day for 5 days.
l Infranuclear: Cerebellopontine angle tumours, acoustic l Surgical decompression is not attempted usually be-
neuroma, meningioma, choleasteatoma. cause majority of patients recover well without any in-
l Meninges: TB meningitis, meningovascular syphilis terferences.
l Temporal bone: Fracture, otitis media, Ramsay-Hunt
syndrome (herpes zoster), glomus jugulare tumours Q. 2. Classify epilepsy. Describe clinical features, diag-
l Extracranial: Leprosy, parotid tumours and surgery, nosis and management of grand mal epilepsy.
facial wounds, sarcoidosis, Bell’s palsy. Or
Mention the types of epilepsy. Describe the clinical fea-
Clinical Features tures and treatment of any one of them.
i. Paralysis of muscles of facial expression Ans.
ii. Lower motor neuron paralysis:
l Absence of furrowing of forehead,
EPILEPSY
l Inability to close eye with uprolling of eyeball

(Bell’s phenomenon), l Epilepsy is defined as a condition in which the sufferer


l Flattening of nasolabial fold, is prone to experience recurrent, usually spontaneous
l Drooping of the angle of the mouth. seizures.
iii. Localization is by the following features. l A seizure is a discrete event resulting from transient,
l Stylomastoid foramen - isolated involvement of facial hypersynchronous abnormal neuronal behaviour.
muscles. l The clinical manifestations of a seizure are highly vari-
l Facial canal-loss of taste from anterior two-thirds of able, based on the location of abnormal neuronal dis-
tongue (chorda tympani), hyperacusis (nerve to sta- charges. It may include motor, psychic and sensory
pedius). phenomenon, with or without loss of consciousness.
Section | I General Medicine 121

Classification of Seizures l There may be postictal headache, confusion, automatic


behaviour and violence.
A. Partial (focal) seizures
i. Simple partial seizures
ii. Complex partial seizures Diagnosis of Grand Mal Epilepsy
iii. Partial seizures evolving to secondarily generalized l Diagnosis of grand mal epilepsy is made by careful as-
seizures sessment of patient’s history documented by the diag-
nostic studies.
B. Primary generalized seizures l These include blood test to assess for the metabolic
i. Absence (petitmal) seizures disarray, brain imaging using MRI or CT scan and
ii. Tonic clonic seizures EEG.
iii. Myoclonic seizures l It also depends upon its onset and symptoms like
iv. Tonic seizures biting of tongue, passing of urine in clothes, injury to
v. Atonic seizures the patient, loss of consciousness and post-epileptic
features.
C. Unclassified seizures
i. Infantile spasm
Management
ii. Neonatal seizures
I. Immediate treatment of an attack of fit:
D. Status epilepticus l The patient should be protected from the injury. He
Grand mal [tonic clonic seizures]: should be moved away from fire and sharp and hard
l This is a common type of epilepsy seen in clinical practice. object.
l Phases recognized during grand mal epilepsy are as l Padded mouth gag is inserted between the teeth to avoid
follows: tongue injury.
i. Prodromal phase l Clear airway should be maintained.
ii. Aura l Diazepam 5 to 10 mg slow IV injection is given.
iii. Tonic and clonic phase
iv. Postictal phase II. Long Term Drug Therapy:
l Phenytoin sodium 200 to 400 mg daily.
i. Prodromal phase: l Carbamazepine 600 to 1800 mg daily in divided dose.
During this phase, patient feels uneasiness or irritability l Sodium valproate 400 to 2000 mg daily.
lasting for hours or days before an attack. l Phenobarbitone 60 to 180 mg daily.
l Primidone 750 to 1500 mg daily in divided dose.
ii. Aura:
Various anticonvulsants that can be used are as follows:
l This is seen when partial seizure becomes generalized. i. Barbiturates - phenobarbitone
l Symptoms of partial seizure, i.e. visual hallucinations, ii. Deoxybarbiturate - primidone
nausea, epigastric discomfort, alteration of psychic iii. Hydantoin - phenytoin
functions, jerking of one limb, etc. constitute an aura. iv. Iminostilbene – carbamazepine
v. Succinimide - ethosuximide
iii. Tonic and clonic phase: vi. Aliphatic carboxylic acid - sodium valproate
l Tonic contraction of muscles, flexion of arms, extension vii. Benzodiazepines - clonazepam, diazepam, clobazam
of legs, a cry due to spasm of respiratory muscles occur viii. Phenyltriazine - lamotrigine
during tonic phase lasting for 10-30 seconds followed ix. Cyclic GABA analogue - gabapentin
by loss of consciousness. x. Newer drugs -vigabatrin, topiramate, tiagabine, leveti-
l This is immediately followed by clonic phase during racetam
which there is violent jerking of face and limbs biting of
the tongue, and in consistence of urine and faeces. This III. Social and psychological aspects:
phase lasts for 1-5 mints. l Patients and relatives should be told about the illness, its
precipitating factors and consequences.
iv. Postictal phase:
l Restriction should be in children as they are more likely
l Deep unconsciousness with flaccid limbs, loss of cor- to be in danger. Cycling, driving and swimming is
neal reflex and plantar extensor response may occur avoided.
during this phase. This phase lasts for few mints to sev- l Patient should be advised to take occupation, in which
eral hours. neither he nor the community is on risk.
122 Quick Review Series for BDS 3rd Year

Q. 3. What is cerebrovascular accident? How will you ii. Secondary (symptomatic) parkinsonism:
manage a patient of hemiplegia? a. post-encephalitic.
b. Toxins, e.g. methylphenyltetrahydropyridine (MPTP),
Ans.
manganese, carbon monoxide.
Cerebrovascular accident or stroke is defined as abrupt on- c. Drugs, e.g. reserpine, phenothiazines, butyrophe-
set neurological disorder of vascular aetiology. Stroke may nones, alpha-methyldopa, metoclopramide.
be haemorrhagic or ischaemic in nature. d. Ischaemic (vascular parkinsonism)
e. Tumours
A. Types of haemorrhagic strokes f. Punch-drunk syndrome in boxers.
l Primary intracerebral haemorrhage. iii. Parkinsonism plus (degenerative disorders):
l Subarachnoid haemorrhage. a. Progressive supranuclear palsy
l Primary intraventricular haemorrhage b. Multisystem degeneration
c. Striatonigral degeneration
B. Types of ischaemic strokes d. Wilson’s disease
l Completed stroke. e. Huntington’s disease in children.
l Transient ischaemic attacks
l Reversible ischaemic neurological deficit Pathology
l Lacunar infarcts.
i. There is loss of pigmented cells in substantia nigra with
deposition of lewy bodies, This results in degeneration
HEMIPLEGIA of nigrostriatal pathway.These cells normally synthe-
Hemiplegia is the paralysis of one side of the body. sise dopamine.
ii. With degeneration of these cells, dopamine levels in the
striatum gets depleted; this is the major chemical pa-
Management of Hemiplegia thology in Parkinson’s disease.
Investigations iii. Additional cell loss is also seen elsewhere in basal
ganglia.
Most useful initial investigation is CT scan.
l Angiogram. Clinical Features
l MRI.
l Histopathology. i. Affects both sexes equally.
ii. Starts usually during fifth decade and later. Occasion-
Treatment ally, it may appear during fifth decade.
l Stroke is managed with supportive measures, anti- iii. Initial symptoms may be muscular ache, either bilater-
oedema measures and care of risk factors. ally or unilaterally.
l Intracerebral haemorrhage and aneurysmal subarach- iv. Mild slowness of activity or depression may be present.
noid haemorrhage may or may not require surgery. v. Later, classical combination of tremors, rigidity and
l Infective conditions will require appropriate anti- hypokinesia appear.
microbial therapy with or without surgery.
l Neoplasms will require surgery for decompression and Diagnosis
radiotherapy, if they are malignant. i. It is purely clinical and there is no definitive diagnostic
Q. 4. Discuss the aetiopathogenesis, clinical features of test to conform Parkinson’s disease.
parkinsonian disease. Outline the drugs used in its ii. If clinical features are not strictly with in the syndrome
treatment. or if the disease occurs in earlier life, Wilson’s disease
will have to be ruled out by estimating serum copper
Ans. ceruloplasmin and urinary copper levels.
Parkinsonism is a syndrome consisting of akinesia and bra- iii. Symptoms that suggest a diagnosis other than Parkin-
dikinesia, rigidity and tremors. It is usually associated with son’s disease include lack of response to levodopa, hal-
abnormalities in postural righting reflexes. lucinations, prominent and early dementia, early pos-
tural instability, severe and early autonomic dysfunction,
upward gaze paralysis, and involuntary movements
Classification other than tremors.
i. Primary parkinsonism: Paralysis agitans or Parkinson’s iv. CT scan or MRI is required, if these additional signs are
disease or idiopathic parkinsonism. present.
Section | I General Medicine 123

Management C. Secondary headaches:


If any offending drug was used, it should be withdrawn. a. Intracranial pathology
l Vascular
Patient should be seen by a physiotherapist and occupa-
l Infective
tional therapist.
l Neoplastic

l Traumatic
Drug Therapy b. Extracranial pathology
l Eyes
i. Anti-cholinergics: Trihexyphenidyl, benzhexol and/or
l Ears
phenadrine are useful in controlling tremors.They
l Sinuses
should be avoided, if the patient is above 65 years.
l Teeth
ii. Amantatine: It potentiates endogenous dopamine and
l Neck
has mild anti-parkinsonian effect.
l Temporomandibular joint
iii. L-dopa: It is administered orally. It is combined with a
peripheral decarboxylase inhibiter. D. Systemic causes:
Currently used combinations are: Fever, hypoxia, hypercapnia, hypertension, anaemia, al-
l L-dopa1carbidopa (4:1 OR 10:1 ratio) lergy, drugs, neuralgias, etc.
l L-dopa1benserazide (4:1 ratio)

iv. Dopamine receptor agonists: Bromocriptine, lisuride E. Depression


and pergolide. These are used as an alternative or an
addition to levodopa therapy. Dosage: It is started with MIGRAINE
2.5 mg/day and increased slowly up to a maximum of
60 mg/day over several months. Migraine is defined as recurrent attacks of headache varied
v. Selegiline in intensity frequency and duration and is commonly unilat-
vi. Catechol-o-methyltransferase (comt) inhibitors: tol- eral in onset and is associated with anorexia and sometimes
capone and entacapone. with nausea and vomiting.

Surgery Clinical Features


i. Stereotactic thalamotomy.
ii. Neural transplant techniques like adrenal medullary i. The headache follows and the pain is confined on one
cells or fetal mid-brain cell implantation into the brain. side, occasionally it may be bilateral.
iii. Deep brain stimulation. ii. Nausea and vomiting may be present and last for the
few hours.
Q. 5. Enumerate the causes of headache. Describe the iii. Light and noise sensitivity is present.
clinical features, management and prevention of mi- iv. Fatigue and stress are present in some of the cases.
graine. v. There is presence of polyuria.
Ans.
Management
Headache can be a symptom with numerous possible
causes, ranging from a mild head injury to a serious brain A. During attack:
tumour, or it could be a disorder itself like migraine. i. Analgesics: Aspirin or paracetamol with metaclo-
pramide 5 to 10 mg.
Classification of Headache ii. Ergotamine: Ergotamine tartrate 0.25 to 0.5 mg IM or l
to 2 mg orally.
A. Physiological: iii. Sumatriptan: It is a specific and selective agonist of
l Hunger 5-HT1 receptors of cranial blood vessels causing va-
l Sleep deprivation soconstriction. 6 mg subcutaneous gives relief in one
l Hangover hour. Oral dose of 100 mg provides relief in two
hours.
B. Primary headaches: iv. General: Lying in dark and quite room.
l Migraine
l Tension headache B. Reducing frequency and severity of attacks:
l Cluster headache i. Elimination of trigger factors
l Benign paroxysmal headaches ii. Relaxation exercises
124 Quick Review Series for BDS 3rd Year

iii. Drugs: l Coma


l Sedatives: Amitryptiline 100-150 mg l Tuberculosis and fungal infections may present as
l Serotonin inhibitors: Calcium antagonists such as chronic meningitis
methysergide 1 to 2 mg TDS should be given for not
more than 3 to 4 months. Investigations
l Propranolol: 20 mg BD is helpful.
l Routine haematology and biochemistry
l Ergotamine tartrate: 0.25 to 0.5 mg IM
l CT scan or MRI of brain
l Hormones: Progesterone given for last eight days is
l Lumbar puncture and CSF analysis
useful for migraine occurring in immediate premen-
strual period.
Treatment
Q. 6. Describe the aetiology, and clinical features of men-
ingitis. How would you proceed to establish the diagnosis? l Specific therapy is to be based on CSF analysis.
l Bacterial infections - Antibiotics (penicillin, ampicillin,
Or ceftriaxone, gentamicin)
What are the causes of meningitis? Describe the clinical l Viral infections - Acyclovir, ganciclovir, HAART
features, complications and treatment of pyogenic men- l Fungal infections - Amphotericin B, fluconazole
ingitis. Parasitic - Metronidazole
l Supportive measures - Steroids, analgesics and antipy-
Ans. eretics, anti-emetics, IV fluids, anti-convulsants, intra-
Inflammation of the meninges and the subarachnoid space venous mannitol or oral glycerol.
is called as meningitis.
Meningitis can be classified into three main categories: Complications
I. Acute purulent (pyogenic, septic, bacterial)
l Neurological deficiencies: Hemiplagia, aphasia, hemi-
II. Viral (aseptic)
anopia, blindness, deafness.
III. Chronic meningitis (tuberculous, fungal)
l Mental deterioration, cerebritis, brain abscess, focal fits,
auditory impairments, sub-dural empyema, internal hy-
drocephalous.
Aetiology
Infection PYOGENIC MENINGITIS
i. Bacterial: Listeria monocytogenes, E.coli, H. influenzae,
l The bacterial or pyogenic meningitis is usually due to
Neisseria, pneumococci, Mycobacterium tuberculosis.
secondary bacteraemic illness or spread of infection
ii. Virus: Enterococci, mumps, influenza virus, herpes
from adjacent structures like ear, nose, sinus or frac-
simplex virus, Epstein-Barr virus, HIV.
tured skull. Old age, alcoholism, diabetes, and splenec-
iii. Protozoan parasite: Toxoplasmosis, amoeba
tomy are additional risk factors.
iv. Fungi: Candida, Histoplasma, Blastomyces.
l Meningococcal meningitis results from air-borne infection.
l The disease requires early diagnosis and prompt treat-
Non-infectious ment otherwise it is associated with high morbidity and
i. Malignant diseases: Breast cancer, lung cancer, leukae- mortality.
mia, lymphoma.
ii. Inflammatory: Sarcoidosis, systemic lupus erythematosus
Clinical Features
Clinical Features i. The classic triad is fever, headache and neck rigidity.
ii. Drowsiness
l Headache iii. Neck stiffness: During neck flexion, there is pain in pos-
l Fever terior part of the neck which is radiating in the nature.
l Nuchal rigidity iv. Kernig’s sign is positive, i.e. a sign of meningeal irri-
l Photophobia tation evidenced by reflex contraction and pain in
l Projectile vomiting hamstring muscles, when attempting to extend the leg
l Altered sensorium often flexing the hip.
l Blurring of vision v. Brudzinskis sign: Elicited when patient is in supine
l Seizures position. Passive flexion of neck causes spontaneous
l Cranial nerve palsies flexion of hips and knee joint.
Section | I General Medicine 125

vi. Leg sign: Flexion of one side of leg causes passive l S. pneumoniae needs antibiotic therapy for two weeks
flexion of other side of leg. whereas L. monocytogenes and gram-negative bacilli
vii. Patients with meningococcal meningitis develop pur- infections require a 3 weeks therapy.
puric skin rashes and adrenocortical failure with shock
(Waterhouse-Friderichsen syndrome). Adjunctive Therapy
viii. Circulatory collapse may occur.
l Dexamethasone decreases morbidity and mortality in
pneumococcal meningitis in adults and meningitis due to
Diagnosis
S. pneumoniae and H. influenzae in infants and children.
l Lumbar puncture and CSF examination: For pressure, l Ten milligrams of dexamethasone is given 15-20 minutes
cell count, protein, glucose, smear stain for bacteria and before or concurrent with the first dose of antibiotic.
culture. l Therapy is continued at the same dosage every 6 hours
l Immediate CT scan is performed to rule out intracranial for 4 days.
space occupying lesions.
l Blood culture: Helpful in detection of organisms. Supportive Therapy
l Patients with raised intracranial pressure (ICP) are man-
Treatment aged in intensive care unit.
l Bacterial meningitis is an emergency. Antimicrobial l Intravenous mannitol, hyperventilation and elevation of pa-
treatment should be started as soon as possible. tient’s head to 30 degrees are measures to reduce raised ICP.
l Empirical treatment is started before the CSF culture Q. 7. Discuss the clinical features, complications and
and Gram stain report is available. It is directed against management of tubercular meningitis.
the most common microorganism present in a particular
age group. Ans.
l Meningeal involvement by the Mycobacterium tubercu-
Antimicrobial Treatment losis may occur through hematogenous spread in cases
l Ceftriaxone or cefotaxime provides adequate coverage of primary or post-primary pulmonary disease.
against S. pneumoniae, H. infuenzae, Group B strepto- l The rupture of the brain tubercle into the subarachnoid
cocci and N. meningitidis. Vancomycin is added to space may also cause meningitis.
cover cephalosporin resistant S. pneumoniae. Ampicil-
lin is added to cover L. monocytogenes in patients less Clinical Features:
than 3 months of age or more than 55 years and also in
immunocompromised patients. Ceftazidime is active The clinical features of tubercular meningitis are:
against P. aeuroginosa and is preferred over ceftriaxone l The onset of symptoms is insidious.
or cefotaxime in hospital-acquired meningitis, post- l Patients have headache, vomiting, low grade fever, and
traumatic and post-surgical meningitis. alteration in sensorium, confusion and behaviour changes.
l The choice of emperical antibiotics in bacterial menin- l Lassitude, i.e. weariness or exhaustion, depression
gitis is as follows: l Signs of meningeal irritation are present.
a. Neonates and infants ,3 months n Ampicillin 1 l Other signs are cranial nerve (oculomotor) palsies, pap-
ceftriaxone or cefotaxime. illoedema and focal neurological deficit.
b. Children and adults n Ceftriaxone or cefotaxime 1 l Hydrocephalus is a common complication.
vancomycin
c. Adults .55 years n Ampicillin 1 ceftriaxone or Investigations
cefotaxime 1 vancomycin
d. Hospital-acquired meningitis, post-traumatic or post- Diagnosis is made by CSF examination:
surgical, immuncompromised patients n Ampicillin l The CSF is straw-coloured clear but when allowed to
1 ceftazidime 1 vancomycin stand, a fine clot (spider web) is formed.
l There is high lymphocyte count, high protein and low
glucose.
Duration of Therapy l In acute cases, polymorphs may predominate.
l The duration of antibiotic therapy depends on the type l The AFB stain may be positive.
of organism. l Culture for AFB is positive in 80% cases.
l A one week therapy is given in case of H. influenzae and l CT or MRI brain may show meningeal enhancement or
N. meningitidis infection. hydrocephalous.
126 Quick Review Series for BDS 3rd Year

Management of Tubercular Meningitis l Painful strong muscle spasms of respiratory muscles


may result in hypoventilation; apnoea and cyanosis may
a. General management:
develop.
i. Maintenance of nutrition, hydration and electrolyte l The laryngeal spasms is a constant threat for reduced
balance. ventilation.
ii. Care of bowel and bladder. l Dysphagia and urinary retention occur due to oesopha-
iii. Nursing should be good. geal and urethral spasm.
iv. If there are convulsions, anticonvulsants are given. l The autonomic disturbances complicate severe tetanus, are
characterized by labile (common) or sustained (uncom-
b. Pharmacological management:
mon) hypertension, tachycardia, arrhythmias, fever,
Antitubercular treatment should be started as soon as sweating with increased catecholamines level.
possible. l Sudden cardiac arrest may occur.
i. Anti-tubercular drugs: Inj. streptomycin 1 gm IM daily l Bradycardia and hypotension have also been reported.
1 Tab isonex 600-900 mg/day 1 Tab ethambutol
ii. Addition of steroids for initial few weeks increases
survival and reduces complications. Prednisolone 40- Diagnosis
60 mg/day to reduce toxicity, pia-arachnoid adhesions Diagnosis is entirely based on clinical features:
and feeling of well-being. i. Risus sardonicus, trismus, opisthotonos with sparing of
iii. Mortality is high, if treatment is delayed. muscles of hand and feet.
ii. Generalised spasms-laryngeal, oesophageal, respira-
Complications tory and urethral muscles.
iii. Nonimmunised or partially immunised status of an
Complications like hydrocephalous, focal deficits, and cra- individual.
nial nerve palsies may occur. l There is no single test that can confirm the diagnosis
Q. 8. Describe the clinical features, diagnosis and treat- including culture of organism from the wound.
ment of tetanus. l Tests that may be helpful in diagnosis are:
i. Total leucocyte count may show leucocytosis.
Ans. ii. The electromyogram (EMG) may show continuous
l Tetanus is a disorder of neuromuscular excitability motor unit discharges and shortening or absence of
caused by an exotoxin (tetanospasmin) elaborated by silent interval that are normally seen on EMG after
Clostridium tetani, characterised by rigidity and power- an action potential.
ful muscle spasms. iii. The electrocardiogram (ECG) may be normal or
l It manifests clinically in many forms, i.e. generalized, may show tachycardia or arrhythmias in case auto-
neonatal, cephalic and localized. nomic dysfunction occurs.
iv. Muscle enzymes (creatinine phosphokinase, aldol-
ases) may be high during acute phase.
Clinical Features v. Serum antitoxin levels ./5 0.01 IU/ml are protec-
l Incubation period is 7 days but it varies from 3-14 days. tive and tetanus is unlikely at this level.
l The rigidy and generalized spasms are the cardinal
manifestations of generalized tetanus. Treatment
l Patient experiences a feeling of unwell and lockjaw or
trismus due to hypertonicity of masseter muscles. The goals of treatment are to eliminate the source of toxin,
l Dysphagia is common. neutralize unbound toxin, prevent muscle spasms, maintain
l Rigidity of abdomen and proximal limb muscles occurs patent airway by tracheostomy and ventilatory support, if
within 24-72 hours of initial symptoms. needed.
l Neck, shoulder and backpain or stiffness appears.
l The hand and feet muscles are spared. A. General measures
l Risus sardonicus (grinning expression due to sustained i. Nursing care-the patient is nursed in a dark, quiet, well
facial muscle spasm) and opisthotonos (arched back ventilated isolated room. In gastric feed may be necessary.
due to sustained contraction of back muscles) occur ii. Nutrition and caloric intake.
either spontaneously or may be induced or provoked iii. Wound debridement and cleansing to be instituted to
by noise, light and handling of the patient, i.e. just put- eliminate the anaerobic environment of the wound.
ting the hand over the abdomen may provoke abdomi- iv. Maintain the vital functions.
nal spasm. v. Tracheostomy.
Section | I General Medicine 127

B. Antibiotics C. Antitoxin
i. Pencillin:10-12 million units IV daily for 10 days or Human tetanus immunoglobulin (TIG) given IM in doses of
benzathine pencillin 2.4 3 10 units provides bacteri- 3000-6000 IU in divided doses, although 1500 IU is consid-
cidal concentration for a period of 5 days. ered sufficient.
ii. Metronidazol (500 mg 6 hourly or 1 g 12 hourly) be-
cause of its anaerobicidal action. In case of pencillin D. Control of muscle spasms
sensitive patients, tetracyclines, clindamycin and eryth- Diazepam is given as continuous IV infusion in doses of
romycin are used. 100-200 mg daily.

SHORT ESSAYS
Q. 1. Migraine. These are contraindicated in patients with coronary ar-
tery disease, uncontrolled hypertension and migraine with
Ans.
neurological deficits. analgesics (meperidine) can be used
l Migraine is defined as episodes of unilateral throbbing in severe acute attacks.
headache, nausea and vomiting and symptoms of neuro-
Q. 2. Bell’s palsy.
logical dysfunction.
l The various types of migraine are as follows: or
a. Classical migraine: In classical migraine, headache
Bell’s palsy-clinical features and management.
is characteristically associated with premonitory
sensory, motor or visual symptoms (aura). Ans.
b. Common migraine: Headache without aura is called
Bell’s palsy is an acute apparently isolated, lower motor
common migraine and is more frequent type of
neuron facial palsy.
migraine.
c. Migraine equivalent: Rarely migraine can present
with focal neurological deficit without headache, it Aetiology
is known as migraine equivalent.
i. Cold: Occurs after exposure to cold.
d. Complicated migraine: Migraine with transient focal
ii. Trauma: Extraction of teeth or injection of local anes-
neurological features or that leaves a persistent neu-
thetic may damage the nerve leading to subsequent
rological deficit is called complicated migraine.
paralysis.
iii. Surgical procedure: Such as parotidectomy in which the
Clinical Features facial nerve is sectioned can also cause facial paralysis.
iv. Tumours: Tumours of the cranial base, parapharyngeal
i. The headache confined usually to one side, but occa-
space and infratemporal fossa.
sionally it may be bilateral.
v. Familial: Familial and hereditary occurrence is also
ii. Nausea and vomiting.
reported in case of Bell’s palsy.
iii. Light and noise sensitivity is present.
vi. Facial canal and middle ear neoplasm.
iv. Fatigue and stress.
vii. Herpes simplex - viral infection.
v. The visual aura is most common and is in the form of
scotomas, hallucinations and fortification spectra. The
later is pathognomonic for migraine. Clinical Features
Symptoms
Management i. Sudden onset following exposure to chills or without
i. Elimination of trigger factors: The trigger tactors are any apparent precipitating factors.
identified and best avoided, e.g. alcohol, red wine, ii. Maximum weakness occurs in 48 hours.
chocolate, etc. ii. Post-auricular pain.
ii. Treatment of acute attack: iii. Spontaneous complaints of loss of sense of taste
l Rest in a quite darkened room is helpful. iv. Hyperacusis.
l Analgesics (aspirin, paracetamol and other NSAIDs) v. Sweating is less on the affected side.
are effective, if taken early at the onset of headache.
iii. Severe attacks may be treated with triptans (sumatrip- Signs
tan, zolmitriptan) and ergotamine. i. Forehead is not wrinkled and frowning is lost.
128 Quick Review Series for BDS 3rd Year

ii. Inability to close the eye on the affected side and on FACIAL PALSY
attempting closure eyeball turns upwards and outwards
(Bell’s phenomenon). Aetiology
iii. On showing teeth, the lips do not separate on the l Supranuclear – cerebrovascular disease, neoplasms,
affected side. demyelinating diseases
iv. Food collects between the teeth and paralysed cheek. l Nuclear- brainstem infarction, neoplasm, poliomyelitis
v. Saliva dribbles from angle of the mouth on paralysed l Infranuclear – cerebellopontine angle tumours, acoustic
side. neuroma, meningioma, choleasteatoma.
vi. Loss of taste in the anterior two-thirds of tongue on the l Meninges – TB meningitis, meningovascular syphilis
same side. l Temporal bone – fracture, otitis media, Ramsay-Hunt
syndrome (herpes zoster), glomus jugulare tumours
Extracranial – leprosy, parotid tumours and surgery,
Investigations
l

facial wounds, sarcoidosis, Bell’s palsy.


i. MRI – may show swelling and enhancement of genicu-
late ganglion.
Clinical Features
ii. EMG – prognostic value.
i. Paralysis of muscles of facial expression
ii. Lower motor neuron paralysis:
Management l Absence of furrowing of forehead.

l Inability to close eye with uprolling of eyeball


i. Local heat: Infrared or moist heat over the face or
parotid region or both, if there is tenderness of nerve (Bell’s phenomenon).
l Flattening of nasolabial fold.
trunk.
l Drooping of the angle of the mouth.
ii. Local treatment of muscles: The patient should mas-
sage the facial muscles with bland oil for twice a day iii. Localization is by the following features.
l Stylomastoid foramen - isolated involvement of facial
for 5 min.
iii. Protection of eye: It is done with dark glass or eye muscles.
l Facial canal-loss of taste from anterior two-thirds
patch. Mild zinc boric solution is used to wash the eye
to prevent conjunctivitis. of tongue (chorda tympani), hyperacusis (nerve to
iv. Prednisolone 90 mg/ day along with amoxicillin 250 mg stapedius).
l Geniculate ganglion - reduced lacrimation and sali-
8 hourly helps in reducing oedema round about the
nerve. vation, involvement of eigth nerve.
l Pons-associated sixth nerve paralysis with contralat-
v. Heavy doses of vitamin B12 1000 gm per day IM is
given. eral pyramidal signs (Millard-Gubler syndrome).
l Supranuclear paralysis - relative sparing of upper
vi. Galvanism: It is given two weeks after the onset of
paralysis three times in a week. half of face, ipsilateral pyramidal signs.
vii. Surgery: Plastic surgery is preferred.
Investigations
l There are no specific confirmatory diagnostic procedures.
Prognosis l Electromyography(EMG)
l Over 80% of patients recover completely in a few weeks
time. Treatment
l Patients with complete paralysis have a less favourable l The patient is asked to wear a pad over the affected eye
prognosis than those with incomplete paralysis. to protect it from irritants.
l If patient is seen within 1 week, a short of predniso-
Q. 3. Facial palsy.
lone:30-40 mg daily in divided doses for 5-7 days and
Ans. then tapering it over next 5-7 days has been found effec-
tive by reducing inflammation and oedema around the
l Infranuclear facial nerve palsy–means involvement of
facial nerve and speeds up the recovery.
7th nerve in pons or along its path from pons to its exit
l Acyclovir at a dosage of 800 mg 5 times a day for 5 days.
from stylomastoid foramen.
l There is a complete paralysis of all the muscles of Surgical decompression is not attempted usually be-
expression of face, hence furrows are lost on the fore- cause majority of patients recover well without any inter-
head on affected side while looking upwards. ferences.
Section | I General Medicine 129

Q. 4. Generalized tonic clonic seizures–clinical features. ii. Long-term drug therapy:


l Phenytoin sodium or carbamazepine or sodium val-
Ans.
proate or phenobarbitone or primidone may be used.
iii. Social and psychological aspects:
GRANDMAL EPILEPSY [TONIC CLONIC l Patients and relatives should be told about the ill-
SEIZURES] ness, its precipitating factors and consequences.
Cycling, driving and swimming are to be avoided.
l This is a common type of epilepsy seen in clinical practice.
l Patient should be advised to take occupation, in
l Phases recognized during grand mal epilepsy are as
which neither he nor the community is on risk.
follows:
i. Prodromal phase Q. 5. Meningitis–aetiology and clinical features.
ii. Aura
Ans.
iii. Tonic and clonic phase
iv. Postictal phase Inflammation of the meninges and the subarachnoid space
is called as meningitis.
i. Prodromal phase:
During this phase, patient feels uneasiness or irritability Aetiology
lasting for hours or days before an attack.
Infection
ii. Aura:
i. Bacterial: Listeria monocytogenes, E.coli, H. influenzae,
l This is seen when partial seizure becomes generalized. Neisseria, pneumococci, Mycobacterium tuberculosis.
l symptoms of partial seizure, i.e. visual hallucinations, ii. Virus: Enterococci, mumps, influenza virus, herpes
nausea, epigastric discomfort, alteration of psychic simplex virus, epstein-barr virus, HIV.
functions, jerking of one limb, etc. constitute an aura. iii. Protozoan parasite: Toxoplasmosis, amoeba.
iv. Fungi: Candida, Histoplasma, Blastomyces.
iii. Tonic and clonic phase:
l Tonic contraction of muscles, flexion of arms, extension Non-infectious
of legs, a cry due to spasm of respiratory muscles occur i. Malignant diseases: Breast cancer, lung cancer, leukae-
during tonic phase lasting for 10-30 seconds followed mia, lymphoma.
by loss of consciousness. ii. Inflammatory: Sarcoidosis, systemic lupus erythematosus
l This is immediately followed by clonic phase during
which there is violent jerking of face and limbs biting of
the tongue, in consistence of urine and faeces. This Clinical Features
phase lasts for 1-5 min. l Headache
l Fever
iv. Postictal phase: l Nuchal rigidity
l Deep unconsciousness with flaccid limbs, loss of corneal l Photophobia
reflex and plantar extensor response may occur during this l Projectile vomiting
phase. This phase lasts for few minutes to several hours. l Altered sensorium
l There may be postictal headache, confusion, automatic l Blurring of vision
behaviour and violence. l Seizures
l Cranial nerve palsies
Diagnosis of Grand Mal Epilepsy l Coma
l Tuberculosis and fungal infections may present as
l Diagnosis of grand mal epilepsy is made by careful as- chronic meningitis.
sessment of patient’s history documented by the diag-
nostic studies. Q. 6. Tuberculus meningitis.
Ans.
Management Meningeal involvement by the Mycobacterium tuberculosis
i. Immediate treatment of an attack of fit: may occur through hematogenous spread in cases of pri-
l The patient should be protected from the injury and mary or post-primary pulmonary disease.
clear airway should be maintained. The rupture of the brain tubercle into the subarachnoid
l Diazepam 5 to 10 mg slow IV injection is given. space may also cause meningitis.
130 Quick Review Series for BDS 3rd Year

The clinical features of tubercular meningitis are: l Followed by dullness after pain.
l The onset of symptoms is insidious. l Attacks may be precipitated by cold water, face wash-
l Patients have headache, vomiting, low grade fever, and ing, shaving, talking, chewing or swallowing.
alteration in sensorium, confusion and behaviour changes.
l Lassitude, i.e. weariness or exhaustion Investigations
l Depression
l Signs of meningeal irritation are present. l CT or MRI scan of the brain.
l Other signs are cranial nerve (oculomotor) palsies, pap-
illoedema and focal neurological deficit. Treatment
l Hydrocephalus is a common complication.
l Carbamazepine 200 mg three or four times a day
l Gabapentin
Investigatlons l Phenytoin
l Amitryptiline
Diagnosis is made by CSF examination:
l Alcohol injection
l The AFB stain may be positive.
l Peripheral neurectomy.
l CT or MRI brain may show meningeal enhancement or
l Radiofrequency ablation of gasserian ganglion.
hydrocephalous.
l Microvascular decompression
Q. 8. Aetiology and clinical manifestations of depression.
Management of Tubercular Meningitis
Ans.
a. General management:
i. Maintenance of nutrition, hydration and electrolyte Depressive disorders are characterised by persistent low
balance. mood, loss of interest and enjoyment, and reduced energy.
b. Pharmacological management: Antitubercular treat- they often impair day to day functioning.
ment should be started as soon as possible.
i. Anti-tubercular drugs: Inj. streptomycin 1 gm IM Clinical Features
daily 1 Tab isonex 600 -900 mg/day 1 Tab etham-
butol Psychological
ii. Addition of steroids for initial few weeks increases l Depressed mood is the most characteristic feature, there
survival and reduces complications. Prednisolone may be diurnal variation of mood, the depressed mood
40-60 mg/day to reduce toxicity, pia-arachnoid ad- being worst in the early morning or in the evening.
hesions and feeling of well-being. l Loss of pleasure in life and loss of interest in oneself
iii. Mortality is high, if treatment is delayed. and others.
l Low self-esteem and ideas of hopelessness.
Q. 7. Trigeminal neuralgia–clinical features and treat-
l Self-blame.
ment.
l Psychotic features (e.g. hallucinations, delusions, etc.)
Ans. may be present, such a condition is known as psychotic
depression.
Trigeminal neuralgia is the most common disorder of the
trigeminal nerve seen in elderly individuals. Somatic
l Sleep disturbances may occur as initial insomnia, early
Aetiology morning wakening or hypersomnia.
l Fatigue, headache and various other pains.
l Usually idiopathic
l Anorexia, weight loss or gain and constipation.
l Secondary (symptomatic) - Pontine neoplasm, basilar
l Poor concentration, psychomotor retardation and re-
artery aneurysm, cholesteatoma, granulomas, inflam-
duced libido.
mation around gasserian ganglion, abnormal vascular
loops on trigeminal nerve. Q. 9. Petit mal epilepsy.
Ans.
Clinical Features
l Petit mal or absence seizure is typically seen in child-
l Paroxysmal, brief attacks of severe lancinating pain in hood and ceases after 20 years of age.
the distribution of one or more divisions of the trigemi- l This is characterized by brief lapses of sensorium with
nal nerve. loss of postural control.
Section | I General Medicine 131

l These are too subtle to be noticed or referred to as day Q.11. Mention the causes of epilepsy.
dreaming.
l The attacks are much briefer (seconds) and more fre- Ans.
quent than complex partial seizures. There is no post- Causes of seizures are as follows:
ictal confusion. i. Idiopathic
l There may be mild motor movements like blinking of ii. Genetic
eyes, chewing and hand clonus. This is associated with l Neurofibroma
characteristic EEG pattern. l Inborn errors of metabolism
l The aetiology depends on the age of the patient. iii. Trauma
Hypoxia, metabolic derangements, congenital defects l Birth trauma
and birth trauma are important causes of seizures in l Head injury
neonates. Febrile seizures are more common in early iv. Metabolic
childhood. Head trauma is common cause of seizures in l Alcohol withdrawal
young adults. l Hypoglycaemia
l In older people, cerebrovascular disease, tumours and l Hypocalcaemia
degenerative disorders are important cause of seizures. l Hyponatraemia
l Tretment: l Hypoxia
i. Ethosuccimide is the drug of choice. l Renal failure
ii. Sodium valproate. l Liver failure
iii. Immediate treatment of an attack of fit. The patient v. Intracranial space occupying lesions:
should be protected from the injury and clear air- l Tuberculoma
way should be maintained. l Neurocystic cercosis

Q. 10. Mention the causes of peripheral neuropathy. l Brain abscess

l Brain tumour
Ans. vi. Cerebrovascular diseases:
l The peripheral neuropathy is the disorder of peripheral l Haemorrhage

nerves either sensory, motor or mixed, symmetrical and l Emboli

affecting distal parts of limbs more. vii. Infections:


l On the basis of types, causes are: l Encephalitis

l Meningitis
A. Mononeuropathy: l HIV

i. Compression, i.e. compression of radial nerve against l Toxoplasmosis

humerus viii. Inflammatory:


ii. Entrapment: l SLE

l It is caused due to wrist fracture. l Sarcoidois

l Due to soft tissue thickening in myxoedema and ix. Degenerative:


acromegaly. l Alzheimer’s disease

iii. Paraproteinaemias and dysproteinaemias x. Drugs:


iv. Other causes are trauma, fractures, operations, lacera- l Penicillins

tions and injections. l Theophylline

l Chloroquine
B. Multiple mononeuropathy: l Mefloquine
i. Vascular, i.e. diabetes, rheumatoid arthritis, etc. l Psychotropic agents
ii. Inflammatory, i.e. leprosy
iii. Infiltration, i.e. malignancy Q. 12. Treatment of hypertensive encephalopathy.
iv. Physical injury Ans.
v. Familial disposition
l Hypertensive encephalopathy is charecterised by a very
C. Polyneuropathy: high blood pressure and neurological disturbances in-
i. Toxins, i.e. alcohol, heavy metals and drugs cluding transient abnormalities in speech or vision,
ii. Avitaminosis paraesthesiae, disorientation, fits, loss of consciousness
iii. Metabolic and endocrine disorders and papilloedema.
iv. Collagen and allied disorders l Neurological deficit is fully reversible, if the hyperten-
v. Malignancy tion is properly controlled.
132 Quick Review Series for BDS 3rd Year

Treatment D. Systemic causes


l A controlled reduction of blood pressure, over a period of Fever, hypoxia, hypercapnia, hypertension, anaemia,
30-60 min to a level of 150/90 mmHg is adequate. Too allergy, drugs, neuralgias, etc.
rapid fall in blood pressure might cause cerebral ischaemia,
E. Depression
blindness, myocardial infarction or renal insufficiency.
l Intravenous sodium nitroprusside (0.3-1.0 micro/kg/ Q. 14. Status epilepticus.
minute) is the most effective drug.
l Alternatively parenteral labetalol (2 mg/minute to a Ans.
maximum of 200 mg), hydralazine (5-10 mg every i. Status epilepticus refers to continuous seizure activity
30 minutes to a maximum of 300 mg) or nitroglycerin or intermittent seizures with impaired consciousness in
(5-100 micrograms/minute as infusion) may be used. interictal period.
l Bed rest, sedation and diuretics. ii. This can be convulsive (tonic clonic) or non-
l Do not use sublingual nifedipine as it can produce pre- convulsive type. Practically, seizure activity lasting
cipitous fall in blood pressure leading to acute myocar- for more than 5 minutes should be managed as status
dial ischaemia or stroke. epilepticus.
Q. 13. Discuss the differential diagnosis of headache. iii. It is a medical emergency because if not rapidly con-
trolled it may be fatal.
Ans. iv. Causes:
a. Abrupt withdrawal of anti-epileptic drugs or non-
Headache can be a symptom with numerous possible
compliance is the most common cause.
causes, ranging from a mild head injury to a serious brain
b. Other causes are: Metabolic disorders, intracranial
tumour, or it could be a disorder itself like migraine.
infections and structural lesions of the brain.
v. Management:
Classification of Headache l status epilepticus should be treated as emergency

and patient should be hospitalized.


A. Physiological l Airway is maintained and oxygen is administered.
l Hunger Intravenous line is started, intravenous dextrose
l Sleep deprivation (50% dextrose 25-50 ml) is promptly given.
l Hangover l Intravenous diazepam (10 mg) or lorazepam

(4 mg) is given slowly in two minutes. This can be


B. Primary headaches repeated after 15 minutes, if seizures are not con-
l Migraine trolled.
l Tension headache l If seizures continue beyond 30 minutes, intravenous
l Cluster headache phenytoin (20 mg/kg) at a rate not more n 50 mg/
l Benign paroxysmal headaches min is given. Alternatively fosphenytoin can be
given.
C. Secondary headaches l Phenobarbitone (IV 20 mg/kg) at a rate not

a. Intracranial pathology more than 50 mg/min is given, if seizures are


l Vascular still uncontrolled. An additional dose of 5-10 mg
l Infective /kg mass
l Neoplastic l Uncontrolled seizures are finally managed by

l Traumatic general anaesthesia and neuromuscular blockade


b. Extracranial pathology along with ventilatory support. Anaesthetic agents
l Eyes used are midazolam, propofol, and pentobarbi-
l Ears tone.
l Sinuses l Once the status is controlled, long-term antiepileptic

l Teeth medications are started.


l Neck l The underlying cause is identified, if any, and

l Temporomandibular joint treated accordingly.


Section | I General Medicine 133

SHORT NOTES
Q. 1. Lock jaw. Clinical Features
Ans. l Tingling or numbness in the limbs.
i. Rigidity and generalized spasms are the cardinal mani- l Sudden darkness before eyes and
festations of generalized tetanus. l Patient has feeling of blacking out.
ii. First of all patient experiences a feeling of unwell and l The patient is cold and sweating and fall suddenly to
lockjaw or trismus due to hypertonicity of masseter ground and become unconsciousness.
muscles. l Patient’s respiration is sighing.
l Pulse of the patient is slow and limbs are cold and
Q. 2. Migraine. clammy.
Or l Pupils may retract to light.
Clinical features of migraine.
Or Management
Migraine-name two drugs for treatment. l Patient must be laid on bed in airy room.
l The foot end of bed is elevated.
Ans. l The sufficient blood pressure is restored for cerebral
l Migraine is defined as episodes of unilateral throbbing perfusion by sympathomimetics, indomethacin or by
headache, nausea and vomiting and symptoms of neuro- cardiac pacing.
logical dysfunction. l For hypoglycaemia, IV glucose is given.
Q. 4. Tonic-clonic seizure.
Clinical Features Ans.
i.
Headache (unilateral).
i. Grand mal or tonic clonic seizure is a common type of
ii.
Nausea and vomiting.
epilepsy seen in clinical practice.
iii.
Light and noise sensitivity.
ii. Phases recognized during grand mal epilepsy are prodro-
iv.
Fatigue and stress.
mal phase, aura, tonic and clonic phase and postictal phase.
v.
The visual aura is most common and is in the form of
iii. Diagnosis is made by careful assessment of patient’s
scotomas, hallucinations and fortification spectra.
history documented by the diagnostic studies.
vi. Drugs used in treatment of migraine are:
iv. Management:
l Analgesics (aspirin, paracetamol and other NSAIDs)
l The patient should be protected from the injury and
are effective, if taken early at the onset of headache.
clear airway should be maintained.
vii. Severe attacks may be treated with triptans (sumatrip-
l Diazepam 5 to 10 mg slow IV injection is given.
tan, zolmitriptan) and ergotamine.
l Long-term drug therapy: Phenytoin sodium or car-
viii. Analgesics (meperidine) can be used in severe acute
bamazepine or sodium valproate or phenobarbitone
attacks.
or primidone may be used.
Q. 3. Sudden loss of consciousness.
Q. 5. Phenytion sodium.
Ans.
Ans.
l Syncope is a transient loss or impairment of conscious-
ness with inability to maintain postural tone due to l Dilantin sodium is a GABA agonist which blocks the
acute decrease in cerebral blood flow. metabolism of GABA and raises its level.
l The drug is well absorbed through oral route. It can also
be administered through parenteral and rectal routes.
Causes l It is a broad-spectrum anticonvulsant and a drug of
Standing for long, starvation, excessive heat and exhaus- choice in primary generalized seizures.
tion, diseases of heart, psychological causes like fear and l It is a drug of second choice in secondary generalized
sudden anxiety, blood loss and hypoglycaemia. seizures.
134 Quick Review Series for BDS 3rd Year

l Side effects: Nausea, anorexia, tremors, alopecia, Q. 8. Bell’s palsy.


thrombocytopenia and hepatic necrosis and weight gain
Or
on long-term use.
l It is known to have teratogenic effects. Bell’s palsy - management.
Q. 6. Syncope. Ans.
or i. Bell’s palsy is an acute apparently isolated, lower motor
neuron facial palsy.
Causes of syncope.
ii. Aetiology: Exposure to cold, trauma during extraction
Ans. of teeth or injection of local anaesthetic, following sur-
gical procedures like parotidectomy in which the facial
i. Syncope refers to generalized weakness of muscles,
nerve is sectioned, tumours of the cranial base, familial,
loss of postural tone, inability to maintain erect posture
facial canal and middle ear neoplasms and viral infec-
and loss of consciousness. Or syncope is a transient loss
tions.
or impairment of consciousness with inability to main-
iii. Clinical features:
tain postural tone due to acute decrease in cerebral
l Characterized by development of unilateral lower
blood flow.
motor neuron type of facial paralysis over a few
ii. Causes: Standing for long, starvation, excessive heat
hours.
and exhaustion, psychological causes like fear and sud-
l May be preceded by ear pain.
den anxiety, blood loss and hypoglycaemia.
l Lesion is at the stylomastoid foramen.
iii. Clinical features:
l Most patients recover fully within a few weeks.
l Tingling or numbness in the limbs.
iv. Treatment consists of protection of eye, facial exercises,
l The patient is cold and sweating and fall suddenly to
massaging the facial muscles, local heat application, fa-
ground and become unconsciousness.
radic stimulation, splints to prevent drooping of face and
l Pulse of the patient is slow and limbs are cold and
medications (steroids and heavy doses of vitamin B12)
clammy.
v. Surgery: Plastic surgery is preferred.
iv. Management:
l A syncope patient must be laid in well-ventilated Q. 9. Trigeminal neuralgia.
room.
l The foot end of bed is elevated.
Or
l For hypoglycaemia, IV glucose is given. Clinical features of trigeminal neuralgia.
Q. 7. Status epilepticus. Ans.
Ans. i. Trigeminal neuralgia is the most common disorder of
i. Status epilepticus refers to continuous seizure activity the trigeminal nerve seen in elderly individuals.
or intermittent seizures with impaired consciousness in ii. Aetiology:
interictal period. l Usually idiopathic

ii. Abrupt withdrawal of anti-epileptic drugs or non- l Secondary: Pontine neoplasm, basilar artery aneu-

compliance is the most common cause. rysm, cholesteatoma, granulomas, inflammation


iii. It is a medical emergency because if not rapidly con- around gasserian ganglion, abnormal vascular loops
trolled it may be fatal. on trigeminal nerve.
iv. Patient should be hospitalized, airway is maintained and iii. Clinical features:
oxygen is administered. Intravenous diazepam (10 mg) l Paroxysmal, brief attacks of severe lancinating pain

or lorazepam (4 mg) is given slowly in two minutes. in the distribution of one or more divisions of the
v. If seizures continue beyond 30 minutes, intravenous trigeminal nerve.
phenytoin (20 mg / kg) is given. Phenobarbitone (IV l Attacks may be precipitated by cold water, face

20 mg/kg) at a rate not more than 50 mg/min is given, washing, shaving, talking, chewing or swallowing.
if seizures are still uncontrolled. iv. Treatment:
vi. Uncontrolled seizures are finally managed by general l Carbamazepine 200 mg three or four times a day.

anaesthesia and neuromuscular blockade along with l Gabapentin, phenytoin or amitryptiline can also

ventilatory support. be used.


vii. Once the status is controlled, long-term antiepileptic l Alcohol injection or radiofrequency ablation of gas-

medications are started and the underlying cause is serian ganglion.


identified, if any, and treated accordingly. l Microvascular decompression.
Section | I General Medicine 135

Q. 10. Hysteria. vi. Management: Neuroleptic drugs (anti-psychotic drugs)


like chlorpromazine 100 mg tid, gradually building up
Ans.
the dose to a maximum of 1500 mg daily or until symp-
l Hysteria is a syndrome characterized by a loss or distor- toms subside.
tion of neurological function not fully explained by or-
Q. 12. Babinski’s sign.
ganic disesase.
l The patient develops symptoms and signs of illness Ans.
(mental, physical or both).
l Extensor plantar reflex-extension, i.e.dorsiflexion of the
l Hysteria is a protean in manifestation and may stimulate
great toe with fanning of other toes (abduction) is called
any disease.
Babinski’s sign.
l Clinical features: Two main variants of hysteria are con-
l Causes: Infants and children up to 2 years of age during
version disorder and dissociation disorder.
deep sleep (physiological) and lesions of corticospinal
a. Conversion disorder: Symptoms mimic lesions of the
tract above S1 segment (pathological).
motor or sensory nervous system. Common presenta-
tions are gait disturbances, loss of function in limbs, Q. 13. Four signs of Horner’s syndrome.
aphonia, pseudoseizures, sensory loss and blindness.
Ans.
b. Dissociation disorder: Hysterical amnesia usually
develops acutely. The memory loss is patchy and i. Horner’s syndrome is produced by damage to the sym-
inconsistent. A characteristic feature is a loss of pathetic pathway.
personal identity (name, address) other personal and ii. Signs:
family details. l Ptosis of eyelid

l Enophthalmos.
Amnesia occurs to escape from intolerable anxiety and
l Loss of ciliospinal reflex.
distress about some problem or situations.
l Blood shot conjunctiva due to loss of vasoconstric-

Q. 11. Schizophrenia. tor activity.


l Anhidrosis of ipsilateral half of face.
Ans.
l Miosis due to reduced pupillodilator activity.

i. Schizophrenia is a group of disorders characterised by


Q. 14. Classify headache.
perturbations in language, perception, cognition and
behaviour. Ans.
ii. Aetiology:
i. Headache can be a symptom with numerous possible
l Genetic.
causes, ranging from a mild head injury to a serious brain
l A highly emotional family environment contributes
tumour, or it could be a disorder itself like migraine.
to relapses.
ii. Classification of headache:
l Psychological stresses and viral vector.
A. Physiological
iii. Types: There are 4 major types of schizophrenia
B. Primary headaches
disorders-catatonic, disorganized, paranoid and un-
C. Secondary headaches:
differentiated.
a. Intracranial pathology
iv. Schizophrenic patients may also be classified as Type 1
b. Extracranial pathology
and Type II.
D. Systemic causes:
Type I patients have a predominance of positive symp-
E. Depression
toms, normal ventricular size and a good response to
anti-psychotic drugs. Q. 15. Tension headache (psychological/psychogenic
Type II patients have predominance of negative symp- headache).
toms, increased ventricular size and poor response to
Ans.
anti-psychotic drugs.
v. Symptoms: l Tension headache is usually bilateral and extends to top
l Thought insertion of the head.
l Auditory hallucinations l The onset is gradual but pain may continue for weeks or
l Delusional perceptions months without interruption.
l Catatonia l Stress and anxiety precipitate tension headache.
l Thought disorder l Provoking factors include fatigue and nervous strains.
l Social withdrawl l It is characterized by dull ache rather than headache or
l Poverty of speech there may be sensation of fullness of head, pressure
136 Quick Review Series for BDS 3rd Year

over head or there may be constricting band around the Q. 18. Anxiety disorder - symptoms, any three.
head.
Ans.
l Associated features-anxiety, depression, nervousness,
insomnia, etc. l Anxiety is persistant excessive and/or unrealistic worry
l Treatment: Anxiolytic drugs-alprazolam 0.25-0.5 mg associated with other features including:
twice a day and antidepressants-amitryptaline. i. Muscle tension
ii. Impaired concentration
Q. 16. Facial paralysis.
iii. Autosomal arousal
Ans. iv. Restlessness
v. Insomnia
l Facial palsy refers to the paralysis of facial muscles. l Patient complains of tachycardia, dyspnoea while palpi-
l It is of two types: tations are rare.
i. Upper motor neuron palsy
ii. Lower motor neuron palsy or Bell’s palsy. Q. 19. Petit mal epilepsy (Classical absence).
Ans.
i. Upper Motor Neuron Palsy
a. It affects mainly muscles of lower part of face and is l Petitmal epilepsy is typically seen in children.
never complete. l During an absence attack, a child stops working, looks
b. It is seldomly isolated palsy. confused, stares in space, may blink or roll-up the eye
c. The emotional movements are preserved. balls and fail to respond to verbal comments.
d. There is no muscle contracture. l The attack is very brief, lasts only for seconds but many
e. There is no reaction of degeneration. such attacks may occur during the day.
f. Electromyography and nerve conduction is normal. l Tretment: Ethosuximide is the drug of choice. While
sodium valproate may also be used.
ii. Lower Motor Neuron Palsy or Bell’s palsy Q. 20. Aetiology of meningitis.
l Bell’s palsy is an acute apparently isolated, lower motor
neuron facial palsy. Ans.
i. Inflammation of the meninges and the subarachnoid
Clinical Features space is called as meningitis.
l Characterized by development of unilateral lower motor
neuron type of facial paralysis over a few hours. Aetiology
l May be preceded by ear pain.
l Lesion is at the stylomastoid foramen. A. Infection
l Most patients recover fully within a few weeks. i. Bacterial: Listeria monocytogenes, E.coli, H. influenzae,
l Treatment consists of protection of eye, facial exer- Neisseria, pneumococci, Mycobacterium tuberculosis.
cises, massaging the facial muscles, local heat applica- ii. Virus: Enterococci, mumps, influenza virus, herpes
tion, faradic stimulation, splints to prevent drooping simplex virus, Epstein-Barr virus, HIV.
of face and medications (steroids and heavy doses of iii. Protozoan parasite: Toxoplasmosis, amoeba.
vitamin B12). iv. Fungi: Candida, Histoplasma, Blastomyces.
Q. 17. Facial pain or facial pain - four causes.
B. Non-infectious
Ans.
i. Malignant diseases: Breast cancer, lung cancer, leukae-
l The pain in the facial area may be due to various mia, lymphoma.
causes. ii. Inflammatory: Sarcoidosis, systemic lupus eryth­
l Most cases are due to dental problems, trigeminal neu- ematosus
ralgia, post-herpetic neuralgia and atypical facial pain.
l The causes of facial pain are: Q. 21. Signs of meningeal irritation.
i. Neuritis of cutaneous nerves of face and scalp. Ans.
ii. Arthralgia of temporomandibular joint.
iii. Trigeminal neuralgia. Signs of meningeal irritation are as follows:
iv. Post-hepatic neuralgia. a. Neck rigidity
v. Temporal arthritis. b. Kernig’s sign
Section | I General Medicine 137

c. Brudzinski’s sign v. Treatment of acute attack with oral drugs is not satis-
a. Neck rigidity: It is is the pathognomonic sign of factory. Inhalation of 100% oxygen (7 L / min for 15
meningeal irritation. This is demonstrated as an minutes) is most effective modality of treatment for
increased resistance to passive flexion of the neck. acute attack.
b. Kernig’s sign is elicited when the patient is lying vi. Subcutaneous sumatriptan (6 mg) may be effective.
supine. The thigh is flexed on the abdomen with the vii. Preventive therapy is generally effective.
knee flexed. Passive extension of the knee causes viii. The drugs used are propranolol, amitriptyline, valpro-
pain and spasm of the hamstrings. This is due to ate, verapamil, cyproheptadine.
irritation of meninges in the lower spine. Q. 25. Status epilepticus—treatment.
c. Brudzinski’s sign is elicited when the patient is in a
supine position. Passive flexion of the neck results in Ans.
a spontaneous flexion of hips and knees. i. Status epilepticus refers to continuous seizure activity
Q. 22. Four drugs used in epilepsy. or intermittent seizures with impaired consciousness in
interictal period.
Ans. ii. Treatment:
l status epilepticus should be treated as emergency
Various drugs used in epilepsy are as follows:
l Beta blockers and patient should be hospitalized.
l Airway is maintained and oxygen is administered.
l Calcium channel blockers
l Antidepressants Intravenous line is started, intravenous dextrose
l Serotonin antagonists (50% dextrose 25-50 ml) is promptly given.
l Intravenous diazepam (10 mg) or lorazepam (4 mg)
l Valproate
is given slowly in two minutes. This can be repeated
Q. 23. TB meningitis. after 15 minutes, if seizures are not controlled.
l If seizures continue beyond 30 minutes, intravenous
Ans.
phenytoin (20 mg/kg) at a rate not more 50 mg/ min
i. The commonest organism for causing tubercular men- is given. Alternatively fosphenytoin can be given.
ingitis is M. tuberculosis. l Phenobarbitone (IV 20 mg/kg) at a rate not more
ii. It is common in childhood. than 50 mg/min is given, if seizures are still uncon-
iii. Hematogenous spread in cases of primary or post- trolled. An additional dose of 5-10 mg/kg mass
primary pulmonary disease. l Uncontrolled seizures are finally managed by gen-
iv. Onset of symptoms is insidious; in some it may be eral anaesthesia and neuromuscular blockade along
acute. with ventilatory support. Anaesthetic agents used
v. Patients have headache, vomiting, low grade fever, and are midazolam, propofol, and pentobarbitone.
alteration in sensorium. Signs of meningeal irritation l Once the status is controlled, long-term antiepileptic
are present. medications are started.
vi. Antitubercular treatment should be started as soon as l The underlying cause is identified, if any, and
possible. treated accordingly.
vii. Addition of steroids for initial few weeks increases
Q. 26. Anti-epileptic drugs.
survival and reduces complications.
Mortality is high, if treatment is delayed. Ans.
Q. 24. Migrainous neuralgia (cluster headache). Various anti-epileptics or anticonvulsants that can be used
are as follows:
Ans.
i. Barbiturates - Phenobarbitone
i. Migrainous neuralgia or cluster headache is common ii. Deoxybarbiturate - Primidone
in middle-aged males and there is no family history. iii. Hydantoin - Phenytoin
ii. The pain occurs periodically at a specific time of the iv. Iminostilbene – Carbamazepine
day, generally in the early morning. The pain is severe v. Succinimide - Ethosuximide
and lasts for 30-90 minutes. vi. Aliphatic carboxylic acid - Sodium valproate
iii. There is unilateral periorbital pain associated with vii. Benzodiazepines - Clonazepam, diazepam, clobazam
nasal congestion, lacrimation, rhinorrhoea, or redness viii. Phenyltriazine - Lamotrigine
of the eye. ix. Cyclic GABA analogue - Gabapentin
iv. Patients may remain asymptomatic for weeks or months x. Newer drugs -Vigabatrin, topiramate, tiagabine, leve-
before another bout of headache (cluster) occurs. tiracetam
138 Quick Review Series for BDS 3rd Year

Topic 8

Diseases of the Kidneys and Genitourinary System


LONG ESSAYS
Q. 1. Define nephrotic syndrome. Describe the aetiology, viii. Urine output is generally normal.
clinical features, investigations and treatment of ne- ix. Hypertension and hematuria are rare.
phrotic syndrome.
Ans. Investigations
Nephrotic syndrome is a group of disorders presenting with
heavy proteinuria, hypoalbuminaemia, massive oedema, i. Urine analysis: Massive proteinuria, albumin 31/41,
and hyperlipidaemia. micro-hyaline cast, 24 hr urine protein estimation
Massive proteinuria is the most important feature of .3.5 grams/24 hr for average 1.7 body surface area.
nephrotic syndrome. ii. Blood examination: It reveals low serum albumin
(, 3 g/dl), low total serum protein and hyperlipidae-
mia. Blood urea level and serum creatinine levels are
Aetiology
normal. Other tests are required in certain settings to
l Primary renal disorders (idiopathic): know the cause of nephrotic syndrome such as blood
a. Minimal lesion nephritic syndrome sugar, antinuclear antibody (ANA), rheumatoid factor,
b. Membranous glomerulonephritis and HIV.
c. Focal and segmental glomerulosclerosis iii. Complement C3 and C4 are lower in disease with a bad
d. Mesangioproliferative glomerulonephritis prognosis.
e. Mesangiocapillary glomerulonephritis iv. Serum electrolyte estimation
f. Membranoproliferative glomerulonephritis v. Serum protein level: lower because of hypoalbuminae-
l Secondary nephrotic syndrome: mia (normal value is 6.6 gm/dl)
a. Infections: malaria vi. Lipid level estimation: LDL levels elevated.
b. Following hepatitis B infection
c. Complication of infective endocarditis
d. Syphilis Treatment
e. Following collagen disease (polyarteritis nodosa and
i. General measures
SLE)
ii. Bed rest with fluid restriction (intake 5 output ml of
f. Metabolic diseases like diabetes mellitus type II
voided urine)
g. Tumours like bronchgenic carcinoma, stomach carci-
iii. Reduced salt intake ,2g NaCl/24hr
noma, chronic lymphatic leukaemia or lung carcinoma.
iv. Fruit juices and K1 restriction
v. Treatment of oedema
Clinical Features l Diuretics

i. Oedema is the main presenting feature of nephrotic l First class proteins (soya protein)

syndrome. l Lipid: antistatynins

ii. Initially, it is present in the dependant parts like lower l Coagulopathy: Anti-platelet drugs like aspirin. Hep-

extremeties, but in later stages it may become general- arin is used when complication has developed.
ized (anasarca). l Others: Vitamin D (1,25-dihydrocholecalciferol)

iii. Bilateral pleural effusion, supplementation is required in patients with evi-


iv. Ascites dence of thrombosis.
v. Pulmonary oedema vi. Management of infection: Antibiotics that are not
vi. Fever may occur due to infection. nephrotoxic (cephalhexins).
vii. Other uncommon features are arterial and venous vii. Specific treatment is done keeping in mind the indi-
thrombosis, pulmonary embolism and renal vein vidual disease. Specific regime: Prednisolone 1 Cy-
thrombosis. clophosphamide 1 Azathioprin.
Section | I General Medicine 139

Q. 2 Define acute nephritic syndrome. Discuss the clini- Investigations


cal features and treatment of post-streptococcal acute a. Urine analysis:
glomerulonephritis. i. Non-selective proteinuria
Ans. ii. Globulin cast and albumin
iii. Microscopically: RBC casts, granular casts
Nephritic syndrome is a clinical syndrome which arises due b. Blood urea and serum creatinine levels are elevated.
to acute glomerular inflammation (glomerulonephritis) and c. Total proteinurea in 24 h is ,3 g then nephritic syn-
is characterized by sudden onset of oliguria, oedema, hy- drome is massive.
pertension, haematuria, subnephrotic proteinuria (, 3 g/ d. Potassium may be raised in acute phase when there is
day) and worsening renal function. renal failure.

Aetiology Treatment
l Most of the cases of acute glomerulonephritis (GN) a. Bed rest: Till the acute symptoms improve.
are post-streptococcal (group A, b haemolytic strepto- b. Diet:
coccal). l Restriction on dietary protein and sodium and potas-
l It can be seen in the following situations: sium intake.
a. Henoch-Schonlein purpura l Also, restriction of fluid intake.
b. Haemolytic urinary purpura l The total amount (ml) of fluid consumption should be
c. Severe, IV haemolysis calculated as follows: ml of previous days urine 1
d. Infection, i.e. hepatitis B, SABE, etc. 500 ml fluid 5 fluid administered per day.
e. IgA nephropathy l No intake of juices.
f. Familial nephropathy c. Hypertension: Moderate to severe hypertension is con-
l Non-streptococcal: Staphylococcal induced or salmo- trolled by the hydralazine, beta-blockers like atenolol
nella or pneumococcal induced. and calcium channel blockers.
l Primary glomerular disease: Depending on where the d. Antibiotics: Penicillin can be administered. Dose: Pro-
immune complexes are deposited caine penincillin, IM, for 6 days to eradicate strepto-
Clinical manifestations are as follows: cocci.
e. Diuretics: It is indicated in cases of acute LVF or pulmo-
Paediatric (symptoms) nary oedema. Furosemide is the drug of choice.
f. In patients with progressive renal failure, dialysis may
a. Abrupt onset
have to be employed.
b. Blood in urine n Frank blood in urine n oliguria
c. Puffiness of face Q. 3 What are the causes and management of glomeru-
d. Hypertension lonephritis.
e. Left ventricular failure n Pulmonary failure
Ans.
f. Because of reduced urine production n Pulmonary
oedema The term glomerulonephritis indicates glomerular inflam-
g. Vomiting because of raised blood urea levels, nausea mation, in which there is immunologically mediated injury
h. Preceding history of infection to glomeruli. It is also known as nephritic syndrome.

Adult
a. Onset is slow
Aetiology
b. Preceding history is absent a. Immunological
c. Oliguria (smoky urine) b. Infectious diseases:
d. Hypertension l Post-streptococcal glomerulonephritis,
e. Pulmonary oedema is absent or not seen l Infective endocarditis
f. Nausea and vomiting l Syphilis

l Mumps
(Signs) l Hepatitis B
a. Raised blood pressure l Epstein-Barr virus infections
b. Pitting pedal oedema c. Multisystem diseases: Inherited conditions like systemic
c. Respiratory system: Bilateral, fine crepitations lupus erythematosus, Goodpasture’s syndrome, Henoch-
d. Puffiness of face Schonlein purpura, Alport’s syndrome (Alport syndrome
140 Quick Review Series for BDS 3rd Year

is characterized by glomerulonephritis, end-stage kidney d. Deposition of substance (e.g. amyloid)


disease and hearing loss). e. Direct injury to glomerulus
d. Primary glomerular diseases: Direct injury to glomerulus,
diffuse proliferative glomerulonephritis, IgA nephropa- Clinical Features
thy, mesangiocapillary glomerulonephritis, membranous
glomerulonephritis. l Most common in the male patients, aged 2 to 14 years,
e. Miscellaneous: Malignancy, eclampsia, penicillamine. who suddenly develops puffiness of the eyelids and
l Management of glomerulonephritis includes investi-
facial oedema because of post-streptococcal infection.
gations, diagnosis and treatment. l Urine: Dark and scanty
l Blood pressure: May be elevated
l Onset of symptoms: Abrupt
i. Investigations l Nonspecific symptoms: Weakness, fever, abdominal
a. Urine analysis: Non-selective proteinuria pain and malaise.
b. Globulin cast and albumin l Post-infectious acute nephritis: A latent period of up
c. Microscopically: RBC casts, granular casts to 3 weeks before onset of symptoms (may vary from
d. Blood urea: Serum creatinine levels elevated. 1-2 weeks for postpharyngitis cases and 2-4 weeks for
e. Total proteinurea in 24 h is , 3 g then nephritic syn- cases of post-dermal infection).
drome is massive. l If nephritis is seen within 1 to 4 days of streptococcal
f. Potassium may be raised in acute phase when there is infection: Pre-existing renal disease.
renal failure. Symptoms of acute glomerulonephritis are as follows:
a. Haematuria (gross haematuria is reported in 30% of
ii. Treatment paediatric patients)
b. Oliguria
a. Bed rest: Till the acute symptoms improve. c. Oedema (peripheral or periorbital, 85% of paediatric
b. Diet: Restriction on dietary protein and sodium and potas- patients)
sium intake. Also, restriction of fluid intake. The total amount d. Oedema may be mild (involving only the face) to severe
(ml) of fluid consumption should be calculated as follows: e. Headache (secondary to hypertension)
ml of previous days urine 1 500 ml fluid 5 fluid admin- f. Shortness of breath or dyspnaea on exertion (due to
istered per day heart failure or pulmonary oedema).
c. No intake of juices. g. Nausea/vomiting, abdominal pain and purpura observed
d. Hypertension: Moderate to severe hypertension is con- with Henoch-Schonlein purpura
trolled by the hydralazine, beta-blockers like atenolol h. Arthralgias associated with SLE
and calcium channel blockers. i. Haemoptysis
e. Antibiotics: Penicillin can be administered.
Dose: Procaine penincillin, IM, for 6 days to eradicate
Complications
streptococci.
f. Diuretics: It is indicated in cases of acute LVF or pulmo- a. CCF (congestive cardiac failure)
nary oedema. Furosemide is the drug of choice. b. Pulmonary oedema
g. In patients with progressive renal failure, dialysis may c. Chronic glomerulonephritis
have to be employed. d. Nephrotic syndrome
Q. 4. Describe the aetiology, clinical features, complica-
tions, investigations and management of acute nephritis. Investigations
Ans. a. Urine analysis: Non-selective proteinuria
i. Globulin cast and albumin
Acute nephritis refers to a specific set of renal diseases where ii. Microscopically: RBC casts, granular casts
an immunologic mechanism causes inflammation and prolif- b. Blood urea: Serum creatinine levels elevated.
eration of glomerular tissue that can cause damage to the c. Total proteinurea in 24 h is , 3 g then nephritic syn-
basement membrane, mesangium or capillary endothelium. drome is massive.
d. Potassium may be raised in acute phase when there is
Aetiology renal failure.

a. Immunological
Management
b. Inherited like Alport’s syndrome (Alport syndrome is
characterized by glomerulonephritis, end-stage kidney a. Bed rest: Till the acute symptoms improve.
disease and hearing loss). b. Diet: Restriction on dietary protein and sodium and po-
c. Metabolic (DM) tassium intake. Also, restriction of fluid intake. The total
Section | I General Medicine 141

amount (ml) of fluid consumption should be calculated l In diseased condition, it becomes positive and pore size
as follows: increases as a result patient develops albuminurea.
ml of previous days urine 1 500 ml fluid 5 fluid admin-
istered per day The sequence of events leading to oedema in nephrotic-
No intake of juices. syndrome is as follows:
c. Hypertension: Moderate to severe hypertension is con-
trolled by the hydralazine, beta-blockers like atenolol Proteinurea
and calcium channel blockers.
d. Antibiotics: Penicillin can be administered. Dose: Pro- Glomerular disease
caine penincillin, IM for 6 days to eradicate streptococci.
e. Diuretics: It is indicated in cases of acute LVF or pulmo-
nary oedema. Furosemide is the drug of choice. Increase in pore size
f. In patients with progressive renal failure, dialysis may
have to be employed.
Hypoproteinaemia
Q. 5. Causes, classification and treatment of acute renal
failure.
Decrease in plasma osmotic pressure
Ans.
The following are the causes and classification:
i. Pre-renal: Massive oedema Decrease in intravascular volume
a. Cardiovascular system: CCF, MI or cardiac tamponade
b. Renal artery stenosis Arginine vasopressin Stimulation of renin-angiotensin-
c. Diarrhoea, vomiting – hypovolaemia aldosterone mechanism
ii. Renal:
a. Vasculitis
b. Glomerulonephritis Increase in sodium and water retention
c. Drugs
iii. Post-renal: Oedema
a. Intrarenal: Stones or canaliculi
b. Extrarenal: Prostitis or pelvic/bladder neoplasm
ii. Congestive cardiac failure:
Treatment In patients without a congenital heart disease, the main
cause of CCF is ventricular dysfunction.
The cause of renal failure must be established and corrected.
a. Hypovolaemia: Volume restored as soon as possible
(blood, plasma or isotonic saline) Ventricular dysfunction
b. Severe metabolic acidosis: 500 ml sodium bicarbonate
solution (1.26%) Fall in cardiac output
c. Generally, restoration of blood volume restores the func-
tion of kidney and acidosis corrected.
d. CVP or pulmonary wedge pressure must be monitored Activation of counter-regulatory neurohormonal mechanisms
(these mechanisms support normal cardiac function)
to establish the rate of fluid infusion.
e. Cardiogenic/septic shock: Invasive haemodynamic mon-
itoring to keep a check on cardiac output. Excessive increase in afterload and preload

Q. 6 Discuss the pathophysiology of oedema in ne-


phrotic syndrome, congestive cardiac failure and cir- A vicious cycle develops
(further fall in CO → neurohormonal activation → Increasing
rhosis of liver. peripheral vascular resistance)
Ans.
Stimulation of renin-angiotensin-aldosterone system
The Pathophysiology of Oedema in:
i. Nephrotic syndrome: Vasoconstriction, salt and water retention

l Glomerulus has negative charge, so albumin does not


Development of oedema
enter because it also has a negative charge.
142 Quick Review Series for BDS 3rd Year

iii. Cirrhosis of liver This accumulation of fluid is called pitting oedema.


l

In cirrhosis of liver, oedema is caused by the following reasons: (Pitting: Pressing a fingertip firmly against an ankle
a. Increased hepatic lymph secretion or leg with oedema causes an indentation in the skin,
b. Portal hypertension which persists for some time after release of the
l When cirrhosis becomes severe, signals are sent to
pressure.)
l As the cirrhosis worsens, water accumulates in
the kidneys and it retains salt and water in the body.
l The excess salt and water accumulates in the tissues
the tissue spaces in and around abdominal cavity
below the skin of the ankles and legs (postural oe- (ascites).
dema, due to gravity). c. Hypoproteinaemia

SHORT ESSAYS
Q. 1. Causes and clinical features of acute nephritis. (Signs)
a. Raised blood pressure
Ans. b. Pitting pedal oedema
The causes and clinical features of acute nephritis are as c. Respiratory system: Bilateral, fine crepitations
follows: d. Puffiness of face
Q. 2. Acute nephritis—investigations and management.
Aetiology: Ans.
a. Most of the cases of acute nephritis are post-streptococcal Investigations and management of acute nephritis are as
(group A, b haemolytic streptococcal). follows:
b. It is seen in the following situations:
i. Henoch-Schonlein purpura Investigations
ii. Haemolytic urinary purpura
iii. Severe, IV haemolysis a. Urine analysis: Non-selective proteinuria
iv. Infection, i.e. hepatitis B, SABE, etc. b. Globulin cast and albumin
v. IgA nephropathy c. Microscopically: RBC casts, granular casts
vi. Familial nephropathy d. Blood urea: Serum creatinine levels elevated.
c. Non-streptococcal: Staphylococci induced or salmonella e. Total proteinurea in 24 h is ,3 g then nephritic syn-
or pneumococcal induced. drome is massive.
d. Primary glomerular disease: Depending on where the f. Potassium may be raised in acute phase when there is
immune complexes are deposited renal failure.

Clinical manifestations are as follows:


Management
Paediatric (Symptoms) a. Bed rest: Till the acute symptoms improve.
a. Abrupt onset b. Diet: Restriction on dietary protein and sodium and
b. Blood in urine n Frank blood in urine n oliguria potassium intake, also, restriction of fluid intake.
c. Puffiness of face The total amount (ml) of fluid consumption should be
d. Hypertension calculated as follows:
e. Left ventricular failure n Pulmonary failure ml of previous days urine 1 500 ml fluid 5 fluid admin-
f. Because of reduced urine production n Pulmonary oedema istered per day
g. Vomiting because of raised blood urea levels, nausea c. No intake of juices.
h. Preceding history of infection d. Hypertension: Moderate to severe hypertension is con-
trolled by the hydralazine, beta-blockers like atenolol
Adult and calcium channel blockers.
a. Onset is slow e. Antibiotics: Penicillin can be administered. Dose: Pro-
b. Preceding history is absent caine penicillin, IM for 6 days to eradicate streptococci.
c. Oliguria (smoky urine) a. Diuretics: It is indicated in cases of acute LVF or
d. Hypertension pulmonary oedema. Furosemide is the drug of choice.
e. Pulmonary oedema is absent or not seen b. In patients with progressive renal failure, dialysis
f. Nausea and vomiting may have to be employed.
Section | I General Medicine 143

Q. 3 Clinical features and treatment of acute post- Q. 4 Nephrotic syndrome—causes and clinical features.
streptococcal glomerulonephritis.
Ans.
Ans.
Causes of nephrotic syndrome are as follows:
Clinical manifestations of post-streptococcal glomerulo- l Primary nephritis (cause not known)
nephritis are as follows: a. Minimal lesion nephritic syndrome
b. Membranous glomerulonephritis
Paediatric (Symptoms) c. Focal segmental glomerulosclerosis
a. Abrupt onset d. Mesangioproliferative glomerulonephritis
b. Blood in urine nFrank blood in urine n oliguria e. Mesangiocapillary glomerulonephritis
c. Puffiness of face l Secondary nephrotic syndrome
d. Hypertension a. Infections: Malaria
e. Left ventricular failure n Pulmonary failure b. Following hepatitis B infection
f. Because of reduced urine production n Pulmonary c. Complication of infective endocarditis
oedema d. Syphilis
g. Vomiting because of raised blood urea levels, nausea e. Following collagen disease (polyarteritis nodosa and
h. Preceding history of infection SLE)
f. Metabolic diseases like diabetes mellitus type II
Adult g. Tumours like bronchgenic carcinoma, stomach
a. Onset is slow carcinoma, chronic lymphatic leukaemia or lung
b. Preceding history is absent carcinoma.
c. Oliguria (smoky urine) Clinical features of nephrotic syndrome are as follows:
d. Hypertension a. Massive oedema
e. Pulmonary oedema is absent or not seen b. Anasarca
f. Nausea and vomiting c. Oliguria in later stage
d. Bilateral pleural effusion
(Signs)
e. Ascites
a. Raised blood pressure
b. Pitting pedal oedema Q. 5 Clinical features and investigations in nephrotic
c. Respiratory system: Bilateral, fine crepitations syndrome.
d. Puffiness of face
Ans.

Clinical features of nephrotic syndrome are as follows:


Treatment a. Massive oedema
i. Bed rest: Till the acute symptoms improve. b. Anasarca
ii. Diet: Restriction on dietary protein and sodium and c. Oliguria in later stage
potassium intake. Also, restriction of fluid intake. The d. Bilateral pleural effusion
total amount (ml) of fluid consumption should be calcu- e. Ascites
lated as follows:
ml of previous days urine 1 500 ml fluid 5 fluid Investigations of nephrotic syndrome are as follows:
administered per day a. Urine analysis: Massive proteinurea, albumin 31/41,
No intake of juices. micro-hyaline cast, 24 hr urine protein estimation
iii. Hypertension: Moderate to severe hypertension is con- . 3.5 grams/24 hr for avg 1.7 body surface area.
trolled by the hydralazine, beta-blockers like atenolol b. Complement C3 and C4 are lower in disease with a bad
and calcium channel blockers. prognosis.
iv. Antibiotics: Penicillin can be administered. c. Serum electrolyte estimation
Dose: Procaine penicillin, IM for 6 days to eradicate d. Serum protein level: Lower because of hypoalbuminae-
streptococci. mia (normal value is 6.6 gm/dl)
v. Diuretics: It is indicated in cases of acute LVF e. Lipid level estimation: LDL levels elevated.
or pulmonary oedema. Furosemide is the drug of Q. 6 Write clinical features and treatment of acute
choice. glomerulonephritis.
vi. In patients with progressive renal failure, dialysis may
have to be employed. Ans.
144 Quick Review Series for BDS 3rd Year

Clinical Features of acute glomerulonephritis: g. Nausea/vomiting, abdominal pain and purpura observed
l Mostly, the patient is a boy, aged 2 to 14 years, who with Henoch-Schonlein purpura.
suddenly develops puffiness of the eyelids and facial h. Arthralgias associated with SLE.
oedema because of post-streptococcal infection. i. Haemoptysis.
l Urine: Dark and scanty
Blood pressure: May be elevated
Treatment
l

l Onset of symptoms: Abrupt


l Nonspecific symptoms: Weakness, fever, abdominal a. Bed rest: Till the acute symptoms improve.
pain and malaise. b. Diet: Restriction on dietary protein and sodium and po-
l Post-infectious acute nephritis: A latent period of up tassium intake. Also, restriction of fluid intake. The total
to 3 weeks before onset of symptoms (may vary from amount (ml) of fluid consumption should be calculated
1-2 weeks for postpharyngitis cases and 2-4 weeks for as follows:
cases of post-dermal infection). ml of previous days urine 1 500 ml fluid 5 fluid admin-
l If nephritis is seen within 1 to 4 days of streptococcal istered per day
infection: Pre-existing renal disease. No intake of juices.
c. Hypertension: Moderate to severe hypertension is con-
Symptoms of acute glomerulonephritis are as follows:
trolled by the hydralazine, beta-blockers like atenolol
a. Haematuria (gross haematuria is reported in 30% of
and calcium channel blockers.
paediatric patients)
d. Antibiotics: Penicillin can be administered. Dose:
b. Oliguria.
Procaine penicillin, IM for 6 days to eradicate strep-
c. Oedema (peripheral or periorbital, 85% of paediatric
tococci.
patients).
d. Oedema may be mild (involving only the face) to severe. Diuretics: It is indicated in cases of acute LVF or pulmo-
e. Headache (secondary to hypertension). nary oedema. Furosemide is the drug of choice.
f. Shortness of breath or dyspnoea on exertion (due to In patients with progressive renal failure, dialysis may
heart failure or pulmonary oedema). have to be employed.

SHORT NOTES
Q. 1 Uraemia. Q. 3 Acute nephritis—three causes.
Ans. Ans.
i. Uraemia refers to the signs and symptoms of renal fail- The three causes of acute nephritis are:
ure that are seen in patients with chronic renal failure, i. Immunological
who show an irreversible deterioration in renal function. ii. Inherited conditions like Alport’s syndrome
ii. Over a period of years, uraemia slowly leads to loss of iii. Metabolic diseases like diabetes mellitus.
excretory, metabolic and endocrine functions of the
kidney. Q. 4 Haematuria—causes.
Q. 2. Aetiology of acute nephritis. Ans.
Ans. The following are the causes of painless haematuria:
Acute nephritis is the inflammation of glomerulus. a. Renal: Trauma, glomerular disease and carcinoma
b. Extrarenal: Trauma, injections, urethritis and prostitis
c. Systemic: Vasculitis and bleeding diseases.
Aetiology Q. 5 Diuretics.
a. Immunological Ans.
b. Inherited like Alport’s syndrome (Alport syndrome is
characterized by glomerulonephritis, end-stage kidney i. Diuretics are drugs used to inhibit sodium reabsorption
disease and hearing loss). at specific sites along the nephron.
c. Metabolic (DM) ii. Types of diuretics are as follows:
d. Deposition of substance (e.g. amyloid) a. Loop diuretics (furosemide)
e. Direct injury to glomerulus b. Thiazides
Section | I General Medicine 145

c. Amiloride and triamterene c. Increased venous pressure or obstruction


d. Spironolactone d. Lymphatic obstruction
e. Carbonic anhydrase inhibitors (dopamine and
Q. 8. Proteinuria.
mannitol)
Ans.
Q. 6 Frusemide—two indications.
i. The presence of excessive protein (chiefly albumin, but
Ans.
globulin may also be seen) in the urine.
Two indications for frusemide are as follows: ii. Causes of proteinuria are as follows:
a. For treatment of oedema (cardiac, hepatic or renal) a. Kidney disease.
b. Forced diuresis b. Increased protein in serum
c. Low reabsorption at PCT.
Q. 7 Oedema—four common causes.
Q.9. Various definition of pyreurea of unknown origin.
Ans.
Ans.
The four causes of oedema are as follows:
a. Lowered oncotic pressure of blood: Liver failure, i. Pyuria is urine that contains pus.
nephrotic syndrome or malnutrition/malabsroption ii. It is the sign of UTI.
b. Increased capillary permeability: Infection, severe sys- iii. Pyuria can be seen in a septic patient or in an older
temic sepsis or drug induced. patient with pneumonia.

Topic 9

Disturbances in Water, Electrolyte


and Acid-Base Balance

SHORT ESSAYS
Q. 1. Dehydration. l Dry mouth
Ans. l Mental confusion, coma.
l Intracranial haemorrhage
Dehydration is otherwise known as primary water depletion l Tachycardia and low systolic blood pressure.
or hypernatraemia.

Aetiology Investigations
i. Decreased intake of water: Coma, depression, inability l Haematocrit usually .50%
to swallow. l Increased blood urea levels
ii. Increased loss from skin: Coma, depression, inability to l Raised plasma sodium
swallow l Urine specific gravity 1.010
iii. Increased respiratory loss: Hyperventilation l Polyuria and urine specific gravity ,1.010 in diabetes
iv. Increased loss in urine: Diabetes insipidus, medullary insipidus.
cystic disease.
Management
Clinical Features l In patients with gradual onset of water depletion over
l Marked thirst more than 2 days, then it should be corrected slowly
l Muscle weakness, rigidity. since rapid lowering of sodium produces shift of water
146 Quick Review Series for BDS 3rd Year

from ECF into brain cells. The rate of correction should l Isotonic saline (500 ml) containing 20 mmol of
not exceed 0.7 mEq/L/hour. potassium chloride should be given over 2-3 hours.
l Due to volume depletion, when hypernatraemia is as- l Repeated measurements of plasma potassium
sociated with hypotension then: are necessary to determine further requirement
a. The initial solution should be - isotonic saline so as of KCl.
to increase the ECF volume. l It is also preferable to monitor the patient using a
b. If neurological features are also present - therapy cardiac monitor.
should be started with N/2 saline.
l Relief of thirst, urine output more than 1,500 ml/ Q. 3. Hyperkalaemia—causes.
24 hours, and normal plasma sodium levels indicate Ans.
adequate correction.
Causes of hyperkalaemia are as follows:
Q. 2. Hypokalaemia. i. Impaired excretion:
l Acute renal failure
Ans.
l Severe CRF
Potassium depletion is known as hypokalaemia. l Addison’s disease

l Hypoaldosteronism

Aetiology l Use of potassium sparing diuretics.

ii. Excessive intake:


Principal causes of hypokalaemia are as follows: l Intravenous fluids containing K
1
i. Loss from GI tract:Vomiting, diarrhoea, fistulae, villous l High K foods.
1
adenoma, intestinal obstruction, etc. iii. Tissue breakdown:
ii. Loss in urine: Primary and secondary hyperaldosteronism, l Haemolysis,
Cushing’s syndrome, renal tubular acidosis, diuretics. l Rhabdomyolysis,
iii. Reduced intake: Diet containing inadequate K1. l Crush injury.
iv. Intracellular shift of K1: Alkalosis, high-dose insulin, iv. Shift of K1 out of cell:
periodic paralysis. l Acidosis

l Insulin deficiency

Clinical Features v. Pseudohyperkalaemia:


l Haemolysed blood sample.
l Generalised muscle weakness
l Depression of tendon reflexes and paraesthesiae
l Apathy, confusion and coma. Clinical Features
l ECG changes like flattening and inversion of T waves,
sagging of the ST segment, and appearance of U wave. l The common clinical features include cardiac arrhyth-
l Atrial and ventricular arrhythmias, especially in patients mias, muscular weakness progressing to flaccid paraly-
receiving digitalis. sis, and respiratory embarrasment.
l Death may occur due to respiratory paralysis or cardiac l Electrocardiographic manifestations of hyperkalaemia
arrest. are: Tall peaked T waves, prolongation of PR interval
and reduced height of P wave.
l Prolongation of QRS complex and ‘Sine wave’
Treatment pattern.
Treatment must be directed towards correcting the hypoka- l Terminally ventricular fibrillation and standstill may
laemia by identifying and eliminating the cause of potas- occur.
sium loss.
i. Potassium supplementation in the form of potassium
Management
chloride (KCl) either given orally or intravenously.
ii. To correct moderately severe deficiency: l Identification and elimination of the underlying cause.
l Oral administration of KCl at a dose of 8-15 g/day l When there are marked ECG changes, 10 ml of 10%
in divided doses calcium gluconate solution is given intravenously
l A diet rich in potassium like fruit juices. slowly over 5 to 10 minutes.
iii. In severe hypokalaemia associated with cardiac arrhyth- l Intravenous administration of glucose along with insu-
mias and muscle weakness: lin encourages shift of potassium from extracellular
l Intravenous KCl is required. compartment to intracellular compartment.
Section | I General Medicine 147

l 50 ml of 50% glucose plus 10 units of soluble (plain) l Nebulisation of b-agonists (salbutamol, terbutaline)
insulin may be given intravenously as a bolus. also can reduce potassium levels.
l Alternatively, 500 ml of 20% glucose plus 10 units l Cation exchange resins such as sodium polystyrene
of soluble insulin may be given as an infusion over sulphonate help in the removal of K1. These resins can
6-12 hours. be given orally or as enema.
l Intravenous administration of 50-100 ml of 8.4% so- l If these measures fail, haemodialysis is indicated.
dium bicarbonate is helpful in correcting acidosis.

SHORT NOTES
Q. 1. Hyponatraemia. Q. 2. Metabolic acidosis.

Ans. Ans.

l Hyponatraemia indicates dilution of body fluids by ex- Metabolic acidosis is characterised by a reduction in plasma
cess of water relative to total solute. bicarbonate and a consequent rise in hydrogen ion concen-
tration.

Aetiology
A. Hyponatraemia with low osmolality: Aetiology
i. Increased ECF volume (oedema states) like i. Excess of acid: Ketoacidosis, lactic acidosis, shock,
l Congestive heart failure methanol poisoning, salicylate poisoning
l Nephrotic syndrome ii. Loss of bicarbonate: Renal tubular acidosis, diarrhoea,
l Cirrhosis of liver fistulae, ureterosigmoidostomy.
ii. Reduced ECF volume (no oedema) iii. Inadequate acid excretion: Chronic renal failure, distal
l Renal loss of sodium (diuretics, ketonuria) renal tubular acidosis, acute renal failure.
l Extrarenal sodium loss (sweating, vomiting, diar-

rhoea, peritonitis, pancreatitis).


B. Hyponatraemia with raised osmolality: Clinical Features
l Hyperglycaemia
l In severe cases, there is deep sighing respiration (Kussmaul’s
l Mannitol administration.
breathing or ‘air hunger’).
l Peripheral vasodilatation and fall in cardiac output may
result in a fall in blood pressure.
Clinical Features
l Severe acidosis may be associated with drowsiness,
Hyponatraemia does not produce any significant clinical confusion and coma.
features. Decrease in osmolality is responsible for following l Low serum bicarbonate
features: l Low blood pH
l Muscle cramps, weakness and fatigue l Partial pressure of carbon dioxide is reduced second-
l Mental confusion, disorientation, coma and convulsions. arily by hyperventilation.

Treatment Treatment
l The rate of correction of hyponatraemia in a symptom- l Correction of the underlying disorder.
atic patient should be around 0.6 mEq/L/ hour. l In severe acidosis, sodium bicarbonate may be given.
l Correction of hyponatraemia requires either addition of l In renal failure with metabolic acidosis, dialysis may be
sodium or removal of water or both. necessary.
l Removal of excessive water can be achieved by admin-
istration of a loop diuretics and hypertonic saline. Q. 3. Oral dehydration.
l When slow correction of hyponatraemia in a volume-
Ans.
expanded subject is desired, it is best to restrict the
fluids. If this is not possible, use a loop diuretic and Dehydration is otherwise known as primary water depletion
increase intake of salt and potassium. or hypernatraemia.
148 Quick Review Series for BDS 3rd Year

Aetiology Use: Used in the treatment of mild hypertension, oe-


dema due to congestive cardiac failure, renal stones due to
i. Decreased intake of water
hypercalciuria, diabetes insipidus.
ii. Increased loss from skin
iii. Increased respiratory loss
III. Low efficacy diuretics:
iv. Increased loss in urine.
i. Potassium sparing diuretics: Triamterene, amiloride,
spironolactone
Clinical Features Use: They are used along with other diuretics in hyper-
l Marked thirst tension to reduce loss of potassium and in treatment of
l Muscle weakness, rigidity. aldosteronism.
l Dry mouth ii. Carbonic anhydrase inhibitors: Acetazolamide.
l Mental confusion, coma. Use: They are used to inhibit formation of aqueous
l Intracranial haemorrhage humour and CSF and reduce intraocular pressure and
l Tachycardia and low systolic blood pressure. intracranial pressure, respectively.
iii. Osmotic diuretics: Mannitol, urea, glycerol.
Use: They are used in the treatment of cranial and intra-
Management ocular pressure.
l In patients with gradual onset of water depletion iv. Methylxanthines: Theophylline.
over more than 2 days, it should be corrected Q. 5. Frusemide—two indications or therapeutic uses.
slowly. The rate of correction should not exceed
0.7 mEq/ L/hour. Ans.
l Due to volume depletion, when hypernatraemia is as-
Frusemide is a high efficacy loop diuretic that acts in the
sociated with hypotension then:
Henle’s loop to inhibit the sodium potassium chloride ion
l The initial solution should be - isotonic saline.
co-transport.
l If neurological features are also present - therapy

should be started with N/2 saline.


l Relief of thirst, urine output more than 1,500 ml/
Therapeutic Uses
24 hours, and normal plasma sodium levels indicate
adequate correction. l Oedema: During the initial stages of renal, hepatic and
cardiac oedema, loop diuretics are preferred.
Q. 4. Diuretics—classification and uses. l May be used in cerebral oedema,
Ans. l Hypertension: Frusemide is used in the presence of re-
nal failure, CCF or hypertensive emergency.
l The drugs that increase urine and solute excretion causing l Forced diuresis.
loss of sodium and water from the body are called diuretics. l Hypercalcaemia and renal stones.
Q. 6. Respiratory alkalosis.
Classification
Ans.
Diuretics are classified based on the efficacy of action as
follows: l Resipiratory alkalosis occurs when there is excessive
loss of carbon dioxide by overventilation of the lungs.
I. High efficacy (loop) diuretics: l There is fall of partial pressure of carbon dioxide and
Frusemide, bumatanide, piretanide, ethacrynic acid, mersalyl. hydrogen ion concentration.
Use: They are used in treatment of pulmonary, cardiac, l Plasma bicarbonate level is decreased.
renal and hepatic oedema, in poisoning with barbiturates l Aetiology includes hysterical overbreathing, lobar
to enhance forced diuresis, in treatment of hyparetension. pneumonia, pulmonary embolism, meningitis, salicy-
late poisoning and hepatic failure.
II. Moderate efficacy diuretics: l In acute respiratory alkalosis, patients may complain of
i. Thiazides: Benzothiadiazines like chlorthiazide, paraesthesia, numbness, tingling and light-headedness.
hydrochlorthiazide, polythiazide, bendroflumethiazide l Treatment is elimination of the underlying disorder. In
ii. Thiazide related drugs: Chlorthalidone, clopamide, hyperventilation syndrome, sedation and rebreathing
indapamide, metolazone, xipamide into a bag terminate the attack.
Section | I General Medicine 149

Topic 10

Endocrine and Metabolic Diseases


LONG ESSAYS
Q. 1. Classify diabetes mellitus. Discuss the complica- Complications of diabetes mellitus
tions and management of diabetes mellitus.
Or
Enumerate the proliferating factors, pathogenesis, in-
a. Acute metabolic complications b. Late systemic complications
vestigations and management of diabetic ketoacidosis.
i. Diabetic ketoacidosis i. Atherosclerosis
Ans. ii. Hyperosmolar nonketotic coma ii. Diabetic microangiopathy
iii. Hypoglycaemia iii. Diabetic nephropathy
Diabetes mellitus is a chronic disorder of carbohydrate, fat iv. Diabetic neuropathy
and protein metabolism. A defective or deficient insulin v. Infections
secretory response, which translates into impaired glucose
use is a characteristic feature of diabetes mellitus resulting a. Acute metabolic complications:
in hyperglycaemia.
i. Diabetic ketoacidosis:
l It is an exclusive complication of type I diabetes. It can
Classification of Diabetes Mellitus develop in patients with severe insulin deficiency com-
bined with glucagon excess. Failure to take insulin and
A. Primary (idiopathic) diabetes exposure to stress are precipitating causes.
i. Type I: Insulin-dependent diabetes mellitus—IDDM. l As the ketogenesis continues unabated the excess ke-
ii. Type II: Non-insulin-dependent diabetes mellitus— tone bodies produced cannot be degraded by the mus-
NIDDM. cles and other tissues resulting in ketosis, which mani-
l Non-obese NIDDM fests as anorexia, nausea, vomiting, deep and fast
l Obese NIDDM breathing, mental confusion and coma. However, most
l Maturity onset diabetes of the young (MODY) of the patients recover.
B. Secondary diabetes ii. Hyperosmolar nonketotic coma:
i. Chronic pancreatitis l It is usually a complication of type II diabetes caused by
ii. Post-pancreatectomy severe dehydration resulting from sustained hypergly-
iii. Hormonal tumours (e.g. pheochromocytoma, pituitary caemic diuresis.
tumours) l The loss of glucose in urine is so intense that the patient
iv. Drugs (corticosteroids) is unable to drink sufficient water to maintain urinary
v. Hemochromatosis fluid loss.
vi. Genetic disorders (e.g. lipodystrophy) l The prominent clinical features are of central nervous sys-
tem. Due to high viscosity of blood, thrombotic and bleed-
Complications of diabetes mellitus ing complications are frequent. The mortality rate is high.
l Complications of diabetes are seen in almost every tis-
sue and organ of the body due to biochemical and struc- iii. Hypoglycaemia:
tural alterations as a result of hyperglycaemia. l It is seen in type I diabetes patients resulting from ex-
l The complications can be broadly divided into acute cessive administration of insulin, missing a meal or due
metabolic complications and late systemic complications. to stress.
150 Quick Review Series for BDS 3rd Year

l These episodes of hypoglycaemia may permanently l When severe insulin deficiency occurs, a starvation-
damage central nervous system and are, therefore, dan- like state develops with breakdown of free fatty acids
gerous. and increasing blood levels of acetoacetic acid, beta-
l These may result in worsening of diabetic control and hydroxybutyric acid, and acetone, resulting in acidosis,
rebound hyperglycaemia called as Somogyi’s effect. This occurs predominantly in type I DM but can occur
in anyone who requires insulin to control glucose.
b. Late systemic complications:
Over a period of 15 to 20 years, either type of diabetes Pathogenesis
produces a number of systemic complications which are
responsible for morbidity and mortality.
Severe lack of insulin

i. Atherosclerosis: causes
l When compared to the general population, development lipolysis in adipose tissue
of atheroma in diabetic patient is accelerated. The dia-
betics also have extensive lesions and often have com- releases
plicated plaques such as ulcerations, calcification and free fatty acids into the circulation
thrombosis.
l Hyperlipidaemia, reduced HDL levels, nonenzymatic which are oxidized in liver to
glycosylation, increased platelet adhesiveness, obesity
ketone bodies (acetoacetic acid and hydroxybutyric acid).
and associated hypertension are the possible contribu-
tory factors for this accelerated atherosclerosis.
l The outcome of this complication is in form of early on-
Presence of glucagon accelerates this ketogenesis.
set of coronary artery disease, silent myocardial infarc-
tion, cerebral stroke and gangrene of the toes and feet.
Symptoms
ii. Diabetic microangiopathy:
l Changes of mental status, rapid respirations (to com-
l The thickening of the basement membrane of small
pensate for acidosis), acetone (fruity) odour of breath,
blood vessels and capillaries of different organs and tis-
nausea and vomiting, dehydration, and a history of dia-
sues like skin, skeletal muscle, eye, kidney, etc. is due
betes (unless first presentation). Patients frequently
to recurrent hyperglycaemia which causes increased
complain of abdominal pain.
glycosylation of haemoglobin and other proteins.
l Hyperglycaemia can be diagnosed with rapid blood
l Similar type of basement membrane like material is also
glucose determination, and ketosis can be determined
deposited in non-vascular tissues such a peripheral
with bedside reagents.
nerves, renal tubules and Bowman’s capsule.
l Urine ketone dipstick is 97% sensitive for serum ketones.
l These and the symptoms are adequate for initiation of
iii. Diabetic nephropathy:
treatment.
It is a common complication leading to death in diabetes.
l Additional laboratory investigations should include true
The four types of lesions seen are:
glucose, serum ketones, electrolytes, BUN, creatinine,
a. Diabetic glomerulosclerosis: which are diffuse and
serum osmolarity, and arterial blood gases.
nodular lesions of glomerulosclerosis.
l Urine and blood cultures should also be done. Be aware
b. Vascular lesions such as hyaline arteriosclerosis of affer-
of hypokalaemia and hypomagnesaemia. Acidosis will
ent and efferent arterioles and atheromas of renal arteries.
give spuriously elevated potassium.
c. Diabetic pyelonephritis and necrotising renal papillitis.
d. Tubular lesions or Armanni-Ebstein lesion. Management of diabetic ketoacidosis:
i. Supportive therapy:
iv. Diabetic neuropathy:
l Airway maintenance, supplemental oxygen as
Affects all parts of the nervous system. The basic patho-
needed, and treatment of shock.
logical changes are segmental demyelination, Schwann cell
ii. Fluid replacement:
injury and axonal damage.
a. 0.9 % saline (NaCl) IV.
1 litre over 30 minutes
DIABETIC KETOACIDOSIS (DKA) 1 litre over 1 hour.
l Diabetic ketoacidosis is a major medical emergency and a 1 litre over 2 hours.
severe cause of morbidity in people with type I diabetes. 1 litre over next 2-4 hours.
Section | I General Medicine 151

b. When blood glucose is ,15 mmol/l (270 mg/dl) or severe hyperkalaemia and especially if hypoten-
l Switch to 5% dextrose, 1 litre 8-hourly. sion is present. Bicarbonate should be given as an
l If still dehydrated, continue 0.9% saline and add infusion of isotonic sodium bicarbonate (1.4%). The
5% dextrose, 1 litre per 12-hour. infusion should be stopped when the pH reaches 7.2.
c. Typical requirement is 6 litres in first 24 hrs but l Monitor serum glucose and potassium as well as
avoid fluid overload in elderly patients. urine output hourly. If bicarbonate therapy was
d. Subsequent fluid requirement should be based on administered, arterial blood gases should also be
clinical response including urine output. followed.
iii. Insulin: vi. Phosphate supplementation:
a. 50 units soluble insulin in 50 ml 0.9% saline IV via l It may be required, if patient is not able to initiate
infusion pump oral intake within first few hours.
6 units/hr initially l Potassium phosphate (4 mEq of K /93 mg of phos-
1
3 units/hr when blood glucose is ,15 mmol/l phorus) may be added to maintenance fluids, if
(270 mg/dl) necessary. Caution is required with presence of re-
2 units/hr if blood glucose declines ,10 mmol/l nal insufficiency.
(180 mg/dl) vi. Magnesium:
b. Check blood glucose hourly initially – if no reduc- l It can be given as magnesium sulphate 2.5 g in
tion in first hour, rate of infusion should be in- 50 ml of NS over first hour.
creased. l Maintenance of fluids should consist of 0.45% sa-
c. Aim for fall in blood glucose of 3-6 mmol/l per hour. line with additives as indicated.
l Regular insulin either IV or IM is used to treat dia- l The 150 to 200 ml/hr is adjusted according to the
betic ketoacidosis. A reasonable goal is to maintain urine output.
the plasma glucose level around 200 mg/dl. vii. Diet:
l Most patients can be treated by ‘low-dose’ insulin Oral intake may resume when mental status and nausea and
schedules in which 8 to 10 units of regular insulin vomiting allow.
IV each hour. Most patients respond to this sched- Initial diet should consist of fluids, and full diet is not
ule, but some may not, probably due to insulin resumed until ketoacidosis is corrected.
resistance.
viii. Additional procedures in the management of dia-
l They are identified by the persistence of acidosis
betic ketoacidosis:
despite several hours of treatment. For them,
l Catheterisation, if no urine is passed after 3 hrs.
larger amounts of insulin should be given.
l Nasogastric tube to keep stomach empty in uncon-
iv. Potassium replacement:
scious or semiconscious patients, or if vomiting is
a. None in first litre of IV fluid unless ,3 mmol/l
protracted.
b. If plasma potassium ,3 mmol/l, give 40 mmol
l Central venous line, if cardiovascular system is
added potassium
compromised, to adjust fluid replacement accu-
Give in 1 litre of fluid
rately.
Avoid infusion rate of .20 mmol/hr.
l If systolic BP is ,90 mmHg or does not rise with
c. If plasma potassium is 3.5- 5.0 mmol/l, give 20 mmol
IV saline, plasma expanders are used.
added potassium.
l Antibiotics are used, if infection is demonstrated or
d. If plasma potassium is .5.0 mmol/l, or patient is
suspected.
anuric, give no added potassium.
l In severe cases monitoring of ECG.
l When the plasma potassium levels drop then po-

tassium supplementation should commence at Prognosis:


that stage.
l Myocardial infarction and infection, particularly pneu-
l If the admission potassium value is normal or low,
monia, and cerebral oedema in children are the major
potassium should be given early, to prevent hypoka-
causes of death in diabetic ketoacidosis.
laemia which can predispose to cardiac arrhythmias.
l Hypotension, azotaemia, deep coma, and associated ill-
l In view of the phosphate depletion, potassium
ness are the poor prognostic signs at admission.
should be administered initially as phosphate salt
rather than as potassium chloride, if initial phos- Q. 2. What are the different types of diabetes mellitus?
phate is below 1 mg/dl. Mention the common symptoms of each type. What are
v. Bicarbonate administration: the precautions you take while carrying out a tooth
l It is controversial but may be indicated in patients extraction in a diabetic patient?
with severe acidosis/coma, where arterial pH #7.0, Or
152 Quick Review Series for BDS 3rd Year

Discuss the aetiopathogenesis, clinical features and l Some cases may present as ketoacidosis during an inter-
complications of diabetes mellitus. Briefly outline the current illness or following surgery.
management. l Occasionally, an initial episode of ketoacidosis is
followed by a symptom-free interval known as ‘hon-
Or
eymoon period’ during which no treatment is re-
Distinguish between the insulin-dependent and non- quired.
insulin-dependent diabetes mellitus. Enumerate the l Characteristically, the plasma insulin is low or unmea-
complications of diabetes mellitus. Add a note on pre- surable.
cautions during surgery. Glucagon levels are elevated but suppressible with
Or insulin.

What are the diagnostic criteria for type-2 diabetes mel- Type-2 DM
litus. Mention the acute and chronic complications,
broadly discuss the management of type-2 diabetes. l Type-2 DM formerly known as non-insulin-dependent
DM usually begins after the age of 40 years and 60% of
Or the patients are obese. However, type-2 DM is being
Mention the complications of diabetes mellitus. Outline increasingly seen in the teenage years.
the principles of dietary management of type-1 diabetes l Type-2 DM occurs with intact beta islet cell function
mellitus. but there is peripheral tissue resistance to insulin.
l There may be some decrease in insulin production or a
Ans. hyperinsulin state. These patients are not ketosis prone
l Diabetes mellitus (DM) is hyperglycaemia secondary to but may develop it under conditions of stress.
decreased insulin production or peripheral tissue resis- l The symptoms begin gradually, over a period of months
tance to insulin. to years. Frequently, hyperglycaemia is detected in an
l Classification and aetiology is based on 1997 Report of asymptomatic person on a routine examination.
the Expert Committee on the Diagnosis and Classifica- l These patients usually do not develop ketoacidosis. In
tion of Diabetes Mellitus. the decompensated state, they are susceptible to the
l Comprises a group of disorders that share a common syndrome of hyperosmolar hyperglycaemic state, i.e.
phenotype of hyperglycaemia hyperosmolar non-ketotic coma.
l The plasma insulin levels are normal to high. Glucagon
Classification levels are elevated, but resistant to insulin.
l Symptoms of complications - burning feet, nocturia,
i. Type-1 DM diminished vision.
ii. Type-2 DM
iii. Gestational diabetes Gestational Onset DM (GODM)
iv. Other causes–Cushing’s syndrome, hypothyroidism,
l Gestational onset DM occurs when diabetes onset is
genetic causes, viral infections of the pancreas.
during pregnancy and resolves with delivery.
l These patients are at a higher risk for developing DM at
Clinical Features a later date.
l The clinical features of type-1 and type-2 DM are
distinctive. Other Specific Types of DM
They include diseases of the exocrine pancreas, various
Type-1 DM endocrinopathies (Cushing’s syndrome, phaeochromocy-
l Type-1 DM insulin-dependent DM usually occurs in toma), drug or chemical-induced DM (beta-blockers, oral
childhood or early adulthood, i.e. usually before the age contraceptives), or genetic syndromes (lipodystrophies) as-
of 40 years and results in ketoacidosis when patients are sociated with diabetes.
without insulin therapy.
This account for 10% of cases of DM. Type-1 DM is
Complications
l

caused by beta islet cell failure, which is of multifacto-


rial causes such as genetic predisposition, viral and au- l Neuropathy
toimmune attacks on the beta islet cells. l Retinopathy
l The abrupt onset of symptoms, with polyuria, polydip- l Nephropathy
sia, polyphagia and weight loss developing over days or l Coronary artery disease
weeks. l Peripheral artery disease
Section | I General Medicine 153

Diagnosis First step


l This involves the estimation of the total daily caloric
History requirement of the individual patient based on a number
l Clinical presentations of DM may include polyuria, of variable factors like age, sex, weight, activity, and
polydipsia, polyphagia associated with weight loss, occupation of the patient.
blurred vision, recurrent candidal vaginitis, soft-tissue An approximate total daily caloric requirement can be
infections, or dehydration. Many cases will be asymp- calculated as:
tomatic and picked up on routine screening. l Sedentary individuals 30 Kcal/kg/day.

l Moderately active individual 35 Kcal/kg/day.


Diagnosis of diabetes mellitus based on various test results
l Heavily active individuals 40 Kcal/kg/day.
is as follows:
a. Random plasma glucose of .200 mg/dl along with Second step
symptoms of diabetes are present.
b. Two readings of fasting plasma glucose of .126 mg/dl. l This involves allocation of the calories in a proper pro-
c. The 2-hour postprandial plasma glucose 200 mg/dl portion to carbohydrate, protein and fat.
during oral glucose tolerance test, after a glucose load l The recommended proportion of calories to be derived
of 75 g. from each of them is given as:
l Carbohydrate 50-60%
d. Elevated HbA1c. However, the HbA1c is not an ade-
l Protein 10-20%
quate screening tool for DM because it may be normal
in those with impaired glucose tolerance. l Fats 10-20%.
e. The patient is said to have impaired glucose tolerance,
However, a few more important factors need be considered
if the fasting plasma glucose is .110 mg/dl and ,126
at this stage are:
mg/dl.
l The minimal protein requirement for a good nutritious
f. Impaired glucose tolerance: 2-hour plasma glucose val-
diet is about 0.9 g/kg/day.
ues between 140 and 200 mg/dl.
l The carbohydrates should be taken in the form of
Differentiating Type-1 and type-2 DM: starches and other complex sugars.
l Occasionally, it may be difficult to differentiate between l Rapidly absorbed simple sugars like glucose should
type-1 and type-2 DM based on the clinical situation. generally be avoided. Use of caloric sweeteners includ-
The diagnosis can be clarified by the use of the C-pep- ing sucrose is acceptable in many patients.
tide, a product of the cleavage of pro-insulin to insulin. l Fish oils containing omega 3 fatty acids have been re-
This will be present in those with type-2 DM and low or ported to be beneficial, as anti-atherogenic.
absent in those with type-1 DM. l A high-fibre diet is beneficial as it has an antiathero-
l If the C-peptide is borderline, checking it after a glucose genic effect mediated through lowering of blood lipids.
load may help. In those with type-2 DM, it will increase
Third step
significantly after glucose load; this response will be
absent in those with type-1 DM. l This involves distribution of the calories throughout the
day. This is particularly important in insulin-requiring
diabetics, to avoid hypoglycaemia.
Treatment l Different distributions may be required for different
lifestyles, a typical pattern of distribution of calories is:
l Oral antidiabetics
l 20% of the total calories for breakfast.
l Glibenclamide
l 35% of the total calories for lunch.
l Glimiperide
l 30% of the total calories for dinner.
l Metformin
l 15% of the total calories for late-evening feed
l Acarbose
l Insulin–short-/intermediate-/long-acting insulin. Q. 3. Describe the aetiology, clinical features and treat-
l Modification of lifestyle ment of acromegaly.
l Food habits - containing more of complex carbohy- Ans.
drates
l Exercise Acromegaly occurs due to excess secretion of GH later in
the life after epiphyseal closure.
Dietary Regimen for a Diabetic Patients
Causes
The preparation of a dietary regimen for a diabetic can be
considered under three steps: l Pituitary adenomas – in 95% of cases.
154 Quick Review Series for BDS 3rd Year

l Pancreatic islet cell tumours – excessive growth hor- Q. 4. Discuss the side effects or complications of sys-
mone secreting pancreatic islet cell tumours. temic corticosteroid therapy with special reference to
l Hypothalamic tumours dental practice.
l Bronchial carcinoid Ans.
l Small cell carcinoma of lung
The remarkable anti-inflammatory properties of glucocorti-
coids have led to their use in wide variety of clinical condi-
Clinical Features tions, equivalent doses of commonly used glucocorticoids
Acromegaly is characterized by: are as follows:
l Hydrocortisone (cortisol)–20 mg
l Increased hand and foot size and enlargement of termi-
l Cortisone acetate–25 mg
nal phalanges of limbs.
l Prednisolone–5 mg
l The ribs also increase in size.
l Betamethasone–0.75 mg
l Face becomes enlarged and mandible is prominent with
l Dexamethasone–0.5 mg
teeth widely spaced.
l Coarse facial features Side effects of corticosteroid therapy can be considered
l Temporal headaches, photophobia and reduction in under two headings:
vision. A. Metabolic effects.
l Lips are thick and voice is coarse and husky. B. Suppression of the hypothalamic-pituitary-adrenal axis
l There is brownish pigmentation of face. (HPA axis).
l General features like fatigue, and weight gain are
present. A. Metabolic effects:
l Cardiac effects: Coronary artery disease, hypertension l Cushingoid features.
and left ventricular hypertrophy are present. l Mood changes ranging from euphoria (steroid psycho-
l Metabolic effects: Intolerance or clinical diabetes sis) to depression.
mellitus. l Fluid retention and oedema.
l Soft tissue changes like thickening of skin, increased l Hypertension.
skin tags, acanthosis nigricans, increased sweat and se- l Glucose intolerance or frank diabetes mellitus.
bum resulting in moist and oily skin, enlargement of l Osteoporosis, bone pains and fractures.
lips, nose and tongue (macroglossia), increased heel pad l Increased susceptibility to infections.
thickness, visceral enlargement (visceromegaly), e.g. l Reactivation of latent tuberculosis.
thyroid, heart (cardiomegaly) and liver, carpal tunnel l Impaired wound healing.
syndrome, myopathy, and sleep apnoea. l Gastric erosions.
l Masked perforation.
l Haemorrhage from stomach and duodenum.
Investigations
l IGF1 (insulin-like growth factor-1) is elevated. B. Suppression of hypothalamic-pituitary-adrenal axis
l GH is elevated. l Occurs with high-dose therapy.
l This makes it difficult to withdraw steroids.
l Can be minimised by giving steroids as a single morning
Treatment dose or better, on alternate days.
a. Surgical: Surgery is the treatment of choice. Surgical
Measures to Reduce the Side Effects
resection of the adenoma is done by transsphanoidal
route followed by radiotherapy. l Giving the lowest possible dose for the shortest possible
b. Radiotherapy: duration.
l Irradiation is advised when initial attempts at surgery l Giving on alternate days rather than daily as a single
do not reduce growth hormone levels to 5 MU /L. dose than in divided doses.
l Implantation of radioactive isotope yittrium 90 causes l Giving in the morning rather than at any other time.
major reduction in growth hormone levels. l Giving for established indications, only under medical
c. Medical therapy: supervision.
Somatostatin analogues: l Monitor caloric intake to prevent weight gain and re-
Bromocriptine 20-30 mg/day orally in divided doses is duce sodium intake.
given. l Use H2 receptor blockers or proton-pump inhibitors.
Octreotide 0.05-0.1 mg subcutaneously is given. l Provide high calcium intake and vitamin D.
Section | I General Medicine 155

Q. 5. Outline the conditions which produce pigmenta- l Elevated blood urea, hyponatraemia and hyperkalaemia.
tion of oral mucosa. Describe the clinical features of l Low blood sugar levels.
Addison’s disease. l Plasma renin activity (PRA) is high and plasma aldoste-
rone levels low or normal.
Ans. l In tuberculous adrenalitis, chest radiograph may show
evidences of pulmonary tuberculosis. CT scan and plain
radiograph of abdomen may show adrenal calcification.
Causes of Addison’s Disease
l Adrenal and other organ-specific antibodies may be
l Autoimmune adrenalitis detected in the serum in autoimmune adrenalitis.
l Tuberculous adrenalitis
Bilateral adrenalectomy
l
Management
l Adrenal haemorrhage or infarction
Drugs (ketoconazole). l Patients with Addison’s disease require life-long gluco-
Metastases in the adrenal corticoid and mineralocorticoid replacement therapy.
l Haemochromatosis l Cortisone (or hydrocortisone) is given at a dose of
l Amyloidosis 20 mg on getting up in the morning and 10 mg in the
l Histoplasmosis. evening at 6 PM. Alternatively, prednisolone is given in
a dose of 5 mg in the morning and 2.5 mg in the evening.
l Fludrocortisone (mineralocorticoid) 0.05 to 0.1 mg
Clinical Features daily.
l Clinical features of Addison’s disease result from gluco- l Addison’s disease due to tuberculous adrenalitis should
corticoid deficiency, mineralocorticoid deficiency, an- be treated with antituberculous chemotherapy.
drogen deficiency and ACTH excess. l During periods of stress and infections, the patient
l The cardinal features of Addison’s disease are hypo- should be told to take additional doses of prednisolone.
tension, pigmentation, and previous history of acute Q. 6. Define hypo - and hyperthyroidism. Discuss clinical
adrenal crisis following stress, or slow recovery from features, investigations and treatment of hypothyroidism.
illness.
l Glucocorticoid deficiency results in malaise, weakness, Or
weight loss, anorexia, nausea, vomiting, diarrhoea or Discuss the aetiology, clinical features and management
constipation, postural hypotension and hypoglycaemia. of myxoedema.
l Mineralocorticoid deficiency manifests as hypotension.
l ACTH excess results in pigmentation of exposed areas, or
pressure areas like elbows, knees and knuckles, palmar Myxoedema. Describe the clinical features, complica-
creases, mucous membranes, conjunctivae and recently tions, treatment and dental considerations.
acquired scars.
l Androgen deficiency results in diminution of body hair, Ans.
especially in females. l Hypothyroidism can be due to either primary disorders
l A number of autoimmune diseases may be associated of the thyroid gland (primary hypothyroidism) or de-
with Addison’s disease. They include Hashimoto’s thy- creased TSH secretion by the pituitary gland (secondary
roiditis, primary atrophic hypothyroidism, pernicious hypothyroidism).
anaemia, type-1 diabetes mellitus, primary ovarian fail- l The term myxoedema indicates severe hypothyroidism
ure and hypoparathyroidism. in which there is accumulation of hydrophilic muco-
l The association of two or more of these endocrinopa- polysaccharides in the ground substance of the dermis
thies is known as type II polyglandular autoimmune and other tissues, leading to thickening of the facial
syndrome. features and doughy induration of the skin.
l The combination of adrenal insufficiency, hypoparathy- l Thyrotoxicosis or hyperthyroidism is the clinical syn-
roidism and chronic mucocutaneous candidiasis consti- drome caused by an excess of circulating free thyroxine
tutes type I polyglandular autoimmune syndrome. and free tri-iodothyronine, or both.

Investigations Hypothyroidism
l ACTH stimulation test: There is failure of plasma corti- l Hypothyroidism can be due to either primary disorders
sol to rise following administration of ACTH. of the thyroid gland or decreased TSH secretion by the
l Plasma ACTH levels are elevated. pituitary gland.
156 Quick Review Series for BDS 3rd Year

l The most common cause of hypothyroidism worldwide l Neuromuscular: Aches and pains, muscle stiffness, de-
is iodine deficiency. However, in areas where iodine layed relaxation of tendon reflexes, carpal tunnel syn-
deficiency is not present, the important causes are auto- drome, deafness, depression, psychosis, cerebellar
immune thyroiditis and iatrogenic. ataxia, and myotonia.
l Dermatological: Myxoedema (non-pitting oedema of
the skin of hands, feet and eyelids), dry flaky skin and
Classification hair, alopecia, vitiligo, purplish lips and malar flush,
l Primary hypothyroidism–due to cause within the carotenaemia, erythema ab igne, and xanthelasmas.
thyroid gland itself. l Reproductive: Menorrhagia, infertility, galactorrhoea,
l Secondary hypothyroidism–due to failure of TSH pro- and impotence.
duction following pituitary or hypothalamic disease. l Haematological: Macrocytosis, and anaemia.
l Miscellaneous: Tiredness, somnolence, cold intoler-
ance, hoarseness of voice, low-pitched voice, and
Common Causes slurred speech.
Primary Hypothyroidism
Spontaneous atrophic hypothyroidism
Investigations
l

l Iodine deficiency.
l Following 131I therapy. l The earliest and most sensitive indicator of primary
l Post-thyroidectomy hypothyroidism is high levels of serum TSH. Where-
l Hashimoto’s thyroiditis as the serum TSH level is low in secondary hypothy-
l Dyshormonogenesis roidism.
l Drugs (PAS, lithium, amiodarone, interferon, methimazole) l Serum T3 and T4 levels are low, but are not reliable for
l Radiation. the diagnosis of hypothyroidism.
l High serum TSH and normal serum T4 indicates sub-
Secondary Hypothyroidism clinical or mild hypothyroidism.
l Destruction of pituitary gland l High titer of antibodies against thyroperoxidase (TPO)
Postsurgery and thyroglobulin are found in patients with Hoshimo-
Post-radiation to’s thyroiditis and atrophic thyroiditis.
Tumour l Other findings are anaemia, high serum cholesterol and
l Disorders of hypothalamus. triglycerides, increased serum creatine kinase and LDH,
hyponatraemia, and low voltage ECG.
Spontaneous Atrophic Hypothyroidism
l This is an organ-specific autoimmune disorder.
l Lymphoid infiltration of thyroid leading to fibrosis and Treatment
atrophy. l Hypothyroidism is treated with thyroxine. Oral levothy-
l TSH-receptor antibodies are ‘present which block the roxine is usually given at a dose of 50 -100 mg per day
effects of endogenous TSH. for three weeks, followed by 100 mg per day for three
l Seen in some patients of Graves’ disease treated with weeks, followed by a maintenance dose of 150 mg per
antithyroid drugs 10-20 years earlier. day. The correct dose of thyroxine is that which restores
l High risk of developing type-1 diabetes mellitus, perni- serum TSH to below 3 mU/L.
cious anaemia and Addison’s disease. l In elderly patients and those with ischaemic heart dis-
ease, thyroxine is started at a lower dose of 25 mg/day
Clinical Features to avoid cardiac side effects.
l In a few patients with ischaemic heart disease, angina
l Enlargement of the thyroid gland. may develop or worsen with thyroxine therapy. They
l General manifestations: Weakness, tiredness, cold intol- should be treated with beta-blockers and vasodilators,
erance, dry coarse skin, pallor, hair loss, puffy face, or CABG, or balloon angioplasty. This allows full re-
hand, and feet, myxoedema, weight gain, poor appetite, placement dosage of thyroxine.
hypothermia, goiter and hoarse voice. l TSH is monitored periodically and the dose adjust-
l Gastrointestinal: Decreased appetite, constipation, and ment of thyroxine is made accordingly. The ideal goal
ascites. is to maintain TSH at the lower half of the reference
l Cardiorespiratory: Angina, bradycardia, hypertension, range.
cardiac failure, pericardial effusion, and pleural effusion. l Treatment is needed lifelong.
Section | I General Medicine 157

Dental Considerations and stimulate thyroid hormone production and goiter


formation.
l Impaired ability of small vessels to contract when cut
l Graves’ disease is characterized by diffuse enlargement
due to deposition of subcutaneous mucopolysaccharides
of the thyroid gland, infiltrative ophthalmopathy (ex-
in hypothyroidism may result in excessive bleeding.
ophthalmos) and pretibial myxoedema (dermopathy).
Hence, local pressure is required for longer period to
l Important manifestations of hyperthyroidism are weight
control bleeding.
loss with increased appetite, heat intolerance, sweating,
l Due to poor healing in hypothyroidism, there is a de-
palpitation, tremors, and nervousness.
layed wound healing and increased risk of infection.
l The signs are tachycardia, atrial fibrillation, fine finger
l In patients with hypothyroidism, use of sedatives,
tremors, moist warm skin, lid retraction, wide palpebral
opioid analgesics and tranquillizers may precipitate
fissure, lid lag and exophthalmos.
myxoedema coma.
l Ophthalmopathy is present in 20-50% patients of Graves’
l Well-controlled hyperthyroidism and hypothyroidism
disease. It may precede the development of thyrotoxicosis
do not pose any additional risk for dental procedures.
or may develop after successful treatment of hyperthyroid-
Q. 7. Discuss the aetiology, clinical features, investiga- ism of Graves’ disease. It usually consists of chemosis,
tions and management of thyrotoxicosis. scleral injection, periorbital oedema and proptosis.
l Proptosis may cause corneal drying and damage. In se-
Or
vere case, exophthalmos, diplopia and optic nerve com-
Describe the clinical features and treatment of thyro- pression may occur.
toxicosis. l Dermopathy occurs in about 5% of patient with Graves’
Ans. disease. Purple or pink patches over anterior and lateral
aspect of the leg (pretibial myxoedema) is commonly
Thyrotoxicosis is defined as clinical manifestations due
seen.
to excessive thyroid hormones.
l Thyroid acropachy is unusual feature of Graves’ disease
Or and manifests as digital clubbing and swelling of fingers
and toes.
A state of excessive thyroid hormone due to hyperfunc-
tion of the thyroid gland is called hyperthyroidism.
Investigations
Aetiology of Thyrotoxicosis l Serum TSH level is suppressed and is the initial diag-
nostic test. Normal TSH levels exclude clinical hyper-
Primary hyperthyroidism thyroidism.
l Graves’ disease l Serum total and unbound (free) T3 and T4 are in-
l Multinodular goiter creased.
l Toxic adenoma (solitary nodule) l In some cases, only T3 levels are raised whereas T4 is
l Subacute thyroiditis (de Quervain’s) normal (T3 toxicosis).
l Iodide induced (Jod-Basedow disease) l TSH-R antibodies levels are increased in about 75%
Drugs – amiodarone cases. ESR may be increased in subacute thyroiditis.
Radiographic contrast media l The uptake of radioactive iodine by thyroid is high in
Excessive iodine ingestion Graves’ disease and toxic nodular goiter whereas it is
l Ingestion of excess thyroid hormone low in subacute thyroiditis.
l Functioning thyroid carcinoma metastasis l Ultrasonography thyroid gland reveals diffuse enlarge-
ment of thyroid gland which helps us to differentiate
Secondary hyperthyroidism Graves’ disease from nodular goiter.
l TSH secreting pituitary tumour
Pregnancy and trophoblastic tumours
Treatment
l

Hyperthyroidism or Graves’ disease is treated by:


Clinical Features l

a. Antithyroid drugs,
l The most common cause of thyrotoxicosis is Graves’ b. Radioactive iodine (131 I) or
disease (Basedow’s disease) 60-80%. c. Subtotal thyroidectomy.
l Graves’ disease is an autoimmune disorder with ge- l The choice of treatment depends on the cause and sever-
netic predisposition. The antibodies (TSH-R antibod- ity of hyperthyroidism, the age of the patient and clini-
ies) bind TSH receptors on thyroid follicular cells cal situation.
158 Quick Review Series for BDS 3rd Year

Antithyroid drugs Radioactive iodine (131I)


l The commonly used drugs are carbimazole, methima- l Iodine (131I) causes progressive destruction of thyroid
zole and propylthiouracil. cells. It can be used as initial treatment after antithyroid
l These drugs reduce synthesis of thyroid hormones by drugs or surgery.
inhibiting the iodination of tyrosine. l Majority of patients develop hypothyroidism following
l These drugs also reduce the thyroid antibody levels. radioactive iodine therapy.
l These are used in young, patients with mild thyrotoxi-
cosis and small goiter. Thyroid surgery (subtotal thyroidectomy)
l The drugs are given for prolonged periods of about l In cases of relapse after antithyroid drugs and in young
1-2 years. After stopping treatment, relapse occurs in males with large goiter or severe hyperthyroidism, sur-
about 50% of patients. gery is indicated. This is also preferred in pregnant
l Rash, fever, and arthalgia are common side effects women.
while agranulocytosis is a serious side effect. l Complications are recurrent laryngeal nerve palsy and
l Propranolol in a dose of 20-40 mg every 6 hours is used hypoparathyroidism.
to control adrenergic symptoms like tachycardia, trem- l Recurrence of hyperthyroidism or development of hy-
ors, sweating and anxiety that occur due to hyperthy- pothyroidism may also occur.
roidism of any origin while it has no effect on thyroid
hormone synthesis and secretion.

SHORT ESSAYS
Q. 1. Mention a note on thyroid function tests and vari- l In patients on thyroxine therapy, fT4 levels are raised
ous imaging techniques used in thyroid diseases. and fT3 levels are normal.
Ans. iii. Total serum thyroxine (tT4) and triiodothyronine
Various thyroid function tests used as laboratory thyroid (tT3):
profile includes measurement of following: l The measured total serum T4 reflects hormone which
i. Serum TSH (thyroid-stimulating hormone). both binds to thyroid-binding globulin and free hormone.
ii. Serum free T3 and T4. l Raised levels are characteristically seen in hyperthy-
iii. Total serum thyroxine (tT4) and triiodothyronine (tT3). roidism, but also in pregnancy, oestrogen therapy,
iv. T3 resin uptake, free thyroxine index (FTI), effective tamoxifen use, and as a congenital anomaly.
thyroxine ratio l Depressed levels are characteristically seen in hypothy-
roidism, but also with nephrotic syndrome, androgen
i. Serum TSH (thyroid-stimulating hormone): therapy, liver failure, or drugs like salicylates, sulpho-
l It is measured by immunoradiometric assays (TSH nylureas and phenytoin.
IRMAs) and is most sensitive index of thyroid function. l Altered by factors which affect the concentration of TBG.
l Suppressed levels indicate primary thyrotoxicosis. l tT3 measurements are subject to the same limitations as
l Elevated levels indicate primary hypothyroidism. for tT4 in relation to TBG.
l Besides primary thyroid disease, TSH levels may also
vary in certain other conditions hence TSH value should iv. T3 resin uptake, free thyroxine index (FTI),
always be interpreted along with T4 assay, e.g. TSH- effective thyroxine ratio:
secreting pituitary tumours, first trimester of pregnancy All these three tests are not used now.
(low TSH), high doses of corticosteroids (low TSH). Some special imaging techniques useful in assessment
of thyroid dysfunction are as follows:
ii. Serum free T3 (f T3) and free T4 (f T4)
l Advantage of serum free T3 and T4 level measurements i. Uptake of radioactive iodine or technetium:
over total T3 and T4 levels is that these are not influ- l An increased uptake of iodine is observed in overactive
enced by changes in the thyroid hormone binding gland synthesizing excess T3. The increased uptake is
globulins (TBG). demonstrated by giving an oral tracer dose of 131I or 121I
l In patients with primary thyrotoxicosis, fT3 and fT4 and measuring the thyroid uptake after 4 hours, using a
levels are elevated. counter over the neck. Alternatively, technetium-99m
l In patients with thyrotoxicosis, fT4 levels are normal (99mTc) may be given intravenously and the thyroid up-
and fT3 levels are raised. take measured.
Section | I General Medicine 159

l Even in the absence of thyrotoxicosis, iodine deficiency ii. Central nervous system dysfunction: Dizziness, head-
or enzyme deficiency may show increased uptake mea- ache, clouding of vision, blunted mental acuity, loss of
surement. fine motor skill, confusion, abnormal behaviour,
l Excess iodine may show diminished uptake measure- convulsions, and loss of consciousness.
ment even in the presence of thyrotoxicosis. l When the hypoglycaemia is of rapid onset adrenergic
symptoms predominate, when the hypoglycaemia is of
ii. Radionuclide scan: gradual onset, central nervous system manifestations
l Radionuclide scanning of thyroid using 1311, 99mTc or predominate.
121
I is very useful in demonstrating the distribution of l Indications of daytime hypoglycaemic episodes are
functioning thyroid tissue. sweating, nervousness, tremor and hunger.
l It is extremely useful in determining the activity of a l Indications of nocturnal hypoglycaemic episodes during
solitary thyroid nodule. A ‘hot’ nodule indicates that it sleep include night sweats, unpleasant dreams, and
is functional, and a ‘cold’ nodule indicates that it is non- early morning headache. Frequently, nocturnal hypo-
functional. glycaemic is asymptomatic.
l Hypoglycaemia from excessive depot-insulin given
iii. Tests to determine aetiology of thyroid disease: before breakfast usually occurs in the late afternoon.
l In autoimmune thyroiditis, antibodies against thyroid Depot-insulin given before the evening meal causes
peroxidase (TPO) and thyroglobulin are present. These hypoglycaemia through the night and in the early
antibodies may also occur in normal population but hours of the morning.
these persons are at a high risk of developing thyroid l Hypoglycaemia occurring with long-acting sulphonyl-
disease late in life. ureas may be prolonged, refractory and dangerous.
l Thyroid-stimulating immunoglobulins (TSI) are present in
Graves’ disease. These antibodies stimulate TSH receptors.
l Serum thyroglobulin (Tg) levels are increased in all Management
types of thyrotoxicosis except thyrotoxicosis factitia. i. Oral carbohydrate: If hypoglycaemia is recognized
Q. 2. Hypoglycemia-clinical features and management. early, it may be corrected by ingestion of easily absorb-
able form of carbohydrate.
Ans. ii. Intravenous dextrose:
l The term hypoglycaemia is used to indicate blood sugar l This is indicated in serious hypoglycaemia, when

levels less than 3.5 mmol/l (63 mg/dl) and is a danger- mental function is impaired, and when prolonged
ous complication. hypoglycaemia is anticipated.
l Prolonged hypoglycaemia may cause permanent brain l A 50 mL of 50% dextrose should be given initially,

damage. followed by infusion of 5-10% dextrose.


l Hypoglycaemia commonly complicates therapy with iii. Glucagon:
insulin and oral hypoglycaemic drugs. l Severe hypoglycaemia may be treated with gluca-

l Less commonly, hypoglycaemia occurs in non-diabetic gon 1 mg subcutaneously or intramuscularly, and


subjects. repeated, if necessary, after 10 minutes.
l Diabetic patients are vulnerable to hypoglycaemia due l It may not be effective in severe and prolonged hy-

to two reasons: poglycaemia due to depot insulins.


i. Insulin excess. l It should not be used to treat hypoglycaemia in-

ii. Counter-regulatory failure. duced by oral hypoglycaemic drugs.


iv. Other measures:
l These are aimed at prevention of recurrence. They
Clinical Features include adjustments in the dosage of oral hypogly-
l Hypoglycaemia usually follows a missed meal, too caemic drugs, changing the timing of insulin injec-
much insulin, or unexpected exercise. tions, adjustments in diet and physical activity, etc.
l Symptoms due to hypoglycaemia mutually occur with a
Q. 3. Hypothyroidism-causes and clinical features.
plasma glucose level of 60 mg/dL in non-diabetic per-
sons. Poorly controlled diabetic patients develop symp- Ans.
toms at higher levels (80 mg/dL), while well-controlled
diabetic patients develop symptoms at lower levels. l Hypothyroidism can be due to either primary disorders
l Symptoms of hypoglycaemia fall into two main categories: of the thyroid gland (primary hypothyroidism) or de-
i. Adrenergic symptoms: Sweating, tremor, tachycar- creased TSH secretion by the pituitary gland (secondary
dia, anxiety, and hunger. hypothyroidism).
160 Quick Review Series for BDS 3rd Year

l The most common cause of hypothyroidism worldwide Aetiology


is iodine deficiency.
l Idiopathic or autoimmune mechanisms.
l Infections
Common Causes l Haemorrhage

Primary hypothyroidism
l Spontaneous atrophic hypothyroidism Clinical Features
l Iodine deficiency. l Weakness
l Following 131I therapy. l Skin and mucosal pigmentation
l Post-thyroidectomy l Weight loss
l Hashimoto’s thyroiditis l Hypotension
l Dyshormonogenesis l Salt craving
l Drugs (PAS, lithium, amiodarone, interferon, me- l Abdominal pain
thimazole) l Diarrhoea
l Radiation. l Vitiligo

Secondary hypothyroidism
Investigations
l Destruction of pituitary gland
Post-surgery l Low cortisol and aldosterone levels and failure to rise
Post-radiation with ACTH administration.
Tumour l Low serum sodium and chloride, high potassium.
l Disorders of hypothalamus.

Treatment
Clinical Features l Hormone replacement therapy with hydrocortisone and
l Enlargement of the thyroid gland. f1udrocortisone.
l General manifestations: Weakness, tiredness, cold intol- Q. 5. Management of type- 2 diabetes.
erance, dry coarse skin, pallor, hair loss, puffy face,
hand, and feet, myxoedema, weight gain, hypothermia, Ans.
goiter and hoarse voice.
The steps in the management of type II diabetic patients are
l Gastrointestinal: Decreased appetite, constipation, ascites.
as follows:
l Cardiorespiratory: Angina, bradycardia, hypertension,
l In obese type-2 diabetic patients, weight reduction
cardiac failure, pericardial effusion, pleural effusion.
should be achieved by diet modification and increased
l Neuromuscular: Aches and pains, muscle stiffness, de-
physical activity.
layed relaxation of tendon reflexes, carpal tunnel syn-
l The first-line therapy in obese patients is metformin. If
drome, psychosis, cerebellar ataxia, myotonia.
this is inadequate, sulfonylurea, thiazolidinediones, or
l Dermatological: Myxoedema (non-pitting oedema of
alpha glucosidase inhibitors are added either singly or
the skin of hands, feet and eyelids), dry flaky skin and
in combination.
hair, alopecia, vitiligo, purplish lips and malar flush.
l If the combination of these drugs fails to control blood
l Reproductive: Menorrhagia, infertility, galactorrhoea,
glucose levels, then start with insulin therapy.
impotence.
l To non-obese patients, a weight maintenance diet is
l Haematological: Macrocytosis, anaemia.
prescribed and the first-line therapy is sulfonylurea.
l Miscellaneous: Tiredness, somnolence, cold intoler-
l If the blood glucose is not controlled by the maximum
ance, hoarseness of voice, low-pitched voice, slurred
dose of sulfonylurea, other agents like biguanide or
speech.
azolidinediones or both are added. If the combination of
l Treatment: Hypothyroidism is treated with thyroxine.
these drugs fails, then insulin is indicated.
Q. 4. Addison’s disease. l Other indications of insulin in type-2 DM are as follows:
a) As initial therapy in patients with severe weight loss,
Ans.
b) Patients with hepatic or renal disease where oral
Addison’s disease is a primary disease of the adrenal agents are contraindicated,
glands, which is unable to elaborate sufficient quantity of c) Patients who are acutely ill and
hormones. d) During pregnancy.
Section | I General Medicine 161

Q. 6. Regulation of calcium metabolism. Symptoms


Or l Changes of mental status, rapid respirations (to com-
Calcium homeostasis. pensate for acidosis), acetone (fruity) odour of breath,
nausea and vomiting, dehydration, and a history of dia-
Ans. betes (unless first presentation). Patients frequently
l The total amount of calcium in the body is about 2% of complain of abdominal pain.
the body weight. Most of it (99%) is in the bones. l Hyperglycaemia can be diagnosed with rapid blood
l The normal total serum calcium level is 9-10.5 mg / dL glucose determination, and ketosis can be determined
(2.22.6 mmol/L). Half of this is present in free form with bedside reagents.
(ionized calcium) and the remainder is bound with pro- l Urine ketone dipstick is 97% sensitive for serum ketones.
teins mainly albumin. l These and the symptoms are adequate for initiation of
l The ionized calcium is responsible for the physiological treatment.
functions of the calcium such as nerve function and l Additional laboratory investigations should include true
muscle contraction. glucose, serum ketones, electrolytes, BUN, creatinine,
serum osmolarity, and arterial blood gases.
Regulation of calcium metabolism is as follows:
l The calcium metabolism is regulated chiefly by the Management of diabetic ketoacidosis:
parathyroid hormone (parathormone) and vitamin D. l Intravenous saline is initially given to correct dehydration.
l Serum calcium level is principal regulator of parathy- l Once blood glucose level falls to 250 mg/dL, the fluid is
roid hormone release. changed to 5% dextrose solution.
l Parathyroid hormone maintains serum calcium level by l Intravenous infusion of regular insulin is administered
the following mechanisms; until the ketoacidosis is corrected.
i. Promoting resorption of calcium from bones. l Potassium and phosphate are replaced as needed.
ii. It also promotes resorption of calcium from renal l Sodium bicarbonate is generally not required. It is
tubules. given, if blood pH is less than 7.0.
iii. It stimulates the synthesis of I, 25-dihydroxycho­ l Antibiotics are given to treat infections.
lecalciferol by the kidneys and thus indirectly pro- l Periodical monitoring of the electrolytes, blood glucose,
motes the absorption of calcium from the intestine. blood urea, serum creatinine, and arterial blood gases is
iv. Vitamin D enhances the absorption of calcium and required.
phosphate from the gut.
Q. 8. Graves’ disease.
Q. 7. Diabetic ketoacidosis.
Or
Or
Clinical features and management of hyperthyroidism.
Diabetic ketoacidotic coma.
Ans.
Ans.
l A state of excessive thyroid hormone due to hyperfunc-
l Diabetic ketoacidosis is a major medical emergency and a tion of the thyroid gland is called hyperthyroidism or
severe cause of morbidity in people with type I diabetes. thyrotoxicosis.
l Pathogenesis:

Severe lack of insulin Clinical Features


causes l The most common cause of thyrotoxicosis is Graves’
lipolysis in adipose tissue
disease (Basedow’s disease) 60-80%.
l Graves’ disease is an autoimmune disorder with genetic
releases predisposition. The antibodies (TSH-R antibodies) bind
TSH receptors on thyroid follicular cells and stimulate
free fatty acids into the circulation
thyroid hormone production and goiter formation.
which are oxidized in liver to l Graves’ disease is characterized by diffuse enlargement
of the thyroid gland, infiltrative ophthalmopathy (ex-
ketone bodies (acetoacetic acid and hydroxybutyric acid).
ophthalmos) and pretibial myxoedema (dermopathy).
l Important manifestations of hyperthyroidism are weight
loss with increased appetite, heat intolerance, sweating,
Presence of glucagon accelerates this ketogenesis.
palpitation, tremors, and nervousness.
162 Quick Review Series for BDS 3rd Year

l The signs are tachycardia, atrial fibrillation, fine finger c. Thyroid surgery (subtotal thyroidectomy)
tremors, moist warm skin, lid retraction, wide palpebral
l In cases of relapse after antithyroid drugs and in young
fissure, lid lag and exophthalmos.
males with large goiter or severe hyperthyroidism sur-
l Ophthalmopathy is present in 20-50% patients of Graves’
gery is indicated. This is also preferred in pregnant
disease. It may precede the development of thyrotoxicosis
women.
or may develop after successful treatment of hyperthyroid-
ism of Graves’ disease. It usually consists of chemosis, Q. 9. Cushing’s syndrome.
scleral injection, periorbital oedema and protosis.
Ans.
l Proptosis may cause corneal drying and damage. In se-
vere case, exophthalmos, diplopia and optic nerve com- l Cushing’s syndrome is a clinical syndrome produced by
pression may occur. the increased production of cortisol from the adrenal
l Dermopathy occurs in about 5% of patient with Graves’ glands.
disease. Purple or pink patches over anterior and lateral
aspect of the leg (pretibial myxoedema) is commonly
seen.
Causes
l Thyroid acropachy is unusual feature of Graves’ disease l Bilateral adrenal hyperplasia
and manifests as digital clubbing and swelling of fingers l Secondary to pituitary ACTH production or ectopic
and toes. ACTH production
l Adrenal neoplasms
Investigations l Prolonged glucocorticoid use.

l Serum TSH level is suppressed and is the initial diag-


nostic test. Normal TSH levels exclude clinical hyper- Clinical Features
thyroidism. l Truncal obesity
l Serum total and unbound (free) T3 and T4 are l Weakness
increased. l Hypertension
l The uptake of radioactive iodine by thyroid is high in l Hirsutism
Graves’ disease and toxic nodular goiter whereas it is l Amenorrhoea
low in subacute thyroiditis. l Striae
l Ultrasonography of thyroid gland reveals diffuse l Depression
enlargement of thyroid gland which helps us to differen- l Ecchymosis
tiate Graves’ disease from nodular goiter. l Proximal myopathy
l Oedema
Treatment
l Hyperthyroidism or Graves’ disease is treated by: Investigations
a. Antithyroid drugs,
b. Radioactive iodine (131 I) or l Serum cortisol - elevated
c. Subtotal thyroidectomy. l Failure to suppress the cortisol levels with dexa­
l The choice of treatment depends on the cause and sever- methasone.
ity of hyperthyroidism, the age of the patient and clini- l Plasma ACTH
cal situation. l X-ray chest for evidence of thymic carcinoma.
l X-ray dorsal spine may show collapse of vertebrae pro-
a. Antithyroid drugs ducing fish spine appearance.
l The commonly used drugs are carbimazole, methima- l Pyelography for adrenocortical tumour.
zole and propylthiouracil. l Tomography, USG, CT scan and MRI to detect
l The drugs are given for prolonged periods of about 1-2 adrenal hyperplasia.
years. After stopping treatment, relapse occurs in about l Glucose tolerance test such as diabetes.
50% of patients.
l Rash, fever, and arthralgia are common side effects Treatment
while agranulocytosis is a serious side effect.
l Resection of pituitary adenoma producing ACTH or
b. Radioactive iodine (131I) radiation.
l Iodine (131I) causes progressive destruction of thyroid l Bilateral adrenalectomy to reduce the adrenal cortisol
cells. It can be used as initial treatment after antithyroid secretion.
drugs or surgery. l Medical treatment with metyrapone, mitotane.
Section | I General Medicine 163

Q. 10. Myxoedema coma. l Hyponatraemia and hypoglycaemia.


l Arterial blood gases often reveal respiratory acidosis,
Ans.
hypoxia and hypercapnia.
l Myxoedema coma is a rare complication of hypothy- l The CSF pressure is low and protein content is elevated.
roidism, seen usually in elderly patients.
Management
Clinical Features
l Treatment of precipitating cause.
l More than 90% of cases occur during the cold winter l Any patient with suspected myxoedema coma should
months. be treated presumptively with thyroid hormone. Since
l The usual precipitating causes are infections, drugs T3 is more biologically active than T4, most physicians
(amiodarone, anaesthetic agents, barbiturates, beta- recommend intravenous Ta (20 µg 8-hourly). However,
blockers, diuretics, lithium, narcotics, phenothi- parenteral Ta is not easily available. Therefore, intrave-
azines), cardiac failure, hyponatraemia, hypoxia and nous T4, initially 100–150 µg and then 75–100 µg daily,
hypercapnia. is recommended. If intravenous formulation is not
l Patients with myxoedema coma usually have long- available, T4 may be administered orally in a dose of
standing hypothyroidism. 300 µg stat followed by 100–300 µg daily.”
l The patients present with hypothermia and neuropsy- l Hydrocortisone sodium succinate 100 mg intravenously
chiatric manifestations. 8 hourly.
l They develop psychosis with delusions and hallucina- l Correction of hyponatraemia and hypoglycaemia.
tions (‘myxoedema madness’), progressing to depressed l Supportive therapy includes gentle warming of the patient
level of consciousness, convulsions and coma (‘myxo- with blankets, intravenous fluids, broad-spectrum antibi-
edema coma’). otics, high-flow oxygen, and rarely assisted ventilation.

SHORT NOTES
Q. 1. Tetany—cardinal signs. ii. Convulsions may occur.
iii. Coma is induced by infection, cold exposure and
Ans. systemic illness.
The cardinal signs of tetany are as follows: iv. It is more commonly seen in the elderly.
l In children, the characteristic triad of carpopedal spasm, v. It has a high mortality rate.
stridor and convulsions occur. l The treatment consists of:
l In adults, there is tingling in the hands, feet and around i. Intravenous administrion of levothyroxine, 400 mg
the mouth (circumoral paraesthesia). as a loading dose followed by 50-100 mg daily.
l Latent tetany may be present when signs of overt tetany ii. If parenteral T4 is not available, T3 can be given IV
are lacking. in the dosage of 10-25 mg 8 hourly.
l It is recognised by eliciting two signs: iii. Supportive measures include external warming, use
i. Trousseau’s sign: Inflation of sphygmomanometer of broad-spectrum antibodies, and correction of
cuff on the upper arm to more than systolic pressure hypoglycaemia and hyponatraemia.
is followed by the characteristic carpal spasm within iv. As there is impaired adrenal reserve in severe hypo-
3 minutes. thyroidism, 50-l00 mg of cortisone is also given
ii. Chvostek sign: Tapping over the branches of facial every 6 hourly.
nerve as they emerge from the parotid gland pro- Q. 3. Acromegaly—any three signs.
duces twitching of the facial muscles.
Ans.
Q. 2. Myxoedema coma. i. Acromegaly occurs due to excess secretion of GH later
Ans. in the life after epiphyseal closure.
ii. Acromegaly is characterized by:
l The term myxoedema indicates severe hypothyroidism l Increased hand and foot size and enlargement of
and is a rare presentation of hypothyroidism. terminal phalanges of limbs.
l Clinical features are: l The ribs also increase in size.
i. Altered sensorium, hypothermia, hypoglycaemia, l Face becomes enlarged and mandible is prominent
hypoventilation, hypoxia, hypercapnia, hyponatrae- with teeth widely spaced.
mia, and hypotension. l Coarse facial features
164 Quick Review Series for BDS 3rd Year

l Temporal headaches, photophobia and reduction in l All these symptoms are directly and indirectly related to
vision. the deprivation of glucose supply to the central nervous
l Lips are thick and voice is coarse and husky. system (particularly the brain) due to a fall in blood
l There is brownish pigmentation of face. glucose level.
l General features like fatigue, weight gain are also l The diagnosis is made by the measurement of plasma or
present. capillary blood glucose, which is low.
l The treatment depends upon the severity of hypoglyce-
Q. 4. Addison’s disease—causes.
mia and consciousness level of the patient.
Ans. l If the patient is able to swallow, rapidly absorbable car-
i. Addison’s disease is a primary disease of the adrenal bohydrates can be administered orally (glucose and
glands, which is unable to elaborate sufficient quantity sugar). Alternatively milk, fruit, candy bars, or biscuits
of hormones. may be given to patients with mild hypoglycaemia.
ii. Aetiology: l If the patient is unable to swallow or is in severe hypo-
l Idiopathic glycaemia, intravenous glucose (20-50 ml of 50% dex-
l Infections trose) is given initially. This is followed by an infusion
l Haemorrhage of 10% dextrose to maintain blood glucose above 100
l Autoimmune adrenalitis mg/dL.
l Tuberculous adrenalitis l Glucagon injection (1 mg SC or IM) may be given in
l Bilateral adrenalectomy severe hypoglycaemia, if intravenous access cannot be
l Adrenal haemorrhage or infarction established promptly.
l Drugs (ketoconazole). l Prevention of hypoglycaemia includes proper education
l Metastases in the adrenal regarding causes and symptoms of hypoglycaemia and
l Haemochromatosis proper adjustment in medication and diet.
l Amyloidosis
Q. 7. Insulin.
l Histoplasmosis

Ans.
Q. 5. Glucosuria.
Human insulin is produced through recombinant DNA
Ans.
technology. Bovine and porcine insulins are now rarely
l Normal urine does not contain reducing substances in used.
urine that can be detected by the usual tests like the Insulins are divided into various groups based on the
Benedict’s. Presence of detectable amount of glucose, onset and duration of action as follows:
fructose or pentose is, therefore, abnormal and is called
glycosuria. I. Ultrashort-acting (rapid acting):
l Excretion of glucose in urine may occur in emotional i. Insulin lispro (lysin1 proline)
states and following anaesthesia or asphyxia. It also oc- ii. Insulin aspart
curs in severe hyperthyroidism. The most common
cause of glucosuria is, however, diabetes mellitus. II. Short-acting (rapid acting):
l The glucose concentration of urine in this condition i. Regular crystalline insulin (rapid action)
may vary form 0.5 to 12.0%. There is also associated ii. Insulin zinc suspension (semilente amorphous)
hyperglycaemia.
l In conditions of diminished renal threshold for glucose III. Intermidiate-acting:
due to a defect in the reabsorption of glucose levels. The i. Isophane insulin suspension (NPH)
condition is called renal glycosuria (glucosuria). 15% of ii. Insulin zinc suspension(lente)
pregnant women also show glucosuria without hyper- iii. Biphasic insulin (ready made ppn)
glycaemia. iv. Glargine (clear, no buffer, no protien.)
Q. 6. Hypoglycaemia. IV. Long-acting:
Ans. i. Glargine
ii. Extended insulin zinc suspension (ultra-lente)
l When the blood glucose concentration falls to less than
iii. Protamine zinc suspention (PZT)
45 mg/dl, the symptoms of hypoglycaemia appear.
l The symptoms include headache, anxiety, confusion, Insulin is usually given subcutaneously. However, regular
sweating, slurred speech, seizures and coma, and, if not insulin can also be given intravenously. Insulin lispro, aspart
corrected, death. and glargine are genetically modified insulin analogues.
Section | I General Medicine 165

Q. 8. Gliblenclamide. l This can be achieved by IM injections of iodised poppy


seed oil (475-950 mg) to women of childbearing age
Ans.
every 3-5 years, by administration of iodised oil orally
l Gilbenclamide (glyburide) belongs to second genera- at 6 months intervals or yearly intervals to adults and
tion sulphonylureas. children or by providing iodised salt for cooking.
l Sulphonylureas were the first oral hypoglycaemic drugs
Q. 10. Investigations/diagnosis of hypothyroidism.
to be introduced and are useful in patients with type II
diabetes mellitus. Ans.
l All these drugs have same mechanism of action, but dif-
l Hypothyroidism can be due to either primary disorders
fer in potency and duration of action. The second gen-
of the thyroid gland or decreased TSH secretion by the
eration drugs are about 100 times more potent than first
pituitary gland.
generation drugs.
l The most common cause of hypothyroidism worldwide
l Gilbenclamide is potent but slow-acting, marked initial
is iodine deficiency.
insulinaemic action in urine as well as bile, single dose
is sufficient despite of short half-life. Investigations used in diagnosis of hypothyroidism are as
l Usual daily dose: 5-15 mg; half-life: 4-6 hr; duration of follows:
action:18-24 hr l The earliest and most sensitive indicator of primary
l Mechanism of action: hypothyroidism is high levels of serum TSH. Where-
as the serum TSH level is low in secondary hypothy-
Sulphonylureas roidism.
l Serum T3 and T4 levels are low, but are not reliable for
the diagnosis of hypothyroidism.
l High serum TSH and normal serum T4 indicates sub-
Bind to specific receptors on B cells of islets of pancreas clinical or mild hypothyroidism.
l High titer of antibodies against thyroperoxidase (TPO)
and thyroglobulin are found in patients with Hoshimo-
to’s thyroiditis and atrophic thyroiditis.
Block the ATP—sensitive potassium channels
l Other findings are anaemia, high serum cholesterol and
triglycerides, increased serum creatine kinase and LDH,
hyponatraemia, and low voltage ECG.
Depolarization and influx of Ca2+ ions into B cells Q. 11. Phenylketonuria.
Ans.
i. Phenylketonuria (PKU) is caused by deficiency of en-
Degranulation and increased release of stored zymatic activity of phenyl alanine hydroxylase.
insulin from B cells ii. It is inherited as an autosomal recessive disorder.
iii. It is a rare metabolic cause of hypopigmentation and
l They are metabolized in the liver and mainly excreted in mental retardation occurs due to accumulation of phe-
urine. nyl alanine in neonates blood.
iv. Diagnosis is made by routine neonatal screening.
Q. 9. Cretinism. v. Treatment involves lifelong adherence to low phenyl-
Ans. alanine diet.
l Cretinism is due to hypothyroidism in children. 1% or Early and adequate dietary treatment will prevent major
more babies are born with cretinism in those areas mental retardation, although with a slight reduction in IQ.
where most women have endemic goiter. Q. 12. Three signs of Graves’ disease.
l The babies affected with cretinism are characterized by
mental and physical retardation, there is high preva- Ans.
lence of deafness, slowed reflexes and poor learning l Graves’ disease (Basedow’s disease) is an autoimmune
capabilities. disorder with genetic predisposition and is character-
l Intraoral manifestations include delayed eruption of ized by diffuse enlargement of the thyroid gland.
teeth and macroglossia. l The signs are tachycardia, atrial fibrillation, fine finger
l The best way of preventing neonatal cretinism is by tremors, moist warm skin, lid retraction, wide palpebral
ensuring adequate levels of iodine during pregnancy. fissure, lid lag and exophthalmos.
166 Quick Review Series for BDS 3rd Year

l Proptosis may cause corneal drying and damage. In se- l All these drugs have same mechanism of action, but dif-
vere case, exophthalmos, diplopia and optic nerve com- fer in potency and duration of action.
pression may occur. l The second generation drugs are about 100 times more
l Dermopathy occurs in about 5% of patient with Graves’ potent than first generation drugs.
disease. l Mechanism of action:
l Thyroid acropachy is unusual feature of Graves’ disease
and manifests as digital clubbing and swelling of fingers Sulphonylureas
and toes.
Q. 13. Hyperpituitarism.
Ans. Bind to specific receptors on β cells of islets of pancreas

i. Excessive secretion of pituitary hormones is known as


hyperpituitarism.
ii. It causes gigantism in infants and acromegaly in adults. Block the ATP-sensitive potassium channels
iii. If pituitary hormones increase before closure of epiphy-
sis of long bones, it results in gigantism
iv. If pituitary hormones increase after closure of epiphysis
of long bones, it results in acromegaly. Depolarization and influx of Ca2+ ions into β cells
v. Intraoral findings: Tongue size is increased; mandible
also is increased in size (class III); diabetes also occurs
in these patients.
Degranulation and increased release of stored
Q. 14. Ocular signs of thyrotoxicosis. insulin from β cells

Ans.
l They are metabolized in the liver and mainly excreted
Occular signs of thyrotoxicosis are as follows: in urine.
i. Exophthalmos l Sulphonylureas are useful in patients with type II diabe-
ii. Chemosis tes mellitus.
iii. Periorbital oedema Q. 16. Oral hypoglycaemic drugs.
iv. Corneal ulceration
v. Ophthalmoplegia Ans.
vi. Papilloedema l Oral hypoglycaemic drugs are used in cases of early and
Ocular symptoms of thyrotoxicosis are as follows: mild non-insulin-dependent diabetes mellitus as they
i. Grittiness, red eyes lower blood glucose levels.
ii. Excessive lacrimation l They are non-invasive drugs as the necessicity of injec-
iii. Diplopia tions is avoided here.
iv. Loss of acuity. l Oral hypoglycaemic drugs are classified as follows:
I. Sulfonylureas:
Q. 15. Sulphonylureas.
Ans.
First generation Second generation
l Sulphonylureas were the first oral hypoglycaemic drugs
to be introduced. They are divided in to two generations. Tolbutamide Gilbenclamide (glyburide)
l Sulfonylureas: Chlopropamide Glipizide
Gliclazide
Glimepride
First generation Second generation
Tolbutamide Gilbenclamide (glyburide)
Chlopropamide Glipizide II. Biguanides: Phenformin, metformin.
III. Meglitinide analogues: Repaglinide, nateglinide
Gliclazide
IV. Thiazolidinediones: Rosiglitazone, pioglitazone
Glimepride. V. a-Glucosidase inhibitors: Acarbose, miglitol
Section | I General Medicine 167

Q. 17. Metformin. l T3 is the active form of the hormone, most of the T4 is


converted into T3 in peripheral tissues such as liver,
Ans. muscles and kidneys.
l Metformin is the only biguanide used clinically. l Thyroid hormones are carried in plasma predominantly
l They inhibit hepatic gluconeogenesis and enhance insu- in the bound form with a plasma protein (thyroid bind-
lin-mediated glucose disposal from muscles and fat. ing globulin, TBG) while only a small amount circu-
l They promote peripheral glucose utilization by enhanc- lates unbound (free hormone). Only the free form can
ing anaerobic glycolysis, and inhibit alimentary absorp- enter cells and exerts its metabolic action.
tion of glucose. l Measurement of thyroxine levels is one of the thyroid
l Metformin is taken orally, well absorbed through GIT function tests.
and is excreted mostly unchanged in urine as it is not Q. 20. Goiter.
metabolized in the body at all.
Ans.
l Very high doses can cause hypoglycaemia.
l Daily dose: 0.5-2 gm; half-life: 1.5-3 hr; duration of i. Goiter is defined as enlargement of the thyroid gland,
action: 6-8 hr. leading to swelling of the neck.
l Metformin is useful in obese patients with type-2 DM ii. The most common cause for goitre is iodine deficiency,
either alone or in combination with sulphonylureas. followed by Hashimoto’s thyroiditis in places where
iodised salt is used.
Q. 18. Gynaecomastia.
iii. Clinical picture: The main symptom of goiter is swell-
Ans. ing of the thyroid gland, which may eventually become
a noticeable lump in the throat.
l Gynaecomastia is the presence of glandular breast tis-
The following symptoms may also exist when the goiter
sue in males.
enlarges significantly in size:
l It results from an imbalance between androgen and
l Hoarseness (voice)
oestrogen activity, which may reflect androgen defi-
l Coughing
ciency and oestrogen excess.
l A feeling of tightness in the throat
l Causes:
l Dysphagia (less common)
i. Idiopathic
l Dyspnoea (less common)
ii. physiological
iv. Treatment:
iii. drug induced: Cimetidine, digoxin, antiandrogens,
l Treatment may not be necessary, if the goitre is small.
anabolic steroids, etc.
l Graves’ disease can be corrected with antithyroid
iv. Hypogonadism
drugs such as propylthiouracil and methimazole,
v. Androgen resistance syndromes
thyroidectomy, and iodine-131.
vi. Oestrogen excess 131
l Thyroidectomy with I may be necessary in euthy-
l Clinical assessment includes drug history, features of
roid goitrous patients who do not respond to levo-
hypogonadism and inspection and palpation of breast
thyroxine treatment, especially if the patients have
tissue as gynaecomastia is often asymmetrical.
difficulty in breathing or swallowing.
l Investigations include ultrasonography, mammography,
random blood sample for testosterone, LH, FSH, oestra- Q. 21. Exophthalmos.
diol, prolactin and hCG. Ans.
l It is self-regressive in adolescent who is progressing
normally through puberty. i. Exophthalmos also called exophthalmia or proptosis is
l Surgical excision is justified for cosmetic reasons. An- defined as “abnormal protrusion of the eyeball anteri-
drogen replacement will usually improve gynaecomas- orly out of the orbit.
tia in hypogonadal males and any other identifiable ii. Mostly the term “exophthalmos” is reserved for those
cause should be addressed, if possible. cases of proptosis which are secondary to endocrino-
logical dysfunction.
Q. 19. Thyroxin. iii. Graves’ disease, an autoimmune disease that causes
hyperthyroidism, is the most common cause of ex-
Ans.
ophthalmos.
l Thyroxin is a harmone secreted by thyroid gland. iv. Thyroid problems generally are common causes.
l The thyroid gland secretes mainly thyroxin (T4) and a v. Measurement of the degree of exophthalmos is per-
small amount of triiodothyronine (T 3). formed using an exophthalmometer.
168 Quick Review Series for BDS 3rd Year

Q. 22. Thyroid function tests. Treatment


Ans. l Oral calcium such as calcium lactate, calcium gluconate
may relieve mild symptoms. Acute cases, injection of
Various thyroid function tests used as laboratory thyroid
calcium gluconate 10% (10 ml) should be given.
profile include:
l If tetany is not relieved, administration of magnesium
i. Serum TSH (thyroid-stimulating hormone).
may be required.
ii. Serum free T3 and T4.
l Correction of alkalosis
iii. Total serum thyroxine (tT4) and triiodothyronine (tT3).
l Persistent vomiting is treated with intravenous isotonic
iv. T3 resin uptake, free thyroxine index (FTI), effective
saline and potassium.
thyroxine ratio
l In alkali excess, their withdrawal and if necessary, am-
Q. 23. Hyperparathyroidism. monium chloride 2 g 4 hourly orally will control tetany.
l Hysterical hyperventilation may be controlled by re-
Ans.
breathing expired air from a suitable bag, or inhalation
l Primary hyperparathyroidism is caused by hypersecre- of 5% carbon dioxide in oxygen.
tion of PTH. In majority of cases, this is due to autono-
Q. 25. Diabetes mellitus—four complications.
mous hypersecretion of PTH.
l Primary hyperparathyroidism (adenoma or hyperplasia) Or
may be familial and part of multiple endocrine neopla-
Diabetes mellitus—six complications.
sia (MEN types I, 2a).
l Secondary hyperparathyroidism is characterized by the Ans.
hypersecretion of PTH due to stimulation by hypocal-
Complications of diabetes mellitus are as follows:
caemia. There is hyperplasia of parathyroid glands.
l Complications of diabetes are seen in almost every
l In tertiary hyperparathyroidism, hyperplastic parathy-
tissue and organ of the body due to biochemical and
roid glands may result in adenoma formation and
structural alterations as a result of hyperglycaemia.
autonomous PTH secretion.
l The complications can be broadly divided into acute
l Majority of patients may be asymptomatic. Howe symp-
metabolic complications and late systemic complications.
toms are generally due to hypercalcaemia. Bone resorption
occurs due as “brown tumours” or cysts of the jaw.
Q. 24. Describe clinical features and treatment of tetany.
Complications of diabetes mellitus
Ans.
Ans. Tetany is a condition where there is hyperexcitability
of peripheral nerves.
a. Acute metabolic complications b. Late systemic complications
i. Diabetic ketoacidosis i. Atherosclerosis
Clinical Features ii. Hyperosmolar nonketotic coma ii. Diabetic microangiopathy
iii. Hypoglycaemia iii. Diabetic nephropathy
i. The first symptoms of tetany are tingling and numbness
iv. Diabetic neuropathy
in the face, fingers, and toes. v. Infections
ii. Carpopedal spasm or Trousseau’s sign:
l It occurs in extreme cases and latent tetany.

l The arm is flexed at elbow, wrist, and metacarpo-

phalangeal joints but the interphalangeal joints are Q. 26. Diagnosis of diabetes mellitus.
extended. The thumb moves towards the palm.
iii. Chovostek’s sign: Ans.
l It also occurs in latent tetany.

l It indicates facial hyperexcitability.


Diagnosis of diabetes mellitus:
l If a tap is given to facial nerve in front of ear, twitch- Based on clinical history:
ing of eyelids, corners of mouth takes occurs. l Clinical presentations of DM may include polyuria,
iv. Laryngeal spasm: polydipsia, polyphagia associated with weight loss,
l Increased excitability of the laryngeal muscles pro- blurred vision, recurrent candidal vaginitis, soft-tissue
duces laryngeal spasm. infections, or dehydration.
l This leads to blockage of respiratory passage and l Many cases will be asymptomatic and picked up on
death may occur. routine screening.
Section | I General Medicine 169

Diagnosis of diabetes mellitus based on various test results Gestational Onset DM (GODM)
is as follows:
i. Gestational onset DM occurs when diabetes onset is
a. Random plasma glucose of .200 mg/dl along with
during pregnancy and resolves with delivery.
symptoms of diabetes are present.
ii. These patients are at a higher risk for developing DM at
b. Two readings of fasting plasma glucose of .126 mg/dl.
a later date.
c. The 2-hour postprandial plasma glucose 200 mg/dl
iii. Risk factors for gestational onset DM are:
during oral glucose tolerance test, after a glucose load
l Obesity
of 75 g.
l Ethinicity
d. Elevated HbA1c. However, the HbA1c is not an ade-
l Family history of type II diabetes
quate screening tool for DM because it may be normal
l Previous macrosomia
in those with impaired glucose tolerance.
l Previous glucose abnormalities in pregnancy.

Q. 27. Anti-thyroid drugs. iv. Management:


l Pregnancy should be planned
Ans. l Maintain strict glycaemic control.

Various anti-thyroid drugs are classified as follows: l Do not strive for normoglycaemia at the expense of

i. Thyroid hormone synthesis inhibitors or antithyroid hypoglycaemia


drugs (thioamides or thiourea derivaties): Propylthio- l Check overnight sample of urine for ketones regularly.

uracil, methimazole, carbimazole. Q. 29. Albuminuria.


ii. Inhibitors of iodide trapping (anion inhibitors): Thio-
cyanates and perchlorates. Ans.
iii. Hormone release inhibitors: Iodine, iodides of Na1 and l The appearance of proteins (albumin) in the urine is
K1 and organic iodide. known as albuminuria.
iv. Thyroid tissue destroying agent: Radio-active iodine l Dipstick testing for albumin is a standard method to
(131I, 125I, 123I) identify the presence of renal disease or urinary infec-
v. Others: Propranalol, atenolol, diltiazem, dexameth­ tion in diabetic individuals.
asone. l This detects urine albumin greater than 300 mg/l, smaller
Q. 28. Gestational diabetes mellitus. amounts of urinary albumin, i.e. microalbuminuria can be
measured and these provide indicators of the risk of devel-
Ans. oping diabetic nephropathy and microvascular disease.

Topic 11

Nutritional Factors in Disease

LONG ESSAYS
Q. 1. Describe the aetiology, clinical features, diagnosis extreme age groups, i.e. early childhood and geriatric
and treatment of scurvy? patients.
l Citrus fruits and tomatoes are the primary sources of
Ans.
vitamin C.
l Scurvy is a systemic disease caused due to deficiency of
vitamin C dietary intake in which vascular integrity and
Aetiology
wound repair mechanisms are defective.
l The lesions and clinical manifestations of vitamin C l Scurvy is usually encountered in third world countries
deficiency are seen more commonly in two where adequate nutrirional intake is not available.
170 Quick Review Series for BDS 3rd Year

Clinical Features l Vitamin C deficiency is now rarely seen in the industri-


alized world, as most individuals have sufficient
The manifestations of vitamin C deficiency include:
vitamin C in their diets.
i. Anaemia: It is common in scurvy and is most often
normocytic and normochromic type. Q. 2. Mention the various B-complex factors and mention
ii. Poor or delayed wound healing: Wound healing is the clinical manifestations due to their deficiency.
delayed due to deranged collagen synthesis, poor pres-
Ans.
ervation and maturation of fibroblasts and localization
of infections in the wounds. Vitamins are essential for growth and normal body func-
iii. Haemorrhagic diasthesis: tions. The various B-complex factors are:
l A marked tendency to bleeding is characteristic of A. Vitamin B1 (thiamine)
scurvy. B. Vitamin B2 (riboflavin)
l Haemarthrosis and GI bleeding C. Vitamin B3 (niacin)
l Lassitude and anorexia D. Vitamin B6 (pyridoxine)
l There may be haemorrhages of skin, mucous mem- E. Pantothenic acid
brane, gums, muscle, joints and under periosteum. F. Biotin
iv. Lesions in teeth and gums: Scurvy may interfere with G. Vitamin B12 (cyanocobalamin)
development of dentin. The gums are soft, swollen and
Deficiency of various B-complex vitamins causes clinical
may bleed readily and get infected commonly. The in-
manifestations as follows:
fected gums are known as scorbutic gingivitis.
v. Skeletal lesions: The most prominent change is de- A. Thiamine (vitamin B1):
ranged formation of osteoid matrix and not deranged
i. It acts as a coenzyme for several metabolic reactions and
mineralization. Growing tubular bones and as well as
maintains proper functioning of neurons.
flat bones are affected. Painful limbs and enlargement
ii. A deficiency of thiamine causes:
of costochondral junction
l Beriberi:
vi. Folliculosis, i.e. presence of abnormal number of
a. Dry beriberi—characterised by multiple neuritis
lymph follicles.
(polyneuropathy).
vii. Scorbic child usually assumes a frog-like position and
b. Cardiac beriberi – characterised by generalized
this may reflect as subperiosteal haemorrhage.
oedema and high output cardiac failure.
l Wernicke’s encephalopathy—confusion, nystagmus,

Oral Manifestations ophthalmoplegia, ataxia and polyneuropathy.


l Korsakoff’s psychosis—amnesia with confabulation.
Oral manifestations of scurvy are as follows: l No oral lesions.
l It chiefly affects the gingival and periodontal regions.
l Atrophy and disorganisation of odontoblasts and irregu- B. Riboflavin (Vitamin B2):
lar dentin formation. i. It is essential for cellular oxidation–reduction reactions.
l The pathognomic sign is the swollen and spongy gums, ii. Its deficiency causes:
particularly the interdental papillae is involved produc- l Angular stomatitis, glossitis and cheilosis
ing the appearance of scurvy buds. l Seborrhoeic dermatitis
l The interdental and marginal gingiva becomes bright l Photophobia, lacrimation and visual fatigue.
red, swollen, smooth, shiny producing appearance
known as scurvy bud. C. Niacin (Vitamin B3):
l In severe cases, haemorrhage in periodontal membranes i. It acts as coenzyme for oxidation–reduction reactions.
followed by loss of bone and loosening of teeth occurs. ii. The deficiency of niacin leads to pellagra characterized
by 3-Ds:
l Dermatitis
Management
l Diarrhoea and
l Vitamin C 250 mg TID orally. l Dementia
l Treatment of anaemia by iron supplementation. iii. There are dark, dry and scaly lesions in sun exposed
l Scurvy is reversed by adequate dietary intake of ascor- areas like back of the hands, lower legs, face and neck
bic acid. (Casal’s necklace).
l The vascular changes are reversible. Periodontal tissue iv. Oral manifestations:
breakdown, however, is permanent, even lost alveolar l Bright red glossitis and stomatitis.
bone will not regenerate after adequate vitamin intake. l Advanced pellagra can result in death.
Section | I General Medicine 171

v. Treatment: Oral nicotinamide or nicotinic acid 10- l Deficiency of vitamins causes various clinical manifes-
150 mg /day. tations as follows:

D. Pyridoxine (Vitamin B6): Thiamine


i. It is involved in amino acid, fat and carbohydrate l Cardiac beriberi - high output cardiac failure
metabolism and haem synthesis. l Dry beriberi - polyneuropathy
ii. Deficiency of vitamin B6 results in: l Wernicke’s encephalopathy - confusion, nystagmus,
l Angular stomatitis, glossitis and cheilosis ophthalmoplegia, ataxia and polyneuropathy
l Convulsions, peripheral neuropathy l Korsakoff’s psychosis - amnesia with confabulation
l Hypochromic, microcytic anaemia

iii. Treatment: Given orally in a dosage of 10-20 mg / day. Riboflavin


Higher doses around 100-200 mg/day are needed in l Angular stomatitis, glossitis and cheilosis.
medication related vitamin deficiency. l Seborrhoeic dermatitis
l Photophobia, lacrimation and visual fatigue
E. Pantothenic acid:
i. It plays a role in fatty acid metabolism and in synthesis Niacin (Pellagra)
of cholesterol and steroid hormones. l Dermatitis
ii. Deficiency symptoms are non-specific and include: l Diarrhoea
l Apathy l Dementia
l Depression

l Paraesthesia (burning feet syndrome). Pyridoxine


l Muscle cramps l Angular stomatitis, glossitis and cheilosis
l Personality changes l Convulsions, peripheral neuropathy
l Hypochromic, microcytic anaemia
F. Biotin:
i. It plays a role in gluconeogenesis and fatty acid synthesis. Pantothenic acid
ii. Deficiency in adults results in: l Apathy, depression
l Depression l Paraesthesia, muscle weakness
l Hallucination l Burning feet, personality changes
l Paraesthesia and

l Anorexia Biotin
iii. The biotin deficiency in infants leads to lethargy, hypo- l Periorofacial dermatitis, conjunctivitis, alopecia
tonia, periorofacial dermatitis, alopecia, ataxia, deaf- l Ataxia, deafness, optic atrophy
ness and optic atrophy.
iv. Treatment: Given in doses of 10 mg/day is effective. Cyanocobalamin
l Megaloblastic anaemia
G. Cyanocobalamin (vitamin B12):
l Peripheral neuropathy
i. Clinical manifestations of cobalamin deficiency are l Subacute degeneration of spinal cord
mainly due to the involvement of haematological, l Megaloblastic madness
gastrointestinal and nervous systems.
ii. Deficiency of cobalamin results in: Folic acid
l Pernicious anaemia
l Megaloblastic anaemia
l Peripheral neuropathy
l Psychosis
l Subacute degeneration of spinal cord
l Neural tube defects
iii. Treatment: Cobalamine is given parenterally in doses of
1000 mg, i.e. 1 mg/week for 8 weeks followed by 1 mg each Ascorbic acid (scurvy)
month. In pernicious anaemia, the treatment is lifelong. l Bleeding gums, petechiae, ecchymosis, purpura
Q. 3. Describe the various manifestations of avitaminosis l Arthralgia, joint effusions
with particular reference to oral cavity. How will you l Depression, hysteria, postural hypotension
treat them?
Vitamin A
Ans.
l Night blindness, bitot’s spots, corneal ulceration, blindness
l Vitamins are essential for growth and normal body l Follicular keratosis
functions. l Recurrent respiratory tract infections
172 Quick Review Series for BDS 3rd Year

Vitamin D l Myopathy
l Rickets l Peripheral neuropathy
l Osteomalacia
Vitamin K
Vitamin E l Ecchymosis
l Haemolytic anaemia l Mucosal bleeding
l Macrocytic anaemia l Internal haemorrhage
l Spinocerebellar syndrome

SHORT ESSAYS
Q. 1 Scurvy. c. Removal antibody-coated bacteria and blood cells.
d. Spleen form blood cells when the bone marrow is
Ans.
unable to do it.
l Deficiency of vitamin C leads to scurvy. l Spleen is normally not palpable. If palpable, the direc-
l Due to vitamin C deficiency, there is defective collagen tion of enlargement is towards right iliac fossa.
formation. l “Massive splenomegaly” is defined as the spleen palpa-
ble more than 8 cm below costal margin. The common
Clinical Features causes of massive splenomegaly are:
a. Chronic myeloid leukaemia
l Anaemia b. Portal hypertension
l Poor or delayed wound healing c. Myelofibrosis
l Haemarthrosis or GI bleeding d. Malaria
l Lassitude and anorexia e. Kala-azar
l Painful limbs and enlargement of costochondral junc- l The symptoms due to splenomegaly are abdominal
tion discomfort and pain.
l Folliculosis, i.e. presence of abnormal number of lymph l Pancytopenia may occur in patients with splenomegaly
follicles. (hypersplenism).
l A scorbic child usually assumes a frog-like position and
this may reflect as subperiosteal haemorrhage. Diagnosis
A detailed history and clinical examination are helpful
Oral Manifestations l

in knowing the underlying cause of splenomegaly.


l It chiefly affects the gingival and periodontal structures. l One should particularly look for the presence of fever,
l The interdental and marginal gingiva becomes bright bleeding, lymphadenopathy and hepatomegaly.
red, swollen, smooth, shiny producing appearance l The following investigations can be helpful in making a
known as scurvy bud. diagnosis;
l There is a typical fetid breath of a patient with fusospi- i. Complete blood count
rochaetal stomatitis. ii. Bone marrow examination
l The women bleed spontaneouslv. iii. Imaging (ultrasonography, CT scan)
iv. Screening for infections and autoimmune diseases
Management v. Endoscopy of the upper gastrointestinal tract.
Q. 3. Malnutrition.
l Vitamin C 250 mg TID orally.
l Treatment of anaemia by iron supplementation. Ans.
Q. 2 Splenomegaly. l Malnutririon occurs due to absolute or relative defi-
ciency of energy and protein. It may be due to inadequate
Ans.
food intake or secondary to illnesses such as diarrhoea,
l Spleen is a reticuloendothelial organ which lies in left respiratory infections, measles and intestinal worms.
upper quadrant of abdomen and its important functions l Classification of malnutrition in adults by body mass
are: index (weight/height) BMI (kg/m2):
a. Removal of old and dead red blood cells from circulation, Malnutrition ,18.5
b. Synthesis of antibodies in the white pulp, and Mild: 17-18.5
Section | I General Medicine 173

Moderate: 16-17 l Deficiency of vitamin A causes interference with


Severe: ,16 growth, reduced resistance to infections and interfer-
l The clinical manifestations of malnutrition ranges from ence with nutrition of cornea, conjunctiva, trachea, hair
mild growth retardation and weight loss to distinct follicle and renal pelvis.
clinical forms like kwashiorkor and marasmus. l Vitamin A deficiency interferes with ability of eyes to
l The clinical features of malnutrition or severe under- adapt to darkness and impairs visual affinity.
nutrition in adults are as follows: l Children with vitamin A deficiency will experience
i. Loss of weight. impaired growth and development.
ii. Thirst, weakness, feeling cold, nocturia, amenor-
rhoea or impotence, craving for food.
iii. Lax, pale, dry skin with loss of turgor and pig- Vitamin E
mented patches. l The most important dietary from of vitamin E is
iv. Hair thinning or loss (except in adolescents) tocopherol.
v. Cold and cyanosed extremities, pressure sores. l Rich sources of vitamin E include vegetable oils, whole-
vi. Muscle wasting. grain cereals and nuts.
vii. Loss of subcutaneous fat, reflected in reduced l Vitamin E has many direct metabolic actions:
skinfold thickness and mid-arm circumference. i. It is an important antioxidant, preventing oxidation
viii. Oedema, which may be present without hypoalbu- of poly-unsaturated fatty acids in cell membranes
minaemia ‘famine oedema’. by free radicals.
ix. Subnormal body temperature, slow pulse, low ii. It plays a role in maintaining cell membrane struc-
blood pressure and small heart. ture.
x. Distended abdomen, with diarrhoea. iii. It affects DNA synthesis and cell signaling and it is
xi. Diminished tendon jerks. involved in the anti-inflammatory and immune system.
xii. Pathy, depression, introversion, aggression, if food l Vitamin E deficiency leads to myopathies, neuropathies
is nearby. and liver necrosis. This results from the effects of cell
xiii. Susceptibility to infections. membrane damage with consequent cell leakage.
l Human deficiency states are rare and have only been
Management described in premature infants and in malabsorption.
l The first feature of human deficiency is a mild haemo-
l People with mild starvation are in no danger; those with lytic anaemia.
moderate starvation need extra feeding. People who are
severely underweight need hospital care. Q. 5. Beriberi—types and clinical features.
l The treatment is directed towards repletion of protein, Ans.
energy, vitamins and micronutrients. The supplementa-
tion can be done orally or parenterally. l Beriberi is caused due to the deficiency of vitamin B1,
i.e. thiamine.
Q. 4. Vitamin—A and E. l The deficiency of vitamin B1 occurs due to:
Ans. i. Severe malnutrition
ii. Chronic alcoholism
iii. Chronic debilitating diseases.
Vitamin A l Types of beriberi:
l Vitamin A or retinol is found only in foods of animal i. Wet beriberi or cardiovascular beriberi: Beriberi is
origin and liver is the richest source. It is also produced wet when there is cardiac involvement.
in the intestine by the splitting of carotenes, which are ii. Dry beriberi or neuritic type: Beriberi is dry when
present in green vegetable, carrot and some fruits. there is CNS involvement.
l Functions of vitamin A are as follows:
i. 11-cis retinaldehyde is the initial part of the photo-
Clinical Features
receptor complex in rods of the retina.
ii. Retinoic acid induces differentiation of epithelial i. Wet beriberi: Palpitation, dyspnoea, cardiomegaly,
cells by binding to specific nuclear receptors, which warm extremities, anasarca, signs of congestive heart
turn on responsive genes. failure in late stage.
iii. Retinoids are necessary for normal growth, fetal ii. Dry beriberi: Cramps, tingling and numbness in the
development, fertility, haematopoiesis and immune limbs, nystagmus, wrist and foot drop, ataxia, loss of
function. equilibrium, paraesthesia and confusion.
174 Quick Review Series for BDS 3rd Year

Treatment Q. 7. Night blindness.


l Vitamin B1 (50-100 mg daily) IM or IV is given. Ans.
l As acute crisis is over, patient has given small dose of
l The most important consequence of vitamin A deficiency is
5 mg daily along with nourishing diet.
blindness. Night blindness is the earliest sign of deficiency
l Diet should consists of high B complex and protein
and is due to an impairment of the dark adaptation process.
content like eggs, milk, nuts and green vegetables.
l The diagnosis of night blindness is supported by low
l Wet beriberi with cardiac involvement require digoxin
plasma retinol concentration and is confirmed by
and diuretics, if congestive heart failure is present.
marked improvement in dark adaptation following ther-
Q. 6. Vitamin A deficiency. apeutic dose of retinol.
l If untreated, the condition progresses with loss of the
Ans:
normal mucous cells from the cornea, which takes on a
l Deficiency of vitamin A causes interference with dull, hazy, lacklustre appearance due to keratinisation;
growth, reduced resistance to infections and interfer- this is described as xerophthalmia.
ence with nutrition of cornea, conjunctiva, trachea, hair l Bitot’s spots may appear as glistening white plaques of
follicle and renal pelvis. desquamated thickened conjunctival epithelium, usually
l Vitamin A deficiency interferes with ability of eyes to triangular in shape and firmly adherent to the underlying
adapt to darkness and impairs visual affinity. conjunctivae.
l Children with vitamin A deficiency will experience im- l The final consequence of deficiency is the development
paired growth and development. of keratomalacia, leading to corneal ulceration, scarring
and irreversible blindness.
On diagnosis of vitamin A deficiency, a single large
Aetiology l

dose of 60 mg retinol as palmitate or acetate (200 000


l Poor intake IU) should be given orally or, if there is vomiting or
l Malabsorption severe diarrhoea, by intramuscular injection. The oral
l Disease of liver and intestine. dose should be repeated the next day and again prior to
discharge or at a follow-up visit.
Clinical Features l Prevention is also an important issue. In countries where
vitamin A deficiency is endemic, pregnant women should
i. Earliest sign of deficiency of vitamin A is difficulty in be advised to eat dark green leafy vegetables and yellow
reading or sewing at night times or finding anything in fruits, to build up stores of retinol in the fetal liver.
darkness. l Babies should also be given such vegetables or locally
ii. Conjunctiva becomes dry and small grayish white available carotene-rich fruits.
raised spots known as Bitot’s spots appear. l In communities where xerophthalmia occurs, single pro-
iii. Microcytic anaemia. phylactic oral dose of 60 mg retinol (200 000 IU) as palmi-
iv. Cornea subsequently becomes lusterless and if there is tate given to pre-school children significantly reduces
lack of treatment, the changes are irreversible. mortality from gastroenteritis and respiratory infections; a
v. Keratomalacia involving the cornea leading to the ul- similar large dose is indicated in any child with measles.
ceration and blindness may result. l Repeated oral administration of these doses to children
vi. The children with vitamin A deficiency not only have every 4-6 months is now used in some endemic areas.
retarded growth but also increased tendency to chest
infection. Q. 8. Pellagra.
vii. Skin become dry and rough. Ans.
viii. Imperfect enamel formation of teeth.
Pellagra is the clinical condition produced due to the
deficiency of nicotinic acid or niacin.
Treatment
Vitamin A deficiency can be prevented by giving good
l
Aetiology
nutrition, intake of fresh leafy green vegetables and ad-
dition of vitamin A to food stuffs. i. Inadequate intake or absorption of niacin.
l Vitamin A may be administered orally as retinol 30 mg ii. Restricted or limited diet in which single serial grain,
daily for three days. i.e. corn is consumed without consumption of wheat,
l In advanced cases where absorption is affected vitamin eggs, beef and another niacin-rich food.
A in dose of 50, 000 IU parenterally for 3 days. iii. Chronic alcoholism
Section | I General Medicine 175

Clinical Features protein produced by the liver, which carries it to the


tissues for utilization.
l Pellagra was formerly endemic among the poor who sub-
l The liver stores enough vitamin B12 for 3 years and this,
sisted chiefly on maize; maize contains niacytin, which is
together with the enterohepatic circulation, means that
a form of niacin that the body is unable to untilise.
vitamin B12 deficiency takes years to become manifest
l Pellagra can develop in only 8 weeks in individuals
even if all dietary intake is stopped.
eating diets that are very deficient in niacin and
tryptophan. Causes of vitamin B12 deficiency:
l It remains a problem in part of Africa, and is occasion-
l Dietary deficiency
ally seen in alcoholics and in patients with chronic small
l Gastric factors
intestinal disease in developed countries.
l Pernicious anaemia
l Pellagra can occur in Hartnup’s disease, a genetic disor-
der characterised by impaired absorption of several Vitamin B12 deficiency—neurological features:
amino acids, including tryptophan. It is also seen occa-
l Severe prolonged vitamin B12 deficiency may cause
sionally in carcinoid syndrome, when tryptophan is
megaloblastic anaemia and/or neurological degener-
utilized in the excessive production of 5-HT, rather than
ation.
being available for the synthesis of niacin.
l In some cases, the neurological disease predominates,
l Pellagra has been called the disease of the three Ds:
perhaps because adequate intake of folate maintains
i. Dermatitis: Characteristically, there is erythema re-
erythropoiesis.
sembling severe sunburn, appearing symmetrically
l Vitamin B12, but not folate, is needed for the integrity of
over the parts of the body exposed to sunlight, par-
myelin. In severe deficiency, there is insidious, diffuse
ticularly the limbs and especially on the neck, but not
and uneven demyelination. It may be clinically mani-
the face. The skin lesions may progress to vesicula-
fested as peripheral neuropathy or spinal cord degenera-
tion, cracking, exudation and secondary infection.
tion affecting both posterior and lateral columns or there
ii. Diarrhoea: This is often associated with anorexia,
may be cerebral manifestations or optic atrophy.
nausea, glossitis and dysphagia, reflecting the pres-
l Treatment with hydroxy cobalamine should produce
ence of a non-infective inflammation that extends
improvement but this may be slow.
throughout the gastrointestinal tract.
iii. Dementia: In severe deficiency, dementia develops Q. 10. Give the clinical features of vitamin D deficiency.
in chronic cases. Give the steps to prevent the same.
Ans.
Treatment
l Treatment is with nicotinamide, given in a dose of VITAMIN-D DEFICIENCY RICKETS
100 mg 8-hourly by mouth or by the parenteral route.
l It occurs generally in growing children.
l The response is usually rapid. Within 24 hours, the ery-
thema diminishes, the diarrhoea ceases and a striking
improvement occurs in the patient’s mental state. Clinical Features
Q. 9. Vitamin B12. l In first six months of life, tetany and convulsions are
common, these above manifestations are due to hypo-
Ans.
calcaemia.
l The average daily diet contains 5-30 mg of vitamin B12, l The wrist and ankles are swollen.
mainly in meat, fish, eggs and milk. l The changes in bone are found in epiphyseal plates,
l In the stomach, gastric enzymes release vitamin B12 metaphysis and shaft.
from food and at gastric pH it binds to a carrier protein l Localized area of thinning are sometime present in skull
termed R protein. so that a finger can produce indentation. This condition
l The gastric parietal cells produce intrinsic factor, a vita- is called as craniotabes.
min B12-binding protein which optimally binds vitamin l Pigeon breast
B12 at pH 8. l Developmental abnormalities of dentin and delayed
l As gastric emptying occurs, pancreatic secretion raises eruption.
the pH and vitamin B12 released from the diet switches l Higher caries index.
from the R protein to intrinsic factor. l Hypoplasia of enamel is present.
l Vitamin B12 is actively transported by the enterocytes to l Pulp chamber is large.
plasma, where it binds to transcobalamin II, a transport l Malocclusion of teeth.
176 Quick Review Series for BDS 3rd Year

VITAMIN-D DEFICIENCY OSTEOMALACIA Clinical Features


l It is also known as adult rickets. Only flat bones and l When thiamine is deficient cells cannot metabolise glu-
diaphysis of long bones are affected. cose aerobically to generate energy as ATP; this is likely
to affect the nervous system first, since it depends
largely on glucose for its energy requirements.
Clinical Features
l The body has very limited stores of thiamine, so defi-
l Pelvic deformities are seen in females. ciency starts after only 1 month on a thiamin-free diet
l Remodelling of bone occurs in absence of adequate cal- and takes several forms:
cium resulting in softening and distortion of skeleton. i. Infantile beriberi is seen in exclusively breastfed
l Bone pain and muscle weakness is present. infants of thiamine-deficient mothers, and is invari-
l Severe periodontitis is present. ably fatal.
ii. Dry (or neurological) beriberi manifests with
chronic peripheral neuropathy, and wrist and/or foot
Management of Rickets and Osteomalacia
drop, and may cause Korsakoff’s psychosis and
l Dietary enrichment of vitamin D in form of milk. Wernicke’s encephalopathy.
l If tetany is present give IV calcium gluconate. Daily iii. Wet (or cardiac) beriberi causes generalised oedema
dose is 1000-2000 IU of vitamin D combined with 500- due to biventricular heart failure with pulmonary
1000 mg of calcium. congestion.
l Curative treatment includes 2000 to 4000 IU of calcium
daily for 6 to 12 weeks followed by daily maintenance
Treatment
dose of 2000 to 4000 IU for long period.
l Patients with osteomalacia require large dose of vitamin D l In dry beriberi, response to thiamine is not uniformly
and calcium, i.e. 40, 000 to 1, 00, 000 IU of vitamin D good, however, multivitamin therapy seems to produce
and 15 to 20 gm of calcium lactate per day. some improvement, suggesting that other vitamin defi-
ciencies may be involved.
Q. 11. Describe the aetiology, clinical features, diagnosis
l Wet beriberi with cardiac involvement requires digoxin
and treatment of thiamine deficiency.
and diuretics, if congestive heart failure is present.
Ans. l Wernicke’s encephalopathy and wet beriberi should be
treated without delay with IV Pabrinex.
l Thiamine (vitamin B1) is widely distributed in foods of
l Korsakoff’s psychosis is irreversible and does not
both vegetable and animal origin, although cereals are
respond to thiamine treatment.
the main sources.
l Vitamin B1 (50-100 mg daily) IM or IV is given. As
l Thiamine pyrophosphate (TPP) is involved in carbohy-
acute crisis is over, patient has given small dose of 5 mg
drate metabolism. This is the bridge between glycolysis
daily along with nourishing diet.
and the tricarboxylic acid (Krebs) cycle. TPP is also the
l Diet should consists of high B complex and protein
co-enzyme for transketolase in the hexose monophos-
content like eggs, milk, nuts and green vegetables.
phate shunt pathway and for decarboxylation of keto-
glutarate to succinate in the Krebs cycle. Q. 12. Riboflavin deficiency.
Ans.
Aetiology l Riboflavin (vitamin B2) is a part of the oxidation chain
l In the developed world, thiamine deficiency is mainly in the mitochondria, acting as a co-enzyme in oxidation
encountered in chronic alcoholics. Poor diet, impaired reduction reactions.
absorption, storage and phosphorylation of thiamine l It is widely distributed in animal and vegetable foods, the
in the liver, and the increased requirements for richest supply coming from milk and its non-fat products.
thiamine. l Levels of the vitamin are low in staple cereals but
l In the developing world, deficiency usually arises as a germination increases its content.
consequence of a diet based on polished rice. l It is destroyed under alkaline conditions by heat and by
l The deficiency of vitamin B1 occurs due to: exposure to UV light.
a. Severe malnutrition l Clinical deficiency is rare in developed countries.
b. Chronic alcoholism It mainly affects the tongue and lips and manifests as
c. Chronic debilitating diseases. glossitis, angular stomatitis and cheilosis.
Section | I General Medicine 177

l The genitals may be affected, as well as the skin areas l Fats


rich in sebaceous glands, causing nasolabial or facial l Proteins
dyssebaceous. l Vitamins and
l Rapid recovery occurs with administration of 10 mg l Inorganic micronutrients like electrolytes, minerals and
daily by mouth. trace elements
l There is accumulation of pyruvic and lactic acids, which l For proper growth, maintenance and development of

produce vasodilatation and increased cardiac output. our body an ideal combination of essential nutrient,
vitamins and minerals should be aimed.
Q. 13. Balanced diet. l A balanced diet not only looks after the wellbeing of

Ans: person but may have role to play in prevention of


certain diseases.
l Balanced diet is a diet which contains different constitu- l An average Indian diet is poor in quality as well as
ents of food like proteins, fats, carbohydrates, vitamins quantity in meeting the daily needs.
and minerals in such quantities and proportions, that need l A balanced diet should contain 30 cal/kg optimum
for energy is adequately met for maintaining health. body weight.
l The balanced diet should contain 60-70% carbohy-
Or
drates, 12 to 18% proteins, 20 to 25% fats of total
l A diet adequate in energy providing substances, i.e. calories.
carbohydrates and fats, tissue building compounds, i.e. l Diet should contain as much as of fresh fruits, vege-
proteins, inorganic chemicals, i.e. water and mineral tables which provide nutrients. Fiber is an important
salts, agents that regulate or catalyze metabolic process, part of the food.
i.e. vitamins and substances for certain physiological l Milk is an ideal food and contains or proximate the
processes such as bulk for promoting peristaltic move- principles of balanced diet.
ment of digestive tract is known as balanced diet. l So while planning a balanced diet, care should be

taken that all essential ingredient in diet are ade-


quately incorporated with fresh fruits, vegetables and
Classification of Nutrients fibre diet.
l Water l Food should be palatable, properly cooked and well

l Carbohydrates balanced providing a balance between food intake


l Dietary fibre and energy output.

SHORT NOTES
Q. 1. Scurvy. iv. Infantile scurvy:
l Scorbic child usually assumes a frog-like position
Ans: and this may reflect as subperiosteal haemorrhage in
i. Scurvy is caused due to deficiency of vitamin C, which to shafts of long bone.
results in defective collagen formation in connective l Scorbutic rosary, i.e. enlargement of costochondral

tissue. joints.
ii. It is of two types: l Lassitude and anorexia

a. Adult scurvy l Painful limbs giving rise to pseudoparalysis.

b. Infantile scurvy v. Management


iii. In adult scurvy: l Consumption of citrus fruits and vegetables

l Swollen spongy gums—scurvy buds l Vitamin C 500 mg daily initially.

l Scorbutic gingivitis
Q. 2. Riboflavin deficiency.
l Perifollicular haemorrhages

l Petechial haemorrhages, ecchymoses, epistaxis and


Ans.
GI bleeding.
l Nailbeds—splinter haemorrhages. l Riboflavin (vitamin B2) is part of the oxidation chain in
l Haemorrhages into muscles and joints the mitochondria, acting as a co-enzyme in oxidation
l Poor wound healing. reduction reactions.
178 Quick Review Series for BDS 3rd Year

l It is widely distributed in animal and vegetable foods, the l Developmental abnormalities of dentin, hypoplasia of
richest supply coming from milk and its non-fat products. enamel and delayed eruption.
l Levels of the vitamin are low in staple cereals but l Higher caries index.
germination increases its content. l Malocclusion of teeth is present.
l Clinical deficiency is rare in developed countries.
It mainly affects the tongue and lips and manifests as Treatment
glossitis, angular stomatitis and cheilosis.
l The genitals may be affected, as well as the skin areas rich in l Dietary enrichment of vitamin D in form of milk.
sebaceous glands, causing nasolabial or facial dyssebaceous. l If tetany is present, give IV calcium gluconate. Daily
l Rapid recovery occurs with 10 mg daily by mouth. dose is 1000-2000 IU of vitamin D combined with 500-
1000 mg of calcium.
Q. 3 Vitamin A deficiency. l Curative treatment includes 2000 to 4000 IU of calcium
Ans. daily for 6 to 12 weeks followed by daily maintenance
dose of 2000 to 4000 IU for long period.
l Deficiency of vitamin A causes interference with
growth, reduced resistance to infections and interfer- Q. 5. Niacin.
ence with nutrition of cornea, conjunctiva, trachea, hair Ans.
follicle and renal pelvis.
l Children with vitamin A deficiency will experience im- Niacin or vitamin B3 refers to nicotinic acid, nicotinamide
paired growth and development. and their derivatives.
l Earliest sign of deficiency of vitamin is difficulty in reading
or sewing at night or finding anything in darkness. Sources
l Conjunctiva becomes dry and small grayish white raised
spots (Bitot’s spots) appear. Important sources of niacin are milk, eggs, beans, and meat.
l Cornea subsequently becomes dry and lusterless and if
there is lack of treatment the changes are irreversible. Functions
l Keratomalacia involving the cornea leading to the ulcer-
l It serves as a precursor of nicotinamide adenine
ation and blindness may result.
dinucleotide (NAD) and nicotinamide adenine dinu-
l Skin becomes dry and rough.
cleotide phosphate (NADP) which are involved in
l Imperfect enamel formation of teeth.
many oxidation-reduction reactions.
l Prevention of vitamin A deficiency is by giving good
nutrition, intake of fresh leafy green vegetables and
addition of vitamin A to food stuffs. Deficiency
l Vitamin A may be administered orally as retinol 30 mg l The deficiency of niacin leads to pellagra.
daily for three days. l Pellagra is characterized by dermatitis, diarrhoea and
Q. 4. Rickets. dementia.
l There are dark, dry and scaly lesions in sun exposed
Or areas such as back of the hands, lower legs, face and
Treatment of rickets. neck (Casal’s necklace).

Ans.
Diagnosis
Vitamin-D deficiency rickets occurs generally in growing
children. Pellagra is generally diagnosed clinically.

Treatment
Clinical Features
Oral nicotinamide or nicotinic acid (10 to150 mg daily)
l In first six months of life, tetany and convulsions are
effectively treats pellagra.
common, these manifestations are due to hypocalcaemia.
l The wrist and ankles are swollen and the changes in bone Q. 6. Vitamin D.
are found in epiphyseal plates, metaphysis and shaft.
Ans.
l Localized area of thinning are sometime present in skull
so that a finger can produce indentation. This condition i. Vitamin D exists in two forms—ergocalciferol (vitamin
is called as craniotabes. D2) and cholecalciferol (vitamin D3). Ergocalciferol is
l Pigeon breast. derived from plants and cholecalciferol is synthesized
Section | I General Medicine 179

in the skin from 7-dehydrocholesterol under the influ- l Severe deficiency results in the peripheral neuropathy,
ence of ultraviolet rays present in sunlight or is syn- depression and irritability.
thetically produced. l Microcytic hypochromic anaemia and seizures may

ii. Vitamin D is stored largely in fat depots. Cholecalcif- also occur.


erol is converted to active forms by hydroxylation in the v. Treatment:
liver and subsequently in kidneys. l Vitamin B6 is given orally in a dosage of 10-20 mg/ day.

iii. The main function of vitamin D is to increase absorp- l In cases of medication related vitamin deficiency,

tion of calcium and phosphate from the intestine. It also higher doses (100-200 mg/day) are needed.
increases the renal tubular resorption of phosphate.
Q. 9. Avitaminosis-A.
iv. Vitamin D promotes bone mineralization. It may also
have other systemic effects since vitamin D receptors Ans.
are found in many body tissues.
v. Vitamin D is actually a hormone and if sufficient expo- i. Vitamin A deficiency is one of the most common vita-
sure of sunlight is given to the skin, it is not required in min deficiency syndromes and is a common cause of
the diet. blindness.
ii. Night blindness, Bitot’s spots, corneal ulceration, and
Q. 7. Vitamin-K. blindness.
Ans. iii. Extraocular manifestations include hyperkeratotic skin
lesions, anorexia and growth retardation.
l Adequate amounts of vitamin-K are normally supplied in
the average diet or synthesised by bacteria in the colon. Q. 10. Hyperavitaminosis-A.
l Vitamin-K is a co-factor in the production of an unusual Ans.
amino acid, g-carboxyglutamate.
l Vitamin-K deficiency leads to prolonged coagulation i. Acute toxicity has been reported after excess consump-
and bleeding. tion or intake of more than 150 mg vitamin A.
l Important roles of vitamin-K: ii. Clinical presentation of acute toxicity includes increased
i. In the newborn, primary deficiency can occur be- intracranial pressure, vertigo, diplopia, seizures, and
cause placental transfer of vitamin-K is inefficient, exfoliative dermatitis.
the neonatal bowel has not yet acquired bacteria and iii. Chronic toxicity (ingestion of 15 mg/day for several
breast milk contains little of the vitamin. Vitamin-K months) manifests as dry skin, cheilosis, glossitis,
is given routinely to newborn babies to prevent alopecia, bone pain, hypercalcaemia and increased
haemorrhagic disease of the newborn. intracranial pressure.
ii. In obstructive jaundice, dietary vitamin-K is not iv. High dose of carotenoids may cause yellowing of skin
absorbed and it is very important to administer the but not the sclera.
vitamin in parenteral form before surgery. Q. 11. Cyanocobalamine deficiency.
iii. Warfarin and related anticoagulants act by antago-
nizing vitamin-K. Or
Q. 8. Vitamin B6. Vitamin B12
Ans. Or
i. Vitamin B6 (pyridoxine) refers to a group of closely Peripheral blood smear in vitamin Bl2 deficiency.
associated substances including pyridoxine, pyridoxal,
Ans.
pyridoxamine and their 5-phosphate esters.
ii. Functions: l Vitamin B12 is also known as wonder vitamin or cyano-
l Acts as a cofactor for a number of enzymes involved cobalamin.
in amino acid metabolism. l It is synthesized by microorganisms and is found in
l It is involved in the metabolism of fat, carbohydrates animal sources such as fish, meat, eggs, yeast and milk.
and several vitamins. l Daily requirement of vitamin B12 are 1 to 2 microgram.
l It is also involved in haem synthesis. l The deficiency of vitamin B12 leads to peripheral neuropa-
iii. Sources: Milk, liver, meat, legumes, whole grain cereals, thy, dementia, optic atrophy, anorexia, pallor, diarrhoea, red
nuts and vegetables are good sources of B6. sour tongue, angular stomatitis and megaloblastic anaemia.
iv. Deficiency: l Vitamin B12 deficiency especially in those with megalo-
l Glossitis, cheilosis, weakness are usual features of blastic anaemia is treated by giving vitamin B12 1000
the vitamin B6 deficiency. micrograms IM twice a week for 3 to 4 weeks.
180 Quick Review Series for BDS 3rd Year

l Oral supplements 1 to 2 microgram are continued as Q. 14. Fat-soluble vitamins—name four.


maintenance therapy.
Ans.
Q. 12. Vitamin-E.
i. Vitamins are organic compounds required by the body
Ans. for variety of essential metabolic functions.
ii. They are grouped as water-soluble and fat-soluble vitamins.
i. The most important dietary from of vitamin E is
iii. Fat-soluble vitamins are vitamin A, D, E and K. Water-
tocopherol.
soluble vitamins are thiamine (B1), riboflavin (B2),
ii. Rich sources include vegetable oils, whole-grain
niacin (B3), pyridoxine (B6), cobalamin (B12), folate,
cereals and nuts.
pantothenic acid, biotin and L-ascorbic acid (vitamin C).
iii. Functions of vitamin E:
l It has many direct metabolic actions. Q. 15. Beriberi—name any two causes.
l It is an important antioxidant.
Ans.
l It plays a role in maintaining cell membrane

structure. l Beriberi is caused due to the deficiency of vitamin B1,


l It affects DNA synthesis and cell signaling. i.e. thiamine.
l It is involved in the anti-inflammatory and immune l The deficiency of vitamin B1 occurs in severe malnutri-
system. tion, chronic alcoholics and those suffering from chronic
iv. Vitamin E deficiency leads to myopathies, neuropathies debilitating diseases.
and liver necrosis. l Treatment consists of administration of vitamin B1
v. Human deficiency states are rare and have only been (50-100 mg daily) IM or IV is given.
described in premature infants and in malabsorption. l As acute crisis is over, patient has given small dose of
vi. The first feature of human deficiency is a mild haemo- 5 mg along with nourishing diet.
lytic anaemia. In chronic fat malabsorption, ataxia and l Diet consists of high B complex and protein content like
visual scotomas occur which respond to vitamin E. eggs, milk, nuts and green vegetables.
l Wet beriberi with cardiac involvement require digoxin
Q. 13. Pellagra.
and diuretics are to be added, if congestive heart failure
Ans. is present.
i. Pellagra is the clinical condition produced due to the Q. 16. Bitot’s spots.
deficiency of nicotinic acid or niacin. l Bitot’s spots may appear as glistening white plaques of
ii. It is caused due to: desquamated thickened conjunctival epithelium, usually
l Inadequate intake or absorption of niacin. triangular in shape and firmly adherent to the underlying
l Restricted or limited diet in which single serial conjunctivae.
grain, i.e. corn is consumed without consumption of l The final consequence of vitamin A deficiency is the
niacin rich food. development of keratomalacia, leading to corneal ulcer-
l Chronic alcoholism ation, scarring and irreversible blindness.
iii. Pellagra has been called the disease of the three Ds: l Vitamin A deficiency can be prevented by giving good
Dermatitis, diarrhoea and dementia. nutrition, intake of fresh leafy green vegetables and
iv. Treatment is with nicotinamide, given in a dose of addition of vitamin A to food stuffs.
100 mg 8-hourly by mouth or by the parenteral route. l Vitamin A may be administered orally as retinol 30 mg
v. The response is usually rapid, within 24 hours, the ery- daily for three days.
thema diminishes, the diarrhoea ceases and a striking l In advanced cases where absorption is affected, vitamin A
improvement occurs in the patient’s mental state. is given in the dose of 50, 000 IU parenterally for 3 days.
Section | I General Medicine 181

Topic 12

Infectious Diseases
LONG ESSAYS
Q. 1. What is the differential diagnosis of ulcers over the Tertiary syphilis
penis? Describe clinical features of syphilis and how do
l Gumma: Involving the skin, mucous membranes and
you treat syphilis?
bones. It is the characteristic feature.
Ans. l CVS: Aortitis, aortic aneurysms, aortic regurgitation,
coronary ostial stenosis, gummatous myocarditis.
The differential diagnoses of ulcers over penis include
l Neurosyphilis: Asymptomatic, meningeal, meningovas-
following lesions:
cular or cerebrovascular syphilis, meningomyelitis,
l Chancre
acute transverse myelitis, tabes dorsalis, general paresis
l Chanchroid (painful nonindurated ulcer)
and gummatous neurosyphilis.
l Lymphogranuloma venereum
Genital herpes and
l
Congenital syphilis
l Neoplasm.
l Preterm delivery, stillbirth, congenital
l Infection or neonatal death.
SYPHILIS l Early: Snuffles, failure to thrive, condyloma lata, osteo-
chondritis, periostitis.
Aetiopathogenesis of Syphilis
l Late: Interstitial keratitis with neovascularization, neu-
l Treponema pallidum. rosyphilis, sensorineural deafness, gummas and bony
l Infection acquired by sexual contact or transplacentally. involvement.
l Chronic systemic infection with periods of latency. l Stigmata: Rhagades, mulberry molars, peg-shaped
Hutchinson teeth, frontal bossing, saddle nose and saber
tibia.
Clinical Features
l Hutchinson’s triad: Hutchinson’s teeth, interstitial keratitis
Primary syphilis and eight nerve deafness.
l In primary syphilis, the typical lesion is chancre, which
develops at the site of inoculation, usually penis in het- Diagnosis
erosexual males.
l Single, rounded, reddish, indurated, painless lesion with l Dark field microscopy
a well-defined edge. l VDRL test
l Heals without treatment in 2 to 6 weeks. l Treponema pallidum haemagglutination assay (TPHA)

Secondary syphilis
Treatment
l This stage develops 6-8 weeks after primary infection.
l Macular, papular, or pustular skin lesions. l Parenteral penicillin is the drug of choice.
l Condyloma lata in genital and anal regions. l Primary, secondary and latent syphilis: Benzathine
l Mucous patches. penicillin 24 lakh units IM stat.
l Alopecia, iritis, hearing loss, arthritis, nephritis, hepatitis, l Tertiary syphilis: Benzathine penicillin 24 lakh units IM
meningitis. once every week for three weeks.
l These are highly infectious. l Alternatives are doxycycline, erythromycin, azithromycin
and ceftriaxone.
Latent syphilis
Q. 2. Describe the clinical features, investigations and
l Latency may remain indefinitely, be interrupted by a treatment of typhoid fever.
relapse of secondary syphilis or progress to tertiary
stage. Or
182 Quick Review Series for BDS 3rd Year

What are the clinical features, diagnosis, complications Laboratory Diagnosis


and management of typhoid fever.
l 1st week: Blood culture is positive for S. typhi and
Or S. paratyphi.
l Culture of bone marrow may be positive when bood
Describe the aetiology, clinical features, diagnosis, com- cultures are negative.
plications and treatment of typhoid fever. l 2nd week: Widal test
l 3rd week: Stool and urine culture will be positive.
Or

Discuss the aetiology, clinical features, complications Complications


and management of enteric fever. l Intestinal: Haemorrhage, paralytic ileus, perforation,
Ans. peritonitis.
l Extraintestinal: Meningitis, cholecystitis, pneumonia,
Typhoid fever or enteric fever is an acute systemic illness myocarditis, bone and joint infection, encephalopathy,
caused due to infection by Salmonella typhi and S. paratyphi. granulomatous hepatitis, nephritis.

Epidemiology Treatment
l Reservoir: Acutely ill or convalescing patients, chronic i. Administration of antibiotics parenterally or orally de-
carriers. pending on patients condition.
l Transmitted by faeco-oral route. ii. Effective antibiotics are fluoroquinolones, third genera-
l Usually seen in children and young adults. tion cephalosporins (ceftriaxone, cefotaxime, cefixime,
and cefodoxime) and azithromycin.
iii. The most commonly used drugs are as follows:
Aetiopathogenesis l Ciprofloxacin 500 mg bid for 10 days

l Invasion of Peyer’s patches and initial transient asymp- l Ofloxacin 400 mg bid for 7 days

tomatic bacteraemia. l Ceftriaxone 1 g IV bid for 10 days

l Phagocytosis by reticuloendothelial system and persis- l Azithromycin 1 g daily for 5 days

tent symptomatic bacteraemia. iv. Dexamethasone is indicated in severe cases as a single


l The typical lesion occurs in Peyer’s patch leading to dose of 3 mg /kg followed by 8 doses of 1 mg/kg every
hyperplasia and necrosis of Peyer’s patches, followed 6th hourly.
by ulceration. v. Carriers are treated with oral antibiotics (amoxycillin,
l Systemic manifestations due to toxaemia and septi- ciprofloxacin, TMP-SMX) for 6 weeks.
caemia. Q. 3. Describe aetiology, clinical features and manage-
ment of diphtheria.
Clinical Features Or
l Onset is insidious. Describe the aetiology, clinical features, complications
l Incubation period is 10-14 days. and treatment of diphtheria.
l Initially, in 1st week, patients present with prodromal
symptoms like - remittent fever, headache, myalgia, Ans.
malaise, constipation, leucopenia, relative bradycardia. l Diphtheria is caused by Corynebacterium diphtheriae,
Children may present with diarrhoea and vomitings. which is a gram-positive rod with a Chinese letter con-
l Fever typically rises in stepwise fashion for few days figuration.
and then attains plateau. l The organism does not invade the tissue, it produces a
l During 2nd week – rashes (rose spots), splenomegaly, powerful exotoxin which damages the heart muscle and
bronchitis, abdominal pain, abdominal distention and nervous system.
diarrhoea.
l Beyond 2nd week - confusion, delirium, complications, Aetiopathogenesis
coma and death.
l Relapse - reappearance of clinical features within l Corynebacterium diphtheriae
2 weeks of recovery. l Humans are the only reservoir.
l Chronic carrier state - seen in 5% of cases, commonly l Infection is transmitted by aerosol inhalation and direct
in females. It is due to persistence of infection in gall- contact.
bladder. l Diphtheria toxin causes tissue necrosis.
Section | I General Medicine 183

Clinical Features l Following recovery patient should be immunized with


diphtheria toxoid.
l Incubation period - one week (2-4 days).
l Supportive measures.
l The presentations may be predominantly nasal, laryn-
geal, pharyngeal or cutaneous. Among which pharyngeal
form is most common one. Prevention
l Low grade fever, pharyngeal erythema, formation of
l Contact tracing, surveillance and prophylactic antibiotics.
membrane in the posterior pharynx.
l Immunization
l Laryngeal involvement causes hoarseness, stridor and
respiratory obstruction.
l Nasal diphtheria - nasal discharge Complications
l Otitis media, conjunctivitis, keratitis, myocarditis and
Acute circulatory failure
neuropathies.
l Death may occur due to acute circulatory failure. l Toxic myocarditis
l It is characterized by the tachycardia, feeble heart
sounds, cardiac enlargement, tic-tac rhythm and ar-
Diagnosis rhythmias.
l Gram’s stain l Sudden death may occur due to congestive heart failure.
l Bacterial culture
l Definitive diagnosis is made on the basis of typical Respiratory complications
clinical presentations including “Pseudomembrane” and l Bronchitis
isolation of C. diphtheriae from the lesions. l Bronchopneumonia
l Respiratory obstruction
l Respiratory paralysis.
Treatment
l Patient is managed in isolation. Toxic neurological damage
l Prompt administration of antitoxin is an important step l Paralysis of palate
as toxin cannot be neutralized once it gets bound to the l Paralysis of accommodation
tissues. l Facial paralysis
l The dose of antitoxin is as follows: l Bulbar paralysis
Mild cases: 4, 000 to 8, 000 units IM. l Paralysis of muscles of respiration
Moderate cases: 16, 000 – 40, 000 units IM l Peripheral neuropathy.
Severe cases: up to 100000 units IV
l Antibiotics are given for 10-14 days to eliminate Renal complications
C. diphtheriae, so that spread of infection is prevented. l Toxic nephritis
The dose of amoxicillin is 500 mg 8th hourly, erythro- l Vascular involvement especially middle cerebral
mycin 500 mg 6th hourly. artery leading to a picture of thrombosis or mono-
l Clarythromycin and azithromycin are equally effective. plegia.
l An urgent tracheostomy may be needed in case of respi- l Other complications include otitis media and arthritis.
ratory obstruction.

SHORT ESSAYS
Q. 1. Oral manifestations of AIDS. Modes of Transmission
Or l Sexual contact
Acquired immune deficiency syndrome (AIDS) - six l Vertical transmission - mother to child
opportunistic infections. l Blood and blood products
l Unsterilised needles
Ans.
HIV/AIDS is a global pandemic.
Oral Manifestations of AIDS
l

l Human immunodeficiency virus, a retrovirus of the


Lentivirus sub-family causes acquired immunodefi- l Oropharyngeal lesions are common in HIV infection
ciency syndrome. and are mostly due to secondary infections.
184 Quick Review Series for BDS 3rd Year

l Oral manifestations in HIV infection are as follows: l Pharyngeal gonorrhoea may be asymptomatic or pres-
i. Fungal infections: ent as sore throat and occurs due to oral sex.
l Candidiasis l Eye involvement occurs in the form of conjunctivitis
l Histoplasmosis due to direct inoculation of gonococci. The purulent
l Cryptococcosis conjunctivitis and oedema of eyelids are found in neo-
ii. Viral infections: nates (ophthalmia neonatorum). It may lead to panoph-
l Oral hairy leukoplakia (EBV) thalmitis. Urgent treatment is required to prevent
l Herpes simplex corneal damage and loss of vision.
l Cytomegalovirus l Dissemination of gonococci through bloodstream from
l Varicella zoster primary site may cause bacteraemia and systemic infec-
l Papilloma virus tion: Intermittent fever, arthritis, tenosynovitis and skin
iii. Bacterial infections: involvement are common. Rash may be maculopapular
l Periodontal infections or haemorrhagic.
l Necrotizing ulcerative gingivitis
Necrotizing stomatitis
Differential Diagnosis
l

iv. Neoplasms:
l Kaposi sarcoma Gonorrhoea should be differentiated from non-gonococcal
l Lymphoma urethritis or vaginitis due to Chlamydia trachomatis, tricho-
v. Idiopathic oral aphthous ulcers moniasis, candidiasis, and bacterial vaginosis.
vi. HIV salivary gland disease (DILS)
vii. Cervical lymphadenopathy
Investigations
l The presence of oral candidiasis or thrush may
be an indicator of underlying HIV disease. l Smear examination of the discharge may show gram-
Thrush appears as white, cheesy exudates negative diplococci in polymorphonuclear leuco-
on erythematous mucosa in the posterior cytes.
orophayrynx. l Diagnosis is confirmed by culture.
l Palatal, glossal or gingival ulcer may also l Ligase chain reaction (LCR) assay may detect both
result from cryptococcal disease or histoplas- N. gonorrhoeae and Chlamydia trachomatis in ure-
mosis. thral or cervical swab and urine.

Q. 2. Gonorrhoea.
Treatment
Ans.
l Antimicrobial is started as soon as possible. A single
l Gonorrhoea is caused by Neisseria gonorrhoeae, a dose is curative in uncomplicated infection while
gram-negative diplococcus. complicated infection requires prolong therapy.
l It is transmitted through sexual activity. Sexually active l Common antimicrobials used are:
young population is the most commonly affected one. Cefixime 400 mg orally
l The incubation period is 2-10 days. Ceftriaxone 125 mg IM
Ciprofloxacin 500 mg orally
Clinical Features Ofloxacin 400 mg orally
Levofloxacin 250 mg orally
l In females, the infection may be asymptomatic in 50% Spectinomycin 2 g IM
cases. l Cefixime and ceftriaxone are first-line agents because
l Urethral infection may present as dysuria, discharge or of fluoroquinolone resistance.
increased frequency. Vaginitis or cervicitis may occur l Complicated gonorrhea (salpingitis, prostatitis arthritis,
and present as vaginal discharge. bacteraemia) are treated by penicillin 10 million units
l Rectum may be involved due to anal sex or spread from intravenously daily for 5 days. Ceftriaxone (1 g intrave-
urogenital infection. Uterus and fallopian tubes may be nously daily for 5 days) or oral fluoroquinolones
involved resulting into salpingitis. Chronic salpingitis (ciprofloxacin 500 mg bd daily for 5 days, levofloxacin
may lead to scarring of fallopian tubes and sterility. 500 mg daily for 5 days),
l Pelvic inflammatory disease (PID) refers to mixed in- l The preventive methods include education, and adoption
fection with gonococci chlamydia and anaerobes. It of mechanical measures like condoms.
may present as lower abdominal pain, discharge, pain l Effective drugs in therapeutic dosage, if taken within
during coitus and vaginal bleeding. 24 hours of exposure, may abort an infection.
Section | I General Medicine 185

Q. 3. Chickenpox. Clinical Features


Ans. l Orofacial and eye infections: Ulcerative gingivostoma-
titis, pharyngitis, herpes labialis, keratoconjunctivitis.
Chickenpox is an erythematous or vesiculopapular lesion l Genital infections: Vulvovaginitis, urethritis, proctitis
caused by varicella-zoster virus. l Nervous system: Encephalitis, Guillian-Barre, trans-
It spreads through aerosol route. verse myelitis.
l Visceral infection: Oesophagitis, pneumonitis, hepatitis.
Clinical Features
l More sever in adults, pregnant women and immuno- Diagnosis
compromised persons.
l Characteristic feature is multiple vesicles over an ery-
l It is more dense on trunk and sparse on limbs.
thematous base.
l The fever is associated with every new crop of leison.
l Lab confirmation by Tzanck test, tissue culture, serol-
l Intense itching infections due to secondary infections.
ogy, or PCR for HSV DNA in CSF.
l It occurs first on mucosal surface and then there is rapid
dissemination in a centripetal distribution.
Treatment
Complications l Mucocutaneous infections: Acyclovir, famcyclovir and
l Cerebral ataxia valacyclovir.
l Myocarditis l Eye infection: Idoxuridine, vidarabine, cidofovir.
l Hepatitis l HSV encephalitis: Intravenous acyclovir (10 mg/kg
l Acute glomerulonephritis 8th hrly for 10 days).
l Pneumonia Q. 5. Mumps.
l Arthritis
l Corneal lesion Or
l Bleeding diathesis.
Complications of mumps.

Management Ans.

l Acyclovir within 48 hours of rash appearances. l Mumps is caused by a paramyxovirus.


l Human varicella-zoster immunoglobulin for sever im- l Mode of spread is by droplet infection.
munocompromised patients, bone marrow recipient l Incubation period is about 15-20 days.
and HIV positive contacts.
Q. 4. Herpes simplex. Clinical Features
Ans. l Fever, headache, vomiting, anorexia and malaise.
l Parotid enlargement
There are 8 herpes viruses infecting humans. These are
l Orchitis followed by testicular atrophy.
double-stranded DNA.
l Neurological complications: Aseptic meningitis, menin-
goencephalitis, neuritis and myelitis.
Herpes Simplex l Arthritis, myocarditis, hepatitis and involvement of
other glands.
Epidemiology and pathogenesis
l Caused by two types of herpes simplex virus, HSV-l and
HSV-2. Diagnosis
l HSV-l causes cutaneous, oropharyngeal and ocular l Isolation of virus from throat swabs
lesion, whereas HSV-2 causes genital herpes. l Serology
l Primary infection is commonly asymptomatic, occur- l Elevated serum amylase levels
ring in childhood via mucosal surfaces or abraded skin.
l The virus remains dormant in sensory and autonomic
ganglia. Treatment
l Reactivation may be triggered by stress, trauma, men- l Supportive care: Prevention
struation and fever. l Attenuated live mumps vaccine at the age of 18 months.
186 Quick Review Series for BDS 3rd Year

Q. 6. Diphtheria. l An urgent tracheostomy may be needed in case of respi-


ratory obstruction.
Ans.
l Following recovery patient should be immunized with
l Diphtheria is caused by Corynebacterium diphtheriae, diphtheria toxoid.
which is a gram-positive rod with a Chinese letter con- l Supportive measures.
figuration.
The organism does not invade the tissue, it produces a
Prevention
l

powerful exotoxin which damages the heart muscle and


nervous system. l Contact tracing, surveillance and prophylactic antibiotics.
l Immunization
Aetiopathogenesis
l Corynebacterium diphtheriae Complications
l Humans are the only reservoir.
l Infection is transmitted by aerosol inhalation and direct Acute circulatory failure
contact. l Toxic myocarditis
l Diphtheria toxin causes tissue necrosis. l It is characterized by the tachycardia, feeble heart sounds,
cardiac enlargement, tic-tac rhythm and arrhythmias.
Sudden death may occur due to congestive heart failure.
Clinical Features l

l Incubation period - one week (2-4 days). Respiratory complications


l The presentations may be predominantly nasal, laryn- l Bronchitis
geal, pharyngeal or cutaneous. Among which pharyn- l Bronchopneumonia
geal form is most common one. l Respiratory obstruction
l Low grade fever, pharyngeal erythema, formation of l Respiratory paralysis.
membrane in the posterior pharynx.
l Laryngeal involvement causes hoarseness, stridor and Toxic neurological damage
respiratory obstruction. l Paralysis of palate
l Nasal diphtheria: Nasal discharge. l Paralysis of accommodation
l Otitis media, conjunctivitis, keratitis, myocarditis and l Facial paralysis
neuropathies. l Bulbar paralysis
l Death may occur due to acute circulatory failure. l Paralysis of muscles of respiration
l Peripheral neuropathy.
Diagnosis
Renal complications
l Gram’s stain
l Toxic nephritis
l Bacterial culture
l Vascular involvement especially middle cerebral artery
l Definitive diagnosis is made on the basis of typical
leading to a picture of thrombosis or monoplegia.
clinical presentations including “Pseudomembrane” and
l Other complications include otitis media and arthritis.
isolation of C. diphtheriae from the lesions.
Q. 7. Enteric fever.
Treatment Or
l Patient is managed in isolation.
l Prompt administration of antitoxin is an important step Enumerate complications of enteric fever.
as toxin cannot be neutralized once it gets bound to the Ans.
tissues.
l The dose of antitoxin is as follows: Enteric fever is an acute systemic illness caused due to
Mild cases: 4, 000 to 8, 000 units IM. infection by Salmonella typhi and S. paratyphi.
Moderate cases: 16, 000 – 40, 000 units IM
Severe cases: up to 100000 units IV
Clinical Features
l Antibiotics are given for 10-14 days to eliminate
C. diphtheriae, so that spread of infection is prevented. l Onset is insidious.
l The dose of amoxicillin is 500 mg 8th hourly, erythro- l Incubation period is 10-14 days.
mycin 500 mg 6th hourly. Clarythromycin and azithro- l Initially in 1st week patients present with prodromal
mycin are equally effective. symptoms like - remittent fever, headache, myalgia,
Section | I General Medicine 187

malaise, constipation, leucopenia, relative bradycardia. Clinical Features


Children may present with diarrhoea and vomitings.
l Malaise, fatigue, headache, fever, chills, rigors, spleno-
l Fever typically rises in stepwise fashion for few days
megaly.
and then attains plateau.
l Relapses seen in case of vivax malaria.
l During 2nd week: Rashes (rose spots), splenomegaly,
l Tropical splenomegaly syndrome.
bronchitis, abdominal pain, abdominal distention and
l Complications (seen in falciparum infections): Cerebral
diarrhoea.
malaria, severe anaemia, acute renal failure, ARDS,
l Beyond 2nd week: Confusion, delirium, complications,
hypoglycaemia, shock, DIC, seizures.
coma and death.
l Relapse: Reappearance of clinical features within
2 weeks of recovery. Diagnosis
l Chronic carrier state: Seen in 5% of cases, commonly in
l Peripheral smear with thick and thin smears.
females. It is due to persistence of infection in gall-
l Quantitative buffy coat analysis.
bladder.

Diagnosis Treatment
l Blood culture, culture of bone marrow, stool and urine l P. vivax, malariae, ovale and uncomplicated falciparum
culture. malaria: Oral chloroquine 600 mg, followed by 300 mg
l Widal test. after 6, 24 and 48 hours.
l Complicated falciparum malaria: Intravenous quinine
10 mg/kg over 4-8 hours thrice daily for 7 days, artesunate
Treatment 2 mg/kg stat and 1 mg/kg daily for 4 days.
i. Administration of antibiotics parenterally or orally de- Q. 9. Amoebic dysentery.
pending on patients condition.
ii. Effective antibiotics are fluoroquinolones, third genera- Ans.
tion cephalosporins (ceftriaxone, cefot xime, cefixime, Dysentery is an acute inflammation of the large intestine
and cefodoxime) and azithromycin. characterized by diarrhoea with blood and mucus in the stools.
iii. The most commonly used drugs are as follows: It is usually caused by bacillary and amoebic infections.
l Ciprofloxacin 500 mg bid for 10 days

l Ofloxacin 400 mg bid for 7 days

l Ceftriaxone 1 g IV bid for 10 days Pathogenesis


l Azithromycin 1 g daily for 5 days
l It results from the infection of the large intestine by
iv. Dexamethasone is indicated in severe cases as a single Entamoeba histolytica.
dose of 3 mg /kg followed by 8 doses of 1 mg/kg every l After ingestion, the cyst undergoes further nuclear
6th hourly. division and trophozoites are released. The trophozoites
v. Carriers are treated with oral antibiotics (amoxycillin, are carried to the large intestine where they produce the
ciprofloxacin, TMP-SMX) for 6 weeks. characteristic flask-shaped amoebic ulcerations.
l Incubation period is 2-6 weeks.
Complications l A localized granuloma, presenting as a palpable mass in
the rectum or causing a filling defect in the colon on
l General – toxaemia, dehydration, peripheral circulatory
radiography, is a common complication.
failure, DIC.
l Amoebae may enter a portal venous radical and be
l Gastrointestinal – perforation of intestine, intestinal
carried to the liver where they multiply and produce
haemorrage.
an amoebic liver abscess.
l Neurological – delirium, psychosis, coma, catotonia,
meningitis, encephalopathy, peripheral neuritis, deafness.
l Miscellaneous – myocarditis, endocarditis, pericarditis, Clinical Features
pyelonephritis, glomerulonephritis, osteomyelitis, arthri-
l In amoebic dysentery, there is intermittent diarrhoea
tis, periostitis, pneumonia, hepatitis, thrombophlebitis.
consisting of one to four, foul smelling, loose, watery
Q. 8. Malaria. stools which may contain mucus and blood.
l Frequent flatulence and abdominal cramping.
Ans.
l Tender hepatomegaly and tenderness over the caecum,
Malaria is a protozoan disease transmitted by the bite of ascending colon and over left iliac fossa (Manson-Barr
Anopheles mosquitoes. point).
188 Quick Review Series for BDS 3rd Year

Complications l Sigmoidoscopy: This may reveal the characteristic


flask-shaped ulcers with normal surrounding mucosa,
l Massive haemorrhage
the aspirated material, or scrapings from the ulcer, or
l Perforation and peritonitis
biopsy of the ulcer may demonstrate the trophozoites.
l Toxic megacolon in fulminant cases
l Amoebic serology: This can detect antibodies.
l Post-dysenteric colitis
l Amoebic liver abscess
l Pleuropulmonary amoebiasis Treatment
l Amoebic pericarditis
l Invasive intestinal amoebiasis is treated with metronida-
l Cutaneous amoebiasis
zole 800 mg thrice daily for 5 days or tinidazole 2 g
daily as a single dose for three days.
Diagnosis l After treatment, furamide should be given at a dose of
500 mg thrice daily for 10 days to eliminate luminal
l Stool examination: microscopic examination of a fresh
cysts, alternative agents include iodoquinol and paro-
sample of stool may demonstrate the motile trophozo-
momycin.
ites. They are about 30 microns in diameter, with a clear
ectoplasm and a granular endoplasm, and usually con-
tains red blood cells,

SHORT NOTES
Q. 1. Rubeola. l A combination vaccine (MMR) can be given at the
age 12-15 months and the booster dose at the age of
Or
4-12 years.
Measles.
Q. 2. Mumps.
Ans.
Ans.
l Measles or rubeola is an acute paramyxoviral infection.
Mumps is a systemic viral infection.
l Transmitted by direct contact with infectious droplets
and by inhalation.
Aetiopathogenesis
Clinical Features l It is caused by paramyxovirus.
l Infection is acquired by the respiratory route.
l Fever, cough, coryza and conjunctivitis. l Single attack gives lifelong immunity.
l Erythematous maculopapular rash, appears first on fore-
head and behind ears, subsequently spreads to trunk and
limbs. Fades in 3-4 days, leaving behind brownish staining. Clinical Features
l Koplik’s spots seen on buccal mucosa. l Fever, headache, vomiting, anorexia and malaise.
l Subacute sclerosing panencephalitis (SSPE) is due to l Parotid enlargement
persistence of the virus and occurs years after the initial l Orchitis followed by testicular atrophy.
infection. l Neurological complications: Aseptic meningitis, menin-
l Complications are otitis media, pneumonia and en- goencephalitis, neuritis and myelitis.
cephalitis. l Arthritis, myocarditis, hepatitis and involvement of
other glands.
Treatment
l Symptomatic and supportive therapy. Diagnosis
l Isolation of patient for one week and advice bedrest. l Isolation of virus from throat swabs
l Parenteral vitamin A administration. l Serology
l Elevated serum amylase levels
Prevention
Treatment
l Live attenuated measles vaccine should be given at the
age of 9 months to all children. l Symptomatic and supportive care
Section | I General Medicine 189

l Prevention is by live-attenuated mumps vaccine. A iv. Low grade fever, pharyngeal erythema, formation of
combined measels, mumps and rubella (MMR) vaccine membrane in the posterior pharynx. Death may occur
is given during 12-15 months of age. due to acute circulatory failure.
v. Definitive diagnosis is made on the basis of typical
Q. 3. German measles.
clinical presentations including “Pseudomembrane”
Ans. and isolation of C. diphtheriae from the lesions.
vi. Treatment consists of isolation of the patient and
i. Rubella or German measles is a systemic viral infec-
prompt administration of antitoxin as toxin cannot be
tion caused by togavirus.
neutralized once it gets bound to the tissues.
ii. The main presenting features are fever, lymphadenopa-
vii. Antibiotics are given for 10-14 days to eliminate
thy and rash. The fever is mild and occurs on the first
C. diphtheriae, so that spread of infection is prevented.
day of the rash.
viii. It can be prevented by contact tracing, surveillance
iii. Rash is maculopapular and nonconfluent which begins
and prophylactic antibiotics and immunization.
on the face and then spreads downwards. Petechial
rash may appear on the soft palate (Forschheimer’s Q. 6. Ascariasis.
spots).
iv. The most devastating effect of rubella is on the fetus, if Ans.
infection occurs in pregnant women during first trimes- l Ascariasis is caused by Ascaris lumbricoides.
ter and early second trimester. Spontaneous abortion l Ascaris bronchopneumonia is characterized by fever,
may occur. cough, dyspnoea, wheeze, eosinophilic leucocytosis and
v. The diagnosis is made by the demonstration of virus or migratory pulmonary infiltrates may occur during the
specific antibodies. The presence of IgM antibody or stage of larval migration through the lungs.
four-fold rise in IgG antibodies in paired acute and l Heavy infections may be associated with abdominal
convalescent sera is diagnostic. pain and malabsorption of fat, protein, carbohydrate and
vi. There is no specific therapy. The fever and joint pain vitamins.
are managed symptomatically. l Adult worms migrate into appendix, bile ducts, or pan-
vii. It can be prevented by the rubella vaccine. MMR is creatic ducts, causing appendicitis, cholangitis, liver
given to children at 12-15 months of age. A booster or abscess or pancreatitis.
second dose is given at 4 years of age. l Diagnosis is by demonstrating typical eggs in the stools.
l Pyrantelpamoate 11mg/kg given orally is effective in
Q. 4. Prevention of measles.
removing the adult worms from the intestine.
Ans. l Mebendazole 100 mg twice daily for three days or a
single dose of 400 mg is also effective.
Prevention of measles includes:
l Administration of live-attenuated measles vaccine at the Q. 7. Hookworm diosease.
age of 9 months to all children.
l However, combination vaccine (MMR) can be given Or
at the age 12-15 months. MMR booster is given at 4 to
Ankylostomiasis.
12 years of age. This provides life long protection.
l Post-exposure prophylaxis is indicated for imunocom- Ans.
promised persons within 6 days of contact.
l Hookworm disease is a symptomatic infection caused
l A dose of immune globulin 0.25 ml/kg is given in
by Ankylostoma duodenale or Nectar americanus.
adults.
l Vaccine and immune globulin should not be given con- l It is one of the main causes of anaemia in the tropics.
l Itching dermatitis (‘ground itch dermatitis’) occurs at
currently.
the site of entry of the larvae. The lesions are maximum
Q. 5. Diphtheria. on the feet especially between the toes.
l A variety of gastrointestinal symptoms are common. The
Ans.
major clinical manifestations are those of iron deficiency
i. Diptheria is caused by Corynebacterium diphtheriae, anaemia consequent to chronic intestinal blood loss.
and infection is transmitted by aerosol inhalation and l Young children often have extreme anaemia, with car-
direct contact. diac insufficiency and anasarca. Older children may
ii. Incubation period - one week (2-4 days) show retarded physical, mental and sexual development.
iii. The presentations may be predominantly nasal, laryn- l In adults, there may be lassitude, dyspnoea, palpitation,
geal, pharyngeal or cutaneous. Among which pharyn- tachycardia, constipation and pallor of skin and mucous
geal form is most common one. membranes.
190 Quick Review Series for BDS 3rd Year

l Diagnosis is made by demonstration of characteristic l Patients are hospitalized promptly. The management
ovum in the stool by microscopic examination. includes appropriate antimalarial chemotherapy, detec-
l Haemoglobin level is usually low and the anaemia is tion and treatment of fluid, electrolyte, acid-base, abnor-
characteristically hypochromic and microcytic. malities and other complications.
l Treatment consists of improvement of nutrition with a
Q. 11. Treatment of falciparum malaria.
high protein diet and anaemia should be treated with
oral iron or blood transfusion. Ans.
l The drugs currently favoured for treatment of hook-
l Several drugs are available for the treatment of
worms are mabendazole, albendazole and pyrantel pa-
malaria.
moate.
l The gametocytes of P. falciparum are killed by prima-
Q. 8. Herpes labialis. quine.
l Primaquine is contraindicated in patients with G6PD
Ans.
deficiency.
i. Recurrent or secondary or herpetic labialis is usually l Chloroquine is the drug of choice in uncomplicated
seen in adult patients. malaria except in chloroquine resistant P. falciparum
ii. First there is a discomfort such as burning sensation, infection.
pruritis or pain at the site. l The resistance to chloroquine is usually observed in
Then there will be eruption of group of vesicles sur- P. falciparum malaria. The treatment is started with oral
rounded by area of redness. quinine sulfate plus doxycycline, tetracycline or clinda-
iii. Vesicles burst after few hours and coalace. On the lips, mycin. Alternative drugs are mefloquine, artesunate
these ruptured vesicles become covered by a brownish followed by mefloquine, and halofantrine.
crust. l Severely ill patients who cannot take orally, are given
iv. Secondary bacterial infection sets in. IV quinine plus doxycyline, tetracycline or clindamy-
v. The characteristic site is mucocutaneous junction of the cin. Oral treatment is started as soon as possible. Alter-
lips. But can occur in other intraoral areas too. native to quinine is parenteral artesunate or artemether
vi. Treatment: By antiviral drugs, e.g. aciclovir. followed preferably by mefloquine.
Q. 9. Diphtheria-two fatal complications. Q. 12. Name three malarial parasites.
Ans. Ans.
Diptheria is caused by Corynebacterium diphtheriae. l Malaria is a protozoan parasite transmitted by the bite
of Anopheles mosquitoes.
Four species of the genus Plasmodium infect humans.
Complications l

These are
l Extension of the membrane into larynx and trachea l P. falciparum

leads to laryngeal obstruction and bronchopulmonary l P. vivax


diphtheria. l P. ovale

l Myocarditis can result in arrhythmias, cardiac failure, l P. malariae

and ECG changes. It often occurs weeks after initial l P. falciparum cause the most fatal form of the diseases.
episode of diphtheria. P. vivax infection is the most common in India.
l Pheripheral neuropathy can occur in glove and stocking
Q. 13. Complications of malaria.
distribution.
l Bulbar paralysis results in III, VI, VII, IX, and X cranial Ans.
nerve palsies and diaphragmatic paralysis.
Complications of falciparum malaria are as follows:
Q. 10. Cerebral malaria. l Cerebral malaria
l Acute noncardiogenic pulmonary oedema (ARDS)
Ans.
l Acute tubular necrosis and renal failure
l Severe falciparum malaria is most dangerous form of l Hyperpyrexia
malaria and is responsible for most of the deaths. l Hypoglycaemia
l Cerebral malaria is a most serious complication of fal- l Severe anaemias
ciparum malaria. l Metabolic acidosis
l It is characterized by confusion, coma, convulsions and l Bleeding and disseminated intravascular coagulation (DIC)
neurological signs. Severe intravascuar haemolysis may l Hypotension and shock
lead to haemoglobinuria (black water fever). l Convulsions
Section | I General Medicine 191

Chronic complications of malaria are: l Relapse: Reappearance of clinical features within


l Tropical splenomegaly (hyperactive malarial spleno- 2 weeks of recovery.
megaly): Abnormal immune response to repeated at- l Chronic carrier state: Seen in 5% of cases, commonly in
tacks of malaria may result in massive splenomegaly, females.
hepatomegaly and raised titers of IgM and malarial
Q. 17. Teeth in congenital syphilis.
antibodies.
l Nephrotic syndrome: Chronic and repeated infections Ans.
with P. malariae may cause immune mediated injury to
l Syphilis is caused by Treponema pallidum.
renal glomeruli and nephrotic syndrome.
l Infection acquired by sexual contact or transplacentally.
Q. 14. Complications of mumps. l Teeth seen in congenital syphilis are: mulberry molars,
peg-shaped laterals, Hutchinson teeth.
Ans. l Hutchinson’s triad: Hutchinson’s teeth, interstitial kera-
l Mumps is caused by paramyxovirus and is spread by titis and eight nerve deafness.
dropelet infection. Q. 18. Lepromatous leprosy.
Complications of mumps are as follows: Ans.
l Epididymo-orchitis, oophoritis.
l Pancreatitis l Lepromatous leprosy develops in patients who demon-
l Encephalomyelitis, mumps meningitis. strate a reduced cell mediated immune response and
l Mumps myocarditis exhibits as a diffuse disease.
l Thrombocytic purpura l Occurs as macules or papules leading to progressive
thickening of skin and characteristic nodules may pro-
Q. 15. Diagnosis of typhoid fever. duce severe disfigurement.
Ans. l In lepromatous leprosy, cutaneous lesions include mac-
ules, papules and nodules. Mucous membranes are in-
Typhoid fever or enteric fever is an acute systemic illness volved.
caused due to infection by Salmonella typhi and l Infiltration of corneal nerves causes keratitis and blindness.
S. paratyphi. l Leonine facies, madarosis, megalobules, saddle nose,
clawing of fingers, testicular atrophy and gynaecomas-
tia are other features.
Laboratory Diagnosis l Smears are highly positive and lepromin test is negative.
l 1st week: Blood culture is positive for S. typhi and
Q. 19. Herpes zoster.
S. paratyphi.
l Culture of bone marrow may be positive when bood Ans.
cultures are negative.
l Herpes zoster is due to infection by the varicella zoster
l 2nd week: Widal test
virus.
l 3rd week: Stool and urine culture will be positive.
l The primary infection is chickenpox, which is com-
Q. 16. Clinical features of typhoid fever. monly acquired during childhood.
l Herpes zoster virus remains dormant in the dorsal root
Ans.
and cranial nerve ganglion, until it gets reactivated,
Clinical features of typhoid fever are as follows: causing herpes zoster.
l Onset is insidious. l Patients are middle aged or elderly.
l Incubation period is 10-14 days. l Vesicular eruption in a dermatomal pattern, with severe
l Initially in 1st week, patients present with prodromal pain preceding the rash by 2 to 3 days.
symptoms like - remittent fever, headache, myalgia, l Ramsay-Hunt syndrome is due to involvement of the
malaise, constipation, leucopenia, relative bradycardia. geniculate ganglion of the facial nerve.
Children may present with diarrhoea and vomitings. l Complications are pneumonitis, encephalomyelitis,
l Fever typically rises in stepwise fashion for few days hepatitis and post-herpetic neuralgia.
and then attains plateau. l Acyclovir 800 mg 5 times a day for 7 to 10 days.
l During 2nd week: Rashes (rose spots), splenomegaly, l Local application of calamine lotion and antiseptic
bronchitis, abdominal pain, abdominal distention and powder.
diarrhoea. l For post-herpetic neuralgia: Analgesics, amitryptiline,
l Beyond 2nd week: Confusion, delirium, complications, carbamazepine, sodium valproate, stellate ganglion
coma and death. block and electrical nerve stimulation.
192 Quick Review Series for BDS 3rd Year

Q. 20. Diagnosis of AIDS. Q. 22. Erysipelas.


Ans. Ans.
l The diagnosis of HIV infection is generally based on the l It is a superficial infection of the skin that is most com-
detection of antibodies against viral proteins in the se- monly associated with b haemolytic streptococci.
rum of the patients. Various tests used in diagnosis of l The infection spreads through the lymphatic channels.
AIDS are as follows: l Usually on the face, the lesions tend to occur on cheeks,
eyelids and bridge of the nose producing butterfly
A. Specific Tests shape.
i. Serological tests by ELISA, western blot and immuno- l The affected area is painful, bright red, well circum-
fluorescense test scribed, swollen, indurated and warm to touch. Affected
ii. Antigen detection test skin exhibits surface texture resembling an orange peel.
iii. Polymerase chain reaction test. l High fever and lymphadenopathy are often.
Q. 23. Gonorrhoea.
B. Indirect tests:
i. CD4 and CD8 cell count Ans.
ii. Lymphopenia l Gonorrhoea is primarily a venereal disease caused by
iii. Lymph node biopsy Neisseria gonorrhoeae a gram-negative organism.
iv. Platelet count: Thrombocytopenia l Infection in males results in acute urethritis, dysuria,
l The ELISA is more sensitive and is used for the and urethral discharge of purulent nature. It may lead to
screening whereas the western blot test is more spe- epididymitis, chronic prostitis, balanitis and posterior
cific and is needed to confirm the diagnosis. urethritis.
l ELISA test may be negative during “window pe- l In females, it manifests as cervicitis with candidal or
riod”. During this period, tests based on detection trichomonalvaginitis.
of antigens (p24) or viral RNA through RT-PCR l Extragenital or oral features include acute painful ulcer-
(reverse transcriptase-polymerase chain reaction) ation of lips, thick dry fissured tongue and inflamed
are recommended. gingiva and tonsils.
l The CD4 count, measured by flow cytometry tells l Oral lesions are commonly accompanied by fever and
about the current immune status of the patient and regional lymphadenopathy.
guides us to start prophylactic therapy and anti- l Can be treated by antibiotics, such as norfloxacin and
retroviral treatment. Viral RNA load suggests the ceftriaxone.
prognosis of the patient.
Q. 24. Chickenpox.
Q. 21. Oral manifestations of HIV and AIDS.
Ans.
Ans.
l Chickenpox is an erythematous or vesiculopapular lesion
Oral manifestations of AIDS are as follows: caused by varicella-zoster virus.
l Oropharyngeal lesions are common in HIV infection l It spreads through aerosol route.
and are mostly due to secondary infections. l Clinical features:
i. Fungal infections: a. More severe in adults, pregnant women and immuno-
l Candidiasis, histoplasmosis and cryptococcosis compromised persons.
ii. Viral infections: b. It is more dense on trunk and sparse on limbs.
l Oral hairy leukoplakia (EBV), herpes simplex, c. The fever is associated with every new crop of leison.
cytomegalovirus, varicella zoster and papilloma d. Intense itching infections.
virus. e. It occurs first on mucosal surface and then there is
iii. Bacterial infections: rapid dissemination in a centripetal distribution.
l Periodontal infections, necrotizing ulcerative l Complications: Cerebral ataxia, myocarditis, hepatitis,
gingivitis or necrotizing stomatitis acute glomerulonephritis, pneumonia, arthritis, corneal
iv. Neoplasms: lesion and bleeding diathesis.
l Kaposi sarcoma or lymphoma l Management: Acyclovir within 48 hours of rash ap-
v. Idiopathic oral aphthous ulcers pearances. Human varicella-zoster immunoglobulin for
vi. HIV salivary gland disease (DILS) severe immunocompromised patient, bone marrow re-
vii. Cervical lymphadenopathy cipient and HIV positive contacts.
Section | I General Medicine 193

Q. 25. Viral exanthema. iv. Congenital syphilis exhibits the following clinical
features:
Ans.
l Hutchinson’s teeth (centrally notched, widely-spaced

l Any eruption or rash appears on the skin is called as peg-shaped upper central incisors)
exanthema. The term is often used to describe the child- l Mulberry molars

hood or infectious rashes. l Frontal bossing

l Rashes on skin are caused by viral disease or infection, l Saddle nose

it is called as viral exanthema, e.g. measles. l Poorly developed maxillae

l In exanthematous stage: On 5th day, the red macules l Enlarged liver and spleen

appear behind the ear, along hair line and on posterior l Anaemia

parts of cheeks and spread rapidly in a few hours all l Lymph node enlargement

over the body. Fully erupted rash deepens in colour, l Jaundice

petechiae may occur. l Pseudoparalysis

l In severe measles, the rash is confluent and the face is


swollen and disfigured together with the eyes creates the Q. 28. Widal test.
typical measly appearance.
Ans.
Q. 26. Drugs used in enteric fever. l Widal test is a diagnostic test for enteric fever or
typhoid.
Or
l This test detects agglutinating antibodies to O, H and Vi
Treatment of enteric fever. antigens.
l A fourflod rise in antibody titre in paired samples
Ans. is a good criterion, but it is of limited practical appli-
cation.
Treatment of enteric fever is as follows:
l In the absence of recent immunisation, a high titre of
i. Administration of antibiotics parenterally or orally
O antibodies (1: 640) is useful, but not specific for
depending on patients condition.
typhoid.
ii. Effective antibiotics are fluoroquinolones, third genera-
l H antibodies may be found in even higher titre, but
tion cephalosporins (ceftriaxone, cefotaxime, cefixime,
because of their broad cross-reactivity, are difficult to
and cepodoxime) and azithromycin.
interpret.
iii. The most commonly used drugs are as follows:
l Vi antibodies typically rise later, after 3-4 weeks of
l Ciprofloxacin 500 mg bid for 10 days
illness, and are of little use in early diagnosis.
l Ofloxacin 400 mg bid for 7 days

l Ceftriaxone 1 g IV bid for 10 days


Q. 29. Cervicofacial actinomycosis.
l Azithromycin 1 g daily for 5 days

iv. Dexamethasone is indicated in severe cases as a single Ans.


dose of 3 mg /kg followed by 8 doses of 1 mg/kg every
6th hourly. i. Actinomycosis is a chronic slowly progressive suppura-
v. Carriers are treated with oral antibiotics (amoxycillin, tive infection of the subcutaneous tissue caused by
ciprofloxacin, TMP-SMX) for 6 weeks. Actinomycetes.
ii. Orofacial actinomycosis is a chronic granulomatous
Q. 27. Congenital syphilis. disease characterized by multiple abscess, tissue
destruction, fibrosis and formation of multiple
Ans.
sinuses.
i. Syphilis is caused by Treponema pallidum. In congenital iii. Laboratory diagnosis: Specimen is the pus from the
syphilis, the infection is transmitted from mother to fetus lesion, sputum, tissue biopsy.
transplacentally. iv. Microscopic examination: Under the microscope, the
ii. In congenital syphilis with superficial lesions, the diag- sulphur granules are found to consist of a dense
nosis may be done by demonstrating spirochaetes in network of thin gram-positive filaments, surrounded by
lesions under dark ground microscope. a peripheral zone of swollen radiating club-shaped
iii. IgM FTA-ABS test, the modification of indirect immu- structures, presenting a sun ray appearance.
nofluorescence test, is useful for diagnosis of congeni- v. Treatment: Intravenous penicillin 20 million units daily
tal syphilis and distinguishing from seropositivity due till healing of disease, followed by oral amoxicillin for
to passively transferred maternal antibodies. several months.
194 Quick Review Series for BDS 3rd Year

Q. 30. DPT or triple vaccine. Q. 32. BCG


Ans. Ans.
i. DPT vaccine or triple vaccine is a combined vaccine l BCG (BacilleCalmette-Guerin) is a live-attenuated and
containing the toxoids of diphtheria and tetanus and a freeze-dried vaccine.
killed suspension of Pertussis bacilli. l Dose and administration: BCG vaccine is available
ii. Route of administration: Given as subcutaneous injec- either as fresh liquid vaccine or in the form of freeze-
tions. dried (lyophilized) vaccine.
iii. Immunisation schedule: Immunoprophylaxis of diph- l The dose of the vaccine is a single intradermal injec-
theria is done by active immunisation, passive immuni- tion of 0.1 ml containing 0.05 to 0.1 mg of the
sation and combined immunisation. constituent vaccine containing about 1-2 million
iv. Adverse reactions: Though rare, consists of transient viable organisms.
local inflammation, fever and occasional convulsions. l It should be administered soon after birth failing which
v. Advantages: it should be given anytime during the first year of life
l Minimizes the number of injections. but not after age of 2 years.
l Improves the immune response as the pertussis l Immune response: Injection of the BCG vaccine
component acts as an adjuvant for the toxoids for induces a self-limited infection. The immunity may last
the diphtheria and tetanus. for 10-15 years and is similar to the immunity following
natural infection.
Q. 31. Tetanus or lock jaw.
Ans.
Role of BCG
i. Tetanus, also known as lock jaw, is an acute infection
of nervous system characterized by intense activity of i. Vaccination does not give absolute protection against
motor neurons and resulting in severe muscle spasms. tuberculosis. However, it makes the disease milder and
ii. Aetiology: It is caused by the exotoxin of the anaerobic non-infectious in immunized children.
gram-positive bacillus Clostridium tetani, which acts at ii. Vaccine prevents serious forms of primary tuberculosis
the synapse of the interneurons of inhibitory pathways such as meningitis, skeletal tuberculosis and military
and motor neurons to produce blockade of spinal inhi- spread of the disease
bition. Q. 33. VDRL test (venereal disease research laboratory
iii. Clinical features: test)
l Incubation period – 6 to 10 days.

l Generalized tetanus is characterized by lock jaw or Ans.


trismus due to spasm of masseter, which is initial l It is the most widely used serological test for diagnosis
symptom. Dysphagia, stiffness or pain in the neck, of syphilis. It is a simple and rapid test which requires
shoulder or back muscles appear concurrently. only a small quantity of serum and is as sensitive and no
Laryngeal spasm may lead to asphyxia. less specific than the other tests.
l Sustained contraction of facial muscles results in
l Procedure: The inactivated serum is mixed with cardio-
grimace or sneer called as risus sardonicus. The lipin antigen on a special slide and rotated for four
contraction of muscles of the back produces an minutes.
arched back called opisthotonus. l Uniform distribution of crystals in the drop indicates the
iv. Treatment: serum is nonreactive while formation of clumps indi-
l General measures: Cardiopulmonary monitoring
cates it is reactive.
should be maintained continuously. Sedation, air- l By testing serial dilutions, the antibody titre can be de-
way and nutrition should be maintained. termined.
l Antibiotics should be given to eradicate vegetative
l Major disadvantages of the standard tests is that the
organisms or source of toxins. antigen is nonspecific and accounts for the biological
l Antitoxin is injected to neutralize circulating toxin and
false positive (BFP) reactions.
unbound toxin with wound. Human tetanus immuno-
globin (TIG) 3000-6000 units IM individual doses. Q. 34. Antiviral agents.
l Prophylaxis: Wound debridement and booster doses
Ans.
of TT should be given.
l Unimmunized individuals: ATS 1500 units or TIG Antiviral agents are the agents that selectively attack one of
250 units should be given. the stages of viral replication without harming the host cells.
Section | I General Medicine 195

For example: Q. 35. Rapid tests for HIV infection.


i. Acyclovir: For herpes group viruses (H. simplex type 1
Ans.
and 2, VZ virus
ii. Amantadine and ribavirin: For influenza A like A number of ‘rapid tests’ have been introduced for the pur-
viruses pose of testing single samples quickly. They are as follows:
iii. Vidarabine (adenine arabinoside): For herpes infection i. Cylinder or cassette ELISA
of brain. ii. Immunochromatographic tests
iv. Idoxuridine and trifluorothymidine: Herpes simplex iii. Coated particle agglutination
type 1 iv. Immunoperoxidase or
v. Zidovudine: For retroviruses (HIV) v. Dip stick tests.
vi. Interferons: Nonselective
Tests using finger prick blood, saliva and urine have also
been developed.

Topic 13

Immunological Factors in Disease


(Anaphylaxis and Drug Allergy)
LONG ESSAYS
Q. 1. Classify allergic reactions. Describe the clinical A. Type I Hypersensitivity (Anaphylactic or
features and management of generalised anaphylaxis. Atopic Reaction)
Ans. For example, systemic anaphylaxis or local anaphylaxis.
l Hypersensitivity or allergy is defined as state of exag- Type I anaphylactic or atopic reaction:
gerated immune response to the antigen, which may A. Systemic anaphylaxis
lead to tissue damage, disease or even death following a. Administration of antisera
contact with specific antigens. b. Administration of drugs
l Coombs and Gell (1963) classified hypersensitivity re- c. Sting by wasp or bee
actions into following four types based on the different B. Local anaphylaxis
mechanisms of pathogenesis: a. Hay fever
A. Type I (anaphylactic, IgE or region-dependant type) b. Bronchial asthma
B. Type II (cytotoxic or cell stimulating) c. Food allergy
C. Type III (immune complex or toxic complex disease) d. Angioedema
D. Type IV (delayed or cell-mediated hypersensitivity)
This classification is now widely used. B. Type II (Antibody-Mediated)
l Depending upon the rapidity and duration of immune Hypersensitivity: Cytologic Reactions
response, two distinct forms of hypersensitivity reaction The cytotoxic reactions are defined as those reactions
are there: which cause injury to cell by combining humoral antibodies
A. Immediate type and with cell surface antigens, blood cells are affected com-
B. Delayed type. monly, e.g. erythroblastosis foetalis, leucopenia.

I. IMMEDIATE TYPE Mechanism


l In this type, on administration of antigen, the reaction i. Cytotoxic antibodies to blood cells:
occurs immediately. The immune response in this type l The mechanism involves direct cytolysis of blood

is mediated by the humoral antibodies. cells by combining cell surface antigen with IgG
l It is of three types, i.e. or IgM.
196 Quick Review Series for BDS 3rd Year

l The complement system is activated resulting in 2. Systemic: Circulating immune complex disease or se-
injury to cell membrane, e.g. autoimmune haemo- rum sickness. The steps involved in cell injury by circu-
lytic anaemia, erythroblastosis foetalis and leuco- lating immune complexes are:
penia. a. After antigen is introduced in circulation, it initiates
ii. Cytotoxic antibodies to tissue components: formation of antibodies which react with antigen to
l Cellular injury may be brought about by antibodies undergo antigen antibody reactions.
reacting with some components of tissue cells in b. These complexes are then deposited to different tis-
certain diseases, e.g. Graves’ disease, myasthenia sue sites containing basement membrane exposed to
gravis, male sterility. circulating blood.
iii. Antibody-dependent cell-mediated cytotoxicity: c. Following deposition of antigen antibody complexes
l Cytotoxicity is mediated by leucocytes like mono- in tissues, there is acute inflammatory reaction and
cytes, neutrophils, eosinophils and NK cells. activation of complement system with elaboration of
l The cellular injury occurs by lysis of antibody chemotactic factors, vasoactive amines and ana-
coated target cells through Fc receptors on leuco- phylatoxins, e.g. arthritis and skin diseases.
cytes.
l The antibodies which involve are IgG and the target II. DELAYED TYPE
cells killed are tumour cells, parasites.
D. T-Cell-Mediated (Type-IV) Hypersensitivity
C. Type III Hypersensitivity (Immune l Here, the reaction is slower and develops within 24-48
hours and the effect is prolonged. It is mediated by cellular
Complex Reaction) response type IV reaction, e.g. tuberculosis and typhoid.
Mechanism l Tuberculin reaction is a classical example of delayed hyper-
The type III reaction results from formation of immune sensitivity. On intradermal injection of tuberculoprotein, an
complexes by direct antigen antibody combination as a re- unsensitised individual develops no response, but in a per-
sult of which complement system gets activated causing son who has developed cell-mediated immunity to tubercu-
cell injury. loprotein due to BCG vaccination or previous tuberculous
Type III reactions are of two types: infection, he develops a typical inflammatory reaction
1. Local Arthus reaction: It is localized inflammatory re- reaching a peak in 48-72 hrs after which it subsides slowly.
action, usually an immune complex vasculitis of skin in l Other examples of delayed hypersensitivity are:
an individual with circulating antibody. In it, fibrinoid i. Tuberculosis
necrosis is caused, e.g. injection of antitetanus serum ii. Tuberculoid leprosy
and Farmer’s lung. iii. Typhoid fever
iv. Contact dermatitis

SHORT ESSAYS
Q. 1. Anaphylaxis. i.e. vasoactive amines called as anaphylactic mediators
which produce the clinical reaction, e.g. histamine, se-
Ans. rotonin, platelet activating factor, etc.
l The type I hypersensitivity is a Bcell-mediated immedi- l The effects of these agents are:
ate hypersensitivity reaction. It is known as anaphylac- i. Increased vascular permeability.
tic atopic reaction. ii. Smooth muscle contraction.
l Anaphylaxis is defined as state of rapidly developing iii. Early vasoconstriction followed by vasodilatation.
immune response to an antigen to which individual is iv. Shock.
previously sensitized. v. Increase gastric secretion.
vi. Increase nasal and lacrimal secretion.
It occurs in two forms:
Mechanism l

A. The acute, potentially fatal systemic form called


l The response is mediated by humoral antibodies of IgE anaphylaxis.
type. B. The chronic or recurrent, non-fatal typically local-
l Antibodies are fixed on the surface of tissue cells, i.e. ized form called atopy.
mast cells and basophils in a sensitized individual. l The clinical examples of anaphylaxis are as follows:
l The antigen combines with the cell fixed antibody, lead- A. Systemic anaphylaxis
ing to release of pharmacologically active substances, a. Administration of antisera
Section | I General Medicine 197

b. Administration of drugs with a whitish “skin,” which is the original roof of the
c. Sting by wasp or bee bulla. These oral lesions may persist for months before
B. Local anaphylaxis the skin becomes involved.
a. Hay fever l The oral mucosa may be affected 2-3 months before the
b. Bronchial asthma skin. Cheeks and vermilion border of the lips are the
c. Food allergy common sites for the bullae.
d. Angioedema l Nikolsky’s sign is positive, i.e. the loss of epithelium
occasioned by rubbing apparently unaffected skin is
Q. 2. Pemphigus vulgaris.
termed as Nikolsky’s sign.
Ans.
Pemphigus vulgaris is the most common form of pem-
l
Histological Features
phigus, accounting for over 80% of cases.
l The pemphigus is characterised microscopically by the
Aetiology formation of a vesicle or bulla entirely intraepithelially
just above the basal layer producing a distinctive supra-
l PV is an autoimmune disorder that is characterized by basilar ‘split’.
the appearance of intraepithelial bullae on uninflamed l The suprabasal separation of epithelium (i.e. floating
skin surface or mucous membranes. epithelium) is the most important diagnostic feature of
pemphigus vulgaris.
Clinical Features l Disappearance of intercellular bridges results in loss of
cohesiveness or acantholysis because of which clumps
l Pemphigus vulgaris is commonly seen in people of 50-
of epithelial cells are found lying free within the ve-
60 years age group.
sicular space these cells are called “Tzanck cells.”
l It is insidious in its onset and can often be fatal.
Tzank cells have large nuclei and hyperchromatic
l Men and women are equally affected. Jewish people are
staining.
more commonly affected.
l Bullae are filled with acantholytic multinucleated epi-
l The characteristic feature of pemphigus vulgaris is
thelial cells known as “Tzank cells” which are diagnos-
rapidly appearing multiple vesicles and bullae which
tic feature of this condition.
vary in diameter from a few millimetres to several
l Immunofluorescent testing is considered to be of great
centimetres.
importance in establishing the diagnosis of pemphigus
l The lesions (bullae) appear on a perfectly normal ap-
vulgaris especially when the clinical or microscopic
pearing mucosa, although a large area of the skin sur-
findings are inconclusive.
face may be affected, the eye is not involved. The bullae
could be rubbed with fingers.
Pemphigus vulgaris can be fatal in several cases as the
l
Treatment
appearance of large bullae all over the skin surface can
lead to rapid fluid loss just like a case of severe burns. l Corticosteroids remain the mainstay of treatment for
l The bullae are flaccid, fragile, regular, and non-inflam- pemphigus vulgaris.
matory. They contain a thin watery fluid initially, which l Taking into account the pre-existing and coexisting
may soon become purulent or sanguineous. conditions, therapy may be tailored for each patient.
l Intraorally, the bullae, if seen sufficiently early, appear as l Various other therapies that have been reported as
vesicles on the palate, oropharynx or inside of the cheeks. beneficial are parenteral gold therapy, dapsone, tetra-
l When the affected epithelium ruptures, it leads to the cycline, and plasmapheres and administration of
formation of shallow painful ulcers that are covered 8-methoxypsoralen.

SHORT NOTES
Q. 1. T cells. T Cells
Ans. l Activated T cells on antigenic stimulation produce an
array of cytokines which are important for production
T and B cells play an important role in immune response. and regulation of immune response.
198 Quick Review Series for BDS 3rd Year

l Some T cells are also involved in killing of viruses by l In anaphylactic reaction, the drug induces synthesis of
recognition of viral antigens. IgE antibodies which are fixed to the mast cells.
l T cells are also involved with allograft rejection. l On subsequent exposure, the antigen-antibody com-
plexes cause degranulation of mast cells releasing the
Q. 2. B lymphocytes.
mediators of inflammation like histamine, leukotrienes,
Ans. prostaglandins and platelet activating factor.
l These mediators of inflammation are responsible for the
i. B cells produce antibody-mediated immune response
characteristic signs and symptoms of anaphylaxis,
by specific differentiation and proliferation of plasma
which could be fatal and are as follows:
cells which produce antigen-specific antibodies.
a. Bronchospasm
ii. B lymphocyte precursors, pro-B cells during embry-
b. Laryngeal oedema
onic life develop in fetal liver and afterwards continu-
c. Hypotension
ously throughout life in the bone marrow.
l Skin tests may predict this type of reactions
iii. When viewed under scanning microscope B cells have
an extensively filamentous surface, with numerous mi- Q. 5. Serum sickness.
crovilli.
iv. B cells have immunoglobulins on their surface. Each B Ans.
cell approximately carries about 105 identical Ig mole- i. Serum sickness is a systemic form of type III hyper-
cules on its surface. The surface Ig on a B cell will have sensitivity, which appears 7-12 days following a single
only single antigen specificity, therefore, it serves as the injection of a high concentration of foreign serum such
antigen recognition unit. as diphtheria antitoxin.
v. B cells produce antibody-mediated immune response ii. The clinical syndrome consists of fever, lymphade-
by specific differentiation and proliferation of plasma nopathy, splenomegaly, arthritis, glomerulonephritis,
cells which produce antigen specific antibodies. endocarditis, vasculitis, urticarial rashes, abdominal
Q. 3. Anaphylaxis. pain, nausea and vomiting.
iii. Pathogenesis: Consists of the formation of immune
Ans. complexes between the foreign serum and antibody to
i. Type I hypersensitivity is a B cell-mediated immediate it, which gets deposited on the endothelial lining of the
type of hypersensitivity. The antibody involved is IgE. blood vessels in the various parts of the body resulting
ii. Antibodies are fixed on the surface of tissue cells, i.e. in inflammatory infiltration.
mast cells and basophils in a sensitized individual. iv. Due to massive complement activation and fixation by
iii. The antigen combines with the cell fixed antibody, lead- antigen - antibody complexes, the plasma concentration
ing to release of pharmacologically active substances, i.e. of the complement falls and the disease is self-limited.
vasoactive amines which produce the clinical reaction. v. When all the foreign antigens are eliminated and free
iv. It occurs in two forms: antibody appears, the symptoms clear without any
a. The acute, potentially fatal systemic form called squelae.
anaphylaxis. vi. With subsequent injections, the disease appears early
b. The chronic or recurrent, non-fatal typically local- as the antibodies are already present in the body.
ized from called atopy. vii. Serum sickness differs from the other types of sensitiv-
ity reactions that here a single injection serves as both
Q. 4. Penicillin anaphylaxis. the sensitising and shocking dose.
Ans. Q. 6. Immuno-suppressive therapy.
l Penicillin allergy develops within minutes and is called Ans.
immediate hypersensitivity reaction.
l Drugs that interfere with division of the B and T lym-
l Drugs can induce both types of allergic reaction, i.e.
phocytes can be used to inhibit or suppress immune
humoral and cell-mediated immunity.
responses.
l Humoral-mediated immunity causes immediate allergic
l These drugs are also used to prevent the rejection of
reactions, which are:
transplants, e.g. azathioprine, cyclophosphamide, meth-
a. Type I – Anaphylaxis
otrexate, cyclosporin, etc.
b. Type II – Cytolytic reaction
l Newer agents include lefunomide, mycophenolate,
c. Type III – Arthus reaction
mofetil, sirolimus and tacrolimus.
Cell-mediated immunity causes delayed hypersensitiv- l These immunosuppressive drugs are not cell-specific
ity reaction. and have serious adverse effects like bone marrow
Section | I General Medicine 199

depression, reactivation of latent tuberculosis and viral iv. Idiopathic urticaria.


and fungal infections. l Formation of wheal and flare cutaneous lesions

involving only superficial portions of dermis.


Q. 7. Pemphigus vulgaris.
l This results in circumscribed wheals with ery-

Ans. thematous raised borders with blanched centres.


l Agio-oedema is a well-demarcated oedema involv-
l Pemphigus vulgaris is the most common form of pem-
ing deeper layers of skin and subcutaneous tissues.
phigus, accounting for over 80% of cases.
Q. 10. Treatment of anaphylactic shock.

Aetiology i. Anaphylaxis is a humoral-mediated Type I hypersensi-


tivity reaction where degranulation of mast cells re-
l It is an autoimmune disorder. leases the mediators of inflammation like histamine,
l Unknown leukotrienes, prostaglandins and platelet activating fac-
tor which are responsible for the characteristic signs
Clinical Features and symptoms of anaphylaxis, like bronchospasm, la-
ryngeal oedema, hypotension, which could be fatal.
l It is commonly seen in people of 50-60 years age group. ii. Adrenaline is the drug of choice (0.3-0.5 ml of 1:1000
l It is insidious in its onset and can often be fatal. solution). It promptly reverses all hypotension, laryn-
l Men and women are equally affected. geal oedema and bronchospasm and is life saving in
l The characteristic feature of pemphigus vulgaris is anaphylactic shock.
rapidly appearing multiple vesicles and bullae which iii. For administration of adrenaline, IM route is preferred
vary in diameter from a few millimetres to several as absorption through SC route is not reliable.
centimetres.
l Constitutional disturbance is severe. Q. 11. Angioedema.
Ans.
Treatment l Angioneurotic oedema is also known as Quincke’s oedema.
l Corticosteroids remain the mainstay of treatment for
pemphigus vulgaris. Aetiology
l Various other therapies that have been reported as
l Food or drug allergy.
beneficial are parenteral gold therapy, dapsone, tetra-
l A biochemical abnormality:
cycline, and plasmapheres, and administration of 8-
l Absence of inhibitor of C1 esterase enzyme from serum
methoxypsoralen.
which then causes increased consumption of C2 and C4
Q. 8. Allergy. and formation of kinin-like substances, which increase
the vascular permeability and oedema.
Ans.
l Allergy is a state of hypersentitivity induced by expo- Clinical Features
sure to a particular antigen.
l This antigenic substance capable of inducing type I IgE l It exhibits as smooth, diffused, oedematous swelling
mediated immune response is called allergen. involving face, lips, chin, eyes, tongue and extremities.
l The first dose or the priming dose of allergenic expo- l Rarely, it leads to oedema of glottis resulting in suffoca-
sure sensitizes the immunologic system (B lympho- tion and even death.
cyte).
l The subsequent exposure or the shocking dose results in Treatment
harmful immunologic activation resulting in expression
l Antihistaminic drugs like corticosteroids can also be
of an allergic reaction.
given.
Q. 9. Utricaria. l a-amino caproic acid: Antifibrinolytic agent like EACA
which blocks the activation of plasmin which then acti-
Ans.
vates the complement cascade.
l The various types of urticaria are as follows: l Hereditary type is treated, with drug danazol.
i. Ig E-dependent urticaria l Avoid use of aetiological agent as food or drugs.
ii. Complement-mediated urticaria l Tracheostomy in severe cases is done when suffocation
iii. Non-immunologic urticaria. occurs.
200 Quick Review Series for BDS 3rd Year

Q. 12. Stevens-Johnson syndrome. Q. 13. Anaphylactic reactions – treatment.


Ans. Ans.

i. Stevens-johnson syndrome is a very severe bullous Anaphylactic reaction is a prototype example of type I
form of erythema multiformae with widespread in- hypersensitivity.
volvement typically including skin, oral cavity, eyes Treatment of anaphylaxis is as follows:
and genitalia. l Maintenance of IV line
ii. It is characterised by the abrupt appearance of symp- l Oxygen 4-6 L/min
toms such as fever, photophobia, malaise and eruptions l Endotracheal intubation or tracheostomy – if laryngeal
over the oral mucosa, skin and the genitalia. oedema is severe.
iii. The oral mucosal lesions are extremely painful and l Adrenaline 0.2-0.5 ml of a 1:1000 should be given intra-
mastication is usually impossible. In many cases, the muscularly together with an antihistamine (chlorpheni-
oral lesions may be the chief complaint. ramine maleate 10 mg IV).
iv. The lips may exhibit ulceration with bloody crusting l Bronchodilators – IV aminophylline 250-500 mg as a
and are painful. bolus dose.
v. Genital lesions include nonspecific urethritis, balanitis, l Hydrocortisone 200 mg IV stat may be considered for
and vaginal ulcers. persistant bronchospasm and hypotension.
vi. Some of the other complications may include tracheo- l Diphenhydramine 50-80 mg IM or IV.
bronchial ulcerations and pneumonia. Patients usually Q. 14. Edema – mention four common causes.
recover unless they are secondarly infected.
vii. Treatment: Ans.
l Cause should be identified and withdrawn and in-
Generalised oedema results from sodium and water excess in the
fections should be appropriately treated. body. The common causes of generalized oedema are as follows:
l Symptomatic treatment including oral antihista-
i. Congestive heart failure
mines, analgesics, local skin care and soothing ii. Nephrotic syndrome
mouthwashes is of great importance. iii. Severe malnutrition
l Topical steroid therapy coupled with antibiotics
iv. Hepatic cirrhosis
may be considered where as systemic steroid ther- v. Acute or chronic renal failure.
apy is controversial. vi. Hypothyroidism, etc.

Topic 14

Diseases of Connective Tissues, Bones and Joints


LONG ESSAYS
Q. 1. Describe the aetiology, clinical features and treat- General manifestations of vitamin D deficiency:
ment of rickets. l Deficiency of vitamin D results in:
a. Rickets in children
Ans.
b. Osteomalacia in adults
l Commonly rickets refers to any disorder in the vitamin
D-calcium-phosphorus axis which results in hypomin-
A. VITAMIN D DEFICIENT RICKETS
eralized bone matrix, but such a defect may result from
a number of aetiologies hence a variety of forms of l The term rickets is derived from the old English word
rickets exist. ‘Wrickken’ which means ‘to twist’.
Section | I General Medicine 201

l Rickets is a clinical disorder seen in growing children Biochemical Changes


from 6 months to 2 years of age due to deficiency of
vitamin D. i. Low levels of 25- hydroxy vitamin D and 1, 25-
dihydroxy vitamin D, active metabolites of vitamin D
The pathogenic mechanism of rickets is as follows: ii. Normal or slightly low plasma calcium levels, low
l The basic derangement in both rickets and osteomalacia plasma phosphatase levels and raised plasma alkaline
is an excess of unmineralized matrix. phosphatase due to osteoblastic activity.
l The changes that occur in the growing bones of chil-
dren with rickets are mostly complicated by inadequate
provisional calcification of epiphyseal cartilage and Osteomalacia
deranging endochondral bone growth. i. Osteomalacia is the adult manifestation of the vitamin
The following sequence of events ensues in rickets: D deficiency characterized by failure of mineralisation
a. Endochondral ossification in long tubular bones: of the osteoid matrix.
i. Over growth of epiphyseal cartilage due to prolifera- ii. In adults, lack of vitamin D deranges the normal bone
tion of cartilage cells at the epiphyses followed by remodelling that occurs throughout the life.
inadequate provisional calcification or mineralisation. iii. The newly formed osteoid matrix laid down by osteo-
ii. Persistence and overgrowth of epiphyseal cartilage, blasts is inadequately mineralized, thus producing the
deposition of osteoid matrix on inadequately miner- excess of persistent osteoid that is characteristic of os-
alised cartilage resulting in enlargement and lateral teomalacia.
expansion of the osteochondral junction. iv. Although the contours of the bone are not affected, the
iii. Deformation of the skeleton due to loss of structural bone is weak and vulnerable to gross fractures or mi-
rigidity of the developing bones. Irregular overgrowth crofractures especially affecting vertebral bodies and
of small blood vessels in disorganised and weak bone. femoral necks.
b. Intramembranous ossification in flat bones: Mesenchy- v. Other clinical features include muscular weakness,
mal cells differentiate into osteoblasts with laying down vague bony pains, fractures following trivial trauma
of osteoid matrix which fails to get mineralised resulting and looser’s zones or pseudofractures at weak places in
in soft and weak flat bones. bones.
vi. Biochemical changes: Normal or low serum calcium
levels, lowered plasma phosphate levels and raised se-
Clinical Features rum alkaline phosphatase due to increased osteoblastic
i. The gross skeletal changes depend on the severity of activity.
the rachitic process, its duration and in particular the
stresses to which individual bones are subjected. B. VITAMIN D RESISTANT RICKETS
ii. Craniotabes is the earliest bony lesion occurring due
to small round unossified areas in the membranous i. It is also known as refractory rickets or phosphate
bones of the skull. diabetes.
iii. During nonambulatory stage of infancy, the head and ii. Results in hypophosphataemia and hyperphosphaturia.
chest sustains greatest stresses. The softened occipital Does not respond to usual doses of vitamin D.
bones may become flattened and the parietal bones iii. Oral manifestations:
can be buckled inward by pressure, with the release of l Retarded dental eruption.

the pressure, elastic recoil snaps the bones back into l Pulp horns elongated often reaching DEJ

their original positions is known as craniotabes. l Periapical involvement of grossly normal appearing

iv. An excess of osteoid produces frontal bossing and a deciduous or permanent teeth.
squared and box-like appearance to the head. l Multiple gingival fistulas.

v. Deformation of the chest results from overgrowth of l Lamina dura poorly defined.

cartilage or osteoid tissue at the costochondral junc- l Irregular or abnormal alveolar bone and cementum.

tion, producing “rachitic rosary”.


vi. The inward pull at the margin of the diaphragm cre-
C. RENAL RICKETS
ates the Harrison groove, girdling the thoracic cavity
at the lower margin of the rib cage. i. It is also known as renal osteodystrophy.
vii. Pigeon chest deformity is the anterior protrusion of ii. It is painful-crippling bone disease, due to inability of
sternum due to action of respiratory muscles. kidneys to synthesise 1alpha-hydroxylase and con-
viii. Bowing of legs occur in ambulatory children. vert 25-hydroxycholecalciferol to the active form of
ix. Lumbar lordosis due to involvement of spine and pelvis. vitamin D.
202 Quick Review Series for BDS 3rd Year

iii. Dramatic increase in faecal calcium excretion and mainly due to decreased synthesis of calcitriol in
negative N2 balance. Secondary hyperparathyroidism kidney.
may cause osteitis fibrosa cystica. v. In rickets, the plasma calcitriol is decreased and alka-
iv. Renal rickets or renal osteodystrophy is seen in pa- line phosphatase activity is elevated.
tients with chronic renal failure. Renal rickets is

SHORT ESSAYS
Q. 1. Osteoporosis-clinical features and treatment. Clinical Features
Ans. i. Muscular weakness and vague bony pains
ii. Fractures following trivial trauma
i. Osteoporosis is a disease characterized by increased po- iii. Incomplete or green stick fractures
rosity of the skeleton resulting from reduced bone mass. iv. Looser’s zones or pseudofractures at weak places in
ii. It may be localized or generalized and generalized os- bones
teoporosis may be primary, or secondary to a large va-
riety of conditions. Biochemical changes include:
iii. Categories of generalized osteoporosis are as follows: Normal or low serum calcium levels, lowered plasma phos-
I. Primary phate levels and raised serum alkaline phosphatase due to
a. Postmenopausal increased osteoblastic activity.
b. Senile
II. Secondary: Investigations in vitamin D deficiency are:
a. Endocrine disorders, e.g. hpyo- and hyperthy- l Serum levels of active metabolites of vitamin D,
roidism, hyperparathyroidism, pituitary tumours [25-hydroxy vitamin D and 1, 25-dihydroxy vitamin D]
b. Neoplasia: Multiple myeloma, carcinomatosis l Plasma calcium and phosphate level
c. Gastrointestinal disorders: Malnutrition, malab- l Plasma alkaline phosphatase level
sorption, hepatic insufficiency, idiopathic
disease Q. 3. Gout.
d. Drugs Anticoagulants, anticonvulsants, cortico-
Ans.
steroids, alcohol.
e. Miscellaneous: Osteogenesis imperfecta, immo- Gout or hyperuricaemia occurs due to increased production
bilisation, homocystinuria, anaemia, etc. of uric acid or decreased renal excretion of uric acid.
The most common forms of osteoporosis are senile and
postmenopausal. Senile osteoporosis affects all aging indi- Aetiology
viduals while postmenopausal osteoporosis affects only
women after menopause. Increased production of uric acid: Glucose-6-
phosphatase deficiency, myeloproliferative disorders,
Q. 2. Osteomalacia. lymphoproliferative disorders, cancer chemotherapy, hae-
molysis, idiopathic, etc.
Ans.
Decreased renal excretion of uric acid: Renal failure, lead
Osteomalacia is the adult manifestation of the vitamin D poisoning, alcohol, lactic acidosis, hyperparathyroidism,
deficiency. myxoedema, Down’s syndrome.
Microscopic picture of osteomalacia:
i. It is characterized by failure of mineralisation of the
osteoid matrix. Clinical Features
ii. Widened and thickened osteoid seams and decreased l The history of gout comprises of four stages: Asymp-
mineralisation at the borders between osteoid and bone tomatic hyperuricaemia, acute gouty arthritis, intercriti-
can be seen as pink and basophilic respectively in H cal period and chronic tophaceous gout.
and E stained smears. l The onset may be insidious or explosively sudden.
iii. Widened and thickened osteoid seams and calcified l Recurrent attacks of a characteristic acute arthritis.
bone can be seen as unstained and black respectively in l Aggregate deposits of monosodium urate monohydrate
von Kossa’s stain. in and around the joints.
iv. Osteoclastic activity and fibrosis of marrow is increased. l Renal disease involving interstitial tissue and blood vessels.
Section | I General Medicine 203

l Uric acid nephrolithiasis. Treatment


l NSAIDs are the agents of choice.
Diagnosis l Colchicine is highly effective.
l Serum uric acid levels are elevated and demonstration l Other NSAIDs used are indomethacin, naproxen, feno-
of urate crystals in synovial fluid. profen and phenylbutazone.

SHORT NOTES
Q. 1. COX-2 inhibitors. Clinical Features
Ans. l The history of gout comprises of four stages.
l The onset may be insidious or explosively sudden.
l COX-2 is an enzyme primarily induced at the site of
l Recurrent attacks of a characteristic acute arthritis.
inflammation, producing prostaglandins that mediate
l Aggregate deposits of monosodium urate monohydrate
inflammation and pain.
in and around the joints.
l Conventional NSAIDs inhibit both the isoforms of COX
l Renal disease involving interstitial tissue and blood
in varying degrees.
vessels.
l COX-2 (cyclooxygenase-2) inhibitors are a new class of
l Uric acid nephrolithiasis.
NSAIDs that play a major role in the management of
inflammation and pain caused by arthritis. These agents
selectively inhibit the cyclo-oxygenase-2 enzyme, e.g.
Diagnosis
celecoxib, rofecoxib and valdecoxib.
l There is no significant difference among various tradi- l Serum uric acid levels are elevated and demonstration
tional NSAlDs and COX-2 inhibitors regarding their of urate crystals in synovial fluid.
analgesic and anti-inflammatory properties.
l These agents have relatively lesser gastrointestinal side
effects compared to the conventional, non-selective Treatment
NSAIDs. The incidence of peptic ulcer is lower when l NSAIDs are the agents of choice.
the patients are treated with COX-2 inhibitors compared l Colchicine is highly effective.
to those treated with non-selective NSAIDs.
l These agents do not have significant effect on platelets. Q. 4. Scleroderma.
l Like traditional NSAIDs, COX-2 inhibitors should be
cautiously used in patients with diabetes, dehydration Ans.
and congestive heart failure as they can produce signifi- Scleroderma is characterized by progressive fibrosis and
cant changes in renal functions. calcification of skin and mucosa.
Q. 2. Rheumatoid arthritis-name three complications.
Ans. Aetiology
l Rheumatoid arthritis is the most common form of l Endocrinal disturbance (of thyroid or parathyroid), ner-
chronic inflammatory disease affecting synovial joints. vous disturbance, vascular disturbance, allergic reac-
l The complications of rheumatoid arthritis are as tions and infections.
follows:
a. Bronchopleural fistula
b. Digital ‘nail fold’ infarcts Clinical Features
c. Irreversible renal failure Cutaneous changes: Indurated oedema and erythema of
d. Respiratory failure skin that later becomes yellowish in appearance.
e. Keratitis and corneal ulcer. l Fixation of epidermis to underlying tissues occurs due
Q. 3. Gout. to sclerotic changes.
l CVS and CNS disturbances are also present.
Ans. l Two types of lesions are seen:
Gout or hyperuricaemia occurs due to increased production i. Morphea:
of uric acid or decreased renal excretion of uric acid. ii. Linear or “Coup de Sabre” lesion
204 Quick Review Series for BDS 3rd Year

l Oral changes: Tongue becomes stiff and board-like, B. Other conditions:


difficulty in mouth opening and xerostomia. i. Age more than 60 years
l Radiographic features: Widening of periodontal liga- ii. Infections like endocarditis, tuberculosis, syphi-
ment space and bone resorption occurs at angle and lis, etc.
ramus of mandible bilaterally. iii. Chronic liver disease
l Histological features: iv. Sarcoidosis
l There is thickening and hyalinization of collagen v. Primary biliary cirrhosis.
fibres in skin. l RF titers generally correlate with exta-articulate mani-
l Atrophy of epithelium also occurs. festations and disease severity in patients with rheuma-
l This may be associated with CREST syndrome. toid arthritis.
l Treatment: Corticosteroids therapy. l RF titers are not useful in following disease progres-
sion, hence it is not required to repeat it later.
Q. 5. Rheumatoid factor.
Q. 6. Rickets.
Ans.
Ans.
l Rheumatoid factors are IgM and IgG autoantibodies
against the Fc portion of IgG. In clinical practice, IgM l Deficiency of vitamin D in early childhood may lead to
rheumatoid factor is widely used. rickets.
l Conditions associated with a positive rheumatoid factor l Skeleton is poorly developed because of defective calci-
are as follows: fications in growing bones due to decreased absorption
A. Rheumatic conditions: of calcium and phosphorus.
i. Rheumatoid arthritis- 80% l Craniotabes (flat, box-like skull), pigeon chest, richitic
ii. Sjogren’s syndrome- 75-95% rosary, Harissons sulcus and bow legs
iii. Systemic lupus erythematosus-15- 30% l Eruption of teeth is delayed.
iv. Systemic sclerosis- 20-30% l Treatment: Vit. D 2000 to 3000 IU daily and orthopae-
v. Mixed connective tissue disease-50-60% dic treatment.

Topic 15

Acute Poisoning and Environmental Emergencies


and Miscellaneous
LONG ESSAYS
Q. 1. Describe the management of acute poisoning. iv. Specific therapy to reduce the toxic effects on the body
v. Supportive care to support the functions of vital
Ans.
organs.
l The diagnosis of poisoning is based on history, exami-
nation and simple laboratory investigations.
i. Resuscitation and Initial Stabilization
l Management of poisoning requires five complementary
steps. These are: l The maintenance of airway, breathing circulation.
i. Resuscitation and initial stabilisation l Blood should be collected for investigations including
ii. Diagnosis of type of poison by history, examination sugar, urea, electrolytes and acid-base status, and
and laboratory investigations toxicological investigations.
iii. Non-specific therapy to reduce the levels of toxin in l Rectal temperature should be obtained in all patients
the body with altered sensorium.
Section | I General Medicine 205

l Hypotension should be treated with crystalloids first as e. Use of activated charcoal and
it is most often due to loss of fluids or toxin-induced f. Whole bowel irrigation.
vasodilatation.
l After initial resuscitation, administer a ‘cocktail’ of a. Removal of unabsorbed poison from gut:
50% dextrose (100 mL), naloxone (0.1 mg to 2 mg stat) l The factors to be considered before performing a proce-
and thiamine (100 mg) to patients with altered senso- dure for gastric emptying are to see whether the ingestion
rium. However, if blood glucose can be estimated using is potentially dangerous, can the procedure remove a sig-
a bedside method, dextrose should be infused only if the nificant amount of toxin, whether the benefits of a proce-
blood glucose is below 100 mg/dL. dure outweigh its risks.
l Do not perform a gastric emptying procedure, if the
patient has ingested a non-toxic agent, non-toxic dose of
ii. Diagnosis of Type of Toxin
a toxic agent, or if he is free of symptoms despite of
l This is based on history, examination and investigations. passage of time during which the toxin is known to
produce features of toxicity.
a. History l Gastric emptying is also not indicated, if the patient had
l A thorough clinical history of patient who has taken poison prior repeated vomiting, or the toxin is absorbed rapidly, or
reveals the type of poison and amount of overdose. patient presents late after ingestion. If the risks of a proce-
dure outweigh the possible benefits, it should be avoided.
b. Examination l If the patient has ingested a high-risk toxin (cyanide,
l Based on the examination findings, it may be possible to paracetamol), gastric emptying is indicated even if he is
define a syndrome associated with certain poisons asymptomatic.
known as toxidrome.
l Examples of toxidromes are as follows: b. Induction of emesis:
a. Cholinergic drugs: Carbamates and organophosphates, l Syrup of ipecac is used to induce emesis with the inten-
nicotine present with salivation, urinary incontinence, tion to remove the poison from the stomach. Vomiting
diarrhoea, vomiting, lacrimation, sweating, abdominal occurs within 30 minutes of ingestion of ipecac. It may
cramps, bronchospasm, bradycardia, miosis, hyperten- be used in an alert conscious and should be avoided in
sion or hypotension, confusion, coma, muscle fascicu- comatose patients.
lations, seizures.
b. Narcotics: e.g. Opiates present with miosis, CNS c. Gastric lavage:
depression, coma, bradycardia, hypothermia, respi- l For orogastric lavage, the patient should be in a left
ratory depression. lateral position with the head lower than the feet, so as
to avoid aspiration. If patient is unconscious, intubate
c. Investigations the patient before passing a lavage tube.
Simple laboratory tests required include: l Gastric lavage may be considered in a patient who has
l Colour of urine ingested a potentially life-threatening amount of a toxic
l Colour of blood agent within the last 60 minutes.
l Crystals in urine (in ethylene glycol poisoning) l It may also be used within 2-4 hours of ingestion or
l Ketonuria chemical poisons but its utility in such situations has not
l Anion gap been proven.
l Osmolol gap l Lavage is contraindicated following ingestion of strong
caustics, non-toxic agents and volatile hydrocarbons.
iii. Non-Specific Treatment d. Cathartics:
l This includes removal of unabsorbed poison from the l Commonly used cathartics are magnesium sulphate,
gut (gastric decontamination) and Enhancing the excre- sorbitol and magnesium citrate.
tion of absorbed poison from the body. l Repetitive doses of these agents should be avoided.
There is no data to support their efficacy and their use is
A. Gastric decontamination includes: not recommended at present.
a. Removal of unabsorbed poison from the gut.
b. Induction of emesis, e. Activated charcoal:
c. Gastric lavage, l Activated charcoal has enormous surface area and can
d. Cathartics, adsorb large amounts of chemicals.
206 Quick Review Series for BDS 3rd Year

l The usual dose is 1 g/kg body wt. or 10 parts of charcoal l It may be useful in patients with ingestion of carbamaze-
for every one part of toxin, whichever is greater. pine, dapsone, phenobarbital, quinine and theophylline.
l It is likely to be beneficial if a patient has ingested a
potentially toxic amount of a poison within 60 minutes c. Dialysis:
of presentation. Peritoneal and haemodialysis are useful in:
l It is contraindicated in patients with unprotected airway l Poisoning with water-soluble compounds of low mo-
and caustic ingestion. lecular weight.
l Toxin with low volume of distribution.
f. Whole bowel irrigation: l Toxin with low serum protein-binding.
l For whole bowel irrigation, isotonic solution of l Toxin not irreversibly bound to the tissues.
polyethylene glycol, electrolytes orally in a dose of
2 litres/hour is used for 4- 6 hours or till the rectal Dialysis is useful in: Ethanol, methanol, salicylates, theophyl-
effluent is clear. line, ethylene glycol, phenobarbital and lithium intoxications.
l The components of this solution are not absorbed
through the intestines; instead, it flushes the gut iv. Specific Therapy
mechanically.
l At present, there are no established indications for the l If the toxin can be identified, then a specific therapy
use of whole bowel irrigation. antidotes can be administered. However, antidotes are
l It is an option for potentially toxic ingestions of sus- available only for a few toxins, some examples of anti-
tained release drugs, enteric-coated drugs, iron and drug dotes are as follows:
packets.
Poison Antidote
B. Enhancement of excretion of absorbed toxin Cholinesterase inhibitors Atropine
from the body: (organophosphates, carbamates)
Important methods for enhancing excretion are: Organophosphates Pralidoxime
a. Forced diuresis with alteration in urinary pH, Opioids Naloxone
b. Multiple doses of activted charcoal,
c. Peritoneal and haemodialysis, haemoperfusion, haemo- Hypoglycaemic agents Dextrose
filtration and exchange transfusion. Tricyclic antidepressants Sodium bicarbonate
Methanol, ethylene glycol Ethanol
a. Alkaline diuresis:
Snake bites Snake antivenin
l Alkaline diuresis is useful in salicylates, phenobarbital
and lithium intoxication. It is performed by infusing 5% Iron Deferoxamine
dextrose containing 20-35 mEq/L of bicarbonate at a Lead, arsenic, mercury BAL (Dimercaprol)
rate so as to produce a urine pH 7.5-8.5. To prevent
Paracetamol N-acetylcysteine
hypokalaemia, add potassium in every second or third
bottle. Diuresis is not required in most cases. Benzodiazepines Flumazenil.
l Monitor vitals of the patient along with input/output,
electrolytes and acid-base status.
l This procedure is contraindicated in patients with
v. Supportive Therapy
shock, hypotension, renal failure and congestive heart l The treatment of most of the cases with poisoning is
failure. largely supportive as antidotes are available only for a
few toxins.
b. Multiple-dose activated charcoal (MDAC): l The aim of the supportive treatment is to preserve the
l Free charcoal remains in the intestines to bind any toxin vital organ functions till poison is eliminated from the
which has significant enterohepatic circulation. body and the patient resumes normal physiological
l Free toxin in the blood also tends to diffuse out of the functions.
blood into the intestines where it binds to the charcoal, l The functions of central nervous system, cardiopulmo-
thereby maintaining the concentration of free toxin in nary system and renal system should be supported with
the intestines near zero (“gastrointestinal dialysis”). proper care in coma, seizures, hypotension, arrhyth-
l Depending upon the severity of poisoning, the doses mias, hypoxia, and acute renal failure. Monitor the fluid,
are: 0.5-1 g/kg body weight every 1-4 hours. electrolyte and acid-base status closely.
Section | I General Medicine 207

SHORT ESSAYS
Q. 1. Mention therapeutic uses of atropine. Aetiology
Ans. Chronic exposure to arsenic in drinking water which
contains more than 10 mg/l of arsenic.
Therapeutic uses of atropine are as follows:

a. As antisecretory: Clinical Features


i. Pre-anaesthetic medication:
The clinical features of chronic arsenic poisoning are as
l Prior use of atropine (anticholinergics) is must to check
follows:
increased salivary and tracheobronchial secretions
i. Anorexia, nausea, vomiting and weight loss.
when irritant general anaesthetics like ether are used.
ii. Low-grade fever.
l It also prevents laryngospasm by reducing respira-
iii. skin lesions:
tory secretions and also prevents vasovagal attack.
l Hyperpigmentation and ‘rain drop’ pigmentation.
ii. Peptic ulcer: Atropine reduces gastric secretion in fast-
l Palmar and plantar keratosis
ing and neurogenic phase and affords symptomatic
l Multiple epitheliomas
relief in peptic ulcer.
l Mee’s lines, i.e. transverse white lines on finger nails.
iii. Pulmonary embolism: Atropine benefits by reducing
reflex secretions. iv. Oral findings:
l Inflammation of mucosa
iv. To check excess seating salivation as in Parkinsonism.
l Excessive salivation and metallic taste in the mouth.

b. As antispasmodic: l Gingival ulcerations.

l Premature shedding of teeth.


i. Intestinal and renal colic, abdominal cramps: In ab-
v. Hepatic portal fibrosis.
sence of mechanical obstruction, atropine offers symp-
vi. Splenomegaly and hypersplenism.
tomatic relief but are less effective in biliary colic.
vii. Bone marrow depression.
ii. Nervous and drug induced diarrhoea, functional diar-
rhoea but not effective in infective diarrhoeas.
iii. Spastic constipation, irritable colon. Treatment
iv. Pylorospasm, gastric hypermotility, gastritis, nervous
dyspepsia l Prevention of exposure to arsenic in drinking water.
v. Dysmenorrhoea. l Removal of arsenic from household drinking water by
using promising medium like iron oxide.
c. Bronchial asthma, asthmatic bronchitis, COPD: Q. 3. Symptoms of lead poisoning.
Atropine dries up secretions in the respiratory tract, leading
Ans.
to its inspissations and plugging of bronchioles.
i. Plumbism or lead poisoning occurs chiefly as an
d. As mydriatics and cycloplegic: occupational hazard, e.g. painters, lead workers,
l Atropine is very valuable in treatment of iritis, iridocy- hair dyes, ointments, tinned foods and contaminated
clitis, choroiditis, keratitis and corneal ulcer. water.
l It affords rest to intraocular muscles and cuts down their ii. Clinical features:
painful spasm. l Serious gastrointestinal disturbances which include

l Atropine prevents adhesions between iris and lens or nausea, vomiting, colic and constipation.
iris and cornea and may even break them, if already l Peripheral neuritis which may produce the charac-

formed. teristic “wrist-drop” or “foot-drop” phenomenon.


l Blood changes are hypochromic anaemia with baso-
e. As cardiac vagolytic: philic stippling of RBCs.
Atropine is useful in counteracting bradycardia and partial iii. Oral manifestations:
heart block in patients where increased vagal tone is l Burtonian lines - gray/bluish black line occurs on

responsible as in MI, digitalis toxicity. marginal gingiva.


l Excessive salivation and metallic taste.
Q. 2 Arsenic poisoning.
iv. Treatment: Treatment of the oral lesions is secondary to
Ans. systemic treatment.
208 Quick Review Series for BDS 3rd Year

Q. 4. Fluorides in health and disease? l Do not perform a gastric emptying procedure, if the
patient has ingested a non-toxic agent, non-toxic dose of
Ans. a toxic agent, or if he is free of symptoms despite of
l 99% of all fluoride in the human body is found in passage of time during which the toxin is known to
calcified tissues. produce features of toxicity.
l Fluorides control caries activity by altering the tooth l If the risks of a procedure outweigh the possible
surface or contour. benefits, it should be avoided.
l The maximum plasma concentration of fluoride follow- l If the patient has ingested a high-risk toxin (cyanide,
ing the oral intake of fluoride on a fasting stomach is paracetamol), gastric emptying is indicated even if he is
reached within 30 minutes and this is independent of asymptomatic.
the amount of fluoride ingested. The plasma half-life
fluoride is in range of 4-10 hours. b. Induction of emesis:
l The presence of excessive quantities of fluorides in l Syrup of ipecac is used to induce emesis with the inten-
drinking water leads to a characteristic sequence of tion to remove the poison from the stomach. It may be
pathological changes in teeth, bone and periarticular used in an alert conscious patients and should be
tissues. avoided in comatose patients.
l A yellowish-brown mottling of teeth especially perma-
nent teeth is an early and easily recognizable feature of c. Gastric lavage:
chronic toxicity. l For orogastric lavage, the patient should be in a left
l This is important, as skeletal involvement may not be lateral position with the head lower than the feet, so as
clinically obvious until advanced changes have taken to avoid aspiration. If patient is unconscious, intubate
place in bone. the patient before passing a lavage tube.
l The changes in bone and periarticular tissues limit l Gastric lavage may be considered in a patient who has
movement of the limbs and may cause back pain. ingested a potentially life-threatening amount of a toxic
l Lesions may progress to cause serious disability, par- agent within the last 60 minutes.
ticularly kyphosis, due to progressive joint ankylosis. l It may also be used within 2-4 hours of ingestion or
l Changes in the bones of the thoracic cage may lead to chemical poisons but its utility in such situations has not
rigidity that causes dyspnoea on exertion. In calcium- been proven.
deficient children, the toxic effects of fluoride manifest l Lavage is contraindicated following ingestion of strong
even at marginally high exposures to fluoride. caustics, non-toxic agents and volatile hydrocarbons.

Q. 5. General management or non-specific treatment of d. Cathartics:


poisoning case. l Commonly used cathartics are magnesium sulphate,
Ans. sorbitol and magnesium citrate.
l Repetitive doses of these agents should be avoided.
General management or non-specific treatment of poison- Their use is not recommended at present.
ing case includes removal of unabsorbed poison from the
gut (gastric decontamination) and enhancing the excretion e. Activated charcoal:
of absorbed poison from the body. l Activated charcoal has enormous surface area and can
adsorb large amounts of chemicals.
A. Gastric decontamination includes: l The usual dose is 1 g/kg body wt. or 10 parts of charcoal
a. Removal of unabsorbed poison from the gut. for every one part of toxin, whichever is greater.
b. Induction of emesis, l It is likely to be beneficial, if a patient has ingested a
c. Gastric lavage potentially toxic amount of a poison within 60 minutes
d. Cathartics, of presentation.
e. Use of activated charcoal and l It is contraindicated in patients with unprotected airway
f. Whole bowel irrigation. and caustic ingestion.

a. Removal of unabsorbed poison from gut: f. Whole bowel irrigation:


l The factors to be considered before performing a proce- l For whole bowel irrigation, isotonic solution of polyeth-
dure for gastric emptying are to see whether the inges- ylene glycol-electrolytes orally in a dose of 2 litres/hour
tion is potentially dangerous, can the procedure remove is used for 4- 6 hours or till the rectal effluent is clear.
a significant amount of toxin, whether the benefits of a l The components of this solution are not absorbed through
procedure outweigh its risks. the intestines; instead, it flushes the gut mechanically.
Section | I General Medicine 209

l At present, there are no established indications for the CLASSIFICATION OF GINGIVAL


use of whole bowel irrigation. ENLARGEMENTS
B. Enhancement of excretion of absorbed toxin from Classification of gingival enlargements based on aetiology
the body: and pathologic changes are as follows:
Important methods for enhancing excretion are: i. Inflammatory enlargement
a. Forced diuresis with alteration in urinary pH, a. Chronic
b. Multiple doses of activted charcoal, b. Acute
c. Peritoneal and haemodialysis, haemoperfusion, haemo- ii. Drug-induced enlargement
filtration and exchange transfusion. a. Dilantin
b. Barbiturates
a. Alkaline diuresis: iii. Associated with systemic diseases
l Alkaline diuresis is useful in salicylates, phenobarbital a. Conditioned enlargement
l Pregnancy
and lithium intoxication. It is performed by infusing 5%
l Puberty
dextrose containing 20-35 mEq/L of bicarbonate at a
l Vitamin C deficiency
rate so as to produce a urine pH 7.5-8.5. To prevent
l Plasma cell gingivitis
hypokalaemia, add potassium in every second or third
l Pyogenic granuloma
bottle. Diuresis is not required in most cases.
l This procedure is contraindicated in patients with shock, b. Enlargements due to systemic diseases like
l Leukaemia
hypotension, renal failure and congestive heart failure.
l Wegener’s granulomatosis

l Sarcoidosis
b. Multiple-dose activated charcoal (MDAC):
l Free charcoal remains in the intestines to bind any toxin iv. Idiopathic gingival enlargement
which has significant enterohepatic circulation. v. Neoplastic enlargements (gingival tumours)
l Benign tumours
l Free toxin in the blood also tends to diffuse out of the
l Malignant tumours, e.g. ameloblastoma, odon-
blood into the intestines where it binds the charcoal,
thereby maintaining the concentration of free toxin in toma, sqamous cell carcinoma, adenocarci-
the intestines near zero. noma, malignant melanoma, fibroma, osteoma,
l It may be useful in patients with ingestion of carbamaze- exostosis, giant cell reparative granuloma, hae-
pine, dapsone, phenobarbital, quinine and theophylline. mangioma.
vi. False enlargement
c. Dialysis:
Peritoneal and haemodialysis are useful in: Based on Location and Distribution
l Poisoning with water-soluble compounds of low
molecular weight. l Localised
l Toxin with low volume of distribution. l Generalised
l Toxin with low serum protein-binding. l Marginal
l Toxin not irreversibly bound to the tissues. l Papillary
l Diffuse
Dialysis is useful in ethanol, methanol, salicylates, theoph- l Discrete – it is an isolated sessile or pedunculated
ylline, ethylene glycol, phenobarbital and lithium intoxications. tumour like enlargement.
Q. 6. Gingival hyperplasia / hypertrophy of gums.
Ans. Based on Degree of Enlargement
l Gingival enlargements are otherwise known as hyper- Grade 0 – No signs of gingival enlargement
trophic gingivitis or gingival hyperplasia. Grade 1 – Enlargement confined to papilla
l It is an increase in size of gingiva and is a very common Grade 2 – Enlargement involves papilla and marginal
feature found in gingival diseases. gingiva
l Various causes of gingival enlargements are as follows: Grade 3 – Enlargement covers three quarters or more of the
i. Influx of normal cells into bloodstream, e.g. gingivitis crown
ii. Influx of abnormal cells into bloodstream, e.g.
Q. 7. What is the organophosphorus poisoning?
leukaemia.
Describe the treatment.
iii. Influx of fibrous tissue, e.g. fibromatosis, epanutin
related. Ans.
210 Quick Review Series for BDS 3rd Year

l Organophosphates are used in agricultural and domestic ii) Maintain BP and patent airway.
insecticides. iii) Atropine is used in the treatment of organophosphorus
l Organophosphorus poisoning can result as accidental, poisoning and mushroom poisoning.
suicidal or homicidal result. l Atropine is highly effective in counteracting the

l Symptoms include, muscarnic, nicotinic and central ef- muscarinic symptoms produced by the organophos-
fects: Vomiting, abdominal cramps, diarrhoea, miosis, phorus compounds.
sweating, increased salivary, tracheobronchial and gas- l In high doses, it antagonizes the central effects also.

tric secretions, hypotension, muscular twitching, weak- iv) Pralidoxine and obidoxime are used in the treatment of
ness, convulsions and coma may occur. Death occurs organophosphorus poisoning. The compounds com-
due to respiratory paralysis. bine with cholinesterase organophosphate complex,
release the binding and set free AChE enzyme. They
should be given within a few hours (,24 hours) after
Treatment poisoning, preferably immediately because the
i) If poisoning is through skin, remove clothes and wash skin complex undergoes ageing and then the enzyme
thoroughly, if by oral route, gastric lavage is necessary. cannot be released.

SHORT NOTES
Q. 1. Arsenic poisoning. l Alkaline diuresis is effective in phenobarbital
poisoning.
Ans.
l Haemodialysis is effective in removing long-acting
i. Chronic exposure to arsenic results in arsenic poison- barbiturates, especially phenobarbital.
ing.
Q. 3. Scorpion bite.
ii. Clinical features: Anorexia, nausea, vomiting, weight
loss and low-grade fever. Ans.
iii. Hyperpigmentation and ‘rain drop’ pigmentation of the
i. Scorpions are the most important venomous animals
skin and as well palmar and plantar keratosis.
after snakes.
iv. Oral findings include inflammation of mucosa, exces-
ii. Two types of scorpion venom exist:
sive salivation, metallic taste in the mouth and prema-
a. The venom of the genera Hadrurus, Vejovis and
ture shedding of teeth.
Uroctonus has local effects only, including sharp
v. Treatment consists of prevention of exposure to arsenic
burning, swelling and discolouration at the bite site.
in drinking water and removal of arsenic from house-
Very rarely, anaphylaxis occurs.
hold drinking water by using promising medium like
b. The second type of venom, produced by the genera
iron oxide.
of the poisonous varieties of centruroides and meso-
Q. 2. Barbiturate poisoning. buthus, contains neurotoxins which block sodium
channels. This leads to spontaneous depolarization
Ans.
of parasympathetic and sympathetic nerves which
Barbiturate poisoning or phenobarbital poisoning: results in tachycardia, hypertension, sweating, pilo-
l In large overdose, there is CNS depression ranging from erection, hyperglycaemia and ultimately pulmonary
lethargy to coma, hypotension, pulmonary oedema, and oedema and seizures.
cardiac arrest. In addition, it causes hypothermia and iii. The sharp pain after a sting is quickly followed by paraes-
ventilatory depression. thesiae and numbness in the area due to peripheral nerve
l Pupils are usually constricted, but may dilate in terminal effects, muscle fasciculation and finally drowsiness.
phases. Bullous skin lesions may be seen with severe iv. Local pain and paraesthesiae are best treated with local
overdose. compresses and oral analgesics.
l With phenobarbital, signs of toxicity usually appear v. Patients with significant envenoming should be hospital-
when serum concentration exceeds 4 mg/dL. ized for at least 12 hours and observed for cardiovascu-
l Prompt gastrointestinal decontamination should be lar and neurological sequelae. More severe symptoms
done by lavage. Repeated oral administration of acti- may require airway support and 1-2 vials of intravenous
vated charcoal 2-4 hourly is very effective. antivenin.
l General measures with particular emphasis on respira-
Q. 4. Lead poisoning.
tory and cardiovascular support, and correction of tem-
perature and electrolyte derangements. Ans.
Section | I General Medicine 211

i. Plumbism or lead poisoning occurs chiefly as an occu- l In the case of ingestion, gastric lavage should be done
pational hazard, e.g. painters, lead workers, hair dyes, within first 4 hours of ingestion.
ointments, tinned foods and contaminated water.
Q. 7. Wrist drop.
ii. Clinical features: Serious gastrointestinal disturbances
which include nausea, vomiting, colic and constipation. Ans.
iii. Peripheral neuritis which may produce the characteris-
i. Plumbism or lead poisoning occurs chiefly as an
tic “wrist-drop” or “foot-drop” phenomenon.
occupational hazard, e.g. painters, lead workers, hair
iv. Blood changes are hypochromic anaemia with baso-
dyes, ointments, tinned foods and contaminated water.
philic stippling of RBCs.
ii. Peripheral neuritis which may produce the characteristic
v. Oral manifestations: Burtonian lines, i.e. gray/bluish
“wrist-drop” or “foot-drop” phenomenon.
black line occurring on marginal gingiva and excessive
iii. Blood changes are hypochromic anaemia with baso-
salivation and metallic taste.
philic stippling of RBCs.
vi. Treatment: Use of sodium calcium edetate.
iv. Oral manifestations include burtonian lines, i.e. gray/
Q. 5. Adverse drug reactions. bluish black line occurring on marginal gingiva and
excessive salivation and metallic taste.
Ans.
v. Treatment: Use of sodium calcium edetate.
i. Adverse drug reaction is any response to a drug that is
Q. 8. Gram stain.
noxious and unintended and that occurs at doses used in
man for prophylaxis, diagnosis or therapy (WHO). Ans.
ii. Various types of adverse drug reactions are:
l The Gram stain is the most widely used differential
a. Side effects
staining procedure in bacteriology.
b. Untoward effects
l It is called a differential stain since it differentiates
c. Toxic effects
between gram-positive and gram-negative bacteria.
d. Allergic and idiosyncratic effects.
l Bacteria that stain purple with the Gram staining proce-
iii. Side effects are urelated pharmacological effects pro-
dure are termed gram-positive; those that stain pink are
duced with therapeutic dose of a drug, e.g. dryness of
said to be gram-negative.
mouth produced by atropine.
l Gram-positive and gram-negative bacteria stain differ-
iv. Untoward effects develop with therapeutic dose of a drug
ently because of fundamental differences in the struc-
and they may be so severe and undesirable that they may
ture of their cell walls.
neccessitate cessation of the treatment, e.g. resistant
l The Gram staining is an essential procedure used in the
staphylococcal dirrhoea following tetracycline therapy.
identification of bacteria and is frequently the only
v. Toxic effects are usually seen when a drug is administered
method required for studying their morphology.
repeatedly or in large doses, e.g. respiratory depression
due to morphine or ototoxicity due to aminoglycosides. Q. 9. Dental fluorosis.
vi. Allergic and idiosyncratic effects are the qualitative
Ans.
intolerance due to immune or other than immune
mechanisms, e.g. penicilline producing anaphylaxis. l Dental fluorosis is mottling of teeth where the enamel loses
its luster, becomes rough, pigmented, pitted and brittle.
Q. 6. Organophosphorus poisoning.
l Skeletal fluorosis: Sclerosis of bones, especially of
Ans. spine, pelvis and limbs.
l Osteoporosis.
i. Organophosphorus poisoning can result as accidental,
l Weakness, anaemia, weight loss, brittle bones and stiff
suicidal or homicidal result.
joints.
ii. Symptoms include, muscarnic, nicotinic and central
effects: Vomiting, abdominal cramps, diarrhoea, mio- Q. 10. Dilantin/phenytoin sodium.
sis, sweating, increased salivary, tracheobronchial and
Ans.
gastric secretions, hypotension, muscular twitching,
weakness, convulsions and coma may occur. Death i. Phenytoin sodium or diphenylhydantoin sodium is a
occurs due to respiratory paralysis. barbiturate analogue and is a major antiepileptic drug.
iii. Treatment ii. Phenytoin acts by stabilizing the neuronal membrane
and thus prevents the spread of seizure discharges.
General measures iii. Therapeutic uses:
l Remove the patient from site of exposure, and wash l It is the first choice of drug in generalized tonic-

with soap and water. clonic seizures, simple and complex partial seizures.
212 Quick Review Series for BDS 3rd Year

l It is second choice to carbamazepine in trigeminal ii. They are widely used in treatment in medical, gynaecologi-
neuralgia and other neuralgias. cal, and other systemic infections but rarely in dentistry.
iv. The adverse effects include hypertrophy and hyperplasia iii. The aminoglycosides are bactericidal antibiotics acting
of gums. by inhibition of protein synthesis.
v. If it is used during pregnancy. It causes fetal hydantoin iv. Aminoglycosides in current therapeutic use are:
syndrome. l For systemic use: Streptomycin, gentamicin, etc.

l For GI infections and gut sterilization: Neomycin,


Q. 11. Ciprofloxacin.
paromomycin
Ans. l For topical use in the eye and on the skin: Neomy-

l Ciprofloxacin belongs to first generation fluoroquino- cin, framycetin, etc.


lones. Q. 14. Quinolones.
l It is highly effective against gram-negative organisms like:
E. coli, Enterobacter, Proteus, K. pneumoniae, Salmo- Ans.
nella, Shigella, H. ducreyi, H. influenzae, N. gonorrhoeae, i. Fluoroquinolones are the synthetic fluorinated ana-
N. meningitidis, etc. logues of nalidixic acid.
l Ciprofloxacin is rapidly absorbed orally but its absorp- ii. They are quinolone antimicrobials having one or more
tion is delayed by food. fluorine substitutions, and are divided into:
l It has high tissue permeability thus achieving higher a. First generation fluoroquinolones: Norfloxacin, Cip-
concentration of the drug in lung, sputum, muscle, rofloxacin, ofloxacin, etc.
bone, prostate and phagocytes than plasma. b. Second generation fluoroquinolones: Lomefloxacin,
l Because of wide spectrum bactericidal activity, oral sparfloxacin, levofloxacin, etc.
efficacy and good tolerability ciprofloxacin is used iii. The fluoroquinolones are bactericidal and act by inhib-
extensively in broad range of infections even as blind iting the enzyme bacterial DNA gyrase.
therapy. iv. Because of wide spectrum bactericidal activity, oral effi-
l The usual dose is 250-750 mg BD orally and cacy and good tolerability ciprofloxacin is used exten-
100-200 mg IV. sively in broad range of infections even as blind therapy.
Q. 12. Phenobarbitone. Q. 15. Adverse effects of cephalosporins.
Ans. Ans.
l Phenobarbitone is an important drug of choice in the Cephalosporins are a group of semisynthetic antibiotics
treatment of epilepsy. chemically related to penicillins. The adverse effects of
l Phenobarbitone inhibits the neurotransmitory action by cephalosporins are as follows:
enhancing the GABA receptors thus facilitating them to i. Pain after i.m. injection mainly with cephalothin and
open the chloride ion channels. thromboplebitis after i.v. injection.
l Phenobarbitone raises the seizure threshold and thus ii. GI disturbances like diarrhoea due to alteration of gut
prevents epileptic attacks. ecology or irritative effect.
l It is used in generalized tonic-clonic seizures and partial iii. Hypersensitivity reactions may manifest as rashes, ana-
seizures. phylaxis, angioedema, asthma and urticaria.
l It is preferred due to its efficacy and low cost. iv. Severe bleeding is seen in cefoperazone and ceftriax-
Q. 13. Aminoglycosides. one. It is produced due to hypoprothrombinaemia com-
monly in patients with cancer, intra-abdominal infec-
Ans.
tion or renal failure.
i. Aminoglycoside antibiotics are a group of natural and v. Neutropenia and thrombocytopenia are reported with
semisynthetic antibiotics having polybasic amino ceftazidime but are rare.
groups linked glycosidically to two or more amino- vi. A disulfiram like interaction with alcohol has been
sugar residues. reported with moxalactam and cefoperazone.
Section | I General Medicine 213

Key Points to Remember


AIMS OF MEDICINE AND CLINICAL METHODS: GIT (DISEASES
OF THE GASTROINTESTINAL SYSTEM) AND DISEASES OF THE LIVER
AND BILIARY SYSTEM
Gastro-oesophageal Reflux
l Reflux of gastric contents into the lower oesophagus is called gastro-oesophageal reflux disease
(GERD).
l Heartburn occurs in reflux oesophagitis.

Causes
i. Sliding hiatus hernia
ii. Cardiomyotomy and vagotomy
iii. Increased intra-abdominal pressure
iv. Reduced tone of lower oesophageal sphincter
v. Impaired gastric emptying
vi. Drugs that reduce the tone of lower oesophageal sphincter, e.g. aminophylline, beta-agonists, nitrates,
calcium channel blockers.

Various appearances of tongue


l Magenta colour—riboflavin deficiency
l Raw beef tongue—nicotinic acid deficiency
l Strawberry—scarlet fever
l ‘Jack in the box’—rheumatic chorea

Jaundice
l Classification of jaundice based upon the underlying derangement of bilirubin metabolism:
i. Predominantly unconjugated hyperbilirubinaemia
ii. Predominantly conjugated hyperbilirubinaemia
l Classification based on the pathological mechanism giving rise to jaundice:
l Haemolytic jaundice

l Hepatocellular jaundice
l Cholestatic jaundice

Hepatitis
Features Hepatitis A Hepatitis B Hepatitis C
Incubation period 30 days (15–45) 90 days (15–150) 50 days (15–160)
Onset Acute Insidious Insidious
Age Children, young Any age Adult, but any age
Route of transmission Faeco-oral Parenteral Parenteral
Severity Mild Severe Moderate
Prognosis Good Worse Moderate
Prophylaxis Immune serum globulin Hyperimmune serum globulin None
and hepatitis B vaccine

Carrier state None 1–30% 1%


214 Quick Review Series for BDS 3rd Year

Some interesting facts:


l ‘Foaming liver’—seen in gas gangrene
l Honeycomb liver—actinomycosis
l Nutmeg liver—chronic venous congestion of liver
l Piecemeal necrosis of liver—chronic active hepatitis

HAEMATOLOGY AND CVS


Coagulation Factors
Factor Synonym
I Fibrinogen
II Prothrombin
III (Tissue) Thromboplastin
IV Calcium
V Proaccelerin/Ac globulin
VI Not designated
VII Proconvertin
VIII Antihaemophilic factor
IX Christmas factor
X Stuart–Prower factor
XI Plasma thromboplastin (PTA)
XII Hageman factor/glass contact factor

XIII Fibrin stabilizing factor

l Coagulation pathways—intrinsic pathway and extrinsic pathway.


l Intrinsic pathway is assessed in vitro by the activated partial thromboplastin time (APTT).
l Extrinsic pathway is assessed by the prothrombin time (PT).
l Bleeding disorders can be due to:
l Coagulation defects

l Platelet disorders

l Vessel wall abnormalities

l Thrombocytopaenia means reduced platelet count. It can occur due to following reasons:
l Failure of megakaryocyte maturation

l Excessive platelet consumption

l Their sequestration in an enlarged spleen

l Spontaneous bleeding does not occur usually until the platelet count falls below about 30 3 1091, unless
their function is also compromised.

Haemophilia A
l An X-linked recessive disorder
l Reduction of factor VIII
l Bleeding into joints and muscles with other bleeding manifestations
l APTT is typically prolonged.

Haemophilia B
l X-linked disorder
l Deficiency of factor IX
l Also known as Christmas disease
Section | I General Medicine 215

Von Willebrand Disease


a. Deficiency or abnormality of Von Willebrand factor
b. Von Willebrand factor acts as a carrier protein for factor VIII
c. It plays an important role in platelet adhesion by forming bridges between platelets and subendothelial
tissue
d. There is prolongation of bleeding time

Syncope (Fainting)
Types of syncope:

Vasovagal syncope l Due to reflex slowing of heart mediated through vagus, marked fall in arterial pressure
and peripheral vascular resistance.
l Causes—emotional stress, warm over-crowded room, sudden pain, mild blood loss,
anaemia, fever and fasting.
l Common in young people.
Cardiac syncope l Due to sudden reduction in cardiac output.

Orthostatic syncope l Due to postural hypertension resulting from loss of vasoconstrictor reflex in the lower
limb vessels.

Pulse
l Pulsus tardus—aortic stenosis
l Pulsus bisferiens—aortic regurgitation
l Pulsus alternans—left ventricular failure
l Pulsus paradoxus—asthma and pericardial tamponade

Types of Murmur
l Austin Flint murmur—mid-diastolic murmur in AR
l Carey Coombs murmur—mid-diastolic murmur in acute rheumatic fever
l Gibson’s murmur—‘train in tunnel’ murmur in PDA
l Graham-Steel murmur—early diastolic murmur of pulmonary regurgitation
l Still’s murmur—vibratory systolic innocent MI
l Musical or seagull—aortic regurgitation
l Mean’s murmur—harsh pulmonary systolic murmur in thyrotoxicosis
Some interesting facts to remember:
l Rise of JVP during inspiration is seen in constrictive pericarditis and is called Kussmaul’s sign.
l Ventricular septal defects (VSD) are the most common congenital defects.
l The first symptom of heart failure—dyspnoea.
l Both central and peripheral cyanoses occur in congestive cardiac failure.
l Normal cardiothoracic ratio should be less than 0.5 in radiographic examination.
l Commonest cause of ventricular tachycardia is acute myocardial infarction.
l Saphenous vein is used in coronal bypass grafting.
l CK-MB is more specific for myocardial damage.
l Commonest cause of death in MI is due to ventricular fibrillation.
l Coarctation of aorta results in notching of ribs.
l Bradycardia—pulse rate less than 60/min.
l Tachycardia—pulse rate more than l00/min.
l Hilar dance (fluoroscopy) is seen in atrial septal defect (ASD).
l Commonest congenital heart disease—VSD.
l Commonest cyanotic—congenital heart disease tetralogy of Fallot.
l Commonest site of coarctation of aorta junction of the arch with descending aorta.
l Trilogy of Fallot—PS 1 VSD 1 overriding of aorta.
l Tetralogy of Fallot—PS 1 VSD 1 overriding of aorta 1 right ventricular hypertrophy.
216 Quick Review Series for BDS 3rd Year

l Wide fixed split of second sound is seen in ASD.


l In mitral stenosis, no symptoms are seen until the valve area is reduced to 2.5 sq. cm.
l Calcification of ascending aorta is seen in cardiovascular syphilis.

RESPIRATORY SYSTEM
Types of Breathing
l Cheyne–Stokes breathing—apneoa followed by gradual increase in amplitude of respiratory movements
until they reach peak and subside into apnoeic phase. This is seen in central respiratory failure.
l Biot’s breathing—two or three inspiratory movements are imposed on expiratory movements. This is
seen in advanced central respiratory failure.
l Kussmaul breathing—deep inspiratory movements and expiratory movements are prolonged—seen in
diabetic acidosis.
l Wheezing sounds heard during inspirations are called rhonchi.
l Crackling sounds produced during inspiration is called crepitations. Inspiratory stridor is due to laryn-
geal obstruction.
l Expiratory stridor (wheeze) is due to bronchial obstruction.
l Common cold is caused due to rhinoviruses.

Sputum
l Pink frothy—pulmonary oedema
l Black sputum—aspergillosis, anthracosis
l Rusty sputum—pneumonia
l Yellow or purulent sputum—bronchiectasis or lung abscess
l Mucopurulent sputum—active bacterial infection
l Foul smell of sputum—lung abscess, bronchiectasis and gas gangrene
l Red or juicy—bronchial carcinoma

Various Appearances of Chest


l Pigeon chest or ‘keel’ breast deformity is seen in asthmatic children.
l Barrel-shaped chest is seen in emphysema.

Some Common Pneumoconioses


Causative agent Disease
Coal dust Coal workers’ pneumoconiosis
Silica Silicosis
Asbestos Asbestos-related diseases
Iron oxide Siderosis

Beryllium Berylliosis
Section | I General Medicine 217

CENTRAL NERVOUS SYSTEM


Signs of Upper and Lower Motor Neuron Lesions
Sign Upper motor neuron lesion Lower motor neuron lesion
Weakness Voluntary movements are disturbed Paralysis of muscles supplied by that
segment or nerve
Tone Hypertonia (clasp-knife spasticity) Hypotonia
Reflex (tendon) Increased, 6 clonus Decreased or absent
Reflex (superficial) Absent or decreased Absent or decreased
Plantar response Extensor Flexor or absent
Muscle nutrition Disuse atrophy Marked atrophy
Fasciculations Absent Present

Reaction of degeneration Absent Present

Trigeminal Neuralgia
l Occurs in the elderly; unknown aetiology.
l May be due to vascular compression at root-entry zone of Vth nerve.
l Maxillary and mandibular divisions are commonly affected.
l Paroxysmal, sharp pain confined to the distribution of the nerve.
l Pain is precipitated by touching trigger zones.
l Most important drug used in the treatment is carbamazepine.
After surgery (for trigeminal neuralgia), up to 80% of patients develop distorted sensation of the facial
skin (facial dysaesthesia) or a combination of anaesthesia and spontaneous pain (anaesthesia dolorosa).

Primary Headaches
Migraine and other types of primary headaches are not ‘vascular headaches’; these disorders do not reliably
manifest vascular changes and treatment outcomes cannot be predicted by vascular effects.
l Primary headaches are those in which headache and associated features are the disorder in itself.
l Cause considerable disability.

Structures involved
l Large intracranial vessels and dura mater.
l Peripheral terminals of trigeminal nerve that innervate these structures.

Types of headache
l Migraine
l Tension-type headache
l Cluster headache
l Idiopathic stabbing headache
l Exertional headache
218 Quick Review Series for BDS 3rd Year

Trigeminal autonomic cephalalgias


It is a group of primary headache disorders related to discharge of trigeminal nerve.
l Autonomic symptoms predominate in such headaches, e.g. lacrimation and nasal congestion.
l Headache occurs due to activation of cranial parasympathetic pathway.
l Vascular changes also seen secondary to parasympathetic activation (e.g. in migraine and cluster
headache) as revealed by functional imaging.
l However, such disorders do not qualify as vascular headaches.
l Include: cluster headache, paroxysmal hemicranias, etc.

Secondary Headaches
An exogenous disorder acts as a trigger.
l Due to systemic infection, e.g. meningitis
l Intracranial haemorrhage: SAH
l Brain tumour
l Vascular disorder: giant cell arthritis (temporal)

Benedikt Syndrome/Dorsal Midbrain Syndrome


l Results from a lesion in the red nucleus or tegmentum of midbrain.
l It is caused by occlusion of the paramedian/penetrating branches of the basilar or posterior cerebral
artery or both.

Signs
l Ipsilateral oculomotor paresis/ptosis and dilated pupil.
l Contralateral involuntary movement—tremor, chorea, athetosis.
l Contralateral hemiparesis may be present, if lesion extends ventrally.
l Contralateral hemianaesthesia may be present, if lesion extends laterally.

Other Midbrain Syndromes in Association with Injury to Fascicle/Nucleus


of CN III
Weber syndrome (injury Nothnagel syndrome (injury Claude syndrome (injury to red
to cerebral peduncle) to superior cerebellar peduncle) nucleus 1 superior cerebellar peduncle)
l Bilateral oculomotor palsy l Ipsilateral oculomotor palsy l Ipsilateral oculomotor palsy
l Contralateral hemiparesis l Contralateral cerebellar ataxia l Contralateral ataxia, tumour, chorea,
athetosis

Most commonly involved cranial nerves in head injury:


i. In head injury as a whole—Ist cranial nerve
ii. In anterior cranial fossa fracture—Ist cranial nerve
iii. In middle cranial fossa fracture—VIIth cranial nerve
iv. In posterior cranial fossa fracture—IXth, Xth, XIth cranial nerves

RENAL SYSTEM AND DISTURBANCES IN WATER, ELECTROLYTE


AND ACID–BASE BALANCE
Nephrotic syndrome is a clinical complex with number of renal and extrarenal features like hypoproteinae-
mia, hypoalbuminaemia, generalized oedema or anasarca, hypercholesterolaemia and hypercoagulability.
Section | I General Medicine 219

Causes of Nephrotic Syndrome


1. Primary glomerular disease
a. Minimal change disease
b. Focal segmental glomerulonephritis
c. Membranous glomerulonephritis
d. Membranoproliferative glomerulonephritis
e. Mesangiocapillary glomerulonephritis
2. Associated with systemic disease
a. Systemic lupus erythematosus
b. Polyarteritis nodosa
c. Amyloidosis
d. Diabetes mellitus
e. Henoch–Schonlein purpura
3. Associated with infection
a. Poststreptococcal glomerulonephritis
b. Endocarditis
c. Hepatitis B
d. Malaria
e. Syphilis
4. Associated with tumours/carcinoma
Hodgkin’s lymphoma
Leukaemia
5. Associated with drugs
a. Gold salts
b. Penicillamine
c. Captopril
d. Antitoxins
e. Antivenoms

Classification of Diuretics Based on the Efficacy of Action


I. High-efficacy (loop) diuretics:
Frusemide, bumetanide, piretanide, ethacrynic acid, mersalyl
Use: They are used in treatment of pulmonary, cardiac, renal and hepatic oedema, in poisoning with
barbiturates to enhance forced diuresis, in treatment of hypertension.
II. Moderate-efficacy diuretics:
i. Thiazides: Benzothiadiazines like chlorothiazide, hydrochlorothiazide, polythiazide, bendroflume-
thiazide
ii. Thiazide-related drugs: Chlorthalidone, clopamide, indapamide, metolazone, xipamide
Use: They are used in the treatment of mild hypertension, oedema due to congestive cardiac failure,
renal stones due to hypercalciuria, diabetes insipidus.
III. Low-efficacy diuretics:
i. Potassium-sparing diuretics: Triamterene, amiloride, spironolactone
Use: They are used along with other diuretics in hypertension to reduce loss of potassium and in
treatment of aldosteronism.
ii. Carbonic anhydrase inhibitors: Acetazolamide
Use: They are used to inhibit formation of aqueous humour and CSF, and reduce intraocular pres-
sure and intracranial pressure, respectively.
iii. Osmotic diuretics: Mannitol, urea, glycerol
Use: They are used in the treatment of cranial and intraocular pressure.
iv. Methylxanthines: Theophylline
220 Quick Review Series for BDS 3rd Year

ENDOCRINE AND METABOLIC DISORDERS AND NUTRITION


Various Inborn Errors of Metabolism
A. Carbohydrate metabolism
Glycogen storage disorders Deficiency
i. Type I von Gierke disease; glucose-6-phosphatase deficiency
ii. Type II Pompe disease; alpha glucosidase deficiency
iii. Type III Cori or Forbes disease; debranching enzymes deficiency

iv. Type VI Hers disease; hepatic phosphorylase deficiency

B. Lipid metabolism
Disease Deficiency
i. Niemann–Pick disease Sphingomyelinase (RBCs appear as foam cells)

ii. Tay–Sachs disease Hexosaminidase A

C. Amino acid metabolism


Disease Deficiency
i. Phenylketonuria Phenylalanine hydroxylase deficiency

Thyrotoxicosis
l A state of thyroid hormone excess.
l It is not synonymous with hyperthyroidism. It is a result of excessive thyroid function.

Causes of thyrotoxicosis
Primary hyperthyroidism, Graves’ disease
Toxic multinodular goitre
Toxic adenoma
Functioning thyroid carcinoma metastases
Drugs: Iodine excess (Jod–Basedow phenomenon)

Vitamins
Normal requirements of vitamin A and D (per day):
i. Vitamin A
(a) Children: 500–750 mcg
(b) Adults: 750 mcg
(c) Pregnancy and lactation: 1200 mcg
ii. Vitamin D
(a) Children below 5 years: 10 mcg 5 400 units
(b) Children above 5 years: 2.5 mcg 5 100 units
(c) Adults: 2.5 mcg 5 100 units
(d) Pregnancy and lactation: 10 mcg 5 400 units
Section | I General Medicine 221

INFECTIONS AND IMMUNOLOGICAL FACTORS IN DISEASE


(ANAPHYLAXIS AND DRUG ALLERGY)
Measles l Paramyxovirus infection; droplet infection.
l Koplik spots are seen in the oral cavity.
l Incubation period—7–14 days.
l Catarrhal stage is highly infective.
Rubella (German measles) l Togavirus infection; droplet infection.
l Incubation period—18 days.
l Congenital rubella—highest risk during first trimester.
Mumps l Paramyxovirus infection; droplet infection.
l Incubation period—14–18 days.
l Complications—epididymo-orchitis, oophoritis, encephalomyelitis, mumps
meningitis, mumps myocarditis.
Sheehan syndrome l Results from infarction of pituitary gland during postpartum period.

Waterhouse–Friderichsen syndrome l Characterized by acute haemorrhagic destruction of both the adrenal glands.
l Characterized by vasomotor collapse and shock.

Incubation Periods of Various Infections


Disease Incubation Period
(a) Anthrax 1–3 days
(b) Diphtheria 2–5 days
(c) Typhoid 5 days
(d) Tetanus 3–21 days
(e) Measles 10 days
(f) Chickenpox 2–3 weeks
(g) Rubella 2–3 weeks (18 days)
(h) Mumps 2–3 weeks (18 days)
(i) Hepatitis A 15–50 day

(j) Hepatitis B 50–160 days

DISEASES OF CONNECTIVE SYSTEM, BONES AND JOINTS


Some Important Radiological Appearances:
i. ‘Pepper pot’ or ‘salt pepper’ appearance—hyperparathyroidism
ii. Moth-eaten appearance—rickets around epiphysis
iii. Honeycomb appearance:
(a) Chronic osteomyelitis on X-ray
(b) Paget’s disease
(c) Osteoclastoma
(d) Adamantinoma
222 Quick Review Series for BDS 3rd Year

iv. ‘Ground glass’ appearance:


(a) Ossifying fibroma of PNS on X-ray
(b) Fibrous dysplasia of bones
(c) Asbestosis lung
(d) Hyaline membrane disease
v. ‘Burst open’ periosteum: Osteosarcoma

ACUTE POISONING AND ENVIRONMENTAL EMERGENCIES


AND MISCELLANEOUS
If the toxin can be identified, then a specific therapy antidote can be administered. However, antidotes are
available only for a few toxins, a few examples of antidotes are as follows:

Poison Antidote
Cholinesterase inhibitors (organophosphates, carbamates) Atropine
Organophosphates Pralidoxime
Opioids Naloxone
Hypoglycaemic agents Dextrose
Tricyclic antidepressants Sodium bicarbonate
Methanol, ethylene glycol Ethanol
Snake bites Snake antivenin
Iron Deferoxamine
Lead, arsenic, mercury BAL (dimercaprol)
Paracetamol N-acetylcysteine

Benzodiazepines Flumazenil
Section II

General Surgery

Topic 1 General Principles of Operative Surgery, Sterilization /Asepsis,


Diathermy, Cryosurgery and Lasers 225
Topic 2 Wounds, Sinus and Fistulae 232
Topic 3 Burns, Skin Grafting and Flaps 245
Topic 4 Haemorrhage and Shock 254
Topic 5 Bacterial Infections and Transmissible Viral Infections 269
Topic 6 Tumours, Cysts and Neck Swellings 287
Topic 7 Diseases of the Oral Cavity (Mouth, Tongue and Lips) 302
Topic 8 Diseases of Salivary Glands 316
Topic 9 Infections and Diseases of the Larynx and Nasopharynx 327
Topic 10 Diseases of Arteries, Veins and the Lymphatic System 335
Topic 11 Nervous System 353
Topic 12 Fractures—General Principles 364
Topic 13 Anomalies of Development of Face (Cleft Lip and Cleft Palate) 376
Topic 14 Thyroid and Parathyroid Glands 382
Topic 15 Local Anaesthesia, Biopsy and Miscellaneous 405
This page intentionally left blank
Section II

General Surgery

Topic 1

General Principles of Operative Surgery,


Sterilization/Asepsis, Diathermy, Cryosurgery
and Lasers
LONG ESSAYS
Q. 1. What is sterilization? What are the methods of B. Chemical methods:
sterilizing surgical materials? Mention the merits and i. Alcohols: Ethyl, isopropyl, triclorobutanol.
demerits. ii. Aldehydes: Formaldehyde, glutaraldehyde.
iii. Dyes
Ans.
iv. Halogens
Sterilisation is defined as the process by which an article, v. Phenols
surface or medium is freed of all living microorganisms vi. Surface active agents
either in the vegetative or spore state. vii. Metallic salts
The term sterilization has evolved to include the dis- viii. Gases: Ethylene oxide, formaldehyde, betapropio-
abling or destruction of infectious proteins such as prions. lactone
Various methods of sterilization are classified as
follows: Methods of Surgical Instrument Sterilization
A. Physical methods:
a. Sunlight In general, surgical instruments must be sterilized to a high
b. Drying sterility assurance level.
c. Dry heat:
i. Flaming Heat sterilization
ii. Incineration l The most widely-used method for heat sterilization in
iii. Hot air surgery is the autoclave. Autoclaves commonly use
d. Moist heating steam heated to 121–134°C (250–273°F).
i. Pasteurization l To achieve sterility, a holding time of at least 15 minutes
ii. Boiling at 121°C (250°F) or 3 minutes at 134°C (273°F) is
iii. Passing steam under normal pressure required. Additional sterilizing time is usually required
iv. Passing steam under pressure (autoclaving) for liquids and instruments packed in layers of cloth, as
e. Filtration by candles, asbestos pads, membranes they may take longer to reach the required temperature
f. Ionising radiations: X-rays, beta rays, gamma rays (unnecessary in machines that grind the contents prior
g. Ultrasonic and sonic vibrations to sterilization).

225
226 Quick Review Series for BDS 3rd Year

l Disadvantages: Proper autoclave treatment will inacti- l Besides moist heat and irradiation, ethylene oxide is the
vate all fungi, bacteria, viruses and also bacterial spores, most common sterilization method, used for over 70%
but will not necessarily eliminate all prions. of total sterilizations, and for 50% of all disposable
medical devices.
Dry heat sterilization l Ethylene oxide is still widely used by medical device
l This can be used to sterilize items, but as the heat takes manufacturers for larger scale sterilization (e.g. by the
much longer to be transferred to the organism, both pallet), but while still used, it is becoming less popular
the time and the temperature must usually be increased, in hospitals. It is now classified by the IARC as a known
unless forced ventilation of the hot air is used. The stan- human carcinogen.
dard setting for a hot air oven is at least two hours at
160°C (320°F). Glutaraldehyde and formaldehyde solutions
l Advantage: It can be used on powders and other heat- l They act as liquid sterilizing agents, but only if immer-
stable items that are adversely affected by steam for sion time is sufficiently long.
instance, it does not cause rusting of steel objects. l To kill all spores in a clear liquid can take up to 22 hours
with glutaraldehyde and even longer with formalde-
Glass bead sterilizer hyde.
l Once a common sterilization method employed in den- l Glutaraldehyde and formaldehyde are volatile, and
tal offices as well as biologic laboratories, is nowadays toxic by both skin contact and inhalation. Glutaralde-
not recommended. hyde has a short shelf life (,2 weeks), and is expensive.

Chemical sterilization Ortho-phthalaldehyde (OPA)


l Chemicals are also used for sterilization. Low tempera- l OPA is a chemical sterilizing agent, used in a 0.55%
ture gas sterilizers function by exposing the items to be solution, OPA shows better mycobactericidal activity
sterilized to high concentrations (typically 5 - 10% v/v) than glutaraldehyde.
of very reactive gases (alkylating agents such as ethyl- l It is also effective against glutaraldehyde-resistant
ene oxide, and oxidizing agents such as hydrogen spores. OPA has superior stability, is less volatile, and
peroxide and ozone). does not irritate skin or eyes, and it acts more quickly
l Liquid sterilants and high disinfectants typically include than glutaraldehyde.
oxidizing agents such as hydrogen peroxide and perace- l On the other hand, it is more expensive, and will stain
tic acid and aldehydes such as glutaraldehyde and more proteins (including skin) grey in colour.
recently o-phthalaldehyde.
l The chemicals used as sterilants are designed to destroy Hydrogen peroxide
a wide range of pathogens and typically the same prop- l It is relatively non-toxic when diluted to low concentra-
erties that make them good sterilants makes them harm- tions, such as 3% solutions although hydrogen perox-
ful to humans. ide is a dangerous oxidizer at high concentrations
(.10% w/w).
Ethylene oxide l Hydrogen peroxide is strong oxidant and these oxidiz-
l Ethylene oxide (EO or EtO) gas is commonly used to ing properties allow it to destroy a wide range of
sterilize objects sensitive to temperatures greater than pathogens and it is used to sterilize heat or temperature
60°C and/or radiation such as plastics, optics and sensitive articles such as rigid endoscopes.
electrics. l In medical sterilization, hydrogen peroxide is used at
l It can kill all known viruses, bacteria and fungi, includ- higher concentrations, ranging from around 35% up to
ing bacterial spores and is compatible with most mate- 90%. The biggest advantage of hydrogen peroxide as a
rials, e.g. medical devices, even when repeatedly sterilant is the short cycle time.
applied. However, it is highly flammable, toxic and l Hydrogen peroxide as sterilization agent has its disad-
carcinogenic. vantages too. Since hydrogen peroxide is a strong oxi-
l A typical process consists of a preconditioning phase, the dant, there are material compatibility issues and article
actual sterilization run and a period of post-sterilization to be sterilized should be compatible with this method
aeration to remove toxic residues, such as ethylene of sterilization.
oxide residues and by-products such as ethylene glycol l Hydrogen peroxide is primary irritant and the contact of
(formed out of EtO and ambient humidity) and the liquid solution with skin will cause bleaching or
ethylene chlorohydrine (formed out of EtO and materials ulceration depending on the concentration and contact
containing chlorine, such as PVC). time.
Section | II General Surgery 227

l The vapour is also hazardous with the target organs be- l Gamma rays are very penetrating and are commonly
ing the eyes and respiratory system. Prolonged exposure used for sterilization of disposable medical equipment,
to even low ppm concentrations can cause permanent such as syringes, needles, cannula and IV sets.
lung damage. l The main disadvantage of gamma radiation is that it
l Hydrogen peroxide can also be mixed with formic acid requires bulky shielding for the safety of the operators;
as needed in the Endoclens device for sterilization of it also requires storage of a radioisotope (usually
endoscopes. Cobalt-60), which continuously emits gamma rays.
l Electron beam processing is also commonly used for
Dry sterilization process (DSP) medical device sterilization. Electron beams use an on-
off technology and provide a much higher dosing rate
l Dry sterilization process is useful for medical and
than gamma or X-rays.
pharmaceutical applications. It uses hydrogen peroxide
l Due to the higher dose rate, less exposure time is needed
at a concentration of 30-35% under low pressure
and thereby any potential degradation to polymers is
conditions. This process achieves bacterial reduction
reduced. The limitation of electron beams is its less
of 1026 . . . 1028.
penetrating power.
l The complete process cycle time is just 6 seconds,
l UV irradiation is useful only for sterilization of surfaces
and the surface temperature is increased only 10-15°C
and some transparent objects. Many objects that are
(18 to 27°F).
transparent to visible light absorb UV.
l UV irradiation is routinely used to sterilize the interiors
Radiation sterilization of biological safety cabinets between uses, but is inef-
l Methods of sterilization exist using radiation such as fective in shaded areas, including areas under dirt.
electron beams, X-rays, gamma rays, or subatomic It also damages some plastics, such as polystyrene
particles. foam, if exposed for prolonged periods of time.

SHORT ESSAYS
Q. 1. Autoclave sterilization. Temperature Air-steam outlet
Pressure gauge and safety valve
Ans. gauge
l Autoclave sterilization is a type of heat sterilization Water tank
which uses an instrument called autoclave.
l Autoclaving is the process of sterilization by saturated
Chamber
steam under high pressure, above 100°C.
l A device to sterilize equipment by subjecting them to
high pressure saturated steam at 121°C or more, typi- Gravity filled Temperature
boiler with
cally for 15–20 minutes depending on the size of the immersion
controller
load and the contents. heater
l It was invented by Charles Chamberland. Autoclave Valve for water entry
sterilization is widely used in microbiology, medicine, and water plus
steam discharge
tattooing, bodypiercing, veterinary science, mycology,
and dentistry. FIGURE 1.1 A simple autoclave

Principle
Autoclave l Water boils when its vapour pressure equals that of the
l The simplest form of autoclave (Fig 1.1) consists of surrounding atmosphere.
a vertical or horizontal cylinder made up of gunmetal l Hence pressure inside a closed vessel increases, the
or stainless steel in a supporting sheet iron case temperature at which water boils also increases to more
(double jacketed). than 100°C.
l The lid or the door is fastened by screw clamps and l This saturated steam has greater penetrative power.
made airtight by a suitable washer. The safety valve can When steam comes into contact with a cooler surface it
be set to blow off at any desired pressure. condenses to water and gives up its latent heat to that
l The heating is done by gas or electricity. surface.
228 Quick Review Series for BDS 3rd Year

l The condensed water ensures the moist conditions for l The first stage of absorption has a linear rate, lasting for
killing of the microbes present. several days to weeks. The second stage is characterized
by loss of suture mass and overlaps the first stage.
Chemical treatments, such as chromic salts, lengthen
Sterilization Time l

the absorption time.


l Usual sterilization time for an autoclave is as follows:
The absorbable surgical sutures are made from either
i. 121°C at 15 lb per square inch for 15-20 minutes.
“natural” or synthetic polymers.
ii. 126°C at 20 lb per square inch for 10 minutes.
iii. 134°C at 30 lb per square inch for 3 minutes. Natural
i. Collagen:
Uses l The collagen sutures are derived from the submuco-

sal layer of bovine small intestine or the serosal


Common articles sterilized in autoclave are:
layer bovine small intestine “gut.”
l Culture media
ii. Surgical gut, plain:
l Rubber articles like tubes, gloves, etc.
l Tensile strength is maintained for 7-10 days postim-
l Syringes and surgical instruments
plantation and absorption is complete within 70
l OT gowns, dressing materials
days.
l Endodontic instruments
l This type of suture is used for repairing rapidly
l Hand instruments, etc.
healing tissues that require minimal support and
Q. 2. Absorbable suture materials. ligating superficial blood vessels.
l Surgical gut, fast-absorbing: This type of suture is
Ans.
indicated for epidermal use (required only for 5-7 d)
l Sutures are classified as either absorbable or non- and is not recommended for internal use.
absorbable depending on whether the body will naturally iii. Surgical gut, chromic (treated with chromium salt):
degrade and absorb the suture material over time. l Tensile strength is maintained for 10-14 days.

The absorption rate is slowed by chromium salt


(90 days).
Sutures
l Tissue reaction is due to the noncollagenous mate-

rial present in these sutures. Also, patient factors


affect rates of absorption and make tensile strength
somewhat unpredictable.
Absorbable Nonabsorbable l Disadvantages of natural fibre absorbable sutures

are as follows:
a. First, these natural fibre absorbable sutures have
a tendency to fray during knot construction.
Natural Synthetic b. Second, there is considerably more variability in
• Collagen • Polyglactin 910 (Vicryl) their retention of tensile strength than is found
• Surgical gut, plain • Poliglecaprone 25 (Monocryl) with the synthetic absorbable sutures.
• Surgical gut, chromic • Polysorb
• Polydioxanone (PDS II)
Synthetic
• Caprosyn
Chemical polymers are absorbed by hydrolysis and cause a
lesser degree of tissue reaction following placement.

l Absorbable sutures are broken down by various pro- i. Polyglactin 910 (Vicryl):
cesses including hydrolysis (polyglycolic acid) and l This is a braided multifilament synthetic suture coated
proteolytic enzymatic degradation. Depending on the with a copolymer of lactide and glycolide (polyglactin
material, the process can be from ten days to eight 370).
weeks. l The water-repelling quality of lactide slows loss of ten-
l They are used in patients who cannot return for suture sile strength, and the bulkiness of lactide leads to rapid
removal, or in internal body tissues. In both cases, they absorption of suture mass once tensile strength is lost.
will hold the body tissues together long enough to allow l The suture is also coated with calcium stearate, which
healing, but will disintegrate so that they do not leave permits easy tissue passage, precise knot placement,
foreign material or require further procedures. and smooth tie-down.
Section | II General Surgery 229

l Tensile strength is approximately 65% at 14 days Q. 3. Cryosurgery in oral cavity.


postimplantation. Absorption is minimal for 40 days
Ans.
and complete in 56-70 days. These sutures cause only
minimal tissue reaction. l Cryotherapy is the deliberate destruction of tissue by
l Vicryl sutures are used in general soft tissue approxima- application of extreme cold.
tion and vessel ligation. l It is well received by patients due to a relative lack of
discomfort, the absence of bleeding and minimal to no
ii. Poliglecaprone 25 (Monocryl): scarring after healing.
l This is a monofilament synthetic suture that is a copoly- l It has many applications in oral medicine and clinical
mer of glycolide and e-caprolactone. oral pathology, and is extremely useful in patients for
l The suture has superior pliability, leading to ease in whom surgery is contraindicated due to either age or
handling and tying. Tensile strength is high initially, medical history.
50-60% at 7 days, and is lost at 21 days. l Various methods have been devised in the use of cryo-
l Absorption is complete at 91-119 days. Poliglecaprone therapy of lesions, they include:
25 sutures are used for subcuticular closure and soft a. the spray freeze technique,
tissue approximations and ligations. b. the applicator technique,
c. the cryoprobe method, and
iii. Polysorb: d. the thermocoupler method.
l Copolymers of glycolide and lactide were synthesized l Cryotherapy is used for the treatment of keratotic, hy-
to produce a Lactomer copolymer that is used to pro- perplastic, granulomatous, vascular, pigmented lesions,
duce a new braided absorbable suture, Polysorb. and salivary gland lesions as well as for gingival lesions.
l The handling characteristics of the Polysorb sutures l Stomatologic purposes need special cryodevices with
were found to be superior to those of the Polyglactin temperature not less than 2150°C, where complicated
910 suture. topographic and anatomic relationships of all com-
l The Polysorb suture is coated with an absorbable mix- plexes of periodontal tissues and tunica mucosa of
ture of caprolactone/glycolide copolymer and calcium mouth are considered in construction of cryoinstrument
stearoyl lactylate to decrease its coefficient friction. working part, and cryoagent supply should be calcu-
l Absorption is essentially complete between 56 and 70 lated for the long time of application.
days. l Main periodontal diseases that can be treated by cryo-
surgery are parodontosis, hypertrophic gingivitis,
iv. Polydioxanone (PDS II): papillitis, epulis, and pericoronitis.
l This is a polyester monofilament suture made of l Practically, there are no contraindications to cryother-
polydioxanone. This suture provides extended wound apy. It is expedient to use cryosurgery at the diseases
support and elicits only a slight tissue reaction. bound to disturbance of a coagulating property of a
l Tensile strength is 70% at 14 days and 25% at 42 days. blood, nephroses, an endocrine pathology, the general
Wound support remains for up to 6 weeks. neurosis with cardiovascular responses.
l Absorption is minimal for the first 90 days and essen- l Cryotherapy in many diseases of tunica mucosa of
tially complete within 6 months. It has a low affinity for mouth, both in elderly and in young age is not only
microorganisms. method of choice, but expedient necessity in compari-
l PDS II suture is used for soft tissue approximation, es- son with surgical treatment or electrocoagulation.
pecially in paediatric, cardiovascular, gynaecologic, l Cryosurgery allows destroying tumoural or any other
ophthalmic, plastic, and digestive (colonic) situations. pathological tissue in strictly defined limits, thus depth
l Another similar suture material is made from polytri- of tissue destruction can be controlled precisely enough.
methylene carbonate (Maxon). This material has a l The focus of cryodestruction is always clearly delimited
similar tensile strength and absorption profile. from intact tissues. Cryosurgery excludes mechanical
trauma of tissues (abruption, dissection) thereof tumour
v. Caprosyn: cells dissemination is excluded. The possibility of com-
l The latest innovation in the development of monofila- plications is negligible. Side effects are not observed.
ment absorbable sutures has been the rapidly absorbing
Caprosyn suture. Q. 4. Chemical sterilization.
l Caprosyn monofilament synthetic absorbable sutures Ans.
are prepared from Polyglytone 621 synthetic polyester,
which is composed of glycolide, caprolactone, trimeth- l In chemical sterilization, low temperature gas steriliz-
ylene carbonate, and lactide. ers function by exposing the items to be sterilized to
230 Quick Review Series for BDS 3rd Year

high concentrations (typically 5 - 10% v/v) of very although hydrogen peroxide is a dangerous oxidizer
reactive gases (alkylating agents such as ethylene ox- at high concentrations (.10% w/w).
ide, and oxidizing agents such as hydrogen peroxide ii. Hydrogen peroxide is strong oxidant and these
and ozone). oxidizing properties allow it to destroy a wide
l Liquid sterilants and high disinfectants typically include range of pathogens and it is used to sterilize
oxidizing agents such as hydrogen peroxide and perace- heat or temperature sensitive articles such as rigid
tic acid and aldehydes such as glutaraldehyde and more endoscopes.
recently o-phthalaldehyde. iii. In medical sterilization, hydrogen peroxide is used
l The chemicals used as sterilants are designed to destroy at higher concentrations, ranging from around 35%
a wide range of pathogens and typically the same to 90%.
properties that make them good sterilants also make iv. The biggest advantage of hydrogen peroxide as a
them harmful to humans. sterilant is the short cycle time.
l The various chemical sterilizing agents are as follows: v. Hydrogen peroxide can also be mixed with formic
acid as needed in the Endoclens device for steriliza-
i. Ethylene oxide (EO or EtO) gas: tion of endoscopes.
l It is commonly used to sterilize objects sensitive to tem- l The disadvantages of hydrogen peroxide as sterilization
peratures greater than 60°C and/or radiation such as agent are:
plastics, optics and electrics. i. Since hydrogen peroxide is a strong oxidant, there
l It can kill all known viruses, bacteria and fungi, includ- are material compatibility issues and article to be
ing bacterial spores and is compatible with most materi- sterilized should be compatible with this method of
als (e.g. medical devices), even when repeatedly sterilization.
applied. ii. Paper products cannot be sterilized in the Sterrad
l It is highly flammable, toxic and carcinogenic. system because of a process called cellulostics, in
which the hydrogen peroxide would be completely
ii. Glutaraldehyde and Formaldehyde: absorbed by the paper product.
l Glutaraldehyde and formaldehyde solutions act as iii. The penetrating ability of hydrogen peroxide
liquid sterilizing agents, but only if immersion time is is not as good as ethylene oxide and so there
sufficiently long. are limitations on the length and diameter of
l To kill all spores in a clear liquid can take up to 22 hours lumens that can be effectively sterilized and
with glutaraldehyde and even longer with formaldehyde. guidance is available from the sterilizer manufac-
l Glutaraldehyde and formaldehyde are volatile, and turers.
toxic by both skin contact and inhalation. Glutaralde- iv. Hydrogen peroxide is primary irritant and the
hyde has a short shelf life (,2 weeks), and is contact of the liquid solution with skin will cause
expensive. bleaching or ulceration depending on the concentra-
tion and contact time.
iii. Ortho-phthalaldehyde (OPA): v. The vapor is also hazardous with the target organs
l OPA is a chemical sterilizing agent, used in a 0.55% being the eyes and respiratory system. Prolonged
solution. exposure to even low ppm concentrations can cause
l OPA shows better mycobactericidal activity than glutar- permanent lung damage
aldehyde. It also is effective against glutaraldehyde-re-
sistant spores.
l OPA has superior stability, is less volatile, and does Dry sterilization process (DSP)
not irritate skin or eyes, and it acts more quickly than i. It uses hydrogen peroxide at a concentration of 30-35%
glutaraldehyde. under low pressure conditions.
l On the other hand, it is more expensive, and will stain ii. This process achieves bacterial reduction of
proteins including skin to grey colour. 10 -6-10 -8.
iii. The complete process cycle time is just 6 seconds, and
iv. Hydrogen peroxide: the surface temperature is increased only 10-15°C
l The advantages of hydrogen peroxide are as follows: (18 to 27°F).
i. It is relatively non-toxic when diluted to low concen- iv. Dry sterilization process is useful for medical and
trations, such as the familiar 3% retail solutions pharmaceutical applications.
Section | II General Surgery 231

SHORT NOTES
Q. 1. Antisepsis. ii. In other words, it is the process or act of destroying
pathogenic microorganisms. Disinfection does not
Ans.
necessarily kill all microorganisms, especially non-
i. Antisepsis is defined as the prevention of infection by resistant bacterial spores.
inhibiting or arresting the growth and multiplication iii. It is less effective than sterilization, which is an extreme
of germs (infectious agents). Antiseptics are the sub- physical and / or chemical process that kills all types of life.
stances that kill or prevent the growth of the microor- iv. A perfect disinfectant would also offer complete and
ganisms. The name comes from the Greek words anti full sterilization, without harming other forms of life,
(against) and sepsis (decay). be inexpensive, and non-corrosive. Unfortunately, ideal
ii. English physician Sir John Pringle (1707-1782) first disinfectants do not exist.
used the word antiseptic and an English surgeon Joseph Q. 4. Cidex (Glutaraldehyde).
Lister (1827-1912) applied his knowledge of bacteria to
develop a successful system of antiseptic surgery. Ans.
iii. Aseptic methods such as sterilization prevent bacteria i. Cidex is one of the brand names for glutaraldehyde, a
from existing in a given area. product is used to disinfect fragile surgical instruments,
iv. Some common antiseptics are: Alcohol {ethanol (60–
most commonly for endoscopy instruments.
90%), 1-propanol (60–70%) and 2-propanol/isopropanol
ii. It is a toxic colourless, oily liquid also available as an
(70–80%)}, boric acid, brilliant green, chlorhexidine
aqueous solution.
gluconate, hydrogen pereoxide, iodine, etc.
iii. Glutaraldehyde is harmful if inhaled or swallowed and
Q. 2. Asepsis in surgery. irritating to eyes and respiratory tract. It can also cause
severe damage to the skin and eyes.
Ans.
Q. 5. Autoclave sterilization.
i. Asepsis is the state of being free from disease-causing
contaminants such as bacteria, viruses, fungi, and para- Ans.
sites or, preventing contact with microorganisms. i. Autoclave sterilization uses a device—autoclave, to
ii. Elimination of infection is the goal of asepsis, not sterility. sterilize equipment and supplies by subjecting them to
The widespread introduction of antiseptic surgical meth- high pressure saturated steam at 121°C or more, typi-
ods followed the publishing of the Paper Antiseptic Prin- cally for 15–20 minutes depending on the size of the
ciple of the Practice of Surgery in 1867 by Joseph Lister. load and the contents.
iii. The modern concept of asepsis evolved in the 19th ii. It was invented by Charles Chamberland.
century. Semmelweis showed that washing the hands iii. Autoclave sterilization is widely used in microbiology, med-
prior to delivery reduced puerperal fever. icine, tattooing, bodypiercing, veterinary science, mycology,
iv. Baron Lister introduced the use of carbolic acid as an dentistry, chiropody and prosthetic fabrication. This means
antiseptic and reduced surgical infection rates. Abraham that all bacteria, viruses, fungi, and spores are inactivated.
Groves of Fergus, Ontario, Canada, was one of the first iv. However, prions may not be destroyed by autoclaving
surgeons who used boiled water to wash his hands and at the typical 134°C for 3 minutes or 121°C for 15 min-
the patient’s incision site and to use heat sterilization by utes. Also, some recently-discovered organisms, such
boiling the surgical instruments and sponges in 1873. as Strain 121, can survive at temperatures above 121°C.
v. Lawson Tait went from antisepsis to asepsis, introducing v. As damp heat is used, heat-labile products (such as some
principles and practices that have remained valid to this plastics) cannot be sterilized this way or they will melt.
day. Ernst von Bergmann introduced the autoclave for Autoclaving is often used to sterilize medical waste prior
the practice of the sterilization of surgical instruments. to disposal in the standard municipal solid waste stream.
Q. 3. Disinfection. Q. 6. Sterilization of hot air.
Ans. Ans.
i. Disinfection is considered to be the primary mechanism i. Hot air sterilization is a method of sterilization using dry
for the inactivation/destruction of pathogenic organ- heat. The temperatures of 160-165°C are generated for
isms to prevent the spread of diseases. at least 2 hours. The device used is called hot air oven.
232 Quick Review Series for BDS 3rd Year

ii. The ovens use dry heat to sterilize articles. Generally, transmissible agents (such as fungi, bacteria, viruses,
they can be operated from 50 to 300°C (122 to 572°F). spore forms, etc.) present on a surface, contained in a
iii. They do not require water and there is not much pres- fluid, in medication, or in a compound such as biologi-
sure build up within the ovens, unlike an autoclave, cal culture media.
making them safer to work with. This also makes them ii. Sterilization can be achieved by applying the proper
more suitable to be used in a laboratory environment. combinations of heat, chemicals, irradiation, high
iv. They are much smaller than autoclaves but can still be pressure, and filtration, i.e. heat sterilization, chemi-
as effective. They can be more rapid than an autoclave cal sterilization, dry sterilization process, radiation
and higher temperatures can be reached compared to sterilization.
other means.
Q. 9. Diathermy.
v. As they use dry heat instead of moist heat, some organ-
isms like prions may not be killed by them every time. Ans.
Hot air sterilization is widely used to sterilize articles
i. Diathermy means “electrically induced heat” and refers
that can withstand high temperatures and not get burnt,
to use of high frequency electric current to produce
like glassware and powders. Linen gets burnt and
heat, used to either cut or destroy tissue or to produce
surgical sharps lose their sharpness.
coagulation.
Q. 7. Incineration. ii. Electrical frequency used by diathermy is in the range
of 300 kHz to 3 MHz, where patients body forms part
Ans.
of the electrical circuit. Diathermy can be monopolar or
i. Incineration is a waste treatment process that involves bipolar.
the combustion of organic substances contained in iii. Effects of diathermy are as follows:
waste materials. a. Coagulation: Is produced by interrupted pulses
ii. Incineration of waste materials converts the waste into of current (50-100 per second) and square wave-
ash, flue gas, and heat. form.
iii. Incineration has particularly strong benefits for the treat- b. Cutting: Produced by continuous current of sinus
ment of certain waste types in niche areas such as clinical wave-form.
wastes and certain hazardous wastes where pathogens
The effects of diathermy depend on the current intensity
and toxins can be destroyed by high temperatures.
and wave-form used.
iv. An incinerator is a furnace for burning waste. Modern
incinerators include pollution mitigation equipment
such as flue gas cleaning. Risks and Complications
v. There are various types of incinerator plant design:
l Can interfere with pacemaker function.
Moving grate, fixed grate, rotary-kiln, and fluidised bed.
l Arcing can occur with metal instruments and implants.
Q. 8. Sterilization. l Superficial burns, if use spirit-based skin preparation.
l Diathermy burns under indifferent electrode, if plate
Ans.
improperly applied.
i. Sterilization is a term referring to any process that l Channeling effects, if used on viscus with narrow
eliminates (removes) or kills all forms of life, including pedicle (e.g. penis or testis).

Topic 2

Wounds, Sinus and Fistulae


LONG ESSAYS
Q. 1. Describe the types of wounds and the manage- Wounds which are encountered in surgical practice can be
ment? classified according to many criteria like:
I. The morphology,
Ans.
II. Thickness,
Section | II General Surgery 233

III. The involvement of skin or other structures, II. According to thickness of the wound
IV. The time elapsing from the trauma and i. Superficial wounds: They involve the epidermis and
V. Depending on the bacterial contamination of the dermis up to the dermal papillae.
wound. ii. Partial-thickness wounds: They involve up to the
lower dermis. (Part of the skin remains and shafts of
Classification of Wounds hair follicles and sweat glands are leftover.)
iii. Full-thickness wounds: They involve the skin and the
I. According to Morphology
subcutaneous tissue.
A) Open wound: This is a wound in which the skin’s con- iv. Deep wounds, including complicated wounds (e.g.
tinuity is interrupted or broken, exposing the under- with laceration of blood vessels and nerves), wounds
neath tissue thus creating a pathway between environ- penetrating into natural cavities, and wounds penetrat-
ment and injury site. They can be further subdivided ing into an organ or tissue.
into:
i. Incised wounds, caused by a clean, sharp-edged III. According to involvement of other structures
object such as a knife, a razor. This wound can i. Simple wounds, comprising only one organ or tissue
bleed a lot and quickly, and deep ones may damage ii. Combined wounds (e.g. in mixed tissue trauma).
tendons and muscles.
ii. Abrasions (grazes): They are superficial wounds in IV. According to time elapsed post-trauma
which the top most layer of the skin (the epider-
i. Fresh wounds, up to 8 hours from the trauma.
mis) is scraped off and are often caused by a slid-
ii. Old wounds, after 8 hours from trauma or skin discon-
ing fall on to a rough surface. Bleeding is of ooz-
tinuity.
ing character.
iii. Lacerations: They are irregular tear-like wounds
V. According to bacterial contamination of
caused by some blunt trauma or contact with a
sharp object. Bleeding may be more brisk or wound
severe. i. Clean wounds: They usually refer to wounds made
iv. Puncture wounds: They are caused by an object by the doctor during an operation or under sterile
puncturing the skin, such as a nail or needle. Bleed- conditions.
ing is usually minimal, and the wound may be ii. Clean-contaminated wounds: In such wounds, the
barely noticeable. contamination of clean wounds is endogenous and
v. Penetration wounds: They are caused by an object comes from the environment, the surgical team, or the
such as a knife entering and coming out from the patient’s skin surrounding the wound.
skin. iii. Contaminated wounds: Such wounds have large con-
vi. Gunshot wounds: They are caused by a bullet or taminates infect the wound.
similar projectile driving into or through the iv. Dirty wounds: In this case, contamination comes from
body. They are usually characterized by one at the established infection.
the site of entry and one at the site of exit, re- Wounds can also be classified as chronic and acute.
ferred to as a “through-and-through.” They are l Chronic wounds are those that are caused by a rela-
heavy bleeding. tively slow process that leads to tissue damage. Chronic
vii. Avulsions: The tearing of tissues from a bullet wounds include pressure, venous, and diabetic ulcers).
wound, animal bite or explosion is considered an l Acute or traumatic wounds are the result of injuries
avulsion. These wounds are often deep with quick that disrupt the tissue.
and heavy blood loss.
B) Closed wounds: In this type of wound, there exists no Management of Wounds
open pathway between the outer environment and the
injury site. They are further subdivided into: The management depends on the type, cause, and depth of
i. Crush wounds: They are caused by a great or ex- the wound as well as whether other structure beyond the
treme amount of force applied over a long period of skin involved, time post-trauma, etc.
time or by a heavy blow of a tool.
ii. Contusions, more commonly known as bruises, A) Open wound’s management
caused by a blunt force trauma that damage tissue l Involves applying pressure (to stop bleeding), cleaning
under the skin. (sterile saline solution/antiseptic solution/hydrogen per-
iii. Haematomas, also called a blood tumour, caused oxide), dressing, stitching and medication (antibiotics).
by damage to a blood vessel that in turn causes l Treatment of lacerations involves examination, cleaning,
blood to collect under the skin. trimming of edges and closing the wound. If the wounds
234 Quick Review Series for BDS 3rd Year

are fresh and less than 12 hours old, they can be closed The phase begins with haemostasis and formation of the
with sutures or staples. Any wound which is more than platelet plug.
24 hours old should be suspected to be contaminated and l Injury to vascular tissue initiates the extrinsic coagu-
not closed completely. Only the deeper tissues can be lation cascade by releasing intracellular calcium and
approximated and the skin should be left open, tissue factor that activate factor VII. The resulting fi-
l Abrasions will heal on their own with skin discoloura- brin plug achieves haemostasis aided by reflex vaso-
tion usually disappears in 1-2 weeks and require no ac- constriction.
tive treatment except keeping the area clean with soap l This plug acts as a lattice for the aggregation of plate-
and water. lets, the most common and “signature” cell type of the
l Puncture wounds may be prone to infection depending early inflammatory phase.
on the depth of penetration. The entry of puncture l In the inflammatory phase, bacteria and debris are
wound is left open to allow for bacteria or debris to be phagocytosed and removed, and factors are released that
removed from inside. cause the migration and division of cells involved in the
l Bites may not be sutured unless a large area is involved proliferative phase.
or the bite involves the face. Many of these become in- l Collagen which were exposed during wound formation
fected and thus are preferred to be left open for daily activates the clotting cascade (both the intrinsic and
wound care. Antibiotics for bite wounds are a must. extrinsic pathways), initiating the inflammatory phase.
l Injured tissues, through activated phospholipase A, si-
B) Closed wound’s management multaneously catalyze arachidonic acids to produce
l Contusions: Rest the injured site, apply ice or a cold potent vasoconstrictors thromboxane A2 and prostaglan-
compress for 10 to 15 minutes every two hours to close din 2-alpha, collectively known as eicosanoids. This
blood vessels, compress or wrap firmly with a bandage initial response helps to limit haemorrhage.
to control swelling, elevate above the heart to prevent l After a short period, capillary vasodilatation occurs
pooling blood in the area. secondary to local histamine release, and the cells of
l Haematomas: The treatment principles are similar to inflammation are able to migrate to the wound bed.
contusion management. Pressure in the form of an elas- l Platelets release platelet-derived growth factor (PDGF)
tic adhesive bandage may be helpful to reduce haemor- and transforming growth factor beta (TGF-b) from their
rhage and swelling. Antibiotics are recommended. alpha granules to attract neutrophils and macrophages.
l Crush wounds: Do not attempt to move the injured l Neutrophils scavenge for bacteria and foreign debris.
person. Treat shock, if present, by keeping the person l Macrophages are the most important mediators of
warm. Start compression on the injury. Medical treat- wound healing. Macrophages continue to emit growth
ments include administration of IV fluids, hyperbaric factors to attract fibroblasts and usher in the next phase
oxygen chamber regiment and amputation. of wound healing.
Q. 2. Discuss the pathology and healing of wounds. ii. Proliferative phase
Ans. l It begins on approximately day 3; it overlaps with the
inflammatory phase. This phase is characterized by an-
The pathology of wound healing in both acute and chronic
giogenesis, collagen deposition, granulation tissue for-
wounds constitutes a cascade of events which can be sum-
mation, and epithelialization.
marized as:
l Fibroblasts are responsible for initiating angiogenesis,
l Inflammation.
epithelialization, and collagen formation. Fibroblasts
l Granulation tissue formation and revascularization.
migrate inward from wound margins over the fibrinous
l Epithelialization.
l Wound contraction. matrix established during this phase.
l During the first week, fibroblasts begin producing
l Scar formation.
glycosaminoglycans and proteoglycans, the ground
The whole pathology of wound healing is divided into substance for granulation tissue, as well as collagen,
three phases: in response to macrophage-synthesized FGF and
i. Inflammatory phase. TGF-b, as well as PDG. Fibroblasts grow and form
ii. Proliferative phase. a new, provisional extracellular matrix (ECM) by
iii. Remodeling phase. excreting collagen and fibronectin.
l Re-epithelialization of the epidermis occurs, in which
i. Inflammatory phase epithelial cells proliferate and ‘crawl’ at the top of the
l It begins at the time of injury and lasts 2-4 days. This wound bed, providing cover for the new tissue. This
phase is characterized by haemostasis and inflammation. process starts from the basement membrane, if the
Section | II General Surgery 235

basement membrane remains intact, otherwise the pro- Q. 3. Discuss the stages of wound healing for closed and
cess initiates from the wound edges. open wounds. What are the factors affecting wound
l Fibroblasts produce mainly type III collagen during this healing?
phase.
Ans.
l Degradation of the fibrin clot and provisional matrix is
accompanied by the deposition of granulation tissue, The process of wound healing is broadly divided into
which continues until the wound is covered. Granula- three stages:
tion tissue, formed in this phase, is important in wounds i. Inflammatory phase.
healing, especially healing by secondary intention. ii. Proliferative phase.
iii. Remodelling phase.
iii. Remodelling
l It continues for 6-12 months after injury. i. Inflammatory phase
l Occurs remodelling of collagen, i.e. type III collagen is l It begins at the time of injury and lasts 2-4 days. This
replaced by type I collagen until a type I: type II ratio of phase is characterized by haemostasis and inflamma-
4:1 is reached. tion. The phase begins with haemostasis and formation
l Collagen reorganizes along lines of tension and cross- of the platelet plug.
links, giving added strength. Tensile strength is 80% of l Injury to vascular tissue initiates the extrinsic coagula-
the original strength approximately 1 year post-injury. tion cascade by releasing intracellular calcium and tissue
l Fibroblasts differentiate into myofibroblasts, causing factor that activate factor VII. The resulting fibrin plug
tissue contraction during this phase of wound healing. achieves haemostasis aided by reflex vasoconstriction.
l Vascularity decreases, producing a less hyperaemic and l This plug acts as a lattice for the aggregation of plate-
more cosmetically appealing wound as this phase pro- lets, the most common and “signature” cell type of the
gresses. early inflammatory phase.
l In the inflammatory phase, bacteria and debris are
Healing can be of three types:
phagocytosed and removed, and factors are released that
i. Healing by primary intention (primary healing) cause the migration and division of cells involved in the
l The wound is surgically closed by reconstruction of the proliferative phase.
skin continuity by simple suturing, by movement (relo- l Collagen which were exposed during wound formation
cation) of skin fragments from the surrounding area activates the clotting cascade (both the intrinsic and
(flaps), or by transplantation of free skin elements extrinsic pathways), initiating the inflammatory phase.
(grafts) of different thickness (e.g. split- or full-thick- l Injured tissues, through activated phospholipase A, si-
ness grafts). multaneously catalyze arachidonic acids to produce
l Primary healing is usually the case in all wounds in potent vasoconstrictors thromboxane A2 and prostaglan-
which the anatomical location and the size allow the din 2-alpha, collectively known as eicosanoids. This
skin continuity to be restored. initial response helps to limit haemorrhage.
l After a short period, capillary vasodilatation occurs
ii. Healing by second intention (secondary healing) secondary to local histamine release, and the cells of
l After wound debridement and preparation, the wound inflammation are able to migrate to the wound bed.
is left open to achieve sufficient granulation for spon- l Platelets release platelet-derived growth factor (PDGF)
taneous closure (re-epithelialization from remaining and transforming growth factor beta (TGF-b) from their
dermal elements [e.g. hair follicle] or from wound alpha granules to attract neutrophils and macrophages.
borders). l Neutrophils scavenge for bacteria and foreign debris.
l Secondary healing is how abrasions or split-thickness l Macrophages are the most important mediators of
graft donor sites heal. wound healing. Macrophages continue to emit growth
factors to attract fibroblasts and usher in the next phase
iii. Healing by third intention (tertiary healing/ of wound healing.
delayed primary closure)
l After wound debridement and preparation (i.e. treat- ii. Proliferative phase
ment of local infection), the wound is left open and then l It begins on approximately day 3; it overlaps with the
closed by primary intention or finally by surgical means inflammatory phase. This phase is characterized by an-
of skin grafting. giogenesis, collagen deposition, granulation tissue for-
l Tertiary healing is how primary contaminated wounds mation, and epithelialization,
or mixed tissue trauma wounds (e.g. after reconstruction l Fibroblasts are responsible for initiating angiogenesis,
of hard tissue) heal. epithelialization, and collagen formation. Fibroblasts
236 Quick Review Series for BDS 3rd Year

migrate inward from wound margins over the fibrinous l Jaundice


matrix established during this phase. During the first l Malignancy
week, fibroblasts begin producing glycosaminoglycans l Malnutrition
and proteoglycans, the ground substance for granulation l Obesity
tissue, as well as collagen, in response to macrophage- l Temperature
synthesized FGF and TGF-b, as well as PDGF. Fibro- l Hypoxia and hypovolaemia,
blasts grow and form a new, provisional extracellular l Vitamin deficiency and trace metal deficiency.
matrix (ECM) by excreting collagen and fibronectin.
Q. 4. Discuss the factors which influence the wound
l Re-epithelialization of the epidermis occurs, in which
healing.
epithelial cells proliferate and ‘crawl’ atop the wound
bed, providing cover for the new tissue. This process Ans.
starts from the basement membrane if the basement
Factors which affect wound healing are as follows:
membrane remains intact, otherwise the process initi-
a) Local factors: Blood supply/oxygen supply, denerva-
ates from the wound edges.
tion (decreased wound contraction), infection, mechani-
l Fibroblasts produce mainly type III collagen during this
cal stress, pressure, types of tissue, surgical technique
phase.
and material.
l Degradation of the fibrin clot and provisional matrix is
b) General factors: Age, anaemias (decreased blood sup-
accompanied by the deposition of granulation tissue
ply), NSIADs, cytotoxic drugs, corticosteroids, system-
(ground substance, collagen, capillaries), which contin-
atic infection, jaundice (impaired clotting factor re-
ues until the wound is covered. Granulation tissue,
lease), malignancy, malnutrition, obesity, temperature,
formed in this phase, is important in wounds healing,
hypoxia, hypovolaemia, vitamin deficiency, and trace
especially healing by secondary intention.
metal deficiency.
iii. Remodeling
i. Hypoxia (decrease in PO)
l It continues for 6-12 months after injury.
l Adequate oxygen level in healing wound is required for
l Occurs remodelling of collagen, i.e. type III collagen is
collagen synthesis, proline hydroxylation, collagen se-
replaced by type I collagen until a type I: type II ratio of
cretion, for leucocyte killing capacity (phagosomal per-
4:1 is reached. Collagen reorganizes along lines of ten-
oxidation), for growth factor secretion and fibroblast
sion and crosslinks, giving added strength. Tensile
proliferation (requires adequate perfusion).
strength is 80% of the original strength approximately 1
l ability to supply oxygen to healing wound process is in-
year post-injury.
hibited by peripheral vascular disease, previous radiation,
l Fibroblasts differentiate into myofibroblasts, causing
chronic inflammation, diabetes mellitus, and infection.
tissue contraction during this phase of wound healing.
l Vascularity decreases, producing a less hyperaemic and ii. Mechanical stress
more cosmetically appealing wound as this phase pro-
l Affects quantity, aggregation and orientation of colla-
gresses.
gen fibres.

Factors Affecting Wound Healing iii. Corticosteroids


l Inhibit macrophage function, decrease inflammatory
Local factors response and inhibit prolyl/lysyl hydroxylase – unstable
l Blood supply collagen bonds.
l Denervation l Inactivate complement system and lead to T and B cell
l Infection dysfunction.
l Mechanical stress l Decrease leucocyte bactericidal activity.
l Types of tissue
l Surgical technique and material. iv. Cytotoxic drugs/anti-neoplastic agents
l They decrease WBCs, fibroblast proliferation, wound
General Factors contraction, and protein synthesis.
l Age l Colchicine – decreases collagen precursors, decreases
l Anaemias collagen secretion (cellular constipation), increases ac-
l NSIADs tivity of collagenases.
l Cytotoxic drugs and corticosteroids l penicillamine – enhanced affinity of chelating ligands
l Systematic infection for calcium ion required for collagen linking.
Section | II General Surgery 237

v. NSIADs xiii. Infection


l Decrease collagen synthesis. l Pus lowers local pO2, has collagenolytic effect, prolongs
inflammatory phase, and inhibits re-epithelialization.
vi. Vitamin E
l Decreases collagen synthesis, inhibits wound healing xiv. Foreign body
and decreases tensile strength. l Clot, necrotic debris, dirt, suture, glass, etc. prolong
inflammatory phase, lead to increased susceptibility for
vii. Malnutrition infection and increased duration of re-epithelialization.
l Catabolic state (net -ve nitrogen balance) inhibits wound
healing. xv. Radiation therapy
l Protein depletion prolongs inflammatory phase, and l Acute radiation changes cause vascular stasis and asso-
impairs fibroplasia. ciated decreased wound healing.
l Methionine deficiency: Impaired formation of disulfide l Chronic radiation changes include irreversible damage
bonds in collagen synthesis. to skin with progressive obliterative endarteritis and an
inability for fibroblasts to replicate or contract resulting
viii. Age in progressive skin changes and inability to heal minor
l Associated with delayed onset of healing, protraction of wounds, the wound is chronically hypoxic.
phases and an inability to reach same level of healing.
l Associated with decreased tensile strength and wound xvi. Temperature
closure rate. l Environment temperature greater than 30°C increases
tensile strength.
ix. Diabetes mellitus
xvii. Pressure
l Affects wound healing in numerous ways due to
associated peripheral vascular disease, neuropathy, im- l Undue pressure may lead to ischaemia of the wound,
muno-dysfunction (decreases chemotaxis, phagocytosis, therefore, all wounds, particularly chronic wounds,
intracellular killing), and decreases collagen synthesis. should be off-loaded.
Q. 5. What are the factors which delay the wound
x. Associated illnesses
healing?
l Cancer, infection, peripheral vascular disease, pulmo-
nary dieases, renal pathology, obesity smoking-all nega- Ans.
tively affect wound healing. The factors which delay wound healing can be summa-
rized as:
xi. Vitamin deficency
Vitamin A Corticosteroids, cytotoxic drugs/anti-neoplastic agents,
l Co-factor collagen synthesis and X-linking-increases NSAIDs, malnutrition, vitamin E, age, diabetes mellitus,
wound strength. hypovolaemia, hypotension, vasoconstriction, oedema, vas-
l Capable of reversing inhibitory effects of steroids and cular disease, infection, obesity, stress, haematopoietic
vitamin E. conditions, malignancy, sepsis, irradiation, renal diseases,
and pulmonary diseases.
Vitamin C
l Co-factor for proline hydroxylation and its deficiency A) Corticosteroids
causes capillary fragility and wound healing delay. l Inhibit macrophage function, decrease inflammatory
response
xii. Trace metals deficency l Inhibit prolyl/lysyl hydroxylase-unstable collagen bonds
Zinc l Inactivate complement
l Enzyme constituent l Lead to T and B cell dysfunction
l Cofactor for collagen synthesis l Decrease leucocyte bactericidal activity.

Copper B) Cytotoxic drugs/anti-neoplastic agents


l Extracellular cofactor l Decrease WBCs, decrease fibroblast proliferation, de-
l Required for collagen crosslinking crease wound contraction, and decrease protein synthesis.
l colchicine–decreases collagen precursors, decreases
Magnesium collagen secretion (cellular constipation), and increases
l Cofactor in glycolization activity of collagenases.
238 Quick Review Series for BDS 3rd Year

l Penicillamine – enhanced affinity of chelating ligands L) Mechanical stress


for calcium ion required for collagen linking. l Affects quantity, aggregation and orientation of colla-
gen fibres.
C) NSIADs
l Decrease collagen synthesis. M) Radiation therapy
D) Vitamin E l Acute radiation changes cause vascular stasis and asso-
ciated decreased wound healing.
l Decreases collagen synthesis, inhibits wound healing
l Chronic radiation changes include irreversible dam-
and decreases tensile strength.
age to skin with progressive obliterative endarteritis
E) Malnutrition and an inability for fibroblasts to replicate or contract
resulting in progressive skin changes and inability
l Catabolic state (net -ve nitrogen balance) inhibits wound
to heal minor wounds, the wound is chronically
healing.
hypoxic.
l Protein depletion – Prolongs inflammatory phase, and
impairs fibroplasia. Q. 6. Classify wound infections. Write briefly about
l Methionine deficiency: Impaired formation of disulfide hospital-acquired infections and their prevention.
bonds in collagen synthesis.
Ans.
F) Age
l Wound infection can be defined as the deposition and
l Associated with delayed onset of healing, protraction of
multiplication of bacteria in tissue with an associated
phases and an inability to reach same level of healing.
host reaction.
l Associated with decreased tensile strength and wound
l Wound infections continue to represent a major medical
closure rate.
problem, both in terms of how they affect the outcome
G) Diabetes mellitus of surgical procedures and for their impact on the length
of hospital stay and medical costs.
l Affects wound healing in numerous ways due to associ-
ated peripheral vascular disease, neuropathy, immuno- Wound infections in surgery can be classified into:
dysfunction (decreases chemotaxis, phagocytosis, i. Superficial wound infections: They are superficial and
intracellular killing), and decreases collagen synthesis. involve only the skin and subcutaneous tissue of the
incision, with physical findings of inflammation.
H) Associated illnesses ii. Deep wound infection: These are also defined at 30
l Cancer, infection, peripheral vascular disease, Pulmo- days post-surgery or at one year, if an implant is in-
nary dieases, renal pathology, obesity smoking - all volved, and involve infection present in the deep soft
negatively affect wound healing tissues of the incision.
iii. Organ/space SSIs: They involve any part of the anat-
I) Hypoxia (decrease in PO) omy other than the incision itself.
l Adequate oxygen level in healing wound is required for
collagen synthesis, proline hydroxylation, collagen se- Major pathogens causing wound infections
cretion, for leucocyte killing capacity (phagosomal per-
l Staphylococcus aureus, coagulase-negative staphy­
oxidation), for growth factor secretion and fibroblast
lococci, enterococci, Escherichia coli, Pseudomonas
proliferation and fibroblast proliferation (requires ade-
aeruginosa, Enterobacter species, Proteus mirabilis,
quate perfusion).
Klebsiella pneumoniae.
l Ability to supply oxygen to healing wound process is in-
hibited by peripheral vascular disease, previous radiation,
chronic inflammation, diabetes mellitus, and infection. Wound Contamination
J) Infection There are a number of ways in which micro-organisms can
l Pus lowers local pO2, has collagenolytic effect, prolongs gain access to a wound:
inflammatory phase, and inhibits re-epithelialization. l Direct contact: Transfer from equipment or the hands of
carriers.
K) Foreign body l Airborne dispersal: Micro-organisms deposited from
l Clot, necrotic debris, dirt, suture, glass, etc. prolong in- the surrounding air.
flammatory phase, lead to increased susceptibility for and l Self-contamination: Physical migration from the pa-
infection and increased duration of re-epithelialization tient’s skin or gastrointestinal tract.
Section | II General Surgery 239

Clinical Picture of Wound Infection l Conditional pathogens that could cause disease
(other than simple localized infections) only in per-
The classic signs of infection include:
sons with lowered resistance to infection or when
l Localised erythema
implanted directly into tissue or normally sterile
l Localised pain
area.
l Localised heat
l Opportunistic pathogens that could cause generalized
l Cellulitus
disease, but only those patients who have a greatly di-
l Oedema.
minished resistance to infection.
Further criteria include:
l Abscess Prevention of Nosocomial infections
l Discharge which may be viscous in nature, discoloured l Isolation
and purulent. l Handwashing and gloving
l Delayed healing not previously anticipated. l Surface sanitation: Modern sanitizing methods such
l Discolouration of tissues both within and at the wound as NAV-CO2 have been effective against gastroenteri-
margins. tis, MRSA, and influenza. Use of hydrogen peroxide
l Friable, bleeding granulation tissue despite gentle han- vapour has been clinically proven to reduce infection
dling of and the non-adhesive nature of wound manage- rates and risk of acquisition. Hydrogen peroxide is
ment materials used. effective against endospore-forming bacteria, such as
l Unexpected pain and/or tenderness either at the time of Clostridium difficile, where alcohol has been shown to
dressing change or reported by the patient as associated be ineffective.
specifically with the wound even when the wound l Aprons
dressing is in place. l Mitigation
l Abnormal smell. l Safe injection practices
l Wound that was assessed as healing starts to develop l Preventing transmission from the environment
strips of granulation tissue in the base as opposed to a l Cleaning of the hospital environment
uniform spread of granulation tissue across the whole of l Use of hot/superheated water
the wound bed. l Disinfection of patient equipment
Treatment l Sterilization

Antibiotics, iodine, silver application, autolytic or enzymatic Q. 7. Discuss about bleeding wounds of the face.
debridement, surgical debridement, and maggot therapy. Ans.
Hospital acquired infections or Nosocomial Wounds of the face which can result in bleeding can be
infections classified as:
l Nosocomial infections are defined as infections that are a
Incisions
result of treatment in a hospital or a healthcare service unit.
l They include any clinical infection that was neither l Straight edges to the wound margins, as if sliced with a
present nor was in its incubation period when the patient knife. These can vary in size, and may be caused by a
entered the hospital. variety of objects, including a scalpel, a knife, any piece
l Infections are considered nosocomial, if they first of straight, sharp metal, or a piece of glass.
appear 48 hours or more after hospital admission or l Tissue is rarely missing from the wound site, and the
within 30 days after discharge. margins of the wound may be easily matched from one
side of the wound to the other for the purposes of
The most common types of nosocomial infections that closure.
could occur in a hospital set up are:
1. Surgical wound and other soft tissue infections. Laceration
2. Urinary tract infections l Jagged edges to the wound margins, more closely re-
3. Respiratory infections sembling a tear than a slice.
4. Gastroenteritis l The wounded tissue is random rather than straight in
5. Meningitis direction, and may have multiple branches.
The causative microorganisms may be broadly classi- l Most often caused by an object with a broken or ser-
fied into the following categories: rated edge, such as a piece of broken glass or metal, but
l Conventional pathogens that could cause disease in healthy may also be caused by a blow from a blunt object to
persons in the absence of any specific immunity to them. tissue with bone immediately behind it.
240 Quick Review Series for BDS 3rd Year

Puncture l Blood oozes out of these vessels into the spaces be-
l Sharp object penetrates the tissue, and travels internally, tween cells or interstitial space, causing swelling and
but does not move laterally in any direction from the discolouration.
point of entry. l Blood loss is generally limited, and not of serious con-
l Such wounds can be misleading, as they may appear sequence.
quite small on surface examination, but extend quite
deeply into the body; even damaging nerves, blood ves- Avulsion
sels, or internal organs. l A full thickness laceration-type wound, often semi-
Abrasion circular in shape.
l This creates a flap which, when lifted, exposes the
l A scraping or scratching. Generally quite superficial, deeper tissues to view, or extrudes them from the wound
and affecting only the surface layers of the epidermis. itself.
Contusion l Avulsions in face often occur in the orbit of the eye.
l Simple bruising. In this type of injury, the capillaries in l Avulsions are difficult to repair, and no avulsion should
the epidermis and dermis are damaged, without break- ever be considered a minor injury.
ing the skin.

SHORT ESSAYS
Q. 1. Discuss briefly about factors affecting wound iv. Cytotoxic drugs/anti-neoplastic agents
healing. l They decrease WBCs, fibroblast proliferation, wound
Ans. contraction, and protein synthesis.

Factors which affect wound healing are as follows: v. NSIADs


a. Local factors: Blood supply/oxygen supply, denerva- l Decrease collagen synthesis.
tion (decreased wound contraction), infection, mechani-
cal stress, pressure, types of tissue, surgical technique vi. Vitamin E
and material. l Decreases collagen synthesis, inhibits wound healing
b. General factors: Age, anaemias, NSIADs, cytotoxic and decreases tensile strength.
drugs, corticosteroids, systematic infection, jaundice ma-
lignancy, malnutrition, obesity, temperature, hypoxia, hy- vii. Malnutrition
povolaemia, vitamin deficiency, and trace metal deficiency. l Catabolic state (net -ve nitrogen balance) inhibits wound
healing.
i. Hypoxia (decrease in PO) l Protein depletion - prolongs inflammatory phase, and
l Adequate oxygen level in healing wound is required for impairs fibroplasia.
collagen synthesis and other processes. l Methionine deficiency: Impaired formation of disulfide
l Ability to supply oxygen to healing wound process is in- bonds in collagen synthesis.
hibited by peripheral vascular disease, previous radiation,
chronic inflammation, diabetes mellitus, and infection. viii. Age
l Associated with delayed onset of healing, protrac-
ii. Mechanical stress tion of phases and an inability to reach same level of
l Affects quantity, aggregation and orientation of colla- healing.
gen fibres. l Associated with decreased tensile strength and wound
closure rate.
iii. Corticosteroids
l Inhibit macrophage function, decrease inflammatory ix. Diabetes mellitus
response and inhibit prolyl/lysyl hydroxylase - unstable l Affects wound healing in numerous ways due to
collagen bonds. associated peripheral vascular disease, neuropathy,
l Inactivate complement system and lead to T and B cell immunodysfunction (decreases chemotaxis, phago-
dysfunction. cytosis, intracellular killing), and decreases collagen
l Decrease leucocyte bactericidal activity. synthesis.
Section | II General Surgery 241

x. Associated illnesses l In this type of wound healing, union of accurately


l Cancer, infection, peripheral vascular disease, pulmo- coated edges of a wound occurs, with an irreducible
nary dieases, renal pathology, obesity smoking - all minimum of granulation tissue.
negatively affect wound healing. l The wound which heals with primary intention has
following characteristics:
xi. Vitamin A and C deficiency: a. Uninfected and clean
Inhibits wound healing. b. Surgical incision
c. Minimum loss of tissues
xii. Trace metals deficency: d. Wound edges are approximated by surgical sutures.
Zinc, copper, magnesium deficiency delays wound healing.
The sequence of events in healing by primary intentions
xiii. Infection: is:
l Pus lowers local pO2, has collagenolytic effect, pro-
longs inflammatory phase, inhibits re-epithelialization. i. Bleeding/haemorrhage
l After injury, the space which exists between the sur-
xiv. Foreign body: faces of the incised wound, approximated by sutures are
l Clot, necrotic debris, dirt, suture, glass, etc. prolong filled by blood.
inflammatory phase, lead to increased susceptibility for l Immediately after injury, disruption of blood vessel ex-
infection and increased duration of re-epithelialization. poses the subendothelial collagen to platelets, resulting
in platelet stimulation and aggregation.
Q. 2. Discuss briefly about delayed wound healing. l Simultaneously clotting factors, released by the injured
Ans. cells, activate the coagulation pathways, resulting in the
formation of clot. These blood clots, sealing the wound
i. Delayed wound healing or healing by tertiary healing and protecting it from dehydration and infection.
is a combination of primary and secondary intentions.
ii. Healing is allowed to occur via secondary intention so ii. Acute inflammation
that the wound can contract and create healthy granulation l It occurs within 24 hrs and characterized by poly-
tissue before the wound is closed via primary intention. morphs.
iii. Delayed wound healing is a characteristic of grossly l Within 10-15 minutes after injury, vasoconstriction is
contaminated wound and is useful for managing replaced by vasodilation and increased capillary perme-
wounds that are too heavily contaminated for primary ability, mediated by vasoactive substances released by
closure but appear clean and well vascularized after damaged cells and clot breakdown.
4-5 days of open observation. l The dilated capillary allows plasma and blood cells to
iv. Over this time, the inflammatory process has reduced leak into the wound area resulting in delivery of multi-
the bacterial concentration of the wound to allow safe ple phagocytic cells (leucocytes) to the wound area
closure. which scavenges for bacteria and foreign debris.
v. In subsequent repair of a wound initially left open or l Macrophages continue to release growth factors to at-
not previously treated, tertiary or delayed healing is tract fibroblasts and usher in the next phase of wound
expected. healing.
vi. For infected or unhealthy wounds with high bacterial
content, wounds with a long time lapse since injury, or
iii. Proliferation
wounds with a severe crush component with signifi-
l Basal cells epidermis of both wound margins proliferate
cant tissue devitalization.
and migrate towards the incision space in the form of
vii. It is often used for infected wounds where bacterial
spurs.
count contraindicates primary closure and the inflam-
matory process can be left to débribe the wound.
iv. Remodelling
viii. Wound edges are approximated within 3-4 days and
l This involves continuous matrix breakdown and new
tensile strength develops as with primary closure.
matrix synthesis.
Q. 3. Discuss briefly about healing by primary intention. l Characterized by invasion of wound area by fibroblasts
on 3rd day, followed by,
Ans.
l formation of new collagen fibers by 5th day and
l Healing by primary intention is defined as healing by l formation of scar tissue with scanty cellular and vascu-
fibrous adhesion, without suppuration or formation of lar elements, some inflammatory cells and epithelialised
granulation tissue. surface by 4th week.
242 Quick Review Series for BDS 3rd Year

Contraindications for healing by primary intention Pathogenesis


l An acute wound .6 hours old (with the exception of l Diabetic ulcers are formed due to neuropathic impairment
facial wounds) of musculoskeletal balance as well as immune compro-
l Foreign debris in the wound that cannot be completely mise from leucocyte dysfunction and peripheral vascular
removed disease, complicating these wounds with infection.
l Active oozing of blood from the wound l Pressure and tissue trauma are major promoting factors,
l Dead space under the skin closure but the neuropathy from the primary disease is the most
l Too much tension on the wound important element.
Q. 4. Discuss briefly about healing by secondary intention. l About 60 to 70% of diabetic ulcers are due to neuropa-
thy, 15 to 20% are due to ischaemia, and another 15 to
Ans. 20% are due to a combination of both.
l Healing by secondary intention is characterized by union l The neuropathy is both sensory and motor, and is sec-
of two wound surfaces by granulation and suppuration. ondary to persistently elevated glucose levels.
l The wound which heals with secondary intention has l The loss of sensory function allows unrecognized injury to
following characteristics: occur from ill-fitting shoes, foreign bodies, or other trauma,
a. Extensive loss of tissues and cells resulting in increased mechanical stress under the metatarsal
b. Open, large tissue defect heads, heels, and callosities. This leads to intermittent or
c. Dehisced surgical wounds continuous ischaemia, resulting in pressure ulceration.
d. Wounds caused by underlying morbidities (pressure l The motor neuropathy or Charcot foot leads to collapse
necrosis, chronic venous insufficiency) or dislocation of the interphalangeal or metatarsopha-
langeal joints, causing pressure on areas with little pro-
The sequence of events in healing by secondary inten- tection. There is also severe micro- and macrovascular
tions is: circulatory impairment.
l Absence/minimal haemorrhage: Bleeding and hemo-
stasis are absent or minimal in healing by secondary
intention, which compromises the healing process, as Treatment
the clot formation, its breakdown and subsequent re- The treatment of diabetic wounds involves local and sys-
lease of growth factors are important for proper healing. temic measures:
The benefit of surgical debridement is that it causes l Standard of care includes off-loading, attentive debride-
bleeding and initiates the repair process. ment, maintenance of a moist wound environment, and,
l Prolonged inflammation: This phase is prolonged due to when cellulitis is present, systemic antibiotics is recom-
the fact that the necrotic tissues and bacterial load in mended.
wounds subject to secondary healing is more. Thus inflam- l Achievement of adequate blood sugar levels is very
mation is considerably longer than 3 days as in the case of important.
healing by primary intention and continues until all ne- l Most diabetic ulcers are infected, and eradication of the
crotic tissues are cleared and bacterial load is controlled. infectious source is paramount to the success of healing.
l Prolonged proliferation: This phase differs from the l Treatment should address the possible presence of os-
sequences in primary healing. In primary healing, first teomyelitis, and should employ antibiotics that achieve
occurs epithelialization, followed by angiogenesis and adequate levels both in soft tissue and bone.
formation of small amount of granulation tissues and l Wide debridement of all necrotic or infected tissue.
contraction. However, in healing by secondary inten- l Off-loading of the ulcerated area by using specialized
tion, proliferation phase begins with granulation forma- orthotic shoes or casts allows for ambulation while pro-
tion (to fill the tissue defect), contraction (to minimize tecting the fragile wound environment.
the defect) and finally delayed epithelialization. l Topical application of PDGF and granulocyte-macrophage
l Prolonged remodelling: The remodelling or organi- colony-stimulating factor has met with limited but signifi-
zation process is similar to that in primary intention cant success in achieving closure.
healing, but is more prolonged, with tensile strength l Prevention and, specifically, foot care play an important
increasing in 2-3 months. role in the management of diabetic ulcers.
Q. 5. Diabetic ulcer. Q. 6. Preauricular sinus.
Ans. Ans.
Diabetic ulcers are chronic ulcers in diabetics typically l Preauricular sinus is a benign congenital malformation
presenting as foot ulcers. of the preauricular soft tissues. It is variably also termed
Section | II General Surgery 243

a preauricular pit, preauricular fistula, preauricular tract l The standard technique for extirpation of the sinus tract
and preauricular cyst. involves an incision around the sinus and subsequent
l Preauricular sinuses are inherited in an incomplete au- dissection of the tract to the cyst near the helix.
tosomal dominant pattern. They can arise spontane- l A supposedly more successful technique is the supra-
ously. The sinus may be bilateral or unilateral. auricular approach.
Q. 7. What is sinus and fistula? What are the causes for
Pathophysiology fistula?
Preauricular sinuses result from incomplete fusion of 2 of the Ans.
6 hillocks that arise from the first and second branchial arches.
Sinus
Clinical Features l In surgical practice, sinus refers to chronically infected
l Most people with preauricular sinuses are asymptomatic. tract such as a passage between an abscess and the skin.
l A small pit is often noted adjacent to the external ear
usually located at the anterior margin of the ascending
Fistula
limb of the helix.
l The opening of the preauricular sinus has also been re- l It is an abnormal connection or passageway between
ported along the posterosuperior margin of the helix, the two epithelium-lined organs or vessels that normally do
tragus or the lobule. not connect. It is generally a disease condition, but a
l The visible pit may represent the full extent of the de- fistula may be surgically created for therapeutic reasons.
formity, or mark a sinus tract that can vary in length, l Various types of fistulas include:
branch and follow a tortuous course. a. Blind: With only one open end.
l Some patients with preauricular sinuses present with b. Complete: With both external and internal openings.
chronic intermittent drainage of purulent material from c. Incomplete: A fistula with an external skin opening,
the opening. Draining sinuses are prone to infection. which does not connect to any internal organ.
Once infected, these sinuses rarely remain asymptom-
The causes of fistulas are varied.
atic, often developing recurrent acute exacerbations.
l Diseases can cause fistulas, as can certain medical treat-
l Patients with preauricular sinuses may present with fa-
ments and trauma or injury to the body.
cial cellulitis or ulcerations located anterior to the ear.
l Conditions such as Crohn’s disease and colitis are com-
mon inflammatory bowel diseases that cause anorectal
Treatment fistulas.
l Trauma to the body, such as prolonged, severe child-
l Once infection occurs, the likelihood of recurrent acute
birth, which can cause obstetric fistulas, can also be a
exacerbations is high, and the sinus tract should be sur-
culprit.
gically removed.
l The formation of fistulas as a result of severe childbirth
l Surgery should take place once the infection has been
occurs most often in parts of the world where profes-
treated with antibiotics and the inflammation has had
sional medical care is either poor or non-existent.
time to subside.

SHORT NOTES
Q. 1. Delayed healing. that are too heavily contaminated for primary closure
but appear clean and well vascularized after 4-5 days of
Ans.
open observation.
l Delayed healing or healing by tertiary intention is a
method of closing a grossly contaminated wound in Q. 2. Secondary healing.
which the wound is left open until contamination has Ans.
been markedly reduced and inflammation has subsided
and then is closed by first intention. l Secondary healing can be defined as union by closure of
l It is a combination of primary and secondary intentions. a wound with granulations.
l Delayed wound healing is a characteristic of grossly l Such healing occurs in open wounds where wound
contaminated wound and is useful for managing wounds edges are not approximated and healing occurs with
244 Quick Review Series for BDS 3rd Year

formation of granulation tissue, contraction and even- ii. Nutrition/malnutrition and


tual spontaneous migration of epithelial cells. iii. Age
l Secondary healing is how abrasions or split-thickness
Q. 7. Reactionary haemorrhage.
graft donor sites heal.
Ans.
Q. 3. Hypertrophic scar.
l Reactionary haemorrhage is defined as the bleeding that
Ans.
occurs within 24 hours (usually 4-6 hours) after injury
l Hypertrophic scar is defined as a raised, erythematous, or operation.
pruritic lesion that remains within the confines of the l It is mainly due to rolling (slipping) of a ligature, dis-
original scar. lodgement of a clot or cessation of reflex vasospasm.
l Histologically consists of collagen fibres arranged in l The precipitating factors of reactionary bleeding are:
nodules containing myofibroblasts and increased den- a. The rise in blood pressure and the refilling of the
sity of blood vessels. venous system on recovery from shock;
l This elevated scar resembles a keloid but does not b. Restlessness, coughing and vomiting which raise the
spread into surrounding tissues and regresses spontane- venous pressure (e.g. reactionary venous haemor-
ously. They are most common over joints. rhage within a few hours of thyroidectomy).
Q. 4. Healing by delayed primary intention. Q. 8. Sinus.
Ans. Ans.
l Delayed healing or healing by tertiary intention is a l In surgical practice, sinus refers to chronically infected
method of closing a grossly contaminated wound in tract such as a passage between an abscess and the skin.
which the wound is left open until contamination has l In anatomy, the term sinus is used in various contexts:
been markedly reduced and inflammation has subsided as a sack or cavity in any organ or tissue or an abnormal
and then is closed by first intention. cavity or passage caused by the destruction of tissue.
l It is a combination of primary and secondary intentions. l In common usage, “sinus” usually refers to the parana-
l Delayed wound healing is a characteristic of grossly sal sinuses, which are air cavities in the cranial bones,
contaminated wound and is useful for managing wounds especially those near the nose and connecting to it.
that are too heavily contaminated for primary closure
Q. 9. Fistula.
but appear clean and well vascularized after 4-5 days of
open observation. Ans.
Q. 5. Keloid. l Fistula is an abnormal connection or passageway be-
tween two epithelium-lined organs or vessels that nor-
Ans.
mally do not connect.
l Keloids are extreme overgrowth of scar tissue beyond l It is generally a disease condition, but a fistula may be
the limits of the original wound, with no tendency of surgically created for therapeutic reasons.
resolving and formed due to overgrowth of granulation l Various types of fistulas include:
tissue (collagen type III) at the site of a healed skin in- a. Blind: With only one open end.
jury which is then slowly replaced by collagen type I. b. Complete: With both external and internal openings.
l A keloid scar is benign, non-contagious, and sometimes c. Incomplete: A fistula with an external skin opening,
accompanied by severe itchiness and pain and changes which does not connect to any internal organ.
in texture. l Treatment: Depends on the cause and extent of the fis-
l Keloid scars are common in Afro-Caribbean and Orien- tula, but often involves surgical intervention combined
tal races. with antibiotics.
l The most common sites of keloid formation are central
Q. 10. Diabetic ulcer.
chest, back, shoulders and ear lobes.
l They are treated by application of pressure, inter- Ans.
lesional steroid (triamcinolone) injection and surgery.
l Diabetic ulcers are chronic ulcers in diabetics typically
Q. 6. Name three factors affecting wound healing. presenting as foot ulcers.
l Pressure and tissue trauma are major promoting factors,
Ans.
but the neuropathy from the primary disease is the most
Three factors which can effect wound healing are: important element.
i. Impaired blood supply (hypoperfusion)/inadequate ox- l The lack of sensation results in increased mechanical
ygen supply (hypoxia). stress under the metatarsal heads, heels, and callosities.
Section | II General Surgery 245

This leads to intermittent or continuous ischaemia, l These ulcers have punched out edge with slough in the
resulting in pressure ulceration. floor thus resembling a gummatous ulcer, e.g. bedsore
l Diabetics are also prone to angiopathy that interferes and perforating ulcers.
with the healing response. Pathogenesis is due to neuro- l Commonly seen in the heel and the ball of the foot in
pathic impairment of musculoskeletal balance as well as ambulatory patients and on the buttock and on the back
immune compromise from leucocyte dysfunction and of the heel in non-ambulatory patients.
peripheral vascular disease, complicating these wounds l These ulcers start with callosity under which suppu-
with infection. ration takes place, the pus comes out and the
l Standard of care includes off-loading, attentive debride- central hole forms the ulcer which gradually burrows
ment, maintenance of a moist wound environment, and, through the muscles and tendons to the bone. The
when cellulitis is present, systemic antibiotics. resulting is a callous ulcer with punched out corny
edge.
Q. 11. Trophic ulcer.
Ans. Q. 12. Snail track ulcer.
l Trophic ulcers can be defined as a pressure sore (decu- Ans.
bitus ulcer) caused by outer injury to a part of the body
that is in poor condition caused by disease, low levels of l Snail track ulcers are oral ulcers in patients who have
blood flow, i.e vascular insufficiency or loss of afferent secondary syphilis.
nerve fibres that carry signals to the brain. l These ulcers present themselves as multiple linear
l Trophic ulcers may be painless or associated with se- ulcers on the sides and undersurface of the tongue.
vere causalgia. l They appear 3-6 weeks after the chancre develops.

Topic 3

Burns, Skin Grafting and Flaps

LONG ESSAYS
Q. 1. Enumerate the classification of burns. l Contact burns: These burns result from contact of
the skin with hot items, including flames.
Ans.
b) Chemical burns: These burns result from contact of the
Burns can be classified by mechanism of injury, depth, skin with chemicals, or by ingestion of chemicals.
extent and associated injuries and comorbidities. c) Electrical burns: These types of burns result when a
person comes into contact with a source of electrical
energy; includes burns caused by electrocution and
According to Mechanism of Injury lightning strike.
Burns may be caused by several different mechanisms, d) Radiation burns: These types of burns result from
each with its own complications: contact with a source of radiation; may include overex-
a) Thermal burns: Consisting of flame, contact, or scald burns. posure, i.e. sunburn.
l Flame burns: This is primarily related to their as-

sociation with structural fires and the accompanying


According to Burn Depth
inhalation injury and/or CO poisoning. It is the most
common cause for hospital admission of burns and Currently, burns are described according to the depth of
also have the highest mortality. injury to the dermis and are loosely classified into first,
l Scalds: These types of burns result when skin comes second, third and fourth degrees.
into contact with hot liquids (spilled liquids or food, This system was devised by the French barber-surgeon
hot bathwater). Ambroise Pare and remains in use today.
246 Quick Review Series for BDS 3rd Year

I. First degree burns: A. Major burns


They involve epidermis only. This degree of burns mani- Features of major burns are as follows:
fests itself with erythema, painful, dry texture and heals in l Age 10-50 yrs: Partial thickness burns .25% TBSA.
a week or less. They usually have no complications. l Full thickness burns .10%.
l Burns involving the hands, face, feet or perineum, burns
II. Second degree burns: that cross major joints.
a) Superficial partial thickness: l Circumferential burns to any extremity.
l These burns extend to superficial (papillary) dermis and l Any burn associated with inhalational injury, electrical burns.
manifests with erythema with clear blisters and has l Burns associated with fractures or other trauma, burns
blanches with pressure. in infants and the elderly, in persons at high-risk of
l They are moist in texture and are painful. developing complications.
l The healing time for this type of burns is about l These burns typically require referral to a specialized
2-3 weeks with possible local infection/cellulitis as burn treatment centre.
complications.
B. Moderate burns
b) Deep partial thickness: Features of moderate burns are as follows:
l These burns extend to deep (reticular) dermis and l Age 10-50 yrs: Partial thickness burns involving 15-
manifests with red and white bloody blisters and has 25% TBSA.
less blanches with pressure. l Age ,10 or .50: Partial thickness burns involving
l They are moist in texture and are painful. They 10-20% TBSA.
take weeks to heal and may progress to third degree l Full thickness burns involving 2-10% TBSA.
burns. l Persons suffering these burns often need to be hospital-
l Scarring and contracture are common complications for ised for burn care.
this degree of burns. C. Minor burns
Minor burns are:
III. Third degree (full thickness):
l Age 10-50 yrs: Partial-thickness burns ,15% TBSA.
l These burns involve the full thickness of dermis and ap- l Age ,10 or .50: Partial thickness burns involving
pear as stiff and white/brown scars with dry, leathery ,10% TBSA.
texture. l Full thickness burns ,2% TBSA without associated
l One characteristic feature of this degree of burns is the injuries.
absence of pain. l These burns usually do not require hospitalization.
l These burns require excision for healing and complicate
with scarring, contracture and amputation. Q. 2. Discuss the pathology and treatment of burns.
Ans.
IV. Fourth degree burns:
l Pathophysiology of burns can broadly be divided as lo-
l This is the most extensive degree of burns and extends cal and systematic inflammatory reaction, the end result
through skin, subcutaneous tissue and into underlying of which is an almost immediate shift of intravascular
muscle and bone. fluid into the surrounding interstitial space.
l They are charred with eschar, dry in texture, and painless.
l They result in amputations and severe functional Local reaction:
impairment. l Burn causes coagulative necrosis of the epidermis and
underlying tissues, with the depth depending on the
temperature to which the skin is exposed and the dura-
According to Burn Severity
tion of exposure. The specific heat of the causative
l The American Burn Association devised a classification agent also affects the depth.
system to aid in the decision-making regarding referral l The area of cutaneous injury is divided into three zones:
to a specialised burn unit. a. zone of coagulation,
l Under this system, burns can be classified as major, b. zone of stasis, and
moderate and minor. c. zone of hyperaemia.
l This is assessed based on a number of factors, including a) Zone of coagulation: This occurs at the point of
total body surface area (TBSA) burnt, the involvement maximum damage. In this zone, there is irrevers-
of specific anatomical zones, age of the person and as- ible tissue loss due to coagulation of the constituent
sociated injuries. proteins.
Section | II General Surgery 247

b) Zone of stasis: This zone represents the surround- or douse the chemically burnt patient with water. Flames
ing tissues (both deep and peripheral) to the co- ascend so lie the patient down.
agulated areas, which are not devitalized initially l Cover the burn with a clean dressing, avoid the patient
but, due to microvascular damage, can progress getting cold and transfer to a hospital as soon as possible.
irreversibly to necrosis over several days, if not l Additional oxygen should be given during transfer.
resuscitated properly. The zone of stasis is charac- Widely used clinical assessment of burn area is by the
terized by decreased tissue perfusion. “rule of nine” which acts as a rough guide to the body
c) Zone of hyperaemia: In this outermost zone, surface area.
tissue perfusion is increased. The tissue here will
invariably recover unless there is severe sepsis or
prolonged hypoperfusion. ii. Primary Management
l Airway: Check the airway is clear. Endotracheal intuba-
tion is necessary, if there are deep burns to the face and
Systemic Reaction
neck, soot in the nostrils, burns of the tongue and phar-
The release of cytokines and other inflammatory mediators ynx, stridor or hoarseness.
at the site of injury has a systemic effect once the burn l History: History including time and nature of the
reaches 30% of total body surface area. incident (wet or dry burn/chemical/electrical/inside or
outside).
Weigh the patient.
Cardiovascular Changes l

l Examine the burn and assess the size with the ‘rule of
l Capillary permeability is increased, leading to loss of nines’ to give a %BSA.
intravascular proteins and fluids into the interstitial l Intravenous access: Obtain large bore venous access,
compartment. even through burnt tissue.
l Peripheral and splanchenic vasoconstriction occurs. l Blood sampling: Samples for haematocrit, electrolytes,
l Myocardial contractility is decreased, possibly due to crossmatch, arterial blood gases and carboxyhaemoglo-
release of tumour necrosis factor a. These changes, bin levels.
coupled with fluid loss from the burn wound, result in l Analgesia: Intravenous morphine, ketamine, or entonox.
systemic hypotension and end organ hypoperfusion. l Catheterise: Assess urine output as a gauge of tissue
perfusion and adequate resuscitation.
l Reassess the patient thoroughly at regular intervals and
Respiratory Changes also the burn.
l Inflammatory mediators cause bronchoconstriction,
and in severe burns adult respiratory distress syndrome
iii. Fluid Resuscitation
can occur.
l Fluids should be instituted as soon as possible. There
are two simple protocols that both depend upon the
Metabolic Changes %BSA, time passed since injury and patients weight.
l The basal metabolic rate increases up to three times its l The rule of nines may over-estimate the BSA, but the
original rate. This, coupled with splanchnic hypoperfu- Lund and Browder chart gives a more accurate assess-
sion, necessitates early and aggressive enteral feeding to ment. Fluid requirements may be greater than the proto-
decrease catabolism and maintain gut integrity. cols suggest.
l Parklands: Crystalloid resuscitation with Hartmanns/RL
Immunological Changes 24 hour fluid requirement 5 4 3 %BSA 3 Wt (kg)
l Non-specific down regulation of the immune response oc- Give half over the first 8 hours, and the remainder over
curs, affecting both cell mediated and humoural pathways. the next 16 hours.
Although there may be pronounced generalised oedema
MANAGEMENT OF THE BURNS initially, as large volumes are required, it is cheap and
produces less respiratory problems later on.
i. First Aid and Assessment of Burn Area l Muir and Barclay: Colloid resuscitation with plasma.
and Depth The first 36 hours are divided into time periods of
4,4,4,6,6,12 hour intervals.
l Remove the casualty from further injury. Extinguish
flames, remove clothing, turn off the electrical source, Each interval 5 0.5 3 %BSA 3 Wt (kg)
248 Quick Review Series for BDS 3rd Year

With colloid resuscitation, less volume is required and b. Closed method: Wound is closed by 3 layers, the
the blood pressure is better supported. However, they inner non-adherent and antiseptic and is oily or
are expensive, often unavailable and tend to leak out of water-based cream. The 2nd layer is ordinary
the circulation and may result in later oedema especially sterile cotton gauge, while 3rd layer is cotton
in the lungs. bandage.
l Inhalational injury may increase fluid requirements by
50%. Both regimes require regular assessment as to the vi. Surgery
adequacy of resuscitation. This includes blood pressure,
pulse, capillary return, urine output, level of conscious- l Circumferential burns will require immediate surgery to
ness and haematocrit. Additional fluid should be given, improve circulation to distal extremities or to permit
if resuscitation is inadequate. adequate breathing, if the chest wall is burnt.
l Water loss is related to evaporative and other extrarenal l Early excision and grafting is preferred as it minimizes
losses and may lead to a hypernatraemia. Salt intake infection and hastens wound healing.
should be balanced against the plasma sodium concen- l The whole burn should be excised within 48 hours.
tration, but is usually about 0.5 mmol/kg/%BSA. Regular dressing changes, further excision and grafting
l If the burn is left exposed in a hot environment, sodium may be required.
free water intake must be increased, but only to achieve l It should be remembered that blood loss may be exces-
a moderate hypernatraemia. Aggressive water load may sive at these times. Blood loss can be reduced by using
lead to a low plasma sodium and result in ‘burn en- diathermy and/or applying gauze soaked in adrenaline
cephalopathy’. Hyperkalaemia usually associated with (1: 200,000) during the burn excision.
severe muscle damage may require correction with in- l The problem with excision of a large burn is often the
sulin and dextrose. lack of donor skin to cover the excised burn.
l The patient’s donor skin can be meshed, so as to in-
crease the size. It can then be covered with cadaveric
iv. Airway Management skin which acts as a biological dressing with growth
l A high index of suspicion is required regarding the pa- stimulating properties. Artificial bovine skin (such as
tient’s airway. Laryngeal oedema develops from direct Integra) may also be used but are expensive. Escharec-
thermal injury leading to early loss of the airway. tomy and skin grafting will hasten the recovery.
l With signs of an airway burn (soot in the nostrils/
stridor/hoarse voice) consider early intubation of the vii. Anaesthesia
patient. A tracheostomy may be necessary, if there is
l If there is any concern over the airway, a gas induction
any delay in securing the airway.
following pre-oxygenation or a fibreoptic intubation are
l The airway is further endangered by an associated loss
the safest options.
of respiratory drive due to a depressed level of con-
l Suxamethonium should be avoided after the first
sciousness (e.g. head injury or carbon monoxide poi-
48 hours up to 2 years after a major burn because it may
soning). Again intubation may be required.
result in a large increase in serum potassium.
l Analgesia requirements are increased. Give Entonox,
v. Burn Management ketamine or morphine (titrated to response).
l Monitoring must include vital signs, temperature and
l Dressings are necessary to reduce infection and adsorb
urine output. Postoperatively the patient should be ad-
exudate. Bactericidal agents, such as silver sulphadia-
mitted to a high dependency unit, so that the continuing
zine 1% and silver nitrate are used.
fluid loss following burn excision can be maintained.
l Antibiotic preparations should be avoided to prevent
resistant colonisation developing. Q. 3. Describe the pathology and management of 50%
l Regular, often daily, dressing changes are recom- of burns in a person aged 40 years.
mended, and the patient should be washed with clean
Ans.
warm water. Debridement cleaning and then dressings
are important aspects of wound care. l Pathophysiology of burns can broadly divided as local
l Two methods of wound care are used: and systematic inflammatory reaction, the end result of
a. Open and which is an almost immediate shift of intravascular fluid
b. Closed. into the surrounding interstitial space.
a. Open method: Wound is kept open with topical l This occurs as a consequence of changes in vascular
application of antiseptic agents like silver sulfa- permeability as the normal capillary barrier is disrupted
diazine. by a host of mediators, including histamine, serotonin,
Section | II General Surgery 249

prostaglandins, platelet products, complement compo- Immunological change


nents, and members of the kinin family. l Non-specific down regulation of the immune response
occurs, affecting both cell-mediated and humoral path-
Local Reaction ways.
l Burn causes coagulative necrosis of the epidermis and Management of 50% of burns in a person aged
underlying tissues, with the depth depending on the 40 years:
temperature to which the skin is exposed and the dura-
i. Fluid resuscitation:
tion of exposure. The specific heat of the causative
l Using Parkland’s formula we can calculate the amount
agent also affects the depth.
of fluid needed to be transfused to a patients with 50%
l The area of cutaneous injury is divided into three zones:
burns and 40 kg weight.
zone of coagulation, zone of stasis, and zone of hyperae-
4 ml 3 total body surface area sustaining 2nd/3rd/4th
mia.
degree burns 3 person’s weight in kg.
a) Zone of coagulation: This occurs at the point of
4 ml 3 50 3 405 8000 ml i.e. fluid requirements, 1st
maximum damage. In this zone, there is irreversible
24 hours is 8 litres.
tissue loss due to coagulation of the constituent
Fluid requirements, first 8 hours (1/2 of total): 4 litres.
proteins.
The rest i.e. 4 liters are infused in next 16 hours.
b) Zone of stasis: This zone represents the surrounding
l Hartmann’s solution/ RL solution is recommended.
tissues (both deep and peripheral) to the coagulated
Regular reassessment of the adequacy of resuscitation
areas, which are not devitalized initially but, due to
should be performed. Blood products and colloid may
microvascular damage, can progress irreversibly to
also be given in addition to these requirements.
necrosis over several days, if not resuscitated prop-
erly. The zone of stasis is characterized by decreased Q. 4. Discuss the management of thermal injuries of
tissue perfusion. face and oropharynx.
c) Zone of hyperaemia: In this outermost zone, tissue
perfusion is increased. The tissue here will invari- Ans.
ably recover unless there is severe sepsis or pro- Management of thermal injuries of face and oropharynx is
longed hypoperfusion. as follows:
l Facial burns vary from relatively minor insults to severe
Systemic Reaction debilitating injuries. Over 50% of burn injuries involve
the head and neck region and can be caused by flame,
l The release of cytokines and other inflammatory media- electrical current, steam, hot substances, and chemicals.
tors at the site of injury has a systemic effect once the l The basic principles of management of facial burns are
burn reaches 30% of total body surface area. similar to that of other burns as described above.
l However, one important aspect of facial burn manage-
Cardiovascular changes ment is the reconstruction of the face post-burns.
l Capillary permeability is increased, leading to loss of l Objectives for reconstruction following a facial burn in-
intravascular proteins and fluids into the interstitial cludes restoration of function, comfort, and appearance.
compartment. l Functional concerns in these patients include airway
l Peripheral and splanchnic vasoconstriction occurs. patency, protection of the cornea, oral continence, and
l Myocardial contractility is decreased, possibly due to neck mobility.
release of tumour necrosis factor a. These changes, l Burns may impart a tight mask-like sensation to the
coupled with fluid loss from the burn wound, result in face, distorting features and limiting facial expression.
systemic hypotension and end organ hypoperfusion. l Appearance is altered by contractures, scarring, and
pigmentary changes. The goal of the reconstructive sur-
Respiratory changes geon is to minimize final deformity by restoring the
l Inflammatory mediators cause bronchoconstriction, and in patient to a near-normal appearance.
severe burns adult respiratory distress syndrome can occur. l Adequate documentation, including photographs, is es-
sential because facial burn reconstruction is typically a
Metabolic changes lengthy process involving multiple procedures.
l The basal metabolic rate increases up to three times its l Certain aesthetic principles are important when ap-
original rate. This, coupled with splanchnic hypoperfu- proaching reconstruction of the burned face.
sion, necessitates early and aggressive enteral feeding to l Reconstruction should proceed within facial aesthetic
decrease catabolism and maintain gut integrity. units. Treat each region (i.e. forehead, eyes, nose, cheeks,
250 Quick Review Series for BDS 3rd Year

ears, upper lip, lower lip and chin, neck) as an individual should be worn for 6 months, if skin grafts have been
entity. applied.
l Place scars at the boundaries of 2 aesthetic units. Often,
sacrificing normal skin and replacing an entire aesthetic
Upper Lip
unit is better than replacing part of an aesthetic unit and
having a scar coursing across that unit. l The upper lip actually consists of 3 aesthetic units: the
l Orientation of scars parallel to relaxed skin-tension 2 lateral lip elements and the central philtral ridge.
lines (RSTLs) is also important. l Usually, an incision is placed transversely at the root
l Primary repair is elected over waiting for scar matura- of the nose, allowing the upper lip to fall back into its
tion and secondary release of contractures that always normal position and maintain fullness.
results in a patch-like appearance on the applied grafts. l The secondary defect is subsequently skin-grafted.
l Scar revision or staged excision may still be necessary l The philtral ridge may be augmented with dermal or
later, and the patient should understand this at the outset. cartilage grafts that otherwise might be discarded.
l Early excision and grafting is the treatment of choice, it l Patients agreeable to permanent moustache wear might
may be a 1-2 stage technique. For larger burn defects, consider moustache reconstruction.
one might consider regional flaps, free flaps, and tissue l This is performed with a free or islandized scalp flap
expansion in addition to skin grafting. based on the superficial temporal vessels.
l Achieving the correct downward-growth orientation of
hair follicles is a crucial consideration.
Eyelid
Eyelid reconstruction has highest priority because the
Nose
l

lid functions to protect the cornea. Initiate reconstruc-


tion even in the presence of an immature scar. l Nasal reconstruction is of intermediate priority.
l Scar release and subsequent grafting is the procedure of l Airway obstruction may require early intervention with
choice. Split-thickness grafts are a better match for the scar release and skin grafting.
upper eyelid, and the optimal donor site is the contra- l Cosmetic nose reconstruction is better postponed until
lateral upper lid, if uninvolved with injury. scar maturation is complete, but total resurfacing with a
l Full-thickness grafts better approximate lower lid thick- skin graft may be performed rather early.
ness and decreased tendency for secondary contraction l Alar deformities may be treated with a local turndown
minimizes chances of postoperative ectropion. flap resurfaced with a full-thickness skin graft or a com-
posite graft from the ear.
Lip and Chin Region
Forehead
l This region is second in priority. Complications of burns
in this region include drooling, microstomia, eating and l The forehead may be resurfaced completely as an
communication difficulties, lip eversion, and oral hy- aesthetic unit with a full-thickness skin graft.
giene inadequacy. l Tissue expansion is an alternative for a forehead burned
l If operative intervention is necessary, thorough scar re- on less than half its area.
lease with sufficient skin and soft tissue should be added
to the lip and chin area, and this frequently requires a
Ear
multiple-step operation.
l Full-thickness grafting is the procedure of choice. Re- l Superficial burns to the ears can be managed like those
construction of very large defects of the lower lip is to the face. However, external pressure should not be
difficult. Consider cheek advancement flaps, rotation applied to the injured helix.
flaps using the residual lip, double cross-lip flaps, or l The cartilage is already poorly vascularized and any
nasolabial flaps. compression will potentiate further injury.
l No pillows or any external pressure are allowed.
In addition, the topical agent, silver sulfadiazine or
Cheek l

mafenide, must be applied multiple times a day, espe-


l Treat the burned cheek with a facial-pressure garment to cially if any cartilage is exposed.
assist in scar maturation. l Chondritis is a major complication that requires an ex-
l This is unnecessary, however, if the entire aesthetic unit tensive (several weeks) course of systemic antibodies.
is to be excised and grafted. l Chondritis invariably leads to loss of cartilage and per-
l Other options include tissue expansion of unburned manent deformity. Pseudomonas is the most common
cervical skin and local flaps. Postoperatively, a mask pathogen.
Section | II General Surgery 251

l Ear reconstruction is generally performed last because Management of Patient with 20% Burns
the first choice for reconstruction uses available local
l Stop the burning process: Flame can be extinguished by
tissue that should be allowed to achieve mature scarring.
wrapping the patient in a fire blanket or any available sheet.
l Reconstructive options include local skin and fascial
l In chemical burn, the area should be diluted by running
flaps with/without tissue expansion, ipsilateral axial
water.
temporoparietal fascial flap, contralateral temporopari-
l The clothing should be removed.
etal, or an ipsilateral random fascial flaps.
l Airway is maintained especially in inhalation injury.
l If the entire skin of the temporoauriculomastoid region
l Immediate cooling of the part should continue for
is lost or badly scared with poor axial vascularity, a bi-
20 minutes with a running tap water (ideal temperature
lobed cervical flap or fascial free flap is an option.
15°C).
l The burnt area should be wrapped in clean linen and
Management of Thermal Injury transported to the hospital.
to Oropharynx
Emergency management
l Thermal injury to the oropharynx is caused by smoke inha-
l Airway maintenance
lation. Thermal damage is usually limited to the oropharyn-
l Breathing and ventilation
geal area, due to the poor conductivity of air and the high
l Circulation
amount of dissipation that occurs in the upper airways.
l Disability - neurological status
l Acute upper airway obstruction (UAO) occurs in ap-
l Exposure and environment control, keep warm
proximately one-fifth to one-third of hospitalized burn
l Fluid resuscitation.
victims with inhalation injury and is a major hazard
because of the possibility of rapid progression from Further management
mild pharyngeal oedema to complete upper airway
l Thorough reassessment of the burns area, depth is done.
obstruction with asphyxia.
l Analgesia preferably with morphine or pethidine IV.
l Its management is as follows:
l Blood transfusion.
i. Deliver high-flow oxygen by mask.
l Early escharectomy and skin grafting will hasten the
ii. If respiratory failure is present, the patient should have
recovery.
assisted ventilation and/or endotracheal intubation.
l Proper antibiotics to combat the infection.
iii. Tracheostomy/cricothyrotomy, if airway obstruc-
l Nutritional support is a must as the patient is under
tion is present or impending and an airway cannot
severe catabolism.
be secured orally.
iv. Secure IV access, but do not delay transport of Dressings
patient to the hospital in any way.
v. Bronchial hygeine therapy: Therapeutic coughing, l Epidermal burns with erythema and no blisters do not
chest physiotherapy, early ambulation, airway suc- need any dressings. Analgesia and moisturizing cream
tioning, therapeutic bronchoscopy and pharmaco- is used.
logic agents have been effective in the removal of l Burns of the face are treated by exposure because of the
retained secretions. difficulty of dressing.
vi. It is important that the magnitude of the swelling in l Where there is lot of crusting, soothing ointments, like
the areas of the face and mandible be closely scru- petroleum jelly, are used.
tinized when making decisions about the need for l Superficial dermal burns with blisters are usually dressed
an artificial airway. to absorb the exudate, prevent desiccation, provide pain
vii. The threshold for intubation should be lower than relief, encourage epithelialisation and prevent infection.
in other patients due to the potential of rapid devel- l Appropriate dressings are plastic films, preserved pig skin
opment of airway oedema. and paraffin gauze. A thick layer of gauze may be placed
on the top to allow transudation fluids to be absorbed.
Q. 5. Define burns and scalds. Discuss management l Partial thickness burn wounds usually heal within
of 20% burns. 2-3 weeks. If they do not heal by 3 weeks, some
intervention like slough excision may be necessary.
Ans. l Deep burns are managed with the topical antimicrobial
l Burn is a type of injury to flesh caused by heat, electric- agents such as silver sulphadiazine cream.
ity, chemicals, light, radiation or friction.
l Scald is thermal injury/burn caused by moist heat, such Surgical treatment
as boiling water, steam, improperly applied poultice, hot l Partial thickness burns should heal without any surgical
oil or tar. intervention.
252 Quick Review Series for BDS 3rd Year

l Full thickness deep burns - two alternative treatments. Surgical reconstruction


l One can wait for spontaneous desloughing and apply The major complication of burn injury is scarring.
skin grafts after 3 weeks. It has got a greater scarring of l Lumpy hypertrophied scars and keloids can be limited
the wound. by application of pressure - Lycra pressure garments.
l Excision of devitalised tissue and early skin grafting. Topical silicon sheeting is also useful.
This has an advantage of rapid healing and minimizes l Contractures at the neck and joints can be treated
the risk of scarring. by releasing the contractures excising all fibrous tissue
and a skin cover either with spilt thickness or pedicle
Mobilization and rehabilitation graft.
Early mobilization of the patient reduces the complications l Plastic reconstruction is necessary for all post-burn
like infection and deep venous thrombosis. Intensive phys- deformities.
iotherapy will avoid formation of contracture resistance.

SHORT NOTES
Q. 1. Write a short note on plasma expanders. vi. Burns may impart a tight mask-like sensation to the
Ans. face, distorting features and limiting facial expression.
vii. Appearance is altered by contractures, scarring, and
i. Plasma expander is a type of blood substitute that has pigmentary changes. The goal of the reconstructive
the function of providing volume for the circulatory surgeon is to minimize final deformity by restoring the
system. It may be used for fluid replacement. patient to a near-normal appearance.
ii. There are two main types of plasma expanders:
l Crystalloids
Q. 3. Write a short note on classification of burns.
l Colloids. Ans.
Crystalloids are aqueous solutions of mineral salts
or other water-soluble molecules.
Colloids contain larger insoluble molecules, such as CLASSIFICATION OF BURNS
gelatin; blood itself is a colloid.
According to Mechanism of Injury
iii. Medical conditions associated with plasma expanders:
l Burns, external a. Thermal burns: Consisting of flame, contact, or scald burns.
l Hypoproteinaemia b. Chemical burns
l Pancreatitis c. Electrical burns
l Peritonitis d. Radiation burns
l Postoperative albumin loss

l Shock
According to Burn Depth
Q. 2. Write a short note on burns of face.
I. First degree burns: Involves epidermis only.
Ans. II. Second degree burns:
a. Superficial partial thickness: These burns extend to
i. Facial burns vary from relatively minor insults to
superficial (papillary) dermis.
severe debilitating injuries.
b. Deep partial thickness: These burns extend to deep
ii. Over 50% of burn injuries involve the head and neck
(reticular) dermis.
region and can be caused by flame, electrical current,
III. Third degree (full thickness): These burns involve the
steam, hot substances, and chemicals.
full thickness of dermis.
iii. The basic principles of management of facial burns are
IV. Fourth degree burns: This is the most extensive degree
similar to that of other burns. However, one important
of burns and extends through skin, subcutaneous tissue
aspect of facial burn management is the reconstruction
and into underlying muscle and bone.
of the face post-burns.
iv. Objectives for reconstruction following a facial Q. 4. Write a short note on skin grafting.
burn include restoration of function, comfort, and
Ans.
appearance.
v. Functional concerns in these patients include airway i. Skin grafting is a surgical procedure involving transplanta-
patency, protection of the cornea, oral continence, and tion of skin or a skin substitute over a burn or non-healing
neck mobility. wound. The transplanted tissue is called a skin graft.
Section | II General Surgery 253

ii. Skin grafting is often used to treat extensive wounding ii. Healing of the donor site occurs by re-epithelialization
or trauma, burns, areas of extensive skin loss due to from the dermis and surrounding skin.
infection such as necrotizing fasciitis or purpura iii. This type of grafts requires less ideal conditions for
fulminans, specific surgeries that may require skin grafts survival and has a much broader range of application
for healing to occur. than FTSGs and is used to resurface large wounds, line
cavities, resurface mucosal deficits, close flap donor
Types of skin grafting sites, and resurface muscle flaps.
l Split-thickness: It includes epidermis and variable iv. Split skin grafts, based on the thickness of the harvested
amounts of dermis which is usually taken from the thigh graft, are categorized further as:
or buttock. Healing of the donor site occurs by a) Thin (0.005-0.012 in)
re-epitheliazation from the dermis and surrounding skin. b) Intermediate (0.012-0.018 in)
l Full-thickness: This graft includes epidermis and all c) Thick (0.018-0.030 in)
the dermis. The donor site is sutured directly or split- v. Disadvantages of split skin grafts:
thickness skin grafted. l Split skin grafts are more fragile.

l Composite graft: They are small grafts containing skin l Usually do not withstand subsequent radiation therapy.

and underlying cartilage or other tissue. Donor sites can l They can contract significantly during healing.

be ear skin and cartilage to reconstruct nasal wings rim l They tend to be hypo- or hyperpigmented, particu-

defects. larly in darker-skinned individuals.


l Their thinness, abnormal pigmentation, and frequent

Classification of grafts lack of smooth texture and hair growth make STSGs
l Allograft: Uses skin obtained from another human being. more functional than cosmetic.
l Xenograft: Uses skin obtained from animal (usually Q. 7. Write a short note on skin grafting indications.
pig).
l Autograft: Uses skin from another area of the patient’s Ans.
own body.

Q. 5 Write a short note on burn wound grafting. Indications for Skin Grafting
They are often used in treatment or rehabilitation of people
Ans.
with:
i. Early excision and grafting is considered as a option for l Large wounds or trauma
treatment for all burns that do not heal by primary in- l Burns
tention within 3 weeks. l Prior infection area with extensive skin loss
ii. Full-thickness burns require grafting unless smaller l Specific surgeries requiring skin grafts for healing to
than 3 to 4 cm in diameter. occur
iii. The more rapidly the wound is closed with skin grafts, l Cosmetic consideration in reconstructive surgeries.
the better. Burns, less than 30% of total body surface
can be, at least theoretically, rapidly closed because Q. 8. Write a short note on scalds.
adequate donor sites are available. Ans.
iv. Larger burns are more difficult to completely graft
early. Initial coverage is often accomplished using l Scald is thermal injury/burn caused by moist heat, such
available skin along with temporary skin substitute to as boiling water, steam, improperly applied poultice, hot
cover affected areas until skin is available. oil or tar.
v. The other option is the use of a permanent skin substitute. l Scald burns can be severe, and sometimes life-threaten-
vi. Deep partial thickness burns are also difficult to assess ing; skin grafting is required in about 20% of all scald
clearly as to time of healing. Considerable judgment cases.
and assessment skills are therefore essential for this l Most full thickness burns requiring grafting will result
approach to yield optimal results. in scarring. Scalds from hot tap water tend to be more
severe than scalds from other causes.
Q. 6. Write a short note on split skin graft. l On average, they result in a higher percentage body
surface area (BSA) burned, a higher percentage of full
Ans.
thickness burns and a longer length of stay in hospital
i. Split skin grafts include epidermis and variable amounts than other scald injuries.
of dermis which is usually taken from the thigh or l International studies indicate that 40% BSA burns are
buttock. not uncommon in children scalded by tap water.
254 Quick Review Series for BDS 3rd Year

Topic 4

Haemorrhage and Shock

LONG ESSAYS
Q. 1. Define shock and describe pathophysiology of Hypoperfusion
shock.
Ans. Energy deficit
Shock is defined as an acute clinical syndrome character-
ized by a significant, systemic reduction in tissue perfusion
Accumulation of lactic acid, fall in pH Anaerobic metabolism
(hypoperfusion), resulting in decreased tissue oxygen
delivery and insufficient removal of cellular metabolic
products, resulting in tissue injury and severe dysfunction Vasoconstriction Metabolic acidosis
of vital organs.

Failure of precapillary Dysfunction of cell membrane,


Pathophysiology sphincters failure of Na pump

The pathophysiology of shock can be summed in two basic


processes: Peripheral Activation of Efflux of K+
pooling of blood intercellular Influx of sodium,
a) Decrease in effective circulating volume: lysosomes water
This reduction in circulating volume is due to two factors:
i. Due to actual loss of blood volume or
Toxic products in circulation
ii. Due to decreased cardiac output without actual loss of
blood volume (normovolaemia).
Damage of capillary endothelium
b) Hypoxia/anoxia of tissue:
As a result of reduction in the effective volume of circulat-
ing blood, there is a decrease in venous return to the heart Further destruction, dysfunction and cell death
resulting in further decrease in cardiac output. This condi-
tion consequently leads to reduced oxygen supply to the
organs and tissues and tissue hypoxia/anoxia.

The pathogenesis of shock involves multiple factors which b) Circulating or local inflammatory mediators:
include: l Role of inflammatory mediators on cellular metabolism
is of great importance in organ dysfunction resulting in
a) Cellular ischaemia: septic shock and also haemorrhagic shock associated
l Hypoperfusion resulting in cellular ischaemia plays a with extensive trauma.
big role in cell injury in most forms of shock. l In most cases, endotoxin from gram-negative bacteria
l Due to the decrease in transport of nutrients to the cells triggers the inflammatory mediators but bacterial
as a result of hypoperfusion leads to diminished ATP antigens and cell injury can also initiate the cascade.
production. l Cytokines like TNF-a and IL-1b produced by macro-
l Essential ATP dependent intracellular metabolic pro- phages are the prime mediators. Other substances
cesses like maintenance of transmembrane potential, involved include IL-2, IL-6, interferon-a, endothelin-1,
mitochondrial function and other energy-dependent en- leukotrienes, thromboxanes, prostaglandins and com-
zyme reactions are thus affected. plement fragments C3a and C5a. Circulating myocar-
l Lysosomal enzyme damage is the point of irreparable dial depressant substance and nitric oxide have a role to
cell damage analogous to clinical irreversibility. play in septic shock.
Section | II General Surgery 255

c) Free radical injury: v. Resorption of fluid from the intracellular to the extra-
l Tissue ischaemia leads to accumulation of adenosine, cellular space.
inosine and hypoxanthine. With resuscutation, reperfu- vi. Renal conservation of body water and electrolytes.
sion of ischaemic areas occurs.
l The availability of O2 generates superoxide (O2-) by iii. Decompensated (irreversible) shock:
xanthine oxidase which is converted to hydrogen perox- This is the stage of no return which is characterized by per-
ide (H2O2) which reacts to produce the highly tissue sistent vasoconstriction, vascular permeability and severe
damaging hydroxyl radicals. hypoxia and cell death which ultimately leads to MODs.
l Oxidant activity directly and through endothelial dam-
age attracts and activates neutrophils causing amplifica- Q. 2. Classify and describe the signs, symptoms and
tion of superoxide generation and further tissue damage management of shock.
due to neutrophil protease release. Ans.
According to pathophysiology, there exist 3 stages l Shock has been classified under various classifications
of shock: but the classification proposed by Hinshaw and Cox
(1972) is the most widely accepted one.
i. Non-progressive (initial, compensated l According to this classification, shock can be classified
reversible) shock: into four broad categories. This is, however, just an
l During this stage, the hypoperfusional state causes hy- artificial separation and there is a frequent, considerable
poxia, leading to the mitochondria being unable to pro- overlapping within these categories:
duce adenosine triphosphate (ATP). Due to this lack of
oxygen, the cell membranes become damaged, they a) Hypovolaemic shock:
become leaky to extracellular fluid, and the cells per- l This type of shock is due to loss of blood plasma or
form anaerobic respiration. body fluid and electrolytes, usually caused by massive
l This causes a build-up of lactic and pyruvic acid which haemorrhage, vomiting, diarrhoea, and dehydration.
results in systemic metabolic acidosis. l This is the most common type of shock. It is characterized
l The process of removing these compounds from the cells by a loss in circulatory volume, which results in decreased
by the liver requires oxygen, which is absent. During this venous return, decreased filling of the cardiac chambers,
phase, adequate cerebral and coronary blood supply is and hence a decreased cardiac output which leads to
tried to be maintained by redistribution of blood. increase in the systemic vascular resistance (SVR).

ii. Progressive decompensated shock: b) Cardiogenic shock:


In this stage, the body will alter its haemodynamic functions l This type of shock is due to the dysfunction of one ven-
to compensate for poor tissue perfusion. This stage is char- tricle or other, usually seen in myocardial infarction,
acterized by the body employing physiological mechanisms, chronic congestive cardiac failure, cardiac arrhythmias,
including neural, hormonal and biochemical mechanisms in pulmonary embolism, etc. resulting in inability of the
an attempt to reverse the condition. The compensatory heart to pump the adequate amount of blood into the
mechanisms in this stage are: lungs and decreased cardiac output. MI is the most
i. Adrenergic discharge: Selective vasoconstriction of common cause of cardiogenic shock.
blood vessels, splanchnic viscera, skin, and vascular
sphincters of kidney to improve the filling of the right c) Distributive shock:
heart, increases the cardiac output and diverts blood to l This type of shock occurs when the afterload is excessively
the heart and brain. reduced due to extensive vasodilatation and is associated not
ii. Hyperventilation: To compensate metabolic acidosis, only with poor vascular tone in the peripheral circulation but
spontaneous deep breathing sucks the blood from extra- maldistribution of blood flow to organs within the body also.
thoracic sites to the heart and lungs. This will increase l Septic shock: This type of shock is mostly due to
the filling of left ventricle and also its stroke volume. release of endotoxins in blood by Gram-negative bacte-
iii. Release of vasoconstrictive hormones: Renin-aldosterone- ria such as Escherichia coli, Proteus species, Klebsiella
angiotensin system, ADH, epinephrine pneumoniae which causes wide-spread vasodilatation
iv. Resorption of fluid from the interstitial tissue: Due to and it is the most common among distributive shock.
selective vasoconstriction, there is decrease in intra- l Neurogenic shock: This type of distributive shock is
vascular hydrostatic pressure which leads to influx of caused by the suppression or outright loss of sympathetic
water, electrolytes from the interstitial tissue space into tone caused any disruption of the sympathetic nervous
the capillaries. system like spinal injury, spinal anesthesia, and drugs.
256 Quick Review Series for BDS 3rd Year

l Anaphylactic shock: This type of shock is a result of l If it is an internal haemorrhage, operation should be
type I hypersensitivity reaction and is caused when the carried out as early as possible after resuscitation.
body’s antibody-antigen response is triggered by some-
thing the person is allergic to. Drugs, like penicillin, C. Extracellular fluid replacement
cephalosporins, iodinated contrast media, serum, etc. l This should start immediately. CVP monitoring will
are common causes of this type of shock. guide us regarding the amount of fluid replacement.
l Non-sugar non-protein crystalloid is preferred. Normal
d) Obstructive shock: saline or ringers lactate should be started first and in-
This type of shock associated with physical obstruction of fused rapidly till the blood pressure is restored.
the great vessels or the heart itself. Pulmonary embolism l At the same time, proper assessment of loss of blood,
and cardiac tamponade are considered forms of obstructive plasma and the fluid is estimated and the same should
shock. Obstructive shock has much in common with cardio- be replaced. If blood is not available, blood substitutes
genic shock and the two are frequently grouped together. like plasma or dextran should be used.

D. Correcting acid-base disturbance


Signs and Symptoms of Shock
This is corrected using molar sodium bicarbonate when the
l Anxiety, restlessness, altered mental state due to de- pH of arterial blood is less than 6.2 and electrolyte abnor-
creased cerebral perfusion and subsequent hypoxia malities specially potassium and calcium.
l Hypotension due to decrease in circulatory volume
l Tachycardia and weak thready pulse due to decreased E. Drugs
blood flow combined with tachycardia a). Sedatives: To alleviate the pain and anxiety, drugs
l Cool, clammy skin due to vasoconstriction and stimula- should be used. Morphine should be given intrave-
tion of vasoconstriction nously as the peripheral vasoconstriction in shock will
l Rapid and shallow respirations (tachypnoea) due to delay the absorption of the drug, if given intramuscu-
sympathetic nervous system stimulation and acidosis larly. Pethidine is also a good agent.
l Hypothermia due to decreased perfusion and evapora- b). Chronotropic agents: If the heart rate is low, atropine is
tion of sweat a good agent followed by isopropranolol.
l Thirst and dry mouth due to fluid depletion c). Inotropic agents: These drugs increase the efficiency of
l Fatigue due to inadequate oxygenation the cardiac muscle contraction. The commonly used
l Cold and mottled skin, especially extremities, due to drugs are dopamine and dobutamine. They increase the
insufficient perfusion of the skin. myocardial activity and selectively increase the renal
l Pallor blood flow by dilating the renal vasculature. They also
l Fainting have a vasoconstrictor effect and hence should be used
l Oliguria/anuria due to decreased renal perfusion and slowly in a small dose.
afferent arteriolar vasoconstriction. d). Vasodilators: Patient with severe septic traumatic and cardio-
genic shock requires these drugs. Nitroprusside, and
Management of Shock nitroglycerins are easily reversible and short-acting. These are
the useful vasodilators to patients of shock with high systemic
Patients in shock are ideally managed in ICU with continu- vascular resistance and high filling pressures of heart.
ous ECG monitoring and life support. The objectives are to e) Vasoconstrictors: These are very useful in neurogenic
increase the cardiac output and improve the tissue perfusion shock. They increase the blood pressure, increase the
especially in coronary, cerebral, renal and mesenteric vas- perfusion pressure for coronary circulation. They also
cular beds. increase the myocardial activity. Phenylephrine and
metaraminol are commonly used. However, they should
A. Resuscitation never be used in hypovolaemic and traumatic shock.
Maintenance of adequate airway ventilation and proper f) Beta-blockers: In cardiogenic shock, there will be stiff
oxygenation. myocardium and rapid heart rate. This will be benefited
Head low position face turned to one side. If necessary, by a beta-blocker. Propranolol is a good drug.
endotracheal intubation. g) Diuretics: These are useful in cardiogenic shock. They
reduce the vascular volume and decrease the filling
B. Control of haemorrhage pressure. Though oliguria is a main clinical manifesta-
l In case of hypovolaemic shock, haemorrhage is con- tion of hypovolaemic shock, diuretics can never correct
trolled immediately by elevation, compression bandage the underlying cause of oliguria, but aggravate the hy-
or ligation of the bleeding vessel. povolaemia. They are also not used in septic shock.
Section | II General Surgery 257

Q. 3. Discuss the different types of haemorrhage and A. General management - hospitalization


their management. l Care of critically ill patients starts with A, B, C
Ans. (airways, breathing, circulation). Oxygen should be ad-
ministered.
Haemorrhage is defined as escape of blood from blood l Intravenous line: Urgent intravenous administration
vessels, i.e. extravasation of blood due to vessel rupture. of isotonic saline or a balanced crystalloid solution to
restore the blood volume to normal. Colloids, such as
Types of Haemorrhage gelatins, have also been used.
l Rest: Anxious, apprehensive, restless persons bleed
I. Haemorrhage may be: more due to excitement and fear. So he should be
l Arterial: Bright red blood comes out like a spurting
sedated.
jet, which rises and falls in time with pulse.
l Venous: Dark red haemorrhage as a steady and co- B. Specific measures to stop haemorrhage
pious flow. Conservative
l Capillary: Blood is a bright red often rapid

oozing. l Pressure: Most of the bleeding can be stopped by firm


l External: Bleeding either through a natural
pressure by a sterile gauge or cotton. If sterile bandages
opening such as the vagina, mouth, nose, ear or are not available, any cloth would do, as controlling the
anus, or through a break in the skin. bleeding is more important. Use of double balloon in
l Internal: Where blood leaks from blood vessels
the oesophagus and stomach to control the bleeding
inside the body. from oesophageal varices.
II. Haemorrhage can also be classified as: l Tourniquet: It comes very handy in an emergency but it
l Primary: Occurs at the time of injury or operation.
has its limitations. It can only be used in the limbs. It
l Reactionary: Reactionary haemorrhage may should be applied on to the arm or thigh, tight enough
follow primary haemorrhage within 24 hours to occlude the veins and arteries. The maximum time to
(usually 4-6 hours) and is mainly due to rolling keep the tourniquet is 1 to 11⁄2 hour.
(slipping) of a ligature, dislodgement of a clot or l Elevation of bleeding area against the gravity. If there is
cessation of reflex vasospasm. bleeding from head to neck and thyroid area, head and
l Secondary: This occurs after 7-10 days post-
neck should be elevated. If it is in the lower limbs, they
trauma. This is due to infection and sloughing of should be elevated. If the oozing is from the veins, it
arterial walls. will stop as the veins get collapsed.
III. Haemorrhage can be classified into four classes by the Operative
American College of Surgeons’ Advanced Trauma
Life Support (ATLS). l Clamping the bleeding vessels with catgut, thread or silk.
l Class I hemorrhage involves up to 15% of blood
l Coagulation by thermocautery or diathermy.
volume. l By local application of adrenaline swabs.
l Class II hemorrhage involves 15-30% of total
l Application of silver clips as in neurosurgery.
blood volume. l Under running or transfusion of vessels, by needle and
l Class III hemorrhage involves loss of 30-40% of
suture.
circulating blood volume. l Application of crushed piece of muscle at the site of
l Class IV hemorrhage involves loss of .40% of
bleeding.
circulating blood volume. l Bone wax or BIPP (bismuth iodoform paraffin paste) is
used for oozing from bone.
l Repair of vascular defect by patches of vein or Dacron
Management of Haemorrhage mash.
l Whenever the haemorrhage is suspected, be it internal l Excision of bleeding organ, e.g. splenectomy.
or external, first and the foremost thing is to assess
C. Restoration of blood volume after
the amount of blood loss and also try to find out
haemorrhage
the cause.
l The severity of the condition and haemorrhage should i. Resuscitation from haemorrhage should include restora-
be assessed quickly by following the guidelines of tion of circulating volume. Ringer’s lactate is preferred
thorough examination, for pallor, tachycardia, shock over normal saline to avoid hyperchloraemic acidosis
and hypotension. Accordingly, a prompt action plan is associated with prolonged use of sodium solutions.
undertaken. Isotonic crystalloid or colloid solutions can be used for
258 Quick Review Series for BDS 3rd Year

volume replacement in haemorrhage. However, there is capillary damage in the lung activation of the coagula-
no place for hypotonic dextrose solutions in the man- tion system, culminating in disseminated intravascular
agement of haemorrhagic shock. coagulation.
ii. Blood transfusion: Many blood products are available l The hypoperfusion resulting from the combined effects
to restore circulating volume, and replace coagulation of widespread vasodilation, myocardial pump failure,
factors and oxygen-carrying capacity. Hypovolaemia is and DIC causes multiorgan system failure that affects
best corrected with crystalloid solution. In haemor- the liver, kidneys, and central nervous system, among
rhagic shock, packed red blood cells (PRBCs) are most others. Unless the underlying infection (and LPS
commonly used to restore intravascular volume and overload) is rapidly brought under control, the patient
oxygen-carrying capacity. Platelet transfusions are usually dies.
indicated in situations of significant thrombocytopenia
(platelet count less than 20,000 to 50,000 per mm3) and
Clinical Features of Septic Shock
continued haemorrhage.
l Pyrexia (fever) with chills and rigors due to increased
Q. 4. Describe the pathophysiology, clinical features and level of cytokines/infection.
treatment of septic shock. l Systemic vasodilation resulting in hypotension (low
Ans. blood pressure).
l Warm and sweaty skin due to vasodilation.
l Septic shock is a subclass of distributive shock caused l Anxiety, restlessness, altered mental state due to
due to release of endotoxins in blood mostly by Gram- decreased cerebral perfusion and subsequent hypoxia.
negative bacteria such as Escherichia coli, Proteus spe- l Hyperventilation with respiratory alkalosis is a common
cies, Klebsiella pneumoniae and as well as other Gram- feature of patients with sepsis secondary to stimulation
positive cocci, such as pneumococci and streptococci, of the medullary respiratory centre by endotoxins and
and certain fungi as well as Gram-positive bacterial other inflammatory mediators.
toxins which causes wide spread vasodilatation.
l Septic shock is characterized by sepsis-induced hypo-
tension (systolic blood pressure ,90 mm Hg or a reduc- Treatment
tion of 40 mm Hg from baseline) despite adequate fluid l Prompt diagnosis and effective management.
resuscitation. l The control of infection by antibiotic treatment.
l Early surgical debridement or radiologically guided
drainage to remove all the dead and septic foci.
Pathophysiology of Septic Shock l Fluid replacement: Correction of pre-existing fluid defi-
l Most cases of septic shock (approximately 70%) are cits is essential using pulmonary capillary wedge pres-
caused by endotoxin-producing gram-negative bacilli. sure and cardiac output as a guide. Large volumes of
l Endotoxins are bacterial wall lipopolysaccharides fluid infusions are required as initial therapy in patients
(LPS), nalogous molecules in the walls of gram-positive with septic shock. The volume resuscitation can be
bacteria and fungi can also elicit septic shock. achieved by either crystalloid or colloid solutions. The
l Free LPS attaches to a circulating LPS-binding protein, crystalloid solutions are 0.9% sodium chloride and lac-
and the complex then binds to a specific receptor tated Ringer solution. The colloids are albumin, dex-
(CD14) on monocytes, macrophages, and neutrophils. trans, and pentastarch.
Engagement of CD14 (results in intracellular signaling l Vasoactive drugs: Phenoxy benzamine, and isopropranolol
via an associated “Toll-like receptor” protein 4 (TLR- (dopamine) are ideal and will improve the cardiac, pulmo-
4), resulting in profound activation of mononuclear cells nary and renal functions with better chances of survival.
and production of potent effector cytokines such as IL-1 l Mechanical ventilation with endotracheal intubation is
and TNF. frequently needed in late septic shock as the shock
l These cytokines act on endothelial cells and have a worsens with inadequate oxygenation.
variety of effects including reduced synthesis of antico- l Steroids: The only indications for steroid treatment in
agulation factors such as tissue factor pathway inhibitor patients with septic shock are hypoadrenalism and for
and thrombomodulin. stress coverage in patients taking steroids (or who re-
l The same cytokine and secondary mediators, now at cently completed a course of steroids) for immunosup-
high levels, result in systemic vasodilation (hypoten- pression or anti-inflammatory purposes. 15-30 mg/kg
sion) and diminished myocardial contractility. body weight of methylprednisolone or dexamethasone
l Widespread endothelial injury and activation, causing is given IV. It may be repeated after 4 hours, if neces-
systemic leucocyte adhesion and diffuse alveolar sary. There is hardly any need for the third dose.
Section | II General Surgery 259

l Vasopressor supportive therapy: If the patient does not l Activation of the renin-angiotensin system: Angiotensin
respond to several liters of volume infusion with iso- II facilitates arteriolar vasoconstrictor that stimulates
tonic crystalloid solution (usually 4 L or more) or renal prostaglandin production as well as the release of
evidence of volume overload is present, the depressed aldosterone and ACTH. This represents the principal
cardiovascular system can be stimulated by inotropic mechanism by which the kidney may excrete the accu-
and vasoconstrictive agents. When proper fluid resus- mulated byproducts of anaerobic metabolism and cel-
citation fails to restore haemodynamic stability and lular damage.
tissue perfusion, initiate therapy with vasopressor l Vasopressin from the posterior pituitary is released,
agents. These agents are dopamine, norepinephrine, causing water retention at the distal tubules.
epinephrine, and phenylephrine. These agents are va- l Tissue hypoperfusion from precapillary vasoconstric-
soconstricting drugs that maintain adequate blood tion leads to anaerobic metabolism and acidosis.Tissue
pressure during life-threatening hypotension and hypoxia, acidosis, and the release of various mediators
preserve perfusion pressure for optimizing flow in lead to a systemic inflammatory response.
various organs.
Q. 5. Describe the pathophysiology, clinical features and Clinical Features
management of haemorrhagic shock. l Anxiety, restlessness, altered mental state due to de-
Ans. creased cerebral perfusion and subsequent hypoxia
l Hypotension due to decrease in circulatory volume
Haemorrhagic shock is defined as shock associated with the l Tachycardia and weak thready pulse due to decreased
sudden and rapid loss of significant amounts of blood. blood flow combined with tachycardia
l Cool, clammy skin due to vasoconstriction and stimula-
Pathophysiology of Haemorrhagic Shock tion of vasoconstriction
l Rapid and shallow respirations (tachypnoea) due to
l Acute haemorrhage causes a decreased cardiac output sympathetic nervous system stimulation and acidosis
and decreased pulse pressure. These changes are l Hypothermia due to decreased perfusion and evapora-
detected by baroreceptors in the aortic arch and atrium. tion of sweat
l As intravascular volume is lost, an increase in periph- l Thirst and dry mouth due to fluid depletion
eral vascular resistance and redistribution of blood from l Fatigue due to inadequate oxygenation
certain non-vital organs, such as the skin, gastrointesti- l Cold and mottled skin (cutis marmorata), especially
nal tract, and kidneys occur. extremities, due to insufficient perfusion of the skin.
l A decrease in circulating blood volume also results in l Pallor
tachycardia in response to decreased stroke volume l Fainting
from inadequate preload. l Oliguria/anuria due to decreased renal perfusion and
The following sequence of events leads to activation of afferent arteriolar vasoconstriction.
compensatory mechanism:
l Increased sympathetic activity, greater myocardial con- Classification of Haemorrhagic Shock
tractility, and enhanced venous return.
l Decreased capillary hydrostatic pressure and mobiliza- Class I Class II Class III Class IV
tion of the interstitial fluid pool into the intravascular Blood loss Up to 750- 1500-2000cc .2000cc
space. 750cc 1500cc
l Arteriolar constriction and loss of circulating volume, % Volume Up to 15% 15-30% 30-40% .40%
which diminish renal blood flow.
Pulse ,100 .100 .120 .140
l Release of epinephrine and norepinephrine, which
produce vasoconstriction and tachycardia, resulting in BP Normal Normal Decreased Decreased
increased cardiac output and blood pressure. Respiratory 14-20 20-30 30-40 .35
l Stimulation of adrenocorticotropic hormone (ACTH) rate
release. Pulse Normal/ Decreased Decreased Decreased
l Decreased insulin secretion, which augments the pressure increased
mobilization of glucose, amino acids, and fat stores.
Urine .30 20-30 5-15 Negligible
l Increased antidiuretic hormone (ADH) secretion, output
which increases water permeability and passive sodium
Mental Slightly Mildly Anxious, Confused,
transport, allowing increased water resorption and
status anxious anxious confused lethargic
splanchnic vasoconstriction.
260 Quick Review Series for BDS 3rd Year

Measurement of blood loss Operative


It is mandatory to detect and measure the amount of i. Clamping the bleeding vessels with catgut, thread or
blood, be it external or internal, so that the lost blood can silk.
be replaced. ii. Coagulation by thermocautery or diathermy.
iii. By local application of adrenaline swabs.
Method of measuring blood loss: iv. Application of silver clips as in neurosurgery.
v. Under running or transfusion of vessels by needle and
i. Weighing the swabs: The swabs are weighed, when suture.
they are dry and after soaking with blood. The differ- vi. Application of crushed piece of muscle at the site of
ence is blood loss 1 gm 5 1 ml of blood. There should bleeding.
be an addition of 50 to 100% according to short or long vii. Bone wax or BIPP (bismuth iodoform paraffin paste)
operation to combat the loss due to evaporation. is used for oozing from bone.
ii. Measurement of blood clot: It can be weighed or viii. Repair of vascular defect by patches of vein or Dacron
roughly seen the size of a clenched fist equals to 500 ml. mash.
iii. Haemoglobin level: Initial stage haemoglobin is com- ix. Excision of bleeding organ, e.g. splenectomy.
pensated and so not lowered. After few hours, it comes
down due to haemodilution. C. Restoration of blood volume after
iv. Central venous pressure (CVP): A good method to haemorrhage
measure the blood volume.
i. Resuscitation from haemorrhage should include restora-
tion of circulating volume by placement of two large-
Management of Haemorrhagic Shock bore IVs, and rapid infusion of a balanced crystalloid
Early resuscitation includes control of bleeding and restora- solution. Ringer’s lactate is preferred over normal iso-
tion of circulating blood volume for oxygenation of tissues. tonic crystalloid or colloid solutions can be used for
volume replacement in haemorrhage.
A. General management - hospitalization ii. Blood and its components transfusion: Many blood prod-
l Care of critically ill patients starts with A, B, C (air- ucts are available to restore circulating volume, and re-
ways, breathing, circulation). place coagulation factors and oxygen-carrying capacity.
l Oxygen should be administered. Hypovolaemia is best corrected with crystalloid solution.
l Intravenous line: Urgent intravenous administration of In haemorrhagic shock, packed red blood cells (PRBCs)
isotonic saline or a balanced crystalloid solution to re- are most commonly used to restore intravascular volume
store the blood volume to normal. Colloids, such as and oxygen-carrying capacity. Platelet transfusions are
gelatins, have also been used. indicated in continued haemorrhage.
l Rest: Anxious, apprehensive, restless persons bleed more Q. 6. Describe indications and complications of blood
due to excitement and fear. So he should be sedated. transfusion.
B. Specific measures to stop haemorrhage Ans.
Conservative Blood transfusion is the process of transferring blood or
l Pressure: Most of the bleeding can be stopped by firm blood-based products from one person into the circulatory
pressure by a sterile gauge or cotton. If sterile bandages system of another.
are not available, any cloth would do, as controlling the Blood transfusions can be grouped into two main types
bleeding is more important. Use of double balloon in depending on their source:
the oesophagus and stomach to control the bleeding l Homologous transfusions or transfusions using the
from oesophageal varices. stored blood of others. These are often called allogeneic
l Tourniquet: It comes very handy in an emergency but it instead of homologous.
has its limitations. It can only be used in the limbs. It l Autologous transfusions, or transfusions using the
should be applied on to the arm or thigh, tight enough patient’s own stored blood.
to occlude the veins and arteries. The maximum time to
keep the tourniquet is 1 to 11⁄2 hour.
Indications of Blood Transfusion
l Elevation of bleeding area should be raised against the
gravity. If there is bleeding from head to neck and thy- l Acute haemorrhage: External or internal.
roid area, head and neck should be elevated. If it is in l A major operation where blood loss is expected.
the lower limbs, they should be elevated. If the oozing l Deep burns where there is considerable tissue destruc-
is from the veins, it will stop as the veins get collapsed. tion, haemolysis and destruction of RBCs.
Section | II General Surgery 261

l Severe anemia: Preoperatively or postoperatively. e. Frusemide 80-120 mg IV should be given for forced
l Severe anaemia where the haemoglobin is less than diuresis. This may be repeated, if urine output is
10 gms%. ln case of chronic anaemias, it is better to increased to 30 ml/hr.
transfuse the packed cells to reduce a load on the heart f. Antihistamine and hydrocortisones should be given.
due to hypervolaemia. g. In very severe cases, haemodialysis should be undertaken.
l Blood dyscrasias like haemophilia, thrombocytopenic
purpura, leukaemia, thalassaemia, aplastic anaemia, etc. II. Pyrexial reactions:
l Erythroblastosis foetalis due to Rh incompatibility. Ex- It is common to see simple reactions like pyrexia, chills,
change transfusion should be performed through um- rigors, restlessness, headache, tachycardia, nausea and
bilical vein of the newborn. vomiting. The causes can be multiple.
l Chemotherapy for malignant diseases can suppress the l Improperly sterilized drip sets.
haemopoeisis (blood forming system). Hence they may l Presence of pyrogens in the donor set.
require blood transfusions. l Transfusion of infected blood.
l Debilitating diseases like uraemia, liver failure, hyper- l Very rapid transfusion
tension, etc. may require blood transfusion.
Prevention: These reactions can be prevented by using
sterilized plastic disposable sets.
Complications of Blood Transfusion Treatment: Transfusion should be stopped immediately
A. Transfusion reactions though temporarily. Antihistaminies and antipyretics should
I. Incompatibility: be given. After his condition returns to normal, blood trans-
fusion can be restarted.
The reasons being:
l Mistake in crossmatching: A technical error, if the sera III. Allergic reactions:
are old or labelling is wrong. Usually, within few hours of transfusion, patients may get
l Transfusion of blood which is already haemolysed by mild urticaria, tachycardia, fever and dyspnoea. He may
warming, over freezing or shaking. even go into severe anaphylactic shock.
l Transfusion of blood after expiry date.
Treatment: Transfusion should be stopped. Antihistamines
Clinical features of mismatched blood transfusion: and cortisones should be given. Shock should be treated.
l The patient develops rigors and fever. He may complain
of nausea, vomiting, headache, pain in the loins, B. Transmission of diseases
tingling sensation in the extremities. 1. Serum hepatitis: Hepatitis B is a common disease in
l Pain in the loin is due to acute tubular necrosis. India which can be transmitted during blood transfu-
l There may be a chest pain and dyspnoea. sion. The symptoms usually appear within 3 months.
l If the patient is already in shock, it may become more 2. AIDS: HIV can be transmitted from the donor’s blood
pronounced instead of curing it. to the recipient.
l Gradually, he will lose consciousness. 3. Bacterial infection: This occurs due to faulty storage
l Urine output gets diminished and haemoglobinuria can technique. This should be treated with higher antibiotics
occur within 2-3 hrs. otherwise patient may go into septicaemia.
l Jaundice may appear within 24-36 hrs which is the
confirmatory evidence of mismatching. C. Reaction due to massive blood transfusion
l Ultimately renal failure sets in due to the blockage of “Massive blood transfusion implies single transfusion of
renal tubules by haematin pigment. 8-10 units of blood in 24 hrs.”
l Acid-base imbalance results in significant metabolic
alkalosis.
Treatment
l Hyperkalaemia
a. Transfusion should be stopped immediately. l Citrate toxicity: After massive blood transfusion, in-
b. Fresh blood specimen of venous blood and urine from creased citrate level consumes ionised calcium from
the patient should be sent to laboratory for rechecking patient’s body. The body compensates it by rapidly
along with the rejected blood pack. mobilizing calcium from the bones. Rarely when
c. IV fluids should be started instead of blood. hypocalcaemia is recognized calcium can be infused.
d. Alkalization of blood to be done by isotonic solution l Hypothermia: During massive blood transfusion, cold
of sodium lactate and 10 ml of sodium bicarbonate to blood is rapidly infused from the refrigerator to the
facilitate precipitation of haematin pigments. patient. His temperature may drop by 3-4°C.
262 Quick Review Series for BDS 3rd Year

l Failure of coagulation: After massive blood transfusion, c) Distributive shock


the natural process of coagulation may fail due to This type of shock occurs when the afterload is excessively
dilution of platelets and various clotting factors. reduced due to extensive vasodilatation and is associated
l Disseminated intravascular coagulation (DIC). This not only with poor vascular tone in the peripheral circula-
may occur after a massive blood transfusion. Actually, it tion but maldistribution of blood flow to organs within the
occurs after mismatched blood transfusion. body also.
l Septic shock: This type of shock is mostly due to re-
D. Complications of over transfusion
lease of endotoxins in blood by Gram-negative bacteria.
These complications may be seen in patients with chronic l Neurogenic shock: This type of distributive shock is
anaemia, in children and elderly patients. They should caused by the suppression or outright loss of sympathetic
receive packed cells rather whole blood. Transfusions tone caused any disruption of the sympathetic nervous
should be given slowly for 4-6 hours and after some inter- system like spinal injury, spinal anaesthesia, and drugs.
vals. Elderly patients should be given packed cells with l Anaphylactic shock: This type of shock is a result of
diuretics. type I hypersensitivity reaction and is caused when the
body’s antibody-antigen response is triggered by some-
E. Complications of intravenous transfusions thing the person is allergic to. Drugs, like penicillin,
Thrombophlebitis and air embolism. cephalosporins, iodinated contrast media, serum, etc.
are common causes of this type of shock.
F. Pulmonary complications
Syndrome of transfusion-related acute lung injury (TRALI) d) Obstructive shock
is defined as noncardiogenic pulmonary oedema related to This type of shock associated with physical obstruction of
transfusion the great vessels or the heart itself.
Q. 7. Define and classify shock. Discuss the management
of hypovolaemic shock. e) Traumatic shock
Management of hypovolaemic shock:
Ans.
The objectives are to increase the cardiac output and im-
Shock is defined as an acute clinical syndrome characterized prove the tissue perfusion especially in coronary, cerebral,
by a significant, systemic reduction in tissue perfusion (hypo- renal and mesenteric vascular beds.
perfusion), resulting in decreased tissue oxygen delivery and A. Resuscitation: Maintenance of adequate airway venti-
insufficient removal of cellular metabolic products, resulting lation and proper oxygenation.
in tissue injury and severe dysfunction of vital organs. B. Head low position face turned to one side: If necessary,
endotracheal intubation.
Classification of Shock C. Control of haemorrhage
i. Fluid resuscitation: This should start immediately. A
a) Hypovolaemic shock large gauge needle or a cannula is introduced into an
l This type of shock is due to loss of blood plasma or appropriate vein in the arm or leg and fluid should be
body fluid and electrolytes, usually caused by massive introduced immediately. CVP monitoring will guide
haemorrhage, vomiting, diarrhoea, and dehydration. us regarding the amount of fluid replacement:
l This is the most common type of shock. It is character- l Lactated Ringer’s solution: This equilibrates rap-
ized by a loss in circulatory volume, which results in idly throughout the extracellular compartment,
decreased venous return, decreased filling of the cardiac restoring the extracellular fluid deficit associated
chambers, and hence a decreased cardiac output which with blood loss.
leads to increase in the systemic vascular resistance l Hypertonic saline: A small volume of hypertonic
(SVR). saline can be an effective initial resuscitative
solution.
b) Cardiogenic shock l Hetastarch: Hydroxyethyl starch (hetastarch) is
l This type of shock is due to the dysfunction of one ven- an artificial colloid derived from amylopectin that
tricle or other, usually seen in myocardial infarction, has colloidal properties similar to those of albu-
chronic congestive cardiac failure, cardiac arrhythmias, min but less expensive.
pulmonary embolism, etc. resulting in inability of the l Dextran: Dextran, in 40- and 70-kD solutions,
heart to pump the adequate amount of blood into can also be used as a plasma expander.
the lungs and decreased cardiac output. MI is the most l Blood substitutes: If blood is not available, blood
common cause of cardiogenic shock. substitutes, like plasma or dextran, should be used.
Section | II General Surgery 263

l Correcting acid base disturbance using molar sodium iii. Vascular compromise to the spinal cord with loss of
bicarbonate when the pH of arterial blood is less than autoregulation, vasospasm, and thrombosis.
6.2 and electrolyte abnormalities specially potassium iv. Neurotransmitter accumulation and release of free radi-
and calcium. cals. Importantly, hypotension contributes to the wors-
D. Drugs: ening of acute spinal cord injury as the result of further
a) Sedatives: To alleviate the pain and anxiety, drugs reduction in blood flow to the spinal cord.
should be used. Morphine should be given intrave- v. Sympathetic input to the heart which normally in-
nously as the peripheral vasoconstriction in shock creases heart rate and cardiac contractility, and input to
will delay the absorption of the drug, if given the adrenal medulla which increases catecholamine re-
intramuscularly. Pethidine is also a good agent. lease, gets disrupted, preventing the typical reflex
tachycardia that occurs with hypovolaemia.
Q. 8. What is shock? What are the varieties of shock?
Describe the neurogenic shock and its management.
Clinical Features of Neurogenic Shock
Ans.
l History of emotional stress or pain of a sudden nature.
Shock is defined as an acute clinical syndrome character- l Hypotension with bradycardia due to absence of reflex-
ized by a significant, systemic reduction in tissue perfusion ive tachycardia due to disrupted sympathetic discharge.
(hypo-perfusion), resulting in decreased tissue oxygen de-
l Warm, dry extremities as a result of loss of peripheral
livery and insufficient removal of cellular metabolic prod-
vasoconstriction.
ucts, resulting in tissue injury and severe dysfunction of
l Motor and sensory deficits indicative of a spinal cord
vital organs.
injury
l Tachypnoea
Varieties of Shock l Unconsciousness
l Anxiety, restlessness.
a) Hypovolaemic shock l Reflexes are usually intact.
b) Cardiogenic shock
c) Distributive shock
l Septic shock Management of Neurogenic Shock
l Neurogenic shock
a) Airway potency and adequate ventilation.
l Anaphylactic shock
b) Fluid resuscitation and restoration of intravascular vol-
d) Obstructive shock
ume: Most patients with neurogenic shock will respond
e) Traumatic shock
to restoration of intravascular volume alone, with satis-
Neurogenic shock: This type of distributive shock is factory improvement in perfusion and resolution of hy-
caused by the suppression or outright loss of sympathetic potension.
tone caused due to any disruption of the sympathetic c) Vasoconstrictors (dopamine, phenylephrine): They im-
nervous system. prove peripheral vascular tone, decrease vascular ca-
pacitance, and increase venous return. However, they
Causes of neurogenic shock: should be considered only after hypovolaemia is ex-
l Spinal cord trauma cluded as the cause of the hypotension.
l Spinal cord neoplasm l The duration of the need for vasopressor support for

l Spinal/epidural anaesthesia neurogenic shock may correlate with the overall progno-
sis or chances of improvement in neurologic function.
Neurogenic shock usually occurs due to spinal cord in- l Appropriate rapid restoration of blood pressure and
juries from vertebral body fractures of the cervical or high circulatory perfusion may improve perfusion to the
thoracic region resulting in disruption of sympathetic regu- spinal cord, prevent progressive spinal cord isch-
lation of peripheral vascular tone. aemia, and minimize secondary cord injury.

Q. 9. What is haemorrhage and types of haemorrhage?


Pathophysiology How will you manage a case of primary haemorrhage
i. Loss of sympathetic tone (parasympathetic response) after a dental extraction?
results in massive vasodilatation, inhibition of the baro-
Ans.
receptor response, and impaired thermoregulation.
ii. Arterial vasodilatation results in hypotension, which in Haemorrhage is defined as escape of blood from blood ves-
turn reduces cardiac output and impaired tissue perfusion. sels, i.e extravasation of blood due to vessel rupture.
264 Quick Review Series for BDS 3rd Year

Types of Haemorrhage l Class III hemorrhage - involves loss of 30-40% of


circulating blood volume.
I. Haemorrhage may be:
l Class IV hemorrhage - involves loss of .40% of
l Arterial: Bright red blood comes out like a spurting
circulating blood volume.
jet, which rises and falls in time with pulse.
l Venous: Dark red haemorrhage as a steady and co- Management of primary haemorrhage after a dental
pious flow. extraction:
l Capillary: Blood is bright red often rapid oozing.
l Post-extraction bleeding usually occurs in the small ves-
l External: Bleeding either through a natural opening
sels. Any clots extending out of the socket are removed
such as the vagina, mouth, nose, ear or anus, or
with gauze, and a 4-in gauze pad (folded) or a tea bag is
through a break in the skin.
placed over the socket.
l Internal: Where blood leaks from blood vessels in-
l Then the patient is instructed to apply continuous pres-
side the body.
sure by biting for 1 h. The procedure may have to be
II. Haemorrhage can also be classified as:
repeated 2 or 3 times.
l Primary: Occurs at the time of injury or operation.
l Patients are told to wait at least 1 h before checking the
l Reactionary: Bleeding that occurs within 24 hours
site so as not to disrupt clot formation. They also are
(usually 4-6 hours) after injury or operation. It is
informed that a few drops of blood diluted in a mouth
mainly due to rolling (slipping) of a ligature, dis-
full of saliva appear to be more blood than is actually
lodgement of a clot or cessation of reflex vasospasm.
present.
l Secondary: This occurs after 7-10 days post-trauma.
l If bleeding continues, the site may be anaesthetized
This is due to infection and sloughing of arterial walls.
by nerve block or local infiltration with 2% lidocaine
III. Haemorrhage can be classified into four classes by the
containing 1:100,000 epinephrine.
American College of Surgeons’ Advanced Trauma
l The socket is then curetted to remove the existing clot
Life Support (ATLS).
and to freshen the bone and is irrigated with normal
l Class I hemorrhage - involves up to 15% of blood
saline. Then the area is sutured under gentle tension.
volume.
l Local haemostatic agents, such as oxidized cellulose,
l Class II hemorrhage - involves 15-30% of total
topical thrombin on a gelatin sponge, or microfibrillar
blood volume.
collagen, may be placed in the socket before suturing.

SHORT ESSAYS
Q. 1 Describe briefly vasovagal shock. Clinical Features
Ans. l History of emotional stress or pain of a sudden nature.
l Bradycardia or pallor.
l Vasovagal shock is response to sudden fear or severe
l Hypotension
pain and manifests itself from slight fainting fit to death.
l Tachypnoea
l This type of shock is also known as neurogenic or psy-
l Fainting
chogenic shock. There is sudden pooling of blood in the
l Reflexes are usually intact.
capacitance vessels of legs and splanchnic arterial bed.
This causes hypoxia of vital centre.
Treatment
Pathophysiology l Place the patient flat or in head low position.
l Ensure potency of airway.
l The nucleus tractus solitarius of the brainstem is acti-
l IV atropine may be needed for persistent or increasing
vated directly or indirectly by the triggering stimulus,
bradycardia.
resulting in simultaneous enhancement of parasympa-
thetic nervous system (vagal) tone and withdrawal of Q. 2. Describe briefly hypovolaemic shock.
sympathetic nervous system tone, this leads to either
Ans.
cardioinhibitory response, characterized by a drop in
heart rate (negative chronotropic effect) and in contrac- l Hypovolaemic shock is due to loss of blood plasma or
tility (negative inotropic effect) leading to a decrease in body fluid and electrolytes, usually caused by massive
cardiac output and unconsciousness or vasodepressor haemorrhage, vomiting, diarrhoea, and dehydration.
response, caused by a drop in blood pressure (to as low l This is the most common type of shock. It is character-
as 80/20) without much change in heart rate. ized by a loss in circulatory volume, which results in
Section | II General Surgery 265

decreased venous return, decreased filling of the cardiac Clinical Features


chambers, and hence a decreased cardiac output which
l Prolonged bleeding and re-bleeding are the diagnostic
leads to increase in the systemic vascular resistance
symptoms of haemophilia. Internal bleeding is common
(SVR).
in people with severe haemophilia and some individuals
with moderate haemophilia.
Causes l The most characteristic type of internal bleed is a recur-
rent painful haemarthroses and muscle haematomas.
i. Loss of extracellular fluid: For example, vomiting, diar-
l First symptoms are often frequent and large bruises and
rhoea, intestinal obstruction, increased sweating, fail-
haematomas from frequent bumps and falls as they
ure of sodium pump caused by hypoxia.
learn to walk. Swelling and bruising from bleeding in
ii. Plasma loss, e.g. burns.
the joints, soft tissue, and muscles may also occur.
iii. Haemorrhage: Due to whole blood loss like:
l Increase in whole blood clotting time and PTT.
a. Surgical: During and following any major
surgery.
b. Traumatic: As a result of any type of major Complications
accident, warfare injuries, homicidal, etc.
l Deep internal bleeding, e.g. deep-muscle bleeding,
c. GI bleeding: Bleeding from peptic ulcer, perfora-
leading to swelling, numbness or pain of a limb.
tion of intestine, bleeding from oesophageal varices,
l Joint damage from haemarthrosis, potentially with se-
etc.
vere pain, disfigurement, and even destruction of the
d. Obstructive bleeding: Incomplete abortion, placenta
joint and development of debilitating arthritis. Transfu-
previa, etc.
sion transmitted infection from blood transfusions that
are given as treatment.
Clinical Features l Adverse reactions to clotting factor treatment, including
the development of an immune inhibitor which renders
l Anxiety, restless, excitation and disorientation
factor replacement less effective.
l Pallor, thirst, cold and clammy skin
l Intracranial haemorrhage is a serious medical emer-
l Fainting in upright position
gency caused by the buildup of pressure inside the skull.
l Tachycardia with rapid, thready pulse, tachypnoea
l Hypotension
l Oliguria or anuria. Treatment
l Haemophilia A: Factor VIII replacement for bleeding
Management or before surgical procedure. Duration and dose of
replacement depends on severity of bleeding.
l Control haemorrhage.
l Haemophilia B: FFP/factor IX concentrates.
l IV fluid to restore circulating blood volume.
l Blood transfusion. Q. 4. Describe briefly complications of blood transfu-
l Rising of foot end of bed. sion.
l Oxygen inhalation.
l If acidosis develops, sodium bicarbonate is infused, Ans.
untill pH is normalized.
Complications of blood transfusion can be broadly classi-
l Recovery is indicated by improvement in conscious-
fied into following categories:
ness, dry and warm skin, CVP is more than 5 cm H2O,
distended veins and increased in urinary output. A. Transfusion reactions:
Q. 3. Describe briefly haemophilia. i. ABO incompatibility: The reasons being:
l Mistake in crossmatching: A technical error, if the
Ans.
sera are old or labelling is wrong.
Haemophilia is a group of hereditary genetic disorders that l Transfusion of blood which is already haemolysed
impair the body’s ability to control blood clotting or coagu- by warming, over freezing or shaking.
lation. It affects the intrinsic pathway of coagulation and l Transfusion of blood after expiry date.
can be classified as: ii. Pyrexial reactions: It is common to see simple reactions
a) Haemophilia A/classic haemophilia (clotting factor like pyrexia, chills, rigors, restlessness, headache, tachy-
VIII deficiency): Recessive X-linked. cardia, nausea and vomiting. The causes can be multiple.
b) Haemophilia B/ christmas disease (factor IX deficiency): l Improperly sterilized drip sets.
Recessive X-linked. l Presence of pyrogens in the donor set.
266 Quick Review Series for BDS 3rd Year

l Transfusion of infected blood. l Blood dyscrasias like haemophilia, thrombocytopenic


l Very rapid transfusion. purpura, leukaemia, thalassaemia, aplastic anaemia, etc.
iii. Allergic reactions: Usually, within few hours of l Erythroblastosis foetalis due to Rh incompatibility.
transfusion, patients may get mild urticaria, tachycar- Exchange transfusion should be performed through
dia, fever and dyspnoea. He may even go into severe umbilical vein of the newborn.
anaphylactic shock. l Chemotherapy for malignant diseases can suppress
the haemopoiesis. Hence they may require blood
B. Transmission of diseases: transfusions.
l Serum hepatitis: Hepatitis B is a common disease l Debilitating diseases like uraemia, liver failure, hyper-
in India which can be transmitted during blood tension, etc. may require blood transfusion.
transfusion. The symptoms usually appear within
3 months. Q. 6. Describe briefly pathophysiology of haemorrhagic
l AIDS: HIV virus can be transmitted from the donor’s shock.
blood to the recipient.
l Bacterial infection: This occurs due to faulty storage Ans.
technique.This should be treated with higher antibiotics
otherwise patient may go into septicaemia. Pathophysiology of haemorrhagic shock:
l Acute haemorrhage causes a decreased cardiac output
C. Reaction due to massive blood transfusion: and decreased pulse pressure. These changes are
“Massive blood transfusion implies single transfusion of detected by baroreceptors in the aortic arch and atrium.
8-10 units of blood in 24 hrs.” l As intravascular volume is lost, an increase in periph-
l Acid-base imbalance results in significant metabolic eral vascular resistance and redistribution of blood from
alkalosis. certain non-vital organs, such as the skin, gastrointesti-
l Hyperkalaemia. nal tract, and kidneys.
l Citrate toxicity. l A decrease in circulating blood volume also results in
l Hypothermia. tachycardia in response to decreased stroke volume
l Failure of coagulation. from inadequate preload.
l Disseminated intravascular coagulation (DIC). l This sequence of events leads to activation of compen-
satory mechanism as given below:
D. Complications of over transfusion: i. Increased sympathetic activity, greater myocardial
These complications may be seen in patients with chronic contractility, and enhanced venous return.
anaemia, in children and elderly patients. ii. Decreased capillary hydrostatic pressure and
mobilization of the interstitial fluid pool into the
E. Complications of intravenous transfusions: intravascular space.
Thrombophlebitis and air embolism. iii. Arteriolar constriction and loss of circulating
volume, which diminish renal blood flow.
F. Pulmonary complications: iv. Release of epinephrine and norepinephrine, which
Syndrome of transfusion-related acute lung injury (TRALI) produce vasoconstriction and tachycardia, result-
is defined as noncardiogenic pulmonary oedema related to ing in increased cardiac output and blood pressure.
transfusion. v. Stimulation of adrenocorticotropic hormone
(ACTH) release.
Q. 5. Describe briefly the indications of blood transfusion. vi. Decreased insulin secretion, which augments the
mobilization of glucose, amino acids, and fat stores.
Ans.
vii. Increased antidiuretic hormone (ADH) secretion,
Indications of blood transfusion are as follows: which increases water permeability and passive
l Acute haemorrhage: External or internal. sodium transport, allowing increased water resorp-
l A major operation where blood loss is expected. tion and splanchnic vasoconstriction.
l Deep burns where there is considerable tissue destruc- viii. Activation of the renin-angiotensin system: Angio-
tion, haemolysis and destruction of RBCs. tensin II facilitates arteriolar vasoconstrictor that
l Severe anaemia: Preoperatively or postoperatively. stimulates renal prostaglandin production as well
l Severe anaemia where the haemoglobin is less than 10 as the release of aldosterone and ACTH. This rep-
gm%. ln case of chronic anaemias, it is better to trans- resents the principal mechanism by which the
fuse the packed cells to reduce a load on the heart due kidney may excrete the accumulated byproducts of
to hypervolaemia. anaerobic metabolism and cellular damage.
Section | II General Surgery 267

ix. Vasopressin from the posterior pituitary is released, II. Specific measures to stop haemorrhage:
causing water retention at the distal tubules. i. Conservative:
x. Tissue hypoperfusion from precapillary vasocon- l Pressure: Most of the bleeding can be stopped by firm
striction leads to anaerobic metabolism and acidosis. pressure by a sterile gauge or cotton.
Tissue hypoxia, acidosis, and the release of various l Tourniquet: The maximum time to keep the tourniquet
mediators lead to a systemic inflammatory response. is 1 to 11⁄2 hour.
Q. 7. Describe briefly management of haemorrhage. l Elevation of bleeding area should be raised against the
gravity.
Ans.
Haemorrhage is defined as escape of blood from blood ves- ii. Operative:
sels, i.e extravasation of blood due to vessel rupture. Clamping, coagulation, repair of vascular defect, excision
of bleeding defect.
Management of Haemorrhage III. Restoration of blood volume after haemorrhage:
I. General management – hospitalization i. The aim is restoration of circulating volume. Ringer’s
l A, B, C (airways, breathing, circulation). Oxygen lactate is preferred over normal saline to avoid hyper-
should be administered. chloraemic acidosis associated with prolonged use of
l Intravenous line: Urgent intravenous administration of sodium solutions.
isotonic saline or a balanced crystalloid solution to ii. Blood and its components transfusion: Many blood
restore the blood volume to normal. products are available to restore circulating volume,
l Rest: Anxious, apprehensive, restless persons bleed more and replace coagulation factors and oxygen-carrying
due to excitement and fear. So he should be sedated. capacity.

SHORT NOTES
Q. 1. Types of shock. ii. Clinical features of septic shock:
l Pyrexia (fever) with chills and rigors.
Ans.
l Systemic vasodilation resulting in hypotension.

Shock can be classified into following types: l Warm and sweaty skin due to vasodilation

i. Hypovolaemic shock: Due to loss of blood plasma or l Hyperventilation with respiratory alkalosis is a

body fluid and electrolytes, usually caused by massive common feature of patients with septic shock.
haemorrhage, vomiting, diarrhoea, and dehydration. iii. Treatment:
ii. Cardiogenic shock: Due to the dysfunction of one l Control of infection by antibiotic treatment

ventricle or other, resulting in inability of the heart to l Early surgical debridement

pump the adequate amount of blood into the lungs and l Fluid replacement

decreased cardiac output. l Vasoactive drugs are ideal to improve the cardiac,

iii. Distributive shock: Due to excessive reduced afterload pulmonary and renal functions with better chances
as a result of extensive vasodilatation. of survival.
l Septic shock l Vasopressor supportive therapy to maintain adequate

l Neurogenic shock blood pressure during life-threatening hypotension.


l Anaphylactic shock
Q. 3. Neurogenic shock.
iv. Obstructive shock: Due to physical obstruction of the
great vessels or the heart itself. Ans.
v. Traumatic shock.
i. Neurogenic shock is a type of distributive shock
Q. 2. Septic shock. and is caused by the suppression or outright loss of
sympathetic tone caused by any disruption of the
Ans.
sympathetic nervous system caused mostly by spinal
i. Septic shock is a subclass of distributive shock caused cord trauma, spinal cord neoplasm and spinal/
due to release of endotoxins in blood mostly by Gram- epidural anaesthesia.
negative bacteria such as Escherichia coli, Proteus spe- ii. Clinical features of neurogenic shock:
cies, Klebsiella pneumoniae, which causes wide-spread l History of emotional stress or pain of a sudden nature.

vasodilatation. l Hypotension with bradycardia.


268 Quick Review Series for BDS 3rd Year

l Warm, dry extremities iii. Treatment:


l Tachypnoea l Application of tourniquet above the level of injury to
l Unconsciousness reduce the haemorrhage and accumulation of blood.
l Reflexes are usually intact. l Multiple tension relieving incisions on the deep fascia.
iii. Management of neurogenic shock: l IV fluids should be restricted to 500 ml.
l Airway potency and adequate ventilation. l Mannitol 20% 1100 ml lV 8th hourly is also very
l Fluid resuscitation and restoration of intravascular helpful. This will prevent acute renal failure.
volume. l Haemodialysis is helpful in acute renal shut down.
l Vasoconstrictors (e.g. dopamine, phenylephrine) to
Q. 7. Rh factor.
improve peripheral vascular tone, decrease vascular
capacitance, and increase venous return. Ans.

Q. 4. Hypovolaemic shock. i. Landsteiner and Weiner in 1940 identified another


agglutinogen in red cells called Rh factor, as it was
Ans. isolated in Rhesus monkey.
i. Hypovolaemic shock is due to loss of blood plasma or ii. It is present in more than 85% individuals and those
body fluid and electrolytes, usually caused by massive having it are called Rh1.
haemorrhage, vomiting, diarrhoea, and dehydration. iii. It differs from agglutinogens A and B, in that no
This is the most common type of shock. corresponding agglutinin exists in plasma.
ii. Clinical features: iv. Six sub-types of Rh factor, i.e. Cc Dd Ee have been de-
l Anxiety, restless, excitation and disorientation
scribed, amongst them D is most common and strongly
l Pallor, thirst, cold and clammy skin
antigenic. It is, therefore, essential that in addition to
l Fainting in upright position
blood groups, Rh factor should also be determined and
l Tachycardia with rapid, thready pulse, tachypnoea
Rh1 blood should not be transfused to an Rh- recipient,
l Hypotension
to avoid hazards during subsequent transfusion.
l Oliguria or anuria.
v. Rh factor determination is done by observing aggluti-
iii. Management: nation of red cells with the anti-D sera.
l Control haemorrhage. Q. 8. Haemophilia.
l IV fluid to restore circulating blood volume.
Ans.
l Blood transfusion.

l Rising of foot end of bed. i. Haemophilia is a group of hereditary genetic disorders


l Oxygen inhalation. that impair the body’s ability to control blood clotting
l If acidosis develops, sodium bicarbonate is infused, or coagulation.
until pH is normalized. ii. It can be classified as:
a. Haemophilia A/classic haemophilia (clotting factor
Recovery is indicated by improvement in consciousness,
VIII deficiency).
dry and warm skin, CVP is more than 5 cm H20,
b. Haemophilia B/ Christmas disease (factor IX deficiency).
distended veins and increased in urinary output.
iii. First symptoms are often frequent and large bruises and
Q. 5. Crush syndrome. haematomas from frequent bumps and falls as they
learn to walk.
Ans. iv. Increase in whole blood clotting time and PTT
i. Crush syndrome (also known as traumatic rhabdomy- v. Treatment:
l Haemophilia A: Factor VIII replacement.
olysis or Bywaters’ syndrome) is a serious medical
l Haemophilia B:FFP/ factor IX concentrates.
condition characterized by major shock and renal
failure following a crushing injury to skeletal muscle. Q. 9. Management of haemorrhage.
For example, cases commonly occur in conditions such
Ans.
as earthquakes, air raids, collapse of building or use of
tourniquet for a long time. i. Haemorrhage is defined as escape of blood from blood
ii. Pathophysiology: vessels, i.e. extravasation of blood due to vessel rupture.
l There is extravasation of blood into the crushed ii. Management of haemorrhage:
muscles and tissues leading to oligaemic shock. a. General management:
l The myohaemoglobin liberated from the crushed l Hospitalization
muscles, enters the circulation which will cause l A, B, C (airways, breathing, circulation)
renal tubular necrosis along with ischaemia of limb. l Rest
Section | II General Surgery 269

b. Specific measures to stop haemorrhage: Q. 12. Reactionary haemorrhage.


l Conservative pressure, tourniquet, elevation of

bleeding area should be raised against the gravity. Ans.


l Operative: Clamping, coagulation, repair of vascu- i. Reactionary haemorrhage is defined as the bleeding that
lar defect, excision of bleeding defect. occurs within 24 hours (usually 4-6 hours) after injury
c. Restoration of blood volume after haemorrhage. or operation. It is mainly due to rolling (slipping) of a
Q. 10. Blood transfusion. ligature, dislodgement of a clot or cessation of reflex
vasospasm.
Ans. ii. The precipitating factors of reactionary bleeding are:
l The rise in blood pressure and the refilling of the
i. Blood transfusion is the process of transferring blood or
blood-based products from one person into the circulatory venous system on recovery from shock.
l Restlessness, coughing and vomiting which raise the
system of another.
ii. Blood transfusions can be grouped into two main types venous pressure.
depending on their source: Q. 13. Complications of blood transfusion.
l Homologous transfusions or transfusions using

the stored blood of others. These are often called Ans.


allogeneic instead of homologous.
Complications of blood transfusion are as follows:
l Autologous transfusions, or transfusions using the
A. Transfusion reactions:
patient’s own stored blood.
i. ABO incompatibility:
iii. Blood transfusion is mainly indicated in:
ii. Pyrexial reactions: It is common to see simple reac-
l Acute haemorrhage: External or internal.
tions like pyrexia, chills, rigors, restlessness, headache,
l A major operation where blood loss is expected.
tachycardia, nausea and vomiting.
l Deep burns
iii. Allergic reactions: Urticaria, purpura, tachycardia,
l Severe anaemia: Preoperatively or postoperatively.
fever, dyspnoea, anaphylactic shock.
Q. 11. Haemophilia-A. B. Transmission of diseases:
l Hepatitis B, HIV, bacterial infections
Ans.
C. Reaction due to massive blood transfusion:
i. Haemophilia-A is the most common form of haemo- l Acid-base imbalance results in significant
philia and is X-linked recessive trait, occurs in males. metabolic alkalosis, hyperkalaemia, citrate toxicity,
ii. It is caused due to deficiency of factor VIII. hypothermia, disseminated intravascular coagulation
iii. First symptoms are often frequent and large bruises and (DIC).
haematomas from frequent bumps and falls as they D. Complications of over transfusion: Seen in patients with
learn to walk. chronic anaemia, in children and elderly patients.
iv. Increase in whole blood clotting time and PTT. E. Complications in general of intravenous transfusions:
v. The most characteristic type of internal bleed is a recurrent Thrombophlebitis and air embolism.
painful haemarthroses and muscle haematomas. F. Pulmonary complications: Syndrome of transfusion-
vi. Severe haemophilia patients require regular supplementa- related acute lung injury (TRALI).
tion with intravenous recombinant or plasma concentrate
factor VIII.

Topic 5

Bacterial Infections and Transmissible Viral Infections

LONG ESSAYS
Q. 1. Describe the etiology, pathology, clinical features Ans.
and management of actinomycosis
Actinomycosis is a chronic bacterial infection.
270 Quick Review Series for BDS 3rd Year

Aetiology l Sinus tracts with drainage from the chest wall (i.e. pleu-
rocutaneous fistula)
Actinomycosis is caused by filamentous, gram-positive,
l Fever, weight loss, fatigue, anorexia.
non-acid-fast, non-spore-forming anaerobic, microaero-
philic bacteria. C. Abdominal actinomycosis
l History of abdominal surgery, perforated viscus, mesen-
Pathophysiology teric vascular insufficiency, or ingestion of foreign bodies.
l Sinus tracts with drainage from either the abdominal
l As these microorganisms are not virulent, there is a
wall, i.e. peritoneocutaneous fistula or the perianal
requirement for a break in the integrity of the mucosa
region.
and the presence of devitalized tissue to invade deeper
l Nonspecific symptoms like: Low-grade fever, weight
body structures and to cause human illness.
loss, fatigue, change in bowel habits, vague, abdominal
l At the same time, actinomycosis is generally a polymi-
discomfort, nausea, vomiting, and sensation of a mass.
crobial infection, human infection requires the presence
of companion bacteria, which participate in the produc- D. Pelvic actinomycosis
tion of infection by elaborating a toxin or enzyme or by
l History of IUCD.
inhibiting host defenses.
l Lower abdominal discomfort, abnormal vaginal bleed-
l Once infection is present, the host mounts an intense
ing or discharge.
inflammatory response followed by fibrosis.
l Infection typically spreads contiguously, frequently
ignoring tissue planes and invading surrounding tissues Management
or organs.
A. Diagnosis
l Ultimately, the infection produces draining sinus tracts.
Haematogenous dissemination to distant organs may i. CBC count: Anaemia and mild leucocytosis.
occur in any stage of actinomycosis, whereas lymphatic ii. ESR and C-reactive protein (CRP) levels are often
dissemination is unusual. elevated.
iii. Culture of organism:
l Direct identification and/or isolation of the infecting
Clinical Symptoms organism from a clinical specimen or from sulphur
A. Cervicofacial actinomycosis granules is necessary for definitive diagnosis in most
cases.
l History of dental trauma to the mouth, poor oral hy- l A Gram-stained smear of the specimen may demon-
giene, dental caries, or periodontal disease; local tissue strate the presence of beaded, branched, gram-
damage caused by neoplasm or osteonecrosis of the jaw positive filamentous rods, suggesting the diagnosis
or maxilla due to radiation treatment may be the cause of actinomycosis.
of disease. l The preliminary diagnosis of actinomycosis also can
l Painless or occasionally painful soft-tissue swelling in- be made by examining sulphur granules.
volving the submandibular or perimandibular region iv. Chest radiography, CT scanning, ultrasound-guided
with increase in size and number which ultimately form fine-needle aspiration and/or biopsy, surgery (e.g. tho-
sinuses that open on to the cheek or submandibular area. racotomy with open lung biopsy, exploratory laparot-
The exudate usually contains sulphur granules. omy) may be required for diagnostic purposes.
l Reddish or bluish discolouration of the skin overlying
the lesion.
l Nodules may be tender in the initial stages are typically B. Treatment
nontender and woody hard in the later stages. l Penicillin G is the drug of choice for treating infections
l Lymphadenopathy is typically absent. caused by Actinomycetes. In cases of penicillin allergy,
l Trismus is present, if the muscles of mastication are doxycyclin is used.
involved. l Sulfonamides such as sulfamethoxazole may be used
l Fever is variably present and difficulty in mastication. as an alternative regimen at a total daily dosage of
2-4 grams. Response to therapy is slow and may take
B. Thoracic actinomycosis months.
l Patient gives history of aspiration (risk factors include l Surgical therapy may include incision and drainage of
seizure disorder, alcoholism, and poor dental hygiene.). abscesses, excision of sinus tracts and recalcitrant
l Dry or productive cough, occasionally blood-streaked fibrotic lesions, decompression of closed-space infec-
sputum, shortness of breath, chest pain. tions, and interventions aimed at relieving obstruction.
Section | II General Surgery 271

Q. 2. Discuss the aetiology, pathology, clinical features l Reflex spasms develop in most patients and can be
and management of tetanus. triggered by minimal external stimuli such as noise,
light, or touch. The spasms last seconds to minutes;
Ans.
become more intense; increase in frequency with
Tetanus is an emergency medical condition characterized disease progression; and can cause apnoea, fractures,
by prolonged contraction of skeletal muscle fibres. dislocations, and rhabdomyolysis.
l Laryngeal spasms can occur at any time and can result
Aetiology and Pathology in asphyxia.
l Other symptoms include elevated temperature,
l Tetanus is caused by Clostridium tetani, an obligate sweating, elevated blood pressure, and episodic rapid
anaerobic gram-positive bacillus. heart rate.
l This bacterium is non-encapsulated and forms spores, l Sustained contraction of facial musculature produces a
which are resistant to heat, desiccation, and disinfec- sneering grin expression known as risus sardonicus.
tants. The spores are ubiquitous and are found in soil, Severe tetanus results in opisthotonos, flexion of the
house dust, animal intestines, and human faeces. arms, extension of the legs, periods of apnoea resulting
l Spores that gain entry can persist in normal tissue for from spasm of the intercostal muscles and diaphragm,
months to years. Under anaerobic conditions, these spores and rigidity of the abdominal wall.
geminate and elaborate tetanospasmin and tetanolysin. l Late in the disease, autonomic dysfunction develops,
l Tetanospasmin is a neurotoxin and causes the clinical with hypertension and tachycardia alternating with
manifestations of tetanus. It is released by the maturing hypotension and bradycardia.
bacilli and distributed via the lymphatic and vascular l Patients may have reflex spasms of the masseter muscles
circulations to the end plates of all nerves. when the posterior pharyngeal wall is stimulated that
l Tetanospasmin enters the nervous system peripherally cause them to bite down as opposed to gag (spatula test).
at the myoneural junction and is transported centripe-
tally into neurons of the central nervous system (CNS).
The neurons become incapable of neurotransmitter re- Types of Tetanus
lease, leading to failure of inhibition of motor reflex a) Generalized tetanus
responses to sensory stimulation. This results in gener-
alized contractions of the agonist and antagonist muscu- l It is the most common type of tetanus, representing
lature characteristic of a tetanic spasm. about 80% of cases.
l The shortest peripheral nerves are the first to deliver the l The generalized form usually presents with a descend-
toxin to the CNS, which leads to the early symptoms of ing pattern. The first sign is trimus or lockjaw, and the
facial distortion and back and neck stiffness. Once the facial spasms called risus sardonicus followed by stiff-
toxin becomes fixed to neurons, it cannot be neutralized ness of the neck, difficulty in swallowing, and rigidity
with antitoxin. of pectoral and calf muscles.
l Recovery of nerve function from tetanus toxins requires l Spasms may occur frequently and last for several
sprouting of new nerve terminals and formation of new minutes with the body shaped into a characteristic form
synapses. called opisthotonous. Spasms continue for up to
4 weeks, and complete recovery may take months.

Clinical Features b) Neonatal tetanus


l The median incubation period is 7 days, and for most l It is a form of generalized tetanus that occurs in new-
cases, incubation ranges from 4-14 days. borns.
l Patients with generalized tetanus present with trismus l Infants who have not acquired passive immunity be-
(i.e. lockjaw) in 75% of cases. cause the mother has never been immunized are at risk.
l Other presenting complaints include stiffness, neck l It usually occurs through infection of the unhealed um-
rigidity, dysphagia, restlessness, and reflex spasms. bilical stump, particularly when the stump is cut with a
l Subsequently, muscle rigidity becomes the major non-sterile instrument.
manifestation. Muscle rigidity spreads in a descending
pattern from the jaw and facial muscles over the next c) Local tetanus
24-48 hours to the extensor muscles of the limbs. l It is an uncommon form of the disease, in which
l Dysphagia occurs in moderately severe tetanus due patients have persistent contraction of muscles in the
to pharyngeal muscle spasms, and onset is usually same anatomic area as the injury. The contractions may
insidious over several days. persist for many weeks before gradually subsiding.
272 Quick Review Series for BDS 3rd Year

l Local tetanus is generally milder, but it may precede the Mild tetanus
onset of generalized tetanus. Mild cases of tetanus can be treated with:
l Localized tetanus is characterized by painful spasms of l Tetanus immunoglobulin IV or IM
the group of muscles in close proximity to the site of l Metronidazole IV for 10 days
injury. This disorder may persist for several weeks but l Diazepam
is usually self-limiting.
Severe tetanus
d) Cephalic tetanus
Severe cases will require admission to intensive care. In
l It is a rare form of the disease, occasionally occurring addition to the measures listed above for mild tetanus:
with otitis media (ear infections) in which C. tetani l Human tetanus immunoglobulin injection intrathecally.
is present in the flora of the middle ear, or following l Attempting endotracheal intubation may induce severe
injuries to the head. reflex laryngospasm; prepare for emergency surgical
l Cephalic tetanus is characterized by variable cranial airway control. Rapid sequence intubation techniques
nerve (CN) palsies; CN VII is most frequently involved. (e.g. with succinylcholine) are recommended to avoid this
complication. Tracheostomy in patients requiring intuba-
Management tion for more than 10 days. Tracheostomy has also been
recommended after onset of the first generalized seizure.
Lab studies l Magnesium, as an intravenous (IV) infusion, to prevent
No specific laboratory tests exist for determining the diagnosis muscle spasm.
of tetanus. The diagnosis is clinically based on the presence of l Diazepam as a continuous IV infusion or other muscle
trismus, dysphagia, generalized muscular rigidity, and/or spasm. relaxants can be given to control the muscle spasms.
l In extreme cases, it may be necessary to paralyze the patient
The spatula test with curare-like drugs and use a mechanical ventilator.
l In order to survive a tetanus infection, the maintenance
This is a diagnostic bedside test and test involves touching
of an airway and proper nutrition are required. An in-
the oropharynx with a spatula or tongue blade. This typi-
take of 3500-4000 calories, and at least 150 g of protein
cally elicits a gag reflex, and the patient tries to expel the
per day, is often given in liquid form through a tube di-
spatula (i.e. a negative test result). If tetanus is present, pa-
rectly into the stomach (percutaneous endoscopic gas-
tients develop a reflex spasm of the masseter and bite the
trostomy), or through a drip into a vein (total parenteral
spatula (i.e. a positive test result).
nutrition). This high-caloric diet maintenance is
required because of the increased metabolic strain
Treatment brought on by the increased muscle activity.
l Full recovery takes 4 to 6 weeks because the body must
l Sanitation of wound. Surgical debridement of dead and
regenerate destroyed nerve axon terminals.
infected tissues.
l Administration of the antibiotic metronidazole de- Q. 3. What are the types of tetanus and its causes? Add
creases the number of bacteria but has no effect on the a note on prevention of tetanus.
bacterial toxin.
l Passive immunization with human anti-tetanospasmin Ans.
immunoglobulin or tetanus immunoglobulin is crucial.
l If specific anti-tetanospasmin immunoglobulin is not Types of Tetanus
available, then normal human immunoglobulin may be
given instead. Tetanus immune globulin (TIG) is recom- a. Generalized tetanus
mended for treatment of tetanus. TIG can only help re- l It is the most common type of tetanus, representing
move unbound tetanus toxin, but it cannot affect toxin about 80% of cases.
bound to nerve endings. A single intramuscular dose of l The generalized form usually presents with a descend-
3000-5000 units is generally recommended for children ing pattern. The first sign is trimus or lockjaw, and the
and adults, with part of the dose infiltrated around the facial spasms called risus sardonicus followed by stiff-
wound, if it can be identified. ness of the neck, difficulty in swallowing, and rigidity
l The World Health Organization recommends TIG of pectoral and calf muscles.
500 units by IM or IV as soon as possible; in addition, l Spasms may occur frequently and last for several
administer age-appropriate TT-containing vaccine (Td, minutes with the body shaped into a characteristic
Tdap, DT, DPT, DTaP, or TT depending on age or aller- form called opisthotonous. Spasms continue for up to
gies), 0.5 cc by intramuscular injection at separate site. 4 weeks, and complete recovery may take months.
Section | II General Surgery 273

b. Neonatal tetanus Q. 4. Define carbuncle. Mention its aetiology, clinical


l It is a form of generalized tetanus that occurs in features and treatment.
newborns. Ans.
l Infants who have not acquired passive immunity be-
cause the mother has never been immunized are at risk. l Carbuncle is defined as a skin infection involving a
l It usually occurs through infection of the unhealed um- group of hair follicles, i.e. it is a confluence of several
bilical stump, particularly when the stump is cut with a furuncles (skin boils).
non-sterile instrument. l It is an abscess larger than a boil, usually with one or
more openings draining pus on to the skin.
c. Local tetanus l It is usually caused by bacterial infection, most
l It is an uncommon form of the disease, in which pa- commonly Staphylococcus aureus. The infection is
tients have persistent contraction of muscles in the same contagious and may spread to other areas of the body
anatomic area as the injury. The contractions may per- or other people.
sist for many weeks before gradually subsiding.
l Local tetanus is generally milder, but it may precede the Aetiology
onset of generalized tetanus.
l Localized tetanus is characterized by painful spasms of l Carbuncle is caused by bacterial infection, most
the group of muscles in close proximity to the site of commonly by Staphylococcus aureus. Because the
injury. This disorder may persist for several weeks but condition is contagious, some household members may
is usually self-limiting. get carbuncles at the same time.
l Generally, the direct cause of a carbuncle infection
d. Cephalic tetanus cannot be determined. Poor personal hygiene, friction
l It is a rare form of the disease, occasionally occurring from clothing or shaving and poor overall health
with otitis media (ear infections) in which C. tetani (troubles with immune system) are things that make
is present in the flora of the middle ear, or following carbuncle infections more likely.
injuries to the head. l Individuals with dermatitis, diabetes and weakened
l Cephalic tetanus is characterized by variable cranial immune systems are more likely to develop staphylo-
nerve (CN) palsies; CN VII is most frequently involved. coccal infections.

Prevention of Tetanus Clinical Features


l Unlike many infectious diseases, recovery from natu- l A carbuncle is made up of several skin boils also known
rally acquired tetanus does not usually result in immu- as furuncles.
nity to tetanus. This is due to the extreme potency of the l The infected mass is filled with pus, fluid, and dead tissue.
tetanospasmin toxin; even a lethal dose of tetanospas- l Fluid may drain out of the carbuncle, but it may occur
min is insufficient to provoke an immune response. that the mass is so deep that it cannot drain on its own.
l Tetanus can be prevented by vaccination with tetanus It may break at any place, but they are most common on
toxoid. Adults are recommended a booster vaccine the back and the nape of the neck.
every 10 years. l Men get carbuncles more often than women.
l It is a standard care practice in many places to give l The carbuncle may be the size of a pea or as large as a
booster to any patient with a puncture wound who is golf ball.
uncertain of when he or she was last vaccinated, or if he l It may be red and irritated, and might hurt when
or she has had fewer than three lifetime doses of the touched. It may also grow very fast and have a white or
vaccine. yellow centre. It may crust or spread to other skin areas.
l The booster may not prevent a potentially fatal case of l Sometimes, other symptoms may occur such as fatigue,
tetanus from the current wound, as it can take up to two fever and a general discomfort or sick feeling. Itching
weeks for tetanus antibodies to form. may occur before the carbuncle develops.
l In children under the age of seven, the tetanus vaccine
is often administered as a combined vaccine, DPT/DTaP
Treatment
vaccine, which also includes vaccines against diphtheria
and pertussis. l Carbuncles usually drain before they heal. This usually
l For adults and children over seven, the Td vaccine (teta- occurs on its own in less than two weeks.
nus and diphtheria) or Tdap (tetanus, diphtheria, and l Placement of a warm moist cloth on the carbuncle
acellular pertussis) is commonly used. and soaking the affected area several times each day
274 Quick Review Series for BDS 3rd Year

helps in its draining and healing. Squeezing the causes cell destruction by hydrolysis of key cell
carbuncle or cutting it open without medical supervision membrane components. This toxin can cause lysis
is contraindicated. of erythrocytes, leucocytes, platelets, fibroblasts, and
l In case the carbuncle is more than two weeks old, muscle cells.
located on spine or typical position on face, recurring l The typical incubation period for gas gangrene is
frequently, with fever, treatment is needed. Treatment frequently short (i.e. ,24 h), but incubation periods
helps to reduce complications related to an infection. of 1 hour to 6 weeks have been reported.
l Antibacterial soaps and antibiotics are recommended. l Self-perpetuating destruction of tissue occurs via a
Deep or large lesions need to be drained. Excision, by rapidly multiplying microbial population and the
cruciate incision, under strict aseptic conditions will production of locally and systemically acting exotoxins.
treat the condition effectively. l Local effects include necrosis of muscle and subcutane-
l Proper hygiene is very important to prevent the spread of ous fat and thrombosis of blood vessels. Marked oedema
infection. Hands should always be washed thoroughly, may further compromise blood supply to the region.
preferably with antibacterial soap, after touching a l Fermentation of glucose is probably the main mecha-
carbuncle. nism of gas production in gas gangrene. Systemically,
l Wash cloths and towels should not be shared or reused. exotoxins may cause severe haemolysis.
Clothing, washcloths, towels, and sheets or other items l Haemoglobin levels may drop to very low levels and,
that contact infected areas should be washed in very hot when occurring with hypotension, may cause acute
(preferably boiling) water. tubular necrosis and renal failure. A rapidly progressive
l Bandages should be changed frequently and thrown infection can quickly result in shock. The mechanism of
away in a tightly-closed bag. If boils/carbuncles recur shock is poorly understood.
frequently, daily use of an antibacterial soap or cleanser
can suppress staph bacteria on the skin.
Clinical Features
Q. 5. Discuss the pathogenesis, clinical features and the
management of gas gangrene. l Mostly patients with post-traumatic gas gangrene have
sustained grave injury to the skin or soft tissues or have
Ans. open fractures.
l Patients with postoperative gas gangrene have a history
l Gas gangrene or clostridial myonecrosis is a bacterial of recent surgery of the GI or biliary tract. In patients
infection of muscle tissue by toxin producing clostridia. with occult malignancy-associated spontaneous gas
l It is a deadly form of gangrene usually caused by gangrene, history is unremarkable.
Clostridium perfringens bacterium and is considered as l The clinical features can manifest as:
a medical emergency. i. Sudden onset of pain which gradually worsens but
spreads only as the underlying infection spreads.
Pathogenesis ii. A low-grade fever and apathetic mental status.
iii. Local swelling and a serosanguineous exudate
l Gas gangrene is caused by an anaerobic, gram-positive, appear soon after the onset of pain.
spore-forming bacillus of the genus Clostridium. iv. The skin characteristically turns to a bronze colour,
l C. perfringens is the most common aetiologic agent that then progresses to a blue-black colour with skin
causes gas gangrene. Other common clostridial species blebs and haemorrhagic bullae.
that cause gas gangrene include Clostridium bifermen- v. Within hours, the entire region may become mark-
tans, Clostridium septicum, Clostridium sporogenes, edly oedematous.The wound may be nonodorous
Clostridium novyi, Clostridium fallax, Clostridium or may have a sweet mousy odour.
histolyticum, and Clostridium tertium. These organisms vi. Crepitus follows gas production; at times, crepitus
are true saprophytes. may not be detected with palpation owing to
l C. perfringens produces at least 20 exotoxins. The most brawny oedema.
important exotoxins and their biologic effects are as vii. Pain and tenderness to palpation disproportionate
follows: Alpha toxin, beta toxin, epsilon toxin, iota to wound appearance are common findings.
toxin, delta toxin, phi toxin, kappa toxin, nu toxin. The viii. Late signs of gas gangrene include hypotension,
precise role of these exotoxins in the pathogenesis of renal failure, and a paradoxical heightening of
gas gangrene is not entirely clear; however, alpha-toxin mental acuity.
is apparently of utmost importance.
l The alpha-toxin is a 370-residue zinc metalloenzyme In summary, the typical signs and symptoms of gas
that has phospholipase-C activity (i.e. lecithinase) and gangrene include severe pain and tenderness, local swelling
Section | II General Surgery 275

to massive oedema, skin discolouration with haemorrhagic the external environment as a result of enamel loss or
blebs and bullae, nonodorous or sweet odour, crepitus, fe- gingival recession.
ver, relative tachycardia, and altered mental status. l Fever
l Gingival bleeding (on occasion with periodontal
abscess)
Management l Decreased intake of fluid, food, or both.
l The combination of aggressive surgical debridement
and effective antibiotic therapy is an important criteria Physical Signs
for successful treatment of gas gangrene.
l Historically, penicillin G in dosages of 10-24 million i. Gingiva
U/d was the drug of choice. Currently, a combination of l Swelling
penicillin and clindamycin is widely used. A combina- l Warmth
tion of clindamycin and metronidazole is a good choice l Erythema
for patients allergic to penicillin. l Fluctuant mass that usually extends toward the buccal
l Other treatment options have included hyperbaric side of the gum and to the gingival-buccal reflection.
oxygen. In view of the frequent catastrophic outcomes l Parulis or “gum boil” (a soft, solitary, reddish papule
in patients with gas gangrene, HBO therapy is an impor- located facial and apical to a chronically abscessed
tant adjunct to surgery and antimicrobial therapy, tooth that occurs at the endpoint of a draining dental
despite the lack of convincing clinical efficacy. sinus tract).
l Surgical care includes fasciotomy for compartment
syndrome, daily debridement as needed to remove all ii. Teeth
necrotic tissue, amputation of the extremity. The tooth that is most frequently involved is the lower third
molar, followed by other lower posterior teeth; upper
Q. 6. Describe the signs, symptoms and treatment of
posterior teeth are involved much less frequently, and
alveolar abscess.
anterior teeth are rarely involved.
Ans. l Increased mobility (mostly periapical abscess)
l Pressure or percussion tenderness (mostly periapical
Alveolar abscess or dentoalveolar abscess is an abscess in
abscess)
the alveolar ridge of the jaw, usually caused by the spread
l Extrusion
of infection from an adjacent nonvital tooth.
In other words, it is a localized suppurative inflamma- iii. Regional lymph node involvement
tion of tissues about the apex of the root of a tooth.
iv. More severe infection results in
Pathology l Trismus, indicating involvement of the masticator space
l Difficulty in swallowing (dysphagia)
The term dentoalveolar abscess comprises 3 distinct l Respiratory difficulty
processes, as follows: l Necrotizing fasciitis
l A periapical abscess originating in the dental pulp and l Neck or facial swelling
secondary to dental caries is the most common dental
abscess in children. Dental caries leads to pulpitis which
in turn progresses to necrosis, causing an abscess. Treatment
l A periodontal abscess involves the supporting structures Assessment of the airway upon respiratory distress, oropha-
of the teeth (periodontal ligaments, alveolar bone). This ryngeal tissue swelling, or inability to handle secretions;
is the most common dental abscess in adults. secure the airway via endotracheal intubation or tracheostomy.
l Pericoronitis describes the infection of the gum flap l Collection of specimen for Gram stain and aerobic and
(operculum) that overlies a partially erupted or anaerobic cultures.
impacted third molar. l Empiric antibiotic therapy
l Analgesics.
Hydrate the patient.
Clinical Features l

l A pulpectomy or incision and drainage is the recom-


l Localized pain and swelling (may progress over a few mended management of a localized acute apical abscess
hours to days). in the permanent dentition.
l Thermal sensitivity (periapical abscess): This is thought l Removal of involved tooth is a common surgical
to occur secondary to exposure of the dentin to procedure.
276 Quick Review Series for BDS 3rd Year

SHORT ESSAYS
Q. 1. Actinomycosis. Treatment
Ans. l Penicillin G is the drug of choice, in cases of penicillin
allergy, doxycyclin is used. Sulfonamides, such as
Actinomycosis is a subacute-to-chronic bacterial infection
sulfamethoxazole, may be used as an alternative regi-
caused by filamentous, gram-positive, non-acid-fast,
men at a total daily dosage of 2-4 grams. Response to
anaerobic-to-microaerophilic bacteria.
therapy is slow and may take months.
l Surgical therapy may include incision and drainage of
Clinical Symptoms abscesses, excision of sinus tracts and recalcitrant
fibrotic lesions, decompression of closed-space infec-
a. Cervicofacial actinomycosis tions, and interventions aimed at relieving obstruction.
l Aetiology: History of trauma to the mouth, poor oral Q. 2. Ludwig’s angina.
hygiene, dental caries, or periodontal disease; local
tissue damage caused by neoplasm or osteonecrosis of Ans.
the jaws due to radiation treatment. l Ludwig’s angina, is a potentially life-threatening cellu-
l Painless or occasionally painful soft-tissue swelling litis of submental and submandibular region combined
involving the submandibular or perimandibular region with inflammatory oedema of mouth, usually occurring
with increase in size and number which ultimately in adults with concomitant dental infections.
form sinuses that open on to the cheek or submandibular l The virulent streptococcal organisms are responsible for
area. The excudate usually contain sulphur granules. infection surrounding submandibular region.
l Reddish or bluish discolouration of the skin overlying
the lesion. Clinical Features
l Nodules may be tender in the initial stages, are typically l Elderly patients who present with diffuse swelling in
nontender and woody hard in the later stages. submandibular region.
l No lymphadenopathy, fever is variably present. l Oedema of floor of the mouth, as a result of which tongue
l Trismus is present, if the muscles of mastication are is pushed upward results in difficulty in swallowing.
involved. l Putrid halitosis.
l High-grade fever with toxicity.
b. Thoracic actinomycosis l Wooden like induration at first is confined to subman-
l History of aspiration. dibular compartment, trismus, hoarseness and dyspnoea.
l Dry or productive cough, occasionally blood-streaked l It may spread to neck - oedema glottis.
sputum, shortness of breath, chest pain.
l Sinus tracts with drainage from the chest wall (i.e. Treatment
pleurocutaneous fistula)
l Rest and hospitalization
l Fever, weight loss, fatigue, anorexia.
l Appropriate antibiotics
l Intravenous fluid to correct dehydration and Ryle’s tube
c. Abdominal actinomycosis feeding.
l History of abdominal surgery, perforated viscus, l If it does not respond to conservative treatment, surgical
mesenteric vascular insufficiency, or ingestion of intervention is recommended.
foreign bodies. l Surgical management includes general anaesthesia,
l Sinus tracts with drainage from either the abdominal 5-6 cm curved incision is made below the mandible in the
wall (i.e. peritoneocutaneous fistula) or the perianal submandibular region over the most prominent part of the
region. swelling, then pus is drained out if pus is not present oe-
l Nonspecific symptoms like:Low-grade fever, weight dematous fluid comes out and give relief, then wound is
loss, fatigue, change in bowel habits, vague, abdominal closed with loose suture, after irrigating the cavity, the
discomfort, nausea, vomiting, sensation of a mass. antiseptic agents are given.
Q. 3. Trismus.
d. Pelvic actinomycosis
Ans.
l History of IUCD
l Lower abdominal discomfort, abnormal vaginal bleeding Trismus is defined as the inability to normally open the
or discharge mouth due to one of many causes.
Section | II General Surgery 277

Aetiology l More severe infections can result in vesicles, bullae and


petechiae with possible skin necrosis.
l Pericoronitis, i.e. the inflammation of soft tissue around
l Lymphatic involvement often is manifested by overly-
impacted third molar represents the most common
ing skin streaking and regional lymphadenopathy.
cause of trismus.
l The infection may occur on any part of the skin includ-
l Molar inflammation of masseter muscles as a sequel to
ing the face, arms, fingers, legs and toes, but it tends to
removal of mandibular third molars.
favour the extremities. Fat tissue is most susceptible to
l Peritonsillar abscess
infection, and facial areas typically around the eyes,
l Temporomandibular joint disorder (TMD)
ears, and cheeks.
l Submucous fibrosis.

Treatment Treatment
l Treatment requires treating the underlying condition l Elevation and rest of the affected limb are recom-
with dental treatments, physical therapy, and passive mended in erysipelas treatment to reduce local swelling,
range of motion devices. inflammation, and pain.
l Additionally, control of symptoms with pain medications l Saline wet dressings should be applied to ulcerated and
(NSAIDs), muscle relaxants, and warm compresses may necrotic lesions and changed every 2-12 hours, depend-
be used. Splints have been used. ing on the severity of the infection.
l Penicillin has remained first-line therapy. Penicillin
Q. 4. Erysipelas. administered orally or intramuscularly is sufficient
Ans. for most cases of classic erysipelas and should be given
for 10-20 days.
l Erysipelas is an acute superficial Streptococcus bacte-
rial infection of the deep epidermis with characteristic Q. 5. Boils.
cutaneous lymphatic spread.
Ans.
Aetiology and Pathology l A boil or furuncle is defined as deep folliculitis, infec-
tion of the hair follicle. It is mostly caused by infection
l Most cases of erysipelas are due to Streptococcus
by Staphylococcus aureus.
pyogenes also known as beta-hemolytic group A
l This results in a painful swollen area on the skin caused
streptococci, although non-group A streptococci can
by an accumulation of pus and dead tissue. Clusters of
also be the causative agent.
boils or furuncles are known as carbuncles.
l Bacterial inoculation into an area of skin trauma is the
initial event in developing erysipelas.
l The local factors, such as venous insufficiency, stasis Clinical Features
ulcerations, inflammatory dermatoses, dermatophyte
l Boils are bumpy red, pus-filled lumps around a hair
infections, insect bites, and surgical incisions, have been
follicle that are tender, warm, and very painful.
implicated as portals of entry.
l Their size ranges from pea-sized to golf ball-sized.
l In erysipelas, the infection rapidly invades and spreads
l A yellow or white point at the centre of the lump can be
through the lymphatic vessels. This can produce overlying
seen when the boil is ready to drain or discharge pus. In
skin “streaking” and regional lymph node swelling and
a severe infection, an individual may experience fever,
tenderness.
swollen lymph nodes, and fatigue.
l Systemic factors that lower resistance commonly are
Clinical Features detectable, including: diabetes, obesity, and haemato-
l Patients typically develop symptoms including high logic disorders.
fevers, shaking, chills, fatigue, headaches, vomiting, and
general illness within 48 hours of the initial infection.
l Erysipelas begins as a small erythematous patch that
Treatment
progresses to a fiery-red, indurated, tense, and shiny l In some instances, draining can be encouraged by
plaque. The erythematous skin lesion enlarges rapidly application of a cloth soaked in warm salt water.
and has a sharply demarcated raised edge. It appears as l Washing and covering the furuncle with antibiotic
a red, swollen, warm, hardened and painful rash, similar cream or antiseptic tea tree oil and a bandage also
in consistency to an orange peel. promotes healing.
278 Quick Review Series for BDS 3rd Year

l Furuncles should never be squeezed or lanced without l It is transmissible by sexual contact with infectious le-
the oversight of a medical practitioner because it may sions, from mother to fetus in utero, via blood products
spread the infection. transfusion, and occasionally through breaks in the skin
l Antibiotic therapy is advisable for large or recurrent that come into contact with infectious lesions.
boils or those that occur in sensitive areas. l If untreated, it progresses through 4 stages: primary,
secondary, latent, and tertiary.
Q. 6. Carbuncle.
Ans. Clinical Features
l Carbuncle is caused by extensive infectious gangrene of Primary syphilis
the adjacent hair follicle and subcutaneous tissue by
l It occurs within 3 weeks of contact with an infected
Staphylococcus aureus.
individual.
l It commonly occurs in diabetic patients.
l It manifests mainly on the glans penis in males and on
l Nape of neck is the commonest site followed by back
the vulva or cervix in females.
and shoulder region. Skin of the site is coarse and has
l Lesions (chancres) are usually solitary, raised, firm, red
poor vascularity.
papules that can be several centimetres in diameter.
l The chancre erodes to create an ulcerative crater within
Pathology the papule, with slightly elevated edges around the
central ulcer. It usually heals within 4-8 weeks, with or
l Since many of the patients are diabetic, there is necrosis
without therapy.
of subcutaneous fats, which gives rise to multiple
abscess, which intercommunicates as open to exterior Secondary syphilis
by multiple openings. This appearance is called as
l It manifests in various ways. It usually presents with a
cribriform appearance.
cutaneous eruption within 2-10 weeks after the primary
chancre and is most florid 3-4 months after infection.
Clinical Features l The eruption may be subtle. A localized or diffuse mu-
cocutaneous rash generally nonpruritic and bilaterally
l Surface is hyperaemic and looks like a red-hot coal.
symmetrical with generalized non-tender lymphade-
l Symptoms like fever with chills and region are severe.
nopathy is typical.
l Sever pain and swelling in nape of neck. Surrounding
l There may be a symmetrical reddish-pink non-itchy
area is indurated.
rash on the trunk and extremities including the palms
l Skin on centre of carbuncle softens and peripheral satel-
and soles.
lite vesicle appears, if rupture discharging pus and gives
l The rash may become maculopapular or pustule. On
rise to cribriform appearance.
mucous membranes, it may form flat, broad, whitish,
l Later development of large crateriform ulcer with centre
wart-like lesions known as condyloma latum.
slough.
Latent syphillis
Management l Latency may last from a few years to as many as
25 years before the destructive lesions of tertiary
l Diabetic control properly with injectable insulin.
syphilis manifest.
l Appropriate parenteral antibiotics are given till com-
l They are asymptomatic during the latent phase, and the
plete resolution occurs, e.g. cloxacillin, erythromycin,
disease is detected only by serologic tests.
or cephalosporins.
l Surgical treatment is required when there is pus. Tertiary syphilis
Cruciate incision is recommended because of multiple
l It is slowly progressive and may affect any organ. The
abscess and extensive subcutaneous necrosis. Edges
disease is generally not thought to be infectious at this
of skin flap are excised, pus is drained, loculi are
stage.
broken, slough is excised and cavity is treated with
l The lesions of benign tertiary syphilis usually develop
antiseptic.
within 3-10 years of infection.
Q. 7. Syphilis. l The typical lesion is a gumma, and patient complaints
usually are secondary to bone pain, which is described
Ans.
as a deep boring pain characteristically worse at night.
l Syphilis is an infectious venereal disease caused by the l CNS involvement may occur, with presenting symp-
spirochaete Treponema pallidum. toms representative of the area affected.
Section | II General Surgery 279

Diagnosis Q. 9. Precautions for surgeon—HIV-infected patient.


l Syphilis may be confirmed either via blood tests or Ans.
direct visualization using microscopy.
The HIV is the virus which results in the causation of the
l Typical diagnosis is with blood tests using nontreponemal
AIDS.
and/or treponemal tests.
Prophylactic measures to be adopted by dental surgeon
l Nontreponemal tests are used initially and include
while treating AIDS patient.
venereal disease research laboratory (VDRL) and rapid
plasma regain.
l Positive confirmation is required with a treponemal test In OPD
such as Treponema pallidum particle agglutination
l Any patient with open wound, gloves are worn when
(TPHA) or fluorescent treponemal antibody absorption
examining a patient.
test (FTA-Abs).
l Use disposable instruments.
l Reusable instruments are cleaned in soap and water and
Treatment immersed in gluteraldehyde.
Penicillin remains the mainstay of treatment and the stan-
dard by which other modes of therapy are judged. The fol- In Operating Room
lowing regimens are recommended for penicillin treatment:
l Primary or secondary syphilis: Benzathine penicillin G l Dental chair is covered with a single sheet of polythene.
2.4 million units intramuscularly (IM) in a single dose. l The number of personnel in dental operating room
l Early latent syphilis: Benzathine penicillin G 2.4 million should be reduced to minimum.
units IM in a single dose. l The staff with abrasion or lacerations on their hands are
l Late latent syphilis or latent syphilis of unknown dura- not allowed inside the operating room.
tion: Benzathine penicillin G 7.2 million units total, l Staff who enter the theatre wear over—shoes, gloves and
administered as 3 doses of 2.4 million units IM each at disposable water resistant gowns and eye protectors.
1-week intervals. l During surgical technique:
l Avoid sharp injury.
l Pregnancy: Treatment appropriate to the stage of syphi-
l Avoid “needle stick” injuries
lis is recommended.
l Proper autoclaving at the end of surgery.
Q. 8. Dangerous area of the face. l AZT—zidovudine, lamivudine and indinavir should be
Ans. given for the health workers following exposure of
susceptible area to infected material from AIDS patient.
l The dangerous triangle of the face or danger area of the
face consists of the area from the corners of the mouth Q. 10. Active immunity.
to the bridge of the nose, including the nose and
Ans.
maxilla.
l Due to the special nature of the blood supply to the hu- l Active immunity is the resistance developed by an indi-
man nose and surrounding area, it is possible for retro- vidual as a result of an antigenic stimulus.
grade infections from the nasal area to spread to the l When B and T cells are activated by a pathogen, mem-
brain. ory B and T cells develop.
l This is possible because of venous communication (via l Throughout the lifetime of the individual, these memory
the ophthalmic veins) between the facial vein and the cells will “remember” each specific pathogen encoun-
cavernous sinus. tered, and are able to mount a strong response, if the
l The cavernous sinus lies within the cranial cavity, be- pathogen is detected again.
tween layers of the meninges and is a major conduit of l This type of immunity is both active and adaptive
venous drainage from the brain. because the body’s immune system prepares itself for
l It is a common misconception that the veins of the head future challenges.
do not contain one way valves like other veins of l Active immunity often involves both the cell-mediated
the circulatory system. In fact, it is not the absence of and humoral aspects of immunity as well as input from
venous valves but the existence of communications the innate immune system.
between the facial vein and cavernous sinus and the l The innate system is present from birth and protects an
direction of blood flow that is important in the spread of individual from pathogens regardless of experiences,
infection from the face. Most, but not all, people have whereas adaptive immunity arises only after an infection
valves in the veins of the face. or immunization and hence is “acquired” during life.
280 Quick Review Series for BDS 3rd Year

Naturally acquired active immunity divides the space into two triangles: An upper or occipital
Naturally acquired active immunity occurs when a person triangle and a lower or supraclavicular triangle.
is exposed to a live pathogen, and develops a primary im-
Contents of the posterior triangle are:
mune response, which leads to immunological memory.
A) Nerves and plexuses:
Artificially acquired active immunity l Spinal accessory nerve (cranial nerve XI)

l Branches of cervical plexus


Artificially acquired active immunity can be induced by a
l Roots and trunks of brachial plexus
vaccine, a substance that contains antigen. A vaccine stimu-
l Phrenic nerve (C3, 4, 5)
lates a primary response against the antigen without caus-
ing symptoms of the disease. B) Vessels:
l Subclavian artery (third part)
Q. 11. Septicaemia. l Transverse cervical artery

l Suprascapular artery
Ans.
l Terminal part of external jugular vein

Septicaemia is defined as the presence of pathogenic organ- C) Lymph nodes:


isms in the bloodstream, leading to sepsis. l Occipital

l Supraclavicular

D) Muscles:
Clinical Features l Inferior belly of omohyoid muscle

l Intermittent high grade pyrexia (fever) l Anterior scalene

l Rigors and chills. l Middle scalene

l Jaundice due to liver damage. l Posterior scalene

l Peripheral circulatory failure. Intravascular coagulation. l Levator scapulae

Patient may go into septic shock. l Splenius

l Septic shock is secondary to sepsis; it is characterized


Q. 13. Anaerobic infections.
by inadequate perfusion of tissue.
l The septic shock differs from all other forms of shock Ans.
by having hot stage before cold stage.
l Anaerobic infections are caused by anaerobic bacteria.
l The symptoms rapidly progress to shock with fever or
l Anaerobic bacteria can be divided into strict anaerobes
decreased body temperature (hypothermia), falling blood
that cannot grow in the presence of more than 0.5% oxy-
pressure, confusion or other changes in mental status,
gen and moderate anaerobeic bacteria that are able of
and blood clotting problems that lead to a specific type
growing between 2 and 8% oxygen. Anaerobic bacteria
of red spots on the skin (petechiae and ecchymosis).
usually do not possess catalase, but some can genrerate
l There may be decreased or no urine output.
superoxide dismutase which protects them from oxygen.
l The clinically important anaerobes in decreasing fre-
Treatment quency are:
i. Six genera of Gram-negative rods (Bacteroides,
l Management of primary focus of infection. Broad-
Prevotella, Porphyromonas, Fusobacterium, Biloph-
spectrum antibodies are given.
ila and Sutterella spp.);
l Blood and fluid transfusion to correct septic shock. Injec-
ii. Gram-positive cocci (primarily Peptostreptococcus
tion of hydrocortisone in case of septic shock may be useful.
spp.);
Q. 12. Boundaries of back of neck. iii. Gram-positive spore-forming (Clostridium spp.)
and nonspore-forming bacilli (Actinomyces, Propi-
Ans.
onibacterium, Eubacterium, Lactobacillus and Bifi-
The posterior triangle or lateral cervical region is a region dobacterium spp.);
of the neck. It has the following boundaries: iv. Gram-negative cocci (mainly Veillonella spp.)
l Apex: Union of the sternocleidomastoid and the trapezius
Anaerobic infections tend to have several features in common.
muscles at the superior nuchal line of the occipital bone
l The infection is usually accompanied by a foul-smelling
l Anterior: Posterior border of the sternocleidomastoideus
gas or pus.
l Posterior: Anterior border of the trapezius
l The infections tend to be located close to membranes,
l Base: Middle one-third of the clavicle
particularly mucosal membranes, as the infection
The posterior triangle is crossed, about 2.5 cm above the typically begins by the invasion of a region that is
clavicle, by the inferior belly of the omohyoideus which bounded by a membrane.
Section | II General Surgery 281

l It tends to involve the destruction of tissue, either be- l Temperature is usually raised due to increase
cause of bacterial digestion or because of destructive metabolic rate.
enzymes that are elaborated by the bacteria. This type of l Risus sardonicus due to stiffness of facial muscle

tissue damage is known as tissue necrosis. and jaw muscles.


l The tissue damage also frequently includes the production vi. Neonatal tetanus is characterized by difficulty in
of gas or a fluid. sucking and excessive crying in elderly stage. Death is
l There are several sites in the body that are prone to usually due to inadequate ventilation and asphyxia.
infection by anaerobic bacteria:
a. Infections in the abdomen can produce the inflam-
Treatment
mation of the appendix that is known as appendicitis.
b. Lung infections can result in pneumonia, infection I. General management
of the lining of the lung (empyema) or constriction II. Specific management.
of the small air tubes known as bronchi (bronchiec-
tasis). I. General management
c. In females, pelvic infections can inflame the lining l Administration in a quiet room, to avoid minor stimuli
of the uterus (endometritis). witch precipitate spasm.
d. Mouth infections can involve the root canals or gums l Wound care: Drainage of pus, excision of necrotic
(gingivitis). tissue, removal of foreign body and proper dressing.
e. Infections of the central nervous system can lead to l Injection tetanus toxoid 0.5 ml IM or anti-tetanus serum
brain and spinal cord infections. (ATS) 50, 000 unit IV or IM or anti-tetanus globulin.
f. Infection of the skin, via bites and other routes l Injection crystalline penicillin 10 lac unit every 6 hours
of entry, causes open sores on the skin and tissue for 7 to 10 days.
destruction. For example: l After recovery, full immunization with tetanus toxoid is
l Massive and potentially lethal tissue degradation, must.
which is known as necrotizing fascitis, caused by
group A b-haemolytic Streptococcus. II. Specific treatment
l Infection of the bloodstream (bacteraemia) can A. Mild cases—only tonic rigidity without spasm or
prelude the infection of the heart (endocarditis). dysphagia.
l These patients are managed by heavy sedation to
Q. 14. Clinical features and management of tetanus.
avoid spasm or convulsions.
Ans. l Benzodiazepines and morphine act centrally to

Tetanus is an emergency medical condition characterized minimize the effect of tetanospasmin.


by prolonged contraction of skeletal muscle fibres. l Chlorpromazine, phynoxy benzamine, phentolamine

also have been used.


B. Seriously ill cases:
Aetiology l They have dysphagia and reflex spasm.
It is caused by anaerobic, gram-positive Bacillus Clostridium l Nasogastric tube for feeding and to administer the drug.

tetani. l Tracheostomy, if breathing difficulties.

C. Dangerously ill cases:


l Patient with major cyanotic convulsion.
Clinical Features
l In addition to sedatives, these patients are paralysed
i. It is more common in young males during their with muscle relaxant (neuromuscular blocking
accident prone years. agents) and positive pressure ventilation is given.
ii. Incubation period—14 days.
iii. Local tetanus is characterized by muscle spasm near the Q. 15. Human immunodeficiency virus and importance
site entry of bacilli. of dental surgery.
iv. Cephalic form: When bacilli are introduced through the Ans.
wounds in head and neck, cranial nerve palsy occurs
with cranial nerve most commonly the 7th cranial nerve.
Aetiology (in Dental Clinic)
v. Generalized form:
l Pain and stiffness in jaw, neck muscles with muscle l Due to infected instruments.
rigidity and dysphagia. l Uses of infected needles.
l Sometimes, entire body may be affected with contrac- l Infected blood transformation during dental procedures.
tion of all somatic muscles leading to opisthotonos. l Contaminated gloves and dressing materials.
282 Quick Review Series for BDS 3rd Year

Epidemiology b) Western blot


l AIDS was first described in US. The disease has now l Like the ELISA procedure, the western blot is an
attained high proportions involving all continents. antibody detection test.
Africa constitutes 50% of all positive cases globally. l Unlike the ELISA method, the viral proteins are separated
l 1 in every 100 sexually active adult worldwide is first and immobilized. In subsequent steps, the binding of
infected with HIV. serum antibodies to specific HIV proteins is visualized.

c) Rapid HIV testing


Pathology This type of HIV testing makes it possible for the patients
It is largely related to depletion of CD41 T cells resulting in: to get pre-test and post-test counselling, their test results,
and any medical referrals they may need all in one visit and
in a very short amount of time.
l OraQuick/OraQuick Advanced Rapid HIV-1/2
Selective tropism and internalization Antibody Test: This type of HIV testing has been ap-
proved for use with venous blood, plasma, and oral
fluids for the detection of HIV-1 and HIV-2.
Uncoating and proviral DNA integration
l Reveal G2 HIV-1 Antibody Test: This type of rapid
HIV testing has been approved for use with plasma or
Budding and syncytia formation serum specimens. While the test only takes 3 minutes
to develop, the test is more complex than the OraQuick
because it requires centrifuged serum or plasma. The
Cytopathic effects test consists of a cartridge with a test area. Like the
OraQuick, any HIV present in the test specimen binds
with the protein impregnated in the test area, causing a
Effects on monocytes and macrophages
HIV infection of nervous system red dot to appear. If a red dot appears along with a red
line used as a control, the test is considered positive,
requiring a confirmatory test.
B cell dysfunction l Uni-Gold Recombigen HIV-1 Test: This HIV testing
has been approved for use with whole blood, plasma, or
serum from a venipuncture or finger stick.
l Multispot HIV-1/HIV-2 Rapid Test: This HIV testing
Precautions in Dental Clinic has been approved for use on frozen and fresh plasma,
l Avoid sharing needle. whole blood, or serum. Multispot consists of a test
l Instrument should be properly sterilized. cartridge and five reagents.
l Testing by ELISA, before blood transfusion. Q. 17. AIDS.
l Before surgery dentist should use asepsis measures of
surgery. Ans.
l Educate the patient about AIDS.
AIDS is the end stage of a progressive state of immunode-
Q. 16. Diagnostic and confirmatory tests in AIDS. ficiency.
Causative organism: Human immunodeficiency virus
Ans.
(HIV).
The tests for HIV are as follows:

a) The ELISA Mode of Transmission


l The standard screening test for HIV is a blood test l Sexual intercourse
known as the enzyme immunoassay (EIA) or ELISA l Mother to foetus
for short. l Through contaminated needles
l This HIV testing requires a small sample of blood l Through contaminated blood transfusion.
from the person being tested. Typically, the test requires
two visits; one to receive pretest counselling and have
Clinical Features
your blood drawn and the second to receive HIV testing
results, post-test counselling and medical referrals for l Prolonged diarrhoea
HIV care, if the results are positive. l Weightloss
Section | II General Surgery 283

l Night sweating l Western blot test is a widely used analytical technique


l Prolonged fever. used to detect specific proteins/antibody in the given
sample of tissue homogenate or extract.
It uses gel electrophoresis to separate native or dena-
Oral Manifestations of AIDS l

tured proteins by the length of the polypeptide (denatur-


l Candidiasis (oral thrush) ing conditions) or by the 3-D structure of the protein
l Angular cheilitis (native/ non-denaturing conditions).
l Kaposi’s sarcoma l The proteins are then transferred to a membrane where
l Hairy leukoplakia they are probed (detected) using antibodies specific to
l Herpes simplex and herpes zoster infection the target protein. Antibodies that do not attach are
l Ulcerations washed away, and enzyme-linked antibodies with the
l Delayed wound healing capability to attach to the person’s antibodies determine
to which HIV proteins the person has antibodies.
There are no universal criteria for interpreting the
Treatment l

western blot test: The number of viral bands that must


l Basically the disease is increasable but certain drugs are be present may vary. If no viral bands are detected, the
given to prevent the progression of disease. result is negative. If at least one viral band for each
l The antiviral agent “azithromidine” is the most effective of the GAG, POL, and ENV gene-product groups are
drug against HIV. present, the result is positive.
l Zidovudine may prolong the life of AIDS patients.
These drugs prevent the replication of virus by inhibit-
Medical Use of Western Blot Test
ing the enzyme revers transcriptase.
l The confirmatory HIV test employs a Western blot to
detect anti-HIV antibody in a human serum sample.
Prevention and Control l A Western blot is also used as the definitive test for
l Safe sexual contact (use of condom). Bovine spongiform encephalopathy (BSE, commonly
l Prevent sharing of needles among drug abusers. Separate referred to as ‘mad cow disease’).
and sterilized needle should be used for each patient. l Some forms of Lyme disease testing employ Western
l Blood transfusion should be done after HIV testing. blotting.
l Western blot can also be used as a confirmatory test for
Q. 18. Western blot test.
Hepatitis B infection.
Ans.

SHORT NOTES
Q. 1. Boils. Q. 3. Abscess.
Ans. Ans.
l A boil or furuncle is defined as deep folliculitis, infec- l Abscess is defined as a collection of pus, accumulated in a
tion of the hair follicle. cavity formed by the tissue in which the pus resides on the
l It is mostly caused by infection by Staphylococcus au- basis of an infectious process or other foreign materials.
reus. l It is regarded as a defensive reaction of the tissue to
l This results in a painful swollen area on the skin caused prevent the spread of infectious materials to other parts
by an accumulation of pus and dead tissue. of the body.
l Clusters of boils or furuncles are known as carbuncles. l Wound abscesses cannot be treated with antibiotics.
They require surgical intervention, debridement and
Q. 2. Sinus.
curettage.
Ans.
Q. 4. Carbuncle.
l The term sinus refers to a sack or cavity in any organ or
Ans.
tissue, or an abnormal cavity or passage caused by the
destruction of tissue. l Carbuncle is caused by extensive infectious gangrene of
l The term is also used for a chronically infected tract the adjacent hair follicle and subcutaneous tissue by
such as a passage between an abscess and the skin. Staphylococcus aureus.
284 Quick Review Series for BDS 3rd Year

l It commonly occurs in diabetic patients. Clinical Features


l Nape of neck is the commonest site followed by back
l Intermittent high grade pyrexia (fever), rigors and chills.
and shoulder region.
l Jaundice due to liver damage.
l Skin of the site is coarse and has poor vascularity.
l Peripheral circulatory failure. Intravascular coagulation.
Q. 5. Erysipelas. Patient may go into septic shock.
l There may be decreased or no urine output.
Ans.
Erysipelas is an acute superficial Streptococcus bacterial Treatment
infection of the deep epidermis with characteristic cutaneous
lymphatic spread. l Management of primary focus of infection. Broad-
spectrum antibodies are given.
l Blood and fluid transfusion to correct septic shock.
Clinical Features Injection of hydrocortisone in case of septic shock may
l Patients develop symptoms including high fevers, shak- be useful.
ing, chills, fatigue, headaches, vomiting, and general Q. 8. Ludwig’s angina.
illness within 48 hours of the initial infection.
l Erysipelas begins as a small erythematous patch that Ans.
progresses to a fiery-red, indurated, tense, and shiny l Ludwig’s angina (angina ludovici) is a potentially life-
plaque. threatening cellulitis of submental and submandibular
l The erythematous skin lesion enlarges rapidly and has a region combined with inflammatory oedema of mouth.
sharply demarcated raised edge. It appears as a red, l Usually occurring in adults with concomitant dental
swollen, warm, hardened and painful rash, similar in infections.
consistency to an orange peel. l The virulent streptococcal organisms are responsible for
l Lymphatic involvement often is manifested by overly- infection surrounding submandibular region.
ing skin streaking and regional lymphadenopathy.
l The infection may occur on any part of the skin includ- Q. 9. Marjolin’s ulcer.
ing the face, arms, fingers, legs and toes, but it tends to Ans.
favour the extremities.
l Fat tissue is most susceptible to infection, and facial l Marjolin’s ulcer refers to an aggressive ulcerating squa-
areas typically around the eyes, ears, and cheeks. mous cell carcinoma presenting in an area of previously
traumatized, chronically inflamed or scarred skin.
Q. 6. Cellulitis. l They are commonly present in the context of chronic
Ans. wounds including burn injuries, venous ulcers, and
ulcers from osteomyelitis and post-radiotherapy scars.
l Cellulitis is a diffuse inflammation of connective tissue l Histologically, the tumour is a well-differentiated
with severe inflammation of dermal and subcutaneous squamous cell carcinoma.
layers of the skin. l This carcinoma is aggressive in nature, spreads locally
l Cellulitis can be caused by normal skin flora or by ex- and is associated with a poor prognosis.
ogenous bacteria, and often occurs where the skin has l Wedge biopsy is the favoured method of diagnosis.
previously been broken: cracks in the skin, cuts, blis- l Treatment is usually surgical, with a wide excision of the
ters, burns, insect bites, surgical wounds, intravenous lesion; typically a 1 cm margin all around is required.
drug injection or sites of intravenous catheter insertion.
l Skin on the face or lower legs is most commonly af- Q. 10. Basal-cell carcinoma.
fected by this infection, though cellulitis can occur on Ans.
any part of the body.
l The mainstay of therapy remains treatment with appro- l Basal-cell carcinoma is the most common type of skin
priate antibiotics, and recovery periods last from 48 cancer. For simplicity, one can also divide basal-cell
hours to six months. carcinoma into 3 groups, based on location and
difficulty of therapy:
Q. 7. Septicaemia. l Superficial basal-cell carcinoma, or some might
consider to be equivalent to “in situ”. Very responsive to
Ans.
topical chemotherapy such as fluorouracil. It is the only
Septicaemia is defined as the presence of pathogenic type of basal-cell cancer that can be effectively treated
organisms in the bloodstream, leading to sepsis. with topical chemotherapy.
Section | II General Surgery 285

l Infiltrative basal-cell carcinoma, which often encom- l They occur in most of chronic wound cases. Venous
passes morpheaform and micronodular basal-cell ulcers develop mostly along the medial distal leg, and
cancer. More difficult to treat with conservative can be very painful.
treatment methods such as electrodessiccation and l Treatment of venous ulcers can be frustrating and
currettage, or with currettage alone. lengthy. Goals should be directed at keeping the ulcer
l Nodular basal-cell carcinoma, which essentially includes infection free, absorbing any excess discharge,
most of the remaining categories of basal-cell cancer. maintaining a moist wound environment, supplying
compression (typically in the range of 40 mmHg),
Q. 11. Actinomycosis.
promoting activity of the patient and the involved
Ans. extremity and managing the patient’s medical problems.
l Actinomycosis is a subacute-to-chronic bacterial infec- Q. 15. Tetanus.
tion caused by filamentous, gram-positive, non-acid-
fast, anaerobic-to-microaerophilic bacteria. Ans.
l Aetiology: History of trauma to the mouth, poor oral l Tetanus is an emergency medical condition character-
hygiene, dental caries, or periodontal disease; local tis- ized by prolonged contraction of skeletal muscle fibres.
sue damage caused by neoplasm or radiation treatment. l It is caused by anaerobic, gram-positive bacillus
l Painless or occasionally painful soft-tissue swelling in- Clostridium tetani.
volving the submandibular or perimandibular region l It is more common in young males during their accident
with increase in size and number which ultimately form prone years.
sinuses that open on to the cheek or submandibular area. l Incubation period 4-14 days.
The excudate usually contain sulphur granules. l Various types of tetanus are as follows:
l Penicillin G is the drug of choice, in cases of penicillin a. Local tetanus
allergy, doxycyclin is used. b. Cephalic form
l Surgical therapy may include incision and drainage of c. Generalized form
abscesses, excision of sinus tracts and recalcitrant d. Neonatal tetanus
fibrotic lesions, decompression of closed-space infec- l Sometimes, entire body may be affected with contrac-
tions, and interventions aimed at relieving obstruction. tion of all somatic muscles leading to opisthotonos.
Q. 12. Hutchinson’s teeth. l Risus sardonicus is due to involvement of facial and jaw
muscles.
Ans.
Q. 16. Tetanus prophylaxis.
l Hutchinson’s teeth which is also known as Hutchinson’s
incisor, Hutchinson’s sign or Hutchinson-Boeck teeth is Ans.
a sign of congenital syphilis.
Babies have teeth that are smaller and more widely
Prevention of Tetanus
l

spaced than normal with notches on their biting surfaces.


It is named after Sir Jonathan Hutchinson, an English l Unlike many infectious diseases, recovery from
surgeon and pathologist, who first described them. naturally acquired tetanus does not usually result in
l It is pathognomonic of congenital syphilis. immunity to tetanus.
l Tetanus can be prevented by vaccination with tetanus
Q. 13. Fistula.
toxoid. Adults are recommended a booster vaccine
Ans. every 10 years.
l It is a standard care practice in many places to give
l Fistula is defined as an abnormal connection between
booster to any patient with a puncture wound who is
two epithelium-lined organs or vessels that normally do
uncertain of when he or she was last vaccinated, or if he
not connect.
or she has had fewer than three lifetime doses of the
l In general, it is considered as a disease condition, but a
vaccine.
fistula may be surgically created for therapeutic reasons.
l In children under the age of seven, the tetanus vaccine
Q. 14. Venous ulcers. is often administered as a combined vaccine, DPT/DTaP
vaccine, which also includes vaccines against diphtheria
Ans.
and pertussis.
l Venous ulcers are wounds that are thought to occur l For adults and children over seven, the Td vaccine
due to improper functioning of venous valves, usually (tetanus and diphtheria) or Tdap (tetanus, diphtheria,
of the legs. and acellular pertussis) is commonly used.
286 Quick Review Series for BDS 3rd Year

l If patient is fully immunized in past but has not received l Syphilis is transmissible by sexual contact with
booster for over 5 years, then should receive 0.5 ml of infectious lesions, from mother to fetus in utero, via
tetanus toxoid IM. blood product transfusion, and occasionally through
l Patients who have not received full course of immuniza- breaks in the skin that come into contact with infectious
tion should not only receive tetanus toxoid but also lesions.
should receive passive immunization l If untreated, it progresses through 4 stages: Primary,
secondary, latent, and tertiary.
Q. 17. Varieties of gangrene.

Ans. Q. 20. Hiltons method of abscess drainage.

Three major types of gangrene are as follows: Ans.


a. Dry gangrene l Hiltons method of abscess drainage: In drainage of
b. Wet gangrene abscess in locations like axilla, groin where there exist
c. Gas gangrene chances of injury to vessels and nerves, the skin and
l Dry gangrene is a condition that results when one subcutaneous tissues are incised by knife.
or more arteries become obstructed, in this type l The deep fascia is not incised, but is pierced with a sinus
of gangrene, the tissue slowly dies because of forceps which is then opened to enlarge the opening in
inadequate or no blood supply. the deep fascia.
Treatment of this type of gangrene is aimed at l A sinus artery forceps is used to break the septa in
improving circulation to the affected area. between loculations to allow free flow of pus.
l Moist, or wet, gangrene is caused by certain bacterial

infection. It can develop following a severe burn, Q. 21. HIV.


frostbite or injury. Administration of antibiotics and
sometimes the surgical removal of the dead tissue is Ans.
the treatment for this type of gangrene. l Human immunodeficiency virus (HIV) is a lentivirus
l Gas gangrene is an infection caused by certain bacteria
which is a member of the retrovirus family and causes
of the genus Clostridium and is often treated with the acquired immunodeficiency syndrome (AIDS).
antitoxin for Clostridium. In several cases, amputation l It is a condition in humans in which progressive failure
may have to be used to keep the infection under control. of the immune system allows life-threatening opportu-
Severe cases have been treated by keeping the patient in nistic infections and cancers to thrive.
an oxygen-rich atmosphere, as in a hyperbaric chamber. l Infection with HIV occurs by the transfer of blood,
Q. 18. Cold abscess. semen, vaginal fluid, pre-ejaculate, or breast milk.
l The four major routes of transmission are unsafe sex,
Ans. contaminated needles, breast milk, and transmission
l Cold abscess is almost always a sequel of tubercular infec- from an infected mother to her baby at birth. HIV in-
tion anywhere in body commonly in lymph node and bone. fection in humans is considered pandemic by the
l Caseation of lymph node forms the cold abscess. (WHO).
l The common sites are at the neck and axilla. Sometimes
cold abscess is seen at loin, at the back or at the side of Q. 22. AIDS.
chest wall. Ans.
l As suggested by the name, cold abscess is cold and non-
reacting in nature. It does not produce hot and painful l AIDS is the end stage of a progressive state of immuno-
abscess as seen in pyogenic abscess. deficiency.
l No local rise in temperature, no tenderness and no redness. l Causative organism: Human immunodeficiency virus
l Soft, cystic and fluctuant swelling and is situated deep- (HIV).
to-deep fascia. l Mode of transmission: Sexual intercourse, mother to
l Treatment includes non-dependent aspiration by using a foetus, through contaminated needles and through
wide bore needle to avoid a persistent sinus. contaminated blood transfusion.
l Clinical features: Prolonged diarrhoea, weightloss,
Q. 19. Syphilis night sweating and prolonged fever.
l Oral manifestations of AIDS include candidiasis
Ans.
(oral thrush), angular cheilitis, Kaposi’s sarcoma, hairy
l Syphilis is an infectious venereal disease caused by the leukoplakia, herpes simplex and herpes zoster infection,
spirochaete Treponema pallidum. ulcerations and delayed wound healing.
Section | II General Surgery 287

Q. 23. Immunity. and chemokines, based on their presumed function, cell


of secretion, or target of action.
Ans.
Q. 25. Cell-mediated immunity.
l Immunity is a biological term used to describe the state
of sufficient biological defenses to avoid and protect from Ans.
infection, disease, or other unwanted biological invasion.
l Cell-mediated immunity can be defined as the immune
l Immunity involves both specific and non-specific
response that does not involve antibodies or complement
components.
but the activation of macrophages, natural killer (NK)
l The non-specific components act either as barriers or as
cells, antigen-specific cytotoxic T lymphocytes, as well as
eliminators of wide range of pathogens irrespective of
the release of various cytokines in response to an antigen.
antigenic specificity.
l Historically, the immune system is distinguised into two
l Other components of the immune system adapt
branches:
themselves to each new disease encountered and are
a. Humoral immunity
able to generate pathogen-specific immunity.
b. Cellular immunity
Q. 24. Cytokines. l In cellular immunity, the protective function of immuni-
zation was associated with cells. CD4 cells or helper
Ans.
T cells provide protection against different pathogens.
l Cytokines represent small cell-signaling protein mole- l Cellular immunity protects the body by activation of
cules secreted by the glial cells of the nervous system antigen-specific cytotoxic T lymphocytes that are able
and as well by numerous cells of the immune system. to induce apoptosis in body cells displaying epitopes of
l They represent a category of signaling molecules used foreign antigen on their surface, such as virus-infected
extensively in intercellular communication. cells, cells with intracellular bacteria, and cancer cells
l Cytokines can be classified as proteins, peptides, or displaying tumour antigens.
glycoproteins; the term “cytokine” encompasses a large l Activation of macrophages and natural killer cells
and diverse family of regulators produced throughout enabling them to destroy pathogens; and stimulation
the body by cells of diverse embryological origin. of cells to secrete a variety of cytokines that influence
l Each cytokine has a matching cell-surface receptor. Cy- the function of other cells involved in adaptive immune
tokines have been classified as lymphokines, interleukins, responses and innate immune responses.

Topic 6

Tumours, Cysts and Neck Swellings

LONG ESSAYS
Q. 1. Describe the various types of cysts of mandible and cysts are defined more by their location than by any histo-
detail the management of each type. logic characteristics.
i. Periapical/radicular cyst: Radicular cyst is the most
Ans.
common odontogenic cyst. The usual aetiology is an
Mandibular cyst can be distinguished into: infected teeth resulting in pulp necrosis and periapical
i. Odontogenic inflammation. Eventually, this epithelium undergoes
ii. Nonodontogenic. necrosis caused by a lack of blood supply, and the
granuloma becomes a cyst. The lesions are not usually
clinically detectable when small but most often are
A. Odontogenic Mandibular Cysts: discovered as incidental findings on radiographic sur-
Odontogenic cysts are defined as epithelial-lined structures vey. They resolve with endodontic therapy of the in-
derived from odontogenic epithelium. Most odontogenic volved tooth. Lesions that fail to resolve with such
288 Quick Review Series for BDS 3rd Year

therapy should be surgically removed and histopatho- vii. Gingival cysts of the adult are found only in soft
logically examined. tissue in the lower premolar areas. These cysts present
ii. Dentigerous cyst: It represents the second most com- as tense, fluctuant, vesicular, or bullous lesions. Histo-
mon odontogenic cyst. It develops within the normal logically, they look like lateral periodontal cysts, and
dental follicle that surrounds an unerupted tooth. The they probably represent the same lesion when found in
dentigerous cyst is not neoplastic. It most frequently is soft tissue.
found in: mandibular third molars, maxillary third viii. Odontogenic keratocyst (OKC) is the most impor-
molars, and maxillary canines, in decreasing order of tant of the odontogenic cysts. This cyst may have any
frequency. These cysts can grow very large and can move clinical appearance; it is a great mimic, and the diag-
teeth, but, more commonly, they are relatively small. nosis is a histologic one. These lesions are different
Most dentigerous cysts are asymptomatic, and their from other cysts; they are aggressive and can be dif-
discovery is usually an incidental finding on radiography. ficult to remove. OKCs can grow quite rapidly, and
Management: Removal of impacted third molars recurrences are frequent. This is the third most com-
with pericoronal radiolucencies; however, impacted mon odontogenic cyst. Histologically, these cysts are
teeth with small pericoronal radiolucencies (suggest- formed with a stratified squamous epithelium that
ing the presence of normal dental follicle rather than produces orthokeratin (10%), parakeratin (83%), or
dentigerous cyst) and monitoring with serial radio- both types of keratin (7%). The lesions grow in a
graphic examination. Any increase in the size of the multilocular bosselated fashion with daughter cysts
lesion should prompt removal and histopathologic that extend into the surrounding bone. Surgical treat-
examination. Any lesion that appears larger than a ment does not result in complete removal of the le-
normal dental follicle indicates removal and histo- sion. Enucleations with peripheral ostectomy and/or
pathologic examination. cryosurgery are the most common forms of treatment.
iii. Primordial cyst: This is a cyst which develops in- Long-term (lifetime) follow-up radiography is im-
stead of a tooth. Presumably, the dental follicle forms perative. If these lesions are left untreated, they can
and subsequently undergoes cystic degeneration with- become quite large and locally destructive. Non-
out ever completing odontogenesis. This is the rarest odontogenic lesions that mimic odontogenic lesions
odontogenic cyst, and lesions designated as primor- include benign fibro-osseous lesions (conventional or
dial cysts may represent residual cysts. juvenile ossifying fibroma, focal or periapical ce-
iv. Residual cyst is a term of convenience because no mento-osseous dysplasia, florid osseous dysplasia),
teeth are left by which to identify the lesion. Most traumatic bone cyst, lingual salivary gland inclusion
commonly, these are actually retained periapical cysts defect, central giant cell granuloma, brown tumour of
from teeth that have been removed. The histology is a hyperparathyroidism, arteriovenous malformation,
nondescript stratified squamous epithelium. and mucoepidermoid carcinoma. The clinical and ra-
v. Lateral periodontal cyst is a misnomer. These cysts diographic features of these mandibular lesions help
are not inflammatory, they do not arise from periodon- establish a differential diagnosis, although micro-
titis, and they are not a phenomenon associated with scopic tissue evaluation is generally necessary to
lateral canals within the tooth structure. These cysts accurately identify the lesion.
are always well demarcated, relatively small, and
radiolucent (sometimes with a radio-opaque roof).
B. Non-Odontogenic Cysts of Mandible:
They are most commonly associated with the
mandibular premolar area and are occasionally found i. Stafne bone cyst is an unusual form of slightly aber-
in the maxillary anterior. rant salivary gland tissue wherein a developmental
vi. Gingival cysts of newborns generally occur in multi- inclusion of glandular tissue is found within or, more
ples but occasionally occur as solitary nodules. They commonly, adjacent to the lingual surface of the body
are located on the alveolar ridges of newborns or of the mandible within a deep and well-circumscribed
young infants. These structures originate from depression. Radiographically, the lesion usually
remnants of the dental lamina and are located in the appears as an ovoid radiolucency located between the
corium below the surface epithelium. They are gener- inferior alveolar canal and the inferior border of the
ally asymptomatic and do not produce any discomfort mandible in the region of the second or third molars.
for the infant. No treatment is required for these These lesions generally represent benign develop-
lesions, which usually disappear either by opening mental anomalies that normally do not require any
on to the surface mucosa or through disruption by treatment.
erupting teeth. These cysts are most likely what older ii. Traumatic bone cyst (solitary bone cyst, haemorrhagic
literature describes as predeciduous dentition. cyst, extravasation cyst, unicameral bone cyst, simple
Section | II General Surgery 289

bone cyst, and idiopathic bone cavity) is a relatively present at birth, it becomes noticeable within the first few
frequent lesion both in the jaws and elsewhere in the weeks of life. The lesion first enlarges to some degree,
skeleton. The lesion is most commonly found in young reaching its maximum size by the age of six months or so,
persons (median age, 18 y); the male-to-female inci- and occasionally becomes ulcerated; but it usually recedes
dence ratio is 3:2. The lesions occasionally have been after the first year of life. Spontaneous complete involution
reported in the maxilla but are far more common in the of the lesion occurs in practically all cases, normally taking
mandible. Radiographically, these lesions tend to ap- place in early childhood. Treatment is rarely necessary.
pear as smoothly outlined radiolucencies that scallop
Cavernous haemangioma, a rare, red-blue, raised tumour,
around the roots of the teeth. They do not displace teeth
is constituted of rather large blood vessels enclosed within
or resorb roots, and the lamina dura is left intact. These
a framework of connective and fatty tissues. Although most
lesions are usually surgically explored to establish a
often associated with the skin, it is also sometimes found in
diagnosis, which is made upon finding an empty cavity.
mucous membranes, the brain, and the viscera. In all cases,
No further treatment is generally necessary because
it is present fully developed at birth; it is rarely malignant
surgical manipulation causes the cavity to fill with
and increases in size only to the same extent as that of the
blood. Soft tissues are closed, and the lesion tends to
body part involved. There is no satisfactory treatment. For
heal without further intervention.
cosmetic purposes, surgery, if feasible, may be considered.
iii. Aneurysmal bone cyst is neither a cyst nor an aneu-
rysm. Aneurysmal bone cysts are more likely to occur Q. 3. Describe the methods of spread of carcinoma and
in the mandible than in the maxilla. They may displace grading and staging of carcinoma in general.
but usually do not resorb the dentition, and sensory
Ans.
disturbances generally are not present. The radiographic
appearance often is described as cystic, honeycomb, or The spread of carcinoma is defined as metastasis. Metas-
soap bubble with eccentric expansion. Histologically, tasis is a complex series of steps in which cancer cells
the aneurysmal bone cyst reveals a fibrous connective leave the original tumour site and migrate to other parts of
tissue stroma with many cavernous or sinusoidal blood- the body via the bloodstream or the lymphatic system. To
filled spaces. Young fibroblasts are present throughout do so, malignant cells break away from the primary tu-
the stroma, and multinucleated giant cells are scattered mour and attach to and degrade proteins that make up the
throughout the lesion. Treatment of the aneurysmal surrounding extracellular matrix (ECM), which separates
bone cyst requires complete removal, and complete re- the tumour from adjoining tissue. By degrading these pro-
moval of the lesion with aggressive curettage is the teins, cancer cells are able to breach the ECM and escape.
most common treatment modality. When oral cancers metastasize, they commonly travel
through the lymph system to the lymph nodes in the neck.
Q. 2. What are the different types of haemangiomas? The body resists metastasis by a variety of mechanisms
Describe the treatment of each. through the actions of a class of proteins known as metas-
Ans. tasis suppressors.

Haemangioma is a congenital, benign tumour, made up of


new-formed blood vessels of the skin. There are three main Routes of Metastasis
types. Metastasis occurs by four routes:
l Spread into body cavities/local invasion: This occurs by
Capillary haemangioma, also called nevus flammeus or
the seeding surface of the peritoneal, pleural, pericardial
port-wine stain, is a common skin lesion resulting from
or subarachnoid spaces.
abnormal local aggregation of capillaries, the smallest
l Spread by lymphatic system: This is followed by the
blood vessels. The stain-like lesion is smooth surfaced, not
transport of tumour cells to regional nodes and ulti-
elevated, and well demarcated. It is pink to dark bluish-red.
mately to other parts of the body; it is common in initial
It varies in size and shape and is seen most frequently on
spread of carcinomas.
the back of the head or neck and less frequently on the
l Haematogenous spread: This is typical of all sarcomas
forehead and about the eyes. There is no satisfactory treat-
but it is the favoured route in certain carcinomas (e.g.
ment, but the lesion usually becomes less noticeable and
those originating in kidneys). Because of their thinner
sometimes disappears as the skin thickens with age.
walls, veins are more frequently invaded than arteries
Immature haemangioma, also called haemangioma sim- and metastasis follows the pattern of the venous flows.
plex or strawberry mark, is a common reddish nubbin on l Transplantation: Mechanical carriage of fragments of
the skin, constituted of aggregations of dilated small blood tumour cells by surgical instruments during operation or
vessels, which may or may not occur singly. If not already the use of needles during diagnostic procedures.
290 Quick Review Series for BDS 3rd Year

Staging of Carcinoma over the world, and every effort should be made to institute
educational programmes on protection against the sun at a
Staging of cancer is the most important predictor of sur-
very early age. The use of protective clothing (e.g. tightly
vival, and cancer treatment. Staging of a carcinoma is de-
woven fabrics, wide-brimmed hats, long sleeves and long
fined as the description (usually numbers I to IV with IV
trousers) is highly effective. When this is not practicable or
having more progression) of the extent carcinoma’s spread.
acceptable, it is important to encourage regular use of sun-
The stage often takes into account the size of a tumour, how
screen products with an SPF rating of at least 15. People
deeply it has penetrated, whether it has invaded adjacent
should understand that the atmosphere has a filtering effect
organs, how many lymph nodes it has metastasized to (if
on ultraviolet light and, therefore, they should avoid expo-
any), and whether it has spread to distant organs. Carci-
sure at midday when the sun is most vertical and the light
noma staging can be divided into a clinical stage and a
passes through less of the atmosphere.
pathologic stage. In the TNM (tumour, node, metastasis)
l Actinic or solar keratoses – these are common asymp-
system, clinical stage and pathologic stage are denoted by a
totic lesions seen mostly on sun-exposed areas of light-
small “c” or “p” before the stage. This staging system is
skinned people.
used for most forms of cancer, except brain tumours and
l Chemical and other keratoses – skin lesions caused by
haematological malignancies.
exposure to arsenic, tar, polycyclic hydrocarbons, infra-
Overall stage grouping is also referred to as Roman
red radiation for a prolonged period at the work place
Numeral Staging. This system uses numerals I, II, III, and
(thermal keratosis) and scar keratosis.
IV (plus the 0) to describe the progression of cancer.
l Large cell acanthoma
l Stage 0 carcinoma in situ.
l Chondrodermatitis nodular helicis
l Stage I cancers are localized to one part of the body.
l Cutaneous horn – a hard, raised nodule with a reddish
l Stage II cancers are locally advanced.
base, usually seen in sun-exposed areas in pale persons.
l Stage III cancers are also locally advanced. Whether a
l Radiation dermatitis
cancer is designated as Stage II or Stage III can depend
l Bowen’s disease – a condition located totally within the
on the specific type of cancer. The specific criteria for
top layer of the skin called the epidermis, and favours
Stages II and III differ according to diagnosis.
the sun-exposed areas of the face, neck and extremities.
l Stage IV cancers have often metastasized, or spread to
l Sebaceous nevi
other organs or throughout the body.
l Porokeratosis – describes a wide variety of skin disorders.
l Fibroepithelioma of Pinkus – appears as flesh-coloured
Within the TNM system, a cancer may also be designated
skin tags hanging from the back.
as recurrent, meaning that it has appeared again after being
l Keratoacanthoma – a rapidly growing, red, raised lesion
in remission or after all visible tumour has been eliminated.
on the sun-exposed areas of middle-aged and elderly
Q. 4. What are the premalignant conditions of skin? people.
l Condylomata acuminata – a virus-induced lesion in
Ans.
the genitals, which appears as sharp pointed red, small
Exposure of skin to sunlight is beneficial in moderation since finger-like projections, either stuck at the base or sus-
ultraviolet radiation is vital to the synthesis of vitamin D and pended by a stalk.
hence to satisfactory skeletal development. Excessive expo- l Buschke-Lowenstein tumour – a giant variety of
sure, however, is hazardous, particularly in light-skinned condylomata acuminata.
persons who tan poorly, and in patients who are photosensi-
tive. There is also evidence that ultraviolet light decreases the Q. 5. What are the premalignant conditions and lesions
immunological responses of the skin. Photodamage is first of oral cavity.
evident as acute sunburn and, in the longer term, as prema- Ans.
ture ageing of the skin. Excessive exposure to sunlight also
predisposes to the development of malignant and premalig- The premalignant conditions of oral cavity are as follows:
nant skin lesions, including actinic keratosis, squamous cell A. Leukoplakia: Leukoplakia describes a white patch or
and basal cell carcinoma, and malignant melanoma. plaque that cannot be characterized clinically or patho-
The incidence of malignant and premalignant condi- logically as any other disease and remains a clinical
tions is particularly high among light-skinned persons liv- diagnosis of exclusion. Leukoplakia occurs most often
ing in hot sunny climates and in dark-skinned persons with in middle-aged and older men and arises most fre-
vitiligo or albinism. Many cases could be prevented by quently on the buccal mucosa, alveolar mucosa, and
persuading parents and individuals directly at risk of the lower lip. However, note that lesions arising on the floor
importance of avoiding sunburn and reducing exposure to of mouth, lateral tongue, and lower lip are the most
solar radiation. Photoprotection in children is a priority all likely to harbour dysplasia or progress to malignancy.
Section | II General Surgery 291

B. Erythroplakia: It is a clinical term used to describe a demarcation of separate tissues between the individual
fiery-red patch that cannot be clinically or pathologi- “toothlets”. It is common in the anterior maxilla.
cally distinguished as any other definable disease. l Complex odontoma is unrecognizable as dental tissues,
Similar to leukoplakia, the erythroplakic lesion is usually presenting as a radio-opaque area with varying
considered as a diagnosis of exclusion. It is described as densities. It is common in the posterior maxilla or in the
a red macule or patch with a soft, velvety texture most mandible.
often occurring on the floor of mouth, lateral tongue, l Ameloblastic odontoma: Generally benign but occa-
retromolar pad, and soft palate. Upon histological sionally aggressive and recurs locally after conservative
analysis, 51% of erythroplakic lesions have been shown surgical removal
to demonstrate invasive squamous cell carcinoma
(SCC), with 40% demonstrating carcinoma in situ, and
9% exhibiting mild-moderate dysplasia. Histopathology
C. Proliferative verrucous leukoplakia: It is a unique form l Complex odontoma
of aggressive disease considered to be within the con- l Poorly differentiated
tinuum of leukoplakia and erythroplakia. Most patients l Variety of calcified patterns
with PVL are women, and many do not have a history of l Not enough coordinated production of enamel,
tobacco use. PVL generally appears on the oral mucosa dentin, or cementum to identify actual tooth
as an irregular white patch or plaque with a varying sur- l Compound odontoma
face. The disease is often characterized by resistance to l Higher degree of differentiation than complex
treatment, recurrence, multifocal proliferation, and a odontoma
progression to carcinoma in up to 87% of patients. l Characteristically small misshapen teeth known as
D. The palatal lesion of reverse smokers is unique to indi- denticles
viduals who place the lit end of a cigarette inside the l Usually only a few
mouth. The resulting palatal lesion may appear l Sometimes as many as 2000
clinically as a red, white, melanotic patch or papule. Up l Ameloblastic odontoma
to 84% of palatal lesions have been demonstrated to l Prominent epithelial component resembling amelo-
harbour dysplasia upon histology. blastoma
l Currently regarded as immature complex odontoma
The premalignant lesions of oral cavity are as follows:
l Dental hard and soft tissues, such as enamel and
A. Oral submucous fibrosis refers to a chronic progressive
condition found predominantly in people of Asian de- dentin
cent. OSF is considered to be the result of the use of the Q. 7. Describe the clinical features, diagnosis and
areca nut product with resultant disruption of the extra- treatment of ameloblastoma.
cellular matrix. The disease often manifests with diffuse
involvement of the oral cavity, pharynx, and upper Ans.
oesophagus that appears clinically as whitish mucosa
Ameloblastoma is an entirely epithelial tumour arising
lacking elasticity. Malignant transformation rate is
from the dental lamina, Hertwig sheath, the enamel organ,
approximately 7% of these lesions.
or the lining of dental follicles/dentigerous cysts. Amelo-
B. Lichen planus, discoid lupus erythematosus: Although
blastoma is the most common epithelial odontogenic
classified as potentially malignant conditions, the data
tumour. Ameloblastomas usually occur in individuals aged
regarding progression to malignancy for these condi-
20-40 years; however, the unicystic variant (see Surgical
tions is controversial. Because of the difficulty in
Considerations, below) most often occurs in adolescents.
classifying and clinically distinguishing the varied
This lesion occurs in both the maxilla and mandible, but the
lesions associated with these conditions, the potential
posterior mandible is the most common location, with
for malignant transformation remains unclear.
equal distribution among males and females. it is extremely
Q. 6. Describe the classification, clinical and histopatho- aggressive and infiltrative. Radiologically, they present as
logical features of odontoma. an expansile multilocular radiolucency in the area of the
lower third molar, but they may be found anywhere in the
Ans.
jaws. These lesions may be unilocular when small, and they
Odontoma can be defined as a hamartoma of odontogenic often resorb the teeth they contact. These lesions are never
origin. They are classified into two types: radiopaque. Histologically, ameloblastoma does not have a
l Compound odontoma which has three separate dental capsule. The six different histopathological variants of am-
tissues (enamel, dentin and cementum), but may eloblastoma are desmoplastic, granular cell, basal cell,
present a lobulated appearance without any definitive plexiform, follicular, and acanthomatous.
292 Quick Review Series for BDS 3rd Year

Clinical Features accomplished with block or segmental resection, depending


on the relationship of the lesion to the inferior cortical border.
Ameloblastomas are often associated with the presence of un-
For peripheral ameloblastoma, a more conservative excision
erupted teeth. Symptoms include painless swelling, facial defor-
with close clinical follow-up is the standard of care.
mity if severe enough, pain if the swelling impinges on other
structures, loose teeth, ulcers, and periodontal (gum) disease. Q. 8. Describe the clinical features, diagnosis and treat-
Lesions will occur in the mandible and maxilla, although 75% ment of the dentigerous cyst.
occur in the ascending ramus area and will result in extensive
Ans.
and grotesque deformitites of the mandible and maxilla. In the
maxilla, it can extend into the maxillary sinus and floor of the Dentigerous cyst represents the second most common odon-
nose. The lesion has a tendency to expand the bony cortices togenic cyst is the dentigerous cyst, which develops within
because slow growth rate of the lesion allows time for perios- the normal dental follicle that surrounds an unerupted tooth.
teum to develop thin shell of bone ahead of the expanding The dentigerous cyst is not thought to be neoplastic. It most
lesion. This shell of bone cracks when palpated and this frequently is found in areas where unerupted teeth are
phenomenon is referred to as “egg shell cracking” or crepitus, found: mandibular third molars, maxillary third molars, and
an important diagnostic feature. Ameloblastoma is tentatively maxillary canines, in decreasing order of frequency. These
diagnosed through radiographic examination and must be cysts can grow very large and can move teeth, but, more
confirmed by histological examination. Radiographically, it commonly, they are relatively small. Most dentigerous cysts
appears as a lucency in the bone of varying size and features— are asymptomatic, and their discovery is usually an inciden-
sometimes it is a single, well-demarcated lesion whereas it often tal finding on radiography.
demonstrates as a multiloculated “soap bubble” appearance. The usual radiographic appearance is that of a well-
Resorption of roots of involved teeth can be seen in some cases, demarcated radiolucent lesion attached at an acute angle to
but is not unique to ameloblastoma. The disease is most often the cervical area of an unerupted tooth. The border of the
found in the posterior body and angle of the mandible, but can lesion may be radio-opaque. The radiographic differentia-
occur anywhere in either the maxilla or mandible. tion between a dentigerous cyst and a normal dental follicle
Ameloblastoma is often associated with bony-impacted is based merely on size.
wisdom teeth—one of the many reasons dentists recom- However, histologically, a distinction other than size is found.
mend having them extracted. The dental follicle is normally lined by the reduced enamel epi-
thelium, while the dentigerous cyst is lined with a stratified
squamous nonkeratinizing epithelium. Dystrophic calcification
Treatment and clusters of mucous cells may be found within the cysts.
While chemotherapy, radiation therapy, curettage and liquid Dentigerous cysts develop from follicular epithelium, and
nitrogen have been effective in some cases of ameloblastoma, follicular epithelium has greater potential for growth, differ-
surgical resection or enucleation remains the most definitive entiation, and degeneration than the epithelium from which
treatment for this condition.The treatment of ameloblastoma radicular cysts arise. Occasionally, other more ominous le-
which is recommended is surgical excision with wide free sions arise within the walls of the dentigerous cyst, including
margins. Appropriate reconstruction may be performed at the mucoepidermoid carcinoma arising from mucous cells within
same time. All patients with ameloblastoma, regardless of the cyst walls, ameloblastoma and squamous cell carcinoma.
surgical treatment method or histologic type, must be moni-
tored radiographically throughout their lifetime. If excision is
Treatment
inadequate, recurrence is common. The maxillary ameloblas-
toma is not confined by the strong cortical plate found in the l Removal of impacted third molars with pericoronal ra-
mandible. In addition, the posterior maxilla lies in close rela- diolucencies.
tionship to many vital structures. These factors make strong l Any increase in the size of the lesion should prompt
arguments for aggressive and definitive surgical treatment of removal and histopathologic examination.
the maxillary ameloblastoma. In the mandible, 1-cm l Any lesion that appears larger than a normal dental fol-
clear margins are considered the standard. This may be licle indicates removal and histopathologic examination

SHORT ESSAYS
Q. 1. Haemangioma. present at birth (infantile haemangioma). The rest ap-
pear in the first several months of life.
Ans.
l Haemangioma, according to location can be classified
l Haemangioma is a benign self-involuting tumour of as:
endothelial cells. About 30% of haemangiomas are a. In the top skin layers-capillary haemangioma
Section | II General Surgery 293

b. Deeper in the skin–cavernous haemangioma l Psychological problems, from skin appearance


c. A mixture of both. l Secondary infections and sores
l Visible changes in the skin
Clinical Features l Vision problems (amblyopia, strabismus)

i. It can present as: Q. 2. Dermoid cyst.


l A red to reddish-purple, raised sore (lesion) on the skin Ans.
l A massive, raised tumour with blood vessels

ii. The appearance depends on location: l Dermoid cyst is a benign cystic teratoma. Histologi-
l If they are on the surface of the skin, they resemble cally, they form due to sequestration of skin along the
a ripe strawberry (referred to as “strawberry hae- lines of embryonic closure.
mangiomas”). l Depending on the location, dermoid cysts may contain
l If they are just under the skin, they present as a blu- substances such as nails and dental, cartilage-like, and
ish swelling. bone-like structures. Dermoid cysts can occur in the
iii. Sometimes they grow in internal organs such as the skin and subcutis and also can be intracranial, intraspi-
liver or larynx. nal, or perispinal. Intra-abdominal cysts, such as cystic
iv. In most cases, haemangiomas will disappear over time. tumours of the ovary or omentum, occur as well.
v. Most haemangiomas are on the face and neck.
Clinical Features
Diagnosis l Congenital dermoid cysts localized on the neck, head,
l Haemangiomas are diagnosed by a physical examination. or trunk are usually visible at birth. When on the head,
l In the case of deep or mixed lesions, a CT or MRI scan dermoid cysts are often adherent to the periosteum. The
may be performed. usual diameter of the lesions is 1-4 cm.
l Intracranial, intraspinal, or intra-abdominal dermoid
Treatment cysts may be suspected due to specific or non-specific
neurologic or gynaecologic symptoms.
l Superficial or “strawberry” haemangiomas often are not l Unilateral upper eyelid swelling may be the first sign,
treated. When they are allowed to disappear on their with imaging studies demonstrating a soft tissue orbital
own, the result is usually normal-appearing skin. dermoid cyst arising from the lacrimal gland.
l In some cases, a laser may be used to remove the small l An eyelid dermoid cyst attached to a tarsus may be evi-
vessels. dent as a firmly adherent non-tender upper-eyelid nodule.
l Cavernous haemangiomas that involve the eyelid and
block vision are generally treated with steroid injections
or laser treatments. These quickly reduce the size of the Treatment
lesions, allowing vision to develop normally. l Treatment for dermoid cyst is complete surgical
l Large cavernous haemangiomas or mixed haemangio- removal, preferably without spillage of cyst contents.
mas may be treated with oral steroids and injections of l Marsupialization is inappropriate for dermoid cyst due
steroids directly into the haemangioma. to the risk of malignancy. Intracranial, intramedullary,
l Surgical removal is sometimes indicated, particularly if and ovarian dermoid cysts are difficult to treat.
there has been delay in commencing treatment and l Sophisticated neurosurgical or gynaecologic surgical
structural changes have become irreversible. techniques are often needed to remove the dermoid cyst
l Surgery may also be necessary to correct distortion of and prevent possible complications.
facial features, again in the case of inadequate or failed
early medical intervention. Q. 3. Sublingual dermoid.
l Recently, lasers have been used to reduce the size of the Ans.
haemangiomas. Lasers that emit yellow light damage
the vessels in the haemangioma without damaging the l Sublingual dermoid is a type of congenital sequestration
skin over it. Some physicians use a combination of ste- dermoid cyst.
roid injection and laser therapy. l The cyst is formed by inclusion of the surface ectoderm
at the fusion line of two mandibular arches.
Complications
Pathology
l Bleeding (especially if the haemangioma is injured)
l Problems with breathing and eating l The cyst is lined by squamous epithelium.
294 Quick Review Series for BDS 3rd Year

l The wall of cyst contains hair follicle, sweat and Diagnosis


sebaceous glands.
X-ray shows:
l Cyst contains the cheesy material.
l Tooth in the cyst
l Soap bubble appearance due to multiple trabeculations
Types of the bone.
l Radiolucent well-defined swelling
l Lateral variety
l Bony margins are well defined and sharp.
l Supramyelohyoid type
l Inframyelohyoid type.
Treatment
Clinical Features Treatment is based on the size of the cyst
l Small cyst: Excision of the cyst by intraoral approach.
l It occurs in young children between the ages of 10 to l Large cyst: Managed by marsupialisation
12 years.
l Congenital, painless swelling in the floor of mouth. Q. 5. Lipoma.
l Swelling is soft and cystic Ans.
l Fluctuation test is positive.
l Transillumination test is negative as it contains thick, l Lipoma is a benign tumour composed of adipose tissue
cheesy, sebaceous material. and is the most common form of soft tissue tumour.
l They are slow-growing, benign fatty tumours form soft,
lobulated masses enclosed by a thin, fibrous capsule.
Treatment l It has been suggested that trauma-related fat herniation
l Total excision is the treatment of choice. through tissue planes creates so-called pseudolipomas.

Q. 4. Dentigerous cyst.
Clinical Features
Ans.
l Lipomas are mostly asymptomatic.
l Dentigerous cyst, also known as follicular odontoma is a l When they arise from fatty tissue between the skin and
common odontogenic cyst of epithelial origin, which en- deep fascia, typical features include a soft, fluctuant
closes the crown of an impacted tooth at its neck portion. feel; lobulation; and free mobility of overlying skin.
l The cyst is lined by squamous epithelium, surrounded l A characteristic “slippage sign” may be elicited by
by connective tissue. Within the cyst, the tooth lies gently sliding the fingers off the edge of the tumour.
obliquely or sometimes embedded in the wall of the The overlying skin is typically normal.
cyst. As cyst grows, it displaces the tooth and prevents
it from eruption. Symptoms in other sites depend on the location
and can include the following:
Lipomas in the major airways can cause respiratory
Clinical Features l

distress due to bronchial obstruction.


l Age and sex: It is usually found in children, equal in l Patients with oesophageal lipomas can present with ob-
both the sexes. struction, dysphagia, regurgitation, vomiting, and reflux.
l Site: Most common site is mandibular third molar and max- l Cardiac lipomas are mainly subendocardial, mostly un-
illary canine which are most commonly impacted teeth. capsulated, appearing as a yellow mass projecting into
the cardiac chamber.
Intramediastinal lipomas can impinge on the superior vena
Symptoms
l

cava, thereby leading to superior vena cava syndrome.


l Cyst remains asymptomatic, if uninfected. l Intestinal lipomas can manifest as classic obstruction,
l On infection, inflammatory signs are present. intussusception, volvulization, or haemorrhage.
l Expansion of mandible: Since the inner table of l Lipomas in intra-articular joint spaces or intraosseous
mandible is strong, the expansion mainly occurs in the sites, such as the calcaneus, can lead to joint dysfunc-
outer aspect of the mandible. The bone gets thinned out tion and pain restricting normal movement.
resulting in egg shell cracking. l Lipomas may also arise in the dural or medullary
l Blue-domed cyst: When it contains blood then it is components of the spinal cord can result in cord
called as blue-domed cyst. compression.
Section | II General Surgery 295

Treatment Treatment
l Lipomas are normally removed by simple excision. The following methods are employed in the treatment of
l New methods are also used to remove lipomas without basal-cell carcinoma (BCC):
scar, using of injection of compounds that trigger lipolysis, l Standard surgical excision with either frozen section
such as steroids or phosphatidylcholine. histology.
l Mohs surgery or Mohs micrographic surgery is an
Q. 6. Basal cell carcinoma. outpatient procedure in which the tumour is surgically
Ans. excised and then immediately examined under a micro-
scope. The base and edges are microscopically examined
l Basal cell carcinoma is the most common type of skin to verify sufficient margins before the surgical repair of
cancer. Basal cell skin cancer tumours typically appear the site.
on sun-exposed skin, are slow growing, and rarely l Chemotherapy: Some superficial cancers respond to
metastasize and cause significant destruction and local therapy with 5-fluorouracil,
disfigurement. l Radiation: Appropriate for all forms of BCC and can be
l Basal cell carcinoma is a non-melanocytic skin cancer, delivered either as external beam radiotherapy or
i.e. an epithelial tumour that arises from basal cells, brachytherapy.
which are small round cells found in the lower layer of l Photodynamic therapy: This is a new modality for treat-
the epidermis. ment of basal-cell carcinoma, which is administrated by
l Basal cells invade the dermis but seldom invade other application of photosensitizers (methyl aminolevu-
parts of the body. linate) to the target area. When these molecules are
l Body distribution of BCCs is as follows: activated by light, they become toxic, therefore, destroy
l On the head (most frequently on the face; most the target cells.
common location – nose ) – 70% l Cryosurgery.
l On the trunk – 25% l Electrodessication and curettage.
l On the penis, vulva, or perianal skin – 5%
Q. 7. Adamantinoma.
Ans.
Types of BCC
l Adamantinoma is a locally aggressive osteolytic tumour
l Nodular – cystic, pigmented, keratotic
arising from ameloblasts.
l Superficial
l It is is an extremely rare, slow growing, low-grade ma-
l Infiltrative
lignant tumour of epithelial origin, that occurs almost
l Micronodular
exclusively in the tibia and the fibula, i.e. the diaphyseal
l Morpheaform
region is the area most commonly affected.
l Superficial
l Adamantinomas are classified into 2 distinct types:
a. Classic and
Clinical Features b. Differentiated.
l Classic adamantinomas usually occur in patients older
l History of chronic sun exposure, including recreational than 20 years, whereas differentiated adamantinomas oc-
sun exposure and occupational sun exposure. cur almost exclusively in patients younger than 20 years.
l Waxy papules with central depression, pearly ap­ l The patient usually has swelling that may be painful.
pearance. The duration of symptoms can vary from a few weeks
l Erosion or ulceration, often central bleeding, especially to years.
when traumatized.
l Crusting, rolled (raised) border
l Translucency Diagnosis
l Telangiectases over the surface l Adamantinoma appears as an eccentric, well-circum-
l Slow growing (0.5 cm in 1-2 y) scribed, and lytic lesion on plain X-ray.
l The anterior cortex of the tibia is by far the most com-
mon location.
Diagnosis l The lesion usually has several lytic defects separated by
To diagnose basal-cell carcinomas, a skin biopsy is taken sclerotic bone which gives a “soap-bubble” appearance.
for pathological study. There is cortical thinning but little periosteal reaction.
296 Quick Review Series for BDS 3rd Year

l The lesion may break through the cortex and extend into Q. 9. Epidermoid cyst.
soft tissue. There may be multiple adjacent lesions with
Ans.
normal intervening bone.
l MRI helps demonstrate the intraosseus and extraosse- l Epidermoid cyst is a benign cyst usually found on the
ous involvement. skin which develops from ectodermal tissue.
l It results from the proliferation of epidermal cells
within a circumscribed space of the dermis.
Treatment l Histologically, epidermoid cyst is made of a thin layer of
l Marginal resection: Delayed local recurrence can be squamous epithelium. Epidermoid cysts commonly result
expected in 32%. from implantation of epidermis into the dermis, as in trauma
l En bloc resection: Local recurrence should not occur. or surgery and also due to obstruction of sebaceous ducts.
l Mean survival with metastatic disease: 12 years.
l Highly radioresistant. Clinical Features
l Chemotherapy has not been shown to be effective.
l Metastasize to lungs, lymph nodes and abdominal l Epidermoid cysts are usually asymptomatic; however,
organs by both haematogenous and lymphatic routes. they may become inflamed or secondarily infected,
resulting in swelling and tenderness.
Q. 8. Burkitt’s lymphoma. l Rarely in event of malignancy, rapid growth, friability,
and bleeding may be reported.
Ans. l Epidermoid cysts appear as flesh-coloured-to-yellow-
l Burkitt’s lymphoma is an uncommon type of non- ish, firm, round nodules of variable size.
Hodgkin’s lymphoma (NHL) which commonly affects l A central pore or punctum may be present.
children.
l It is a highly aggressive type of B cell lymphoma. This Treatment
condition is named after Denis Parsons Burkitt, who
first described the disease in 1956 in equatorial Africa. l Asymptomatic epidermoid cysts do not need to be treated.
l Burkitt’s lymphoma can be divided into three main l Intralesional injection with triamcinolone may decrease
clinical variants: and treat inflammation.
a. the endemic, l Epidermoid cysts can be removed by simple excision or
b. the sporadic and incision with removal of the cyst and cyst wall though
c. the immunodeficiency-associated variants. the surgical defect.
l The endemic variant is found in equatorial Africa and l Incision and drainage may be performed, if a cyst is infected.
is the most common malignancy of children in this area. Q. 10. Ranula.
l The sporadic type is found outside of Africa. The jaw is
less commonly involved, compared to the endemic variant. Ans.
The ileocaecal region is the common site of involvement. l Ranula is a clinical term for a pseudocyst associated
l The immunodeficiency-associated Burkitt’s lymphoma with mucus extravasation into the surrounding soft
is usually associated with HIV infection or occurs in tissues or mucoceles that occur in the floor of the
the setting of post-transplant patients who are taking mouth and usually involve the major salivary glands.
immunosuppressive drugs. l They are usually caused by trauma to salivary duct and
occasionally salivary obstruction.
Treatment l Ranulas are divided into 2 types:
i. Oral ranulas and
l Chemotherapy is the best option. ii. Cervical or plunging ranulas.
i) Methotrexate–1 mg/kg/day for 4-5 days. l Oral ranulas are secondary to mucus extravasation that
ii) Cyclophosphamide orally or intravenously 30-40 pools superior to the mylohyoid muscle, whereas cervi-
mg/kg in a single or divided doses for 2-3 days in a cal ranulas are associated with mucus extravasation
month. along the facial planes of the neck.
iii) Vincristine sulphate is given intravenously 0.05 to
0.15 mg/kg.
The Oral Ranula
l Treatment includes dose-adjusted EPOCH with Rituxan
(rituximab). Other treatments are immunotherapy, bone l It is a relatively large unilateral blue to translucent mass
marrow transplants, surgery to remove the tumour, and in the floor of the mouth that remotely resembles the
radiotherapy. belly of a frog (Rana species).
Section | II General Surgery 297

l The consistency of the lesion is that of mucus, and the l At the level of cricoid cartilage.
lesion does not blanch on compression. l At the floor of the mouth.
l Individuals with an oral ranula may complain of swell-
ing of the floor of the mouth that is usually painless. The Clinical Features
mass may interfere with speech, mastication, respira-
tion, and swallowing because of the upward and medial l Thyroglossal cyst appears around the age of 15 to
displacement of the tongue. 30 years.
l When oral ranulas are large, the tongue may place pres- l They are more common in females.
sure on the lesion, which may interfere with subman- l They are painless, midline swellings. Pain occurs, if the
dibular salivary flow. As a result, obstructive salivary cyst becomes infected.
gland signs and symptoms may develop, such as pain or l The cyst is soft, cystic, fluctuant. Cyst moves with de-
discomfort when eating, a feeling of fullness at that site, glutition.
and increased swelling of the submandibular gland. l Transillumination is negative.

The Cervical Ranula Complications


l It appears as an asymptomatic, continuously enlarging l Infection
mass that may fluctuate in size. The overlying skin is l Fistula formation
usually intact. l Carcinoma
l The mass is fluctuant, freely movable, and non-tender.
The mass is not associated with the thyroid gland or Treatment
lymph node chains.
l Sistrunk operation: Excision of the cyst along with en-
tire thyroglossal duct, which may include part of the
Treatment hyoid bone.
i) Complete excision: It is often difficult to excise the cyst Q. 12. Classify the cyst of the jaw.
as it bursts open while dissecting. It is very thin walled.
In the tense cyst, it may be advisable to aspirate some Ans.
fluid and then it will become easy to carry out complete Cysts of jaws are classified as follows:
excision. A. Odontogenic cysts:
ii) Partial excision and marsupialisation: The major part a. Depending upon the origin:
of the cyst along with mucous membrane of the floor I. Developmental cysts:
of the mouth is excised. The cut edges of the cyst wall l Eruption cyst
are sutured to the cut mucous membrane. The cyst is l Dentigerous cyst
wide open and it will not have any opportunity to form l Gingival cyst of adult
a retention cyst again. l Gingival cyst of infants
iii) Deep plunging ranula: Should be excised completely. l Odontogenic keratocyst.
The cyst is dissected and removed completely. Any bit II. Inflammatory cysts:
of a cyst wall missed during the dissection will surely l Radicular (dental cyst)
recur. l Apical and lateral cyst

Q. 11. Thyroglossal cyst. l Residual cyst

l Paradental cyst.
Ans. b. Depending upon the tissue of origin:
Thyroglossal cyst is a congenital tubular dermoid cyst. It l From reduced enamel epithelium: Dentigerous

arises from thyroglossal duct, which extends from foramen cyst, eruption cyst.
caecum at the base of tongue to the isthmus of the thyroid l From epithelial rest of Malassez: Radicular cyst,

gland. residual cyst.


l From dental lamina: Gingival cyst of infants,

gingival cyst of adults.


Sites of Occurrence B. Non-odontogenic cyst
l Subhyoid is the most common site. l Nasopalatine duct cyst

l At the level of thyroid cartilage. l Globulomaxillary cyst

l Suprahyoid. Double chin appears. l Nasolabial cyst

l At the foramen caecum. l Median palatine, median alveolar and mandibular cyst.
298 Quick Review Series for BDS 3rd Year

SHORT NOTES
Q. 1. Sebaceous cyst. midline of the palate, discharge, pain defined as a
burning sensation in the anterior part of the maxilla that
Ans.
occasionally radiates into the bridge of the nose, and
i. Sebaceous cyst is also called as epidermoid cyst. in a person who wears a dental prothesis, a pressure
ii. The obstruction of sebaceous ducts, resulting in accu- sensation underneath the prosthesis.
mulation of sebaceous material, which becomes v. Nasopalatine duct cysts are treated by enucleation via a
enlarged due to retention of its sebum. So it is also palatine or buccal approach.
called retention cyst. Q. 4. Lipoma.
iii. The cyst is lined by squamous epithelium material with
an unpleasant smell. In the centre of cyst, a black spot Ans.
is found, it is keratin-filled punctum. i. A lipoma is a benign tumour composed of adipose tis-
iv. The scalp, ears, back, face, and upper arm are common sue. It is the most common form of soft tissue tumour.
sites for sebaceous cysts, though they may occur ii. Lipomas are soft to the touch, usually movable, and are
anywhere on the body except the palms of the hands generally painless.
and soles of the feet. iii. Usually, treatment of a lipoma is not necessary, unless
v. Sebaceous cysts generally do not require medical treat- the tumour becomes painful or restricts movement.
ment. However, if they continue to grow, they may iv. They are usually removed for cosmetic reasons, if they
become unsightly, painful, infected, or all of the above grow very large, or for histopathology to check that
and surgical excision is recommended. they are not a more dangerous type of tumour such as a
Q. 2. Dermoid cyst. liposarcoma.
v. Lipomas are normally removed by simple excision,
Ans. under local anaesthesia.
i. Dermoid cyst is a benign cystic teratoma. Q. 5. Haemangioma.
ii. Histologically, dermoid cysts form due to sequestration
Ans.
of skin along the lines of embryonic closure.
iii. Depending on the location, dermoid cysts may contain i. Haemangioma is a benign self-involuting tumour endo-
substances such as nails and dental cartilage-like, and thelial cells.
bone-like structures. ii. About 30% of haemangiomas are present at birth
iv. Dermoid cysts can occur in the skin and subcutis and (infantile haemangioma). The rest appear in the first
also can be intracranial, intraspinal, or perispinal. Intra- several months of life.
abdominal cysts, such as cystic tumours of the ovary or iii. The appearance depends on location. If they are on the
omentum, occur as well. surface of the skin, they are reminiscent of a ripe straw-
v. Treatment for dermoid cyst is complete surgical berry hence, they are sometimes referred to as “straw-
removal, preferably without spillage of cyst contents. berry haemangiomas”; however, if they are just under
Sophisticated neurosurgical or gynaecologic surgical the skin, they present as a bluish swelling. Sometimes
techniques are often needed to remove the dermoid cyst they grow in internal organs such as the liver or larynx.
and prevent possible complications. In most cases, haemangiomas will disappear over time.
iv. It can present as:
Q. 3. Nasopalatine duct cyst. l A red to reddish-purple, raised sore (lesion) on the

Ans. skin
l A massive, raised tumour with blood vessels.
i. Nasopalatine duct cysts (NPDCs) are developmental, epi-
thelial, non-neoplastic cysts that are considered to be the Most haemangiomas are on the face and neck.
most common (about 70%) of the non-odontogenic cysts. Q. 6. Neurofibroma.
ii. It develops in only a single location, which is the
Ans.
midline anterior maxilla.
iii. Small cysts in the early stages of their development are i. Neurofibroma, is a benign tumour derived from an
frequently (40-87%) asymptomatic and are likely to be admixture of Schwann cells and perineural fibroblast
recognized on routine radiographic examinations dur- proliferations.
ing dental treatment. ii. The oral or pharyngeal neurofibroma is usually diag-
iv. Large cysts can be responsible for a variety of symp- nosed in teenagers and young adults, although all ages
toms, including swelling in the anterior part of the are susceptible.
Section | II General Surgery 299

iii. Neurofibroma is seen either as a solitary lesion or as l Invasion of lymphatics. This is followed by the transport
part of the generalized syndrome of neurofibromatosis of tumour cells to regional nodes and ultimately to other
called von Recklinghausen disease of the skin. parts of the body; it is common in initial spread of car-
iv. The World Health Organization (WHO) has subdi- cinomas.
vided neurofibromas into 2 broad categories: Dermal l Haematogenous spread. This is typical of all sarcomas
and plexiform. Dermal neurofibromas arise from a but it is the favoured route in certain carcinomas (e.g.
single peripheral nerve, while plexiform neurofibromas those originating in kidneys). Because of their thinner
are associated with multiple nerve bundles. walls, veins are more frequently invaded than arteries
v. Other clinicopathologic subtypes include localized neuro- and metastasis follows the pattern of the venous flows.
fibroma (sporadic neurofibroma), diffuse neurofibroma, l Transplantation. Mechanical carriage of fragments of
plexiform neurofibroma, and epithelioid neurofibroma. tumour cells by surgical instruments during operation or
vi. Patients usually present with an uninflamed, slowly the use of needles during diagnostic procedures.
enlarging, asymptomatic lesion which is rarely painful,
Q. 9. Dentigerous cyst.
that varies greatly in size from tiny nodules to large
pendulous masses. Ans.
vii. Solitary oral neurofibromas are usually treated by surgical
i. Dentigerous cyst, also known as follicular odontoma is a
excision, depending on the extent and the site. Surgical
common odontogenic cyst of epithelial origin, which en-
removal may result in recurrence, and multiple recurrences
closes the crown of an impacted tooth at its neck portion.
have been associated with malignant transformation.
ii. The cyst is lined by squamous epithelium, surrounded
Q. 7. Epithelioma (squamous cell carcinoma). by connective tissue. As cyst grows, it displaces the
teeth deeper and deeper and prevent from eruption.
Ans.
iii. Symptoms:
i. Epithelioma is an abnormal growth of the epithelium. Epi- l Cyst remains asymptomatic, if uninfected while on

theliomas can be benign growths or malignant carcinomas. infection inflammatory signs are present.
ii. They are classified according to the specific type of l Expansion of mandible: Since the inner table of

epithelial cells that are affected. The most common mandible is strong the expansion mainly occurs in
epitheliomas are basal cell carcinoma and squamous the outer aspect of the mandible. The bone gets
cell carcinoma (skin cancers). thinned out resulting in egg shell cracking.
iii. Squamous cell carcinoma represents a histologically l Blue domed cyst: When it contains blood then it is

distinct form of cancer which arises from the uncon- called as blue domed cyst.
trolled multiplication of transformed malignant cells iv. Treatment is based on the size of the cyst.
showing squamous differentiation and tissue architecture. l Small cyst: Excision of the cyst by intraoral
iv. Squamous cell carcinoma is one of the most common approach.
cancers, and frequently forms in a large number of l Large cyst: Managed by marsupialisation.
body tissues and organs, including skin, lips, mouth,
Q. 10. Glomus tumour.
oesophagus, urinary bladder, prostate, lung, vagina,
and cervix, among others. Ans
v. Squamous cell carcinoma may be classified into the
i. Glomus jugulare tumours refer to rare, slow-growing,
following types:
hypervascular tumours arising within the jugular fora-
l Adenoid squamous cell carcinoma
men of the temporal bone. They are included in group
l Basaloid squamous cell carcinoma
paragangliomas.
l Clear cell squamous cell carcinoma
ii. They occur predominantly in women in the fifth and
l Keratoacanthoma
sixth decades of life.
l Signet ring cell squamous cell carcinoma
iii. Due to their insidious onset of symptoms, these tumours
l Spindle cell squamous cell carcinoma
often go unnoticed, and delay in diagnosis is frequent.
l Verrucous carcinoma
iv. Glomus tumours are encapsulated and locally invasive
Q. 8. Spread of malignant tumours. tumours.
v. The most common symptoms are conductive hearing
Ans
loss and pulsatile tinnitus. Other aural signs and symp-
Metastasis or spread of malignant tumours occurs by four toms are ear fullness, otorrhoea, haemorrhage, bruit,
routes: and the presence of a middle ear mass.
l Spread into body cavities. This occurs by the seeding surface vi. Histological structure of glomus tumours is a dense
of the peritoneal, pleural, pericardial or subarachnoid spaces. matrix of connective tissue among nerve fascicles.
300 Quick Review Series for BDS 3rd Year

vii. Radiation therapy and surgery can be involved in the in 1885 by the French physician Louis-Charles Malassez
treatment of these tumours. until been finally renamed to the modern name ameloblas-
toma in 1930 by Ivey and Churchill. The resulting tumours
Q. 11. Odontoma.
or cysts are usually not malignant (benign) but the tissue
Ans. growth may be aggressive in the involved area. On occasion,
tissue near the jaws, such as around the sinuses and eye
l Odontoma can be defined as a hamartoma of odonto-
sockets, may become involved as well. The tissues involved
genic origin. They are classified into two types:
are most often those that give rise to the teeth so that amelo-
a. Compound odontoma
blastoma may cause facial distortion. There are three main
b. Complex odontoma
clinical subtypes of ameloblastoma: Unicystic, multicystic,
l A compound odontoma which has three separate dental
peripheral. The peripheral subtype composes 2% of all am-
tissues (enamel, dentin and cementum), but may present
eloblastomas. Of all ameloblastomas in younger patients,
a lobulated appearance without any definitive demarca-
unicystic ameloblastomas represent 6% of the cases. Amelo-
tion of separate tissues between the individual “tooth-
blastomas are often associated with the presence of un-
lets”. It is common in the anterior maxilla.
erupted teeth. Symptoms include painless swelling, facial
l The complex odontoma is unrecognizable as dental tis-
deformity if severe enough, pain if the swelling impinges on
sues, usually presenting as a radio-opaque area with
other structures, loose teeth, ulcers, and periodontal (gum)
varying densities. It is common in the posterior maxilla
disease. Although chemotherapy, radiation therapy, curet-
or in the mandible.
tage and liquid nitrogen have been effective in some cases of
Q. 12. Basal cell carcinoma. ameloblastoma, surgical resection or enucleation remains
the most definitive treatment for this condition.
Ans.
Basal cell carcinoma (BCC) is the most common skin can- Q. 14. Malignant melanoma.
cer in humans, which paradoxically accounts for less than Ans.
0.1% of patient deaths. Basal cell skin carcinomas typically
appear on sun-exposed skin, are slow growing, and rarely Malignant melanoma refers to a neoplasm of melanocytes
metastasize. BCC usually appears as a flat, firm, pale area or of the cells that develop from melanocytes. Melanomas
that is small, raised, pink or red, translucent, shiny, and have 2 growth phases, radial and vertical. During the radial
waxy, and the area may bleed following minor injury. Tu- growth phase, malignant cells grow in a radial fashion in
mour size can vary from a few millimetres to several centi- the epidermis. With time, most melanomas progress to the
metres in diameter. BCC is a non-melanocytic skin cancer vertical growth phase, in which the malignant cells invade
that arises from basal cells. Body distribution of BCCs is as the dermis and develop the ability to metastasize. Melano-
follows: mas are classified as thin if they are 1 mm or less in depth;
l On the head (most frequently on the face; most common moderate if they are 1-4 mm in depth; and thick if they are
location is the nose, specifically the nasal tip and alae) - 70% greater than 4 mm in depth. There are 5 histologic types of
l On the trunk- 25% melanoma:
l On the penis, vulva, or perianal skin - 5% i. Superficial spreading melanomas
ii. Nodular melanomas
Patients present with a shiny, pearly nodule. However, iii. Lentigo maligna melanomas
superficial basal-cell cancer can present as a red patch-like iv. Acral lentiginous melanomas
eczema. Infiltrative or morpheaform basal-cell cancers can v. Mucosal lentiginous melanomas
present as a skin thickening or scar tissue. To diagnose
basal-cell carcinomas, a skin biopsy (shave biopsy) is rec- Wide local excision with sentinel lymph node biopsy and/
ommended. Treatment includes standard surgical excision, or elective lymph node dissection (LND) is considered
Mohs surgery, chemotherapy, immunotherapy, radiation, the mainstay of treatment. Conservative management is
photodynamic therapy, cryosurgery, electrodessication and reserved for adjuvant therapy of patients with advanced
curettage. melanoma.
Q. 13. Ameloblastoma. Q. 15. Premalignant lesions of the skin.
Ans. Ans.
Ameloblastoma is a benign tumour of odontogenic epithe- Premalignant conditions of skin are as follows:
lium (ameloblasts) much more commonly appearing in the l Actinic or solar keratosis: These are common asymp-
lower jaw involving abnormal tissue growth. This type of totic lesions seen mostly on sun-exposed areas of light-
odontogenic neoplasm was referred to as an adamantinoma skinned people.
Section | II General Surgery 301

l Chemical and other keratoses: Skin lesions caused by l Congenital, painless swelling in the floor of mouth.
exposure to arsenic, tar, polycyclic hydrocarbons, infra- l Swelling is soft and cystic. Fluctuation test is positive.
red radiation for a prolonged period at the work place l Transillumination test is negative as it contains thick,
(thermal keratosis) and scar keratosis. cheesy, sebaceous material.
l Large cell acanthoma
l Chondrodermatitis nodular helicis Treatment
l Cutaneous horn: A hard, raised nodule with a reddish
base, usually seen in sun-exposed areas in pale persons. Total excision is the treatment of choice.
l Radiation dermatitis Q. 18. Implantation dermoid.
l Bowen’s disease: A condition located totally within the
Ans.
top layer of the skin called the epidermis, and favours
the sun-exposed areas of the face, neck and extremities. Implantation dermoid is an acquired condition that occurs
l Sebaceous nevi as the result of implantation of epidermal fragments into the
l Porokeratosis: Describes a wide variety of skin disorders. dermis by a penetrating injury. The epidermis continues
l Fibroepithelioma of Pinkus: Appears as flesh-coloured to grow and forms a cyst lined with stratified squamous
skin tags hanging from the back. epithelium and filled with keratin.
l Keratoacanthoma: A rapidly growing, red, raised lesion on
the sun-exposed areas of middle-aged and elderly people. Clinical Features
l Condylomata acuminata: A virus-induced lesion in the
genitals, which appears as sharp-pointed red, small l History of penetrating injury - more common in adults
finger-like projections, either stuck at the base or sus- who are likely to suffer minor hand trauma, e.g.
pended by a stalk. gardener, manual worker.
l Buschke-Lowenstein tumour: A giant variety of condy- l Cyst is smooth, small, spherical swelling in the subcu-
lomata acuminata. taneous tissues.
l Often an implantation dermoid occurs under the scar of
Q. 16. Cystic hygroma. a previous laceration.
Ans. l The cyst may be quite hard.

Cystic hygroma is a cystic lymphatic lesion which can affect Excision of the cyst under local anaesthesia is the
any anatomic site in the human body. Cystic hygroma usually treatment of choice.
affects the head and neck (approximately 75%), with a left- Q. 19. Cavernous haemangioma.
sided predilection. Within the neck, the posterior triangle tends
to be most frequently affected. They tend to predominate be- Ans.
low the mylohyoid muscle and can involve both the anterior Cavernous haemangioma is a vascular disorder character-
and posterior triangles of the neck. The cysts are typically large ized by grossly dilated blood vessels with a single layer of
and thick-walled and have little involvement of surrounding endothelium and an absence of neuronal tissue within the
tissue. The overlying skin can take on a bluish hue or may ap- lesions. This condition is classified as neoplastic or hamar-
pear normal. Cystic hygromas are typically soft, painless, tomatous and manifest as a harmless proliferation of blood
compressible (doughy) masses and typically transilluminates. vessels which form a tumour-like mass of blood-filled
The conservative treatment of CH consists of the administra- spaces which can occur anywhere in the body and is present
tion of sclerosing agents like OK-432, bleomycin, pure etha- at birth or develops soon after.
nol, bleomycin, sodium tetradecyl sulfate, and doxycycline.
Q. 20. Complication of lipoma.
However, the mainstay of treatment is surgical excision.
Ans.
Q. 17. Sublingual dermoid.
Complication of lipoma are as follows:
Ans.
l Bleeding
Sublingual dermoid is a type of congenital sequestration l Pain
dermoid cyst. The cyst is formed by inclusion of the surface l Lipomas in the major airways can cause respiratory
ectoderm at the fusion line of two mandibular arches. distress due to bronchial obstruction.
l Patients with oesophageal lipomas can present with ob-
struction, dysphagia, regurgitation, vomiting, and reflux.
Clinical Features l Cardiac lipomas are mainly subendocardial, mostly un-
l It occurs in young children between the ages of 10 to capsulated, appearing as a yellow mass projecting into
12 years. the cardiac chamber.
302 Quick Review Series for BDS 3rd Year

l Intramediastinal lipomas can impinge on the superior vena absent in poorly differentiated type). Intraepithelial
cava, thereby leading to superior vena cava syndrome. keratin in the shape of a whorl is termed squamous eddy
l Intestinal lipomas can manifest as classic obstruction, or pearl.
intussusception, volvulization, or haemorrhage. l At high power, intercellular bridges are commonly seen.
l Lipomas in intra-articular joint spaces or intraosseous
sites, such as the calcaneus, can lead to joint dysfunc- Q. 23. Adamantinoma.
tion and pain restricting normal movement. Ans.
l Lipomas may also arise in the dural or medullary
components of the spinal cord can result in cord Adamantinoma is a locally aggressive osteolytic tumour. It
compression. is a tumour arising from ameloblasts (enamel-forming
cells).It is is an extremely rare, slow growing, low-grade
Q. 21. Clark’s level of invasion of malignant melanoma. malignant tumour of epithelial origin, that occurs almost
Ans. exclusively in the tibia and the fibula. The diaphyseal re-
gion is the area most commonly affected. Adamantinomas
Clark staging of melanoma invasion is as follows: are classified into 2 distinct types: classic and differenti-
l Level I: All tumour cells above basement membrane ated. Classic adamantinomas usually occur in patients older
(in situ) than 20 years, whereas differentiated adamantinomas occur
l Level II: Tumour extends into papillary dermis almost exclusively in patients younger than 20 years. The
l Level III: Tumour extends to interface between papil- patient usually has swelling that may be painful. The dura-
lary and reticular dermis tion of symptoms can vary from a few weeks to years.
l Level IV: Tumour extends between bundles of collagen
of reticular dermis (extends into reticular dermis)
l Level V: Tumour invasion of subcutaneous tissue Treatment
Q. 22. Microscopic picture of squamous cell carcinoma. l Marginal resection: Delayed local recurrence can be
expected in 32%.
Ans.
l En bloc resection
In the histological slide, a squamous cell carcinoma has the l Local recurrence should not occur.
following features: l Mean survival w/metastatic disease: 12 years.
l Dermal invasion by abnormal cells from the epidermis l Highly radioresistant
l Pleomorphism of the tumour cells l Chemotherapy has not been shown to be effective.
l Presence of keratinization within the cells which l Metastasize to lungs, lymph nodes and abdominal
give the cells abundant pink cytoplasm, (this may be organs by both haematogenous and lymphatic routes.

Topic 7

Diseases of the Oral Cavity (Mouth, Tongue and Lips)

LONG ESSAYS
Q. 1. Discuss the aetiology, clinical features and man- Discuss aetiopathogensis, clinical features and treat-
agement of carcinoma of tongue. ment of carcinoma of anterior two-thirds of tongue.
Or Ans.
Describe the signs, symptoms and management of carci- Carcinoma of tongue accounts for more than half of all oral
noma of posterior third of tongue. carcinomas.
Or
Section | II General Surgery 303

Aetiology x. Palpable lymph nodes in the neck which are hard,


nodular and may get fixed in advanced stages.
l Various premalignant conditions like:
xi. Features of bronchopneumonia.
i. Leukoplakia.
ii. Erythroplakia.
l Six Ss Spread of Carcinoma of Tongue
i. Smoking
i. Local spread
ii. Syphilis
Carcinoma of anterior two-thirds of tongue is limited to the
iii. Spices
half of the tongue and does not extend to the other half
iv. Spirits
across the midline. Carcinoma of posterior one-third of
v. Sharp tooth
tongue spreads to tonsils, soft palate and epiglottis.
vi. Sepsis due to betel nut chewing and tobacco.
ii. Lymphatic spread
Types The lymphatic spread occurs from:
a. Tip of the tongue to submental lymph nodes.
A. Pathological types b. Lateral margins of the tongue to submandibular lymph
i. Non-healing ulcer, commonly on lateral border of nodes, juguloomohyoid.
tongue in 60% of cases, with slough. c. Posterior one-third of tongue to pharyngeal nodes and
ii. A proliferative growth, with everted edge. upper deep cervical lymph nodes.
iii. Frozen tongue or indurated variety.
iv. Fissure variety. iii. Blood spread
It is rare and occurs extremely late.
B. Histological types
i. Squamous cell carcinoma—commonest.
ii. Adenocarcinoma may be from minor salivary glands or
Investigations
mucous glands. l Edge biopsy: Biopsy from posterior third growth should
iii. Melanomas. be done under general anaesthesia.
l FNAC of lymph nodes.
C. According to site of involvement l Indirect and direct laryngoscopy to see posterior third growth.
i. Lateral margin—commonest—47-50%. l CT scan to know the extension of posterior third growth
ii. Posterior third—20%. or to see the status of advanced secondaries.
iii. Dorsum—6.5%. l Chest X-ray to see bronchopneumonia.
iv. Ventral surface—9%.
Treatment
Clinical Features
l Treatment of carcinoma of the tongue is same as that of
i. Elderly persons of both sexes are affected. any other cancer in the oral cavity.
ii. Pain in the tongue due to infection or ulceration l The parameters like, preserving the function of the
or the involvement of lingual nerve. In case of tongue, widespread disease in the posterior one-third
carcinoma of posterior third of tongue, pain occurs on tumours, general health of patient may decide the
swallowing, treatment in favour of radiotherapy.
iii. Excessive salivation. l Various types of treatments for carcinoma of tongue are:
iv. Dysphagia either due to fixed tongue or due to the A. Surgery
involvement of genioglossus or growth in the posterior B. Radiotherapy and
third of the tongue. C. Chemotherapy.
v. Visible ulcer in anterior two-thirds of tongue. Growth
or ulcer in posterior third is usually not visible. A. Surgery
vi. Ankyloglossia Various types of surgeries indicated in case of carcinoma of
vii. Inability to articulate. tongue are as follows:
viii. Foetororis (halitosis): Due to infection and necrosis in
the oral cavity. i. Wide excision:
ix. Change in voice: Occurs in posterior third tumours. l Wide excision with 1 cm margin and a depth of 1 cm is
Tumour in posterior third area is more aggressive. sufficient.
304 Quick Review Series for BDS 3rd Year

l The reconstruction of the tongue is not necessary. B. Radiation


l Usually done in cases of carcinoma in situ. i. Interstitial radiation: Small growth of 1 cm can be
treated with radium needles, radon seeds and radioactive
ii. Partial glossectomy: tantalum wires or iridium wires placed in the growth.
l It is indicated when the lesion is less than 2 cm (T1) and ii. Tele therapy: Cobalt-60 is used when the growth is larger.
confined to the lateral border of the tongue. It is very useful in posterior one-third of tongue where
l About one-third of the anterior two-thirds of the tongue surgery is not practical and the tumours are radiosensitive.
is removed. iii. Radiation is also useful in late stages of the disease,
l The wide excision should include at least 2 cm of tissue either for palliation or to step down the staging of the
away from the palpable indurated edge of the tumour. disease. After that surgery can be undertaken.
iii. Hemiglossectomy: C. Chemotherapy
l It refers to removal of half of the anterior two-thirds of Methotrexate 50 mg/kg/day for 5 days will reduce the size
tongue and is reconstructed. of the tumour. But long-term results are not encouraging.
l This is indicated in a radio-residual tumour, radio-
recurrent tumour or where radiotherapy facilities are Treatment of secondary lymph nodes:
not available. a. Prophylactic block dissection of neck: Even though
l Reconstruction of the tongue can be done by using the lymph nodes are not palpable, prophylactic supra-
PMMC flap. omohyoid block dissection is carried out and prognosis
is better. However, one can wait for the appearance of
iv. Total glossectomy:
lymph nodes and then block dissection is done.
l Very extensive growths involving the entire tongue are b. When the lymph nodes are enlarged along with the
given radiotherapy initially, to reduce the size of the carcinoma of tongue, en block radical neck dissection is
tumour. Surgery can then be undertaken. done at the time of resection of the tongue, floor of the
l Total glossectomy carries significant mortality and mouth and mandible (Commando operation).
morbidity. c. When the glands are fixed and inoperable palliative
radiation and chemotherapy is tried although the glands
v. Commando’s operation:
are resistant. It will have poor prognosis.
l This is indicated when carcinoma of tongue is fixed to
the mandible with infiltration of the floor of the mouth. Mode of a death in carcinoma of tongue:
l Hemiglossectomy with hemimandibulectomy and re- i) Cancer cachexia and starvation.
moval of the floor of the mouth and radical neck dissec- ii) Aspiration and bronchopneumonia
tion is described as Commando’s operation. iii) Haemorrhage from the growth or glands invading a
The choice of above operations can be selected based on size deep artery.
of the lesion and involvement of lymph nodes as follows: iv) Asphyxia due to oedema and compression of larynx.
l Early growth ,2cm in size: Requires wide excision
Prognosis—5 year survival is about 25%.
l Growth .2 cm in size: Requires hemiglossectomy.
l Larger primary tumour: It can be given preoperative Q. 2. Discuss the differential diagnosis of ulcers over
radiotherapy, and then later hemiglossectomy is done. tongue.
l Same side palpable, mobile lymph nodes present: They
Ans.
are removed by radical neck block dissection.
l Bilateral mobile lymph nodes: They are dealt with one Various ulcers considered in differential diagnosis of the
side radical block and other side functional block dissec- ulcers over the tongue are as follows:
tions with essentially retaining internal jugular vein to i. Aphthous ulcer
maintain the cerebral venous blood flow. Or other option ii. Dental ulcer—traumatic
is same side radical neck dissection and the opposite side iii. Chronic non-specific ulcer
suprahyoid block dissectior can be done. iv. Syphilitic ulcer
l Wide excision when growth is in the tip. v. Tuberculous ulcer
l Posterior third growth can be approached by lip split vi. Carcinomatous ulcer
and mandible resection, so as to halt total glossectomy. vii. Herpetic ulcer
l When mandible is involved hemimandibulectomy is done. viii. Glossitis causing ulcers
l The procedure that involves wide excision or hemiglos- ix. Post-pertussis ulcer (whooping cough)
sectomy, hemimandibulectomy and radical neck dissec- l Among the ulcers of tongue, few are painful while

tion together is called as Commando operation. others are painless.


Section | II General Surgery 305

The painful ulcers of the tongue are: l Such ulcers are common on the lateral margin of the
i. Aphthous ulcers tongue and they heal when the tooth is extracted.
ii. Tubercular ulcers
iii. Dental ulcers, etc. iii. Chronic nonspecific ulcer:
l It is a solitary, painless, slightly indurated ulcer.
While the following ulcers are painless:
l Usually located in the anterior part of the tongue.
i. Carcinomatous ulcers
l The patient complains of burning sensation while taking
ii. Gummatous ulcers
spicy food.
iii. Systemic diseases.
l Mouthwashes and cauterization of the ulcer may be suf-
i. Aphthous ulcers: ficient.
l The term aphthous ulcer is applied to three specific iv. Syphilitic or gummatous ulcer:
conditions: l Snail track ulcers are seen in various parts of the mouth
a. Recurrent aphthous minor in 2nd stage of syphilis. A gumma can occur in the
b. Recurrent aphthous major and tertiary stage of syphilis.
c. Herpetiform aphthous ulceration. l A primary chancre may occur in the tongue.
l The primary chancre occur on the tip, in the midline
a. Recurent aphthous minor: while mucous patches occur any where on the tongue.
l Appears in crops between one and many ulcers on a l Gumma is a complication of tertiary syphilis resulting
cyclical basis. in a firm swelling in the midline in the anterior
l They are, multiple, very painful ulcers that can occur at two-thirds of the tongue.
any age group, more common in females at the time of l Ulcer is nontender.
menstruation. These are called as minor aphthous ulcers. l Severe endarteritis obliterans results in the necrosis of
l They are 0.5 cm in diameter, round or oval in shape with gumma giving rise to gummatous ulcer.
yellow floor and a red erythematous margin. l It has punched out edges and wash leather slough on the
l These ulcers are superficial ulcers, they heal in 10-14 days. floor.
l These ulcers are rare nowadays. After the advent of
b. Recurrent aphthous major: penicillin, syphilis has become rare and less violent.
l When they are larger, deeper, painful, they are called as
major aphthous ulcers. v. Tubercular ulcer of tongue:
l They are due to viral infection. l Tongue is not common site for tuberculosis. It affects tip
l They take longer time to heal, and they heal with of the tongue.
scarring. l Young adults with pulmonary tuberculosis may show
multiple shallow ulcers with undermined edges.
Treatment of both major and minor aphthous: l These ulcers are very painful with enlargement of
l Chlorhexidine gluconate mouth wash. regional lymph nodes. There may be matted lymph
l Temporary relief can be obtained by applying choline nodes in the neck.
salicylate gel. l It occurs in patients with fulminating pulmonary
l Vitamin B complex is usually given as a placebo. tuberculosis.
l These ulcers are sometimes multiple with serous
c. Herpetiform aphthous ulcers: discharge.
l They are quite small and occur in crops of many ulcers. l Treatment follows antituberculous line of treatment.
l They are not caused by herpes simplex virus, aetiology Antitubercular treatment will heal these ulcers.
is different.
l Chlorhexidine or tetracycline mouthwashes will lead to vi. Systemic diseases:
rapid healing. Ulcers may be found on the tongue in certain systemic
diseases like:
ii. Dental ulcer: l Pemphigus
l Dental ulcer is an example for traumatic ulcer. l Systemic lupus erythematosus (SLE)
l These ulcers occur due to broken tooth, sharp tooth, l Lichen planus
ill-fitting dentures, prosthesis, etc.
l They are very painful ulcers. vii. Herpetic ulcer:
l They are firm and tender and cervical lymph nodes may l Herpetic ulcer is due to herpetic infection of the lingual
enlarged. nerve.
306 Quick Review Series for BDS 3rd Year

l Acute neuralgic pain usually precedes the appearance of the cheek which gradually increases in size in a
the vesicles gradually. patient with history of chewing pan or smoking.
l The vesicles may rupture into multiple small painful ii. Edges are everted with induration at the base as well
ulcers. as at the edge. Everted edge, induration are the typical
l Herpes zoster can be treated with antiviral drugs. features of the ulcer and is a diagnostic feature of
a squamous cell carcinoma. Possibly, it is a host
viii. Glossitis causing ulcers: reaction indicating a good immunity.
l Chronic superficial glossitis is seen in smokers (smoker’s iii. Ulcer bleeds on touch.
ulcer). iv. Pain occurs when it involves the skin, bone or if
l The ulcers are superficial multiple with hyperaemia. secondarily infected. Due to secondary infection, most
l They are painful. of the oral cancers are tender to touch.
v. Fixity to the underlying structures such as mandible
ix. Post-pertussis ulcer (whooping cough): may be present.
l It occurs in children due to repeated coughing. vi. Involvement of retromolar trigone indicates that it
l Typical location of the ulcer on the under surface of the is an advanced disease, as the lymphatics here
tongue and on the frenulum clinches the diagnosis. communicate freely with the pharyngeal lymphatics.
vii. Trismus and dysphagia signifies involvement of
Q. 3. Describe aetiology, clinical features diagnosis and pterygoids, or posterior extension.
management of carcinoma cheek. viii. Halitosis is very characteristic.
Or ix. Submandibular lymph nodes and upper deep cervical
lymph nodes are involved which are hard, nodular,
Describe the aetiology, pathology of carcinoma of cheek initially mobile but later get, fixed to each other and
and how will you manage if it is involved the mandible? then to deeper structure.
Ans.
l Squamous cell carcinoma is the most common type of Investigations
carcinoma of the cheek. i. Wedge biopsy from the edge of the lesion. Edge biopsy
l Occasionally, it can be adenocarcinoma arising from the usually taken from two sites. Malignant squamous cells
minor salivary glands or rarely it can also be melanoma. with epithelial pearls (keratin pearls) are the histological
l Carcinoma of the cheek is very common in India due to features.
the habit of keeping the tobacco quid in the cheek pouch. ii. OPG of the mandible to rule out extension.
iii. FNAC from lymph nodes.
iv. CT scans—to assess the extension of tumour and its
Aetiology secondaries.
l All ‘S’—smoking, spirit, syphilis, sharp tooth and
spices.
l Premalignant conditions:
Treatment
a. Leukoplakia. Treatment of carcinoma of cheek may be curative or palliative,
b. Erythroplakia. Treatment strategies for carcinoma of cheek are as follows:
c. Hyperplastic candidiasis a. Surgery: Wide excision, hemimandibulectomy, neck
d. Submucosal fibrosis. lymph nodes block dissection.
l Betel nut chewing (pan with pan quid kept in the cheek) b. Radiotherapy: Curative or palliative; external or
brachytherapy.
c. Chemotherapy: Intra-arterial, IV or orally.
Pathological Types
i. A non-healing ulcer. a. Surgery
ii. An exophytic growth, or a proliferative growth, i.e. i. A small superficial ulcer (T1, T2) is treated by wide
verrucous carcinoma excision followed by split skin graft (SSG).
iii. An infiltrative lesion slowly involves the adjacent structures. ii. Early growth without bone involvement:
l Curative radiotherapy using Caesium137 needles or

Iridium192 wires, i.e. brachytherapy.


Clinical Features l As it is a squamous cell carcinoma, primary is

i. A non-healing ulcer or cauliflower-like growth, i.e. radiosensitive.


verrucous carcinoma is an exophytic growth. Ulcer in l Other option is wide excision with 3 cm clearance.
Section | II General Surgery 307

l Presently advanced technology in radiotherapy, Q. 4. Surgical anatomy of maxillary sinus, clinical


facilitates the use of external radiotherapy also. features and treatment of carcinoma of maxillary sinus.
The incidence of dreaded complication like osteora-
Ans.
dionecrosis of mandible has reduced due to better
RT methods. l The maxillary sinus is the largest of all paranasal
iii. A small infiltrative ulcer is treated by wide excision sinuses, located in the body of maxilla.
followed by a flap reconstruction
The features of maxillary sinus:
The forehead flap in males and deltopectoral flap in i. The maxillary sinus is the first paranasal sinus to
females are now replaced by pectoralis major myocutane- develop.
ous flap (PMMC flap). ii. The maxillary sinus is pyramidal in shape, with its:
l Base: Directed medially towards the lateral wall of
b. Radiotherapy the nose.
l The early lesions can be managed with radiotherapy. l Apex: Directed laterally towards zygomatic pro-
l Indications for radiotherapy are as follows: cess of the maxilla.
i. T1 and T2 lesions l Roof: Is formed by the floor of the orbit.
ii. Patient who are not fit for surgery l Floor: Formed by the alveolar process of the maxilla.
iii. Lesions near the commisure The floor lies about 1cm below the level of the floor of
l Two types of radiotherapy used are: the nose.
a. External radiotherapy iii. The size of the sinus is variable, average measure-
b. Interstitial radiotherapy ments are: Height: 3.5 cm, width: 2.5 cm and antero-
l In external radiotherapy large total dose of 6000-8000 posterior depth is 3.5 cm.
cGy units are given; 200 cGy units/day. iv. It opens into the middle meatus of the nose in the lower
l Interstitial radiotherapy is indicated in infiltrative small part of the hiatus semilunaris, a second opening is of-
lesions. Caesium 137 or iridium wires are placed within ten present at the posterior end of the hiatus. Both
the tumour. Minimal tissue reaction is the advantage of openings are near to the roof than the floor of the sinus.
this method. v. The size of the opening is 3 – 4 mm as it is overlapped
by the following:
c. Chemotherapy (a) From above, by the uncinate process of the eth-
l Drugs used are methotrexate, vincristine, bleomycin, moid, and the descending part of the lacrimal bone;
and adriamycin. (b) From below, by the inferior nasal concha; and
l Often it is given intra-arterially through external carotid (c) From behind, by the perpendicular plate of the
artery using arterial pump or by increasing the height of palatine bone.
the drip more than 13ft so as to attain a pressure more It is still further reduced in size by the thick mucosa of
than systolic pressure. the nose.
l Chemotherapy can also be given IV or orally. vi. Arterial supply: Facial, infraorbital and greater
palatine arteries.
In advanced carcinoma of the cheek: vii. Venous drainage: The maxillary sinus drains into the
Surgery: T3 and T4 lesions require surgery as the main facial vein and the pterygoid plexus of veins.
modality of the treatment followed by postoperative radio- viii. Lymphatic drainage: Into the submandibular nodes.
therapy. Most of the lesions require full thickness resection ix. Nerve supply: Infraorbital, and anterior, middle and poste-
leaving behind large defects. Such defects can be repaired rior superior alveolar nerves innervate the maxillary sinus.
by myocutaneous flap.
Reconstruction after surgery can be done by: Clinical Features of Carcinoma of Maxillary
Split skin graft.
Sinus
l

l Deltopectoral cutaneous flap—based on 2nd perforator


of internal mammary artery. l Histologically, two varieties of carcinoma of maxillary
l Forehead flap—based on anterior branch of superficial sinus are identified:
temporal artery. a. Columnar cell carcinoma of maxillary antrum.
l Pectoralis major myocutaneous flap—based on thora- b. Squamous cell carcinoma of the epithelium of hard
coacromial artery. palate and gum.
l Mandible reconstruction by cortical bone graft or rib, l Usually elderly people are affected.

fibula or synthetic material. l Occurs with equal distribution in both sexes.

l Free flaps (microvascular) l Foul smelling blood-stained nasal discharge.


308 Quick Review Series for BDS 3rd Year

l Toothache and a bulge in the hard palate, if the Treatment


floor is involved.
l Radiotherapy is the main modality of treatment in car-
l Swelling on the maxillary area, if it bulges on the
cinoma of maxillary sinus. Curative rate is around 70%
anterolateral wall.
in early cases. In advanced cases, radiotherapy is given
l Nasal obstruction and epiphora, if medial wall
first followed by maxillectomy.
enlarges.
l Surgical treatment is maxillectomy. Complete maxillary
l Lymph node metastasis is very common.
resection will leave behind a badly deformed face. But
nowadays sophisticated prosthesis is available which
Investigations will bring the disfigured face back to normal. So,
l CT scan surgery is acceptable.
l Sinoscopy l Radiotherapy and chemotherapy are available for
l Biopsy will confirm the diagnosis. palliation and recurrence.

SHORT ESSAYS
Q. 1. Describe the lymphatic drainage of tongue. Q. 2. Glossitis.
Ans. Ans.
Glossitis is the inflammation of the tongue.
LYMPHATIC DRAINAGE OF THE TONGUE
(Fig 7.1) Aetiology
Anterior 2/3rd of tongue Posterior 1/3rd of tongue
i. Smoking
ii. Syphilis
iii. Sepsis
iv. Spiritis
Submental nodes
v. Spices
vi. Sharp tooth
Jugulo-omohyoid lymph node
Submandibular nodes
Omohyoid muscle
Clinical Features
Internal jugular vein Glosssitis passes through 5 stages:
Lymphatic drainage of tongue. a. Hypertrophic papillae
FIGURE 7.1 Lymphatic drainage of tongue b. Hyperkeratosis—surface is dried up[leukoplakia]
c. Raw red tongue
i. The tip of the tongue drains bilaterally to the submental d. Fissured red tongue
lymph nodes. Lymphatics in the midline of tongue e. Carcinoma in situ
freely cross communicate with each other and so spread
of malignancy can occur to bilateral neck Iymph nodes.
ii. The right and left halves of the remaining part of the Treatment
anterior two-thirds of the tongue drain unilaterally to
i. Remove source of irritation, e.g. sharp teeth, ill-fitting
the submandibular nodes.
dentures.
iii. The posterior one-third of the tongue drains bilaterally to
ii. Stop pipe smoking.
the jugulo-omohyoid nodes, these are known as the lymph
iii. Anti-syphilitic treatment when VDRL test is positive.
nodes of the tongue. Lateral margin drains to submandibular
iv. Mouthwash with carbolic acid 1:1000 acts as an anaes-
lymph nodes and then into upper deep cervical lymph nodes.
thetic as well as an antiseptic.
iv. Many lymphatic vessels pass as subperiosteal lymphat-
v. Smear white petroleum jelly over the tongue before
ics of mandible. So carcinoma can involve the bone
sleeping.
through this route.
v. Lymphatic vessels of tongue are named as follows: Q. 3. Erythroplakia.
l Apical vessels.
Ans.
l Central vessels.

l Marginal vessels. Erythroplakia is a premalignant lesion. The premalignant


l Basal vessels. lesions are defined as morphologically altered tissue in
Section | II General Surgery 309

which cancer is more likely to occur then in its apparently iii. Giant cell epulis
normal counter part, e.g. leukoplakia, erythroplakia, iv. Carcinomatous epulis
nicotianapalati, stomatitis, etc.
i. Fibrous Epulis
This is the commonest form.
ERYTHROPLAKIA l

l It arises from the periosteum of the jaw.


l It is a red patch that cannot be clinically or pathologi- l It grows, separates the teeth and slowly loosens them.
cally diagnosed as any other condition and is relatively l It is a fibroma containing fusiform cells and blood vessels.
rare compared to leukoplakia. l It is a firm non-tender nodule.
l Erythroplakia is almost associated with premalignant l It may become malignant fibrosarcoma; then it grows
changes histologically and is, therefore, a most impor- rapidly and often bleeds.
tant precancerous lesion. l Complete excision along with the wedge of the bone
with its root. Incomplete excision invites recurrence.
Aetiology ii. Granulomatous Epulis
i. Unknown l It is not a true tumour but a granulation tissue at a
ii. Smoking and alchohol abuse are important aetiological carious tooth or due to an ill-fitting denture.
factors. l It is a soft to firm mass of granulation tissue, bleeds on
touch.
Clinical Features l Pregnancy epulis (gingivitis gravidarum) refers to this
variety.
l No apparent sex predilection.
l Most cases occurred in sixth and seventh decades of life.
l Site: Floor of mouth, tongue, retromolar area, soft Treatment
palate, mandibular mucosa and sulcus. Treat the cause like caries tooth should be removed, denture
l Appears clinically well demarcated, erythematous, needs to be trimmed to fit properly, scraping of granulation
macule/papule with a soft velvety tissue. tissue with diathermy.
l Usually asymptomatic
l Classification: iii. Giant Cell Epulis
i. Homogenous l It is also called myeloid epulis.
ii. Erythroplakia interspersed with patches of leukoplakia l This is a bone tumour - osteoclastoma arising from un-
iii. Granular or speckled erythroplakia derlying bone.
l Histologically, it contains giant cells and spindle cells.
Histopathology l It expands the bone under cover of mucoperiosteum
which becomes vascular and plum coloured.
l Multicentric in origin and severe epithelial dysplasia l It grows rapidly loosening the adjoining teeth.
is seen. l X-ray shows ‘soap bubble’appearance of osteoclas-
l The epithelium shows lack of keratin production and is toma.
often atrophic but it may be hyperplastic.
l Lack of keratinization and thining of epithelium allows Treatment:
underlying vasculature to be seen causing red colour. l Small tumours are treated by curettage.
l Connective tissue rete pegs are high into the epithelium. l Large tumours require radical excision.
l The underlying connective tissue often demonstrates
chronic inflammation. iv. Carcinomatous Epulis
l This is an epithelioma of the gums arising from mucous
Treatment membrane of the alveolar margin.
l It presents as a non-healing, painless ulcer, which
l Remove irritating agents or any underlying cause.
slowly infiltrates the bone.
l Prompt biopsy is mandatory.
l It starts like a small ulcerated nodule and infiltrates deep
l Recurrence rate of less than 5% is reported.
into the bone and fungates to become painful.
Q. 4. Epulis. l Regional lymph nodes are hard and enlarged due to
metastasis.
Ans.
l Histologically, it is a squamous cell carcinoma.
i. Fibrous epulis l Treated by wide excision, which include removal of
ii. Granulomatous epulis segment of the bone.
310 Quick Review Series for BDS 3rd Year

l Radiotherapy may be considered to prevent recurrence Q. 6. Carcinoma of lip.


and also in inoperable cases for palliation.
Ans.
Q. 5. Leukoplakia. l Carcinoma of lip is more common in men (90%).
Ans. Common in lower lip (90%).
l Agriculturists who are commonly exposed to sunlight
Leukoplakia appears clinically as a white patch in the get carcinoma lip more often and so it is called as
mouth and cannot be scraped off. It is irreversible and not countryman’s lip.
attributable to any known diseases. It is important to biopsy l Usually it is a well-differentiated squamous cell carcinoma.
leukoplakic portion to rule out malignancy.

Aetiology
Aetiology
l Ultraviolet rays
The causes for leukoplakia are as follows: l Smoking
l Smoking results in hyperkeratosis. Nicotine in the l Leukoplakia
form of cigarettes, chewed tobacco, powdered snuff l Khaini chewers (tobacco1 lime).
produces premalignant changes in the oral cavity.
l Spices
l Spirits have synergistic action with smoking Clinical Features of Carcinoma of Lip
l Sharp tooth, sepsis, poor oral hygiene l Initially it starts as a red, granular dry lesion, which
l Sunlight—actinic rays eventually gets ulcerated and forms an ulceroprolifera-
l Syphilis tive lesion.
l Susceptibility of a person l Commonly presents as non-healing progressive ulcer,
l Superficial glossitis. painless to begin with.
l Chronic hypertrophic candidiasis (long-standing candida l Ulcer exhibits everted edge with indurations.
infection) l Growth moves with the lip.
l Betelnut, and slaked lime with betel leaf and tobacco l Submental, submandibular and upper deep neck nodes
(Pan) are involved.
l Fungation, bleeding.
Types of Leukoplakia
a. Homogenous Diagnosis
b. Nodular Edge biopsy, FNAC of lymph nodes.
c. Speckled
Treatment
Stages in the Development of Leukoplakia Surgery and radiotherapy are the two modalities of treatment
i. Keratosis appears as milky blush on the surface. available for treatment of carcinoma of lip.
ii. Acanthosis refers to elongation of rete pegs. This
appears as a smooth, white, dry patch. Surgery
iii. Dyskeratosis means the formation of keratin cell layer l If lesion is less than 2 cm, then curative radiotherapy is
in the deeper aspect of epidermis, before they reach the given, either brachytherapy or external beam radiother-
surface. apy. It gives a good prognosis.
iv. Speckled leukoplakia appears as multiple, small white l If tumour is more than 2 cm, wide excision is done.
patches. l Excision up to one-third of lower lip can be sutured
v. Carcinoma in situ. primarily in layers keeping vermillion border in proper
apposition without causing any microstomia.
Excision of more than one-third of the lip requires re-
Treatment l

construction using different flaps like Estlander’s rotat-


i. Pan chewing and smoking has to be formed. ing flap (modified lip-switch flap) or Fries’ modified
ii. Excision, if required skin grafting has to be done. Bernard facial flap or Abbe lip-switch flap.
iii. Even though leukoplakia is irreversible, isoretinoin l Lymph nodes are dealt by radical neck dissection on one
(13-cis-retinoic acid) can reverse some cases of leuko- side and functional bloc or suprahyoid block dissection
plakia possibly. on other side.
Section | II General Surgery 311

Radiotherapy the tumour. Minimal tissue reaction is the advantage of


l It is indicated in all stages of carcinoma of lip. this method.
l Elderly patients with carcinoma of lip and fixed nodes, c. Chemotherapy:
who are not fit for surgery are treated by irradiation.
l Commissure involvement is treated with radiotherapy l Drugs used are methotrexate, vincristine, bleomycin,
than surgery. adriamycin.
l Often it is given intra-arterially through external carotid
artery using arterial pump or by increasing the height of
Prognosis the drip more than 13ft so as to attain a pressure more
Good. 5-year survival is 70%. than systolic pressure.
l Chemotherapy can also be given IV or orally.
Q. 7. Treatment of carcinoma of cheek.
Ans. In Advanced Carcinoma of the Cheek
Treatment of carcinoma of cheek may be curative or palliative, Surgery: T3 and T4 lesions require surgery as the main mo-
Treatment strategies for carcinoma of cheek are as follows: dality of the treatment followed by postoperative radio-
a. Surgery: Wide excision, hemimandibulectomy, neck therapy. Most of the lesions require full thickness resection
lymph nodes block dissection. leaving behind large defects. Such defects can be repaired
b. Radiotherapy: Curative or palliative; external or brachy- by myocutaneous flap.
therapy. Reconstruction after surgery can be done by:
c. Chemotherapy: Intra-arterial, IV or orally. l Split skin graft.
l Deltopectoral cutaneous flap—based on 2nd perforator
a. Surgery: of internal mammary artery.
i. A small superficial ulcer (T1, T2) is treated by wide l Forehead flap—based on anterior branch of superficial
excision followed by split skin graft (SSG). temporal artery.
ii. Early growth without bone involvement: l Pectoralis major myocutaneous flap—based on thora-
l Curative radiotherapy using Caesium137 needles or coacromial artery.
Iridium192 wires, i.e. brachytherapy. l Mandible reconstruction by cortical bone graft or rib,
l As it is a squamous cell carcinoma, primary is fibula or synthetic material.
radiosensitive. l Free flaps (microvascular)
l Other option is wide excision with 3 cm clearance.
Q. 8. Ulcers of tongue.
l Presently advanced technology in radiotherapy, facili-

tates the use of external radiotherapy also. The inci- Ans.


dence of dreaded complication like osteoradionecrosis
l Various ulcers of the tongue are as follows:
of mandible has reduced due to better RT methods.
i. Aphthous ulcer
iii. A small infiltrative ulcer is treated by wide excision
ii. Dental ulcer—traumatic
followed by a flap reconstruction.
iii. Chronic nonspecific ulcer
The forehead flap in males and deltopectoral flap in iv. Syphilitic ulcer
females are now replaced by pectoralis major myocutane- v. Tuberculous ulcer
ous flap (PMMC flap). vi. Carcinomatous ulcer
vii. Herpetic ulcer
b. Radiotherapy: viii. Glossitis causing ulcers
l The early lesions can be managed with radiotherapy. ix. Post-pertussis ulcer (whooping cough)
l Indications for radiotherapy are as follows: l Small among the ulcers of tongue, few are painful while
i. T1 and T2 lesions others are painless.
ii. Patient who are not fit for surgery
The painful ulcers of the tongue, are:
iii. Lesions near the commisure
i. Aphthous ulcers
l Two types of radiotherapy used are:
ii. Tubercular ulcers
a. External radiotherapy
iii. Dental ulcers, etc.
b. Interstitial radiotherapy
l In external radiotherapy, large total dose of 6000-8000 While the following ulcers are painless:
cGy units are given; 200 cGy units/day. i. Carcinomatous ulcers
l Interstitial radiotherapy is indicated in infiltrative small ii. Gummatous ulcers
lesions. Caesium 137 or iridium wires are placed within iii. Systemic diseases
312 Quick Review Series for BDS 3rd Year

Aphthous Ulcers Carcinomatous Ulcer


l They are, multiple, very painful ulcers that can occur at l It is usually seen on the lateral margin of tongue and
any age group, more common in females at the time of bleeds on touch.
menstruation. These are called as minor aphthous ulcers. l Non-healing ulcer with everted edge, edge and base are
l When they are larger, deeper, painful, they are called as indurated.
major aphthous ulcers. l Fixity to the underlying structures.
l They are due to viral infection. These ulcers are super- l Significant lymph nodes are present in the neck.
ficial ulcers with erythematous margin.
Q. 9. Discuss in brief the premalignant lesions of oral
l They subside within a few days.
cavity.
l Temporary relief can be obtained by applying salicylate gel.
l Vitamin B complex is usually given as a placebo. Ans.
The premalignant lesions are defined as morphologically
Dental Ulcer altered tissue in which cancer is more likely to occur then
l These ulcers occur due to broken tooth, sharp tooth, ill- in its apparently normal counter part. For example,
fitting dentures, prosthesis, etc. They are very painful l Leukoplakia
ulcers. l Erythroplakia
l Such ulcers are common on the lateral margin and they l Nicotianapalati
heal when the tooth is removed. This is an example for l Stomatitis
traumatic ulcer. l Dyskeratosis congenitis
The more common ones are leukoplakia and erythroplakia,
Tubercular Ulcer of Tongue
l Tuberculosis affects tip of the tongue. These ulcers are Leukoplakia
very painful with enlargement of regional lymph nodes.
l It is a keratotic white lesion on oral mucosa that cannot
l It occurs in patients with fulminating pulmonary tuber-
be characterised clinically or histologically as any other
culosis.
disease.
l Ulcers have undermined edges. These ulcers are some-
l It is the most common potentially malignant lesion of
times multiple with serous discharge.
the oral mucosa.
l The common predisposing factors of leukoplakia are:
Syphilitic or Gummatous Ulcer Smoking, sepsis, superficial glossitis, syphilis, sharp
l Gumma is a complication of tertiary syphilis resulting in a tooth, chronic hypertrophic candidiasis, pan chewing,
firm swelling in the midline in the anterior two-thirds of using areca, tobacco, slaked lime.
the tongue. l Clinically, the lesion appears as white or greyish co-
l Ulcer is non-tender. loured well-localized patch in the cheek, tongue, palate
l Severe endarteritis obliterans results in the necrosis of or other areas of oral cavity.
gumma giving rise to gummatous ulcer. l Treatment:
l Pan chewing and smoking has to be stopped.
l It has punched out edges and wash leather slough on the
l Excision of the lesion and if required skin grafting
floor. These ulcers are rare nowadays.
has to be done.
l Regular follow up is necessary.
Systemic Diseases
Ulcers may be found on the tongue in certain systemic dis- Erythroplakia
eases like:
l Pemphigus l It is a red velvety appearance of the oral mucosa which
l Systemic lupus erythematosus (SLE) cannot characterize any recognized condition.
l Lichen planus l It is 17 to 20 times more potentially malignant than
leukoplakia.
l Diagnosis is by biopsy.
Post-pertussis Ulcer l Treatment—biopsy and surgical excision.
l It occurs in children due to repeated coughing. Q. 10. Describe the anatomy of maxillary antrum.
l Typical location of the ulcer on the under surface of the
tongue and on the frenulum clinches the diagnosis. Ans.
Section | II General Surgery 313

l The maxillary sinus is the largest of all paranasal often present at the posterior end of the hiatus. Both
sinuses, located in the body of maxilla. openings are near to the roof than the floor of the sinus.
v. The size of the opening is 3 – 4 mm as it is overlapped
The features of maxillary sinus:
by the following:
i. The maxillary sinus is the first paranasal sinus to
(a) From above, by the uncinate process of the
develop.
ethmoid, and the descending part of the lacrimal
ii. The maxillary sinus is pyramidal in shape, with its:
bone;
l Base—directed medially towards the lateral wall of
(b) From below, by the inferior nasal concha; and
the nose.
(c) From behind, by the perpendicular plate of the
l Apex—directed laterally towards zygomatic pro-
palatine bone.
cess of the maxilla.
It is still further reduced in size by the thick mucosa of
l Roof—is formed by the floor of the orbit.
the nose.
l Floor—formed by the alveolar process of the
vi. Arterial supply: Facial, infraorbital and greater pala-
maxilla.
tine arteries.
The floor lies about 1cm below the level of the floor of
vii. Venous drainage: The maxillary sinus drains into the
the nose.
facial vein and the pterygoid plexus of veins.
iii. The size of the sinus is variable, average measure-
viii. Lymphatic drainage: Into the submandibular nodes.
ments are: Height: 3.5 cm, width: 2.5 cm and antero-
ix. Nerve supply: Infraorbital, and anterior, middle
posterior depth is 3.5 cm.
and posterior superior alveolar nerves innervate the
iv. It opens into the middle meatus of the nose in the lower
maxillary sinus.
part of the hiatus semilunaris, a second opening is

SHORT NOTES
Q. 1. Angular stomatitis. l Buccal mucosa and commissural areas are most
frequently affected sites followed by alveolar ridge,
Ans. tongue, lip, hard and soft palate, etc.
l Angular stomatitis is caused due to deficiency of riboflavin. l Oral leukoplakia often presents as solitary or multiple
l Paleness of lips, especially at angles of mouth, followed white patches.
by cheilosis is the earliest sign of deficiency disease. l The surface of lesion may be smooth or finely wrinkled
l In mild deficiency state, glossitis begins with soreness or even rough on palpation and lesion cannot be
of the tip and lateral margins of tongue. removed by scrapping.
l The filiform papillae become atrophic, while fungiform l In most of the cases, these lesions are asymptomatic,
papillae remain normal or become engorged and mush- however, in some cases they may cause pain, feeling of
room-shaped, giving the tongue surface a reddened, thickness and burning sensation.
coarsely granular appearance.
In many cases, tongue becomes magenta coloured.
l
Treatment Modalities
Q. 2. Leukoplakia. l Surgical excision, cryosurgery, CO2-laser surgery, reti-
Ans. noids and other drugs
l Photodynamic therapy.
Leukoplakia is a keratotic white lesion on oral mucosa that
cannot be characterized clinically or histologically as any Q. 3. Tongue tie.
other disease. Ans.
i. Tongue tie is a developmental anomaly.
Aetiology ii. Lingual frenum is short, thicker and fibrous.
Tobacco, alcohol, candidiasis, dietary deficiency, syphilis, iii. It holds the tip of the tongue close to the lower central
trauma-like ill-fitting dentures, sharp dentures, tooth brush incisors.
trauma, galvanism, idiopathic, UV radiation, etc. iv. It results in ankyloglossia, i.e. inability to protrude the
tongue.

Clinical Features Q. 4. Cancrumoris.


l It occurs in 4th, 5th, 6th and 7th decades of life. Ans.
314 Quick Review Series for BDS 3rd Year

i. Cancrumoris is always an extension of an acute ulcer- l Malignant ulcers forms a crust.


ative gingivitis (i.e. Vincent’s disease) into the adjacent l Growth moves with lip.
soft tissues. iv. Diagnosis: Edge biopsy, FNAC of lymph nodes.
ii. The bone exposed by the necrosis of the soft tissues v. Treatment:
usually sequestrates in about 3 weeks. l Early cases: Radiotherapy (either brachytherapy or

iii. After removal of loose bone, and following the external beam radiotherapy) gives good cure.
antibiotic therapy, considerable scarring occurs which l Late cases: Surgery (radical resection with plastic

produces facial asymmetry and trismus. repair of the lip).


Q. 5. Lingual thyroid. Q. 8. Impacted teeth.
Ans. Ans.
i. Lingual thyroid is a round reddish swelling seen at the i. Impacted teeth are those, which are prevented from
back of the tongue, at the foramen caecum. eruption by some physical barier, in eruptive path.
ii. Thyroid develops from the thyroglossal bud which ii. Physical barrier may be formed due to:
starts from the foramen caecum and descends into a) Premature loss of deciduous teeth causing crowding
the neck. The descent can be arrested at any stage. If it of dental arches.
stops at the foramen caecum, it develops into lingual b) Rotation of tooth bud.
thyroid. iii. Incidence: The mandibular molars are most frequently
iii. It should be remembered that this is the only excisting impacted, then maxillary third molars, maxillary cus-
thyroid tissue. In case it has to be removed, it should be pid, premolars and supernumerary teeth comes in the
reimplanted elsewhere. sequence.
iv. It may cause: Haemorrhage, respiratory obstruction, iv. Classification: Impacted third molar
dysphagia and speech impairment. v. Multiple impacted supernumerary teeth present in
v. Diagnosis: “Gardner’s syndrome”.
l Radioisotope study shows iodine uptake by lingual vi. Treatment: These impacted teeth should be
thyroid and the status of thyroid in normal location. removed surgically, if creating pain or causing any
l Ultrasound neck–to see the absence of normal thyroid. complications.
vi. Treatment:
Q. 9. Pre-malignant conditions of the tongue.
l L-thyroxine orally can reduce the swelling.

l Surgical excision. Ans.


l Radioisotope therapy for ablation.
Premalignant conditions of the tongue are as follows:
Q. 6. Name various ulcers of tongue. i. Leukoplakia
ii. Erythroplakia
Ans.
iii. Submucous fibrosis
The various ulcers of the tongue are: iv. Chronic hyperplastic candidiasis.
i. Dental ulcers v. Syphilitic glossitis.
ii. Aphthous ulcers
Q. 10. Signs and symptoms of carcinoma of tongue.
iii. Ulcers in lichenplanus
iv. Syphylitic ulcers Ans.
v. Tuberculous ulcers
vi. Malignant ulcers. The various signs and symptoms of tongue are as
follows:
Q. 7. Carcinoma of lip. i. Pain in the tongue due to infection or ulceration or the
involvement of lingual nerve.
Ans.
ii. Dysphagia - difficulty in swallowing,
i. The carcinoma of lip is usually squamous cell type of iii. Excessive salivation.
carcinoma. It is also known as country man’s lip. iv. Visible ulcer in anterior two-thirds of tongue.
ii. Aetiology: v. Ankyloglossia
l UV rays, smoking, leukoplakia, Khaini chewers. vi. Inability to articulate.
iii. Clinical features: vii. Foetororis (halitosis)
l It is more common in men (90%) and especially viii. Change in voice: Occurs in posterior third tumours.
involves lower lip (90%). ix. Palpable lymph nodes in the neck which are hard,
l Non-healing progressive painless ulcer. nodular and fixed in advanced stages.
l Edge is everted and indurated. x. Features of bronchopneumonia.
Section | II General Surgery 315

Q. 11. Macroglossia. l It clinically appears as retrusion of chin. Steep man-


dibular angle. Deficient chin button.
Ans.
i. Macroglossia is a developmental disturbances of tongue. Treatment
ii. It is also called as tongue hypertrophy; enlarged tongue. l Sliding ostectomy.
iii. Types are:
Q. 14. Ankyloglossia.
a. True macroglossia
b. Psuedomacroglossia Or
iv. Clinical features: Severe retrognathia, unusually small
Tongue tie.
maxilla or mandibular size,
v. It may be associated with Down syndrome, Beckwith- Ans.
Wiedemann syndrome.
i. Ankyloglossia is also known as tongue tie.
vi. Treatment—by surgical intervention.
ii. It occurs due to fusion between tongue and floor of
Q. 12. Predisposing factors—carcinoma of tongue. mouth.
iii. It may be complete or partial.
Ans.
iv. Partial ankyloglossia or tongue tie is due to short lin-
Various predisposing factors of carcinoma of tongue are: gual frenum.
a. Leukoplakia. v. It causes difficulty in speech.
b. Erythroplakia. vi. Treatment: Surgical clipping of lingual frenum.
c. Six Ss
Q. 15. Lymphatic drainage of tongue.
i. Smoking
ii. Syphilis Ans.
iii. Spices
The lymphatic drainage of tongue is as follows:
iv. Spirits
i. The tip of the tongue drains bilaterally to the submental
v. Sharp tooth
lymph nodes.
vi. Sepsis due to betel nut chewing and tobacco.
ii. The right and left halves of the remaining part of the
Q. 13. Micrognathia or micrognathism. anterior two-thirds of the tongue drain unilaterally to
the submandibular nodes. A few central lymphatics
Ans.
drain bilaterally to the same nodes.
l Micrognathia is characterized by smaller jaw either iii. The posterior one-third of the tongue drains bilaterally
maxilla or mandible. to the jugulo-omohyoid nodes, these are known as the
i. True micrognathia: Jaw is truly smaller with normal lymph nodes of the tongue.
relation in between the two jaws.
Lateral margin drains to submandibular lymph nodes
ii. Apparent micrognathia: Jaw is not smaller but
and then into upper deep cervical lymph nodes.
appears smaller due to abnormal relation between
the two jaws. Q. 16. Dysplasia.
Ans.
True Micrognathia l Dysplasia is a histologic connotation to premalignancy
It is of following types: marked by abbarent and uncoordinated cellular prolif-
(a) Congenital: It is found along with Pierre Robins eration depicted at the cellular level as atypia which is
syndrome and congenital heart disease. reflected at tissue level as dysplasia.
(b) Acquired: This occurs after birth, e.g. ankylosis of TMJ l Mild, moderate or severe dysplasia presents as:
and infection or trauma at the region of TMJ, ear and a) Increased abnormal mitoses.
mastoid. b) Individual cell keratinisation.
c) Epithelial pearls are only present when malignant
In maxilla:
changes have occurred.
l Mouth breathing is presumed as its causative factor.
d) Loss of basal polarity and disorientation of cells is seen.
l This causes deficiency of premaxillary area.
e) Hyperchromatism of cell.
l Clinically, it appears as retraction of middle third of
f) Increased nuclear cytoplasmic ratio.
face or retrusion of maxilla.
g) Dyskaryosis or nuclear atypia.
In mandible: h) Large prominant nuclei are also found.
l Mandibular retrusion is mostly due to agenesis of i) Division of nuclei without division of cytoplasm
condyle or posterior positioning of mandible. j) Basilar hyperplasia.
316 Quick Review Series for BDS 3rd Year

Q. 17. Haemagioma of tongue. vii. Treatment: By antifungal antibiotics—nystatin or am-


photericin suspensions or lozenges. For AIDS patients,
Ans. oral fluconazole is used.
i. Cavernous type of haemangiomas occurs in the tongue, Q. 19. Macrognathia.
lips, etc.
ii. It presents since birth but manifests during Ans.
childhood. l Macrognathia means abnormally large jaw.
iii. It presents as soft, cystic, fluctuant swelling, at times l It may be present as:
pulsatile. i. Generalized enlargement of skeleton as in pituitary
iv. Trauma due to teeth or food results in bleeding. gigantism.
v. Haemangioma of the tongue is treated on the same lines ii. Localized as in Paget’s disease, acromegaly and
as lymphangioma. leontosis ossea.
vi. It is much more difficult to excise it, especially a large l Factors favouring micrognathia are:
haemangioma. Preoperative angiography and ligation l Increase height of ramus.
of lingual artery on both sides may be necessary. l Increase length of body of mandible.

l Anterior positioning of glenoid fossa.


Q. 18. Oral thrush.
l Decreased maxillary length.
Ans. l Posterior positioning of maxilla with relation to cranium.
i. Acute pseudomembranous oral candidiasis is also
known as oral thrush. It is caused by fungus “Candida Treatment
albicans”.
l Ostectomy to a portion of jaw.
ii. Common sites of occurrence: Buccal mucosa, tongue
and palate. Q. 20. Enumerate various types of epulis.
iii. Aetiology: Prolonged antibiotic therapy and immuno-
Ans.
suppression.
iv. Clinical features: i. Epulis means “up on the gum”, it refers to solid swell-
l It is more common in women. ing situated on the gum.
l Characterised by the presence of scrapable adherent ii. It arises from alveolar margin of the jaw.
white plaques that resemble cottage cheese or iii. Various types of epulis are as follows:
curdled milk. a. Congenital epulis
l Burning sensation and foul taste. b. Fibrous epulis
v. Histological features: c. Pregnancy epulis
l Invasion of mycelia is seen in epithelium and d. Giant cell epulis
epithelium is undergoing necrosis. e. Myelomatous epulis and
vi. Diagnosis is established by: History, clinical signs, f. Sarcomatous epulis.
exfoliative cytologic examination or biopsy. iv. They are treated by surgical curettage or by radical excision.

Topic 8

Diseases of Salivary Glands

LONG ESSAYS
Q. 1. Describe classification of salivary gland tumours The salivary glands are exocrine glands which produces
and clinical features, diagnosis and management of saliva. Salivary glands are enveloped by a connective tissue
carcinoma of parotid gland. capsule and composed of lobules.
Ans.
Section | II General Surgery 317

The various salivary glands are as follows: B. Mucoepidermoid tumours


l Parotid glands: This is the largest salivary gland. Its se- C. Acinic cell tumour
cretion is mainly serous in nature and enters the oral D. Carcinoma
cavity via Stensen’s duct. i. Carcinoma in pleomorphic adenoma
l Submandibular glands: They are located in the floor of ii. Adenoid cystic carcinoma
the lower jaws, superior to the digastric muscles and iii. Undifferentiated carcinoma
their secretion is a mixture of both serous fluid and mu- iv. Adenocarcinoma
cus, and enters the oral cavity via Wharton’s ducts. v. Epidermoid carcinoma
l Sublingual glands: They are a pair of glands located vi. Acinic cell tumour
beneath the tongue to the submandibular glands. The vii. Mucoepidermoid carcinoma
secretion produced is mainly mucous in nature, how- viii. Malignant mixed tumour.
ever, it is categorized as a mixed gland. They have 8-20
excretory ducts. II. Non-Epithelial tumours
l Minor salivary glands: They are over 600 minor salivary A. Lipoma
glands and are located throughout the oral cavity within B. Lymphoma
the lamina propria of the oral mucosa. Their secretion is C. Neurofibroma
mainly mucous in nature (except for Von Ebner’s glands). D. Lymphangioma
l Von Ebner’s glands: These glands found in circumval- E. Sarcoma.
late papillae of the tongue. They secrete a serous fluid
Most (70%) salivary gland tumours (SGTs) originate in
important for lipid hydrolysis. They are an essential
the parotid gland. The remaining tumours arise in the sub-
component of taste.
mandibular gland (8%) and minor salivary glands (22%).
Although 75% of parotid gland tumours are benign,
SIMPLE CLASSIFICATION OF SALIVARY slightly more than 50% of tumours of the submandibular
GLAND TUMOURS gland and 80% of minor SGTs are found to be malignant.

A. Benign salivary gland tumours:


i. Pleomorphic adenoma, Clinical Features of Carcinoma of Parotid
ii. Warthin’s tumour, Gland
iii. Oncocytoma, Carcinoma expleomorphic adenoma shows following features:
iv. Monomorphic adenoma. l It starts growing rapidly
B. Malignant salivary gland tumours: l Skin infiltration occurs
i. Mucoepidermoid carcinoma, l Facial nerve paralysis
ii. Adenoid cystic carcinoma, l Exhibits fixation to masseter muscle
iii. Acinic cell carcinoma, l Red, dilated veins over the surface
iv. Carcinoma ex-pleomorphic adenoma, l Presence of lymph nodes in the neck
v. Squamous cell carcinoma, l Tumour feels stony hard
vi. Adenocarcinoma.

Investigations
INTERNATIONAL CLASSIFICATION
i. Fine needle aspiration cytology (FNAC): It is done to
OF SALIVARY GLAND TUMOURS
confirm the diagnosis and rule out malignancy.
According to this classification, the salivary gland tumours ii. Diagnostic imaging techniques;
are devided into: a. X-ray of the bones: To see bony resorption.
I. Epithelial tumours b. Computer tomography (CT scan):
II. Non-epithelial tumours. l It allows direct, bilateral visualization of the sali-

vary gland tumour and provides information


I. Epithelial tumours about overall dimension and tissue invasion.
A. Adenoma l CT is excellent for demonstrating bony invasion.

i. Pleomorphic adenoma l It helps to define the extraglandular spread and

ii. Monomorphic adenomas cervical lymph nodes.


l Adenolymphoma(Warthin’s tumour) c. Magnetic resonance imaging (MRI):
l Oxiphilic adenoma (oncocytoma) l It provides superior soft tissue delineation such

l Other types. as perineural invasion.


318 Quick Review Series for BDS 3rd Year

iii. FNAC of the lymph nodes: It is done in case of lymph l The tumour may be whitish-tan to grey to bluish in colour.
nodes which are palpable in case of malignancy of pa- l In size, pleomorphic adenomas may vary from a few
rotid gland. millimetres to large or even giant.
l It is usually mobile unless found in the palate and can
cause atrophy of the mandibular ramus when located in
Treatment of Malignant Pleomorphic the parotid gland.
Adenoma l When found in the parotid tail, it may present as an
i. Radical parotidectomy: eversion of the ear lobe.
l It includes removal of both lobes of parotid gland, facial l With tumour in the parotid gland, signs of facial nerve
nerve, parotid duct, fibres of masseter, buccinators, weakness are seldom encountered.
pterygoids and radical block dissection of the neck. l In large neglected tumours, facial nerve weakness is
l If facial nerve is not involved, it should be preserved likely to arise as the result of malignant change.
to avoid morbidity. l Pleomorphic adenoma in the deep lobe of the parotid
l Advanced tumours with fixed nodes in the neck may gland presents as an oral retrotonsillar mass/parapha-
require radiotherapy even though the response rate is ryngeal space tumour.
poor. l Patients with minor salivary gland tumours may present
ii. Postoperative radiotherapy: It is indicated, with a variety of symptoms, depending on the site of the
l If the deep lobe is involved. tumour, such symptoms include dysphagia, dyspnoea,
l If the lymph nodes are involved. hoarseness, difficulty in chewing, and epistaxis.
l High grade tumours

l If margins are positive.


Diagnosis
Q. 2. What is mixed parotid tumour, its clinical features a) Tissue sampling procedures:
and management. l Fine needle aspiration (FNA): Sensitivity around 90%
l Core needle biopsy: Sensitivity around 97%.
Ans.
l Needle biopsy is highly recommended prior to surgery
l Pleomorphic adenoma (mixed parotid tumour) is a be- to confirm the diagnosis.
nign parotid tumour, derived from a mixture of ductal
b) Diagnostic imaging techniques:
(epithelial) and myoepithelial cells, thus demonstrating
both epithelial and mesenchymal differention in nature. l Ultrasound: Can determine and characterize superficial
l It derives its name from the architectural pleomorphism parotid tumours.
(variable appearance) seen by light microscopy. l Computer tomography (CT): Allows direct, bilateral
l The tumour has 3 components: An epithelial cell com- visualization of the salivary gland tumour and provides
ponent; a myoepithelial cell component; and a stromal information about overall dimension and tissue inva-
(mesenchymal) component. sion. CT is excellent for demonstrating bony invasion.
l Among the major salivary glands, the tail of the super- l Magnetic resonance imaging (MRI): Provides superior
ficial lobe of the parotid salivary gland is the most com- soft tissue delineation such as perineural invasion.
mon site of occurrence (70-80%), although it can occur
in any parotid location. It is less commonly seen in the Treatment
submandibular salivary gland (10%) and is seldom en- l Overall, the mainstay of the treatment for pleomorphic
countered in the sublingual gland. adenoma is surgical resection.
l Though it is classified as a benign tumour, pleomorphic l Generally, pleomorphic adenomas of the parotid gland
adenomas have the capacity to grow to large proportions are treated with superficial or total parotidectomy with
and may undergo malignant transformation, to form the latter being the more commonly practiced due to
carcinoma ex-pleomorphic adenoma. high incidence of recurrence. The facial nerve should be
l Although it is “benign”, the tumour is aneuploid, it can preserved whenever possible.
recur after resection, it invades normal adjacent tissue l The benign tumours of the submandibular gland is
and distant metastases have been reported after long treated by simple excision with preservation of man-
(110 years) time intervals. dibular branch of the trigeminal nerve, the hypoglossal
nerve, and the lingual nerve.
Clinical Features Q. 3. What are the causes of acute parotitis? Describe
the clinical features and treatment.
Symptoms and signs depend on the location.
l Pleomorphic adenoma is usually a solitary, slow growing, Ans.
firm, mobile, well-circumscribed painless nodular mass. Parotitis is defined as inflammation of parotid glands.
Section | II General Surgery 319

Aetiology of Acute Parotitis l The patient quickly becomes extremely ill. In the hospi-
tal setting, S. aureus is apt to be methicillin-resistant
Acute parotitis can be caused by bacterial or viral infection.
(MRSA).
i. Acute bacterial parotitis
l Gram stains and culture and sensitivity testing is or-
ii. Acute viral parotitis (mumps)
dered.
Acute bacterial parotitis l Intravenous vancomycin 500 mg every 6 hours is begun
empirically.
l Presently acute bacterial parotitis is rare. l Therapy may be altered based upon cultures or infec-
l Before the introduction of antibiotics and intravenous tious disease consultation.
administration of fluids, bacterial parotitis occurred in
postoperative patients or other severely ill patients. Q. 4. Describe the pathology, clinical features and man-
l Acute bacterial parotitis is most often caused by a bacte- agement of submandiblar salivary calculus.
rial infection of Staphylococcus aureus but may be Ans.
caused by any commensal bacteria.
l Submandibular salivary calculus/sialolithiasis is most
Acute viral parotitis common among other salivary glands, which is also the
l It represents itself as one of the classic childhood infec- most frequent cause of recurrent sialadenitis.
tions which spread by droplets or by direct spread from l The higher frequency of calculi formation in the sub-
oropharyngeal secretions that contain the paramyxovirus. mandibular gland is related to the:
l Universal immunization from 1977 has made the clini- a. pH of saliva (alkaline in the submandibular gland),
cal disease unusual in developed countries. b. viscosity of saliva (more mucous in the submandibu-
l The child should receive the first measles, mumps and lar gland) and
rubella (MMR) vaccine at age one year and a second at c. anatomical factors (uphill course of Wharton’s duct).
age 4-6 years. Submandibular sialolithiasis is most often situated near
l Occasional outbreaks of mumps, mostly in teenagers or the orifice of Wharton’s duct or at the bend of the duct pass-
patients in their early twenties are seen in case where ing behind the mylohyoid muscle.
the second shot has not been received.
l The disease was characterized by grossly enlarged and
modestly tender parotid glands. Parotid stimulation Pathology
causes pain in the gland and ear. l As the concentration of mucus is higher in the subman-
l Mumps was a benign disease in the vast majority of dibular gland, it results in viscous nature of its secre-
cases but was occasionally complicated by meningoen- tions compared to the other salivary glands.
cephalitis, pancreatitis, orchitis, or deafness especially l This increased viscosity, and subsequent relatively
in young adults. slower flow, contributes to the formation of salivary
gland calculi and stasis.
Clinical Features
l Acute bacterial parotitis: The patient reports progressive Clinical Features
painful swelling of the gland and fever; chewing aggra- l A palpable lump or visible swelling in the area of the
vates the pain. gland.
l Acute viral parotitis (mumps): Pain and swelling of the l Pain, if present, originates from the floor of the mouth.
gland lasts for 5-9 days. Moderate malaise, anorexia, l In many cases, stones cause only intermittent swelling.
and fever occur. Bilateral involvement is present in most l Symptoms tend to increase during meals.
instances. l Complications include persistent obstruction of the
l The acutely inflamed gland is exquisitely tender. duct, leading to bacterial invasion, overgrowth and in-
l Massaging the gland from posterior to anterior ex- fection (sialoadenitis).
presses clear saliva from the parotid duct in normal
glands.
l Purulent saliva is expressed with bacterial parotitis. Treatment
Some current treatment options are:
Treatment l For small stones, hydration, moist heat, NSAIDs oc-
casionally, and having the patient take any food or
l Acute bacterial parotitis is caused by bacteria that as- beverage that is bitter and/or sour. Sucking on citrus
cends from the mouth and most frequently occurs in fruits, such as a lemon or orange, may increase saliva-
chronically ill patients. tion and promote spontaneous expulsion of the stone.
320 Quick Review Series for BDS 3rd Year

l Some stones may be massaged out by a specialist. l Sialendoscopy


l Duct cannulation with stone removal, gland excision in l Antibiotics are recommended.http://en.wikipedia.org/
recurrent cases. wiki/Sialendoscopy

SHORT ESSAYS
Q. 1. Acute parotitis. Treatment
Ans. l In the hospital setting, S. aureus is apt to be methicillin
resistant (MRSA).
l Acute parotitis can be caused by bacterial or viral
l Gram stains and culture and sensitivity testing is or-
infection.
dered.
a. Acute bacterial parotitis
l Intravenous vancomycin 500 mg every 6 hours is begun
b. Acute viral parotitis (mumps)
empirically.
l Therapy may be altered based upon cultures or infec-
Aetiology tious disease consultation.
l Presently acute bacterial parotitis is rare, before the in- Q. 2. Pleomorphic adenoma.
troduction of antibiotics and intravenous administration
of fluids, bacterial parotitis occurred in postoperative Ans.
patients or other severely ill patients. l Pleomorphic adenoma (mixed parotid tumour) is a
l Acute bacterial parotitis is most often caused by a bacte- benign parotid tumour, derived from a mixture of duc-
rial infection of Staphylococcus aureus but may be tal (epithelial) and myoepithelial cells, thus demon-
caused by any commensal bacteria. strating both epithelial and mesenchymal differention
l Acute viral parotitis represents itself as one of the clas- in nature.
sic childhood infections which spread by droplets or by l Among the major salivary glands, the tail of the super-
direct spread from oropharyngeal secretions that con- ficial lobe of the parotid salivary gland is the most com-
tain the paramyxovirus. mon site of occurrence (70-80%), although it can occur
l Universal immunization from 1977 has made the clini- in any parotid location. It is less commonly seen in the
cal disease unusual in developed countries. The child submandibular salivary gland (10%) and is seldom en-
should receive the first measles, mumps and rubella countered in the sublingual gland.
(MMR) vaccine at age one year and a second at age 4-6 l Though it is classified as a benign tumour, pleomorphic
years. adenomas have the capacity to grow to large proportions
l The disease was characterized by grossly enlarged and and may undergo malignant transformation, to form
modestly tender parotid glands. Parotid stimulation carcinoma expleomorphic adenoma.
caused pain in the gland and ear. l Although it is “benign”, the tumour is aneuploid, it can
l Mumps was a benign disease in the vast majority of recur after resection, it invades normal adjacent tissue
cases but was occasionally complicated by meningoen- and distant metastases have been reported after long
cephalitis, pancreatitis, orchitis, or deafness especially (110 years) time intervals.
in young adults.

Clinical Features Clinical Features


l Acute bacterial parotitis: The patient reports progressive Symptoms and signs depend on the location.
painful swelling of the gland and fever; chewing aggra- l Pleomorphic adenoma is usually a solitary, slow grow-
vates the pain. ing, firm, mobile, well-circumscribed painless nodular
l Acute viral parotitis (mumps): Pain and swelling of the mass.
gland last 5-9 days. Moderate malaise, anorexia, and l The tumour may be whitish-tan to grey to bluish in colour.
fever occur. Bilateral involvement is present in most l In size, pleomorphic adenomas may vary from a few
instances. millimetres to large or even giant.
l The acutely inflamed gland is exquisitely tender. l It is usually mobile unless found in the palate and can
l Massaging the gland from posterior to anterior ex- cause atrophy of the mandibular ramus when located in
presses clear saliva from the parotid duct in normal the parotid gland.
glands. l When found in the parotid tail, it may present as an
l Purulent saliva is expressed with bacterial parotitis. eversion of the ear lobe.
Section | II General Surgery 321

l With tumour in the parotid gland, signs of facial nerve c. the anatomical factors (uphill course of Wharton’s
weakness are seldom encountered. duct).
l In large neglected tumours, facial nerve weakness is l Submandibular sialolithiasis is most often situated near
likely to arise as the result of malignant change. the orifice of Wharton’s duct or at the bend of the duct
l Pleomorphic adenoma in the deep lobe of the parotid passing behind the mylohyoid muscle.
gland present as an oral retrotonsillar mass/parapharyn-
geal space tumour.
Pathology
l Patients with minor salivary gland tumours may present
with a variety of symptoms, depending on the site of the l As the concentration of mucus is higher in the subman-
tumour, such symptoms include dysphagia, dyspnoea, dibular gland, it results in viscous nature of its secre-
hoarseness, difficulty in chewing, and epistaxis. tions compared to the other salivary glands.
l This increased viscosity, and subsequent relatively
slower flow, contributes to the formation of salivary
Diagnosis gland calculi and stasis.
a) Tissue sampling procedures:
l Fine needle aspiration (FNA): Sensitivity around 90% Clinical Features
l Core needle biopsy: Sensitivity around 97%.
l Needle biopsy is highly recommended prior to surgery l A palpable lump or visible swelling in the area of the
to confirm the diagnosis. gland.
l Pain, if present, originates from the floor of the mouth.
b) Diagnostic imaging techniques: l In many cases, stones cause only intermittent swelling.
l Ultrasound can determine and characterize superficial l Symptoms tend to increase during meals.
parotid tumours. l Complications include persistent obstruction of the
l Computer tomography (CT) allows direct, bilateral vi- duct, leading to bacterial invasion, overgrowth and in-
sualization of the salivary gland tumour and provides fection (sialoadenitis).
information about overall dimension and tissue inva-
sion. CT is excellent for demonstrating bony invasion.
Treatment
l Magnetic resonance imaging (MRI) provides superior
soft tissue delineation such as perineural invasion. Some current treatment options are:
l For small stones, hydration, moist heat, NSAIDs occa-
sionally, and having the patient take any food or bever-
Treatment age that is bitter and/or sour. Sucking on citrus fruits,
l Overall, the mainstay of the treatment for pleomorphic such as a lemon or orange, may increase salivation and
adenoma is surgical resection. promote spontaneous expulsion of the stone.
l Generally, pleomorphic adenomas of the parotid gland l Some stones may be massaged out by a specialist.
are treated with superficial or total parotidectomy with l Duct cannulation with stone removal, gland excision in
the latter being the more commonly practiced due to recurrent cases.
high incidence of recurrence. The facial nerve should be l Sialendoscopy
preserved whenever possible. l Antibiotics are recommended.http://en.wikipedia.org/
l The benign tumours of the submandibular gland is wiki/Sialendoscopy
treated by simple excision with preservation of man-
Q. 4. Surgical anatomy of parotid gland.
dibular branch of the trigeminal nerve, the hypoglossal
nerve, and the lingual nerve. Ans.
Q. 3. Salivary gland calculus. l The parotid gland lies beneath the skin, in front of and
below the ear. It is contained within the investing
Ans.
layer of the deep fascia of the neck, called the parotid
l Submandibular salivary calculus/sialolithiasis is most fascia.
common among other salivary glands. l It is separated from the submandibular gland by a
l The higher frequency of calculi formation in the sub- fascial thickening, the stylomandibular ligament.
mandibular gland is related to: l The parotid gland occupies the parotid space.
a. the pH of saliva (alkaline in the submandibular gland), l From the anterolateral edge of the gland, the parotid
b. the viscosity of saliva (more mucous in the subman- duct (Stensen’s) passes lateral to the masseter muscle. It
dibular gland) and turns medial at the anterior margin of the muscle, where
322 Quick Review Series for BDS 3rd Year

it is related to the buccal fat pad or “boule de Bichat”. l The most common location to find a mucocele is the
The buccal pad is located medial to the parotid duct, surface of the lower lip. It can also be found on the buc-
between the masseter and buccinator muscles. cal mucosa, on the anterior ventral tongue, and the floor
l The buccinator muscle is pierced by the duct. It enters the of the mouth (ranula).
oral cavity at the level of the upper second molar tooth. l Superficial lesions take on a bluish to translucent hue,
Accessory parotid tissue may extend along the parotid whereas deep lesions have normal mucosal colouration.
duct. A short accessory duct may enter the main duct. l Their size varies from 1 mm to several centimetres and
is more commonly found in children and young adults.
Parotid Fascia l They may rapidly enlarge and then appear to involute
because of the rupture of the contents into the oral cav-
l The parotid fascia is the splitting of the general invest- ity or resorption of the extravasated mucus.
ing layer that envelops both the parotid and subman- l The patient may relate a history of recent or remote
dibular (submaxillary) glands, forming the superficial trauma to the mouth or face, or the patient may have a
and deep layers. habit of biting the lip. However, in many cases, no insult
l The superficial layer is dense and tough in comparison can be identified.
to the deep, which is thin and weak. However, the stylo- l The duration of the lesion is usually 3-6 weeks; how-
mandibular ligament between the styloid process and ever, it may vary from a few days to several years in
the angle of the mandible is derived from the deep layer. exceptional instances.
It is tough, and separates the parotid from the subman- l Patients with superficial mucoceles report small fluid-
dibular gland. filled vesicles on the soft palate, the retromolar pad, the
l The parotid space communicates medially with the lat- posterior buccal mucosa, and, occasionally, the lower
eral pharyngeal space and with the posterior area of the labial mucosa.
masticator space. The posterior area of the masticator l Mucoceles appear as granulation tissue surrounding
space contains the masseter muscle, the pterygoid mus- mucin. Since inflammation occurs concurrently, neutro-
cles, the small pterygomandibular space and the space phils and foamy histiocytes usually are present.
of the body of the mandible. l The mucosa lining is usually intact; however, repeated
l Since many intraparotid anatomic entities radiate from sucking on the lesion may result in a white, rough, kera-
the gland, the surgeon should be familiar with all of totic surface. Occasionally, a punctate sinus tract is ob-
them, especially those that must not be sacrificed. served from which mucoid material is expressed.
l These vesicles spontaneously rupture and leave an
Bed of the Parotid Gland ulcerated mucosal surface that heals within a few
days.
l Complete removal of the parotid gland reveals the fol- l Typically, affected individuals report a chronic and re-
lowing structures: current history. Frequently, the patient has a history of
l One Vein: Internal jugular vein
lichen planus, lichenoid drug reaction, or chronic graft
l Two Arteries: carotis externa and interna
versus host disease involving the oral mucosa.
l Four Nerves: IX, X, XI, and XII cranial nerves

Q. 5. Mucous cyst/ mucocele.


Treatment
Ans.
l Some mucoceles spontaneously resolve on their own
l Mucous cyst of the oral mucosa or mucocele refers to after a short time while others are chronic and require
two related phenomena: mucus extravasation phenom- surgical removal.
enon, and mucus retention cyst. l Recurrence may occur, and thus the adjacent salivary
l Mucus extravasation phenomenon refers to a swelling gland is excised as a preventive measure.
of connective tissue containing collected mucin due to a l Several types of procedures are available for the surgi-
ruptured salivary gland duct usually caused by local cal removal of mucoceles. These include laser and
trauma. minimally-invasive techniques which means recovery
l In case of mucus retention cyst, mucin collection is due times are reduced drastically.
to an obstructed or ruptured salivary duct.
Q. 6. Ranula.

Clinical Features Ans.


l Mucoceles are asymptomatic swellings that have a rela- l Ranula is a pseudocyst that is associated with mucus
tively rapid onset and fluctuate in size. extravasation into the surrounding soft tissues.
Section | II General Surgery 323

l Ranulas are mucoceles that occur in the floor of the Treatment


mouth and usually involve the major salivary glands.
Specifically, the ranula originates in the body of the a. Oral ranula
sublingual gland, in the ducts of Rivini of the sublingual l With most oral ranulas, surgical management is pre-
gland, in the Wharton’s duct of the submandibular ferred which includes complete excision of the ranula
gland, and, infrequently from the minor salivary glands and associated major salivary gland.
at this location. l Laser ablation and cryosurgery, either alone or after
l Ranulas are divided into 2 types: marsupialization, have been used for some patients with
a. oral ranulas and oral ranula.
b. cervical or plunging ranulas. l Isolated reports demonstrate that oral ranulas have
been successfully treated with intracystic injection
Oral ranulas are secondary to mucus extravasation that of the streptococcal preparation, OK-432 and botuli-
pools superior to the mylohyoid muscle, whereas cervical num toxin A. Currently, the use of this sclerosing
ranulas are associated with mucus extravasation along the agent for the treatment of oral ranulas is considered
facial planes of the neck. experimental.

Clinical Features b. Cervical ranula


Oral ranula l The elimination of cervical ranulas depends on the com-
plete surgical excision of the oral portion of the ranula
l Oral ranula usually presents as asymptomatic swelling
with the associated sublingual salivary gland or, rarely,
of the floor of the mouth.
the submandibular gland.
l It is a relatively large unilateral blue to translucent mass
in the floor of the mouth that remotely resembles the Q. 7. Xerostomia.
belly of a frog (Rana species).
Ans.
l The lesion may cross the midline when especially
large, making the offending salivary gland difficult to l Xerostomia is defined as dry mouth resulting from re-
localize. duced or absent salivary flow.
l Large oral ranulas displace the tongue superiorly and l Xerostomia is sometimes colloquially called pasties,
medially, and may interfere with speech, mastication, cottonmouth, drooth, doughmouth.
respiration, and swallowing.
l When oral ranulas are large, the tongue may place
pressure on the lesion, which may interfere with sub-
Aetiology
mandibular salivary flow. As a result, obstructive sali- l It may be a symptom of various medical conditions
vary gland signs and symptoms may develop, such as (Sjögren’s syndrome, sarcoidosis and amyloidosis or a
pain or discomfort when eating, a feeling of fullness at side effect of a radiation to the head and neck, or a side
that site, and increased swelling of the submandibular effect of many medications (antihistamines, antidepres-
gland. sants, anticholinergics, anorexiants, antihypertensives,
l The consistency of the lesion is that of mucus, and the antipsychotics, anti-Parkinson agents, diuretics and sed-
lesion does not blanch on compression. atives), and chemotherapy.
l If the mass is located in the deeper aspect of the floor of l It may or may not be associated with decreased salivary
the mouth, it loses its bluish translucent colour. gland function.
l Xerostomia is often a contributing factor for both minor
Cervical ranula and serious health problems. It can affect nutrition and
l It appears as an asymptomatic, continuously enlarging dental, as well as psychological, health.
mass that may fluctuate in size. The overlying skin is
usually intact.
Clinical Features
l The mass is fluctuant, freely movable, and nontender.
The mass is not associated with the thyroid gland or l Individuals with xerostomia often complain of problems
lymph node chains. with eating, speaking, swallowing and wearing den-
l The mass may not be well defined but follows the fas- tures. Denture wearers may have problems with denture
cial planes of the neck and may extend into the medi- retention, denture sores and the tongue sticking to the
astinum. Similar to the oral ranula, the mass tends to palate.
cause a lateral swelling; however, it may cross the l Dry, crumbly foods, such as cereals and crackers, may
midline. be particularly difficult to chew and swallow.
324 Quick Review Series for BDS 3rd Year

l Patients with xerostomia often complain of taste dis- Diagnosis


orders (dysgeusia), a painful tongue (glossodynia)
Diagnosis of xerostomia may be based on evidence ob-
and an increased need to drink water, especially at
tained from the patient’s history, an examination of the oral
night.
cavity and/or sialometry.
l Xerostomia can lead to markedly increased dental car-
ies, parotid gland enlargement, inflammation and fissur-
ing of the lips (cheilitis), inflammation or ulcers of the
Treatment
tongue and buccal mucosa, oral candidiasis, salivary
gland infection (sialadenitis), halitosis and cracking and l Treatment includes finding causes and removing them,
fissuring of the oral mucosa. if possible.
l Some common problems associated with xerostomia l Patients with xerostomia should avoid the use of decon-
include a constant sore throat, burning sensation, diffi- gestants and antihistamines, and pay careful attention to
culty in speaking and swallowing, hoarseness and/or oral hygiene.
dry nasal passages. l Sipping non-carbonated sugarless fluids frequently,
l If left untreated, xerostomia decreases the oral pH chewing xylitol-containing gum and using a carboxy-
and significantly increases the development of plaque methyl cellulose saliva substitute may help.
and dental caries. Oral candidiasis is one of the most l Pilocarpine may be prescribed to treat xerostomia.
common oral infections seen in association with l Non-systemic relief can be found using an oxidized
xerostomia. glycerol triesters treatment used to coat the mouth.

SHORT NOTES
Q. 1. Write a short note on ranula. ii. Indications:
l Salivary gland’s stones, salivary duct calculus,
Ans.
Sjögren’s syndrome, salivary gland tumours, sali-
i. Ranula is a pseudo cyst associated with mucus extrava- vary gland narrowing or obstruction, etc.
sation into the surrounding soft tissues or mucoceles iii. Technique:
that occur in the floor of the mouth and usually involve l A baseline radiograph of the required salivary gland

the major salivary glands. would be taken, a cannula then is inserted in this
ii. Aetiology: Trauma to salivary duct and occasionally salivary gland duct’s opening in the mouth, and then
saliva obstruction. a radio-opaque fluid (contrast medium) is injected in
iii. These lesions are divided into 2 types: the duct through a small tube.
a. Oral ranulas and l A series of radiographs would then be taken to de-

b. Cervical or plunging ranulas. termine the flow of the fluid, identify any obstruc-
iv. Oral ranulas are secondary to mucus extravasation that tions and its location, the rate of fluid excretion from
pools superior to the mylohyoid muscle, whereas cervi- the gland.
cal ranulas are associated with mucus extravasation
Q. 3. Write a short note on parotid fistula.
along the facial planes of the neck.
v. Treatment: Ans.
l With most oral ranulas, surgical management is
i. Parotid fistula is most commonly a post-traumatic situ-
preferred.
ation. In post-traumatic cases, spontaneous closure of
l Oral ranulas have been successfully treated with in-
the fistula is the general rule.
tracystic injection of the streptococcal preparation,
ii. Conservative approaches to the treatment of a parotid
OK-432.
fistula are eliminating oral intake by the patient and ap-
l The elimination of cervical ranulas depends on the
plying a pressure dressing while maintaining nutrition
complete surgical excision of the oral portion of the
by the intravenous route.
ranula with the associated sublingual salivary gland
iii. Anticholinergic drugs decrease the production of saliva
or, rarely, the submandibular gland.
and thus would appear to be beneficial.
Q. 2. Write a short note on sialogram. iv. When a parotid fistula does not heal under these condi-
tions, then more aggressive treatment is indicated.
Ans.
v. Treatment should be based on whether the fistula is
i. Sialogram or sialography is diagnostic imaging of the ductal or glandular in origin. Several methods of
salivary glands using X-ray. treatment have been advocated in the past. Low dose
Section | II General Surgery 325

radiotherapy has been mentioned by some authorities vii. Most of these tumours are treated with surgical re-
as the treatment of choice for parotid fistula. moval. Recurrence is rare, occurring in 6–12% of
cases.
Q. 4. Acute parotitis.
Q. 6. Frey’s syndrome.
Ans.
Ans.
i. Parotitis is defined as inflammation of parotid gland.
ii. Aetiology: Acute parotitis can be caused by bacterial or i. Frey’s syndrome is a food-related syndrome which can
viral infection. be congenital or acquired specially after parotid surgery
a. Acute bacterial parotitis and can persist for life.
b. Acute viral parotitis (mumps) ii. The symptoms of Frey’s syndrome are redness and
iii. Presently acute bacterial parotitis is rare. sweating on the cheek area adjacent to the ear.
iv. Acute bacterial parotitis is most often caused by a bac- iii. Sweating appears when the affected person eats, sees,
terial infection of Staphylococcus aureus but may be thinks about or talks about certain kinds of food which
caused by any commensal bacteria. produce strong salivation. Observing sweating in the
v. Acute viral parotitis represents itself as one of the clas- region after eating a lemon wedge may be diagnostic.
sic childhood infections which spread by droplets or by iv. Frey’s syndrome often results as a side effect of parotid
direct spread from oropharyngeal secretions that con- gland surgery.
tain the paramyxovirus. The auriculotemporal branch of the trigeminal nerve
vi. The child should receive the first measles, mumps and carries sympathetic fibres to the sweat glands of the
rubella (MMR) vaccine at age one year and a second at scalp and parasympathetic fibres to the parotid gland.
age 4-6 years. As a result of severance and inappropriate regeneration,
v. The disease was characterized by grossly enlarged and the fibres may switch courses, resulting in “gustatory
modestly tender parotid glands. Parotid stimulation sweating” or sweating in the anticipation of eating, in-
caused pain in the gland and ear. stead of the normal salivatory response.
vi. Mumps was a benign disease in the vast majority of v. Treatments:
cases but was occasionally complicated by meningoen- l Injection of botulinum toxin type A.

cephalitis, pancreatitis, orchitis, or deafness especially l Surgical transection of the nerve fibres.

in young adults. l Application of an ointment containing an anticholin-

ergic drug such as scopolamine.


Q. 5. Warthin’s tumour.
Q. 7. Salivary calculus.
Ans.
Ans.
i. Warthin’s tumour also known as papillary cystade-
noma lymphomatosum or adenolymphoma, is a type of i. Sialolithiasis refers to the formation of stones in the
benign tumour of the salivary glands. salivary glands. Stones are most commonly found in
ii. Its aetiology is unknown, but there is a strong associa- the submandibular gland, where stones can obstruct
tion with cigarette smoking. Smokers are at 8 times Wharton’s duct.
greater risk of developing Warthin’s tumour than the ii. It is frequently associated with chronic infection
general population. (Staphylococcus aureus, Streptococcus viridans) of the
iii. It primarily affects older individuals (age 60–70 years) glands, dehydration, Sjögren’s syndrome and/or in-
and there is a slight female predilection according to creased local levels of calcium, but in many cases can
recent studies. arise idiopathically.
iv. The tumour is slow growing, painless, and usually ap- iii. Pain, when present, usually originates from the floor of
pears in the tail of the parotid gland near the angle of the mouth, although in many cases the stones cause
the mandible. only intermittent swelling.
v. In 5–14% of cases, Warthin’s tumour is bilateral, but iv. Since chewing promotes release of saliva, symptoms
the two masses usually are at different times. Warthin’s tend to increase during meals. A palpable lump or vis-
tumour is highly unlikely to become malignant. ible swelling in the area of the gland is often noted.
vi. The histological appearance of this tumour is unique. v. Complications include persistent obstruction of the
There are cystic spaces surrounded by two uniform rows duct, leading to bacterial invasion, overgrowth and
of cells with centrally placed pyknotic nuclei. The cystic infection.
spaces have epithelium referred to as papillary infoldings vi. Diagnosis is usually made by characteristic history and
that protrude into them. Additionally, the epithelium has physical examination and can be confirmed by X-ray
lymphoid stroma with germinal centre formation. (80%) or by sialogram or ultrasound.
326 Quick Review Series for BDS 3rd Year

vii. Current treatment options are: v. Xerostomia can lead to markedly increased dental car-
l For small stones, hydration, moist heat, NSAIDs ies, parotid gland enlargement, inflammation and fis-
occasionally, and having the patient take any food suring of the lips (cheilitis), inflammation or ulcers of
or beverage that is bitter and/or sour. Sucking on the tongue and buccal mucosa, oral candidiasis, sali-
citrus fruits, such as a lemon or orange, may in- vary gland infection (sialadenitis), halitosis and crack-
crease salivation and promote spontaneous expul- ing and fissuring of the oral mucosa.
sion of the stone.
Q. 10. Plunging ranula.
l Some stones may be massaged out by a specialist.

l Duct cannulation with stone removal, gland exci- Ans.


sion in recurrent cases.
i. A ranula below the mylohyoid muscle is referred to as
l Sialendoscopy
a “plunging or cervical ranula” which can manifest as
l Antibiotics are recommended.http://en.wikipedia.
neck swelling in conjunction with, or independent of, a
org/wiki/Sialendoscopy
floor-of-mouth cyst.
Q. 8. Pleomorphic adenomas. ii. Most reported plunging ranulas are 4-10 cm in size and
are usually found in the submandibular space.
Ans.
iii. Treatment: Surgical
i. Pleomorphic adenoma (mixed parotid tumour) is a l Transoral approach: This provides better access for

benign parotid tumour, derived from a mixture of duc- complete removal of the sublingual gland.
tal (epithelial) and myoepithelial cells, thus demon- l Transcervical approach: Complete removal of the

strating both epithelial and mesenchymal differention sublingual gland is difficult with this approach, re-
in nature. quiring division of the mylohyoid muscle and dis-
ii. Among the major salivary glands, the tail of the super- section up to the floor of the mouth.
ficial lobe of the parotid salivary gland is the most com-
Q. 11. Sjogren’s syndrome.
mon site of occurrence (70-80%), it is less commonly
seen in the submandibular salivary gland (10%) and is Ans.
seldom encountered in the sublingual gland.
i. Sjögren’s syndrome also known as “ Mikulicz disease”
iii. Clinical features:
and “Sicca syndrome”, is a systemic autoimmune dis-
l Slow growing, painless tumour.
ease in which immune cells attack and destroy the
l Tumour in the parotid gland, signs of facial nerve
exocrine glands that produce tears and saliva.
weakness are seldom encountered.
ii. Sjögren’s syndrome can exist as a disorder in its own
l In large neglected tumours, facial nerve weakness is
right (primary Sjögren’s syndrome) or it may develop
likely to arise as the result of malignant change.
years after the onset of an associated rheumatic disor-
l Pleomorphic adenoma in the deep lobe of the pa-
der such as rheumatoid arthritis, systemic lupus erythe-
rotid gland present as an oral retrotonsillar mass/
matosus, scleroderma, primary biliary cirrhosis, etc.
parapharyngeal space tumour.
(secondary Sjögren’s syndrome).
iv. Surgical excision.
iii. The hallmark symptom of the disorder is a generalized
Q. 9. Xerostomia. dryness, typically involving dry mouth and dry eyes
(part of what are known as Sicca symptoms). In addi-
Ans.
tion, Sjögren’s syndrome may cause skin, nose, and
i. Xerostomia is defined as dry mouth resulting from re- vaginal dryness.
duced or absent saliva flow. iv. Treatment:
ii. Aetiology: Postoperative period, dehydration, enteric l There is neither a known cure for Sjögren’s syn-

fever, septicaemia, post-radiotherapy for oral cancer. drome nor a specific treatment.
iii. Individuals with xerostomia often complain of prob- l Treatment is generally symptomatic and supportive

lems with eating, speaking, swallowing and wearing like moisture replacement therapies such as artifi-
dentures. Denture wearers may have problems with cial tears, additionally, ciclosporin (restasis) is
denture retention, denture sores and the tongue sticking available by prescription to help treat chronic dry
to the palate. eye by suppressing the inflammation that disrupts
iv. Patients with xerostomia often complain of taste dis- tear secretion.
orders (dysgeusia), a painful tongue (glossodynia) l Prescription drugs are also available that help to

and an increased need to drink water, especially at stimulate salivary flow, such as cevimeline (Evoxac)
night. and pilocarpine.
Section | II General Surgery 327

Topic 9

Infections and Diseases of the Larynx and Nasopharynx

LONG ESSAYS
Q. 1. Describe the signs, symptoms and treatment of l Direct laryngoscopy also allows thorough evaluation of
carcinoma of larynx. the larynx and the entire upper aerodigestive tract for
accurate staging of the disease.
Ans.
Laryngeal carcinoma is a malignancy of the larynx in-
l
Treatment
volving the true vocal cords and anterior and posterior
commissures. l The goal of treatment is cure of the disease and second-
l Most laryngeal cancers are squamous cell carcinomas, ary objective is the preservation or reconstruction of
reflecting their origin from the squamous cells which voice and the ability to swallow without aspiration.
form the majority of the laryngeal epithelium. l Surgery, radiation, or multiple-modality treatment can ac-
l For the purposes of tumour staging, the larynx is complish the management of this disease. Chemotherapy
divided into three anatomical regions: is used as an induction agent in some treatment protocols.
i. The glottis (true vocal cords, anterior and posterior l The factors to be considered to determine the best treat-
commissures); ment modality for management of laryngeal carcinoma
ii. The supraglottis (epiglottis, arytenoids and aryepi- in each patient are:
glottic folds, and false cords); and a. Tumour stage and characteristics,
iii. The subglottis. b. Patient factors, and
c. Treatment facilities.
Out of these, glottic laryngeal carcinoma is most fre- l In general, early stages T1 and T2 are managed with a
quent, followed by supraglottic and subglottic carcinoma. single modality, such as radiation, endoscopic excision,
or conservation laryngeal surgery. Moderately advanced
Clinical Features lesions T3 are typically treated with a combination of
radiation and chemotherapy.
l Persistent hoarseness or other voice changes l Radiation is the primary nonsurgical treatment for
l Neck lump early-stage. Standard course of radiation for laryngeal
l Laryngeal tumours can also cause hemoptysis and air- carcinoma cancer usually consists of a total of 60-70 Gy
way compromise, if the tumours are large administered in single daily fractions over 6 weeks.
l Sore throat l Advantages of radiotherapy include the avoidance
l Persistent cough of surgery and the subsequent hospitalization and a
l Stridor and superior voice outcome.
l Bad breath l Recent studies indicate improved disease control with
radiation and chemotherapy given simultaneously.
Surgery, usually a total laryngectomy, is used in patients
Diagnosis
l

who do not respond to chemotherapy.


l Diagnosis is made on the basis of a medical history, l Surgical options include conservation laryngeal surgery
physical examination, and imaging diagnostics like or total laryngectomy, depending on the extent of
chest X-ray, CT or MRI scans, and tissue biopsy. the disease. Endoscopic laser surgery can be used as
l CT scanning of the neck is useful to evaluate tumour an alternative to laryngectomy or chemoradiation in
extension, especially cartilage invasion and nodal selected T3 laryngeal carcinoma.
metastases. l Invasive tumours (i.e. T4 due to cartilage destruction)
l Chest radiograph or chest CT scan can rule out lung are usually managed with a total laryngectomy and ap-
metastases and a second primary malignancy. propriate neck dissections.
l A tissue biopsy for confirmation of the diagnosis is l Endoscopic management of premalignant lesions
imperative before initiating any treatment. and stage I and some stage II glottic carcinomas can be
328 Quick Review Series for BDS 3rd Year

performed during direct laryngoscopy using an opera- iii. Lower oesophageal rings: Schatzki’s ring:
tive microscope. l A lower oesophageal ring is a 2 to 4 mm mucosal stric-
l Vertical partial laryngectomy is indicated in the treat- ture, probably congenital, causing a ring-like narrowing
ment of tumours that arise on the true vocal cord with of the distal oesophagus at squamocolumnar junction.
limited involvement of the anterior commissure. l These rings cause intermittent dysphagia for solids.
l In the case of anterior commissure involvement, a fron- This symptom can begin at any age but usually does not
tolateral partial laryngectomy may be considered. This begin until after age 25.
procedure extends the resection to the contralateral l The swallowing difficulty comes and goes and is
cord, including the anterior commissure. especially aggravated by meat and dry bread. Symptoms
l Contraindications for both types of vertical partial usually occur only when the oesophageal lumen is
laryngectomy include tumour involvement of the ,12 mm in diameter and never when it is .20 mm.
interarytenoid area, subglottic extension greater than
10 mm, and poor medical condition, especially signifi- iv. Oesophageal web (Plummer-Vinson syndrome;
cant pulmonary disease. Paterson-Kelly syndrome; sideropenic dysphagia):
l Total laryngectomy is the standard for treatment of l An oesophageal web is a thin mucosal membrane that
advanced laryngeal cancer. Adequate treatment of grows across the lumen. Rarely, webs develop in patients
tumours that have invaded beyond the confines of the with untreated severe iron-deficiency anaemia; they
larynx involves resection of the disease with a margin of develop even more rarely in patients without anaemia.
normal extralaryngeal tissue. l Webs usually occur in the upper oesophagus, causing
l In this procedure, the entire larynx, hyoid bone, and dysphagia for solids. They are best diagnosed by barium
overlying strap muscles are resected in an en bloc man- swallow.
ner. If tumour is involved, the thyroid gland is removed
with the larynx. v. Dysphagia lusoria:
Q. 2. Discuss the differential diagnosis of obstructive l It is caused by compression of the oesophagus from any
lesions in the oesophagus. of several congenital vascular abnormalities.
l The vascular abnormality is usually an aberrant right
Ans. subclavian artery arising from the left side of the aortic
Various differential diagnosis of obstructive lesions in the arch, a double aortic arch, or a right aortic arch with left
oesophagus are as follows: ligamentum arteriosum.
l The dysphagia may develop in childhood or later in life
i. Oesophageal stricture: as a result of arteriosclerotic changes in the aberrant
l It is usually a complication of acid reflux, most commonly vessel.
due to gastroesophageal reflux (GERD). These patients l Barium swallow shows the extrinsic compression, but
are usually older and have had GERD for a long time. arteriography is necessary for absolute diagnosis.
l Oesophageal stricture can also be due to other causes, l Most patients require no treatment, but surgical repair is
such as acid reflux from Zollinger-Ellison syndrome, sometimes done.
trauma from NG tube placement, and chronic acid ex-
vi. Scleroderma:
posure in patients with poor oesophageal motility from
scleroderma. l It is characterized by atrophy and sclerosis of the gut
l Other non-acid-related causes of peptic strictures in- wall, most commonly of the distal oesophagus (90%).
clude infectious oesophagitis, ingestion of chemical ir- Consequently, the lower oesophageal sphincter cannot
ritant, pill irritation, and radiation. Peptic stricture is a close and this can lead to severe gastroesophageal reflux
progressive mechanical dysphagia, meaning patients disease (GERD).
will complain of initial intolerance to solids followed by l Patients typically present with progressive dysphagia to
inability to tolerate liquids. both solids and liquids secondary to motility problems
or peptic stricture from acid reflux.
ii. Oesophageal cancer:
l It presents with progressive mechanical dysphagia. Pa- vii. Achalasia:
tients usually come with rapidly progressive dysphagia l It is characterized by failure of lower oesophageal sphinc-
first with solids then with liquids, weight loss (.10 kg), ter (LES) relaxation as well as loss of peristalsis in the
and anorexia (loss of appetite). distal oesophagus, which is mostly smooth muscle.
l Oesophageal cancer usually affects the elderly, they can l Both of these features impair the ability of the oesopha-
be either squamous cell carcinoma or adenocarcinoma. gus to empty contents into the stomach.
Section | II General Surgery 329

l Patients usually complain of dysphagia to both solids l Palpate the landmarks (e.g. thyroid notch, sternal notch,
and liquids. Dysphagia to liquids, in particular, is a cricoid cartilage), and mark them with an ink pen. Plan
characteristic of achalasia. a 3-cm vertical incision that extends inferiorly from the
l Other symptoms of achalasia include regurgitation, cricoid cartilage and infiltrate lidocaine (1%) with
night coughing, chest pain, weight loss, and heartburn. 1:1,50,000 parts epinephrine. Make the vertical incision.
l Other conditions like diverticulum, cervical osteo- l Horizontal skin incision, is also recommended as they
phytes, enlarged aorta, mediastinal tumour can also are made along relaxed skin tension lines and gives
cause obstruction of oesophagus. better cosmetics. A horizontal incision may trap
more secretions. Meticulous haemostasis is important
Q. 3. Mention the indications for tracheostomy.
throughout, beginning with the skin edges.
Describe the steps of operation.
l Subcutaneous fat may be removed with electrocautery
Ans. to aid in exposure and to prevent later fat necrosis. Dis-
section proceeds through the platysma until the midline
Tracheostomy is an operative procedure that creates a
raphe between the strap muscles is identified.
surgical airway in the cervical trachea.
l Palpate the inferior limit of the field to assess the proxim-
Indication for tracheostomy are as follows: ity of the innominate artery. Cauterize or ligate aberrant
i. To bypass obstruction: anterior jugular veins and smaller vessels. The strap
l Congenital anomaly (e.g. laryngeal hypoplasia, muscles are separated and retracted laterally, exposing
vascular web) the pretracheal fascia and the thyroid isthmus. Elevate the
l Foreign body that cannot be dislodged with isthmus off the trachea with a haemostat and divide it.
Heimlich and basic cardiac life support (BCLS) l When preparations for transfer of circuitry tubes are
manoeuvres. complete, deflate the endotracheal tube balloon and en-
l Supraglottic or glottic pathologic condition (e.g. ter the trachea. Injection of topical anaesthesia can stem
infection, neoplasm, bilateral vocal cord paralysis) the cough reflex of an awake patient. Absolute haemo-
ii. Neck trauma that results in severe injury to the thyroid stasis before this point obviates the threat that blood
or cricoid cartilages, hyoid bone, or great vessels. could enter the trachea and exacerbate the cough reflex.
iii. Subcutaneous emphysema. l Securing the cricoid with a hook and elevating it supe-
iv. Facial fractures that may lead to upper airway obstruc- riorly facilitates control of the tracheal entry. Several
tion (e.g. comminuted fractures of the midface and options for the tracheal stoma are available, including
mandible) the following:
v. Oedema associated with: a. T-shaped tracheal opening: Make a 2-cm incision
l Trauma horizontally through the membrane between the
l Burns second and third or third and fourth tracheal rings.
l Infection b. U- or H-shaped tracheal opening: Reflect tracheal
l Anaphylaxis flaps inferiorly or both inferiorly and superiorly. This
vi. To provide a long-term route for mechanical ventila- is beneficial in the patient with obesity.
tion in cases of respiratory failure c. Permanent stoma: Create a permanent stoma with
vii. To provide pulmonary toilet: skin flaps developed and sutured to a rectangular
l Inadequate cough due to chronic pain or weakness tracheal opening. This is desirable only in patients
l Aspiration and the inability to handle secretions. who are expected to require secure transluminal
This intervention can prevent aspiration and pro- access indefinitely.
vide for the removal of any aspirated substances. l After the trachea is entered, suction secretions and
viii. Prophylaxis (as in preparation for extensive head and blood out of the lumen and slowly withdraw the endo-
neck procedures and the convalescent period) tracheal tube to a point just proximal to the opening.
ix. Severe sleep apnoea not amendable to continuous Replace the lateral retractors into the trachea and insert
positive airway pressure (CPAP) devices or other, less the previously tested tracheostomy tube.
invasive surgery. l After the airway is confirmed intact based on carbon
dioxide return and bilateral breath sounds, secure the
tracheostomy tube to the skin with 4-0 permanent
Tracheostomy Procedure sutures.
l Position the unconscious or anaesthetized patient supine l Attach a tracheostomy collar with the head flexed to
with the neck extended and the shoulders elevated on a avoid unnecessary slack in the collar.
small roll. In case the patient is conscious, the procedure l To avoid the risk of subcutaneous emphysema and
is performed in a sitting or semirecumbent position. subsequent pneumomediastinum, the skin is not closed.
330 Quick Review Series for BDS 3rd Year

l Place a sponge soaked with iodine or petrolatum gauze l Additional mucolytic agents (e.g. acetylcysteine [muco-
between the skin and the flange for 24 hours to deflect myst], guaifenesin) may be used. If uncorrected, mucus
infection and anxiety about minor oozing of the skin edge. that plugs the inner cannula can cause a life-threatening
obstruction.
Q. 4. Discuss the indications, postoperative manage- l The original tube is left sutured in place for 5-7 days to
ment and complications of tracheostomy. allow the tract to heal. The sutures are then removed,
Ans. and the tube is replaced.
l For patients in whom the tracheostomy was an acute
Tracheostomy is an operative procedure that creates a intervention, this is an opportunity to downsize the tube
surgical airway in the cervical trachea. or to change to a metal (Jackson) tube. The site should
Indication for tracheostomy are as follows: be kept clean and dry to minimize infection from what
i. To bypass obstruction: is a chronically colonized location. Patient and family
l Congenital anomaly (e.g. laryngeal haypoplasia, education should begin as soon as possible.
vascular web)
l Foreign body that cannot be dislodged with Heimlich

and basic cardiac life support (BCLS) manoeuvres. Complications of Tracheostomy


l Supraglottic or glottic pathologic condition (e.g.
A) Immediate complications of tracheostomy
infection, neoplasm, bilateral vocal cord paralysis)
ii. Neck trauma that results in severe injury to the thyroid l Apnoea due to loss of hypoxic respiratory drive:
or cricoid cartilages, hyoid bone, or great vessels. l Bleeding: Intraoperative bleeding arises from the
iii. Subcutaneous emphysema cut edges of the very vascular thyroid gland and from
iv. Facial fractures that may lead to upper airway obstruc- lacerated vessels in the field that should be cauterized or
tion (e.g. comminuted fractures of the midface and ligated.
mandible) l Pneumothorax or pneumomediastinum: These can
v. Oedema associated with: result from direct injury to the pleura or the cupola of
l Trauma the lung (especially in children) or from high negative
l Burns inspiratory pressures of patients who are awake and
l Infection distressed.
l Anaphylaxis l Injury to adjacent structures: The paratracheal struc-
vi. To provide a long-term route for mechanical ventila- tures vulnerable to injury are the recurrent laryngeal
tion in cases of respiratory failure. nerves, the great vessels, and the oesophagus.
vii. To provide pulmonary toilet: l Postobstructive pulmonary oedema.
l Inadequate cough due to chronic pain or weakness l Endotracheal tube ignition: This rare complication is
l Aspiration and the inability to handle secretions. associated with opening the trachea with electrocautery
This intervention can prevent aspiration and pro- or laser.
vide for the removal of any aspirated substances.
viii. Prophylaxis (as in preparation for extensive head and B) Early complications of tracheostomy:
neck procedures and the convalescent period) l Early bleeding: This is usually the result of increased
ix. Severe sleep apnoea not amendable to continuous blood pressure as the patient emerges from anaesthesia
positive airway pressure (CPAP) devices or other, less (and relative hypotension) and begins to cough.
invasive surgery l Plugging with mucus
l Tracheitis
l Cellulitis
Postoperative Management l Displacement
l Postoperative care is critical. The recently insulted tra- l Subcutaneous emphysema: This results from a tight clo-
chea produces copious secretions, and irrigation with sure of tissue around the tube, tight packing material
saline and suctioning every 15 minutes are not initially around the tube, or false passage of the tube into pretra-
unreasonable. cheal tissue. It can progress to pneumothorax, pneumome-
l Suctioning should be limited to the length of the tube to diastinum, or both and should be treated with loosening of
avoid tracheal ulceration and tracheitis and should be the closure or packing and with performance of a tube
limited to no more than 15 seconds because the act of thoracotomy.
suctioning blocks the airway and sucks the air out of the l Atelectasis: An overly long tube can mimic a unilateral
lungs. Humidified oxygen helps to prevent inspissation mainstem intubation, causing atelectasis or collapse of
of the secretions. the opposite lung.
Section | II General Surgery 331

C) Late complications of tracheostomy l Tracheo-oesophageal fistula


l Bleeding l Tracheocutaneous fistula
l Tracheomalacia l Granulation

SHORT ESSAYS
Q. 1. Dysphagia. l Direct strategy usually refers to treatment that involves
food, whereas indirect strategy refers to an exercise
Ans. regimen performed without food bolus.
l Dysphagia typically refers to difficulty in eating as l Direct techniques include modifications of food
a result of disruption in the swallowing process consistency.
and can represent a serious threat to one’s health be- l Indirect techniques include stimulation of the oropha-
cause of the risk of aspiration pneumonia, malnutri- ryngeal structures and the adoption of behavioural
tion, dehydration, weight loss, and airway obstruction. techniques, such as those involving postural changes or
l Disorders leading to dysphagia may affect the oral, the swallow manoeuvre.
pharyngeal, or oesophageal phases of swallowing. Q. 2. Indications for tracheostomy.
Ans.
Pathophysiology
Tracheostomy is an operative procedure that creates a
Dysphagia may be categorized according to the swallowing surgical airway in the cervical trachea.
phase affected. A number of dysphagic problems can be Indication for tracheostomy are as follows:
identified during each phase of deglutition. i. To bypass obstruction:
l Congenital anomaly (e.g. laryngeal hypoplasia,

vascular web)
Clinical Features l Foreign body that cannot be dislodged with

Oral or pharyngeal dysphagia Heimlich and basic cardiac life support (BCLS)
manoeuvres.
l Coughing or choking with swallowing
l Supraglottic or glottic pathologic condition (e.g.
l Difficulty initiating swallowing
infection, neoplasm, bilateral vocal cord paralysis)
l Food sticking in the throat
ii. Neck trauma that results in severe injury to the thyroid
l Sialorrhoea
or cricoid cartilages, hyoid bone, or great vessels.
l Unexplained weight loss
iii. Subcutaneous emphysema
l Change in dietary habits
iv. Facial fractures that may lead to upper airway obstruc-
l Recurrent pneumonia
tion (e.g. comminuted fractures of the midface and
l Change in voice or speech (wet voice)
mandible)
l Nasal regurgitation
v. Oedema associated with:
l Trauma
Oesophageal dysphagia
l Burns
l Sensation of food sticking in the chest or throat l Infection
l Oral or pharyngeal regurgitation l Anaphylaxis
l Change in dietary habits vi. To provide a long-term route for mechanical ventila-
l Recurrent pneumonia tion in cases of respiratory failure.
vii. To provide pulmonary toilet:
l Inadequate cough due to chronic pain or weakness
Diagnosis
l Aspiration and the inability to handle secretions.
The gold-standard for diagnosing oropharyngeal dysphagia This intervention can prevent aspiration and
is a modified barium swallow study or videofluoroscopic provide for the removal of any aspirated
swallow study (fluoroscopy). substances.
viii. Prophylaxis (as in preparation for extensive head and
neck procedures and the convalescent period)
Treatment
ix. Severe sleep apnoea not amendable to continuous
In treatment of dysphagia, direct and indirect strategies for positive airway pressure (CPAP) devices or other, less
treating dysphagia have been described. invasive surgery.
332 Quick Review Series for BDS 3rd Year

Q. 3. Give in brief note on post operative complications l RPA can lead to airway obstruction or sepsis, both
of tracheostomy. life-threatening emergencies.
l RPA is usually caused by a bacterial infection originat-
Ans.
ing from the nasopharynx, tonsils, sinuses, adenoids or
middle ear. Any upper respiratory infection (URI) can
Complications of Tracheostomy be a cause.
RPA can also result from a direct infection due to
A) Immediate complications of tracheostomy l

penetrating injury or a foreign body.


l Apnoea due to loss of hypoxic respiratory drive.
l Bleeding: Intraoperative bleeding arises from the cut edges
of the very vascular thyroid gland and from lacerated Clinical Features
vessels in the field that should be cauterized or ligated. l Symptoms may include stiff neck (limited neck mobil-
l Pneumothorax or pneumomediastinum: These can result ity or torticollis) some form of palpable neck pain,
from direct injury to the pleura or the cupola of the lung malaise, difficulty swallowing, fever, stridor, drooling,
(especially in children) or from high negative inspiratory or enlarged cervical lymph nodes. Any combination of
pressures of patients who are awake and distressed. these symptoms should arouse suspicion of RPA.
l Injury to adjacent structures: The paratracheal struc-
tures vulnerable to injury are the recurrent laryngeal
nerves, the great vessels, and the oesophagus. Treatment
l Postobstructive pulmonary oedema. l RPAs frequently require surgical intervention.
l Endotracheal tube ignition: This rare complication is l A tonsillectomy approach is typically used to access/
associated with opening the trachea with electrocautery drain the abscess, and the outcome is usually positive.
or laser. l In complex cases, tracheotomy may be required to
prevent upper airway obstruction caused by oedema in
B) Early complications of tracheostomy
the neck.
l Early bleeding: This is usually the result of increased l Antibiotics are also given.
blood pressure as the patient emerges from anaesthesia
(and relative hypotension) and begins to cough. Q. 5. Quinsy.
l Plugging with mucus Ans.
l Tracheitis
l Cellulitis l Quinsy or peritonsillar abscess refers to a complication
l Displacement of tonsillitis and consists of a collection of pus beside
l Subcutaneous emphysema: This results from a tight closure the tonsil (peritonsillar space).
of tissue around the tube, tight packing material around the l Peritonsillar space is anatomically contiguous with
tube, or false passage of the tube into pretracheal tissue. several deeper spaces, and infections can potentially
It can progress to pneumothorax, pneumomediastinum, involve the parapharyngeal and retropharyngeal spaces.
or both and should be treated with loosening of the closure l Quinsy usually progresses from tonsillitis to cellulitis
or packing and with performance of a tube thoracotomy. and ultimately to abscess formation.
l Atelectasis: An overly long tube can mimic a unilateral
mainstem intubation, causing atelectasis or collapse of
Clinical Features
the opposite lung.
History
C) Late complications of tracheostomy
Symptoms of quinsy usually begin 3-5 days prior to
l Bleeding evaluation.
l Tracheomalacia l Fever
l Tracheo-oesophageal fistula l Malaise
l Tracheocutaneous fistula l Headache
l Granulation l Neck pain
Q. 4. Retropharyngeal abscess. l Throat pain markedly more severe on the affected side
and occasionally referred to the ipsilateral ear
Ans.
l Dysphagia
l Retropharyngeal abscess (RPA) is an abscess located in l Change in voice
the tissues in the back of the throat behind the posterior l Otalgia
pharyngeal wall (the retropharyngeal space). l Odynophagia
Section | II General Surgery 333

Physical evaluation l Contralateral deviation of the uvula


Physical findings of peritonsillar abscess include the l Erythema of the tonsil
following: l Exudates on the tonsil
l Mild/moderate distress
Fever
l
Treatment
l Tachycardia
l Dehydration l Needle aspiration should be performed to drain the
l Drooling, salivation, trouble handling oral secretions abscess and should provide moderate pain relief.
l Trismus resulting from pain from inflammation and l Larger abscesses may require incision and drainage.
spasm of masticator muscles l An empiric antibiotics should be administered. Clinda-
l Hot potato/muffled voice mycin is the preferred drug. Antipyretics and analgesic
l Rancid or fetor breath are recommended.
l Cervical lymphadenitis in the anterior chain l If the patient’s airway is compromised, he or she needs
l Asymmetric tonsillar hypertrophy immediate endotracheal intubation. If this cannot be
l Localized fluctuance completed, then a cricothyroidotomy or a tracheotomy
l Inferior and medial displacement of the tonsil may need to be performed.

SHORT NOTES
Q. 1. Tonsillitis. distress, and high WBC counts. Cough is frequent and
not painful.
Ans. l The prodrome is usually an upper respiratory infection,
i. Tonsillitis is inflammation of the pharyngeal tonsils. The followed by progression to higher fever, cough, inspira-
inflammation usually extends to the adenoid and the tory stridor, and a variable degree of respiratory distress.
lingual tonsils. l Patients may acutely decompensate with worsening
ii. Clinical features: respiratory distress due to airway obstruction from a
l Red and/or swollen tonsils, white or yellow patches purulent membrane that has loosened.
on the tonsils, tender, stiff, and/or swollen neck, sore
throats painful or difficulty in swallowing.
l Cough, headache, sore eyes, body aches, otalgia, Treatment
fever, chills and nasal congestions. l In minor bacterial cases, it can be simply treated with a
l Acute tonsillitis is caused by both bacteria and course of antibiotics.
viruses and will be accompanied by symptoms of ear l In more severe cases, it is treated by admission to an
pain when swallowing, bad breath, and drooling intensive care unit (ICU) and intubation is performed.
along with sore throat and fever.
l In this case, the surface of the tonsil may be bright Q. 3. Pharyngitis.
red or have a grayish-white coating, while the lymph
nodes in the neck may be swollen. Ans.
l Pharyngitis is an infection or irritation of the pharynx
Q. 2. Tracheitis.
and/or tonsils.
Ans. l The aetiology is usually infectious, mostly of viral ori-
gin while bacterial pharyngitis is self-limiting, but more
l Tracheitis is defined as inflammation of trachea. concerning due to suppurative and nonsuppurative
l In ICD-10, tracheitis is classified under “acute upper complications. Other causes include allergy, trauma,
respiratory infections”. toxins, and neoplasia.
l Bacterial tracheitis is a bacterial infection of the trachea, l The most significant bacterial agent causing pharyngitis
mostly caused by Staphylococcus aureus and is capable in both adults and children is GAS infection (Strepto-
of producing airway obstruction. coccus pyogenes).
l Pharyngitis can be acute or chronic.
Pharyngitis can result in enlarged tonsils which cause
Clinical Features l

trouble swallowing and breathing.


l In the classic presentation, patients present acutely with l If pharyngitis is caused by a systemic infection, it can
fevers, toxic appearance, stridor, tachypnoea, respiratory be accompanied by a cough or fever.
334 Quick Review Series for BDS 3rd Year

Q. 4. Arterial supply of tonsils. inferiorly from the cricoid cartilage or a horizontal skin
incision, is made along relaxed skin tension lines and
Ans.
gives better cosmetics. Attach a tracheostomy collar.
Arterial supply of tonsils: iv. To avoid the risk of subcutaneous emphysema and
Each tonsil is supplied by 5 arteries. subsequent pneumomediastinum, the skin is not closed.
l Dorsal linguae branches of lingual artery v. Place a sponge soaked with iodine between the skin and
l Tonsillar branch of facial artery the flange for 24 hours to deflect infection and anxiety
l Ascending palatine branch of facial artery about minor oozing of the skin edge. Postoperative care
l Ascending pharyngeal artery from external carotid is critical.
l Descending palatine branch of maxillary artery
Q. 8. Endotracheal intubation.
Q. 5. Collar-stud abscess.
Ans.
Ans.
i. Endotracheal intubation refers to placement of flexible
l A collar stud abscess is an acute suppurative infection plastic tube into the trachea to maintain an open airway
of a digit presenting as a stud-like blister. or to administer certain drugs.
l The abscess tracks to deeper tissue; cold abscess tracks ii. It is performed in critically injured/ill or anaesthetized
through the neurovascular bundle that pierce the deep patients to provide ventilation, including mechanical
fascia and enters the subcutaneous plane to form the ventilation and to prevent the possibility of asphyxiation/
abscess. airway obstruction.
l Simple incision of the blister does not resolve matters. iii. The most widely used route is orotracheal, in which
The finger must be explored with identification of the an endotracheal tube is passed through the mouth
deep collection and delayed closure. and vocal apparatus into the trachea. In a nasotracheal
l Systemic antibiotics are recommended. procedure, an endotracheal tube is passed through the
nose and vocal apparatus into the trachea.
Q. 6. Pharyngocele. iv. Other methods of intubation involve surgery and
Ans. include the cricothyrotomy and the tracheotomy, used
primarily in situations where a prolonged need for
l Pharyngocele refers to the lateral pharyngeal wall her- airway support is anticipated.
niation located in the pyriform recess or at the vallecula.
l It is considered as a rare condition and the patient Q. 9. Nasogastric intubation.
complains of dysphagia, lateral cervical mass which
Ans.
increases in size with the Valsalva manoeuvre, and other
symptoms of the upper digestive way. i. Nasogastric intubation refers to insertion of a nasogas-
l The elective diagnostic study is pharyngo-oesophageal tric tube, through the nose, into the gaster.
swallow. ii. It is used for feeding purposes and administration of
drugs and other oral agents.
Q. 7. Tracheostomy. iii. Contraindication for NG intubation:
Ans. l Base skull fractures

l Severe facial fractures especially to the nose and


i. Tracheostomy is an operative procedure that creates a obstructed oesophagus
surgical airway in the cervical trachea. l Oesophageal varices, and/or obstructed airway.
ii. Indication of tracheostomy: l Gastric bypass surgery patients.
l To bypass obstruction
l Neck trauma that results in severe injury to the Q. 10. Quinsy.
thyroid or cricoid cartilages, hyoid bone, or great
Ans.
vessels.
l Subcutaneous emphysema l Quinsy or peritonsillar abscess refers to a complication
l Facial fractures that may lead to upper airway of tonsillitis and consists of a collection of pus beside
obstruction the tonsil (peritonsillar space).
l To provide a long-term route for mechanical ventila- l Peritonsillar space is anatomically contiguous with
tion in cases of respiratory failure, etc. several deeper spaces, and infections can potentially
iii. Position the patient with the neck extended and the involve the parapharyngeal and retropharyngeal spaces.
shoulders elevated on a small roll and palpate the land- l Quinsy usually progresses from tonsillitis to cellulitis
marks and place a 3-cm vertical incision that extends and ultimately to abscess formation.
Section | II General Surgery 335

Q. 11. Dysphagia. Q. 12. Retropharyngeal abscess.


Ans. Ans.
l Dysphagia is defined as difficulty in swallowing. It is associ- i. Retropharyngeal abscess refers to the abscess which is
ated with obstructive or motor disorders of the oesophagus. located in the tissues in the back of the throat behind the
l Patients with obstructive disorders such as oesophageal posterior pharyngeal wall, i.e. the retropharyngeal
tumour or lower oesophageal ring are unable to swallow space.
solids but can tolerate liquids. ii. Retropharyngeal abscess is mostly caused by bacterial
l Persons with motor disorders, such as achalasia, are infections of nasopharynx, sinuses, tonsils, adenoids.
unable to swallow solids or liquids. They are also formed due to direct infection due to
l Diagnosis of the underlying condition is made through penetrating injury or a foreign body.
barium studies, the observed clinical signs, and evalua- iii. Clinical features include stiff neck, neck pain, malaise,
tion of the patient’s symptoms. dysphagia, fever, stridor, drooling, enlarged cervical
l Oesophageal dysphagia is caused by an abnormality in lymph nodes. Any combination of these symptoms
the oesophagus, such as a smooth muscle disorder that should arouse suspicion of RPA.
interferes with peristalsis or an obstruction from exter-
nal compression or a stricture.
Treatment
l Oropharyngeal dysphagia is caused by difficulty in ini-
tiating the swallowing process, so that solids and liquids l Incision and drainage of abscess.
cannot move out of the mouth properly.

Topic 10

Diseases of Arteries, Veins and the Lymphatic System

LONG ESSAYS
Q. 1. Describe the clinical features, diagnosis and treat- Pathophysiology:
ment of thromboangiitis obliterans (Buerger’s disease).
l The mechanism underlying Buerger disease is unclear.
Ans. l An immunologic phenomenon that leads to vasodys-
function and inflammatory thrombi is suspected.
Thromboangiitis obliterans (also known as Buerger’s dis-
l Patients with the disease show hypersensitivity to intra-
ease) is a recurring progressive segmental occlusive inflam-
dermally injected tobacco extracts, have increased
matory condition with thrombosis and re-canalization of
cellular sensitivity to types I and III collagen, have
small- and medium-sized arteries and veins of the upper
elevated serum anti-endothelial cell antibody titers, and
and lower extremities.
have impaired peripheral vasculature endothelium-
dependent vasorelaxation.
Aetiology l Increased prevalence of HLA-A9, HLA-A54, and
HLA-B5 is observed in these patients, which suggests
l Unknown but, it has strong association with tobacco use
a genetic component to the disease.
or exposure.

Epidemiology Clinical Features


l Buerger’s disease is more common among men than women. The typical presentations of Buerger’s disease are:
l It is more common in Israel, Japan, India, and Manipur l Claudication of affected area. As the condition pro-
along the “old silk route”. gresses, ischemic rest pain appears.
336 Quick Review Series for BDS 3rd Year

l Paraesthesias (numbness, tingling, burning, hypoaesthe- l The following strategies are important in prevention of
sia) of the feet and hands and impaired distal pulses in complications from Buerger disease:
the presence of normal proximal pulses. l Use of well-fitting protective footwear to prevent

l Unremitting ischaemic ulcerations. foot trauma and thermal or chemical injury.


l Gangrene of the digits of hands and feet. The disease l Early and aggressive treatment of extremity injuries

evolves; the patients may require several surgical ampu- to protect against infections.
tations. l Avoidance of cold environments.

l Although classic to affecs the vessels of the extremities, l Avoidance of drugs that lead to vasoconstriction.

a few cases of aortic, cerebral, coronary, iliac, mesen-


teric, pulmonary, and renal thromboangiitis obliterans Surgical care
have been reported. The proposed surgical treatments for Buerger disease are as
follows:
Omental transfer
Diagnosis l

l Sympathectomy
Several diagnostic criteria have been proposed but the l Spinal cord stimulator implantation
commonly followed diagnostic criteria are outlined below l Ultimate surgical therapy for refractory Buerger disease
although the criteria tend to differ slightly from author to author. is distal limb amputation for non-healing ulcers,
gangrene, or intractable pain.
Diagnosis criteria of Olin (2000)
Q. 2. Define gangrene, describe the types, clinical
l Typically between 20 and 40 years old and male, al-
manifestations and management of them.
though recently females have been diagnosed.
l Current (or recent) history of tobacco use. Ans.
l Presence of distal extremity ischaemia (indicated by
l Gangrene is defined as the necrosis of an organ or tissue
claudication, pain at rest, ischaemic ulcers or gangrene)
caused by insufficient blood supply, i.e. ischaemia.
documented by non-invasive vascular testing such as
l The term gangrene derives from the Latin word
ultrasound.
“gangraena” and from the Greek gangraina which
l Exclusion of other autoimmune diseases, hypercoagu-
means “putrefaction of tissues”.
lable states, and diabetes mellitus by laboratory tests.
l It is a complication resulting from infectious or inflam-
l Exclusion of a proximal source of emboli by echocar-
matory processes, injury, or degenerative changes asso-
diography and arteriography.
ciated with chronic diseases, such as diabetes mellitus.
l Consistent arteriographic findings in the clinically
involved and non-involved limbs.
Aetiology
No specific laboratory tests confirm or exclude the diag-
nosis of Buerger disease. The primary goal of a laboratory The following conditions are risk factors for the develop-
workup in patients is to exclude other disease processes in ment of gangrene:
the differential diagnosis. Imaging studies like angiography l Injury or trauma, such as a crush injury, a severe burn,
and echocardiography are helpful. or frostbite.
l Diseases that affect the peripheral circulation of
blood, such as arteriosclerosis, diabetes, smoking, or
Treatment Raynaud’s disease.
Medical care l Infection of wounds.
l Absolute discontinuation of tobacco use is the only
strategy proven to prevent the progression of Buerger Types of Gangrene
disease.
l Treatment with intravenous iloprost (a prostaglandin I. Dry gangrene
analogue), has been shown to be somewhat effective in l Dry gangrene is a condition that results when one or
improving symptoms, accelerating resolution of distal more arteries become obstructed.
extremity trophic changes, and reducing the amputation l In this type of gangrene, the tissue slowly dies because
rate. of inadequate or no blood supply.
l The use of thrombolytic therapy in the treatment of l This occurs mostly in the extremities and it may
Buerger disease has been proposed, but the data for this develop in people with diabetes or arteriosclerosis. It
treatment remain inconclusive and the treatment is thus may also develop after prolonged exposure to freezing
considered experimental. temperatures.
Section | II General Surgery 337

l In dry gangrene, the tissue first becomes bluish and they l Gas gangrene can cause necrosis, gas production, and
feel cold to the touch. As time progresses, a line of de- sepsis. Progression to toxaemia and shock is often very
marcation appears between the healthy and devitalized rapid.
tissue, which becomes dry and dark black. l Gas gangrene is often treated with the antitoxin for
l The dark colouration is due to liberation of haemoglobin Clostridium. In a number of cases, amputation may
from haemolyzed red blood cells, which is acted upon by have to be used to keep the infection under control.
hydrogen sulfide (H2S) produced by the bacteria, resulting l Severe cases have been treated by keeping the patient in
in formation of black iron sulfide that remains in the tissues. an oxygen-rich atmosphere, as in a hyperbaric chamber.
l Eventually, there may be a separation of the dead tissue
from the living tissue; with spontaneous amputation IV. Other gangrenes
(autoamputation) of the involved extremity. l Noma is a gangrene of the face.
l Treatment of this type of gangrene is aimed at improving l Necrotizing fasciitis affects the deeper layers of the
circulation, revascularization (i.e. restoration of blood skin.
flow) to the affected area. This may be accomplished with l Fournier gangrene usually affects the male genitals and
drugs or through the surgical removal of the obstruction. groin.

II. Wet gangrene Q. 3. Classify gangrene. Discuss the clinical features and
management of diabetic gangrene.
l Wet gangrene is gangrene which develops as a compli-
cation of an untreated infected wound, caused by certain Ans.
bacterial infection. l Gangrene is defined as the necrosis of an organ or tissue
l The tissue is infected by saprogenic microorganisms caused by insufficient blood supply, i.e. ischaemia.
(Bac. perfringens, fusiformis, putrificans, etc.), which l It is a complication resulting from infectious or inflam-
cause tissue to swell and emit a fetid smell. matory processes, injury, or degenerative changes asso-
l Wet gangrene usually develops rapidly due to blockage ciated with chronic diseases, such as diabetes mellitus.
of venous and/or arterial blood flow. The affected part is
saturated with stagnant blood, which promotes the rapid The gangrene is classified into following types:
growth of bacteria. I. Dry gangrene: Dry gangrene is a condition that results
l The toxic products formed by bacteria are absorbed when one or more arteries become obstructed. In this
causing systemic manifestation of septicaemia and type of gangrene, the tissue slowly dies because of
finally death. inadequate or no blood supply. This occurs mostly in
l Wet gangrene occurs in naturally moist tissue and or- the extremities and it may develop in people with dia-
gans such as the mouth, bowel, lungs, cervix, and vulva. betes or arteriosclerosis.
l Bedsores occurring on body parts such as the sacrum, II. Wet gangrene: Wet gangrene is gangrene which devel-
buttocks, and heels — although not necessarily moist ops as a complication of an untreated infected wound,
areas — are also categorized as wet gangrene infections. caused by certain bacterial infection. The tissue is in-
l The affected part is oedematous, soft, putrid, rotten and fected by saprogenic microorganisms (Bac. perfrin-
dark. The darkness in wet gangrene occurs due to the gens, fusiformis, putrificans, etc.), which cause tissue
same mechanism as in dry gangrene. to swell and emit a fetid smell.
l Administration of antibiotics and sometimes the surgi- III. Gas gangrene: Gas gangrene is a bacterial infection
cal removal of the dead tissue to keep the infection from that produces gas within tissues. It is caused by bacte-
spreading is the treatment. rial exotoxin-produced by Clostridium perfringens
which are mostly found in soil and other anaerobes
III. Gas gangrene (e.g. Bacteroides and anaerobic streptococci).
l Gas gangrene is a bacterial infection that produces gas IV. Other gangrenes:
within tissues. It is caused by bacterial exotoxin- l Noma is a gangrene of the face.

produced by Clostridium perfringens which are mostly l Necrotizing fasciitis affects the deeper layers of the

found in soil and other anaerobes (e.g. Bacteroides and skin.


anaerobic streptococci). l Fournier gangrene usually affects the male genitals

l Infection spreads rapidly as the gases produced by bac- and groin.


teria expand and infiltrate healthy tissue in the vicinity.
l After an incubation period of one to four or five days, DIABETIC GANGRENE
the affected tissue is swollen, painful and cold. A wa-
tery, brownish, foul-smelling fluid drains from the l Diabetic gangrene is classified under as dry gangrene in
wound, and little bubbles of gas develop in the tissues. which the lower extremities are most affected.
338 Quick Review Series for BDS 3rd Year

l Diabetic foot gangrene is a result of the compromised Pathophysiology


blood circulation, an insufficient oxygen-rich and nutrient-
l Incompetence in the superficial venous system alone usu-
dense blood supply usually caused by arterial—femoral,
ally results from failure at valves located at the SFJ and SPJ.
popliteal or tibial—obstruction.
l The gravitational weight of the column of blood along
l The lumen of the artery becomes progressively narrowed
the length of the vein creates hydrostatic pressure, which
up to the point of complete occlusion (blockage), causing
is worse at the more distal aspect of the length of vein.
normal blood flow to stop. In other words, gangrene de-
l Incompetence of the perforating veins leads to hydrody-
velops, if the blood supply deteriorates to a stage where
namic pressure. The calf pump mechanism helps to empty
insufficient blood is available to keep the tissues alive.
the deep venous system, but if perforating vein valves fail,
then the pressure generated in the deep venous system by
Clinical Features and Management the calf pump mechanism are transmitted into the superfi-
of Diabetic Gangrene cial system via the incompetent perforating veins.

l In initial stages of diabetic dry gangrene, the tissue first


becomes bluish, with dull, aching pain, the affected area Stages
being extremely painful to touch and cold. l C0 no visible or palpable signs of venous disease
l As time progresses, a line of demarcation appears l C1 telangiectasia or reticular veins
between the healthy and devitalized tissue, which l C2 varicose veins
becomes dry and dark black. l C3 oedema
l Eventually, there may be a separation of the dead tissue l C4a skin changes due to venous disorders: pigmentation,
from the living tissue; with spontaneous amputation eczema
(autoamputation) of the involved extremity. l C4b skin changes due to venous disorders: lipodermato-
l Treatment of this type of gangrene is aimed at improv- sclerosis, atrophie blanche
ing circulation, revascularization (i.e. restoration of l C5 as C4 but with healed ulcers
blood flow) to the affected area. This may be accom- l C6 skin changes with active ulcers (venous insuffi-
plished with drugs or through the surgical removal of ciency ulceration)
the obstruction.
l Standard medical approach to diabetic foot gangrene
frequently leads to the “cut-and-medicate” treatment, Clinical Features
usually resulting in:
l Visible distension of superficial veins.
i. Debridement—procedures that involve the surgical
l Painful, achy, or “heavy” legs, oedema (often worse by
removal of devitalized tissues, and/or
the end of day/evening and after exercise).
ii. Amputations—procedure involving the surgical
l Burning, swelling, throbbing, cramping of gastrocne-
removal of entire parts of the body (toe, foot, or leg).
mius muscles.
Q. 4. Describe the symptoms, signs and treatment of l Leg fatigue.
varicose veins of leg.
Ans. Signs of Venous Insufficiency
l Varicose veins are tortuous, dilated, prominent superfi- l Varicose eczema
cial veins in the lower limbs, often in the anatomical l Hemosiderin staining / pigmentation
distribution of the long and short saphenous veins. l Atrophie blanche
l Varicose veins actually represent underlying chronic l Lipodermatosclerosis
venous insufficiency with ensuing venous hypertension. l Oedema
l Ulceration (trophic ulcers in lower one-third and medial
malleolus).
Aetiology
Due to genetic or developmental weakness in the vein
l
Diagnosis
wall resulting in diminished elasticity, dilation over time
and valvular incompetence. l Duplex ultrasound
l The cause of primary varicose veins is incompetent l Trendelenburg test: This physical examination tech-
venous valves that result in venous hypertension. nique distinguishes patients with reflux at the SFJ from
l Secondary varicose veins result from deep venous throm- those with incompetent deep venous valves.
bosis and its sequelae or congenital anatomic abnormalities. l Doppler auscultation
Section | II General Surgery 339

Treatment l Tuberculous lymphadenitis is defined as a chronic


specific granulomatous inflammation with caseation
A. Conservative necrosis of the lymph node.
l Elevation of lower extremities. l TB lymphadenitis refers to involvement of lymph nodes
l Compression stockings with variable pressure gradients by members of the M. tuberculosis complex which in-
(class II or III) clude M. tuberculosis, M. bovis, M. africanum, M. canetti
l Diosmin/hesperidine and other flavonoids and M. caprae.
l Anti-inflammatory medication such as ibuprofen or l Tuberculous lymphadenitis is among the most frequent
aspirin can be used as part of treatment for superficial presentations of extrapulmonary tuberculosis (TB).
thrombophlebitis along with graduated compression l The characteristic morphological element is the tubercu-
hosiery – but there is a risk of intestinal bleeding. lous granuloma (caseating tubercule): Giant multinucle-
l In extensive superficial thrombophlebitis, consideration ated cells (Langhans cells), surrounded by epithelioid
should be given to anticoagulation, thrombectomy or cell aggregates, T cell lymphocytes and few fibroblasts.
sclerotherapy of the involved vein. l Granulomatous tubercules evolve to central caseous
necrosis and tend to become confluent, replacing the
B. Medical intervention lymphoid tissue.
Medical intervention in varicose veins can be divided into l The most common site of tuberculous lymphadenitis is
surgical and non-surgical treatments. in the neck along the sternocleidomastoid muscle.
l It is usually unilateral and causes little or no pain.
I. Surgical: Advanced cases of tuberculous lymphadenitis may
i. Saphenous stripping: Stripping consists of removal suppurate and form a draining sinus.
of all or part the saphenous vein main trunk. The
complications include deep vein thrombosis, pulmo-
nary embolism, and wound complications including
Stages
infection. Pathologically, active tuberculous lymphadenitis passes
ii. CHIVA: CHIVA is an acronym from the scientific pa- through four stages:
per “Conservatrice et Hémodynamique de l’Insuffisance l Stage 1 is lymphoid hyperplasia, with formation of tu-
Veineuse en Ambulatoire” published in France in 1988. bercles and granulomas without caseation necrosis. The
It generally consists of 1 to 4 small incisions under lymph nodes are hard and as big as a bean, movable and
local anaesthesia in order to disconnect the varicose covered with normal skin without hotness or tenderness.
veins from the abnormal flow due to valvular incompe- l Stages 2 and 3 are caseation necrosis in the affected
tence which dilates them. lymph nodes followed by destruction of capsules of
iii. Ambulatory phlebectomy lymph nodes and adherence of multiple nodes with peri-
iv. Vein ligation adenitis. The lymph nodes are bigger and fused together
v. Cryosurgery to form a mass, immovable and adhere to the skin; after
putrefaction, the skin becomes dark red in colour and
II. Non-surgical treatment slightly hot, and the mass shows a sensation of fluctua-
i. Sclerotherapy: The commonly used sclerosants are tion. There may be central softening due to abscess
polidocanol (POL), sodium tetradecyl sulphate (STS), formation.
Sclerodex (Canada), hypertonic saline, glycerin and l Stage 4 is rupture of caseous materials into the sur-
chromated glycerin. rounding soft tissue, forming a confluent abscess cavity.
ii. Endovenous thermal ablation: Endovenous laser The mass may rupture to form a creeping cavity with
treatment/ablation (ELA) for varicose veins “appears greyish white wall and fistulae, discharging thin pus
to be more effective in the short term, and at least with flocculent substance, and the superficial skin is
as effective overall, as the comparative procedure dark purple in colour.
of junction ligation and vein stripping for the treatment
of varicose veins. Complications for ELA include
minor skin burns and temporary paraesthesia.
Clinical Features
iii. Radiofrequency (RF) ablation. l The most common presentation is persistent, painless
swelling of the lymph nodes. With progress of disease,
Q. 5. Describe the aetiology, stages, clinical features
the swollen nodes can begin to release a discharge of
and management of tuberculous lymphadenitis
fluid.
(scrofula).
l Systematic features: Fever, weight loss, fatigue, night
Ans. sweats.
340 Quick Review Series for BDS 3rd Year

Diagnosis thyrohyoid membrane and becomes larger with the


Valsalva manoeuvre. There might be associated stridor
l Positive tuberculin test
or hoarseness, and a radiograph of the neck might show
l Chest X-ray
an air fluid level in the mass.
l CT scan
l Dermoid cyst. A dermoid cyst is a midline cyst
l Cytology/biopsy (FNAC)
that contains solid and cystic components; it seldom
l AFB staining
transilluminates as brilliantly as a cystic hygroma, and a
l Mycobacterial culture
radiograph might show that it contains calcifications.
l Cervical (unilateral) lymphadenopathy which is
Treatment perhaps the most common type of adenopathy, frequently
results from pharyngitis (viral, streptococcal, gonococcal)
l Chemotherapy: HRZE 2 months followed by HR for
or oral, head and neck or intraoral infection or malignancy.
next 4 months
l Cervical bilateral: Mononucleosis, sarcoidosis, toxo-
l DOTS: HRZ 2 months followed by HR for next
plasmosis, pharyngitis.
4 months
l Anterior cervical lymphadenopathy often results
l Surgery: Surgery is indicated in following cases:
from head and neck infections.
l Persistant fluctuant lesions
l Failure with chemotherapy and Q. 7. Aetiology, pathogenesis, clinical features and man-
l Symptomatic relief agement of cervical TB lymphadenitis.
Q. 6. Discuss the differential diagnosis of cervical lymph- Ans.
adenopathy.
Tuberculous cervical lymphadenitis refers to a lymphadeni-
Ans. tis of the cervical lymph nodes associated with tuberculosis.
Clinical conditions with which differential diagnosis of This condition was previously referred to as scrofula.
cervical lymphadenopathy is carried out are as follows:
l Mumps. The swelling of mumps parotitis crosses the Aetiology and Pathology
angle of the jaw. On the other hand, cervical lymph
nodes are usually below the mandible. l Tuberculous cervical lymphadenitis cases are caused by
l Thyroglossal cyst. A thyroglossal cyst is a mass that Mycobacterium tuberculosis. The cervical TB lymphad-
can be distinguished by the midline location between enitis has 4 pathological stages:
the thyroid bone and suprasternal notch and the upward i. Stage 1 is lymphoid hyperplasia, with formation of
movement of the cyst when the child swallows or sticks tubercles and granulomas without caseation necrosis.
out his or her tongue. ii. Stages 2 and 3 are caseation necrosis in the affected
l Branchial cleft cyst. A branchial cleft cyst is a smooth lymph nodes followed by destruction of capsules of
and fluctuant mass located along the lower anterior bor- lymph nodes and adherence of multiple nodes with
der of the sternomastoid muscle. periadenitis.
l Sternomastoid tumour. A sternomastoid tumour is a iii. Stage 4 is rupture of caseous materials into the sur-
hard, spindle-shaped mass in the sternomastoid muscle rounding soft tissue, forming a confluent abscess
resulting from perinatal haemorrhage into the muscle cavity. Stage 4 nodes are uncommon in mediastinal
with subsequent healing by fibrosis (Leung & Robson, disease.
1991). The tumour can be moved from side to side but
not upward or downward. Torticollis is usually present.
l Cervical ribs. Cervical ribs are orthopaedic anomalies
Clinical Features
that are usually bilateral, hard, and immovable. Diagno- l Presence of chronic painless, enlarging, or persistent
sis is established with a radiograph of the neck. mass with grows in time. Any cervical node, although
l Cystic hygroma. Cystic hygroma is a multiloculated, anterior cervical chain is more common. The nodes are
endothelial-lined cyst that is diffuse, soft, and com- firm, rubbery which become more firm and matted as
pressible, contains lymphatic fluid, and typically transil- disease progresses. The mass is referred to as a “cold
luminates brilliantly. abscess”, as there is no accompanying local hyperaemia
l Haemangioma. Haemangioma is a congenital vascular or warmth and the overlying skin acquires a violaceous
anomaly that often is present at birth or appears shortly (bluish-purple) colour.
thereafter. The mass is usually red or bluish. l As the lesion progresses, skin becomes adhered to the
l Laryngocele. A laryngocele is a soft, cystic, compress- mass and may rupture, forming a sinus and an open
ible mass that extends out of the larynx and through the wound, draining fistula.
Section | II General Surgery 341

l Multiple masses in two-thirds of patients, bilateral Methods of Spread of Carcinoma


nodes in one-third of patients.
l Death from cancer often comes not from the primary
l Systemic symptoms include fever/chills, weight loss, or
site but from the metastases or the spread site. Cancer
malaise in 43% of patients.
spreading, or metastases take place in many ways:
a. Through the lymphatic system.
Diagnosis b. Through the bloodstream.
c. By spreading through body spaces such as the bron-
i. Fine-needle aspiration:
chi or abdominal cavity.
l Fine-needle aspiration is a useful initial procedure
d. Through implantation.
with a sensitivity of 77% and specificity of 93%. The
l The most common way for cancer to spread is through the
positive predictive value approaches 100%.
lymphatic system. This process is called “embolization”.
l The most reliable criteria for diagnosing infection
l Cancer can also spread through the bloodstream. Cancer
are the presence of stainable acid-fast bacilli and
cells, like healthy cells, must have a blood supply in
cultured organisms on aspirate.
order to live, so all cancer cells have access to the blood-
l Granuloma formation is highly suggestive but not
stream. Malignant cells can break off from the tumour
definitive.
and travel through the bloodstream until they find a suit-
ii. Cultures: They take 4-6 weeks for growth, however, the
able place to start forming a new tumour.
newer polymerase chain reaction (PCR) techniques are
l Sarcomas spread through the bloodstream, as do certain
promising.
types of carcinomas, like carcinoma of the kidneys,
iii. Excisional or incisional biopsies:
testicular carcinoma, and Wilms’ tumour, a type of kid-
l Tuberculosis (TB): Biopsies are potentially hazard-
ney cancer seen in young children. Cancers may spread
ous because they may spread the disease and give
by more than one route.
rise to sinus formation.
l Cancers can also spread by local invasion — that is, by
l NTM: Excisional biopsy has both diagnostic as well
intruding on the healthy tissue that surrounds the tumour.
as therapeutic efficacy.
l A very rare type of metastasis is caused by implantation
iv. Skin testing with partial purified protein derivative (PPD)
or inoculation. This can happen accidentally when a
has been fairly reliable. Generally, more than 85%
biopsy is done or when cancer surgery is performed.
of patients have a positive test of greater than 10 mm of
In this case, malignant cells may actually drip from a
induration. PPD should be the first-line of investigation
needle or an instrument this is also called a “spill”.
in the workup of a patient with a neck mass.

Management Block Neck Dissection


l In general, M. tuberculosis infection is not considered a l The main goal of this procedure was to remove, en bloc,
localized disease; therefore, systemic chemotherapy the entire ipsilateral lymphatic structures from the man-
should be instituted. dible superiorly to the clavicle inferiorly and from the
l Medical treatment alone is the standard treatment for infrahyoid muscles to the anterior border of the trapezius.
scrofula. The best approach is to use conventional l The resection includes, the spinal accessory nerve, the
treatment of tuberculosis with antibiotics. internal jugular vein, the sternocleidomastoid muscle,
l Surgery alone has had disappointing results and is plagued and the submandibular gland.
by a high rate of recurrence and fistulizations. Surgery is l The anatomic structures that remained were the carotid
reserved for establishing the diagnosis, advanced local arteries, vagus nerve, hypoglossal nerve, brachial
disease, persistent disease, or draining fistula. plexus, and phrenic nerve.
l Failure to provide adequate chemotherapy at time of l The skin incision is made through the platysma, and
surgery may lead to postoperative fistulas and haema- the flap is elevated in the subplatysmal plane. In the
togenous spread. superior lateral aspect of the flap, leaving the greater
auricular nerve and the external jugular vein on the
Q. 8. What are the methods of spread of carcinoma. sternocleidomastoid muscle is important.
Describe the block dissection of neck. l The posterior flap is elevated toward the trapezius
muscle. The sternocleidomastoid muscle is exposed and
Ans.
incised above the clavicle with electrocautery.
l Cancer/tumours are called “malignant” because they l The anterior and posterior belly of the omohyoid is
have the ability to invade normal tissues and to metasta- identified. Omohyoid crosses the internal jugular vein
size or spread to other parts of the body. laterally. The internal jugular vein is identified in the
342 Quick Review Series for BDS 3rd Year

lower aspect of the neck, and a 2-0 silk suture is then point is performed with a 2-0 silk suture and a distal
passed around the vein and tied. suture ligature.
l The supraclavicular fatty tissue is opened using blunt
Modifications to the radical neck dissection include the
dissection with identification of the phrenic nerve. The
following:
phrenic nerve appears as a white cord down the midline
l Type I: The spinal accessory nerve is preserved.
of the anterior scalenus muscle.
l Type II: The spinal accessory nerve and the internal
l The internal jugular vein is ligated and transected. The
jugular vein are preserved.
submental fatty tissue, the submandibular nodes, and
l Type III: The spinal accessory nerve, the internal jugular
the submandibular gland is removed and displaced
vein, and the sternocleidomastoid muscle are preserved.
inferiorly. The internal jugular vein is identified superi-
l Extended radical neck dissection: The lymph node
orly, medial to the posterior belly of the digastric
groups and/or additional structures not included in the
muscle. The ligation of the internal jugular vein at this
classic neck dissection are resected.

SHORT ESSAYS
Q. 1. Gangrene. B. Wet gangrene
Ans. l Wet gangrene is gangrene which develops as a compli-
cation of an untreated infected wound, caused by certain
l Gangrene is defined as the necrosis of an organ or tissue bacterial infection.
caused by insufficient blood supply, i.e. ischaemia. l The tissue is infected by saprogenic microorganisms,
l It is a complication resulting from infectious or inflam- which cause tissue to swell and emit a fetid smell.
matory processes, injury, or degenerative changes asso- l Wet gangrene usually develops rapidly due to blockage
ciated with chronic diseases, such as diabetes mellitus. of venous and/or arterial blood flow. The affected part is
saturated with stagnant blood, which promotes the rapid
Aetiology growth of bacteria.
l The toxic products formed by bacteria are absorbed
The following conditions are risk factors for the develop- causing systemic manifestation of septicaemia and
ment of gangrene: finally death.
l Injury or trauma, such as a crush injury, a severe burn, l Wet gangrene occurs in naturally moist tissue and or-
or frostbite. gans such as the mouth, bowel, lungs, cervix, and vulva.
l Diseases that affect the peripheral circulation of blood, l Administration of antibiotics and sometimes the surgi-
e.g. arteriosclerosis, diabetes, smoking, or Raynaud’s cal removal of the dead tissue to keep the infection from
disease. spreading is the treatment.
l Infection of wounds.
C. Gas gangrene
Types of Gangrene l Gas gangrene is a bacterial infection that produces
gas within tissues. It is caused by bacterial exotoxin-
A. Dry gangrene
produced by Clostridium perfringens.
l Dry gangrene is a condition that results when one or l Infection spreads rapidly as the gases produced by bac-
more arteries become obstructed. teria expand and infiltrate healthy tissue in the vicinity.
l In this type of gangrene, the tissue slowly dies because l Gas gangrene can cause necrosis, gas production, and
of inadequate or no blood supply. This occurs mostly sepsis. Progression to toxaemia and shock is often very rapid.
in the extremities and in people with diabetes or l It is often treated with the antitoxin for Clostridium. In
arteriosclerosis. a number of cases, amputation may have to be used to
l In dry gangrene, the tissues first become bluish and they keep the infection under control.
feel cold to the touch. As time progresses, a line of de- l Severe cases have been treated by keeping the patient in
marcation appears between the healthy and devitalized an oxygen-rich atmosphere, as in a hyperbaric chamber.
tissue, which becomes dry and dark black.
l Eventually, there may be a separation of the dead tissue Q. 2. Aneurysm.
from the living tissue; with spontaneous amputation
Ans.
(autoamputation) of the involved extremity.
l Treatment of this type of gangrene is aimed at improv- l Aneurysm is a localized dilation of the wall of a blood ves-
ing circulation, revascularization to the affected area. sels, usually caused by atherosclerosis and hypertension,
Section | II General Surgery 343

or less frequently, by trauma, infection, or a congenital Prevention of Aneurysms


weakness in the vessel wall.
l Congenital aneurysms cannot be prevented.
l Aneurysms can be congenital or they can occur as a
l A healthy lifestyle, i.e. a low-fat diet, regular exercise
result of aging or disease.
and abstinence from smoking.
l Hypertension should be carefully controlled to prevent
Clinical Features aneurysm formation or extension.
l Sign of an arterial aneurysm is a pulsating swelling that Q. 3. Bedsores.
produces a blowing murmur on auscultation. Ans.
l Common sites include the abdominal aortic artery, the
intracranial vessels and the aorta. l Bedsores (pressure ulcers/decubitus ulcers) are areas
l Many aneurysms are present without symptoms and are of damaged skin caused by staying in one position for
discovered by feeling or on X-ray films during a routine too long.
examination. l These lesions can be caused by many factors such as:
l When symptoms occur, they include a pulsing sensa- Unrelieved pressure; friction; humidity; shearing forces;
tion, and there may be pain, if the aneurysm is pressing temperature; age; continence and medication; to
on internal organs. any part of the body, especially portions over bony or
l A ruptured aneurysm usually produces sudden and severe cartilaginous areas such as sacrum, elbows, knees, and
pain, and depending on the location and amount of ankles.
bleeding, shock, loss of consciousness and death. Emer-
gency surgery is necessary to stop the bleeding. Pathophysiology
l In some cases, the aneurysm may leak blood, causing
pain without the rapid deterioration characteristic l The main cause for a pressure sore is compression of the
of a rupture. Thrombi may form in the dilated pouch tissues by an external force, such as a mattress, wheel-
and give rise to emboli that may obstruct smaller chair pad, or bed rail. Other traumatic forces that may
vessels. be present include shear forces and friction.
l In some cases, the aneurysm may dissect into the wall l These forces cause microcirculatory occlusion as pres-
of an artery, blocking some of the branches. Dissecting sures rise above capillary filling pressure, resulting in
aneurysms usually occur in the aortic arch near its ischaemia. Ischaemia leads to inflammation and tissue
origin, as it leaves the heart or start in the descending anoxia.
thoracic portion of the aorta after it gives off the l Tissue anoxia leads to cell death, necrosis, and ulcer-
branches to the head and arms. ation. Irreversible changes may occur after as little as
l Symptoms vary according to the part of the body that is 2 hours of uninterrupted pressure.
being deprived of blood; they are usually sudden, severe
and require emergency treatment. Stages of Bedsores
Tests to diagnose aneurysms include: l Many classification systems for staging pressure sores
l Angiogram. have been presented in the literature, the most widely
l Magnetic resonance imaging (MRI). used is being that of Shea, which has been modified to
l Spinal tap. represent the present National Pressure Ulcer Advisory
l Ultrasound. Panel classification system.
l Echocardiography l This system consists of 4 stages of ulceration but is
l X-ray. not intended to imply that all pressure sores follow a
standard progression from stage I to stage IV.
Treatment of Aneurysms i. Stage I
l Drugs may be prescribed to lower blood pressure and Stage I bedsores represents intact skin with signs of im-
reduce the risk of rupture. pending ulceration. Initially, this would consist of blanch-
l Abdominal aneurysms that are large or increasing in able erythema from reactive hyperaemia that should resolve
size should be treated surgically. within 24 hours of the relief of pressure. Warmth and indu-
l Enlarging thoracic aneurysms should be considered for ration also may be present. Continued pressure creates ery-
surgery. thema that does not blanch with pressure. This may be the
l A dissecting or ruptured aneurysm requires emergency first outward sign of tissue destruction. Finally, the skin
surgery. may appear white from ischaemia.
344 Quick Review Series for BDS 3rd Year

ii. Stage II l In case a large acquired arteriovenous fistula is left


It represents a partial-thickness loss of skin involving epi- untreated, a large volume of blood flows under high
dermis and possibly dermis. This lesion may present as an pressure from the artery into the veins which are not
abrasion, blister, or superficial ulceration. strong enough to withstand such high pressure, so the
walls stretch and the veins enlarge and bulge.
iii. Stage III l In addition, blood flows more freely into the enlarged
It represents a full-thickness loss of skin with extension into veins than it would if it continued its normal course
subcutaneous tissue but not through the underlying fascia. through the arteries. As a result, blood pressure falls.
This lesion presents as a crater with or without undermining l To compensate for this fall in blood pressure, the heart
of adjacent tissue. pumps more forcefully and more rapidly, thus greatly
increasing its output of blood.
iv. Stage IV l Eventually, the increased effort may strain the heart,
It represents full-thickness loss of skin and subcutaneous causing heart failure. The larger the fistula, the more
tissue and extension into muscle, bone, tendon, or joint quickly heart failure can develop.
capsule. Osteomyelitis with bone destruction, dislocations, l On auscultation, machinery murmur is characteristic for
or pathologic fractures may be present. Sinus tracts and AV fistula.
severe undermining commonly are present. l Doppler ultrasonography is used to confirm the diagnosis
Unstageable pressure ulcers are covered with dead cells, or and to determine the extent of the problem.
eschar and wound exudate, so the depth cannot be determined. l For fistulas between deeper blood vessels (such as the
aorta and vena cava), magnetic resonance imaging
(MRI) is more useful.
Treatment
Multidisplinary treatment, involving multiple special-
l
Treatment
ists is the best choice of management.
l Pressure-distributive mattresses, debridement, infection l Small congenital arteriovenous fistulas can be cut out or
control, nutritional supports, wound intervention with eliminated with laser coagulation therapy.
free flaps, negative wound pressure therapy are impor- l Acquired arteriovenous fistulas are corrected by a sur-
tant components of bedsore management. geon as soon as possible after diagnosis.
l In case AV fistulas are located in inaccessible place like
Q. 4. Arteriovenous fistula.
brain, complex injection techniques that cause clots to
Ans. form may be used to block blood flow through the fistula.
l Arteriovenous fistula defined as an abnormal channel or Q. 5. Venous ulcer.
passageway between an artery and a vein.
l AV fistula may be congenital, surgically created for Ans.
haemodialysis or acquired due to pathologic process,
l Venous ulcers or stasis ulcers are ulcers which develop
such as trauma or erosion of an arterial aneurysm.
due to chronic venous insufficiency.
Congenital AV fistula are uncommon.
l Venous or stasis ulcers occur due to venous hyperten-
sion that develops because of inadequate calf muscle
Pathophysiology pump action and after the onset of either primary or
l When an arteriovenous fistula is formed involving a secondary valvular incompetence.
major artery like the abdominal aorta, it can lead to a l Venous stasis ulcers are common in patients who have a
large decrease in peripheral resistance. history of leg swelling, long-standing varicose veins, or a
l This lowered peripheral resistance causes the heart to history of blood clots in either the superficial or the deep
increase cardiac output in order to maintain proper veins of the legs. Ulcers may affect one or both legs.
blood flow to all tissues.
l The physical manifestations of this would be a relatively Pathophysiology
normal systolic blood pressure with a decreased diastolic
blood pressure resulting in a wide pulse pressure. Two hypotheses have been proposed to explain venous ulcers:
i. The first states that distension of the capillary beds
occurs because of increased stasis. This leads to leakage
Clinical Features of fibrinogen into the surrounding dermis. Over time,
l Congenital arteriovenous fistulas near the surface of the a fibrinous pericapillary cuff is formed, impeding the
skin, may appear swollen and reddish blue. delivery of oxygen and other nutrients or growth factors
Section | II General Surgery 345

to the affected tissue. The resulting hypoxic injury leads l The risk factors of thrombophlebitis (“white leg”):
to fibrosis and then ulceration. Prolonged sitting, varicose veins, obesity, old age,
ii. The other hypothesis suggests that the endothelium is smoking, caustic material IV, hypercoagulable state.
damaged by increased venous pressure and leucocyte
activation. Proteolytic enzymes and free radicals are
Aetiology
released, escape through the leaky vessel walls, and
damage the surrounding tissue, leading to injury and l Specific disorders associated with thrombophlebitis in-
ulceration. clude superficial thrombophlebitis (affects veins near
the skin surface) and deep venous thrombosis (affects
deeper, larger veins).
Clinical Features l Thrombophlebitis migrans can be a non-metastatic
l Typically, venous ulcers are located circumferentially manifestation of malignancies such as pancreatic carci-
around the lower leg from approximately mid-calf to noma.
just below the medial and lateral malleoli. l Thrombophlebitis at the site of an intravenous infusion
l Larger but shallower than other ulcers, stasis ulcers have as a result of irritating drugs, hypertonic solutions, or
a moist granulating base and an irregular border. This the intraluminal catheter or cannula itself is by far the
base oozes venous blood when manipulated. The tissue most common type of thrombophlebitis encountered.
surrounding these ulcers may exhibit signs of stasis l Usually, redness and pain signal its presence while the
dermatitis. infusion is being given. Iatrogenic form of traumatic
l Patients often report mild pain that is relieved by elevation. (chemical) phlebitis may be deliberately produced by
l The base of a venous ulcer is usually red. It may also be sclerotherapy.
covered with yellow fibrous tissue or there may be a
green or yellow discharge, if the ulcer is infected. Clinical Features
l Fluid drainage can be significant with this type of ulcer.
It may even feel warm or hot. The skin may appear l Pain, tenderness in the part of the body affected.
shiny and tight, depending on the amount of oedema l Skin redness or inflammation (not always present).
(swelling). l Swelling (oedema) of the extremities (ankle and foot).
l The skin may also have brown or purple discolouration
about the lower leg, known as “stasis skin changes.” Diagnosis
l Evaluation for a hypercoagulable state should include
Treatment tests for factor V leiden and prothrombin gene
l Venous ulcers are costly to treat, and there is a signifi- mutations, protein C and protein S, antithrombin C,
cant chance that they will recur after healing. antiphospholipid antibodies, lupus anticoagulant, factor
l Elevation of the leg and topical therapy which absorb VIII, and homocysteine.
exudate and maintain a moist wound environment are l In migratory thrombophlebitis, a more detailed evalua-
common treatment modalities. tion of the patient in search of a malignant lesion should
l Special treatment protocols: include selective application of serum carcinoembry-
i. Compression therapy onic antigen (CEA), prostate-specific antigen (PSA),
l Elastic wrap or multiple layer bandage systems colonoscopy, CT scans, and mammography.
l Compression stockings l Duplex ultrasound is helpful in assessment of the prog-
l Unna boot ress of the condition.
ii. TIRS technique: The TIRS (terminal interruption of
reflux source) technique entails blocking off the Treatment
veins that drain the ulcer bed using sotradecol or
polidocanol foam, administered by ultrasound guid- l For the superficial, localized, mildly tender area of
ance. It has been documented that most ulcers thrombophlebitis that occurs in a varicose vein, treat-
treated with this technique have healed in 3–8 weeks. ment with mild analgesics, such as aspirin, and the use
of some type of elastic support usually are sufficient.
Q. 6. Thrombophlebitis. l In case of severe thrombophlebitis, as indicated by
the degree of pain and redness and the extent of the
Ans.
abnormality, bed rest with elevation of the extremity
l Thrombophlebitis is an inflammatory reaction with and the application of massive, hot, wet compresses is
thrombus. recommended.
346 Quick Review Series for BDS 3rd Year

l Patients who present with thrombosis of the long or Diagnosis


short saphenous veins should be considered for antico-
l The diagnosis of NF1 is not a difficult one being evident
agulation or ligation of the saphenous vein.
on clinical grounds.
l If the thrombophlebitis is associated with a cannula or a
l The diagnositic criteria for NF1 are listed below.
catheter, the device should be immediately removed and
A patient meeting two or more of the following criteria
cultured. If the patient is septic, appropriate antibiotics
can be diagnosed as suffering from NF1:
should be given. If suppurative thrombophlebitis is sus-
a) Neurofibromas: Two or more, or one plexiform neu-
pected, immediate and complete excision of all of the
rofibroma
involved veins is indicated.
b) Café-au-lait macules: Six or more measuring 1 to
Q. 7. Von Recklinghausen’s disease. 5 cm in their greatest dimension
c) Freckling: In the axillary or inguinal areas
Ans. d) Optic glioma
l Neurofibromatosis type 1 (NF-1), formerly known as e) Iris hamartomas (Lisch nodules): Two or more
von Recklinghausen disease is a human genetic disorder f) Sphenoid dysplasia or thinning of the cortex of the
with autosomal dominancy. long bones
l NF-1 is caused by a mutation of a gene on the long arm of g) First-degree relative
chromosome 17 which encodes a protein known as neuro- l In difficult cases biopsy of a neurofibroma can be
fibromin which plays a role in intracellular signaling. diagnostic.
l It is inherited as an autosomal dominant trait, but about l Magnetic resonance and computed tomography are
50% of cases arise due to spontaneous mutation. important in the diagnosis of spinal nerve root tumours,
l Von Recklinghausen disease (NF1) is a tumour disorder intracranial tumours and hamartomas.
causing non-cancerous lumps. NF-1 often comes with
scoliosis (curvature of the spine), learning dificulties, Treatment
eye problems, and epilepsy.
There is no treatment for the disease, but small cutaneous
or subcutaneous neurofibromas can be removed, if they are
Epidemiology painful.
Its incidence is 1 per 3,000 births and present in about Q. 8. Non-Hodgkin’s lymphoma.
30 persons per 10,000 population.
Ans.
Non-Hodgkin’s lymphomas are a diverse group of
Clinical Manifestations l

cancers that develop in B or T lymphocytes and include


l The pigmented spots are irregular in shape with rela- any kind of lymphoma except Hodgkin’s lymphomas.
tively even borders, vary in size, and are of brownish l The WHO/REAL classification of NHL is considerably
coffee colour (café au lait). They are most proeminent more complex and recognizes 27 distinct pathological
over the trunk, in the axilla (axillary freckles), and about subtypes.
the pelvis. l Most non-Hodgkin’s lymphomas (85%) are from
l The tumours are usually multiple and vary in size from B cells. Less than 15% develop from T cells. Non-
minute lesions to large tumours. The majority are Hodgkin’s lymphoma is more common than Hodgkin’s
smoothly rounded or lobulated, and can sometimes be lymphoma.
seen or felt along the course of peripheral nerves. Often
they sink into the subcutaneous fat on gentle pressure.
Clinical Features
They are also more frequent over the trunk.
l Plexiform neurofibromas may grow a lot, leading to l The first symptom is often rapid and usually painless
grotesque overgrowth of soft tissue and bone in a limb enlargement of lymph nodes in the neck, under the
or around the orbit. arms, or in the groin.
l Tumours of the spinal nerve roots may compress the l Enlarged lymph nodes within the chest may press against
spinal cord and at the same time extend though the in- airways, causing cough and difficulty in breathing.
tervertebral foramens to form a large mass in the poste- l Deep lymph nodes within the abdomen may press against
rior mediastinum (dumb-bell tumours). various organs, causing loss of appetite, constipation,
l Stenosis of the aqueduct of Sylvius with obstructive abdominal pain, or progressive swelling of the legs.
hydrocephalus is at times observed in NF1. There is also l Too few red blood cells can cause anaemia, leading
an association of vascular stenoses and NF1. to fatigue, shortness of breath, and pale skin. Too
Section | II General Surgery 347

few white blood cells can lead to infections. Too few l On the basis of this staging, the patient will be classified
platelets may lead to increased bruising or bleeding. according to a staging classification (the Ann Arbor
l Non-Hodgkin’s lymphomas also commonly invade staging classification scheme is a common one):
the bone marrow, digestive tract, skin, and occasionally i. Stage I is involvement of a single lymph node
the nervous system, causing various symptoms. Some region (I) (mostly the cervical region) or single
people have persistent fever of unknown origin. This extralymphatic site (Ie);
type of fever commonly reflects an advanced stage of ii. Stage II is involvement of two or more lymph
disease. node regions on the same side of the diaphragm (II)
l In children, the first symptoms—anaemia, rashes, and or of one lymph node region and a contiguous
neurologic symptoms, such as weakness and abnormal extralymphatic site (IIe);
sensation—are likely to be caused by infiltration of iii. Stage III is involvement of lymph node regions on
lymphoma cells into the bone marrow, blood, skin, both sides of the diaphragm, which may include the
intestine, brain, and spinal cord. spleen (IIIs) and/or limited contiguous extralym-
l Depending on progression of disease, NHL can be phatic organ or site (IIIe, IIIes);
broadly divided into: iv. Stage IV is disseminated involvement of one
i. Indolent lymphomas are characterized by: or more extralymphatic organs. The absence
l A long survival period (many years) of systemic symptoms is signified by adding ‘A’
l Rapid response to many treatments to the stage; the presence of systemic symptoms
l Lack of cure when standard therapies are used is signified by adding ‘B’ to the stage. For
ii. Aggressive lymphomas are characterized by: localized extranodal extension from mass of nodes
l Rapid progression without therapy that does not advance the stage, subscript ‘E’ is
l High rates of cure with standard chemotherapy added.

Q. 10. Hodgkin’s lymphoma of neck


Treatment
Ans.
i. Stages I and II Non-Hodgkin’s lymphomas:
l They are treated with radiation therapy limited to the
l According to classification by the World Health Organi-
site of the lymphoma and adjacent areas. zation (WHO), Hodgkin disease (Hodgkin’s lymphoma)
l People with aggressive lymphomas at a very early
exists in 5 types.
stage need to be treated with combination chemo- l Four of these, nodular sclerosis, mixed cellularity, lym-
therapy and sometimes radiation therapy. phocyte depleted, and lymphocyte rich, are referred to
ii. Stages III and IV Non-Hodgkin’s lymphomas: as classic Hodgkin disease (Hodgkin’s lymphoma). The
l Almost all people with indolent lymphomas fifth type, nodular lymphocyte predominant Hodgkin
have stage III or IV disease. They do not always disease (NLPHD), is a distinct entity with unique
require treatment initially, but they are monitored clinical features and a different treatment paradigm.
for evidence of progression, which could signal a
Nodular sclerosing CHL:
need for therapy, sometimes years after the initial
diagnosis. l It is the most common subtype and is composed of large
l Treatment may include therapy with monoclonal tumour nodules showing scattered lacunar classical
antibodies rituximab alone or chemotherapy with or Reed-Sternberg cells set in a background of reactive
without rituximab. lymphocytes, eosinophils and plasma cells with varying
l For people with aggressive stage III or IV non-Hodgkin’s degrees of collagen fibrosis/sclerosis.
lymphomas, combinations of chemotherapy drugs are l NSHD is frequently observed in adolescents and young
given promptly, often together with rituximab. adults and usually involves the mediastinum and other
supradiaphragmatic sites.
Q. 9. Lymphoma staging.
Mixed-cellularity Hodgkin lymphoma:
Ans.
l A common subtype and is composed of numerous
l The staging for both Hodgkin’s as well as non-Hodgkin’s classic RS cells admixed with numerous inflammatory
lymphomas is same. cells including lymphocytes, histiocytes, eosinophils,
l After Hodgkin’s lymphoma is diagnosed, a patient will and plasma cells without sclerosis.
be staged. l This type is most often associated with EBV infection
l Positron emission tomography (PET) scan is now used and may be confused with the early, so-called ‘cellular’
instead of the gallium scan for staging. phase of nodular sclerosing CHL.
348 Quick Review Series for BDS 3rd Year

l MCHD is the histologic type most commonly observed l The mass is referred to as a “cold abscess”, because there
in patients with human immunodeficiency virus (HIV) is no accompanying local colour or warmth and the over-
infection. lying skin acquires a violaceous (bluish-purple) colour.
l Tuberculous cervical lymphadenitis usually accompanied
Lymphocyte-depleted Hodgkin disease (LDHD) by other symptoms of the disease, such as fever, chills,
(less than 1% of cases): malaise and weight loss in about 43% of the patients.
l It is a rare subtype, composed of large numbers of often l As the lesion progresses, skin becomes adhered to the
pleomorphic RS cells with only few reactive lymphocytes. mass and may rupture, forming a sinus and an open
l The infiltrate in LDHD is diffuse and often appears wound.
hypocellular. Large numbers of Reed-Sternberg cells
and bizarre sarcomatous variants are present.
Diagnosis
l It is associated with older age and HIV-positive status.
It is usually performed by needle aspiration biopsy or exci-
Lymphocyte-rich classic Hodgkin disease (LRHD) sional biopsy of the mass and the histological demonstra-
(5% of cases): tion of stainable acid-fast bacteria in the case of infection
l It is a rare subtype, which shows many features which by M. tuberculosis (Ziehl-Neelsen stain).
may cause diagnostic confusion with nodular lymphocyte
Q. 12. What are the different stages of tuberculous
predominant B-cell Non-Hodgkin’s lymphoma (B-NHL).
lymphadenitis?
l This form has the most favourable prognosis. In this
type, RS cells of the classic or lacunar type are ob- Ans.
served, with a background infiltrate of lymphocytes.
l Tuberculous lymphadenitis (or tuberculous adenitis) is a
Nodular lymphocyte-predominant Hodgkin disease chronic specific granulomatous inflammation with case-
(NLPHD) (5% of cases): ation necrosis of the lymph node.
l The characteristic morphological element is the tubercu-
l In contrast to the other histologic subtypes, the typical
lous granuloma (caseating tubercule): Giant multinucle-
Reed-Sternberg cells are either rare or absent.
ated cells (Langhans cells), surrounded by epithelioid
l Instead, lymphocytic and histiocytic (L&H) cells, are
cells aggregates, T cell lymphocytes and few fibroblasts.
seen within a background of inflammatory cells, which
l Granulomatous tubercles evolve to central caseous ne-
are predominantly benign lymphocytes.
crosis and tend to become confluent, replacing the
Q. 11. Enumerate the clinical features of tuberculous lymphoid tissue.
cervical lymphadenitis. l Pathologically active tuberculous lymphadenitis passes
through four stages:
Ans.
Stage I is lymphoid hyperplasia, with formation of
l Tuberculous cervical lymphadenitis refers to a lymph- tubercles and granulomas without caseation necrosis.
adenitis of the cervical lymph nodes associated with Stages 2 and 3 are caseation necrosis in the affected
tuberculosis. It was previously known as “scrofula”. lymph nodes followed by destruction of capsules
l It is most often observed in immunocompromised patients. of lymph nodes and adherence of multiple nodes with
periadenitis.
Stage 4 is rupture of caseous materials into the sur-
Clinical Features rounding soft tissue, forming a confluent abscess cavity.
l The most usual signs and symptoms are the appearance Stage 4 nodes are uncommon in mediastinal disease.
of a chronic, painless mass in the neck, which is persis-
The duration of disease increases progressively from
tent and usually grows with time.
stage 1 to stage 4 tuberculosis lymphadenitis.

SHORT NOTES
Q. 1. Lymphadenitis. Clinical Features:
Ans. l It usually presents as one or more enlarged or swollen
Lymphadenitis is the inflammation or enlargement of lymph lymph nodes under the neck, in the axilla, or in the
nodes and is relatively common, and most often indicates groin.
bacterial, viral and fungal infections. Lymphadenitis can l Lymph nodes that are swollen may feel slightly hard-
also occur near an underlying tumour. ened, and may be painful when touched.
Section | II General Surgery 349

l The skin covering the lymph node can sometimes feel l The mass is referred to as a “cold abscess”, because
hot to the touch or may appear slightly hyperaemic. there is no accompanying local colour or warmth and
l Presence of fever is usually noticed. the overlying skin acquires a violaceous (bluish-purple)
colour.
Tuberculous cervical lymphadenitis is usually accompa-
Treatment l

nied by other symptoms of the disease, such as fever,


Modalities of treatment of lymphadenitis include: chills, malaise and weight loss.
l Analgesia l As the lesion progresses, skin becomes adhered to the
l Antibiotics mass and may rupture, forming a sinus and an open
l Abscess drainage wound.
l Diagnosed with the help of needle aspiration biopsy or
Q. 2. Lymphosarcoma. excisional biopsy of the mass.
Ans. l Treatment: The best approach is to use conventional
treatment of tuberculosis with antibiotics.
l Lymphosarcoma is defined as a malignant neoplastic
disorder of the lymphoid tissues characterized by prolif- Q. 5. Commando operation.
eration of atypical lymphocytes and their localization in
Ans.
various parts of the body.
l The term lymphosarcoma is absolute now. Instead the l Commando operation is a term attributed to Hayes
term non-Hodgkin’s lymphoma is used. Martin, a pioneer in head and neck surgery, for the en
l They are classified according to the “Revised European- bloc removal of an advanced 1° malignancy of the oral
American Lymphoma classification” (REAL). This sys- cavity, usually SCC.
tem attempts to group lymphomas by cell type (i.e. the l lymphoma is amenable to RT or chemotherapy;
normal cell type that most resembles the tumour) and l The Commando operation is a very aggressive proce-
defining phenotypic, molecular or cytogenetic charac- dure, and entails partial removal of the mandible, floor
teristics. There are three large groups: the B cell, T cell, of the mouth and/or tongue, accompanied by a radical
and natural killer cell tumours. neck dissection.
Q. 3. Hodgkin’s disease—neck. Q. 6. Waldayer’s ring.
Ans. Ans.
l Hodgkin’s lymphoma, previously known as Hodgkin’s l Waldeyer’s or Pirogov tonsillar ring is a circle of lym-
disease, is a type of lymphoma, which is a cancer phoid tissue located in the pharynx and to the back of
originating from white blood cells called lymphocytes. the oral cavity.
l It was named after Thomas Hodgkin, who first l It was named after the nineteenth century German
described abnormalities in the lymph system in 1832. anatomist Heinrich Wilhelm Gottfried von Waldeyer-
l A quarter of all extranodal lymphomas occur in the head Hartz.
and neck, and 8% of findings on supraclavicular fine- l The ring consists of (from superior to inferior):
needle aspirate biopsy yield a diagnosis of lymphoma. i. Pharyngeal tonsil
l Classical Hodgkin’s lymphoma can be subclassified into ii. Tubal tonsil
four pathologic subtypes based upon Reed-Sternberg iii. Palatine tonsils (“faucial tonsils”)
cell morphology and the composition of the reactive cell iv. Lingual tonsils
infiltrate seen in the lymph node biopsy specimen. v. The tubopharyngeal plicae: Lateral bands which run
almost vertically at the junction of the Lateral pos-
Q. 4. Tuberculous cervical lymphadenitis. terior walls of the oro- and nasopharynx.
Ans. vi. Lymphoepithelial collections in the laryngeal ven-
tricle.
l Tuberculous cervical lymphadenitis refers to lymphad-
enitis of the cervical lymph nodes associated with tuber- Q. 7. Cystic hygroma.
culosis. It was previously known as “scrofula”.
Ans.
Cystic hygroma (CH) is the most common form of
Clinical Features l

lymphangioma.
l The most usual signs and symptoms are the appearance l It is congenital multiloculated cystic lymphatic lesion
of a chronic, painless mass in the neck, which is persis- containing watery fluid and can affect any anatomic
tent and usually grows with time. subsite in the human body.
350 Quick Review Series for BDS 3rd Year

l Cystic hygroma most commonly affects the head and l Tuberculous lymphadenitis is a chronic specific granu-
neck (approximately 75%), with a left-sided predilection, lomatous inflammation with caseation necrosis.
the posterior triangle being the most frequently affected. l The characteristic morphological element is the tubercu-
l About 20% of CHs occur in the axilla. lous granuloma (caseating tubercule): Giant multinucle-
l Microscopically, cystic hygroma consists of multiple ated cells (Langhans cells), surrounded by epithelioid
locules filled with lymph. In the depth, the locules are cells aggregates, T cell lymphocytes and few fibroblasts.
quite big but they decrease in size towards the surface. l Granulomatous tubercules evolve to central caseous
l CHs tend to form in loose areolar tissue. necrosis and tend to become confluent, replacing the
l CHs present themselves as typically soft, painless, com- lymphoid tissue.
pressible (doughy) masses which typically transillumi-
Q. 11. Embolism.
nate. Although CHs are benign, they can be disfiguring.
l The medical treatment of CH consists of the administra- Ans.
tion of sclerosing agents, but the mainstay of treatment
remains surgical excision. i. Embolism is defined as the obstruction caused by a
foreign substance or a blood clot, i.e. lodging of an
Q. 8. Lymphatic drainage of tongue. embolus into a narrow capillary vessel of an arterial
Ans. bed which causes a blockage (vascular occlusion) in a
distant part of the body.
l Lymphatic drainage of the tongue is very complex. ii. Embolisms are usually categorised by where they occur
There are three types of lymphatic vessels: in the body. Most common are listed below:
a) Marginal vessels: They are present in the margins of Types of embolism:
the tongue and drain lymphatic vessels to submental l Pulmonary embolism
lymph nodes. l Cerebral embolism
b) Central vessels: They line the central part of the l Amniotic embolism
tongue. They drain into the submandibular and l Air embolism
jugulo-omohyoid and jugulo-digastric lymph nodes. l Thromboembolism
c) Dorsal vessels: They are present in the dorsal side of l Fat embolism
the tongue and drain into jugulo-omohyoid and l Septic embolism
jugulo-digastric lymph nodes.
l All the nodes drain into the deep cervical nodes from Q. 12. Gangrene.
where they pass on to the jugular trunk. The jugular trunk Ans.
enters the thoracic duct on the left and the right lymph duct
on the right or the jugulosubclavian junction on the right. i. Gangrene is defined as the necrosis of an organ or tissue
caused by insufficient blood supply, i.e. ischaemia. It is
Q. 9. Classification of Hodgkin’s lymphoma. a complication resulting from infectious or inflamma-
Ans. tory processes, injury, or degenerative changes associ-
ated with chronic diseases, such as diabetes mellitus.
l According to classification by the World Health Organi- ii. Types of gangrene:
zation (WHO), Hodgkin disease (Hodgkin’s lymphoma) l Dry gangrene: Caused due to obstruction of one or
exists in 5 types. more arteries.
i. Nodular sclerosing CHL l Wet gangrene: It develops rapidly due to blockage of
ii. Mixed-cellularity Hodgkin lymphoma venous and/or arterial blood flow.
iii. Lymphocyte-depleted Hodgkin disease l Gas gangrene: It is a bacterial infection that
iv. Lymphocyte-rich classic Hodgkin produces gas within tissues.
v. Nodular lymphocyte-predominant Hodgkin l Others which include Fournier gangrene, necrotising
l Four of these, nodular sclerosis, mixed cellularity, lym- fasciitis, etc.
phocyte-depleted, and lymphocyte-rich, are referred to iii. Treatment include debridement (or, in severe cases, am-
as classic Hodgkin disease (Hodgkin’s lymphoma). putation) of the affected body parts, antibiotics, vascular
l The fifth type, nodular lymphocyte predominant Hodg- surgery, maggot therapy or hyperbaric oxygen therapy.
kin disease (NLPHD), is a distinct entity with unique
clinical features and a different treatment paradigm. Q. 13. Arterial ulcer.
Q. 10. Microscopic appearance in tuberculous lymph- Ans.
adenitis
i. Arterial ulcers or ischaemic ulcers are caused by insuf-
Ans. ficient blood circulation to the capillary beds of the
Section | II General Surgery 351

lower extremities, due to peripheral vascular disease, iii. The definitive treatment for an aortic aneurysm is
diabetes mellitus, trauma, or advanced age. surgical repair. This usually involves the insertion of an
ii. When pressure is placed on the skin, the skin is dam- endovascular stent inside the aortic walls for the length
aged and is unable to be repaired due to the lack of of the aneurysm.
blood perfusing the tissue.
Q. 16. Pulmonary embolism.
iii. The wound which is very painful, has a characteristic deep,
punched out look, often extending down to the tendons. Ans.
iv. Wound beds may be pale, gray or yellow with no evi-
i. Pulmonary embolism is defined as the obstruction of the
dence of new tissue growth; necrosis or cellulitis may
main artery of the lung, pulmonary artery or one of its
be present; commonly accompanied by dry necrotic
branches by emboli, i.e. a substance that has travelled
eschar and exposed tendons.
from elsewhere in the body through the bloodstream.
Q. 14. Haemangioma. ii. Pulmonary embolism is a complication of venous
thromboembolism, most commonly deep venous
Ans.
thrombosis (DVT).
i. Haemangioma is an abnormal proliferation of blood iii. Symptoms of pulmonary embolism include dyspnoea,
vessels that may occur in any vascularized tissue. chest pain on inspiration, and palpitations.
ii. It usually appears during the first weeks of life and re- iv. Clinically PE manifests with low blood oxygen saturation
solves by age 10. It is the most common tumour in infancy. and cyanosis, tachypnoea and tachycardia. Severe cases
iii. The presentation of haemangioma depends on its loca- of PE results in collapse, hypotension, and sudden death.
tion. If they are on the surface of the skin, they are re- v. Diagnosis is based on these clinical findings in combina-
ferred to as “strawberry haemangiomas”; if they are just tion with laboratory tests (such as the D-dimer test) and
under the skin, they present as a bluish swelling. imaging studies, usually CT pulmonary angiography.
iv. The haemangioma may be: vi. Treatment is typically with anticoagulant medication,
l In the top skin layers (capillary haemangioma) including heparin and warfarin. Severe cases require
l Deeper in the skin (cavernous haemangioma) thrombolysis with tissue plasminogen activator (tPA) or
l A mixture of both may require surgical intervention via pulmonary throm-
v. Treatment: bectomy.
l Most haemangiomas disappear without treatment,
Q. 17. Carotid body tumour.
leaving minimal or no visible scars.
l In case of large haemangiomas, surgical removal is Ans.
sometimes indicated, particularly if there has been
i. Carotid body tumour (chemodectoma or paragangli-
delay in commencing treatment and structural
oma) is a tumour which develop within the adventitia of
changes have become irreversible.
the medial aspect of the carotid bifurcation.
l A pulsed dye laser can be useful for very early, flat,
ii. The following 3 different types of carotid body tumours
superficial lesions.
(CBTs) have been described:
Q. 15. Aneurysm of aorta. l Familial

l Sporadic: Most common


Ans.
l Hyperplastic

i. Aneurysm of aorta is a dilation of the aorta, usually iii. Aetiology: Known risk factors include the presence
representing an underlying weakness in the wall of the of chronic hypoxic stimulation and the genetic
aorta at that location. predisposition.
ii. Classification of aortic aneurysm: Aortic aneurysms are iv. Clinical features:
classified according to the location of its occurrence in l The mean age of onset is reported to be 45 years.

aorta as follows: l They present most commonly as an asymptomatic

l An aortic root aneurysm (aneurysm of sinus of Val- palpable neck mass in the anterior triangle of the neck.
salva): Arises from the sinuses of Valsalva or aortic root. l They are slow-growing tumours that can remain

l Thoracic aortic aneurysms arise from the thoracic asymptomatic for many years.
aorta and are further classified into ascending, aortic l On examination, the mass is typically vertically

arch, or descending aneurysm. fixed because of its attachment to the bifurcation of


l Abdominal aortic aneurysms, the most common form the common carotid (Fontaine sign).
of aortic aneurysm, are found on the abdominal aorta. l A bruit can be felt; approximately 10% of the cases

l Thoracoabdominal aortic aneurysms involve both present with cranial nerve palsy with paralysis of the
the thoracic and abdominal aorta. hypoglossal, glossopharyngeal, recurrent laryngeal,
352 Quick Review Series for BDS 3rd Year

or spinal accessory nerve, or involvement of the l In this type of gangrene, the tissue slowly dies because
sympathetic chain. of inadequate or no blood supply. This occurs mostly
l Carotid body tumours (CBTs) may, therefore, be in the extremities and in people with diabetes or
associated with pain, hoarseness, dysphagia, Horner arteriosclerosis.
syndrome, or shoulder drop. l In dry gangrene, the tissue first becomes bluish and they
v. Treatment: They are treated with either surgery or feel cold to the touch. As time progresses, a line of de-
radiotherapy. marcation appears between the healthy and devitalized
tissue, which becomes dry and dark black.
Q. 18. Thromboangiitis obliterans. l Eventually, there may be a separation of the dead tissue
Ans. from the living tissue; with spontaneous amputation
(autoamputation) of the involved extremity.
i. Thromboangiitis obliterans also known as Buerger’s disease l Treatment of this type of gangrene is aimed at improv-
is a recurring progressive segmental occlusive inflammatory ing circulation, revascularization to the affected area.
condition with thrombosis and re-canalization of small- and
medium-sized arteries and veins of the upper and lower Q. 21. Arteriography.
extremities.
Ans.
ii. Clinical features: The typical presentations are:
l Claudication of affected area. As the condition pro- i. Arteriography is defined as a method of radiologic
gesses, ischaemic rest pain appears. visualization of arteries performed after a radio-opaque
l Paraesthesias of the feet and hands and impaired contrast medium is introduced into the bloodstream or
distal pulses in the presence of normal proximal into a specific vessel by injection or through a catheter.
pulses. Unremitting ischaemic ulcerations. ii. It can be:
iii. Treatment: l Catheter arteriography: Radiography of vessels after

l Medical care: Absolute discontinuation of tobacco, introduction of contrast material through a catheter
treatment with intravenous iloprost. inserted into an artery.
l Surgical care: Proposed surgical treatments for l Selective arteriography: Radiography of a specific

Buerger disease are as follows: Omental transfer, vessel which is opacified by a medium introduced
sympathectomy, spinal cord stimulator implantation. directly into it, usually via a catheter.
l Ultimate surgical therapy for refractory Buerger iii. Arteriography is used to evaluate the following conditions:
disease is distal limb amputation for non-healing l Blood vessel injury secondary to trauma

ulcers, gangrene, or intractable pain. l Aneurysm

l Blood clots
Q. 19. Hamartomas. l Peripheral vascular disease.

Ans. Q. 22. Treatment of cervical rib.


i. Hamartoma is a benign tumour-like nodule, disordered pro-
Ans.
liferation of mature tissues that are native to a site of origin
with disorganization and often with one element predomi- i. Cervical rib is the presence of a supernumerary (or extra)
nating, e.g. exostoses, nevi and soft tissue hamartomas. rib which arises from the seventh cervical vertebra.
ii. Although most hamartomas are benign, some histo- ii. It is a congenital abnormality located above the normal
logic subtypes, e.g. neuromuscular hamartoma, may first rib.
proliferate aggressively. iii. A cervical rib is present in only about 1 in 500 (0.2%)
iii. Although hamartoma is not a malignant tumour, it of people and in even rarer cases, an individual may
grows at the same rate as the surrounding tissues. have two cervical ribs.
iv. They occur in many different parts of the body and are iv. The presence of a cervical rib can cause a form of
most often asymptomatic and undetected unless seen thoracic outlet syndrome due to compression of the
on an image taken for another reason. lower trunk of the brachial plexus or subclavian artery.
v. Hamartomas result from an abnormal formation of
Q. 23. Subclavian steal syndrome.
normal tissue, the exact reason still unknown.
Ans.
Q. 20. Dry gangrene.
i. Subclavian steal syndrome is defined as constellation of
Ans.
signs and symptoms that arise due to retrograde blood
l Dry gangrene is a condition that results when one or flow in the vertebral artery associated with proximal
more arteries become obstructed. ipsilateral subclavian artery stenosis or occlusion.
Section | II General Surgery 353

ii. This syndrome may occur when the subclavian artery is l Signs of venous insufficiency: Varicose eczema, he-
occluded proximal to the origin of the vertebral artery. mosiderin staining/pigmentation, atrophie blanche,
iii. The term subclavian steal syndrome should really be lipodermatosclerosis, oedema, ulceration.
reserved for those patients who develop neurological iv. Treatment:
symptoms as a consequence of brain ischaemia that l Conservative treatment: Sclerotherapy elastic stock-

occurs during or immediately following exercise of the ings, elevating the legs, and exercise.
ipsilateral arm. l Surgical treatment include: Vein stripping, CHIVA.

iv. Signs and symptoms: l Minimal invasive treatments: Ultrasound-guided

l Presyncope foam sclerotherapy, radiofrequency ablation and


l Syncope endovenous laser treatment.
l Neurologic deficits
Q. 25. Aneurysm.
l Blood pressure differential between the arms

v. Treatment: The goal of surgical therapy is to restore Ans.


antegrade blood flow in the vertebral artery. Following
surgical methods are used: i. Aneurysms are localized dilation of the wall of a blood
l Endarterectomy vessels, usually caused by atherosclerosis and hyper-
l Stents tension, or less frequently, by trauma, infection, or a
l Extrathoracic carotid-subclavian bypass congenital weakness in the vessel wall.
l Transposition ii. Aneurysms can be congenital or they can occur as a
l Axillary-axillary bypass result of aging or disease. The most common disease
associated with aneurysms is atherosclerotic disease.
Q. 24. Varicose vein. iii. Tests to diagnose aneurysms include:
Ans. l Angiogram

l Magnetic resonance imaging (MRI)


i. Varicose veins are tortuous, dilated, prominent superficial
l Spinal tap
veins in the lower limbs, often in the anatomical distribu-
l Ultrasound
tion of the long and short saphenous veins.
l Echocardiography
ii. Aetiology: Due to genetic or developmental weakness
l X-ray
in the vein wall resulting in diminished elasticity, dila-
iv. Treatment of aneurysms:
tion over time and valvular incompetence.
l Drugs may be prescribed to lower blood pressure
iii. Clinical features:
and reduce the risk of rupture.
l Visible distension of superficial veins
l Enlarging thoracic aneurysms should be considered
l Painful, achy, or “heavy” legs, oedema.
for surgery.
l Burning, swelling, throbbing, cramping of gastroc-
l A dissecting or ruptured aneurysm requires emergency
nemius muscles
surgery.
l Leg fatigue

Topic 11

Nervous System

LONG ESSAYS
Q. 1. Migraine. Pathophysiology
Ans. l Although the mechanisms of migraine remain not
l Migraine is a paroxysmal headache disorder character- completely understood, various theories have been
ized by varying degrees of recurrent vascular-quality put forward: Vascular theory, depolarization theory,
headache, photophobia, sleep disruption, and depression. serotonin theory, and neural theory.
354 Quick Review Series for BDS 3rd Year

Variants of Migraine l Blurred vision, delirium, nasal stuffiness, diarrhoea,


According to ICHD-2, there are following subclasses of polyuria, pallor, or sweating may be noted during the
migraines (some of which include further subdivisions): headache phase.
l Migraine without aura or common migraine.
Postdrome phase
l Migraine with aura
l Childhood periodic syndromes l The effects of migraine may persist for some days after
l Retinal migraine the main headache has ended.
l Complications of migraine l Many sufferers report a sore feeling in the area where
l Probable migraine the migraine was, and some report impaired thinking for
a few days after the headache has passed.
l The patients may feel tired or “hungover” and have head
Clinical Features pain, cognitive difficulties, gastrointestinal symptoms,
The signs and symptoms of migraine vary among patients. mood changes, and weakness.
There are four phases to a migraine attack:
l Prodrome phase which occurs hours or days before the Diagnosis
headache.
l Aura phase which immediately precedes the headache. Diagnosis of migraine without aura, according to the Inter-
l Pain phase, also known as headache phase. national Headache Society, can be made according to the
l Postdrome phase following criteria: 5 or more attacks, 4-72 hours with nau-
sea/vomiting, photophobia/phonophobia and 2 or more of
Prodrome phase these symptoms: unilateral, moderate/severe pain intensity,
l Prodromal symptoms occur in 40–60% of migraine suf- pulsating, worsened by physical activity.
ferers.
l This phase may consist of altered mood, irritability, Management
excessive sleepiness, depression or euphoria, fatigue,
yawning, craving for certain food, muscle stiffness (es- Conventional treatment focuses on three areas: Trigger
pecially in the neck), hot ears, constipation or diarrhoea, avoidance, abortive therapy and prophylactic pharmaco-
increased urination, and other visceral symptoms. logical drugs.

Aura phase A. Trigger avoidance:


l About 20-30% patients experience migraine with aura Avoid alcohol, birth control pills, caffeine, anxiety, smok-
which comprises focal neurological phenomena that ing or exposure to smoke, hunger, loud noises.
precede or accompany the attack.
B. Abortive therapy:
l They appear gradually over 5 to 20 minutes and gener-
ally last fewer than 60 minutes. The pain phase may l Analgesics/NSIADs: High dose aspirin 900 mg/6 hr or
begin before the aura has completely subsided. paracetamol 1 g/6 hr PO with metoclopramide
l Symptoms of migraine aura can be visual, sensory, or l Serotonin agonists: Sumatriptan
motor in nature out of which visual aura is the most
C. Phophylatic therapy:
common.
l Other symptoms of the aura phase can include auditory, Propranolol, atenolol, metoprolol, flunarizine, sodium val-
gustatory or olfactory hallucinations, temporary dys- proate, topiramate, and tricyclics are some of the com-
phasia, vertigo, tingling or numbness of the face and monly used drugs.
extremities, and hypersensitivity to touch. Q. 2. Bell’s palsy—clinical features and management.
Pain phase Ans.
l The typical headache of migraine is unilateral, throb- Bell’s palsy is defined as idiopathic unilateral facial nerve
bing or pulsatile, localized in the frontotemporal and paralysis, characterized by rapid onset of partial or com-
ocular area, then builds up over a period of 1-2 hours, plete palsy, usually in a single day.
progressing posteriorly and becoming diffuse.
l It typically lasts from several hours to a whole day. The
pain of migraine is invariably accompanied by other Clinical Features
features like nausea, vomiting, photophobia, phonopho- l Acute onset of unilateral upper and lower facial paraly-
bia, and osmophobia and seek a dark and quiet room. sis (over a 48-h period): Facial drooping on the affected
Section | II General Surgery 355

half, facial palsy is typified by inability to control move- Clinical Features


ment in the facial muscles.
Grandmal seizures have 2 phases:
l The paralysis is of the infranuclear/lower motor neuron
type. The paralysis must include the forehead and lower i. Tonic phase:
aspect of the face.
l The person will quickly lose consciousness, and the
l The patient may report inability to close the eye or to
skeletal muscles will suddenly tense, often causing the
smile on the affected side. He or she also may report
extremities to be pulled towards the body or rigidly
increased saliva on the side of the paralysis.
pushed away from it, which will cause the person to fall
l Posterior auricular pain
if standing.
l Decreased lacrimation
l The tonic phase is usually the shortest part of the sei-
l Hyperacusis in the ear ipsilateral to the paralysis, which
zure, usually lasting only a few seconds. The person
is secondary to weakness of the stapedius muscle.
may also express vocalizations like a loud moan during
l Partial or absent eye closure (Bell phenomenon)
the tonic stage, due to air forcefully expelled from the
l The affected side has decreased taste as compared to the
lungs.
normal side.
l Initial inspection of the patient demonstrates flattening ii. Clonic phase:
of the forehead and nasolabial fold on the side affected
l The person’s muscles will start to contract and relax
with the palsy.
rapidly, causing convulsions.
l When the patient is asked to raise the eyebrows, the side
l These may range from exaggerated twitches of the
of the forehead with the palsy will remain flat.
limbs to violent shaking or vibrating of the stiffened
l When the patient is asked to smile, the face becomes
extremities.
distorted and lateralizes to the side opposite the palsy.
l The person may roll and stretch as the seizure spreads.
Convulsions usually last for less than two minutes.
Management l The eyes typically roll back or close and the tongue
l In general, persons with true Bell’s palsy have an excel- often suffers bruising sustained by strong jaw con-
lent prognosis. Thus, treatment of Bell’s palsy is still tractions.
controversial, because spontaneous recovery is fairly l Incontinence is seen in some cases.
common. l Due to physical and nervous exhaustion, postictal sleep
l The goals of treatment are to improve function of the follows a tonic–clonic seizure.
facial nerve and reduce neuronal damage. Treatment l Postictal confusion and complete amnesia which slowly
may be considered for patients who have the onset of wears off as the person becomes gradually aware that a
paralysis within 1-4 days of the initial visit. seizure occurred.
l Corticosteroids: Prednisone is 1 mg/kg or 60 mg/d for l The seizure in most cases is preceded by Aura, which
6 days, followed by a taper, for a total of 10 days. may last a few minutes or several hours or absent at all.
l Anti-virals (such as acyclovir) are ineffective in improv- During this period, the patient experiences a sense of
ing recovery from Bell’s palsy beyond steroids alone. strong dejavu, dizziness, lightheadedness, discomfort,
They were, however, commonly prescribed due to a altered vision/hearing.
theoretical link between Bell’s palsy and the herpes l The aura stage occurs because tonic–clonic seizures
simplex and varicella zoster virus. often start in an isolated area of the brain, known as the
l Eye care is imperative in Bell’s palsy is accepted univer- seizure focus, and gradually spreads to the whole brain,
sally. The patient’s eye is at risk for drying, corneal whereupon loss of consciousness occurs and becomes a
abrasion, and corneal ulcers. Eye care includes artificial tonic–clonic seizure.
tears for use during the day as well as eyeglasses or
Q. 4. Status epilepticus—treatment.
shields. At night, eye lubricants may be used.
l Surgery decompression of nerve for Bell’s palsy is Ans.
controversial.
i. Status epilepticus refers to continuous seizures or re-
Q. 3. Generalized tonic-clonic seizures—clinical features. petitive, discrete seizures with impaired consciousness
in the interictal period.
Ans.
ii. The duration of seizure activity sufficient to meet the
Generalized tonic-clonic seizures or grandmal seizures are definition of status epilepticus has traditionally been
defined as recurrent sudden attack or convulsion character- specified as 15 to 30 min.
ized by generalized muscle spasms and loss of conscious- iii. However, a more practical definition is to consider sta-
ness, affecting the entire brain. tus epilepticus as a situation in which the duration of
356 Quick Review Series for BDS 3rd Year

seizures prompts the acute use of anticonvulsant ther- anti-seizure activity without causing general depression
apy, typically when seizures last beyond 5 min. of the CNS. In toxic doses, it may produce excitatory
iv. Status epilepticus is an emergency and must be treated signs and at lethal levels a type of decerebrate rigidity.
immediately, since cardiorespiratory dysfunction, hy- l Carbamazepine: Useful in patients with generalized
perthermia, and metabolic derangements can develop tonic-clonic and both simple and complex partial sei-
as a consequence of prolonged seizures, and these can zures. Its is the DOC for partial seizures.
lead to irreversible neuronal injury. l Valproic acid: It is a broad-spectrum antiepileptic drug,
effective in the treatment of GTCS, absence, myoclonic,
partial, and tonic-clonic seizures.
Management of Status Epilepticus l Lamotrigine: Useful for monotherapy and add-on ther-
l The first step in the management of a patient in status apy of partial and secondarily generalized tonic-clonic
epilepticus is to attend to any acute cardiorespiratory seizures in adults and Lennox-Gastaut syndrome in both
problems or hyperthermia, perform a brief medical and children and adults. Lennox-Gastaut syndrome is a dis-
neurologic examination, establish venous access, and order of childhood characterized by multiple seizure
send samples for laboratory studies to identify meta- types, mental retardation, and refractoriness to antisei-
bolic abnormalities. Anticonvulsant therapy should then zure medication.
begin without delay. l Clonazepam: Useful in the therapy of absence seizures
l Assess for evidence of respiratory/cardiovascular insuf- as well as myoclonic seizures in children.
ficiency. Treat hyperthermia. Establish IV and adminis- l Diazepam: Effective agent for treatment of status epi-
ter 50 ml 50% dextrose in water, 100 mg thiamine, and lepticus, its short duration of action is a disadvantage,
0.4 naloxane. leading to the more frequent use of lorazepam. Diaze-
l Administer lorazepam, 0.1 mg/kg (4-8 mg) 2 mg/min. pam per rectal is the DOC for febrile seizures.
l Immediately after lorazepam, administer phenytoin, l Phenobarbitone: Effective agent for generalized tonic-
20 mg/kg. Phenytoin can cause precipitous fall in blood clonic and partial seizures. Its efficacy, low toxicity, and
pressure. If seizures are not controlled, a repeat bolus of low cost make it an important agent for these types of
phenytoin (5-10 mg/kg) or fos phenytoin (5-10 mg/kg) epilepsy. It is the DOC antiepileptic in pregnancy. How-
is given. ever, its sedative effects and its tendency to disturb behav-
l If the seizures still persists, administer phenobarbital iour in children have reduced its use as a primary agent.
20 mg/kg slowly over 30 min. In this stage, endotra- l Ethosuximide: Effective against absence seizures but
cheal intubation is required. If seizures persists, addi- not tonic-clonic seizures. It is the DOC for absence
tional dose of phenobarbital 5-10 mg/kg is given. seizures in children ,3 years.
l In case the seizures remain refractory even after l Gabapentin: Effective for partial seizures, with and
60-90 min, consider placing the patient in midazolam, without secondary generalization, when used in addi-
propofol or pentobarbital coma. tion to other antiseizure drugs.
Q. 5. Anti-epileptic drugs. Q. 6. Meningitis—clinical features.
Ans. Ans.
Anti-epileptic drugs can be grouped broadly according to l Meningitis is inflammation of the protective membranes
their mechanism of action: covering the brain and spinal cord, known collectively
i. Inhibitors of sodium channels: The drugs in this cate- as the meninges.
gory includes phenytoin, carbamazepine, valproate and l The inflammation may be caused by infection with vi-
lamotrigine. ruses, bacteria, or other microorganisms, and less com-
ii. Inhibitors of neurotransmission: The drugs in this group monly by certain drugs. This condition is classified as a
include barbiturates (phenobarbitone) and benzodiaze- medical emergency.
pines (diazepam, clonazepam and clobazam).
iii. Inhibitors of calcium channels: Ethosuxide, valproate,
lamotrigine) Clinical Features of Meningitis
iv. Drugs which decrease excitatory neurotransmission: l Severe headache is the most common symptom of men-
Felbamate, glutamate ingitis – occurring in almost 90% of cases of bacterial
Individual characteristic and uses of some of these anti- meningitis.
epileptic drugs are summarized below: l Nuchal rigidity (inability to flex the neck forward
l Phenytoin: Effective against all types of partial and passively due to increased neck muscle tone and
tonic-clonic seizures but not absence seizures. It exerts stiffness).
Section | II General Surgery 357

l Classic triad of diagnostic signs consists of nuchal ri- tactile stimuli, e.g. washing the face, brushing the teeth,
gidity, sudden high fever, and altered mental status; or exposure to a draft of air, generate excruciating pain.
however, all three features are present in only 44–46% l The objective signs of sensory loss cannot be demon-
of all cases of bacterial meningitis. strated on examination.
l Other signs commonly associated with meningitis in- l Onset is typically sudden, and bouts tend to persist for
clude photophobia (intolerance to bright light) and weeks or months before remitting spontaneously. Re-
phonophobia (intolerance to loud noises). missions may be long, but in most patients recurrence
l Small children often do not exhibit the mentioned occurs.
symptoms, and may only be irritable and looking
unwell.
l In infants up to 6 months of age, bulging of the fonta- Treatment
nelle (the soft spot on top of a baby’s head) may be l Drug therapy with carbamazepine is effective in
present. ,50 to 75% of patients. Carbamazepine should be
l Other signs of meningism: Positive Kernig’s sign or started as a single daily dose of 100 mg taken with
Brudzinski’s sign. food, and increased gradually (by 100 mg daily every
Kernig’s sign is assessed with the patient lying supine, 1 to 2 days) until substantial (.50%) pain relief is
with the hip and knee flexed to 90 degrees. In a patient achieved. Most patients require a maintenance dose
with a positive Kernig’s sign, pain limits passive exten- of 200 mg qid. Doses .1200 mg daily provide no
sion of the knee. A positive Brudzinski’s sign occurs additional benefit.
when flexion of the neck causes involuntary flexion of l If treatment is effective, it is usually continued for ap-
the knee and hip. proximately 1 month and then tapered as tolerated.
l Another test, known as the “jolt accentuation manoeu- l If carbamazepine is not well tolerated or is ineffective,
vre” helps determine whether meningitis is present in phenytoin, 300 to 400 mg daily, can be tried.
patients reporting fever and headache. The patient is l Baclofen may also be administered, either alone or in
told to rapidly rotate his or her head horizontally; if this combination with carbamazepine or phenytoin. The
does not make the headache worse, meningitis is un- initial dose is 5 to 10 mg tid, gradually increasing as
likely. needed to 20 mg qid.
l Adults presenting with community-acquired acute bac- l If drug treatment fails, surgical therapy should be of-
terial meningitis, present with at least two of the follow- fered. The most widely applied procedure creates a
ing four symptoms: heat lesion of the trigeminal (gasserian) ganglion or
a. Headache nerve, a method termed radiofrequency thermal rhi-
b. Fever zotomy.
c. Neck stiffness l A third treatment, microvascular decompression, re-
d. Altered mental status (as defined by a score below quires a suboccipital craniotomy. This procedure has a
14 on the Glasgow Coma Scale) .70% efficacy rate and a low rate of pain recurrence in
Q. 7. Trigeminal neuralgia—clinical features and treat- responders.
ment. Q. 8. Clinical manifestations of depression.
Ans. Ans.
Clinical manifestations of trigeminal neuralgia are as fol- l Depression is defined as depressed mood on a daily
lows: basis for a minimum duration of 2 weeks.
l Excruciating paroxysms of pain in the lips, gums, l An episode may be characterized by sadness, indiffer-
cheek, or chin and, very rarely, in the distribution of the ence, apathy, or irritability and is usually associated
ophthalmic division of the fifth nerve. with: Changes in sleep patterns, appetite, and weight;
l The pain rarely lasts more than a few seconds or a min- motor agitation or retardation; fatigue; impaired con-
ute or two but may be so intense that the patient winces. centration and decision-making; feelings of shame or
l The paroxysms, experienced as single jabs or clusters, guilt; and thoughts of death or dying.
tend to recur frequently, both day and night, for several l Patients with depression have a profound loss of plea-
weeks at a time. They may occur spontaneously or with sure in all enjoyable activities, exhibit early morning
movements of affected areas evoked by speaking, chew- awakening, feel that the dysphoric mood state is quali-
ing, or smiling. tatively different from sadness, and often notice a diur-
l Presence of trigger zones, typically on the face, lips, or nal variation in mood (worse in morning hours).
tongue, that provoke attacks; patients may report that
358 Quick Review Series for BDS 3rd Year

Criteria for Major Depressive Episode ii. Criteria for ideal nerve graft:
l Should be immunologically acceptable.
l Depressive mood for most of the day, nearly every day.
l Should undergo Wallerian degeneration.
l Anhedonia: Markedly diminished interest or pleasure in all,
l Should contain active nerve cells.
or almost all, activities most of the day, nearly every day.
l Should become vascularized after being placed in a
l Significant weight loss when not dieting or weight gain
favourable nourished bed.
or decrease or increase in appetite nearly every day.
iii. The primary sources of nerve grafts include:
l Insomnia or hypersomnia nearly every day.
l Sural nerves
l Psychomotor agitation or retardation nearly every day
l Superficial radial nerves
(observable by others, not merely subjective feelings of
l Medial and lateral cutaneous nerves of the forearm
restlessness or being slowed down).
l Medial and lateral cutaneous nerves of the arm
l Fatigue or loss of energy nearly every day.
l Posterior cutaneous nerve of the thigh
l Feelings of worthlessness or excessive or inappropriate
l Anterolateral femoral cutaneous nerve of the thigh.
guilt (which may be delusional) nearly every day (not
iv. New techniques depend primarily upon microsurgical
merely self-reproach or guilt about being sick).
techniques and instruments, requiring surgeons to su-
l Diminished ability to think or concentrate, or indeci-
ture under the microscope using fine suture materials.
siveness, nearly every day (either by subjective account
v. The 4 key elements of the grafting process:
or as observed by others).
a. The detrimental role of tension at the suture line
l Recurrent thoughts of death (not just fear of dying), re-
b. The deleterious effect of postoperative stretching on
current suicidal ideation without a specific plan, or a
successful functional recovery
suicide attempt or a specific plan for committing suicide.
c. The regeneration axons advanced more easily through
Q. 9. Electrocoagulation of trigeminal ganglion. nerve grafts of 2 cm with 2 tension-free anastomoses
compared with a single suture under tension
Ans.
d. The epineurium was the primary source of connec-
i. Electrocoagulation of trigeminal ganglion or radiofre- tive tissue proliferation.
quency thermal rhizotomy refers to percutaneous heat
ablation of gasserion ganglion at the base of the skull. Q. 11. Neuropraxia.
ii. This is an outpatient procedure performed by placing a Ans.
needle into the gasserian ganglion, through which an
electrical current passes, heating the probe and produc- i. Neurapraxia is part of Seddon’s classification scheme
ing a thermal lesion in the ganglion. used to classify nerve damage. It is a transient episode
iii. The reported rate of pain recurrence is the lowest com- of motor paralysis with little or no sensory or auto-
pared with other percutaneous procedures, with the nomic dysfunction.
average patient experiencing 3 years of excellent pain ii. Neuropraxia is the mildest type of peripheral nerve
relief. This procedure has a high success rate. injury and is characterized by a temporary interruption
of conduction without loss of axonal continuity.
Side effects: Facial numbness, which can range from iii. In neuropraxia, there is a physiologic block of nerve
mild to severe. Results can be temporary and additional conduction in the affected axons.
treatment may be necessary. iv. Neuropraxia describes nerve damage in which there is
no disruption of the nerve or its sheath. In this case,
Complications there is an interruption in conduction of the impulse
down the nerve fibre, and recovery takes place without
l Unintended nerve damage
true regeneration.
l Failure to access the trigeminal nerve or gasserion ganglion
v. This is probably a biochemical lesion caused by con-
l Bleeding from puncture site
cussion or shock-like injuries to the fibre. In the case of
l Apnoea
the role nerve, neurapraxia is brought about by com-
Q. 10. Nerve grafting. pression or relatively mild, blunt blows, including
some low-velocity missile injuries close to the nerve.
Ans.
vi. There is a temporary loss of function which is revers-
i. Nerve grafting is defined as replacement of a damaged ible within hours to months of the injury (the average
nerve with a section of a healthy nerve that has been is 6–8 weeks).
removed from another part of the body. vii. There is frequently greater involvement of motor than
l Nerve grafting is indicated when nerve suture is ei- sensory function with autonomic function being re-
ther impossible or undesirable. tained. This form of nerve-impulse interruption should
Section | II General Surgery 359

not be ignored and the patient should refer to a physi- vi. Other characteristics of neurotmesis are as follows:
cian as soon as possible. l Wallerian degeneration occurs below to the site of

viii. In sports, neuropraxia is common and happens to most injury.


athletes during their sports lifespan. Patients should l There is connective tissue lesion that may be partial

also refrain from playing sports or putting stress on or complete.


the injured area. l Sensory-motor problems and autonomic function

ix. Other characteristics of neuropraxia are: defect are severe.


l There are sensory-motor problems distal to the site l There is not nerve conduction distal to the site of

of injury. injury (3 to 4 days after lesion).


l The endoneurium, perineurium, and the epineu- l EMG and NCV findings are as axonotmesis.

rium are intact. l Because of lack of nerve repair, surgical intervention

l There is not wallerian degeneration. is necessary.


l In neuropraxia, conduction is intact in the distal
Q. 13. Axonotmesis.
segment and proximal segment, but no conduction
occurs across the area of injury. Ans.
l Recovery of nerve conduction deficit is full, and
i. Axonotmesis is a type of nerve injury accompanied by
requires days to weeks.
damage of axons and their myelin sheaths but Schwann
l EMG shows lack of fibrillation potentials (FP) and
cells, the endoneurium, perineurium and epineurium
positive sharp waves.
are intact.
Q. 12. Neurotmesis. ii. Axonotmesis is usually the result of a more severe
crush or contusion than neuropraxia.
Ans.
iii. In axonotmesis, the proximal section is repaired by
i. Neurotmesis is a total severance or disruption of the creating a sprout with its growth cone, but in the distal
entire nerve fibre. Neurotmesis may be partial or com- section occurs axonal degeneration.
plete. iv. The rate of outgrowth of regenerating nerve fibres is
ii. It is the most serious nerve injury. In this type of injury, about 1 to 2 mm per day, so that the recovery of conduc-
both the nerve and the nerve sheath are disrupted. tion to a target structure depends on not only regrowth
While partial recovery may occur, complete recovery is into the appropriate endoneurial tube (endoneurium),
impossible. but also on the distance involved.
iii. It occurs in cases of severe contusion, stretch, lacera- v. Other characteristics of axonotmesis are:
tions. Not only the axon, but the encapsulating connec- l Wallerian degeneration occurs below to the site of

tive tissues loses their continuity. injury.


iv. The last (extreme) degree of neurotmesis is trans­ l There are sensory and motor deficits distal to the site

section, but most neurotmetic injuries do not produce of lesion.


gross loss of continuity of the nerve but rather than in- l There is not nerve conduction distal to the site of

ternal disruption of the architecture of the nerve suffi- injury (3 to 4 days after injury).
cient to involve perineurium and endoneuruim as well l EMG shows fibrillation potentials (FP), and positive

as axons and their covering. sharp waves (2 to 3 weeks post-injury).


v. There is a complete loss of motor, sensory and auto- l Axonal regeneration occurs and recovery is possi-

nomic function. If the nerve loss has been completely ble without surgical treatment. Sometimes surgical
divided, axonal regeneration causes a neuroma to form intervention because of scar tissue formation is
in the proximal stump. required.

SHORT NOTES
Q. 1. Write short notes on lockjaw. ii. Other causes of lockjaw are as follows: Pericoronitis,
inflammation of muscles of mastication, peritonsillar
Ans.
abscess, and temporomandibular joint disorder (TMD).
i. Lockjaw or trismus refers to a pathological condition in iii. Lockjaw is also a common temporary side effect of
which the mouth is held shut by sustained spasm of the many stimulants of the sympathetic nervous system
masseter (jaw) muscle. This condition is most often such as recreational drugs methylenedioxymeth­
observed in cases of tetanus. amphetamine (MDMA), methamphetamine (meth),
360 Quick Review Series for BDS 3rd Year

methylphenidate(Ritalin), mephedrone(4-MMC), am- of prostaglandins, which are involved in the patho-


phetamine. physiology of migraine headaches.
l Despite the introduction more than a decade ago of a
new class of migraine-specific drugs with superior
Treatment
efficacy, the triptans, NSAIDs remain the most com-
l Treatment of the underlying condition: Dental treatments, monly used therapies for the migraine attack.
physical therapy, and passive range of motion devices. l This is in part due to their wide availability as over-the-
l Symptomatic therapy: With pain medications (NSAIDs), counter drugs and their pharmaco-economic advan-
muscle relaxants, and warm compresses may be used. tages, but also to a favourable efficacy/side effect pro-
Splints have been used. file at least in attacks of mild and moderate intensity.
Q. 2. Write short note on migraine. Q. 4. Write short note on headache.
Ans. Ans.
i. Migraine is a paroxysmal headache disorder. Migraine l Headache (cephalgia) refers to pain anywhere in the
is a syndrome and not a disease and is characterized by head and neck region. This pain is caused by distur-
paroxysmal headache associated with others signs and bance of the pain-sensitive structures around the brain,
symptoms. which are divided in two categories: Within the cranium
ii. Although migraine refers to certain headaches with a (blood vessels, meninges, and the cranial nerves) and
vascular quality, it is a dominantly inherited disorder outside the cranium (the periosteum of the skull, mus-
characterized by varying degrees of recurrent vascular- cles, nerves, arteries and veins, subcutaneous tissues,
quality headache, photophobia, sleep disruption, and eyes, ears, sinuses and mucous membranes).
depression. l Headaches are classified by the International Headache
iii. Types of migraine variants: According to ICHD-2, Society’s.
there are following subclasses of migraines (some of l International Classification of Headache Disorders
which include further subdivisions): (ICHD), which is most widely used. The top, diagnostic
l Migraine without aura or common migraine. level includes 13 headache groups.
l Migraine with aura l 1-4 are classified as primary headaches, groups 5-12 as
l Childhood periodic syndromes secondary headaches, cranial neuralgia, central and pri-
l Retinal migraine mary facial pain and other headaches for the last two
l Complications of migraine groups.
l Probable migraine l The ICHD-2 classification defines migraines, tension-
iv. Treatment: type headaches, cluster headache and other trigeminal
l Treatment of migraine includes: Eliminating partic- autonomic cephalalgias as the main types of primary
ular triggers, acute management of the specific at- headaches. Also, according to the same classification,
tack, and long-term preventive approach. headaches due to stabbing, cough, exertion and sexual
l Most commonly used medications in case of mi- activity are classified as primary headaches as well as
graine are: NSIADs, sumatriptan and related selec- daily-persistent headaches along with the hypnic head-
tive serotonin receptor agonists. ache and thunderclap headaches.
l Secondary headaches are classified based on their aeti-
Q. 3. Migraine—name two drugs for treatment.
ology and not on their symptoms and includes those that
Ans. are due to head or neck trauma such as whiplash injury,
intracranial haematoma, post-craniotomy or other head
The two drugs used to treat migraine are as follows:
or neck injury.
i. Sumatriptan:
l Headaches caused by cranial or cervical vascular disor-
l It is structurally similar to serotonin (5HT), and is a
ders such as ischaemic stroke and transient ischaemic
5-HT (types 5-HT1D and 5-HT1B) agonist. The spe-
attack, non-traumatic intracranial haemorrhage, vascular
cific receptor subtypes it activates are present on the
malformations are also defined as secondary headaches.
cranial arteries and veins.
l Acting as an agonist at these receptors, sumatriptan Q. 5. Write short note on tension headache.
reduces the vascular inflammation associated with
Ans.
migraine.
ii. NSIADs: l Tension headache is classified under primary headaches
l By blocking cyclooxygenase nonsteroidal anti- in ICHD classification and is the most common type of
inflammatory drugs (NSAIDs), decrease the synthesis headache.
Section | II General Surgery 361

l Tension-type headaches can be episodic (fewer than Q. 7. Write a short note on Bell’s palsy.
15 days a month) or chronic (15 days or more a month
for at least 6 months). Ans.
l Tension-type headaches can last from minutes to days, l Bell’s palsy is a unilateral, peripheral facial paresis or
months or even years, though a typical tension headache paralysis that has an abrupt onset and no detectable
lasts 4–6 hours. cause. This is cause by dysfunction of cranial nerve VII
(the facial nerve) that results in inability to control facial
muscles on the affected side.
Aetiology l Bell’s palsy is the most common acute mononeuropathy
l Stress, sleep deprivation, uncomfortable stressful posi- (disease involving only one nerve) and is the most com-
tion and/or bad posture, hunger, eyestrain, caffeine mon cause of acute facial nerve paralysis.
withdrawal, and dehydration. l The characteristic feature is rapid onset of partial or
complete palsy, usually in a single day. The paralysis is
of the infranuclear/lower motor neuron type.
Symptoms l Diagnosis: Bell’s palsy is a diagnosis of exclusion.
l Tension headache pain is described as a constant pres- l Treatment: Corticosteroid such as prednisone significantly
sure. The pain is frequently bilateral. improves recovery at 6 months and is thus recommended.
Q. 8. Facial pain—four causes.
Treatment: Ans.
l Over the counter analgesics: Ibuprofen, paracetamol/ l Facial pain can be associated with pathologic conditions
acetaminophen, and aspirin. or disorders related to somatic and neurologic structures.
l For chronic tension headaches: Amitriptyline, mirtazap- l These pain disorders may originate from intracranial
ine, topiramate, and sodium valproate (as prophylaxis). lesions, such as neoplasms, aneurysms, or abscesses.
Q. 6. Write short note on trigeminal neuralgia. l Pain can also be characterized as primary headaches,
which may be neurovascular, in the case of migraine
Ans. pain, or tension-type.
l Trigeminal neuralgia or tic douloureux (also known as l Several neurogenic pain disorders, of which the most
prosopalgia the ‘suicide disease or ‘Fothergill’s disease) common may be trigeminal neuralgia, also occur in the
is a neuropathic disorder characterized by episodes of facial area.
intense pain (stabbing or shock-like) in the face, origi- l Pain may also arise from the associated structures of the
nating from one of the three trigeminal nerves. head, such as ears, eyes, nose, paranasal sinuses, throat,
l Trigeminal neuralgia is characterized by pain often ac- lymph nodes, salivary glands, and the neck.
companied by a brief facial spasm or tic. l On the other hand, facial pain can originate in the oral
l Pain distribution is unilateral and follows the sensory area and associated structures, including pain from the
distribution of cranial nerve V, typically radiating to the dental pulp, periodontium, mucogingival tissues, and
maxillary (V2) or mandibular (V3) area. At times, both the tongue.
distributions are affected. l Additionally, temporomandibular disorders are a com-
l Pain is brief and paroxysmal, but it may occur in volleys mon cause of facial pain. Besides somatic pain, psycho-
of multiple attacks. logical factors can either contribute to pain or actually
cause the pain disorder.

Treatment Q. 9. Anxiety disorder—any three symptoms.

l Anticonvulsants: These agents may help control parox- Ans.


ysmal pain by limiting the aberrant transmission of Following are three symptoms specific to specific anxiety
nerve impulses. Carbamazepine is the first-line drug; disorders:
second-line drugs include baclofen, lamotrigine, oxcar- i. Non-specific persistent fear and worry and become
bazepine, phenytoin, gabapentin, and sodium valproate. overly concerned with everyday matters (in cases of
l Surgery: Microvascular decompression (Janetta proce- GAD – generalized anxiety disorder).
dure) ii. Person suffers from brief attacks of intense terror and
l Stereotactic radiation therapy: Nerve can be damaged to apprehension, often marked by trembling, shaking,
prevent pain signal transmission using gamma knife or confusion, dizziness, nausea, difficulty breathing (in
a linear accelerator-based radiation therapy. case of panic disorder).
362 Quick Review Series for BDS 3rd Year

iii. Intense fear and avoidance of negative public scrutiny, vii. Treatment: The usual treatment for meningitis is the
public embarrassment, humiliation, or social interac- prompt application of antibiotics and sometimes anti-
tion (in case of SAD—social anxiety disorder). viral drugs. In some situations, corticosteroid drugs
can also be used to prevent complications from overac-
Q. 10. Write a short note on petit mal epilepsy.
tive inflammation.
Ans.
Q. 13. Write a short note on signs of meningial irritation.
i. Petit mal epilepsy are generalized seizures which oc-
Ans.
curs most often in those younger than 20, and are char-
acterized by an abrupt but brief interruption in con-
sciousness, accompanied by small jerks of the hands or a) Nuchal Rigidity
facial muscles. l involuntary muscle spasm limits passive neck flexion
ii. Because of the lack of awareness patients with this type l Patient cannot flex neck to place chin on chest
of seizure experience, a petit mal is sometimes called
an absence seizure. b) Spinal Rigidity
iii. The seizures in these conditions are called typical l Erector spinae muscle spasm limits spine movement
absence seizures and are usually associated with l Opisthotonos (rigid arched back) may occur
generalized 3-4 Hz spike-and-slow-wave complexes
on EEG. c) Kernig’s Signs
Technique
Q. 11. Write a short note on status epilepticus?
l Patient supine
Ans. l Flex both hip and knee to 90 degrees
l Hold hip immobile and extend knee
i. Status epilepticus (SE) can be defined as one continuous
unremitting seizure lasting longer than 30 minutes, or Positive Test suggesting Meningeal Irritation
ii. recurrent seizures without regaining consciousness be- l Resistance to knee extension
tween seizures for greater than 30 minutes (or shorter l Pain in hamstrings
with medical intervention).
iii. In this condition the brain is in a state of persistent sei- d) Brudzinski’s Sign
zure. It is always considered a medical emergency. Technique
Q. 12. Write a short note on meningitis l Patient supine
l Immobilize trunk against bed
Ans. l Flex neck, chin to chest
i. Meningitis is inflammation of the protective mem- l Positive Test suggesting Meningeal Irritation
branes covering the brain and spinal cord, known col- l Involuntary hip flexion
lectively as the meninges. Q. 14. Write a short note on drugs used in epilepsy.
ii. The inflammation may be caused by infection with
viruses, bacteria, or other microorganisms, and less Ans.
commonly by certain drugs. This condition is classified Following group of drugs are used as anti-epileptics:
as a medical emergency.
iii. The most common symptoms of meningitis are head- A) Sodium channel blocker
ache and neck stiffness associated with fever, confu- l Carbamazepine: CBZ is a major first-line AED for par-
sion or altered consciousness, vomiting, and an inabil- tial seizures and generalized tonic-clonic seizures.
ity to tolerate light (photophobia) or loud noises l Phenytoin:first-line AED in the treatment of partial and
(phonophobia). secondary generalized seizures
iv. Sometimes, especially in small children, only nonspe- l Oxcarbazepine: This is a recently developed analog of
cific symptoms may be present, such as irritability and CBZ.
drowsiness. l Lamotrigine
v. If a rash is present, it may indicate a particular cause of
meningitis; for instance, meningitis caused by menin- B) GABA receptor agonist
gococcal bacteria may be accompanied by a character- l Clobazam: This is a potent anticonvulsant for partial
istic rash. epilepsy
vi. Diagnosis: Lumbar puncture may be used to diagnose l Clonazepam: This is the drug of choice for myoclonic
or exclude meningitis. seizures and subcortical myoclonus
Section | II General Surgery 363

l Phenobarbital: It is a very potent anticonvulsant with a Q. 17. Local anesthesia in dentistry


broad spectrum of action. Currently, its use is limited
because of its adverse effects. PHB still is a first-line Ans.
drug for treatment of status epilepticus. However, be- i. In dentistry the most commonly used local anesthetic is
cause of its adverse-effect profile, it is a second-line lidocaine (also called xylocaine or lignocaine), a mod-
agent in the treatment of partial onset and secondarily ern replacement for novocaine and procaine.
generalized tonic-clonic seizures. ii. Other local anesthetics in current use include septo-
caine, marcaine (a long-acting anesthetic), and mepiva-
C) GABA reuptake inhibitor: caine. A combination of these may be used depending
l Tiagabine: its use is limited to adjunctive therapy in re- on the situation. Also, most agents come in two forms:
fractory partial epilepsy with and without epinephrine.
iii. The most common technique, effective for the lower
teeth and jaw, is inferior alveolar nerve anesthesia.
D) AED with potential GABA mechanism
iv. The upper teeth are usually anesthetized locally by in-
l Valproate (VPA): It is the drug of choice for primary serting the needle beneath the oral mucosa surrounding
generalized epilepsies, and is also approved for the the teeth.
treatment of partial seizures.
Q. 18. Write a short note on Frey’s syndrome
There exist many more antiepileptic drugs which has
been discussed in detail in long essay questions Ans.

Q. 15. Write a short note on dilantin sodium. i. Frey’s syndrome (also known as Baillarger’s syndrome,
Dupuy’s syndrome, Auriculo-temporal syndrome or
Ans. Frey-Baillarger syndrome) is a syndrome associated
with injury to parotid gland post surgery.
i. Dilantin sodium is the trade name for diphenyl hydan-
ii. Symptoms:
toin sodium. It is a commonly used anti-epileptic
l Redness and sweating on the cheek area adjacent to
drug.
the ear.
ii. It acts to suppress the abnormal brain activity seen in
l Gustatory Sweating” or sweating in the anticipation
seizure by reducing electrical conductance among brain
of eating, instead of the normal salivatory response.
cells by stabilizing the inactive state of voltage-gated
iii. Treatment:
sodium channels.
l injection of botulinum toxin type A
iii. It is used as first line therapy for partial and generalized
l surgical transection of the nerve fibers
tonic clonic seizures.
l application of an ointment containing an anticholin-

Q. 16. Write a short note on neuropraxia ergic drug such as scopolamine

Ans. Q. 19. Write a short note on Horner’s syndrome

i. Neuropraxia is the mildest type of peripheral nerve in- Ans.


jury and is charterized by a temporary interruption of i. Horner’s syndrome or Horner syndrome is a clinical
conduction without loss of axonal continuity. syndrome caused by damage to the sympathetic ner-
ii. In neuropraxia, there is a physiologic block of nerve vous system.
conduction in the affected axons. ii. It is also known by the names Bernard-Horner syn-
iii. Other characteristics of neuropraxia are: drome orClaude Bernard-Horner syndrome or as oculo-
l There are sensory-motor problems distal to the site sympathetic palsy.
of injury. iii. Signs found in all patients on affected side of face
l The endoneurium, perineurium, and the epineurium include:
are intact. l ptosis, anhidrosis, dilation lag, Enophthalmos, loss
l There is not wallerian degeneration. of ciliospinal reflex and bloodshot conjunctiva may
l In neuropraxia, conduction is intact in the distal seg- occur depending on the site of lesion.
ment and proximal segment, but no conduction oc- iv. Diagnosis: Three tests are useful in confirming the
curs across the area of injury. presence and severity of Horner syndrome:
l Recovery of nerve conduction deficit is full, and re- a. Cocaine drop test – Cocaine eyedrops block the reup-
quires days to weeks. take of norepinephrine resulting in the dilation of a
l EMG shows lack of fibrillation potentials (FP) and normal pupil. Due to the lack of norepinephrine in the
positive sharp waves. synaptic cleft, the pupil will fail to dilate in Horner’s
364 Quick Review Series for BDS 3rd Year

syndrome. Nowadays alpha-agonist apraclonidine iii. In axonotmesis, the proximal section is repaired by
are applied to both eyes and observe the reversal of creating a sprout with its growth cone, but in the distal
miosis on the affected side of Horner syndrome. section occurs axonal degeneration.
b. Paredrine test: This test helps to localize the cause of iv. Other characteristics of axonotmesis are:
the miosis. l Wallerian degeneration occurs below to the site of

c. Dilation lag test injury.


l There are sensory and motor deficits distal to the site
Q. 20. Write a short note on axonotmesis of lesion.
l There is not nerve conduction distal to the site of
Ans.
injury (3 to 4 days after injury).
i. Axonotmesis is a type of nerve injury accompanied by l EMG shows fibrillation potentials (FP), and positive
damage of axons and their myelin sheaths but sharp waves (2 to 3 weeks post injury).
schwanncells, the endoneurium, perineurium and epi- l Axonal regeneration occurs and recovery is possi-
neurium are intact. ble without surgical treatment. Sometimes surgical
ii. Axonotmesis is usually the result of a more severe intervention because of scar tissue formation is
crush or contusion than neuropraxia. required

Topic 12

Fractures—General Principles

LONG ESSAYS
Q. 1. Le Fort’s classification of fractures of maxilla. II. Le Fort II fractures (pyramidal #):
Ans. l They may result from a blow to the lower or mid-maxilla.
l Such a fracture has a pyramidal shape and extends from
Renee Le Fort (1901) provided the earliest classification the nasal bridge at or below the nasofrontal suture
system of maxillary fractures. through the frontal processes of the maxilla, inferolater-
He found 3 distinct fracture patterns. In the Le Fort ally through the lacrimal bones and inferior orbital floor
I fracture, the palate is separated from the maxilla; in the and rim through or near the inferior orbital foramen, and
Le Fort II fracture, the maxilla separates from the face; inferiorly through the anterior wall of the maxillary
and in the Le Fort III fracture, craniofacial dysjunction is sinus; it then travels under the zygoma, across the ptery-
present. According to this classification, there exist 3 types gomaxillary fissure, and through the pterygoid plates.
of maxillary fractures: l Only the Le Fort II fracture violates the orbital rim.
Because of this proximity to the infraorbital foramen,
I. Le Fort I fractures (horizontal #): Le Fort II fractures are associated with the highest
l They may result from a force of injury directed low on incidence of infraorbital nerve hypesthesias.
the maxillary alveolar rim in a downward direction. l The Le Fort II fracture extends posteriorly to the ptery-
l The fracture extends from the nasal septum to the lateral goid plates at the base of the skull. A Le Fort I fracture
pyriform rims, travels horizontally above the teeth api- is characterized by a low septal fracture, whereas a
ces, crosses below the zygomaticomaxillary junction, Le Fort II fracture results in a high septal fracture.
and traverses the pterygomaxillary junction to interrupt
the pterygoid plates. III. Le Fort III fractures (transverse #):
l In other words, the Le Fort I fracture is a low transverse l They are also termed as craniofacial dysjunctions, may
fracture that crosses the floor of the nose, pyriform ap- follow impact to the nasal bridge or upper maxilla.
erture, canine fossa, and lateral wall from the maxilla, l These fractures start at the nasofrontal and frontomaxil-
resulting in separation of the palate from the maxilla. lary sutures and extend posteriorly along the medial
Section | II General Surgery 365

wall of the orbit through the nasolacrimal groove and maxillary sinuses on axial CT and inferior to the orbital
ethmoid bones. The thicker sphenoid bone posteriorly rim on coronal slices.
usually prevents continuation of the fracture into the
optic canal. Instead, the fracture continues along the Treatment
floor of the orbit along the inferior orbital fissure and
continues superolaterally through the lateral orbital l The goals of treatment of Le Fort fractures are to rees-
wall, through the zygomaticofrontal junction and the tablish preinjury occlusion with normal height and
zygomatic arch. Intranasally, a branch of the fracture projection of the face.
extends through the base of the perpendicular plate of l To accomplish this, the structural buttress of the maxilla
the ethmoid, through the vomer, and through the inter- must be aligned and stabilized to provide the necessary
face of the pterygoid plates to the base of the sphenoid. support and contour to the midface.
l Because of their location, Le Fort III fractures are as- l The proper occlusal relationship between the dental arches
sociated with the highest rate of cerebrospinal fluid is established with intermaxillary fixation (IMF), or more
(CSF) leaks. appropriately termed maxillomandibular fixation.

Le Fort I fractures
Clinical Picture l For stable, nondisplaced Le Fort I fractures, maxillo-
Le Fort fracture I # mandibular fixation (MMF) alone may be sufficient to
provide stable restoration of bony support.
l Slight swelling of the upper lip.
l Partial or segmental alveolar ridge fractures can also be
l Ecchymosis is present in the buccal sulcus beneath each
treated with MMF alone after proper reduction.
zygomatic arch.
l However, unstable fractures require an additional means
l Malocclusion, mobility of teeth.
of fixation. Some prefer to place additional fixation even
l Impacted type of fractures may be almost immobile and
to nondisplaced fractures, with the goal of allowing
it is only by grasping the maxillary teeth and applying
earlier removal of MMF and return to mastication.
little firm pressure that a characteristic grate can be felt
l The method of choice for fixation is through miniplates
which is diagnostic of the fracture.
placed via an open approach.
l Percussion of upper teeth results in cracked pot sound.
l For true Le Fort I fractures, one plate across the naso-
l Guerin’s sign is present, characterised by ecchymosis in
maxillary or zygomaticomaxillary buttress on each side
the region of greater palatine vessels.
is usually adequate for stable fixation.
l The most common method is low-profile titanium plates
Le Fort fracture II # and Le Fort fracture
secured with monocortical self-tapping screws.
III #(common features)
l Gross oedema of soft tissue over the middle third of the Le Fort II fractures
face, bilateral, circumorbital ecchymosis, bilateral sub- l Just as for Le Fort I fractures, disimpaction, MMF, and
conjunctival haemorrhage. sublabial incisions and exposure of maxillary bone and
l Epistaxis, CSF rhinorrhoea, dish face deformity. fracture lines are performed.
l Diplopia, enophthalmos, cracked pot sound. l Additional exposure is often necessary superiorly for
l Le Fort fracture II also manifests by step deformity at adequate exploration of the orbital rim.
infraorbital margin, mobile mid-face, anaesthesia or l In general, the pyramidal free maxillary segment is sta-
paraesthesia of cheek. bilized to the intact zygoma.
l Le Fort III—tenderness and separation at frontozygo- l As rigid fixation is a traumatic procedure, it is recom-
matic suture, lengthening of face, depression of occular mended to be performed until reduction is optimised.
levels, enophthalmos, hooding of eyes, tilting of occlu- l Fixation may be completed directly using noncompres-
sal plane with gagging on one side. sion miniplates that span the break in the region of the
zygomaticomaxillary buttresses.
If instability persists, additional plates may be placed in
Diagnosis l

the nasomaxillary buttresses or inferior orbital rim.


Diagnosis is made on the basis of: Monocortical, self-tapping screws are ideal.
l Physical examination and confirmation by axial CT.
l X-ray of the head and neck is performed and if fracture Le Fort III fractures
signs are present, CT is performed. l In repairing Le Fort III fractures, stabilize the mobile
l To qualify for Le Fort fractures, the pterygoid plates segments of bone to the stable mandible below and
must be involved. These are seen posterior to the cranium above.
366 Quick Review Series for BDS 3rd Year

l Initially, the maxilla must be disimpacted and MMF l CT scanning


implemented. l CT scanning allows complete assessment of
l Soft tissue incisions may be made in the same locations fractures.
as for Le Fort II fractures. l 3D reconstruction is useful.

l Allows production of a stereolithograph and a 1:1


Q. 2. Discuss the management of maxillofacial injuries.
resin model from the digital image.
Ans.
D. Classification
Maxillofacial injuries or facial and orbital fractures are:
l Usually seen following sporting accidents, interpersonal l Fractures are usually classified as:
l Upper third—frontal bones
violence and RTAs.
l Middle third—zygoma, nasal bones, and maxilla
l May be associated with multisystem trauma.
l Lower third—mandible and teeth
l Assessment should follow ATLS principles.
l Le Fort fractures: Fractures of mid-portion of face
have been classified as:
Assessment l Le Fort I: Fracture detaching palate and maxillary

alveolus.
A. Primary survey
l Le Fort II: Pyramidal fracture through sinus wall
l Airway compromise from a fracture or haemorrhage laterally and nasal bones medially.
has to be identified. l Le Fort III: Fracture through frontozygomatic
l Bilateral anterior mandibular fractures has the risk of sutures and orbits detaching facial skeleton from
tongue falling back. base of skull.
l Orotracheal intubation may be required.
l Haemorrhage is controlled or reduced with mouth props E. Principles of treatment
and epistaxis balloons.
l Primary repair produces the best cosmetic results.
l Anterior and posterior nasal packing may be required.
l May be delayed for 2 or 3 days, if multidisciplinary
B. Secondary survey approach required.
l Open reduction and internal fixation is treatment of
l Palpation of orbital rims, zygomatic arches and mandi- choice allowing:
ble to identify fractures. l Anatomical reduction of fractures
l Examination of eyes. l Stable internal fixation
l Reduced/restricted eye movement may suggest orbital l Early jaw mobilisation
fracture.
l Subconjunctival haemorrhage may suggest skull fracture. Q. 3. Classify the fractures. Describe the treatment of
l Proptosis and ophthalmoplegia may suggest retrobulbar fractured mandible.
haemorrhage.
Ans.
l Assess sensation in maxillary branch of trigeminal
nerve. A fracture is defined as a disruption in the integrity of a
l Intercanthal distance should be 30 - 35 mm. living bone, involving injury to the bone marrow, perios-
l Intercanthal distance greater than 35 mm suggests a teum, and adjacent soft tissues.
nasoethmoid fracture.
Interpupillary distance should be 55 mm.
Classification of Fractures
l

l Intraoral examination is essential.


l Assessment of occlusion and intraoral haematomas. Fracture classification has been described depending on
various criteria:
C. Radiology
l May be difficult to obtain films in the acute setting A. Based on fracture pattern
l Useful radiographs include: l Linear fractures: Fracture that runs parallel to the long
l Occipitomental views (15° and 30°) for orbital and axis of a bone.
zygomatic fractures l Transverse fractures: Fracture in right angle with the
l Posteroanterior views of facial bones axis of the bone.
l Submentovertex view for zygomatic arch fractures l Oblique fractures: Fracture with oblique fracture line.
l Orthopantomogram (OPG) for mandibular fractures l Spiral fractures: Fracture with spiral fracture line.
l Reverse Townes view for condyle neck fractures l Comminuted fractures: Fracture with splintered or
l Occlusal films for dentoalveolar fractures crushed bone with more than two fragments.
Section | II General Surgery 367

l Segmental fractures: Fracture where a part of bone is com- l Type III fractures are those in which the fragments
pletely separated from parent bone, with same diameter. are not in contact, and the head is displaced medially
and forward. The fragments are confined within the
B. Based on displacement pattern area of the glenoid fossa. The capsule is torn, and the
l Angulation head is outside the capsule.
l Shift/translation l Type IV fractures of the condylar head articulate on
l Rotation or in a forward position with regard to the articular
l Length eminence.
l Type V fractures consist of vertical or oblique
C. Based on condition of Soft tissue fractures through the head of the condyle.
l Closed/simple fracture: Fracture which lacks communi-
cation between the site of fracture and exterior of body. Management of fracture of mandible
l Open/compound fracture: Fracture in which a wound l The indications for closed versus open reduction have
through the adjacent or overlying soft tissue communi- changed dramatically over the last century.
cates with the outside of the body; this must be consid- l The ability to treat fractures with open reduction
ered a surgical emergency. and rigid internal fixation (ORIF) has dramatically
revolutionized the approach to mandibular fractures.
Special Types of Fracture l Traditionally, closed reduction (CR) and ORIF with
wire osteosynthesis have required an average of 6 weeks
l Greenstick fracture one in which one side of a bone is of immobilization by maxillomandibular fixation
broken and the other is bent, most commonly seen in (MMF) for satisfactory healing.
children. l The use of preoperative and perioperative antibiotics
l Avulsion fracture separation of a small fragment of in the treatment of mandible fractures, especially in
bone cortex at the site of attachment of a ligament or the dentate portion is well established to reduce the risk
tendon. of infection.

Indications of open reduction and fixation are:


Fracture of Mandible
l Displaced unfavourable fractures through the angle of
Mandible fractures are a frequent injury because of the the mandible
mandible’s prominence and relative lack of support. Frid- l Severely atrophic edentulous mandibles
rich and associates showed that most fractures occur in the l Complex facial fractures
body (29%), condyle (26%), and angle (25%) of the man- l Condylar fractures
dible. The symphyses account for 17% of mandibular frac- l Mandibular non-unions
tures, whereas fractures of the ramus (4%) and coronoid l Malunions after improper reduction
process (1%) have lower occurrence rates. In automobile
accidents, the condylar region was the most common frac- The procedure used for ORIF are wire osteosynthesis, in-
tured site. In motorcycle accidents, the symphysis was traoral approach, submandibular approach, retromandibular
fractured most often. When assault was the cause, the angle and preauricular approach.
demonstrated the highest incidence of fracture. Indications of closed reduction are:
Nondisplaced favourable fractures.
Classification of mandibular fracture: l

l Grossly comminuted fractures: Generally, these are best


i. Angle fractures may be classified as: treated by closed reduction to minimize stripping of the
l Vertically favourable or unfavourable and
periosteum of small bone fragments.
l Horizontally favourable or unfavourable.
l Fractures in children involving the developing dentition:
ii. Condylar fractures are classified as extracapsular, Such fractures are difficult to manage by open reduction
subcondylar, or intracapsular. Five types of condylar because of the possibility of damage to the tooth buds or
fractures are described in order of increasing severity: partially erupted teeth.
l Type I is a fracture of the neck of the condyle with
l Coronoid fractures: These fractures usually require
relatively slight displacement of the head. The angle no treatment unless impingement on the zygomatic arch
between the head and the axis of the ramus varies is present.
from 10-45°.
l Type II fractures produce an angle from 45-90°, The procedures used for closed reductions are:
resulting in tearing of the medial portion of the joint Erich arch bars, bridle wires, Ivy loops, and biphasic pin
capsule. fixation.
368 Quick Review Series for BDS 3rd Year

Q. 4. Clinical signs, symptoms and general principles Treatment of Fracture


of treatment of fracture.
l Treatment of fractures can be classified as surgical or
Ans. conservative.
Signs and symptoms of fracture: l The term conservative refers to any non-surgical proce-
l Pain at or near the seat of fracture. dure, like as pain management, immobilization or other
l Tenderness of discomfort on getle pressure over the non-surgical stabilization.
affected area. l Pain management
l Swelling about the seat of fracture. Swelling frequently l Immobilization
render it difficult to perceive other signs of fracture and l The fractured limb is usually immobilized with a plaster
care must be taken, therefore, not to treat the condition or fiberglass cast or splint.
as a less serious injury. l When the initial post-fracture oedema or swelling goes
l Loss or power; the injured part cannot be moved normally. down, the fracture may be placed in a removable brace
l Deformity of the limb; the limb may assume an unnatural orthosis.
position and be mis-shapen. l If being treated with surgery, surgical nails, screws,
l The contracting muscles may cause the broken ends of plates and wires are used to hold the fractured bone
the bone to override, thereby producing shortening of together more directly.
the limp. l Alternatively, fractured bones may be treated by the
l Irregularity of the bone. If the fracture is near the skin, Ilizarov method which is a form of external fixator.
the irregularity of the bone may be felt. l By allowing only limited movement, fixation helps
l Crepitus (bony grating) may be heard or felt. preserve anatomical alignment while enabling callus
l Unnatural movement at the seat of the fracture. formation, towards the target of achieving union.

SHORT ESSAYS
Q. 1. Fracture mandible. v. Ramus #: Bounded by the superior aspect of the an-
gle to two lines forming an apex at the sigmoid notch.
Ans.
vi. Condylar process #: Area of the condylar process
l Mandible fractures are a frequent injury because of the superior to the ramus region.
mandible’s prominence and relative lack of support. vii. Coronoid process #: Includes the coronoid process
l Most fractures occur in the body (29%), condyle (26%), of the mandible superior to the ramus region.
and angle (25%) of the mandible. viii. Alveolar process #: Region that normally contains
The symphyses account for 17% of mandibular teeth.
fractures, whereas fractures of the ramus (4%) and
coronoid process (1%) have lower occurrence rates. Management of Mandible Fracture
l The most common fractured sites in:
Automobile accidents → the condylar region The steps involved in management of mandibular fractures
Motorcycle accidents → the symphysis are as follows:
Assault cases → the angle. i. Reduction
l Classification of mandible fractures according to ana- ii. Fixation
tomical regions: iii. Immobilization
i. Symphysis #: Fracture in the region of the central
incisors that runs from the alveolar process through i. Reduction of the fracture
the inferior border of the mandible. It involves realigning the bony parts in their original ana-
ii. Parasymphyseal #: Fractures occurring within the tomic relationship. Temporary holding devices may be
boundaries of vertical lines distal to the canine utilized to assist in this endeavour and reduction may be
teeth. accomplished in an open (the fracture is visibly inspected)
iii. Body #: From the distal symphysis to a line coin- or closed (the fracture is not visualized) fashion.
ciding with the alveolar border of the masseter
muscle. ii. Fixation
iv. Angle #: Triangular region bounded by the anterior It is the procedure by which the fractured bone ends are
border of the masseter muscle to the posterosupe- fixed in reduced position.
rior attachment of the masseter muscle usually l Indirect fixation by IMF
distal to the third molar. l Direct fixation by bone plate, screws.
Section | II General Surgery 369

iii. Immobilization irrigation and debridement procedures are necessary to


The reduced and fixed fragments of the bone are immobi- ensure a clean surgical wound.
lized for certain period for healing to occur. Fixation and l IIIC: Any open fracture associated with an arterial
immobilization are achieved together by the means of IMF injury that requires repair is considered type IIIC.
or with the help of bone plates and screws.
Management
Methods of immobilization:
A) Intramaxillary fixation: l The treatment goals for open fractures are to prevent
a) Dental wiring infection, to allow the fracture to heal, and to restore
Direct wiring/Glimmer’s direct method of wiring function in the injured limb.
Eyelet wiring/lvy eyelet wiring/interdental eyelet l Once the initial assessment, evaluation, and manage-
wiring. ment of any life-threatening injury are completed, the
b) Arch bar wiring open fracture is treated as follows:
c) Splints: Cast metal splint, acrylic cap splint. i. Acheivement of haemostasis.
d) Circumferential wiring. ii. Antibiotic administration: Cefazolin/clindamycin is
B) Intermaxillary fixation with osteosynthesis: preferred for type I and II open fractures while for
a) Direct intra-osseous wiring/Transosseous wiring type III fractures aminoglycoside is recommended
l Upper border intraosseous wiring to be added to the treatment.
l Lower border intraosseous wiring iii. Tetanus vaccination
l Figure 8 wiring iv. Urgent irrigation and debridement (I&D) of the
l Four-hole system wound.
l Two-hole system v. For type II and type III injuries, serial I&Ds are
b) Transfixation with Kirschner’s wiring recommended every 24-48 hours after the initial
c) External pin fixation debridement until a clean surgical wound is
d) Bone clamps ensured. The wound is closed when it is clean.
e) Implants/grafts. vi. Management of the open fracture depends on the
site of injury and type of open fracture. The wound
Q. 2. Compound fractures. is subsequently stabilized either temporarily or
Ans. definitively.
vii. If soft-tissue coverage over the injury is inadequate,
Open (compound) fractures involve wounds that communi- soft-tissue transfers or free flaps are performed
cate with the fracture, or where fracture haematoma is when the wound is clean and the fracture is defini-
exposed, and may thus expose bone to contamination and tively treated.
prone to infection.
Q. 3. Pathological fractures.
Classification of open (compound fractures):
Gustilo et al Ans.
a) Type I: The wound is smaller than 1 cm, clean, and A pathological fracture is one that results in a bone that has
generally caused by a fracture fragment that pierces the a suboptimal structure secondary to an underlying patho-
skin (i.e. inside-out injury). This is a low-energy injury. logical process.
b) Type II: The wound is longer than 1 cm, not contami-
nated, and without major soft-tissue damage or defect.
This is also a low-energy injury.
Common Predisposing Factors
c) Type III: The wound is longer than 1 cm, with signifi- l Osteoporosis—thinning of cortical bone by erosion
cant soft-tissue disruption. The mechanism often in- (particularly in femur and vertebral column in elderly).
volves high-energy trauma, resulting in a severely un- l Osteomalacia—failure of mineralisation of osteoid (frac-
stable fracture with varying degrees of fragmentation. tures are often microfractures without displacement).
l Paget’s disease—excessive uncontrolled destruction of
Type III fractures are also subdivided into the following: bone, with excessive production of osteoid (bone is
l IIIA: The wound has sufficient soft tissue to cover the weak despite increase in bulk).
bone without the need for local or distant flap coverage. l Primary or metastatic tumours (metastases mainly from
l IIIB: Disruption of the soft tissue is extensive, such breast, bronchus, thyroid, kidney).
that local or distant flap coverage is necessary to cover l Congenital bone disorders—collagen type I gene mutation,
the bone. The wound may be contaminated, and serial therefore, abnormal osteoid (e.g. osteogenesis imperfecta).
370 Quick Review Series for BDS 3rd Year

l Fragility fracture is a type of pathologic fracture that i. Reactive phase


occurs as result of normal activities, such as a fall from a. Fracture and inflammatory phase
standing height or less. b. Granulation tissue formation
l There are three fracture sites said to be typical of fragility c. Hematoid (blood clot) formation
fractures:
l Vertebral fractures, ii. Reparative phase
l Fractures of the neck of the femur, and a. Cartilage callus formation
l Colles fracture of the wrist. b. Lamellar bone deposition
l Pathological fractures present as a chalkstick fracture in
long bones, and appear as a transverse fracture nearly iii. Remodelling phase
90 degrees to the long axis of the bone. a. Remodelling to original bone contour
l In a pathological compression fracture of a spinal
vertebra, fractures will commonly appear to collapse the
Reactive Phase
entire body of vertebra.
l In circumstances where other pathologies are excluded l After fracture, the first change seen by light and electron
(e.g. cancer), a pathologic fracture is diagnostic of os- microscopy is the presence of blood cells within the
teoporosis irrespective of bone mineral density. tissues which are adjacent to the injury site.
l Soon after fracture, the blood vessels constrict, stopping
any further bleeding. Within a few hours after fracture,
Management of Pathological Fractures the extravascular blood cells form a blood clot, known
a) Labs: as a haematoma.
l Serum lab studies will help rule out systemic conditions l All of the cells within the blood clot degenerate and die.
l Sedimentation rate Some of the cells outside of the blood clot, but adjacent
l CBC, serum calcium to the injury site, also degenerate and die. Within this
b) Skeletal survey (helps rule out pending fractures in axial same area, the fibroblasts survive and replicate. They
skeleton, femurs, and humeri): Bone scan may reveal form a loose aggregate of cells, interspersed with small
other bone lesions which are not evident from radio- blood vessels, known as granulation tissue.
graphs; note that some tumours such as multiple
myeloma do not show increased uptake on bone scans.
Reparative Phase
c) Biopsy:
l Performed thru fine needle or open l Days after fracture, the cells of the periosteum replicate
l Biopsy; culture for osteomyelitis. and transform. The periosteal cells proximal to the
fracture gap develop into chondroblasts which form
Prophylactic management hyaline cartilage. The periosteal cells distal to the
a) Scoring system of risk (Mirels in 1989) fracture gap develop into osteoblasts which form wo-
l Based on 4 characteristics: Tumour location, associ- ven bone. The fibroblasts within the granulation tissue
ated pain, type of lesion (either lucent, mixed, or develop into chondroblasts which also form hyaline
blastic), and lesion size. cartilage. These two new tissues grow in size until they
l Tumour is scored from 1 to 3 in each category and a unite with their counterparts from other parts of the
total score is obtained that correlates to fracture risk. fracture.
l Prophylactic fixation is advised for lesions with l These processes culminate in a new mass of heteroge-
scores of higher than 8, and consideration for stabili- nous tissue which is known as the fracture callus.
zation are considered for scores of 8. The fracture gap is bridged by the hyaline cartilage and
woven bone, restoring some of its original strength.
Q. 4. Fracture healing. l The next phase is the replacement of the hyaline carti-
lage and woven bone with lamellar bone. The replace-
Ans.
ment process is known as endochondral ossification with
Fracture healing is a proliferative physiological process respect to the hyaline cartilage and bony substitution
facilitating the repair of a bone fracture. with respect to the woven bone.
l Substitution of the woven bone with lamellar bone
precedes the substitution of the hyaline cartilage with
Phases of Fracture Healing lamellar bone. The lamellar bone begins forming
There are three major phases of fracture healing, two of which soon after the collagen matrix of either tissue becomes
can be further sub-divided to make a total of six phases. mineralized.
Section | II General Surgery 371

l At this point, the mineralized matrix is penetrated by by a sudden impact to the joint. The ligaments always
channels, each containing a microvessel and numerous become damaged as a result of a dislocation.
osteoblasts. l A subluxation is a partial dislocation. Dislocations
l The osteoblasts form new lamellar bone upon the re- are usually caused by a sudden impact to the joint. This
cently exposed surface of the mineralized matrix. This usually occurs following a blow, fall, or other trauma.
new lamellar bone is in the form of trabecular bone. l The most common sites where dislocation or sublux-
l Eventually, all of the woven bone and cartilage of the ation is seen in the human body are: Shoulders, fingers,
original fracture callus is replaced by trabecular bone, knees, wrists and elbows.
restoring most of the bone’s original strength. l Symptoms of joint dislocation: Swelling, intense pain
and immobility of the affected joint.
Remodelling Phase
Management
l The remodelling process substitutes the trabecular bone
with compact bone. l A joint dislocation can be successfully ‘reduced’ into its
l The trabecular bone is first resorbed by osteoclasts, normal position by a trained medical professional.
creating a shallow resorption pit known as a “Howship’s l Trying to reduce a joint without proper training is
lacuna”. Then osteoblasts deposit compact bone within not recommended and can result in making the injury
the resorption pit. substantially worse.
l Eventually, the fracture callus is remodelled into a new l Radiographs are taken to confirm a diagnosis and detect
shape which closely duplicates the bone’s original any fractures which may also have occurred at the time
shape and strength. of dislocation. Once a diagnosis is confirmed, the joint
is manipulated back into position.
Q. 5. Malunion. l As this process can be very painful, it is preferably done
Ans. either under sedation or in an operating room under a
general anaesthetic.
l Malunion of a fracture refers to the healing of a fracture
with incorrect anatomical alignment. After care
l In malunion, the bone may have healed at a bent angle l After a dislocation, injured joints are usually held in
(angulated), may be rotated out of position, or the frac- place by a splint (for straight joints like fingers and toes)
tured ends may be overlapped causing bone shortening. or a bandage (for complex joints like shoulders).
l Malunion may be caused by inadequate immobilization of l Additionally, the joint muscles, tendons and ligaments
the fracture, misalignment at the time of immobilization, must also be strengthened. This is usually done through
or premature removal of the cast or other immobilizer. a course of physiotherapy.
l Symptoms of malunion are as follows:
i. Deformity Q. 7. Non-union of fractures.
ii. Swelling Ans.
iii. Pain
l Non-union is permanent failure of healing following a
iv. Tenderness
broken bone and is a very serious complication of fracture.
v. Difficulty in bearing weight
l Certain types of fractures are associated with a high risk
l Management:
of non-union, such as fractures of the wrist (carpus),
i. Malunion is treated by surgically breaking the mal-
including scaphoid bone; certain fractures of the foot
union (osteotomy), followed by open surgery to realign
and fractures of the fifth metatarsal, shoulder, long bone
the fracture fragments into their normal anatomical
fractures (proximal humerus fractures); and some shin
position (open reduction) and stabilize the fracture by
bone (tibial) fractures.
use of metal plates, rods, screws, and/or wires (internal
fixation), i.e. ORIF. Factors predisposing to non-union are:
ii. Infection requires surgical removal of any infected
l The broken ends of bone may be separated too much
bone or tissue (débridement), followed by intensive
(over distraction), which can occur if excess traction
antibiotic treatment.
was applied.
Q. 6. Definition of dislocation and partial dislocation. l Excessive motion at the fracture site, either from inad-
equate immobilization after the injury or from having a
Ans.
cast removed prematurely.
l Joint dislocation, or luxation is defined as displacement l Muscle or other tissue caught between the fracture
or misalignment of bones in a joint. It is often caused fragments also can prevent healing.
372 Quick Review Series for BDS 3rd Year

l Presence of infection l Those with an acute lesion on CT scan or evidence of


l Inadequate blood supply to the fracture site. diffuse cerebral oedema should be urgently discussed
l Bone disease (e.g. bone cancer) also can prevent healing with the local neurosurgical unit, with the CT images
transferred immediately, either by computer image-link
or courier.
Types of Nonunions l All CT scans should be accompanied by a provisional
i. Fibrous nonunion: Fractures that have healed by form- radiology report from the referring hospital.
ing fibrous tissue rather than new bone. l Other indications for neurosurgical referral include
ii. False joint (pseudarthrosis): Nonunions in which con- compound depressed skull fracture, severely depressed
tinuous movement of the fracture fragments has led to fracture, deteriorating GCS score even with a normal
the development of a false joint. scan and cerebrospinal fluid otorrhoea and rhinorrhoea.
l The following details are necessary when making a
neurosurgical referral: Name, age, sex, date, time and
Clinical Feature and Diagnosis mechanism of injury, initial GCS on scene (documented
l A history of a broken bone is usually apparent. by paramedics) and GCS following resuscitation
l The patient complains of persistent pain at the fracture (before administration of anaesthetic agents should
site and may also notice abnormal movement or click- they be required), evidence of deteriorating GCS, pupil
ing at the level of the fracture. reaction, vital observations, previous medical and drug
l An X-ray plate of the fractured bone shows a persistent history, previous functional ability and mobility in the
radiolucent line at the fracture. Callus formation may be case of elderly patients, other injuries and management
evident but callus does not bridge across the fracture. of the patient since injury.
l If there is doubt about the interpretation of the X-ray, stress l Indications to operations in head-injured patient is
X-rays, tomograms or CT scan may be used to make sure. based on a number of factors: Premorbid state (previous
medical history and functional ability of the patient),
the severity of initial injury, the onset and rapidity of
Treatment neurological deterioration and patient assessment on
l Surgical treatment includes removal of all scar tissue from arrival at the neurosurgical unit.
between the fracture fragments, immobilization of the l Important radiological features on CT scan include size
fracture with metal plates, rods and or pins and bone graft. of focal lesion(s) together with any associated surround-
l In simple cases, healing may be evident within 3 months. ing oedema and midline shift.

Q. 8. General management of a patient with head injury. Q. 9. Subdural haematoma.

Ans. Ans.

l Modern management of head injuries at the neurosurgi- i. Subdural haematoma, also known as a subdural haemor-
cal unit involves continued ventilation, surgery, inten- rhage (SDH), is a type of haematoma, a form of traumatic
sive care unit management of intracranial pressure and brain injury in which blood gathers within the outermost
cerebral perfusion pressure, oxygenation. meningeal layer, between the dura mater, which adheres
l The management of head-injured patients depends on to the skull, and the arachnoid mater enveloping the brain.
the GCS (Glasgow Coma Scale) following resuscitation. ii. Subdural haematomas are divided into acute, subacute,
l Patients with a mild head injury (GCS 14-15) should be and chronic, depending on their speed of onset.
admitted to a ward where thorough and frequent neuro-
logical observations can be ensured.
Clinical Feature
l In case such a patient subsequently deteriorate neurologi-
cally (e.g. deteriorating GCS or increasing focal neurological l Symptoms of subdural haemorrhage have a slower on-
deficit), a CT scan of the patient’s head should be performed set than those of epidural haemorrhages because the
promptly, and the local neuro-surgical unit contacted. lower pressure veins bleed more slowly than arteries.
l Patients with a mild head injury should be placed under l If the bleeds are large enough to put pressure on the
observation until complete neurological recovery and brain, signs of increased ICP or damage to part of the
are only discharged, if a responsible adult can supervise brain will be present.
them at home for a further few days. l Other signs and symptoms of subdural haematoma can
l All patients with a GCS of 13 or less should receive a include any combination of the following:
CT scan of their head although many authorities would A history of recent head injury, loss of consciousness
advocate a CT scan on all whose GCS is not normal. or fluctuating levels of consciousness, irritability,
Section | II General Surgery 373

seizures, headache, dizziness, disorientation, amnesia, Treatment


lethargy
l Treatment of a subdural haematoma depends on its size
Nausea or vomiting, loss of appetite, personality
and rate of growth.
changes, inability to speak or slurred speech, ataxia,
l Some small subdural haematomas can be managed by
altered breathing patterns, hearing loss, blurred vision,
careful monitoring until it heals itself. Other small
abnormal movement of the eyes.
subdural haematomas can be managed by inserting a
temporary small catheter through a hole drilled through
Diagnosis the skull and sucking out the haematoma; this procedure
l On a CT scan, subdural haematomas are classically cres- can be done at the bedside.
cent-shaped, with a concave surface away from the skull. l Large or symptomatic haematomas require a craniotomy.

SHORT NOTES
Q. 1. Lockjaw. i. A black eye (periorbital haematoma) or ‘shiner’, is
bruising around the eye commonly due to an injury to
Ans.
the face.
i. Lockjaw or trismus refers to a pathological condition in ii. The black eye is caused by bleeding beneath the skin
which the mouth is held closed by sustained spasm of and around the eye. Sometimes a black eye indicates a
the masseter muscle. This condition is most often ob- more extensive injury, even a skull fracture, particularly
served in cases of tetanus. if the area around both eyes is bruised (raccoon eyes) or
ii. Other causes of lock jaw are as follows: if there has been a head injury.
l Pericoronitis, iii. The most common cause of a black eye is a blow to the
l Inflammation of muscles of mastication, eye or nose. Other causes include: Surgical procedures
l Peritonsillar abscess, to the face, such as a facelift, jaw surgery, or nose sur-
l Temporomandibular joint disorder (TMD). gery, basilar skull fracture.
iii. Lock jaw is also a common temporary side effect iv. Pain and swelling are the most common signs and
of many stimulants of the sympathetic nervous symptoms of a black eye. Swelling increases as
system, e.g. recreational drugs methylenedioxymeth- discolouration progresses.
amphetamine (MDMA), methamphetamine (meth), v. Initially, the swelling and discolouration may be mild.
methylphenidate (Ritalin), mephedrone (4-MMC), The eye often starts off slightly reddened, and then
amphetamine. progresses to a darker shade.
iv. Treatment: vi. The skin around the eye becomes deep violet, yellow,
l Treatment of the underlying condition: Dental green, or black in colour.
treatments, physical therapy, and passive range of
Over the course of a few days, the area becomes lighter
motion devices.
and the swelling decreases.
l Symptomatic therapy: With pain medications
(NSAIDs), muscle relaxants, and warm compresses Q. 4. Compound fracture.
may be used. Splints have been used.
Ans.
Q. 2. Aetiology of Jaw clicking. i. Compound fracture is defined as a fracture in which
Ans. broken bone fragments lacerate soft tissue and protrude
through an open wound in the skin, i.e. it involves
The following medical conditions are some of the possible wounds that communicate with the fracture, or where
causes of jaw clicking: fracture haematoma is exposed, and may thus expose
i. Normal jaw mechanics bone to contamination. Open injuries carry a higher risk
ii. Temporomandibular joint disorder (TMJ disorder) of infection.
iii. Jaw muscle disorders ii. The treatment of a compound fracture involves:
iv. Jaw joint disorders l Realignment of the fractured bones,
v. Jaw alignment disorders l Immobilization.
vi. Bruxism iii. Usually immediate surgery is required for compound
fractures as they are highly susceptible to infection due to
Q. 3. Black eye.
their exposure to bacteria and debris. Hence, early treat-
Ans. ment in case of a compound fracture is very important.
374 Quick Review Series for BDS 3rd Year

iv. Compound fractures take much longer to heal, due to l The femur can also be treated with traction by pulling
the amount of injury that the bone and the surrounding on the leg with a weight to align the broken bone.
tissue sustains, and also because of the higher rate of
Q. 7. Immediate complications of fracture.
complications caused by non-union of the joints and
infections. Ans.
Q. 5. Classify fractures of long bones. Immediate complications of fractures are usually caused
Ans. by the force of violence producing the fracture and are as
follows:
i. Fractures of the “long bones” are one of the most com- i. Injury to muscles, tendons, blood vessels, nerves and
mon injuries associated with a variety of accidents. organs close to fractured bone.
ii. The long bones fracture can be of following types: ii. Contamination in an open fracture may result in infection.
a. Fracture types of the diaphyseal segment: iii. Blood loss from the fracture site within or outside (in
open injuries) may produce shock.
iv. Gangrene of a part of the limb due to block in the blood
Simple (type A) Multi-fragmentary
supply.
Q. 8. Delayed and late complications of fracture.
Ans.
Wedge fractures(type B) Multi-fragmentary A few complications may set in after a few days or a few
weeks after fracture.
b. Fracture types of the proximal and distal The following are but only a few of the complications to
segments: be anticipated, diagnosed and treated in time:
l Infections
l Fat embolism
Extra-articular (type A) Articular l Development of contractures and stiffness, and deformities
l New bone formation in muscles, known as myositis
ossificans
l Compression in a closed compartment producing compart-
Partial articular (type B) Complete articular (type C) ment syndromes and subsequent ischaemic contractures.
l Delayed nerve palsies
c. Exceptions: l Delayed union, non-union and mal-union, including
The 3 exceptions are: cross-unions
The proximal humerus (A 5 extra-articular unifocal, l Shortening or lengthening on bones after union.
B 5 extra-articular bifocal, and C 5 articular), Q. 9. Head injury management.
The proximal femur (A 5 trochanteric area, a 5
neck, C 5 head), and the malleolar segment Ans.
(A 5 infrasyndesmotic, B 5 transsyndesmotic, and
i. Modern management of head injuries at the neurosurgi-
C 5 suprasyndesmotic).
cal unit involves continued ventilation, surgery, inten-
Q. 6. Write the symptoms and treatment of long bone sive care unit management of intracranial pressure and
fractures. cerebral perfusion pressure, oxygenation.
ii. The management of head-injured patients depends on
Ans.
the GCS (Glasgow Coma Scale).
i. The symptoms of long bone fracture are as follows: iii. Patients with a mild head injury (GCS 14-15) should be
l Pain admitted to a ward where thorough and frequent neuro-
l Unable to put weight on that limb, limited movement logical observations can be ensured.
l Shortening, limb at an odd angle iv. In case such a patient subsequently deteriorate neuro-
l Bruising and swelling logically, a CT scan of the patient’s head should be
ii. The long bones can be treated with: performed promptly, and the local neuro surgical unit
l A cast (on the outside of the skin), contacted.
l By attaching a device to the bone (plate or external v. Indications to operations in head-injured patient is
fixator) or based on a number of factors like premorbid state,
l By inserting a rod inside of the bone (intramedullary nail). the severity of initial injury, the onset and rapidity of
Section | II General Surgery 375

neurological deterioration and patient assessment on l Epidural bleeds can quickly expand and compress
arrival at the neurosurgical unit. the brainstem, causing unconsciousness, abnormal
posturing, and abnormal pupil responses to light.
Q. 10. Artificial respiration.
Ans. Diagnosis
i. Artificial respiration is defined as the act of simulating l On CT scans and MRIs, epidural haematomas usually
respiration, which provides for the overall exchange of appear convex in shape.
gases in the body by pulmonary ventilation, external l The lens like shape of the haematoma leads the appear-
respiration and internal respiration. ance of these bleeds to be called “lentiform”.
ii. Artificial respiration is a part of most protocols for
performing cardiopulmonary resuscitation (CPR). Treatment
iii. Pulmonary ventilation or external respiration is
achieved through manual insufflation of the lungs. This l The blood may be aspirated surgically to remove the
method is also known as expired air resuscitation mass and reduce the pressure it puts on the brain.
(EAR), expired air ventilation (EAV), mouth-to-mouth l The haematoma is evacuated through a burr hole or
resuscitation, rescue breathing or colloquially the kiss craniotomy.
of life. l If transfer to a facility with neurosurgery is prolonged
iv. Different types of artificial respiration include: trephination may be performed in the emergency
l Mouth to mouth: This involves the rescuer making a department.
seal between their mouth and the patient’s mouth Q. 12. Depressed fracture skull.
and ‘blowing’, to pass air into the patient’s body.
l Mouth to nose: In some instances, the rescuer may
Ans.
need or wish to form a seal with the patient’s nose. i. A depressed skull fracture is a type of fracture usually
Typical reasons for this include maxillofacial resulting from blunt force trauma, such as getting struck
injuries, performing the procedure in water or the with a hammer, rock or getting kicked in the head.
remains of vomit in the mouth. ii. These types of fractures, which occur in about 10% of
l Mouth to mouth and nose: Used on infants (usually severe head injuries, are comminuted fractures in which
up to around 1 year old), as this forms the most broken bones are displaced inward.
effective seal. iii. Depressed skull fractures carry a high risk of increased
l Mouth to mask: Most organisations recommend the pressure on the brain, crushing the delicate tissue.
use of some sort of barrier between rescuer and pa- iv. Compound depressed skull fractures occur when there is
tient to reduce cross-infection risk. One popular type a laceration over the fracture, resulting in the internal
is the ‘pocket mask’. This may be able to provide cranial cavity being in contact with the outside environ-
higher tidal volumes than a bag valve mask. ment increasing the risk of contamination and infection.
l Bag valve mask (BVM): This is a simple device v. Complex depressed fractures are those in which the
manually operated by the rescuer, which involves dura mater is torn.
squeezing a bag to expel air into the patient. vi. Treatment: Depressed skull fractures may require
l Mechanical resuscitator: An electric unit designed to surgery to lift the bones off the brain, if they are placing
breathe for the patient. pressure on it.
Q. 11. Extradural haematoma. Q. 13. Cardiopulmonary resuscitation.
Ans. Ans.
l Epidural or extradural haematoma is a type of traumatic i. Cardiopulmonary resuscitation (CPR) is an emergency
brain injury (TBI) in which a buildup of blood occurs procedure which is attempted in an effort to return life
between the dura mater and the skull. to a person in cardiac arrest.
l The condition is present in 1 to 3% of head injuries. ii. It is indicated in those who are unresponsive with no
Between 15 and 20% of patients with epidural haema- breathing or only gasps. It may be attempted both in
tomas die of the injury. and outside of a hospital.
l Epidural bleeds, like subdural and subarachnoid haem- iii. CPR involves chest compressions at a rate of at least
orrhages, are extra-axial bleeds, occurring outside of the 100 per minute in an effort to create artificial circula-
brain tissue, while intra-axial haemorrhages, including tion by manually pumping blood through the heart.
intraparenchymal and intraventricular haemorrhages, iv. Current recommendations place emphasis on high
occur within it. quality chest compressions over artificial respirations.
376 Quick Review Series for BDS 3rd Year

v. A universal compression to ventilation ratio of 30:2 is ii. The mandible can dislocate in the anterior, posterior,
recommended for adult and in children and infant, if lateral, or superior position. Anterior dislocations are
only a single rescuer is present. the most common.
iii. These dislocations are classified as acute, chronic
If at least 2 rescuers are present, a ratio of 15:2 is pre-
recurrent, or chronic.
ferred in children and infants. In newborns, a rate of 3:1 is
iv. Clinical features:
recommended unless a cardiac cause is known in which
l Usually pain and difficulty with jaw movement is
case a 15:2 ratio is reasonable.
present in all patients with mandible injury.
Q. 14. Mandibular dislocation. l Anterior mandible dislocations usually result in a

visible and palpable periauricular depression from


Ans.
displacement of the condyle.
i. Mandible dislocation is the displacement of the v. Treatment: Classic reduction technique, recumbent
mandibular condyle from the articular groove in the approach, wrist pivot method, ipsilateral approach.
temporal bone.

Topic 13

Anomalies of Development of Face (Cleft Lip


and Cleft Palate)

LONG ESSAYS
Q. 1. Classification of cleft lip and palate and their effect joined and it occurs due to the failure of fusion of the
upon teeth and eating. lateral palatine processes, the nasal septum, and/or the
median palatine processes (formation of the secondary
Ans.
palate).
Cleft lip (cheiloschisis) and cleft palate (palatoschisis) are l This cleft results in velopharyngeal inadequacy (air leak
variations of a type of clefting congenital deformity caused by into the nasal cavity from oral cavity leads to hypernasal
abnormal facial development during gestation. Approximately voice resonance and nasal emissions.)
1 in 700 children born have a cleft lip and/or a cleft palate. The
The typical distributions of cleft types are:
highest prevalence rates are reported for Native Americans and
l Cleft lips alone (15%),
Asians and Africans have the lowest prevalence rates.
l Cleft lips and palate (45%) and
l Isolated cleft palate (40%).
Cleft Lip Isolated cleft palate is more common in females while
l In cases when the cleft does not affect the palate struc- cleft lip and palate is predominant in males.
ture of the mouth, it is referred to as cleft lip.
Failure of merging between the medial nasal and maxil-
l
Aetiology
lary processes at 5 weeks’ gestation, on one or both
sides, results in formation of cleft lip. Aetiological factors responsible for cleft lip and cleft palate
l Cleft lip usually occurs at the junction between the cen- include both genetic and environmental components.
tral and lateral parts of the upper lip on either side. l Enviromental factors: Maternal epilepsy and antiepilep-
tic drugs (phenytoin, diazepams), steroids, maternal
smoking, rubella. Other teratogens, such as alcohol,
Cleft Palate anticonvulsants, and retinoic acid, are associated with
l In this condition, the two plates of the skull that form malformation patterns that include cleft lip and palate,
the hard palate (roof of the mouth) are not completely but have not been directly related to isolated clefts.
Section | II General Surgery 377

l Genetic factors: First degree relatives are 25 times more dental plates to help with the feeding process, though
likely to have deformities than normal population and their long-term benefits are still unproven.
combined CP and CL have a more predominant genetic
factor. Most common inclusive syndrome is Pierre-
Effects on the Teeth and Jaw
Robin sequence. Over 150 syndromes are associated
with CL/P, the most frequently encountered being Stick- l Due to the condition being based in an area so close to
ler, Van der Woude syndrome, Shprirntzen, Down, Apert the teeth and jaw, it is sometimes the case that they will
and Treacher Collins. also be affected.
l In some cases, there may be first teeth missing close to
where the cleft lip is situated or the teeth may come
Classification Systems for Cleft Lip and Cleft through but be crooked.
Palate l In other instances, there are too many teeth and this can
Various classifications of CL and CP exist. The most com- also lead to crookedness due to overcrowding. Dentist
monly used are as follows: and orthodontist have to be recommended in such situa-
l Kernahan’s striped Y or its modification (the most com- tions. Braces are used at the time of second teeth eruption
monly used classification): Kernohan has provided a to help force the teeth into a straighter position. This may
pictorial description, the striped Y classification for cleft require the removal of teeth, if there is overcrowding or
lip and palate. This system was modified by Millard to implants, if there are absent teeth.
include descriptions of the nasal tip and nasal floor. l The jaw can also be affected by a cleft lip as it may
l LAHSHAL: Complete clefts of lips, alveolus, hard and cause the growth of the upper jaw to fall behind the
soft palate are designated by capital letters, whereas lower jaw. If this is the case, then the assistance of a
small letters designate incomplete clefts. Microform medical expert in orthodontic surgery can help put the
clefts are designated by asterisk. condition right.
l Syndromic versus non-syndromic. Q. 2. Describe the congenital anomalies of maxillofacial
l Complete versus incomplete. region and describe the management of cleft palate.
l Primary versus secondary palate.
l Classification of the alveolar segment. Ans.
Congenital anomalies of the maxilla-facial region are:
Classification of Clefts l Cleft lip
l Cleft palate
i. Cleft lips: Unilateral/bilateral, incomplete/complete, l Developmental anomalies of teeth and jaws: Anodontia,
ii. Cleft lips and alveolus hyperdontia (supernumerary teeth), structural anomalies
iii. Cleft lip and palate of teeth.
iv. Cleft hard palate
v. Cleft soft palate Of these, from surgical point of view, cleft lip and cleft
vi. Submucous cleft palate are important. Cleft lip (cheiloschisis) and cleft palate
vii. Bifid uvula: Incomplete form of cleft palate. (palatoschisis) are variations of a type of clefting congenital
deformity caused by abnormal facial development during
gestation.
Effects on Feeding
Cleft lip is a type of maxilla facial anomaly formed due to the
l Although there may be some affect on the ability of a
failure of merging between the medial nasal and maxillary
baby with cleft lip and palate to feed, they are still usu-
processes at 5 weeks’ gestation, on one or both sides. Cleft lip
ally able to do so.
usually occurs at the junction between the central and lateral
l However, there are instances in which the child may
parts of the upper lip on either side. Cleft lip is formed in the
have difficulty suckling on the nipple to get enough
top of the lip as either a small gap or an indentation in the lip
milk, be taking in too much air while feeding or may
(partial or incomplete cleft) or it continues into the nose (com-
even have milk coming up through their nose.
plete cleft). Cleft lip can unilateral or bilateral.
l If this is the case, then feeding the baby in different
positions and angles or gently pressing the baby against Cleft palate is a congenital anomaly where the two plates
nipple to help them latch on properly is recommended. of the skull that form the hard palate (roof of the mouth) are
l If the child is unable to latch on properly, other options not completely joined and it occurs due to the failure of
are available. Special teats and soft bottles, specifically fusion of the lateral palatine processes, the nasal septum,
modified for cleft lips and cleft palate, are available to and/or the median palatine processes (formation of the sec-
help with feeding. There is also the possibility of using ondary palate).
378 Quick Review Series for BDS 3rd Year

Aetiology l The goal of repair in patients with cleft palate is to


separate the oral and nasal cavities; this separation in-
Aetiological factors responsible for cleft lip and cleft palate
volves the formation of a valve that is both watertight
include both genetic and environmental components.
and airtight. The valve is necessary for normal speech.
a. Enviromental factors: l The repair also helps with the preservation of facial
growth and the development of proper dentition.
l Maternal epilepsy and antiepileptic drugs (phenytoin,
l The basic surgical techniques for soft palate repair are:
diazepams),
Double reverse z-plasty (Furlow) palatoplasties, von
l Steroids,
Langenbeck, Schweckendiek, 2-flap, 3-flap (V-to-Y).
l Maternal smoking,
l Two methods of repairing a cleft palate are widely
l Rubella.
practiced:
l Other teratogens, such as alcohol, anticonvulsants, and
a. One is the double-opposing Z-plasty, which involves
retinoic acid, are associated with malformation patterns
realignment of the levator muscles and lengthening
that include cleft lip and palate, but have not been di-
of the palate and helps with closure of the palate.
rectly related to isolated clefts.
b. The other is a straight-line repair with direct dissec-
b. Genetic factors: tion of levator musculature and reorientation of that
musculature in a similar coronal fashion.
l First degree relatives are 25 times more likely to have
l Hard palate cleft closure is done using a flap of nasal
deformities than normal population and
mucosa from vomer, with realignment of muscles in
l Combined CP and CL have a more predominant genetic
their correct anatomical position.
factor.
l Most common inclusive syndrome is Pierre-Robin
sequence. Follow-up and Secondary Management
l Over 150 syndromes are associated with CL/P, the most
l Speech therapy
frequently encountered being Stickler, Van der Woude
l Orthodontic management: Alveolar grafts, orthognathic
syndrome, Shprirntzen, Down, Apert and Treacher Collins.
surgery, septorhinoplasty
l Monitor facial growth
Clinical Features l Audiological assessment
A child with cleft lip or palate may have one or more of Q. 3. Describe the Aetiology, pathophysiology, clinical
these conditions at birth: features and management of cleft lip and palate.
l The most prominent symptom is the presence of a fis-
Ans.
sure or cleft lip/cleft palate.
l A cleft lip may be just a small notch in the lip. It may Cleft lip (cheiloschisis) and cleft palate (palatoschisis) are
also be a complete split in the lip that goes all the way variations of a type of clefting congenital deformity caused
to the base of the nose. by abnormal facial development during gestation.
l A cleft palate can be on one or both sides of the roof of
the mouth. It may go the full length of the palate.
Cleft Lip
l Misaligned teeth
l Change in nose shape (amount of distortion varies)

Problems that may be present because of a cleft lip or FIGURE 13.1 Unilateral complete
palate are:
l Failure to gain weight
l Feeding problems
l Flow of milk through nasal passages during feeding
l Misaligned teeth
l Poor growth FIGURE 13.2 Unilateral incomplete
l Recurrent ear infections
l Speech difficulties

The goal of cleft palate treatment is:


l Repair of cleft palate is usually done before months
(6-12 months). FIGURE 13.3 Bilateral complete
Section | II General Surgery 379

Aetiology
Aetiological factors responsible for cleft lip and cleft palate
FIGURE 13.4 Bilateral incomplete include both genetic and environmental components.
l Enviromental factors: Maternal epilepsy and anti-
epileptic drugs (phenytoin, diazepams), steroids,
l In cases where the cleft does not affect the palate struc- maternal smoking, rubella. Other teratogens, such
ture of the mouth, it is referred to as cleft lip. as alcohol, anticonvulsants, and retinoic acid, are
l Cleft lip usually occurs at the junction between the cen- associated with malformation patterns that include
tral and lateral parts of the upper lip on either side. cleft lip and palate, but have not been directly re-
l Cleft lip is formed in the top of the lip as either a small gap lated to isolated clefts.
or an indentation in the lip Figs 13.2 and 13.4 (partial or l Genetic factors: First degree relatives are 25 times more
incomplete cleft) or it continues into the nose Figs 13.1 likely to have deformities than normal population and
and 13.3 (complete cleft). Cleft lip can unilateral or bilateral. combined CP and CL have a more predominant genetic
factor. Most common inclusive syndrome is Pierre-
Robin sequence. Over 150 syndromes are associated
Cleft Palate with CL/P, the most frequently encountered being Stick-
l In this condition, the two plates of the skull that form ler, Shprirntzen, Van der Woude syndrome, Down, Ap-
the hard palate (roof of the mouth) are not completely ert and Treacher Collins.
joined and it occurs due to the failure of fusion of the
lateral palatine processes, the nasal septum, and/or the
median palatine processes (formation of the secondary
Pathophysiology
palate). l Facial tissues, including lip and palate, are produced
l Palate cleft can occur as complete (soft and hard palates, from mesenchymal migration, penetration, and fusion
possibly including a gap in the jaw) or incomplete (a of cranioneural crest cells.
‘hole’ in the Figs 13.5 and 13.6 roof of the mouth, usu- l Each of the 3 main facial prominences (nose, lips, palate)
ally as a cleft soft palate). is derived in embryonic period from bilateral converging
l This cleft in the roof of the mouth connects the oral cav- facial processes.
ity directly to the nasal cavity which results in velopha- l Formation of the primary palate begins at approxi-
ryngeal inadequacy (air leak into the nasal cavity from mately 35 days of gestation. Complete lip development
oral cavity leads to hypernasal voice resonance and nasal generally occurs by the sixth week of gestation, fol-
emissions.) lowed by palatal fusion.
l Three paired anatomic processes are involved with this
phenomenon of migration and penetration: (i) the medial
nasal process coalesces with the (ii) maxillary process,
followed by the coalescence of the (iii) lateral nasal pro-
cess with the medial nasal process.
l Failure of merging between the medial nasal and maxil-
lary processes at 5 weeks’ gestation, on one or both
FIGURE 13.5 Postincisive foramen cleft
sides, results in formation of cleft lip.
l Failure of fusion of the palatal shelves, due to any reason
lead to cleft palate.
l Possible reasons for palatal clefting involve abnormal
growth of the palatal shelves, cell death (post-fusion),
and failure of mesenchymal consolidation and differ-
entiation.

FIGURE 13.6 Transincisive cleft Clinical Features


A child with cleft lips or palate may have one or more of
The typical distributions of cleft types are: Cleft lips alone these conditions at birth.
(15%), cleft lips and palate (45%) and isolated cleft palate l The most prominent symptom is the presence of a fis-
(40%). Isolated cleft palate is more common in females sure or cleft lip/cleft palate. A cleft lip may be just a
while cleft lip and palate is predominant in males. small notch in the lip.
380 Quick Review Series for BDS 3rd Year

l It may also be a complete split in the lip that goes all the The realigning of the muscles of lip in their correct
way to the base of the nose. A cleft palate can be on one anatomical position is an important and essential com-
or both sides of the roof of the mouth. ponent of this repair.
l It may go the full length of the palate. l Millard “rotation–advancement flap” repair: The medial
l Misaligned teeth lip element is rotated inferiorly and the lateral lip ele-
l Change in nose shape (amount of distortion varies) ment is advanced into the resulting upper lip defect. The
columellar flap is then used to create the nasal sill.
Problems that may be present because of a cleft lip or
l Hagedorn-LeMesurier repair: The medial lip element is
palate are:
lengthened by introducing a quadrilateral flap devel-
l Failure to gain weight
oped from the lateral lip element.
l Feeding problems
l Tennison-Randall repair: The medial lip element is
l Flow of milk through nasal passages during feeding
lengthened by introducing a triangular flap from the
l Misaligned teeth
inferior portion of the lateral lip element.
l Poor growth
l Recurrent ear infections
l Speech difficulties Cleft Palate Treatment
l Repair of cleft palate is usually done before months
Management of Cleft Lips and Cleft Palate (6-12 months).
l The goal of repair in patients with cleft palate is to
l The management depends on the severity and type of separate the oral and nasal cavities; this separation in-
cleft lip and cleft palate. volves the formation of a valve that is both watertight
l A multidisciplinary team approach including plastic and airtight. The valve is necessary for normal speech.
surgeon, otolaryngologist, paediatric dentist, orthodon- The repair also helps with the preservation of facial
tist, oral surgeon, speech therapist, audiologist, nutri- growth and the development of proper dentition.
tionist, paediatrician, psychologist, and social worker l The basic surgical techniques for soft palate repair are:
are recommended. Double reverse z-plasty (Furlow) palatoplasties, von
Langenbeck, Schweckendiek, 2-flap, 3-flap (V-to-Y).
Two methods of repairing a cleft palate are widely
Cleft Lip Treatment l

practiced:
l Principles of cleft lip repair include layered repair of the a. One is the double-opposing Z-plasty, which involves
skin, muscle, and mucous membrane to restore sym- realignment of the levator muscles and lengthening
metric length and function. of the palate and helps with closure of the palate.
l Cleft lip repair is usually done at 3 months after birth. b. The other is a straight-line repair with direct dissec-
While surgery to repair a cleft lip can be performed tion of levator musculature and reorientation of that
soon after birth, the often preferred age is at approxi- musculature in a similar coronal fashion.
mately 10 weeks of age, following the “rule of 10s” l Hard palate cleft closure is done using a flap of nasal
coined by surgeons Wilhelmmesen and Musgrave mucosa from vomer, with realignment of muscles in
(the child is at least 10 weeks of age; weighs at least their correct anatomical position.
10 pounds, and has at least 10 g haemoglobin).
If the cleft is bilateral and extensive, two surgeries may
Follow-up and Secondary Management
l

be required to close the cleft, one side first, and the


second side a few weeks later. l Speech therapy
l A wide range of surgical techniques are used for repair l Orthodontic management: Alveolar grafts, orthognathic
of cleft lips but the most widely used technique is surgery, septorhinoplasty
“rotation–advancement flap” technique of Millard. l Monitor facial growth
This technique corrects both lip and nasal deformity. l Audiological assessment

SHORT ESSAYS
Q. 1. How do you classify cleft lip and principles of Cleft lip, a congenital anomaly of maxillo-facial region, is
management? formed due to the failure of merging between the medial
nasal and maxillary processes at 5 weeks’ gestation, on one
Ans. or both sides.
Section | II General Surgery 381

Classification of Cleft Lip Follow-up and Secondary Management


Cleft lip can be classified as follows: l Speech therapy
i. Unilateral cleft lip l Orthodontic management: Alveolar grafts, orthognathic
ii. Bilateral cleft lip surgery, septorhinoplasty
iii. Incomplete cleft lip (Figs 13.2 and 13.4): Formed in the top l Monitor facial growth
of the lip as either a small gap or an indentation in the lip. l Audiological assessment
iv. Complete cleft lip (Figs 13.1 and 13.3): Cleft continues
Q. 2. Describe the pathogenesis, types and treatment of
into the nose.
cleft palate.
v. Combination of cleft lip and palate.
The typical distributions of cleft types are: Ans.
Cleft lips alone (15%), l Cleft palate is a congenital condition, when the two
Cleft lips and palate (45%). plates of the skull that form the hard palate (roof of the
mouth) are not completely joined and it occurs due to
Management of Cleft Lips the failure of fusion of the lateral palatine processes, the
nasal septum, and/or the median palatine processes (for-
l The management depends on the severity and type of
cleft lip. mation of the secondary palate).
l Palate cleft can occur as complete (soft and hard palate,
l A multidisciplinary team approach including plastic sur-
possibly including a gap in the jaw) or incomplete (a ‘hole’
geon, otolaryngologist, paediatric dentist, orthodontist, oral
in the roof of the mouth, usually as a cleft soft palate).
surgeon, speech therapist, audiologist, nutritionist, paedia-
trician, psychologist, and social worker are recommended.
Pathogenesis
Cleft Lip Treatment l Facial tissues, including lip and palate, are produced
l Principles of cleft lip repair include layered repair of the from mesenchymal migration, penetration, and fusion
skin, muscle, and mucous membrane to restore sym- of cranioneural crest cells.
metric length and function. l Each of the 3 main facial prominences (nose, lips, palate)
l Cleft lip repair is usually done at 3 months after birth. is derived in embryonic period from bilateral converging
While surgery to repair a cleft lip can be performed facial processes.
soon after birth, the often preferred age is at approxi- l Formation of the primary palate begins at approxi-
mately 10 weeks of age, following the “rule of 10s” mately 35 days of gestation. Complete lip development
coined by surgeons Wilhelmmesen and Musgrave (the generally occurs by the sixth week of gestation, fol-
child is at least 10 weeks of age; weighs at least 10 lowed by palatal fusion.
pounds, and has at least 10 g haemoglobin). l Three paired anatomic processes are involved with this
l If the cleft is bilateral and extensive, two surgeries may phenomenon of migration and penetration: The medial
be required to close the cleft, one side first, and the nasal process coalesces with the maxillary process, fol-
second side a few weeks later. lowed by the coalescence of the lateral nasal process
l A wide range of surgical techniques are used for repair with the medial nasal process.
of cleft lips but the most widely used technique is l Failure of fusion of the palatal shelves, due to any reason
“rotation–advancement flap” technique of Millard. leads to cleft palate. Possible reasons for palatal clefting
l This technique corrects both lip and nasal deformity. involve abnormal growth of the palatal shelves, cell
The realigning of the muscles of lip in their correct ana- death (post-fusion), and failure of mesenchymal consoli-
tomical position is an important and essential compo- dation and differentiation.
nent of this repair.
Millard “rotation–advancement flap” repair: The medial
Types of Cleft Palate
l

lip element is rotated inferiorly and the lateral lip ele-


ment is advanced into the resulting upper lip defect. The l Cleft of soft palate
columellar flap is then used to create the nasal sill. l Cleft of hard palate
l Hagedorn-LeMesurier repair: The medial lip element is l Cleft of hard and soft palate (complete cleft palate)
lengthened by introducing a quadrilateral flap developed
from the lateral lip element.
Management Protocal of Cleft Palate
l Tennison-Randall repair: The medial lip element is
lengthened by introducing a triangular flap from the The most common treatment protocol presently used in
inferior portion of the lateral lip element. most cleft treatment:
382 Quick Review Series for BDS 3rd Year

l Newborn: Diagnostic examination, general counselling l Age 6 months: Presurgical orthodontics, if necessary;
of parents, feeding instructions, palatal obturator (if first speech evaluation
necessary); genetic evaluation and specification of diag- l Age 9 months: Speech therapy begins
nosis; empiric risk of recurrence of cleft calculated; l Age 9-12 months: Repair of cleft palate (placement of
recommendation of a protocol for the prevention of a ventilation tubes, if not done at the time of cleft lip repair)
cleft recurrence in the family. l Age 1-7 years: Orthodontic treatment
l Age 3 months: Repair of cleft lip (and placement of l Age 7-9 years: Alveolar bone graft
ventilation tubes) l Older than 8 years: Orthodontic treatment continues.

SHORT NOTES
Q. 1. Write a short note on cleft lip. Q. 2. Write a short note on cleft palate.
Ans. Ans.
l Cleft lip is a type of maxillofacial anomaly formed due to l Cleft palate is a congenital anomaly where the two
the failure of merging between the medial nasal and maxil- plates of the skull that form the hard palate are not com-
lary processes at 5 weeks’ gestation, on one or both sides. pletely joined and it occurs due to the failure of fusion
l Cleft lip usually occurs at the junction between the cen- of the lateral palatine processes, the nasal septum, and/
tral and lateral parts of the upper lip on either side. or the median palatine processes.
l Cleft lip is formed in the top of the lip as either a small l This cleft results in velopharyngeal inadequacy.
gap or an indentation in the lip (partial or incomplete
cleft) or it continues into the nose (complete cleft). Cleft
Treatment
lip can unilateral or bilateral.
l Repair of cleft palate is usually done before months
(6-12 months).
Treatment l The goal of repair in patients with cleft palate is to
l A wide range of surgical techniques are used for repair of separate the oral and nasal cavities.
cleft lips but the most widely used technique is “rotation– l The repair also helps with the preservation of facial
advancement flap” technique of Millard. This technique growth and the development of proper dentition.
corrects both lip and nasal deformity. l The basic surgical techniques for soft palate repair
l Hagedorn-LeMesurier repair: The medial lip element is are: Double reverse z-plasty (Furlow) palatoplasties,
lengthened by introducing a quadrilateral flap devel- von Langenbeck, Schweckendiek, 2-flap, 3-flap
oped from the lateral lip element. (V-to-Y).
l Tennison-Randall repair: The medial lip element is l Hard palate cleft closure is done using a flap of nasal
lengthened by introducing a triangular flap from the mucosa from vomer, with realignment of muscles in
inferior portion of the lateral lip element. their correct anatomical position.

Topic 14

Thyroid and Parathyroid Glands

LONG ESSAYS
Q. 1. Describe clinical features, diagnosis and manage- Thyroid gland carcinomas are classified into 4 types:
ment of carcinoma of thyroid. i. Papillary carcinoma (75-85%)
Ans. ii. Follicular carcinoma (10-20%)
Section | II General Surgery 383

iii. Medullary carcinoma (5%) l Characterized by rapid and infiltrative growth and
iv. Anaplastic carcinoma (,5%) spread.
l Worst prognosis with very low cure rate.
Papillary, follicular and anaplastic carcinomas of thy-
l Local infiltrative growth and spread is common and
roid gland arise from follicular epithelium whereas med-
metastasis is very common.
ullary carcinoma develops from C-parafollicular cells.

Pathogenesis and Aetiology


Characteristics of Papillary Carcinoma of
Thyroid There are several factors, genetic and environmental, impli-
cated in the pathogenesis of thyroid cancers.
l 30-50 years is peak onset age for papillary carcinoma l Genetic factors: They include two important possibilities:
with female: male ratio of 3:1. i. Mutated proto-oncogenes, resulting in change in
l Associated with thyroglossal cyst, thyroid cyst and protein production and accelerated growth, and
radiation exposure to neck. ii. Secondly alterations in growth suppression genes,
l Cure rate is very high. which after initiation result in unregulated cell
l Microscopically characterized by presence of Orphan growth. Oncogenes ras, c-myc, c-fos and PTC/RET
annie eye nucleus and Psammoma bodies. proto-oncogene are mostly responsible for this gene
l They are not encapsulated. Their spread is lymphatic. mutation.
l Predisposing factors include: Irradiation in childhood, l Environmental factors:
long-standing thyroglossal cyst, thyroid cyst. i. Exposure to ionizing radiation, particularly during
the first two decades of life.
Characteristics of follicular carcinoma of thyroid
ii. Long-standing multinodular goitre as in iodine defi-
l The peak onset age for follicular carcinoma of thyroid ciency-related endemic goitre.
is 40-60 yrs with female: male ratio of 3:1. iii. Though most thyroid lymphomas arise from pre-
l Encapsulated but invasion into vascular structures (veins existing Hashimoto thyroiditis, conclusive evidence
and arteries) within the thyroid gland is common. suggesting association of autoimmune thyroiditis
l Most common metastasis is to bones (pulsating bony with thyroid carcinoma is not present.
metastasis) and spread is via lymphatic mostly.
l Also has a overall high cure rate.
l Long-standing goitre is associated with this type of car- Clinical Features
cinoma. Symptoms of thyroid carcinoma is usually not present in
l Hurtle cell carcinoma is a variant of follicular carci- early stage of the disease and mostly manifest itself in later
noma with worse prognosis and characterized by pres- stages.
ence of hurtle cell. As the carcinoma increases in size, it can present itself
with following clinical features:
Characteristics of medullary carcinoma of thyroid l A lump or nodule in the neck— especially in the front
l Presence of amyloid stroma and not TSH dependent. of the neck, in the area of the Adam’s apple. This is the
l Occurs in 4 clinical conditions sporadic, MEN II-A, most common presentation of thyroid cancer as an
MEN II-B and inherited medullary carcinoma without asymptomatic thyroid mass or a nodule that can be felt
associated endocrine pathologies. in the neck.
l It is more common in females than males (except for l Enlargement of the neck.
inherited cancers). l Enlarged lymph nodes in the neck.
l Regional metastases, i.e. spread to neck lymph nodes l Hoarseness, difficulty in speaking normally, voice
occurs early in the disease. changes.
l Usually originates in the upper central lobe of the l Difficulty in swallowing, or a choking feeling or diffi-
thyroid from C-parafollicular cell. culty in breathing.
l Poor prognostic factors include age .50, male, distant l Pain in the neck or throat, including pain from the neck
spread (metastases), and when seen in patients with to the ears.
other endocrine tumours due to MEN II-B syndrome. l Sensitivity in the neck— discomfort with neckties, tur-
tlenecks, scarves, necklaces.
Characteristics of anaplastic carcinoma of thyroid l Persistent or chronic cough not due to allergies or
l Peak onset age for anaplastic carcinoma of thyroid is illness.
65 yrs and more. l Asymmetry in the thyroid (big nodule on one side, noth-
l The female: male ratio is 1:2, i.e. more in males. ing on the other).
384 Quick Review Series for BDS 3rd Year

l Nodules that when manipulated give the impression that i. Papillary carcinoma:
the entire thyroid is moving. l For lesions smaller than 1 cm, lobectomy with isthmec-
l Nodules that cause the wind pipe to go to one side of the tomy is appropriate.
neck, as well as cause superior vena cava syndrome. l In all others cases, total thyroidectomy is the treatment
l Some particularly aggressive thyroid tumours can go to of choice.
the brain and cause neurological symptoms. l Lymph node metastases are treated with modified radi-
cal neck dissection.
Diagnosis ii. Follicular carcinoma:
Diagnosis of thyroid cancer typically involves a number of l For follicular carcinoma diagnosed by FNAC lobec-
procedures and tests. tomy, including the isthmus and pyramidal lobe is rec-
i. Physical examination: Palpation of thyroid gland and ommended.
lymph nodes to feel for enlargement and lumps, as well l Intraoperative frozen sections should be performed
as the gland’s size and firmness. when there is evidence of capsular or vascular invasion.
ii. Biopsy: Biopsy can be performed by “fine needle aspi- l Total thyroidectomy is indicated for carcinoma, except
ration biopsy”(FNA) or a surgical biopsy, where the in patients with minimally invasive follicular cancers.
nodule, or the thyroid gland itself, is removed surgi-
cally can be used. iii. Hurtle cell carcinoma:
iii. Imaging tests: A variety of imaging scans are used to l Total thyroidectomy with central node dissection is
evaluate thyroid nodules for possible thyroid cancer. preferred.
These include: l If palpable lymph nodes are present, modified radical
a. Nuclear scan, also known as radioactive iodine up- neck dissection should be performed and the patient
take (RAI-U) scan: Nodules that absorb more radio- treated with T4 postoperatively.
active iodine, and, therefore, are more visible, are
known as “hot nodules” and are more likely to be iv. Medullary carcinoma:
benign. l Total thyroidectomy is the treatment of choice because
b. CT scan: It is used to evaluate the thyroid. A CT of the high incidence of multicentricity and more ag-
scan cannot detect smaller nodules, but may help gressive course.
to detect and diagnose a goitre, or larger thyroid l Dissection of cervical lymph nodes is done, if nodes are
nodules. involved or for tumours larger than 2 cm.
c. Magnetic resonance imaging (MRI): It can help to
detect enlargement in the thyroid, as well as v. Anaplastic carcinoma:
tumours, tumour size, and may be able to detect l Treatment is not effective.
tumour spread. l Combination of radiation therapy with chemotherapy
d. Thyroid ultrasound: It can tell whether a nodule is a and debulking provides a bit of relief for the patient.
fluid-filled cyst, or a mass of solid tissue, but it can- l The constricting tumour tissue should be taken out to
not determine, if a nodule or lump is malignant. relieve the tracheal obstruction. Tracheostomy may be
iv. Blood tests: Thyroid stimulating hormone (TSH) blood necessary, if there is airway obstruction.
tests is used to evaluate the thyroid’s activity in papil- l Post-total thyroidectomy all patients of papillary and
lary and follicular carcinoma as these carcinomas are follicular carcinomas are treated with L-thyroxine.
TSH-dependent.When medullary thyroid cancer is sus-
pected, test for raised serum calcitonin or CEA level Q. 2. Describe clinical features, diagnosis and manage-
can be an indicator. Genetic testing to identify the ab- ment of primary thyrotoxicosis.
normal gene associated with some cases of medullary Ans.
thyroid cancer can also be done.
Clinical feature of primary thyrotoxicosis/Graves diease are
as follows:
Treatment
l Primary treatment for carcinoma of thyroid is surgical,
Symptoms
except in the case of anaplastic carcinoma when all l Excitability, hyperactivity, irritability and dysphoria
types of treatment are not able to avoid the worst prog- l Heat intolerance and swelling
nosis in most cases. l Hyperreflexia, muscle wasting, and proximal myopathy
l The modalities of treatment in different types of thyroid without fasciculation
gland carcinoma are as follows: l Profuse sweating
Section | II General Surgery 385

l Fatigue and weakness l These drugs block the binding of iodine and coupling of
l Weight loss with increased appetite iodotyrosines. The most dangerous side-effect is agran-
l Diarrhoea and polyuria. ulocytosis. Other potential side effects include granulo-
l Oligomenorrhoea and loss of libido. cytopenia and aplastic anaemia. The most common side
l Goitre/enlarged thyroid gland effects are rash and peripheral neuritis.
l If treatment with antithyroid drugs fails to induce remis-
Signs sion, RAI or surgery must be considered.
l Tachycardia
l Hot moist palms C. Radioiodine
l Systolic hypertension is present. l Radioiodine (radioactive iodine-131, abbreviated as
l Presence of fine tremors, i.e. involuntary movement of RAI) is suitable for most patients, although some prefer
body parts is present. to use it mainly for older patients.
l Cardiac arrhythmias, i.e. atrial fibrillation and atrial l Indications for RAI include failed medical therapy or
tachycardia develops. surgery, or when medical or surgical therapies are con-
l Diaphoresis is present, i.e. excessive sweating is present. traindicated.
l There is presence of powerful wide pulse pressure and l Contraindications to RAI are pregnancy (absolute),
good bounding pulse is present. ophthalmopathy (relative; it can aggravate thyroid eye
l Exaggerated deep tendon reflexes are seen. disease), and solitary thyroid nodules.
l Ophthalmopathy l The radio-iodine treatment acts slowly (over months to
l Pretibial myxoedema: Thickiening of skin due to mucin years) to partially or completely destroy the thyroid
deposition on tibia. gland.
l Patients must, therefore, be monitored regularly with
Ophthalmopathy and pretibial myxoedema are characteris- thyroid blood tests to ensure that they don’t evolve to
tics of primary thyrotoxicosis. hypothyroidism.
Eye signs include: D. Surgery
l Exophthalmous with staring look.
l This modality is suitable for young patients and preg-
l Lid lag/ Von Graffe sign: Lagging of upper eyelids.
nant patients.
l Lid retraction
l Indications are: A large goitre, suspicious nodules or
l Mobieus sign: Absence of convergence of eyeballs.
suspected cancer and patients with ophthalmopathy.
l Joffroy’s sign: Absence of wrinkling of forehead when
l Preoperative administration of Lugol’s iodine solution
patient is asked to look upward.
decreases intraoperative blood loss during thyroidec-
l Stellwag’s sign: Staring look of patient.
tomy in patients with Graves’ disease.
l Ophthalmoplegia: Paralysis of ocular muscles.
l Choice can be made between partial or total removal of
the thyroid gland (subtotal thyroidectomy vs total thy-
Diagnosis roidectomy). A total removal excludes the difficulty in
determining how much thyroid tissue must be removed.
l TFT: Elevation of T3, T4 and decrease in TSH levels.
l Thyroid scan: Radioisotope scan using 131 Iodine or 99Tc. E. Thyroid hormones
l Ultrasound: For search of cyst or STN.
l Many Graves’ disease patients will become lifelong
thyroid patients, due to the surgical removal or radioac-
Treatment tive destruction of their thyroid.
l In effect, they are then hypothyroid patients, requiring
A. Symptomatic perpetual intake of artificial thyroid hormones.
l Beta blockers (e.g. propranolol) can be used to inhibit l Given the one-week plasma half life of levothyroxine (T4),
the sympathetic nervous system symptoms like tachy- it takes about five-six weeks (half-lives) before a steady
cardia until antithyroid treatments start to take effect. state is attained after the dosage is initiated or changed.
l After the optimal thyroxine dose has been defined, long-
B. Antithyroid drugs term monitoring of patients with an annual clinical
l Antithyroid medications is recommended to be given evaluation and serum TSH measurement is appropriate.
for six months to two years. Upon cessation of the
Q. 3. Describe the signs, symptoms and treatment of
drugs, the hyperthyroid state may recur.
common primary malignant lesions in thyroid.
l The main antithyroid drugs are carbimazole, methima-
zole and propylthiouracil/PTU. Ans.
386 Quick Review Series for BDS 3rd Year

The most common primary malignant lesions in thyroid l Asymmetry in the thyroid (big nodule on one side, noth-
glands are: ing on the other).
i. Papillary carcinoma (75-85%) l Nodules that when manipulated give the impression that
ii. Follicular carcinoma (10-20%) the entire thyroid is moving, this is often a sign of an
aggressive cancer.
Characteristics of papillary carcinoma of thyroid:
l Nodules that cause the wind pipe to go to one side of the
l 30-50 years is peak onset age for papillary carcinoma
neck, as well as cause superior vena cava syndrome
with female: male ratio of 3:1.
Some particularly aggressive thyroid tumours can go to
l Associated with thyroglossal cyst, thyroid cyst and ra-
the brain and cause neurological symptoms.
diation exposure to neck.
l Cure rate is very high.
l Microscopically characterized by presence of Orphan Diagnosis
annie eye nucleus and Psammoma bodies.
Diagnosis of thyroid cancer typically involves a number of
l They are not encapsulated. Their spread is lymphatic.
procedures and tests.
l Predisposing factors include: irradiation in childhood,
i. Physical examination: Palpation of thyroid gland and
long-standing thyroglossal cyst, thyroid cyst.
lymph nodes to feel for enlargement and lumps, as well
l They have the best prognosis.
as the gland’s size and firmness.
Characteristics of follicular carcinoma of thyroid: ii. Biopsy: Biopsy can be performed by “fine needle aspi-
l The peak onset age for follicular carcinoma of thyroid ration biopsy”(FNA) or a surgical biopsy, where the
is 40-60 yrs with female: male ratio of 3:1. nodule, or the thyroid gland itself, is removed surgi-
l Also has a overall high cure rate. cally can be used.
l Encapsulated but invasion into vascular structures (veins iii. Imaging tests: A variety of imaging scans are used to
and arteries) within the thyroid gland is common. Most evaluate thyroid nodules for possible thyroid cancer.
common metastasis is to bones (pulsating bony metas- These include:
tasis) and spread is via lymphatic mostly. a. Nuclear scan: It is also known as radioactive iodine
l Long-standing goitre is associated with this type of car- uptake (RAI-U) scan. Nodules that absorb more ra-
cinoma. dioactive iodine, and therefore, are more visible, are
l Hurtle cell carcinoma is a variant of follicular carci- known as “hot nodules” and are more likely to be
noma with worse prognosis and characterized by pres- benign.
ence of hurtle cell. b. CT scan: It is used to evaluate the thyroid. A CT scan
l The prognosis is poor. cannot detect smaller nodules, but may help to detect
and diagnose a goitre, or larger thyroid nodules.
c. Magnetic resonance imaging (MRI): It can help to
Clinical Features detect enlargement in the thyroid, as well as tumours,
Symptoms of thyroid carcinoma is usually not present in tumour size, and may be able to detect tumour spread.
early stage of the disease and mostly manifest itself in later d. Thyroid ultrasound: It can tell whether a nodule is a
stages. As the carcinoma increases in size, it can present fluid-filled cyst, or a mass of solid tissue, but it can-
itself with following clinical features: not determine, if a nodule or lump is malignant.
l A lump or nodule in the neck — especially in the front iv. Blood tests:
of the neck, in the area of the Adam’s apple. This is the l Thyroid-stimulating hormone (TSH) blood tests is

most common presentation of thyroid cancer as an as- used to evaluate the thyroid’s activity in papillary
ymptomatic thyroid mass or a nodule that can be felt in and follicular carcinoma as these carcinomas are
the neck. TSH-dependent.
l Enlargement of the neck and enlarged lymph nodes in l When medullary thyroid cancer is suspected, test for

the neck. raised serum calcitonin or CEA level can be an indi-


l Hoarseness, difficulty in speaking normally, voice cator.
changes, difficulty in swallowing, or a choking feeling l Genetic testing to identify the abnormal gene associ-

and difficulty in breathing. ated with some cases of medullary thyroid cancer
l Pain in the neck or throat, including pain from the neck can also be done.
to the ears.
Sensitivity in the neck — discomfort with neckties,
Treatment
l

turtlenecks, scarves, necklaces.


l Persistent or chronic cough not due to allergies or Primary treatment for carcinoma of thyroid is surgical,
illness. except in the case of anaplastic carcinoma when all types of
Section | II General Surgery 387

treatment are not able to avoid the worst prognosis in most l After initiation of the process sensitized T-helper
cases. lymphocytes stimulates B lymphocytes to produce
The modalities of treatment in different types of thyroid antibodies against the thyroid hormone receptor. TSIs or
gland carcinoma are as follows: antibodies that stimulate the TSH-R, as well as
TSH-binding inhibiting immunoglobulins or antibodies
a. Papillary carcinoma: have been described.
l For lesions smaller than 1 cm, lobectomy with isthmec- l The thyroid-stimulating antibodies stimulate the thyrocytes
tomy is appropriate. to grow and synthesize excess thyroid hormone, which is
l In all others cases, total thyroidectomy is the treatment a hallmark of Graves’ disease.
of choice. l Macroscopically, the thyroid gland is diffusely and
l The advantages of total thyroidectomy include the effi- smoothly enlarged, with a increased vascularity. Micro-
cient use of radioiodine postoperative treatment. scopically, the gland is hyperplastic with minimal colloids.
l Lymph node metastases are treated with modified radi-
cal neck dissection. Clinical Features
l Graves’ disease is characterized by a classical triad of
b. Follicular carcinoma: (a) goitre, (b) thyrotoxicosis and (c) exophthalmos.
l For follicular carcinoma diagnosed by FNAC lobec- l Female to male ratio is 6:1.
tomy, including the isthmus and pyramidal lobe is rec- l Nervous system is more predominantly affected than
ommended. the circulatory system.
l Intraoperative frozen sections should be performed l Symptoms may occur singly or in combination.
when there is evidence of capsular or vascular invasion.
a) Goitre
l Total thyroidectomy is indicated for carcinoma, except
in patients with minimally invasive follicular cancers. Diffusely enlarged vascular thyriod gland.

Q. 4. Discuss Graves’ disease in detail. b) Thyrotoxicosis


Ans. l Anxious, nervous, altered sleep patterns, emotional
mood swings, fatigue, excitability, emotional liability,
l Graves’ disease (diffuse toxic goitre, Basedow’s dis- agitation, and insomnia.
ease) is a thyroid-specific autoimmune disorder charac- l Muscle weakness, wasting, fatigue and myopathy.
terized by production of antibodies by the body against l Hyperkinesia and tremors of the hands.
thyroid-stimulating hormone receptor (TSHR), leading l Tachycardia, ↑systolic BP, palpations, widened pulse
to hyperthyroidism, or an abnormally strong release of pressure and bounding pulse.
hormones from the thyroid gland. l Hyperactive tendon reflexes.
l It is considered to be a systemic autoimmune disease l Heat intolerance, hot, moist palms.
characterized by: l Increased sweating and thirst.
l Hyperthyroidism/thyrotoxicosis l Weight loss even though the appetite is good.
l Exophthalmos l Menstrual disturbances like amenorrhoea, infertility.
l Dermatopathy l Dermopathy: Pretibial myxoedema — mucin like de-
l Pretibial myxoedema posit and thickening skin of pretibial region and dorsum
of the foot.
Gastrointestinal — diarrhoea.
Aetiology and Pathophysiology l

l In elderly, cardiovascular complications such as atrial


l The exact aetiology of the autoimmune process in fibrillation and congestive heart failure.
Graves’ disease is unknown.
l But conditions like postpartum state, iodine excess, c) Exophthalmos
lithium therapy, and bacterial and viral infections are l Protrusion of the eyeball caused by infiltration of the
considered as potential triggers. retrobulbar tissue with fluid and round cells.
l Graves’ disease is associated with certain human leukocyte l Because of the ophthalmoplegia and weakness of the
antigen (HLA) haplotypes — HLA-B8 and HLA-DR3 muscles of the eyeball, various eye signs are noticed.
and HLADQA1*0501 in Caucasian patients whereas l Ocular signs in thyrotoxicosis:
HLA-DRB1*0701 is protective against it. l Von Graefe’s sign (Lid lag) — when the eyeball

l Polymorphisms of the cytotoxic T-lymphocyte antigen moves down, the upper lid does not.
4 (CTLA-4) gene is also associated with development of l Proptosis: Widening of palpebral fissure, supraor-

Graves’ disease. bital and infraorbital swelling.


388 Quick Review Series for BDS 3rd Year

l Moebius sign: Lack of convergence of eyeballs should be monitored carefully and dosage can be ad-
l Joffroy’s sign: Absence of wrinkling of forehead. justed to maintain the euthyroid state.
l Dalrymple sign: Spasm of the upper eyelid reveal- l Propylthiouracil 100-300 mg every 6-8 hours. It has got
ing the sclera above the corneoscleral limbus. more side effects and hence not used commonly. Most seri-
l Stellwag sign: Infrequent blinking and staring look. ous side effects are agranulocytosis and aplastic anaemia.
l Diplopia: Double vision l Ideally, all the patients of thyrotoxicosis should be made
l Chemosis: Swelling (or oedema) of the conjunctiva. euthyroid with the drugs. Eventually, one can decide
whether to continue with the drugs or opt either for ra-
dioactive iodine or surgery.
Diagnosis
II. Radioactive iodine therapy
A. Thyroid function test
l Radioactive iodine destroys the thyroid cells and re-
l Suppressed TSH levels.
duces the mass of functioning tissue.
l Elevated total serum thyroxine (TT4) and triiodothyro-
l Indications for RAI are failed medical therapy or surgery,
nine T3 levels.
and where medical or surgical therapy is contraindicated.
l Antithyroid antibodies: Elevated levels of anti-TPO,
l Contraindications to RAI are pregnancy (absolute),
antimicrosomal, or antithyroglobulin antibodies (in or-
ophthalmopathy (relative; it can aggravate thyroid eye
der of sensitivity) usually confirm the autoimmune na-
disease), and solitary thyroid nodules.
ture of the thyrotoxicosis.
l TSI or TBI (thyroid-stimulating or thyroid-binding im- III. Surgery
munoglobulins): Measures of these antibodies by in vi- l Subtotal thyroidectomy (8 gm of thyroid tissue are left
tro bioassay confirm Graves disease but are rarely nec- behind) or total thyroidectomy are preferred.
essary for diagnosis. l As operating on a frankly hyperthyroid patient is dan-
gerous, prior to thyroidectomy preoperative treatment
B. Imaging studies
with antithyroid drugs is given to render the patient
l Thyroid scanning and radioactive iodine uptake: Thy- “euthyroid” (i.e. normothyroid).
roid scanning is rarely indicated for the diagnosis of l Preoperative administration of iodine, usually by Lugol’s
classic Graves disease as the whole of thyroid gland is iodine solution, decreases intraoperative blood loss dur-
visualized as hot. I-123 (123I), 99Tc scan are usually ing thyroidectomy in patients with Graves’ disease.
performed. Due to higher radiation exposure, RAIUs
using iodine I-131 (131I) are now limited to patients Q. 5. Classify thyroid tumours. Discuss the aetiopath-
who undergo RAI therapy for treatment of thyrotoxico- hology and the management of differentiated thyroid
sis or for visualization of residual thyroid malignancy. malignancies.
l Ultrasonography: Ultrasonography of the thyroid may Ans.
help to define anatomy in puzzling cases but is almost
never indicated in classic Graves’ disease. l Thyroid gland tumours are classified into benign tumours
and malignant tumours. Benign tumour of thgyroid gland
is called follicular adenoma which is diagnosed by FNAC
Treatment and treated by hemithyroidectomy or lobectomy.
The treatment modalities of Graves’ disease includes l Thyroid gland malignant tumours are classified into
three modalities: 4 types:
i. Anti-thyroid drugs i. Papillary carcinoma (75-85%)
ii. Radioactive iodine therapy ii. Follicular carcinoma (10-20%)
iii. Surgery iii. Medullary carcinoma (5%)
iv. Anaplastic carcinoma (,5%)
Along with these modalities, supportive therapy like rest, l According to WHO classification, differentiated thyroid
general nutrition, sedation and beta adrenergic blocking carcinoma can be subdivided into papillary and follicu-
agents are useful in the management of Graves’ disease. The lar thyroid carcinoma.
aim is to restore and maintain the euthyroid state permanently. l Papillary, follicular and anaplastic carcinoma of thyroid
gland arise from follicular epithelium whereas medul-
I. Antithyroid drugs lary carcinoma develops from C-parafollicular cells.
l Carbimazole: It should be started with high dose 10 mg
four times a days. When there is an improvement, the Pathogenesis and Aetiology
maintenance dose of 5 mg 3 times a day should be given
for 6 to 12 months. Thyroxine of 0.2 mg can be given There are several factors, genetic and environmental, impli-
concurrently to minimize the size of the gland. Patient cated in the pathogenesis of thyroid cancers.
Section | II General Surgery 389

Genetic factors l A lump or nodule in the neck — especially in the front


They include two important possibilities: of the neck, in the area of the Adam’s apple. This is the
a. Mutated proto-oncogenes, resulting in change in protein most common presentation of thyroid cancer as an as-
production and accelerated growth, and ymptomatic thyroid mass or a nodule that can be felt in
b. Secondly alterations in growth suppression genes, which the neck.
after initiation result in unregulated cell growth. l Enlargement of the neck.
l Enlarged lymph nodes in the neck.
Oncogenes ras, c-myc, c-fos and PTC/RET proto-oncogene l Hoarseness, difficulty in speaking normally, voice changes.
are mostly responsible for this gene mutation. l Difficulty in swallowing, or a choking feeling or diffi-
culty in breathing.
Environmental factors l Pain in the neck or throat, including pain from the neck
l Exposure to ionizing radiation, particularly during the to the ears.
first two decades of life. l Sensitivity in the neck — discomfort with neckties,
l Long-standing multinodular goitre as in iodine defi- turtlenecks, scarves, and necklaces.
ciency-related endemic goitre. l Persistent or chronic cough not due to allergies or
l Though most thyroid lymphomas arise from pre-existing illness.
Hashimoto thyroiditis, conclusive evidence suggesting l Asymmetry in the thyroid (big nodule on one side, noth-
association of autoimmune thyroiditis with thyroid car- ing on the other).
cinoma is not present. l Nodules that when manipulated give the impression that
the entire thyroid is moving.
Characteristics of Papillary Carcinoma of l Nodules that cause the wind pipe to go to one side of the
neck, as well as cause superior vena cava syndrome.
Thyroid l Some particularly aggressive thyroid tumours can go to
l 30-50 years is the peak onset of age for papillary carci- the brain and cause neurological symptoms.
noma with female: male ratio of 3:1.
Associated with thyroglossal cyst, thyroid cyst and ra-
l
Diagnosis
diation exposure to neck.
l Cure rate is very high. Diagnosis of thyroid cancer typically involves a number of
l Microscopically characterized by presence of Orphan procedures and tests.
annie eye nucleus and Psammoma bodies. i. Physical examination: Palpation of thyroid gland and
l They are not encapsulated. Their spread is lymphatic. lymph nodes to feel for enlargement and lumps, as well
as the gland’s size and firmness.
ii. Biopsy: Biopsy can be performed by “fine needle aspi-
Characteristics of Follicular Carcinoma of
ration biopsy”(FNA) or a surgical biopsy, where the
Thyroid: nodule, or the thyroid gland itself, is removed surgi-
l The peak onset age for follicular carcinoma of thyroid cally can be used.
is 40-60 yrs with female: male ratio of 3:1. Also has a iii. Imaging tests: A variety of imaging scans are used to
overall high cure rate. evaluate thyroid nodules for possible thyroid cancer.
l Encapsulated but invasion into vascular structures These include:
within the thyroid gland is common. a. Nuclear scan, also known as radioactive iodine up-
l Most common metastasis is to bones (pulsating bony take (RAI-U) scan: Nodules that absorb more radio-
metastasis) and spread is via lymphatic mostly. active iodine, and therefore, are more visible, are
l Long-standing goitre is associated with this type of car- known as “hot nodules” and are more likely to be
cinoma. benign.
l Hurtle cell carcinoma is a variant of follicular carci- b. CT scan: It is used to evaluate the thyroid. A CT
noma with worse prognosis and characterized by pres- scan cannot detect smaller nodules, but may help
ence of hurtle cell. detect and diagnose a goitre, or larger thyroid
nodules.
c. Magnetic resonance imaging (MRI): It can help to
Clinical Features detect enlargement in the thyroid, as well as tu-
Symptoms of thyroid carcinoma is usually not present in mours, tumour size, and may be able to detect tu-
early stage of the disease and mostly manifest itself in later mour spread.
stages. d. Thyroid ultrasound: It can tell whether a nodule is a
As the carcinoma increases in size, it can present itself fluid-filled cyst, or a mass of solid tissue, but it can-
with following clinical features: not determine, if a nodule or lump is malignant.
390 Quick Review Series for BDS 3rd Year

iv. Blood tests: l Thyroid cyst.


l Thyroid-stimulating hormone (TSH) blood tests is l Colloid nodules
used to evaluate the thyroid’s activity in papillary
and follicular carcinoma as these carcinomas are
Clinical Features
TSH-dependent.
l When medullary thyroid cancer is suspected, test l Common in female, seen in age group 20-40 year. 80%
for raised serum calcitonin or CEA level can be an solitary thyroid nodules occur in women.
indicator. l Most patients will be clinically and biochemically eu-
l Genetic testing to identify the abnormal gene associ- thyroid.
ated with some cases of medullary thyroid cancer l Obstructive signs — stridor, tracheal deviation, neck
can also be done. vein engorgement.
l Long duration of swelling in front of neck, dyspnoea,
dysphagia.
Treatment l Single nodule is present.
l Primary treatment for carcinoma of thyroid is surgical, l Hard area may suggest calcification and soft area
except in the case of anaplastic carcinoma when all necrosis.
types of treatment are not able to avoid the worst prog- l Sudden increase in size may occur due to haemorrhage.
nosis in most cases. l Solitary nodule has more tendencies to change in malig-
l The modalities of treatment in different types of thyroid nancy then MNG (multi-nodular goitre).
gland carcinoma are as follows:

i. Papillary carcinoma Investigations


l For lesions smaller than 1 cm, lobectomy with isthmec- l Thyroid scan: Radioactive scan is useful to make out a
tomy is appropriate. hot or cold nodule. Hot nodules are unlikely to be ma-
l In all others cases, total thyroidectomy is the treatment lignant and likely to be toxic. Cold nodules are likely to
of choice. be malignant, but they can also be a simple cyst.
l Lymph node metastases are treated with modified radi- l Thyroid function tests: T3, T4 and TSH estimations are
cal neck dissection. not of much use as most of these nodules are euthyroid.
l Ultrasound scanning: It will show cystic or solid nodule.
ii. Follicular carcinoma Solid swellings are likely to be adenoma or carcinoma.
l For follicular carcinoma diagnosed by FNAC lobec- l FNAC: Cytology of the fluid from the cyst will throw a
tomy, including the isthmus and pyramidal lobe is rec- light.
ommended. l Needle biopsy: 16 to 18 gauge needle aspiration will
l Intraoperative frozen sections should be performed pick up small clump of tissue for better histological
when there is evidence of capsular or vascular invasion. study.
l Total thyroidectomy is indicated for carcinoma, except l CT and MRI: May suggest the type of the nodule.
in patients with minimally invasive follicular cancers.
Q. 6. Describe the aetiopathology, clinical features and Management
treatment of solitary nodule in thyroid gland. l If the nodule is non-toxic and does not have any suspi-
Ans. cion of malignancy, it could be left alone and kept under
observation. In future, any time it might turn malignant
l Solitary thyroid nodules can be classified into benign or toxic. Then prompt action should be taken.
and malignant nodules. l If there are any pressure symptoms on trachea or oe-
l Generally, most (90%) thyroid nodules are benign and can sophagus or if there is any evidence of sudden increase
be classified as adenomas, colloid nodules, cysts, infectious in the size of the swelling which signifies haemorrhage
nodules, lymphocytic or granulomatous nodules, hyper- or malignancy, surgery, should be undertaken.
plastic nodules, thyroiditis, and congenital abnormalities. l Cosmetic reason is also a definite indication for surgery.
l In the thyroid surgery, whole of the thyroid gland should
be explored properly not to miss any similar nodule in
Aetiopathogenesis:
other part of the gland. lf there are no other nodules and
l Thyroiditis if there is a solitary nodule, it should be resected along
l Follicular adenoma: Most common cause of STN. with the normal surrounding thyroid tissue (resection,
l Carcinoma enucleation).
Section | II General Surgery 391

l If the nodule is situated at the junction of isthmus and more compared to CNS. Ophthalmic signs and pretibial
the lobe, hemithyroidectomy should be done. The ex- myxoedema are rarely seen.
cised nodule should be subjected to histopathology. If l Thyrotoxic symptoms: Most patients with toxic
there is any evidence of malignancy (carcinoma) im- multinodular ngoitre present with symptoms typical
mediate total thyroidectomy should be done. of hyperthyroidism, including heat intolerance, dys-
pnoea, palpitations, tremor, weight loss, hunger,
Q. 7. Classify goitre. Discuss clinical features, treatment
cardiovascular complications and frequent bowel
of multinodular goitre and mention four complications
movements.
of multinodular goitre.
l Obstructive symptoms: A significantly enlarged goitre
Ans. can cause symptoms related to mechanical obstruction:
Dysphagia, dyspnoea, or frank stridor. Involvement of
Classification of Goitre the recurrent or superior laryngeal nerve may result in
complaints of hoarseness or voice change.
Goitre is classified in different ways: l Many patients are asymptomatic or have minimal symp-
toms and are incidentally found to have hyperthyroid-
a. Non-toxic:
ism during routine screening. The most common labora-
l Simple goitre tory finding is a suppressed TSH with normal free
l Diffuse hyperplastic goitre thyroxine (T4) levels.
l Simple multinodular goitre l Findings of hyperthyroidism may be more subtle
than those of Graves disease with features like tachy-
b. Toxic:
cardia; hyperkinesis; moist, smooth skin; tremor;
l Diffuse toxic goitre/Graves’ diease. proximal muscle weakness; and brisk deep tendon
l Toxic multinodular goitre reflexes.
l Toxic nodules l The size of the thyroid gland is variable.
Goitre can also be classified according to its size: l Multiple irregular, variably sized nodules are typically
l Class I—palpation struma—in normal posture of the present. In a small gland, multinodularity may be appar-
head, it cannot be seen; it is only found by palpation. ent only on an ultrasonogram.
l Class II—the struma is palpative and can be easily seen. l Mechanical obstruction may result in superior vena
l Class III—the struma is very large and is retrosternal; cava syndrome, with engorgement of facial and neck
pressure results in compression marks. veins (Pemberton sign).
l Features of primary thyrotoxicosis (e.g. orbitopathy,
pretibial myxoedema, acropachy) are rare.
Non-Toxic Multi nodular Goitre l If treatment of a multinodular goitre is indicated, radio-
l A non-toxic multinodular goitre is a nodular enlarge- active iodine or surgery is generally more effective in
ment of the thyroid gland which is irreversible in nature. achieving a long-lasting solution to the problem com-
l Initially, uniform follicular epithelial hyperplasia (dif- pared to the use of medications alone.
fuse goitre) is present, with an increase in thyroid mass. l Although the hyperthyroidism may be treated initially
l As the disorder persists, TSH stimulates some thyroid with antithyroid medications, these medications will not
lobules (active lobules) and spares some, i.e. inactive usually achieve a permanent remission in patients with
lobules. The thyroid architecture loses uniformity, with a multinodular goitre and hyperthyroidism.
the development of areas of involution and fibrosis in-
terspersed with areas of focal hyperplasia. This process
results in multiple nodules. Complications of Multinodular Goitre
l The natural history of a multinodular goitre involves
variable growth of individual nodules; this may progress A. Complications of untreated MNG
to haemorrhage and degeneration, followed by healing l Compression of neck structures
and fibrosis. l Tracheal compression and upper airway obstruction
l Simple or non-toxic multinodular goitre can complicate (rare with benign goitres)
into secondary thyrotoxicosis, i.e. toxic multinodular goitre. l Dysphagia
l Superior vena cava syndrome
Recurrent nerve palsy
Toxic Multinodular Goitre l

l Horner’s syndrome
l Toxic multinodular goitre has gradual onset and more l Pain due to haemorrhage into a nodule
common in old age. Cardiovascular system is affected l Thyrotoxicosis
392 Quick Review Series for BDS 3rd Year

B. Complications of radioiodine treatment l Cure rate is very high.


l Radiation-induced hypothyroidism l Microscopically characterized by presence of Orphan
l Post-treatment autoimmune thyrotoxicosis (5% of patients annie eye nucleus and Psammoma bodies.
with nontoxic MNG) l They are not encapsulated. Their spread is lymphatic.
l Radiation thyroiditis (3%) l Predisposing factors include: Irradiation in childhood,
l Acute compression due to radioiodine-induced swelling long-standing thyroglossal cyst, and thyroid cyst.
(rare)
Clinical Features
C. Complications of thyroidectomy
l Recurrent laryngeal nerve injury l A lump or nodule in the neck — especially in the front
l Inadvertent surgical hypoparathyroidism of the neck, in the area of the Adam’s apple. This is the
most common presentation of thyroid cancer as an as-
D. Complications of FNA biopsy ymptomatic thyroid mass or a nodule that can be felt in
l Local discomfort the neck.
l Haematomas (rare) l Enlargement of the neck and enlarged lymph nodes in
l Infection (rare) the neck.
l Hoarseness, difficulty in speaking normally, voice
Q. 8. How do you classify goitre? Discuss the clinical changes, difficulty in swallowing, or a choking feeling
features, investigation and management of papillary and difficulty in breathing.
carcinoma. l Pain in the neck or throat, including pain from the neck
to the ears.
Ans.
l Sensitivity in the neck — discomfort with neckties,
turtlenecks, scarves, and necklaces.
Classification of Goitre l Persistent or chronic cough not due to allergies or
illness.
Goitre is classified in different ways:
l Asymmetry in the thyroid (big nodule on one side, noth-
a. Non-toxic: ing on the other).
l Nodules that when manipulated give the impression that
l Simple goitre
the entire thyroid is moving (this is often a sign of an
l Diffuse hyperplastic goitre
aggressive cancer).
l Simple multinodular goitre
l Nodules that cause the wind pipe to go to one side of the
b. Toxic: neck, as well as cause superior vena cava syndrome.
l Some particularly aggressive thyroid tumours can go to
l Diffuse toxic goitre/Graves’ disease.
the brain and cause neurological symptoms.
l Toxic multinodular goitre
l Toxic nodules
Goitre can also be classified according to its size:
Diagnosis
a. Class I—palpation struma—in normal posture of the The diagnosis of papillary carcinoma of thyroid typically
head, it cannot be seen; it is only found by palpation. involves a number of procedures and tests.
b. Class II—the struma is palpative and can be easily seen. i. Physical exam: Palpation of thyroid gland and lymph
c. Class III—the struma is very large and is retrosternal; nodes to feel for enlargement and lumps, as well as the
pressure results in compression marks. gland’s size and firmness.
ii. Biopsy: Biopsy can be performed by “fine needle aspi-
ration biopsy”(FNA) or a surgical biopsy, where the
Papillary Carcinoma of Thyroid Gland
nodule, or the thyroid gland itself, is removed surgi-
Papillary carcinoma of thyroid gland arise from follicular cally can be used.
epithelium and is the most common malignancy of thyroid iii. Imaging tests: A variety of imaging scans are used to
gland. evaluate thyroid nodules for possible thyroid cancer.
These include:
Characteristics of papillary carcinoma of thyroid l Nuclear scan: It is also known as radioactive iodine

l 30-50 years is peak onset age for papillary carcinoma uptake (RAI-U) scan. Nodules that absorb more ra-
with female: male ratio of 3:1. dioactive iodine, and therefore, are more visible, are
l Associated with thyroglossal cyst, thyroid cyst and known as “hot nodules” and are more likely to be
radiation exposure to neck. benign.
Section | II General Surgery 393

l CT scan: It is used to evaluate the thyroid. A CT scan l Patients younger than 40 years who have papillary
cannot detect smaller nodules, but may help to detect thyroid carcinoma nodules that are smaller than 1 cm,
and diagnose a goitre, or larger thyroid nodules. well-defined, minimally invasive, and isolated may be
l Magnetic resonance imaging (MRI): It can help to treated with hemithyroidectomy and isthmectomy.
detect enlargement in the thyroid, as well as tumours, l Total thyroidectomy:
tumour size, and may be able to detect tumour spread. l Perform a total thyroidectomy, i.e. removal of all

l Thyroid ultrasound: It can tell whether a nodule is a thyroid tissues but preserving the contralateral
fluid-filled cyst, or a mass of solid tissue, but it can- parathyroid glands in patients who are older than
not determine, if a nodule or lump is malignant. 40 years with papillary carcinoma and in any pa-
iv. Blood tests: tient with bilateral disease.
l Thyroid-stimulating hormone (TSH) blood tests is l In papillary tumours of the thyroid, total thyroidec-

used in papillary carcinoma as it is TSH-dependent. tomy is the surgical treatment of choice for a number
Tumour marker thyroglobulin is used. of reasons. This surgical procedure also facilitates
earlier detection and treatment of recurrent or meta-
static carcinoma.
Treatment l This surgical option is mandatory in patients with

The following medical care is appropriate for patients with papillary carcinoma discovered based on postopera-
papillary carcinoma: tive histology findings (i.e. if a very well-differentiated
l Approximately 4-6 weeks after surgical thyroid re- tumour is discovered) after a one-sided lobectomy,
moval, patients must have radioiodine therapy to detect with or without isthmectomy.
and destroy any metastasis and residual tissue in the l When the primary tumour spreads outside the thy-

thyroid. Radioiodine treatment may be used again roid and involves adjacent vital organs (e.g. larynx,
6-12 months after initial treatment of metastatic disease trachea, oesophagus), preserve these organs at the
where disease recurs or has not fully responded. first surgical approach.
l When a large, unresectable tumour is present and the l However, the surrounding soft tissues, including the
uptake of radioiodine is limited, when intractable bone muscles and involved areas of the trachea and/or oe-
pain exists, or if resection is not feasible, external beam sophagus, may be sacrificed, if they are directly in-
radiation may be performed to control local tumour volved with the differentiated thyroid carcinoma and
growth, including, but not limited to, the neck, lungs, local resection is feasible.
mediastinum, bone, and CNS. l External beam radiotherapy: This has been used as ad-
l Administer the thyroid hormone replacement levothy- juvant therapy in patients with papillary thyroid cancer
roxine to patients for life, especially after total thyroid- who were older than 45 years and had locally invasive
ectomy. Treatment consists of administering levothy- disease. Some improvements in 10-year survival rates
roxine at 2.5-3.5 mcg/kg/d. have been reported with this approach.
l Chemotherapy with cisplatin or doxorubicin has limited Q. 9. Describe the pathology, clinical features and man-
efficacy, producing occasional objective responses (gen- agement of follicular carcinoma of thyroid gland.
erally for short durations). Because of the high toxicity
of chemotherapy with cisplatin or doxorubicin, chemo- Ans.
therapy may be considered in symptomatic patients Follicular carcinoma of thyroid gland arise from follicular
with recurrent or advancing disease. epithelium.
Characteristics of follicular carcinoma of thyroid are as
follows:
Surgical Care l The peak onset of age for follicular carcinoma of thy-
l Surgery is the definitive management of thyroid cancer, roid is 40-60 yrs with female: male ratio of 3:1. Also has
and various types of operations may be performed. an overall high cure rate.
l Lobectomy with isthmectomy: This procedure is the mini- l Encapsulated but invasion into vascular structures (veins
mal operation for a potentially malignant thyroid nodule. and arteries) within the thyroid gland is common. Most
l Subtotal thyroidectomy: common metastasis is to bones (pulsating bony metas-
l This is a near-total thyroidectomy. The argument for tasis) and spread is via lymphatics mostly.
this form of surgical intervention is that total thyroid- l Long-standing goitre is associated with this type of car-
ectomy does not improve long-term prognosis and cinoma.
the incidence of complications (e.g. hypoparathy- l Hurtle cell carcinoma is a variant of follicular carci-
roidism, superior and/or recurrent laryngeal nerve noma with worse prognosis and characterized by pres-
injury) is lower with subtotal thyroidectomy. ence of hurtle cell.
394 Quick Review Series for BDS 3rd Year

Pathogenesis and Aetiology iii. Imaging tests: A variety of imaging scans are used to
evaluate thyroid nodules for possible thyroid cancer.
There are several factors, genetic and environmental, impli-
These include:
cated in the pathogenesis of follicular cancers.
a. Nuclear scan, also known as radioactive iodine up-
l Genetic factors: They include two important possibilities:
take (RAI-U) scan. Nodules that absorb more radio-
a. Mutated proto-oncogenes, resulting in change in
active iodine, and therefore, are more visible, are
protein production and accelerated growth, and
known as “hot nodules” and are more likely to be
b. Secondly alterations in growth suppression genes,
benign.
which after initiation result in unregulated cell
b. CT scan, used to evaluate the thyroid. A CT scan
growth.
cannot detect smaller nodules, but may help to de-
Oncogenes ras, c-myc, c-fos and PTC/RET proto-onco-
tect and diagnose a goitre, or larger thyroid nodules.
gene are mostly responsible for this gene mutation.
c. Magnetic resonance imaging (MRI) can help to detect
l Environmental factors:
enlargement in the thyroid, as well as tumours, tu-
l Long-standing multinodular goitre as in iodine
mour size, and may be able to detect tumour spread.
deficiency-related endemic goitre.
d. Thyroid ultrasound, can tell whether a nodule is a
fluid-filled cyst, or a mass of solid tissue, but it can-
Clinical Features not determine, if a nodule or lump is malignant.
iv. Blood tests:
l A lump or nodule in the neck – especially in the front of l Thyroid-stimulating hormone (TSH) blood tests is
the neck, in the area of the Adam’s apple. This is the used to evaluate the thyroid’s activity in papillary
most common presentation of thyroid cancer as an as- and follicular carcinoma as these carcinomas are
ymptomatic thyroid mass or a nodule that can be felt in TSH-dependent.
the neck. l When medullary thyroid cancer is suspected, test
l Enlargement of the neck and enlarged lymph nodes in for raised serum calcitonin or CEA level can be an
the neck. indicator.
l Hoarseness, difficulty in speaking normally, voice l Genetic testing to identify the abnormal gene associ-
changes, difficulty in swallowing, or a choking feeling ated with some cases of medullary thyroid cancer
and difficulty in breathing. can also be done.
l Pain in the neck or throat, including pain from the neck
to the ears.
l Sensitivity in the neck – discomfort with neckties, turtle- Treatment
necks, scarves, and necklaces. l For follicular carcinoma diagnosed by FNAC, lobectomy,
l Persistent or chronic cough not due to allergies or illness. including the isthmus and pyramidal lobe is recommended.
l Asymmetry in the thyroid (big nodule on one side, noth- Intraoperative frozen sections should be performed when
ing on the other). there is evidence of capsular or vascular invasion.
l Nodules that when manipulated give the impression that l FTC is a highly vascular lesion. In patients with bone
the entire thyroid is moving, this is often a sign of an metastases who experience severe pain that does not
aggressive cancer. respond to palliative radiation, arterial embolization of
l Nodules that cause the wind pipe to go to one side of the the tumour might be considered.
neck, as well as cause superior vena cava syndrome. l Patients younger than 40 years who have FTC nodules
l Some particularly aggressive thyroid tumours can go to that are less than 1 cm in size, well defined, minimally
the brain and cause neurological symptoms. invasive, and isolated may be treated with hemithyroid-
ectomy and isthmectomy.
Total thyroidectomy is recommended in patients who
Diagnosis l

are older than 40 years with FTC and in any patient with
Diagnosis of thyroid cancer typically involves a number of bilateral disease. This surgical option is mandatory in
procedures and tests. patients with FTC ascertained by postoperative histo-
i. Physical examination: Palpation of thyroid gland and logic studies (i.e. if a very well-differentiated tumour is
lymph nodes to feel for enlargement and lumps, as well discovered) after a one-side lobectomy, with or without
as the gland’s size and firmness. isthmectomy.
ii. Biopsy: Biopsy can be performed by “fine needle aspi- l The most useful drugs for postsurgical treatment of FTC
ration biopsy”(FNA) or a surgical biopsy, where the are L-thyroxine (L-T4) and radioiodine. Antineoplastic
nodule, or the thyroid gland itself, is removed surgi- drugs such as cisplatin and doxorubicin may be useful
cally can be used. for palliation in patients with metastases.
Section | II General Surgery 395

SHORT ESSAYS
Q. 1. Hypocalcaemia. l Maintenance doses of both calcium and vitamin-D
(often as 1, 25-(OH)2-D3, i.e. calcitriol) are often nec-
Ans.
essary to prevent further decline.
Hypocalcaemia is defined as the presence of low serum
Q. 2. Thyroid crisis.
calcium levels in the blood, usually taken as less than
2.1 mmol/l or 9 mg/dl or an ionized calcium level of less
Ans.
than 1.1 mmol/l (4.5 mg/dl). It is a type of electrolyte
disturbance. Thyroid crisis or thyroid storm is an acute, life-threatening,
hypermetabolic state induced by excess release of thyroid
hormones in individuals with thyrotoxicosis previously.
Causes
A. Absent parathyroid hormone (PTH) Causes
l Hereditary hyperparathyroidism

l Following thyroidectomy, the parathyroid glands l Though previously thyroid storm was a common com-
are located very close to the thyroid and are easily plication of toxic goitre surgery during intraoperative
injured or even accidentally removed during thy- and postoperative stages.
roidectomy. l Now with better understanding of the condition, better
l Hypomagnesaemia preoperative control of the thyrotoxic state and use of
l In DiGeorge syndrome, a disease characterized by radioiodine ablation has greatly reduced cases of thyro-
the failure of the third and fourth pharyngeal pouches toxic storms.
to develop, the parathyroid glands do not form and l Nowadays, they are more commonly seen in a thyrotoxic
there is thus a lack of PTH. patient with intercurrent illness or surgical emergency.
B. Deficient PTH l The most common cause of thyroid storm is intercurrent
l Severe acute hyperphosphataemia illness or infection (i.e. medical storm).
l Tumour lysis syndrome l Some other causes that rapidly increase the thyroid hor-
l Acute renal failure mone levels include the following:
C. Alkalosis, often caused by hyperventilation i. Radioiodine therapy
D. Chelation therapy ii. Withdrawal of antithyroid drug therapy
iii. Vigorous thyroid palpation
iv. Iodinated contrast dye
Clinical Features v. Thyroid hormone ingestion
l Perioral tingling and paraesthesia, ‘pins and needles’ vi. Sepsis or infection
sensation over the extremities of hands and feet. This is
the earliest symptom of hypocalcaemia. Clinical Feature
l Tetany, carpopedal spasm are seen.
l Latent tetany l Hyperpyrexia, temperature in excess of 106°C, dehy-
Trousseau sign of latent tetany: Eliciting carpal spasm dration.
by inflating the blood pressure cuff and maintaining the l Heart rate faster than 140 beats/min, hypotension, atrial
cuff pressure above systolic. dysrhythmias, congestive heart failure.
Chvostek’s sign: Tapping of the inferior portion of the l Cardiac arrhythmia—supraventricular arrhythmias are
zygoma will produce facial spasms. more common, e.g. atrial flutter and fibrillation, but
l Tendon reflexes are hyperactive. ventricular tachycardia may also occur.
l Life-threatening complications like laryngospasm and l Nausea, vomiting, severe diarrhoea, abdominal pain,
cardiac arrhythmias. hepatocellular dysfunction—jaundice.
l ECG changes include: Prolonged QTc and prolonged l Confusion, agitation, delirium, frank psychosis, sei-
ST interval. zures, stupor or coma.
l Tachycardia disproportionate to fever.

Treatment
Treatment
l Two ampoules of intravenous calcium gluconate 10% is
given slowly in a period of 10 minutes, or if the hypo- l The patient should be sedated immediately with mor-
calcaemia is severe, calcium chloride is given instead. phine or pethidine.
396 Quick Review Series for BDS 3rd Year

l Hyperpyrexia should be controlled by ice bag, rapid Follicular Adenomas


sponging, hypothermic blanket, and rectal ice irrigation.
i. Follicular adenomas are the most common type of ad-
l Oxygen is administered and IV glucose saline.
enomas and arise from the follicular epithelium within
l IV solution should be combat dehydration.
the thyroid gland.
l Potassium may be added to control tachycardia.
ii. They are typically homogeneous, solitary, and encapsu-
l Glucocorticoids to decrease peripheral conversion of T4
lated tumours that are histologically distinct from adja-
to T3.
cent thyroid tissue.
l Lugol’s iodine should be given IV.
iii. Follicular adenomas are further classified according to
l Propanolol should be used 20-40 mg 6 hourly.
their cellular architecture and relative amounts of cel-
l For atrial fibrillation, digitalis may be cautiously admin-
lularity and colloid into:
istered.
l fetal (microfollicular),
l Large dose of propyl thiouracil orally, rectally or
l colloid (macrofollicular),
through Ryle’s tube.
l embryonal (atypical), and
l Saturated solution of potassium iodide.
l Hürthle (oxyphil) cell types.
Q. 3. Radioiodine therapy. iv. Colloid adenomas do not have any potential for micro-
invasion, while the fetal, embryonal, and Hürthle cell
Ans.
adenomas all have the potential for microinvasion.
l Radioiodine therapy is used in nuclear medicine v. Follicular adenoma ranges in diameter from 3 cm on an
therapeutically and can also be seen with diagnostic average, but sometimes is larger up to 10 cm or smaller.
scanners, if it has been used therapeutically. vi. The typical thyroid adenoma is solitary, spherical and
l Use of the131I as iodide salt exploits the mechanism of encapsulated lesion that is well demarcated from the
absorption of iodine by the normal cells of the thyroid surrounding. Thyroid parenchyma. The colour ranges
gland. from grey-white to red-brown, depending upon:
l Examples of its use in radiation therapy are those where l The cellularity of the adenoma
tissue destruction is desired after iodine uptake by the l The colloid content.
tissue.
l Major uses of 131I include the treatment of thyrotoxi-
cosis and some types of thyroid cancer that absorb
Papillary Adenomas
iodine. Papillary adenomas are the least common type of thyroid
l The 131I isotope is also used as a radioactive label for adenoma.
certain radiopharmaceuticals that can be used for ther- A thyroid adenoma may be clinically silent (“cold” or
apy, e.g. 131I-metaiodobenzylguanidine (131I-MIBG) for “warm” adenoma), or it may be a functional tumour, pro-
imaging and treating pheochromocytoma and neuro- ducing excessive thyroid hormone (“hot” adenoma). In this
blastoma. case, it may result in symptomatic hyperthyroidism, and
l In all of these therapeutic uses, 131I destroys tissue by may be referred to as a toxic thyroid adenoma.
short-range beta radiation.
Q. 5. Multinodular goitre.
l It can be seen in diagnostic scans after its use as therapy,
because it is also a gamma-emitter. Ans.
l Because of the carcinogenicity of its beta radiation in
Multinodular goitre can be classified into:
the thyroid in small doses, I-131 is rarely used primarily
a) Non-toxic multinodular goitre
or solely for diagnosis, instead the more purely gamma-
b) Toxic multinodular goitre
emitting radioiodine Iodine-123 is used in diagnostic
testing.
l The longer half-lived iodine-125 is also occasionally Non-Toxic Multi nodular Goitre
used when a longer half-life radioiodine is needed for
l A non-toxic multinodular goitre is a nodular enlarge-
diagnosis, and in brachytherapy treatment, where its
ment of the thyroid gland which is irreversible in
low-energy gamma radiation without a beta component,
nature.
makes it useful.
l Initially, uniform follicular epithelial hyperplasia (dif-
Q. 4. Thyroid adenoma. fuse goitre) is present, with an increase in thyroid mass.
l As the disorder persists, TSH stimulates some thyroid
Ans.
lobules (active lobules) and spares some (inactive
Thyroid adenomas are benign neoplasms, which are usually lobules).
classified as follicular or papillary. l The thyroid architecture loses uniformity, with the de-
Section | II General Surgery 397

velopment of areas of involution and fibrosis inter- nodules, lymphocytic or granulomatous nodules, hyperplastic
spersed with areas of focal hyperplasia. This process nodules, thyroiditis, and congenital abnormalities.
results in multiple nodules.
The natural history of a multinodular goitre involves
Aetiopathogenesis
l

variable growth of individual nodules; this may progress


to haemorrhage and degeneration, followed by healing l Thyroiditis
and fibrosis. l Follicular adenoma: Most common cause of STN
l Simple or non-toxic multinodular goitre can complicate l Carcinoma
into secondary thyrotoxicosis, i.e. toxic multinodular goitre. l Thyroid cyst.
l Colloid nodules

Toxic Multi nodular Goitre


Clinical Features
The sequence of events leading to toxic multinodular goitre
is as follows: l Common in female, seen in age group 20-40 year. 80%
l Iodine deficiency leads to low levels of T4; this induces solitary thyroid nodules occur in women.
thyroid cell hyperplasia to compensate for the low levels l Most patients will be clinically and biochemically eu-
of T4. thyroid.
l Increased thyroid cell replication predisposes single l Obstructive signs: Stridor, tracheal deviation, neck vein
cells to somatic mutations of the TSH receptor. engorgement.
l Constitutive activation of the TSH receptor may gener- l Long duration of swelling in front of neck, dyspnoea,
ate autocrine factors that promote further growth, result- dysphagia.
ing in clonal proliferation. l Single nodule is present.
l Cell clones then produce multiple nodules. Somatic l Hard area may suggest calcification and soft area necrosis.
mutations of the TSH receptors and G a protein confer l Sudden increase in size may occur due to haemorrhage.
constitutive activation to the cyclic adenosine mono- l Solitary nodule has more tendencies to change in malig-
phosphate (cAMP) cascade of the inositol phosphate nancy then MNG (multi-nodular goitre).
pathways.
l These mutations may be responsible for functional au- Investigations
tonomy of the thyroid in 20-80% of cases.
l These mutations are found in autonomously functioning i. Thyroid scan
thyroid nodules, solitary and within a multinodular l Radioactive scan is useful to make out a hot or cold
gland. Nonfunctioning thyroid nodules within the same nodule.
gland lack these mutations. l Hot nodules are unlikely to be malignant and likely to
l Toxicmulti nodular goitre has gradual onset and more be toxic.
common in old age. Cardiovascular system is affected l Cold nodules are likely to be malignant, but they can
more compared to CNS. also be a simple cyst.
l Ophthalmic signs and pretibial myxoedema are rarely
seen. ii. Thyroid function tests
l If treatment of a multinodular goitre is indicated, radio- T3, T4 and TSH estimations are not of much use as most of
active iodine or surgery is generally more effective in these nodules are euthyroid.
achieving a long-lasting solution to the problem com-
pared to the use of medications alone. iii. Ultrasound scanning
l Although the hyperthyroidism may be treated initially l It will show cystic or solid nodule. Solid swellings are
with antithyroid medications, these medications will not likely to be adenoma or carcinoma.
usually achieve a permanent remission in patients with
a multinodular goitre and hyperthyroidism. iv. FNAC
Cytology of the fluid from the cyst will throw a light.
Q. 6. Solitary nodule of thyroid.
v. Needle biopsy
Ans.
16 to 18 gauge needle aspiration will pick up small clump
Solitary thyroid nodules can be classified into benign and of tissue for better histological study.
malignant nodule.
Generally, most (90%) thyroid nodules are benign and can vi. CT and MRI
be classified as adenomas, colloid nodules, cysts, infectious l May suggest the type of the nodule.
398 Quick Review Series for BDS 3rd Year

Management Clinical Feature


l If the nodule is non-toxic and does not have any suspi- Most cases of FTC are subclinical and manifests as asymp-
cion of malignancy, it could be left alone and kept under tomatic solitary thyroid nodule. Pressure symptoms appear
observation. In future, any time it might turn malignant in later stages presented as dyspnoea, dysphagia, and voice
or toxic. Then prompt action should be taken. hoarseness.
l If there are any pressure symptoms on trachea or oe-
sophagus or if there is any evidence of sudden increase
Diagnosis and Management
in the size of the swelling which signifies haemorrhage
or malignancy, surgery should be undertaken. l TST: Available studies are not specific for FTC.
l Cosmetic reason is also a definite indication for surgery. l Tumour markers: Thyroglobulin
l In the thyroid surgery, whole of the thyroid gland should l FNAC
be explored properly not to miss any similar nodule in l Imaging: Ultrasound, CT, radioscanning
other part of the gland. If there are no other nodules and
if there is a solitary nodule, it should be resected along Treament
with the normal surrounding thyroid tissue. Treatment is usually surgical, followed by radioiodine.
l If the nodule is situated at the junction of isthmus and l If follicular cells are found on cytological testing,
the lobe, hemithyroidectomy should be done. The ex- it is common to carry out hemithyroidectomy to dis-
cised nodule should be subjected to histopathology. If tinguish between follicular adenoma and follicular
there is any evidence of malignancy (carcinoma) im- carcinoma on histopathological examination, pro-
mediate total thyroidectomy should be done. ceeding to completion thyroidectomy and postopera-
Q. 7. Follicular carcinoma of thyroid. tive radioiodine ablation where carcinoma is con-
firmed. This way total thyroidectomy is not carried
Ans. out unnecessarily.
l Follicular thyroid carcinoma (FTC) is a well-differentiated l Thyroidectomy is invariably followed by radioiodine
malignant tumour. treatment at levels from 50 to 200 millicuries following
l FTC originates in follicular cells and is the second most two weeks of a low iodine diet (LID).
common cancer of the thyroid, after papillary carcinoma. Q. 8. Management of papillary carcinoma.
l Follicular thyroid cancer accounts for 15% of thyroid
cancer which occurs more commonly in women of over Ans.
50 years old.
l Papillary thyroid cancer or papillary thyroid carcinoma
l This carcinoma is encapsulated but the capsule gets in-
is the most common type of thyroid cancer, representing
vaded very early in the disease.
75 to 85% of all thyroid cancer cases.
l It occurs more frequently in women and presents in the
Pathophysiology 30-40 year age group.
l Predisposing factors for papillary thyroid carcinoma
l Activating point mutations in the ras oncogene are
are: previous radiation to the head and neck in child-
well known in patients with follicular adenoma and
hood, long-standing thyroglossal cyst, thyroid cyst.
carcinoma.
l They spread mainly through lymphatics to cervical lymph
l As a result of such mutations, p21-RAS becomes locked
nodes. Papillary carcinomas are slow growing and thus
in its active conformation, leading to the constitutive
have best prognosis among thyroid gland tumours.
activation of the protein and tumour development.

Mortality/Morbidity Management of Papillary Carcinoma


l Most FTCs are slow growing and are associated with a Diagnosis
very favourable prognosis. l Papillary thyroid carcinoma is usually discovered on
l Mean mortality rates are 1.5% in females and 1.4% in routine examination as an asymptomatic thyroid nodule
males. that appears as a neck mass.
l Despite its well-differentiated characteristics, follicular l FNAC is the investigation of choice followed by ultra-
carcinoma may be overtly or minimally invasive. sound and radioactive scanning.
l Metastases are due to angioinvasion and haematoge- l Histolgy demonstrates 2 type of cells: Orphan annie
nous spread, bone being the most common site of me- eyed nuclei and psamomma bodies. Tumour marker
tastasis and osteolytic in nature. thyroglobulin is used.
Section | II General Surgery 399

Medical care Clinical Features


The following medical care is appropriate for patients with Patient present initially as STN/goitre. The symptoms of
papillary carcinoma, a relatively common well-differentiated Hashimoto’s thyroiditis are similar to those of hypothyroid-
thyroid cancer: ism in general, which are often subtle.
l After surgical thyroid removal, the patient waits around The possible symptoms are as follows:
4–6 weeks to then have radioiodine therapy. l Poor appetite
l This therapy is intended to both detect and destroy any l Weight gain
metastasis and residual tissue in the thyroid. l Cold intolerance
l The treatment may be repeated 6–12 months after initial l Constipation
treatment of metastatic disease where disease recurs or l Menorrhgia, oligomenorrhoea
has not fully responded. l Mental lethargy
l Patients are administered hormone replacement levo- l Bradycardia
thyroxine for life after surgery, especially after total l Bradykinesis
thyroidectomy. l Periorbital puffiness
l Chemotherapy with cisplatin or doxorubicin has proven l Less sweating/dry skin and hair.
limited efficacy, however, it could be helpful for patients
with bone metastases to improve their quality of life.
Diagnosis
Surgical care
l FNAC
l Minimal disease (diameter up to 1.0 centimetres): l Antithyroid antibodies
Hemithyroidectomy (or unilateral lobectomy) and isth- l TSH test
mectomy may be sufficient. There is some discussion
whether this is still preferable over total thyroidectomy
for this group of patients. Treatment
l Gross disease (diameter over 1.0 centimetres): Total thy- Medical management
roidectomy, and central compartment lymph node re-
moval is the therapy of choice. Additional lateral neck l The treatment of choice for Hashimoto’s thyroiditis (or
nodes can be removed at the same time, if an ultrasound- hypothyroidism of any cause) is thyroid hormone re-
guided FNA and thyrobulin TG cancer washing was placement.
positive on the pre operative neck node ultrasound evalu- l The drug of choice is orally administered levothyroxine
ation. sodium, usually for life.
l The goal of therapy is to restore a clinically and bio-
Q. 9. Autoimmune thyroiditis/ Hashimoto’s thyroiditis. chemically euthyroid state.
l The standard dose is 1.6-1.8 mcg/kg lean body weight
Ans.
per day, but the dose is patient dependent. It is recom-
l Hashimoto’s thyroiditis (also called autoimmune or mended that the TSH levels be kept under 3.0.
chronic lymphocytic thyroiditis) is the most common
Surgical care is indicated in following conditions:
thyroiditis.
l A large goitre with obstructive symptoms, such as dys-
l Hashimoto’s thyroiditis is characterized by the pro-
phagia, voice hoarseness, and stridor, caused by extrin-
duction of immune cells and autoantibodies by the
sic obstruction of airflow.
body’s immune system, which can damage thyroid
l Presence of a malignant nodule, as found by cytologic
cells and compromise their ability to make thyroid
examination by fine-needle aspiration.
hormone.
l Presence of a lymphoma diagnosed on fine-needle aspi-
l Physiologically, antibodies against thyroid peroxidase
ration.
(anti-TPO), thyroglobulin (anti-Tg) and TSH receptors
l Thyroid lymphoma responds very well to radiotherapy
cause gradual destruction of follicles in the thyroid
and is the treatment modality of choice in this situation.
gland.
l Cosmetic reasons for large, unsightly goitres.
l Colloid hormones get released in the blood from the
follicles, resulting in elevated levels of T3 and T4 hor- Q. 10. Retrosternal goitre.
mones.
Ans.
l However, this elevation is transient. But as antibodies
destroy all follicles, functional part of gland is depleted l Retrosternal goitre is usually defined as enlarged thy-
and no more hormones are produced, resulting is hypo- roid gland with greater than 50% of its mass below the
thyroidism. thoracic inlet.
400 Quick Review Series for BDS 3rd Year

l Retrosternal goitre can be classified into two groups. Primary Thyrotoxicosis


l One is the truly intrathoracic or aberrant goitre. Its

presence in the mediastinum is congenital, with the Symptoms


blood supply derived entirely from intrathoracic ves- l Hyperactivity, irritability and dysphoria
sels, and it has no direct connection to the cervical l Heat intolerance and swelling
thyroid gland. l Palpitations
l The much more common second group represents l Fatigue and weakness
the acquired retrosternal goitre. It arises in the cervi- l Weight loss with increased appetite
cal thyroid gland and, while growing, descends along l Diarrhoea and polyuria.
a fascial plane, through the thoracic inlet into the l Oligomenorrhoea and loss of libido.
mediastinum.
l According to the degree of descent, these goitres are Signs
classified:
l Tachycardia is main sign which is present due to activa-
i) Substernal goitre: Goitre can be palpable from the
tion of thyrocardiac component.
neck by insinuating the fingers behind the sternum.
l Systolic hypertension is present.
ii) Plunging goitre: Normally, this goitre is not palpa-
l Presence of tremors, i.e. involuntary movement of body
ble in the neck. But by increasing the intrathoracic
parts is present.
pressure such as coughing and sneezing, the goitre
l Cardiac arrhythmias, i.e. atrial fibrillation and atrial
is pushed up into the neck.
tachycardia develops.
iii) Intrathoracic goitre: It is completely inside the tho-
l Diaphoresis is present, i.e. excessive sweating is
rax and never palpable in the neck.
present.
l There is presence of powerful wide pulse pressure and
Clinical Features good bounding pulse is present.
l Exaggerated deep tendon reflexes are seen.
l Usually symptomless but can cause obstruction.
l There is protrusion of eyes, i.e. exophthalmus with star-
l Dyspnoea, dysphagia if obstructing the trachea or oe-
ing look.
sophagus.
l Veins in the neck may get engorged due to the obstruc-
tions to the great veins (superior vena caval syndrome). Treatment
l Goitres with these symptoms are usually toxic or i. General:
malignant. a. Allow the patient to take mental and physical rest.
b. Maintain nutrition of patient by giving nutritious
diet.
Investigations c. If patient is anxious alprazolam 0.25 to 0.5 mg BD
l X-ray of chest may show compression or deviation of is given.
the trachea and calcification of goitre. d. Most of the patients of thyrotoxicosis are fearful
l Barium swallow to know the position and amount of because of increased sympathetic activity. In
compression of the oesophagus. such type of patient, propanolol 40-80 mg BD is
l Thyroid scan—to differentiate the goitre from mediasti- given.
nal tumour. ii. Drug therapy:
l Assessment of thyroid function for evidence of toxicity. a. Carbimazole is commonly used drug. It is started
with—30 mg/day, adjustment of doses is made
when patient come under control and maintenance
Treatment dose is given, i.e. 10 - 20 mg/ day.
l In case obstructive symptoms along with thyrotoxicosis b. Potassium per chlorate 800 mg/ day in divided doses
are evident, it is unwise to treat a retrosternal goitre with is given and the dose is reduced with improvement
antithyroid drugs or radioiodine as these may enlarge in patient’s condition.
the goitre. c. Iodides are given, i.e. sodium or potassium iodide
l Resection can almost always be carried out from the 6 to 10 mg/day.
neck, but median sternotomy is sometimes preferred. iii. Surgery: The commonly performed surgery is subtotal
thyroidectomy, prior surgery anti-thyroids are given to
Q. 11. Describe clinical symptoms, signs and treatment
make patient euthyroid. Potassium iodide 50 to 100 mg/
of primary thyrotoxicosis.
day is given for 10 to 14 days.
Ans. iv. Radioiodine treatment:
Section | II General Surgery 401

SHORT NOTES
Q. 1. TSH. Q. 3. Thyroid scan 131I.
Ans. Ans.

i. Thyroid-stimulating hormone is also known as TSH or l A thyroid scan 131I is a nuclear medicine examination
thyrotropin and is a peptide hormone synthesized and using radioactive iodine isotope 131 to determine func-
secreted by thyrotrope cells in the anterior pituitary tioning of thyroid gland.
gland, which regulates the endocrine function of the l The thyroid scan can be used to determine the following:
thyroid gland. i. Determine functioning of gland
ii. Thyroid-stimulating hormone controls the rate of secre- ii. To diagnose pathology of thyroid gland, such as
tion of thyroxine and triiodothyronine by the thyroid hyperthyroidism, cancer or other growths
gland, and these hormones control the rates of most iii. Assess the nature of a nodule discovered in the gland
intracellular chemical reactions in the body. iv. Detect areas of abnormality, such as lumps (nod-
iii. TSH stimulates the thyroid gland to secrete the hor- ules) or inflammation
mones thyroxine (T4) and triiodothyronine (T3). v. Determine whether thyroid cancer has spread be-
iv. TSH production is controlled by thyrotropin-releasing yond the thyroid gland
hormone (TRH), vi. Evaluate changes in the gland following surgery,
v. TSH secretion by the anterior pituitary is regulated via radiotherapy or chemotherapy.
a negative feedback loop by T4 and T3; when the levels Q. 4. Thyrotoxicosis.
of T3 and T4 are low, the production of TSH is in-
creased, and, on the converse, when levels of T3 and T4 Ans.
are high, TSH production is decreased. i. Thyrotoxicosis or hyperthyroidism is the clinical syn-
drome caused by an excess of circulating free thyroxine
Q. 2. Goitre. and free triiodothyronine, or both.
Ans. ii. It is common, affecting about 2% of women and 0.2%
of men.
i. Goitre is defined as enlargement of the thyroid gland, iii. Aetiology: Graves’ disease accounts for 70-80% of
leading to swelling of the neck. thyrotoxicosis followed by toxic thyroid adenoma,
ii. The most common cause for goitre is iodine deficiency, toxic multi-nodular goitre, and subacute thyroiditis.
followed by Hashimoto’s thyroiditis in places where iv. Clinical features: Thyrotoxicosis usually develops in-
iodised salt is used. sidiously, and most patients have had symptoms for at
iii. Clinical picture: The main symptom of goitre is swell- least 3-6 months before presentation. Almost every
ing of the thyroid gland, which may eventually become system is affected:
a noticeable lump in the throat. l Palpitation, tachycardia, cardiac arrhythmias, car-
The following symptoms may also exist when the goi- diac failure
tre enlarges significantly in size: l Sweating, tremor, hyperkinetic movements, nervous-
l Hoarseness (voice) ness, myopathy, irritability, tiredness and lethargy.
l Coughing l Weight loss
l A feeling of tightness in the throat l Heat intolerance
l Dysphagia (less common) l Diarrhoea and vomiting
l Dyspnoea (less common) l Emotional disturbance, behavioural abnormalities
iv. Treatment: l Ophthalmic signs
l Treatment may not be necessary, if the goitre is l Irregular menstruation and amenorrhoea
small. l Pretibial myxoedema
l Graves’ disease can be corrected with antithyroid l Vitiligo, loss of libido
drugs such as propylthiouracil and methimazole, v. Diagnosis: It is based on measurement of thyroid stimulat-
thyroidectomy, and iodine-131. ing hormone and free thyroxine in the serum, which are
l Levothyroxine can also be used in euthyroid patients usually present in low and high concentrations, respectively.
for the treatment of goitre. vi. Treatment:
131
l Thyroidectomy with I may be necessary in euthy- l Rapid symptomatic relief can be achieved with beta-
roid goitrous patients who do not respond to levo- blockers.
thyroxine treatment, especially if the patients have l Thyroid function can be reduced by anti-thyroid
difficulty breathing or swallowing. drugs, radioactive iodine and surgery.
402 Quick Review Series for BDS 3rd Year

Q. 5. Thyroid crisis. Q. 7. Tetany.


Ans. Ans.
i. Thyroid crisis or storm is an acute, severe, life-threatening, l Tetany is a medical sign, the involuntary contraction of
hypermetabolic state or thyrotoxic reaction induced by muscles, caused by diseases and other conditions that
excessive release of thyroid hormones (THs) and appear increase the action potential frequency.
suddenly within 3 or 4 days after thyroidectomy. l This condition is characterized by hyperexcitability of
peripheral nerves.
Clinical Features l The most common cause of tetany is deficiency of
calcium, but excess of phosphate high phosphate-to-
Thyroid crisis is an acute exacerbation of hyperthyroidism calcium ratio can also trigger the spasms.
and usually characterized by: l Other factors which can result in tetany are: hypofunc-
l Tachycardia disproportionate to fever. tion of the parathyroid gland, hyperventilation (low
l Hyperpyrexia, temperature in excess of 106°C, dehy- levels of carbon dioxide), low levels of magnesium.
dration. l Diagnosis: Trousseau’s sign and Chovostek’s sign.
l Profuse sweating, respiratory distress, fatigue l Treatment: Oral calcium such as calcium lactate, cal-
l Confusion, agitation, delirium, frank psychosis, seizures, cium gluconate may relieve mild symptoms. Acute
stupor or coma cases, injection calcium gluconate 10% (10 ml) should
l Hypertension with wide pulse pressure be given.
l Signs of high-output heart failure
l Nausea, vomiting, severe diarrhoea, abdominal pain, Q. 8. Describe clinical features and treatment of tetany.
hepatocellular dysfunction-jaundice Ans.
Tetany is a condition where is hyperexcitability of periph-
Treatment eral nerves.
l The patient should be sedated immediately with mor-
phine or pethidine.
Clinical Features
l Hyperpyrexia should be controlled by ice bag, rapid
sponging, hypothermic blanket, and rectal ice irrigation. i. The first symptoms of tetany are tingling and numbness
l Oxygen is administered and IV glucose, saline solution in the face, fingers, and toes.
should be combat dehydration. ii. Carpopedal spasm or Trousseau’s sign:
l Potassium may be added to control tachycardia. l It occurs in extreme cases and latent tetany.

l Cortisone is often highly effective. l The arm is flexed at elbow, wrist, and metacarpopha-

l Propanolol should be used 20-40 mg 6 hourly. langeal joints but the interphalangeal joints are ex-
l For atrial fibrillation, digitalis may be cautiously admin- tended. The thumb moves towards the palm.
istered. iii. Chovostek’s sign:
l Large dose of propyl thiouracil orally, rectally or l It also occurs in latent tetany.
through Ryle’s tube. l It indicates facial hyperexcitability.

l Saturated solution of potassium iodide. l If a tap is given to facial nerve in front of ear, twitch-

ing of eyelids, corners of mouth occurs.


Q. 6. Exophthalmos.
iv. Laryngeal spasm:
Ans. l Increased excitability of the laryngeal muscles pro-

duces laryngeal spasm.


i. Exophthalmos also called exophthalmia or proptosis is
l This leads to blockage of respiratory passage and
defined as “abnormal protrusion of the eyeball anteri-
death may occur.
orly out of the orbit.
v. Treatment: Oral calcium such as calcium lactate, calcium
ii. Mostly the term “exophthalmos” is reserved for those
gluconate may relieve mild symptoms. Acute cases, in-
cases of proptosis which are secondary to endocrino-
jection calcium gluconate 10% (10 ml) should be given.
logical dysfunction.
iii. Graves’ disease, an autoimmune disease that causes Q. 9. Retrosternal goitre.
hyperthyroidism is the most common cause of exoph-
Ans.
thalmos.
iv. Thyroid problems generally are common causes. i. Retrosternal goitre is usually referred to as enlarged
v. Measurement of the degree of exophthalmos is per- thyroid gland with greater than 50% of its mass below
formed using an exophthalmometer. the thoracic inlet.
Section | II General Surgery 403

ii. Retrosternal goitres are classified into two groups: l Tracheomalacia: Due to long-standing goitre, trachea
a. Congenital (intrathoracic or aberrant goitre) cartilage become soft and eventually collapse post-
b. Acquired (more common and it arises in the cervical thyroidectomy.
thyroid gland) l Bilateral recurrent laryngeal nerve injury.
iii. Clinical features: l Hypocalcaemia: Damage to parathyroid gland results in
l Most retrosternal goitres are symptomless and are spasm of respiratory muscles. Dyspnoea in this case
discovered on a routine chest X-ray. manifest only 2-3 days post-surgery.
l There may be severe symptoms like dyspnoea, mostly
Q. 12. Medullary carcinoma of thyroid.
at night, cough and stridor, dysphagia, engorgement
of facial, neck and superficial chest wall veins. Ans.
l In severe cases, there may be obstruction of the su-
i. Medullary carcinoma of the thyroid (MTC) is a thyroid
perior vena cava. carcinoma which originates in the parafollicular C cells
iv. Treatment: of the thyroid gland.
l In case obstructive symptoms along with thyrotoxi-
ii. There are two forms of MTC:
cosis are evident, it is unwise to treat a retrosternal a) Sporadic MTC (most common)
goitre with antithyroid drugs or radioiodine as these b) Inherited MTC (occurs in association with multiple
may enlarge the goitre. endocrine neoplasia (MEN) type 2A and 2B syn-
l Resection may be carried out from the neck, but
dromes, and non-MEN familial MTC).
median sternotomy is sometimes preferred. iii. Clinical feature: There are no specific symptoms of
Q. 10. Parathyroid tetany. medullary thyroid carcinoma (MTC) but, one or more
of the following symptoms may be observed:
Ans. l Presence of lump at the base of the neck, which may

i. Parathyroid tetany is defined as spasms caused by insuf- interfere with or become more prominent during
ficiency or complete absence of function of the parathy- swallowing.
roid glands, i.e. hypofunction of the parathyroids. l In advanced stages: Hoarseness, dysphagia, and re-

ii. The principal clinical manifestations of parathyroid spiratory difficulty.


tetany are: l Distant metastases (e.g. lung, liver, bone) may produce

l Muscle twitching, symptoms of weight loss, lethargy, and bone pain.


l Sharp flexion of the wrist and ankle joints (carpo- iv. Surgery remains the main treatment of MTC.
pedal spasm), Q. 13. Metastasis in papillary carcinoma of thyroid.
l Cramps, and convulsions, sometimes with attacks of

stridor. Ans.
l Tonic muscular spasms of varying duration in differ- l Papillary thyroid carcinomas typically invade the lym-
ent parts of the body. phatics and spread to other sites within the thyroid
l Increased electromechanical excitability of the mo- gland, as well as to the regional lymph nodes.
tor and sensory nerves, and l Lymph node metastases have been reported in the range
l Increased excitability of the autonomic nervous sys- of 46-90% of cases of papillary carcinoma.
tem leading to the malfunction of internal organs. l Vascular invasion is uncommon; however, if it does oc-
l During an attack of tetany, sudden death may occur cur, the spread of tumour is usually to the lungs and
from asphyxia or heart failure. bones.
l In latent tetany, an attack may be provoked by some l Direct extension into the soft tissues of the neck occurs
factor such as infection or intoxication. in approximately 25% of cases.
l Treatment of parathyroid tetany is based on replace-

ment therapy, i.e. the injection of such substances as Q. 14. Thyrotoxicosis—medical management.
parathyroid hormone or calcium preparations. Ans.
Q. 11. Causes for dyspnoea in goitre. Principles of treatment of thyrotoxicosis with the exception
of low 123I uptake hyperthyroidism (e.g. subacute thyroid-
Ans.
itis). The treatment of hyperthyroidism includes symptom
Causes of dyspnoea in goitre can be the following: relief and therapy with anti-thyroid medications, therapy
l Pressure symptom: Due to enlargement of thyroid with radioactive iodine131I, or thyroidectomy.
gland. l Symptom relief: Many of the neurologic and cardiovas-
l Bleeding: Formation of haematoma resulting in pres- cular symptoms of thyrotoxicosis are relieved by beta-
sure on trachea. blocker therapy.
404 Quick Review Series for BDS 3rd Year

l Antithyroid drugs: Methimazole, propylthiouracil. These ii. Clinical features:


drugs inhibit multiple steps in the synthesis of T4 and T3, l Thyroglossal cysts are usually found in subhyoid

leading to a gradual reduction in thyroid hormone levels portion of tract.


over 2-8 weeks or longer. Nodular forms of hyperthyroid- l Usually present as midline swellings/lump which is

ism (toxic multinodular goitre and toxic adenoma) are smooth, painless and cystic.
permanent conditions and will not go into remission. l Remainder can be found as far lateral as lateral tip of

l Radioactive iodine therapy: Radioactive iodine ther- hyoid bone.


apy is the most common treatment for hyperthyroid- l The cyst elevates/protudes on protrusion of the tongue.

ism in adults. Although the effect is less rapid than l Male: female ratio is approximately equal.

it is in antithyroid medication or thyroidectomy, it is iii. Treatment:


effective and safe and does not require hospitalization. Sistrunk operation: Involves excision not only of the
It is administered orally as a single dose, in capsule or cyst but also of the path’s tract and branches.
liquid form. Generally, the dose of 131I administered is
Q. 17. Discuss differences in—physiological, colloidal
75-200 mCi/g of estimated thyroid tissue divided by
and nodular goitre.
the percent of 123I uptake in 24 hours.
l Surgical care: Thyroidectomy. Ans.
Q. 15. Complications of thyroid surgery.
Physiological Goitre
Ans.
l It is called sporadic goitre.
In general, complications of thyroid surgery can be divided l The goitre is soft and diffuse
into minor, rare, or major. l Puberty, pregnancy goitre.
A. Minor complications: Post-operative surgical site sero-
mas, poor scar formation.
B. Rare complications: Damage to the sympathetic trunk. Colloidal Goitre
C. Major complications: l It is the late stage of diffuse hyperplasia.
l Bleeding: Present with neck swelling, neck pain, and/ l TSH levels have gone down and many follicles are inac-
or signs and symptoms of airway obstruction (e.g. tive and full of colloid.
dyspnoea, stridor, hypoxia). l May be due to iodine deficiency.
l Injury to recurrent laryngeal nerve: Present with post-

operative hoarseness or breathiness.


l Hypoparathyroidism: Signs of hypocalcaemia—
Nodular Goitre
circumoral paraesthesias, mental status changes, l The formation of nodules takes place due to fluctuating
tetany, carpopedal spasm, laryngospasm, seizures, TSH stimulation and its level in circulation.
QT prolongation on ECG, and cardiac arrest. l The nodule may be solid or cellular.
l Thyrotoxic storm l It may occur due to adenoma or carcinoma.
l Injury to superior laryngeal nerve: Mild hoarseness
Q. 18. Plunging goitre.
or decreased vocal stamina.
l Infection: Manifests as superficial cellulitis or as an Ans.
abscess.
l Plunging goitre (goitre plongeant), initially described
Q. 16. Thyroglossal cyst. and so named by Malard is a cervical goitre disappear-
ing within the chest, only to bob up into the neck fol-
Ans.
lowing a straining effort or during an attack of coughing
i. Thyroglossal cyst is a fibrous cyst formed from a per- brought on by a choking spell.
sistent thyroglossal duct (foramen caecum). They l It is a freely movable goitre that is sometimes above and
arise at junction of anterior two-thirds and posterior sometimes below the sternal notch.
one-third of the tongue. l It is also called wandering goitre/diving goitre.
Section | II General Surgery 405

Topic 15

Local Anaesthesia, Biopsy and Miscellaneous

SHORT ESSAYS
Q. 1. Local anaesthesia anaesthesia usually extends from the legs to the ab-
domen or chest.
Ans.
l Intravenous regional anaesthesia (Bier’s block)—
i. Local anaesthesia is defined as any technique to render blood circulation of a limb is interrupted using a
part of the body insensitive to pain without affecting tourniquet, then a large volume of local anaesthetic
consciousness. is injected into a peripheral vein. The drug fills the
ii. It allows patients to undergo surgical and dental proce- limb’s venous system and diffuses into tissues where
dures with reduced pain and distress. peripheral nerves and nerve endings are anaesthe-
iii. Local anaesthetic can block almost every nerve be- tized. The anaesthetic effect is limited to the area
tween the peripheral nerve endings and the central that is excluded from blood circulation and resolves
nervous system. quickly once circulation is restored.
iv. Small and large peripheral nerves can be anaesthetized l Local anaesthesia of body cavities (e.g. intrapleu-
individually (peripheral nerve block) or in anatomic ral anaesthesia, intra-articular anaesthesia).
nerve bundles (plexus anaesthesia).
v. Various clinical techniques of local anaesthesia include:
l Surface anaesthesia—application of local anaes-
Use of Local Anaesthesia
thetic spray, solution or cream to the skin or a mu- Typical operations performed under conduction anaesthesia
cous membrane. The effect is short lasting and is include:
limited to the area of contact. l Dentistry: Surface anaesthesia, infiltration anaesthesia
l Infiltration anaesthesia—injection of local anaes- or intraligamentary anaesthesia during restorative oper-
thetic into the tissue to be anaesthetized. Surface and ations or extractions, regional nerve blocks during
infiltration anaesthesia are collectively topical an- extractions and surgeries.
aesthesia. l Eye surgery: Surface anaesthesia with topical anaesthet-
l Field block—subcutaneous injection of a local ics, retrobulbar block.
anaesthetic in an area bordering on the field to be l ENT operations, head and neck surgery: Infiltration
anaesthetized. anaesthesia, field blocks, peripheral nerve blocks,
l Peripheral nerve block—injection of local anaes- plexus anaesthesia.
thetic in the vicinity of a peripheral nerve to anaes- l Heart and lung surgery: Epidural anaesthesia combined
thetize that nerve’s area of innervation. with general anaesthesia.
l Plexus anaesthesia—injection of local anaesthetic l Abdominal surgery: Epidural/spinal anaesthesia, often
in the vicinity of a nerve plexus, often inside a tissue combined with general anaesthesia.
compartment that limits the diffusion of the drug l Gynaecological, obstetrical and urological operations:
away from the intended site of action. The anaes- Spinal/epidural anaesthesia.
thetic effect extends to the innervation areas of sev- l Bone and joint surgery of the pelvis, hip and leg: Spinal/
eral or all nerves stemming from the plexus. epidural anaesthesia, peripheral nerve blocks, intrave-
l Epidural anaesthesia—a local anaesthetic is in- nous regional anaesthesia.
jected into the epidural space where it acts primarily l Surgery of skin and peripheral blood vessels: Topical
on the spinal nerve roots. Depending on the site of anaesthesia, field blocks, peripheral nerve blocks, spinal/
injection and the volume injected, the anaesthetized epidural anaesthesia.
area varies from limited areas of the abdomen or
chest to large regions of the body. Q. 2. Complications of local anaesthesia.
l Spinal anaesthesia—a local anaesthetic is injected
Ans.
into the cerebrospinal fluid, usually at the lumbar
spine (in the lower back), where it acts on spinal l Local anaesthetics are safe and have few major side ef-
nerve roots and part of the spinal cord. The resulting fects. But in high doses, local anaesthetics can have toxic
406 Quick Review Series for BDS 3rd Year

effects caused by being absorbed through the blood- l It is believed that smaller-diameter needles, i.e.
stream into the rest of the body (systemic toxicity). 30 gauge, are more likely to break than larger-diameter,
l These adverse events may be classified as either systemic i.e. 25 gauge.
or localized. Systemic complications can occur as a result l Needle breakage usually occurs at the hub, which is
of psychogenic reactions induced by anxiety, toxicity the reason for never inserting a needle completely into
secondary to high levels of the drug in blood, or allergy. tissue.
l Localized complications may manifest in a number of
ways as follows: vi. Soft tissue injury:
With the loss of sensation that accompanies a successful
i. Paraesthesia: block, a patient can easily bite into his or her lip or tongue.
Complete anaesthesia or an altered sensation in the lip or
tongue may persist beyond the expected duration of action vii. Facial nerve paralysis:
of a local anaesthetic. Anaesthesia of the facial nerve may occur, if the needle has
l Commonly referred to as a paraesthesia, these neuropa- penetrated the parotid gland capsule and local anaesthetic is
thies may manifest as a total loss of sensation (anaesthe- then administered within this nerve.
sia), a burning or tingling, pain to non-noxious stimuli l The seventh cranial nerve is contained within the
(dysesthesia), or increased pain to noxious stimuli (hy- parotid gland and provides motor function through its
peresthesia). five branches — the temporalis, zygomatic, buccal,
mandibular and cervical.
ii. Trismus: l Needle placement into the parotid may occur, if there is
Limited jaw opening, or trismus, is a relatively common overinsertion during an inferior alveolar nerve block or
complication following local anaesthetic administration. the Vazirani-Akinosi block.
l It can be caused by spasm of the muscles of mastication,
which in turn may be a result of needle insertion into or viii. Infection:
through one of them. With the introduction many years ago of sterile disposable
l The most common muscle to be the source of trismus is needles, infection is now an extremely rare complication of
the medial pterygoid, which can be penetrated during an local anaesthetic administration.
inferior alveolar nerve block using any of the three main l It may occur, if the needle has been contaminated prior
techniques: the conventional approach, the Vazirani- to insertion.
Akinosi (closed-mouth) technique, or the Gow-Gates.
ix. Mucosal lesions:
iii. Haematoma: Occasionally, the intraoral mucosa may show signs of
A haematoma is a localized mass of extravasated blood that sloughing or ulceration.
may become clinically noticeable following an injection. l The epithelial layer may desquamate from prolonged
l The vessels most commonly associated with haemato- application of topical anaesthetic.
mas are the pterygoid plexus of veins, the posterior l It is possible, but not common, that necrosis of tissues
superior alveolar vessels, the inferior alveolar vessels, may result from high concentrations of vasoconstrictor,
and the mental vessels. such as 1:50, 000.
l The patient will notice development of swelling and the l Sites of ulceration consistent with a diagnosis of
discolouration of a bruise lasting seven to 14 days. aphthous stomatitis may also result following local
anaesthetic administration.
iv. Pain:
Occasionally, injection of local anaesthetic can be accom- Q. 3. FNAC.
panied by pain or a burning sensation.
Ans.
l Passing the needle through a sensitive structure such as
muscle or tendon may cause pain. l Fine needle aspiration cytology (FNAC), also known as
l It may occur during injection, if the solution is adminis- needle aspiration biopsy (NAB), fine needle aspiration
tered too quickly and, therefore, distends the tissue rapidly. biopsy (FNAB) and fine needle aspiration (FNA), is a
l Local anaesthetic solutions that are too cold or too diagnostic procedure used to investigate superficial
warm may also cause discomfort. lumps or masses.
l In this technique, a thin, hollow needle is inserted into
v. Needle breakage: the mass to extract cells that, after being stained, will be
This event is very rare. examined under a microscope.
l Sudden, unexpected movement of the patient is the l Fine needle aspiration biopsies are very safe, minor
primary cause. surgical procedures. A needle aspiration biopsy is safer
Section | II General Surgery 407

and less traumatic than an open surgical biopsy, and l Gastrointestinal symptoms may include crampy abdomi-
significant complications are usually rare, depending on nal pain, diarrhoea, and vomiting.
the body site. l Due to the presence of histamine-releasing cells in the
l Common complications include bruising and sore- heart, coronary artery spasm may occur with subsequent
ness. There is a risk, because the biopsy is very small myocardial infarction or dysarhythmia.
(only a few cells), that the problematic cells will be l A drop in blood pressure may result in a feeling of light
missed, resulting in a false negative result. There is headedness and loss of consciousness.
also a risk that the cells taken will not enable a
definitive diagnosis.
Management
Resuscitation measures such as airway management,
Applications l

supplemental oxygen, large volumes of intravenous


i. A biopsy is performed on a lump or a tissue-mass when fluids, and close monitoring.
its nature is in question. l Administration of epinephrine is the treatment of
ii. For known tumours, this biopsy is performed to assess choice with antihistamines and steroids often used as
the effect of treatment or to obtain tissue for special adjuncts.
studies. l A period of hospital observation for between 6 and
24 hours is recommended for people once they
Q. 4. Anaphylaxis.
have returned to normal due to concerns of biphasic
Ans. anaphylaxis.

Anaphylaxis is defined as an acute multi-system severe Q. 5. T lymphocyte.


type I hypersensitivity reaction.
Ans.
l T lymphocyte is a component of white blood cells
Classification known as lymphocytes, which has a central role in cell-
l Biphasic anaphylaxis: Biphasic anaphylaxis is the mediated immunity.
recurrence of symptoms within 72 hours with no further l They are distinguished from other lymphocyte types, by
exposure to the allergen. the presence of a special receptor on their cell surface
l Anaphylactic shock: Anaphylactic shock is anaphylaxis called T cell receptors (TCR).
associated with systemic vasodilation that results in l Thymus is the principal organ responsible for the
low blood pressure. It is also associated with severe T cell’s maturation.
bronchoconstriction to the point where the individual is l Several different subsets of T cells have been discovered,
unable to breathe. each with a distinct function:
l Pseudoanaphylaxis: It does not involve an allergic
reaction but is due to direct mast cell degranulation. i. T helper cells (TH cells):
l Active anaphylaxis/anaphylaxis: Two weeks or so after l They assist other white blood cells in immunologic pro-
an animal, including humans, is exposed to certain cesses, including maturation of B cells into plasma cells
allergens, active anaphylaxis (which is simply called and B memory cells, and activation of cytotoxic T cells
“anaphylaxis”) would be elicited upon exposure to the and macrophages, among other functions.
same allergens. l These cells are also known as CD41 T cells because
l Passive anaphylaxis: Passive anaphylaxis is induced in they express the CD4 protein on their surface.
native animals that receive transfer of the serum experi- l Helper T cells become activated when they are pre-
mentally from sensitized animals with certain allergens. sented with peptide antigens by MHC class II molecules
Passive anaphylaxis would be provoked in the recipient that are expressed on the surface of antigen presenting
animals after exposure to the same allergens. cells (APCs).
l Once activated, they divide rapidly and secrete small
proteins called cytokines that regulate or assist in the
Clinical Picture active immune response.
l These usually develop over minutes to hours.
l Skin involvement may include generalized hives, itchi- ii. Cytotoxic T cells (TC cells, or CTLs):
ness, flushing, and swelling of the lips, tongue, or l They destroy virally infected cells and tumour cells, and
throat. are also implicated in transplant rejection.
l Respiratory symptoms may include shortness of breath, l These cells are also known as CD81 T cells since they
wheezes or stridor. express the CD8 glycoprotein at their surface.
408 Quick Review Series for BDS 3rd Year

l These cells recognize their targets by binding to antigen Radiation therapy is the term used to define the medical use
associated with MHC class I, which is present on the of ionizing radiation as part of cancer treatment to control
surface of nearly every cell of the body. malignant cells. Radiotherapy may be used for curative or
adjuvant treatment as well as palliative treatment or as
iii. Memory T cells: therapeutic treatment (where the therapy has survival ben-
l They are a subset of antigen-specific T cells that persist efit and it can be curative). Total body irradiation (TBI) is a
long-term after an infection has resolved. radiotherapy technique used to prepare the body to receive
l They quickly expand to large numbers of effector a bone marrow transplant. Radiotherapy has several appli-
T cells upon re-exposure to their cognate antigen, thus cations in non-malignant conditions, such as the treatment
providing the immune system with “memory” against of trigeminal neuralgia, severe thyroid eye disease, pteryg-
past infections. ium, pigmented villonodular synovitis, prevention of keloid
scar growth, and prevention of heterotopic ossification.
iv. Regulatory T cells (Treg cells):
The types of radiotherapy are:
l They are formerly known as suppressor T cells, are l External beam radiotherapy (EBRT or XRT)
crucial for the maintenance of immunological tolerance. l Teletherapy, brachytherapy or sealed source radiotherapy
l Their major role is to shut down T cell-mediated im- l Systemic radioisotope therapy or unsealed source
munity toward the end of an immune reaction and to
radiotherapy.
suppress auto-reactive T cells that escaped the process
of negative selection in the thymus. Radiation therapy works by damaging the DNA of cancer-
ous cells. This DNA damage is caused by one of two types
v. Natural killer T cells (NKT cells): of energy, photon or charged particle. This damage is either
l They are a special kind of lymphocyte that bridges the direct or indirect ionizing the atoms which make up the
adaptive immune system with the innate immune system. DNA chain. Indirect ionization happens as a result of the
l Unlike conventional T cells that recognize peptide anti- ionization of water, forming free radicals, notably hydroxyl
gen presented by MHC molecules, NKT cells recognize radicals, which then damage the DNA. Direct damage to
glycolipid antigen presented by a molecule called CD1d. cancer cell DNA occurs through high-LET charged parti-
l Once activated, these cells can perform functions as- cles such as proton, carbon or neon ions which have an
cribed to both Th and Tc cells (i.e. cytokine production antitumour effect which is independent of tumour oxygen
and release of cytolytic/cell killing molecules). supply because these particles act mostly via direct energy
transfer usually causing double-stranded DNA breaks.
Q. 6. Transillumination.
Q. 8. Oral submucous fibrosis.
Ans.
Ans.
l In medicine, transillumination refers to the transmission
of light through tissues of the body. l Oral submucous fibrosis is a chronic disease of the oral
l Organs analysed include the sinuses, the breasts and the cavity characterized by inflammation and progressive
testes. Transillumination is widely used by paediatri- fibrosis of the submucosal tissues (lamina propria and
cians to shine light in bodies of infants and observe the deeper connective tissues).
amount of scattered light. Since their skeleton is not l With progression of this condition, the jaws become
fully calcified, light can easily penetrate tissues. Com- rigid to the point that the sufferer is unable to open his
mon examples of diagnostic applications include: mouth.
l Hydrocephalus: Light penetrates to the inside of the l The condition is linked to oral cancers and is associated
skull of the infant. If there is an excess of CSF, light with areca nut chewing, the main component of betel quid.
is scattered to different parts of the skull, producing
patterns characteristic to hydrocephalus.
l Pneumothorax: Bright light penetrates the thin front
Clinical Features
chest wall and reflects off the back chest wall to in- l Initially, the mucosa feels leathery with palpable fibrotic
dicate the degree of pneumothorax. bands. In the advanced stage, the oral mucosa loses its
l Dentistry: Bright transilluminated light can highlight resiliency and becomes blanched and stiff. Other fea-
dental caries and sign of dental trauma such as tures of the disease include:
enamel infractions. l Pain in the ear or deafness, nasal intonation of voice,

restriction of the movement of the soft palate


Q. 7. Radiation therapy.
l A bud-like shrunken uvula, thinning and stiffening of

Ans. the lips, pigmentation of the oral mucosa.


Section | II General Surgery 409

l Dryness of the mouth and burning sensation l Maintaining proper oral hygiene.
l Decreased mouth opening and tongue protrusion l Supplementing the diet with foods rich in vitamins A, B
complex, and C and iron.
Employing a dental surgeon to round off sharp teeth and
Treatment l

extract third molars.


Biopsy screening is mandatory before treatment and l Surgical treatment is recommended in cases of progres-
treatment includes: sive fibrosis when interincisor distance becomes less
l Abstention from chewing areca nut (also known as betel than 2 cm (0.79 in). Multiple release incisions deep to
nut) and tobacco. mucosa, submucosa and fibrotic tissue and suturing the
l Minimizing consumption of spicy foods, including gap or dehiscence so created by mucosal graft obtained
chillies. from tongue and Z-plasty.

SHORT NOTES
Q. 1. Pain. l Among older adolescents and young adults, the
most common cause of ear infections is Haemophilus
Ans.
influenzae.
l Pain is defined as an unpleasant sensory and emotional
experience associated with actual or potential tissue
Classification
damage, or described in terms of such damage.
l Pain is considered as part of the body’s defense system, l Acute otitis media (AOM)
producing a reflexive retraction from the painful stimulus l Secretory otitis media (SOM)
and tendencies to protect the affected body part while it l Chronic suppurative otitis media (CSOM)
heals as well as avoid that harmful situation in the future.
Clinical Picture
Classification of Pain l Intense pain (due to pressure behind the tympanic mem-
The International Association for the Study of Pain brane). It may result in bullous myringitis, in which the
(IASP) classification: tympanic membrane is inflamed and blistered.
l It classifies pain into five categories based on duration l Discharge (otorrhoea)
and severity, anatomical location, body system l Non-intact tympanic membrane (i.e. perforated eardrum)
involved, cause, and temporal characteristics (intermit-
tent, constant, etc.).
Treatment
l However, this system has been criticized as inadequate
for guiding research and treatment and an additional l Oral and topical analgesics are effective to treat the pain
category based on neurochemical mechanism has also l Antibiotic treatment (amoxicillin)
been proposed. l Insertion of tympanostomy tube (grommet) into the
eardrum in CSOM.
Management of Pain Q. 3. Chemotherapy.
l Acute pain is usually managed with medications such as Ans.
analgesics and anaesthetics.
l Chemotherapy refers to antineoplastic drugs for treat-
l Management of chronic pain is much more difficult and
ment of cancer or the combination of these drugs into a
may require the coordinated efforts of a pain management
cytotoxic standardized treatment regimen.
team, including medical practitioners, clinical psycholo-
l In its non-oncological use, the term may also refer to
gists, physiotherapists, occupational therapists, and nurse.
antibiotics (antibacterial chemotherapy).
Q. 2. Otitis media. l The majority of chemotherapeutic drugs can be divided
into alkylating agents, antimetabolites, anthracyclines,
Ans.
plant alkaloids, topoisomerase inhibitors, and other
l Otitis media is inflammation of the middle ear. antitumour agents.
l Otitis media is most commonly caused by infection with l Side effects of chemotherapy include: Immunosuppres-
viral, bacterial, or fungal pathogens. The most common sion and myelosuppression, fatigue due to mild to severe
bacterial pathogen is Streptococcus pneumoniae. anaemia. Gastrointestinal distress, nausea, vomiting,
410 Quick Review Series for BDS 3rd Year

diarrhoea or constipation, apolecia and damage to spe- l Although oxidation reactions are crucial for life, they
cific organs leading to cardiotoxicity, hepatotoxicity, can also be damaging; hence, plants and animals main-
nephrotoxicity, ototoxicity, etc. tain complex systems of multiple types of antioxidants,
l Less common side effects include pain, erythema, dry such as glutathione, vitamin C, and vitamin E as well as
skin, damaged fingernails, xerostomia, oedema, sexual enzymes such as catalase, superoxide dismutase and
impotence. various peroxidases.
l Antioxidants are widely used as ingredients in dietary
Q. 4. Penicillin.
supplements and have been investigated for the preven-
Ans. tion of diseases such as cancer, coronary heart disease
and even altitude sickness.
l The discovery of penicillin is attributed to Scottish sci-
l In addition to these uses of natural antioxidants in
entist and Nobel laureate Alexander Fleming in 1928.
medicine, these compounds have many industrial uses,
l Penicillin is a group of antibiotics derived from
such as preservatives in food and cosmetics and
Penicillium fungi. They include penicillin G, procaine
preventing the degradation of rubber and gasoline.
penicillin, benzathine penicillin, and penicillin V.
l Penicillins are the first drugs that were effective against Q. 6. Local anaesthesia.
many previously serious diseases such as syphilis and
Staphylococcus infections. Ans.
l All penicillins are beta-lactam antibiotics and are used
l Local anaesthesia is any technique to render part
in the treatment of bacterial infections caused by
of the body insensitive to pain without affecting
susceptible, usually Gram-positive organisms.
consciousness.
l Penicillin acts by inhibiting the formation of peptido-
l It allows patients to undergo surgical and dental proce-
glycan cross-links in the bacterial cell wall, in other
dures with reduced pain and distress.
words, the antibiotic causes cytolysis or death due to
l Local anaesthetic can block almost every nerve between
osmotic pressure.
the peripheral nerve endings and the central nervous
Q. 5. Antioxidants. system.
l The most peripheral technique is topical anaesthesia
Ans.
to the skin or other body surface. Small and large
l An antioxidant is a molecule capable of inhibiting the peripheral nerves can be anaesthetized individually
oxidation of other molecules. (peripheral nerve block) or in anatomic nerve bundles
l Antioxidants terminate these chain reactions by (plexus anaesthesia).
removing free radical intermediates, and inhibit other l Clinical techniques include: Surface anaesthesia, infil-
oxidation reactions. tration anaesthesia, field block, peripheral nerve block,
l They do this by being oxidized themselves, so antioxi- plexus anaesthesia, epidural anaesthesia, spinal anaes-
dants are often reducing agents such as thiols, ascorbic thesia, intravenous regional anaesthesia (Bier’s block)
acid or polyphenols. and local anaesthesia of body cavities.
Section | II General Surgery 411

Key Points to Remember

GENERAL PRINCIPLES OF OPERATIVE SURGERY, STERILIZATION/ASEPSIS,


DIATHERMY, CRYOSURGERY AND LASERS
‘Sterilization’ is defined as the process by which an article, surface or medium is freed of all microorgan-
isms either in the vegetative or spore state.
‘Disinfection’ is defined as the process of destruction or removal of all pathogenic microorganisms or
organisms giving rise to infection but not necessarily their spore forms.
‘Antisepsis’ indicates the prevention of infection by inhibiting the growth of bacteria in wounds or tissues.

Methods of Sterilization
(a) Dry heat sterilization
(b) Moist heat sterilization

a. Dry heat method includes:


Hot air oven
Time and temperature: Usually 160°C for 2 h or 120°C for 6 h is widely used for sterilization of cutting
instruments.

b. Moist heat method includes:


Autoclave
Time and temperature: Usually temperature of 12°C at 15 lb of pressure is recommended for 15 minutes or
134°C at 30 lb of pressure is recommended for 3 minutes.

Other methods include:


(a) Chemicals
Agents such as 70% alcohol, benzalkonium chloride or hexachlorophene are used to kill vegetative
bacteria.
(b) Radiation
X-rays, gamma rays, UV light are used to kill microorganisms.

WOUNDS, SINUS, FISTULAE AND ULCERS


Types of Wounds
a. Incised wounds
b. Lacerated wounds
c. Penetrating wounds
d. Crushed wounds

Components of wound healing


Wound healing takes place through four steps:
a. Inflammation
b. Wound contraction
c. Epithelialization
d. Granulation tissue formation
412 Quick Review Series for BDS 3rd Year

Factors influencing wound healing:

General factors Local factors


Age Position of skin wound
Nutrition Blood supply
Hormones Tension
Anaemia Infection
Uraemia Movement
Jaundice Exposure to ionizing radiation
Diabetes Foreign bodies
Blood dyscrasias Adhesions to bony surfaces
Malignant disease Necrosis

Cytotoxic drugs Lymph drainage

Sinus
Sinus is a blind track leading from the surface down to the tissues. The sinus is lined by granulation tissue
which may be epithelialized.

Fistula
Fistulas are communicating track between two epithelial surfaces commonly between hollow viscus and the
skin (external fistula) or between two hollow viscera (internal fistula).

Examples of fistulas
Some diseases or conditions where multiple fistulas and sinuses are seen:
l ‘Watering can’ perineum—multiple perianal fistulas
l Crohn disease—multiple anal fistulas
l Actinomycosis—multiple sinuses
l Ulcerative colitis—multiple fistulae

Ulcer
An ulcer is defined as a discontinuity of the skin or mucous membrane.
Types of ulcers:
(i) Meleney’s ulcer
(ii) Marjolin’s ulcer
(iii) Bazin’s ulcer
(iv) Martorell’s ulcer
(v) Painful ulcer
(vi) Painless ulcer
Various types of edges of an ulcer are as follows:
l Undermined edge
l Punched-out edge
l Sloping edge
l Raised and pearly white-beaded edge
l Rolled edge (everted edge)
Section | II General Surgery 413

BURNS, SKIN GRAFTING AND FLAPS


Burns
Burns can be classified into two types:
(a) Partial thickness burns
(b) Full thickness burns
Rule of 9 (rule of Wallace)
It helps to assess the degree of burns.
Burns of head and neck 9%
Burns of upper limbs 9 3 2 5 18%
Burns of thorax (front and back) 9 3 2 5 18%
Burns of abdomen (front and back) 9 3 2 5 18%
Burns of lower limbs (front and back) 18 3 2 5 36%
Burns of external genitalia 1%

Total body surface area covered 100%

Types of burn:
(i) First degree burn
(ii) Second degree burn
(iii) Third degree burn

Skin Grafting
Different types of grafts used are as follows:
(a) Cancellous graft (for osteogenesis) iliac crest
(b) Cortical graft (for rigid fixation) fibula, tibia, etc.
(c) Thiersch graft—partial thickness graft
(d) Wolfe graft—full thickness graft

HAEMORRHAGE AND SHOCK


Haemorrhage
Classification of haemorrhage depending upon the nature of vessels:
a. Arterial haemorrhage
b. Venous haemorrhage
c. Capillary haemorrhage
Classification of haemorrhage depending upon the timing of haemorrhage:
a. Primary haemorrhage
b. Secondary haemorrhage
c. Reactionary haemorrhage
Classification of haemorrhage depending upon the duration of haemorrhage:
a. Acute haemorrhage.
b. Chronic haemorrhage
414 Quick Review Series for BDS 3rd Year

Shock
Shock is defined as a condition in which circulation fails to meet the nutritional needs of the cells and at the
same time fails to remove the metabolic waste products.
Types of shock are as follows:
a. Haematogenic/hypovolaemic shock
b. Traumatic shock
c. Neurogenic shock
d. Cardiogenic shock
e. Septic shock
f. Miscellaneous (e.g. anaphylactic shock)

BACTERIAL INFECTIONS AND TRANSMISSIBLE VIRAL INFECTIONS


Actinomycosis
l It is caused by Actinomyces israelii.
l Its types include:
(i) Cervicofacial
(ii) Abdominal
(iii) Pulmonary
l Cervicofacial actinomycosis is common in patients with poor oral hygiene, bad caries tooth.
l Treatment include injection of crystalline penicillin—10 lakh units once a day for 6–12 months.
l Actinomycosis of the right iliac fossa may need right hemicolectomy.

Leprosy
l It is also known as Hansen disease.
l It is caused by Mycobacterium leprae, an acid-fast bacillus.
l Nasal secretion is the main source for infection.
l Main causes for the spread are:
(i) Poverty
(ii) Poor oral hygiene
(iii) Overcrowding
l Types of leprosy include:
(i) Tuberculoid leprosy
(ii) Lepromatous leprosy
(iii) Borderline leprosy
l Treatment includes:
(i) For lepromatous and borderline lepromatous leprosy
Dapsone 100 mg/dL, clofazimine 50 mg/dL, rifampicin 600 mg, for a minimum of 2 years
(ii) For tuberculoid
Dapsone 100 mg daily, rifampicin 600 mg, for a period of 6 months

Syphilis
l It is also known as French disease.
l It is a sexually transmitted disease caused by Treponema pallidum.
l Clinical presentation includes:
(i) Early syphilis
(ii) Late syphilis or tertiary syphilis
l Early syphilis is again divided into:
(i) Primary syphilis
Section | II General Surgery 415

(ii) Secondary syphilis


(iii) Latent syphilis
l Chancre is seen in early or primary syphilis.
l Chancre is shallow, indurated, painless, ulcer called as hunterian chancre.
l Secondary syphilis:
(a) It appears after 6–12 weeks of spirochaetaemia.
(b) It is characterized by snail track ulcers and mucous patches.
l In case of congenital syphilis, clinically Hutchinson’s triad is seen, which includes:
(i) Interstitial keratitis
(ii) 8th nerve deafness
(iii) Hutchinson’s teeth
l Tertiary syphilis:
(i) This stage develops after 5–15 years of primary syphilis.
(ii) It causes neurosyphilis and cardiovascular syphilis.
(iii) Characteristic lesion is ‘gumma’.

TUMOURS, CYSTS AND NECK SWELLINGS


Cysts
Congenital cysts include:
a) Thyroglossal cyst
b) Branchial cyst
c) Epidermoid cyst
d) Dermoid cyst
Acquired cysts include:
a) Retention cyst of mouth
b) Hydatid cyst
c) Haematogenous cyst

Thyroglossal cyst
It arises from thyroglossal tract/duct, which extends from foramen caecum at the base of the tongue to the
isthmus of the thyroid gland.

Dermoid cyst
It is lined by squamous epithelium containing desquamated cells.
The contents are thick and toothpaste-like.

Branchial cyst
It arises from the vestigial remnants of 2nd branchial arch.
It is located in the anterior triangle of the neck partly under cover of the upper one-third of anterior border
of sternomastoid.

Tumours
(i) Most common bone tumour in children—osteosarcoma.
(ii) Most common primary bone tumour—osteosarcoma.
Some common tumours include:
(a) Desmoid tumour: A kind of fibroma, which arises from deeper parts of rectus sheath
(b) Brown’s tumour: Osteitis fibrosa cystica (hyperparathyroidism)
(c) Cock’s peculiar tumour: Ulcerated multiple sebaceous cysts of scalp
416 Quick Review Series for BDS 3rd Year

(d) Giant cell tumour: Osteomyelitis of cranial bones


(e) Pott’s puffy tumour: Oedema of scalp in osteomyelitis of cranial bones
(f) Grawitz tumour: Hypernephroma (adults)
(g) Wilms’ tumour: Nephroblastoma (children)
(h) Potato tumour or carotid body tumour: Berry tumour or Hutchinson’s tumour
(i) Ubiquitous tumour: Lipoma or universal tumour

DISEASES OF THE ORAL CAVITY (MOUTH, TONGUE AND LIPS)


l Oral cancer is the commonest malignant neoplasm in the head and neck.
l Leukoplakia is the premalignant condition for oral cancer.
l Stages in the development of leukoplakia:
(a) Keratosis
(b) Acanthosis
(c) Dyskeratosis
(d) Speckled leukoplakia
(e) Carcinoma in situ
l Carcinoma of the cheek is the most common malignant neoplasm in India because of habit of keeping
the tobacco quid in the cheek pouch.
l Carcinoma of cheek has three pathological types:
(i) Nonhealing ulcer
(ii) Exophytic growth
(iii) Infiltrative lesion
l Pathological types of carcinoma of tongue are:
(i) Nonhealing ulcer
(ii) Proliferative growth with everted edge
(iii) Frozen tongue or indurated variety
l Carcinoma of lip is seen in individuals who are constantly exposed to sunlight, so it is also called
countryman’s lip.
l Radiotherapy are of two types:
(i) External radiotherapy
(ii) Interstitial radiotherapy
l The dose for radiotherapy—6500–7500 cGy is required to eradicate squamous cell carcinoma of head and neck.

SALIVARY GLANDS
l The commonest salivary gland malignancy—adenoid cystic carcinoma
l The commonest salivary tumour seen in children—mucoepidermoid carcinoma
l Some causes of salivary gland enlargement are:
(i) Sjogren syndrome
(ii) Mikulicz disease
(iii) Drugs
l Parotid fistula: It is a condition that commonly occurs after surgery on the parotid gland.
Section | II General Surgery 417

INFECTIONS AND DISEASES OF THE LARYNX AND NASOPHARYNX


Diseases of the Larynx
Carcinoma larynx
(i) Common in age group—40–70 years
(ii) It is classified as supraglottic, glottic, subglottic
(iii) Male–female ratio—10:1
(iv) Carcinoma supraglottis is commonly seen in epiglottis, false cords and aryepiglottic folds.
(v) Common site for carcinoma glottis is anterior and middle one-third on its free edge and upper surface.
(vi) Treatment includes:
(a) Radiotherapy
(b) Surgery

DISEASES OF ARTERIES, VEINS AND THE LYMPHATIC SYSTEM


Diseases of Vein
Varicose veins and deep vein thrombosis
l Varicose vein is defined as dilated, tortuous and elongated superficial veins of the limb
l Types of varicose vein are:
(i) Primary varicose veins
(ii) Secondary varicose veins
l Primary varicose vein occurs as a result of congenital weakness in the vessel wall.
l Complications of varicose vein include:
(i) Eczema and dermatitis
(ii) Lipodermatosclerosis
(iii) Haemorrhage
(iv) Thrombophlebitis
(v) Venous ulcer
(vi) Periostitis of the tibia
(vii) Marjolin’s ulcer
l Deep vein thrombosis is also known as phlebothrombosis.
l Causes of deep vein thrombosis are:
(i) Trauma
(ii) Hormones
(iii) Road traffic accidents
(iv) Operations
(v) Malignancy
(vi) Blood disorder
(vii) Orthopaedic surgery, obesity
(viii) Serious illness
(ix) Immobilization
(x) Splenectomy
418 Quick Review Series for BDS 3rd Year

l Tests done in case of deep vein thrombosis:


(a) Homan’s test
(b) Moses test
l Complications of deep vein thrombosis:
(a) Permanent oedema of the limb
(b) Pulmonary embolism
(c) Secondary varicosity and nonhealing ulcer

Diseases of Arteries
(i) Acute arterial occlusion
(ii) Peripheral aneurysms
l Acute arterial occlusion—causes:
(a) Mural thrombosis following a myocardial infarction
(b) Mural thrombosis due to mitral stenosis
(c) Atheromatous plaque
(d) Aneurysm
(e) Valve vegetation
l Symptoms are seen in the form of five Ps:
(a) Pain
(b) Pallor
(c) Paresis
(d) Pulselessness
(e) Paraesthesia
l Peripheral aneurysms affect the popliteal artery, femoral artery, iliac artery, etc.
l Mainly seen in elderly patients
l Complications are:
(a) Thrombosis
(b) Embolization
(c) Compression on the popliteal vein
l Treatment includes:
(a) Proximal and distal ligation of the artery

Disease of Lymphatic System


(i) Lymphoedema of leg,
(ii) Hodgkin’s lymphoma,
(iii) Non-Hodgkin’s lymphoma
(iv) Burkitt’s lymphoma
l In case of lymphoedema of leg, there is enlargement of limb due to accumulation of lymph in the
subcutaneous tissues.
l Common sites for lymphoedema are lower and upper limbs.
l Hodgkin’s lymphoma is a malignant neoplasm of lymphoreticular system.
l It is mainly seen in children and middle-aged people.
Burkitt’s lymphoma:
l Burkitt’s lymphoma is a type of high-grade non-Hodgkin’s lymphoma.
l It is caused by Epstein–Barr virus.
l Types:
(a) Endemic
(b) Sporadic
l Treatment:
(a) Endemic cases respond well to cyclophosphamide.
(b) Sporadic cases should be treated with combination chemotherapy.
Section | II General Surgery 419

NERVOUS SYSTEM
Nerve injuries are classified as:
(i) Neuropraxia
(ii) Axonotmesis
(iii) Neurotmesis
(i) Neuropraxia:
Here axons are intact, spontaneous recovery is complete.
(ii) Axonotmesis:
Here axons are divided, connective tissue is intact; Wallerian degeneration occurs and axons then
regenerate slowly.
(iii) Neurotmesis:
Whole nerve severed, recovery occurs if cut ends are apposed.

FRACTURES—GENERAL PRINCIPLES
l Fracture is defined as a structural break in the normal continuity of bone.
l Dislocation is defined as a total disruption of a joint with no remaining contact between articular sur-
faces.
l Subluxation is defined as a disruption of a joint with some contact between articular surfaces.
l Types of fracture:
(i) Closed or simple fracture
(ii) Open or compound fracture
l Fractures of mandible include:
(i) Dentoalveolar fracture
(ii) Condylar fracture
(iii) Fracture of ramus/angle of mandible
(iv) Fracture of symphysis of mandible
l Fracture of middle third includes:
(i) Nasoethmoidal complex fracture
(ii) Zygomatic complex fracture
l Le Fort I fracture is also known as Guerin’s fracture, floating maxilla.
l Le Fort II fracture is also known as pyramidal fracture.
l Le Fort III fracture is also known as craniofacial disjunction, and is a high-level fracture or transverse
fracture.
l Management of mandibular fracture includes:
(i) Closed reduction and indirect skeletal fixation
(ii) Open reduction and direct skeletal fixation
l Management of middle third fracture includes:
(i) Direct/internal fixation of maxilla
(ii) External fixation of maxilla
(iii) Internal suspension
l Stages of healing of a fracture:
(i) Stage of haematoma formation
(ii) Cellular stage
(iii) Stage of callus formation
(iv) Stage of new bone formation
(v) Stage of remodelling
l Complications of maxillofacial injuries are:
(i) Anaesthesia
(ii) Malunion and deformities
420 Quick Review Series for BDS 3rd Year

(iii) Infection
(iv) Superior orbital fissure syndrome
(v) Nonunion and delayed union
(vi) Derangement of the occlusion
(vii) Ankylosis of temporomandibular joint

ANOMALIES OF DEVELOPMENT OF FACE (CLEFT LIP AND CLEFT PALATE)


l Cleft lip results from an abnormal development of the median nasal and maxillary process.
l Types of cleft lip are:
(i) Central
(ii) Lateral
(iii) Complete or incomplete
(iv) Simple or compound
l Cleft palate results from a failure of fusion of the two palatine processes.
l Types of cleft palate are:
(i) Complete
(ii) Incomplete
l Cleft lip operated at around 3 months of age.
l In repair of cleft lip and anterior palate ‘RULE OF 10’ is followed. The rule implies that at the time
of repair, haemoglobin should be more than 10 gm%, age approximately 10 weeks, weight more than
10 lb and total leucocyte count should be less than 10,000 cells/cumm.
l Common methods employed for unilateral cleft lip repair are:
i) Millards rotation advancement flap
ii) Tennison-Randall triangular flap
l Cleft palate is operated at at the age of 1.5 to 2 years.

DISEASES OF THYROID AND PARATHYROID GLANDS


Thyroid Gland
l Thyroid gland is present in the neck, enclosed by pretracheal fascia, which is a part of deep cervical
fascia.
l Goitre—it is a diffused enlargement of the thyroid gland.
l Multinodular goitre is the end result of diffuse hyperplastic goitre.
l Multinodular goitre is common in females.
l Toxic multinodular goitre is also called as Plummer disease.
l The most common site of a nodule is at the junction of isthmus with one lobe.
l Retrosternal goitre is a multinodular goitre, which develops in the neck and is slowly pulled down
behind the sternum.
l Clinical types of retrosternal goitre are:
(i) Substernal
(ii) Intrathoracic
(iii) Plunging goitre
l Pemberton’s sign is seen in retrosternal goitre.

Parathyroid Gland
l Parathyroid glands are the endocrinal glands situated in the neck, which secrete the hormone parathormone.
l Hyperparathyroidism is a condition that occurs due to an increased activity of parathyroids and
manifests as hypercalcaemia.
Section | II General Surgery 421

l Hyperparathyroidism is more commonly seen in females.


l Types of hyperparathyroidism are:
(i) Primary hyperparathyroidism
(ii) Secondary hyperparathyroidism
(iii) Tertiary hyperparathyroidism
l Most common presentation is asymptomatic hypercalcaemia and renal stones.
l Treatment of hyperparathyroidism includes surgery of parathyroid gland.

BIOPSY, LOCAL ANAESTHESIA AND MISCELLANEOUS


l Biopsy includes:
(i) Excisional biopsy
(ii) Incisional biopsy
(iii) Punch biopsy
l Total excision of a small lesion for microscopic study is called excisional biopsy.
l Excisional biopsy is done is solitary nodule of thyroid and parotid gland tumours.
l When a small section is removed for examination, it is called as incisional biopsy.
l Incisional biopsy is contraindicated in salivary gland tumours except in minor salivary gland tumours.
l Injectable anaesthetics include:
(i) Low potency—procaine, chloroprocaine
(ii) Medium potency—lidocaine, prilocaine
(iii) High potency—tetracaine, bupivacaine, ropivacaine
l Surface anaesthetics include:
(i) Soluble—cocaine, lidocaine
(ii) Insoluble—benzocaine
l In case of cleft palate, antrostomy, Caldwell Luc operation: General anaesthesia is indicated.
This page intentionally left blank
Section III

Oral Pathology

Topic 1 Developmental Disturbances of Oral and Paraoral Structures 425


Topic 2 Benign and Malignant Tumours of the Oral Cavity 439
Topic 3 Tumours of Salivary Glands 456
Topic 4 Cysts and Tumours of Odotogenic Origin 468
Topic 5 Bacterial Infections of the Oral Cavity 484
Topic 6 Viral Infections of the Oral Cavity 495
Topic 7 Mycotic Infections of the Oral Cavity 503
Topic 8 Diseases of the Periodontium 508
Topic 9 Dental Caries 516
Topic 10 Diseases of the Pulp and Periapical Tissues 534
Topic 11 Spread of Oral Infections 543
Topic 12 Physical and Chemical Injuries of the Oral Cavity 545
Topic 13 Regressive Alterations of the Teeth 551
Topic 14 Healing of Oral Wounds 560
Topic 15 Oral Aspects of Metabolic Disease 567
Topic 16 Allergic and Immunologic Diseases of the Oral Cavity 574
Topic 17 Diseases of Bone and Joints 578
Topic 18 Diseases of Blood and Blood Forming Organs 592
Topic 19 Diseases of the Skin 603
Topic 20 Diseases of the Nerves and Muscles 613
Topic 21 Forensic Odontology 617
This page intentionally left blank
Section III

Oral Pathology

Topic 1

Developmental Disturbances of Oral


and Paraoral Structures

LONG ESSAYS
Q. 1. Enumerate and describe briefly developmental i.a. Gemination
disturbances affecting the shape of teeth. l Gemination is a developmental anomaly which refers
Ans. to division of single tooth germ into incomplete or
complete formation of two teeth.
The developmental disturbances affecting shape of teeth are l Crowns may be partially or totally separated from each other.
as follows: l Roots are fused and single root canal is present within
i. Gemination, fusion and concrescence the root.
ii. Accessory cusps l The structure is usually one with two completely or
l Cusp of Carabelli incompletely separated crowns that have a single root
l Talon cusp and a root canal.
l Dens invaginatus l The condition is seen in both deciduous and permanent
l Dens evaginatus dentitions, with a higher frequency in the anterior and
iii. Ectopic enamel maxillary region.
l Enamel pearls

l Cervical enamel extensions i.b. Fusion


iv. Taurodontism l Fusion is defined as single enlarged tooth or joined
v. Dilaceration tooth in which the tooth count reveals a missing tooth
vi. Supernumerary roots. when the anamolous tooth is counted as one.

425
426 Quick Review Series for BDS 3rd Year

l One of the most important criteria for fusion is the fused l Teeth most frequently involved are maxillary lateral
tooth must exhibit confluent dentin. and maxillary central incisors.
l Both permanent and deciduous dentitions are affected in l Radiographic features: Appearance of tooth within tooth
case of fusion, although it is more common in decidu- due to deep pear-shaped invagination from lingual pits,
ous teeth. approximating to pulp.
l Fusion can be complete or incomplete and its extent will l Treatment: This anomaly makes teeth prone to caries so
depend on stage of odontogenesis at which fusion takes endodontic therapy should be done. Restoratory proce-
place. dures are unsuccessful because of this deep invagina-
l The incisor teeth are more frequently affected in both tion, that generally approximates pulp.
the dentitions.
d. Dens evaginatus (occlusal tuberculated
i.c. Concrescence premolar, Leong’s premolar, evaginated odontome,)
l Concrescence is defined as union of two adjacent teeth l Dens evaginatus is a developmental condition appears
by cementum only without confluence of the underlying as an accessory cusp or globule of enamel on occlusal
dentin. surface between buccal and lingual cusps of premolars
l It is the type of fusion which is limited only to the roots unilaterally or bilaterally.
of teeth and it occurs after the root formation of l This is opposite of invagination. That means, there
involved teeth is completed. occurs extrusion of the dental papilla outwards into the
l Aetiology: Concrescence may be developmental or enamel organ.
post-inflammatory or due to traumatic injury. l Clinical findings:
l Common between maxillary 2nd molar and unerupted l This condition is more common in people of Chinese
3rd molar. race.
l More common in maxillary 1st premolars but also
ii. Accessory Cusps occurs rarely on molars, cuspids and incisors.
l Presents a tubercle of enamel with a core of dentin
a. Cusp of Carabelli
with a narrow pulp chamber.
b. Talon’s cusp
l When the tooth erupts, this bit of enamel is higher
c. Dens invaginatus (dens in dente)
than the cusps, and covers the underlying mass of
d. Dens evaginatus
dentin.
l If present in deciduous teeth, it causes difficulty in
a. Cusp of Carabelli
feeding.
l Present on mesiopalatal cusp of maxillary 1st molars.
l When the thin surface enamel of the tubercle breaks
l An analogous accessory cusp is seen occasionally on
down, infection of the tooth takes place resulting in
the mesiobuccal cusp of a mandibular permanent or
death of the pulp and abscess formation.
deciduous molar known as protostylid.
l Treatment consists of extraction of the tooth.
b. Talon’s cusp
l Talon’s cusp is an anomalous projection resembling iii. Ectopic Enamel
eagle’s talon projects lingually from cingulum area of l Enamel pearls
permanent incisors. l Cervical enamel extensions
l A developmental groove is present at the site, where this l Ectopic enamel or enamel pearls or enameloma or
projection meets with the lingual surface of tooth. enamel drop usually occurs in furcation area below
l This groove is prone to caries, so it should be removed. If the crest of gingiva.
pulp exposure is present, then endodontic therapy is done. l Cervical enamel extension also occurs along the
l Found in association with “Rubinstein-Taybi syndrome”. surface of dental roots.
l Maxillary and mandibular molars are most com-
c. Dens in dente or dens invaginatus monly affected.
l Dens in dente is a developmental variation which arises l Predisposes to development of buccal bifurcation cysts.
as a result of enamel epithelial invagination of the
crown surface before calcification.
l Several causes of this condition are: It is because of iv. Taurodontism (Bull-like Teeth)
focal growth proliferation and focal growth retardation l Taurodontism is a dental anomaly in which the body of
that takes place in certain areas of tooth bud, increased the tooth is enlarged at the expense of roots.
localized external pressure. l Aetiology:
l After calcification, it appears as accentuation of lingual pit. l A specialized or retrograde character
Section | III Oral Pathology 427

l A primitive pattern l It is of two types:


l An atavistic feature a) Hereditary (also called as amelogenesis imperfecta).
l Mendelian recessive trait b) Environmental.
l A mutation

l Associated with Klinefelter’s syndrome. A) AMELOGENESIS IMPERFECTA


l It is due to failure of Hertwig’s epithelial root sheath

to invaginate at proper horizontal level. l It is a group of hereditary defects of enamel associated


l Clinical findings: with other generalized defects, dentin is usually normal.
l It may affect both deciduous and permanent denti-
l It may be of three types:
tions, but more common in permanent dentition. a) Hypoplastic type: It is the defect of enamel organic
l Molars are commonly affected.
matrix formation.
l Tooth morphology is normal.
b) Hypocalcification type: It is the defect of mineraliza-
l Radiographic features: tion of enamel.
l Enlarged and rectangular pulp chamber is present.
c) Hypomaturation type: It is the defect of enamel crys-
l No constriction of pulp at cervical area.
tal maturation.
l Roots are very short.

l Furcation is present just above root apex. Clinical Features


l Treatment: No treatment is required.
l Hypoplastic type: Enamel thickness is not complete.
l Hypocalcification type: Enamel is soft and can be
v. Dilaceration removed by prophylactic instrument.
l Hypomaturative type: Enamel can be pierced with ex-
l Dilaceration refers to angulation or curve in root or
plorer point and chipped off.
crown of tooth.
l Teeth are brownish in colour. Vertical lines or grooves
l Angulation is caused due to trauma to the tooth during
may be present on surface.
formative stage of tooth.
l Enamel is chalky and it can be chipped off with expo-
l Curve is present at apical, middle or at cervical portion
sure of underlying dentin.
depending on the portion which is forming at the time
l Contact points are abraded.
of trauma.
l Treatment: There is no treatment except for improvement
l Occlusal trauma in deciduous tooth may also cause
of cosmetic appearance by veneering or capping of teeth.
dilaceration of permanent tooth.
l More common in the maxillary anterior region.
l Significance-tooth with bent root is difficult to extract. B) ENVIRONMENTAL ENAMEL HYPOPLASIA
l In this type of enamel hypoplasia, both enamel and
vi. Supernumerary Roots dentin are affected.
l One or more extra roots may be present in tooth.
l Usually single-rooted teeth such as mandibular cuspids Causes
and bicuspids are involved. l Nutritional deficiencies.
l Third molars of both jaws also present one or more l Exanthematous diseases.
extra roots. l Congenital syphilis.
Q. 2. Enumerate the various causes of enamel hypopla- l Hypocalcaemia.
sia and describe hypoplasia of teeth associated with l Birth injuries.
fluorosis. l Ingestion of fluoride.
l Local infection.
Or l Idiopathic
Describe enamel hypoplasia in detail. l Nutritional deficiencies: Vitamin A, C and D deficien-
cies and exanthematous diseases (e.g. chickenpox, mea-
Or sles) and scarlet fever cause enamel hypoplasia of pit-
Classify enamel hypoplasia. Describe the pathogenesis ting type, if deficiency occurs during teeth formation.
and clinical features of enamel hypoplasia due to con- l Enamel-hypoplasia due to congenital syphilis: In con-
genital syphilis. genital syphilis, crowns of maxillary central incisors
become screwdriver-shaped. Crown of first molar at oc-
Ans. clusal surface arranged into agglomerated mass of
l Enamel hypoplasia is defined as an incomplete or defec- globuel and termed as mulberry molars. The teeth
tive formation of the organic enamel matrix of teeth. affected are called as Hutchinson’s teeth.
428 Quick Review Series for BDS 3rd Year

l Enamel hypoplasia due to hypocalcaemia: Calcium level l Dentin dysplasia is otherwise known as rootless teeth.
less than 6 to 8 mg/dl may cause enamel hypoplasia of l It is an autosomal dominant disturbance.
pitting type. l Sields and his associates have divided it into:
l Enamel hypoplasia due to birth injuries: Premature chil- a. Radicular dentin dysplasia (type I)
dren, affected with Rh haemolytic diseases at birth, may b. Coronal dentin dysplasia (type II)
suffer from enamel hypoplasia.
l Enamel hypoplasia due to ingestion of fluoride: If Type I (radicular dentin dysplasia/rootless
drinking water contains fluoride content more than teeth)
1 p.p.m. at the time amelogenesis, it can cause death of Clinical features:
ameloblasts so it leads to defective matrix formation.
l It may also result in disturbances in calcification. l Both dentitions are affected, teeth appear normal in
Enamel affected is termed as mottled enamel. morphologic appearance and colour. Sometimes there
l In mild cases, white flecks appear in enamel. Moderate may be amber translucency.
degree of fluoride toxicity may lead to white opaque l Teeth exhibit normal eruption pattern while delayed
areas in enamel. In more severe case, brown discoloura- eruption is reported in few cases.
tion of enamel occurs. l Roots are so short that teeth characteristically exhibit
l Still more severe toxicity of fluoride causes corroded abnormal mobility and premature exfoliations after only
crown surface. a minor trauma.
Radiographic features:
Treatment l In both dentitions, the roots are short, blunt and conical
l Defluoridation of drinking water is done, if drinking or similarly malformed.
water contains excess fluoride content. l Roots of primary teeth may be thin spicules.
l The staining of enamel surface can be removed by l In deciduous teeth, pulp chambers and root canals are
bleaching with hydrogen peroxide (30% solution), completely obliterated while a crescent or chevron-
grinding or capping. shaped pulp chambers are seen in permanent teeth.
l Enamel hypoplasia due to local infection or trauma: l Periapical radiolucencies representing granulomas,
l Only single tooth, generally maxillary incisor or cysts or abscesses associated with apparently otherwise
premolar, is affected. intact teeth are a characteristic feature.
l The apically infected deciduous tooth may affect the
Histologic features:
ameloblastic layer of permanent tooth.
l A portion of coronal dentin is normal.
l Ameloblastic layer may also get disturbed due to
l Radicular dentin is tubular dentin.
occlusal trauma to deciduous tooth.
l Pulp is obliterated with dentin formation in cascade
l The permanent tooth beneath infected or traumatized
pattern known as “lava flowing around boulders”.
deciduous tooth is discoloured or pitted.
l The teeth are called Turner’s teeth and this type of

enamel hypoplasia is termed as Turner’s hypoplasia. Type II (coronal dentin dysplasia)


Clinical features:
Q. 3. Enumerate the developmental disturbances in the l Both dentitions are affected.
formation of dentin. Write in detail the clinical types, l Deciduous teeth resemble dentinogenesis imperfecta,
clinical features, radiological features and histopatho- they appear yellowish brown or bluish grey.
logical features of dentin dysplasia. l The clinical appearance of permanent teeth is normal.
Ans. Radiographic features:
Developmental disturbances in the formation of dentin are l Obliterated pulp chamber in deciduous teeth is seen.
as follows: l Abnormally large pulp chambers in coronal portion
i. Dentinogenesis imperfecta of permanent teeth are described as “thistle-tube” in
ii. Dentin dysplasia shape.
iii. Regional odontodysplasia l Permanent teeth exhibit pulp stones.
iv. Dentin hypocalcification. l No periapical radiolucencies are seen.
Histological features:
Dentin dysplasia l In deciduous teeth, coronal dentin is relatively normal
l Rare disturbance of dentin formation, characterized by while radicular dentin is amorphous and atubular.
normal enamel and atypical dentin with normal pulpal l Permanent teeth shows relatively normal coronal dentin
morphology. but pulp contains multiple pulp stones or denticles.
Section | III Oral Pathology 429

Q. 4. List the developmental abnormalities of teeth with roots and obliterated pulp canals and chambers in
respect to its structure. Give a detailed account on den- Type I DI.
tinogenesis imperfecta.
Histological features:
Ans. l Enamel is normal while in dentin irregular tubules are
seen with large areas of uncalcified matrix.
Developmental abnormalities of teeth with respect to its
l Pulp chamber is obliterated by continuous deposition of
structure are as follows:
dentin with entrapped ‘odontoblasts’.
i. Amelogenesis imperfecta
l Dentinoenamel junction is without scalloping, so
ii. Enamel hypoplasia.
occlusal surface is lost due to attrition.
iii. Dentinogenesis imperfecta
iv. Dentin dysplasia Dentinogenesis imperfecta II (brandywine type
v. Regional odontodysplasia
dentinogenesis imperfecta)
vi. Dentin hypocalcification
This disorder was found in Brandywine triracial isolate in
southern Maryland.
Dentinogenesis Imperfecta This corresponds to dentinogenesis imperfecta type III of
l Dentinogenesis imperfecta is an autosomal dominant Shields classification.
condition. Clinical features:
l It is defect of collagen formation hence only dentin is l The crowns of the deciduous and permanent teeth wear
affected rather than enamel. rapidly after eruption and multiple pulp exposures are
seen in primary teeth.
Dentinogenesis imperfecta revised classification l Colour of teeth has unusual transluscent/opalescent hue.
Dentinogenesis imperfecta I: Dentinogenesis imperfecta l Enamel lost earlier due to fracture; defective DEJ and
without osteogenesis imperfecta (opalscent dentin), this dentin is worn off.
corresponds to dentinogenesis imperfecta type II of Shields
Radiographic features:
classification.
l Deciduous dentition shows ‘shell teeth’ appearance due
Dentinogenesis imperfecta II (brandywine type dentino-
to enlarged pulp chambers and root canals. The perma-
genesis imperfecta): This corresponds to dentinogenesis
nent teeth have pulpal spaces that are either smaller than
imperfecta type III of Shields classification.
normal or completely obliterated.
There is no substitute for then category designated as DI
Histological features:
type I of the previous classification (Shields).
Histopathology of teeth in Shields type III has not been
Dentinogenesis imperfecta I adequately documented.
Clinical features: Treatment for both Type I and Type II DI:
l Enamel may be thinner than normal, with amber-like l Treatment of these patients is directed towards preventing
translucency and teeth have variety of colours blue-grey the loss of enamel and subsequent loss of dentin through
or amber-brown and are opalescent. attrition.
l Crown capping may be done on teeth to avoid attrition.
Radiographic features:
Cast metal crowns on posterior teeth and jacket crowns
l In both primary/permanent teeth, radiographs show bul-
on anterior teeth have been used with considerable
bous crowns with a cervical constriction, short slender
success.

SHORT ESSAYS
Q. 1. Write briefly on anodontia. iv. Syndrome associated
v. Radiation injury to the developing tooth germ.
Ans.
l Anodontia can also be divided into following types:
l Anodontia is defined as the condition in which there is a. True anodontia: It occurs due to failure of develop-
congenital absence of teeth in oral cavity. ment or formation of tooth in jaw bone.
l Aetiology: The causes of anodontia are: b. Pseudo anodontia: It refers to the condition in which
i. Hereditary factor teeth are present within the jaw bone but are not
ii. Environmental factor clinically visible in the mouth, as they have not
iii. Familial factor erupted, e.g. impacted teeth.
430 Quick Review Series for BDS 3rd Year

c. Induced or false anodontia: It is the condition in process. Recent studies show that it is a defect of mes-
which teeth are missing in the oral cavity because of enchymal tissues.
their previous extractions. l It is unilateral or bilateral defect, but never a midline
l True anodontia is of two types: defect.
a. Complete anodontia: There is congenital absence of l Mandibular cleft is very rare and is due to failure of
all the teeth. copula to give rise to the mandibular arch or due to per-
b. Partial anodontia: Congenital absence of one or few sistence of the central groove of the mandibular process.
teeth.
Cleft palate
Complete anodontia: l Cleft palate is due to lack of forces, interference by the
l It is the condition in which there is neither any deciduous tongue or disparity in the size of palatal processes of
tooth nor any permanent tooth present in the oral cavity. maxilla and/or the globulomaxillary process.
l A complete anodontia is a common feature of heredi- l It results from non-union of palatal shelves during
tary ectodermal dysplasia, however, in many cases embryogenesis.
cuspids are present in this disease.
Complete anodontia occurs among children those who
l
Clinical Features
have received high doses of radiation to the jaws as
infants for therapeutic reasons. l Mongoloids are mostly affected.
l Cleft lip alone is more common in males while cleft
Partial anodontia: palate alone is more common in females.
l It is a common phenomenon and is characterized by l Cleft lip can occur as unilateral or as a bilateral
congenital absence of one or few teeth. anomaly.
l In partial anodontia, any tooth can be congenitally missing, l The clefting anterior to the incisive foramen is also
e.g. the third molars are most frequently observed congeni- defined as cleft of primary palate.
tally missing teeth. The mandibular first molars and the l The clefting posterior to the incisive foramen is defined
mandibular lateral incisors are least likely to be missing. as cleft of secondary palate. A complete cleft palate
constitutes a cleft of hard palate, soft palate, and cleft
Q. 2. Cleft lip and palate. uvula.
Ans. l Four of the various accepted classification schemes for
CLP are as follows:
l Incidence of cleft lip and cleft palate is 1 in 800 births. i. Davis and Ritchie classification
l Mode of inheritance is multifactorial. ii. Veau classification
l Cleft lip and cleft palate are formed due to developmen- iii. Kernahan and Stark symbolic classification.
tal or maturational defect of embryonic processes. iv. International Confideration of Plastic and Recon-
structive Surgery.
Aetiology
Treatment of Cleft Lip
i. It is a hereditary condition. It appears as:
a. Monogenic/syndromic form: It is associated with l Surgery of lip is performed before child begins to
Down’s syndrome and other syndromes. speaks; either with Millard or Tennyson’s method.
b. Polygenic form: This form is determined by multi- l ‘Rule of 10’ is followed:
ple genes. 10 weeks of age.
ii. Other factors that have been suggested as possible 10 pounds of weight.
causes of cleft palate are: Hb% not less than 10 gm%.
a. Vascular insufficiencies.
b. Mechanical disturbances. Treatment of Cleft Palate
c. Circulating substances, such as alcohol, certain
drugs and toxins. l Cleft palate is treated at about 18 months of age.
d. Infections Q. 3. Median rhomboid glossitis.
e. Lack of inherent developmental forces.
Ans.
Maxillary cleft lip l It is an asymptomatic, elongated, erythematous patch of
l It is formed due to failure of fusion of medial nasal the atrophic mucosa on the mid dorsal surface of the
process with lateral nasal process and maxillary tongue.
Section | III Oral Pathology 431

Aetiology II. Apparent micrognathia: Jaw is not smaller but ap-


pears smaller due to abnormal relation between the
l It occurs due to failure of tuberculum impar to retract or
two jaws.
withdraw before fusion of the lateral halves of tongue.

Clinical Features True Micrognathia


l The condition is seen mostly in the young adults. It is of following types:
l Three times more frequent in men than in woman. (a) Congenital: It is found along with Pierre Robins
l The lesion is located in the posterior midline immedi- syndrome and congenital heart disease.
ately anterior to the foramen caecum and circumvallate (b) Acquired: This occurs after birth, e.g. ankylosis of TMJ
papilla on the dorsum of tongue. and infection or trauma at the region of TMJ, ear and
l The fully developed lesion of the median rhomboid mastoid.
glossitis appears as diamond or lozenge-shaped area In maxilla:
devoid of papillae. l Mouth breathing is presumed as its causative factor.
l The lesions are typically less than 2 cm in greatest l This causes deficiency of premaxillary area.
dimension and most demonstrate a smooth, flat surface l Clinically, it appears as retraction of middle third of
although some times surface may be lobulated. face or retrusion of maxilla.
l It is usually asymptomatic but occasionally causes
slight soreness or burning sensation. In mandible:
l The colour of lesion varies from pale-pink to bright-red. l Mandibular retrusion is mostly due to agenesis of con-
The lesions with atrophic candidiasis are usually more dyle or posterior positioning of mandible.
erythematous. l It clinically appears as retrusion of chin. Steep
l Infected cases may also demonstrate a midline soft pal- mandibular angle. Deficient chin button.
ate erythema in the area of routine contact with the un-
derlying tongue involvement, this is commonly reffered Treatment
to as a kissing lesion.
l Sliding ostectomy.
l Localized chronic fungal infection specifically Candida
albicans has been found. Fungi are best visualized by
PAS stain. MACROGNATHIA
l It means abnormally large jaw.
Histopathology l It may be present as:
l Median rhomboid glossitis shows a smooth or nodular i. Generalized enlargement of skeleton as in pituitary
surface covered by atrophic stratified squamous epithe- gigantism.
lium overlying a moderately fibrosed stroma with some- ii. Localized as in Paget’s disease, acromegaly and
what dilated capillaries. leontosis ossea.
l Loss of fungiform and filiform papillae. l Factors favouring macrognathia are:
l Underlying connective tissue is vascular and is infil- l Increased height of ramus.
trated by chronic inflammatory cells, proliferation of l Increased length of body of mandible.
spinous layer and elongation of rete pegs. l Anterior positioning of glenoid fossa.
l Presence of Candida hyphae and spores in the superfi- l Decreased maxillary length.
cial layers of the epithelium. l Posterior positioning of maxilla with relation to

cranium.
Treatment
l No treatment is required. Treatment
Q. 4. Micrognathia and Macrognathia. l Ostectomy to a portion of jaw.
Ans. Q. 5. Taurodontism.

MICROGNATHIA Ans.

Micrognathia is characterized by smaller jaw either


TAURODONTISM (BULL-LIKE TEETH)
l

maxilla or mandible.
I. True micrognathia: Jaw is truly smaller with normal l Taurodontism is a dental anomaly in which the body of
relation in between the two jaws. the tooth is enlarged at the expense of roots.
432 Quick Review Series for BDS 3rd Year

Aetiology permanent teeth have pulpal spaces that are either


smaller than normal or completely obliterated.
l A specialized or retrograde character
l A primitive pattern
l An atavistic feature Histological Features
l Mendelian recessive trait
Histopathology of teeth in Shields type III has not been
l A mutation
adequately documented.
l Associated with Klinefelter’s syndrome.
l It is due to failure of Hertwig’s epithelial root sheath to
invaginate at proper horizontal level. Treatment
l Treatment of these patients is directed towards prevent-
Types of Taurodontism ing the loss of enamel and subsequent loss of dentin
i. Hypotaurodont (mild form) through attrition.
ii. Mesotaurodont l Crown capping may be done on teeth to avoid attrition.
iii. Hypertaurodont (severe form) Cast metal crowns on posterior teeth and jacket crowns on
anterior teeth have been used with considerable success.
Clinical Findings Q. 7. Write briefly on supernumerary teeth.
l It may affect both deciduous and permanent dentition, Ans.
but more common in permanent dentition.
l Molars are commonly affected. l Presence of any extra tooth in dental arch in addition to
l Tooth morphology is normal. normal series of teeth is known as supernumerary teeth.
l Splitting of normal tooth bud or extra tooth bud may
give rise to additional tooth.
Radiographic Features l These may be either supernumerary teeth or supplemen-
l Enlarged and rectangular pulp chamber is present. tal teeth.
l No constriction of pulp at cervical area.
l Roots are very short. Classification
l Furcation is present just above root apex.
I. According to the morphology
Treatment a. Conical: This small peg-shaped conical tooth is
supernumerary tooth.
No treatment is required. b. Tuberculate: This type of supernumerary tooth pos-
Q. 6. Shell teeth or ghost teeth. sesses more than one cusp or tubercle. It is of barrel-
shaped and may be invaginated.
Ans. c. Supplemental: It refers to the duplication of the teeth in
l Shell teeth or ghost teeth are characteristic feature of normal series. The most common tooth is permanent
dentinogenesis imperfecta II, i.e. Brandywine type maxillary lateral incisor.
dentinogenesis imperfecta. d. Odontome: This represents the hamartomatous
l This disorder was found in Brandywine triracial isolate malformation.
in southern Maryland.
II. According to location
Clinical Features a. Mesiodens: They are located between two upper central
incisors.
l The crowns of the deciduous and permanent teeth wear b. Distomolars: They are located on the distal aspect of
rapidly after eruption and multiple pulp exposures are regular molar teeth in dental arch.
seen in primary teeth. c. Paramolars: They are located either on buccal or lingual
l Colour of teeth has unusual transluscent/opalescent hue. aspect of normal molars.
l Enamel lost earlier due to fracture; defective DEJ and d. Extra lateral incisors: They are more common in maxil-
dentin is worn off. lary arch.

Radiographic Features Clinical Features


l Deciduous dentition shows ‘shell teeth’ appearance i. Supernumerary teeth do not resemble normal teeth and
due to enlarged pulp chambers and root canals. The are conical in shape.
Section | III Oral Pathology 433

ii. Mostly present between maxillary central incisors l Both permanent and deciduous dentitions are affected in
termed as mesiodens. case of fusion, although it is more common in decidu-
iii. Supplemental teeth resemble with teeth of normal series. ous teeth.
These are found either distal to molars (distomolars) or l The incisor teeth are more frequently affected in both
at buccal or lingual aspect of molars (paramolars). the dentitions.
iv. Supernumerary teeth cause crowding or malocclusion
and give rise to cosmetic problem. i. c. Concrescence
v. These extra teeth are responsible for the increased car- l Concrescence is defined as union of two adjacent teeth by
ies incidence and periodontal problem. cementum only without confluence of the underlying dentin.
vi. Multiple supernumerary teeth can occur in association l It is the type of fusion which is limited only to the roots of
with the conditions like Gardener’s syndrome and teeth and it occurs after the root formation is completed.
cleidocranial dysplasia. l Common between maxillary 2nd molar and unerupted
vii. Dentigerous cyst may sometimes develop from an 3rd molar.
impacted supernumerary tooth.
ii. Accessory Cusps
Treatment
a. Cusp of Carabelli
l The supernumerary teeth should be extracted. b. Talon’s cusp
l Impacted supernumerary teeth should also be removed c. Dens invaginatus (dens in dente)
surgically since they can interfere with normal tooth d. Dens evaginatus
alignment.
Q. 8. Developmental anomalies affecting the shape of a. Cusp of Carabelli
teeth. l Present on mesiopalatal cusp of maxillary 1st molars.
l An analogous accessory cusp is seen occasionally on
Ans. the mesiobuccal cusp of a mandibular permanent or
The developmental disturbances affecting shape of teeth are deciduous molar known as protostylid.
as follows:
i. Gemination, fusion and concrescence b. Talon’s cusp
ii. Accessory cusps l Talon’s cusp is an anomalous projection resembling
l Cusp of Carabelli eagle’s talon projects lingually from cingulum area of
l Talon cusp permanent incisors.
l Dens invaginatus l Found in association with “Rubinstein-Taybi syndrome”.
l Dens evaginatus

iii. Ectopic enamel c. Dens in dente or dens invaginatus


l Enamel pearls l Dens in dente is a developmental variation which arises
l Cervical enamel extensions as a result of enamel epithelial invagination of the
iv. Taurodontism crown surface before calcification.
v. Dilaceration l Teeth most frequently involved are maxillary lateral and
vi. Supernumerary roots. maxillary central incisors.

i.a. Gemination d. Dens evaginatus (occlusal tuberculated


l Gemination is a developmental anomaly which refers to premolar, Leong’s premolar, evaginated
division of single tooth germ into incomplete or com- odontome)
plete formation of two teeth. l Dens evaginatus is a developmental condition appears
l Crowns may be partially or totally separated from each as an accessory cusp or globule of enamel on occlusal
other while roots are fused and single root canal is pres- surface between buccal and lingual cusps of premolars
ent within the root. unilaterally or bilaterally.
l The condition is seen in both deciduous and permanent l It occurs due to extrusion of the dental papilla outwards
dentitions. into the enamel organ.
i.b. Fusion
iii. Ectopic Enamel
l Fusion is defined as single enlarged tooth or joined
tooth in which the tooth count reveals a missing tooth l Enamel pearls
when the anamolous tooth is counted as one. l Cervical enamel extensions
434 Quick Review Series for BDS 3rd Year

l Ectopic enamel or enamel pearls or enameloma or odontogenesis often manifest as horizontal line of small
enamel drop usually occurs in furcation area below the pits or grooves or enamel surfaces.
crest of gingiva. l This line on tooth surface indicates zone of enamel hy-
l Cervical enamel extension, also occurs along the sur- poplasia and corresponds to time of development and
face of dental roots. duration of insult.
l The factors in following stages:
iv. Taurodontism (Bull-Like Teeth) a. Prenatal period:
l The prenatal infections are rubella, syphilis.
l Taurodontism is a dental anomaly in which the body of l There is presence of internal disease
the tooth is enlarged at the expense of roots. l There are excess fluoride ions

b. Neonatal period: During this period, enamel hypo-


v. Dilaceration: plasia is caused due to:
l Haemolytic disease of newborn
l Dilaceration refers to angulation or curve in root or l Birth injury
crown of tooth. l Premature delivery
l Angulation is caused due to trauma to the tooth during l Prolong labour
formative stage of tooth. l Low birth weight
l Curve is present at apical, middle or at cervical portion c. Postnatal period: During this period, enamel hypo-
depending on the portion which is forming at the time plasia is due to:
of trauma. l Sever childhood infection.

l Prolong fever due to infectious disease in childhood


vi. Supernumerary Roots l Nutritional deficiency

l Hypocalcaemia
l One or more extra roots may be present in tooth.
l Rickets
l Usually single-rooted teeth, such as mandibular cuspids
l Celiac disease.
and bicuspids, are involved.
l Third molars of both jaws also present one or more Q. 10. Mottled enamel.
extra roots.
Q. 9. Write briefly on factors causing acquired enamel Ans.
hypoplasia. l The mottled enamel is a type of hypoplasia due to fluo-
Ans. ride and was first described by GV Black and Frederick
S Mckay.
Factors associated with acquired enamel hypoplasia are of l If drinking water contains fluoride content more than
two types: 1 p.p.m. at the time of amelogenesis, it can cause death
A. Local factors of ameloblasts so it leads to defective matrix formation.
B. Environmental or systemic factors l It may also result in disturbances in calcification.
Enamel affected is termed as mottled enamel.
A. Local factors l The mottled enamel is due to the disturbance of amelo-
l Infection blasts during formative stage of tooth development.
l Trauma l In mild cases, white flecks appear in enamel.
l Radiotherapy and l Moderate degree of fluoride toxicity may lead to white
l Idiopathic factors. opaque areas in enamel,
l When local infection or trauma causes damage to ame- l In more severe case, brown discolouration of enamel
loblast cells during odontogenesis, it may result in de- occurs.
fect in enamel formation in isolated permanent tooth l Still more severe toxicity of fluoride causes corroded
and this is known as focal enamel hypoplasia. crown surface.
l The focal enamel hypoplasia is caused due to periapical
spread of infection from a carious deciduous tooth or
trauma to the deciduous tooth, the tooth affected in this Treatment
process is commonly known as Turner’s tooth. l Defluoridation of drinking water is done, if drinking
water contains excessive fluoride content.
B. Environmental or systemic factors l The staining of enamel surface can be removed by
l The systemic or environmental disturbances in the func- bleaching with hydrogen peroxide (30% solution),
tioning of ameloblasts at specific period of time during grinding or capping.
Section | III Oral Pathology 435

Q. 11. Regional odontodysplasia. Radiographic features


Ans. l “Ghost” like appearance of tooth with large pulp, thin
enamel and dentin.
l One or several teeth in a localized area are affected.
l Anterior teeth are affected mostly. Histological features
l Maxillary arch is more commonly involved. l Enamel is hypoplastic and hypomineralized.
l Dentin has thickened predentin layer.
Clinical features
l Reduced enamel epithelium around unerupted tooth has
l Delayed or no eruption of teeth may be seen. irregular calcified bodies.
l Defective mineralization causes irregular shape of teeth.

SHORT NOTES
Q. 1. Anodontia. l Crowns may be partially or totally separated from each
other while roots are fused and single root canal is pres-
Ans.
ent within the root.
i. Anodontia means the absence of teeth in oral cavity. l The condition is seen in both deciduous and permanent
ii. True anodontia: dentitions.
l Congenital absence of teeth is termed as true anodontia.

l Total absence of teeth is found in case of hereditary


Q. 4. Fusion.
ectodermal dysplasia. Ans.
l Partial absence of teeth is termed as true partial ano-

dontia and affects third molars, lateral incisors and l Fusion is a development disturbance in the shape of the
premolars usually. tooth.
iii. False anodontia: l Fused teeth arise through union of two normally sepa-
l This condition results due to extraction of teeth.
rated tooth germs.
iv. Pseudoanodontia: It results from non-eruption of mul- l Fusion is defined as single enlarged tooth or joined
tiple teeth. tooth in which the tooth count reveals a missing tooth
v. Aetiology: Mutation when the anomalous tooth is counted as one.
l Both permanent and deciduous dentitions are affected in
Q. 2. Xerostomia. case of fusion, although it is more common in decidu-
Ans. ous teeth.
l The incisor teeth are more frequently affected in both
i. Xerostomia refers to dry mouth. the dentitions.
ii. Aetiology: Psychological (anxiety and depression)
Duct calculi (blockage of submandibular gland duct, Q. 5. Dens-in-dente.
producing dryness) Ans.
Sialoadenitis, salivary gland aplasia, Sjogren’s syndrome
Drug therapy(anticholinergic and sympathomimetic l Dens in dente is also known as dens invaginatus/dilated
agents) composite odontome.
Others (anaemia, vitamin A, riboflavin deficiencies) l It is a developmental variation which arises as a result
iii. Clinical features: of an invagination in the surface of the tooth crown be-
Unilater or bilateral pain, discomfort, dryness fore calcification has occurred.
Swelling of affected gland l Causes: Increased localized external pressure, facial
Lymph node enlargement growth retardation.
iv. Management: Removal of aetiological cause, promote l Permanent maxillary lateral incisors are commonly
salivary stimulation by using sugarfree chewing gums, involved followed by maxillary central incisors.
salivary substitutes. l Condition is usually bilateral.

Q. 3. Gemination. Q. 6. Taurodontism.
Ans. Ans.
l Gemination is a developmental anomaly which refers to l Taurodontism is a dental anomaly in which the body of
division of single tooth germ into incomplete or com- the tooth is enlarged at the expense of the roots.
plete formation of two teeth. l The term taurodontism refers to “bull-like teeth”
436 Quick Review Series for BDS 3rd Year

l Cause: Failure of Hertwig’s epithelial sheath to invagi- l It is a rare disturbance of dentin formation characterized
nate at the proper horizontal level. by normal enamel but atypical dentin formation with
l Most commonly involves permanent dentition followed abnormal pulpal morphology.
by deciduous dentition, molars are commonly l Two types:
involved. Type1/radicular dentin dysplasia
l Condition may be unilateral or bilateral. Type 2/coronal dentin dysplasia
l Involved teeth are rectangular in shape.
Q. 11. Dentinogenesis imperfecta.
Q. 7. Unerupted teeth.
Ans.
Ans.
l Dentinogenesis imperfecta is a developmental distur-
l Unerupted teeth is an uncommon condition in which bance in the structure of the teeth.
more or less permanently delayed eruption of teeth. l This is an autosomal condition affecting both deciduous
l The person affected may or may not have retained his and permanent teeth.
deciduous teeth, but the permanent teeth failed to erupt. l Affected teeth are grey to yellowish brown and have
l The condition may be associated with cleidocranial broad crowns with a constriction area of the cervical
dysplasia. area resulting in a “tulip” shape.
l Types:
Q. 8. Tooth ankylosis.
Dentinogenesis imperfecta type I (opalescent dentin)
Ans. Dentinogenesis imperfecta type II (brandywine type)
l Ankylosed teeth are submerged teeth, most commonly Q. 12. Amelogenesis imperfecta.
deciduous mandibular second molars, that have under-
Ans.
gone a variable degree of root resorption and have
become ankylosed to the bone. l Amelogenesis imperfecta is a developmental distur-
l Causes: Trauma, infection, disturbed local metabolism, bance in the structure of the teeth.
genetic influence. l It is also known as hereditary enamel dysplasia/heredi-
l This condition is usually treated by surgical removal of tary brown enamel/hereditary brown opalescent teeth.
ankylosed teeth to prevent the development of maloc- l It is a structural defect of the tooth enamel.
clusion, local periodontal disturbance or dental caries. l It is differentiated into 3 main groups:
Hypoplastic
Q. 9. Supernumerary teeth.
Hypocalcified
Ans. Hypomature
l Supernumerary teeth are a developmental disturbance in Q. 13. Mottled enamel.
the number of teeth.
Ans.
l A supernumerary tooth is an additional entity to the
normal series and is seen in all quadrants of the jaw l Mottled enamel is a developmental disturbance in the
l Morphological types of supernumerary teeth: structure of teeth.
Conical l Enamel hypoplasia due to fluoride is called mottled
Tuberculate enamel.
Supplemental l Aetilogy: Ingestion of fluoride containing drinking wa-
Odontome ter during the time of tooth formation.
l Multiple supernumerary teeth can occur in association l Clinical features:
with the conditions like Gardener’s syndrome and i. Occasional white flecking or spotting of the enamel.
cleidocranial dysplasia. ii. Mild changes manifested by white opaque areas
l Impacted supernumerary teeth should be removed involving more of the tooth structure.
surgically since they can interfere with normal tooth iii. Moderate and severe changes showing pitting and
alignment. brownish staining of the surface.
iv. Corroded appearance of the teeth.
Q. 10. Dentin dysplasia.
Q. 14. Hypoplasia of teeth.
Ans.
Ans.
l Dentin dysplasia is a developmental disturbance in the
structure of teeth. l Enamel hypoplasia may be defined as incomplete or
l It is also known as “rootless teeth”. defective formation of organic enamel matrix of teeth
Section | III Oral Pathology 437

l Types: l Clinical features: Palatal cysts of the newborn typi-


a. Hereditary type cally present as multiple (usually less than six) 1-4 mm
b. Environmental yellow-white, sessile mucosal papules of the posterior
l Aetiology: hard palate and occasionally of the anterior soft palate.
Nutritional deficiency (vitamins A, C, D)
Q. 18. Fordyce’s granules.
Exanthematous diseases (measles, chickenpox, scarlet
fever) Ans.
Congenital syphilis
l Fordyce’s granules are a developmental anomaly char-
Hypocalcaemia
acterized by heterotopic collections of sebaceous glands
Birth injury, prematurity
at various sites in the oral cavity.
Local infection or trauma
Ingestion of chemicals
Idiopathic. Clinical Features
l Treatment: There is no treatment except for improve-
ment of cosmetic appearance by veneering or capping l They appear as small yellow spots, either discretely
of teeth. separated or large plaques, often projecting slightly
above the surface of tissue.
Q. 15. Epidermoid cyst. l They are found most frequently in a bilaterally sym-
Ans. metrical pattern on the mucosa of cheeks opposite to
molar teeth, inner surface of the lips, retromolar region
l Epidermoid cyst is also known as epidermal inclusion and occasionally tongue, gingival, frenum, and palate.
cyst, keratin cyst or sebaceous cyst.
Q. 19. Median rhomboid glossitis.
Clinical Features Ans.
i. Most common in third and fourth decades of life. i. Median rhomboid glossitis is a developmental distur-
ii. Discharge of a foul smelling cheese-like material is a bance of tongue.
common compliant.
iii. Epidermoid cysts appear as firm, round, mobile, flesh-
coloured to yellow or white subcutaneous nodules of Clinical Features
variable size.
l It presents as oval-shaped reddish patch or plaque in
iv. The cysts can cause difficulty in feeding, swallowing,
the posterior midline of the dorsum of the tongue, just
or speaking.
anterior to the v-shaped grouping of the circumvalate
v. The cysts may be associated with Gardner’s syndrome,
papillae.
basal cell nevus syndrome.
l It appears in the childhood and is of 3:1 male predilection.
Q. 16. Globulomaxillary cyst. l Lesion with atrophic candidiasis appears erythematous.
l Infected lesions show midline soft palate erythema in
Ans.
the area of routine contact with tongue involvement
i. Globulomaxillary cyst is described as a fissural cyst referred as a ‘kissing lesion’.
found within the bone between the maxillary lateral l They may be caused due to localized chronic fungal
incisor and canine teeth. infection specially Candida.
ii. Clinical features: Cysts become infected and patient
Q. 20. Macroglossia.
may complain of local discomfort or pain in the area.
iii. Radiologically, it is a well-defined radiolucency which Ans.
frequently causes the roots of the adjacent teeth to
i. Macroglossia is a developmental disturbance of tongue.
diverge.
ii. It is also called as tongue hypertrophy; enlarged tongue.
iv. Treatment: Cyst should be surgically removed.
iii. Types are:
Q. 17. Bohn’s nodules. a. True macroglossia
b. Psuedo macroglossia
Ans.
iv. Clinical features: Severe retrognathia, unusually small
l Bohn’s nodules are the gingival cysts of the newborn. maxilla or mandibular size.
l Bohn’s nodules were used for cysts which originated v. It may be associated with Down syndrome, Beckwith-
from palatal gland structures and were scattered more Wiedemann syndrome.
widely over the hard and soft palates. vi. Treatment: By surgical intervention.
438 Quick Review Series for BDS 3rd Year

Q. 21. Shell teeth. l Hairy tongue may appear as brown, green, pink or any
of a variety of hues depending on specific aetiology and
Ans.
secondary factors.
i. In type III dentinogenesis imperfecta (Bradywine type), l Filliform papillae in hairy tongue measure more than
the dentin appears very thin and the pulp chambers and 15 mm in length.
the root canals are extremely large. l Over growth of Candida albicans may result in glosso-
ii. Because of this, all the teeth appear as thin ‘shells’ of pyrosis (burning tongue).
enamel and dentin which is described as “shell
Q. 26. Dilaceration.
theeth”.
iii. They are seen in combination with enamel aplasia. Ans.
Q. 22. Ghost teeth. l Dilaceration refers to an angulation, or a sharp bend or
curve, in the root or crown of a formed tooth.
Ans.
l This condition is caused due to trauma during the period
i. Ghost teeth is also known as regional odontodysplasia. in which the tooth is forming.
ii. It is a unique dental anamoly that involves enamel as l Dilacerated teeth frequently present difficult problems
well as dentin. at the time of extraction.
iii. Permanent teeth are usually affected which exhibits
Q. 27. Talon’s cusp.
delayed eruption and defective mineralization.
iv. Radiographs show marked reduction in the radioden- Ans.
sity of the teeth and hence assume a “ghost-like”
l Talon’s cusp is an anomalous structure resembling an
appearance.
eagle’s talon, projects lingually from the cingulum areas
v. Both the enamel and dentin appear very thin and pulp
of a maxillary or mandibular permanent incisors.
chamber is exceedingly large. The enamel layer often is
l It consists of deep developmental groove where the cusp
not evident.
blends with the sloping lingual tooth surface.
Q. 23. Dermoid cyst. l It is composed of normal enamel and dentin and
contains a horn of pulp tissue.
Ans.
l It may be associated with Rubinstein-Taybi syndrome.
i. Dermoid cyst is also known as cystic teratoma.
Q. 28. Concresence.
ii. It is a hamartomatous tumour containing multiple seba-
ceous glands. Ans.
iii. It occurs mostly on the face, neck, or scalp.
l Concresence is a developmental anomaly where the
iv. Three subclasses of congenital mouth cysts are:
fusion of teeth occurs along the cementum only.
a. Epidermoid (simple) cysts
l It occurs after the root formation is completed.
b. Dermoid (complex) cysts
l This may be due to trauma, crowding of teeth with
c. Teratoid (complex) cysts
v. Treatment: Surgical excision. resorption of interdental bone.
Q. 29. Melkerson-Rosenthal syndrome.
Q. 24. Name few developmental cysts.
Ans. Ans.

Various developmental cysts are as follows: Melkerson-Rosenthal syndrome consists of following


l Dentigerous cyst recurrent attacks of facial paralysis, identical with Bell’s
l Eruption cyst palsy associated with multiple episodes of non-pitting,
l Odontogenic keratocyst noninflammatory, painless oedema of the face, chelitis
l Lateral periodontal cyst and granulomatosa (swelling of the lips), and fissured tongue.
l Calcifying odontogenic cyst The facial oedema involves usually the upper lip.

Q. 25. Hairy tongue. Q. 30. Hutchinson’s triad.

Ans. Ans.
l Hairy tongue is also known as lingua nigra, lingua l Hutchinson’s traid is a pathognomonic feature of con-
villosa, black hairy tongue. genital syphilis.
l It is a condition of defective desquamation of filliform l The Hutchinson’s triad includes: Hypoplasia of the
papillae that results from a variety of precipitating incisors and molar teeth, eighth nerve deafness and
factors. intestinal keratitis.
Section | III Oral Pathology 439

Q. 31. Fordyce granules. Q. 32. Turner’s hypoplasia.


Ans. Ans.
l Fordyce granules is a developmental anomaly charac- l Turner’s hypoplasia is enamel hypoplasia due to local
terized by heterotopic collection of sebaceous glands at infection or trauma.
various sites in the oral cavity. l Only single tooth, generally maxillary incisor or premo-
l They appear as small yellow spots, either discretely lar is affected.
separated or large plaques, often projecting slightly l The apically infected deciduous tooth may affect the
above the surface of tissue. ameloblastic layer of permanent tooth.
l They are found most frequently in a bilaterally l Ameloblastic layer may also get disturbed due to
symmetrical pattern on the mucosa of cheeks opposite occlusal trauma to deciduous tooth.
to molar teeth, inner surface of the lips, retromolar l The permanent tooth may get discoloured or pitted. The
region and occasionally tongue, gingiva, frenum, and teeth are called Turner’s teeth and this type of enamel
palate. hypoplasia is termed as Turner’s hypoplasia.

Topic 2

Benign and Malignant Tumours of the Oral Cavity

LONG ESSAYS
Q. 1. Mention malignant tumours of epithelial tissue of l Generally occurs in elderly patients, the mean age of
oral mucosa. Describe clinical and histological features occurrence is 60-70 years and males are more com-
of verrucous carcinoma. monly affected.
l The common locations for verrucous carcinoma are
Ans.
gingiva, alveolar mucosa and buccal mucosa.
The malignant tumours of epithelial tissue of oral mucosa l Verrucous carcinoma presents as slow growing, exo-
are as follows: phytic, papillary growth having white pebbly surface.
i. Basal cell carcinoma (rodent ulcer) l The carcinoma occurs either as single entity or there
ii. Epidermoid carcinoma (squamous cell carcinoma) can be multiple lesions involving different parts of oral
iii. Verrucous carcinoma cavity.
iv. Spindle cell carcinoma l The lesions on buccal mucosa are sometimes very ex-
v. Adenoid squamous cell carcinoma tensive and often cause pain, tenderness and difficulty in
vi. Basaloid squamous cell carcinoma taking the food.
vii. Lymphoepithelioma and transitional cell carcinoma l The regional lymph nodes are often enlarged and tender.
viii. Nasopharyngeal carcinoma l Less aggressive than squamous cell carcinoma.
ix. Malignant melanoma. l Lateral invasion is more and vertical invasion is less.
l The lesion has pebbly surface and is sometimes covered
VERRUCOUS CARCINOMA by white leukoplakic film which is keratinized.
l Verrucous carcinoma is a diffused papillary, non-
metastasizing well-differentiated malignant neoplasm
Histopathology
of oral epithelium.
l It is also known as Ackerman’s tumour or pseudo- l The hyperplastic epithelium often exhibits a papillary
epithaliomatous papilloma. surface being covered by the thick layer of parakeratin.
l Massively enlarged bulb-like acanthotic rete ridges are
seen which invaginated into underlying connective
Clinical Features tissue stroma.
l A very high percentage of patients with this disease are l Many deep cleft-like spikes lined by thick layer of para-
tobacco chewers and few of them reported ill-fitting dentures. keratin, these extend from the surface of epithelium and
440 Quick Review Series for BDS 3rd Year

project deep into the centre of bulbous rete ridges. This Treatment
is known as parakeratin plugging.
Surgical excision conservatively and recurrence is rare.
l All bulbous rete ridges of the epithelium projects into
the connective tissue and is known as pushing margin. Q. 3. Describe and enumerate the precancerous lesions
l The basement membrane is intact and underlying con- of oral cavity. Describe the aetiology, clinical features
nective tissue shows inflammatory cell infiltration. and histopathology of leukoplakia.
l Formation of epithelial pearls and microcytes are seen.
Ans.
The premalignant lesions are defined as morphologically
Treatment
altered tissue in which cancer is more likely to occur then
l Surgical excision or laser therapy is done and prognosis in its apparently normal counter part, e.g.
should be good. l Leukoplakia
l Erythroplakia
Q. 2. Describe the pathogenesis, clinical X-ray appear-
l Nicotina palati
ance of central ossifying fibroma with histological
l Stomatitis
features.
l Dyskeratosis congenitis
Ans.
Central ossifying fibroma is a central neoplasm of bone. LEUKOPLAKIA
It is a keratotic white lesion on oral mucosa that cannot
Clinical Features be characterised clinically or histologically as any other
disease.
l It occurs in young adults and is more common in
It is the most common potentially malignant lesion of
females.
the oral mucosa.
l More predilection for mandible than maxilla.
l Occurs in the body of the mandible from premolar to
molar region. Aetiology of Leukoplakia
l The lesion is asymptomatic until the growth produces a
The common predisposing factors of leukoplakia are:
noticeable swelling and mild deformity.
i. Tobacco: Use of tobacco in various forms such as
l Displacement of teeth may be an early clinical feature.
smoking of cigarette, cigar, biddies and pipes. All these
l It is relatively slow growing tumour and the cortical
types of tobacco habits are important for development
plates of bone and overlying mucosa or skin are invari-
of leukoplakia. It is believed that during smoking a
ably intact.
large amount of tobacco end products are produced in
l Rarely paraesthesia and pain.
oral cavity. The products in association with heat cause
severe irritation to oral mucous membrane and finally
Radiological Features result in development of leukoplakia.
ii. Alcohol: Many people who consume alcohol develop
l Shows variable appearance depending on its stage. leukoplakia.
l The lesion is well circumscribed and demarcated from iii. Candidiasis: Chronic candidal infections are associ-
surrounding bone. ated with leukoplakia.
l In early stages, it appears as radiolucent area with no iv. Dietary deficiency: Deficiency of vitamin A causes
evidence of internal radio-opacities. metaplasia and hyperkeratinization of epithelium which
l As the tumour matures, calcification increases and may result in development of leukoplakia.
hence the flecks of radio-opacity appears until it be- v. Syphilis: The syphilitic infections play minor role in
comes uniformly radio-opaque mass. causation of leukoplakia.
l Displacement of adjacent teeth is common and as well vi. Hormonal imbalance: Imbalance or dysfunction of
as impingement upon other adjacent structures. both male and female sex hormones causes kerato-
genic changes in oral epithelium. These changes lead
to the development of leukoplakia.
Histological Features
vii. Trauma like ill fitting dentures, sharp dentures, tooth
l The lesion is composed basically of many delicate inter- brush trauma
lacing collagen fibres. viii. Galvanism
l The connective tissue characteristically presents many ix. Idiopathic
small foci of irregular bony trabeculae. x. UV radiation
Section | III Oral Pathology 441

Clinical Features Changes in cellular layer


l The various forms of leukoplakia and subdivisions as l Epithelial dysplasia is the hallmark of histologic changes
recommended by WHO in 1980 are as follows: seen in epithelium in case of leukoplakia.
i. Homogenous: Lesions that are uniformly white. l The histopathological features of epithelial dysplasia are:
ii. Non-homogenous: Lesions in which part of the i. Loss of polarity of basal cells
lesion is white and rest appears reddened. ii. Increased nuclear cytoplasmic ratio
l More elaborative subdivisions may be used such as: iii. Irregular epithelial stratification
i. Homogenous: iv. Cellular pleomorphism
a. Smooth v. Nuclear hyperchromatism
b. Furrowed (fissured) vi. Reduction of cellular cohesion.
c. Ulcerated vii. Enlarged nucleoli
ii. Non-homogenous nodulospekled: Well-demarcated viii. Increased number of mitotic figures.
raised white areas, interspersed with reddened areas.
Thickness of the epithelium
l Usually, the lesion occurs after 30 years of age with
peak incidence above the age of 50 years. In leukoplakia, the thickness of epithelium is altered and it
l Buccal mucosa and commissural areas are most frequent occurs in the form of epithelial atrophy or acanthosis.
affected sites followed by alveolar ridge, tongue, lip,
hard and soft palate, etc. Alteration in underlying connective tissue
l Oral leukoplakia often presents solitary or multiple In leukoplakia, there is often variable degree of destruction of
white patches. collagen fibres and moreover chronic inflammatory cell infil-
l The size of lesion may vary from small well-localized trate is also present in underlying connective tissue stroma.
patch measuring few millimetres in diameter to several
centimetres. A Modified Classification and Staging
l The surface of lesion may be smooth or finely wrinkled
or even rough on palpation and lesion cannot be
System for Oral Leukoplakia
removed by scrapping. A proposal for a modified classification and staging system
l The lesion is whitish or greyish or in some cases it is for oral leukoplakia has been presented by van der Wall et al,
brownish yellow in colour due to heavy use of tobacco. 2000 in which size of the leukoplakia and the presence
l In most of the cases, these lesion are asymptomatic, or absence of epithelial dysplasia are taken into account.
however, in some cases, they may cause pain, feeling of Altogether four stages are recognized:
thickness, burning sensation, etc. L1–Size of leukoplakia ,2 cm
L2–Size of leukoplakia 2-4 cm
L3–Size of leukoplakia .4 cm.
Histologic Features
Lx–Size not specified.
l Leukoplakia exhibits variety of histologic changes P–pathology
which are related to keratinization pattern, changes in P0–No epithelial dysplasia
cellular layer, thickness of epithelium and alteration in P1–Distinct epithelial dysplasia
underlying connective tissue stroma. Px–Dysplasia not specified in the pathology report.

Keratinization pattern OLEP Staging System


l Leukoplakia generally presents hyperorthokeratiniza-
tion or hyperparakeratinization or both with or without Stage I–L1 P0
the presence of epithelial dysplasia. Stage II–L2P0
l In case of leukoplakia, an abnormal increase in the Stage III–L3P0 or L1 L2 P1
thickness of orthokeratin layer is seen in area of epithe- Stage IV–L3 P1.
lium which is usually keratinized.
l An important histological criterion of leukoplakia is Diagnostic Procedures
presence of hyperkeratinization of normally keratin-
ized epithelium or some amount of perakeratin depo- l Elimination of possible causes
sition in area of epithelium which are usually not l Biopsy
keratinized. While diagnostic methods other than histological ex-
l Epithelial dysplasia is more frequently associated with amination like Toludine blue staining or Lugol’s iodine and
hyperkeratinized lesion. exfoliative cytology are of limited value.
442 Quick Review Series for BDS 3rd Year

Treatment Modalities l The epithelium shows lack of keratin production and is


often atrophic but it may be hyperplastic.
l Surgical excision l Lack of keratinization and thining of epithelium
l Cryosurgery allows underlying vasculature to be seen causing red
l CO2-laser surgery colour.
l Retinoids and other drugs l Connective tissue rete pegs are high into the epithelium.
l Photodynamic therapy. l The underlying connective tissue often demonstrates
Q. 4. Enumerate the precancerous lesions. Write in de- chronic inflammation.
tail about aetiology, clinical features and histopathology
of erythroplakia.
Treatment
Ans.
l Remove irritating agents.
The premalignant lesions are defined as morphologically l Prompt biopsy is mandatory.
altered tissue in which cancer is more likely to occur then l Recurrence rate of less than 5% is reported.
in its apparently normal counter part, e.g.
Q. 5. Enumerate pre-malignant conditions of the
l Leukoplakia
oral cavity. Give an account on oral submucous fibro-
l Erythroplakia
sis with reference to aetiology, clinical features and
l Nicotina palati
histology.
l Stomatitis
l Dyskeratosis congenitis Or
Pathogenesis of oral submucous fibrosis.
ERYTHROPLAKIA Ans.
l It is a red patch that cannot be clinically or pathologi-
The premalignant condition is defined as generalized state
cally diagnosed as any other condition and is relatively
of body, which is associated with significantly increased
rare compared to leukoplakia.
risk of cancer, e.g. oral submucus fibrosis, syphilis, lichen
l Erythroplakia is almost associated with premalignant
planus, white sponge nevus, etc.
changes histologically and is, therefore, a most impor-
tant precancerous lesion.
ORAL SUBMUCOUS FIBROSIS (OSMF)
Aetiology OSMF is a precancerous condition arising from oral cavity.
i. Unknown The lesion is characterized by juxtaepithelial inflammatory
ii. Smoking and alchohol abuse are important aetiological reaction in oral mucosa followed by fibroelastic transforma-
factors. tion of lamina propria leading to mucosal atrophy, rigidity
and trismus.

Clinical Features
Synonyms
l No apparent sex predilection.
l Most cases occurred in sixth and seventh decades of l Atropia idiopathica
life. l Idiopahic scleroderma of mouth
l Sites: Floor of mouth, tongue, retromolar area, soft pal- l Idiopathic palatal fibrosis
ate, mandibular mucosa and sulcus. l Sclerosing stomatitis
l Appears clinically well-demarcated, erythematous, l Submucosal fibrosis of palate
macule/papule with a soft velvety tissue
l Usually asymptomatic Aetiology
l Classification:
i. Homogenous The OSMF is caused due to:
ii. Erythroplakia interspersed with patches of leukoplakia l Excessive consumption of red chillies.
iii. Granular or speckled erythroplakia l Excessive “areca nut” chewing.
l Nutritive deficiency
l Immunological factors
Histopathology l Genetic factors
l Multicentric in origin and severe epithelial dysplasia is l Protracted tobacco use
seen. l Patient with deficiency of micronutrients.
Section | III Oral Pathology 443

Pathogenesis Q. 6. Write briefly about gaint cell lesions of oral cavity.


Describe in detail central giant cell granuloma.
Aetiological factors cause an alteration in the oral mucosa
which increases the risk of hypersensitivity reaction to many Ans.
potential irritants resulting in oral mucous membrane changes
Various giant cell lesions of oral cavity are as follows:
which may lead to malignant transformation up to 4.5-7.6%.
l Neoplasms
i. Giant cell tumour of bone
Clinical Features ii. Central and peripheral giant cell granuloma
l It is caused during 20 to 40 years of age. iii. Giant cell epulis
l Females are affected more often than males. iv. Giant cell tumour of hyperthyroidism
l In OSMF, fibrotic changes are frequently seen in buccal v. Giant cell fibroma
mucosa, retromolar area, uvula, tongue, etc. vi. Malignant fibrous histiocytoma
l Initially patient complains of burning sensation in the l Other lesions:
mouth, particularly during taking hot and spicy foods. i. Osteoblastoma
l There can be excessive salivation, decreased salivation ii. Chondroblastoma
and defective gustatory sensation. iii. Fibrous dysplasia of bone
l In initial phase of disease, palpation of mucosa elicits a iv. Hodgkin’s disease
“wet leathery” feeling. v. Giant cell arteritis
l Patient complains of vesiculation, ulceration, recurrent vi. CEOT
stomatitis, pigmentation changes. vii. Sarcoidosis.
l In advanced stage, the oral mucosa loses its resilience and
becomes blanched and stiff and thereby causing trismus. CENTRAL GIANT CELL GRANULOMA
l Lips become leathery and difficulty in everting them,
i.e. microchelia. Central giant cell granuloma is relatively a common benign
l Opaque marble-like oral mucosa. intraosseous destructive giant cell lesion which affects an-
l Deglutition is difficult with referred pain in ear. terior part of jaw bone.
l Circular fibrous bands around rima oris. It is more aggressive in nature.
l Palpation of mucosa often reveals vertical fibrous beds.
l Uvula gets shrunken and gives a hockeystick appearance. Clinical Features
l Gingiva fibrotic, deep pigmented and devoid of stippling.
l The lesion occurs in young individuals especially
under 30 years of age and female predilection is
Histopathology present.
Microscopically, OSMF reveals following features: l Central giant cell granuloma affects the mandible more
l Overlying hyperkeratinized, atrophic epithelium often often than maxilla and occurs more commonly in ante-
shows flattening and shortening of rete pegs. rior segments of the jaws.
l There can be variable degrees of cellular atypia or epi- l The central giant cell granuloma presents no signs or
thelial dysplasia. symptoms and may be discovered accidentally.
l In OSMF, dysplastic changes are found in epithelium l It is slowly enlarging bony hard swelling of jaw which
which include nuclear pleomorphism, severe intercel- is painful on palpitation.
lular oedema, etc. l The lesion causes expansion and distortion of cortical
l The stromal blood vessels are dilated and congested and plates and there is presence of displacement or mobility
there can be areas of haemorrhage. and root resorption of regional teeth.
l The underlying connective tissue stroma in advanced l Granuloma following an aggressive course produces
stage of disease shows homogenization and hyaliniza- fast enlarging, large, painful swelling in the jaw.
tion of collagen fibres. l Thus depending on clinical and radiographic features,
l Decreased number of fibroblastic cells and narrowing of central giant cell granuloma can be classified into non-
blood vessels due to perivascular fibrosis are present. aggressive and aggressive types.

Treatment Histopathology
l Stoppage of all causative habits. l The central giant cell granuloma exhibits loose fibril-
l Definitive treatment of OSMF includes intralesional injec- lar connective tissue stroma, consisting of numerous
tion of collagenase, corticosteroids and fibrinolysin, etc. stromal cells which are plum and spindle-shaped and
l Systemic administration of steroids in severe cases. undergo frequent mitosis.
444 Quick Review Series for BDS 3rd Year

l Several areas of haemorrhage and hemosiderin pigmen- Aetiology


tations are also evident.
l Physical trauma
l Multiple multinucleated giant cells of varying size are
l Alcohol
dispersed throughout fibrous tissue stroma.
l Tobacco smoking
l The giant cells are found around the blood capillaries
l Syphilis, sepsis, chronic dental trauma
or near the area of haemorrhage and giant cells consist
l Chronic superficial glossitis
of 5 - 20 nuclei.
l Small foci of osteoids are often found near periphery of
lesion.
l Little amount of chronic inflammatory cell infiltration
Clinical Features
in connective tissue stroma. l Commonly seen in the age groups of 40 to 60 years.
l Males are more affected than females.
It is a painless growth in early stages.
Radiological Features l

l Becomes painful after ulceration.


l Radiographically, the lesion produces multilocular ra- l Enlarged lymph nodes.
diolucent area in jaw with soap-bubble appearance. l Excessive salivation, foetor oris, sore throat, immobility
l The margin of lesion is scalloped and well demar- of tongue, hoarseness of voice and dysphagia.
cated. l Carcinoma of tongue is seen in four different varieties:
l Resorption of roots of nearby teeth or divergence of i. Ulcerative: Seen near the edge of tongue with ir-
roots is common feature. regular, raised and everted edges, base is indurated,
floor is yellowish grey slough.
ii. Warty growth: Broad and indurated base.
Treatment
iii. Indurated plaque or mass.
l Surgical excision is done with curettage. iv. A fissure: It is a chronic fissure which does not heal.
l If aggressive, then removal of section of bone in-
volved.
Q. 7. Describe the clinical features and histopathology
Spread of Carcinoma
of epidermoid carcinoma of oral cavity. Local spread by infiltration and invasion
Lymphatic spread
Or
Blood stream
Describe the aetiology, clinical features and histopathol-
ogy of oral squamous cell carcinoma.
Treatment
Ans.
Surgery,
l Synonyms of oral squamous cell carcinoma are epider-
Radiotherapy
moid carcinoma or epithelioma.
l It is most common oral carcinoma with 60% cases aris-
ing from anterior two-thirds of tongue and remainder Prognosis
from the base of the tongue.
l It is a malignant neoplasm exhibiting squamous differ- The 5 years survival rate of cancer tongue is not more
entiation. than 25%.

SHORT ESSAYS
Q. 1. Tori. Aetiology
l Genetic or environmental
Ans.
l Masticatory stresses
l Tori is also called as mandibular torus.
l It is an exostosis or outgrowth of bone found on the Clinical Features
lingual surface of the mandible. l Occurs in 8% of population
l It consists primarily of compact bone. l Seen in middle-aged
Section | III Oral Pathology 445

l It may occur singly, multiply, unilaterally but is bilateral l Management: No treatment as it has no premalignant
in premolar region. potential.
l There is growth on the lingual surface of mandible above
Q. 3. Give clinical and histological features of speckled
mylohyoid line, usually opposite to bicuspid teeth.
and homogenous types of erythroplakia.
l Size is variable ranging from an outgrowth to the one
that contacts opposite torus. Ans.

Clinical Features
Radiographic Features
l Radio-opaque shadow superimposed over roots of pre- Homogenous erythroplakia
molars and molars, occasionally on incisors. l Commonly found on buccal mucosa and soft palate and
l Margins are sharply demarcated anteriorly on the peri- rarely on tongue and floor of mouth.
apical film and less dense and less well defined as they l Appears as a bright red, soft velvety lesion with straight
extend posteriorly. or scalloped, well-demarcated margins.
l The shape of the shadow tends to be oval with long axis l It is quite extensive in size.
in the posteroanterior direction. l Margins: It is quite sharply demarcated from the sur-
l On occlusal radiograph, tori appear as radio-opaque, rounding pink mucosa
homogenous, knobby protruberances from lingual sur- l Surface is typically smooth and regular in colouration.
face of mandible.
Speckled erythroplakia
l It is a type of erythroplakia based on clinical appear-
Treatment ance. It is also called as granular erythroplakia.
l It is soft to palpation and has often been described as
l Does not require any treatment.
having a velvety feel.
l Removal advised, if mandibular denture is planned.
l The pebbled lesions tend to be somewhat firm, but
Q. 2. Leukoedema. erythroplakia never actually becomes hard or indurated,
until an invasive carcinoma develop within it.
Ans.
Leukoedema is an abnormality of buccal mucosa, which Histopathology
clinically resembles early leukoplakia.
l Multicentric in origin.
l Severe epithelial dysplasia is seen.
Aetiology l Carcinoma in situ or superficial invasive lesion.
l Tobacco l Lack of keratin production.
l Racial predilection l Atrophic epithelium and sometimes hyperplastic
l Poor oral hygiene epithelium.
l Lack of keratinization and thining of epithelium allows
underlying vasculature to be seen.
Clinical Features
Q. 4. Histology of leukoplakia.
l Common in age group of 15 to 35 years of age.
l Male predilection in ratio of 2:1. Ans.
l Common sites are buccal mucosa and lip. l Leukoplakia exhibits variety of histologic changes
l The lesion is usually bilateral in majority of the cases. which are related to keratinization pattern, changes in
l Buccal mucosa retains the normal softness and flexibil- cellular layer, thickness of epithelium and alteration in
ity but exhibits greyish white, slightly folded opalescent underlying connective tissue stroma.
appearance that is described as epithelium covered with
diffuse oedematous film or velvet-like feel and mother
Keratinization Pattern
of pearl appearance.
l Signs: Desquamation leaves the surface of lesion eroded. l Leukoplakia generally presents hyperorthokeratiniza-
l It can be eliminated by stretching and scraping of mu- tion or hyperparakeratinization or both with or without
cosa but re-establishes itself almost immediately. the presence of epithelial dysplasia.
l Differential diagnosis: Leukoplakia, cheek biting lesion, l In case of leukoplakia, an abnormal increase in the
white sponge nevus, hereditary benign intraepithelial thickness of orthokeratin layer is seen in area of epithe-
dyskeratosis. lium which is usually keratinized.
446 Quick Review Series for BDS 3rd Year

l An important histological criterion of leukoplakia is l Management: Surgical excision, if irritated by clothing


presence of hyperkeratinization of normally keratinized or suddenly increase in size, deepen in colour or be-
epithelium or some amount of parakeratin deposition in come ulcerated.
area of epithelium which are usually not keratinized.
Q. 6. Osteogenic sarcoma.
l Epithelial dysplasia is more frequently associated with
hyperkeratinized lesion. Ans.
l Osteogenic sarcoma is the most common malignant
Changes in cellular layer tumour of bone.
l Epithelial dysplasia is the hallmark of histologic changes l It is derived from osteoblasts in which tumour cells
seen in epithelium in case of leukoplakia. contain high level of alkaline phosphatase.
l The histopathological features of epithelial dysplasia are:
i. Loss of polarity of basal cells
Aetiology
ii. Increased nuclear cytoplasmic ratio
iii. Irregular epithelial stratification i. Following radiation therapy
iv. Cellular pleomorphism ii. Traumatic irritation
v. Nuclear hyperchromatism iii. Fibro-osseous diseases like fibrous dysplasia and Paget’s
vi. Reduction of cellular cohesion disease
vii. Enlarged nucleoli iv. Genetic mutation
viii. Increased number of mitotic figures. v. Viral causes

Thickness of the Epithelium Clinical Features


l Common in males in age group of 30’s.
In leukoplakia, the thickness of epithelium is altered and it
l Common in long bones like femur and tibia and in jaw
occurs in the form of epithelial atrophy or acanthosis.
bones.
Alteration in underlying connective tissue
l Grows rapidly and shows early metastasis via blood-
In leukoplakia, there is often variable degree of destruction
stream.
of collagen fibres and moreover chronic inflammatory cell in-
l Signs of exophthalmos, blindness, nasal obstruction and
filtrate is also present in underlying connective tissue stroma.
epistaxis.
Q. 5. Blue nevus. l Oral manifestations are rare, maxilla and mandible are
equally affected.
Ans.
l Affected tooth is displaced or loose with swelling.
l Nevus is a general term that refers to any congenital le- l There may be trismus and haemorrhage.
sion of various cell types or tissue types.
l Blue nevus is a type of nevomelanocytic nevus, in Radiologic Features
which histologically collection of pigmented spindled
nevus cells are found deep in the connective tissue, i.e. l Radiographic appearance is variable. It may develop into
submucosa. frankly osteolytic stage, mixed stage and entirely osteo-
l Majority of blue nevi are found at birth and early child- blastic; mixed stage shows honey comb appearance.
hood and they persist unchanged throughout life. l Osteogenic stage shows sunray appearance, Codman’s
l The lesions are smooth and exhibit hair growing from triangle and onion peel appearance
surface.
l The colour occurs as the melanocytes reside deep in Differential Diagnosis
connective tissue and overlying vessels dampen the
brown colouration of melanin and thus yield a blue tint. Carcinoma, chondrosarcoma, fibrous dydplasia, osteomyelitis.
l It is composed of dermal melanocytes, which rarely
undergo malignant transformation. Treatment
l Orally, they may be macular or nodular in appearance.
Radical resection with amputation of bone.
l Histologically, elongated melanocytes with long branch-
ing dentritic processes lie in bundles and there is no Q. 7. Oral submucous fibrosis.
junctional activity.
Ans.
l It has stellate and fusiform cells that contain melanin
and are located deep in lamina propria. Few pigmented Oral submucous fibrosis is a precancerous condition
macrophages may be present. arising from oral cavity. The lesion is characterized by
Section | III Oral Pathology 447

juxta-epithelial inflammatory reaction in oral mucosa fol- l There can be variable degrees of cellular atypia or epi-
lowed by fibroelastic transformation of lamina propria lead- thelial dysplasia.
ing to mucosal atrophy, rigidity and trismus. l In OSMF, dysplastic changes are found in epithelium
which include nuclear pleomorphism, severe intercel-
lular oedema, etc.
Aetiology l The underlying connective tissue stroma in advanced
The OSMF is caused due to: stage of disease shows homogenization and hyaliniza-
l Excessive consumption of red chillies. tion of collagen fibres.
l Excessive “areca nut” chewing. l Decreased number of fibroblastic cells and narrowing of
l Nutritive deficiency blood vessels due to perivascular fibrosis are present.
l Immunological factors
l Genetic factors
l Protracted tobacco use Treatment
l Patient with deficiency of micronutrients. l Stoppage of all causative habits.
l Definitive treatment of OSMF includes intralesional
Pathogenesis injection of collagenase, corticosteroids and fibrino-
lysin, etc.
Aetiological factors cause an alteration in the oral mucosa l Systemic administration of steroids in severe cases.
which increases the risk of hypersensitivity reaction to
many potential irritants resulting in oral mucous membrane Q. 8. Capillary haemangioma.
changes which may lead to malignant transformation up to Ans.
4.5-7.6%.
Haemangiomas are benign endothelial cell neoplasms that
are typically absent at birth and characteristically have
Clinical Features rapid growth in infancy with spontaneous involution later
l It is caused during 20 to 40 years of age. in life.
l Females are affected more often than males.
In OSMF, fibrotic changes are frequently seen in buccal
Capillary Haemangioma
l

mucosa, retromolar area, uvula, tongue, etc.


l Initially, patient complains of burning sensation in the l Age: They are one of the most common benign tumours
mouth, particularly during taking hot and spicy foods. of infancy (6 months).
l There can be excessive salivation, decreased salivation l Sex: Female patients outnumber male patients with
and defective gustatory sensation. haemangiomas by a ratio of 3:1.
l In initial phase of disease, palpation of mucosa elicits a l Capillary haemangiomas are believed to be hamartoma-
“wet leathery” feeling. tous proliferations of vascular endothelial cells.
l Patient complains of vesiculation, ulceration, recurrent l They are now thought to be of placental origin due to a
stomatitis, pigmentation changes. unique microvascular phenotype shared by juvenile
l In advanced stage, the oral mucosa loses its resilience haemangiomas and human placenta.
and becomes blanched and stiff and thereby causing l They generally exhibit 2 phases of growth, a prolifera-
trismus. tive phase and an involutional phase.
l Lips become leathery and difficulty in everting them, l The proliferative phase of rapid growth typically occurs
i.e. microchelia. from 8-18 months. Pathologically, it is characterized by
l Opaque marble-like oral mucosa an increased number of endothelial and mast cells, the
l Deglutition is difficult with referred pain in ear, circular latter being a stimulus for vessel growth. Endothelial
fibrous bands around rima oris. cell proliferation returns to normal following the prolif-
l Palpation of mucosa often reveals vertical fibrous bands. eration phase.
l Uvula gets shrunken and gives a hockeystick appear- l The involutional phase is characterized by slow regres-
ance. sion of the haemangiomas. One-half of all lesions will
involute by age 5 years, and 75% will involute by age
7 years. During this phase, mast cell numbers decrease
Histopathology to normal and there is a decrease in endothelial and mast
Microscopically, OSMF reveals following features: cell activity. These vascular spaces become lined with
l Overlying hyperkeratinized, atrophic epithelium often endothelial cells without muscular support.javascript:sh
shows flattening and shortening of rete pegs. owrefcontent(‘refrenceslayer’);
448 Quick Review Series for BDS 3rd Year

Types of Capillary Haemangioma Q. 11. Briefly write about verrucous carcinoma.


i. Salmons patch: Midline or forehead Ans.
ii. Portwine stain: Head and neck
l Verrucous carcinoma is a diffused papillary, non-
iii. Strawberry angioma: Compressible
metastasizing well-differentiated malignant neoplasm
of oral epithelium.
Treatment l It is also known as Ackerman’s tumour or pseudoepi-
thallomatous papilloma.
Wait and watch phenomenon is best.
Q. 9. Carcinoma in situ. Aetiolgy
Ans. Ill-fitting dentures
l Carcinoma in situ is also called as intraepithelial carci- Tobacco
noma.
l Carcinoma in situ (CIS) is an early form of carcinoma Clinical Features
defined by the absence of invasion of surrounding tissues.
l Occurs during 60 years and males are more commonly
l It is a condition which frequently arises on skin but
affected.
occurs also on mucous membranes.
l The common locations for verrucous carcinoma are
l In other words, the neoplastic cells proliferate in their
gingiva, alveolar mucosa and buccal mucosa.
normal habitat, hence the name “in situ” (Latin for “in
l Verrucous carcinoma presents as slow growing, exo-
its place”).
phytic, papillary growth having white pebbly surface.
l For example, carcinoma in situ of the skin, also called
l The carcinoma occurs either as single entity or there can
Bowen’s disease, is the accumulation of neoplastic epi-
be multiple lesions involving different parts of oral cavity.
dermal cells within the epidermis only.
l The lesions on buccal mucosa are sometimes very exten-
l It represents a precancerous dyskeratotic process or
sive and often cause pain, tenderness and difficulty in
laterally spreading, intraepithelial type of superficial
taking the food.
epithelioma or carcinoma.
l The regional lymph nodes are often enlarged and tender.
l CIS is usually treated in much the same way as a malig-
l Less aggressive than squamous cell carcinoma.
nant tumour.
l Lateral invasion is more and vertical invasion is less.
l Carcinoma in situ is synonymous with high-grade dys-
l Keratinized and whitish surface gives the lesion pebbly
plasia in most organs. The risk of transforming into
appearance.
cancer is high.
Q. 10. Briefly explain changes in epithelial dysplasia. Treatment
Ans. l Surgical excision or laser therapy is done.
Dysplasia means disordered cellular development. Epithe- Q. 12. Burkitt’s lymphoma.
lial dysplasia is characterized by cellular proliferation and
cytological changes. Ans.
The histopathological features of epithelial dysplasia l Burkitt’s lymphoma is also called as African jaw lym-
are as follows: phoma and was described by Dennis Burkitt in 1958.
l Loss of polarity of basal cells
l Increased nuclear cytoplasmic ratio
l Irregular epithelial stratification Aetiology
l Cellular pleomorphism EBV virus
l Nuclear hyperchromatism
Reduction of cellular cohesion
l
Clinical Features
l Enlarged nuclei and cells
l Enlarged and prominent nucleoli l In Africa lymphoma accounts for 50% of childhood
l Increased number of mitotic figures. malignancies while in USA and Europe, it accounts for
l Dyskeratosis 6-10%.
l Basilar hyperplasia l The endemic form of Burkitt’s lymphoma has a peak
l Tear drop rete pegs incidence between 3 and 8 years of age and a 2-to -1
l Poikilokaryosis. male predominance.
Section | III Oral Pathology 449

l The sporadic form affects slightly older age group with a Aetiology
mean age around 11 years and has no gender predilection.
l Physical trauma
l Endemic Burkitt’s lymphoma typically involves the
l Alcohol
mandible, maxilla and abdomen with extranodal in-
l Tobacco smoking
volvement of the retroperitoneum, kidneys, liver, ovaries
l Syphilis, sepsis, chronic dental trauma
and endocrine glands.
l Chronic superficial glossitis
l Maxillary molar area is more commonly involved.
l Peripheral lymphadenopathy is common.
l It is a rapidly growing tumour mass of jaws destroying Clinical Features
the bone extending into maxillary, ethmoid and sphe- l Commonly seen in the age groups of 40 to 60 years.
noid sinus as well as orbit. l Males are more affected than females.
l It is a painless growth in early stages.
Oral Manifestations l Becomes painful after ulceration.
l Enlarged lymph nodes.
l Mobility and loss of teeth with asymmetrical face due to
l Excessive salivation, foetor oris, sore throat, immobility
gross swelling.
of tongue, hoarseness of voice and dysphagia.
l Gingiva and mucosa are swollen, ulcerated and necrotic.
l Carcinoma of tongue is seen in four different varieties:
l Derangement of arch and occlusion.
i. Ulcerative: Seen near the edge of tongue with ir-
regular, raised and everted edges, base is indurated,
Histopathology floor is yellowish-grey slough.
l It is a lymphoreticular cell malignancy, especially it is a ii. Warty growth: Broad and indurated base.
B cell neoplasm. iii. Indurated plaque or mass
l Burkitt’s lymphoma is a neoplastic B cell proliferation iv. A fissure: It is a chronic fissure which does not heal.
that cantains cell-surface B-lineage differentiation anti-
gens and monoclonal surface immunoglobulin. Spread of Carcinoma
l Throughout the lymphoid proliferation are the numer- l Local spread by infiltration and invasion
ous scattered macrophages containing nuclear debris, l Lymphatic spread
contributing to the so-called starry sky appearance. l Blood stream
l Histologic differential diagnosis includes other sub-
types of Non-Hodgkin’s lymphoma, undifferentiated
carcinoma and sarcoma, metastatic neuroblastoma and Treatment
acute leukaemia. Surgery
Radiotherapy
Management
l Burkitt’s lymphoma was at once invariably fatal within
Prognosis
4 to 6 months of diagnosis. The 5 years survival rate of cancer tongue is not more than 25%.
l Because of high proliferation rate, it is proved to be
Q. 14. Gaint cell granulomas.
extremely sensitive to combination chemotherapy and
is, therefore, potentially curable. Ans.
l The over-all 2-year survival rate with combination che-
Gaint cell granuloma is of two types: Central and peripheral.
motherapy is 55%, with a range of 80% for low stage
disease and 40% for advanced stage disease.
Q. 13. Squamous cell carcinoma.
CENTRAL GIANT CELL GRANULOMA
Central giant cell granuloma is relatively a common benign
Ans.
intraosseous destructive giant cell lesion which affects an-
l Synonyms of oral squamous cell carcinoma are epider- terior part of jaw bone.
moid carcinoma or epithelioma. It is more aggressive in nature.
l It is most common oral carcinoma with 60% cases aris-
ing from anterior two-thirds of tongue and remainder
from the base of the tongue.
Clinical Features
l It is a malignant neoplasm exhibiting squamous differ- l The lesion occurs in young children and female predi-
entiation. lection is present.
450 Quick Review Series for BDS 3rd Year

l Central giant cell granuloma affects the mandible more l Its site in dentulous patient is interdental papilla. Man-
often than maxilla and occurs anterior to first molar. dible is more frequently affected than maxilla. Periph-
l The central giant cell granuloma is a small, slow en- eral giant cell granuloma appears as a small, exophytic,
larging bony hard swelling of jaw which is painful on well circumscribed, pedunculated lesion on gingival
palpation. surface.
l The lesion causes expansion and distortion of cortical l The colour of lesion varies from purplish-red to
plates and there is presence of displacement or mobility darkish-red.
of regional teeth. l There can be bleeding from the surface of either spon-
l Granuloma follows an aggressive course and in such taneously or on provocation from instrument. Some-
cases they produce fast enlarging, large, painful swell- times the peripheral cell granuloma can be aggressive in
ing in the jaws. nature and such lesion may attain very large size and
involves some teeth.
In some cases, the lesion may develop with an ‘hour-
Histopathology l

glass shape.’
l The central giant cell granuloma exhibits fibrovascular
connective tissue stroma, consisting of numerous stromal Histopathology
cells which are plum and spindle-shaped and undergo
frequent mitosis. l The overlying covering epithelium is ulcerated with
l Several areas of haemorrhage and haemosiderin pig- areas of haemorrhage.
mentations are also evident. l The underlying connective tissue stroma reveals numer-
l Multiple multinucleated giant cells of varying size are ous proliferating fibroblasts, blood capillaries and
dispersed throughout fibrous tissue stroma. multi-nucleated giant cells, which are scattered through-
l The giant cells are found around the blood capillaries or out the lesion.
near the area of haemorrhage and giant cells consist of l The fibroblasts present in hypercellular stroma are
5 - 20 nuclei. spindle-shaped and have oval-shaped nuclei.
l Small foci of osteoids are often found near periphery of l The giant cells are large in size and contain more number
lesion. of nuclei as compared to true giant cell tumour.
l Little amount of chronic inflammatory cell infiltration l Areas of haemorrhage and haemosiderin pigment are
in connective tissue stroma. present within connective tissue stroma.
Treatment: Surgical excision.
Radiological Features Q. 15. TNM classification.
l Radiographically, the lesion produces multicellular radio- Ans.
lucent area in jaw with soap-bubble appearance.
l The margin of lesion is scalloped and well demarcated. In oncology, an international system for staging malig-
l Resorption of roots of nearby teeth or divergence of nancy which measures 3 major parameters of a cancer are
roots is common feature. as follows:
T–Size or extent of the primary tumour.
N–Regional lymph nodes.
Treatment M–Distant metastasis.
l Surgical excision is done with curettage.
l If aggressive removal of section of bone involved. TNM Classification of Lip and Oral Cavity
for Staging Malignancy
PERIPHERAL GIANT CELL GRANULOMA T–Primary tumour:
Peripheral giant cell granuloma is the most common of Tx: Primary tumour cannot be assessed.
giant cell lesions which arises from tooth-bearing areas T0: No evidence of primary tumour
of jaw and appears as a purplish-red nodule. Tis: Carcinoma in situ
T1: Tumour 2 cm or less in greatest dimension.
T2: Tumour more than 2 cm but not more than 4 cm in
Clinical Features greatest dimension.
l The lesion usually arises during mixed dentition or T3: Tumour more than 4 cm in greatest dimension.
during third and fourth decade of life. T4: Lip: Tumour invades adjacent structures, e.g. through
l It is most common in males. cortical bone, tongue, and skin of neck.
Section | III Oral Pathology 451

Oral cavity: Tumour invades adjacent structures, e.g. through It is an intresting solitary lesion of bone given by Jaffe and
cortical bone, into deep (extrinsic) muscle of tongue, maxil- Lichtenstein in 1942.
lary sinus, and skin.

N–Regional lymph nodes: Clinical Features


Nx: Regional lymph nodes cannot be assessed. l Age: Young persons of age under 20 years.
N0: No regional lymph node metastasis. l Found equally in males and females and seen more in
N1: Metastasis in a single ipsilateral lymph node, 3 cm or mandible.
less in greatest dimension. l Associated often with the history of traumatic injury
N2: Metastasis in a single ipsilateral lymph node, more preceding development of lesion.
than 3 cm but not more than 6 cm in greatest dimension. Or l It occurs generally in long bones and vertebral column.
in multiple ipsilateral lymph nodes none more than 6 cm in l Lesions are also seen frequently in clavicle, rib, skull
greatest dimension. and bones of hands and feet.
N2a: Metastasis in a single ipsilateral lymph node, more l Symptoms: Lesions are usually tender with swelling
than 3 cm but not more than 6 cm in greatest dimension. and hence limit the movement of affected bone.
N2b: Metastasis in multiple ipsilateral lymph nodes none l Excessive bleeding is encountered on entering the
more than 6 cm in greatest dimension. lesion at the time of operation.
N2c: Metastasis in multiple ipsilateral lymph nodes none l Hence, the tissue has been described as blood-soaked
more than 6 cm in greatest dimension. sponge with large pores representing cavernous spaces
N2c: Metastasis in bilateral or contralateral lymph nodes, of the lesion
none more than 6 cm in greatest dimension.
N3: Metastasis in a lymph node, more than 6 cm in greatest Radiographically
dimension.
Honeycomb or soap-bubble appearance.
Note: Midline nodes are considered ipsilateral nodes.

M–Distant metastasis: Histologially


Mx: Presence of distant metastases cannot be assessed. Fibrous connective tissue stroma containing many cavern-
M0: No distant metastases. ous or sinusoidal blood-filled spaces.
M1: Distant metastases.
Q. 16. Aneurysmal bone cyst. Treatment
Ans. l Surgical curettage or excision.

SHORT NOTES
Q. 1. Bence-Jones protein. l Clinical features: In age group of 3rd, 4th and 5th decades
with no sex predilection.
Ans.
l Occurs on gingiva, tongue, buccal mucosa and palate.
l Bence-Jones protein is present in urine in about 60 – 85% l They are usually painless and if injured they are painful.
of multiple myeloma patients. l They are sessile, dome-shaped or slightly pedunculated.
l This is an unusual protein which coagulates when urine l Tumour is soft and myxomatous to firm and elastic.
is heated to 40 – 60°C and then disappears when urine l Treatment: Surgical excision.
is boiled and it reappears as the urine is cooled.
Q. 3. Papilloma.
l Other diseases in which it is seen occasionally are
leukaemia and polycythaemia. Ans.
Q. 2. Fibroma. l It is common, benign neoplasm of unknown origin,
which arises from the surface of the epithelium.
Ans.
l May be caused from papilloma virus.
l It is benign soft tissue tumour found in oral cavity. l Seen in age groups of 3rd and 4th decades of life.
l True beign neoplasm of fibrous tissue is relatively an l Commonly seen on tongue, palate, buccal mucosa,
infrequent lesion. gingiva, lip, mandibular ridge, and floor of mouth.
452 Quick Review Series for BDS 3rd Year

l It is a typically an exophytic lesion with cauliflower-like l Inflammatory cell infiltration is common.


surface or with finger-like projection. l In late stage, lesion consists of well-defined nodules or
l It is generally arising from pedunculated base and is a lesions with diffuse involvement of lamina propria.
small mass on mucosa with papillomatous shape.
Q. 6. CREST syndrome.
l Lesion with keratinisation is white and firm while one
without is soft, grayish pink in colour. Ans.
l Multiple oral papillomas are seen in focal dermal hy-
Crest syndrome is usually associated with hereditary haem-
poplasia syndrome and also in others like Cowden
orrhagic telangiectasia and it includes following features:
syndrome, Down syndrome and nevus unius lateris.
C – Calcinosis cutis
l Application of formaldehyde at night on wart may cure
R – Raynaud’s phenomenon
the condition.
E –Oesophageal dysfunction
Q. 4. Leukoplakia. S – Sclerodactyly
T – Telangiectasia.
Ans.
Q. 7. Hairy leukoplakia.
Leukoplakia is a keratotic white lesion on oral mucosa that
cannot be characterized clinically or histologically as any Ans.
other disease.
i. Hairy leukoplakia is HIV-associated mucosal disorder,
which often involves lateral and ventral surfaces of
Aetiology of Leukoplakia tongue. It can also occur on floor of the mouth, buccal
mucosa, etc.
Tobacco, alcohol, candidiasis, dietary deficiency, syphilis,
ii. The lesion often appears as white patch and is charac-
trauma-like ill-fitting dentures, sharp dentures, tooth brush
terized by an irregular surface, exhibiting numerous
trauma, galvanism, idiopathic, UV radiation, etc.
linear vertical folds or projections, sometimes so
marked to as resemble “Hairs”.
Clinical Features iii. Hairy leukoplakia probably occurs as an opportunistic
infection caused by Epstein-Barr virus. The lesions are
l It occurs in 4th, 5th, 6th and 7th decades of life.
always colonized by Candida albicans.
l Buccal mucosa and commissural areas are most fre-
iv. Hairy leukoplakias are asymptomatic lesions and when-
quently affected sites followed by alveolar ridge, tongue,
ever they occur they occur on buccal mucosa.
lip, hard and soft palate, etc.
v. Histopathologically, a very characteristic finding in
l Oral leukoplakia often presents as solitary or multiple
hairy leukoplakia is presence of subcorneal upper spi-
white patches.
nus layer zone made up of cytopathologically altered
l The surface of lesion may be smooth or finely wrinkled
keratinocytes. Parakeratin layer is thick often colonized
or even rough on palpation and lesion cannot be
by candidal hyphae.
removed by scrapping.
vi. Differential diagnosis includes lichen planus, verrucous
l In most of the cases, these lesions are asymptomatic,
leukoplakia, chronic tongue biting habits.
however, in some cases they may cause pain, feeling of
thickness and burning sensation. Q. 8. Erythroplakia speckled.
Ans.
Treatment Modalities
i. Speckled erythroplakia is also known as granular eryth-
l Surgical excision, cryosurgery, CO2-laser surgery, reti- ropakia.
noids and other drugs ii. It is a type of erythroplakia based on clinical appearance.
l Photodynamic therapy iii. It is soft to palpation and has often been described as
Q. 5. Histology of Kaposi’s sarcoma. having a velvety feel.
iv. The pebbled lesions tend to be some what firm, but
Ans. erythroplakia never actually becomes hard or indurated,
Histological features of Kaposi’s sarcoma are as follows: until an invasive carcinoma develop within it.
l It consists of interweaving band of spindle-shaped and/ Q. 9. Oral submucous fibrosis.
or plump endothelial cell and atypical vascular channels,
Ans.
enmeshed in reticular or collagen fibres.
l It consists of numerous, small capillary type blood ves- i. Oral submucous fibrosis is a precancerous condition
sels which may or may not contain blood. arising from oral cavity.
Section | III Oral Pathology 453

ii. The lesion is characterized by juxtraepithelial inflam- l These are the characteristic histological feature of
matory reaction in oral mucosa followed by fibroelastic Schwann-cell tumours.
transformation of lamina propria leading to mucosal l Verocay noted a “peculiar arrangement of nuclei in
atrophy, rigidity and trismus. transverse bands.”
l These bands of fusiform nuclei alternated with clear
zones devoid of nuclei.
Aetiology l Later investigators named these structures “verocay
Mainly due to excessive consumption of red chillies. bodies” and defined them as stacked arrangements of
elongated palisading nuclei alternating with anuclear
zones containing cell processes.
Clinical Features l These verocay bodies are typically found in the more
l It is commonly seen in females at age of 20 – 40 yrs. densely packed Antoni A regions, rather than in the
l Initially patient complains of burning sensation in the loose or microcystic Antoni B areas.
mouth, particularly during taking hot and spicy foods.
Q. 12. Basal cell carcinoma.
l In initial phase of disease, palpation of mucosa elicits a
“wet leathery” feeling. Ans.
l In advanced stage, the oral mucosa loses its resilience
i. Basal cell carcinoma is mainly caused due to exposure
and becomes blenched and stiff and thereby causing
to sunlight through ultraviolet radiation.
trismus.
ii. Sites of occurrence: Exposed surface of the skin of the
l Opaque marble-like oral mucosa
head and neck, majority occur on the face above a
l Uvula gets shrunken and gives a hockeystick
line drawn from the tragus of the ear to the angle of the
appearance.
mouth and when it occurs here it is also called rodent
ulcer.
Treatment iii. It occurs in middle-aged or elderly and in males.
iv. Most common form of the lesion is noduloulecrative
l Stoppage of all causing habits.
which starts as a small, hard lump on the skin (papule)
l Intralesional injection of collagenase, corticosteroids
which slowly increases in size and breaks down to form
and fibrinolysin, etc.
an ulcer surrounded by a rolled edge.
l Systemic administration of steroids in severe cases.
v. Slow growth causes local tissue destruction involving a
Q. 10. Traumatic neuroma. large area.
vi. Treatment: Surgical excision and radiotherapy.
Ans.
Q. 13. Congenital epulis.
i. Traumatic neuroma is also known as amputation
neuroma. Ans.
ii. Aetiology: Due to injury to the nerve, e.g. during diffi-
i. Congenital epulis is a growth of gingiva in infants.
cult extraction of a tooth.
ii. This is considered similar to granular cell myoblastoma
iii. This is not a true tumour but an excessive formation of
because of its histologic picture. However, it could be a
tissue during repair of the damaged nerve trunk.
separate pathological entity.
iv. Clinical findings:
iii. Clinical features:
l It may appear as a small nodule or swelling of the
l Occurs in newborn infants and females.
mucosa near the mental foramen, on the alveolar
l The maxilla is more affected in anterior region along
ridge, on the lips or tongue.
the gum margin.
l Pain on pressing and along the course of the nerve
l Patient complains of smooth swelling showing
involved.
irregular lobulations.
l Pain may be referred to face, eye, throat or head.
l Usually pedunculated and of size less than 1- 1.5 cm
v. Histopathology: The growth consists of nerve bundles
in diameter.
growing irregularly and there is fibrous septa.
iv. Histopathology:
vi. Treatment: Surgical excision.
l The tumour consists of large, closely packed cells

Q. 11. Verocay bodies. with little stroma and with many blood capillaries.
l Cell cytoplasm contains fine acidophilic granules.
Ans.
l The pseudoepitheliomatous hyperplasia found in

l Verocay bodies are a group of uniform, fusiform cells granular cell myoblastoma may not be present.
arranged in whorls, herring bones or palisades. l No mitotic figures seen.
454 Quick Review Series for BDS 3rd Year

v. Treatment: Q. 17. Histology of papilloma.


l A benign lesion that can be easily removed by surgical
Ans.
excision.
l Does not recur. Histological features of papilloma are characteristic and are
as follows:
Q. 14. Neurofibroma. l Narrow connective tissue core extends from tissue
Ans. beneath the lesion and branches into numerous folds
which are covered by epithelium.
l Neurofibroma is also called as neurofibromatosis or von l It consists of many long, tiny, finger-like projections
Recklinghausen disease of skin or fibroma molluscum. extending from the epithelium and containing a thin,
l Inherited as a simple autosomal dominant trait with central connective tissue core, which supports the nutri-
variable penetrance. ent blood vesels.
l It arises from connective tissue sheath of Schwann cells l The presence of inflammatory cell infiltrate in supporting
and axons usually from inferior dental canal, substance fibrous connective tissue stoma is often prominent.
of bone and beneath periosteum.
l Occurs at any age and equal in males and females. Q. 18. Torus palatinus.
l Sites – trunk, face and extremities. Ans.
l The nodules are sessile, pedunculated which are scat-
tered over skin surface. l Torus palatinus is a slow growing, flat based bony pro-
l May show even café-au-lait spots. truberance or excrescence which occurs in the midline
l Orally, they occur on both jaws simultaneously, expand- of hard palate.
ing and filling maxillary sinus. Pain and paraesthesia, if l It is a hereditary condition, thought to follow mendelian
mandibular nerve is involved. dominant pattern.
l Treatment – excised, if solitary lesion. l Mostly seen in females at age group of 30 yrs.
l They may appear in varied shapes like flat, spindle,
Q. 15. Cavernous haemangioma. nodular or lobular.
Ans. l Mucosa is intact and may become ulcerated.
l It is composed of dense or shell of compact bone with a
l Haemangiomas are relatively common benign prolifera- centre of cancellous bone.
tive lesion of vascular tissue origin which may be present
either at birth or may arise during early childhood. Q. 19. Keratoacanthoma.
l Cavernous haemangioma occurs in place where venous Ans.
space is abundant, i.e. lip, cheek, tongue, posterior tri-
angle of neck. l Keratoacanthoma is also called as self-healing carci-
l Patients usually manifest symptomatically during the noma.
third to fifth decades of life. l The lesion clinically and pathologically resembles squa-
l Compressible swelling bluish warm, non-tender swelling, mous cell carcinoma.
associated with arteriovenous communication. l It is relatively common low grade malignancy that
l Treatment consists of injection of boiling water or originates in pilosebaceous glands.
hypertonic saline or sodium tetradecyl sulphate and l Sunlight is the main aetiological factor and others,
surgical excision. trauma, HPV, genetic factors, etc.
l Face, neck and dorsum of extremities are common sites.
Q. 16. Exostoses. l It is seen in all age groups and mostly in men.
Ans. l Lesions are typically solitary and begin as firm, round
skin-coloured or reddish papules that rapidly progress
l Multiple exostoses are also called as hyperostosis and to dome-shaped nodules with smooth shiny surface
are some what less common than the tori. and a central crateriform ulceration or keratin plug
l They are usually found on the buccal surface of maxilla that project like a horn.
below the mucobuccal fold in the molar region. l Treatment consists of surgical excision.
l Clinically, they appear as small nodular protruberances
over which the mucosa may appear blanched. Q. 20. Carcinoma in situ.
l Aetiology is unknown. Ans.
l They have no clinical significance and instead they may
interfere with the preparation or insertion of prosthetic l Carcinoma in situ (CIS) is an early form of carcinoma
appliance. defined by the absence of invasion of surrounding
l No treatment is required. tissues.
Section | III Oral Pathology 455

l In other words, the neoplastic cells proliferate in their l Pigmentation is ipsilateral to the side of bony lesion.
normal habitat, hence the name “in situ” (Latin for “in l They are related to increased amounts of melanin in the
its place”). basal cells of the epidermis.
l For example, carcinoma in situ of the skin, also called l They tend to appear in linear or segmented pattern near
Bowen’s disease, is the accumulation of neoplastic epi- midline of body–back, buttocks, sacral areas, neck,
dermal cells within the epidermis only. shoulders, lips and oral mucosa.
l CIS is usually treated in much the same way as a malig- l While café-au-lait spots do not cause any ailment them-
nant tumour. selves, having multiple spots has been linked with neu-
l Carcinoma in situ is synonymous with high-grade dys- rofibromatosis and the rare McCune-Albright syndrome.
plasia in most organs. The risk of transforming into l Rarely undergo malignant transformation, e.g. fibrosar-
cancer is high. comas.
Q. 21. Features of epithelial dysplasia. Q. 24. Kaposi’s sarcoma.
Ans. Ans.

The histopathological features of epithelial dysplasia are. i. Kaposi’s sarcoma is a multicentric neoplasm originating
l Loss of polarity of basal cells from endothelial cells of blood vessels.
l Increased nuclear cytoplasmic ratio ii. Clinical types: Involving skin and oral mucosa (in
l Irregular epithelial stratification elderly men). Involving lymph nodes (in children).
l Cellular pleomorphism iii. Though considered as malignant, doesn’t metastatise
l Nuclear hyperchromatism and never causes death of the patient.
l Reduction of cellular cohesion iv. Kaposi’s sarcoma is common in HIV infection and so
l Enlarged nuclei and cells its lesions in AIDS patients is of diagnostic value.
l Enlarged and prominent nucleoli v. Oral lesions may be red, blue or purple nodules and
l Increased number of mitotic figures lesions may also begin as flat, red macules of different
l Dyskeratosis size and shape which finally coalesce to form plaques.
l Basilar hyperplasia vi. Nodular growth may involve the whole of palate and
l Tear drop rete pegs extend below the plane of occlusion. The lesions do not
l Poikilokaryosis. blanch with pressure and may ulcerate.

Q. 22. Reed-Sternberg cells. Treatment: Electrocautery, conventional excision of the


lesion, intralesional injection of chemotheraputic agents
Ans. like 1% vinblastine sulfate.
l Reed-Sternberg cells are characteristic malignant cells Q. 25. Verrucous carcinoma.
of Hodgkin’s disease.
l They are large cells of 20 – 50 micrometres in diameter, Ans.
abundant, amphophilic, finely granular or homogenous l Verrucous carcinoma is a diffused papillary, non-
cytoplasm. metastasizing well-differentiated malignant neoplasm
l They show 2 mirror image nuclei – owl eyes each of oral epithelium.
with an eosinophilic nucleolus and a thick nuclear
membrane. Clinical Features
l They are identified as large binucleated cells with
prominent nucleoli and an unusual CD151, CD301 l It occurs during 60 years of age and males are more
immunophenotype. commonly affected.
l They are frequently infected by EB virus in 50% of l The common locations for verrucous carcinoma are
cases of Hodgkin’s disease. gingiva, alveolar mucosa and buccal mucosa.
l Verrucous carcinoma presents as slow growing, exo-
Q. 23. Cafe-au-lait spots. phytic, papillary growth having white pebbly surface.
Ans. l The lesions on buccal mucosa are sometimes very ex-
tensive and often cause pain, tenderness and difficulty in
l Café-au-lait spots or Café-au-lait macules are pig- taking the food.
mented birthmarks. l Lateral invasion is more and vertical invasion is less.
l The name café-au-lait is French for “milky coffee” and
refers to their light-brown colour. They are also called
Treatment
“giraffe spots.”
l These are most common extraskeletal manifestation. Surgical excision or laser therapy is done.
456 Quick Review Series for BDS 3rd Year

Q. 26. Rodent ulcers. l A junctional nevus is a mole found in the junction


(border) between the epidermis and dermis layers of
Ans.
the skin.
i. Rodent ulcer is also called as basal cell carcinoma. l These moles may be brown or black pigmented and
ii. Aetiology: Exposure to sunlight through ultraviolet ra- slightly raised, and have a higher risk of developing into
diation. malignant melanoma.
iii. Site of occurrence: Exposed surface of the skin of the l It is less common as compared to intradermal nevi.
head and neck. Majority occur on the face above a line l It occurs early in life and occurs from basal layers of
drawn from the tragus of the ear to the angle of the melanocytes.
mouth and when it occurs here it is called rodent ulcer. l It affects both skin and oral mucosa.
iv. This is rare in the oral cavity proper.
Q. 29. Tori.
v. Clinical features:
l It occurs in middle-aged or elderly and in males Ans.
l Most common form of the lesion is noduloulecrative
l It is an exostosis or outgrowth of bone found on the
which starts as a small, hard lump on the skin (pap-
lingual surface of the mandible.
ule) which slowly increases in size and breaks down
l It consists primarily of compact bone.
to form an ulcer surrounded by a rolled edge.
l Aetiology: Genetic or environmental, masticatory stresses.
l Slow growth causes local tissue destruction involving
l It is seen in middle-aged individuals and appears as
a large area.
singly, multiply, unilaterally but is bilateral in premolar.
vi. Treatment: Surgical excision and radiotherapy.
l There is growth on the lingual surface of mandible
Q. 27. Radiological findings of multiple myeloma. above mylohyoid line, usually opposite to bicuspid
teeth.
Ans.
l No treatment is required and its removal is advised, if
l Multiple myeloma is seen bilaterally and in mandibular mandibular denture is planned.
posterior region and ramus.
Q. 30. Hamartomas.
l Appears as sharply punched-out appearance in a variety
of bones, which may include vertebrae, ribs, skull, jaws Ans.
and ends of long bones.
l A hamartoma is a benign, focal malformation that re-
l Lesion size may range from few millimetres to a centi-
sembles a neoplasm in the tissue of its origin.
metre or more in diameter.
l This is not a malignant tumour, and it grows at the same
l Teeth may appear radio-opaque due to mineral loss and
rate as the surrounding tissues.
hence has osteopenic back ground.
l It is composed of tissue elements normally found at that
l Diffuse destructive lesions of bone may also occur.
site, but which are growing in a disorganized mass.
Q. 28. Junctional nevus. l They occur in many different parts of the body and are
most often asymptomatic and undetected unless seen on
Ans.
an image taken for another reason.

Topic 3

Tumours of Salivary Glands

LONG ESSAYS
Q. 1. Describe clinical features, histological features of l Mucoepidermoid carcinoma is a malignant epithelial
mucoepidermoid carcinoma. tumour which is composed of both mucus secreting
Ans. cells and epidermoid type cells.
Section | III Oral Pathology 457

l Most common neoplasm found in major and minor High grade tumour:
salivary glands, occurs mostly in the parotid gland. l Cells are present as cords and nests.
l Lips, buccal mucosa, tongue, retromolar areas can also be l Prominent nuclear pleomorphism and mitotic activity is
involved. Intraorally, palate is more commonly affected. noted.
l Cysts are lesser.
l Necrosis and perineural invasion may be present.
Clinical Features
l Female predilection Q. 2. Classify salivary gland tumours; write about histo-
l Usually occurs in the 3rd or 5th decades of life. genesis, clinical features of pleomorphic adenoma.
l Clinically, the tumour resembles pleomorphic adenoma.
Or
a. Low grade tumour:
Enumerate benign tumours of salivary glands; describe
l Is not completely encapsulated and does not increase clinical and histopathological features of pleomorphic
more than 5 cm in diameter. adenoma.
l More often contains cysts because of this feature, they
resemble mucocele, especially the ones on the retromo- Ans.
lar pads.
Classification of salivary gland tumours is as follows:
b. High grade tumour:
l It grows rapidly and produces pain. A. Epithelial tissue neoplasm:
l It is not encapsulated and hence metastasis to regional i. Adenoma:
lymph nodes and distant metastasis to lung, bone, brain l Pleomorphic adenoma
and subcutaneous tissues are also common. l Monomorphic adenoma
l Facial nerve paralysis is frequent in parotid tumours. l Adenolymphoma
l Patient also complains of trismus, drainage from the ear, l Oxyphil adenoma
dysphagia, numbness of the adjacent areas and ulcer-
ation, mostly in the minor salivary glands.
ii. Other types:
l Mucoepidermoid tumour
Histological Features l Acinic cell tumour
l Carcinoma
The tumour is composed of mucus secreting cells, epider-
l Adenoid cystic carcinoma
moid type cells, and intermediate cells.
l Adenocarcinoma
a. Mucus cells are of different shapes and have abundant
l Epidermoid carcinoma
foamy cytoplasm.
l Undifferentiated carcinoma
b. Epidermoid cells have squamoid features, polygonal
l Carcinoma in situ
shape, intercellular bridges, and rarely keratinization.
c. Intermediate cells are the basaloid cells that recognize
B. Connective tissue neoplasms:
the mucoepidermoid carcinoma.
l Fibroma
Some times clear cells may be present which are devoid l Fibrosarcoma
of mucin and glycogen. l Lipoma
l Histologically also the tumours can be graded into low l Neurilemmoma
grade, intermediate grade and high grade. l Haemangioma
l Melanoma.
Low grade tumour:
l Lymphoma
l Well formed glandular structures.
l Prominent cystic spaces. BENIGN TUMOURS of salivary glands are as follows:
l Lesser cellular atypia. l Pleomorphic adenoma
l More mucus cells. l Monomorphic adenoma
l Warthin’s tumour
Intermediate grade tumour: l Basal cell tumour
l Less prominent cyst formation. l Canalicular adenoma
l Intermediate cells predominate. l Necrotizing sialometaplasia
458 Quick Review Series for BDS 3rd Year

PLEOMORPHIC ADENOMA Microscopic


Morphologic diversity is the characteristic feature.
Histogenesis l

l Types
l Many theories have been put forward. But the latest is i. Myxoid
focussed on the myoepithelial and the reserve cell in the ii. Myxoid and cellular
intercalated duct. iii. Predominantly cellular
l Ductal and myoepithelial cells are present in this iv. Extremely cellular
tumour. l The epithelial component forms ducts and small cysts,
l The myoepithelial cell is responsible for the morpho- containing coagulum.
logic diversity of the tumour, the production of fibrous, l May also occur as cellular nests, sheets of cells, cords
mucinous, chondroid and osseous areas. or spindle cells.
l The intercalated duct reserve cell can differentiate into l Myoepithelial cells appear as angular or spindled, some
ductal and myoepithelial cells and these myoepithelial cells are rounded with eccentric nuclei and hyalinized
cells can undergo mesenchymal metaplasia. eosinophilic cytoplasm (hyaline cells).
l The pleomorphic adenoma gene (PLAG1) has been l Vacuolar degeneration of myoepithelial cells results in
mapped to another gene – 8q12. cartilagenous appearance.
l Foci of hyalinization, bone and fat can be seen in the
connective tissue of the tumour.
Clinical Features
Q. 3. Classify tumours of salivary glands. Write in detail
l Pleomorphic adenoma is the most common tumour of about adenoid cystic carcinoma.
salivary glands.
l The parotid gland is the most commonly affected espe- Or
cially the superficial lobe of the gland.
l Very rarely affects the sublingual glands. Enumerate the malignant tumours of the salivary
l The palate is the most common site of involvement of glands. Describe in detail adenoid cystic carcinoma.
the minor salivary glands. Ans.
l Occurs more in females than in males.
l Seen in 4th to 6th decades of life with the average age Classification of salivary gland tumours is as follows:
being 43 years.
A. Epithelial tissue neoplasm
l The symptoms reported usually are small, painless,
nodule which slowly begins to increase in size. i. Adenoma:
l The tumour is mobile, firm in consistency, cystic degen- l Pleomorphic adenoma
eration can be palpated sometimes, if the tumour is su- l Monomorphic adenoma
perficial. l Adenolymphoma
l Skin over the swelling is normal. l Oxyphil adenoma
l It is a painless tumour but local discomfort is definitely
present. Facial nerve involvement with paralysis is rare. ii. Other types:
l Pleomorphic adenomas of the accessory glands do not l Mucoepidermoid tumour
grow more than 1-2 cm in diameter as they start causing l Acinic cell tumour
difficulties in breathing, talking, mastication. l Carcinoma
l The tumour in the palate appears fixed but is not invasive. l Adenoid cystic carcinoma
l Recurrent lesions occur as multiple nodules and are less l Adenocarcinoma
mobile. l Epidermoid carcinoma
l Undifferentiated carcinoma
l Carcinoma in situ
Histopathological Features
B. Connective tissue neoplasms
Macroscopic l Fibroma
l Irregular to ovoid mass with irregular borders. l Fibrosarcoma
l Fibrous incomplete capsule or unencapsulated in major l Lipoma
glands and unencapsulated in minor glands. l Neurilemmoma
l The cut surface may be rubbery, fleshy, mucoid or glis- l Haemangioma
tening. l Melanoma.
l Haemorrhage and infarction sites may be seen. l Lymphoma
Section | III Oral Pathology 459

Malignant tumours l Histologically, the low grade adenoid cystic carcinoma


The various malignant tumours of the salivary glands are: merges gradually into an extensive dedifferntiated com-
l Malignant pleomorphic adenoma ponent that is composed of solid sheets and cords of
l Adenoid cystic carcinoma anaplastic tumour cells with focal gland formation.
l Acinic cell carcinoma l Molecular studies show the presence of p53 gene muta-
l Mucoepidermoid carcinoma tion selectively in the dedifferentiated component, sug-
l Clear cell carcinoma gesting the p53 gene alteration in the dedifferentiation
process of the carcinoma.

ADENOID CYSTIC CARCINOMA


Treatment
(CYLINDROMA)
l Surgical.
l Slow growing but aggressive neoplasm. l Surgical 1 radiation.
l High capacity of recurrence. l Radiation alone is not recommended.
l Proliferation of ductal and myoepithelial cells in cribri-
form, tubular, solid and cystic patterns can be seen. Q. 4. Classify the salivary gland neoplasms. Discuss in
detail the clinical features and histopathology of Sjogren’s
syndrome.
Clinical Features
Ans.
l Most commonly involves the parotid, the submaxillary
and the accessory glands in the palate and tongue. The salivary gland tumours are classified as follows:
l Occurs during 5th and 6th decades of life.
A. Epithelial tissue neoplasm
l More common in the females.
l Symptoms include local pain, facial nerve paralysis, i. Adenomas:
fixation to deeper structures and local invasion. l Pleomorphic adenoma
l Intraoral lesions usually exhibit ulceration. l Monomorphic adenoma
l Can spread through perineural spaces. l Adenolymphoma
l Oxyphil adenoma

Histological Features ii. Other types:


l Mucoepidermoid tumour
l Adenoid cystic carcinoma is composed of myoepithelial
l Acinic cell tumour
cells and ductal cells which have various arrangements.
l Carcinoma
l Morphologically, 3 patterns are seen as follows:
l Adenoid cystic carcinoma
i. Cribriform (classic)
l Adenocarcinoma
ii. Tubular
l Epidermoid carcinoma
iii. Solid (basaloid)
l Undifferentiated carcinoma
l The cribriform pattern is the most common and the solid
l Carcinoma in situ
pattern is the least common.
l The cribriform pattern shows epithelial cell nests that B. Connective tissue neoplasms:
form multiple cylindrical cyst-like patterns resembling
a Swiss cheese or honeycomb. l Fibroma
l The tubular pattern reveals tubular structures that are l Fibrosarcoma
lined by stratified cuboidal epithelium. l Lipoma
l The solid pattern shows solid groups of cuboidal cells. l Neurilemmoma
l Haemangioma
l Melanoma
Variants l Lymphoma
Dedifferentiation of adenoid cystic carcinoma:
It is a rare variant of adenoid cystic carcinoma.
l
SJOGREN’S SYNDROME
l Characterized histologically by 2 components:
i. Conventional low grade adenoid cystic carcinoma It is a triad consisting of keratoconjunctivitis sicca, xerosto-
ii. High grade dedifferentiated carcinoma. mia, and rheumatoid arthritis.
l Due to the frequent recurrence, the clinical course is l Primary Sjogren’s syndrome (sicca complex): Only dry
short, with a predominant solid growth pattern. eyes and dry mouth.
460 Quick Review Series for BDS 3rd Year

l Secondary Sjogren’s syndrome: Primary Sjogren’s 1 l In the sicca complex, there is parotid gland enlargement
systemic lupus erythematosus, polyarteritis nodosa, which is usually absent in secondary Sjogren’s syndrome.
polymyositis, scleroderma or rheumatoid arthritis. l Lymphadenopathy is twice common in the primary
form of the disease.
Clinical Features Histological Features
l Female predilection and over 40 yrs.
Three types of histological patterns are seen in the major
l Typical features are dryness of mouth and eyes as a re-
salivary glands:
sult of hypofunction of the salivary glands and lacrimal
a) Intense lymphocytic infiltration of the gland replacing
glands.
all the acinar structures.
l Painful, burning sensation of the oral mucosa.
b) Proliferation of the ductal epithelium and myoepithe-
l Other secretory glands involved in dryness are of the
lium to form ‘epimyoepithelial islands’
nose, larynx, pharynx, tracheobronchial tree and the
c) Atrophy of the glands following the lymphocytic infiltration.
vagina.
l Sialochemistry studies have shown increased levels of Similar changes were seen in the accessory salivary glands
IgA, potassium and sodium in the saliva. in the lips.

SHORT ESSAYS
Q. 1. Mumps. Treatment
Ans. l Conservative.
l Maintaining hydration.
Mumps is an acute contagious viral infection usually seen
l Prevention is by means of vaccination.
in children.
l Characterized by unilateral or bilateral parotid swelling. Q. 2. Mucocele.
l Other than salivary glands, it may also involve nerves,
Ans.
meninges, pancreas and gonads.
l Incubation period of 2 to 3 weeks. The mucocele occurs due to retention of mucous material
due to trauma, involving salivary glands and their ducts.
Pathogenesis
Clinical Features
l It is transmitted through droplet infection.
l The virus then replicates in the respiratory epithelium l Occurs most frequently on the lower lip, can also occur
and spreads to the local lymph nodes and develops on the palate, cheek, tongue, and floor of the mouth.
viraemia. l The lesion may lie superficial or deep in the tissue.
l The affected area shows mononuclear cell infiltrates l The superficial lesion appears raised, bluish, translus-
with oedema. cent, several mm to a cm in diameter.
l Necrosis of acinar and epithelial duct cells is seen in l The deep lesion also appears as a swelling but shows no
salivary glands and in the seminiferous tubules. colour.
l The mucocele begins, reaches a certain size and will
remain that way unless treated.
Clinical Features l Recurrence is common.
l Disease preceded by headaches, chills, moderate fever,
vomiting and pain below the ear.
Histological Features
l After this firm, rubbery or elastic swelling of the sali-
vary glands elevating the ear which lasts for 1 week. l Histologically, the cavity is not lined by epithelium and
l Produces pain on mastication. is, therefore, not a true cyst. It represents a lesion of the
l The swelling reaches its maximum in 3 days and then retention type.
gradually subsides. l The wall is made of compressed fibrous connective tis-
l The papilla of the opening of the parotid duct is often sue and fibroblasts. There is infiltration of PMN leuko-
puffy and reddened. cytes, lymphocytes, and plasma cells.
Section | III Oral Pathology 461

l The lumen is filled with an eosinophilic coagulum con- Treatment


taining different cells, mainly leucocytes, and mononu-
l Symptomatic treatment.
clear phagocytes.
l Artificial tears and artificial saliva used to treat dry eyes
l Sometimes lined with an intact, flattened epithelial
and mouth, respectively.
lining, referred to as epithelium of the ‘feeder duct.’
l The salivary gland acini adjacent to the mucocele show Q. 4. Sjogren’s syndrome.
areas of interstitial inflammation, dilatation of intralob-
Ans.
ular and interlobular ducts with collection of mucus,
breakdown of acinar cells resulting in the formation of Sjogren’s syndrome is characterized by a triad of symptoms
pooled mucus areas. consisting of keratoconjunctivitis sicca, xerostomia, and
l Treatment: Excision. rheumatoid arthritis.
l Primary Sjogren’s syndrome (sicca complex)–only dry
Q. 3. Mickulicz’s disease.
eyes and dry mouth.
Ans. l Secondary Sjogren’s syndrome – primary Sjogren’s 1
systemic lupus erythematosus, polyarteritis nodosa,
l Mickulicz’s disease is a benign lymphoepithelial
polymyositis, scleroderma or rheumatoid arthritis
lesion.
l It is a chronic condition characterized by the abnormal
enlargement of the salivary and the lacrimal glands. Clinical Features
l Tonsils and glands in the soft tissue of the face and neck
l Female predilection and age of occurrence is over 40 yrs.
may also be involved.
l Typical features are dryness of mouth and eyes as a result
of hypofunction of the salivary glands and lacrimal glands.
Aetiology l Painful, burning sensation of the oral mucosa.
l Other secretory glands involved in dryness are of the nose,
l Exact cause is not known.
larynx, pharynx, tracheobronchial tree and the vagina.
l Thought to be autoimmune.
l Sialochemistry studies have shown increased levels of
The symptoms occur due to accumulation of lympho- IgA, potassium and sodium in the saliva.
cytes in the involved glands. l In the sicca complex, there is parotid gland enlargement
which is usually absent in secondary Sjogren’s syndrome.
Lymphadenopathy is twice common in the primary
Clinical Features l

form of the disease.


l Females are affected more commonly than males.
l Seen more in the middle-aged adults. Treatment
l Characterized by sudden onset of xerostomia which
leads to difficulty in swallowing and results in tooth l Symptomatic treatment.
decay. l Keratoconjunctivitis is treated with ocular lubricants
l Enlarged lacrimal glands leading to decreased or no such as artificial tears containing methylcellulose.
tears. l Xerostomia is treated by saliva substitutes.
l Painless swelling of parotid gland, submaxillary l Oral hygiene and fluoride application done to prevent
glands. and treat problems associated with dry mouth like
l Other symptoms are recurring fever accompanied by dental caries.
dry eyes, decreased lacrimation and uveitis. Q. 5. Necrotizing sialometaplasia.
Ans.
Histologic Features
It is a non-neoplastic inflammatory condition of the salivary
l Orderly lymphocytic infiltration of the salivary gland glands.
tissue, destroying and replacing the acini.
The epithelium may consist of ducts showing cellular
l
Aetiology
proliferation forming nests or clumps of poorly defined
epithelial cells which are termed as ‘epimyoepithelial In most cases, it occurs spontaneously.
islands.’ Other causes may be:
l In advanced lesion, there is deposition of eosinophilic, l Trauma
hyaline material in the epithelial islands. l Radiation therapy.
462 Quick Review Series for BDS 3rd Year

l Surgery. l The symptoms reported usually are, small, painless,


l Tobacco use. nodule which slowly begins to increase in size.
l Vascular ischaemia. l The tumour is mobile, firm in consistency, cystic de-
generation can be palpated sometimes, if the tumour
is superficial.
Clinical Features l Skin does not ulcerate most of the times.
l Usually involves the minor salivary glands, particularly l It is a painless tumour but local discomfort is definitely
the ones in the palate. It can also attack the minor glands present. Facial nerve involvement with paralysis is rare.
of the retromolar pad area, buccal mucosa, tongue, inci- l Pleomorphic adenomas of the accessory glands do not
sive canal, and labial mucosa. grow more than 1-2 cm in diameter as they start causing
l Also seen in major salivary glands and minor mucous difficulties in breathing, talking, mastication.
glands in the lung, nasal cavity, larynx, trachea, naso- l The tumour in the palate appears fixed but is not invasive.
pharynx and maxillary sinus. l Recurrent lesions occur as multiple nodules and are less
l Lesions found in the skin are called ‘syringometaplasia.’ mobile.
l Male predilection.
l The lesion manifests as a painless, swelling with or Treatment
without ulceration. l Surgical excision.
l The typical ulcer presents as a crateriform ulcer on the
palate, usually unilateral. Erosion of the palate may Q. 7. Histopathology of pleomorphic adenoma.
occur. Ans.
Histopathological features of pleomorphic adenoma:
Histological Features
Macroscopic:
l The features seen are coagulative necrosis of glandular
acini and squamous metaplasia of its ducts. l Irregular to ovoid mass with irregular borders.
l Mucin pooling is present. l Fibrous incomplete capsule or unencapsulated in major
l Inflammatory infiltrate consists of macrophages, neu- glands and unencapsulated in minor glands.
trophils, less commonly – lymphocytes, plasma cells l The cut surface may be rubbery, fleshy, mucoid or glis-
and eosinophils. tening.
l Pseudoepitheliomatous hyperplasia of overlying mu- l Haemorrhage and infarction sites may be seen.
cosa can also be present. Microscopic:
l Morphologic diversity is the characteristic feature.
Treatment l Types
l Resolves spontaneously. i. Myxoid
l The average healing time is approximately 5 weeks. ii. Myxoid and cellular
iii. Predominantly cellular
Q. 6. Pleomorphic adenoma. iv. Extremely cellular
Ans. l The epithelial component forms ducts and small cysts,
contains coagulum.
Pleomorphic adenoma is a benign neoplasm which can l May also occur as cellular nests, sheets of cells, cords
differentiate to epithelial cells or a mesenchymal cells and or spindle cells.
hence given the term mixed tumour. l Myoepithelial cells appear as angular or spindled, some
cells are rounded with eccentric nuclei and hyalinized
Clinical Features eosinophilic cytoplasm (hyaline cells).
l Vacuolar degeneration of myoepithelial cells results in
l Most common tumour of salivary glands. cartilagenous appearance.
l The parotid gland is the most commonly affected and l Foci of hyalinization, bone and fat can be seen in the
the superficial lobe of the gland in particular. connective tissue of the tumour.
l Very rarely affects the sublingual glands.
l The palate is the most common site of involvement of Q. 8. Warthin’s tumour.
the minor salivary glands. Ans.
l Occurs more in females than in males.
l Seen in 4th to 6th decades of life with the average age l Warthin’s tumour is the second most common tumour of the
being 43 yrs. salivary glands and is most common in the parotid gland.
Section | III Oral Pathology 463

Clinical Features Histopathologic Features


l Occurs equally in men and women. Macroscopic:
l Presents in 6th and 7th decades. l Smooth, soft mass and well encapsulated when located
l The tumour is generally superficial and painless, firm to in the parotid.
palpate. l The tumour has cysts consist of clear fluid.
l Areas of focal haemorrhage can also be seen.
Histologic Features
Microscopic:
Macroscopic: l Tumour has two histologic components.
l Well encapsulated smooth, soft mass. a. Epithelial tissue
l The tumour has cysts consist of clear fluid. b. Lymphoid tissue
l Areas of focal haemorrhage can also be seen. l The lesion is an adenoma exhibiting cyst formation.
l The cysts are lined by papillary proliferations of bilay-
Microscopic: ered oncocytic epithelium.
l Tumour has two histologic components. l The inner cells are tall columnar with finely granular
a. Epithelial tissue and eosinophilic cytoplasm, due to the presence of mi-
b. Lymphoid tissue tochondria and slightly hyperchromatic nuclei.
l The lesion is an adenoma exhibiting cyst formation. l The outer cells are oncocytic triangular and sometimes
l The cysts are lined by papillary proliferations of bilay- fusiform basaloid cells.
ered oncocytic epithelium. l Focal areas of squamous metaplasia and mucous cell
l The inner cells are tall columnar with finely granular protoplasia may be seen.
and eosinophilic cytoplasm, due to the presence of mi- l Chocolate-coloured eosinophilic coagulum is present
tochondria and slightly hyperchromatic nuclei. within the cystic spaces.
l The outer cells are oncocytic triangular and sometimes
fusiform basaloid cells. Q. 10. Histology and laboratory investigations in
l Focal areas of squamous metaplasia and mucous cell Sjogren’s syndrome.
protoplasia may be seen. Ans.
l Chocolate-coloured eosinophilic coagulum is present
within the cystic spaces. Histological features of Sjogren’s syndrome are as
follows:
Treatment Three types seen in the major salivary glands.
a) Intense lymphocytic infiltration of the gland replacing
l Surgical excision. all the acinar structures.
Q. 9. Aetiopathogenesis and histopathology of Warthin’s b) Proliferation of the ductal epithelium and myoepithe-
tumour. lium to form ‘epimyoepithelial islands’
c) Atrophy of the glands following the lymphocytic
Ans. infiltration.
l Similar changes were seen in the accessory salivary
Warthin’s tumour is the second most common tumour in the
salivary glands. Most common in the parotid gland. glands in the lips.

Lab findings:
Aetiopathogenesis l 75% of the patients have polyclonal hyperglobulinae-
l Many theories have been put forward. mia and many develop cryoglobulins.
l Most accepted: Tumour arises in the salivary gland tis- l Multiple organ-specific or tissue-specific antibodies are
sue entrapped within paraparotid or intraparotid lymph found including anti-salivary duct antibodies, rheuma-
nodes during embryogenesis. toid factor and antinuclear antibodies.
l According to Allegra, it is most likely a delayed hypersen- l Increased sedimentation rate.
sitivity disease, the lymphocytes being an immune reac- l The presence of antisalivary duct antibody is three times
tion to the salivary ducts which undergo oncotic change. more common in those with secondary Sjogren’s syn-
l A strong association between smoking and the tumour drome to those with the sicca complex.
has been reported. Q. 11. Histology of mucoepidermoid carcinoma.
l Epstein-Barr virus has also been implicated in the
pathogenesis of this tumour. Ans.
464 Quick Review Series for BDS 3rd Year

Histological features of mucoepidermoid carcinoma are as l The tumour has cysts consist of clear fluid.
follows: l Areas of focal haemorrhage can also be seen.
l The tumour is composed of mucus secreting cells, epi-
dermoid type cells, and intermediate cells. Microscopic:
a. Mucus cells are of different shapes and have abun- l Tumour has 2 histologic components.
dant foamy cytoplasm. a. Epithelial tissue
b. Epidermoid cells have squamoid features, polygo- b. Lymphoid tissue
nal shape, intercellular bridges, and rarely kerati- l The lesion is an adenoma exhibiting cyst formation.
nization. l The cysts are lined by papillary proliferations of bilay-
c. Intermediate cells are the basaloid cells that recog- ered oncocytic epithelium.
nize the mucoepidermoid carcinoma. l The inner cells are tall columnar with finely granular
and eosinophilic cytoplasm, due to the presence of mi-
Some times clear cells may be present which are devoid
tochondria and slightly hyperchromatic nuclei.
of mucin and glycogen.
l The outer cells are oncocytic triangular and sometimes
The tumours can be graded into low grade, intermediate
fusiform basaloid cells.
grade and high grade.
l Focal areas of squamous metaplasia and mucous cell
protoplasia may be seen.
Low grade tumour: l Chocolate-coloured eosinophilic coagulum is present
Well-formed glandular structures. within the cystic spaces.
l Prominent cystic spaces. l Treatment:
l Lesser cellular atypia. Surgical excision.
l More mucus cells.
Q. 13. Xerostomia.
Intermediate grade tumour: Ans.
l Less prominent cyst formation. Xerostomia is also known as dry mouth.
l Intermediate cells predominate.
Aetiology
High grade tumour:
l Temporary causes:
l Cells are present as cords and nests.
i. Psychological–anxiety and depression
l Prominent nuclear pleomorphism and mitotic activity is
ii. Duct calculi
noted.
iii. Sialadenitis
l Cysts are lesser.
iv. Drug therapy–Zyban
l Necrosis and perineural invasion may be present.
l Permanent causes:
Q. 12. Warthin’s tumour. i. Salivary gland aplasia
ii. Sjogren’s syndrome
Ans.
iii. Radiation therapy
Warthin’s tumour is the second most common tumour in iv. Surgical desalivation
the salivary glands and most common in the parotid
gland. Clinical Features
l Unilateral dryness with pain and discomfort is indica-
Clinical Features tive of duct calculus.
l Sjogren’s syndrome always produces bilateral swelling.
l Equal in men and women. l When the deficiency of saliva is more, there is extreme
l Presents in 6th and 7th decades. discomfort.
l The tumour is generally superficial. l The mucosa will appear dry and atrophic, inflamed, pale
l It is painless, firm to palpate. or transluscent.
l The tongue shows atrophy of papillae, inflammation,
Histologic Features fissuring, cracking and in severe cases, denudation.
l Soreness, burning and pain of the mucous membrane
Macroscopic: and tongue are common symptoms.
l Smooth, soft mass and well encapsulated when located l This dry mouth leads to dental caries and subsequent
in the parotid. loss of teeth.
Section | III Oral Pathology 465

Treatment l Advise to use sugar free chewing gum to stimulate sali-


vary flow.
l Basic aetiologies have to be identified and eliminated
l Salivary substitutes can be given.
like the calculi, emotional problems and drugs.

SHORT NOTES
Q. 1. Sialolith. iv. Histologic features:
l Orderly lymphocytic infiltration of the salivary
Ans. gland tissue, destroying and replacing the acini.
i. The round, ovoid calcified structure present in the sali- l The epithelium may consist of ducts showing cellu-

vary duct or gland is called a ‘sialolith’. lar proliferation forming nests or clumps of poorly
ii. It is formed by the deposition of calcium salts around a defined epithelial cells which are termed as ‘epimy-
central nidus. oepithelial islands.’
iii. Composition of stone (calculi): v. Treatment:
l Tricalcium phosphate Ca3(PO4)2 l Symptomatic treatment.

l Calcium carbonate CaCO3 l Artificial tears and artificial saliva used to treat dry

l Soluble salts eyes and mouth, respectively.


l Organic matter Q. 3. Pleomorphic adenoma.
l Water

iv. Clinical features: Ans.


l Common in adults. i. Pleomorphic adenoma is a benign neoplasm which
l Swelling and pain which is related to mealtime. consists of cells that can differentiate into epithelial and
l Stones may be palpable. mesenchymal cells and hence called mixed tumours.
l Occurs more frequently in submandibular duct and ii. Clinical features:
glands l Parotid gland is the most commonly affected sali-
v. Diagnosis: By sialography. vary gland.
vi. Treatment: l It develops in 5th and 6th decades of life.
l Small calculi can be removed by manipulation or by l Begins as a small painless nodule overlying angle of
increasing the salivation by sucking a lemon, leading mandible or anterior to external ear and grows slowly.
to expulsion of the stone. l Irregular nodular lesion with firm consistency.
l The larger stones need to be removed by surgical iii. Histopathology: The neoplasm often exhibits prolifera-
exposure. tion of glandular epithelial cells in form of diffuse sheet
l If present near the gland, the gland has to be removed. or clusters. The neoplastic cells are polygonal, spindle
Q. 2. Mikulicz’s disease. or stellate shape and have tendency to form duct-like
structures which are widely distributed within lesions.
Ans. iv. Treatment–surgical excision.
i. Mikulicz’s disease is a benign lymphoepithelial lesion. Q. 4. Mucocele and ranula.
It is a chronic condition characterized by the abnormal
Ans.
enlargement of the salivary and the lacrimal glands.
ii. Aetiology: May be: MUCOCELE
l Autoimmune or Retention of mucous material due to trauma, involving
l Viral or salivary glands and their ducts is known as mucocele.
l Hereditary i. Clinical features:
iii. Clinical features: l Occurs most frequently on the lower lip, can also occur

l Females are affected more commonly than males. on the palate, cheek, tongue, and floor of the mouth.
l Characterized by sudden onset of xerostomia which l The lesion may lie superficial or deep in the tissue.

leads to difficulty in swallowing and results in tooth ii. Histological features:


decay. l The cavity is not lined by epithelium and is, therefore,

l Enlarged lacrimal glands leading to decreased or no not a true cyst.


tears. l The wall is made of compressed fibrous connective

l Painless swelling of parotid gland, submaxillary glands. tissue and fibroblasts.


466 Quick Review Series for BDS 3rd Year

iii. Treatment: Q. 7. Histopathology of adenoid cystic carcinoma.


l Excision.
Ans.
RANULA (Ranula means–swollen abdomen of frog) Histological features of adenoid cystic carcinoma are as
i. This is a type of retention cyst involving submandibular follows:
and sublingual glands and occurring in the floor of the i. Adenoid cystic carcinoma is composed of myoepithelial
mouth (more often sublingual gland) cells and ductal cells which have various arrangements.
ii. Cause – trauma ii. Morphologically, 3 patterns:
iii. Clinical findings: a. Cribriform(classic)
l Slow growing, painless swelling on one side in the
b. Tubular
floor of the mouth. c. Solid (basaloid)
l Tongue is pushed up.
iii. The cribriform pattern shows epithelium cell nests that
l Treatment: Complete surgical excision along with
form multiple cylindrical cyst-like patterns resembling
the whole of the sublingual salivary gland. a Swiss cheese or honeycomb pattern.
Q. 5. Sialography. iv. The tubular pattern reveals tubular structures that are
lined by stratified cuboidal epithelium.
Ans.
v. The solid pattern shows solid groups of cuboidal cells.
i. Sialography is a technique in which ducts and ductules vi. The cribriform pattern is the most common and the
of the salivary glands are demonstrated radiographically solid pattern is the least common.
after a radio-opaque liquid has been injected along them.
Q. 8. Histopathology of pleomorphic adenoma.
ii. Indications:
l To demonstrate – calculi, strictures, recurrent paroti- Ans.
tis, tumours, etc. Histopathological features of pleomorphic adenoma are as
l Salivary fistula
follows:
l Relationship of salivary glands and ducts to sur-
i. Macroscopic:
rounding structures. l Irregular to ovoid mass with irregular borders.
iii. Contraindication: l Fibrous incomplete capsule or unencapsulated in
l Active or recent infection of the gland.
major and minor salivary glands.
iv. Variety of abnormalities seen in sialography are: ii. Microscopic: Morphologic diversity is the characteristic
l Spherical dilatation (sialangiectasis),
feature.
l Dilatation with “beading” effect of the duct.

l Presence of tumors Various types observed microscopically are:


l Irregularity of duct i. Myxoid
l Obliteration of duct ii. Myxoid and cellular
l Destruction of the gland iii. Predominantly cellular
iv. Extremely cellular
Q. 6. Histopathology of Warthin’s tumour.
Q. 9. Histogenesis of pleomorphic adenoma.
Ans.
Ans.
Histopathologic features of Warthin’s tumour are as follows:
i. Tumour has two histologic components. Histogenesis of pleomorphic adenoma is as follows:
a. Epithelial tissue i. Many theories have been put forward, but the latest is
b. Lymphoid tissue focussed on the myoepithelial and the reserve cell in the
ii. The lesion is an adenoma exhibiting cyst formation, intercalated duct.
with papillary projections into the cystic spaces and ii. Ductal and myoepithelial cells are present in this tumour.
lymphoid matrix showing germinal centres. The myoepithelial cell is responsible for the morpho-
iii. The cysts are lined by papillary proliferations of bilay- logic diversity of the tumour, the production of fibrous,
ered oncocytic epithelium. mucinous, chondroid and osseous areas.
iv. The inner cells are tall columnar with finely granular iii. The intercalated duct reserve cell can differentiate into
and eosinophilic cytoplasm, due to the presence of mi- ductal and myoepithelial cells and these myoepithelial
tochondria and slightly hyperchromatic nuclei. cells can undergo mesenchymal metaplasia.
v. The outer cells are oncocytic triangular and sometimes iv. All these cytogenic abnormalities involve the chromo-
fusiform basaloid cells. some at the region 12q13-15.
vi. Chocolate-coloured eosinophilic coagulum is present v. The pleomorphic adenoma gene (PLAG1) has been
within the cystic spaces. mapped to another gene – 8q12.
Section | III Oral Pathology 467

Q. 10. Warthin’s tumour. i. Histologically, mucoepidermoid carcinoma is com-


posed of mucus secreting cells, epidermoid type cells,
Ans.
and intermediate cells.
i. Warthin’s tumour is the second most common tumour in a. Mucus cells are of different shapes and have abun-
the salivary glands. Most common in the parotid gland. dant foamy cytoplasm.
ii. The tumour is generally superficial, painless, firm to b. Epidermoid cells have squamoid features, polygo-
palpate. nal shape, intercellular bridges, and rarely kerati-
iii. Macroscopically, it is a smooth, soft mass and well nization.
encapsulated when located in the parotid. c. Intermediate cells are the basaloid cells that recog-
iv. Microscopically, the tumour exhibits two histologic com- nize the mucoepidermoid carcinoma.
ponents:
Some times clear cells may be present which are devoid
a. Epithelial tissue
of mucin and glycogen.
b. Lymphoid tissue
ii. The tumours can be graded into:
v. Treatment: Surgical excision.
l Low grade

Q. 11. Aetiopathogenesis of Warthin’s tumour. l Intermediate grade and

l High grade.
Ans.
iii. Low grade tumour is characterized by well-formed
i. Many theories have been put forward to explain aetio- glandular structures, prominent cystic spaces, lesser
pathogenesis of Warthin’s tumour. cellular atypia and more mucus cells.
ii. Most accepted theory is that tumor arises in the salivary iv. Intermediate grade tumour is characterized by less
gland tissue entrapped within paraparotid or intrapa- prominent cyst formation and predominant intermedi-
rotid lymph nodes during embryogenesis. ate cells.
iii. According to Allegra, it is most likely a delayed hypersen- v. High grade tumour is characterized by cells are present
sitivity disease, the lymphocytes being an immune reac- as cords and nests.
tion to the salivary ducts which undergo oncotic change. Prominent nuclear pleomorphism and mitotic activity is
iv. A strong association between smoking and the tumour noted. Cysts are lesser.
has been reported. Necrosis and perineural invasion may be present.
v. Epstein-Barr virus has also been implicated in the
Q. 14. Adenoid cystic carcinoma.
pathogenesis of this tumour.
Ans.
Q. 12. Histology and laboratory investigations in
Sjogren’s syndrome. i. Adenoid cystic carcinoma is also known as cylindroma.
ii. It is a malignant neoplasm arising from glandular epi-
Ans.
thelium of either major or minor glands. Slow growing
i. Histological features of Sjogren’s syndrome are as follows: but aggressive neoplasm.
l Three types seen in the major salivary glands are: iii. Clinical features:
a) Intense lymphocytic infiltration of the gland re- l Most commonly involves the parotid, the submaxil-

placing all the acinar structures. lary and the accessory glands in the palate and tongue
b) Proliferation of the ductal epithelium and myo- l The tumour arises at the age of 50-70 years and is

epithelium to form ‘epimyoepithelial islands’ more common in females.


c) Atrophy of the glands following the lymphocytic l Symptoms include local pain, facial nerve paralysis

infiltration. (parotid), fixation to deeper structures and local


l Similar changes were seen in the accessory salivary invasion.
glands in the lips. l Can spread through perineural spaces.

ii. Lab findings of Sjogren’s syndrome are as follows: iv. Histopathology:


l 75% of the patients have polyclonal hyperglobulinae- It is characterized by the presence of numerous, small,
mia and many develop cryoglobulins. darkly stained polygonal or cuboidal cells. Double layer
l Multiple organ-specific or tissue-specific antibodies of tumour cells are arranged in duct-like pattern and
are found including anti-salivary duct antibodies, contains eosinophilic coagulum at centre, because of this
rheumatoid factor and antinuclear antibodies. lesion typically produces “Swiss Cheese appearance”
l Increased sedimentation rate. The stromal of connective tissue tumour is hyalinised
which surrounds tumour cells by forming structural
Q. 13. Histology of mucoepidermoid carcinoma.
pattern of many cylinders.
Ans. v. Treatment: Wide surgical excision.
468 Quick Review Series for BDS 3rd Year

Q. 15. Salivary analysis. iii. The salivary parameters analyzed included: Sodium,
potassium, calcium, inorganic phosphate, magnesium,
Ans.
total protein, albumin, lactate dehydrogenase, amylase,
i. Salivary analysis is a new diagnostic tool used in oral total immunoglobulin G, secretory immunoglobulin
cancers. A(Sec IgA), epidermal growth factor, insulin growth
ii. This analysis evaluates biochemical and immunologi- factor I (IGF-I) and metalloproteinases (MMP-2 and
cal parameters in the saliva of oral squamous cell carci- MMP-9).
noma (OSCC) patients.

Topic 4

Cysts and Tumours of Odotogenic Origin

LONG ESSAYS
Q. 1. Define cyst. Classify cysts of odontogenic origin. l Median palatine, median alveolar and median man-
Write in detail about dentigerous cyst. dibular cyst.
Or ii. Inflammatory cysts
Classify cysts of odontogenic origin. Describe pathogen- l Residual cyst
esis, radiological appearances and histological features l Radicular cyst
of dentigerous cyst. l Paradental cyst and mandibular infected buccal cyst
l Inflammatory collateral cyst.
Ans.
A cyst is defined as a pathological cavity having fluid, II. Non-epithelial cysts:
semifluid or gaseous contents and which is not created by i. Solitary or simple bone cyst
accumulation of pus (Kramer 1974). ii. Aneurysmal bone cyst.

CLASSIFICATION OF CYSTS B. Cyst Associated with the Maxillary Antrum


i. Benign mucosal cyst of maxillary antrum
A. Cysts of the Jaws ii. Postoperative maxillary cyst.
I. Epithelial cysts
Cysts of soft tissues of mouth, face and neck
i. Developmental:
l Dermoid cyst and epidermoid cyst
a. Odontogenic cysts: l Branchial cyst
l Gingival cyst of infants l Thyroglossal duct cyst
l Gingival cyst of adults l Anterior median lingual cyst
l Eruption cyst l Cystic hygroma
l Dentigerous cyst l Nasopharyngeal cyst
l Odontogenic keratocyst l Cysts of salivary gland
l Lateral periodontal cyst l Hydatid cyst.
l Glandular odontogenic cyst
l Calcifying odontogenic epithelial cyst.
DENTIGEROUS CYST
b. Non-odontogenic cysts: l It is also called as follicular cyst or pericoronal cyst.
l Nasopalatine duct cyst l It is the odontogenic cyst that surrounds the crown of
l Nasolabial cyst the impacted tooth by expansion of its follicle and is
l Globulomaxillary cyst attached to neck.
Section | III Oral Pathology 469

Pathogenesis ii. Lateral – mesioangular impaction grow laterally


along root surfaces.
l It is formed due to fluid accumulation in the layer of iii. Circumferential – encloses entire tooth.
reduced enamel epithelium and the crown of the un-
erupted tooth.
l It may originate initially by the proliferation and cystic Complications
transformation of the islands of epithelium in connec-
tive tissue wall of dental follicle. l Ameloblastoma
l Squamous cell carcinoma
l Mucoepidermoid carcinoma
Clinical Features
l Most lesions are present in the 2nd, 3rd and 4th decades
with male predilection. Treatment
l The most common site of the cysts is the mandibular l Enucleation.
and maxillary third molar and maxillary cuspid areas, l Marsupilization in young adults.
since these are the most commonly impacted teeth.
l Generally, it is asymptomatic but may be painful, if it is Q. 2. Classify the benign and malignant tumours of the
secondarily infected. oral cavity. Write in detail about clinical and histologi-
l The dentigerous cyst has potential to become an aggres- cal features of ameloblastoma.
sive lesion with expansion of bone and subsequent Ans.
facial asymmetry.
l There is extreme displacement of teeth, severe root Classification of odontogenic tumours is as follows:
resorption of adjacent teeth.
l It has thin cortical plates with crepitus-like sensation on A. Benign Tumours:
palpation. i. Odontogenic epithelium without odontogenic ectomes-
l Extreme bone loss causing fluctuations, paraesthesia and enchyme
pathological fractures. a. Ameloblastoma
b. Squamous odontogenic tumour
c. Calcifying epithelial odontogenic tumour
Histopathological Features d. Adenomatoid odontogenic tumour.
l It is usually composed of thin connective tissue wall ii. Odontogenic epithelium with odontogenic ectomesen-
with a thin layer of stratified squamous epithelium of chyme with or without hard tissue formation:
2 – 3 layers lining the lumen. a. Ameloblastic fibroma
l Rete pegs formation is absent except in case of second- b. Ameloblastic fibrodentinoma
arily infected cyst. c. Ameloblastic fibro-odontoma
l The connective tissue wall is frequently quite thickened d. Odontoameloblastoma
and composed of very loose fibrous connective tissue. e. Calcifying odontogenic cyst
l Inflammatory cells commonly infiltrate the connective f. Complex odontoma
tissue consisting of neutrophils and plasma cells. g. Compound odontoma
l It also shows Rushton bodies within the lining epithelium iii. Odontogenic ectomesenchyme with or without included
which are peculiar linear and often curved hyaline bodies. odontogenic epithelium
l The content of cystic lumen is usually thin watery a. Odontogenic fibroma
yellow fluid and is occasionally blood tinged. b. Myxoma
l Protein content in lumen is greater than 5 gm/dl. c. Cementoblastoma.

B. Malignant tumours:
Radiographic Features
i. Odontogenic carcinomas:
l Radiographically, dentigerous cyst reveals well-defined a. Malignant ameloblastoma
unilocular radiolucent area, enclosing the crown of b. Primary intraosseous carcinoma
infected tooth. c. Ghost cell odontogenic carcinoma
l The periphery of lesion is bordered by the well corti- d. Clear cell odontogenic carcinoma.
cated or sclerotic margin. ii. Odontogenic sarcomas:
l It shows three different types of appearances: a. Ameloblastic fibrosarcoma
i. Central – symmetric from all sides surrounding b. Ameloblastic fibrodentinosarcoma
crown of tooth. c. Ameloblastic fibro-odontosarcoma.
470 Quick Review Series for BDS 3rd Year

AMELOBLASTOMA Intrafollicular
cyst formation
It is also known by other terms like: Connective
tissue stroma
i. Adamantoblastoma.
ii. Adamantinoma – coined by Malassez in 1885 Peripheral
ameloblast-like cells
iii. Ameloblastoma – coined by Churchill in 1934
iv. Epithelial odontoma. Central stellate
reticulum-like cells

Definition as given by Robinson:


Ameloblastoma is a tumour of odontogenic origin usually Ameloblastomatous
follicles
unicentric nonfunctional, intermittent in growth anatomi-
cally benign and clinically persistent. FIGURE 4.1 Histological features of follicular ameloblastoma

Definition according to WHO:


Polymorphic neoplasm consisting of proliferating odonto- l Each follicle-like structure is bordered on the periphery
genic epithelium usually occurring in two main forms in the by the single layer of tall columnar cells resembling
follicular types of the growth. The tumour consists of ameloblasts.
enamel organ like islands of epithelium cells while in the l The cells located at the centre of follicle are loosely
plexiform type it forms continuous anastomosing islands. arranged and resemble stellate reticulum cells.
l Microcyst formation is often observed inside these
Classification follicles.

i. Central or intraosseous: ii. Plexiform type


Solid or multicystic l In this pattern, ameloblast-like tumour cells are
Unicystic arranged in irregular masses or more frequently as anet-
Desmoplastic work of interconnecting strands of cells (Fig 4.2).
ii. Periapical or extraosseous l Each of these masses are bounded by a layer of colum-
nar cells, between these layers may be found stellate
Pathogenesis of Ameloblastoma reticulum-like cells.
l The stellate reticulum-like tissue is much less promi-
The tumour may be derived from the: nent in the plexiform type than in the follicular type.
a. Cell rests of enamel organ, either remnants of dental l The areas of cystic degeneration of stroma are also
lamina or remnants of Hertwig’s sheath, the epithelial common.
rest cells of Malassez.
b. Epithelium of odontogenic cyst. iii. Acanthomatous type:
c. Disturbance of the developing enamel organ. l The cells occupying the position of the stellate reticulum
d. Basal cell of the surface epithelium of the jaw. undergo squamous metaplasia, sometimes with keratin
formation in the anterior portion of tumour islands.
Occasionally, epithelial or keratin pearls may be observed.
Histopathology l

Histologically, the ameloblastoma shows neoplastic prolif-


eration of odontogenic epithelial cells mostly in distinct
patterns; Cystic degenaration
of connective tiisue
l Follicular type
l Plexiform type
Stellate reticulum-
l Acanthomatous type like cells
l Granular cell type
l Basal cell type
Ameloblast-
i. Follicular type like cells

l It is the most commonly encountered variant. Connective tissue


stroma
l In Figure 4.1, the neoplastic odontogenic epithelial cells
proliferate in form of multiple discrete follicles and
islands within fibrous connective tissue stroma. FIGURE 4.2 Histological features of plexiform ameloblastoma
Section | III Oral Pathology 471

iv. Granular type Classify cysts of the orofacial region. Describe patho-
l There is marked transformation of the cytoplasm, usu- genesis and histopathology of odontogenic keratocyst.
ally of the stellate reticulum-like cells that it takes a Or
very coarse granular eosinophilic appearance.
Classify the cysts of orofacial region. Give clinical signs,
v. Basal cell type symptoms and histopathological features of primordial
Bears considerable resemblance to the basal cell carcinoma cyst.
of the skin. Ans.
A cyst is defined as a pathological cavity having fluid,
Clinical Features semifluid or gaseous contents and which is not created by
l The ameloblastoma occurs in wide age range from accumulation of pus (Kramer 1974).
10 years through 90 years.
l The average age of occurrence is 33 -39 years.
l The males are affected more commonly than females.
CLASSIFICATION OF CYSTS OF ORAL
l Ameloblastoma occurs in all the areas of the jaws but CAVITY
mandible is most commonly affected. A. Cysts of the Jaws
l In the mandible, the molar-angle-ramus area is involved
three times more commonly than premolar and anterior I. Epithelial cysts
regions combined. i. Developmental:
l Clinically, ameloblastoma presents slow enlarging, pain-
a. Odontogenic cysts:
less, ovoid and fusiform bony hard swelling of the jaw.
l Gingival cyst of infants
l Pain, paraesthesia and mobility of regional teeth is pres-
l Gingival cyst of adults
ent in some cases.
l Eruption cyst
l Pathological fractures may occur in many affected bones.
l Dentigerous cyst
l Odontogenic keratocyst
Radiological Appearance l Lateral periodontal cyst
l Glandular odontogenic cyst
l Radiographically, ameloblastoma is a well-defined,
l Calcifying odontogenic epithelial cyst.
multilocular, radiolucent area in a bone with “honey-
comb” appearance.
l In radiograph, the lesion typically exhibits an irregular b. Non-odontogenic cysts:
and scalloped margin. l Nasopalatine duct cyst
l Nasolabial cyst
l Globulomaxillary cyst
Differential Diagnosis l Median palatine, median alveolar and median man-
l Residual cyst dibular cyst.
l Lateral periodontal cyst
l Giant cell granuloma ii. Inflammatory cysts
l Traumatic bone cyst l Residual cyst
l Primordial cyst l Radicular cyst
l Paradental cyst and mandibular infected buccal cyst
Treatment l Inflammatory collateral cyst.
l Complete removal of neoplasm II. Non-epithelial cysts
l Curretage
l Peripheral ostectomy i. Solitary or Simple bone cyst
l Intraoral block excision ii. Aneurysmal bone cyst.
l Extraoral en bloc resection
Q. 3. Define cyst. Classify odontogenic cysts. Discuss in B. Cyst Associated with the Maxillary
detail odontogenic keratocyst. Add a note on the syn- Antrum
drome associated.
i. Benign mucosal cyst of maxillary antrum
Or ii. Postoperative maxillary cyst.
472 Quick Review Series for BDS 3rd Year

Cysts of soft tissues of mouth, face and neck Radiological Features


l Dermoid cyst and epidermoid cyst l Small round to ovoid radiolucent areas, unilocular or
l Branchial cyst multilocular.
l Thyroglossal duct cyst l Well-defined radio-opaque margin which may be
l Anterior median lingual cyst smooth or scalloped.
l Cystic hygroma l Radiological subclassification:
l Nasopharyngeal cyst i. Replacement – found in place of tooth
l Cysts of salivary gland ii. Follicular or envelopmental – surrounding crown of
l Hydatid cyst. impacted tooth
iii. Collateral – adjacent to tooth
ODONTOGENIC KERATOCYST iv. Extraneous – involving ascending ramus.
l OKC was earlier called as primordial cyst.
l Odontogenic keratocyst is a common cystic lesion of the Histological Features
jaw, which arises form the remnants of dental lamina.
l The epithelial lining is highly characteristic.
l The term OKC was first introduced by Philipsen–1956.
l A parakeratin surface which is usually corrugated rip-
l It is named as keratocyst because the cyst epithelium
pled or wrinkled.
produces so much keratin that it fills the cyst lumen.
l Uniformity of thickness of epithelium and is generally
l Odontogenic cysts have more aggressive course than
between 6 and 10 cells in depth.
any other cystic lesion of jaw and for this reason these
l Prominent palisaded, polarized basal cell layer often
are sometimes known as benign cystic neoplasms.
described as having a “picket fence” or “tombstone”
appearance.
Pathogenesis l Occasionally, orthokeratin is found but if present, para-
Odontogenic keratocyst mainly arises from the: keratin is evident.
l Dental lamina or its remnants. l Connective tissue shows “daughter cells” or “satel-
l Primordium of developing tooth germ or enamel organ. lite cysts”because of which it shows high recurrence
l Sometimes from basal cell layer of oral epithelium. rate.
l The lumen of keratocyst may be filled with thin straw-
coloured fluid or with thick creamy material with a
Clinical Features protein content of 3.5gm%.
l Age: The cyst may occur at any age from very young to l Sometimes a lumen contains a great deal of keratin
very old. The peak incidence is between 2nd and 3rd while at other times it has little cholesterol as well as
decades of life. hyaline bodies at the site of inflammation.
l Sex: It is found more frequently in males as compared l Epithelium and connective tissue interface is weak.
to females. l No rete ridges are present.
l Site: The mandible is affected more commonly than
maxilla. Differential Diagnosis
l In the mandible, the majority of cysts occur in ramus
and third molar area, followed by first and second molar l Unicystic ameloblastoma
area and then the anterior mandible. l Glandular odontogenic cyst
l It is asymptomatic unless they become secondarily in- l Dentigerous cyst
fected in which case patient complains of pain, soft tis- l Lateral periodontal cyst
sue swelling and drainage. l Aneurysmal bone cyst
l The size varies from 1-10 cm mean 4-5 cm. l Residual cyst
l Occasionally, they experience paraesthesia of lower lip
and teeth. Treatment
l There is often one tooth missing from the dental arch.
l Larger lesions show expansion and thinning of bone l Surgical excision
may result in pathological fracture and has egg shell l Enucleation and curettage
crackling effect on palpation. l Enucleation and peripheral ostectomy
l The maxillary odontogenic keratocyst tends to be sec- l Osseous resection
ondarily infected with greater frequency than the man- l Liquid nitrogen therapy
dibular ones, due to its vicinity to maxillary sinus. l Decompression and marsupialization
Section | III Oral Pathology 473

Prognosis Derived from reduced enamel epithelium:


l High recurrence rate 5 – 62% l Dentigerous cyst
l Malignant transformation 10% l Eruption cyst

Derived from dental lamina (rests of Serres)


Syndrome Associated with OKC l Gingival cyst of infants
Gorlin and Goltz syndrome l Gingival cyst of adults
l Odontogenic keratocyst
It is also called as jaw cyst, basal cell nevus, bifid rib syn-
l Lateral periodontal cyst
drome or nevoid basl cell carcinoma syndrome or heredi-
l Glandular odontogenic cyst
tary cutaneomandibular polyoncosis.
Features of this syndrome are as follows: Unclassified
l Multiple odontogenic keratocysts
l Paradental cyst
l Autosomal dominant trait
l Calcifying odontogenic epithelial cyst.
l Skin cancer – multiple nevoid basal cell carcinoma of
skin
l Frontal and temoparal bossing GORLIN CYST
l Hypertelorism
The calcifying odontogenic cyst (COC) was first described
l Mandibular prognathism
as a separate entity by Gorlin in 1962. The other names are
l Abnormal calcium and phosphate metabolism
calcifying odontogenic cyst or cystic keratinizing tumour.
l Calcification of flax cerebri.
Q. 4. Classify odontogenic cysts of the oral cavity write
in detail the aetiology and histological features of Gorlin Special Features
cyst. The lesion is unusual that it has some features of cyst but
Ans. also has many charachteristics of solid neoplasms.
l A rare well-circumscribed solid or cystic lesion derived
from odontogenic epithelium that resembles follicular
CLASSIFICATION OF ODONTOGENIC ameloblastoma but contains ghost cells and spherical
calcifications.
CYSTS
l Most likely to affect the anterior areas of the jaws.
Classification by Aetiology l It is most common in people in their second to third
decades but can be seen at almost any age.
i. Developmental cysts: Unknown origin l In one-third of cases, an impacted tooth is involved.
l Gingival cyst of infants l The cystic forms occur in 3 forms:
l Gingival cyst of adults Type 1A – simple unicystic type
l Eruption cyst Type 1B – the odontome producing type
l Dentigerous cyst Type 1C – the ameloblastous proliferating type
l Odontogenic keratocyst
l Lateral periodontal cyst
l Glandular odontogenic cyst Radiological Findings
l Calcifying odontogenic epithelial cyst. l The central intrabony lesions appear radioluscency.
l Well circumscribed
ii. Inflammatory cysts: Result of inflammation l Variable mounts of calcified radio-opaque material are
l Residual cyst usually scattered throughout the radioluscency ranging
l Radicular cyst from tiny flecks to large masses.
l Paradental cyst and mandibular infected buccal cyst l Size – may be very large in diameter
l Inflammatory collateral cyst. l Small lesions are very common.

Classification by Tissue of Origin Histologic Features


Derived from rests of Malassez Microscopically, there are many cells that are described as
l Periapical cyst “ghost cells”, enlarged eosinophilic epithelial cells without
l Residual cyst nuclei (Fig 4.3).
474 Quick Review Series for BDS 3rd Year

ODONTOMA
Cystic lumen
Odontomas are hamartoma that contain both epithelial and
Calcifications
mesodermal dental tissue components.
Ghost cells
l Generally, there are two types of odontoma:
Stellate reticulum- i. Complex odontoma
like spinous cells
ii. Compound odontoma.
Dentinoid l BROCA 1866 first coined the term odontoma.
Ameloblast-like
basal cells i. Complex odontoma:
Connective tissue
l It is always benign and contains enamel, dentin and
FIGURE 4.3 Histologic features of Gorlin’s cyst cementum which are not differentiated, so that structure
of actual tooth is not identifiable.

ii. Compound odontoma:


Q. 5. Classify odontogenic tumours. Describe in detail
about odontomes. In compound odontomas, the enamel and dentin are laid
down in such a fashion that the structure bears considerable
Ans. anatomic resemblance to normal teeth, except that they are
Classification of odontogenic tumours is as follows: often smaller than typical teeth.

A. Benign Tumours: Clinical Features


i. Odontogenic epithelium without odontogenic
ectomesenchyme: l The lesion occurs at any age but more common among
a. Ameloblastoma children or young adults with mean age of 14.8 years.
b. Squamous odontogenic tumour l Both sexes are equally affected or slight male predomi-
c. Calcifying epithelial odontogenic tumour nance is present.
d. Adenomatoid odontogenic tumour. l Maxilla is commonly affected.
l Odontomas are commonly seen in pericoronal area of
ii. Odontogenic epithelium with odontogenic ectomes- permanent teeth.
enchyme with or without hard tissue formation: l The odontoma produces large, bony, hard swellings of
a. Ameloblastic fibroma jaw, with expansion of cortical plates and displacement
b. Ameloblastic fibrodentinoma of regional teeth.
c. Ameloblastic fibro-odontoma l If odontoma is located high in alveolus, they may
d. Odontoameloblastoma tend to erupt in oral cavity by resolving overlying
e. Calcifying odontogenic cyst bone and as a result there may be pain, inflammation,
f. Complex odontoma ulceration, etc.
g. Compound odontoma
Histopathology
iii. Odontogenic ectomesenchyme with or without
included odontogenic epithelium Compound odontome
a. Odontogenic fibroma l Fully developed compound odontoma reveals the
b. Myxoma presence of encapsulated mass of separate denticles,
c. Cementoblastoma. embedded in fibrous tissue stroma.
l The fully developed complex odontoma reveals an
B. Malignant tumours: irregularly arranged but well-formed mass of enamel,
i. Odontogenic carcinomas dentin and cementum which is surrounded by fibrous
a. Malignant ameloblastoma tissue capsule.
b. Primary intraosseous carcinoma
c. Ghost cell odontogenic carcinoma Complex odontoma
d. Clear cell odontogenic carcinoma. l The fully developed complex odontoma reveals irregu-
larly arranged but well-formed mass of enamel, dentin,
ii. Odontogenic sarcomas cementum and pulp which is surrounded by fibrous
a. Ameloblastic fibrosarcoma tissue capsule.
b. Ameloblastic fibrodentinosarcoma l The dentinal tissues lie in the direct contact with
c. Ameloblastic fibro-odontosarcoma. connective tissue that resembles dental pulp.
Section | III Oral Pathology 475

l Most of enamel tissues are fully calcified and appear as CALCIFYING EPITHELIAL ODONTOGENIC
small empty space. TUMOUR
l Ghost cells are often seen.
Also called as PINDBORG TUMOUR as it was described
by Dr. Jens J Pindborg in 1956.
Treatment The Pindborg’s tumour is locally aggressive neoplasm,
l Both the types of odontomes are treated by surgical which is uncommon, benign, odontogenic neoplasm that is
enucleation. exclusively epithelial in origin.

Q. 6. Classify odontogenic neoplasms. Describe the


pathogenesis, clinical features, histopathology and dif- Clinical Features
ferential diagnosis of calcifying epithelial odontogenic l The tumour occurs in middle age persons.
tumour. l The mandible is involved more often than maxilla. Mo-
Ans. lar region is more common site of occurrence followed
by premolar region.
l The tumour presents a slow enlarging, painless swelling
CLASSIFICATION OF ODONTOGENIC of jaw with expansion and distortion of cortical plates.
The swelling is bony hard and clinically it is well de-
TUMOURS l

fined or diffused.
Benign Tumours l Pain, paraesthesia may develop on rare occasions and
few lesions may be completely asymptomatic.
i. Odontogenic epithelium without odontogenic
ectomesenchyme
Histopathology
a. Ameloblastoma
b. Squamous odontogenic tumour l In Figure 4.4, the tumour reveals sheet of closely packed,
c. Calcifying epithelial odontogenic tumour polyhedral cells, non-inflamed connective tissue stroma.
d. Adenomatoid odontogenic tumour. The tumour cells contain oval-shaped nuclei and homog-
enous eosinophilic cytoplasm.
ii. Odontogenic epithelium with odontogenic
ectomesenchyme with or without hard tissue
formation
a. Ameloblastic fibroma
b. Ameloblastic fibro-odontome Fibrous stroma
c. Complex odontome
d. Compound odontome Liesegang rings

iii. Odontogenic ectomesenchyme with or Sheets of polyhedral


cells with hyperchromatic
without included odontogenic epithelium nuclei and prominent
intercellular bridges.
a. Odontogenic fibroma
Amyloid-like miterial
b. Myxoma
FIGURE 4.4 Histological features of CEOT
c. Cementoblastoma.
l Prominent intracellular bridges and distinct cell bound-
Malignant Tumours aries are often found in the lesions.
l Some amount of homogenous, hyaline material is
i. Odontogenic carcinomas
often deposited in between tumour cells called amy-
a. Malignant ameloblastoma loid material.
b. Primary intraosseous carcinoma l One of the most important histological characteristics of
c. Ghost cell odontogenic carcinoma CEOT is the presence of several calcified masses in and
d. Clear cell odontogenic carcinoma. around the tumour cells.
l Some Liesegang rings are also found.
ii. Odontogenic sarcoma
a. Ameloblastic fibrosarcoma
b. Ameloblastic fibrodentinosarcoma
Treatment
c. Ameloblastic fibro-odontosarcoma. Surgical enucleation.
476 Quick Review Series for BDS 3rd Year

Q. 7. Write in detail about periapical cyst. Histopathological Features


Ans. l It is lined by stratified squamous epithelium which is
varied in thickness with rare keratin formation (Fig 4.5).
l It is an inflammatory cyst lined by epithelium and is
l Hyaline and rushton bodies are found in great numbers
filled with fluid.
in epithelium.
l Radicular cyst is otherwise known as apical periodontal
l Connective tissue makes up the wall and is composed of
cyst or root end cyst or periapical cyst.
collagen fibres, fbroblasts and small blood vessels.
l Lumen contains fluid with low concentraton of protein
Aetiology and it also contains cholesterol or keratin.
l The cholesterol clefts and multinucleated giant cells are
a. Infection from microorganisms and their toxins in the
seen in the cyst wall.
pulp canal of nonvital tooth.
b. Trauma
Cystic lumen

Pathogenesis Lining epithelium


(stratified squamous
l Usually begins as periapical granuloma. epithelium)
l In the midst of the rich vascularised area provided by Connective tissue
the granuloma, the rests of Malassez proliferate and enclosed in
eventually forms a large mass of cells. cascades of
epithelium
l With continuous growth, the inner cells of mass are
Connective tissue
deprived of nourishment and they undergo liquefaction capsule with dense
necrosis. inflammatory cell
l This leads to the formation of cavity which is located in infiltrate
the centre of granuloma, giving rise to a radicular cyst. More fibrous deeper
portion of capsule
FIGURE 4.5 Histopathological features of radicular cyst
Clinical Findings
l Usually found in adult life although may occur at Differential Diagnosis
any age. l Periapical granuloma
l More common in maxillary anterior region. l Periapical scar
l Most common in males. l Traumatic bone cyst
l The tooth involved is generally nonvital and hence l Periodontal abscess
asymptomatic. l Mandibular infected buccal cyst
l Septic tooth or root is usually present.
l Presents as slowly growing painless swelling on the
buccal aspect of the jaw, because it is a thinner bone. Treatment
l Swelling which is round and hard at first, later when l Enucleation and extraction of affected tooth.
the bone becomes thin to the thickness of an egg l Root canal treatment and apicoectomy.
shell, crackling sensation is felt on pressure with the
fingers. Q. 8. Describe clinical and histopathological features of
l Finally, as the bony wall is resorbed fully, pressure adeno-ameloblastoma.
produces fluctuation. Or
l Swelling looks bluish in colour under the mucous
membrane. Classify odontogenic tumours. Describe in detail about
l Infection of the cyst causes redness and pain. adenomatoid odontogenic tumour.
l Pus formation in the cyst will lead to sinus opening and Ans.
discharge of pus.
l Adenomatoid odontogenic tumour is also known as ade-
noameloblastoma or ameloblastic adenomatoid tumour.
Radiological Features l The adenomatoid odontogenic tumour is uncommon,
It is round or oval radiolucency of variable size which is well-circumscribed, odontogenic neoplasm character-
generally well delineated and is most likely with marked ized by the formation of multiple duct-like structures by
radio-opaque rim. neoplastic epithelial cells.
Section | III Oral Pathology 477

Clinical Features
Rosettes of tumour cells
l Prevalence 3 -7%.
Eosinophilic material
l This tumour usually occurs in younger age group.
l Females are more commonly affected. Polyhedral cells arranged
to form nests
l The lesion most typically appears in maxillary anterior
region. Foci of calcified material
l The tumour presents a slow enlarging, small, bony hard
Duct-like structures lined
swelling in maxillary anterior region. by ameloblast-like cells
l Sometimes it occurs in premolar region.
Scanty connective tissue
l There is displacement of regional teeth, mild pain and
FIGURE 4.6 Histological features of adeno-ameloblastoma
expansion of cortical bones.
l Types:
i. Intraosseous 96%
ii. Extraosseous 4 % l Small foci of calcifications are often seen, which are
scattered throughout the lesion.
l Droplets of amorphous (PAS positive) eosinophilic ma-
Histopathology terial are found between neoplastic cells.

l Microscopically, adenoid odontogenic tumour reveals


neoplastic odontogenic epithelial cells, proliferating in Differential Diagnosis
multiple “duct-like” patterns (Fig 4.6). l Dentigerous cyst
l The presence of these duct-like structures often gives l Odontomes
glandular lesion. l Unicystic ameloblastoma CEOT
l Each duct-like structure is bordered on periphery by l CEOC.
a single layer of tall columnar cells resembles amelo-
blastoma.
l The lumens of duct-like structures are filled with the Treatment
homogenous eosinophilic coagulum. The treatment is surgical enucleation.

SHORT ESSAYS
Q. 1. Classify odontogenic cysts. B. Classification by Tissue of origin:
Ans. i. Derived from rests of Malassez:
l Periapical cyst
Odontogenic cysts are classified as follows: l Residual cyst
A. Classification by Aetiology:
i. Developmental cysts: ii. Derived from reduced enamel epithelium:
l Gingival cyst of infants l Dentigerous cyst
l Gingival cyst of adults l Eruption cyst
l Eruption cyst
l Dentigerous cyst iii. Derived from dental lamina (rests of Serres)
l Odontogenic keratocyst l Gingival cyst of infants
l Lateral periodontal cyst l Gingival cyst of adults
l Glandular odontogenic cyst l Odontogenic keratocyst
l Calcifying odontogenic epithelial cyst. l Lateral periodontal cyst
l Glandular odontogenic cyst
ii. Inflammatory cysts:
l Residual cyst
l Radicular cyst iv. Unclassified:
l Paradental cyst and mandibular infected buccal cyst l Paradental cyst
l Inflammatory collateral cyst. l Calcifying odontogenic epithelial cyst.
478 Quick Review Series for BDS 3rd Year

Q. 2. Pindborg’s tumour or CEOT. Q. 3. Gorlin’s cyst.


Ans. Ans.
l Calcifying epithelial odontogenic tumour also called l The other names of Gorlin’s cyst are calcifying odonto-
as Pindborg tumour as it was described by Jens J genic cyst or cystic keratinizing tumour. It was first de-
Pindborg in 1956. scribed as a separate entity by Gorlin in 1962.
l The Pindborg’s tumour is locally aggressive neoplasm,
which is uncommon, benign, odontogenic neoplasm Special Features
that is exclusively epithelial in origin.
l The lesion is unusual that it has some features of cyst
but also has many characteristics of solid neoplasms.
Clinical Features l A rare well circumscribed solid or cystic lesion derived
l The tumour occurs in middle age persons. from odontogenic epithelium that resembles follicular
l The mandible is involved more often than maxilla. ameloblastoma but contains ghost cells and spherical
l Molar region is more common site of occurrence fol- calcifications.
lowed by premolar region. l Most likely to affect the anterior areas of the jaws.
l The tumour presents a slow enlarging, painless swelling l It is most common in people in their second to third
of jaw with expansion and distortion of cortical plates. decades but can be seen at almost any age.
l The swelling is bony hard and clinically it is well de- l In one-third of cases, an impacted tooth is involved.
fined or diffused.
l Pain, paraesthesia may develop on rare occasions and Radiological Findings
few lesions may be completely asymptomatic. l The central intrabony lesions appear radioluscency.
l Variable mounts of calcified radio-opaque material are
Histopathology usually scattered throughout the radiolucency ranging
from tiny flecks to large masses.
The tumour reveals sheet of closely packed, polyhedral
cells, non-inflamed connective tissue stroma. The tumour
cells contain oval-shaped nuclei and homogenous eosino- Histologic Features
philic cytoplasm. Microscopically, there are many cells that are described as
l Prominent intracellular bridges and distinct cell bound- “ghost cells”, enlarged eosinophilic epithelial cells without
aries are often found in the lesions. nuclei.
l Some amount of homogenous, hyaline material is of-
Q. 4. Ameloblastic fibroma.
ten deposited in between tumour cells called amyloid
material. Ans.
l One of the most important histological characteristics of
l Ameloblastic fibroma may develop from the dental fol-
CEOT is the presence of several calcified masses in and
around the tumour cells. licle before or after the onset of calcification of tooth.
l Some Liesegang rings are also found.
Clinical Features
Radiological Features l Young individuals are affected mostly and is seen
before 20 years of age.
l The tumour shows considerable variations. l Common site – premolar molar area in mandible.
l It is either diffuse or a well circumscribed. l It occurs as slow growing and asymptomatic swelling.
l Unilocular radiolucent area (or) l Enlarges by gradual expansion so that the periphery of
l Combined pattern of radiolucency or opacity with many bone often remains smooth.
irregular bony trabeculae traversing the radiolucent area
in many directions producing multilocular or honey-
Histological Features
comb pattern.
l Scattered flecks of calcification throughout radiolu- l Both epithelium and connective tissue are neoplastic
cency giving it a snow driven appearance. (Fig 4.7).
l Cells of epithelium are ameloblast-like cells that are
surrounding stellate reticulum-like cells.
Treatment l Islands of epithelium may be seen in fibromyxoid con-
Surgical enucleation. nective tissues.
Section | III Oral Pathology 479

l The border is definite and is even surrounded by a thin


layer of sclerotic bone.

Stands of odontogenic
epithelium
Hitological Features
Amtloblast like cells l Thin non-keratinized epithelium of 1-5 layers resem-
bling reduced enamel epithelium.
Embryonic connective l Cuboidal or even columnar lining cells having clear,
tissue
vacuolated, glycogen rich cytoplasm.
l This lining is incomplete and easily sloughs away.
FIGURE 4.7 Histological features of ameloblastic fibroma
Treatment
The lateral periodontal cyst is treated by surgical enucle-
Radiological Features ation.
Unilocular or multilocular radiolucency associated with Q. 6. Write briefly histopathology of keratocyst.
unerupted or missing tooth.
Ans.
Treatment Histological features of keratocyst are as follows:
l The epithelial lining is highly characteristic.
Amelobastic fibroma is treated by conservative resection.
l A parakeratin surface which is usually corrugated rip-
Q. 5. Lateral periodontal cyst. pled or wrinkled.
l Uniformity of thickness of epithelium and is generally
Ans.
between 6 and 10 cells in depth.
l Lateral periodontal cyst is a slow growing non-expansile l Prominent palisaded, polarized basal cell layer often
developmental odontogenic cyst. described as having a “picket fence” or “tombstone”
l It is quite uncommon but well recognised. appearance.
l Occurs on a lateral periodontal location, located mid- l Occasionally, orthokeratin is found but if present, para-
distance between the apex and the cervical area of the keratin is evident.
affected tooth. l Connective tissue shows “daughter cells” or “satellite
l Tooth is asymptomatic and vital. cysts” because of which it shows high recurrence rate.
l It is of developmental origin arising from cystic degen- l The lumen of keratocyst may be filled with thin straw-
eration of clear cells of the dental lamina containing an coloured fluid or with thick creamy material with a
embryonic lining of one to three cuboidal cells and dis- protein content of 3.5 gm%.
tinctive focal thickenings (plaques). l Sometimes a lumen contains a great deal of keratin
while at other times it has little cholesterol as well as
Clinical Features hyaline bodies at the site of inflammation.
l Epithelium and connective tissue interface is weak.
l Age occurs chiefly in adults of mean age of 50 yrs with l No rete ridges are present.
an age range of 22–85 yrs.
l Sex: Predilection for males over females. Q. 7. Liesegang’s ring calcification.
l Site: Mandibular premolars/cuspid / incisor Ans.
l Very less frequently found in maxillary lateral incisor area.
l Majority of cases show no clinical signs and symptoms and l Liesegang rings are laminated ring-like structures
are discovered during radiographic examination of teeth. occasionally found in benign cysts and abscesses.
l Occasionally, if it is present on the labial aspect of root, there l They have been confused with parasites (especially
may be slight mass obvious with normal overlying mucosa. eggs), algae, calcifications, and psammoma bodies.
l If infected, it resembles lateral periodontal abscess. l Liesegang rings were best observed with Papanicolaou,
hematoxylin-eosin, Masson’s trichrome, acid-fast
(AFB), and Gram stains, which accentuate the concen-
Radiographic Features
trically laminated morphology.
l The majority of the cases are seen as a well delineated, l An amorphous electron-dense core and fibrillary lucent
round, small (not exceeding 1 cm in diameter), radiolu- concentric rings were seen with transmission electron
cency with a radio-opaque rim. microscopy with no significant energy peaks by electron
480 Quick Review Series for BDS 3rd Year

probe microanalysis or distinct diffraction patterns by l Hence the tissue has been described as blood soaked
X-ray diffraction. sponge with large pores representing cavernos spaces of
l This investigation indicates that Liesegang rings are the lesion.
composed of organic substances most likely formed by l Four phases of pathogenesis:
periodic precipitation from a supersaturated solution i. Osteolytic initial phase
within cystic fluid. ii. Active growth phase
l Awareness of the Liesegang phenomenon within iii. Mature stage – stage of stabilization
cystic lesions will decrease the possibility of erro- iv. Healing phase
neous misdiagnosis as another type of pathologic l Radiographically: Honeycomb or soap-bubble appear-
process. ance.
l It is a charachteristic feature of Pindborg’s tumour. l Histologically: Fibrous connective tissue stroma con-
l The calcification actually appears to occur in some in- taining many cavernous or sinusoidal blood-filled spaces.
stances in the globules of amyloid material many of
which are transformed from being PAS (periodic acid
Treatment
Schiff) negative to PAS positive during this calcification
process. l Surgical curettement or excision
l Low doses of radiation
Q. 8. Aneurysmal bone cyst.
Q. 9. Enumerate benign epithelial odontogenic tumours.
Ans.
Ans.
It is an intresting solitary lesion of bone given by Jaffe and
Lichtenstein in 1942. Benign tumours are classified as follows:
i. Odontogenic epithelium without odontogenic ectomes-
enchyme
Clinical Features a. Ameloblastoma
l Most commonly seen in young persons of age under b. Squamous odontogenic tumour
20 years. c. Calcifying epithelial odontogenic tumour
l Found equally in males and females. d. Adenomatoid odontogenic tumour.
l Found mostly in mandible than maxilla. ii. Odontogenic epithelium with odontogenic ectomesen-
l Associated often with the history of traumatic injury chyme with or without hard tissue formation:
preceding development of lesion. a. Ameloblastic fibroma
l It occurs generally in long bones and vertebral column. b. Ameloblastic fibro-odontome
l Lesions are also seen frequently in clavicle, rib, skull c. Complex odontome
and bones of hands and feet. d. Compound odontome
l Symptoms: Lesions are usually tender or painful and iii. Odontogenic ectomesenchyme with or without in-
hence limit the movement of affected bone. cluded odontogenic epithelium
l Swelling over the involved bone area. a. Odontogenic fibroma
l Excessive bleeding is encountered on entering the le- b. Myxoma
sion at the time of operation. c. Cementoblastoma

SHORT NOTES
Q. 1. Odontomas. fashion that the structure bears considerable ana-
tomic resemblance to normal teeth, except that
Ans.
they are often smaller than typical teeth.
l Odontomas are hamartomas that contain both epithelial l Treatment: By surgical enucleation.
and mesodermal dental tissue components.
l Generally there are two types of odontoma:
i. Complex odontoma: It is always benign and contains Q. 2. Radicular cyst.
enamel, dentin and cementum which are not differenti- Ans.
ated, so that structure of actual tooth is not identifiable.
ii. Compound odontoma: In compound odontomas, Radicular cyst is also known as apical periodontal cyst or
the enamel and dentin are laid down in such a dental root end cyst, periapical cyst.
Section | III Oral Pathology 481

Clinical Findings l Histologically, it shows neoplastic proliferation of


odontogenic epithelial cells mostly in distinct patterns.
l Common in maxillary incisor region with male pre-
l Follicular type
dominance.
l Plexiform type
l The tooth involved is generally nonvital and is asymp-
l Acanthomatous type
tomatic.
l Granular type and
l Large lesions often produce a slow enlarging bony hard
l Basal cell type.
swelling of the jaw with expansion of cortical plates.
l Radiographical appearance: “Honeycomb” appearance.
l If the cyst is secondarily infected, it leads to the forma-
l Treatment: Complete removal of neoplasm, curretage.
tion of the abscess, which is called “cyst abscess”.
l Pus formation in the cyst will lead to sinus and dis- Q. 5. Dentigerous cyst.
charge of pus.
Ans.
l Dentigerous cyst is also called as follicular cyst or peri-
Treatment coronal cyst.
l Complete enucleation and extraction of affected tooth. l It is the odontogenic cyst that surrounds the crown of
the impacted tooth by expansion of its follicle and is
Q. 3. Primordial cyst. attached to neck.
Ans.
l Primordial cyst is also called as odontogenic keratocyst. Clinical Features
l It mainly arises from the dental lamina or its remnants, l Seen in the 2nd and 3rd and 4th decades with male pre-
primordium of developing tooth germ or enamel organ dilection.
or from basal cell layer of oral epithelium. l The most common sites of the cyst are the mandibular
l The peak incidence is between 2nd and 3rd decades of and maxillary third molar and maxillary cuspid areas,
life. It is found more frequently in males as compared to since these are the most commonly impacted teeth.
females. l Generally, it is asymptomatic and is painful, if is
l The mandible is affected more commonly than maxilla. infected with gross swelling.
l It is asymptomatic unless they become secondarily in- l There is extreme displacement of teeth, root resorption
fected in which case patient complains of pain, soft tis- of adjacent teeth.
sue swelling and drainage.
l Histopathologically, connective tissue shows “daughter
cells” or “satellite cysts” because of which it shows high Treatment
recurrence rate. l Marsupilization – in children with larger cyst.
l The treatment of choice is enucleation. l Enucleation–in adults.
l Syndrome associated with OKC Gorlin Goltz syn-
drome. Q. 6. Pindborg’s tumour.

Q. 4. Ameloblastoma. Ans.

Ans. l Pindborg tumour is also called as calcifying epithelial


odontogenic tumour.
l Ameloblastoma is also known as adamantoblastoma, l Pindborg tumour as it was described by Jens J Pindborg
adamantinoma, epithelial odontoma. in 1956 is locally aggressive neoplasm, which is uncom-
l Definition of Robinson: Tumour of odontogenic origin mon, benign, odontogenic neoplasm that is exclusively
usually unicentric, nonfunctional, intermittent ingrowth epithelial in origin.
anatomically benign and clinically persistent.

Clinical Features
Clinical Features
l The tumour occurs in middle age persons.
l The ameloblastoma occurs in 2nd, 3rd, 4th and 5th de- l The mandible is involved more often than maxilla. Mo-
cades of life. lar region is more common site of occurrence followed
l The males are affected more commonly involves man- by premolar region.
dible in molar ramus area. l The tumour presents a slow enlarging, painless swell-
l Clinically, it presents as slow enlarging, painless, ovoid ing of jaw with expansion and distortion of cortical
and fusiform bony hard swelling of the jaw. plates.
482 Quick Review Series for BDS 3rd Year

l The swelling is bony hard and clinically it is well de- l The gingival cyst of the adult is a cyst which arises from
fined or diffused. the rest cells of the dental lamina.
l Pain, paraesthesia may develop on rare occasions and l It is most common in the mandible near the canines and
few lesions may be completely asymptomatic. premolars. Middle-aged adults are more likely to be
affected.
Treatment l It appears as a bluish swelling on attached gingiva or
interdental papilla. Usually do not exceed 1cm in size.
Surgical enucleation. l The epithelial lining of these cysts is thin non-kera-
Q. 7. Epidermoid cyst. tinised.
l It is circumscribed radioluscent cystic lesion of alveolar
Ans. bone with some swelling of soft tissue.
l An epidermoid cyst is a benign cyst usually found on l Treatment consists of local surgical excision.
the skin.
Q. 10. Malignant potential of dentigerous cyst.
l Several synonyms exist for epidermoid cysts, including
epidermal cyst, epidermal inclusion cyst, infundibular Ans.
cyst and keratin cyst.
l Mandibular lesions develop from both odontogenic and
l Epidermoid cysts commonly result from implantation
nonodontogenic origins and have varying degrees of
of epidermis into the dermis, as in trauma or surgery.
destructive potential.
l The cyst develops out of ectodermal tissue and mostly
l Malignant tumours that often involve the mandible in-
seen in women.
clude squamous cell carcinomas, osteosarcomas, and
l Histologically, it is made of a thin layer of squamous
metastatic tumours.
epithelium.
l Dentigerous cyst has the potentiality to give rise to fol-
l They are also seen in Gardner’s syndrome on the head
lowing malignant tumours:
and neck.
Squamous cell carcinoma
l They can be infected by bacteria and form a pimple-like
Mucoepidermoid carcinoma
shape.
l It is from the lining epithelium of dentigerous cyst
l Epidermoid cyst may be totally asymptomatic, or it may
which contains mucus secreting cells or cells with this
hurt when touched.
potential.
l Treatment: Surgical excision.
l Despite this information, however, many lesions are
Q. 8. Odontogenic keratocyst. impossible to differentiate without biopsy.
l In such cases, defining the degree of malignant potential
Ans.
is very helpful.
l OKC is also called as primordial cyst. l Although imaging will not always provide a specific
l It is named as keratocyst because the cyst epithelium diagnosis, it should help narrow the differential diagno-
produces so much keratin that it fills the cyst lumen. sis, thereby helping to guide patient treatment.
l It mainly arises from the dental lamina or its remnants,
Q. 11. Histopathology of odontogenic keratocyst.
primordium of developing tooth germ or enamel organ
and sometimes from basal cell layer of oral epithelium. Ans.
l The peak incidence is between 2nd and 3rd decades of life.
Histological features of odontogenic keratocyst are as
l It is found more frequently in males as compared to
follows:
females.
l The epithelial lining is highly characteristic. A para-
l The mandible is affected more commonly than maxilla.
keratin surface which is usually corrugated rippled or
l It is asymptomatic unless they become secondarily in-
wrinkled.
fected.
l Uniformity of thickness of epithelium and is generally
l Histopathologically, connective tissue shows “daughter
between 6 and 10 cells in depth.
cells” or “satellite cysts” because of which it shows high
l Prominent palisaded, polarized basal cell layer often
recurrence rate.
described as having a “picket fence” or “tombstone”
l Syndrome associated with OKC Gorlin-Goltz syndrome.
appearance.
Q. 9. Gingival cyst of an adult. l Connective tissue shows “daughter cells” or “satellite
cysts” because of which it shows high recurrence rate.
Ans.
l The lumen of keratocyst may be filled with thin straw-
l Gingival cysts of adult are rare odontogenic cysts of coloured fluid or with thick creamy material with a
developmental origin. protein content of 3.5 gm%.
Section | III Oral Pathology 483

l Sometimes a lumen contains a great deal of keratin l Rete pegs formation is absent except in case of second-
while at other times it has little cholesterol as well as arily infected cyst.
hyaline bodies at the site of inflammation. l The connective tissue wall is frequently quite thick-
l Epithelium and connective tissue interface is weak, no ened and composed of very loose fibrous connective
rete ridges are present. tissue.
l Inflammatory cells commonly infiltrate the connective
Q. 12. Haemorrhagic cyst.
tissue consisting of neutrophils and plasma cells.
Ans. l It also shows Rushton bodies within the lining epithe-
l Haemorraghic cyst is also called as traumatic or solitary lium which are peculiar linear and often curved hyaline
bone cyst. bodies.
l This is not a true cyst as it is not lined by epithelium but l The content of cystic lumen is usually thin watery yel-
by a fibrous tissue, so it is just a intrabony cavity. low fluid and is occasionally blood tinged.
l Aetiology: History of trauma which patient may or may l Protein content in lumen is greater than 5 gm/dl.
not remember it. Origin is due to failure of organization Q. 15. Potential complications of dentigerous cyst.
of blood clot and subsequent liquefaction.
l Cyst contents: Varies, may be fluid, or viscid yellow Ans.
fluid or the cyst may be empty with only few pieces of The potential complications of dentigerous cyst are as
blood clot and old blood pigment. follows:
l Clinical findings: Asymptomatic swelling or no swell- l Ameloblastoma arises either from lining epithelium or
ing. More often in males and young individuals of age from rests of odontogenic epithelium in the wall of the
18-20 yrs and in the posterior part of the mandible. cyst.
Teeth remain vital in the affected area. l Squamous cell carcinoma
l Unilateral paraesthesia of lower lip due to pressure of l Mucoepidermoid carcinoma arises from the lining epi-
cyst on the nerve. thelium of dentigerous cyst which contains mucus se-
l Treatment: Open the lesion surgically, curette the lining, creting cells or cells with this potential.
cause fresh bleeding to encourage bone deposition.
Q. 16. Cholestrol crystals.
Q. 13. Adenomatoid odontogenic tumour.
Ans.
Ans.
l Cholestrol crystals appear microscopically as clear
l Adenomatoid odontogenic tumour is also known as
needle-like spaces or clefts owing to the dissolving of
adenoameloblastoma, ameloblastic adenomatoid tumour.
the contained cholesterol by the agents used in the pre-
l It is uncommon, well-circumscribed, odontogenic neo-
pation of the tissues for histological examination.
plasm characterized by the formation of multiple duct-
l They are associated with multinucleated gaint cells of
like structures by neoplastic epithelial cells.
the foreign body type.
l Commonly seen in radicular cysts and OKC.
Clinical Features l Source: By disintegration of RBC, degeneration and
l Tumour usually occurs in younger age. Females are disintegration of lymphocytes and plasma cells, or from
more commonly affected and generally appears in max- giant cells, or circulating plasma lipids.
illary anterior region. l Complications include fistula formation, osteomyelitis,
l The tumour presents a slow enlarging, small, bony hard cellulitis, suamous cell carcinoma.
swelling in maxillary anterior region. Q. 17. Rushton bodies.
l There is displacement of regional teeth, mild pain and
expansion of cortical bones. Ans.
l Types: Intraosseous–96% and extraosseous–4 %
l Rushton bodies or hyaline bodies are peculiar linear,
l Treatment: Surgical enucleation
often curved with variable stainability.
Q. 14. Histopathology of dentigerous cyst. l Source: Keratinized secondary enamel cuticle, odonto-
genic epithelium or haematogenous origin from thrombi.
Ans.
l They present within the lining epithelium of the cyst
Histopathological features of dentigerous cyst are as follows: especially those exhibiting inflammation, e.g. dentige-
l It is usually composed of thin connective tissue wall nous cysts.
with a thin layer of stratified squamous epithelium of l They are haematogenous in origin and are brittle and
2 – 3 layers lining the lumen. fracture immediately.
484 Quick Review Series for BDS 3rd Year

Q. 18. Enamel pearl. l The ghost cell is a unique cell type occurring in a
variety of odontogenic and non-odontogenic lesions.
Ans.
l The ghost cells contained nuclear remnants of cytoplas-
l Enamel pearl is also called as ectopic enamel. mic organelles and numerous tonofilaments.
l This is a condition where pearl of an enamel is seen in l They are larger often vacuolated and the remnants of
unusual location. nuclear membranes are more prominent.
l These are hemispheric or dome-shaped white calcified l This may be due to intracellular oedema and the pres-
projections of enamel. ence of dilated degenerated membranous organelles.
l A localized bulging of odontoblastic layer which is l There presence within the proliferative odontogenic
radio-opaque. epithelium is the essential characteristic for the di-
l Mostly seen in roots of maxillary molars in the furcation agnosis.
areas composed of normal enamel. l The ghost cell keratinisation is observed in odontomas,
ameloblastomas, ameloblastic fibro-odontomas, and in
Q. 19. Ghost cells.
ameloblastic odontomas.
Ans.

Topic 5

Bacterial Infections of the Oral Cavity


LONG ESSAYS
Q. 1. Enumerate the various bacterial infections and I. IMPETIGO
describe them in detail.
l It is a superficial infection of the skin that is caused by
Streptococcus pyogenes and Staphylococcus aureus pre-
Ans.
senting as a scabbing eruptions.
The various bacterial infections are as follows: l Occurs in two clinically distinctive patterns:
i. Impetigo i. Nonbullous impetigo and
ii. Erysipelas ii. Bullous impetigo.
iii. Streptococcal tonsillitis and pharyngitis l Treatment:
iv. Scarlet fever i. Topical mupirocin is effective and fusidic acid is
v. Diphtheria also very effective.
vi. Gonorrhoea ii. Removal of crusts with a clean cloth soaked in
vii. Tuberculosis warm soapy water is recommended before applica-
viii. Leprosy tion of topical therapy.
ix. Actinomycosis iii. For bullous or more extensive lesions, the treatment
x. Noma of choice is one week course of systemic oral antibi-
xi. Syphilis, etc. otics. Cephalexin, trimethoprim-sulfamethoxazole,
Section | III Oral Pathology 485

dicloxacillin, flucloxacillin, and amoxicillin-clavu- VI. GONORRHOEA


lanic acid represent good current choices.
l Gonorrhoea is primarily a venereal disease caused by
Neisseria gonorrhoeae a gram-negative organism.
II. ERYSIPELAS l Infection in males results in acute urethritis, dysuria,
l It is a superficial infection of the skin that is most com- and urethral discharge of purulent nature. It may lead to
monly associated with b haemolytic streptococci. epididymitis, chronic prostitis, balanitis and posterior
l The infection spreads through the lymphatic channels. urethritis.
l Usually, on the face, the lesions tend to occur on cheeks, l In females, it manifests as cervicitis with candidal or
eyelids and bridge of the nose producing butterfly shape. trichomonal vaginitis.
l The affected area is painful, bright red, well circum- l Extra genital or oral features include acute painful ul-
scribed, swollen, indurated and warm to touch. Affected ceration of lips, thick dry fissured tongue and inflamed
skin exhibits surface texture resembling an orange peel. gingiva and tonsils.
l High fever and lymphadenopathy are often present. l Oral lesions are commonly accompanied by fever and
regional lymphadenopathy.
III. STREPTOCOCCAL TONSILLITIS l Can be treated by antibiotics, such as norfloxacin and
ceftriaxone.
AND PHARYNGITIS
Tonsillitis and pharyngitis are extremely common in
l
VII. TUBERCULOSIS
school-aged children and are caused by group A,b-
haemolytic streptococci, adenoviruses, influenza and l In India, TB is the most common oppurtunistic infection
Epstein-Barr virus. caused by Mycobacterium tuberculosis an acid-fast
bacillus.
IV. SCARLET FEVER l TB is a specific infectious granulomatous disease.
l Clinical features:
l Scarlet fever is a systemic infection in children caused i. General clinical signs and symptoms are remarkably
by b-haemolytic type of streptococci, i.e. St. pyogenes. inconspicuous. The patient may suffer from episodic
l Skin rashes are produced due to injury to vascular endo- fever and chills, but easy fatigability and malaise are
thelium and hyperaemia and give ‘sand paper ‘feel to skin. often the chief early features of the disease.
l Charateristic bright scarlet skin rash appears on 2nd or ii. Miliary TB: Microorganism may become dissemi-
3rd day of illness. This rash is prominent in skin folds nated by either bloodstream or lymphatics to in-
and is called ‘Pasta Lines’. volve many organs like kidney, liver.
l The oral manifestations are known as “stomatitis scar- iii. Scrofula: Tuberculous lymphadenitis of submaxil-
lantina”. lary and cervical lymph nodes.
l “Strawberry tongue” and “raspberrv tongue” are char- iv. Lupus vulgaris - primary TB of skin.
acteristic of scarlet fever. l Oral manifestations:
l Treatment: Antibiotics particularly penicillins. i. Oral lesions are usually secondary to pulmonary
infection.
V. DIPHTHERIA ii. Oral ulcers have undermined edges.
iii. Most commonly affected intraoral site is tongue fol-
l Diphtheria is caused by Corynebacterium diphtheriae
lowed by palate, lips, buccal mucosa, gingiva and
also known as Klebs-Loefller bacillus through droplet
frenulae.
infection.
iv. Upper and lower jaws involved by anachoretic
l Clinical features: Malaise, fever, sore throat, tachycar-
effect in areas of PA inflammation forming tuber-
dia are the signs and symptoms. Bull neck or cervical
culoma.
lymphadenopathy may be seen.
l Histological features: Lesion characteristically consists
l Oral manifestations includes “diphtheritic membrane”,
of granuloma along with central caseous necrosis.
a pseudomembrane greyish green-coloured, which usu-
l Lab diagnosis:
ally begins on tonsils, pharynx or larynx.
l There is temporary paralysis of soft palate during 3rd to Common method of D:
5th weeks of disease causing airway obstructions. i. Demonstration of the acid-fast bacilli (AFB) in the
l Complications: Myocarditis and perineuritis. sputum smear is gold standard for diagnosis of TB.
l Treatment includes prophylaxis with DPT vaccine and ii. Chest X-ray.
antibiotics. iii. Montoux test (tuberculin test)
486 Quick Review Series for BDS 3rd Year

iv. CT scan: Used to diagnose mediastinal or hilar lymph- iii. Peripheral nerves such as facial and trigeminal nerves
adenopathy, cavities and intralesional calcification. are frequently involved leading to facial paralysis, dif-
v. MRI: Most useful for diagnosis of extrapulmonary ficulty in phonation and mastication.
tuberculosis. iv. Features of advanced disease are nodules or lump in
vi. Blood culture skin of face and ears, plantar ulcers, loss of fingers and
toes, nasal dipression, foot drop, claw toe and others.
New methods of D are:
v. The disease is a crippling and disfiguring one and runs
i. RIAs (radioimmune assay)
a chronic course but rarely causes sudden death.
ii. Soluble antigen fluorescent antibody test (SAFA)
vi. Oral lesions consists of small tumour-like masses
iii. Enzyme-linked immunosorbant assay (ELISA)
called lepromas on tongue, lips and hard palate.
iv. DNA probes
vii. The nodules show a tendency to breakdown and ulcer-
v. Polymerase chain reaction (PCR)
ate. Gingival hyperplasia with loosening of teeth.
l Treatment: Multiple drug therapy is recommended

as M. tuberculosis mutates and resists single drug


therapy. Histologic Features
Antitubercular drugs should be prescribed as follows: i. Typical granulomatous nodule shows collections of
i. Isoniazid (INH) 1 Rifampicin n for 9 months. Or epithelioid histiocytes and lymphocytes in a fibrous
Isoniazid (INH) 1 Rifampicin 1 Pyrizinamide n for stroma.
2 months followed by Isoniazid (INH) 1 Rifampicin for ii. Langhans type giant cells are variably present.
4 months. iii. Sheets of lymphocytes with vacuolated macrophages
ii. Other drugs used are streptomycin and ethambutol. called “lepra cells” are sattered throughout the lesions.
iv. There is an abundance of organisms in lepromatous type
while in tuberculoid type there is a paucity of organisms.
VIII. LEPROSY
Leprosy is also known as Hansen’s disease. It is a
Diagnosis
l

chronic granulomatous infection caused by Mycobacte-


rium leprae. l Based on clinical presentation and bacteriological ex-
l Leprosy manifests in two forms: amination.
A. Tuberculoid type and
B. Lepromatous type
Treatment
Specific long-term chemotherapy is initiated up on di-
Tuberculoid Type l

agnosis. Rifampicin and dapsone for 6 months in case


l Develops in patients with a high immune reaction to the of tuberculoid type and rifampicin and dapsone along
organism and the disease is usually localized. with clofazimine in case of lepromatous type is usually
l It manifests as single or multiple, macular, ery- advocated.
thematous lesions with dermal and peripheral nerve l Once infection is treated, therapy is directed towards
involvement. reconstruction of the damage, physiotherapy and educa-
tion of patients.
Lepromatous Type
IX. ACTINOMYCOSIS
l Develops in patients who demonstrate a reduced cell-
mediated immune response and exhibits as a diffuse l Actinomycosis is a chronic granulomatous suppurative
disease. and fibrosing disease.
l Occurs as macules or papules leading to progressive l Endogenous, opportunistic infection.
thickening of skin and characteristic nodules may pro-
duce severe disfigurement.
Aetiology
It is caused most commonly by Actinomyces israelii, al-
Clinical Features l

though A. naeslundi, A. viscosus, A. odontolyticus and A.


General features of leprosy are: propionica have been shown to cause the human disease.
i. Hypopigmented patches l This bacterium is anaerobic gram-positive, fungus like
ii. Partial or total loss of cutaneous sensation in the af- filamentous and branched and normally present in
fected areas. crypts of tonsils and cavities over teeth.
Section | III Oral Pathology 487

Clinical Features l The overlying skin is inflamed, oedematous and finally


necrotic.
i. Actinomycosis is classified anatomically into three
forms according to location of lesion as:
(a) Cervicofacial (most common) Treatment
(b) Abdominal and
l Antibiotics should be given.
(c) Pulmonary
l Immediate treatment of any existing malnutrition further
ii. Cervicofacial actinomycosis is characterized by “lumpy
improves the probability of saving the patient.
jaw” and swelling of the soft tissue with formation ex-
tra oral draining sinuses over skin or mucosal surface.
iii. Pus collected from sinus shows typical “sulfur granules” XI. SYPHILIS
or tiny yellow grains which are colonies of organism.
l Syphilis is caused by Treponema pallidum a spirochaete/
iv. The skin over the sinus is scarred. No lymphadenopathy
demostrated best by darkfield microscopy in silver
is present.
impregnation.
v. It may also cause osteomyelitis, if not treated.
l Syphilis may be classified as:
vi. Abdominal actinomycosis is an extremely serious form
a) Acquired and
of the disease and carries high mortality rate.
b) Congenital

Treatment
A) Acquired Syphilis
l Treatment of this disease is difficult and has not been
Three stages:
uniformly successful.
l Primary
l Long-standing fibrosis cases are treated by draining the ab-
l Secondary
scess, excising the sinus tract with high doses of antibiotics.
l Tertiary
l Antibiotics such as penicillins and tetracyclines have
been most frequently used. Primary and secondary stages are infectious and painless.

X. NOMA Primary stage (chancre):


l Chancre develops at the site of inoculation approxi-
l Noma is also called as cancrum oris or gangrenous sto-
mately 3-90 days after contact with the infection.
matitis.
l Chancre is usually solitary but may be multiple at times.
l It is specific infection by Vincent’s organism.
It occurs mainly on genitalia, may occur on oral mucosa
l It is rapidly spreading, mutilating gangrenous stomatitis
and fresh extraction wound, as painful ulcers.
of oral and facial tissues occurring usually in debilitated
l Highly infectious, and exhibits positive serologic reac-
or nutritionally deficient patients.
tion despite the presence of spirochaete.
l Unilateral lymphadenopathy, non-tender, rubbery nodes.
Predisposing Factors l The Chancre appears microscopically as a superficial
l Undernourished persons. ulcer showing intense inflammatory infiltrate especially
l Debilitated from infections, e.g. diphtheria, measles, plasma cells.
pneumonia, scarlet fever, TB and blood dyscrasias. l Chancre heals spontaneously in 3 weeks to 2 months
l Excessive mechanical injury. time.

Secondary or metastatic stage (mucous patches):


Clinical Features
l Usually commences after 6 weeks after primary lesions.
l It is seen chiefly in malnourished children. l The lesions are typically multiple and occur on skin as
l Begins as small ulcers of gingival mucosa which spreads painless macules or papules. The oral lesions are called
rapidly and involves surrounding tissue of jaw, lips and ‘mucous patches’ and are usually painless, multiple,
cheeks by gangrenous necrosis. greyish white plaques overlying an ulcerated surface.
l The initial sites are commonly the areas of stagnation l The mucous patches occur more frequently on, tongue,
around the fixed bridge or crown. gingiva, or buccal mucosa or as a split papule on lips
l Commencement of gangrene is denoted by blackening and are highly infectious.
of skin. l Serologic reaction is always positive.
l The odor is extremely foul and patients have high tem- l Secondary syphilis can present as explosive and wide
perature. spread form known as ‘lues maligna’.
488 Quick Review Series for BDS 3rd Year

Tertiary or late syphilis l Syphilis may be classified as:


l Tertiary or late syphilis is non-infectious and occurs a) Acquired and
several years later. b) Congenital
l Diffuse form may involve cardiovascular and central
nervous system, i.e. cardiosyphilis and neurosyphilis. A) Acquired Syphilis
l Gumma is classic of tertiary or late syphilis, it is a
granuloma with central necrosis occurring most com- Aetiology
monly on tongue or palate. l Acquired form of syphilis is contracted:
l Palatal perforation by ulcer after vigorous antibiotic i. Primarily as a venereal disease
use, known a Herxheimer reaction. ii. After sexual intercourse with an infected partner
l Atrophic/interstitial glossitis is most characteristic lesion iii. Innocently by dentists and other health care persons
and has malignant potential to sqamous cell carcinoma. working on infected patients in contagious stage.
l The disease, if untreated, manifests 3 distintive stages
B. Congenital Syphilis throughout its course:
a. Primary
l Congenital syphilis is transmitted to the offspring only
b. Secondary
by an infected mother and is not inherited.
c. Tertiary
l Congenital (prenatal) syphilis manifest following great
variety of lesions: Primary and secondary stages are infectious and painless.
i. Frontal bossing of Parrot.
ii. Hypoplastic maxilla and Primary stage (chancre):
iii. Relative protuberance of mandible. l Chancre develops at the site of inoculation approxi-
iv. High arched palate. mately 3-90 days after contact with the infection.
v. Saddle nose l Chancre is usually solitary but may be multiple at times.
vi. Mulberry molars It occurs mainly on genitalia, may occur on oral mucosa
vii. Higoumenakis’s sign - irregular thickening of sterno- and fresh extraction wound, as painful ulcers.
clavicular portion of clavicle. l Highly infectious, and exhibits positive serologic reac-
viii. Saber shin tion despite the presence of spirochaete.
ix. Rhagades l Unilateral lymphadenopathy, non-tender, rubbery nodes.
l Pathognomonic of the disease is the occurrence of l The chancre appears microscopically as a superficial
Hutchinson’s triad: ulcer showing intense inflammatory infiltrate especially
i. Hypoplasia of incisors and molars (screw driver-shaped plasma cells.
incisors, mulberry molars/moon’s/Fournier’s molar). l Chancre heals spontaneously in 3 weeks to 2 months
ii. Eighth nerve deafness and time.
iii. Interstitial keratitis
Secondary or metastatic stage (mucous patches):
Diagnosis l Usually commences after 6 weeks after primary lesions.
l The lesions are typically multiple and occur on skin as
Diagnosisn can be made by:
painless macules or papules. The oral lesions are called
i. Darkfield microscopy
‘mucous patches’ and are usually painless, multiple,
ii. TPHA (Treponema pallidum haemagglutination assay)
greyish white plaques overlying an ulcerated surface.
iii. Fluorescent treponemal antibody absorption test.
l The mucous patches occur more frequently on tongue,
iv. RPCF (Reiter’s protein complement fixation)
gingiva, or buccal mucosa or as a split papule on lips
and are highly infectious.
Treatment l Serologic reaction is always positive.
l Penicillin up to 15 days is the drug of choice. l Secondary syphilis can present as explosive and wide
l Tetracycline or doxycycline may be used, if patient is spread form known as ‘lues maligna’.
allergic to pencillin. After second stage, patients are free from lesions and
l Surgical correction of facial defects. symptoms and enter a latent stage which lasts for 1-30 years
Q. 2. Describe the clinical features and oral manifesta- till the next stage.
tions of congenital and acquired syphilis.
Tertiary or late syphilis:
Ans.
l Tertiary or late syphilis is non-infectious and occurs
l Syphilis is caused by Treponema pallidum a spirochaete/ several years later.
demostrated best by darkfield microscopy in silver im- l Diffuse form may involve cardiovascular and central
pregnation. nervous system, i.e. cardiosyphilis and neurosyphilis.
Section | III Oral Pathology 489

l Gumma is classic of tertiary or late syphilis, it is a vii. Higoumenakis’s sign - irregular thickening of ster-
granuloma with central necrosis occurring most com- noclavicular portion of clavicle.
monly on tongue or palate. viii. Saber shin
l Lesions appear as firm nodular mass in the tissue, which ix. Rhagades
subsequently ulcerate to form a deep painless ulcer. l Pathognomonic of the disease is the occurrence of
l Lesions of palate cause perforation by sloughing of Hutchinson’s triad:
necrotic mass of tissue. i. Hypoplasia of incisors and molars (screwdriver-shaped
l palatal perforation by ulcer after vigorous antibiotic use, incisors, mulberry molars/moon’s/Fournier’s molar).
known a Herxheimer reaction ii. Eighth nerve deafness and
l Atrophic/interstitial glossitis is most characteristic iii. Interstitial keratitis
lesion and has malignant potential to sqamous cell
carcinoma.
Diagnosis
Diagnosisn can be made by:
B. Congenital Syphilis i. Darkfield microscopy
l Congenital syphilis is transmitted to the offspring only ii. TPHA (Treponema pallidum haemagglutination assay)
by an infected mother and is not inherited. iii. Fluorescent treponemal antibody absorption test.
l Congenital (prenatal) syphilis manifest following great iv. RPCF (Reiter’s protein complement fixation)
variety of lesions:
i. Frontal bossing of Parrot.
Treatment
ii. Hypoplastic maxilla and
iii. relative protuberance of mandible. l Penicillin up to 15 days is the drug of choice.
iv. High arched palate. l Tetracycline or doxycycline may be used, if patient is
v. Saddle nose allergic to pencillin.
vi. Mulberry molars l Surgical correction of facial defects.

SHORT ESSAYS
Q. 1. Oral manifestations of tuberculosis. iv. Tuberculosis may involve upper and lower jaws by ana-
choretic effect in areas of periapical inflammation form-
Or
ing tuberculous periapical granuloma or tuberculoma.
Tuberculosis ulcer of oral cavity. v. Tuberculous leison of jaw bone: Chronic osteomyeli-
tis of maxilla and mandible may occur and infection
Ans. reaches to bone via blood or root canal or extraction
socket. Tuberculous osteomyelitis of jaw bone pro-
l Tuberculous lesions of the oral cavity are relatively duces pain, swelling, sinus or fistula formation.
uncommon. l Histological features: Lesion characteristically consists
l Oral manifestations: of granuloma exhibiting foci of caseous necrosis sur-
i. Oral lesions are seldom primary but are usually rounded by epitheloid cells, lymphocytes and occa-
secondary to pulmonary infection. sional multinucleated giant cells.
ii. Primary oral tuberculosis is associated with regional l Lab diagnosis:
lymphadenopathy and usually involves gingiva and
presents as diffuse hyperaemic, nodular or papillary Common method of D:
proliferation of gingival tissues. i. Demonstration of the acid-fast bacilli (AFB) in the
iii. In secondary tuberculosisis, commonly affected in- sputum smear is gold standard for diagnosis of TB.
tra oral site is tongue followed by palate, lips, buc- ii. Chest X-ray.
cal mucosa, gingiva and frenula. Usually presents as iii. Montoux test (tuberculin test)
irregular, superficial or deep, painful ulcers which iv. CT scan
tend to increase slowly in size. v. MRI (extrapulmonary tuberculosis)
iv. Tuberculous lesions of tongue develop on the lateral vi. Blood culture
borders and appear as single or multiple ulcers New methods of D are:
which are well defined, painful, firm and yellowish i. RIAs (radioimmunoassays)
grey in colour. ii. Soluble antigen fluorescent antibody test (SAFA)
v. Occasional mucosal lesions present as nodules, iii. Enzyme-linked immunosorbant assay (ELISA)
vesicles, fissures, plaque, granulomas or verrucous iv. DNA probes
lesions but no obvious clinical ulceration. v. Polymerase chain reaction (PCR)
490 Quick Review Series for BDS 3rd Year

Treatment Primary stage (chancre)


Multiple drug therapy with antitubercular drugs is recom- l Chancre develops at the site of inoculation approxi-
mended as M. tuberculosis mutates and resists single drug mately 3-90 days after contact with the infection.
therapy. l Chancre is usually solitary but may be multiple at times
and occurs mainly on genitalia, may also occur on oral
Q. 2. Various types and lab investigations of tuberculosis. mucosa and fresh extraction wound, as painful ulcers.
Ans. l Highly infectious and heals spontaneously in 3 weeks to
2 months time.
l In India, TB is the most common oppurtunistic infection
caused by Mycobacterium tuberculosis an acid-fast bacillus. Secondary or metastatic stage (mucous patches)
l TB is a specific infectious granulomatous disease.
l Usually commences after 6 weeks after primary lesions.
Various types of tuberculosis are as follows: l The oral lesions are called ‘mucous patches’ and are
i. Pulmonary TB—is the chief form of disease. usually painless, multiple, greyish white plaques overly-
ii. Phithisis—TB of lungs ing an ulcerated surface.
iii. Scrofula—TB of lymph nodes l The mucous patches occur more frequently on, tongue,
iv. Lupus vulgaris—TB of skin gingiva, or buccal mucosa or as a split papule on lips
v. Pott’s disease—TB of spine, also known as cervical caries and are highly infectious.
vi. Miliary TB—if spreads by blood
vii. Cold abscess—formed by TB Tertiary or late syphilis
l Lab diagnosis: l Tertiary or late syphilis is non-infectious and occurs
Common method of D: several years later.
i. Demonstration of the acid-fast bacilli (AFB) in the l Gumma is classic of tertiary or late syphilis, it is a
sputum smear is gold standard for diagnosis of TB. granuloma with central necrosis occurring most com-
ii. Chest X-ray. monly on tongue or palate.
iii. Montoux test (tuberculin test) l Lesions appear as firm nodular mass in the tissue, which
iv. CT scan: Used to diagnose mediastinal or hilar lymph- subsequently ulcerate to form a deep painless ulcer.
adenopathy, cavities and intralesional calcification. l Lesions of palate cause perforation by sloughing of
v. MRI: Most useful for diagnosis of extrapulmonary necrotic mass of tissue.
tuberculosis. l palatal perforation by ulcer after vigorous antibiotic use,
vi. Blood culture known as Herxheimer reaction.
l Atrophic/interstitial glossitis is most characteristic lesion
New methods of D are: and has malignant potential to sqamous cell carcinoma.
i. RIAs (radioimmunoassays)
ii. Soluble antigen fluorescent antibody test (SAFA)
iii. Enzyme-linked immunosorbant assay (ELISA) b) Oral Manifestations of Congenital Syphilis
iv. DNA probes
Congenital (prenatal) syphilis manifest following great
v. Polymerase chain reaction (PCR)
variety of lesions:
Q. 3. Write briefly on oral manifestations of syphilis. l Hypoplastic maxilla and relative protuberance of
mandible.
Ans.
l High arched palate.
Syphilis is caused by Treponema pallidum a spirochaete/ l Mulberry molars, screwdriver-shaped incisors occur
demostrated best by darkfield microscopy in silver impreg- due to involvement of developing tooth germs.
nation. l Rhagades, i.e. fissuring and scaring at the corners of the
l Syphilis may be classified as: mouth.
a) Acquired and l Delayed eruption of teeth is present.
b) Congenital l Hypodontia and enamel hypoplasia is seen.
l Pathognomonic of the disease is the occurrence of
Hutchinson’s triad which includes:
a) Oral Manifestations of Acquired Syphilis i. Hypoplasia of incisors and molars (screwdriver-
l It manifests 3 distintive stages throughout its course: shaped incisors, mulberry molars/moon’s/Fournier’s
i. Primary molar).
ii. Secondary ii. Eighth nerve deafness and
iii. Tertiary iii. Interstitial keratitis
Section | III Oral Pathology 491

Diagnosis Q. 5. Diphtheria.
i. Darkfield microscopy Ans.
ii. TPHA (Treponema pallidum haemagglutination assay)
Diphtheria is a life-threatening, acute infectious communi-
iii. Fluorescent treponemal antibody absorption test.
cable disease of skin and mucous membrane caused by
iv. RPCF (Reiter’s protein complement fixation)
Corynebacterium diphtheriae.

Treatment
Aetiology
Penicillin up to 15 days is the drug of choice and surgical
Corynebacterium diphtheriae, also known as Klebs-
correction of facial defects.
Loefller bacillus.
Q. 4. Scarlet fever (scarlatina)
Ans. Clinical Features
Scarlet fever is highly contagious systemic bacterial infec- l Onset is gradual and it manifests as malaise, fever, sore
tion occurring predominantly in children. throat, tachycardia and change of voice. There may be
swelling of the neck (bull neck) and tender cervical
Cause lymphadenopathy.
l Oral manifestations:
Beta-haemolytic streptococci, erythrogenic toxins. There is formation of patchy diphtheritic membrane
which begins on the tonsils and enlarges becoming con-
Clinical Features fluent over the surface.
This pseudomembrane is greyish green-coloured and
l Scarlet fever is a systemic infection in children caused usually begins on tonsils, pharynx or larynx and is thick,
by b-haemolytic type of streptococci, i.e. St. pyogenes. fibrinous, gelatinous containing dead cells, leucocytes
l After incubation period of 3-5 days, the patient exhibits and bacteria.
severe pharyngitis and tonsillitis, headache, chills, fever, l There is temporary paralysis of soft palate during 3rd to
abdominal pain and vomiting. 5th weeks of disease. The patients will have peculiar na-
l Enlargement and tenderness of regional lymph nodes. sal twang and may exhibit nasal regurgitation of liquids.
l Skin rashes due to injury to vascular endothelium and l If uncausing airway obstructions checked in respiratory
hyperaemia and give ‘sand paper’ feel to skin. tract, the larynx may become oedematous and covered
l Because of 3 exotoxins (A,B,C), i.e. erythrogenic or by pseudomembrane which results in husky voice, this
scarlet fever toxins, characteristic bright scarlet skin may produce a mechanical respiratory obstruction and
rash appears on 2nd or 3rd day of illness. This rash is the typical cough or diphtheritic croup.
prominent in skinfolds and is called ‘Pasta Lines’.
l The oral manifestations are known as “stomatitis scarla-
tina”. Complications
l The tongue exhibits a white coating and the fungiform Myocarditis and perineuritis may result due to toxaemia.
papilla are hyperaemic, projecting above the surface. Incubation period (respiratory diphtheria): 2-5 days rarely
This is known as “strawberry tongue”. up to 8 days.
Later, the coating is lost and the tongue becomes red
and smooth except for swollen hyperaemic papilla. This is Diagnosis
known as “raspberrv tongue”.
l Incubation period: 3-5 days i. Based on clinical signs and symptoms.
l D: Clinical findings, routine blood examination, Dick ii. Shick test, Elek’s test
test, Scultz-Charlton test. iii. Definite D is based on culturing organisms on various
media like Pai agar and cystine tellurite agar and spe-
cial stains like Albert’stain,
Treatment
iv. Ponder’s stain or Neissers stains are used to demon-
l Antibiotics particularly penicillin, dicloxacillin and strate metachromatic granules.
cephalexin will ameliorate the disease and also help in
controlling possible complications.
l To relieve the discomfort local applications like mupiro-
Treatment
cin topical ointment can be used. Includes prophylaxis with DTP vaccine and antibiotics.
492 Quick Review Series for BDS 3rd Year

Q. 6. Actinomycosis. ii. Cervicofacial actinomycosis is characterized by


“lumpy jaw” and swelling of the soft tissue with for-
Ans.
mation extraoral draining sinuses over skin or muco-
l Actinomycosis is a chronic granulomatous suppurative sal surface.
and fibrosing disease. iii. Pus collected from sinus shows typical “sulfur granules”
l Endogenous, opportunistic infection. or tiny yellow grains which are colonies of organism.
iv. The skin over the sinus is scarred. No lymphadenopathy
is present.
Aetiology v. The infection of soft tissues may extend to involve the
l It is caused most commonly by Actinomyces israelii, al- mandible or maxilla resulting in osteomyelitis, if not
though A. naeslundi, A. viscosus, A. odontolyticus and A. treated.
propionica have been shown to cause the human disease. vi. Abdominal actinomycosis is an extremely serious form
l This bacterium is anaerobic gram-positive, fungus-like of the disease and carries high mortality rate.
filamentous and branched and normally present in
crypts of tonsils and cavities over teeth.
Treatment
Treatment of this disease is difficult and has not been
Clinical Features
l

uniformly successful.
i. Actinomycosis is classified anatomically into 3 forms l Long-standing fibrosis cases are treated by draining the
according to location of lesion as abscess, excising the sinus tract with high doses of anti-
(a) Cervicofacial (most common) biotics.
(b) Abdominal and l Antibiotics such as penicillins and tetracyclines have
(c) Pulmonary been most frequently used.

SHORT NOTES
Q. 1. Enumerate various types of tuberculosis. opisthotonus, i.e. entire body mucles affected; cephalic
tetanus, i.e. mostly 7th nerve palsy.
Ans. iv. Incubation period—6-10 days.
l In India, TB is the most common oppurtunistic infection Q. 3. Oral manifestation of syphilis.
caused by Mycobacterium tuberculosis and is a specific Ans.
infectious granulomatous disease.
l Various types of tuberculosis are as follows: a) Oral manifestations of acquired syphilis are:
i. Pulmonary TB—is the chief form of disease. i. Primary stage (chancre):
ii. Phithisis—TB of lungs l Chancre is usually solitary but may be multiple at
iii. Scrofula—TB of lymph nodes times occurring on oral mucosa.
iv. Lupus vulgaris—TB of skin ii. Secondary or metastatic stage (mucous patches):
v. Pott’s disease—TB of spine, also known as cervical l The oral lesions are called ‘ mucous patches’ and are
caries usually painless, multiple, greyish white plaques
vi. Miliary TB—if spreads by blood overlying an ulcerated surface.
vii. Cold abscess—formed by TB iii. Tertiary or late syphilis:
l Gumma is classic of tertiary or late syphilis, it is a
Q. 2. Tetanus/lockjaw.
granuloma with central necrosis occurring most
Ans. commonly on tongue or palate.
l Lesions of palate cause perforation.
i. Tetanus is an acute infection of nervous system charac-
terized by intense activity of motor neurons and severe b) Oral manifestations of congenital syphilis:
muscle spasm. l Hypoplastic maxilla and relative protuberance of mandible.
ii. Cause: Clostridium tetani, exotoxins, anaerobic G 1 l High arched palate.
bacillus l Mulberry molars, screwdriver-shaped incisors.
iii. Clinical features: Lockjaw/trismus; stiffness of jaw/ l Rhagades.
neck muscles, risus sardonicus, i.e. rigid facial muscles, l Delayed eruption of teeth is present.
Section | III Oral Pathology 493

Q. 4. Cervicofacial actinomycosis. Q. 8. Hutchinson’s triad.


Ans.
Ans.
Pathognomonic of the congenital syphilis is the occurrence
i. Cervicofacial actinomycosis is the most common form
of Hutchinson’s triad which includes:
of disease and is characterized by swelling of the soft
i. Hypoplasia of incisors and molars (screwdriver-shaped
tissue with formation extraoral draining sinuses over
incisors, mulberry molars/moon’s/Fournier’s molar).
skin or mucosal surface.
ii. Eighth nerve deafness and
ii. Pus collected from sinus shows typical “sulfur granules”
iii. Interstitial keratitis
or tiny yellow grains which are colonies of organism.
iv. The skin over the sinus is scarred and disfigured and no Q. 9. Gumma.
lymphadenopathy is present.
Ans.
v. The infection of soft tissues may extend to involve the
mandible or maxilla resulting in osteomyelitis, if not i. The gumma is a classical lesion of tertiary syphilis.
treated. ii. It occurs most frequently in the skin and mucous mem-
brane, liver and bone.
Q. 5. Tularaemia/rabbit fever.
iii. It consists of focal, granulomatous inflammatory process
Ans. with central necrosis.
iv. Intraoral gumma seen in tongue and palate. Lesion ap-
Tularaemia is also known as rabbit fever.
pears as a firm nodular mass in the tissue which may
i. Cause: Gram –ve bacillus Francisella tularensis also
ulcerate to form deep painless ulcer.
known as Bacterium tularense and Pasteurella tularensis
ii. Clinical features: Q. 10. Noma.
l Based on site of infection, six characteristic clinical
Ans.
syndromes of tularaemia are as follows:
l Ulceroglandular (most common type), glandular, i. Noma is a gangrenous stomatitis that occurs in debili-
oropharyngeal, pneumonic, occuloglandular and ty- tated or nutritionally deficient persons by vincent’s
phoidal. organisms.
l Eyes and lymph nodes get involved. ii. Clinical features: It begins as a small ulcer of the gin-
iii. Incubation period: up to 7 days. giva which rapidly spreads and involves jaws, lips and
iv. Treatment: Streptomycin is the drug of choice. cheeks by gangrenous necrosis.
iii. The overlying skin becomes inflamed, oedematous and
Q. 6. Histopathology of tuberculosis ulcer.
necrotic which results in line of demarcation between
Ans. healthy and dead tissue.
iv. Treatment: Antibiotic therapy with adequate nutritional
i. Characteristic histological appearance of tuberculosis
support.
ulcer is due to cell-mediated hypersensitivity reaction.
ii. Lesion characteristically consists of granuloma exhibit- Q. 11. Primary stage of syphilis.
ing foci of caseous necrosis surrounded by epithelioid
Ans.
cells, lymphocytes and occasional multinucleated giant
cells. i. Primary stage of syphilis is also known as chancre.
ii. Chancre develops at the site of inoculation approxi-
Q. 7. Sarcoidosis (Boeck’s sarcoid, Besnier Boeck–
mately 3-90 days after contact with the infection.
Schaumann disease).
iii. Chancre is usually solitary but may be multiple at times
Ans. and occurs mainly on genitalia, may also occur on oral
mucosa and fresh extraction wound, as painful ulcers.
i. Sarcoidosis is also known as Boeck’s sarcoid, Besnier
iv. Highly infectious and heals spontaneously in 3 weeks
Boeck–Schaumann disease)
to 2 months time.
ii. It is a multisystem granulomatous disease of unknown
origin. Q. 12. Congenital syphilis.
iii. Clinical features: Depressed delayed hypersensitivity,
Ans.
i.e. chronic CMI; increased abnormal serum immuno-
globulins suggesting lymphoproliferation. i. Congenital syphilis is transmitted to the offspring only
iv. Histologically, it resembles TB, but no caseous necro- by an infected mother and is not inherited.
sis; no AFB. ii. Congenital (prenatal) syphilis manifest following great
v. D: Kveim-Siltzbatch test. variety of lesions: Frontal bossing, hypoplastic maxilla
494 Quick Review Series for BDS 3rd Year

and relative protuberance of mandible, high arched pal- by draining the abscess, excising the sinus tract with
ate, saddle nose, mulberry molars, Higoumenakis’s high doses of antibiotics.
sign, Saber shin, rhagades, etc.
Q. 16. Ghon’s lesion.
iii. Pathognomonic of the disease is the occurrence of
Hutchinson’s triad: Ans.
a. Hypoplasia of incisors and molars
b. Eighth nerve deafness and i. Primary tuberculosis or Ghon’s complex or childhood
c. Interstitial keratitis. tuberculosis is the infection of an individual who has
not been previously infected or immunized.
Q. 13. Cancrum oris. ii. The most commonly involved tissues for primary com-
plex are lung and hilar lymph nodes.
Ans. iii. Primary complex or Ghon’s complex in lungs consists
of three components: Pulmonary, lymphatic vessel and
i. Cancrum oris is always an extension of an acute ulcer-
lymph node components.
ative gingivitis (i.e. Vincent’s disease) into the adjacent
iv. In pulmonary component, the lesion in the lung is
soft tissues.
called primary focus or Ghon’s focus. Tubercle bacilli,
ii. The bone exposed by the necrosis of the soft tissues
either free or within phagocytes drain to the regional
usually sequestrates in about 3 weeks.
lymph nodes, which often caseate. This combination of
iii. After removal of loose bone, and following the antibi-
parenchymal lesion and nodal involvement is referred
otic therapy, considerable scarring occurs which pro-
to as Ghon’s complex.
duces facial asymmetry and trismus.
Q. 17. Diagnosis of tuberculosis.
Q. 14. Treponema pallidum.
Ans.
Ans.
The various laboratory investigations for diagnosis of
l Treponema pallidum is the causative agent of syphilis. tuberculosis are:
‘Trepo’ means to turn, ‘nema’ means thread; ‘pallidum’ i. Demonstration of the acid-fast bacilli (AFB) in the
refers to its pale staining. sputum smear is gold standard for diagnosis of TB.
l It is a thin delicate spirochaete with tapering ends. It has ii. Blood culture
about 10 regular spirals, which are sharp and angular, at iii. Montoux test (tuberculin test)
regular intervals of about 1 micron. iv. Chest X-ray, CT scan and MRI (extrapulmonary tuber-
l It can be seen by dark ground microscope or negative culosis).
staining. It can be stained by silver impregnation meth- v. New methods of D are: RIAs (radioimmunoassays),
ods. It stains light rose red with Giemsa stain. Soluble antigen fluorescent antibody test (SAFA),
l Pathogenicity: Natural infection with T. pallidum occurs Enzyme-linked immunosorbant assay (ELISA), DNA
only in human beings. probes and polymerase chain reaction (PCR).
Q. 15. Actinomycosis. Q. 18. Tuberculin test.

Ans. Ans.
i. Actinomycosis is a chronic granulomatous suppurative and i. The tuberculin test or the mantoux test involves subcu-
fibrosing disease. endogenous, opportunistic infection. taneous injection of 0.1 ml of 5 tuberculin units of
ii. Aetiology: Actinomyces israelii purified protein derivative of siebert stabilized with
iii. Clinical features: Tween 80 or one tuberculin unit of PPDRT into the
l Actinomycosis is classified anatomically into 3 forearm.
forms according to location of lesion as cervicofa- ii. It is positive, if induration is seen after 48 - 72 hrs.
cial (most common), abdominal and pulmonary. iii. The maximum diameter of induration measured by
l Cervicofacial actinomycosis is characterized by palpation is recorded and interpreted as follows:
swelling of the soft tissue with formation of extra- l . 15 mm of ulceration is n strongly positive

oral draining sinuses over skin or mucosal surface. l . 10 mm means n positive

l Pus collected from sinus shows typical “sulphur l 5-9 mm n indeterminate

granules” or tiny yellow grains which are colonies of l , 5 mm n negative.

organism. A strongly positive test may indicate recent infection.


iv. Treatment of this disease is difficult and has not been iv. The tuberculin test has great value in excluding tuber-
uniformly successful. Long-standing cases are treated culosis than in diagnosing it.
Section | III Oral Pathology 495

Q. 19. Lumpy jaw. ii. Aetiology:


l Infection either by staphylococci or streptococci.
Ans.
l Sulfahydryl radical is most essential stimulating

i. Actinomycosis bovis produces the ‘lumpy jaw’ in cattle agent.


but is rarely found to be a pathogen in humans. iii. Clinical features:
ii. The usual pattern of the disease is characterized by for- l Arises more frequently on gingiva, may also occur

mation of abscesses that tend to drain by the formation on lips, tongue and buccal mucosa.
of sinus tracts. l Overzealous proliferation of a vascular type of con-

nective tissue.
Q. 20. Mucous patches.
l Intravenous pyogenic granuloma occurs on neck and

Ans. upper extremities.


l It is deep red or reddish purple, painless and soft in
l Secondary or metastatic stage of syphilis is known as
consistency.
mucous patch.
l Pregnancy tumour is a lesion histologically similar
l Usually commences after 6 weeks after primary lesions.
to pyogenic granuloma, occurs in pregnancy.
l The oral lesions are called ‘mucous patches’ and are
iv. Treatment: Surgical excision.
usually painless, multiple, greyish white plaques overly-
ing an ulcerated surface. Q. 22. Kveim-Siltzbatch test.
l The mucous patches occur more frequently on, tongue,
gingiva, or buccal mucosa or as a split papule on lips Ans.
and are highly infectious.
l It is an intracutaneous test for the diagnosis of sar-
Q. 21. Pyogenic granuloma. coidosis.
l It has been devised utilizing suspension of human
Ans.
known sarcoidal tissue as a test agent.
i. Pyogenic granuloma or granuloma pyogenicum origi- l This test may be an important aid in the early and ac-
nates as a response to non-specific infection. curate diagnosis of the disease.

Topic 6

Viral Infections of the Oral Cavity

LONG ESSAYS
Q. 1. Enumerate viral infections of the oral cavity. De- The viral infections of oral cavity are classified depending
scribe general and oral manifestations of lesions caused on the presence of the major viruses as follows:
by herpes simplex viruses.
Or I. RNA viruses:
A. Orthomyxovirus
Enumerate the various viral infections affecting the oral cav-
l Influenza
ity; write in detail the pathogenesis, clinical features and oral
B. Paramyxovirus
manifestations of herpes simplex virus type 1 and type 2.
l Measles (rubeola) and

Or l Mumps

C. Rhabdovirus
Enumerate the viral lesions of the oral cavity. Describe in
l Rabies
detail the clinical features, histological features and lab
D. Arena virus
investigations of primary herpetic gingivostomatitis.
l Lassa fever

Ans. l Lymphocytic choriomeningitis


496 Quick Review Series for BDS 3rd Year

E. Calicivirus A. PRIMARY HERPETIC STOMATITIS


F. Coronavirus
l Upper respiratory tract infection
Clinical Features
G. Bunyavirus l Herpetic gingivostomatitis is a common oral disease
H. Picornavirus transmitted by droplet spread or contact with the
I. Reovirus lesions.
J. Togavirus l This infection occurs in the persons who are not
K. Retrovirus infected previously with herpes virus or they don’t have
circulatory antibodies against virus.
II. DNA viruses:
l It affects children and young adults.
A. Herpes virus l Disease occurring in children is frequently the primary
l Herpes simplex virus 1 and 2 attack and is characterized by the development of fever,
l Varicella zostervirus irritability, headache, pain upon swallowing and
l Cytomegalovirus regional lyphadenopathy.
l Epstein-Barr virus l Within a few days, mouth becomes painful, and the
l Human herpes virus 6, 7 and 8. gingiva becomes intensely inflamed and appears
B. Poxvirus erythematous and oedematous.
l Smallpox l Lips, tongue, buccal mucosa, palate and tonsils may be
l Molluscum contagiosum involved. Shortly, yellowish fluid-filled vesicles develop
C. Adenovirus in oral cavity.
l Pharyngoconjunctival fever l These vesicles rupture to form painful ulcers covered by
D. Parvovirus gray membrane and surrounded by erythematous halo.
E. lridovirus l Healing occurs in 7 to 14 days and leave no scar.
F. Papovavirus Herpetic whitlows in hands of hospital staff and
l Human warts or papillomas. disseminated infection of newborn are examples of pri-
mary herpetic infections.
HERPES SIMPLEX VIRUS
l Herpes simplex is a DNA virus which causes the Histological Features
disease in the man. l Intraepithelial fluid-filled vesicles
l The tissues preferentially involved by herpes simplex l Ballooning degeneration
virus, are often referred to as herpes virus hominis and l Intranuclear inclusions known as Lipschutz bodies are
are derived from ectoderm principally the skin, mucous present. These are eosinophilic, ovoid homogeneous
membranes, eyes and central nervous system. structures within the nucleus.
l Two types of infections occur with herpes simplex virus: l Perinuclear halo in nucleus produced by displacement
A. Primary infection of chromatin peripherally by Lipschutz bodies.
B. Secondary or “recurrent” infection. l Cytoplasm of infected cells forms giant cells and subja-
l Primary infection occurs in persons who do not have cent connective tissue is usually infiltrated by inflamma-
circulating antibodies whereas secondary or “recurrent” tory cells.
infection occurs in persons who have circulating anti-
bodies.
l Primary herpetic infections may manifest clinically as
Diagnosis
primary gingivostomatitis, primary vulvovaginitis, l It can be diagnosed both clinically and by laboratory
inoculation herpes simplex, varicelli form eruption, procedures.
meningoencephalitis, disseminated herpes simplex. l HSV can be demonstrated in laboratory by isolation of
Subclinical primary infection is common in 99% of virus in tissue culture or by DNA in the scrapings from
cases the primary infection is subclinical with no visible the lesion.
clinical disease. l Most sensitive and accurate method for diagnosis is
l Recurrent herpetic manifestations include fever blister, PCR technique.
genital herpes simplex and dendritic corneal ulcers.
HSV 2 is associated with carcinoma of uterine cervix;
l
Treatment
HSV does not remain latent at site of original infection;
reaches regional ganglia along the nerve path; HSV1 in l Antiviral drugs: If diagnosed early, the antiviral drugs
trigeminal and HSV 2 in lumbosacral ganglions. have significant impact on the course of the disease.
Section | III Oral Pathology 497

l Antibiotics: They help in the prevention of secondary l Epidermolysis bullosa.


infection. l Erythema multiforme.
l NSAIDs and topical anaesthetic gel: May relieve the l Epidermolysis bullosa
discomfort considerably. l Smallpox
l Pemphigus
Food or drug allergies
B. SECONDARY ”RECURRENT” HERPES l

l Drug or chemical burns.


LABIALIS AND STOMATITIS
Q. 2. Classify viruses and viral infections
l After primary infection, virus may travel along the
nerves and remains in dormant phase in trigeminal or Ans.
lumbosacral ganglion.
Classification of viral lesions of oral cavity: The viruses
and viral infections of oral cavity are classified depending
Clinical Features on the presence of the major viruses.
l Precipitating factors: Fever, allergy, anxiety, GI
upset may cause reactivation of virus with formation I. RNA viruses
of greyish vesicular lesion on lips and oral mucosa in A. Orthomyxovirus
cluster form. i. Influenza
l Recurrent intraoral herpetic lesions invariably develop B. Paramyxovirus
on mucosa tightly bound to periosteum (aphthous on i. Measles (rubeola) and
loose mucosa). ii. Mumps
l The most common sites of occurrence for recurrent in- C. Rhabdovirus
traoral herpetic lesions are hard palate, attached gingiva i. Rabies
and alveolar ridge. ii. Haemorrhagic fever
l These vesicles rupture to form ulcers. They may form D. Arenavirus
crust when present over lip. i. Lassa fever
l The lesions gradually heal within 7 to 10 days and leave ii. Lymphocytic choriomeningitis
no scar. E. Calicivirus
l These develop in an interval of months or years usually. F. Coronavirus
i. Upper respiratory tract infection
G. Bunyavirus
Histological Features
H. Picornavirus
l Both primary and secondary lesions show identical i. Poliomyelitis
histological features. ii. Coxsackie diseases
l Ballooning degeneration in cells. iii. Common cold
l Lipschultz bodies in some cells within the nuclei iv. Foot and mouth disease
causing perinuclear halo. v. Encephalomyocarditis.
l Secondary lesion contains additional multinuclear giant I. Reovirus
cells. J. Togavirus
i. Rubella
ii. Yellow fever
Treatment
iii. St. Louis encephalitis.
Until recently, little could be provided in the way of actual K. Retrovirus (RNA tumour virus)
therapy except for symptomatic relief.
l Antiviral drugs: Acyclovir vidarabine, idoxuridine II. DNA viruses
l Suppurative treatment: A. Herpes virus
l For fever—asprin or acetaminophen a. Herpes simplex virus 1
l For painful ulcers—dicyclonin hydrochloride is used i. Gingivostomatitis
topically. ii. Keratoconjunctivitis
iii. Genital and skin lesions
Differential Diagnosis b. Herpes simplex virus 2
i. Genital and skin lesions
l Herpes zoster. ii. Keratoconjunctivitis
l Impetigo iii. Neonatal infections and meningitis.
498 Quick Review Series for BDS 3rd Year

c. Varicella zoster virus sensory nerves, usually unilaterally. Often the trunk is
i. Varicella (chikenpox) affected.
d. Cytomegalovirus l Within few days, the patient has a linear papular or
i. Cytomegalic inclusion disease vesicular eruption of the skin or mucosa supplied by
e. Epstein-Barr virus the affected nerves. It is typically unilateral and derma-
i. Infectious mononucleosis tomic in distribution.
ii. Hepatitis l The dorsal root ganglion is also inflamed with vesicular
iii. Encephalitis eruptions unilaterally along the sensory nerve path, over
f. Human herpes virus 6 the skin or mucosa.
i. Otitis media l Triggering factors for infections are: Malignancy,
ii. Encephalitis trauma, radiations, etc.
g. Human herpes virus 7
i. Roseola infantum
Oral Manifestations
h. Human herpes virus 8
i. Infectious mononucleosis l Lesions of oral mucosa are fairly common and ex-
ii. Febrile exanthema tremely painful vesicles may be found on the buccal
i. Simian herpes virus mucosa, tongue, uvula, pharynx and larynx. They gen-
i. Mucocutaneous lesions erally rupture to leave the areas of erosion.
ii. Encephalitis l One of the characteristic clinical features of the disease
B. Poxvirus involving the face or oral cavity is the unilaterality of
i. Smallpox the lesions. Typically when large the lesions will extend
ii. Molluscum contagiosum up to midline and stop abruptly.
C. Adenovirus
i. Pharyngoconjunctival fever
ii. Epidemic keratoconjunctivitis
James Ramsay Hunt Syndrome
D. Parvovirus l A special form of zoster infection of the geniculate
E. lridovirus ganglion with the involvement of external ear and
F. Papovavirus oral mucosa, has been termed as James Ramsay Hunt
i. Human warts or papillomas. syndrome.
ii. Tumorigenic viruses in animals. l The clinical manifestations include facial paralysis,
pain in external auditory meatus and pinna of the
Q. 3. Herpes zoster. ear. In addition, vesicles occur in the oral cavity and
Ans. oropharynx with hoarseness, tinnitus, vertigo and other
disturbances.
l Herpes zoster is also known as shingles or zona.
l It is an acute infectious viral disease of an extremely
painful and incapacitating nature. Diagnosis
l The viral infection affects sensory nerves with trigemi- l Characteristic distribution of lesions
nal nerve most frequently involved and other sensory l Cytological smears
nerves involved are C3, T5, Ll and L2. l Fluorescent antibody staining techniques
l Viral culture and serologic diagnosis.
Aetiology
l Virus causing infection is “varicella zoster” virus. Treatment
l Antiviral drugs such as acyclovir.
For the treatment of post-herpetic neuralgia: Intrale-
Clinical Features l

sional corticosteroids.
l The disease is most common in adult life and affects l Topical application of capsaicin.
both the sexes equally. l The newer antiviral drugs are under intensive clinical
l Initially adult patient exhibits fever, a general malaise, testing for potential effectiveness in treatment of herpes
pain and tenderness along the course of involved zoster.
Section | III Oral Pathology 499

SHORT ESSAYS
Q. 1. Mumps. l The incubation period is probably 2-10 days.
l It begins with sore throat, cough, rhinorrhoea, low grade
Ans.
fever, headache, sometimes vomiting, prostration and
l Mumps is an acute contagious viral disease character- abdominal pain.
ized by unilateral or bilateral swelling of salivary glands l Small vesicles which rupture to form crops of ulcers are
mostly parotid. more common on pharynx and posterior oral mucosa and
are less frequent on tongue, buccal mucosa and palate.
The ulcers do not tend to be extremely painful although
Aetiology l

the patients will have dysphagia.


l Virus causing mumps is Paramyxo virus. l A permanent immunity develops to the infecting strains
l Incubation period varies from 14-18 days. rapidly and antibodies are found.

Clinical Features Laboratory Findings


l The disease is usually preceded by the onset of head- The coxsackie virus can be isolated in suckling mice or
ache, chills, fever and pain during mastication. hamsters by inoculation of scrapings from throat lesions or
l These symptoms are followed by rubbery or elastic stool specimens.
swelling of salivary glands, frequently elevating the ear,
which lasts for about one week.
l The submandibular gland also may be involved. The Treatment
involvement of sublingual glands is rare. No treatment is necessary as the disease appears to be self-
l The papilla of the opening of the parotid duct on the limiting.
buccal mucosa is often puffy and reddened.
Q. 3. Herpes zoster or shingles.
l Complications are:
l Epididymo-orchitis Ans.
l Pancreatitis
l This is due to reactivation of the varicella zoster virus
l Mastoiditis
which remained dormant after chickenpox attack
l Oophoritis
earlier in life.
l This is an acute condition in which there occurs group
Treatment of vesicles on inflamed base, along with neuralgic pains
and itching sensations of unilateral distribution.
It is conservative consisting of:
l Maintaining hydration and alimentation
l Hot and cold compressions. Clinical Features
l Analgesics and antipyretics.
i. Facial pain precedes the eruption of versicles.
Prevention ii. Vesicles are more on the upper lip when 2nd division
of 5th cranial nerve is involved and over the mental
Prophylaxis is done by active vaccines.
foramen on the skin when the 3rd division of 5th cranial
Q. 2. Herpangina. nerve is involved.
iii. Eruptions on face and in the mouth are strictly unilat-
Ans.
eral and confined to the area supplied by the concerned
Herpangina is a specific viral infection caused by coxsackie nerve. Bullae appear only on the area where a nerve
group A virus. comes to the surface.
iv. If geniculate ganglion of 7th cranial nerve is involved,
patient will have atypical neuralgia.
Clinical Features v. A special form of zoster infection of the geniculate
l In herpangina or apthous pharyngitis, the clinical ganglion with the involvement of external ear and oral
features are mild and of short duration (1 week). mucosa, has been termed as “James Ramsay Hunt
l It is commonly seen in young children. syndrome”.
500 Quick Review Series for BDS 3rd Year
t.me/desidentist
Treatment Diagnosis
l Antiviral drugs, i.e. aciclovir orally, idoxuridine skin l It can be diagnosed both clinically and by laboratory
ointment. procedures.
l Isolation of HSV in tissue culture or by DNA in the
Q. 4. Primary herpetic stomatitis.
scrapings from the lesion.
Ans. l Most sensitive and accurate method for diagnosis is
PCR technique.
l Herpes simplex is a DNA virus which causes the dis-
ease in the man.
l Two types of infections occur with herpes simplex virus: Treatment
A. Primary infection l Antiviral drugs: If diagnosed early, the antiviral drugs
B. Secondary or “recurrent” infection. have significant impact on the course of the disease.
Primary infection occurs in persons who do not have circu- l Antibiotics help in the prevention of secondary infection.
lating antibodies whereas secondary or “recurrent” infec- l NSAIDs and topical anaesthetic gel may relieve the
tion occurs in persons who have circulating antibodies. discomfort considerably.
Q. 5. AIDS.
A. PRIMARY HERPETIC STOMATITIS Ans.
Clinical Features i. AIDS (acquired immuno deficiency syndrome) is a vi-
l Herpetic gingivostomatitis is a common oral disease ral disease caused by human immunodeficiency
transmitted by droplet spread or contact with the lesions. virus (HIV).
l It affects children and young adults. ii. Modes of transmission are: Sexual contact with
l Disease occurring in children is frequently the primary affected person, infected needles, infected blood or
attack and is characterized by the development of fever, blood product transfusion and from mother to child
irritability, headache, pain upon swallowing and re- (prenatal).
gional lyphadenopathy. iii. The virus is present in body fluids like saliva, semen,
l Within a few days, mouth becomes painful, and the milk, etc. and may enter inside the body through any of
gingiva becomes intensely inflamed and appears ery- the route and binds with CD4 protein receptors in host
thematous and oedematous. cells. This protein is present mostly in T helper cell.
l Lips, tongue, buccal mucosa, palate and tonsils may be iv. Clinical features:
involved. Shortly, yellowish fluid-filled vesicles de- l Initially, there is a latent period of about (8-12 weeks).

velop in oral cavity. Then antibodies appear in blood {seroconversion) and


l These vesicles rupture to form painful ulcers covered by general sign and symptoms appear. Then asymptom-
grey membrane and surrounded by erythematous halo. atic phase appears for many years.
l Healing occurs in 7 to 14 days and leave no scar. l Finally, the signs and symptoms of AIDS are ap-

peared. These may include:


a. Infections, eg. CMV infection, bacterial septicae-
Histological Features mia, toxoplasmosis, histoplasmosis, etc.
l Intraepithelial fluid-filled vesicles b. Secondary neoplasms, e.g. Kaposi’s sarcoma,
l Ballooning degeneration non-Hodgkin’s lymphoma, etc.
l Intranuclear inclusions known as Lipschutz bodies and- v. Oral diseases in AIDS include:
perinuclear halo are present. l Angular stomatitis, aphthous ulcers, hairy leukoplakia,

l Cytoplasm of infected cells forms giant cells and subjacent candidiasis, CMV/herpes simplex infection.
connective tissue is usually infiltrated by inflammatory cells. l Warts, Kaposi’s sarcoma and gingivitis, etc.

SHORT NOTES
Q. 1. Lipshutz bodies. ii. The herpetic vesicle is an intraepithelial blister filled
with fluid which characteristically contains intranuclear
Ans. inclusions known as Lipshutz bodies.
iii. These are eosinophilic, ovoid homogenous structures
i. Lipshutz bodies are a characteristic histological feature within the nucleus which tend to displace the nucleolus
seen in primary herpetic stomatitis. and nuclear cromatin peripherally.
Section | III Oral Pathology 501

iv. The displacement of cromatinoffen produces a perinu- v. A special form of zoster infection of the geniculate gan-
clear halo. glion with the involvement of external ear and oral mucosa,
has been termed as James Ramsay Hunt syndrome.
Q. 2. Mention the oral lesions in HIV infection (AIDS).
vi. The newer antiviral drugs are under intensive clinical
Ans. testing for potential effectiveness in treatment of herpes
zoster.
i. AIDS is predominantly lethal type of viral infection
caused by the HIV and is characterized by the depletion Q. 5. Measels.
of T4 lymphocytes in the body.
Ans.
ii. Lesions strongly associated with HIV infection are:
l Candidiasis i. Measels is an acute viral infection caused by rubeola
l Hairy leukoplakia virus and spreads due to direct contact or droplet infec-
l Kaposi’s sarcoma tion.
l Non-Hodgkin’s lymphoma ii. Ocular involvement causes photophobia, lacrimation
l Periodontal diseases like necrotising ulcerative gingi- and conjunctivitis.
vitis and necrotising ulcerative periodontitis. iii. The red, irregular, small macules with bluish white
centre in oral mucous membrane known as the Koplik’s
Q. 3. Clinical features of herpangia.
spots appear.
Ans. iv. Lastly, skin eruptions appear starting from forehead and
spread downward involving rest of body.
Herpangina is a specific viral infection caused by coxsackie v. These eruptions fade in 6 days with desquamation.
group A virus. vi. Treatment consists of passive immunization, isolation
of patient and active immunization.
Clinical Features Q. 6. Tzanck cells.
l Sudden onset
Ans.
l Constitutional symptoms are milder
l Short duration of course—1 week l “Tzank cells” are diagnostic feature of herpes zoster
l Lesions are confined to oral cavity—vesicular rash on and pemphigus.
the fauces, soft palate. l In cases of herpes zoster, multinucleated giant cells
l Lesions are small, vesicular and uniform and are more known as “Tzank cells” are characteristic feature of
common on pharynx posterior oral mucosa and are less cytologic smear examination.
frequent on tongue, buccal mucosa and palate. l In case of pemphigus as a result of acantholysis, clumps
l The ulcers do not tend to be extremely painful although of epithelial cells are found lying free within the vesicular
the patients will have dysphagia. space, these cells are called “Tzanck cells”.
l “Tzank cells” have large nuclei and condensation of
chromatin along cell wall.
Treatment
Q. 7. Reiter’s syndrome.
No treatment is necessary as the disease appears to be self-
limiting. Ans.
Q. 4. Herpes zoster. i. Reiter’s syndrome is caused due to PPLO (pleuro-
pneumonia-like organism)
Ans. ii. It arises as a complication of non-specific urethritis.
i. Herpes zoster is also known as shingles or zona. iii. It consists of tetrad:
l Urethritis
ii. It is an acute infectious viral disease of an extremely
l Polyarthritis
painful and incapacitating nature caused by “varicella
l Conjunctivitis and
zoster” virus.
l Mucocutaneous lesions
iii. The patient has a linear papular or vesicular eruptions
of the skin or mucosa supplied by the affected nerves. iv. Maculopapular lesions are characteristics of skin and
It is typically unilateral and dermatomic in distribution. mucous membranes.
iv. One of the characteristic clinical features of the disease Q. 8. Infectious mononucleosis/glandular disease/
involving the face or oral cavity is the unilaterality of kissing disease.
the lesions. Typically, when large the lesions will ex-
tend up to midline and stop abruptly. Ans.
502 Quick Review Series for BDS 3rd Year

i. Infectious mononucleosis or glandular disease or h. Human herpes virus 8


kissing disease is caused due to Epstein-Barr virus i. Infectious mononucleosis
(EBV). ii. Febrile exanthema
ii. It is thought to be arising through oral exchange of i. Simian herpes virus
saliva (kissing) with affected person. i. Mucocutaneous lesions
iii. Splenomegaly, hepatitis and cervical nodes enlarge- ii. Encephalitis
ment are seen.
Q. 11. James Ramsay Hunt syndrome.
iv. Oral manifestations include palatal petechiae, acute
gingivitis and appearance of white or grey membrane in Ans.
various areas.
i. A special form of zoster infection of the geniculate
v. Diagnosed by Monospot test as screening test.
ganglion with the involvement of external ear and oral
vi. Non-specific treatment is done.
mucosa, has been termed as James Ramsay Hunt
Q. 9. Herpes labialis. syndrome.
ii. The clinical manifestations include facial paralysis,
Ans.
pain in external auditory meatus and pinna of the ear.
i. Recurrentor secondary or herpetic labialis is usually iii. In addition, vesicles occur in the oral cavity and oro-
seen in adult patients. pharynx with hoarseness, tinnitus, vertigo and other
ii. First there is a discomfort such as burning sensation, disturbances.
pruritis or pain at the site. Then there will be eruption
Q. 12. Histopathological features of primary herpes
of group of vesicles surrounded by area of redness.
simplex infection.
iii. Vesicles burst after few hours and coalace. On the lips,
these ruptured vesicles become covered by a brownish Ans.
crust.
Histological features of primary herpes simplex infection
iv. Secondary bacterial infection sets in.
are as follows:
v. The characteristic site is mucocutaneous junction of the
i. Intraepithelial fluid-filled vesicles
lips. But can occur in other intraoral areas too.
ii. Ballooning degeneration
vi. Treatment: By antiviral drugs, e.g. aciclovir
iii. Intranuclear inclusions known as Lipschutz bodies
Q. 10. Herpes virus infections. and perinuclear halo are present.
iv. Cytoplasm of infected cells forms giant cells and
Ans.
subjacent connective tissue is usually infiltrated by
Various infections caused by group of herpes viruses are as inflammatory cells.
follows:
Q. 13. Chickenpox.
a. Herpes simplex virus 1
i. Gingivostomatitis Ans.
ii. Keratoconjunctivitis
i. Chickenpox is also known as varicella.
iii. Genital and skin lesions
ii. It is caused by VZV (varicella zoster virus).
b. Herpes simplex virus 2
iii. Clinical features:
i. Genital and skin lesions
l Inflammation of dorsal nerve root ganglia or extra-
ii. Keratoconjunctivitis
medullary cranial nerve ganglia, i.e. sensory nerves.
iii. Neonatal infections and meningitis.
l It is always unilateral.
c. Varicella zoster virus
l Small blister-like lesions occasionally involve oral
i. Varicella (chikenpox)
mucosa.
d. Cytomegalovirus
iv. This disease runs its clinical course in a week to 10
i. Cytomegalic inclusion disease
days, rarely leaving any after effects.
e. Epstein-Barr virus
v. Secondary infection of vesicles results in formation of
i. Infectious mononucleosis
pustules, which may leave small pitting scars upon
ii. Hepatitis
healing.
iii. Encephalitis
f. Human herpes virus 6 Q. 14. Rubella.
i. Otitis media
Ans.
ii. Encephalitis
g. Human herpes virus 7 i. Rubella is also known as German measles.
i. Roseola infantum ii. It is cause by myxovirus.
Section | III Oral Pathology 503

iii. It affects oral structures, if encountered in first trimester ii. They are the intraoral lesions reported to occur in about
of pregnancy 97% of all patients with measles. The oral lesions are
iv. Clinical features: No Koplik’s spots; if occurs during prodromal, frequently occurring 2-3 days before the
first trimester of pregnancy, it causes congenital defects. cutaneous rash and are pathognomonic of this disease.
iii. These characteristic spots occurring on buccal mucosa
Q. 15. Koplik’s spots. are small, irregularly shaped flecks which appear as
Ans. bluish white specks surrounded by a bright-red margin.
iv. Skin rashes—red maculopapular rashes occur on
i. Koplik’s spots are characteristic of measels or the back of the ear, forehead first, and then whole skin
rubeola. followed by Koplik’s spots.

Topic 7

Mycotic Infections of the Oral Cavity

LONG ESSAYS
Q. 1. Enumerate fungal infections of the oral cavity. Oral Candidiasis
Write in detail about the classification, clinical features
and lab investigations of oral candidiasis.

Ans.
Acute Chronic
The various fungal infections affecting oral cavity are as Acute pseudomembranous oral candidiasis
(thrush)
follows:
Acute atrophic (erythematous) oral
i. Candidiasis candidiasis
ii. Histoplasmosis
iii. Blastomycosis
iv. Paracoccidioidomycosis
v. Coccidioidomycosis i. Chronic hyperplastic ii. Chronic mucocuta- iii. Chronic atrophic
vi. Sporotrichosis oral candidiasis neous candidiasis oral candidiasis
vii. Aspergillosis
viii. Cryptococcosis a. Chronic familial mucocutaneous candidiasis
ix. Toxoplasmosis b. Chronic localized mucocutaneous candidiasis
l The “candidiasis” refers to a multiplicity of c. Chronic diffuse mucocutaneous candidiasis
diseases caused by a yeast-like fungus called as d. Candidiasis endocrinopathy syndrome
Candida albicans. It is the most common oral fun-
gal infection in humans.
l The classification of oral candidiasis is as Clinical Features
follows:
l Predisposing factors of candidiasis are: A. Acute candidiasis
i. Infancy i. Acute pseudomembranous type:
ii. Debilitating illness l Common sites of occurrence: Buccal mucosa, tongue
iii. Metabolic diseases: and palate.
Diabetes l Aetiology: Prolonged antibiotic therapy and immuno-
Hypothyroid suppression.
Adrenocortical syndrome l Appearance is milk curd-like creamy white plaques
iv. Antibiotic therapy (broad-spectrum mainly.) which are removable.
504 Quick Review Series for BDS 3rd Year

l Adherent and bleed on forceful removal. d. Candidiasis endocrinopathy syndrome:


l Burning sensation and foul taste. l Appears as white plaques most of which are not

removable.
ii. Acute atrophic type: l Common sites of occurrence: Tongue, buccal mu-

l Common sites of occurrence: Posterior hard palate, cosa, and palate.


buccal mucosa and dorsal tongue. l It is rare and endocrine disorder develops after

l Aetiology: Prolonged antibiotic therapy, xerostomia, candidiasis.


immunosuppression and idiopathic.
l Appears as red erythematous lesion with papillary Histological Features
atrophy or like a wiped off lesion of acute pseudomem-
branous type. l Histopatholic pattern of oral candidiasis may vary
l This is the only painful variety. slightly depending on the type of clinical form.
l Invasion of mycelia is seen in epithelium and epithe-
lium is undergoing necrosis.
B. Chronic candidiasis
i. Chronic hypertrophic type:
l Common sites of occurrence: Anterior buccal mucosa, Lab Investigations
lip, tongue or cheeks. l Diagnosis is established by clinical signs in conjunc-
l Aetiology: Idiopathic, immunosuppression. tion with exfoliative cytologic examination of smears
l Asymptomatic firm persistant white plaques that are not for presence of many budding cells (spores) and
removable. filaments.
l It is leukoplakia like and persists for years, may trans- l White patches of candidiasis are easily wiped out with
form into epidermoid carcinoma. wet gauge.
l Biopsy for mycological examination.
ii. Chronic atrophic type (denture stomatitis or l Positive culture on corn meal sugar.
denture sore mouth):
Common sites of occurrence: It is found confined to
l
Treatment
denture bearing areas often with angular cheilitis.
l Appears as asymptomatic red lesion. i. Several antifungal agents have been developed for
managing oral candidiasis, they are as follows:
iii. Chronic mucocutaneous type A. Polyene agents:
l Nystatin
l Common sites of occurrence: Tongue, buccal mucosa,
l Amphotericin B
palate. It also occurs on skin, scalp, nails and mucous
membranes. B. Imidazole agents:
l Clotrimazole
l Asymptomatic firm persistant white plaques some of
l Ketoconazole
which may be removable, red areas.
l Aetiology: Rare, inherited or sporadic, idiopathic C. Triazoles:
l Fluconazole
immune dysfunction.
l Itraconazole
Various types of chronic mucocutaneous candidiasis are as D. Other antifungal agents:
follows: l Iodoquinol
a. Chronic familial ii. The development of new specific antifungal agents
l It occurs in infants. such as nystatin has been beneficial in the treatment of
l It is autosomal recessive candidiasis.
b. Chronic localized iii. Triazoles are the newest group of antifungal drugs. In
l It appears as granulomatous, horny masses on face immunosuppressed patients systemic administration of
and scalp. amphotericin B and fluconazole is given.
l Wide-spread involvement of skin and nails also iv. Although iodoquinol is not strictly an antifungal drug,
present. it has antifungal and antibacterial properties. When
c. Chronic diffused compounded in a cream base with corticosteroid, it is
l Seen in elderly persons above 55 years of age. very effective as topical therapy for angular cheilitis.
l Limbs, face, scalp, nails and mouth are involved. v. Improvement of oral hygiene is essential.
Section | III Oral Pathology 505

SHORT ESSAYS
Q. 1. Thrush. l clinical signs in conjunction with exfoliative cytologic
examination of smears for presence of many budding
Ans. cells (spores) and filaments.
Acute pseudomembranous oral candidiasis is also known as l Biopsy
thrush. It is caused by fungus “Candida albicans”. l Positive culture on corn meal sugar.
i. Common sites of occurrence: Buccal mucosa, tongue
and palate. Complication
ii. Aetiology: Prolonged antibiotic therapy and immuno-
suppression. Noma in malnourished children
iii. Predisposing factors causing candidiasis are:
l Infancy
Treatment
l Debilitating illness

l Metabolic diseases: By antifungal antibiotics—nystatin or amphotericin sus-


Diabetes pensions or lozenges.
Hypothyroid adrenocortical syndrome For AIDS patient, oral fluconazole is used.
l Antibiotic therapy (broad-spectrum mainly)
Q. 2. North American blastomycosis.
Ans.
Clinical Features
i. North American blastomycosis is also known as
l Pseudomembranous oral candidiasis is characterized by Gilchrist’s disease. It is a granulomatous infection
presence of adherent white plaques that resemble cot- caused by blastomyces dermatitidis.
tage cheese or curdled milk. ii. Clinical features:
l This white patches can be removed by scraping them l It typically occurs in middle age and is far more
with a tongue blade or rubbing them with a dry guaze common in men than in the women.
sponge. l Cutaneous lesions begin as small red papular lesions,
l Infants get this from mother’s affected vulva, from which ulcerate and discharge pus from sinuses. Thus
bottle feeding or overcrowded nurseries. the lesion resembles actinomycosis.
l In adults, one finds inflammation, erythema and eroded l Crateriform lesions are typical and they often exhibit
areas. Deeper invasion by the organism leaves an ulcer- indurated and elevated borders.
ative lesion upon removal of patch. l Systemic signs are similar to pulmonary tuberculo-
l Adults get this when they are on broadspectrum antibi- sis such as fever, weight loss, and productive cough.
otic or corticosteroid therapy for prolonged period. It l Oral lesions are also similar to actinomycosis with
also occurs in AIDS patients. ulcer formation. Tiny ulcers may be the chief feature.
l It is more common in women. l Resembles SCC, giant cells contain microorgan-
l Burning sensation and foul taste. isms; mostly in dogs, outdoor workers.
l White patches of candidiasis are easily wiped out with iii. Histopathology: Reveals microabscess and pseudoepi-
wet gauge which leaves erythematous area or atrophic theliomatous hyperplasia.
area. iv. Diagnosis: Clinically as the lesions resemble squamous
cell carcinoma, biopsy and histopathological examina-
tion are required.
Histological Features
v. Treatment:
l Histopathologic pattern of oral candidiasis may vary l Most patients with blastomycosis requires no treatment.
slightly depending on the type of clinical form. l Even in the case of symptomatic acute blastomyco-
l Invasion of mycelia is seen in epithelium and epithe- sis, administration of systemic amphotericin B is
lium is undergoing necrosis. indicated only if the patient is seriously ill (AIDS
and other immunosuppression disorder) or not im-
proving clinically and is ill for more than 2-3 weeks.
Diagnosis l Patients with chronic blastomycosis or extrapulmo-

l History of development nary lesions need treatment. Itraconazole is gener-


l History of medication ally recommended.
506 Quick Review Series for BDS 3rd Year

Q. 3. Write about the laboratory diagnosis for investiga- shows filamentous outgrowths (Reynolds-
tion of candidiasis. Braude phenomenon).
b. Chlamydospores: These develop in a nutrition-
Ans.
ally poor medium such as cornmeal agar at
Methods used in laboratory diagnosis of candida are as 28°C.
follows: iii. Biochemical reactions: Candida albicans can be iden-
i. Direct examination: Scrapings from the lesions of skin, tified by the assimilation and fermentation of sugar.
nails or mucosa are examined in a wet film in KOH or iv. Serology: Candida albicans can also be identified by
Gram stained smear. The Candida albicans appears as the precipitation test with a carbohydrate extract of
budding yeast cells. group A antigens.
ii. Culture: v. Antigen detection: It is done by ELISA and RIA
l Specimens are inoculated on the Sabouraud’s dex- which detects cell wall manner or cytoplasmic
trose chloramphenicol agar medium at 25-37°C for constituents.
24 hours. Candida produces creamy white, smooth vi. Skin test: Delayed hypersensitivity to candida extracts
colonies with a yeasty odour. is a useful indicator of functional integrity of CMI.
l The Candida albicans is identified by: vii. Animal inoculation: Candida albicans kills the animal
a. Germtubes: When candida is grown in human (rabbit, guinea pig and mice) in 4-5 days with typical
serum at 37°C for 3 hours, a wet KOH film renal abscess on intravenous inoculation.

SHORT NOTES
Q. 1. Thrush or acute pseudomembranous candidiasis ii. Clinical features:
or oral moniliasis. l Lymphadenopathy

l Hepatosplenomegaly
Ans.
l Fever
i. Acute pseudomembranous oral candidiasis is also known l Productive cough.
as thrush. It is caused by fungus “Candida albicans”. iii. Oral manifestations:
ii. Common sites of occurrence: Buccal mucosa, tongue l Oral lesions are nodular and ulcerative.
and palate. l Ulcers are indurated with covering of grayish
iii. Aetiology: Prolonged antibiotic therapy and immuno- membrane.
suppression. iv. Treatment: Amphotericin B.
iv. Clinical features:
Q. 3. Rhinosporidiosis
l It is more common in women.

l Characterised by the presence of scrapable adherent Ans.


white plaques that resemble cottage cheese or curdled i. Rhinosporidiosis is a chronic granulomatous disease
milk. caused by Rhinosporidium seeberi.
l Burning sensation and foul taste.
ii. It affects nose and cheeks, oropharynx, skin, larynx,
v. Histological features: eyes and genital mucosa.
l Invasion of mycelia is seen in epithelium and epi-
iii. Skin lesions resemble warts while oral lesions are soft, red-
thelium is undergoing necrosis. dish tumour-like growths with discharge and bleed easily.
vi. Diagnosis is established by: iv. Smears of lesions show spores.
l History
v. Surgical removal is the treatment of choice.
l Clinical signs

l Exfoliative cytologic examination. Q. 4. Mention predisposing factors of candidiasis.


l Biopsy Ans.
l Positive culture on corn meal sugar.

vii. Treatment: By antifungal antibiotics—nystatin or i. Candidiasis is caused by fungus “Candida albicans”.


amphotericin suspensions or lozenges. ii. Predisposing factors of candidiasis are:
l Infancy
For AIDS patients, oral fluconazole is used. l Debilitating illness

Q. 2. Histoplasmosis/Darling’s disease. l Metabolic diseases like:

Diabetes
Ans.
Hypothyroid
i. Histoplasmosis is also known as Darling’s disease and Adrenocortical syndrome
is caused by Histoplasma capsulatum. l Antibiotic therapy (broad-spectrum mainly).
Section | III Oral Pathology 507

Q. 5. Coccidioidomycosis/valley fever. Q. 8. Opportunistic fungal infection.


Ans. Or
i. Coccidioidomycosis is also known as valley fever or Oppurtunistic mycosis.
san Joaquin valley fever.
Ans.
ii. It is caused by Coccidioides immitis
iii. There are two basic forms of the disease: i. Opportunistic infections are caused by some sapro-
a. Non-disseminated and phytic fungi which are ubiquitous in environment,
b. Progressive disseminated. e.g. Aspergillus, Mucor, Penicillium and Rhizopus
iv. The lesions of oral mucosa and skin are proliferative species.
granulomatous and non-specific ulcerative lesions. ii. They produce serious and fatal infections in patients
v. Lytic lesions of jaw are also common. with AIDS or other debilitating diseases like cancer,
vi. Histologically, organisms are found in cytoplasm of diabetes or in those persons where the physiological
giant cells. state has been upset by immunosuppressive drugs,
steroids, X-ray or broad-spectrum antibiotics.
Q. 6. South American blastomycosis.
iii. Aspergillosis and mucormycosis are important oppor-
Ans. tunistic systemic infections.
i. South American blastomycosis is also known as Q. 9. Identification of Candida organisms.
Lutz’s disease or paracoccidioidomycosis caused by
Ans.
B. brasiliensis
ii. The organism may enter the body through periodontal i. Two forms of Candida albicans exist and both are
tissues and subsequently reaches lymph nodes. gram-positive.
iii. It can cause widespread oral ulcers. a. Spherical or ovoid budding cells of 2-3 3 5-6 mm
iv. Has been isolated from periodontal ligament and peri- size. These are yeast or Y forms.
apical granuloma. b. Elongated filamentous cells joined end-to-end re-
v. Histopathology shows “Pilots wheel” or “Micky sembling hyphae and producing spores.
mouse” cells. ii. Candidiasis is an opportunistic infection, the common-
est predisposing factor being diabetes.
Q. 7. Classification of oral candidiasis.
iii. It is identified by following laboratory diagnosis:
Ans. a. Direct microscopic examination:
l KOH preparation shows yeast all with budding
l The classification of oral candidiasis is as follows:
and pseudohyphae.
Oral Candidiasis l Gram stain smear shows gram-positive yeast

cells.
b. Culture and identifications: On Sabouraud’s dex-
trose with chloramphenicol, after 1 to 7 days
Acute Chronic incubation at 37°C shows creamy white smooth
Acute pseudomembranous oral candidiasis colonies.
(thrush) c. Other laboratory tests are:
Acute atrophic (erythematous) oral l Precipitation test
candidiasis
l Agglutination test.

l Indirect fluorescent test.

i. Chronic hyperplastic ii. Chronic mucocuta- iii. Chronic atrophic


oral candidiasis neous candidiasis oral candidiasis

a. Chronic familial mucocutaneous candidiasis


b. Chronic localized mucocutaneous candidiasis
c. Chronic diffuse mucocutaneous candidiasis
d. Candidiasis endocrinopathy syndrome
508 Quick Review Series for BDS 3rd Year

Topic 8

Diseases of the Periodontium

LONG ESSAYS
Q. 1. Classify periodontal diseases. Describe in detail l Mucogingival deformities and conditions around
acute necrotizing ulcerative gingivitis. teeth.
l Mucogingival deformities and conditionson edentulous
Or
ridges.
Briefly describe aetiology, histopathology and clinical
features of acute necrotizing gingivitis. Occlusal trauma

Ans.
ACUTE NECROTIZING ULCERATIVE
GINGIVITIS (ANUG)
CLASSIFICATION OF PERIODONTAL l It is an inflammatory and destructive endogenous oral
DISEASES infection which is characterized by the necrosis of gin-
Gingival diseases: gival tissue.
l It is also known as trench mouth, Vincent’s infection,
l Plaque-induced gingival diseases
acute ulceromembranous gingivitis and acute ulcerative
l Non-plaque-induced gingival diseases
gingivitis.
Chronic peridontitis
l Localised Aetiology
l Generalised
i. Role of bacteria
Aggressive periodontitis It is caused mainly by specific bacteria: Fusiform bacillus
l Localised and spirochaetes.
l Generalised The constant flora is composed of Prevotella intermedia, in
addition to Fusobacterium, Treponema, and Selenomonas
Periodontitis as a manifestation of systemic diseases:
species.
Necrotising periodontal diseases
l Necrotising ulcerative gingivitis (NUG) ii. Local predisposing factors
l Necrotising ulcerative periodontitis (NUP) a. Poor oral hygiene, pre-existing marginal gingivitis and
faulty dental restoration.
Abscesses of periodontium
b. Area of gingiva traumatized by opposing in maloc-
l Gingival abscess cluded teeth such as, the palatal surface behind maxil-
l Periodontal abscess lary incisors and labial gingival surface of mandibular
l Pericoronal abscess incisors
c. Smoking – due to direct toxic effect of nicotine
Periodontitis associated with endodontic lesions d. Emotional stress.
l Endodontic – periodontal lesion
l Periodontic – endodontic lesion iii. Systemic predisposing factors
l Combined lesion a. Nutritional deficiency like vitamin A, C, B2
b. Marked malnutrition
Developmental or acquired deformities and condi- c. Chronic diseases – syphilis, cancer
tions d. Gastrointestinal diseases – ulcerative colitis
l Localised tooth-related factors that predispose to e. Blood dyscrasias – leukaemia, aplastic anaemia, and
plaque-induced gingival diseases or periodontitis. AIDS.
Section | III Oral Pathology 509

Clinical Features l Numerous plasma cells may appear in periphery of in-


filtrate.
l It is identified as an acute disease characterized
by sudden onset, sometimes followed by an episode
of debilitating diseases or acute respiratory tract Treatment
infections.
l The conservative treatment is superficial cleaning of
l It is seen commonly in age group 16 to 30 years.
oral cavity by chlorhexidine, diluted hydrogen peroxide
Oral signs and symptoms or warm salt water. This is followed by scaling and pol-
ishing under topical anaesthesia.
Signs:
l Use of antibiotics is coupled with local treatment in
l Charachterised by punched out, crater-like depressions patients with toxic systemic complications.
at the crest of the interdental papillae, subsequently l Nutritional supplements like vitamin B and vitamin C.
involving marginal gingiva and rarely attached
gingiva. Q. 2. Enumerate causes of gingival enlargement. Discuss
l Craters are covered by greyish pseudomembranous pathogenesis, clinical features and histopathology of
slough with a marked demarcation of linear erythema leukaemic enlargement.
from the normal mucosa. Ans.
l Spontaneous bleeding from gingival tissue.
l Fetid odour and increased salivation. l Gingival enlargements are otherwise known as hyper-
trophic gingivitis or gingival hyperplasia.
Symptoms: l Various causes of gingival enlargements are as
l Extremely tender with radiating pain on eating hot and follows:
spicy foods. i. Influx of normal cells into bloodstream, e.g. gingivitis.
l Metallic foul taste ii. Influx of abnormal cells into bloodstream, e.g.
l Pasty saliva. leukaemia.
iii. Influx of fibrous tissue, e.g. fibromatosis, epanutin
Extraoral and systemic signs and symptoms: related.
Mild to moderate stages: l Classification of gingival enlargements based on aetiol-
l Local lymphadenopathy ogy and pathologic changes are as follows:
l Slight elevation of temperature i. Inflammatory enlargement
a. Chronic
Severe cases: b. Acute
l High fever with increased pulse rate ii. Drug-induced enlargement
l Loss of appetite and general lassitude a. Dilantin
b. Barbiturates
Systemic reactions: iii. Associated with systemic diseases
l They are severe in children. a. Conditioned enlargement
l Rarely noma, gangrenous stomatitis, fusospirochaetal l Pregnancy

meningitis, peritonitis, toxaemia and fatal brain abscess l Puberty

may occur. l Vitamin C deficiency

l Plasma cell gingivitis

l Pyogenic granuloma
Histopathology b. Enlargements due to systemic diseases like
l It involves both stratified squamous epithelium and l Leukaemia

underlying connective tissue. l Wegener’s granulomatosis

l The surface epithelium is destroyed and is replaced l Sarcoidosis

by pseudomembranous meshwork of fibrin, necrotic iv. Idiopathic gingival enlargement


epithelial cells, polymorphonuclear neutrophils and v. Neoplastic enlargements (gingival tumours)
various microorganisms which appear as a surface l Benign tumours

psuedomembrane. l Malignant tumours, e.g. ameloblastoma, odon-

l the underlying connective tissue is hyperaemic with toma, sqamous cell carcinoma, adeno-carcinoma,
numerous engorged capillaries and dense infiltration of malignant melanoma, fibroma, osteoma, exostosis,
polymorphonuclear neutrophils which appears as a lin- giant cell reparative granuloma, haemangioma.
ear erythema. vi. False enlargement
510 Quick Review Series for BDS 3rd Year

LEUKAEMIC GINGIVAL ENLARGEMENT l Juvenile periodontitis is characterised by rapid


loss of attachment and bone loss occurring in an
Pathogenesis otherwise clinically healthy patient with the amount
Leukaemic gingival enlargement consists of basic infiltra- of microbial deposits inconsistent with the disease
tion of the gingival corium by leukaemic cells that increases severity and familial aggregation of diseased indi-
the gingival thickness and creates gingival pockets where viduals.
bacterial plaque accumulates, initiating a secondary inflam-
matory lesion that contributes to the enlargement of gingiva. Aetiology
a. May be genetic (hereditary) in origin
Clinical Features b. May have racial predisposition
l It may be diffused or marginal, localized or generalized. c. May be of fungal
l It may increase in size most often in the interdental pa-
pilla partially covering the tooth crown.
Classificaton
l Gingiva appears bluish red with a shiny surface.
l Gingiva is sponge-like and friable and bleeds spontaneously. l Prepubertal
l The bleeding tendency can also manifest in skin and l Localised
oral mucosa as petechiae. l Generalised
l Diffuse submucosal bleeding manifests as ecchymosis.
The greatly altered tissue is susceptible to bacterial in-
l
LOCALISED JUVENILE PERIODONTITIS
fections and may cause ANUG.
l Is generally not seen in edentulous and chronic leukae- Also known as localised aggressive periodontitis or early
mic patients. onset periodontitis.

Histopathology Clinical Features


l The reticular connective tissue layer is infiltrated l Age of occurrence–between puberty and 20 years.
densely by immature and proliferating leukaemic cells. l Site – first molars and incisors.
l The nature of leucocytes depend on the type of leukaemia. l Sex – more common in females.
l The papillary layer contains comparatively fewer l Lack of clinical inflammation despite the presence of
leucocytes. deep periodontal pockets and advanced bone loss.
l The blood vessels contain predominantly these leukae- l Minimal plaque with elevated levels of bacteria.
mic cells and RBCs are reduced in number. l Causative organism: Acinobacillus actinomycetemcomi-
l Epithelium may be thinned or hyperplastic. tans and Porphyromonas gingivalis.
l Inter- and intracellular oedema is seen with diminished l Progresses rapidly with a rapid bone loss.
surface keratinization. l Distolabial migration of maxillary incisors causing
l Isolated surface areas of acute necrotizing inflammation diastema formation.
with pseudomembranous meshwork of fibrin, necrotic l Increasing mobility of incisors and first molars.
epithelial cells, polymorphonuclear neutrophil bacteria l Sensitivity of denuded root surfaces to thermal and
are frequently seen. tactile stimulation.
l Deep, dull and radiating pain during mastication.
Treatment l Periodontal abscess with regional lymph node en-
largement.
l Rationale is to remove local deposists, i.e. by scaling
and curettage under topical anaesthesia with antibiotics
administered systemically the evening before and for Radiographic Findings
24 hours after each treatment.
l Vertical or angular bone loss around first molars and
l After acute symptoms subside, attention is directed to-
incisors in an healthy teenager.
wards correcton of gingival enlargement
l Pattern appears as arc-shaped loss of alveolar bone from
Q. 3. Juvenile periodontitis. distal surface of second premolar to mesial surface of
second molar.
Ans.
l Frequently, bone loss pattern resembles mirror image
l Juvenile periodontitis is also known as aggressive with bilateral symmetrical pattern of bone loss.
periodontitis. l Wider bone defects than chronic periodontitis.
Section | III Oral Pathology 511

Histopathological Findings Clinical Findings


l Pocket epithelium – ulcerated l Age of occurrence – puberty to 30 years.
l Connective tissue – accumulation of inflammatory cells l Site: All or most of the teeth are affected.
mainly leucocytes, plasma cells and few lymphocytes l Sex: No sex predilection.
and macrophages. l Severe, acutely inflamed tissue.
l Bacterial invasion of connective tissue may reach bone l Often proliferating ulcerated and fiery red, bleeds
surface. spontaneously.
l Suppuration is seen.
l Attachment and bone loss is seen.
Treatment l Systemically shows weight loss, mental depression and
malaise.
l Extraction and transplantation of third molar into the
l Patients produce a poor antibody response to the
socket of first molar.
pathogens present.
l Standard periodontal therapy like scaling, root planning,
l Small amounts of bacterial plaque is associate with it.
curettage.
l Organisms found are A. actinomycetemcomitans,
l Flap surgery, root amputation, hemisection.
P. gingivalis, Tannerella forsythia
l Antibiotic therapy, e.g. tetracycline 250 mg qid for
l Smokers show more number of teeth involved in destruction.
3 weeks.

Radiographic Findings
GENERALISED JUVENILE PERIODONTITIS l Severe bone loss associated with minimal number of teeth.
l Osseous destruction of 25 to 60% during 9-week period.
l It is also known as generalised aggressive periodontitis
l Extreme bone loss in one site with absolute no bone loss
or rapidly progressive periodontitis.
in other site for the same individual.

SHORT ESSAYS
Q. 1. Gingival enlargements. iv. Neoplastic (gingival tumours)
l Benign tumours
Ans.
l Malignant tumours
Gingival enlargements: It is increased in size of gingiva. v. Idiopathic gingival enlargement.
It is a very common feature found in gingival diseases. vi. False enlargement
Other terms:
l Hypertrophic gingivitis

l Gingival hyperplasia
Based on location and distribution
l Localised
Classification l Generalised
l Marginal
Based on aetiology l Papillary
i. Inflammatory enlargement l Diffuse
a. Chronic l Discrete – it is an isolated sessile or pedunculated
b. Acute tumour like enlargement.
ii. Drug-induced enlargement
iii. Associated with systemic diseases Based on degree of enlargement
Conditioned enlargement
l Pregnancy
Grade 0 – No signs of gingival enlargement
l Puberty
Grade 1 – Enlargement confined to papilla
l Vitamin C deficiency
Grade 2 – Enlargement involves papilla and marginal
l Plasma cell gingivitis
gingiva
l Pyogenic granuloma
Grade 3 – Enlargement covers three quarters or more of the
Systemic diseases like crown
l Leukaemia
Q. 2. Acute necrotizing ulcerative gingivitis.
l Wegener’s granulomatosis

l Sarcoidosis Ans.
512 Quick Review Series for BDS 3rd Year

Acute necrotizing ulcerative gingivitis (ANUG) is an in- Extraoral and systemic signs and symptoms:
flammatory and destructive endogenous oral infection Mild to moderate stages:
which is characterized by the necrosis of gingival tissue.
l Local lymphadenopathy
Other terms: Trench mouth, Vincent’s infection, acute
l Slight elevation of temperature
ulceromembranous gingivitis and acute ulcerative gingivitis.
Severe cases:
Aetiology l High fever with increased pulse rate
l Loss of appetite and general lassitude
i. Role of bacteria
It is caused mainly by specific bacteria: Fusiform bacillus Systemic reactions are severe in children.
and spirochaetes. Rarely noma, gangrenous stomatitis, fusospirochaetal men-
ingitis, peritonitis, toxaemia and fatal brain abscess may
ii. Local predisposing factors occur.
l Poor oral hygiene
l Area of gingiva traumatized by opposing in maloc- Q. 3. Differences between supragingival and subgingival
cluded teeth calculus.
l Smoking – due to direct toxic effect of nicotine. Ans.

iii. Systemic predisposing factors l Dental calculus a hard, stone-like concretion, varying in
l Nutritional deficiency like vitamin A, C, B2 colour from creamy yellow to black, that forms on the
l Chronic diseases – syphilis, cancer teeth or dental prostheses through calcification of dental
l Gastrointestinal diseases – ulcerative colitis plaque.
l Blood dyscrasias – leukaemia, aplastic anaemia, and l Differences between supragingival and subgingival
AIDS. calculus are as follows:

iv. Psychosomatic factors Supragingival calculus Subgingival calculus


l Psychiatric disturbances
l Location – coronal to the l Location – below the crest
gingival margin of gingival margin
Clinical Features l Visible in the oral cavity l Not visible on routine
clinical examination
l It is identified as an acute disease characterized by sud-
den onset, sometimes followed by an episode of debili- l Colour – white, yellow in l Colour – dark brown or
colour greenish black in colour
tating diseases or acute respiratory tract infections.
l It is seen commonly in age group 16 to 30 years. l Consistency – hard and l Consistency – hard and
clay-like dense
l Can be easily detatched l Firmly attached and
Oral Signs and Symptoms from tooth surface difficult to remove

Signs l Source – derived from l Source – formed from


salivary secretions gingival exudate
l Charachterised by punched out, crater-like depressions
at the crest of the interdental papillae, subsequently l Composition: l Composition:
More brushite and octacalcium Less brushite and octacalcium
involving marginal gingiva and rarely attached phosphate phosphate
gingiva. Less magnesium whitlockite More magnesium whitlockite
l Craters are covered by greyish pseudomembranous
l Salivary proteins – present l Salivary proteins – absent
slough with a marked demarcation of linear erythema
from the normal mucosa. l Sodium content lesser l Sodium content increases
l Spontaneous bleeding from gingival tissue with fetid with depth of pocket.
odour and increased salivation.

Symptoms Q. 4. Write a note on aetiology of gingivitis.


l Extremely tender with radiating pain on eating hot and
spicy foods. Ans.
l Metallic foul taste Gingivitis is the inflammation of gingiva and occurs in
l Pasty saliva. acute, subacute and chronic forms.
Section | III Oral Pathology 513

Local factors: v. Mouth breathing: It leads to drying of oral mucous


i. Microorganisms: The plaque associated with gingivi- membrane which results in gingival irritation with ac-
tis is complex and heterogeneous. The Actinomyces companying irritation.
group of organisms are dominant in supragingival vi. Predisposing malocclusion
plaque. vii. Lack of oral hygiene
ii. Calculus: Supragingival or subgingival or both causes
irritation of contacting gingival tissue. The irritation is Systemic factors:
produced by the mechanical friction resulting form i. Nutritional disturbances: The nutritional imbalance if
hard, ruf surface of calculus. frequently manifestated as changes in gingiva.
iii. Food impaction: The impaction of food and accumula- ii. Pregnancy: During pregnancy, gingiva undergoes changing
tion of debris on teeth due to oral negligence results in causing pregnancy gingivitis.
gingivitis. iii. Diabetes mellitus
iv. Faulty and irritating restorations or appliance may act iv. Hormonal changes: Puberty and mensturation may
as irritant to gingival tissue and induce gingivitis. result in gingival inflammation.

SHORT NOTES
Q. 1. Vincent infection. l Papillon-Lefevre syndrome is a autosomal recessive
inherited disease.
Ans.
l Sex: Males and females are equally affected.
l Vincents infection is also known as acute necrotizing
ulcerative gingivitis (ANUG).
l It is an inflammatory and destructive endogenous oral Clinical Features
infection which is characterized by the necrosis of
l Charachterised by hyperkeratotic skin lesions, severe
gingival tissue.
destruction of periodontium and calcification of dura.
l It is seen commonly in age group of 16 to 30 years.
l The cutaneous and periodontal changes appear before
l Characterised by punched out, crater-like depressions
age of 4 years.
covered by grayish pseudomembranous slough at the
l Periodontal involvement exhibits early bone loss and
crest of the interdental papillae, subsequently involving
exfoliation of teeth.
marginal gingival and rarely attached gingival.
l Primary teeth are lost within 5 years of age and perma-
l Spontaneous bleeding from gingival tissue, fetid odour
nent dentition is lost by age of 15 years and patient
and increased salivation.
becomes edentulous.
l Extremely tender with radiating pain on eating hot and
spicy foods. Q. 4. Dilantin gingival hyperplasia.
l Palliative treatment.
Ans.
Q. 2. Microorganisms causing ANUG.
l Phenytoin (dilantin) gingival hyperplasia is the first
Ans.
drug-induced gingival enlarment.
Acute necrotising ulcerative gingivitis is caused by spe- l The gingival enlargement is seen in more than 50% of
cific bacteria: patients receiving the drug.
l Fusiform bacillus and spirochaetal organism forming a l Most commonly seen in younger individuals.
fusospirochaetal complex. l It starts as a painless swelling in the interdental papilla
l The complex consists of Treponema microdentium, Pre- and extends to facial and lingual margins and grow
votella intermedia, intermediate spirochaetes, vibrios, massively to interfere with the occlusion.
fusiform bacilli, and filamentous organisms, Selenomo- l The lesion is mulberry-shaped, firm pale pink and
nas species in addition to Borrelia species. resilient, minutely lobulated surface.
l The exudate from NUG lesions consists of fusospiro- l No tendency to bleed.
chaetal abscesses. l The enlargement is generalised with more severity seen
in anterior regions.
Q. 3. Papillon-Lefevre syndrome.
l Overgrowth does not depend on oral hygiene.
Ans. l It disappears from which the teeth are extracted.
514 Quick Review Series for BDS 3rd Year

Q. 5. Juvenile periodontitis. l Borrelia vincentii is an organism that exists normally in


the human mouth in low concentrations and safe pro-
Ans.
portions.
l Juvenile periodontis is characterised by rapid loss of l It is an anaerobic spirochaete that is also gram-negative.
attachment and bone loss occurring in an otherwise l Discovered by French physician Henri Vincent, its
clinically healthy patient with the amount of microbial common name is Vincent’s disease or Vincent’s angina –
deposits inconsistent with the disease severity and fa- an infection of oropharynx and throat.
milial aggregation of diseased individuals. l Patients have a painful membranous ulceration of throat.
l Causative organisms are Acinobacillus actinomycetem- l Symptoms of infection with Borrelia vincentii include
comitans and Porphyromonas gingivalis. foul breath, ulcers in the interdental papillae, ulcers on
l Juvenile periodontitis is classified as: the gums that easily bleed, increase in the space between
a. Localised juvenile periodontitis: Commonly seen in teeth, gums covered with white/grey layer of dead skin,
females in relation to first molars and incisors, and fever accompanied with fatigue.
b. Generalised juvenile periodontitis: Involves all or
Q. 9. Granulomatous diseases of periodontium.
most of the teeth with no sex predilection.
Q. 6. Marginal gingivitis. Ans.

Ans. The granulomatous diseases of periodontium are as follows:


i. Eosinophilic granuloma
Gingivitis is inflammation of the gums without involve- ii. Giant cell granuloma
ment of the mucous membrane of the mouth. iii. Wegener’s granulomatosis
iv. Pyogenic granuloma or granuloma pyogenicum
Marginal gingivitis:
Gingivitis involving the gingival margin is called as mar- Q. 10. Infectious acute periodontitis.
ginal gingivitis. It may include a portion of the contiguous Ans.
attached gingiva.
l It is of 2 types: l Infectious acute periodontitis is also called as aggres-
a. Localised marginal gingivitis: It is confined to one sive periodontitis.
or more areas of the marginal gingiva. l This is a rapidly progressing type of periodontitis with-
b. Generalised marginal gingivitis: It involves the out large accumulations of plaque or calculus.
gingival margins in relation to all teeth. The inter- l A sharply localized, acute inflammatory process involving
dental papillae are usually affected. the interproximal and marginal areas of two or more
adjacent teeth.
Q. 7. Scorbutic gum. l It may be localized or generalized form.
Ans. l It is characterized by severe pain, purulent exudate from
oedematous inflamed gingivae, general malaise, fever,
l Scorbutic gum is seen in scurvy caused due to pro- and sequestration of the crestal aspects of the alveolar
longed vitamin C deficiency for at least 3 months. process.
l “Ascorbic acid” comes from the New Latin word “scor-
butus” meaning scurvy. Q. 11. Periapical periodontitis.
Ans.
Clinical Features l Periapical periodontitis is the inflammation of the tissue
l It is an early manifestation of vitamin C deficiency. adjacent to the tip of the tooth’s root, known as the peri-
l Gingiva becomes brilliant red, tender and grossly swollen. apical tissue.
l Oral symptoms are like that of chronic gingivitis involv- l Symptoms can include pain, swelling, the loosening of
ing free, attached gingiva and alveolar mucosa. teeth, redness and pockets, where the gum and bone
l Degeneration of collagen fibres occurs in gingiva. recede from the tooth.
l Scurvy buds are seen on gingiva. l Types: General, unspecified and acute.
l The spongy tissues are extremely hyperaemic and bleed l If left untreated, the disease can advance, causing perma-
spontaneously. nent gum, tooth and bone damage, finally the tooth loss.
l Ulcerated gums Q. 12. Gum boil or parulis.
l Treatment includes vitamin C supplementation.
Ans.
Q. 8. Borrelia vincentii.
l Gum boil, known medically as Parulis, is defined as a
Ans. drainage point for abscessed teeth.
Section | III Oral Pathology 515

l This is a dental infection that occurs around the root of Q. 16. Periodontal (lateral) abscess.
the tooth.
Ans.
l It can be caused by tooth decay, gum infections, failed
root canal, weak immune system. l Periodontal abscess is a localized accumulation of pus
l Gum boil appears as a small red bulging in the gum within the gingival wall of a periodontal pocket.
which when pressed drains out pus or occasionally blood. l The tooth shows tenderness to lateral percussion.
l This condition is very painful and may also cause fever, l Affected tooth may be vital or non-vital.
swelling of the face and earache. l It may be acute or chronic.
l The patient may experience a foul salty taste in his l Treatment: Draining the abscess, and subgingival curet-
mouth and bad breath due to the pus draining out. tage, antibiotics.
Q. 13. Intrinsic staining in teeth. Q. 17. Mention the differences between acute and
chronic periodontal abscess.
Ans.
l Pigmented deposits on tooth surface are called as dental Ans.
stains.
l The stains that are incorporated into the tooth surface
are called as intrinsic stains. Acute periodontal abscess Chronic periodontal abscess
l Intrinsic staining is generally seen in: Ovoid elevation of gingiva A sinus that opens on to
i. Porphyria, along lateral aspect of root gingival mucosa somewhere
along the length of the root
ii. Erythroblastosis fetalis and
iii. Tetracycline therapy. Symptoms: Symptoms:
l Throbbing radiating pain l Asymptomatic or dull
Q. 14. Dental stains. l Tooth mobility and gnawing pain
sensitivity l Slight elevation of tooth and
Ans. l Lymphadenopathy a desire to bite down and
l Pigmented deposits on tooth surface are called as dental grind
stains. Pus may be expressed on slight Patient gives history of
l Dental stains are of two types: digital pressure intermittent exudation of pus
a. Intrinsic stains: For example, in porphyria, erythro-
blastosis fetalis and tetracycline therapy.
b. Extrinsic stains: Q. 18. Chronic periodontitis (pyorrhoea alveolaris).
i. Yellowish brown stain from tobacco smoking.
ii. Black stains caused by chromogenic bacteria. Ans.
iii. Brown stains caused by mesenteric line at the l Chronic periodontitis is also known as peridontoclasia
cervical margin of tooth composed of pigmented or pyorrhoea alveolaris.
salivary mucin. l Clinical findings are as follows:
iv. Green stains mostly found in children formed by In early stages:
the remnants of Nasmyth’s membrane possibly No pain, no pus
by blood pigments. Red margins of gums
Q. 15. Desquamative gingivitis. Loss of interdental papillae
In later stages:
Ans.
Pain and soreness, bleeding gums with bad smell
l Desquamative gingivitis is a peculiar condition charac- Pockets formation and sensitivity of teeth
terised by intense erythema, desquamation, and ulcer- Pus at the gingival margin, itching of gums and un-
ation of free and attached gingiva. easiness
l The clinical description of desquamative gingivitis repre- Loss of alveolar bone with mobility, extrusion and
sents gingival manifestations of a variety of diseases such shifting of teeth.
as lichen planus, cicatricial pemphigoid, pemphigus vul- In last stage:
garis, bullous pemphigoid, adult linear IgA dermatosis, Constant bleeding from gums. Bad taste in the
dermatitis, herpetiformis, and drug reactions. mornings. Loss of teeth on their own.
l The symptoms range from mild to severe form. May spread to maxillary sinus or body of the mandible.
l Severe form is charachterised by denuded gingiva while the l Treatment includes removing all deposits around
epithelium is shredded, friable, and peeled of small patches. teeth, causes of irritation to the gum margins and the
l The condition is painful pockets, i.e. gingivectomy, subgingival curettage and
l Oral cavity is dry with burning sensation. home care.
516 Quick Review Series for BDS 3rd Year

Topic 9

Dental Caries

LONG ESSAYS
Q. 1. Define and classify dental caries. Describe l The enamel directly bordering the pit and fissure may
the clinical and histopathological features of dentinal appear opaque, bluish white and is undermined.
caries l The lateral spread of caries at the DEJ as well as pene-
tration into the dentin along the dentinal tubules may be
Ans.
extensive without fracturing away the overhanging
Dental caries has been defined as: enamel.
l An irreversible progressive microbial disease of the Histopathological features of dentinal caries:
calcified tissues of the teeth, characterized by the de- l Destruction of dentin is by decalcification and by prote-
mineralization of the inorganic portion and distortion of olysis of collagenous matrix.
the organic substances of the tooth, which often leads to l Decalicification of dentin alone does not cause distint-
cavitation. egration (but softens) because protein material forming
up to 25% of weight of the tissue, remains intact until
Classification of Caries destroyed by the bacterial enzymes.
l Once the caries reaches the dentin, the lesion rapidly
l Dental caries can be classified in a number of ways. spreads laterally along the dentinoenamel junctions.
l Three basic factors – morphology, dynamics and chro- This is because:
nology are considered for classification. Based on these a. Dentin is less mineralized than enamel, so acids will
factors, caries can be classified as follows. destroy more of the mineralized matrix.
b. Dentin is cellular, which allows easy bacterial move-
a. Based on the morphology or anatomical site: ment into the deeper portions of the lesion.
i. Pit and fissure caries
ii. Smooth surface caries
Zones of Dentinal Caries
b. Based on the dynamics with regard to rate of caries As the carious lesion progresses, various zones of carious
progression: dentin may be distinguished which tend to assume a trian-
i. Acute caries gular shape with apex towards pulp and the base towards
ii. Chronic caries the enamel. The dentinal caries histologically presents 5
zones as follows:
c. Based on the nature of attack:
i. Primary (virgin) caries Zone 1: Zone of fatty degeneration of Tome’s
ii. Secondary or recurrent caries fibres:
l This zone represents the inner most layer of the carious
d. Based on chronology, it can also be classified as: dentin, here the dentinal tubules appears normal.
l Infancy (nursing bottle) caries l There is evidence of fatty degeneration of the Tome’s
l Adolescent caries fibres.
l No crystals in the lumen of the tubules.
l No bacteria in the tubules.
Clinical Features of Dentinal Caries l Intertubular dentin has normal cross banded collagen
l It occurs on pit and fissures, smooth surfaces and roots and normal dense appetite crystals.
of the teeth.
l It usually occurs in pit and fissures with high steep wall Zone 2: Zone of dentinal sclerosis:
and narrow bases. l This is the zone of dentinal sclerosis and is character-
l It appears brown or black and will feel slightly soft and ized by the deposition of very fine crystal structures
catch a fine explorer point. calcium salts within the dentinal tubules.
Section | III Oral Pathology 517

l Superficial layer shows area of demineralization and l The aetiology of dental caries is a very complex pro-
damage of the odontoblastic processes. cess, which is often explained with the help of some of
l No bacteria in the tubules. the following theories.
l The dentin is capable of remineralization. i. Acidogenic theory
ii. Proteolytic theory
Zone 3: Zone of decalcification of dentin: iii. Proteolytic chelation theory
l This zone is a narrow zone, preceding bacterial invasion iv. Sucrose chelation theory
and appears transparent because of decalcification of v. Autoimmune theory.
dentin.
It is softer than normal dentin.
l
I. ACIDOGENIC THEORY
l No bacteria in tubules.
l Cross banded intertubular collagen is still intact. l This theory is also known as Miller’s chemicoparasitic
l This zone is capable of self-repair and remineralization. theory.
l Acidogenic theory states that the process of dental car-
Zone 4: Zone of bacterial invasion of decalcified ies is a chemicoparasitic process consisting of two
but intact dentin: stages:
l This zone is marked by widening and distortion of den- a. Preliminary stage: Production of organic acid occurs
tinal tubules, which are packed with microorganisms. as a result of fermentation of the carbohydrates by
l There is very little amount of minerals in dentin. the plaque bacteria. The decalcification of enamel
l Denaturation of collagen fibres also takes place. which results in its total destruction and decalcifica-
l The zone cannot undergo self-repair or remineralization. tion of dentin.
l This zone must be removed before restoration. b. Later stage: Preliminary stage followed by dissolu-
tion of the softened residue.
Zone 5: Zone of decomposed dentin: l Acids resulting in primary decalcification is pro-
l This is the outermost zone of the carious dentin. duced by the fermentation of dietary carbohydrates,
l It is characterized by complete destruction of dentinal i.e. starches and sugar from the retaining centres of
tubules. teeth.
l In this zone, the area of decomposition of dentin, which l According to Miller, there are four important factors,
occurs along the direction of dentinal tubules, is called which can influence the process of tooth destruction in
“liquefaction foci of Miller”, and which occurs perpen- dental caries:
dicular to dentinal tubules is called “transverse clefts” i. Dietary carbohydrates
l In the process, the entire dentinal structures become ii. Microorganisms
destroyed and cavitation begins from dentinoenamel iii. Acids
junction. iv. Dental plaque.

Q. 2. Define dental caries. Write in detail about the i. Role of carbohydrates:


theories of dental caries. l Fermentable dietary carbohydrates play an important
Or role in the causation of caries, e.g. glucose, fructose and
sucrose. Among them, sucrose is more potent.
Discuss about various theories of dental caries. l These sugars are easily and rapidly fermented by cario-
genic bacteria in the oral cavity to produce acid at or
Or near the tooth surface and causes dissolution of the hy-
droxyapetite crystals of the enamel followed by the
Define caries. Discuss in detail about acidogenic theory.
dentin.
Or l The cariogenicity of a dietary carbohydrate varies with
the frequency of ingestion, physical form and chemical
Define dental caries and describe the Miller’s acido- composition, route of administration and presence of
genic theory. other food constituents.
l Risk of caries incidence increases greatly, if the dietary
Ans.
sugar is solid, sticky and soft in nature which remains
l “Dental caries is an irreversible progressive microbial adherent to the tooth surface for long time after taking
disease of the calcified tissues of the teeth, characterized the meal.
by the demineralization of the inorganic portion and l Following the ingestion of these sugars, the pH of the
distruction of the organic substances of the tooth, which plaque falls to 4.5 to 5 within 1 to 3 minutes and neu-
often leads to cavitation”. tralization occurs after 10 to 30 minutes.
518 Quick Review Series for BDS 3rd Year

l The glucose, sucrose and fructose, etc. are rapidly de- l Drawbacks of this theory are that it cannot explain the
fused into the plaque due to their low molecular weight. role of sucrose, pH, fluoride, etc. in dental caries.
l This theory summarises that the interplay between l This theory concluded that proteolysis in the intiation of
Bacteria 1 Sugars 1 Teeth n Organic acids n Caries. dental caries is likely to be of no significance, but its
role in the progression of the more advanced carious
ii. Role of microorganisms: lesions cannot be ruled out.
l A large number of microorganisms play individual role
in dental caries production and among them the most
C. PROTEOLYTIC CHELATION THEORY
important one is Streptococcus mutans.
l It readily ferments the dietary carbohydrate to produce This theory proposed by Schatz et al (1955 ) implies a si-
acid, which causes tooth destruction. multaneous microbial degradation of the organic compo-
l It synthesizes dextran from sucrose, which helps in ad- nents and dissolution of the minerals of the process known
hering the plaque bacteria as well as the acid on to the as chelation.
tooth surface. l According to this theory during caries, first of all pro-
l S. mutans has the ability to adhere and to grow on hard teolytic breakdown of the organic portion of the enamel
and smooth surface of the teeth. matrix takes place.
l The actinomycosis group, e.g. Actinomycosis israelii, l Following this a chelating agent is formed by the com-
Actinomycosis viscosus, etc. are the important organ- bination of the proteolytic breakdown products, ac-
isms to cause root caries. The Lactobacillus acidophilus quired pellicle, food debris, etc.
is important organism for the progress of dental caries. l The chelating agent (negatively charged) released the
calcium ion (positively charged) from enamel and den-
iii. Role of acids: tin, this process is called as chelation and eventually
l During the process of caries formation, a large variety results in tooth decay.
of acids are produced in the oral cavity due to the bacte- l This theory finally stated that both organic and inor-
rial fermentation of dietary carbohydrate. ganic portion of enamel may be attacked simultane-
l These acids are lactic acid, aspartic acid, acetic acid, bu- ously.
tyric acid, and glutamic acid. They can cause demineral-
ization of enamel and dentin and causes the tooth decay.
D. SUCROSE CHELATION THEORY
iv. Role of bacterial plaque: l Egglers–Lura (1967) proposed that sucrose itself, and
l Plaque is a thin, transparent film produced on tooth not the acid derived from it, can cause dissolution of
surface and it consists of microorganisms suspended in enamel by forming an ionized calcium saccharate.
salivary mucin, also contain desquamated epithelial l This theory proposes that if there is very high concen-
cells, leucocytes, food debris, etc. tration of sucrose in the mouth, there can be formation
l The dental plaque helps in initiation of dental caries by: of complex substances like calcium saccharide and cal-
i. Harbouring the cariogenic bacteria on the tooth surface. cium complexing intermediaries. These complexes
ii. Holding the acids on the tooth surface for long cause release of calcium and phosphorus ions from the
duration. enamel and thereby results in tooth decay.
iii. By protecting the acids from getting neutralized by
buffering action of saliva.
E. AUTOIMMUNE THEORY
This theory suggests that few odontoblast cells at some
B. PROTEOLYTIC THEORY l

specific sites, within the pulp of few specific teeth are


l According to this theory, the proteolytic enzymes liber- damaged by autoimmune mechanisms.
ated by cariogenic bacteria causes destruction of organic
matrix of enamel, as a result of which the inorganic crys-
Current Concept of Caries Aetiology
tals of the enamel become detached from one another
and finally the whole structure collapses, leading to a l Dental caries is a multifactorial disease in which there
cavity formation. is interplay of three primary factors: the host, micro-
l Gottlieb and Gottlieb; Diamond and Applebaum stated flora and the substrate. In addition, a fourth factor, i.e.
that “Caries is essentially a proteolytic process: the mi- time.
croorganisms invade the organic pathways and destroy l Caries formation requires a host, a cariogenic flora and
them in their advance. Acid formation accompanied a suitable substrate that must be present for a sufficient
proteolysis”. length of time.
Section | III Oral Pathology 519

Q. 3. Discuss in detail the role of carbohydrates in Advanced enamel caries


dental caries. Write about histopathology of enamel l It presents several zones in the tissues, out of which four
caries. zones are clearly visible, starting from the inner advanc-
ing front of the lesion, the zones are:
Ans.
i. Zone 1: Translucent zone
l The role of dietary factors in the occurrence of dental ii. Zone 2: Dark zone
caries is well established. iii. Zone 3: Body of the lesion
l Carbohydrates are one of the main dietary factors that iv. Zone 4: Surface zone
are thought to be associated with increased incidence of
dental caries. Zone 1: Translucent zone:
l It is the deepest zone which lies at the advancing front
Role of Carbohydrates in Dental Caries of the enamel lesion.
l It is the first recognizable zone of alteration from nor-
l Fermentable dietary carbohydrates (e.g. glucose, fruc- mal enamel.
tose and sucrose) play an important role in the causation l This zone is more porous than normal enamel and the
of caries. pores are larger than the normal enamel.
l The glucose, sucrose and fructose, etc. are rapidly dif- l The pores are located at prism boundaries and other
fused into the plaque due to their low molecular weight. junctional sites. The pore volume is 1%.
l Among all the carbohydrates, sucrose is more potent. l This zone appears structureless and contains more fluo-
l These sugars are easily and rapidly fermented by cario- ride than normal enamel.
genic bacteria in the oral cavity to produce acid, at or
near the tooth surface that causes dissolution of the hy- Zone 2: Dark zone:
droxyapatite crystals of the enamel followed by the
l This zone is often referred to as ‘positive zone’.
dentin.
l The dark zone is located just superficial to the translu-
l Risk of caries incidence increases greatly, if the dietary
cent zone.
sugar is sticky in nature which remains adherent to the
l Its dark appearance is due to the excessive demineral-
tooth surface for long time after taking the meal.
ization of the enamel.
l Following the ingestion of these sugars, the pH of the
l The zone is narrower in rapidly advancing caries and
plaque falls to 4.5 to 5 within 1 to 3 minutes and neu-
wider in slowly advancing lesion.
tralization occurs after 10 to 30 minutes.
l The zone contains 2 to 4% pore volume. The pores are
l The dental caries occurs due to the interplay of factors
larger than normal enamel, but smaller than those of
such as oral microorganisms, local carbohydrates and
translucent zone.
tooth surface.
l This zone reveals some degree of remineralisation of
l The following equation describes the role of these factors
carious lesion.
in caries
Bacteria 1 Sugars 1 Teeth n Organic acids n Zone 3: Body of lesion:
Dental caries l The zone is situated between the dark zone and the sur-
face layer of enamel.
l It represents the area of greatest demineralization.
Histopathological Features of Caries in Enamel l The pore volume is 5 to 25%.
A wide range of histopathological changes may be noted l This zone contains apatite crystals larger than those of
based on the severity of the enamel caries. the normal enamel.
l The large crystals results from the reprecipitation of
Early enamel caries minerals dissolved from the deeper zone.
l There will be loss of interprismatic or inter-rod sub-
stances with increase in prominence of these enamel Zone 4: Surface zone:
rods. l The surface zone when examined by the polarizing light
l Dark line often appears at right angles of the enamel appears relatively unaffected.
rods, suggesting segments. l It may be due to the surface remineralization by the
l Accentuation of the incremental striae of Retzius often salivary mineral ions.
occurs. l Pore volume is less than 5%.
l Accentuation of striae of Retzius and accentuation of l The surface zone is usually demineralised when the
perikymata are often the early microscopic changes. lesion has penetrated into the dentin.
520 Quick Review Series for BDS 3rd Year

Q. 4. Describe the structure of dental plaque and its role bacterial surface of certain bacteria recognize spe-
in the aetiology of dental caries. cific carbohydrate structure in the pellicle and be-
come linked (selective process).
Ans.
iv. Adhesion of Streptococcus mutans takes place through
Dental plaque is a film of adhesive material which forms the production of an enzyme glycosyltransferase (GFT)
continuously on the surface of the teeth, especially in stag- by the bacteria. GFT is adhesive and it is adsorbed on
nation area. It is a sticky film of bacteria that adheres to the the tooth surface.
tooth surface. l In summary, multiple processes are involved in the

interactions with salivary glycoproteins, extracellu-


lar polysaccharides and direct cell-to-cell adhesion.
Structure of Dental Plaque l In this initial colonization of bacteria, gram-positive

l Characteristically forms on uncleaned tooth or denture cocci adhere to irregularities in pellicle covered enamel.
surfaces. l During this initial adherence, reactions occur mainly

l Composed of 80% water and 20% solids. between specific bacteria and the pellicle.
l About one-half of the dry weight of plaque is attributed l Then, further bacterial colonization by rods to already

to bacteria and salivary proteins. attached cells takes place.


l In addition to proteins, plaque also contains carbohy- l In this second phase, bacteria, bacterial products,

drates and lipids. These constitute 25% of the dry interbacterial matrix, host and dietary factors are
weight of plaque. involved.
l Inorganic components constitute 5-10% of dry weight
of plaque. Has high concentration of calcium and phos-
Some Important Factors that Increase
phate.
l Classified as supra- or subgingival plaque based on the the Risk of Caries
anatomical region of appearance. Dental plaque harbors caries causing microorganisms and
keeps them in constant touch with the tooth surfaces:
Role of Plaque in Aetiology of Dental Caries l Plaque also contains carbohydrates and lipids that help
in adhesion of the bacteria and also acts a source of
l Supragingival plaque plays an essential role in dental fermentable substrates
caries. l It acts as a barrier to salivary buffers which can neutral-
l The plaque is the site of the greater concentration of ize the acids formed in plaque.
bacteria in the mouth. l Certain cariogenic bacteria in plaque are capable of
l About 85% of the plaque in individuals suffering from storing polysaccharides that can act as a carbohydrate
caries had a high concentration of aciduric streptococci. reserve to be used when the individual is consuming a
l If plaque contains more of acid producing bacteria, then sugar-free diet.
caries will result.
l Oral streptococci convert disaccharides (sucrose) into Q. 5. Define and describe dental caries. Describe the
polysaccharides. They produce glucans (glucose units) clinical and histopathological features of caries in
and levans (fructose units) which by acting as “biologic enamel.
glue” hold the plaque mass together and allow the ad- Ans.
herence of plaque to the tooth surface.
l Increased concentration of microorganisms in the Dental caries has been defined as an irreversible progres-
plaque increases the risk of dental caries. sive microbial disease of the calcified tissues of the teeth,
characterized by the demineralization of the inorganic por-
tion and distortion of the organic substances of the tooth,
Mechanism of Bacterial Adherence which often leads to cavitations.
i. Initial bacterial attachment or adherence takes place
through electrostatic interactions, i.e. negatively
Caries Description
charged bacterial cell surfaces and pellicle become
linked by cations such as calcium. l It is considered as a disease of the modern civilization.
ii. Bacterial attachment also takes place through hydro- l The prevalence of caries is lowest in Asian and African
phobic interactions, i.e. a close structural fit occurs be- countries while it is highest in American and other
tween molecules on the pellicle and bacterial surfaces. Western countries.
iii. Bacterial attachment also takes place through spe- l It is more common in children while adults are also
cific lectin like interactions. Lectins (proteins) in the frequently affected; it occurs equally among both sexes
Section | III Oral Pathology 521

l Dental caries is a multifactorial disease in which there l This zone is more porous than normal enamel and the
is an interplay of three primary factors: the host, micro- pores are larger than the normal enamel.
flora and the substrate. In addition, a fourth factor, i.e. l The pores are located at prism boundaries and other
time is also responsible. junctional sites. The pore volume is 1%.
l Pits and fissures are the most commonly affected areas l This zone appears structureless and it contains more
while the smooth surfaces are also frequently affected. fluoride than normal enamel.
l Plaque is generally considered as the initiating factor of
dental caries. Zone 2: Dark zone:
l Untreated caries can result in complications such as l This zone is often referred to as ‘positive zone’.
pulpitis and periapical abscess. l The dark zone is located just superficial to the translu-
cent zone.
Clinical Features of Enamel Caries l Its dark appearance is due to the excessive demineral-
ization of the enamel.
l It occurs on pit and fissure, smooth surface and root l The zone is narrower in rapidly advancing caries and
surfaces. wider in slowly advancing lesion.
l It frequently affects the pit and fissures with high steep l The zone contains 2 to 4% pore volume. The pores are
wall and narrow bases. larger than normal enamel, but smaller than those of
l It appears brown or black and will feel slightly soft and translucent zone.
catch a fine explorer point. l This zone reveals some degree of remineralisation of
l The enamel directly bordering the pit and fissure may carious lesion.
appear opaque, bluish white and is undermined.
l The lateral spread of caries at the DEJ as well as penetra- Zone 3: Body of Lesion
tion into the dentin along the dentinal tubules may be ex- l The zone is situated between the dark zone and the sur-
tensive without fracturing away the overhanging enamel. face layer of enamel.
l It represents the area of greatest demineralization.
Histopathological Features of Caries in Enamel l The pore volume is 5 to 25%.
l This zone contains apatite crystals larger than those of
A wide range of histopathological changes may be noted
the normal enamel.
based on the severity of the enamel caries.
l The large crystals results from the reprecipitation of
Early enamel caries minerals dissolved from the deeper zone.
l There will be loss of interprismatic or inter-rod substances Zone 4: Surface Zone
with increase in prominence of these enamel rods.
l The surface zone when examined by the polarizing light
l Dark line often appears at right angles of the enamel
appears relatively unaffected.
rods, suggesting segments.
l It may be due to the surface remineralization by the
l Accentuation of the incremental striae of Retzius often
salivary mineral ions.
occurs.
l Pore volume is less than 5%.
l Accentuation of striae of Retzius and accentuation of
l The surface zone is usually demineralised when the
perikymata are often the early microscopic changes.
lesion has penetrated into the dentin.
Advanced enamel caries Q. 6. Describe the histopathology of caries in dentin and
l It presents several zones in the tissues, out of which four discuss the pioneer bacteria.
zones are clearly visible, starting from the inner advanc-
Ans.
ing front of the lesion, the zones are:
i. Zone 1: Translucent zone Histopathological features of dentinal caries:
ii. Zone 2: Dark zone l Destruction of dentin is by decalcification and by prote-
iii. Zone 3: Body of the lesion olysis of collagenous matrix.
iv. Zone 4: Surface zone l Once the caries reaches the dentin, the lesion rapidly
spreads laterally along the dentinoenamel junction, this
Zone 1: Translucent zone: is because:
l It is the deepest zone which lies at the advancing front a. Dentin is less mineralized than enamel, so acids will
of the enamel lesion. destroy more of the mineralized matrix.
l It is the first recognizable zone of alteration from nor- b. Dentin is cellular, which allows easy bacterial move-
mal enamel. ment into the deeper portions of the lesion.
522 Quick Review Series for BDS 3rd Year

Zones of Dentinal Caries Pioneer Bacteria


As the carious lesion progresses, various zones of carious l Pioneer bacteria or the first binders are the first group of
dentin may be distinguished which tend to assume a trian- bacteria that adhere to the dental plaque.
gular shape with apex towards pulp and the base towards l Studies have shown that the pioneer bacteria are strep-
the enamel. The dentinal caries histologically presents tococci, neisseria, actinomyces and capnocytophaga.
5 zones as follows: l Other bacteria coadhere to the pioneer bacteria to
increase the complexity of the association
Zone 1: Zone of fatty degeneration of Tome’s l Dental caries results from the metabolism of this asso-
fibres: ciation which attack the tooth enamel.
l This zone represents the inner most layer of the carious l Initial bacterial attachment (adherence) takes place
dentin, here the dentinal tubules appear normal. through electrostatic interactions, i.e. negatively charged
l There is evidence of fatty degeneration of the Tome’s bacterial cell surfaces and pellicle become linked by
fibres. cations such as calcium.
l No crystals in the lumen of the tubules. l Bacterial attachment also takes place through hydro-
l No bacteria in the tubules. phobic interactions, i.e. a close structural fit occurs be-
tween molecules on the pellicle and bacterial surfaces.
Zone 2: Zone of dentinal sclerosis: l Bacterial attachment also takes place through specific lectin
l This is the zone of dentinal sclerosis and is character- like interactions. Lectins (proteins) in the bacterial surface
ized by the deposition of very fine crystal structures of certain bacteria recognize specific carbohydrate structure
calcium salts within the dentinal tubules. in the pellicle and become linked (selective process).
l Superficial layer shows area of demineralization and l Adhesion of Streptococcus mutans takes place through
damage of the odontoblastic processes. the production of an enzyme glycosyltransferase (GFT)
l No bacteria in the tubules and the dentin are capable of by the bacteria. GFT is adhesive and it is adsorbed on
remineralization. the tooth surface.
l In summary, multiple processes are involved in the
Zone 3: Zone of decalcification of dentin: interactions with salivary glycoproteins, extracellular
l This zone is a narrow zone, preceding bacterial invasion polysaccharides and direct cell-to-cell adhesion.
and appears transparent because of decalcification of Q. 7. Define caries. Discuss in detail about acidogenic
dentin. theory.
l It is softer than normal dentin.
l No bacteria in tubules. Or
l This zone is capable of self-repair and remineralization. Define dental caries and describe the Miller’s acidogenic
theory.
Zone 4: Zone of bacterial invasion of decalcified
but intact dentin: Ans.
l This zone is marked by widening and distortion of den- l “Dental caries is an irreversible progressive microbial
tinal tubules, which are packed with microorganisms. disease of the calcified tissues of the teeth, characterized
l There is very little amount of minerals in dentin. by the demineralization of the inorganic portion and
l Denaturation of collagen fibres also takes place. distruction of the organic substances of the tooth, which
l The zone cannot undergo self-repair or remineralization. often leads to cavitation”.
l The aetiology of dental caries is a very complex pro-
Zone 5: Zone of decomposed dentin: cess, which is often explained with the help of some of
l This is the outermost zone of the carious dentin. the following theories.
l It is characterized by complete destruction of dentinal i. Acidogenic theory
tubules. ii. Proteolytic theory
l In this zone, the area of decomposition of dentin, iii. Proteolytic chelation theory
which occurs along the direction of dentinal tubules, is iv. Sucrose chelation theory
called “liquefaction foci of Miller”, and which occurs v. Autoimmune theory.
perpendicular to dentinal tubules is called “transverse
clefts”
I. ACIDOGENIC THEORY
l In the process, the entire dentinal structures become
destroyed and cavitation begins from dentinoenamel l This theory is also known as Miller’s chemicoparasitic
junction. theory.
Section | III Oral Pathology 523

l Acidogenic theory states that the process of dental car- l It synthesizes dextran from sucrose, which helps in
ies is a chemico-parasitic process consisting of two adhering the plaque bacteria as well as the acid on to the
stages: tooth surface.
a. Preliminary stage: Production of organic acid occurs l S. mutans has the ability to adhere and to grow on hard
as a result of fermentation of the carbohydrates by and smooth surfaces of the teeth.
the plaque bacteria. The decalcification of enamel l The actinomycosis group, e.g. Actinomycosis israelii,
which results in its total destruction and decalcifica- Actinomycosis viscosus, etc. are the important organ-
tion of dentin. isms to cause root caries. The Lactobacillus acidophilus
b. Later stage: Preliminary stage followed by dissolu- is important organism for the progress of dental caries.
tion of the softened residue.
Acids resulting in primary decalcification are produced
iii. Role of Acids
l

by the fermentation of dietary carbohydrates, i.e.


starches and sugar from the retaining centres of teeth. l During the process of caries formation, a large variety
l According to Miller, there are four important factors of acids are produced in the oral cavity due to the bacte-
which can influence the process of tooth destruction in rial fermentation of dietary carbohydrate.
dental caries: l These acids are lactic acid, aspartic acid, acetic acid, bu-
i. Dietary carbohydrates tyric acid, and glutamic acid. They can cause demineral-
ii. Microorganisms ization of enamel and dentin and cause the tooth decay.
iii. Acids
iv. Dental plaque.
iv. Role of Bacterial Plaque
Plaque is a thin, transparent film produced on tooth
i. Role of Carbohydrates l

surface and it consists of microorganisms suspended in


l Fermentable dietary carbohydrates play an important salivary mucin, also contains desquamated epithelial
role in the causation of caries, e.g. glucose, fructose and cells, leucocytes and food debris, etc.
sucrose. Among them, sucrose is more potent. l The dental plaque helps in initiation of dental caries by:
l These sugars are easily and rapidly fermented by cario- i. Harbouring the cariogenic bacteria on the tooth
genic bacteria in the oral cavity to produce acid at or surface.
near the tooth surface and causes dissolution of the hy- ii. Holding the acids on the tooth surface for long duration.
droxyapetite crystals of the enamel followed by the iii. By protecting the acids from getting neutralized by
dentin. buffering action of saliva.
l The cariogenicity of a dietary carbohydrate varies with
the frequency of ingestion, physical form and chemical Q. 8. Define dental caries. Describe the immunology of
composition, route of administration and presence of dental caries. Give a brief account of caries activity tests.
other food constituents. Ans.
l Risk of caries incidence increases greatly, if the dietary
sugar is solid, sticky and soft in nature which remains Dental caries has been defined as an irreversible progres-
adherent to the tooth surface for long time after taking sive microbial disease of the calcified tissues of the teeth,
the meal. characterized by the demineralization of the inorganic por-
l Following the ingestion of these sugars, the pH of the tion and distortion of the organic substances of the tooth,
plaque falls to 4.5 to 5 within 1 to 3 minutes and neu- which often leads to cavitations.
tralization occurs after 10 to 30 minutes.
The glucose, sucrose and fructose, etc. are rapidly de-
Immunology of Dental Caries
l

fused into the plaque due to their low molecular weight.


l This theory summarises that the interplay between l Salivary immunoglobulin IgA protects the tooth from oral
Bacteria 1 Sugars 1 Teeth n Organic acids n Caries. pathogenic bacteria by acting as a specific agglutinin.
l It binds and agglutinates specific oral bacteria, prevent-
ing their adherence to tooth, and thereby removing from
ii. Role of Microorganisms
the mouth.
l A large number of microorganisms play individual role l These immunoglobulins inactivate surface glycosyl-
in dental caries production and among them the most transferases, resulting in reduced synthesis of extracel-
important one is Streptococcus mutans. lular glucans which in turn reduces plaque formation.
l It readily ferments the dietary carbohydrate to produce l Antibodies also increase opsonization leading to phago-
acid, which causes tooth destruction. cytosis by lymphocytes and polymorphs.
524 Quick Review Series for BDS 3rd Year

Caries Activity Test IV. S. mutans dip slide method


A number of caries activity test have been evolved which l Salivary samples are classified based on estimates
are as follows: of S. mutans colonies on modified Mitis Salivarius
Agar.
I. Lactobacillus colony test: l Saliva is collected for 5 minutes following stimulation
l Most widely used microbiological test. and then poured over the agar medium.
l It is also called LB colony count and it measures the l Incubated for 48 hours.
number of aciduric bacteria in the patient’s saliva by l S. mutans are visible as blue colonies.
counting the number of colonies which appear on Ro- l Colony density is compared against a standard chart and
gosa medium. classified as 0 (negligible), 1 (,100, 000), 2 (100, 000 –
1000, 000), 3 (more 1000, 000)
Snyder test:
V. Buffer capacity test:
l This test measures the ability of salivary microorganisms
to produce organic acids from carbohydrate metabolism. l Evaluates the quantity of acid required to lower the
l Glucose agar media containing an indicator dye, i.e. pH level of saliva.
bromocresol green is useful. l pH meter or colour indicators are used to evaluate the
l The indicator dye changes from green to yellow in pH of saliva.
range of pH between 5.4 and 3.8. l Lower buffering capacity of saliva indicates higher caries
l Paraffin-stimulated saliva is added into the medium, risk.
change of the medium from blue –green at pH 4.7 – 5 to
VI. Enamel solubility test:
yellow at pH 4 is indicative of degree of caries activity.
l Measures the milligrams of enamel dissolved in 4 hours
II. Swab test: when the patient’s saliva is mixed with glucose and
l Same as Snyder test. powdered enamel.
l Involves sampling of the oral flora with help of an oral swab. l Enamel is dissolved by the acid formed.
l The swab is incubated. l Not suited for office procedures.
l Colour comparator used to measure the colour change
that occurs due to change in pH. VII. Salivary reductase test:
l pH 4.1 and below suggests marked caries activity; pH of l It measures the activity of reductase enzyme present in
4.5 to 4.6 indicates slightly active. salivary bacteria.
l Paraffin-stimulated saliva is collected in the plastic con-
III. Salivary S mutans level test: tainer and an indicator dye “Diazoresorcinol” is added
l Involves incubation of plaque samples on Mitis Salivarius to it which colours the saliva blue.
Agar (MSA). l The reductase enzyme liberated by the cariogenic bacte-
l The agar plates are incubated at 37° C for 48 hours. ria causes colour changes in the medium from blue to
l Levels of S. mutans higher than 105 indicative of unac- other colours, which indicates the caries “conductive-
ceptable cariogenic challenge. ness” of the patients.

SHORT ESSAYS
Q. 1. What is plaque? Describe its role in the pathogen- l In addition to proteins, plaque also contains carbohy-
esis of dental caries. drates and lipids. These constitute 25% of the dry
weight of plaque.
Ans.
l Inorganic components constitute 5-10% of dry weight
l Plaque is a soft, non-mineralised, bacterial deposit of plaque. Has high concentration of calcium and phos-
which forms on the teeth and dental prosthesis that are phate.
not adequately cleaned. l It is classified as supra- or subgingival plaque based on
the anatomical region of appearance.
Structure of Dental Plaque
Role of Bacterial Plaque in Dental Caries
l Plaque is composed of 80% water and 20% solids.
l About one-half of the dry weight of plaque is attributed l Dental plaque harbours caries causing microorganisms
to bacteria and salivary proteins. and keeps them in constant touch with the tooth surfaces.
Section | III Oral Pathology 525

l Plaque also contains carbohydrates and lipids that help iii. Systemic factors:
in adhesion of the bacteria and also act a source of fer- l Hereditary, pregnancy and lactation factors have been
mentable substrates. suggested as aetiological factors for dental caries.
l It acts as a barrier to salivary buffers which can neutralize
the acids formed in plaque. iv. Host factors:
l Certain cariogenic bacteria in plaque are capable of l Poor oral hygiene and improper brushing technique
storing polysaccharides that can act as a carbohydrate may lead to dental caries.
reserve to be used when the individual is consuming a
sugar-free diet. v. Immunological factors:
l The dental plaque, therefore, helps in the initiation of l The functional role of circulating antibodies as protec-
dental caries by helping in the following processes: tive agents against tooth decay has been demonstrated
l It harbors the cariogenic bacteria on the tooth in non-human primates.
surface.
Q. 3. Describe the pathogenesis of dental caries.
l It holds the acids on the tooth surface for long

duration. Ans.
l It protects the acids from getting neutralized by buff-
Several different factors play an important role in the
ering action of saliva.
occurrence of caries. These include the following:
l An important component of dental plaque is the ac-
a. Dietary factors:
quired pellicle which is a glycoprotein derived from the
l Carbohydrates, like monosaccharides, disaccharides
saliva and may serve as a nutrient for the microorgan-
or polysaccharides, have a major role in the occur-
isms of the plaque.
rence of dental caries.
Q. 2. Describe in brief aetiology of dental caries. b. Microorganisms:
l Cariogenic bacteria, e.g. Streptococcus mutans, Lac-

Ans. tobacillus and Actinomycoses that are present in the


mouth and accumulated within the dental plaque,
l Dental caries is a multifactorial disease in which there metabolize the carbohydrates to produce acids that
is an interplay of three primary factors: the host, micro- dissolve enamel, leading to formation of cavities.
flora and the substrate. In addition, a fourth factor, i.e. c. Systemic factors:
time, is also responsible. l Heredity, pregnancy and lactation have been sug-
l Therefore, caries formation requires: gested as aetiological factors for dental caries.
l A host, d. Host factors:
l Cariogenic flora and, l Poor oral hygiene and improper brushing technique
l A suitable substrate that must be present for a suffi- may lead to dental caries.
cient length of time. e. Immunological factors:
l The functional role of circulating antibodies as pro-
Several different factors play an important role in the
occurrence of caries. These include the following: tective agents against tooth decay has been demon-
strated in non-human primates.
i. Dietary factors: Some important features in the pathogenesis of caries
l Carbohydrates, like monosaccharides, disaccharides or are as follows:
polysaccharides, have a major role in the occurrence of l Dental caries is a multifactorial disease in which there
dental caries. is an interplay of three primary factors: the host, micro-
l The risk of caries is mainly determined by the type of flora and the substrate. In addition, a fourth factor, i.e.
carbohydrates consumed. time, is also responsible.
l Sticky solid carbohydrates are more prone to result in l Pits and fissures are the most commonly affected areas
caries than liquids. while the smooth surfaces are also frequently affected.
l Plaque is generally considered as the initiating factor of
ii. Microorganisms: dental caries.
l Cariogenic bacteria that are present in the mouth and l Dental plaque harbors caries causing microorganisms
accumulated within the dental plaque metabolize the and keeps them in constant touch with the tooth
carbohydrates to produce acids that dissolve enamel, surfaces.
leading to formation of cavities. l Plaque also contains carbohydrates and lipids that help
l Major ones include acidogenic Streptococcus mutans, in adhesion of the bacteria and also act a source of fer-
Lactobacillus and Actinomycoses. mentable substrates.
526 Quick Review Series for BDS 3rd Year

l It acts as a barrier to salivary buffers which can neutralize l Occurs singly or in pairs or in chains and can be cul-
the acids formed in plaque. tured on agar, in broth, in litmus, milk, etc.
l Certain cariogenic bacteria in plaque are capable of l Can undergo both fermentative and oxidative metabo-
storing polysaccharides that can act as a carbohydrate lism.
reserve to be used when the individual is consuming a l Produce lactic acid and grow in the relatively acidic
sugar-free diet. environment.
l Pioneer bacteria or the first binders are the first group of
bacteria that adhere to the dental plaque. Q. 5. Acidogenic theory of dental caries.
l Studies have shown that the pioneer bacteria are strep- Ans.
tococci, neisseriae, actinomyces and capnocytophaga.
l Other bacteria coadhere to the pioneer bacteria to in- l Acidogenic theory is also known as Miller’s chemico-
crease the complexity of the association. parasitic theory.
l Dental caries results from the metabolism of this asso- l This theory states that the process of dental caries is a
ciation which attack the tooth enamel. chemicoparasitic process consisting of two stages:
a. Preliminary stage: Production of organic acid occurs
as a result of fermentation of the carbohydrates by
Mode of Cavity Formation the plaque bacteria. The decalcification of enamel
l Streptococcus mutans is the main causative organism of which results in its total destruction and decalcifica-
smooth surface caries of enamel. tion of dentin.
l This organism ferments sugars to lactic acid which dis- b. Later stage: Preliminary stage followed by dissolu-
solves the enamel matrix. tion of the softened residue.
l The quick increase in the number of streptococci alters l Acids resulting in primary decalcification is produced
the pH, nutrients and oxidation-reduction potentials in by the fermentation of dietary carbohydrates, i.e.
the bacterial plaque which allows growth of other or- starches and sugar from the retaining centres of teeth.
ganisms. l According to Miller, there are four important factors,
l The acid formation continues unabated leading to fur- which can influence the process of tooth destruction in
ther destruction of the enamel and dentin resulting in dental caries:
cavitations. i. Dietary carbohydrates
ii. Microorganisms
Q. 4. Describe the microbiological aspects of dental iii. Acids
caries. iv. Dental plaque.
Ans.
i. Role of carbohydrates:
l Cariogenic bacteria that are present in the mouth and l Fermentable dietary carbohydrates play an important
accumulated within the dental plaque metabolize the role in the causation of caries, e.g. glucose, fructose and
carbohydrates to produce acids that dissolve enamel, sucrose.
leading to formation of cavities. l These sugars are easily and rapidly fermented by cario-
l Major organisms include Streptococcus mutans, Lacto- genic bacteria in the oral cavity to produce acid at or
bacillus and Actinomycoses. near the tooth surface and causes dissolution of the hy-
droxyapetite crystals of the enamel followed by the
Streptococcus mutans
dentin.
l Streptococcus mutans and Streptococcus salivarius l Following the ingestion of these sugars, the pH of the
cause enamel caries (i.e. initial decalcification of plaque falls to 4.5 to 5 within 1 to 3 minutes and neu-
enamel). tralization occurs after 10 to 30 minutes.
l These organisms ferment many sugars and produce acid l This theory summarises that the interplay between
which causes large drop in the pH. Bacteria 1 Sugars 1 Teeth n Organic acids n Caries.
l Streptococcus mutans is a-haemolytic streptococci
which ferment mannitol and sorbitol, but do not hydro- ii. Role of microorganisms:
lyze arginine. l A large number of microorganisms play individual role
in dental caries production and among them the most
Lactobacillus—Characteristics important one is Streptococcus mutans.
l Aerobic and anaerobic bacteria. l It synthesizes dextran from sucrose, which helps in ad-
l Non-sporing, long and thin, non-moving, Gram-positive hering the plaque bacteria as well as the acid on to the
organisms. tooth surface.
Section | III Oral Pathology 527

l The Actinomycosis group, e.g. Actinomycosis israelii, B. Substance which interfere with carbohydrate degrada-
Actinomycosis viscosus, etc. are the important organ- tion through enzymatic alteration:
isms to cause root caries. i. Vitamin K: It prevents acid formation in mixture of
l The Lactobacillus acidophilus is important organism glucose into saliva.
for the progress of dental caries. ii. Sarcoside.
C. Substances which interfere with bacterial growth and
iii. Role of acids: metabolism:
l During the process of caries formation, a large variety i. Urea and ammonium compounds
of acids are produced in the oral cavity due to the bacte- ii. Chlorophyll
rial fermentation of dietary carbohydrate. iii. Nitrofurans
l These acids are lactic acid, aspartic acid, acetic acid, bu- iv. Penicillin.
tyric acid, and glutamic acid. They can cause demineral-
ization of enamel and dentin and causes the tooth decay. II. Nutritive measures:
l Groups of patients whose diet is high in fat, low in
iv. Role of bacterial plaque: carbohydrate and practically free from sugar have low
l Plaque is a thin, transparent film produced on tooth caries activity.
surface and it consists of microorganisms suspended in l Phosphate diet causes significant reduction in indication
salivary mucin, also contains desquamated epithelial of caries.
cells, leucocytes and food debris, etc.
l The dental plaque helps in initiation of dental caries by: III. Mechanical measures:
i. Harbouring the cariogenic bacteria on the tooth sur- a) Toothbrushing: Toothbrushing reduces the number of
face. oral microorganisms.
ii. Holding the acids on the tooth surface for long dura- b) Mouth rinsing: The use of mouthwash looses the food
tion. debris from the teeth and prevents the caries.
iii. Protecting the acids from getting neutralized by c) Dental floss: It removes plaque from an area of gingiva
buffering action of saliva. to contact area on proximal surfaces of teeth, an area
impossible to reach with the toothbrush.
Q. 6. Explain the preventive measures taken for control- d) Detergent foods: Fibrous foods prevent lodging of food
ling caries. in the pit and fissures of the teeth and in addition acts as
Ans. detergent.
e) Pit and fissure sealants: The pits and fissures of oc-
The preventive and controlling methods of caries may be clusal surface are most difficult areas on teeth to
classified into three general types as follows: keep clean, so pits and fissure sealants are generally
I. Chemical measures used.
II. Nutritional measures
III. Mechanical measures. Q. 7. Describe briefly the clinical types of caries.

I. Chemical measures: Ans.


A. Substances which alter the tooth surfaces or tooth
structures.
Classification of Caries
i. Fluorine
l It has cariostatic activity and makes the teeth l Dental caries can be classified in a number of ways
more resistant to acid attack into oral cavity. l Three basic factors, morphology, dynamics and chro-
l It decreases microbial acid production and en- nology, are considered for classification. Based on these
hances the remineralization of underlying enamel. factors, caries can be classified as follows.
l It is given in the form of:

a. Communal water fluoridation a. Based on the morphology or anatomical site:


b. Fluoride containing dentifrices i. Pit and fissure caries
c. Fluoride mouth rinses ii. Smooth surface caries
d. Dietary fluoride supplements.
ii. Bisbiguanides chlorhexidine and alexidine. They b. Based on the dynamics with regard to rate of
are anti-plaque agents. caries progression:
iii. Silver nitrate i. Acute caries
iv. Zinc chloride and potassium ferrocyanide. ii. Chronic caries
528 Quick Review Series for BDS 3rd Year

c. Based on the nature of attack: vi. Chronic caries:


i. Primary (virgin) caries l Slow progress and so there is more of secondary dentin
ii. Secondary or recurrent caries formation.
l Cavity is wide, with stained enamel with more destruc-
d. Based on chronology, it can also be classified as: tion due to secondary enamel caries.
l Infancy (nursing bottle) caries l Less painful because of blockage of dentinal tubules
l Adolescent caries and other pulp reactions to the lesion.
l Occurs in patients above 30 years of age.

Clinical Features of Dentinal Caries vii. Arrested caries:


i. Pit and fissure caries: l This is an area of remineralization of the carious lesion
l These areas are favourable for caries development due to change in pH.
because of their anatomical nature. l It is seen as discoloured (brown or black spot) area
l Streptococcus mutans, S. sanguis and Lactobacillus which is more resistant to subsequent caries attack.
sp. play an important role as an aetiological factor. l The cavities become self-cleansing and with no stagna-
l The lesion begins beneath a bacterial plaque with decal- tion, caries is arrested.
cification of the enamel. l The secondary dentine forms below the cavity.
l The lesion is characteristically triangular or cone- l This is seen on occlusal surface as brown-stained, pol-
shaped with its apex at the outer surface and its base ished smooth area (eburnated dentin) which is hard and
towards the dentinoenamel junction. shiny.
l Thus the cavities are larger in size, and more undermin-
viii. Primary caries:
ing of enamel will be found.
l Caries which occurs for the first time on any surface of
ii. Smooth surface caries: tooth.
l Early sign in these areas is that of smooth, chalky white
ix. Recurrent caries
area of decalcification.
l The lesion in these areas is cone-shaped with the apex l Caries which recurs near the margin of a restoration due
towards the dentinoenamel junction and the base towards to the leaky margins of a restoration.
the surface of the tooth. Q. 8. Rampant caries.
iii. Cervical caries: Ans.
l This occurs as chalky, crescent-shaped area of decalcifi- l Carious condition characterized by sudden, rapid and
cation at the gingival margin of labial and buccal surfaces uncontrollable destruction of teeth.
which subsequently forms a cavity. l The surfaces that are generally caries-free are involved.
l The cavities are saucer-shaped and wider. l Common areas affected include the proximal and cervi-
l This occurs due to poor oral hygiene. cal surfaces of mandibular incisors.
l Caries increment of 10 or more new lesions over a pe-
iv. Root caries:
riod of 1 year is the characteristic feature of rampant
l This is caries of cementum and the underlying dentin, caries.
and occurs in later life (40 - 50) due to recession of the l Affects primary dentition and permanent dentition.
gingival margin. l Dietary factors, oral microbial flora and physiological
l The lesion appears as shallow saucer-shaped ill-defined factors that affect saliva have a significant role in occur-
area, which is discoloured and quite soft. rence of rampant caries.
l The lesion starts at the cementoenamel junction and l Nursing bottle caries is a form of rampant caries.
spreads laterally rather than in depth.
Q. 9. Classify dental caries. Write briefly about nursing
v. Acute caries: bottle syndrome.
l The lesion progresses rapidly and involves pulp quickly.
Ans.
l Seen in children and adolescents.
l Cavity is deep in spite of small opening in the enamel. l Dental caries can be classified in a number of ways.
l Enamel is widely undermined, dentin is soft and pale. l Three basic factors, morphology, dynamics and chro-
l Painful, condition where pulp is soon exposed and nology, are considered for classification.
infected. l Based on these factors, caries can be classified as
l Secondary dentin is little or absent. follows:
Section | III Oral Pathology 529

a. Based on the morphology or anatomical site: Advanced enamel caries


l Pit and fissure caries l It presents several zones in the tissues, out of which four
l Smooth surface caries zones are clearly visible, starting from the inner advanc-
ing front of the lesion, the zones are:
b. Based on the dynamics with regard to rate of i. Zone 1: Translucent zone
caries progression: ii. Zone 2: Dark zone
l Acute caries iii. Zone 3: Body of the lesion
l Chronic caries iv. Zone 4: Surface zone

c. Based on the nature of attack:


Zone 1: Translucent zone:
l Primary (virgin) caries
l It is the deepest zone which lies at the advancing front
l Secondary or recurrent caries
of the enamel lesion.
d. Based on chronology, it can also be classified as: l It is the first recognizable zone of alteration from normal
enamel.
l Infancy (nursing bottle) caries
l This zone is more porous than normal enamel and the
l Adolescent caries
pores are larger than the normal enamel.
Nursing bottle syndrome: l The pores are located at prism boundaries and other
junctional sites. The pore volume is 1%.
l It is also called as nursing caries or baby bottle syn-
l This zone appears structureless and it contains more
drome and bottle mouth syndrome.
fluoride than normal enamel.
l This is a type of rampant caries affecting deciduous
dentition.
l It occurs due to prolonged use or overnight use of: Zone 2: Dark zone:
a. A nursing bottle containing milk or fluids like formu- l This zone is often referred to as ‘positive zone’ and it is
lated fruit juice or sweetened water. a dark zone and is located just superficial to the translu-
b. Breastfeeding cent zone.
c. Sugar or honey sweetened pacifiers. l Its dark appearance is due to the excessive demineral-
l First the four maxillary anteriors are affected, followed ization of the enamel.
by 1st molar and then the cuspids are involved, if the l The zone is narrower in rapidly advancing caries and
habit is prolonged. wider in slowly advancing lesion.
l The absence of caries in mandibular incisors distin- l The zone contains 2 to 4% pore volume. The pores are
guishes the disease from ordinary rampant caries. larger than normal enamel, but smaller than those of
l The carious process in affected teeth may be so severe translucent zone.
that only root stumps remain. l This zone reveals some degree of remineralisation of
l The mandibular incisors usually escape because they carious lesion.
are covered and protected by the tongue.
Q. 10. Histopathology of caries in enamel. Zone 3: Body of lesion:
Ans. l The zone is situated between the dark zone and the sur-
face layer of enamel.
l It represents the area of greatest demineralization and
Histopathological Features of Caries in the pore volume is 5 to 25%.
Enamel l This zone contains apatite crystals larger than those of
A wide range of histopathological changes may be noted the normal enamel.
based on the severity of the enamel caries. l The large crystals results from the reprecipitation of
minerals dissolved from the deeper zone.
Early enamel caries
l There will be loss of interprismatic or inter-rod substances Zone 4: Surface zone:
with increase in prominence of these enamel rods. l The surface zone when examined by the polarizing light
l Dark line often appears at right angles of the enamel appears relatively unaffected.
rods, suggesting segments. l It may be due to the surface remineralization by the
l Accentuation of the incremental striae of Retzius often salivary mineral ions.
occurs. l Pore volume is less than 5%.
l Accentuation of striae of Retzius and accentuation of l The surface zone is usually demineralised when the
perikymata are often the early microscopic changes. lesion has penetrated into the dentin.
530 Quick Review Series for BDS 3rd Year

Q. 11. Histopathology of caries in dentin. “liquefaction foci of Miller”, and which occurs perpen-
dicular to dentinal tubules is called “transverse clefts”
Ans.
l In the process, the entire dentinal structures become
destroyed and cavitation begins from dentinoenamel
Histopathological Features of Dentinal junction.
Caries
Q. 12. Cemental caries.
l Once the caries reaches the dentin, the lesion rapidly
spreads laterally along the dentinoenamel junctions. Ans.
l Cemental caries is also referred to as root caries.
Zones of Dentinal Caries l Caries of cementum and the underlying dentin occurs in
later life (40 – 50 years) due to recession of the gingival
As the carious lesion progresses, various zones of carious margin.
dentin may be distinguished which tend to assume a trian- l The lesion appears as shallow saucer-shaped ill-defined
gular shape with apex towards pulp and the base towards area, which is discoloured and quite soft.
the enamel. The dentinal caries histologically presents 5 l The lesion starts at the cementoenamel junction and
zones as follows: spreads laterally rather than in depth.
The plaque shows gram-positive filamentous actinomy-
Zone 1: Zone of fatty degeneration of Tome’s l

ces as predominant bacteria along with other types of


fibres:
bacteria.
l This zone represents the inner most layer of the carious l Organisms invade along the direction of the calcified
dentin, here the dentinal tubules appear normal. Sharpey’s fibres or between the bundles of fibres.
l There is evidence of fatty degeneration of the Tome’s l Ultimately, cementum becomes split-up and is de-
fibres. stroyed by decalcification and proteolysis.
l No crystals and bacteria in the lumen of the tubules. l Xerostomia (decreased salivary flow) increases the inci-
dence of root surface caries (e.g. following irradiation
Zone 2: Zone of dentinal sclerosis: therapy).
l This is the zone of dentinal sclerosis and is character- l Therefore, dietary control and strict oral hygiene and
ized by the deposition of very fine crystal structures use of topical fluorides are important to prevent this
calcium salts within the dentinal tubules. type of caries.
l Superficial layer shows area of demineralization and
damage of the odontoblastic processes. Q. 13. Microbiology of dental caries.
l No bacteria in the tubules and the dentin is capable of Ans.
remineralization.
l Cariogenic bacteria that are present in the mouth and
Zone 3: Zone of decalcification of dentin accumulated within the dental plaque metabolize the
l This zone is a narrow zone, preceding bacterial invasion carbohydrates to produce acids that dissolve enamel,
and appears transparent because of decalcification of leading to formation of cavities.
dentin. l Major ones include acidogenic Streptococcus mutans,
l No bacteria in tubules. Lactobacillus and Actinomycoses.
l This zone is capable of self-repair and remineralization.
Streptococcus mutans
Zone 4: Zone of bacterial invasion of decalcified l Streptococcus mutans and Streptococcus salivarius
but intact dentin: cause enamel caries (i.e. initial decalcification of
l This zone is marked by widening and distortion of den- enamel).
tinal tubules, which are packed with microorganisms. l These organisms ferment many sugars and produce acid
l There is very little amount of minerals in dentin. which causes large drop in the pH.
l Denaturation of collagen fibres also takes place. l Streptococcus mutans is a-haemolytic streptococci
l The zone cannot undergo self-repair or remineralization. which ferment mannitol and sorbitol, but do not hydro-
lyze arginine.
Zone 5: Zone of decomposed dentin:
l This is the outermost zone of the carious dentin and is Lactobacillus
characterized by complete destruction of dentinal tubules. l Lactobacilli are aerobic and anaerobic bacteria.
l In this zone, the area of decomposition of dentin, which l Non-sporing, long and thin, non-moving, Gram-positive
occurs along the direction of dentinal tubules, is called organisms.
Section | III Oral Pathology 531

l Occurs singly or in pairs or in chains and can be cultured l Early sign in these areas is that of smooth, chalky white
on agar, in broth, in litmus, milk, etc. area of decalcification.
l Can undergo both fermentative and oxidative metabo- l Another feature in these lesions is prominence of
lism. striae of Retzius as they cross the carious lesion,
l Produce lactic acid and grow in the relatively acidic and it is due to loss of minerals which by contrast
environment. make the organic structure more prominent under the
microscope.
Q. 14. Smooth surface caries. l The lesion in these areas is cone-shaped with the apex
towards the dentinoenamel junction and the base
Ans.
towards the surface of the tooth (opposite to that of pit
l Smooth surface caries refers to the caries of the proxi- and fissure caries).
mal surfaces of the teeth or caries on the gingival third l The lesion has:
of buccal and lingual surfaces of the teeth. l Zone of initial decalcification
l It is generally preceded by formation of microbial l Zone of advanced decalcification
plaque. l Zone of complete decalcification.

SHORT NOTES
Q. 1. Lactobacillus. Q. 4. Cariogenic microorganisms.
Ans. Ans.
l Lactobacilli are aerobic and anaerobic bacteria. l Streptococcus mutans is the main causative organism of
l They are responsible for progression of caries. smooth surface caries of enamel.
l Non-sporing, long and thin, non-moving, Gram-positive l Both Streptococcus mutans and lactobacilli are the
organisms. causative organisms of pit and fissure caries.
l Occurs singly or in pairs or in chains and can be cultured l Lactobacilli are the main causative organisms of den-
on agar, in broth, in litmus, milk, etc. tinal caries.
l Can undergo both fermentative and oxidative me- l The organisms involved in root surface caries are:
tabolism. Actinomyces, Nocardia and Streptococcus mutans.
l Produce lactic acid and grow in the relatively acidic
Q. 5. Name the organisms causing cervical caries.
environment.
Ans.
Q. 2. Streptococci.
The following organisms are involved in cervical caries:
Ans. l Actinomyces
l Streptococcus mutans and Streptococcus salivarius l Nocardia
cause enamel caries (i.e. initial decalcification of l Streptococcus mutans
enamel). Q. 6. Dental plaque.
l These organisms ferment many sugars and produce acid
which causes large drop in the pH. Ans.
l Streptococcus mutans is a-haemolytic streptococci l Plaque is a soft, non-mineralised, bacterial deposit
which ferment mannitol and sorbitol, but do not hydro- which forms on the teeth and dental prosthesis that are
lyze arginine. not adequately cleaned.
l It is composed of 80% water and 20% solids.
Q. 3. Pioneer bacteria.
l Classified as supra- or subgingival plaque based on the
Ans. anatomical region of appearance.
l Dental plaque harbors caries causing microorganisms
l Pioneer bacteria or the first binders are the first group of
and keeps them in constant touch with the tooth surfaces.
bacteria that adhere to the dental plaque.
l Studies have shown that the pioneer bacteria are strep- Q. 7. Define caries.
tococci, neisseriae, actinomyces and capnocytophaga.
Ans.
l Other bacteria coadhere to the pioneer bacteria to in-
crease the complexity of the association. Dental caries has been defined as an irreversible progres-
l Dental caries results from the metabolism of this asso- sive microbial disease of the calcified tissues of the teeth,
ciation which attack the tooth enamel. characterized by the demineralization of the inorganic
532 Quick Review Series for BDS 3rd Year

portion and distortion of the organic substances of the iii. Zone of the body of the lesion: This is the largest
tooth, which often leads to cavitations. portion of the initial carious lesion and contains in-
vading bacteria.
Q. 8. Enamel caries.
iv. Surface zone: This zone though intact, is slightly
Ans demineralized which appears as pitting.
l Enamel caries occurs on pit and fissure, smooth surface Q. 12. Smooth surface caries.
and root.
Ans.
l It frequently affects the pits and fissures with high steep
wall and narrow bases. l Refers to the caries of the proximal surfaces of the teeth
l It appears brown or black and will feel slightly soft and or caries on the gingival third of buccal and lingual sur-
catch a fine explorer point. faces of the teeth.
l The enamel directly bordering the pit and fissure may l It is generally preceded by formation of microbial
appear opaque, bluish white and is undermined. plaque.
l Early sign in these areas is that of smooth, chalky white
Q. 9. Dentinal caries.
area of decalcification.
Ans. l The lesion has:
l Zone of initial decalcification
l Destruction of dentin is by decalcification and by prote-
l Zone of advanced decalcification
olysis of collagenous matrix.
l Zone of complete decalcification.
l Decalicification of dentin alone does not cause disinte-
gration (but softens) because protein material forming Q. 13. Arrested caries.
up to 25% of weight of the tissue, remains intact until
Ans
destroyed by the bacterial enzymes.
l Once the caries reaches the dentin, the lesion rapidly l This is an area of remineralization of the carious lesion
spreads laterally along the dentinoenamel junctions. due to change in pH.
l It is seen as discoloured (brown or black spot) area
Q. 10. Zones in dentinal caries.
which is more resistant to subsequent caries attack.
Ans. l The cavities become self-cleansing and with no stagna-
tion, caries is arrested.
The dentinal caries histologically presents 5 zones as fol-
l The secondary dentin forms below the cavity.
lows:
l This is seen on occlusal surface as brown-stained, polished
Zone 1: Zone of fatty degeneration of Tome’s fibres
smooth area (eburnated dentin) which is hard and shiny.
Zone 2: Zone of dentinal sclerosis
Zone 3: Zone of decalcification of dentin Q. 14. Nursing bottle caries.
Zone 4: Zone of bacterial invasion of decalcified but intact
Ans
dentin
Zone 5: Zone of decomposed dentin l Nursing bottle caries is also called as nursing caries or
baby bottle syndrome or bottle mouth syndrome.
Q. 11. Zones in enamel caries.
l It occurs due to prolonged use or overnight use of:
Ans. a. A nursing bottle containing milk or fluids like for-
mulated fruit juice or sweetened water.
Advanced enamel caries presents several zones in the
b. Breastfeeding
tissues, out of which four zones are clearly visible, start-
c. Sugar or honey sweetened pacifiers.
ing from the inner advancing front of the lesion, the
l First the four maxillary anteriors are affected, followed
zones are:
by 1st molar and then the cuspids are involved, if the
i. Zone 1: Translucent zone
habit is prolonged.
ii. Zone 2: Dark zone
l The absence of caries in mandibular incisors distin-
iii. Zone 3: Body of the lesion
guishes the disease from ordinary rampant caries.
iv. Zone 4: Surface zone
i. Translucent zone: This is the innermost (deepest) Q. 15. Caries susceptibility.
zone, and this represents the advancing area of the
Ans
carious lesion.
ii. Dark zone: This is the next zone towards the surface, l Caries susceptibility refers to the inherent tendency of
over the translucent zone. It is dark because it does the host and target tissue, the tooth to be afflicted by the
not transmit light. caries process.
Section | III Oral Pathology 533

l Microbiological test helps to measure the number of l The secondary dentine forms below the cavity.
Streptococcus mutans and Lactobacillus acidophilus. l This is seen on occlusal surface as brown-stained,
l Two samples of paraffin-stimulated saliva is collected polished smooth area (eburnated dentin) which is hard
and diluted 10 times and each is cultivated in two differ- and shiny.
ent special media.
Q. 19. Liquefaction foci of Miller.
l After incubation, the numbers of colonies that develop in
two separate media are counted and then are multiplied Ans
by 10 to estimate the number of bacteria in 1 ml of saliva.
l If count is more than 10, 00, 000 S. mutans and more l Liquefaction foci of Miller are noted in advanced caries
than 1, 00, 000 L. acidophilus, then the caries suscepti- of dentin and are described by Miller.
bility of individual is very high. l They are formed by focal coalescence and breakdown
of a few dentinal tubules.
Q. 16. Snyder’s tests for caries susceptibility. l They occur along the direction of dentinal tubules.
Ans. Q. 20. Acidogenic theory.
l Snyder’s test measures the ability of salivary microor- Ans.
ganisms to produce organic acids from carbohydrate
l Acidogenic theory is also known as Miller’s chemico-
metabolism.
parasitic theory.
l Glucose agar media containing an indicator dye, i.e.
l It proposes that acid formed due to the fermentation of
bromocresol green is useful.
dietary carbohydrates by oral bacteria leads to progres-
l The indicator dye changes from green to yellow in
sive decalcification of the tooth.
range of pH between 5.4 and 3.8.
l Acidogenic theory states that the process of dental
l Paraffin-stimulated saliva is added into the medium,
caries involves two stages:
change of the medium from green to yellow is indicative
i. Preliminary stage – involves decalcification of the
of degree of caries activity.
enamel that results in its total destruction and the
Q. 17. Role of saliva in dental caries. decalcification of dentin.
ii. Subsequent stage – dissolution of the softened
Ans.
residue.
Saliva has a protective role in preventing dental caries. l According to Miller, there are four important factors,
l Salivary flow and its flushing effect clear the tooth which can influence the process of tooth destruction in
surfaces. dental caries:
l Saliva with its wide variety of substances acts as the i. Dietary carbohydrates
host’s main defense against dental caries. ii. Microorganisms
l The defense factors of saliva can be divided into: iii. Acids
a. Active defense factors – such as lysozyme, lactoper- iv. Dental plaque.
oxidase, lactoferrin and agglutinins
b. Passive defensive factors or salivary buffers - Q. 21. Rampant caries.
bicarbonate and carbonate system, immunoglobulins. Ans.
Q. 18. What is arrested caries? l Rampant caries is the carious condition characterized
by sudden, rapid and uncontrollable destruction of
Ans
teeth.
l Arrested caries is an area of remineralization of the l Common areas affected include the proximal and cervi-
carious lesion due to change in pH. cal surfaces of mandibular incisors.
l It is seen as discoloured (brown or black spot) area l Caries increment of 10 or more new lesions over a
which is more resistant to subsequent caries attack. period of 1 year is the characteristic feature of rampant
l The cavities become self-cleansing and with no stagna- caries.
tion, caries is arrested. l Nursing bottle caries is a form of rampant caries.
534 Quick Review Series for BDS 3rd Year

Topic 10

Diseases of the Pulp and Periapical Tissues

LONG ESSAYS
Q. 1. Define and classify pulpitis, write in detail about Clinical Features
the pathogenesis, clinical features and histopathological
l Acute pulpitis usually occurs in a tooth with large cari-
features of acute pulpitis.
ous lesion or a defective restoration around which there
Ans. has been recurrent caries.
l It may be either sequela of focal reversible pulpitis or
l Pulpitis is the inflammation of dental pulp resulting
acute exacerbation of chronic pulpitis.
from untreated caries, trauma, or multiple restora-
l The tooth is sensitive to cold and hot stimuli.
tions.
l Thermal changes also increase the intensity of pain.
l Pulpitis is classified in variety of ways as follows:
l Application of hot or cold stimuli causes an increase
The simplest division is into:
in intensity to pain and such pain persists for longer
i. Acute pulpitis and
duration even after the stimuli is removed.
ii. Chronic pulpitis
l The intensity of pain increases during the sleep, i.e.
l Both acute and chronic pulpitis are further classified in
supine position and occurs due to increase in local
several different ways:
blood pressure in head and neck region.
A. Depending on extent of involvement of pulp:
l Acute pulpitis helps in spread of inflammation through-
a. Partial pulpitis or focal pulpitis
out pulp with subsequent necrosis.
b. Total or generalized pulpitis
l Acute pulpitis is often associated with microabscess
B. Based on the presence or absence of a direct
formation in pulp along with liquefaction degeneration.
communication between dental pulp and oral
l Pain subsides when drainage is established or when
environment:
pulp undergoes complete necrosis.
a. Open pulpitis (pulpitis aperta)
l The tooth is non-tender to percussion unless the pulpal
b. Closed pulpitis (pulpitis clausa)
inflammation has spread beyond the root apex into peri-
apical region.
ACUTE PULPITIS l When intrapulpal pressure becomes very high during
acute inflammation it causes collapse of apical blood
Acute pulpitis is an irreversible condition characterized by
vessels. This is known as “pulp strangulation”.
acute, intense inflammatory reaction in pulp tissue.
Acute inflammation of dental pulp is a frequent immediate
Histopathology
sequela of focal reversible pulpitis.
l Initially, severe oedema in the pulp with vasodilatation
takes place.
Aetiology l Now margination and migration of WBC (neutrophils)
l Pulp exposure due to faulty cavity preparation. and extravasation of RBCs take place.
l Caries progressing beyond the dentinal barriers and l Moderate to dense infiltration of polymorphonuclear
reaching the pulp. leucocytes .
l Recurrent caries around the pre-existing restoration l Many microabscess formations in pulp characterized by
l Chemical irritation to pulp the area of liquefaction degeneration in pulp being sur-
l Cracked tooth syndrome rounded by dense band of neutrophils and microorgan-
l Metallic restoration in a tooth without proper thermal isms.
insulation. l There may be complete liquefaction and necrosis of
l Blow to tooth with subsequent damage to pulp. pulp with total destruction of odontoblastic cell layer.
l Galvanic current produced due to dissimilar metallic This is known as acute suppurative pulpitis.
restoration may transmitted to pulp and causing l Death of pulp is due to tissue dehydration. This is
pulpitis. known as “dry gangrene of pulp”.
Section | III Oral Pathology 535

Management l Jaw swelling is common features but mobility of teeth


and sinus tract formation are rare.
l Early stages: Pulpotomy can be done in which coronal l Anaesthesia and paraesthesia of lip is uncommon.
portion of the pulp is removed and placement of cal- l Regional lymphadenopathy is common.
cium hydroxide with proper restoration is done. l There is thickened, woodened feeling of bone and slow
l RCT (pulpectomy) is done, if there is large area of increase in jaw size.
pulpal destruction.
Q. 2. Describe the pathogenesis, histopathology, radio-
Histopathology
graphic appearance and clinical features of chronic os-
teomyelitis of the jaw. l Chronic inflammatory reaction of bone with accumula-
tion of exudate and pus within medullary spaces.
Ans.
l The lymphocytes, macrophages and plasma cells pre-
Chronic osteomyelitis is the persistent abscess of the bone dominate among the inflammatory cells.
characterized by the complex inflammatory process includ- l Osteoblastic and osteoclastic cavities occur partially
ing necrosis of mineralized and marrow tissues, suppura- with formation of irregular bony trabeculae having re-
tion, resorption, sclerosis and hyperplasia. versal lines.
l Sequestrum may develop in later stages of the disease.
Colonies of bacteria are also seen within the inflamed
Pathogenesis
l

tissue.

Infection of bone marrow from infected pulp


Radiological Findings
l Moth eaten appearance with irregular enlarged radiolu-
Extension of infection into cancellous bone cent areas separated by islands of normal appearing
bone.
l The irregular calcified areas separated from normal
Thrombus formation in nutrient vessels of the living bone bone are present known as sequestra.
l External resorption of roots of the teeth and lamina dura
becomes less apparent.
Death of cancellous bony trabeculae with l Fistulous tract may appear.
formation of sequestrum
Q. 3. Define osteomyelitis. Describe the aetiology, pa-
thology and clinical features of acute suppurative osteo-
Spread of infection via Volkmann’s canal in cortical plates myelitis of mandible in adults.

Ans.
Periostitis l Osteomyelitis is defined as an inflammatory condition
of the bone that begins as an infection of medullary cav-
ity and the haversian system and extends to involve the
Multiple sinus tract formation periosteum of the affected area.

Necrosis of cortical bone ACUTE SUPPURATIVE OSTEOMYELITIS


Acute suppurative osteomyelitis is serious sequela of peri-
apical infection, resulting into diffuse spread of infection
Discharge of the pus form involucrum through throughout the medullary space.
sinuses known as clocae

Causative Microorganisms
Clinical Features
l Staphylococcus aureus
l The molar area of mandible is more frequently affected. l b-haemolytic streptococci
l Pain is usually mild and insidious and is not related to l Haemophilus influenzae
the severity of the disease. l Pseudomonas species
536 Quick Review Series for BDS 3rd Year

Predisposing Factors Treatment


l Debilitating conditions: Diabetes, anaemia, leukaemia l Drainage of pus is must along with antibiotic therapy.
l Avitaminosis and malnutrition. l “Sequestrectomy” or “saucerization” is done.
l Syphilis, typhoid.
Q. 4. Describe the aetiology, histopathological, clinical
and radiological features of periapical granuloma and
Aetiology mention its consequences.
l Direct spread of infection from dental pulp into the Ans.
mandible.
l Spread of infection in the mandible from presenting Periapical granuloma or chronic apical periodontitis or dental
suppurative odontogenic infections. granuloma is a low grade infection with a localized mass of
l Spread of infection following removable of tooth with- granulation tissue around the root apex of non-vital tooth
out proper asepsis and antibiotic coverage. which develops in relation to infection and inflammation.
l Compound fracture of mandible with exposure of bone
outside the mucosa. Aetiology
l Post-radiation secondary infection.
l Extension of pulpal inflammation
Clinical Features l Acute trauma due to blows on the tooth.
l Spread of periodontal infection in the root apex.
l Usually occurs after the age of 50 years and males are l Perforation of root apex into endodontic therapy.
more commonly affected. l Orthodontic tooth movements with excessive uncon-
l The mandible is most commonly involved and lesions trolled force.
are diffused in nature.
l Severe throbbing deep seated pain and swelling are the
initial complaints. Clinical Features
l Usually has trismus and paraesthesia of lip. l The affected tooth is usually non-vital.
l Regional lymphadenopathy l The tooth may be slightly tender or asymptomatic and
l There is loosening and soreness of the regional teeth produce a dull sound.
with difficulty in mastication. l The tooth involved produces sensitivity to percussion
l Multiple intraoral and extraoral pus discharging sinuses which occurs due to oedema, hyperaemia and inflam-
develop. mation of apical periodontal ligament.
l Discharge of pus is seen from gingival cervice of the l Mild pain and discomfort in tooth during chewing solid
affected teeth. foods.
l Fetid oral odour. l The involved tooth feels slightly elongated from the
l Patient is slightly febrile and general symptoms include socket.
fever, malaise, anorexia and vomiting. l If pus has formed, a small reddish swelling may be
found on the buccal gum or a sinus may be present.
Histological Features l Granuloma remains attached to the extracted tooth.
l Enlargement and tenderness of regional lymph nodes.
l The bone marrow undergoes liquefaction and purulent
l Fever and malaise occur.
exudates occupy the marrow space.
l A large number of acute inflammatory cell infiltrations
are present which show PMNs with occasional presence Histopathological Features
of lymphocytes and plasma cells.
l The lesion appears as granulation tissue mass consisting
l Some areas of affected bone undergo necrosis with
of proliferating fibroblasts, endothelial cells and numer-
degeneration of osteoblasts and osteocytes and, there-
ous immature blood capillaries.
fore, results in development of sequestrum.
l Chronic inflammatory cells, e.g. macrophages, lympho-
l When acute phase of infection subsides then this
cytes and plasma cells are present in the lesion.
sequestrum is surrounded by newly formed living
l There is presence of epithelial islands, cholesterol
bone and is termed as “involucrum”.
clefts, foam cells, plasma cells and T lymphocytes in
the lesion.
Differential Diagnosis l The epithelial rests of Malassez proliferate in response
l Metastasis in bone with secondary infection. to chronic inflammation and these proliferating cells
l Primary intra-alveolar carcinoma. undergo liquifaction.
Section | III Oral Pathology 537

l The bony tissue at the periphery of lesion is lined by the Pathogenesis


osteoclast cells with area of bone resorption.
l Usually begins as periapical granuloma.
l Few bacteria are present in the lesions which are not
l In the midst of the rich vasclarised area provided by the
affected by the cellular immune mechanism.
granuloma, the rests of Malassez proliferate and eventu-
l Occasionally, Russell bodies are also found.
ally form a large mass of cells.
l Resorption of cementum and dentin often occurs as a
l With continuous growth, the inner cells of mass are
result of chronic inflammation.
deprived of nourishment and they undergo liquefaction
l In some areas, along root, cementoblastic activity pre-
necrosis.
dominates leading to hypercementosis.
l This leads to the formation of cavity which is located
in the centre of granuloma, giving rise to a radicular
Radiological Features cyst.
l Earliest change is the periodontal ligament PDL appears
as a thickening of ligament at the apex. Histopathological Features
l Loss of lamina dura.
l Periapical radiolucency is less than 1.5 cm in diameter. l It is lined by stratified squamous epithelium which is
l The radiolucency is the well-circumscribed lesion defi- varied in thickness with rare keratin formation.
nitely demarcated from surrounding bone. l Hyaline and Rushton bodies are found in great numbers
l Some of root and perapical bone resorption is occasion- in epithelium.
ally observed. l Connective tissue makes up the wall and is com-
posed of collagen fibres, fbroblasts and small blood
vessels.
Treatment l Lumen contains fluid with low concentraton of protein
l Extraction of involved tooth or and it also contains cholesterol or keratin.
l Root canal therapy with apical curettage. l The cholesterol clefts and multinucleated giant cells are
seen in the cyst wall.
Q. 5. Enumerate the periapical lesions. Describe briefly
the pathogenesis, histopathological and clinical features
of the radicular cyst. Clinical Findings
Ans. l Usually found in adult life although may occur at any
age.
Various periapical lesions are as follows:
l More common in maxillary anterior region.
l Acute apical periodontitis
l Most common in males.
l Periapical abscess
l The tooth involved is generally non-vital and hence as-
l Periapical granuloma
ymptomatic.
l Periapical cyst
l Septic tooth or root is usually present.
l Dentigerous cyst
l Presents as slowly growing painless swelling on the
l Periapical scar
buccal aspect of the jaw, because it is a thinner bone.
l Giant cell granuloma
l Swelling which is round and hard at first, later when
l Osteomyelitis
the bone becomes thin to the thickness of an egg
l Periapical cemental dysplasia
shell, crackling sensation is felt on pressure with the
l Langerhans cell disease
fingers.
l Finally, as the bony wall is resorbed fully, pressure
RADICULAR CYST produces fluctuation.
l It is an inflammatory cyst lined by epithelium and is l Swelling looks bluish in colour under the mucous
filled with fluid. membrane.
l Radicular cyst is otherwise known as apical periodontal l Infection of the cyst causes, redness and pain.
cyst or root end cyst or periapical cyst. l Pus formation in the cyst will lead to sinus opening and
discharge of pus.
Aetiology
a. Infection from microorganisms and their toxins in the
Treatment
pulp canal of non-vital tooth. l Enucleation and extraction of affected tooth
b. Trauma l Root canal treatment and apicoectomy.
538 Quick Review Series for BDS 3rd Year

SHORT ESSAYS
Q. 1. Acute pulpitis. Q. 2. Sequestrum.
Ans. Ans.
Acute pulpitis is an irreversible condition characterized by i. The sequestrum is a dead piece of bone separated from
acute, intense inflammatory reaction in pulp tissue. the living bone and is hard, rough, porous, light in
weight and even lighter in colour than normal bone.
ii. Sequestra are more dense and better defined due to fol-
Aetiology lowing reasons:
l Pulp exposure due to faulty cavity preparation. l Sclerosis that was induced before the bone became

l Caries progressing beyond the dentinal barriers and necrotic.


reaching the pulp. l Dead bone has affinity for calcium. Hence it absorbs

l Recurrent caries around the pre-existing restoration calcium.


l Chemical irritation to pulp l Inflammatory reaction is the reason of demineraliza-

l Cracked tooth syndrome tion of bone surrounding the sequestra. This en-
l Blow to tooth with subsequent damage to pulp. hances the contrast.
iii. These sequestra must be removed surgically to initiate
new bone formation and repair.
Clinical Features iv. New bone is formed under the periosteum where it has
l It may be either sequela of focal reversible pulpitis or been raised from the jaw. This new bone called involu-
acute exacerbation of chronic pulpitis. crum will be formed around the original bone which
l Thermal changes also increase the intensity of pain. has died.
l The intensity of pain increases during the sleep, i.e. su- v. New bone is also formed in the areas from where dead
pine position and occurs due to increase in local blood bone is removed by forming the granulation tissue
pressure in head and neck region. which in due course changes into new bone.
l Acute pulpitis helps in spread of inflammation through- Q. 3. Classification of osteomyelitis.
out pulp with subsequent necrosis.
l Acute pulpitis is often associated with microab- Ans.
scess formation in pulp along with liquefaction Osteomyelitis is defined as an inflammatory condition of
degeneration. the bone that begins as an infection of medullary cavity and
l Pain subsides when drainage is established or when the haversian system and extends to involve the periosteum
pulp undergoes complete necrosis. of the affected area.
l The tooth is non-tender to percussion unless the pulpal
inflammation has spread beyond the root apex into peri-
apical region. Classification of Osteomyelitis
A. Based on location:
Histopathology l Intramedullary
l Subperiosteal
l Initially, severe oedema in the pulp with vasodilatation l Periosteal
takes place.
l Moderate to dense infiltration of polymorphonuclear B. Based on duration and severity:
leucocytes. l Acute
l Many microabscess formations in pulp. l Chronic
l There may be complete liquefaction and necrosis of
pulp with total destruction of odontoblastic cell layer. C. Based on presence or absence of suppuration:
l Death of pulp is due to tissue dehydration. This is Suppurative:
known as “dry gangrene of pulp”. l Acute suppurative osteomyelitis

l Chronic suppurative osteomyelitis – i. Primary,

ii. Secondary
Management l Infantile osteomyelitis

l In early stages – pulpotomy can be done. Non-suppurative:


l If there is large area of pulpal destruction, pulpectomy l Chronic non-suppurative: i. Focal sclerosing, ii. If-

is done. fuse sclerosing


Section | III Oral Pathology 539

l Radiation osteomyelitis l The affected tooth has a large open carious cavity,
l Garre’s sclerosing osteomyelitis which is present for long duration.
l Osteomyelitis due to specific infection like actino- l It is a hyperplastic tissue growth (granulation tissue)
mycosis, tuberculosis, syphilis, etc. through the opening of the pulp chamber into the tooth
cavity and bleeds profusely on provocation.
Q. 4. Pulp polyp.
Ans.
Histopathology
l Pulp polyp is the productive pulpal inflammation due to
l The hyperplastic pulp tissue lesion presents the feature
an extensive carious exposure of a young pulp.
of granulation tissue mass, consisting of numerous pro-
l It is characterised by the development of granulation
liferating fibroblasts and young blood capillaries.
tissue, covered at times by epithelium and resulting
l Inflammatory cell infiltration by lymphocytes, plasma
from long-standing low grade infection.
cells and sometimes polymorphonuclear neutrophils in
l Pulp polyp is also known as:
tissue are common.
Chronic open hyperplastic pulpitis or
l Stratified squamous epithelium is present on the surface
Pulpitis aperta.
of hyperplastic pulpitis which resembles oral epithelium.
l The epithelium surface shows well-formed rete peg
Predisposing Factors formation.
l The epithelial cells on surface are believed to be des-
It occurs when:
quamated epithelial cells which came either from buccal
a) Host immunity is strong.
mucosa or from salivary gland ducts.
b) Bacterial virulency is low.
c) Large carious cavity and good blood supply due to large
apical foramina. Treatment
l Endodontic treatment or
Pathogenesis l Tooth extraction.
The sequence of polyp formation is as follows: Q. 5. Define periapical granuloma and describe its his-
topathological features.
Irritation/infection Ans.
Periapical granuloma or chronic apical periodontitis or
dental granuloma is a low grade infection around the root
Chronic inflammation apex of non-vital tooth.

Aetiology
Proliferation of granulation tissue
l Extension of pulpal inflammation
l Acute trauma.
l Perforation of root apex into endodontic therapy.
Epithelialization

Clinical Features
Fibrosis l The tooth involved is usually non-vital.
l The tooth involved produces sensitivity to percussion.
Mild pain and discomfort in tooth during chewing solid
Clinical Features
l

foods.
l Seen often in deciduous 2nd molar and permanent 1st l The involved tooth feels slightly elongated from the
molar in children or young adults. socket.
l The involved tooth is asymptomatic and is feebly sensi- l If pus has formed, a small reddish swelling may be
tive to thermal stimuli. found on the buccal aspect of tooth or a sinus may be
l Pulp polyp appears as small, pinkish, red lobulated present.
mass, which protrudes from pulp chamber and fills up l Enlargement and tenderness of regional lymph nodes.
the carious cavity. l Fever and malaise occur.
540 Quick Review Series for BDS 3rd Year

Histopathological Features Clinical Findings


l The lesion appears as granulation tissue mass consisting l Usually found in adults although it may occur at
of proliferating fibroblasts, endothelial cells and numer- any age.
ous immature blood capillaries. l More common in males and maxillary anterior region.
l Presence of epithelial islands, cholesterol clefts, foam l The tooth involved is generally non-vital.
cells, plasma cells. l Presents as slowly growing painless swelling on the
l Occasionally, Russell bodies are also found. buccal aspect of the jaw.
l In some areas, along root, hypercementosis occurs. l Swelling which is round and hard at first, later when
the bone becomes thin to the thickness of an egg
shell, crackling sensation is felt on pressure with the
Radiological Features fingers.
l A thickening of periodontal ligament. l Swelling looks bluish in colour and fluctuation can be
l Loss of lamina dura. elicited in later stages.
l Well-circumscribed periapical radiolucency less than l Pus formation in the cyst will lead to sinus opening and
1.5 cm in diameter at the root apex. discharge of pus.
l Some of root and perapical bone resorption is occasion-
ally observed.
Treatment
Enucleation and extraction of affected tooth
Treatment l

l Root canal treatment and apicoectomy.


l Extraction of involved tooth or
Q. 7. Garre’s osteomyelitis.
l Root canal therapy with apical curettage.
Ans.
Q. 6. Periapical cyst.
l Garre’s osteomyelitis is otherwise known as chronic
Ans.
osteomyelitis with proliferative periostitis or periostitis
Periapical cyst is an inflammatory cyst lined by epithelium ossificans or Garre’s chronic non-suppurative sclerosing
and is filled with fluid. osteitis.
l It is otherwise known as radicular cyst or apical peri- l It represents a reactive periosteal osteogenesis in re-
odontal cyst or root end cyst. sponse to low grade infection or trauma.

Aetiology Aetiology
a. Infection in the pulp canal of non-vital tooth. l Mild infection
b. Trauma l Chronic periapical abscess
l Chronic trauma in the jaw bone.
Pathogenesis
Clinical Features
l Usually begins as periapical granuloma.
l The rests of Malassez proliferate and eventually form a l It is common in young children and adults.
large mass of cells. l The posterior region of mandible is commonly
l With continuous growth, the inner cells of mass are involved.
deprived of nourishment and they undergo liquefaction l The involved jaw bone has a carious non-vital tooth.
necrosis. l There is a slight tenderness or a vague pain.
l This leads to the formation of radicular cyst. l Hard non-tender swelling with medial and lateral ex-
pansion of jaw.
Lymphadenopathy, slight pyrexia and leucocytosis may
Histopathological Features l

be present, but ESR is normal.


l It is lined by stratified squamous epithelium which is
varied in thickness with rare keratin formation
Radiological Features
l Hyaline and Rushton bodies are found in great numbers
in epithelium. l It presents a central jaw lesion with mottled pre-
l Connective tissue makes up the wall. dominantly radiolucent appearance and having few
l Lumen contains fluid with low concentraton of protein radio-opaque foci.
and it also contain cholesterol or keratin. l Characteristic onion skin appearance.
Section | III Oral Pathology 541

Treatment and Prognosis Clinical Features


l Elimination of causative agent. l Patients are of less than 20 years.
l Extraction of carious infected tooth and antibiotic therapy. l Most commonly affected tooth is mandibular first
l Prognosis is good. molar.
l Tooth is non-vital and grossly carious.
Q. 8. Chronic focal sclerosing osteomyelitis.
Ans.
Radiographic Features
l Chronic sclerosing osteomyelitis is a non-suppurative
inflammatory condition often seen in dentulous jaw. l Radio-opacity at the apex of root with intact lamina
l It is of two types: Focal and diffuse. dura.
l This is uncommon reaction of bone to exceptionally low l This lesion has dense bony trabeculae with little
grade periapical infections or due to very high tissue stroma.
resistance.
Treatment
Aetiology l Endodontic therapy of affected tooth.
High resistance of alveolar bone to odontogenic infection. l Extraction of affected tooth.

SHORT NOTES
Q. 1. Pulp polyp. l It is of developmental origin.
l Commonly involved teeth are mandibular premolars.
Ans.
l Radiographically, it is seen as a well-delineated radio-
i. Pulp polyp is chronic form of pulpitis with excessive, lucency, round, small not exceeding 1 cm in diameter,
exuberant proliferation of pulp tissue. with a radio-opaque margin.
ii. It occurs when there is a large carious cavity and good l The treatment of the lateral periodontal cyst is surgical
blood supply due to large apical foramina. ablation.
iii. It is mostly seen in children and young adults in first molars. Q. 4. Acute alveolar abscess.
iv. Usually painless and appears as pinkish-coloured glob-
ular mass. Ans.
v. Histologically, it is composed of granulation tissues i. Acute alveolar abscess is also known as dentoalveolar
with chronic inflammatory infiltrate covered by strati- abscess or periapical abscess.
fied squamous epithelium. ii. It is defined as acute suppurative infection in periapical
v. Involved tooth may be treated by endodontic therapy or region of tooth.
extraction. iii. Clinical features:
Q. 2. Radicular cyst. l Pain in the affected tooth.

l Localized swelling which may be tender during pal-


Ans. pation.
i. Radicular cyst is an inflammatory cyst and is otherwise l Application of heat intensifies pain, whereas appli-

known as apical periodontal cyst or root end cyst. cation of cold relieves pain temporarily.
ii. More common in males and maxillary anterior region. l Pus discharging sinus often develops on alveolar

iii. The tooth involved is generally non-vital. mucosa of the affected root apex.
iv. Histologically, it is lined by stratified squamous epithe- iv. Treatment consists of establishing drainage through the
lium which is varied in thickness. pulp and then extraction of tooth or RCT and apicectomy.
v. Hyaline and Rushton bodies are found in great numbers Q. 5. Periapical granuloma.
in epithelium.
vi. It is treated by enucleation or extraction of affected tooth. Ans.
Q. 3. Periodontal cyst. i. Periapical granuloma or chronic apical periodontitis or
dental granuloma is a low grade infection around the
Ans. root apex of non-vital tooth.
l Periodontal cyst occurs on a lateral periodontal region, ii. Clinical features:
located mid-distance between the apex and the cervical l The tooth involved is usually non-vital.

area of the affected tooth. l Mild pain and discomfort in tooth during chewing

l Involved tooth is asymptomatic and vital. solid foods.


542 Quick Review Series for BDS 3rd Year

l The involved tooth feels slightly elongated from the Q. 8. Pyogenic granuloma.
socket.
Ans.
l Enlargement and tenderness of regional lymph nodes.

iii. Histologically, the lesion appears as granulation tissue l Pyogenic granuloma is one of the inflammatory lesions
mass consisting of proliferating fibroblasts, endothelial seen in the oral cavity.
cells and numerous immature blood capillaries. l Caused due to low grade local irritation, traumatic injury
iv. Radiographic features include a thickening of periodontal or hormonal factors.
ligament, loss of lamina dura and well-circumscribed l Occurs in second decade of life in young females.
periapical radiolucency less than 1.5 cm in diameter at l Clinically, it is smooth, lobulated, exopthytic lesion man-
the root apex. ifesting as a small red erythematous papules on a pedun-
v. Treatment consists of extraction of involved tooth or culated or a sessile base seen on the interdental papilla.
root canal therapy with apical curettage. l The development of lesion is slow, asymptomatic.
Q. 6. Garre’s osteomyelitis. l It is usually haemorrhagic and compressible and hence
bleeds on minor trauma.
Ans. l It is developed in 5% of pregnancies and hence also
l Garre’s osteomyelitis is also known as chronic osteomy- called as pregnancy tumour or granuloma gravidarum.
elitis with proliferative periostitis or periostitis ossificans l Treatment consists of surgical excision.
or Garre’s chronic non-suppurative sclerosing osteitis. Q. 9. Histopathology of periapical granuloma.
l It represents a reactive periosteal osteogenesis in re-
sponse to low grade infection or trauma. Ans.
l It is common in young children and adults in the poste-
Histopathological features of periapical granuloma are as
rior region of mandible.
follows:
l The involved jaw bone has a carious non-vital tooth.
l The lesion appears as granulation tissue mass consisting
l Lymphadenopathy, slight pyrexia and leucocytosis may
of proliferating fibroblasts, endothelial cells and numer-
be present but ESR is normal.
ous immature blood capillaries.
l Radiologically, it exhibits characteristic onion skin
l Chronic inflammatory cells, e.g. macrophages, lympho-
appearance.
cytes and plasma cells are present in the lesion.
l Treatment consists of elimination of causative agent and
l There is presence of epithelial islands, cholesterol clefts
extraction of carious infected tooth and antibiotic ther-
and foam cells.
apy. Prognosis is good.
l The plasma cells often produce immunoglobulin. There
Q. 7. Acute suppurative osteomyelitis. is also present T lymphocytes in the lesion.
Ans. l The epithelial cell rests of Malassez proliferate in re-
sponse to chronic inflammation and these proliferating
i. Acute suppurative osteomyelitis is serious sequela of cells undergo cystification.
periapical infection, resulting in diffuse spread of in- l The bony tissue at the periphery of lesion is lined by the
fection throughout the medullary spaces of bone. osteoclast cells with area of bone resorption.
ii. Causative microorganisms include Staphylococcus l Few bacteria are present in the lesions which are not
aureus, b-haemolytic streptococci, Haemophilus in- affected by the cellular immune mechanism.
fluenzae and Pseudomonas species. l Occasionally, Russell bodies are also found.
iii. Usually occurs after the age of 50 years and males are
more commonly affected. Q. 10. Fibroepithelial polyp.
iv. The mandible is most commonly involved and severe
Ans.
throbbing deep seated pain and swelling are the initial
complaints. l Fibroepithelial polyp is characterized by a pink, red or
v. Regional lymphadenopathy, loosening and soreness of white knob-like growth.
the regional teeth with difficulty in mastication. l They may arise anywhere on the mucosa of the oral
vi. Multiple intraoral and extraoral pus discharging cavity, but more commonly seen in the gingiva, tongue
sinuses develop. and the lip.
vii. Patient is slightly febrile and general symptoms l This is caused by minor trauma or irritation, usually
include fever, malaise, anorexia and vomiting. following accidental biting.
viii. Histologically, the bone marrow undergoes liquefac- l They are small and generally painless.
tion and purulent exudates occupy the marrow space l They rarely continue to grow, unlike papillomas and
and development of sequestrum. fibromas, which have similar clinical pictures.
ix. It is treated by establishing drainage of pus along with l They are usually single, but occasionally multiple.
antibiotic therapy and “sequestrectomy” or “saucer- Removal of the irritant and excision is the treatment of
ization” is done. choice.
Section | III Oral Pathology 543

Topic 11

Spread of Oral Infections

LONG ESSAYS
Q. 1. Describe in detail about Ludwig’s angina. Clinical Findings
Ans. l Painful, rapid developing board-like swelling in the
floor of the mouth.
l Ludwig’s angina is a form of firm, acute, toxic and
l Elevation of tongue.
severe diffuse cellulitis causing board swelling of sub-
l Enlarged painful lymph nodes.
mandibular, sublingual and submental spaces bilaterally.
l Difficulty in swallowing and opening the mouth.
l It is a disease primarily of dental origin following infec-
l High fever, rapid pulse, fast breathing.
tion of second and third inferior molars.
l Headache, malaise and other signs of toxaemia.
l This is a condition of acute infection of the floor of the
l Swelling of the upper part of the neck.
mouth (i.e. cellulitis) involving the cellular tissue and
l Oedema of glottis causing respiratory obstruction.
tissues of the neck and submandibular area.
l Infection may spread to pharyngeal spaces, to carotid
sheath or to pterygopalatine fossa.
Aetiology l Cavernous thrombosis with subsequent meningitis is
l Infection from mandibular molar teeth sequel to this type of spread of infection.
l Infection within the floor of the mouth
l Vincent’s angina
l Periodontal disease Treatment
l Acute tonsillitis l Incision and drainage of pus
l Peritonsillar abscess (Quinsy) l Antibiotic therapy
l Pericoronitis l Mouthwashes
l Fracture of the mandible l Liquid diet
l Erysipelas l Emergency tracheostomy, if symptoms of asphyxia are
present.
Predisposing Factors
Lowered resistance and poor oral hygiene.

SHORT ESSAYS
Q. 1. Focal infection. preference is probably an environmental phenomenon
rather an inherent or developed phenomenon of micro-
Ans.
organism. Examples:
i. A focal infection is a localized or generalized infection l Scarlet fever: The remarkable cutaneous features of

caused by the dissemination of microorganisms or toxic the disease being due to the erythrogenic toxin liber-
products from a focus of infection. ated by infecting streptococci
ii. Two mechanisms of focal of infection: either haema- l Rheumatic fever

togenous or lymphogenous spread l Infected periapical lesions such as periapical granu-

a. Metastasis of microorganisms from infected focus loma, cysts, and abscesses.


b. Toxins are carried from focus to distant site l Periodontal disease with reference to tooth

iii. Certain organisms have predilection for isolating them- extraction.


selves in specific sites in the body. This localization iv. Significance of oral foci of infection.
544 Quick Review Series for BDS 3rd Year

They cause or aggravate a great number of systemic l Female to male ratio of patients affected with haeman-
diseases like: giomas is 3:1.
l Arthritis l They are one of the most common benign orbital tu-
l Valvular heart diseases mours of infancy (6 months).
l Gastrointestinal diseases
l Ocular diseases
Skin diseases
l
Pathophysiology and Clinical Features
l Renal diseases
l Capillary haemangiomas are believed to be hamartoma-
Q. 2. Ludwig’s angina. tous proliferations of vascular endothelial cells.
Ans. l They are now thought to be of placental origin due to a
unique microvascular phenotype shared by juvenile
l Ludwig’s angina is a form of firm, acute, toxic and haemangiomas and human placenta.
severe diffuse cellulitis causing board swelling of sub- l They generally exhibit 2 phases of growth, a prolifera-
mandibular, sublingual and submental spaces bilaterally. tive phase and an involutional phase.
l It is a disease primarily of dental origin following infec- l The proliferative phase of rapid growth typically occurs
tion of second and third inferior molars. from 8-18 months. Pathologically, it is characterized by
an increased number of endothelial and mast cells, the
latter being a stimulus for vessel growth. Endothelial
Aetiology
cell proliferation returns to normal following the prolif-
l Infection from mandibular molar teeth, Vincent’s an- eration phase.
gina, periodontal disease, pericoronitis or fracture of the l The involutional phase is characterized by slow regres-
mandible. sion of the haemangiomas. One-half of all lesions will
involute by age 5 years, and 75% will involute by age
7 years. During this phase, mast cell numbers decrease
Clinical Findings to normal and there is a decrease in endothelial and mast
l Painful, rapid developing board-like swelling in the cell activity. These vascular spaces become lined with
floor of the mouth. endothelial cells without muscular support.javascript:sh
l Elevation of tongue. owrefcontent(‘refrenceslayer’);
l Enlarged painful lymph nodes. l Types of capillary haemangiomas are:
l Difficulty in swallowing and opening the mouth. Salmons patch: midline – forehead
l High fever, rapid pulse, fast breathing. Portwine stain: head and neck
l Oedema of glottis causing respiratory obstruction. Strawberry angioma: compressible
l Infection may spread to pharyngeal spaces, to carotid l Treatment: Long-term observation, i.e. wait and watch
sheath or to pterygopalatine fossa. phenomenon is best.
l Cavernous thrombosis with subsequent meningitis is
sequel to this type of spread of infection.
CAVERNOUS HAEMANGIOMAS
Treatment l They occur in place where venous space is abundant
l Consists of incision and drainage of pus, antibiotic i.e. lip, cheek, tongue, posterior triangle of neck.
therapy, mouthwashes, liquid diet. l No predilection exists for race or ethnicity.
l Emergency tracheostomy, if symptoms of asphyxia are l Equally distributed in both sexes.
present. l Patients usually manifest symptoms during the third
to fifth decades of life.
Q. 3. Capillary and cavernous haemangiomas. l Compressible swelling, bluish warm, non-tender
swelling
Ans.
l Associated with arteriovenous communication
l Associated with lipoma.
DD: Lymphangioma and lipoma
CAPILLARY HAEMANGIOMAS l

l Treatment includes injection of boiling water or hy-


l They begin as endothelial cell neoplasms that are pertonic saline or sodium tetradecyl sulphate or
typically absent at birth and characteristically have excision.
rapid growth in infancy with spontaneous involution l Complications include ulceration, bleeding, infection
later in life. and high output cardiac failure.
Section | III Oral Pathology 545

SHORT NOTES
Q. 1. Cellulitis. Clinical Findings
Ans. l Painful, rapid swelling in the floor of the mouth causing
elevation of tongue.
l Cellulitis is an inflammation and infection of cellular l Enlarged painful lymph nodes.
tissue especially of loose subcutaneous tissue. l Swelling of the upper part of the neck.
l It occurs in the facial spaces or muscular spaces or take l Oedema of glottis causing respiratory obstruction.
the form of deep-seated phlegmons. l Difficulty in swallowing and opening the mouth.
l It occurs due to spread of dental infection, i.e. from api- l High fever, rapid pulse, fast breathing.
cal abscess, osteomyelitis, pericoronal infection, peri- l Headache, malaise and other signs of toxaemia.
odontal infection; after extraction of tooth. Fracture of
the jaw followed, by secondary infection. Q. 4. Spaces involved in Ludwig’s angina.
l The soft tissue filling the facial planes and spaces is the Ans.
common site.
l The condition may progress rapidly leading to serious l It is a severe form of cellulitis which can spread into
complications which are surgical emergencies, requir- parapharyngeal space.
ing incision and drainage. l Ludwig’s angina is a bilateral infection of the submandibu-
l Infection may be localized to one space or may spread lar space that consists of two compartments in the floor of
along the various facial planes at the same time. the mouth, the sublingual space and the submental space.

Q. 2. Focal infection. Q. 5. Definitions of focus of infection and focal infection.

Ans. Ans.
l Focus of infection: It is the circumscribed area of the
l A focal infection is a localized or generalized infection
tissue, which is infected with exogenous pathogenic
caused by the dissemination of microorganisms or toxic
microorganisms and is usually located near a mucous or
products from a focus of infection.
cutaneous surface.
l Two mechanisms of focal of infection: By either haema-
l Focal infection: A focal infection is a localized or gener-
togenous or lymphogenous spread
alized infection caused by the dissemination of microor-
l Metastasis of microorganisms from infected focus.
ganisms or toxic products from a focus of infection.
l Toxins are carried from focus to distant site.
Q. 6. Pioneer bacteria.
Q. 3. Ludwig’s angina.
Ans.
Ans.
l In earliest stages, if caries when only a few tubules are
l Ludwig’s angina is form of firm, acute, toxic and severe involved, microorgsnisms may be found penetrating
diffuse cellulitis which can spread into parapharyngeal these tubules before there is any clinical evidence of the
space. carious process.
l It is a disease of primarily of dental origin. l These microorganisms are termed as pioneer bacteria.

Topic 12

Physical and Chemical Injuries of the Oral Cavity

LONG ESSAYS
Q. 1. Describe the predisposing factors and clinical Osteoradionecrosis is a radiation-induced pathologic pro-
features of osteoradionecrosis. cess. It is an acute form of osteomyelitis caused by damage
Ans. to the intraosseous blood vessels and is characterized by
546 Quick Review Series for BDS 3rd Year

the chronic and painful infection and necrosis accompa- iv. After the infection has gained entry to the bone, there is
nied by the late sequestration and sometimes permanent relatively diffuse spread of the process and minimum
deformity. localization of infection and there may be necrosis of
considerable amount of bone, periosteum and overlying
Factors leading to osteoradionecrosis are as follows:
mucosa.
i. Irradiation of an area of previous surgery before
v. The necrotic process may extend throughout the
adequate healing had taken place.
radiated bone. The devitalized bone may undergo
ii. Irradiation of lesions in close proximity to bone.
sequestration.
iii. Poor oral hygiene and continued use of irritants.
iv. Use of high dose of irradiation with or without proper
fractionation. Histology:
v. Combined use of external radiation and intraoral
i. There is destruction of osteocytes, absence of osteo-
implants.
blasts and lack of new bone or osteoid formation.
vi. Poor patient’s corporation in managing irradiated tis-
ii. The walls of regional blood vessels are thickened by
sues or fulfilling homecare programmes.
fibrous connective tissue and are also seat of endarteri-
vii. Surgery in the irradiated area.
tis and periarteritis.
viii. Failure to prevent trauma to irradiated bony areas.
iii. The loose connective tissue which usually replaces the
ix. Use of prosthetic appliances indiscriminately follow-
bone marrow is infiltrated by lymphocytes, plasma cells
ing radiation therapy.
and macrophages.
x. Presence of numerous physical and nutritional prob-
iv. Radiation causes proliferation of intima of blood
lems prior to therapy.
vessels leading to thrombosis of arteries which results
in non-vital bone.
Clinical Features
i. The occurrence of osteoradionecrosis is unpredictable
Treatment
and it may arise without gross infection or trauma.
ii. Mandible is affected more commonly than maxilla. l Debridment of necrotic tissue should be done along
iii. Aetiology is unknown. Although exact pathogenesis with removal of sequestrum.
not completely understood, it is generally agreed that l Administration of intravenous antibiotic and hyperbaric
there are three factors involved: Radiation, trauma and oxygen therapy are essential.
infection. l Oral hygiene maintenance.

SHORT ESSAYS
Q. 1. Sialolithiasis. l Occurs more frequently in submandibular duct and
glands due to tenacity of submandibular saliva,
Ans.
because of its high mucin content adheres to any
i. Sialolithiasis is also known as salivary duct stone or foreign particle. Submandibular duct is also long
salivary duct calculus. and irregular in course.
ii. A round, ovoid calcified structure present in the salivary v. Diagnosis: By sialography.
duct or gland is called a ‘sialolith’ and is formed by the vi. Treatment:
deposition of calcium salts around a central nidus. l Small calculi can be removed by manipulation or by

iii. Composition of stone (calculi): increasing the salivation by sucking a lemon, leading
l Tricalcium phosphate Ca3(PO4)2 to expulsion of the stone.
l Calcium carbonate CaCO3 l The larger stones need to be removed by surgical

l Soluble salts exposure.


l Organic matter l If present near or in the gland, the gland has to be

l Water extirpated.
iv. Clinical features: l Peizoelectric shock wave lithotripsy may be an
l May occur at any age, but common in middle age alternative procedure for removal of sialolith.
group.
Q. 2. Classify fractures of teeth.
l Swelling and pain which is related to mealtime.

l Stones may be palpable. Ans.


Section | III Oral Pathology 547

According to Ellis classification, traumatized anterior teeth l The lesion may lie superficial or fairly deep in the
are divided into nine classes as follows: tissue.
Class 1: Fracture of crown, involving little or no dentin, i.e. l The superficial lesion appears as raised, bluish, translus-
enamel only. cent, circumscribed vesicle measuring several mm to a
Class 2: Fracture with involvement of dentin but no pulp i.e. cm in diameter. While the deep lesion also appears as a
enamel and dentin only. swelling but shows no colour.
Class 3: Considerable dentin is involved with pulpal l Begins to increase in size, reaches a certain size and
exposure, i.e. enamel, dentin and pulp are involved. may persist as such unless treated.
Class 4: Traumatized tooth becomes non-vital with or l Recurrence is common.
without loss of crown structure.
Class 5: Teeth lost due to trauma, i.e. avulsion
Histological Features
Class 6: Fracture of root with/without loss of crown
structure. l Mucocele is not a true cyst. It represents a lesion of the
Class 7: Displacement of tooth without fracture of crown or retention type.
root. l Histologically, the cavity of mucocele is not lined by
Class 8: Fracture of “crown en masse” and its replace- epithelium and its wall is made of compressed fibrous
ment. connective tissue and fibroblasts. There is infiltration of
Class 9: Traumatic injuries to deciduous teeth. PMN leucocytes, lymphocytes, and plasma cells.
l The lumen is filled with an eosinophilic coagulum
Histological Features containing different cells, mainly leucocytes, and
mononuclear phagocytes.
l Histological features during most satisfactory form of l The salivary gland acini adjacent to the mucocele show
healing are similar to that of bony fractures. areas of interstitial inflammation, dilatation of intralob-
ular and interlobular ducts with collection of mucus,
Treatment breakdown of acinar cells resulting in the formation of
l If enamel is fractured: restoration of the missing tooth small areas of pooled mucus.
structure is done.
l If dentin is involved: placement of sedative base (zinc Treatment
oxide eugenol) is done at fractured dentin and tooth is
restored. l Excision.
l If pulp is exposed: Q. 4. Enumerate the effects of radiation on the tissues.
a. Pulp capping is done for small pulpal exposures.
b. Pulpotomy is carried out for coronal pulp in imma- Ans.
ture permanent teeth. The effects of radiation on oral and paraoral structures are
c. Pulpectomy, i.e. endodontic treatment may be done dependent on various factors:
for mature permanent teeth. i. The source of radiation
Q. 3. Describe the pathogenesis and histopathology of ii. Total amount of radiation administered
mucocele. iii. Fractionation (The period of time over which the
radiation was administered.)
Ans. iv. The type of filtration used.
The mucocele occurs due to retention of mucous material v. The total area of tissue irradiated.
due to trauma, involving salivary glands and their ducts. The effects of radiation described here are those frequently
Mucoceles often have been classified as: seen after delivery of local therapeutic doses of X-ray radiation
a. Extravasation mucocele in the treatment of neoplasms about head and neck.
b. Retention mucocele
EFFECTS OF RADIATION ON SKIN
The extravasation type is far more common than retention
type of mucocele. l Erythema is the earliest visible reaction that appears
first and then gradually disappears.
Secondary erythema appears and fades slowly.
Clinical Features l

l After heavy irradiation, secondary erythema may be


l It occurs most frequently on the lower lip, but may also accompanied by oedema along with desquamation of
occur on the palate, cheek, tongue, and floor of the mouth. epithelial cells resulting in denudation of the surface.
548 Quick Review Series for BDS 3rd Year

l Re-epithelization and disappearance of secondary l X-rays have the damaging effect on bone forming cells.
erythema occurs in 10-14 days. The blood vessels also become thrombosed and when
l Alteration of sebaceous glands activity occurs within a these changes are associated with trauma and infection,
week after beginning of irradiation leading to dryness of osteoradionecrosis occurs.
skin. l Osteoradionecrosis is a radiation-induced pathologic
l Epithelium becomes thin, atrophic and blood vessels process. It is an acute form of osteomyelitis caused by
become telangiectatic or occluded. damage to the intraosseous blood vessels and is charac-
terized by the chronic and painful infection and necrosis
EFFECTS OF RADIATION ON ORAL MUCOSA accompanied by the late sequestration and sometimes
l The changes noted in the oral mucosa after radiation are permanent deformity.
same as those seen in the skin and are related to dose
Q. 5. Acrodynia (Pink disease).
and duration of therapy. Erythema of mucosa develops
some what at lower dose compared to skin. Ans.
l Then mucositis develops earlier than dermatitis.
l Acrodynia is also known as Pink disease or Swift’s
Mucosa become hyperaemic and oedematous.
disease or erythroedema polyneuritis.
l As treatment continued, mucosa becomes denuded, ul-
l This condition is due to mercurial toxicity reaction
cerated and covered with fibrinous exudate.
either an actual ‘mercury poisoning’ or as idiosyncrasy
l Mucositis persists through out the treatment period with
to the metal.
radiotherapy and then disappears following termination
of radiation therapy.
l Taste sense is also lost due to damage to microvilli and Clinical Features
outer surface of taste cells. Taste acuity was returned, i. Age of occurrence: More common in young infants be-
within 2 to 4 months (60-120 days) after completion of low the age of 2 years but can occur at a later age as well.
radiotherapy. ii. The skin becomes red or pink and has cold clammy
feeling particularly on hands, feet, nose, ears and
EFFECTS OF RADIATION ON SALIVARY GLANDS
cheeks. This particular appearance of skin has been
l Xerostomia or dryness of mouth occurs due to loss of described as resembling ‘raw beef’.
acinar cells, decrease in secretory granules and inflam- iii. During the course of disease, the skin over the affected
mation in connective tissue of salivary glands. areas peels frequently.
l These changes may cause permanent dryness of mouth iv. The patients have maculopapular rash which is ex-
due to permanent loss of secretion. tremely pruritic. Severe sweating is a constant feature
l There are no remarkable changes in the ducts of salivary of acrodynia.
glands. v. Other features are extreme irritability, photophobia
with lacrimation, muscular weakness, tachycardia,
EFFECTS OF RADIATION ON TEETH
hypertension, insomnia, G.I. upset, stomatitis and
l Radiation exposure of developing teeth in formitive patchy loss of hair.
stage may cause anodontia or defective root formation.
l Radiation after development of teeth may cause cervical
caries that may lead to fracture of crown at cervical Oral Findings
third. i. Profuse salivation
l Caries may either be due to direct effect of X-rays on ii. Painful and swollen gingiva which may exhibit ulceration.
teeth or associated with xerostomia that has occurred iii. Difficulty in mastication because of pain.
secondarily due to radiotherapy. iv. Loosening and premature shedding of deciduous teeth.
l Radiation caries is a destruction of the tooth substance
developing at the cervical area of teeth due to deminer-
Treatment
alization, but clinically looks like caries.
l Identify and remove the source of mercury.
EFFECTS OF RADIATION ON BONE l Immediate chelation therapy is the standard of care for
l The bone is resistant to X-ray radiation, although osteo- a patient showing the symptoms of severe mercury
blasts are sensitive. poisoning.
l In case of sufficiently intense radiation, the normal bal- l Administration of dimercaprol, D-pencillamine or
ance between bone formation and resorption is dis- DMSA, 2, 3-dimercapto-1-propanesulfonic acid has
turbed, general bone vitality is decreased and localized been proved most successful in many cases unless the
osteoporosis may result. disease is of long duration.
Section | III Oral Pathology 549

SHORT NOTES
Q. 1. Mucocele. l It is bluish translucent, fluctuant swelling that can be
palpated bimanually.
Ans.
l A plunging type of ranula may be present in

i. The mucocele is not a true cyst, it occurs due to retention submandibular region.
of mucous material due to trauma, involving salivary l It may interfere with speech and mastication.

glands and their ducts. ii. Histological features are similar to retention type of
ii. Clinical features: mucous cyst with a definite epithelial lining present
l It occurs most frequently on the lower lip, but may sometimes.
also occur on the palate, cheek, tongue, and floor of iii. Treatment:
the mouth. l Marsupialization is done to unroof the gland.

l The extravasation type is far more common than re- l Excision of entire sublingual gland.

tention type of mucocele.


Q. 4. Plumbism.
l The superficial lesion appears as raised, bluish,

transluscent, circumscribed vesicle measuring sev- Ans.


eral mm to a cm in diameter.
i. Plumbism or lead poisoning occurs chiefly as an occu-
l Recurrence is common.
pational hazard, e.g. painters, lead workers, hair dyes,
iii. Histological features:
ointments, tinned foods and contaminated water.
l Histologically, the cavity of mucocele is not lined
ii. Clinical features:
by epithelium and its wall is made of compressed
l Serious gastrointestinal disturbances which include
fibrous connective tissue and fibroblasts.
nausea, vomiting, colic and constipation.
l The lumen is filled with an eosinophilic coagulum.
l Peripheral neuritis which may produce the charac-
iv. Treatment: Excision.
teristic “wrist-drop” or “foot-drop” phenomenon.
Q. 2. Sialolithiasis. l Blood changes are hypochromic anaemia with baso-

philic stippling of RBCs.


Ans. iii. Oral manifestations:
i. Sialolithiasis is also known as salivary duct stone or l Burtonian lines—grey/bluish black line occurs on

salivary duct calculus. marginal gingiva.


ii. It is formed by the deposition of calcium salts around a l Excessive salivation and metallic taste.

central nidus. iv. Treatment: Treatment of the oral lesions is secondary to


iii. Clinical features: systemic treatment.
l May occur at any age, but common in middle age
Q. 5. Amalgam tattoo.
group.
l Swelling and pain which is related to meal time. Ans.
l Stones may be palpable.
l Amalgam tattoo of oral mucous membrane is a rela-
l Occurs more frequently in submandibular duct and
tively common finding in dental practice.
glands.
iv. Diagnosis: By sialography.
v. Treatment: Aetiology
l Small calculi can be removed by manipulation or by
l Silver enters inside oral mucosa by:
increasing the salivation. The larger stones need to
a. Therapeutic use of AgNO3.
be removed by surgical exposure.
b. Locally due to condensation of amalgam in gingiva
l Peizoelectric shock wave lithotripsy.
during restoration work.
Q. 3. Ranula. c. From broken pieces introduced into socket during
extraction of restored tooth.
Ans.
d. From particles entering surgical wound during RCT
i. Ranula is retention type of mucocele arising from sub- with retrograde amalgam filling.
lingual or submandibular salivary gland.
ii. Clinical features:
Clinical Features
l It is unilateral and present mostly on the floor of

mouth and appears as frog (“rana”) belly, so named l Systemic treatment of silver nitrate causes permanent
as ranula. bluish grey pigmentation of skin and mucosa.
550 Quick Review Series for BDS 3rd Year

l Local causes may lead to amalgam tattooing in oral i. Traumatic cyst/solitary bone cyst/haemorrhagic bone
mucosa mostly in gingival area. They appear as black or cyst/simple bone cyst is a pseudo-cyst, not lined by epi-
olive brown granules. thelium.
l It lies above the mandibular canal.

l Associated teeth are vital.


Differential Diagnosis l Most common site is mandibular molar area.

l This may be confused with pigmentation due to malig- l It contains only air.

nant melanoma. l Treatment is surgical exploration to induce bleeding

in the lesion.
Q. 6. Pink’s disease.
Q. 9. Bruxism (night grinding).
Ans.
i. Acrodynia is also known as Pink disease or Swift’s Ans.
disease. Bruxism is habitual grinding or clenching of teeth either
ii. Aetiology: Mercurial toxicity reaction. during sleep or while awake.
iii. Clinical features:
l Age—more common in young infants ,2 years.

l ‘Raw beef’ appearance of skin. Aetiology


l Maculopapular rash
Occlusal disharmony, gastrointestinal disturbances, nutri-
l Severe sweating.
tional deficiencies, endocrine disturbance, emotional ten-
l Other features are extreme irritability, photophobia
sion or nervous tension.
with lacrimation, patchy loss of hair, etc.
iv. Oral findings:
l Profuse salivation. Effects of Bruxism
l Painful and swollen gingiva.
l Attrition, mobility or displacement of teeth
l Loosening and premature shedding of deciduous teeth.
l Receding gum
v. Treatment:
l Loss of alveolar bone
l Identify and remove the source of mercury.
l Temperomandibular joint dysfunction
l Immediate chelation therapy.

Q. 7. Osteoradionecrosis.
Treatment
Ans.
l Removal of cause
i. Osteoradionecrosis is a radiation-induced pathologic l Use of splints at night
process.
ii. Aetiology is unknown. Q. 10. Tooth ankylosis.
iii. Clinical features: Ans.
l Mandible is affected more commonly than maxilla.

l Three factors involved in its pathogenesis are: Ra- i. Ankylosis of tooth is the fusion of tooth with bone.
diation, trauma and infection. ii. Aetiology: Any traumatic episode (like occlusal trauma)
l After the infection has gained entry to the bone, or periapical inflammatory processes or after root canal
there may be necrosis of considerable amount of treatment.
bone, periosteum and overlying mucosa. iii. Clinical features:
l The devitalized bone may undergo sequestration. l Tooth shows lack of mobility.
iv. Histology: There is destruction of osteocytes, absence l There may be pulpal disease.

of osteoblasts and lack of new bone formation. l A characteristic solid sound on percussion over

v. Treatment: tooth.
l Debridment of necrotic tissue 1 removal of sequestrum. l Deciduous tooth, if affected, does not exfoliate and

l Administration of intravenous antibiotic and hyper- becomes submerged because of the eruption of the
baric oxygen therapy are essential. adjacent permanent teeth and growth of dental
l Oral hygiene maintenance. arch.
iv. Radiographic features: Blending of bone with tooth
Q. 8. Traumatic bone cyst.
root is seen in radiograph.
Ans. v. No treatment is required, prognosis is good.
Section | III Oral Pathology 551

Topic 13

Regressive Alterations of the Teeth

LONG ESSAYS
Q. 1. Describe briefly the regressive alterations of the l There is flattening of occlusal and proximal surfaces of
teeth. teeth.
l Eposure of dentin due to wearing of tooth surfaces.
Or
l Reduced cuspal height of teeth and in extreme cases,
Describe the reparative and regressive changes associated teeth are worn down to gum level.
with dentin and pulp. l There may be shortening of length of dental arch due
to reduction in mesiodistal diameter of teeth through
Ans.
proximal attrition.
l Regressive changes in dental tissues are considered to l Normally, the dentinal tubules may be irritated by these
be the lesions of retrograde nature. Various regressive processes and secretes secondary dentin in order to pro-
alterations of the teeth may be the result of general tect the teeth.
aging process of an individual or injury to the tissues or l Both deciduous and permanent teeth may be affected.
pathologic processes.
l The various regressive alterations of the teeth are as follows: Treatment: Aetiology should be sought out and removed.
i. Attrition Restorative treatment may be done to teeth.
ii. Abrasion
iii. Erosion ABRASION
iv. Abfraction
v. Dental sclerosis/transparent dentin Pathological wearing of tooth substance through some
vi. Dead tracts abnormal mechanical process, or loss by wear of dental
vii. Secondary dentin/irregular dentin tissue caused by abrasion by foreign substance, e.g. tooth-
viii. Pulp calcifications brush, dentrifrice.
ix. Resorption of teeth
x. Hypercementosis/cementum hyperplasia Aetiology
xi. Cementicles
l Wrong toothbrushing.
l Abrasive tooth powder, gritty toothpaste and clasps on
ATTRITION dentures.
Physiologic wearing away of the tooth surface or restora- l Wrong use of silk floss.
tion caused by tooth to tooth contact during mastication or l Habits, e.g. pipe smoking, opening of bobby pins and
parafunction. occupation of the patient, e.g. carpenters, shoe makers
or tailors, etc.

Aetiology
Clinical Features
l Mastication of food
l Abnormal chewing habits l Usually occurs at exposed root surfaces of the tooth
l Bruxism but under certain circumstances it may also be seen on
l Occlusal abnormalities. incisal or proximal surfaces.
l Usually located at cervical areas of teeth and is more
common on the left side of the mouth.
Clinical Features l Lesions are more wide than deep.
l The first clinical manifestation of attrition may be ap- l Exposed dentin is hard and appears highly polished.
pearance of small polished facets on a cusp tip or ridge l Premolars and cuspids are commonly affected.
or incisal edges. l In pipe smokers, surface is discoloured yellowish brown.
552 Quick Review Series for BDS 3rd Year

l May be sensitive to thermal changes, sweets, acids and l If excessive dietary intake of acidic foods or beverages
instrumentation. is discovered n patient education and counselling are
important.
Treatment: Remove the cause, advise proper toothbrushing
l If the patients has symptoms of GERD n They should
method and restorative treatment.
be referred to a medical doctor for complete evaluation
and therapy.
EROSION l A patient with salivary hypoplasia n Benefits from
use of sugarless chewing gum or mints to increase the
Erosion is defined as irreversible loss of dental hard tissue
salivary flow.
by a chemical process that does not involve bacteria.

Aetiology ABFRACTION
Extrinsic causes: Loss of tooth surface at the cervical areas of teeth caused
l Acidic beverages like fruit juices and soft drinks. by tensile and compressive forces during tooth flexure.
l Foods, medications, e.g. iron tonics with a low pH or
environmental acids. Clinical Findings
l Swimmers who workout regularly in pools with exces-
sive acidity. l Commonly affects single teeth with excursive interfer-
l Wine tasters ences or eccentric occlusal loads.
l Fumes in chemical factories where acids are used. l It affects buccal or labial cervical areas of teeth.
Intrinsic causes:
l Lower salivary flow rate (xerostomia). DENTAL SCLEROSIS (TRANSPARENT
l Gastric acid regurgitated into oesophagus and mouth: DENTIN)
Regurgitation of food from stomach, or chronic vomiting,
and this will bring gastric acid into mouth and cause ero- l It is characterized by calcification of dentinal tubules
sion of tooth, e.g. gastroesophageal reflux disease (GERD). which occurs as a result of injury to the dentin by
l Chronic excessive vomiting, e.g. anorexia nervosa or caries or abrasion or as a manifestation of normal
bulimia. aging process. The source of calcium for calcification
is probably the fluid (dental lymph) within the
tubules.
Clinical Findings l Decreased conductivity of odontoblastic processes as a
i. Broad concavities with smooth, highly polished, result of mineralization of the tooth.
scooped out depression on enamel adjacent to CEJ. l Shows an advancing carious process. Sclerotic dentin
ii. Wear on non-occluding surfaces. under carious lesion shown to be harder than adjacent
iii. Clean non-tarnished appearance of amalgams. normal dentin.
iv. Common site of occurrence. l It is regarded as a protective reaction in senile teeth,
l More often in maxillary than mandibular teeth.
sclerosis prevents irritation being transmitted from the
l Usually on the labial or buccal surfaces and at the
periphery of the tubules.
cervical margins.
l Erosion caused by vomiting typically affects palatal

surfaces of the maxillary teeth.


DEAD TRACTS
iii. In the maxilla, the incisors and in the manbidle, the l Dead tracts in dentin is an optical phenomenon due to
premolars are more often affected. differences in the refractive indices of the affected
iv. Pulp exposure in deciduous teeth and in severe cases of tubules and normal tubules.
adults. l The dentinal tubules remain uncalcified and are perme-
v. The surface is hard and polished, which is often hyper- able to penetration of dyes.
sensitive. l These occurring in the dentin appear as:
vi. React to tactile stimulation. a. Dark in transmitted light and
b. White in reflected light in ground sections
Dead tracts appear as a response to carious process, to
Treatment l

attrition and abrasion. The purpose is to protect the


l As a first step in the management of erosion, identifica- pulp. At the pulpal end of the tract, secondary dentin is
tion of the aetiology is important. laid down.
Section | III Oral Pathology 553

SECONDARY DENTIN/IRREGULAR l They greatly resemble secondary dentin as tubules are


less and irregular.
DENTIN
l They are more common in pulp chamber than in the root
l Secondary dentin is formed and deposited in response canal.
to a normal or slightly abnormal stimulus after the com- l True denticles may be subdivided according to their
plete formation of the tooth. attachment to the wall of pulp chamber as follows:
l It is also known as adventitious dentin or irritation den- a. Free denticles: Denticles which are not attached to
tin as it is stimulated by trauma, caries, attrition, etc. dentinal wall and lying entirely free within the pulp
l It is associated with normal aging process. tissue are called ‘free denticles’.
l It is localised exclusively adjacent to the irritated zone; b. Attached denticles: Denticles which are continuous
tubules are less in number and very irregular. with dentinal walls are called ‘attached denticles’.
l Secondary dentin is less mineralized and contains They are more common than free denticles.
6-10% less mineral than primary dentin.
l Its purpose is to protect the pulp. ii. False denticles:
l There are two types of secondary dentin: l They are composed of localised masses of the calcified
a. Physiological secondary dentin tissue and do not contain dentinal tubules.
b. Reparative secondary dentin l The nodule appears to be made up of concentric layers
or lamellae deposited around a central nidus.
a. Physiological secondary dentin: l They are more common in pulp chamber and are larger
l This type of dentin is a regular, uniform layer of dentin than true denticles.
around pulp chamber that is laid down throughout the l The false denticles may also be subdivided according to
life of the tooth as a result of physiological factors espe- their attachment to the wall of pulp chamber as:
cially age and tooth eruption. a. Free denticles or
l This is formed more slowly than primary dentin. b. Attached denticles

b. Reparative (adventitious) secondary dentin: iii. Diffuse calcifications:


l It is the dentin that forms around the pulp chamber as a l They are most commonly seen in root canals of teeth as
result of irritation or attrition, which is a form of tooth ‘calcific degenerates’.
wear. l Its usual pattern is in amorphous, unorganized linear
strands or columns paralleling the blood vessels and
Clinical Features nerves.

l It decreases tooth sensitivity by acting as additional Significance of pulp stones


insulating layer. l Not completely understood, nothing special.
l Protects pulp. l Pulp calcification is a purely coincidental finding without
l Allows large resortation to be made beyond the limits of clinical significance.
the original pulp chamber. l Difficulty may be encountered in extirpating pulp during
l Histologically: It is irregular in nature being composed root canal therapy.
of few tubules that may be tortuous in appearance. l No treatment indicated.

PULP CALCIFICATIONS RESORPTION OF TEETH:


Various forms of calcifications may be located in any por-
Resorption of calcified dental tissues occurs in same fash-
tion of the pulp tissue, certain calcifications are more com-
ion as that of bone. One of the main factors involved in
mon in the pulp chamber and others in the root canal.
tooth resorption is pressure. It is a cellular process in which
The two chief forms of pulp calcifications are:
osteoclasts take active part.
A. Discrete pulp stones (denticles, pulp nodules)
The chief causes or situations in which tooth resorption
B. Diffuse calcification
can occur are classified as follows:
Depending on their microscopic structure, the pulp
stones have been classified as either true or false stones.
A. External Resorption
i. True denticles: a. Periapical inflammation
l They are made up of localised masses of the calcified b. Reimplantation of teeth
tissue and resemble dentin due to their tubular structure. c. Tumours and cysts
554 Quick Review Series for BDS 3rd Year

d. Impaction of teeth Clinical Features


e. Traumatic occlusion
l No clinical signs and symptoms.
f. Excessive mechanical or occlusal forces like heavy orth-
odontic forces
g. Idiopathic Radiological Features
Larger root with blunting of root apex.
B. Internal Resorption (Pink Tooth)
a. Idiopathic: Histological Features
l Internal resorption is also known as chronic perforat-
Secondary or cellular cementum is deposited over primary
ing hyperplasia of pulp, odontoclastoma or pink tooth
acellular cementum, this is arranged in concentric layers
of Mummery and is due to inflammatory hyperplasia
and termed as osteocementum.
of the pulp.

HYPERCEMENTOSIS (CEMENTUM Treatment


HYPERPLASIA) No treatment is indicated as the condition itself is innocuous.
l Hypercementosis is a non-neoplastic condition in which
excessive cementum is deposited in continuation with CEMENTICLES
the normal radicular cementum.
Cementicles are small foci of calcified tissue, not necessar-
l It is a regressive change of teeth characterized by the
ily true cementum which lie free in periodontal ligaments
deposition of excessive amounts of secondary cemen-
of lateral or apical root areas.
tum on the root surface.
l Generally, whole root surface is involved. Sometimes
only apical area is involved. Aetiology
l It may occur either in a generalized form involving all
Cementicles may arise from:
the teeth or in a localized form involving only one tooth.
l Calcification of epithelial rests of Malassez in the peri-
odontal ligament as a result of degenerative change.
Aetiology l Calcification of connective tissues between Sharpey’s
bundles with no apparent central nidus.
i. Accelerated elongation of teeth: When a tooth is lost,
l Calcification of thrombosed capillaries in periodontal
the tooth in opposite arch is elongated due to deposition
ligament.
of cementum at root apex.
l Small spicules or broken pieces of cementum or alveo-
ii. Mild periapical infection
lar bone may be present in periodontal ligaments, i.e.
iii. Periodontitis: In this, hypercementosis occurs at some
cemental tears resemble cementicles.
distance away from the root apex.
iv. Functionless teeth, embedded or impacted teeth.
v. Tooth repair: Resorption due to occlusal trauma is re- Radiographic Features
paired by cementum deposition, fracture of the root is
Usually not visible in intraoral radiographs due to too
repaired due to deposition of cementum at fractured site.
smaller size (0.2-0.3 mm).
vi. Osteitis deformans or Paget’s disease of bone.
In Paget’s disease, generalised deposition of secondary ce-
Treatment
mentum at the roots occurs and apparent disappearance of
lamina dura. No treatment is needed.

SHORT ESSAYS
Q. 1. Hypercementosis.
surfaces of the tooth root due to excessive cemento-
Ans. genesis.
i. Hypercementosis is also known as cementum hyperplasia iii. Generally, whole root surface is involved. Sometimes
ii. Apposition of excessive amount of cementum at root cementum formation is focal involving only apical area
surface. It increases the thickness of cementum on of a tooth.
Section | III Oral Pathology 555

Aetiology l Treatment of primary condition is obviously necessary


in cases where hypercementosis is due to inflammation
Various circumstances favour the deposition of excessive
of pulpal origin.
amounts of cementum which are as follows:
i. Accelerated enlogation of a tooth owing to loss of Q. 2. Age changes in the pulp.
antagonist.
Ans.
ii. Aging
iii. Inflammation about a tooth Age changes or regressive changes of pulp mean aging of
l Pulpal infection pulp. The various degenerative or regressive changes of the
l Periapical inflammation pulp include:
l Periodontitis A. Reticular atrophy of pulp
iv. Mechanical stimulation B. Pulpal calcification
l Functionless or unerupted tooth

v. Tooth repair:
l Resorption due to occlusal trauma is repaired by
A. RETICULAR ATROPHY OF PULP
cementum deposition. Clinical Features
l Fracture of the root is repaired by deposition of
cementum between the root fragments. l It is age-related change in the pulp with decreased sen-
l Cemental tears
sitivity to various stimuli.
vi. Osteitis deformans or Paget’s disease of bone. l Affected teeth are asymptomatic and respond normally
vii. Excessive occlusal trauma. to vitality tests.
Histological features:
Clinical Features l Few stellate cells are present along with increased intra-
cellular fluid.
l No clinical signs and symptoms. l Large vacuolization areas are also seen in pulp.
l The involved teeth are completely asymptomatic. l Degeneration and disappearance of the odontoblasts.
l When the tooth with hypercementosis is extracted the
roots appear larger in diameter than normal and present
rounded apices. B. Pulp Calcifications
Various forms of calcifications may be located in any
Roentgenographic Features portion of the pulp tissue, certain calcifications are more
l Radiograph shows excessive cemental thickening with common in the pulp chamber and others in the root
typical rounding off of the root apex. canal.
l On an IOPA radiograph, most cases of hypercementosis The two chief forms of pulp calcifications are:
are distinguished by the thickening and apparent blunt- a. Discrete pulp stones (denticles, pulp nodules)
ing of the roots. b. Diffuse calcifications

a. Pulp stones:
Histological Features Depending on their microscopic structure, the pulp stones
l Hypercementosis exhibits a characteristic appearance of have been classified as either true or false stones.
excessive amount of secondary or cellular cementum
deposited directly over the typically thin layer of pri- i. True denticles:
mary acellular cementum. l They are made up of localised masses of the calcified
l The area involved may be the entire root or only the tissue and resemble dentin due to their tubular
apical region. structure.
l The secondary cementum also known as ‘osteocementum’ l They greatly resemble secondary dentin as tubules are
is arranged in the concentric layers around the root and fre- less and irregular.
quently shows numerous resting lines indicated by deeply l They are more common in pulp chamber than in the root
staining hematoxyphilic lines parallel to the root surface. canal.
l True denticles may be subdivided according to their
attachment to the wall of pulp chamber as follows:
Treatment a. Free denticles: Denticles which are not attached to
l No treatment is indicated for teeth exhibiting hyperce- dentinal wall and lying entirely free within the pulp
mentosis since the condition is itself innocuous. tissue are called ‘free denticles’.
556 Quick Review Series for BDS 3rd Year

b. Attached denticles: Denticles which are continuous Q. 4. Attrition, abrasion and erosion.
with dentinal walls are called ‘attached denticles’.
Ans.
They are more common than free denticles.

ii. False denticles: ATTRITION


l They are composed of localised masses of the calcified
Physiologic wearing away of the tooth surface or restora-
tissue and do not contain dentinal tubules.
tion caused by tooth to tooth contact during mastication or
l The nodule appears to be made up of concentric layers
parafunction.
or lamellae deposited around a central nidus.
Aetiology: *Mastication of food, abnormal chewing
l They are more common in pulp chamber and are larger
habits, bruxism and occlusal abnormalities.
than true denticles.
l The false denticles may also be subdivided according to
their attachment to the wall of pulp chamber as: Clinical Features
a. Free denticles or
b. Attached denticles l The first clinical manifestation of attrition may be ap-
pearance of small polished facets on a cusp tip or ridge
b. Diffuse calcifications: or incisal edges.
l There is flattening of occlusal and proximal surfaces
l They are most commonly seen in root canals of teeth as
of teeth.
‘calcific degenerates’.
l Exposure of dentin due to wearing of tooth surfaces.
l Its usual pattern is in amorphous, unorganized linear
l Reduced cuspal height of teeth and in extreme cases,
strands or columns paralleling the blood vessels and
teeth are worn down to gum level.
nerves.
l Both deciduous and permanent teeth may be

Significance of pulp stones: affected.


l Not completely understood, nothing special.
Treatment: Aetiology should be sought out and removed.
l Pulp calcification is a purely coincidental finding with-
Restorative treatment may be done to teeth.
out clinical significance.
l Difficulty may be encountered in extirpating pulp dur-
ing root canal therapy. ABRASION
l No treatment indicated.
Pathological wearing of tooth substance through some
Q. 3. Pink tooth of Mummery. abnormal mechanical process. Or loss by wear of dental
tissue caused by abrasion by foreign substance, e.g. tooth-
Ans.
brush, dentrifrice.
i. Internal resorption is also known as chronic perforating
hyperplasia of pulp, odontoclastoma or pink tooth of
Mummery. Aetiology
ii. It is an unusual form of resorption that begins centrally l Wrong toothbrushing, abrasive tooth powder, gritty
within the pulp, apparently initiated by a peculiar in- toothpaste and clasps on dentures.
flammatory hyperplasia of the pulp. l Habits like pipe smoking and occupation of the patient,
iii. Aetiology: Idiopathic e.g. carpenters, shoe makers or tailors, etc.
iv. Clinical features:
l No early clinical signs and symptoms.
l Tooth may show pink spot (pink tooth) when more Clinical Features
of dentin is resorbed from one area of the crown,
l Usually occurs at exposed root surfaces of the tooth.
leaving a covering of translucent enamel.
l Usually located at cervical areas of teeth and is more
l It appears as a pink area due to vascular pulp visible
common on the left side of the mouth.
through the translucent enamel.
l Lesions are more wide than deep.
v. X-ray appearance: Pink spot appears as round or ovoid
l Exposed dentin is hard and appears highly polished.
area of radiolucency in the central portion of the
l Premolars and cuspids are commonly affected.
tooth.
l May be sensitive to thermal changes, sweets, acids and
vi. Treatment: If condition is discovered before perforation
instrumentation.
of crown, root canal therapy may be carried out. Once
perforation has occurred, extraction of tooth is the treat- Treatment: Remove the cause, advise proper toothbrushing
ment. method and restorative treatment.
Section | III Oral Pathology 557

EROSION in tooth resorption is pressure. It is a cellular process in


which osteoclasts take active part.
Erosion is defined as irreversible loss of dental hard tissue
l The tooth resorption may be classified as follows:
by a chemical process that does not involve bacteria.
A. External resorption
B. Internal resorption (pink tooth)
Aetiology
A. External resorption:
Extrinsic causes:
Causes:
l Acidic beverages like fruit juices and soft drinks.
i. Periapical inflammation
l Foods, medications, e.g. iron tonics with a low pH or
ii. Reimplantation of teeth
environmental acids.
iii. Tumours and cysts
l Swimmers who work-out regularly in pools with exces-
iv. Impaction of teeth
sive acidity.
v. Traumatic occlusion
l Wine tasters
vi. Excessive mechanical or occlusal forces like heavy
l Fumes in chemical factories where acids are used.
orthodontic forces
vii. Idiopathic:
Intrinsic causes:
l Resorption of calcified dental tissues occurs in same
l Lower salivary flow rate (xerostomia).
fashion as that of bone.
l Gastric acid regurgitated into oesophagus and mouth:
l One of the main factors involved in tooth resorption
Regurgitation of food from stomach, or chronic vomit-
is pressure.
ing, and this will bring gastric acid into mouth and
l It is a cellular process in which osteoclasts take
cause erosion of tooth, e.g. gastroesophageal reflux dis-
active part.
ease (GERD).
l External resorption can create singnificant defects
l Chronic excessive vomiting, e.g. anorexia nervosa or
in the crown of teeth before resorption.
bulimia.
l The resorption can extend apically into the pulp or

coronally under the enamel.


Clinical Findings l The cervical pattern of external resorption often is

rapid and has been termed invasive cervical resorption.


i. Broad concavities with smooth, highly polished,
l Histologically, it consists of numerous multinucle-
scooped out depression on enamel adjacent to CEJ.
ated dentinoclasts located in the areas of structure
ii. Wear on nonoccluding surfaces.
loss. Areas of resorption are repaired by the deposi-
iii. Common site of occurrence:
tion of osteodentin.
l More often in maxillary than mandibular teeth.
l Extensive bony replacement in areas of external
l Usually on the labial or buccal surfaces and at the
resorption can lead to ankylosis.
cervical margins.
l Treatment: The first step is identification and elimi-
iv. In the maxilla, the incisors and in the mandible, the
nation of causative and accelerating factors.
premolars are more often affected.
l Cases located in the cervical areas can be treated by
v. Pulp exposure in deciduous teeth and in severe cases of
surgical exposure, removal of all soft tissue from
adults.
the defects and restoration of the lost structure of
vi. The surface is hard and polished, which is often hyper-
the tooth.
sensitive.
B. Internal resorption (pink tooth):
Treatment a. Idiopathic:
i. Internal resorption is also known as chronic perforating
l Identification of the aetiology.
hyperplasia of pulp, odontoclastoma or pink tooth of
l Patient education and counselling are important regard-
Mummery.
ing dietary intake of acidic foods or beverages.
ii. It is an unusual form of resorption that begins centrally
l Use of sugarless chewing gum or mints to increase the sali-
within the pulp, apparently initiated by a peculiar
vary flow is beneficial in patients with salivary hypoplasia.
inflammatory hyperplasia of the pulp.
Q. 5. Classify resorption of teeth. Describe external and iii. Aetiology: Idiopathic
internal resorption of teeth. iv. Clinical features:
l No early clinical signs and symptoms.
Ans.
l Tooth may show pink spot (pink tooth) when more
l Resorption of calcified dental tissues occurs in same of dentin is resorbed from one area of the crown,
fashion as that of bone. One of the main factors involved leaving a covering of translucent enamel.
558 Quick Review Series for BDS 3rd Year

lIt appears as a pink area due to vascular pulp visible vi. Treatment:
through the translucent enamel. l If condition is discovered before perforation of crown,
v. X-ray appearance: Pink spot appears as round or ovoid root canal therapy may be carried out.
area of radiolucency in the central portion of the l Once perforation has occurred, extraction of tooth is
tooth. the treatment.

SHORT NOTES
Q. 1. Erosion. Q. 3. Abrasion.
Ans.
Ans.
i. Pathological wearing of tooth substance through some
Erosion of dental tissues is defined as irreversible loss of abnormal mechanical process or loss by wear of
dental hard tissue by a chemical process that does not dental tissue caused by abrasion by foreign substance,
involve bacteria. e.g. toothbrush, dentrifrice.
i. Aetiology: ii. Causes:
l Chemicals or acids, acid fruit juices and soft drinks,
l Wrong toothbrushing, abrasive tooth powder, gritty
chronic vomiting, habitual taking of acidic medi- toothpaste, clasps on dentures, etc.
cines or iron tonics with a low pH, etc. l Occupations where tools, nails, etc. are held
ii. Clinical findings: between the teeth and habits like pipe smoking.
l Broad concavities with smooth, highly polished,
iii. Site of occurrence:
scooped out depression on enamel adjacent to l More often in maxilla than in mandible.
CEJ. l Labial and buccal surfaces more often affected.
l Occurs more often in maxillary teeth usually on
iv. Clinical findings:
the labial or buccal surfaces and at the cervical l In toothbrush abrasion, V-shaped groove or shallow,
margins. saucer-shaped surface develops on the labial aspect
l Pulp exposure in deciduous teeth and in severe cases
of the cervical margins of teeth.
of adults. l Exposed dentine is hard and highly polished.
l React to tactile stimulation.
l May be sensitive to thermal changes, sweets, acids
iii. Treatment: and instrumentation.
l Identification of the aetiology.
v. Treatment: Remove the cause, advise about proper
l Patient education and counselling, a medical con-
toothbrushing method, restorative treatment.
sultation for complete evaluation and therapy,
use of sugarless chewing gum or mints to increase Q. 4. Cementicles.
the salivary flow are some of the treatment Ans.
modalities.
i. Cementicles are small foci of calcified tissue, not nec-
Q. 2. Attrition. essarily true cementum which lie free in periodontal
ligaments of lateral or apical root areas.
Ans. ii. Aetiology: Cementicles may arise from:
l Calcification of epithelial rests of Malassez in the
i. Physiologic wearing away of the tooth surface or resto-
ration caused by tooth to tooth contact during mastica- periodontal ligament.
l Calcification of connective tissues between Sharpey’s
tion or parafunction.
ii. Aetiology: Mastication of food, abnormal chewing hab- bundles.
l Calcification of thrombosed capillaries in periodon-
its, bruxism, occlusal abnormalities.
iii. Clinical features: tal ligament.
l Appearance of small polished facets on a cusp tip or
iii. Radiographic features: Usually not visible in intraoral
ridge or incisal edges. radiographs due to too smaller size (0.2-0.3 mm).
l Exposure of dentin due to wearing of tooth surfaces.
iv. Treatment: No treatment is needed.
l Reduced cuspal height of teeth and in extreme cases, Q. 5. Hypercementosis.
teeth are worn down to gum level.
Ans.
l Both deciduous and permanent teeth may be

affected. i. Hypercementosis is a non-neoplastic condition in


iv. Treatment: Aetiology should be sought out and re- which excessive cementum is deposited in continuation
moved. Restorative treatment may be done to teeth. with the normal radicular cementum.
Section | III Oral Pathology 559

ii. Aetiology: Accelerated elongation of teeth, mild peri- iv. Clinical features:
apical infection, functionless teeth, embedded or im- l No early clinical signs and symptoms.

pacted teeth, tooth repair, osteitis deformans or Paget’s l Tooth may show pink spot (pink tooth) when more

disease of bone. of dentin is resorbed from one area of the crown,


iii. Clinical features: No significant clinical signs and leaving a covering of translucent enamel.
symptoms. l It appears as a pink area due to vascular pulp visible

iv. Radiological features: Larger root with blunting of root through the translucent enamel.
apex. v. X-ray appearance: Pink spot appears as round or ovoid
v. Treatment: No treatment is indicated as the condition area of radiolucency in the central portion of the tooth.
itself is innocuous. vi. Treatment: If condition is discovered before perforation
of crown, root canal therapy may be carried out. Once
Q. 6. Secondary dentin. perforation has occurred, extraction of tooth is the treat-
ment.
Ans.
Q. 8. Factors causing external resorption of teeth.
i. Secondary dentin is formed and deposited in response
to a normal or slightly abnormal stimulus after the Ans.
complete formation of the tooth. The factors causing external resorption of teeth are as follows:
ii. It is also known as adventitious dentin or irritation a. Chronic periapical inflammation
dentin as it is stimulated by trauma, caries, attrition, b. Reimplantation of teeth
etc. c. Tumours and cysts
iii. It is associated with normal aging process. d. Impaction of teeth
iv. It is localised exclusively adjacent to the irritated e. Traumatic occlusion
zone; tubules are less in number and very irregular and f. Excessive mechanical or occlusal forces like heavy orth-
is less mineralized. odontic forces
v. Its purpose is to protect the pulp. g. Root canal filling that protrudes beyond the apex of the
vi. There are two types of secondary dentin: tooth.
a. Physiological secondary dentin h. Idiopathic
b. Reparative secondary dentin
vii. It decreases tooth sensitivity by acting as additional Q. 9. Dentinal sclerosis (transperant dentin)
insulating layer and protects pulp Ans.
viii. Histologically: It is irregular in nature being
composed of few tubules that may be tortuous in i. Dentinal sclerosis (transperant dentin) is a highly calci-
appearance. fied dentin than normal dentin occurring as a response
to slowly progressive caries and also to attrition, or
Q. 7. Pink tooth of Mummery. abrasion of the crown or as a manifestation of normal
aging process.
Ans.
ii. Decreased conductivity of odontoblastic processes as a
i. Internal resorption is also known as chronic perforating result of mineralization of the tooth.
hyperplasia of pulp, odontoclastoma or pink tooth of iii. Shows an advancing carious process. Sclerotic dentin
Mummery. under carious lesion shown to be harder than adjacent
ii. It is an unusual form of resorption that begins centrally normal dentin.
within the pulp, apparently initiated by a peculiar in- iv. It is regarded as a protective reaction in senile teeth,
flammatory hyperplasia of the pulp. sclerosis prevents irritation being transmitted from the
iii. Aetiology: Idiopathic. periphery of the tubules.
560 Quick Review Series for BDS 3rd Year

Topic 14

Healing of Oral Wounds

LONG ESSAYS
Q. 1. Write briefly about healing of extraction wound l New delicate capillaries penetrate to the centre of the clot.
and mention about its complications. l The remnants of PDL gradually undergo degeneration
and are no longer recognizable.
Ans.
l The wall of boney socket now appears slightly frayed.
The average sequence of events in healing process of an In some instances, trabeculae of osteoid can be seen
extraction wound are as follows: extending outwards from the wall of the alveolus.
l Margins of alveolar socket exhibit prominent osteoclas-
i. Immediate reaction following an extraction: tic resorption and fragments of necrotic bone which
l The first stage in the healing of an extraction wound is may have been fractured from the rim of the socket dur-
the formation of the blood clot. ing extraction, are seen in the process of resorption or
l After the removal of tooth, the socket is filled with sequestration.
blood which coagulates, red blood cells being entrapped
fibrin strands and the ends of torn blood vessels in the iv. Third week wound:
periodontal ligament become sealed off. l The clot is completely organized by maturation of
l Various phenomena consisting of alterations in vascular granulation tissue.
bed occur during first 24-48 hours after extraction. l Very young trabeculae of osteoid or new uncalcified
l Blood vessels immediately surrounding the wound are bone is formed around the periphery of wound from the
dilated and mobilization of leucocytes to the immediate socket wall.
area around the clot. l Original cortical bone of alveolar socket undergoes
l The surface of blood clot is covered by a thick layer of remodelling.
fibrin. The clot itself shows areas of contraction. l Crest of alveolar bone is rounded off by osteoclastic
l The collapse of unsupported gingival tissue into the resorption.
opening of a fresh extraction wound is of great aid in l By this time, surface of wound may have become
maintaining the clot in position. completely epithelialized.

ii. Healing in first week wound: v. Fourth week wound:


l There is proliferation of fibroblasts from connective tis- l During the fourth week after extraction, the wound be-
sue and these fibroblasts grow into the clot around the gins the final stage of healing.
entire periphery. l There is continuous deposition, remodelling resorption
l There is endothelial proliferation which signals the be- of the bone filling the alveolar socket.
ginning of capillary ingrowth. l Due to resorption of alveolar crest the bone filling, the
l Crest of alveolar bone shows beginning of osteoclastic socket does not extend above the alveolar crest, hence
activity. obviously the crest of the healed socket is below that of
l During this period, blood clot begins to undergo organi- adjacent teeth.
zation by in growth of fibroblasts and occasionally by
small capillaries from residual periodontal ligament. The major complications of healing of extraction wounds
l An extremely thick layer of leucocytes forms over the are as follows:
surface of the clot and the edge of the wound continues i. Dry socket
to exhibit epithelial proliferation. ii. Fibrous healing of extraction wound.

iii. Healing of second week wound:


DRY SOCKET (ALVEOLAR OSTEITIS)
l During this period, the blood clot becomes organized by
fibroblasts growing into the clot on the fibrinous mesh l The dry socket or alveolar osteitis is the most common and
work. the painful complication in the healing of extraction wounds.
Section | III Oral Pathology 561

l It is basically a focal osteomyelitis in which the blood Prevention of Dry Socket


clot has been disintegrated or lost with the production
l Doing extractions as gently as possible.
of foul odour and severe pain of throbbing type but no
l Instruct the patient not to rinse for 24 hours.
suppuration.
l Prescribe vit B complex with vit C.
l The condition derives its name from the fact that after
the clot is lost the socket has a dry appearance due to the
exposed bone. Treatment
l The socket is irrigated with hot normal saline solution
Aetiology thoroughly.
l Remove any loose spicules and sharp edges of bone.
l Excessive trauma during extraction of tooth. l Socket is dressed with a material containing obtundent
l Pre-existing low grade infection. gently and loosely.
l Lack of bleeding soon after extraction or diminished l Dressing is repeated until healthy granulation tissue
local blood supply appears and penicillin is prescribed.
l Too early and vigorous mouthwashing after extraction.
l Vitamins B and C deficiencies, diabetes mellitus, blood
dyscrasias, etc. FIBROUS HEALING OF EXTRACTION
WOUND
It is an uncommon complication of extraction wound
Pathophysiology l

usually following a difficult, complicated or surgical


l Destruction of the clot is caused by the action of proteo- extraction of tooth.
lytic enzymes produced by the bacteria or local fibrino- l It occurs most frequently following the tooth extraction
lytic activity. accompanied by loss of both lingual and labial or buccal
l Fibrinolytic activity is currently thought to be respon- cortical plates and periosteum.
sible for premature clot loss and severe pain in dry l Exact aetiology is unknown.
socket. l The lesion is generally asymptomatic.
l Clot lysis occurs by two mechanisms: l Excision of the lesion.
a. Plasminogen dependent pathway as proposed by
Birn and Q. 2. Describe the healing of fracture wound and
b. Plasminogen independent pathway. complication in healing of the fracture.
l Plasminogen is hepatically synthesized and released in Ans.
to circulation.
The fractures of jaw are common injuries and range from
Plasminogen minor alveolar process fracture to destructive injuries of
↓ maxillofacial region.
Plasmin The sequence of events in healing of fracture wound is as
↓ acts on follows:
Fibrinogen and fibrin → Clot dissolution.
i. Immediate effects of a fracture:
l Anaerobic microorganisms may also play a significant l After fracture, haversian vessels of the bone are torn at
role in the development of this condition. the fracture site as are the vessels of periosteum and
marrow cavity that happen to cross the fracture line.
There is considerable extravasation of blood in frac-
Clinical Findings l

tured area but at the same time there is lack of circula-


l More common in the women and tobacco users. tion and loss of blood supply.
l It usually starts by the second or third postoperative day l Due to disruption of blood supply and tearing of blood ves-
and lasts for 7-10 days. sels, there is death of bone marrow adjacent to fracture line.
l Extremely painful. l Circulation actually stops as proximal to the fracture
l It is associated with low grade fever and ipsilateral site as there is anastomosis of undamaged vessels.
lymphadenopathy. l The blood clot which forms once thought to play an
l No pus formation but foul odour. important role in healing of fracture through replacement
l The ‘dry socket’ usually starts by second or thitd by granulation tissue and its subsequent replacement
postoperative day and lasts for 7-10 days. of bone.
562 Quick Review Series for BDS 3rd Year

l Most authorities now feel that role of blood clot in healing l This differentiation of cells into osteoblast and subse-
process is only a passive one and is not necessary for inva- quent formation of bone occurs in deepest part of callus
sion of osteogenic cells and the newly forming bone and collar.
the callus forms outside the granulation tissue replacement. l In rapidly growing area of collar, varying number of
cells of the osteogenic layer differentiates into chondro-
ii. Callus formation: blasts rather than osteoblasts and forms cartilage.
l The callus is the structure which unites the fracture ends l This cartilage cells begins to mature and the cartilage
of bone and consists of fibrous tissue, cartilage and bone. begins to calcify in a fashion similar to endochondral
l There are two types of callus: bone formation. The calcified cartilage is gradually
a. External callus: It consists of new tissue which resorbed and replaced by the bone.
forms around the outside of two fragments of bone. l Shortly after the fracture occurs, endosteum begins to
b. Internal callus: It consists of new tissue arising from proliferate and within a week or two begins formation
the marrow cavity. of new bone and cartilage which unites and establishes
l Preservation of periosteum is essential as it is an impor- the continuity of the bone.
tant structure in callus formation and ultimate healing of
the fracture. iii. Remodelling of the callus:
l Cells of the periosteum torn at the fracture line usually l After the callus formation unites, the two fragments of
die but peripheral to this area there is a flurry of cellular bone, the external and internal callus remodelling
activity within hours after the injury. occurs to form indistinguishable bone.
l The outer or fibrous layer of periosteum is inert and is
actually lifted away from the surface of bone by prolif-
Complications of Fracture Healing
eration of cells in osteogenic or inner layer of perios-
teum which assumes features of osteoblasts which, in i. Delayed union and non-union of the fragments of bones
turn, begins the formation of small amount of new bone are the complication of the healing process. They result
at some distance from the fracture site. when the calluses of osteogenic tissue over each of two
l There is continuous proliferation of osteogenic cells fragments fail to meet and fuse.
forming collar of callus over the surface of fracture. ii. Fibrous union in fracture arises usually as a result of
l The new bone which begins to form in the external cal- lack of immobilization of damaged bone. The fracture
lus consists of irregular trabeculae often laid down at ends of fragments are united by fibrous tissue as there
right angles to the surface. is failure of ossification.

SHORT ESSAYS
Q. 1. Dry socket. l Too early and vigorous mouthwashing after extraction.
l Vitamin B and C deficiencies, diabetes mellitus, blood
Ans.
dyscrasias, etc.
l The dry socket or alveolar osteitis is the most common
and the painful complication in the healing of extraction
wounds. Pathophysiology
l It is basically a focal osteomyelitis in which the blood clot
has been disintegrated or lost with the production of foul l Destruction of the clot is caused by the action of proteo-
odour and severe pain of throbbing type but no suppuration. lytic enzymes produced by the bacteria or local fibrinolytic
l The condition derives its name from the fact that after activity.
the clot is lost the socket has a dry appearance due to the l Fibrinolytic activity is currently thought to be respon-
exposed bone. sible for premature clot loss and severe pain in dry
socket.
Clot lysis occurs by two mechanisms:
Aetiology l

a. Plasminogen-dependent pathway as proposed by


l Excessive trauma during extraction of tooth. Birn and
l Pre-existing low grade infection. b. Plasminogen independent pathway.
l Lack of bleeding soon after extraction or diminished l Plasminogen is hepatically synthesized and released
local blood supply. into circulation.
Section | III Oral Pathology 563

Plasminogen Technique of exfoliative cytology:


↓ l The surface of lesion is cleaned by removing all debris
Plasmin and mucins.
↓ acts on l Vigorously scrapping the entire surface of lesion several
Fibrinogen and fibrin → Clot dissolution. times with a metal cement spatula or a moistened
tongue blade or a cytobrush.
l Anaerobic microorganisms may also play a significant l The collected material is then quickly and evenly spread
role in the development of this condition. over a microscopic slide and is immediately fixed with
95% of alcohol.
The slide is then air-dried and is stained by a special
Clinical Findings l

stain called as PAP stain (Papanicolaou stain).


l More common in the women and tobacco users.
l It usually starts by the second or third postoperative day Findings in exfoliative cytology as reported by cytologist
and lasts for 7-10 days. fall into one of the following five classes:
l Extremely painful. Class I (normal): lt indicates that only normal cells were
l It is associated with low grade fever and ipsilateral present in the smear.
lymphadenopathy. Class II (atypical): It indicates the presence of minor
l No pus formation but foul odour. cellular atypia but no evidence of malignant changes.
l The ‘dry socket’ usually starts by second or thitd post- Class III (intermediate): This is an inbetween cytology that
operative day and lasts for 7-10 days. separates cancer from non-cancer diagnosis, the cells display
wider atypia that may be suggestive of cancer, but they are
not clear cut and may represent precancerous lesions or car-
Prevention of Dry Socket cinoma in situ. Biopsy is recommended for further diagnosis.
Class IV (suggestive cancer): It indicates that in the lesion there
l Doing extractions as gently as possible.
is presence of few cells with malignant characteristics or many
l Instruct the patient not to rinse for 24 hours.
cells with borderline characteristics. Biopsy is mandatory.
l Prescribe vit B complex with vit C.
Class V (positive for cancer): The cells exhibit definite fea-
tures of malignancy. Biopsy is mandatory.
Treatment Q. 3. Biopsy.
l The socket is irrigated with hot normal saline solution
Ans.
thoroughly.
l Remove any loose spicules and sharp edges of bone. Biopsy is a procedure of removing a tissue from a living
l Socket is dressed with a material containing obtundent person for histopathological examination and for diagnosis.
gently and loosely.
Indications for biopsy are as follows:
l Dressing is repeated until healthy granulation tissue
i. Whenever there is any doubt about a lesion and it
appears and penicillin is prescribed.
cannot be diagnosed clinically.
Q. 2. Exfoliative cytology. ii. When a lesion is in an early stage and shows only a
superficial ulceration or swelling.
Ans.
iii. When a lesion in the oral cavity does not heal.
Exfoliative cytology is the microscopic study of cells which iv. When a lesion increases in size, despite therapy.
exfoliate or abrade from the surface of an organ or lesion v. When the lesion is unusual and not diagnosable.
after suitable staining. vi. Diagnosis by biopsy is particularly important in early
ulcerations and neoplasms.
Indications of Exfoliative Cytology
Types of Biopsy
The exfoliative cytology is used in establishing the diagnosis
of following oral lesions: i. Excisional biopsy: If a lesion is totally excised for
i. Herpes simplex and herpes zoster histological evaluation, it is called as excisional biopsy.
ii. Pemphigus vulgaris and pemphigoid ii. Incisional biopsy: When only a small section of tissue
iii. Squamous cell carcinoma is removed from the lesion for the purpose of histologi-
iv. Apthous ulcer, etc. cal evaluation, it is called incisional biopsy.
564 Quick Review Series for BDS 3rd Year

iii. Fine needle aspiration cytology (FNAC): It is done by ii. Healing in first week wound:
aspirating tissue material inside a lesion which is later l There is proliferation of fibroblasts from connective
on diagnosed microscopically after preparing a smear. tissue.
iv. Frozen section biopsy: It is performed in order to get an l There is endothelial proliferation which signals the
immediate histological report of a lesion. The tissue is beginning of capillary ingrowth.
obtained from the lesion and is kept in deep freeze and l Crest of alveolar bone shows beginning of osteoclastic
then frozen tissue is sectioned and stained to get a activity.
prompt diagnosis. l During this period, blood clot begins to undergo organi-
v. Punch biopsy: zation.
l A punch is used to remove a circular piece of the l An extremely thick layer of leucocytes forms over the
tissue that are to be examined. surface of the clot and the edge of the wound continues
l It is convenient to remove a piece of the tissue, to exhibit epithelial proliferation.
particularly from posterior oropharyngial region.
iii. Healing of second week wound:
Biopsy Technique l During this period, the blood clot becomes.
l New delicate capillaries penetrate to the centre of the
Following points should be considered to ensure obtaining
clot.
a proper specimen during biopsy technique:
l The remnants of PDL gradually undergo degeneration
l Do paint the surface of the area to be biopsied with
and are no longer recognizable.
iodine or highly coloured antiseptic.
l The wall of boney socket now appears slightly frayed.
l Inject 0.5 to 2 ml of local anaesthetic solution with adrena-
In some instances, trabeculae of osteoid can be seen
line around the periphery of lesion but not into the lesion.
extending outwards from the wall of the alveolus.
l Use a sharp scalpel to avoid tearing tissue.
l Margins of alveolar socket exhibits prominent osteo-
l Remove a border of normal tissue with the specimen, if
clastic resorption.
at all possible.
l Use care not to mutilate the specimen when holding it iv. Third week wound:
with the forceps.
l Fix the tissue immediately upon removal in 10% l The clot is completely organized by maturation of
formalin or 70% alcohol. If the specimen is thin, place granulation tissue.
it upon a piece of glazed paper and drop it into fixative l Very young trabeculae of osteoid or new uncalcified
to prevent curling of tissues. bone is formed around the periphery of wound from the
l With the help of scalpel 4 to 8 mm piece of the lesion is socket wall.
removed and put in a bottle containing 10% solution of l Original cortical bone of alveolar socket undergoes
formalin and sent to lab. remodelling.
l The biopsy specimen is sent to the histopathologist for l Crest of alveolar bone is rounded off by osteoclastic
diagnosis after labelling it properly. resorption.
l By this time, surface of wound may have become
Q. 4. Healing of extraction socket. completely epithelialized.
Ans.
v. Fourth week wound:
The average sequence of events in healing process of an l During this period, the wound begins the final stage of
extraction wound are as follows: healing.
i. Immediate reaction following an extraction: l There is continuous deposition, remodelling resorption
of the bone filling the alveolar socket.
l The first stage is the formation of the blood clot.
l Due to resorption of alveolar crest, obviously the crest
l After the removal of tooth, the socket is filled with
of the healed socket is below that of adjacent teeth.
blood which coagulates and RBCs being entrapped, fi-
brin strands and the ends of torn blood vessels in the Q. 5. Healing of the biopsy wound.
periodontal ligament become sealed off.
Ans.
l Blood vessels immediately surrounding the wound are
dilated and mobilization of leucocytes to the immediate The mechanism of healing of a biopsy wound is exactly
area around the clot. similar to any wound healing anywhere else in the body.
l The collapse of unsupported gingival tissue into the The only difference is that, whether the edges of the wound
opening of a fresh extraction wound is of great aid in can be brought in contact with each other or not.
maintaining the clot in position. Accordingly, two methods of healing are possible:
Section | III Oral Pathology 565

i. Healing by 1st intention or primary healing: ii. Immobilization: It is also important in healing reaction
In this type, the opposing edges of the wound are in close especially in the fractures.
contact by drawing them together and suturing. Then the iii. Physical factors:
healing process follows as usual, as shown below: a. Mild traumatic injury favours the healing process
while severe trauma is deterrent to rapid wound
Blood c1ots healing.
b. The local temperature in the area of wound
influences the rate of healing by its effect on local
Infiltration of inflammatory cells circulation and cell multiplication.
iv. Circulating factors: Good blood supply of wound
tissues promotes healing process. Anaemia has been
Differentiation of connective tissue cells into fibroblasts reported to delay the wound healing.
v. Nutritional factors:
l Presence of protein enhances the speed of wound

Proliferation of fibroblasts healing.


l Presence of vitamin C accelerates the rate of wound

healing process.
l Vitamins A and D accelerate the wound healing and
Collagen formation and at the same time epithelial cells
proliferate and close the surface. vitamin B complex promotes wound healing.
vi. Age of patient: Wounds in younger patients heal rap-
idly compared to elderly individuals due to increased
circulation and presence of protein synthesis. The rate
New blood capillaries are formed and the
wound heals rapidly. of healing appears to be in an inverse proportion to the
age of the patient.
vii. Infection: A wound which is exposed to mild physical
ii. Healing by second intention or secondary irritation or expose to bacteria heals quickly.
healing:
Q. 7. Complications of wound healing.
l In this case, the opposing edges of the wound cannot be
brought in close contact with each other. Ans.
l Only thing that can be done is to reduce the size of
the area left to heal by granulation, by advancing Complications of wound healing are as follows:
the undercut mucosa and suturing it to the underlying i. Infection: Wounds may provide a portal entry to micro-
mesodermal tissues. scopic infections of the wound and delay the healing
l This type of wound occurs when a lesion is removed process.
from the palate or alveolar ridge and the edges cannot ii. Keloid and hypertrophic scar formation:
be brought together in close contact because the mucosa l Keloids are the overgrowth scar tissues with ten-

cannot be stretched so far. dency for resolution. They occur in wounds which
l Healing process of course takes place as usual like in heal without any complication.
primary healing but it is slower because of the large area l Hypertrophic scars occur in wounds where healing

to be healed. is delayed.
iii. Pigmentory changes: These are common in healing of
When healing is complete, what is left is a small depressed wounds on skin appear as hypopigmented or hyperpig-
area of mucosa. mented, e.g. lichen planus and lichenoid reactions.
iv. Cicatrisation: It refers to late reduction in the size of
Q. 6. Enumerate the factors affecting healing of oral
the SCE contrast to immediate wound contraction.
wound.
Complication due to burns of skin.
Ans. v. Implantation cyst: Epithelial cells may slide or get
entrapped in wound and later may proliferate to form
A number of factors which influence the healing process of implantion cyst.
wounds in the oral cavity are as follows: vi. Healing after pulpal diseases: It depends upon the
i. Location of wound: It is important as it may modify the degree of infection, inflammation, amount of the pulpal
rate of healing. Wounds in the area in which there is tissue involved and age of the patient.
good vascular bed heal more rapidly than wounds in an vii. Healing after periapical diseases: It may result in the
avascular area. formation of fibrosis in the involved area.
566 Quick Review Series for BDS 3rd Year

SHORT NOTES
Q. 1. Biopsy. fixed with 95% of alcohol and is stained by a special
stain called as PAP stain.
Ans.
v. Findings in exfoliative cytology as reported by cytolo-
i. Biopsy is a procedure of removing a tissue from a gist fall into one of the following five classes:
living person for histopathological examination and Class I (normal)
for diagnosis. Class II (atypical)
ii. Biopsy is indicated whenever there is any doubt about Class III (intermediate)
a lesion and it cannot be diagnosed clinically. Class IV (suggestive cancer)
iii. Various types of biopsy are as follows: Class V (positive for cancer)
a. Excisional biopsy
Q. 4. Factors affecting wound healing.
b. Incisional biopsy
c. Fine needle aspiration cytology (FNAC) Ans.
d. Frozen section biopsy
A number of factors which influence the healing process of
iv. After obtaining a proper specimen of biopsy, it should
wounds in the oral cavity are as follows:
be fixed immediately in 10% formalin or 70% alcohol.
i. Location of wound: The vascularity of the region may
v. The biopsy specimen is sent to the histopathologist for
modify the rate of healing.
diagnosis after labelling it properly.
ii. Immobilization: It is also important in healing reaction
Q. 2. Dry socket. especially in the fractures.
iii. Physical factors like mild traumatic injury and local tem-
Ans. perature in the area of wound influences the rate of healing.
l The dry socket or alveolar osteitis is the most common iv. Circulating factors: Good blood supply of wound
complication in the healing of extraction wounds. tissues promotes healing process.
l It is basically a focal osteomyelitis in which the blood clot v. Nutritional factors: Presence of proteins and vitamins
has been disintegrated or lost with the production of foul accelerates the rate of wound healing process.
odour and severe pain of throbbing type but no suppuration. vi. Age of patient: Wounds in younger patients heal
l The condition derives its name from the fact that after rapidly compared to elderly individuals.
the clot is lost the socket has a dry appearance due to the vii. Infection: A wound which is exposed to mild physical
exposed bone. irritation or expose to bacteria heals quickly.
l One of the commonest aetiological factors is excessive Q. 5. Mention the types of biopsy.
trauma during extraction of tooth.
l More common in the women and tobacco users and usu- Ans.
ally starts by the second or third postoperative day and Various types of biopsy are as follows:
lasts for 7-10 days. i. Excisional biopsy: If a lesion is totally excised for
l Treatment consists of irrigation of socket with hot nor- histological evaluation, it is called as excisional biopsy.
mal saline solution thoroughly and dressing it with a ii. Incisional biopsy: When only a small section of tissue
material containing obtundent gently and loosely. is removed from the lesion for the purpose of histologi-
l Dressing is repeated until healthy granulation tissue cal evaluation, it is called incisional biopsy.
appears. iii. Fine needle aspiration cytology (FNAC): It is done by
Q. 3. Exfoliate cytology. aspirating tissue material inside a lesion which is later
on diagnosed microscopically after preparing a smear.
Ans. iv. Frozen section biopsy: It is performed in order to get an
immediate histological report of a lesion. The tissue is
i. Exfoliative cytology is the microscopic study of cells
obtained from the lesion and is kept in deep freeze and
which exfoliate or abrade from the surface of an organ
then frozen tissue is sectioned and stained to get a
or lesion after suitable staining.
prompt diagnosis.
ii. The exfoliative cytology is used in establishing the di-
v. Punch biopsy: A punch is used to remove a circular
agnosis of oral lesions like herpes simplex and herpes
piece of the tissue that are to be examined.
zoster, pemphigus vulgaris and pemphigoid, squamous
cell carcinoma, etc. Q. 6. Reimplantation of tooth.
iii. Technique consists of vigorously scrapping the entire
Ans.
surface of lesion several times with a metal cement
spatula or a moistened tongue blade or a cytobrush. i. This is a procedure in which tooth is reinserted back
iv. The collected material is then quickly and evenly into its socket after its removal or after its accidental
spread over a microscopic slide and is immediately dislocation.
Section | III Oral Pathology 567

ii. Intentional replantation: Intentional removal of tooth i. Transplantation is the removal of a tooth or tooth bud
for various endodontic procedures, which can’t be done from one socket and transplanting it into another
if the tooth is present in its socket and replacement of socket.
the tooth in its socket after endodontic therapy. ii. In this procedure, the pulp becomes acellular and peri-
iii. Reimplantation after avulsion of tooth is having best odontal ligament and cementum gradually disappear,
prognosis, if: and ankylosis results.
l Tooth is inserted within three hour. iii. Types of transplantation are:
l Tooth has been kept in saliva, milk or saline and not a. Autotransplantation: Tooth of the same individual is
dehydrated. transplanted, from one place in mouth to another
l Tooth is not contaminated highly. place, e.g. developing 3rd lower molar into the posi-
iv. This tooth generally needs endodontic therapy on later tion of a lost 1st lower molar, etc.
visits. It is ankylosed gradually and resorbed finally b. Homotransplantation: Transplanting the tooth of
after 5 to 10 years. one person to another person.
iv. Autotransplantation has obviously more chances of
Q. 7. Transplantation of tooth.
success than homotransplantation.
Ans.

Topic 15

Oral Aspects of Metabolic Disease

SHORT ESSAYS
Q. 1. Hypopituitarism. ii. Retardation of tooth eruption and delayed exfoliation
of teeth.
Ans.
iii. The clinical crowns appear smaller in size.
iv. The roots of teeth are shorter than normal.
Aetiology: v. Thickening of dentinal walls at expense of pulp cham-
l Hypopituitarism, i.e. decreased levels of pituitary hor- bers and disturbances in amelogenesis.
mones, is usually due to compression or infarction of vi. Simmonds’ disease in adults is due to an infarction of
anterior pituitary cells or defect in the hypothalamic pituitary characterized by:
l Loss of weight, atrophic alterations of skin and
control of hormonal secretion.
l Before puberty, the hypofuctioning leads to dwarfisim diminished sexual functions.
l Markedly lowered basal metabolic rate.
which manifests with features of growth hormone defi-
l No specific dental changes.
ciency.
l As Simmond’s disease represents a panhypopituita-
Common causes of postpartum pituitary insufficiency in rism, there is decrease in activity of the pituitary
adults are: gland and those glands that are under pituitary regu-
l Pituitary adenoma lation.
l Simmond’s disease or hypophyseal cachexia l Changes in the head include thin eyebrows, loss of
l Sheehan’s syndrome (pituitary infarction in postpartum eyelashes, thin lips and immobile expression.
woman) l There will be decreased salivary flow due to hypo-
l Hypofunction of posterior lobe leads to deficiency of functioning of salivary glands which leads to
vasopressin (diabetes insipidus). increased caries activity and periodontal disease.

Clinical Features Diagnosis


i. Patient is characterized by dwarfism with well propor- l Radiograph and CT scan
tionate body, fine, silky, and sparse hair, eyebrows, l Growth hormone assay
wrinkled atrophic skin and hypogonadism. l Fluoride treatment
568 Quick Review Series for BDS 3rd Year

l Administration of supplementary corticosteroids during Treatment


minor oral surgical procedures.
l Transsphenoidal surgery.
Q. 2. Hyperpituitarism. l Octerotide lowers the growth hormone.
l Dopamine antagonists are used.
Ans.
l Hyperpituitarism results from increase in the number of Q. 3. Addison’s disease.
acidophilic cells or adenoma of anterior lobe of pitu-
itary gland, which significantly increases production of Ans.
growth hormone. Addison’s disease is a primary disease of the adrenal
l Gigantism results, if the increase occurs before the glands, where they are unable to elaborate sufficient quanti-
epiphyses of long bones are closed. Acromegaly occurs, ties of hormones. Chronic insufficiency of adrenal cortex
if the increase occurs later in the life after epiphyseal results in Addison’s disease.
closure.

Aetiology
Clinical Features
It usually develops following:
Gigantism l Autoimmune distruction of adrenal glands
l Generalized symmetrical overgrowth of body (height of l May occur in conjunction with other autoimmune
.8 feet). disorders.
l Later in life, genital under development and excessive l Infections (TB, HIV)
perspiration are also present.
l They complain of headache, lassitude, fatigue, muscle
and joint pains and hot flushes. Clinical Features
l It is also characterized by broad enlarged nose, thick l Early manifestations include lethargy, fatigue and mus-
and furrowed oily skin. cular weakness.
l Organomegaly and hypertension are common findings. l Other features include weight loss, hypotension, salt
l There is enlargment of maxilla and mandible with man- craving, abdominal pain, diarrhoea and vitiligo.
dibular prognathism. Class III malocclusion with inter- l Vomiting, diarrhoea, severe anaemia, feeble heart action,
dental spacing. general debility.
l Teeth in gigantism are proportional to the size of the l Hypoglycaemia, dehydration, hypertension, elevated
jaws and the rest of the body. The roots may be longer serum potassium and postural dizziness develop due to
than normal. low levels of cortisol.
l Hypercementosis and supraeruption of teeth. l Females develop irregular menustration and loss of
l Lips are thick and tongue is enlarged with crenations on body hair.
lateral border. l Increased levels of ACTH stimulates MSH and results
in skin and mucosal pigmentation, i.e. bronzing of
Acromegaly
skin.
It is characterized by: l In the oral mucous membrane, pale brown to deep
l Temporal headaches, photophobia and reduction in chocolate pigmentation, spreading over buccal mucosa
vision. from angles of mouth and developing on gingiva,
l Enlargement of terminal phalanges of limbs. The ribs tongue, lips, may be the first evidence of the disease.
also increase in size. l Patient carries BLUE WARNING CARD (as an indica-
l Lips are thick and mandible is enlarged. tion on systemic steroid Rx)
l Macroglossia causing buccal or labial inclination of
teeth.
Diagnosis is based on:
Investigations l Clinical signs
l Radiographs exhibiting: Enlarged sella turcica and para- l Characteristic changes in the blood chemistry: Low serum
nasal sinuses, tufted terminal phalanges and widened sodium and chloride, high potassium levels.
carpal joint spaces. l Low diurnal plasma cortisol and ACTH levels
l Abnormal glucose tolerance l Elevated levels of blood urea nitrogen
l Hypertention l Biopsy of the oral lesion shows acanthosis with silver
l Hyperphosphataemia. positive granules in the cells of stratum germinativum.
Section | III Oral Pathology 569

Treatment l Random blood glucose .200 mg/dl with symptoms of


hyperglycaemia.
l Hormone replacement therapy with hydrocortisone and
fludrocortisone.
Q. 4. Diabetes mellitus.
Diagnosis
l Clinical signs and symptoms
Ans.
l Blood sugar estimation:
Diabetes mellitus comprises a group of disorders that share a. FBG
a common phenotype of hyperglycaemia. b. PPBG
It is a condition in which there is increase in the blood glu- l Glucose tolerance test
cose level due to lack of insulin secretion from the pancreas
and this upsets the metabolism of carbohydrates, proteins
and fats.
Complications
Complications are due to microangiopathy and macroangi-
Classification opathy which include:
l Neuropathy: Peripheral neuritis with severe pain, loss of
It is classified into two types: sensation in hands and feet
i. Type I or insulin-dependant diabetes mellitus (IDDM). l Retinopathy
(juvenile onset or type I or brittle or ketosis prone or l Nephropathy: Renal failure
labile diabetes) l Myocardial degeneration—affecting heart rate, rhythm
ii. Type II or non-insulin-dependant diabetes mellitus (NI- and conduction.
DDM) (adult onset or type II or maturity onset diabetes). l Coronary artery disease: Peripheral artery disease
l Gangrene of the extremities
Aetiology l Autonomic insufficiency and
l Susceptibility to infections.
Type 1 diabetes mellitus
a. Genetic factors, i.e. autoimmune disease with destruc-
Treatment
tion of insulin producing pancreatic b cells.
b. Circulating antibodies are found. l Diet—containing more of complex carbohydrates
c. Other causes—Cushing’s syndrome, hypothyroidism, ge- l Oral antidiabetics—glibenclamide, glimiperide, metfor-
netic causes, viral infections (mumps virus, Epstein-Barr min, etc.
virus) of the pancreas, diet and stress. l Insulin—short/intermediate/long-acting insulin.
l Modification of lifestyle
Type 2 diabetes mellitus l Exercise
a. Genetic factor
b. Obesity Q. 5. Oral manifestations and dental management of
c. Advancing age (middle aged and elderly) diabetes mellitus patients.
d. Inactive lifestyle Ans.
l Oral manifestations of diabetes mellitus are mainly due to
Clinical Features inflammation and infection resulting from abnormal neu-
l Polyuria trophil function, microangiopathy, and altered oral flora.
l Polydypsia l Most of the patients exhibit symptoms of dry mouth,
l Polyphagia persistant gingivitis, multiple carious lesions, periodon-
l Malaise tal disease and candidiasis.
l Weight loss l Gingiva becomes deep red colour and bleed easily, oe-
l Abnormal and accelerated metabolism of amino acids dematous and slightly enlarged, generalized, painful,
and fats resulting in ketoacidosis. suppuration of marginal gingivae and interdental papil-
l Symptoms of complications—burning feet, nocturia, lae with periodontal pockets.
diminished vision. l Fulminating periodontal disease of some form or other
is seen in untreated or inadequately controlled diabetic
patients due to decreased resistance to infections.
Criteria for Diagnosis l Recurrent periodontal and root abscesses leading to
l Fasting plasma glucose .126 mg/dl. loosening of teeth due to loss of periodontal membrane.
l 2 hr plasma glucose .200 mg/dl. l Caries—sudden increase in the number of new cavities.
570 Quick Review Series for BDS 3rd Year

l Decreased salivary flow, dryness of mouth and throat, Secondary hyperparathyroidism (von
burning throat, mouth, tongue and redness. Recklinghausen’s disease of bones)
l Dry socket occurring after tooth extraction. Slow l Due to end stage renal disease.
healing of wounds after oral surgery and delayed for l “Brown’s tumour” is common finding.
week. l Serum alkaline phosphatase is increased.
l Diabetic breath—which is sweetish or of acetone odour.

Dental management of diabetic patient: Oral Manifestations


l Detection of diabetic condition, and by joint action of l Jaws may show enlarged marrow spaces.
dental and medical specialists, patients condition is at- l Some cystic areas may also present.
tended to. l Loss of lamina dura.
l All foci of infection should be removed and trauma
should be avoided during dental treatment.
l Local anaesthetic with minimum amount of vasocon- Radiographic Appearance
strictor should be used.
l Lesions may show “ground glass” appearance of bone
l Precautions should be taken with respect to preopera-
in radiographs.
tive, operative and postoperative procedures.
l Pathological calcification of soft tissues.
l Never operate in a known uncontrolled diabetic
l “Pepper pot” skull.
patient.
l Do dental treatment on a patients whose blood sugar
levels are controlled. Histological Features
Q. 6. Clinical and oral manifestations of hyperparathy- l The cystic areas show osteoclasts along with fibroblas-
roidism. tic proliferation.
Ans. l The cystic spaces contain hemosiderin pigment and
termed as “Brown’s tumour”.
l Hyperparathyroidism is a condition that occurs due to l This disease is called as von Recklinghausen’s disease
excessive parathormone secretion. or osteitis fibrosa cystica.
l It is of three types:
a. Primary hyperparathyroidism: Excessive parathor-
mone secretion due to adenoma of parathyroid Treatment
gland. Surgical removal of one or more parathyroid glands.
b. Secondary hyperparathyroidism: Parathormone se-
cretion is excessive due to prolonged hypocalcae- Q. 7. Scurvy.
mia. This restores blood calcium level at the expense
of bones. Ans.
c. Tertiary hyperparathyroidism: When secondary hy- Scurvy is prolonged deficiency of vitamin C, which is char-
perthyroidism becomes autonomous it is termed as acterized by:
tertiary hyperparathyroidism. i. Microvessels having least muscular support.
ii. Defective synthesis of osteoids.
Clinical Features iii. Impaired wound healing.

Primary hyperparathyroidism
Functions of Vitamin C
l Bone pain, stiffness of joints and pathological fractures
are earliest symptoms. l Antioxidant
l Later calcification of cornea and extremities may also l It increases uptake of iron in intestine.
seen. l Collagen synthesis (hydroxyproline formation)
l Renal calculi, peptic ulcers, psychiatric problems, gen-
eralized osteoporosis.
Clinical Features
l “Giant cell tumour” or “cyst” of jaws.
l “Osteitis fibrosa cystica/generalisata”. l Lassitude, anorexia, painful limbs and enlargement of
l Malocclusion due to drifting of teeth. costochondral junction.
l “Brown’s tumour”. l Hair follicle rises above the skin and there is perifollicu-
l Serum Ca level is increased. lar haemorrhage.
Section | III Oral Pathology 571

l Haemorrhage may occur in the joints, in the nerve l There is presence of typical fetid breath of the patient
sheath, under the nails or conjunctiva. with fusospirochaetal stomatitis.
l Scorbutic child usually assumes the frog-like position
and is anaemic.
Radiographic Findings
l Delayed wound healing
l Subcutaneous bruising and haematomas l Widening of epiphysis
l Osteoid matrix is small but highly calcified. l Fraenkel’s white line and radio-opaque Trummerfeld
l Scorbutic rosary zone appears.
l Radiolucent widening of anterior ends of ribs.
Oral Manifestations
Diagnosis
l Swollen bleeding gums.
l Interdental and marginal gingiva is bright red, swollen, Long history of inadequate diet, purpura, anaemia, etc.
smooth, shiny surface producing scurvy bud.
In severe cases, haemorrhage and swelling of peri-
Treatment
l

odontal ligament membrane occurs followed by loss of


bone and loosening of teeth which results in early Administration of vitamin C in the dose of 250 mg tid daily
tooth loss. is given.

SHORT NOTES
Q. 1. Rickets. Oral Manifestations
Ans. l Swollen bleeding gums.
l Loss of bone and loosening of teeth which results in
l Deficiency of vitamin D in early childhood may lead to
early tooth loss.
rickets.
l Typical fetid breath.
l Skeleton is poorly developed because of defective calci-
fications in growing bones due to decreased absorption
of calcium and phosphorus. Radiographic Findings
l Radiolucent widening of epiphysis and anterior ends of
Clinical Features ribs.
l Craniotabes (flat, box-like skull), pigeon chest, richitic l Fraenkel’s white line and radio-opaque Trummerfeld
rosary, Harisson’s sulcus and bow legs. zone appear.
l Eruption of teeth is delayed.
Treatment
Treatment Administration of vitamin C in the dose of 250 mg tid daily
l Vit. D 2000 to 3000 IU daily. is given.
l Orthopaedic treatment.
Q. 3. Acromegaly.
Q. 2. Scurvy.
Ans.
Ans.
i. Acromegaly occurs, if increase in production of growth
l Scurvy is due to prolonged deficiency of vitamin C. hormone occurs later in the life after epiphyseal
closure.
ii. Acromegaly is characterized by:
Clinical Features l Temporal headaches, photophobia and reduction in

l Lassitude, anorexia, painful limbs and enlargement of vision.


costochondral junction. l Enlargement of terminal phalanges of limbs. The ribs

l Haemorrhage may occur in the joints, in the nerve also increase in size.
sheath or under the nails. l Lips are thick and mandible is enlarged.

l Delayed wound healing. l Macroglossia causing buccal or labial inclination of

l Subcutaneous bruising and haematomas. teeth.


572 Quick Review Series for BDS 3rd Year

Q. 4. Amyloidosis. vi. Treatment:


l X-ray therapy.
Ans.
l Curettage

i. Amyloid is an abnormal proteinaceous substance that is


deposited extracellularly in the tissues. Q. 7. Hyperparathyroidism.
ii. Now the cheif predisposing causes of amyloidosis are: Ans.
Rheumatoid arthritis and multiple myeloma.
iii. Clinical features: i. This condition occurs due to excessive parathormone
l Organs most commonly affected with amyloid are secretion.
kidneys, heart, liver and spleen. ii. It is of three types:
l Tongue when affected results in macroglossia and a. Primary hyperparathyroidism: Excessive parathor-
gingiva is also seen to be commonly affected. mone secretion due to adenoma of parathyroid gland.
l Skin, eyes and respiratory tract may get involved. b. Secondary hyperparathyroidism: Parathormone se-
iv. Histologically: cretion is excessive due to prolonged hypocalcaemia.
l Amyloid is best demonstrated by special stains such c. Tertiary hyperparathyroidism: When secondary hy-
as Congo red, thioflavin-T and crystal violet. perparathyroidism becomes autonomous it is termed
l It gives characteristic “apple green birefringence” as tertiary.
with “Congo red” stain under polarized light. This iii. Clinical features:
reaction is shared by all forms of amyloid. l Bone pain, stiffness of joints and pathological frac-

tures are earliest symptoms.


Q. 5. Hyperthyroidism. iv. Oral manifestations:
Ans. l Jaw may show enlarged marrow spaces and some

cystic areas.
i. Hyperthyroidism is a condition where there is excessive l Lesions may appear as “ground glass” in radio-
production of thyroxine hormones. graph.
ii. Aetiology: Thyroid gland is enlarged either diffusely or v. Histological features:
becomes nodular. l The cystic areas show osteoclasts along with fibro-
iii. Clinical features: blastic proliferation.
l Weight loss
l The cystic spaces contain hemosiderin pigment and
l Exophthalmos
termed as “Brown’s tumour”.
l Heat intolerance and hot moist palms
l This disease is called as von Recklinghausen’s dis-
l Excessive appetite
ease ‘or osteitis fibrosa cystica.
l Palpitation
vi. Treatment:
iv. Oral manifestations: Surgical removal of one or more parathyroid glands
l Severe malocclusion

l Early eruption of teeth Q. 8. Vitamin C-deficiency.


v. Treatment:
Enlarged gland may be removed partially. Ans.
Q. 6. Eosinophilic granuloma. Vitamin C is water-soluble vitamin, general manifestations
of vitamin C deficiency include:
Ans.
i. Normocytic and normochromic type of anaemia.
i. Eosinophilic granuloma affects only skeletal system but ii. Poor or delayed wound healing due to deranged colla-
overlying soft tissues may be swollen and tender. gen synthesis, poor preservation and maturation of
ii. Initial lesion is soft and brownish, which later on be- fibroblasts.
comes grey and fibrosed. iii. Haemorrhagic diasthesis:
iii. Sites involved are usually jaws, femur, humerus and l A marked tendency to bleeding is characteristic of

ribs. scurvy.
iv. Radiographic features: l Haemarthrosis and GI bleeding

l Jaw lesion is well circumscribed and radiolucent. l Lassitude and anorexia

l Teeth appear ”floating in air”. l Subperiosteal haemorrhage

v. Histologically: iv. Lesions in teeth and gums: Defective development of


l Proliferating histeocytes and eosinophils and multi- dentin and scorbutic gingivitis.
nucleated giant cells are found in lesion. v. Skeletal lesions: Deranged formation of osteoid matrix
l Later lesion gets fibrosed. affecting growing tubular bones and as well as flat
Section | III Oral Pathology 573

bones. Painful limbs and enlargement of costochondral l Depapillation of tongue.


junction. l Nasolabial dysseborrhoea.
vi. Scorbic child usually assumes a frog-like position. l Paleness of lips, especially at angles of mouth, fol-
lowed by cheilosis.
Q. 9. Oral manifestation of vitamin C deficiency.
Ans. Q. 12. Diabetic coma.
Ans.
Oral manifestations of scurvy are as follows:
l It chiefly affects the gingival and periodontal regions. i. Diabetic coma results from hyperglycaemia and acidosis.
l Atrophy and disorganisation of odontoblasts and irregu- ii. This occurs in a patient who is not taking insulin treat-
lar dentin formation. ment and is not so dangerous because it takes 2 to
l The pathognomonic sign is the swollen and spongy 3 days to develop.
gums, particularly the interdental papillae is involved iii. Symptoms include, increased thirst, fever, drowsiness,
producing the appearance of scurvy buds. nausea and vomiting, increased restlessness, polyuria,
l In severe cases, haemorrhage in periodontal mem- constipation, abdominal pain, unconsciousness and
branes followed by loss of bone and loosening of teeth finally death.
occurs.
Q. 13. Niemann-Pick disease.
Q. 10. Gigantism.
Ans.
Ans.
i. Niemann-Pick disease occurs due to disturbance of
Gigantism results, if the increase in production of growth
sphingomyelin metabolism.
hormone occurs before the epiphyses of long bones are
ii. It results from lysosomal accumulation of sphingomy-
closed.
elin resulting from inherited deficiency of enzyme
sphingomyelinase.
Clinical Features iii. Histological features: Pathognomonic cells known as
Niemann-Pick cells are present which are foamy, lipid
l Generalized symmetrical overgrowth of body (height of
laden cells distributed throughout RE system.
.8 feet).
iv. Treatment:
l Patient complains of headache, lassitude, fatigue, mus-
l Enzymatic replacement therapy is currently being
cle and joint pains and hot flushes.
explored.
l It is also characterized by broad enlarged nose, thick
l Prognosis is poor.
and furrowed oily skin.
l Organomegaly and hypertension are common findings. Q. 14. Pituitary dwarfism.
Ans.
Oral Manifestations
Hypopituitarism before puberty leads to dwarfisim which
l There is enlargment of maxilla and mandible with man- manifests with features of growth hormone deficiency.
dibular prognathism.
l Class III malocclusion with interdental spacing.
l Lips are thick and tongue is enlarged with crenations on Clinical Features
lateral border.
i. Dwarfism with well-proportionate body, fine, silky, and
l Teeth in gigantism are proportional to the size of the
sparse hair, eyebrows, wrinkled atrophic skin and hypo-
jaws and the rest of the body.
gonadism.
l Hypercementosis and supraeruption of teeth.
ii. Retardation of tooth eruption and delayed exfoliation of
Q. 11. Riboflavin deficiency. teeth.
iii. The clinical crowns appears smaller in size.
Ans. iv. The roots of teeth are shorter than normal.
l Riboflavin is used as FAD and FMN coenzymes in elec- v. Thickening of dentinal walls at expense of pulp cham-
tron transport chain. bers and disturbances in amelogenesis.
l Deficiency of riboflavin results in:
l Angular stomatitis.
Diagnosis
l Glossitis

l Magenta-coloured tongue. l Radiograph and CT scan, growth hormone assay.


574 Quick Review Series for BDS 3rd Year

Q. 15. Insulin shock. iii. Mucous membrane lesions affect the tongue, oral
cavity and vagina.
Ans.
iv. Mental symptoms and weight loss also occurs.
i. Insulin shock occurs due to hypoglycaemia in a patient v. Fiery red oral mucosa that is painful and epithelium
who are taking insulin treatment but taken too high dose of entire tongue desquamates.
of insulin or failed to take sufficient food at the proper
Q. 17. Gaucher’s disease.
time. This is more dangerous because it comes on within
a matter of hours. Ans.
ii. Symptoms: Weakness, pallor, hunger, nervousness, nau-
l Gaucher’s disease is a common lysosomal storage dis-
sea and vomiting, sweating, unconsciousness, dizziness,
ease, characterized by deposition of glucocerebroside in
convulsions, coma and death.
cells of the macrophage–monocyte system.
Treatment l It occurs due to deficiency of specific lysosomal hydro-
lase, glucocerebrosidase.
l Give glass of orange juice or few lumps of sugar, if pa- l It occurs in three clinical forms:
tient is conscious. Or a. Type I: Chronic non-neuronopathic form.
l Give 1 mg glucagon subcutaneously or intramuscularly. b. Type II: Infantile or acute neuronopathic form.
l Consult the physician immediately and shift the patient c. Type III: Juvinile or Norbottnian form.
to the hospital. l Gaucher cells are pathognomonic and are lipid-filled
Q. 16. Pellagra. cells that are pale, round cells with “crumpled cyto-
plasm” measuring 20-80 m in diameter, containing a
Ans. small eccentric nucleus.
i. Pellagra is caused due to deficiency of niacin. l Gaucher cell accumulations are found in reticuloendo-
ii. Pellagra is characterized by 3 Ds: thelial cells of liver, spleen, bone marrow and lymph
l Diarrhoea nodes.
l Dementia and

l Dermatitis

Topic 16

Allergic and Immunologic Diseases of the Oral Cavity

LONG ESSAYS
Q. 1. Classify aphthous ulcers and describe its aetiology, l Immunological abnormalities
clinical features and differential diagnosis. Add a note l Iron, vitamin B12 and folic acid deficiency
on Behcet’s syndrome.

Ans. Precipitating Factors


Recurrent aphthous stomatitis is characterized by develop- l Traumatic incidents, e.g. self-inflicted bites, toothbrush-
ment of painful solitary or multiple ulcers on oral mucosa. ing, dental procedures, etc.
l Endocrinal conditions: Pregnancy, menstrual period.
Psychogenic disorders.
Aetiology l

l Allergic factors, e.g. asthma, hay fever, food or drug


l Bacterial infections allergy.
Section | III Oral Pathology 575

Classification l Anitschkow cells are nothing but epithelial cells with


elongated nuclei containing a linear bar of chromatin
Based on clinical manifestations, recurrent aphthous stoma-
with radiating processes of chromatin extending
titis has been classified into four chief varieties:
towards the nuclear membrane.
i. Recurrent apthous minor
ii. Recurrent apthous major
iii. Recurrent herpetiform ulcerations Differential Diagnosis
iv. Recurrent apthae associated with Behcet’s syndrome.
Lesions which may be mistaken for recurrent aphthous
stomatitis include:
Clinical Features l Herpetic stomatitis.
l Herpangina
Recurrent apthous minor: l Traumatic ulcers
l Occurs more frequently in young women than in men. l Erosive lichenplanus
l They are most common ulcers, painful and smaller in l Pemphigus and
size varying from 2-3 mm to over 10 mm in diameter l Pemphigoid
and found mostly on non-keratinized mucosa.
l The most common sites of occurrence are labile mucosa not
bound to periosteum like buccal and labial mucosa, buccal Treatment
and lingual sulci, tongue, soft palate, pharynx, gingiva, etc. There is no specific treatment for recurrent aphthous
l The aphthous ulcers begin as single or multiple superfi- stomatitis.
cial erosions covered by a greyish membrane and l Symptomatic treatment:
surrounded by red inflammatory halo. i. Orabase gel.
l Healing occurs in 7-14 days with little or no evidence of ii. Xylocaine and choline salicylate application.
scarring. iii. Benadryl, topical.
iv. Camphor-phenol.
Recurrent apthous major: l Diet supplementation:
l Large-sized painful ulcers occur mostly on keratinized i. Vitamin B12, folic acid and iron supplements can be
mucosa ranging from 1 to 10 in number. prescribed.
l The lesions may exceed 1 cm in diameter persist up to l Antibiotics and antiseptics:
6 weeks and healing occurs with scarring. i. Broad-spectrum antibiotics
l The ulcers recur in waves over a long period of time ii. Chloramphenicol (suspension or topical)
Their incidence is more in patients with HIV infection. iii. Tetracycline mouthwash (250 mg/5 ml) should be
used, four times a day
Recurrent herpetiform ulcerations: iv. Chlorhexidine.
l They are characterized by crops of multiple ulcers often v. Silver nitrate gentian violet, etc.
up to 100 in number and are extremely painful occur- l Immune enhancement or modulation of immune
ring any where in the oral cavity. system: Can be done with drug levamisole.
l Lesions begin as small pinhead size erosions which It
gradually enlarge and coalesce. l Immunosuppression (inflammatory suppression):
l These ulcers persist continuously for 1-3 years and heal i. Prednisone
without scarring. ii. Betamethasone-17-benzoate.
iii. Antihistamine.
iv. Tetracycline
Histological Features
Minor aphthous ulcers exhibit fibrinopurulent mem-
Behcet’s Syndrome
l

brane covering over ulcerated epithelium.


l An intense inflammatory cell infiltration is present in l Behcet’s syndrome is a disease of uncertain aetiology. It
the connective tissue beneath the ulcer. is characterised chiefly by triad: Recurrent oral and
l Granulation tissue may be found at the base of ulcers. genital ulcers; ocular inflammation and skin lesions.
l Anitskow cells are found abundantly in recurrent aph- l Possible cause of the syndrome: PPLO virus; auto-
thous stomatitis but are not pathognomonic of the disease. immune.
576 Quick Review Series for BDS 3rd Year

Clinical features: l Men are affected 5-10 times more as compared to


l It is more common in young adults between 25 and 40 women.
years of age. Treatment: There is no specific treatment for the disease.

SHORT ESSAYS
Q. 1. Angioneurotic oedema. ii. In non-immunologic drug reactions, drugs directly
affect mast cells, causing release of chemical media-
Ans.
tors.
l Angioneurotic oedema is also known as Quincke’s oe- l Several other pharmacologic mechanisms are:
dema exhibits as smooth, diffused, oedematous swelling i. Deposits of heavy metals can occur in the gingivae,
involving face, lips, chin, eyes, tongue and extremities. e.g. bismuth, lead.
l Rarely, it leads to oedema of glottis resulting in suffoca- ii. Toxic effects - cause erosive lesions.
tion and even death. iii. Intolerance and idiosyncracy - can result in dilantin
gingivitis.
iv. Granulomatous lesions due to iodides and
Aetiology bromides.
l Food or drug allergy. v. Allergic reactions - will cause vesicular and bullous
l A biochemical abnormality: Absence of inhibitor of C1 eruptions, e.g. barbiturates, antibiotics and sulphon-
esterase enzyme from serum which then causes amides.
increased consumption of C2 and C4 and formation vi. Ecologic, i.e. in relation to environment. For
of kinin-like substances, which increase the vascular example, monilial infection following use of
permeability and oedema. antibiotics.

Treatment Clinical Features


l Antihistaminic drugs, like corticosteroids, can also be l Common reactions produced in oral cavity are stomatitis,
given. ulceration and necrosis, haemorrhage, gingival hyperpla-
l £-aminocaproic acid: Antifibrinolytic agent like EACA sia, pigmentation, altered salivary function and altered
which blocks the activation of plasmin which then acti- taste sensation.
vates the complement cascade. l The most common type of allergic reaction of oral mu-
l Hereditary type is treated with drug danazol. cosa is erythema multiforme and other common patterns
l Avoid use of aetiological agent as food or drugs. of oral mucosal disease are anaphylactic stomatitis, in-
l Tracheostomy in severe cases is done when suffocation traoral fixed drug eruptions, lichenoid drug reactions,
occurs. lupus erythematosus and nonspecific vesiculoulcerative
Q. 2. Stomatitis medicamentosa. lesions.
l Oral lesions of gingiva often resemble necrotizing gin-
Ans. givitis or Vincent’s infection. Hairy togue black, brown
l An allergic reaction of oral mucosa to the systemic ad- or yellow is a complication of antibiotic therapy.
ministration of certain drugs is known as stomatitis
medicamentosa.
Treatment
l In general, any drug may produce any type of reaction,
namely erythema, or vesicle and bulla or ulcer. i. The signs and symptoms of drug allergy regress with
l Pathogenesis of drug allergy: Several mechanisms are discontinuance of the causative agent.
involved in drug allergy ii. Localized acute signs may be relieved by the adminis-
i. IgE-mediated reactions may occur when drug reacts tration of antihistaminic drugs or cortisone.
with IgE antibody bound to mast cells, with subse- iii. Recurrence can be prevented only by complete absti-
quent release of chemical mediators. nence from use of drugs involved.
Section | III Oral Pathology 577

SHORT NOTES
Q. 1. Contact stomatitis. ii. They are cells with elongated nuclei containing a linear
bar of chromatin with radiating processes of chromatin
Ans.
extending towards the nuclear membrane.
l Contact stomatitis is an allergic reaction due to local iii. Although they are abundant in recurrent aphthous sto-
application or contact of certain drugs, foods, restor- matitis, they are not pathognomonic of the disease.
ative materials, dentrifices, etc. iv. Antischkow cells are also found in patients with:
l Contact stomatitis or stomatitis venenata is due to Ag-Ab l Sickle cell disease

reactions at the site of contact, e.g. procaine; resin, etc. l Megaloblastic anaemia

l Allergic contact stomatitis can be acute or chronic. l Iron deficiency anaemia

l Tretment: l Children receiving chemotherapy for cancer and

i. The mild cases of acute contact stomatitis can be l Normal people also.

treated by removal of suspected allergen and in more Q. 5. Delayed hypersensitivity.


severe cases, antihistamine therapy combined with
topical anaesthetics is beneficial. Ans.
ii. Chronic reactions respond to removal of antigenic i. Delayed hypersensivity or type IV hypersensitivity
source and application of a topical corticosteroid like constitutes one aspect of cell-mediated immune re-
fluocinonide gel or dexamethasone elixir. sponse.
Q. 2. Histamines. ii. It is typically provoked by intracellular infections or
haptens like simple chemicals applied on skin, evolve
Ans. slowly and consists of a mixed cellular reaction involv-
l Histamine is an amine of the tissues, present in all the ing lymphocytes and especially macrophages.
tissues of the body in an inactive or bound form. It is iii. The reaction is induced by sensitized T cells which on
liberated as active histamine during. contact with specific antigen, release cytokines that
a. Injury to tissues (i.e. wounds, inflammation) cause biological effects on leucocytes, macrophages
b. Antigen-antibody reaction (of allergy) and tissue cells.
l Action: iv. There are two types of delayed hypersensivity:
i. Increases capillary permeability and dilates the a. The tuberculin (infection) type
arterioles. b. The contact dermatitis type.
ii. Constricts the smooth muscles of the bronchi. v. Examples of delayed hypersensitivity response are:
l Tuberculosis – tuberculin testing
l It is destructed by antihistaminic drugs.
l Typhoid fever
Q. 3. Immunoglobulin. l Allergic contact dermatitis

l Graft rejection
Ans.
i. Immunoglobulin is defined as a protein of animal origin Q. 6. Bechet’s syndrome.
endowed with known antibody activity. Ans.
ii. Immunoglobins are synthesized by plasma cells and
i. Behcet’s syndrome is a disease of uncertain aetiology.
also by lymphocytes.
ii. Possible cause of the syndrome:
iii. All antibodies are immunoglobins but all immunoglo-
PPLO virus; autoimmune
bins may not be antibodies.
iii. Clinical features:
iv. Various classes of immunoglobulins are: IgG, IgA,
l It is more common in young adults between 25 and
IgM, IgD, IgE.
40 years of age.
Q. 4. Antischkow cells. l Men are affected 5-10 times more as compared to

women.
Ans.
l It is characterised chiefly by triad: Recurrent oral

i. Antischkow cells are modified epithelial cells with char- and genital ulcers; ocular inflammation and skill
acteristic changes in their nuclei found in cytological lesions.
smears taken from around recurrent aphthous ulcers. iv. There is no specific treatment for the disease.
578 Quick Review Series for BDS 3rd Year

Q. 7. Recurrent aphthous major. l The lesions may exceed 1 cm in diameter persist up to


6 weeks and healing occurs with scarring.
Ans.
l The ulcers recur in waves over a long period of time
Recurrent aphthous major is simply a more severe form of Their incidence is more in patients with HIV infection.
recurrent aphthous minor. l Histological features: Anitschkow cells are also found
l Large-sized painful ulcers occur mostly on keratinized with elongated central chromatid in nucleus.
mucosa ranging from 1 to 10 in number. l Treatment: Symptomatic treatment.

Topic 17

Diseases of Bone and Joints

LONG ESSAYS
Q. 1. Classify the diseases of TMJ. Write aetiology and Malignant:
clinical features of ankylosis. l Osteosarcoma

l Chondrosarcoma
Ans.
l Synovial sarcoma

Temporomandibular joint (TMJ) disorder refers to a cluster l Malignant lymphoma

of conditions characterized by pain in the TMJ or its E. Neuropathic:


surrounding tissues, functional limitations of the mandible, l Charcoat joint

or clicking in the TMJ during motion. l Ritter’s sympathetic dystrophy

F. Internal derangement:
l Disc displacement
CLASSIFICATION OF TMJ DISORDERS l Disc fracture

I. Based on the type of the disorder:


(or)
A. Developmental disturbances of TMJ:
l Aplasia of mandibular condyle

l Hypoplasia of mandibular condyle


II. Based on the origin:
l Hyperplasia of mandibular condyle A. Intra-articular origin or intrinsic disorders
B. Traumatic disturbances of TMJ: a) Trauma
l Ankylosis (hypomobility) l Dislocation, subluxation

l Injuries of articular disc (meniscus) l Haemarthrosis

l Fractures of condyle l Intracapsular fracture, extracapsular fracture

l Luxation and subluxation b) Disc displacement


C. Inflammatory disturbances of TMJ: l Anterior disc displacement with reduction

l Arthritis due to specific infections l Anterior disc displacement without reduction

l Rheumatoid arthritis c) Arthritis


l Osteoarthritis (degenerative joint disease, hypertrophic l Osteoarthritis

arthritis) l Rheumatoid arthritis

D. Neoplasms: l Juvenile rheumatoid arthritis

Benign: l Infectious arthritis

l Osteoma d) Developmental defects


l Chondroma l Condylar agenesis or aplasia

l Osteochondroma l Condylar hypoplasia

l Fibromyxoma l Condylar hyperplasia

l Synovioma e) Ankylosis
Section | III Oral Pathology 579

f) Neoplasms Clinical Features


l Benign tumours: Osteoma, osteochondroma,
l Occurs at any age mostly before the age of 10 years.
chondroma
l Equal sex distribution.
l Malignant tumours: Chondrosarcoma, fibrosar-
l Restriction in the opening of the mouth to appreciable
coma, synovial sarcoma
extent depending on the type of ankylosis.
B. Extra-articular origin or extrinsic disorders
l In cases of complete ankylosis, i.e. bony ankylosis, there
a) Masticatory muscle disorders
is absolute limitation of motion and in fibrous ankylosis
l Masticatory muscle spasm (MPD syndrome)
movement of joint is better than bony ankylosis.
l Masticatory muscle inflammation (myositis)
l Injury at infancy or childhood, there is associated facial
b) Problems that result from extrinsic trauma
deformity.
l Traumatic arthritis
l In unilateral ankylosis, the chin is displaced laterally
l Fracture
and backward on the affected side.
l Myositis, myospasm
l Whereas bilateral ankylosis, results in underdeveloped
l Tendonitis
lower portion of the face, i.e. receding chin and
l Contracture of elevator muscle, i.e. myofibrotic
micrognathia.
contractures
l In intra-articular ankylosis, the joint undergoes progres-
sive destruction of the meniscus with flattening of
ANKYLOSIS mandibular fossa, thickening of head of the condyle
Ankylosis of TMJ is one of the most incapacitating of all and narrowing of the joint space.
diseases involving this structure. l Extra-articular ankylosis results in a “splitting” of TMJ
by a fibrous or bony mass external to the joint proper.
Aetiology
Radiographic Features
l Abnormal intrauterine development
l Birth injury (by forceps) Radiograph exhibits abnormal or irregular shape of head of
l Trauma to the chin forcing the condyle against the the condyle and a radio-opacity indicative of the dense
glenoid fossa, with bleeding into the joint space bone filing the joint space.
l Malunion of condylar fractures
Injuries associated with the fracture of the malar-zygomatic
l
Treatment
compound
l Loss of tissues with scarring l Bony ankylosis is treated by surgical methods like
l Congenital syphilis condylectomy, gap arthroplasty.
l Primary inflammation of the joint (rheumatoid arthritis, l Fibrous ankylosis may be treated by functional methods.
infectious arthritis, Marie-Strumpell disease)
Q. 2. Enumerate bone disorders affecting the jaws. De-
l Inflammation of the joint secondary to a local inflam-
scribe the pathogenesis, clinical features, radiological
mation process (e.g. otitis media, mastoiditis, etc.)
appearances and histopathology of fibrous dysplasia.
l Inflammation of the joint secondary to the bloodstream
infection (e.g. septicaemia, scarlet fever, etc.) Or
l Metastatic malignancies
Describe in detail about fibrous dyaplasia.
l Inflammation secondary to radiation therapy
Ans.
Classification Various bone disorders affecting jaws are:
l TMJ ankylosis is divided into following types: i. Osteogenesis imperfecta
a. Intra-articular ankylosis and ii. Osteopetrosis
b. Extra-articular ankylosis. iii. Fibrous dysplasia
a. Monostotic fibrous dysplasia
Or b. McCune-Albright syndrome (polyostotic fibrous
a. Fibrous ankylosis. dysplasia)
b. Bony ankylosis. iv. Cherubism
v. Mandibulofacial dysostosis
Or vi. Pierre Robin malformation
a. True ankylosis. vii. Achondroplasia
b. False ankylosis. viii. Chondroectodermal dysplasia
580 Quick Review Series for BDS 3rd Year

ix. Cleidocranial dysplasia l Female:male – 3:1


x. Down syndrome l The skin lesions consist of irregularly pigmented, light
xi. Marfan syndrome brown melanotic spots described as cafe-au-lait spots.
xii. Infantile cortical hyperostosis l Two distinct forms of polyostotic fibrous dysplasia are
xiii. Paget’s disease as follows:
xiv. Massive osteolysis a. Jaffe’s type: Fibrous dysplasia involving a variable
xv. Diseases of the temporomandibular joint number of bones, although most of the skeleton is
normal, accompanied by pigmented lesions of the
skin or ‘cafe-au-lait spots’ is Jaffe’s type.
FIBROUS DYSPLASIA b. Albright’s type: An even more severe fibrous dysplasia
Fibrous dysplasia is a skeletal developmental anomaly of involving nearly all bones in the skeleton and associ-
the bone-forming mesenchyme that manifests as a defect in ated pigmented lesions of the skin and in addition en-
osteoblastic differentiation and maturation. docrine disturbances of varying types is Albright’s type.

iii. Craniofacial form of fibrous dysplasia:


Aetiology l This form of disease occurs in 10-25% of patients with
l Exact cause is not known. monostotic fibrous dysplasia and in 50% of patients
l It is usually caused by mutation in GNAS1 gene (gua- with polyostotic fibrous dysplasia. It also occurs as
nine nucleotide-binding protein, a-stimulating activity isolated craniofacial form.
polypeptide). l In isolated variety, no extracranial lesions are present,
l It appears to have no familial, hereditary or congenital this form involves maxillary sinus, the zygomatic pro-
basis. cess, floor of the orbit and sometimes extends towards
the base of the skull.
Hypertelorism, cranial asymmetry, severe malocclusion,
Clinical Features l

marked facial deformity, visual impairement, exophthal-


The following four disease patterns of fibrous dysplasia are mos and blindness occur because of involvement of or-
recognised: bital and periorbital bones.
i. Monostotic form, l Involvement of sphenoid wing and temporal bones may
ii. Polyostotic form, result in vestibular dysfunction, tinnitus and deafness.
iii. Craniofacial form, and l When cribriform plate is involved it leads to hyposmia
iv. Cherubism or anosmia.

i. Monostotic form of fibrous dysplasia: iv. Cherubism:


l It accounts for 70-80% of fibrous dysplasias. l It is an autosomal dominant, special variant of fibrous
l It occurs in children younger than 10 years. dysplasia with variable penetrance.
l Equal sex predilection. l It occurs in children and more severe in boys.
l This form most frequently occurs in the ribs, femur, l Regression may occur after adolescence.
tibia, craniofacial bones and less commonly humerus.
It is a condition in which single bone is involved and is
Oral Manifestations
l

replaced by abnormal fibrous connective tissue which


undergoes osseous metaplasia and the bone is trans- l Maxilla is more commonly affected than the mandible
formed into dense lamellar bone. and most of the changes occur in posterior region.
l The degree of bone deformity is less severe as compared l The most common area involved is premolar area.
with polyostotic type. l There is presence of unilateral facial swelling which is
slow growing with intact overlying mucosa. Swelling is
ii. Polyostotic form of fibrous dysplasia: painless but patient may feel discomfort.
l 20-30% of fibrous dysplasias are polyostotic. It is a l There is presence of enlarging deformities of alveolar
condition which involves multiple bones. process mainly buccal and labial cortical plates.
l Polyostotic form most frequently involves the skull and l In mandible, there is protrusion of inferior border of
facial bones, pelvis, spine and shoulder girdle. mandible.
l The sites of involvement are the femur, tibia, pelvis, l The teeth present in the affected area are malaligned and
ribs, skull and facial bones, upper extremities, lumbar tipped or displaced.
spine, clavicle and cervical spine in the decreasing order l Supernumerary teeth are often impacted and affected
of frequency. the eruption of the teeth.
Section | III Oral Pathology 581

Radiological Features Aetiology


l In long and short bones, radiolucent lesions with endos- l It is still unknown, there may be a family history of the
teal scalloping and with or without bone expansion and disease.
the absence of periosteal reaction seen. This lucent l Various aetiologies suggested are as follows:
lesion is known as Rind sign. a. Inflammatory, as Paget suggested.
l In skull and facial bones, single or multiple, symmetric b. Viral infection
or asymmetric, radiolucent or sclerotic lesions are c. Autoimmune, connective tissue and vascular disorders.
seen. d. Breakdown of normal bone mechanism in which
l Maxillary and mandibular involvement has mixed ra- deposition and resorption takes place (Jaffe).
diolucent and radio-opaque pattern with displacement e. Recent evidence suggests that it may be due to vari-
of the teeth and distortion of the nasal cavities. ous paramyxovirus infections of osteoclasts modi-
fied by genetic or environmental factors.

Histopathological Features
Clinical Features
Monostotic form
l The lesion is essentially a fibrous bone made up of l It occurs most commonly after the age of 50 years and
proliferating fibroblasts in compact stroma of inter- rarely diagnosed in people younger than 20 years.
lacing collagen fibres. l Male: female ratio is 2:1.
l Irregular trabeculae of bone are scattered throughout the l Site: Maxilla more often affected and other commonly
lesion. affected bones are skull, femur, tibia, sacrum, lumbar
l Some of the trabeculae are C-shaped and described as spine, pelvis and mandible.
Chinese character-shaped. l The most common presenting complaint is bone pain. It
l There is permanent maturation arrest in woven bone is pereceived as a dull constant aching pain deep below
stage. the soft tissues.
l Because of increased vascularity, the involved bones
Polyostotic form become warm to touch.
l As head enlarges, patients hat sizes need to be increased
l The lesions are composed of fibrillar connective tissue
or changed due to skeletal deformity and enlargement of
within which are numerous trabaculae of coarse, woven
skull bones.
bone.
l Other typical findings of Paget’s disease with skull in-
l The osteocytes are quite large and collagen fibres are
volvement include pathological fractures resulting from
seen into the fibrous tissue.
weakened pagetic bone, nonspecific headache, deaf-
l These trabaculae have typically wide osteoid seams.
ness, blindness or other eye problems.
l Softened bone at the base of the skull may lead to platy-
Treatment basia. Secondary to platybasia or vertebral fractures
progressive pain, paraesthesias, limb paresis, gait diffi-
l Treatment is usually conservative like osseous contour-
culties (waddling gait) or bowel or bladder inconsis-
ing to prevent facial deformity.
tency may develop.
l Some evidences suggest that vitamin D and bisphospho-
l Bowing of femur and tibia is characteristic feature. Broad-
nates may be helpful in ameliorating pain and possibibly
ening and flattening of the chest and spinal curvature.
reconstituting lesions with normal bone.
l Facial appearance has been described as leontiasis ossea
l Surgical removal of the lesion.
(lion-like facies).
Q. 3. Write in detail about Paget’s disease of bone.
Ans. Oral Manifestations
l Paget’s disease or osteitis deformans is a multifocal l Involvement of the jaws is a common finding.
benign tumour of osteoclasts occurring in people over l The maxilla exhibits progressive enlargement, the alveo-
the age of 40 years. lar ridges become widened and palate becomes flattened.
l It is characterized by excessive and abnormal remodelling l Teeth may become loose and migrate, producing some
of bone. spacings, as a result of the underlying changes in the
l The excessive remodelling gives rise to bones that are supporting bone.
extensively vascularised, weak, enlarged and deformed l Difficulty in extraction because of heavy cementosis
with subsequent complications. and dry socket results following extractions.
582 Quick Review Series for BDS 3rd Year

l Edentulous patients commonly complain of inability to Histologic Features


wear their appliances due to expansion of jaws.
i. Very extensive typical mosaic pattern of the trabeculae
l Skull vault enlarged and triangular
of bone, due to repeated deposition and resorption of
l Ill-defined ache in the affected bone
bone, in which more bone is formed than resorbed.
l Hypercementosis: Ankylosed and submerged teeth
ii. Fibrosis of the marrow secondary to resorptive process.
l Atypical facial neuralgias.
Marrow is loose and vascular and formation of new
subperiosteal bone.
Laboratory Findings iii. Hypercementosis following resorption of roots and mosaic
pattern of this new cementum due to osteoclastic activity.
l The serum alkaline phosphatase levels are elevated
iv. Repeated episodes of bone removal and formation
to extreme limits as high as over 250 Bodansky units
result in appearance of many small irregularly shaped
especially in osteoblastic phase of the disease.
bone fragments that appear to be joined in a jigsaw
l Serum calcium and phosphorus levels are within normal
puzzle or mosaic pattern with deeply staining hema-
limits.
toxyphylic reversal lines.
l Urinary hydroxyproline levels are elevated reflecting
v. Increased abnormal local vascularity consists of increased
osteoclastic activity and bone resorption.
number of patent capillaries and dilated arterioles as well
l More recently, urinary excretion of bone-specific pyri-
as large venous sinuses.
dinium collagen crosslinks has been found to be a sensi-
tive and specific index of bone resorption.
l Recently, urinary N-telopeptide (NTX) and alpha-C Differential Diagnosis
telopeptide (CTX) have emerged as sensitive biochemi- l Fibrous dysplaisa of bone
cal markers for bone resorption. l Hyperparathyroidism
l An abnormally high alpha-CTX: beta-CTX ratio pres- l Osteomyelitis
ent with active Paget’s disease returns to reference l Fibroosteoma
range following treatment with bisphosphonates. l Osteogenic sarcoma, metastatic carcinoma, multiple
myeloma
Radiological Features
l The radiographic features are varied depending on the Complications of Paget’s Disease
stage of the disease. l Sarcoma of bone (malignant transformation)
l Paget’s disease is characterized by two phases: l Arteriosclerosis
i. Intial phase of deossification and softening and l Cardiac enlargement
ii. Bizarre dysplastic type of reossification phase. l Osteomyelitis
l The skull X-rays, demonstrate cotton wool appearance.
Radiographs of the jaws demonstrate poorly defined
l
Treatment
areas of osteoporosis, loss of normal trabeculation and
appearance of irregular osteoblastic activity giving rise l There is no specific treatment.
to typical cotton-wool appearance of ‘Paget’s bone’. l Recently drugs like calcitonin or biphosphonates have
l The osteolytic areas in skeleton are multiple and diffuse been used with promising results.
or isolated. The isolated lesion in the skull when large is l Cytotoxic antibiotics, like mithramycin, have been used
referred to as ‘osteoporosis circumscripta’. therapeutically but have serious side effects.
l Teeth exhibit significant radiographic changes such as l If treatment is administered before major changes in the
loss of well-defined lamina dura, pronounced hyperce- bone have occurred, the general outlook of patients with
mentosis, root resorption and displacement. Paget’s disease is good.

SHORT ESSAYS
Q. 1. Paget’s disease.
Aetiology
Ans.
l It is still unknown, there may be a family history of the
l Paget’s disease or osteitis deformans is a multifocal disease.
benign tumour of osteoclasts occurring in people over l Various aetiologies suggested are as follows:
the age of 40 years. a. Inflammatory
Section | III Oral Pathology 583

b. Viral infection l Teeth exhibit loss of well-defined lamina dura,


c. Autoimmune, connective tissue and vascular pronounced hypercementosis, root resorption and
disorders. displacement.

Clinical Features Histologic Features


l It occurs most commonly after the age of 50 years and i. Very extensive typical mosaic pattern of the trabeculae
rarely diagnosed in people younger than 20 years. of bone due to repeated deposition and resorption of
l Male: female ratio is 2:1. bone, in which more bone is formed than resorbed.
l Site: Maxilla more often affected and other commonly ii. Hypercementosis following resorption of roots and mo-
affected bones are skull, femur, tibia, sacrum, lumbar saic pattern of this new cementum due to osteoclastic
spine, pelvis and mandible. activity.
l The most common presenting complaint is bone pain. iii. Repeated episodes of bone removal and formation re-
l Because of increased vascularity, the involved bones sult in appearance of a jigsaw puzzle or mosaic pattern
become warm to touch. with deeply staining hematoxyphylic reversal lines.
l Other typical findings of Paget’s disease with skull in-
volvement include pathological fractures resulting from
weakened pagetic bone, nonspecific headache, deaf- Treatment
ness, blindness or other eye problems. l There is no specific treatment.
l Softened bone at the base of the skull may lead to platy- l Recently drugs, like calcitonin or biphosphonates, have
basia. Secondary to platybasia or vertebral fractures been used with promising results.
progressive pain, paraesthesias, limb paresis, gait diffi- l If treatment is administered before major changes in the
culties (waddling gait) or bowel or bladder inconsis- bone have occurred, the general outlook of patients with
tency may develop. Paget’s disease is good.
l Facial appearance has been described as leontiasis ossea
(lion-like facies). Q. 2. Write about histopathology of Paget’s disease of
bone.
Oral Manifestations Ans.
l The maxilla exhibits progressive enlargement, the alveo- Histologic features of Paget’s disease are as follows:
lar ridges become widened and palate becomes flattened. i. Very extensive typical mosaic pattern of the trabeculae
l Teeth may become loose and migrate, producing some of bone due to repeated deposition and resorption of
spacings. bone, in which more bone is formed than resorbed.
l Hypercementosis. ii. Fibrosis of the marrow secondary to resorptive process.
l Edentulous patients commonly complain of inability to Marrow is loose and vascular and formation of new
wear their appliances due to expansion of jaws. subperiosteal bone.
iii. Hypercementosis following resorption of roots and
Laboratory Findings mosaic pattern of this new cementum due to osteoclastic
activity.
l The serum alkaline phosphatase levels are elevated to as iv. Repeated episodes of bone removal and formation
high as over 250 Bodansky units. result in appearance of many small irregularly shaped
l Serum calcium and phosphorus levels are within normal bone fragments that appear to be joined in a jigsaw
limits. puzzle or mosaic pattern with deeply staining hema-
l Urinary hydroxyproline levels are elevated reflecting toxyphylic reversal lines.
osteoclastic activity and bone resorption. v. Increased abnormal local vascularity cosists of increased
l More recently, urinary excretion of bone-specific number of patent capillaries and dilated arterioles as
pyridinium collagen crosslinks has been found to be well as large venous sinuses.
a sensitive and specific index of bone resorption.
Q. 3. Write briefly about fibrous dysplasia.

Radiological Features Ans.


l The skull X-rays demonstrate cotton wool appearance. Fibrous dysplasia is a skeletal developmental anomaly of
l The isolated osteolytic lesion in the skull when large is the bone-forming mesenchyme that manifests as a defect in
referred to as ‘osteoporosis circumscripta’. osteoblastic differentiation and maturation.
584 Quick Review Series for BDS 3rd Year

Aetiology l The teeth present in the affected area are malaligned and
tipped or displaced.
l Exact cause is not known.
l Supernumerary teeth are often impacted and affected
the eruption of the teeth.
Clinical Features
The following four disease patterns of fibrous dysplasia are Radiological Features
recognised: l In skull and facial bones, single or multiple, symmetric
i. Monostotic form, or asymmetric, radiolucent or sclerotic lesions are seen,
ii. Polyostotic form, Rind sign is observed.
iii. Craniofacial form, and l Maxillary and mandibular involvement has mixed ra-
iv. Cherubism diolucent and radio-opaque pattern with displacement
of the teeth and distortion of the nasal cavities.
i. Monostotic form of fibrous dysplasia:
l It accounts for 70-80% of fibrous dysplasias and occurs
in children younger than 10 years with equal sex predi- Histopathological Features
lection. Monostotic form
l This form most frequently occurs in the ribs, femur,
tibia, craniofacial bones and less commonly humerus. l Irregular trabeculae of bone are scattered throughout the
l It is a condition in which single bone is involved. lesion.
l Some of the trabeculae are C-shaped and described as
ii. Polyostotic form of fibrous dysplasia: Chinese character-shaped.
l 20-30% of fibrous dysplasias are polyostotic. It is a l There is permanent maturation arrest in woven bone stage.
condition which involves multiple bones.
Polyostotic form
l Polyostotic form most frequently involves the skull and
facial bones, pelvis, spine and shoulder girdle. l The lesions are composed of fibrillar connective tissue
l Two distinct forms of polyostotic fibrous dysplasia are within which are numerous trabaculae of coarse, woven
as follows: bone.
a. Jaffe’s type: Fibrous dysplasia involving a variable l The osteocytes are quite large and collagen fibres are
number of bones, accompanied by pigmented le- seen into the fibrous tissue.
sions of the skin or ‘cafe-au-lait spots’ is Jaffe’s type. l These trabaculae have typically wide osteoid seams.
b. Albright’s type: An even more severe fibrous dys-
plasia involving nearly all bones in the skeleton and Treatment
associated pigmented lesions of the skin and in addi-
tion endocrine disturbances of varying types is Al- l Treatment is usually conservative like osseous contouring.
bright’s type. l Vitamin D and bisphosphonates may be helpful in ame-
liorating pain and possibibly reconstituting lesions with
iii. Craniofacial form of fibrous dysplasia: normal bone.
l Surgical removal of the lesion.
l This form of disease occurs in 10-25% of patients with
monostotic fibrous dysplasia and in 50% of patients Q. 4. Clinical and histopathological features of mono-
with polyostotic fibrous dysplasia. It also occurs as stotic fibrous dysplasia.
isolated craniofacial form.
Ans.
iv. Cherubism: l Monostotic fibrous dysplasia is a condition in which sin-
l It is an autosomal dominant, special variant of fibrous dys- gle bone is involved and is replaced by abnormal fibrous
plasia usually occurs in children below the age of 3 years. connective tissue which undergoes osseous metaplasia
l Regression may occur after adolescence. and the bone is transformed into dense lamellar bone.
l ‘Leontiasis ossea’ is the term applied to the cases of
fibrous dysplasia which affect the maxilla or facial
Oral Manifestations bones giving the patient a leonine appearance.
l Maxilla is more commonly affected than the mandible.
l There is presence of unilateral painless facial swelling Clinical features of monostotic fibrous dysplasia:
which is slow growing with intact overlying mucosa, l It accounts for 70-80% of fibrous dysplasias.
but patient may feel discomfort. l It occurs in children younger than 10 years.
Section | III Oral Pathology 585

l Equal sex predilection. l Cleidocranial dysostosis or Marie and Santon’s disease


l This form most frequently occurs in the ribs, femur, is a hereditary (congenital) condition transmitted as
tibia, craniofacial bones and less commonly humerus. autosomal dominant trait.
l It is a condition in which single bone is involved and is l It is manifested as retardation or partial failure of the
replaced by abnormal fibrous connective tissue which development of the bones of the clavicle and of the skull
undergoes osseous metaplasia and the bone is trans- but not of mandible.
formed into dense lamellar bone.
The degree of bone deformity is less severe as compared
Aetiology
l

with polyostotic type.


l Maxilla is more commonly affected than the mandible l It is familial and transmitted as true dominant mende-
and the most common area involved is premolar area. lian characteristic.
l There is presence of unilateral facial swelling which is l Mutations in the core-binding factor alpha-1 (CBFA1)
slow growing with intact overlying mucosa. Swelling is gene, located on chromosome 6p21 shown to be the
painless but patient may feel discomfort. cause of cleidocranial dysplasia.
l In mandible, there is protrusion of inferior border of
mandible.
Clinical Features
l The teeth present in the affected area are malaligned and
tipped or displaced. l Cleidocranial dysplasia is primarily characterized by
abnormalities of skull, dentition, jaws, clavicle and
shoulder girdle as well as by occasional stunting of the
Histopathology long bones.
l The defect of shoulder girdle ranges from unilateral or
bilateral aplasia of clavicle in about 10% of cases to
Cellular connective tissue partial absence or even a simple thinning of one or both
clavicles.
l As a result of clavicular disturbance, the patients have
Woven bone with out
osteoblastic rimming
an unusual mobility of the shoulders and they may be
able to bring the shoulders together in front of the chest
until they meet in the midline.
Irregular bony trabeculae l Defects of the vertebral column, anamolies of pelvis
(Chinese letter pattern)
and long bones, as well as bones of the digits are also
FIGURE 17. 1 Histologic features of fibrous dysplasia relatively common.
l In the skull, fontanels often remain open or exhibit de-
l The lesion is essentially a fibrous one made up of pro- layed closure. Frontal, parietal and occipital bones are
liferating fibroblasts in compact stroma of interlacing prominent. Wormian bones (i.e. bones in the suture)
collagen fibres. may be present in the vault.
l Irregular trabeculae of bone are scattered throughout the l Delayed closure of the fontanels and sutures results in
lesion. broad, square and flat skull.
l Some of the trabeculae are C-shaped and described as
Chinese character-shaped.
l These trabeculae are usually coarse woven bone but
Oral Manifestations
may be lamellar, although not as well organized as l High, narrow, arched palate and cleft palate appear to be
normal lamellar bone. common.
l There is permanent maturation arrest in woven bone l Maxilla, zygoma and nasal bones are underdeveloped
stage. resulting in narrow face and sunken nose.
l Maxilla is almost invariably underdeveloped resulting
in relative mandibular prognathism.
Treatment
l Hypertelorism: Large width between the eyes, means
l Surgical removal of lesion eyes wide apart.
l Osseous countering is necessary for characterizing l Prolonged retention of deciduous teeth and subsequent
deformity for aesthetic purposes. delay in eruption of permanent teeth.
l Defects of enamel, supernumerary and geminated teeth.
Q. 5. Cleidocranial dysplasia.
l Roots of teeth are somewhat short and thinner than
Ans. usual and may be deformed.
586 Quick Review Series for BDS 3rd Year

l There is absence or paucity of cellular cementum on the (where fibroblasts, loose fibrillar connective tissue, vas-
roots of permanent teeth, and it may be related to failure cular tissue and multinucleated giant cells are present).
of eruption so frequently seen. l A characteristic perivascular eosinophilic cuffing of
small capillaries is found.
Treatment Differential Diagnosis
l There is no specific treatment.
l Aneurysmal bone cyst
l Dental care should be taken.
l Giant cell granuloma (central) of jaws
Q. 6. Write briefly about cherubism. l Osteoclastoma
l Fibrous dysplasia
Ans.
l Hyperparathyroidism
l Cherubism is also known as juvenile fibrous dysplasia,
or familial fibrous dysplasia. Treatment
l It is an autosomal dominant condition with unusual
l Lesion becomes static or regresses with onset of puberty
clinical appearance and facial deformity.
l If necessary, surgery is undertaken after puberty, when
lesions reach their remission phase.
Clinical Features Q. 7. Osteogenesis imperfecta.
l Affected children are normal at birth until 14 months Ans.
to 3 years of age without any clinical or radiographic
evidence of disease. l Osteogenesis imperfecta is also called ‘brittle bones’ or
l Around 3 years of age, symmetrical enlargement of the Lobstein’s disease or fragilitas ossium or osteopsathyrosis.
jaw begins and this self-limited bone growth usually l It presents a hereditary autosomal dominant trait.
begins to slow down when patient reaches 5 years of age l This is the disease of mesodermal tissues with defect of
and stops by the age of 12-15 years. collagen maturation while calcification of bone takes
l Angle of mandible is affected symmetrically giving a place in normal manner.
typical cherub-like chubby face.
l Fullness of cheek is present with firm, non-tender, hard Clinical Features
swelling of jaw. l The chief clinical characteristic of osteogenesis imper-
l Regional lymphadenopathy. fecta is the extreme fragility or porosities of bones with
l Eyes look starring into sky with a classic ‘eye to proneness to fracture. Increased tendency to fracture of
heaven’ appearance with a rim of sclera visible beneath long bones.
the iris. l Second characteristic clinical feature is occurrence of pale
blue sclera. The sclera are abnormally thin and pigmented
Oral Manifestations choroid shows through and produces the blue colour.
l Additional signs and symptoms described as characteristic
l Numerous dental abnormalities have been reported, like of osteogenic imperfecta are: Deafness due to otosclerosis,
absence of numerous teeth, displacement of teeth, etc. laxity of ligaments and peculiar shape of skull, dentinogen-
l Premature shedding of deciduous teeth, delayed perma- esis imperfecta and abnormal electrical reaction of the
nent tooth eruption; partial anodontia of permanent teeth. muscles.
l Increased tendency for capillary bleeding although no
Radiographic Features specific blood dyscrasia or defect.

l Multilocular radiolucent areas are seen bilaterally caus- Sillence Classification of Osteogenesis
ing expansion and destruction of cortical plates. Imperfecta
l Teeth appear as floating in cyst-like spaces referred to as
‘floating tooth syndrome’. a. Type I osteogenesis imperfecta:
l There is a classic ground glass appearance because of l Most common and mildest form and it includes subtype
small tightly compressed trabecular pattern. A and B.
l In subtype A, dentinogenesis imperfecta is absent and in
B it is present.
Histologic Features l Symptoms of both include blue sclera, in utero frac-
l Lesions reveal numerous multinucleated giant cells and tures, mild to moderate bone fragility, kyphoscoliosis,
are indistinguishable from central giant cell granulomas hearing loss, easy bruising, and short stature.
Section | III Oral Pathology 587

b. Type II osteogenesis imperfecta: Treatment


l It exhibits extreme bone fragility and frequent fractures. No known treatment.
In utero fractures are present in 100% of cases. Many
are stillborn, 90% die before 4 weeks of age. Q. 8. Osteopetrosis (marble bone disease)
l Blue sclera may be present. Ans.
l Hearing loss is not common.
l Dentinogenesis may be present with small nose, micro- l Osteopetrosis is also known as marble bone disease or
gnathia and short trunk. Albers-Schonberg disease, osteosclerosis fragilis generalisata.
l This is the disease affecting bones which become
c. Type III osteogenesis imperfecta: sclerotic, dense and fragile.
l Type III OI is associated with dentinogenesis imper- l Spontaneous fracture is seen in such bones.
fecta, sclera of variable hue, limb shortening, frontal l This genetic defect occurs due to abnormal osteoclasts
bossing and pulmonary hypertension. activity. Bones lack medullary cavity.
l In utero fractures occur in 50% of cases. The remaining l Three distinct forms of the disease are:
half have fractures in neonatal period. i. Adult onset,
l No hearing loss. ii. Infantile and
iii. Intermediate.
d. Type IV osteogenesis imperfecta:
l In subtype A, DI is absent, while in subtype B DI is i. Infantile osteopetrosis (malignant osteopetrosis):
present. l Autosomal recessive mode of transmission.
l Symptoms of both the subtypes include normal sclera, l It is diagnosed early in the life, if untreated failure to
normal hearing, mild angulationa and shortening of survive and growth retardation are symptoms.
long bones. l Bony defects occur, nasal stuffiness due to mastoid and
paranasal sinus malformation is usually the presenting
feature of infantile osteopetrosis.
Oral Manifestations l Manifestations of cranial nerve entrapment neuropa-
l Structure of the dentin is affected, but not of enamel, i.e. thies include: Deafness, proptosis and hydrocephalus.
odontoblastic activity is affected just like osteoblastic l Dentition delayed.
activity. l Osteomyelitis of the mandible is common due to defi-
l Class III malocclusion, anterior and posterior crossbites cient blood supply.
and open bites are also frequent. l Bones are fragile and can fracture easily.
l Defects are seen in both deciduous and permanent teeth. l Anaemia, easy bruising and bleeding.
Deciduous teeth are poorly calcified and semitranslu- l Hepatosplenomegaly, hypersplenism, and haemolysis
cent appearing as faintly dirty pink in colour, half nor- may ccur due to extramedullary haematopoiesis.
mal size, with globular crown and relative short roots, in l Other manifestations include sleep apnoea and blindness.
proportion to other dimensions.
l In permanent dentition, OI patients often have un- ii. Adult osteopetrosis (benign osteopetrosis):
erupted first and second molars. l Diagnosed at late adolescence or adulthood.
l Bones are fragile, frequent fractures of long bones and
Radiographic Features osteomyelitis may be the presenting features.
l Many patients have bone pain, bony defects are com-
Osteopenia, bowing, angulation or deformity of long mon and include cranial nerve entrapment neuropathies
bones, multiple fractures and wormian bones in the skull like deafness, facial palsy, etc., carpal tunnel syndrome
are hallmarks of osteogenesis imperfecta. and osteoarthritis.
l Enlargement of head (hydrocephalus)
Histological Findings l Other manifestations include visual impairment and
psychomotor retardation.
l Osteoblastic activity appears as retarded and imperfect.
l The basic defect appears to lie in organic matrix with
failure of foetal collagen to be transformed into mature Radiographic Features
collagen. Qualitative defects and quantitative defects l Bones may be uniformly sclerotic, but alternating sclerotic
both exist. and lucent bands may be noted near the ends of long bones.
l The trabeculae of cancellous bone are delicate and often l Thickened and clubbed ends of long bones.
show fracture. l Vertebrae are extremely radiodense and may show
l Calcification proceeds normally. ‘rugger-jersey’ sign.
588 Quick Review Series for BDS 3rd Year

l Radiographs may show evidence of fractures or osteo- l It is a group of idiopathic disorders characterized by the
myelitis. clonal proliferation of specialized bone marrow derived
l When jaws are affected they exhibit diffuse radio- antigen presenting dendritic cells called Langerhans
opacity of affected bones; roots of the teeth are not cells and mature eosinophils.
identifiable on dental radiograph. l The clinical spectrum includes the following group of
diseases:
a. Letterer-Siwe disease
Laboratory Findings
b. Eosinophilic granuloma
l Patients manifest myelophthisic anaemia. c. Hand-Schuller-Christian disease
l Hypocalcaemia
l Increased PTH, acid phosphatase and creatinine kinase Oral manifestations of Hand-Schuller-Christian
levels. disease:
l They are non-specific and include sore mouth, halitosis,
Histological Features gingivitis, swelling of the jaws and bad taste.
l Loose and sore teeth with precocious exfoliation of
l Osteopetrosis is charaterised by the endosteal produc- teeth and failure of socket to heal after extraction of
tion of bone with an apparent concomitant lack of loose teeth.
physiologic bone resorption. l Loss of supporting alveolar bone mimicking advanced
l Osteoblasts are prominent but osteoclasts are seldom periodontal disease is characteristic of the child suffering
found in significant numbers in tissue sections. from this disease.
l Persistence of cartilaginous core of bony trabeculae. l The bones affected may be, skull, mandible, femur,
humerus, ribs or other bones as well.
Treatment
l Infantile type: Calcitriol appears to stimulate bone Radiographic Features
resorption by stimulating dormant osteoclasts. X-ray of jaw may show solitary intrabony lesion,
l Erythropoietin can be used to correct anaemia. multiple “scooped-out” and sclerotic bony lesion with
l Treatment with gamma interferon shown to produce distinct border, periosteal new bone formation, root
long-term benefits. resorption.
l Adult osteopetrosis requires no treatment by itself. l The lesions are destructive and well demarcated.
Q. 9. Write briefly about histiocytoses-X-disease (Lang-
erhans cell histiocytosis) and oral manifestations of Treatment
Hand-Schuller-Christian disease.
l Local curettage or excision of the lesions.
Ans.
l However, the condition is benign and may undergo
l Langerhans cell histiocytosis (LCH ) is a disease that pri- spontaneous remission over a period of years and heal
marily affects bone but occasionally may also affect other on its own.
organ systems and present in a multisystemic pattern. l The prognosis is good.

SHORT NOTES
Q. 1. Grading system of cherubism. Grade III: McCune-Albright syndrome invovement of
Ans. the whole maxilla and mandible except the coronoid
process and condyles.
l Cherubism is also known as familial fibrous dysplasia
of jaws. Q. 2. Clinical features of Paget’s disease.
l It is an autosomal dominant fibro-osseous lesion of the
Ans.
jaws occurring in early childhood from the age of 2 to
4 years. Paget’s disease is also known as osteitis deformans.
l Grading system of cherubism as suggested by Arnott is
as follows:
Clinical Features
Grade I: Involves both mandibular ascending rami.
Grade II: Involves both maxillary tuberosities as well as l It occurs most commonly after the age of 50 years with
ascending mandibular rami. male: female ratio of 2:1.
Section | III Oral Pathology 589

l Maxilla more often affected and other commonly Treatment


affected bones are skull, femur, tibia, sacrum, lumbar
l Osseous contouring for esthetic purposes.
spine, pelvis and mandible.
l The most common presenting complaint is bone pain. Q. 4. Leontiasis ossea
l Other typical findings of Paget’s disease with skull in-
Ans.
volvement include pathological fractures, nonspecific
headache, deafness, blindness or other eye problems. l The term leontiasis ossea has often been applied to
l Edentulous patients commonly complain of inability to cases of fibrous dysplasia.
wear their appliances due to progressive expansion of jaws. l The cases in which fibrous dysplasia affects the
l Teeth may exhibit spacings and roots demonstrate maxilla or facial bones give the patient a leonine
hypercementosis. appearance.
l There is no specific treatment. l Thus it is appreciated that fibrous dysplasia of bone has
Q. 3. Fibrous dysplasia. come to include a number of lesions once described by
other terms.
Ans. l In the Paget’s disease, involvement of the facial bones is
l Fibrous dysplasia is a skeletal developmental anomaly occasionally seen and has been called leontiasis ossea
of the bone-forming mesenchyme that manifests as a (lion-like facies).
defect in osteoblastic differentiation and maturation. Q. 5. Cleidocranial dysplasia.
l Exact cause of disease is not known.
Ans.

Clinical Features i. Cleidocranial dysostosis or Marie and Santon’s dis-


ease is a hereditary condition transmitted as autosomal
i. The four disease patterns of fibrous dysplasia are: dominant trait.
Monostotic form, polyostotic form, craniofacial form, ii. It is primarily characterized by abnormalities of skull,
and cherubism. dentition, jaws, clavicle and shoulder girdle as well as
ii. Monostotic form of fibrous dysplasia accounts for 70- by occasional stunting of the long bones.
80% of cases and occurs in children younger than 10 iii. The patients have an unusual mobility of the shoulders.
years involving single bone. iv. Delayed closure of the fontanels and sutures, results in
iii. Polyostotic form of fibrous dysplasia accounts 20-30% broad, square and flat skull.
of cases and involves multiple bones; presents in two v. Wormian bones may be present in the vault.
distinct forms Jaffe’s type and Albright’s type. vi. High, narrow, arched palate and cleft palate appears to
iv. Craniofacial form occurs in 10-25% of patients with be common.
monostotic fibrous dysplasia and in 50% of patients vii. Prolonged retention of deciduous teeth and subse-
with polyostotic fibrous dysplasia. It also occurs as quent delay in eruption of permanent teeth.
isolated form. viii. There is no specific treatment.
v. Cherubism is an autosomal dominant, special variant of
fibrous dysplasia usually occurs in children below the Q. 6. Osteogenesis imperfecta (brittle bones).
age of 3 years. Ans.
vi. In all the forms, maxilla is more commonly affected
than the mandible. i. Osteogenesis imperfecta is also called ‘brittle bones’ or
Lobstein’s disease or fragilitas ossium or osteopsathy-
rosis.
Radiological Features ii. It is inherited disease of bone.
l In skull and facial bones, single or multiple, symmetric ii. Congenital form is present at birth, and the childhood
or asymmetric, radiolucent or sclerotic lesions are seen, type is present later in life.
Rind sign is observed. iii. In the childhood type, the signs and symptoms are:
Blue sclera, increased tendency to fracture of long
bones, deafness due to otosclerosis, relaxation of liga-
Histopathological Features ments, leading to changes of shape of skull and dislo-
l The lesions are composed of fibrillar connective tissue cations, dentinogenesis imperfecta and hypertension.
within which are numerous trabaculae of coarse, woven iv. Oral manifestations include defects in both deciduous
bone. and permanent teeth.
l Some of the trabeculae are C-shaped and described as v. Dentin is soft, pulp chambers become obliterated and
Chinese character-shaped. lack of scalloping of amelodentinal junction.
590 Quick Review Series for BDS 3rd Year

vi. Histologically, the basic defect appears to lie in organic l Open mouth and prognathism.
matrix with failure of foetal collagen or type-l collagen l Sexual underdevelopment.
to be transformed into mature collagen. l Hypermobility of joints

vii. There is no known treatment. l Cardiac abnormalities

iii. Oral manifestations:


Q. 7. Osteopetrosis (marble bone disease). Fissured and enlarged tongue (macroglossia), high
Ans. arched palate and enamel hypoplasia
iv. There is increased incidence of acute leukaemia (ALL)
i. Osteopetrosis is also known as marble bone disease in children affected with Down’s syndrome.
or Albers-Schonberg disease, osteosclerosis fragilis
generalisata. Q. 10. Blue sclera.
ii. This is the disease affecting bones which become scle- Ans.
rotic, dense and fragile. Spontaneous fracture is seen in
such bones. i. Blue sclera (China-blue eyes) is seen in osteogenesis
iii. This genetic defect occurs due to abnormal osteoclasts imperfecta. It is due to unusually transparent or thin
activity. Bones lack medullary cavity. sclera which causes increased visibility of choroid, i.e.
iv. Three distinct forms of the disease are: Adult onset, pigment of eye through the sclera.
infantile and intermediate forms. ii. Blue sclera are also seen in:
v. Radio-opaque sclerosis of all bones occurs in diffuse form. l Marfan’s syndrome

vi. Laboratory findings: l Cherubism

l Patients manifest myelophthisic anaemia and hypo- l Ehlers-Danlos syndrome

calcaemia. l Osteopetrosis

l Increased PTH, acid phosphatase and creatinine ki- l Fetal rickets and

nase levels. l Normal infants

vii. Treatment includes bone marrow transplantation and Q. 11. Albright’s syndrome.
vitamin A administration
Ans.
Q. 8. Histopathology of Paget’s disease of bone.
l Also known as McCune-Albright syndrome.
Ans. l Albright syndrome includes: Precocious puberty 1
Histologic features of Paget’s disease of bone are as follows: Polyostotic fibrous dysplasia 1 Cafe-au-lait pigmentation.
i. Very extensive typical mosaic pattern of the trabeculae l Severe polyostotic fibrous dysplasia involving nearly all
of bone due to repeated deposition and resorption of bones of the skeleton.
bone, in which more bone is formed than resorbed. l Pigmented lesions of the skin, i.e. café-au-lait spots seen.
ii. Fibrosis of the marrow secondary to resorptive process. l Endocrine disturbances include hyperfunction of one or
Marrow is loose and vascular and formation of new more endocrine glands.
subperiosteal bone. Q. 12. Marfan’s syndrome.
iii. Hypercementosis following resorption of roots and
mosaic pattern of this new cementum due to osteoclas- Ans.
tic activity. l Also known as arachnodactyly.
iv. Repeated episodes of bone removal and formation re- l Skeleton typically includes arachnodactyly, dolicho-
sult in appearance of many small irregularly shaped stenomelia (long limbs).
bone fragments that appear to be joined in a jigsaw l Long and narrow face with frontal bossing.
puzzle or mosaic pattern with deeply staining hema- l Ears are large and eyes are sunken.
toxyphylic reversal lines. l Aortic aneurysm and regurgitation.
v. Increased abnormal local vascularity. l TMJ dysarthrosis, multiple odontogenic cysts of the
jaws.
Q. 9. Trisomy 21 or Down syndrome.
l High arched palate, bifid uvula.
Ans.
Q. 13. Philadelphia chromosome.
i. Down syndrome or mongolism or trisomy 21 syndrome
Ans.
is due to trisomy of 21st chromosome.
ii. Clinical features: l Philadelphia chromosome is now recognized as a
l Mental defects. translocation of chromosomal material from chromo-
lFlat face, small slanting eyes (mongolism). some 22 to chromosome 9.
Section | III Oral Pathology 591

l It is usually seen in the leukaemic patients. Q. 16. Serum alkaline phosphatase.


l Mongolism or Down syndrome is due to the defect of
Ans.
trisomy chromosome 21, hence leukaemia is 3-15 times
more common in mongoloids. l Alkaline phosphatase occurs in many tissues of body
notably in osteoblasts.
Q. 14. Achondroplasia.
l Increase in serum concentration of this enzyme results in
Ans. increased osteoblastic activity in association with liver
diseases and other conditions like sarcoidosis, leukaemia.
l Achondroplasia is most common type of genetic dwarf-
l Increase is also seen during periods of rapid bone
ism transmitted as an autosomal dominant triat.
growth in infancy and childhood, during pregnancy and
l It is caused by failure of normal cartilage proliferation
in healing fractures.
in epiphysis and bones of skull.
l Its values are markedly increased in Paget’s disease and
also occasionally increased in rickets, hyperparathy-
Clinical Features roidism and polyostotic fibrous dysplasia.
l Dwarfism with short limbs and normal head which ap- Q. 17. Pierre Robin syndrome.
pears larger and disproportionate to body.
l Stubby fingers Ans.
l Lumbar lordosis l Pierre Robin syndrome (bird facies) or Robin anomalad
l Prominent buttocks is characterized by triad of symptoms which include
l Bowed legs cleft palate 1 micrognathia (mandible) 1 glossoptosis.
l Intelligence is normal but patient is usually strong built. l Primary defect is arrested development of mandible.
l Often seen in circus clowns and wrestlers. l Others features include bird facies and respiratory dif-
ficulty due to glossoptosis.
Oral Manifestations l Systemic findings include congenital heart diseases,
mental retardation, ocular lesion and skeletal defect.
Retruded maxilla, restricted growth of base of the skull, l There is an U-shaped cleft palate and feeding difficulty.
mandibular prognathism.
Q. 18. Cotton wool appearance of bone.

Treatment Ans.

No treatment is required for this disease. l It is an radiological appearance seen in Paget’s disease.
l In the osteoblastic phase of osteitis deformans, the os-
Q. 15. Mandibulofacial dysotosis. teoblastic areas appear as opacities in X-ray which tend
Ans. to be patchy in distribution, this patchiness is termed as
cotton wool appearance.
l Mandibulofacial dysotosis is also known as Treacher l In early osteitis deformans, there is poorly defined areas
Collins or Franceschetti syndrome. of osteoporosis but more significantly there is loss of
l It is caused due to anatomic alternation of the structures, normal trabaculation and the appearance of irregular
embryologically derived from first branchial arch. osteoblastic activity giving rise to cotton wool appear-
ance of pagetic bone.
Clinical Features Q. 19. Cafe-au-lait spots.
l Antimongoloid palpabral fissures
Ans.
l Deficient eye lashes
l Hypoplasia of the facial bones, especially of malar bone l Cafe-au-lait spots are pigmented macules seen in McCune-
and mandible. Albright syndrome or polyostotic fibrous dysplasia.
l Malformed external ears. l Café-au-lait spots are characteristic feature related to in-
l Blind fistulae are found between angle of mouth and creased amounts of melanin in the basal cells of epidermis.
ears. l These macules help to differentiate fibrous dysplasia
l Bird or fish facies. from neurofibromatosis.
l These pigmentations occur at birth and occasionally
precede the development of skeletal and endocrine
Treatment abnormalities. They are arranged in linear or segmental
l No treatment is done and prognosis of disease is good. pattern near midline of the body.
592 Quick Review Series for BDS 3rd Year

Q. 20. Infantile cortical hyperostosis (Caffey’s disease). l Infants will develop fever and become hyperirritable.
l Intermittent soft tissue swelling, inflammation of the
Ans.
overlying skin.
l Infantile cortical hyperostosis is also known as Caffey- l Oral manifestations: Malocclusion, enamel hypoplasia,
Silverman syndrome. residual asymmetric deformity of angle and ramus of
l It is characterised by unusual cortical thickening of the mandible.
certain bones especially skull, jaws and clavicles are l Radiographic features include unilateral or bilateral
most frequently involved. thickening of cortex.
l It occurs in two forms, i.e. autosomal dominant and l Lab findings inlude increased ESR and serum alkaline
sporadic forms. phosphatase levels.
l Commonly seen in infants before 6 months of age. l Treatment: Regression occurs in months.

Topic 18

Diseases of Blood and Blood Forming Organs

LONG ESSAYS
Q. 1. Describe leukaemia in detail. l Other predisposing factors may be chromosomal abnor-
malities, genetics, age, hormones, immune competence,
Ans.
and stress.
l Leukaemia is a disease characterized by the progressive
overproduction of white blood cells which usually
Clinical Features
appear in the circulating blood in an immature form.
Acute leukaemia
Classification l Acute lymphoblastic leukaemia is common in children
while acute myeloid leukaemia is common in adults.
l Depending on the onset and the course, the leukaemia is l Sudden onset.
classified as: l Characterized by weakness, fever, headache, petechial
a. Acute leukaemia and or ecchymotic haemorrhages in the skin and mucous
b. Chronic leukaemia membranes.
l Depending on the type of cell of origin, leukaemia is l Lymphadenopathy is often the first sign of the disease.
also classified into: l Gingival bleeding, epistaxis, haemorrhagia may occur
a. Myeloid (mylogenous) leukaemia and due to thrombocytopenia.
b. Lymphoid (lymphoblastic, lymphocytic) leukaemia. l Bleeding may occur due to disseminated intravascular
Acute and chronic leukaemias are broadly classified as: coagulation (DIC) which is mainly in patients with
i. Acute lymphoblastic leukaemia (ALL) acute promyelocytic leukaemia.
ii. Acute myeloblastic leukaemia (AML) l Hepatomegaly, splenomegaly, gum hyperplasia, stoma-
iii. Chronic lymphocytic leukaemia (CLL) titis, sternal tenderness, enlargement and infiltration of
iv. Chronic myelocytic leukaemia (CML) skin may be seen.

Chronic leukaemia
Aetiology
l Disease is present before the symptoms are seen.
l Aetiology of leukaemia is unknown. l Patient may appear with excellent health or exhibit
l However, some are associated with ionizing radiation, emaciation suggestive of a chronic debilitating disease.
cytotoxic drugs, chemical carcinogens and infectious l Lymph node enlargement common in CLL but uncom-
origin of unknown organism. mon in CML.
Section | III Oral Pathology 593

l Splenomegaly and hepatomegaly are fully developed bone marrow, and normal clinical status). The initial
due to protracted course of the disease. induction phase is followed by the consolidation phase
l Enlargement of salivary glands and tonsils leading to and the maintenance phase.
leukaemic infiltration and xerostomia.
l Petechiae, ecchymosis of skin, papules, pustules, bullae, Radiotherapy
areas of pigmentation, herpes zoster, itching and burn- l Cranial irradiation along with intrathecal methotrexate
ing sensations are also seen. is given in ALL patients for CNS prophylaxis.

Oral Manifestations Bone marrow transplantation


l Following initial remission of disease with chemotherapy,
l Oral lesions occur in both acute and chronic forms. further treatment is given to cure the disease.
l Gingivitis, gingival hyperplasia, haemorrhage, pete-
chiae and ulceration of the mucosa. Acute myeloid leukaemia:
l Rapid loosening of the teeth due to necrosis of PDL.
l Daunorubicin
l Alterations in the developing tooth crypts.
l Cytosine arabinoside
l Osseous changes in jaws.
l Etoposide
l All-trans-retinoic acid
Diagnosis
Acute lymphoblastic leukaemia:
l Peripheral blood examination reveals the presence of
l Vincristine
blast cells with high, low or normal total leucocyte
l Prednisolone
count.
l Daunorubicin
l There is also the evidence of anaemia and thrombocyto-
l L-asparaginase
penia.
l Methotrexate
l The bone marrow examination shows hypercellularity
l Etoposide
along with the presence of .20 % leukaemic blast cells.
l Cytosine arabinoside
l Cytochemical staining, cytogenetics and immunophe-
l Mercaptopurine
notyping of the cells help in differentiating different
types of leukaemia. Indications for BMT are:
l T Common (pre-B) ALL in second remission
Treatment l T and B cell ALL in first remission
The management of acute leukaemia consists of supportive l B cell ALL in first remission
and specific treatment. l AML in first remission
l Common (pre-B) ALL in second remission and
Supportive treatment l B cell ALL in first remission
Anaemia is managed with infusion of red cell concen-
l
Prognosis
trate. Platelet transfusion is needed to treat bleeding
manifestations and to maintain platelet count above The cure rate in AML is around 25-30% with chemotherapy
10,000-20,000/ Cu mm. and 50-60% with BMT. The cure rate of .90% can be
achieved by chemotherapy in children with ALL (pre-B type).
Specific treatment Q. 2. Classify anaemias. Describe pernicious anaemia in
l The objective of specific treatment is to eliminate leu- detail.
kaemic cells without affecting the normal cells. How-
ever, the therapy may be associated with high morbidity Or
and mortality. Hence, the decision to administer a spe- What is anaemia? Classify anaemias. Write about clini-
cific therapy to a particular patient is based on the age, cal features and treatment of pernicious anaemia.
type of leukaemia, and the presence of other associated
illnesses. Ans.
Anaemia is defined as an abnormal reduction in the number
Chemotherapy of circulating red blood cells, the quantity of haemoglobin
l In chemotherapy, a combination of various cytotoxic and the volume of packed red cells in a given unit of blood.
drugs is given under a standard protocol. The first step The normal haemoglobin level varies from 14-16 g/l in
is to achieve remission (normal blood counts, normal the adult male and 12-14 g/dl in the female.
594 Quick Review Series for BDS 3rd Year

Classification l Erythropoietin deficiency


Renal disease
Anaemia has been traditionally classified into:
Endocrine deficiencies
l Dyshaematopoietic,
l Hypoplastic anaemia
l Haemorrhagic and
Aplastic anaemia
l Haemolytic anaemia
Pure red cell aplasia
According to the morphology (MCV, MCH, and MCHC) l Infiltration

into: Leukaemia
l Normocytic Metastatic carcinoma
l Microcytic Myelofibrosis
l Macrocytic and B. Ineffective
l Normochromic l Megaloblastic

l Hypochromic types Vitamin B12 deficiency


Folate deficiency
However, the recent classification is based on reticulo- Other causes
cyte index, which is a measure of RBC production: l Microcytic
l The reticulocyte index is increased (.2.5) due to increase in Thalassaemia
erythropoiesis in haemolytic and haemorrhagic anaemias. Certain sideroblastic anaemia
l A low reticulocyte index ,2 % shows decreased marrow l Normocytic
production or maturation defects during erythropoiesis. 2. Increased erythrocyte production (reticulocyte
production index greater than 3)
Aetiologic classification of anaemia l Haemolytic anaemia
I. Blood loss Hereditary
l Acute post-haemorrhagic blood loss Acquired
l Chronic post-haemorrhagic blood loss l Treated nutritional anaemia
II. Deficiency of haemopoietic factors
l Iron deficiency

l Folate and vitamin B12 deficiency


PERNICIOUS ANAEMIA (VITAMIN B12
l Protein deficiency, i.e. diarrhoea, malabsorption
DEFICIENCY, ADDISONIAN ANAEMIA)
III. Bone marrow aplasia l Pernicious anaemia is a type of a chronic progressive,
l Aplastic anaemia megaloblastic anaemia of adults and is caused by
l Pure red cell aplasia deficiency of intrinsic factors in stomach.
IV. Anaemia due to systemic infections or systemic disorders l It is probably an autoimmune disorder with a genetic
l Anaemia due to chronic infection predisposition and the disease is associated with human
l Anaemia due to chronic renal disease leucocyte antigen (HLA) types A2, A3, B7 and A blood
l Anaemia due to chronic liver disease group.
l Disseminated malignancy

l Endocrinal diseases

V. Anaemia due to bone marrow infiltration Clinical Features


l Leukaemias l Occurs rarely before 30 years of age and increases in
l Lymphomas frequency with advancing age.
l Myelofibrosis/myelosclerosis l No racial predilection in other countries, except in USA.
l Congenital sideroblastic anaemia Females are more commonly affected.
VI. Anaemia due to increased red cell destruction (haemo- l The four major cardinal features of pernicious anaemia
lytic anaemia) are:
l Intracorpuscular defect (hereditary or acquired) a. Abnormally large RBCs
l Extracorpuscular defect (acquired) b. Hypochlorhydria
c. Neurologic and gastrointestinal symptoms
Kinetic classification of anaemia d. A fatal outcome unless the patient receives lifelong
1. Impaired erythrocyte production (reticulocyte produc- injections of vitamin B12.
tion index less than 2) l Generalized weakness, fatigue, headache, palpitation,
A. Hypoproliferative nausea, vomting, anorexia, diarrhoea and dyspnoea.
l Iron-deficient erythropoiesis l Shortness of breath, loss of weight, pallor and abdominal
Iron deficiency pain.
Anaemia of chronic disorders l Patients have smooth, dry and yellow skin.
Section | III Oral Pathology 595

l Neurological manifestations include tingling sensation l Erythroid precursors are large and oval, their nucleus is
in hands and feet, paraesthesia of extremities due to large and contains coarse motley chromatin clumps,
peripheral nerve degeneration. providing a checker board appearance.

Oral Manifestations Treatment


l Glossitis, glossodynia (painful tongue) and glossopyrosis l Administration of folic acid and vitamin B12.
(itching, and burning tongue) l Early recognition and treatment of pernicious anaemia
l Tongue appears beefy red in colour. provides a normal uncomplicated lifespan.
l Sometimes loss of papilla produces a bald appearance l Delayed treatment permits progression of the anaemia
of tongue which is referred to as Hunter’s glossitis or and neurological complications.
Moeller’s glossitis.
l Sometimes hyperpigmentation occurs in mucosa. Q. 3. Describe in detail haemophilia.

Ans.
Histopathology l Haemophilia is also known as bleeder’s disease, the
l Oral epithelial cells in pernicious anaemia reveal disease of kings.
enlarged and hyperchromatic nuclei with prominent l It is a blood disease characterized by a prolonged
nucleoli and serrated nuclear membrane. coagulation time and haemorrhagic tendencies.
l There is atrophy of epithelium with intra-or subepithe- l The disease is hereditary, the defect being carried by the
lial chronic inflammatory cell infiltration. X chromosome, and is transmitted as a gender-linked
l Cellular atypia is sometimes present. mendelian recessive triat, thus it occurs only in males.

Laboratory Findings Aetiology


i. Blood Haemophilia Types
l RBC count is seriously decreased to 1,000,000 or less i. Haemophilia A: Due to deficiency of factor VIII known
per Cubic millimetre. as antihaemophilic globulin (AHG) which is a clot
l Macrocytosis, haemoglobin content of RBCs is in- promoting factor.
creased proportional to their size. ii. Haemophilia B or Christmas disease: Due to deficiency
l Great many red blood cell abnormalities have been de- of factor IX known as Christmas factor.
scribed in advanced cases of anaemia like polychro- iii. Haemophilia C: Due to deficiency of factor XI and
matophilic cells, stippled cells, nucleated cells, Howell- plasma thromboplastin antecedent.
Jolley bodies and Cabot’s rings punctuate basophilia. These are closely similar diseases due to inherited, sex-
l Mild to moderate thrombocytopenia. linked recessive gene-that means only the males are affected
l Iron deficiency. and the females who are unaffected transmit the disease.
ii. Serum
l The indirect bilirubin may be elevated. Clinical Features
l Serum lactic dehydrogenase usually is markedly l It occurs only in males, while females are only the carriers.
increased. l Haemorrhagic tendency even in infancy
l The serum potassium, cholesterol and skeletal alkaline l Ecchymoses even from minor trauma
phosphatase are often decreased. l Persistent oozing of blood and sudden bleeding into the
l Serum antibodies for IF are highly specific. muscles and joints (knee). Joint becomes swollen and
painful.
iii. Gastric secretions l Symptoms of anaemia.
l Total gastric secretions are decreased to 10% of refer- l Blood test shows prolonged clotting time and normal
ence range. bleeding time.
l Achlorhydria l Haemophilia is classified according to the clinical se-
l IF is either absent or markedly decreased. verity as mild, moderate and severe.

iv. Bone marrow


Oral Manifestations
l Bone marrow biopsy and aspirate are hypercellular and
show trilineage differentiation. l Gingival haemorrhage
596 Quick Review Series for BDS 3rd Year

l Eruption and exfoliation of teeth are associated with Aetiology


severe haemorrhage
No evidence
l Mandibular pseudotumour

Laboratory Findings Clinical Features


l It may occur at any age commonly seen in infants and
l Characteristic defect of haemophilia is a prolonged
young children.
coagulation time.
l Patients manifest fever, malaise, sore throat, stomatitis,
l The bleeding time is normal, as is the prothrombin time
and regional lymphadenopathy.
and platelet aggregation.
l Headache, arthritis, cutaneous infection, and conjunctivitis.
l Activated partial thromboplastin time is prolonged.
l Oral manifestations include severe gingivitis, stomatitis
l Functional assay of the factors is useful in diagnosing
with ulceration.
haemophilia caused due to dysfunction of coagulation
l Isolated painful ulcers may occur which persist for
factors.
10-14 days and heal with scarring.
l In children due to infection loss of supporting bone
Treatment and Prognosis around the teeth.
l There is no known cure for haemophilia. The affected
persons should be protected from traumatic injuries. Radiological Features
l If surgical procedure such as tooth extraction must be
l IOPA typically exhibits mild to severe loss of superficial
carried out, it should be considered as a major one to be
alveolar bone, even in children, as a result of repeated
performed only in a hospital.
cyclic gingivitis advancing to periodontitis.
l Preoperative transfusion of whole blood and administration
l In children, loss of bone around the teeth is termed as
of antihaemophilic factor concentrate are recommended.
prepubertal periodontitis.

Dental Considerations
Laboratory Findings
l Tooth extraction should be done with minimum trauma.
l Cyclic neutropenia is an unusual disease which mani-
l Extraction of no more than 8 teeth in a mild case and no
fests the clinical signs and symptoms and blood changes
more than 4 teeth in a moderate or severe case is done.
in a periodic fashion.
l To avoid extractions in haemophiliac patients, regular
l The cycle commonly occurs every three weeks, in some
dental care to attend to caries and chronic gingivitis is
cases it may be several months or even longer in duration.
undertaken at regular intervals.
l Patient may exhibit normal blood count which, over a
Q. 4. Cyclic neutropenia. period of 4 to 5 days, begins to show perseptious decline
in the neutrophil count compensated by an increase in
Ans.
the monocytes and lymphocytes.
l Cyclic neutropenia is also known as periodic neutropenia l At the height of the disease, neutrophils completely
or periodic agranulocytosis or cyclic agranulocytic angina. disappear for a period of 1 or 2 days.
l Cyclic neutropenia is a unusual form of agranulocytosis l Soon, however, the cell begins to reappear and within 4
characterised by a periodic or cyclic diminution in cir- to 5 days differential count are essentially normal.
culating polymorphonuclear neutrophilic leucocytes as
a result of bone marrow maturation arrest accompanied
by mild clinical manifestations which spontaneously
Treatment
regress to attain a rhythmic pattern. No specific treatment, splenectomy is done.

SHORT ESSAYS
Q. 1. Agranulocytosis. l It is characterised by decreased number of circulating
granulocytes.
Ans.
Classification
l Agranulocytosis is also known as granulocytopenia.
Agranulocytosis is a serious disease involving white i. Primary agranulocytosis
blood cells. ii. Secondary agranulocytosis
Section | III Oral Pathology 597

Aetiology l The four major cardinal features of pernicious anaemia


are:
l Ingestion of drugs (antithyroid, marcolides, procain-
a. Abnormally large RBCs
amide, aprindine, dipyrone, digitalis, corticosteroids,
b. Hypochlorhydria
salicylates, others)
c. Neurologic and gastrointestinal symptoms
l infections
d. A fatal outcome unless the patient receives lifelong
injections of vitamin B12.
Clinical Features l Generalized weakness, fatigue, headache, palpitation,
nausea, vomting, anorexia, diarrhoea and dyspnoea.
l It occurs at any age but common in adults particularly in
l Shortness of breath, loss of weight, pallor and abdominal
women.
pain.
l It frequently affects workers in the health professions
l Patients have smooth, dry and yellow skin.
and in hospitals.
l Neurological manifestations include tingling sensation
l Commences with high fever, chills and soar throat, mal-
in hands and feet, paraesthesia of extremities due to
aise, weakness and prostration.
peripheral nerve degeneration.
l Skin appears pale anaemic and sometime jaundiced.
l Presence of infection in oral cavity, throughout GIT, RT,
and skin. Oral Manifestations
l Regional lymphadenitis.
l Glossitis, glossodynia (painful tongue) and glossopyro-
sis (itching, and burning tongue)
Oral Manifestations l Tongue appears beefy red in colour.
l Sometimes loss of papilla produces a bald appearance
l Necrotizing ulcers on oral mucosa, tonsils and pharynx
of tongue which is referred to as Hunter’s glossitis or
l No purulent discharge noticed
Moeller’s glossitis.
l Tooth extraction is contraindicated.

Laboratory Findings LABORATORY FINDINGS


i. Blood
l WBC count is below 2000 cells/cu mm with an almost
l RBC count is seriously decreased to 1,000,000 or less
absence of polymorphonuclear cells.
per Cubic millimetre.
l RBC and platelet count is normal.
l Macrocytosis
l Bone maroow is relatively normal except absence of
l Howell-Jolley bodies and Cabot’s rings punctuate
granulocytes, metamyelocytes, and myelocytes.
basophilia.
l Promyelocytes and myeloblasts are near normal numbers.
l Mild to moderate thrombocytopenia
l Iron deficiency
Treatment
l Not specific ii. Serum
l Withdrawal of causative drug and administration of an- l The indirect bilirubin, serum lactic dehydrogenase are
tibiotic to control infections. usually increased.
l The serum potassium, cholesterol and skeletal alkaline
Q. 2. Pernicious anaemia or addisonian anaemia. phosphatase are often decreased.
l Serum antibodies for IF are highly specific.
Ans.
l Pernicious anaemia is a type of a chronic progressive, iii. Gastric secretions
megaloblastic anaemia of adults caused due to defi- l Total gastric secretions are decreased to 10% of
ciency of intrinsic factors in stomach. reference range.
l It is probably an autoimmune disorder with a genetic l Achlorhydria
predisposition. l IF is either absent or markedly decreased.

iv. Bone marrow


Clinical Features
l Bone marrow biopsy and aspirate are hypercellular.
l Occurs rarely before 30 years of age and increases in
frequency with advancing age.
l No racial predilection in other countries, except in USA.
Treatment
Females are more commonly affected. l Administration of folic acid and vitamin B12..
598 Quick Review Series for BDS 3rd Year

l Early recognition and treatment provide a normal ii. Parenteral iron therapy
uncomplicated lifespan. l Intravenous iron therapy is indicated when the patient is
l Delayed treatment permits progression of the anaemia unable to tolerate oral iron, or has malabsorption or his
and neurological complications. needs are relatively acute.
l Previously used iron compound, iron dextran has been
Q. 3. Iron deficiency anaemia.
associated with the risk of anaphylaxis which is almost
Ans. never seen with newer preparations like sodium ferric
gluconate and iron sucrose.
l Iron deficiency anaemia is the most common form of
l Red blood cell transfusion: It is indicated in patients
anaemia worldwide.
with severe anaemia where cardiorespiratory conditions
warrant immediate intervention or when there is contin-
Aetiology ued and excessive blood loss.
Causes of iron deficiency anaemia are as follows: Q. 4. Plummer-Vinson syndrome.
i. Blood loss:
Ans.
a. Acute blood loss: Accident and surgery
b. Chronic blood loss: Gastritis, peptic ulcer, hook- l The Plummer-Vinson syndrome is the one manifesta-
worm infestation, haemorrhoids, and menstrual loss tion of the iron deficiency anaemia by Plummer in 1914
ii. Increased demand: Infancy, adolescence, and pregnancy and Vinson in 1922 under the term hysterical dysphagia.
iii. Malabsorption: Post-gastrectomy, sprue, and Crohn’s l It is a predisposition for the development of the carcinoma
disease in the upper alimentary tract.
iv. Inadequate diet l Depletion of iron oxidative enzymes may produce
myasthenic changes.
Clinical Features l It is also an autoimmune phenomenon as it is associated
with rheumatoid arthritis, pernicious anaemia, celiac
l It occurs at any age. disease and thyroiditis.
l Presents general symptoms of anaemia.
l Pagophagia, i.e. craving for ice, cheilosis and spoon- Clinical Features
shaped nails (koilonychia).
l Dysphagia due to formation of cricoid web (Plummer- l Occurs at any age chiefly in women in the 4th or 5th
Vinson or Patterson-Kelly syndrome). decade of life.
l Angular cheilitis, pallor of the skin, smooth red painful l Present general symptoms of anaemia
tongue with atrophy of filliform and fungiform papillae. l Pagophagia, i.e. craving for ice, cheilosis and spoon-
l Post-cricoid web is a premalignant lesion. shaped nails (koilonychia).
l Splenomegaly is uncommon. l Dysphagia due to formation of cricoid web (Plummer-
Vinson or Patterson-Kelly syndrome).
l Angular cheilitis, pallor of the skin, smooth red painful
Laboratory Findings tongue with atrophy of filliform and fungiform papillae.
l The general blood picture is microcytic hypochromic. l Carcinoma of hypopharynx and upper part of oesophagus.
l Serum iron and ferritin are low while total iron-binding
capacity (TIBC) is increased. Laboratory Findings
l Transferrin saturation is below 16%.
l Bone marrow stains for iron reveal decreased or absent l The general blood picture is microcytic hypochromic.
iron stores. l Serum iron and ferritin are low while total iron-binding
l Stool examination for parasites and occult blood is useful. capacity (TIBC) is increased.
l Bone marrow stains for iron reveal decreased or absent
iron stores.
Treatment l Stool examination for parasites and occult blood is useful.
i. Oral iron therapy
l The drug of choice is ferrous sulfate 200 mg thrice a Treatment
day (elemental iron 60 mg thrice a day) orally taken in
between meals. Oral iron therapy
l The treatment with oral iron is usually given for a long l The drug of choice is ferrous sulfate 200 mg thrice a day
duration and is sustained for 6-12 months even after (elemental iron 60 mg thrice a day) orally taken in
normalization of haemoglobin. between meals.
Section | III Oral Pathology 599

l The treatment with oral iron is usually given for a long l The haemoglobin content of RBC is increased but is pro-
duration and is sustained for 6-12 months even after portional to their increased size. Many other RBC abnor-
normalization of haemoglobin. malities are indicated along with the cells like polychro-
matophilic cells, stippled cells, nucleated cells, Howell-Jolly
Parenteral iron therapy bodies, and Cabot’s rings punctate basophilia.
l Intravenous iron therapy is indicated when the patient is l Leucocytes are remarkabely reduced in number but are
unable to tolerate oral iron, or has malabsorption or his increased in average size, in number of lobes to the nu-
needs are relatively acute. The newer preparations like cleus, and anicopoikilocytosis
sodium ferric gluconate and iron sucrose may be used. l Mild to moderate bocytopenia is noticed.
l Red blood cell transfusion is indicated in patients with
Q. 6. Clinical features and lab findings of cyclic neutro-
severe anaemia where cardiorespiratory conditions war-
penia.
rant immediate intervention or when there is continued
and excessive blood loss, Ans.
Q. 5. Describe the pathogenesis and blood picture of
pernicious anaemia. Clinical Features
Ans. l Occurs at any age commonly seen in infants and young
The pathogenesis of pernicious anaemia is as follows: children.
l Pernicious anaemia is a type of chronic progressive, l Patients manifest fever, malaise, sore throat, stomatitis,
megaloblastic anaemia of adults and is caused by defi- and regional lymphadenopathy.
ciency of intrinsic factors in stomach. l Headache, arthritis, cutaneous infection, and conjunc-
l It is probably an autoimmune disorder with a genetic tivitis.
predisposition and the disease is associated with human
leucocyte antigen (HLA) types A2, A3, B7 and A blood Oral Manifestations
group.
l Binding and blocking antibodies to IF are found in most l Severe gingivitis, stomatitis with ulceration
patients. l In children due to infection, loss of supporting bone
l A greater association between pernicious anaemia and around the teeth.
other autoimmune disorders are present.
l An association with Helicobacter pylori is also seen. Laboratory Findings
Blood picture of pernicious anaemia is as follows: l Normal blood count which, over a period of 4 to 5 days,
l The chronic disease often exhibits periods of remission begins to show perseptious decline in the neutrophil
and exacerbation. count compensated by an increase in the monocytes and
l The RBC count is seriously decreased often to 1,000,000 lymphocytes.
or less per cubic mm. l At the height of the disease, neutrophils completely
l Many of the cells exhibit macrocytosis, although disappear for a period of 1 or 2 days.
poikilocytosis, or variation in the shape of the cells is l Soon, however, the cell begins to reappear and within
also present. 4 to 5 days differential count is essentially normal.

SHORT NOTES
Q. 1. Thrombocytopenic purpura. Clinical Features
Ans. l Petechiae in skin, mucous membrane occurs.
l Pupura is defined as purpulish discolouration of skin l Epistaxis (bleeding from nose)
and mucous membrane due to subcutaneous and sub- l Bleeding from GIT (malena, haematemasis)
mucus extravasation of blood. l Intracranial haemorrhages
l In thrombocytopenic purpura due to thrombocytopenia, l Gingival bleeding and palatal petechiae.
purpuric spots of focal haemorrhages may occur in skin
and mucous membranes.
Investigations
l This is of two types:
i. Idiopathic/primary thrombocytopenic purpura l Platelet counts is below 1.5 lac / cu mm (usually below
ii. Secondary (due to various other causes) 50,000 / cu mm)
600 Quick Review Series for BDS 3rd Year

l BT prolonged, CT normal Treatment


l Increased megakaryocytes in bone marrow.
l Removal of the cause.
l Broad-spectrum antibiotics for oral ulcers should be
Treatment prescribed.
l For mild cases, no treatment is required. Q. 4. Strawberry tongue.
l Severe cases need transfusion of platelets, steroid
treatment and splenectomy. Ans.
Q. 2. Rh hump. l Strawberry tongue is an oral manifestation of scarlet
fever.
Ans. l The tongue exhibits white coating and fungiform papillae
l Rh hump is seen in erythroblastosis fetalis. are oedematous and hyperaemic.
l Erythroblastosis fetalis is manifestated by enamel hypo- l They project above the surface of the tongue as small
plasia in some cases. red knobs, this is called as strawberry tongue.
l This unusually involves the incisal edges of the anterior
Q. 5. Cyclic neutropenia.
teeth and the middle portion of the desciduous cuspids
and first molar crowns. Ans.
l Here a ring-like characteristic defect occurs known as
Rh hump. l Cyclic neutropenia is also known as periodic neutropenia
or periodic agranulocytosis.
Q. 3. Agranulocytosis. l It is characterised by a periodic or cyclic diminution in
Ans. circulating polymorphonuclear neutrophilic leucocytes
as a result of bone marrow maturation arrest.
l Agranulocytosis is a serious disease involving white
blood cells where the granulocytes specially neutrophils
are decreased below their normal lower limit. Clinical Features
l It occurs at any age commonly seen in infants and
Causes young children.
l Patients manifest fever, malaise, sore throat, stomatitis,
l Various drugs intake, e.g. chloramphenicol, quinine,
and regional lymphadenopathy.
sulfonamide, thiouracil, tolbutamide.
l Headache, arthritis, cutaneous infection, and conjunctivitis.
l Other causes are similar to leukocytopenia.

Clinical Features Oral Manifestations


l Occurs at any age and common in adults particularly in l Severe gingivitis, stomatitis with ulceration
women. l In children due to infection, loss of supporting bone
l Commences with high fever, chills and soar throat, mal- around the teeth.
aise, weakness and prostration
Skin appears pale anaemic and sometime jaundiced.
l
Radiographic Features
l Presence of infection in oral cavity, throughout GIT, RT,
and skin. Loss of alveolar bone may be seen on radiograph.
l Regional lymphadenitis
Treatment
Oral Manifestations
No specific treatment is present.
l Necrotizing ulcers on oral mucosa, gingiva, tonsils and
pharynx. Q. 6. Plummer-Vinson syndrome.
l Tooth extraction is contraindicated.
Ans.
l Since WBCs are reduced in number, the chances of in-
fections are increased which may cause bacteraemia l Plummer-Vinson syndrome is the one manifestation of
and septicaemia. the iron deficiency anaemia.
Section | III Oral Pathology 601

Clinical Features l Chediak-Higashi syndrome is a genetic disease, an au-


tosomal recessive trait.
l Occurs at any age chiefly in women in the 4th or 5th
l Oculocutaneous albinism, photophobia, nystagmus,
decade of life.
recurrent infections.
l Presents general symptoms of anaemia.
l Melanocytes with abnormal granules cause skin and
l Cheilosis and spoon-shaped nails (koilonychia).
hair changes.
l Dysphagia due to formation of cricoid web.
l Hair will have gray streaks.
l Angular cheilitis, pallor of the skin.
l Neuropathy and ataxia are prominent features in some
l Smooth red painful tongue with atrophy of filiform and
patients.
fungiform papillae.
l Oral ulceration, severe gingivitis, glossitis, periodontal
l Carcinoma of hypopharynx and upper part of oesophagus.
breakdown.
l Giant abnormal granules in circulating leucocytes are
Laboratory Findings the hallmark of the syndrome.
l No specific treatment.
l Microcytic hypochromic anaemia
l Serum iron and ferritin are low. Q. 9. Infectious mononucleosis.
l Bone marrow stains for iron reveal decreased or absent
iron stores. Ans.
l Stool examination for parasites and occult blood is useful. l Infectious mononucleosis is also known as glandular
fever or kissing disease.
Treatment l It is caused by EBV.
l Chiefly occurs in children and young adults.
Oral and parentral iron therapy. l Oral lesions include stomatitis, acute gingivitis, appear-
Q. 7. Von Willebrand’s disease. ance of a white or grey membrane in various areas,
palatal petechiae and occasional ulcers.
Ans.
Von Willebrand disease is a quantitative and qualitative
l
Laboratory Findings
defect of Von Willebrand factor.
l This factor carries factor ‘VIII’ and synthesizes a colla- l Increased neutrophil antibody titre (1:4096), i.e. posi-
gen bridge which causes adherence of platelets to dam- tive Paul-Bunnell test.
aged endothelial wall. So deficiency causes low factor l Thrombocytopenia.
‘VIII’ level and platelet dysfunction.
Treatment
Clinical Features
l Bed rest, adequate diet and short-term steroid therapy is
l Bruising, epistaxis the usual form of therapy.
l Gingival bleeding during brushing and extraction
Q. 10. Oral manifestations of pernicious anaemia.

Investigations Ans.

l Prolonged bleeding time The oral manifestations of pernicious anaemia are as


l Reduced factor ‘VIII’ activity follows:
l Low level of Van Willebrand factor l Glossitis, glossodynia (painful tongue) and glossopyro-
sis (itching, and burning tongue)
l Tongue appears beefy red in colour.
Treatment l Sometimes hyperpigmentation occurs in mucosa.
l Mild bleeding can be controlled with desmopressin. l Sometimes loss of papilla produces a bald appearance
l Severe bleeding can be controlled with factor ‘VIII’ of tongue which is referred to as Hunter’s glossitis or
concentrate. Moeller’s glossitis.
Q. 8. Chediak-Higashi syndrome. Q. 11. Oral manifestations of leukaemia.
Ans. Ans.
602 Quick Review Series for BDS 3rd Year

The oral manifestations of leukaemia in both acute and Oral manifestations:


chronic forms are as follows: l Purpulish red discolouration of tongue and gingiva.
l Gingivitis, gingival hyperplasia, haemorrhage, pete- l Gingiva is swollen and bleeds spontaneously without
chiae and ulceration of the oral mucosa. tendency to ulceration.
l Rapid loosening of the teeth due to necrosis of PDL.
l Alterations in the developing tooth crypts. Q. 14. Thalassaemia.
l Osseous changes in jaws.
Ans.
l Petechiae, bullae and burning sensation.
Q. 12. Mention the clinical features of haemophilia. l Thalassaemia is also called as Cooley’s anaemia.

Ans.
Clinical Features
Haemophilia is a potentially fatal inherited bleeding disor-
der characterized by the profuse haemorrhage due to defi- l Congenital disorder which is characterized by deficient
ciency of clotting factors. synthesis of haemoglobin, either a or b chain.
l Two types:
(a) Heterozygous or thalassaemia minor or thalassae-
Types mia trait.
i. Haemophilia A or classic haemophilia: In this, factor (b) Homozygous or b thalassaemias or thalassaemias
VIII deficiency is present. major.
ii. Haemophilia B or Christmas disease: In this, factor IX l Two forms of a-thalassaemia:
deficiency is present. (a) Hb-H disease (mild).
iii. von Willebrand’s disease: In this, factor VIII deficiency (b) Hb Bart’s disease with hydrops fetalis.
is present along with defective platelet function. l Mongoloid features, flaring of maxillary anteriors de-
pressed bridge of nose, unusual prominence of premax-
illa, poor spacing of teeth, a marked open bite, prominent
Clinical Features malar bone.
l It occurs only in males, and females are the carriers. l Ashen grey skin due to combination of pallor, jaundice
l Haemorrhagic tendency even in infancy. and haemosiderosis.
l Ecchymoses even from minor trauma.
Persistent oozing of blood.
l
Laboratory Findings
l Sudden bleeding in the muscles and joints (knee) and
joints becomes swollen and painful. l Hypochromic microcytic anaemia.
l Symptoms of anaemia. l WBC count elevated.
l Blood examination reveals prolonged clotting time l Presence of nucleated RBCs, “safety-pin” cells and
while bleeding time is normal. “target-cells”.
l Heinz bodies are formed by the precipitation of alpha
chains.
Oral Manifestations l Increased serum bilirubin.
l Gingival haemorrhage l Cellular hyperplasia of bone marrow.
l Eruption and exfoliated with severe haemorrhage
Mandibular pseudotumours.
Radiographic Features
l

Q. 13. Polycythaemia.
l Extreme thickening of diploe producing “crew-cut” or
Ans. “hair-on-end” appearance of surface of skull.
l Osteoporosis of skull and long bones.
l Polycythaemia is also called as Osler’s disease.
l Intraoral radiographs show “salt and pepper appearance”.
l There is an uncontrolled proliferation of erythroid stem
cells leading to excess of erythroid cell mass in the body
(RBCs). Treatment
l Common in males, occurs in middle age.
l Blood transfusion
l Symptoms: Lasstitude, headache, dizziness, blackout,
l Desferrioxamine is given for iron overload.
slurring of speech, pruritis, mental confusion and
indigestion. Q. 15. Sickle cell anaemia.
l Marked purpulish red discolouration of head and neck,
feet and hands giving angry appearance. Ans.
Section | III Oral Pathology 603

l Sickle cell anaemia is an autosomal dominant disorder. l Haemophilia A or classic haemophilia is a condition
l In sickle cell anaemia, the erythrocytes assume crescent where factor VIII (AHG) deficiency is present.
shape and undergo lysis.
Clinical Features
Clinical Features l Mild cases are asymptomatic with prolonged bleeding
after tooth extraction and any major surgery. The levels
l It is more common in females.
of factor ‘VIII’ lie between 7 and 50% (normal level is
l Malaise, weakness and jaundice with yellow sclera, pal-
50 to 150%)
lor, loss of appetite and muscular rigidity.
l In moderate cases, haematoma formation occurs after
l Fever, pain in abdomen, back and joints.
minor trauma or surgery (level 1 to 7%).
l Swelling joints, hand and feet.
l In severe case, level of factor ‘VIII’ is less than 1%. This
l There may be delayed eruption and hypoplasia of
causes spontaneous bleeding in muscles (haematomas)
dentition.
and weight-bearing joints (haemarthroses).
l There is presence of mongoloid faces with high cheek
bones and bimaxillary prognathism.
Oral Findings
Treatment l Haemorrhage from many sites in oral cavity.
l Tumour-like outgrowth in mandible (due to subperios-
l No specific treatment. teal bleeding and subsequent new bone formation).
l Oxygen and blood transfusion in serious situation. l TMJ haemarthroses.
Q. 16. Haemophilia A.
Treatment
Ans.
l Factor ‘VIII’ concentrate
l Haemophilia is a potentially fatal inherited bleeding l Fresh frozen plasma (FFP)
disorder characterized by the profuse haemorrhage due l Cryoprecipitate
to deficiency of clotting factors. l Desmopressin acetate (in mild cases).

Topic 19

Diseases of the Skin

LONG ESSAYS
Q. 1. Enumerate the vesiculobullous lesions and describe l Herpes labialis
the pemphigus vulgaris. l Measles

Ans.
B. Immunologic Conditions
The various types of vesiculobullous lesions that affect the l Pemphigus vulgaris
oral cavity have been categorised as follows: l Bullous form of lichen planus

A. Viral Diseases C. Hereditary Conditions


l Herpes gingivostomatitis l Epidermolysis bullosa
l Primary varicella zoster l Familial benign pemphigus (Hailey-Hailey disease)
l Secondary varicella zoster l Keratosis follicularis (Darier’s disease )
604 Quick Review Series for BDS 3rd Year

D. Miscellaneous l Intraorally, the bullae, if seen sufficiently early, appear


as vesicles on the palate, oropharynx or inside of the
l Impetigo
cheeks.
l When the affected epithelium ruptures, it leads to the
PEMPHIGUS VULGARIS formation of shallow painful ulcers that are covered
with a whitish “skin” which is the original roof of the
l Pemphigus vulgaris is the most common form of pem-
bulla. These oral lesions may persist for months before
phigus, accounting for over 80% of cases.
the skin becomes involved.
l The oral mucosa may be affected 2-3 months before the
Aetiology skin. Cheeks and vermilion border of the lips are the
common sites for the bullae.
l PV is an autoimmune disorder that is characterized by
l Nikolsky’s sign is positive, i.e. the loss of epithelium
the appearance of intraepithelial bullae on uninflamed
occasioned by rubbing apparently unaffected skin is
skin surface or mucous membranes.
termed as Nikolsky’s sign.
Mechanism of bullae formation is as follows: l The pemphigus vegetans is an uncommon form of pem-
l It is an autoimmune disease where circulating autoanti- phigus vulgaris, occurring in 1-2% of pemphigus vul-
bodies are directed against epidermal cell surface glyco- garis cases. Here bullae may not be observed, but firm
proteins, desmoglein 3 and desmoglein 1, these are nodular granulations covered with epithelium and
components of desmosomes which are responsible for deeply fissured area seen on cheeks and tongue.
intercellular adhesion in the epidermis. l In pemphigus vegetans, the affected mucosa has a cor-
l The autoantibodies attach to these desmosomal compo- rugated irregular surface referred to as vegetations and
nents effectively inhibiting the molecular interaction also, there are microabscesses full of eosinophils.
that is responsible for adherence.
As a result of this immunological attack on the desmo-
Histological Features
l

somes, a split develops within the epithelium causing a


blister to form. l The pemphigus is characterised microscopically by the
l Patients who have developed autoantibodies against formation of a vesicle or bulla entirely intraepithelially
desmoglein 3 with or without desmoglein 1 will histo- just above the basal layer producing a distinctive supra-
pathologically show intraepithelial clefting suprabasally basilar ‘split’.
and clinically oral mucosal blisters of pemphigus vul- l The suprabasal separation of epithelium (i.e. floating
garis will form. epithelium) is the most important diagnostic feature of
pemphigus vulgaris.
Disappearance of intercellular bridges results in loss of
Clinical Features l

cohesiveness or acantholysis because of which clumps


l Pemphigus vulgaris is commonly seen in people of of epithelial cells are found lying free within the ve-
50-60 years age group. sicular space, these cells are called “Tzanck cells.”
l It is insidious in its onset and can often be fatal. Tzank cells have large nuclei and hyperchromatic
l Men and women are equally affected. Jewish people are staining.
more commonly affected. l Bullae are filled with acantholytic multinucleated epi-
l The characteristic feature of pemphigus vulgaris is thelial cells known as “Tzank cells” which are diagnos-
rapidly appearing multiple vesicles and bullae which tic feature of this condition.
vary in diameter from a few millimetres to several l Immunofluorescent testing is considered to be of great
centimetres. importance in establishing the diagnosis of pemphigus
l The lesions (bullae) appear on a perfectly normal ap- vulgaris especially when the clinical or microscopic
pearing mucosa, although a large area of the skin sur- findings are inconclusive.
face may be affected, the eye is not involved. The bullae
could be rubbed with fingers.
l Pemphigus vulgaris can be fatal in several cases as the
Treatment
appearance of large bullae all over the skin surface l Corticosteroids remain the mainstay of treatment for
can lead to rapid fluid loss just like a case of severe pemphigus vulgaris.
burns. l Taking into account the pre-existing and co-existing
l The bullae are flaccid, fragile, regular, and non-inflammatory. conditions, therapy may be tailored for each patient.
They contain a thin watery fluid initially, which may soon l Various other therapies that have been reported
become purulent or sanguineous. as beneficial are parenteral gold therapy, dapsone,
Section | III Oral Pathology 605

tetracycline, and plasmapheres and administration of l Acanthosis of spinous cell layer and ‘saw-tooth’ appear-
8-methoxypsoralen. ance of rete pegs.
l Patient may continue to experience mild disease activity l There is presence of necrosis or liquefaction degenera-
while under optimal treatment. tion of basal cell layer of epithelium.
l Band-like subepithelial mononuclear infiltrate consist-
Q. 2. Describe the aetiology, histopathology, clinical fea- ing of T cells and histiocytes.
tures and differential diagnosis of oral lichen planus. l Chronic inflammatory cell infiltration is present in
Ans. juxta-epithelial region.
l Degenerating basal keratinocytes form rounded or
l Lichen planus is a common chronic, dermatologic dis- ovoid, amorphous eosinophilic bodies known as
ease of the skin and mucous membranes. “civatte, hyaline, cytoid” bodies.
l Degeneration of basal keratinocytes and disruption of
Aetiology anchoring elements of epithelial basement membrane
weakens the epithelial connective tissue interface re-
l Lichen planus is primarily an immune-related disorder sulting in histological clefts known as Max-Joseph
which may also be caused due to other factors. spaces.
l One of the major factors that can cause exacerbations or
remission of the condition is emotional trauma.
l Such cases are common following severe psychological Differential Diagnosis
stress such as death of a close friend or relative, marital Lichen planus must be differentiated from the lesions
or sexual problems, failure in a career, loss of job and which may present a similar clinical appearance, they are as
security, or exam tensions. follows:
i. Lichenoid reactions: Some of the varieties of medi-
Clinical Features cations may induce lesions that appear clinically very
similar to idiopathic form of lichen planus termed as
l Lichen planus commonly occurs among the middle lichenoid mucositis or lichenoid dermatitis.
aged and elderly people with slight predilection for ii. Leukoplakia: Some of the distinguishing features of
females. leukoplakia from lichen planus are that it is more
l Oral lichen planus can involve several areas of oral cav- common in men, found in slightly younger age group,
ity including the buccal mucosa, vestibule, tongue, lips, may have a family history, has no history of remission
floor of mouth, palate, and gingiva. and recurrence, usually involves commissures of the
l Patients may often report with burning sensation of oral mouth, surrounding mucosa is normal in appearance
mucosa. and symptom like soreness is felt.
l The oral lesion is generally characterized by radiating iii. Mucous patches of secondary syphilis: They are
white and grey velvety thread-like papules in linear, distinguished from lichen planus by the tendency for
angular or retiform arrangement. Tiny white elevated the papules to ulcerate in the centre, favour commis-
dots are usually present at the intersection of these white sure of lips and tonsils. Other manifestations include
lines and are known as “Wickham’s striae”. glandular enlargement and Treponema palladium
l Types of pattern of lichen planus in the oral cavity: found in oral lesions.
i. Linear pattern iv. Candidiasis (thrush): The white patches of candidia-
ii. Papular pattern sis can be easily scraped off and microscopically,
ii. Reticular spores and mycelia can be seen from the collected
iii. Annular pattern, i.e. circular specimens.
iv. Vesicular or bullous v. Recurrent apathae (ulcer): They are usually associ-
v. Erosive or atrophic ated with trauma.
vi. Hypertrophic vi. Pemphigus: It is characterised by bullous lesions re-
sulting on a normal looking mucosa and histologically
presence of acantholytic cells is diagnostic.
Histopathology vii. Lupus erythematosus: It is characterised by the area
The following histopathological changes may be noted- of atrophy and scarring remains stationary over many
which are suggestive of lichen planus: months or years and is firm on palpation.
l The overlying surface epithelium exhibits hyperortho- viii. Erythema multiforme: Can be differentiated by his-
keratosis or parakeratosis. tory and biopsy, its acute nature and severe involve-
l Thickening of granular cell layer. ment of labial mucosa.
606 Quick Review Series for BDS 3rd Year

Q. 3. Enumerate the mucocutaneous disorders. Describe Only the first two of them affect the oral mucosa
the pathogenesis, clinical features and histopathology of and also among them only pemphigus vulgaris is more
pemphigus. common.

Ans.
Clinical Features
There are several different skin disorders that involve the There are several different clinical features noted in pa-
mucous membranes including the oral mucosa. Some of tients suffering from pemphigus. Some of the characteris-
these include: tic clinical features are as follows:
l Ectodermal dysplasia l It occurs during the 4th, 5th and 6th decades of life and
l Lichen planus is more prevalent among the females.
l Psoriasis l It is characterised by rapidly developing vesicle or bulla
l Erythema multiforme on several areas of skin and mucous membrane, which
l Pemphigus contain clear fluid initially. Later on, formation of pus is
l Epidermolysis bullosa noted in these vesicles.
l Dermatitis herpetiformis l The vesicle ruptures very soon and leaves painful, ery-
l Systemic lupus erythematosus (SLE) thematous ulcers that bleed profusely.
l Systemic sclerosis l Gentle traction or oblique pressure on an affected area
around the lesion causes stripping of the normal skin or
mucous membrane. This feature is known as “Nikol-
PEMPHIGUS sky’s sign.”
Pemphigus is a group of vesiculobullous lesions of skin and l It is a fatal condition as the patient may die due to dehy-
mucous membrane, which is characterized by the formation dration and septicaemia.
of intraepithelial vesicles or bulla, causing separation of
epithelial membrane. Histopathology
Characteristic histopathological changes noted in the
Pathogenesis skin layers in patients affected by pemphigus include the
following:
l Pemphigus is an autoimmune disorder that affects
l The formation of a vesicle or bulla entirely intraepitheli-
the keratinocyte skin cell surfaces. The IgG antibod-
ally just above the basal layer producing a distinctive
ies bring about a loss of cell-to-cell adhesion by
suprabasilar ‘split’.
binding to the keratinocyte desmosomes and also to
l The suprabasal separation of epithelium (i.e. floating
the desmosome-free areas of the keratinocyte cell
epithelium) is the most important diagnostic feature of
membranes.
pemphigus vulgaris.
l The binding of these autoantibodies also activates
l Disappearance of intercellular bridges results in loss of
the complement system in the body bringing about the
cohesiveness or acantholysis because of which clumps
release of inflammatory mediators.
of epithelial cells are found lying free within the vesicu-
l The T cells are also activated and recruited in the in-
lar space, these cells are called “Tzanck cells.” Tzank
flammatory areas. All these changes bring about the
cells have large nuclei and hyperchromatic staining.
characteristic appearance of the bullous lesions noted in
l Bullae are filled with acantholytic multinucleated epi-
pemphigus.
thelial cells known as “Tzank cells” which are diagnos-
tic feature of this condition.
The different types of pemphigus include:
l Small number of polymorphonuclear leucocytes (PMNs)
i. Pemphigus vulgaris
and lymphocytes may be found.
ii. Pemphigus vegetans
iii. Pemphigus foliaceus Immunofluorescent testing is considered to be of great
iv. Pemphigus erythematosus importance in establishing the diagnosis of pemphigus vul-
v. Familial benign chronic pemphigus garis especially when the clinical or microscopic findings
vi. Benign mucous membrane pemphigus are inconclusive.
Section | III Oral Pathology 607

SHORT ESSAYS
Q. 1. Erythema multiforme. ii. Vesicles may form within epithelium or at epithelial
connective tissue junction.
Ans.
iii. Subepithelial connective tissue shows oedema and peri-
Erythema multiforme is an acute self-limiting inflamma- vascular infiltration of lymphocytes and macrophages.
tory dermatological disorder that involves skin, mucous
membrane and sometimes, internal organs.
Treatment
i. Cause should be identified and withdrawn.
Aetiology ii. Infections should be appropriately treated
i. Infectious agents: Mycoplasma pneumoniae, herpes iii. For all the forms of erythema multiforme, symptomatic
simplex. treatment including oral antihistamines, analgesics,
ii. Drug hypersensitivity: Barbiturates, sulfa drugs, salicy- local skin care and soothing mouthwashes is of great
lates. importance.
iii. Hyperimmune reaction: Due to formation of antigen- iv. Topical steroid therapy coupled with antibiotics may be
antibody complex against submucosal and dermal considered whereas systemic steroid therapy is contro-
blood vessels. versial.

Q. 2. Aetiopathogenesis of ectodermal dysplasia and its


Types oral manifestations.
a. EM minor or erythema multiforme minor: It represents Or
the localized eruptions of skin with mild or no mucosal
involvement. Oral manifestations of hereditary ectodermal dyspla-
b. EM major or erythema multiforme major or Stevens- sia.
Johnson syndrome (SJS): It is more severe mucosal and Ans.
skin disease and is potentially life-threatening disorder.
l Ectodermal dysplasia is also known as hereditary ecto-
dermal dysplasia or ectodermal dysplasia syndrome.
Clinical Features l Ectodermal dysplasia syndrome is a large heterogenous
i. Occurs chiefly in young adults between the age of 15 to group of inherited disorders primarily involving ecto-
40 years and males are more commonly affected than dermal structures involving skin, hair, nails, eccrine
females. glands and teeth.
ii. Characterised by the occurrence of asymptomatic vividly l The most common syndromes within this group are
erythematous discrete macules, papules or occasionally hypohydrotic or anhydrotic ED and hidrotic ED.
vesicles and bullae that appear symmetrically distributed
over hands and arms, legs and feet, face and neck. Aetiology
iii. The classical dermal lesions of erythema multiforme
which often appear on extremities are concentric ring- l It is a genetic disorder caused due to the presence of a
like resulting from varying shades of erythema giving sex-linked recessive gene which results in the aberrant
rise to terms ‘target’ ‘iris’ or “bull’s eye” lesions. development of the ectodermal derivatives during early
iv. The vesicles of mucosal surface develop rapidly and are embryonic life.
short lived and become eroded or ulcerated and bleed
profusely.
v. Recurrence is common, patient also develops tracheo- Clinical Features
bronchial ulceration and pneumonia. l Males are affected more commonly than females al-
though women have a higher risk of carrying this gene.
Skin, hair, eccrine glands, nails and teeth are the most
Histopathology l

commonly affected ectodermal derivatives.


i. The microscopic appearance of erythema multiforme is l Apart from the skin findings like sparse to complete ab-
not diagnostic. It usually consists of changes such as inter- sence of sclap hair, sweat glands may be absent or sparse.
cellular or intracellular oedema and necrosis of epithelium. Nails are often brittle and thin or show abnormal ridging.
608 Quick Review Series for BDS 3rd Year

Oral Manifestations angular or retiform arrangement. Tiny white elevated


dots are usually present at the intersection of these white
Several oral findings are of particular interest:
lines and are known as “Wickham’s striae”.
l Patients invariably manifest anodontia or oligodontia,
i.e. complete or partial absence of teeth and frequent
malformation of both deciduous and permanent teeth. Histopathology
l Abnormal morphogenesis of teeth like truncated or
cone-shaped teeth. The following histopathological changes may be noted-
l Dry and cracked protruberant lips with pseudorhagades which are suggestive of lichen planus:
formation. l The overlying surface epithelium exhibits hyperortho-
l Dry mouth due to the hypoplasia of salivary glands. keratosis or parakeratosis
l High palatal arch, cleft lip and cleft palate. l Thickening of granular cell layer.
l Acanthosis of spinous cell layer and ‘saw-tooth’ appear-
Q. 3. Microscopic features of pemphigus vulgaris. ance of rete pegs.
Ans. l There is presence of necrosis or liquefaction degenera-
tion of basal cell layer of epithelium.
Microscopic features of pemphigus vulgaris are as follows: l Band-like sub-epithelial mononuclear infiltrate consist-
l The pemphigus is characterised microscopically by the ing of T cells and histiocytes.
formation of a vesicle or bulla entirely intraepithelially l Chronic inflammatory cell infiltration is present in
just above the basal layer producing a distinctive supra- juxta-epithelial region.
basilar ‘split’. l Degenerating basal keratinocytes form rounded or
l The suprabasal separation of epithelium (i.e. floating ovoid, amorphous eosinophilic bodies known as
epithelium) is the most important diagnostic feature of “Civatte, hyaline, cytoid” bodies.
pemphigus vulgaris. l Degeneration of basal keratinocytes and disruption of
l Disappearance of intercellular bridges results in loss of anchoring elements of epithelial basement membrane
cohesiveness or acantholysis because of which clumps weakens the epithelial connective tissue interface re-
of epithelial cells are found lying free within the vesicu- sulting in histological clefts known as Max-Joseph
lar space these cells are called “Tzanck cells.” Tzank spaces.
cells have large nuclei and hyperchromatic staining.
l Bulla are filled with acantholytic multinucleated epithe- Q. 5. Write briefly about Stevens-Johnson syndrome.
lial cells known as “Tzank cells” which are diagnostic
Ans.
feature of this condition.
l Immunofluorescent testing is considered to be of great i. Stevens-Johnson syndrome is a very severe bullous
importance in establishing the diagnosis of pemphigus form of erythema multiforme with widespread in-
vulgaris especially when the clinical or microscopic volvement typically including skin, oral cavity, eyes
findings are inconclusive. and genitalia.
ii. It is characterised by the abrupt appearance of symp-
Q. 4. Clinical features and histopathology of lichen
toms such as fever, photophobia, malaise and erup-
planus.
tions over the oral mucosa, skin and the genitalia.
Ans. iii. Widespread involvement of different regions like the
skin and the eyes are also noted which are commonly
l Lichen planus is a common chronic, dermatologic disease
haemorrhagic and are often vesicular or bullous.
of the skin and mucous membranes.
iv. The oral mucosal lesions are extremely painful and
mastication is usually impossible. In many cases, the
oral lesions may be the chief complaint.
Clinical Features
v. The lips may exhibit ulceration with bloody crusting
l It is commonly noted during the 5th or 6th decade of life and are painful.
and is more common in females. vi. Genital lesions include nonspecific urethritis, balanitis,
l Oral lichen planus can involve several areas of oral cav- and vaginal ulcers.
ity including the buccal mucosa, vestibule, tongue, lips, vii. Some of the other complications may include tracheo-
floor of mouth, palate, and gingiva. bronchial ulcerations and pneumonia. Patients usually
l Patients may often report with burning sensation of oral recover unless they are secondarly infected.
mucosa. viii. Treatment:
l The oral lesion is generally characterized by radiating l Cause should be identified and withdrawn and
white and grey velvety thread-like papules in linear, infections should be appropriately treated.
Section | III Oral Pathology 609

l Symptomatic treatment including oral antihista- l The influence of insulin or insulin-like growth factor
mines, analgesics, local skin care and soothing (IGF) which stimulates the epidermal cells is well
mouthwashes is of great importance. established.
l Topical steroid therapy coupled with antibiotics l The malignant forms are thought to occur due to

may be considered whereas systemic steroid ther- the stimulation of these cells by some substance
apy is controversial. secreted by the tumours.
l Stimulation of the epidermal keratinocytes and der-
Q. 6. Enumerate the differences between leukoplakia mal fibroblasts bring about their proliferation lead-
and lichen planus. ing to acanthosis.
l Exogenous medications are also implicated as aetio-
Ans.
logical factors.
The differences between leukoplakia and lichen planus are iv. Clinical features:
as follows: l Acanthosis nigricans is more common in dark-

skinned individuals affecting both men and women


Leukoplakia Lichen planus equally.
l The skin lesions may range from symmetric, mild
Is a premalignant lesion due Is a common chronic, derma-
to external irritation, e.g. tologic disease of the skin hyperpigmentations to heavily pigmented, aggressive
jagged tooth and mucous membranes verrucous lesions.
due to drugs, emotional l Oral mucosa is involved in about 25-50% of the
disturbance
cases. The tongue and the lips are most frequently
More common in males, the More common in females and involved and is characterised by features such as
age of onset is above 30 years is usually observed during the hypertrophy of the filliform papillae and enlarged
with peak incidence above 5th or 6th decade of life
lips that are covered by papillomatous growths.
the age of 50 years.
l Buccal mucosa or palate are less frequently involved,
Generalized hyperkeratosis Localized hyperkeratosis and shows a velvety white appearance with papillary
distribuited asymetrically Symmetrically distributed
lesions occasionally. Gingival enlargement may be
Patient is careless type with Seen in nervous and high similar to that noted in idiopathic fibromatosis.
many habits like chewing strung patients. v. Treatment:
and smoking tobacco, taking
l Correction of the underlying disease process such as
alcohol, etc.
hyperinsulinaemia often reduces the symptoms.
Site of location anterior Site of location posterior part
l Treatment of lesions is usually for cosmetic pur-
region of the oral cavity, of oral cavity, no epithelial
epithelium shows mitosis mitosis poses only.
l Malignant form of acanthosis nigricans has a poor
Complains of soreness Asymptomatic and gives only
prognosis.
rough feeling
No remissions and exacerba- The course of disease is long Q. 8. Hailey-Hailey disease.
tions of the lesions with periods of remissions
and exacerbations Ans.
Changes to papillomatous Changes to atrophic and i. Familial benign pemphigus is also known as Hailey-
stage and even up to bullous forms
neoplastic
Hailey disease as it was originally described by Hailey
brothers in 1939.
ii. It is supposed to be resulting from a genetic defect in
Q. 7. Acanthosis nigricans. calcium pump protein. In addition to primary gene
defect, contributing factors like heat, friction and in-
Ans.
fection are known to exacerbate the disease.
i. Acanthosis nigricans is an unusual form of dermatosis iii. The disease usually manifests during adolescence or
which is divided into benign and malignant forms. young adult life. No sex predilection.
ii. The patients with benign form experience very few iv. The lesions develop as small groups of vesicles ap-
complications following the occurrence of skin lesions, pearing on normal or erythematous skin which eventu-
those with malignant forms are associated with signifi- ally rupture to leave eroded, crusted areas. Nikolsky’s
cant complications. sign is present.
iii. Aetiology: v. Oral mucosa is affected in all cases off amilial benign
l The definitive cause of acanthosis nigricans is not chronic pemphigus and lesions develop slowly caus-
yet ascertained. ing superficial erosions with well-defined margins.
610 Quick Review Series for BDS 3rd Year

vi. The oral lesions develop as crops of vesicles which iii. Clinical features: It is characterised by a wide variety of
rapidly rupture leaving raw eroded areas. features. Some of them are as follows:
vii. There is history of involvement of other mucous mem- l Relative focal absence of dermis.

branes like urethral, anal, nasal, rectal, vulval. The le- l Skin atrophy, streaky pigmentation and telangiec-

sions heal with scarring, after a benign, chronic course. tasia.


viii. Histological appearance of epithelial lesions is similar l Multiple skin or mucosal papillomas

to pemphigus vulgaris and keratosis follicularis. The- l Anomalies of extremities like syndactyly, polydac-

acantholysis is more extensive than pemphigus vul- tyly and adactyly.


garis and there is less damage to acantholytic cells. l Asymmetrical face with pointed chin and notched

‘Dilapidated brick wall effect’ is characteristic histo- nasal alae.


logic feature of this disease. l Eye abnormalities most frequently iris and choroid

ix. Treatment: colobomata and strabismus.


l Trasient improvement is obtained by the use of iv. Oral manifestations:
soothing compresses like aluminium acetate fol- l Papillomas of lips have been striking feature in

lowed by intermittent use of mild corticosteroid many patients. Papillomas of buccal mucosa and
preperations and topical antibiotics like clindamy- gingiva may also be present.
cin and erythromycin. l Size, shape and structure of teeth are affected resulting

l Systemic antibiotics may be required to suppress in microdontia, enamel hypoplasia, etc.


more widespread flares. l Cleft lip and/or cleft palate is seen in several cases.

v. Histological features:
Q. 9. Goltz’s syndrome.
l Atrophic reticulated patches of skin.

Ans. l Partial or complete absence of significant portions of

dermis
i. Focal dermal hypoplasia syndrome is also known as
l Appearance of adipose cells in dermis
Goltz’s syndrome and is a rare genetic disorder.
vi. Treatment:
ii. It is characterised by the appearance of skin lesions and
l The goal of treatment is to correct the skeletal and
defects of the eyes, teeth and several other organ sys-
soft tissue anomalies that would have resulted from
tems such as the urinary, skeletal, cardiovascular, gas-
this disorder.
trointestinal, and central nervous system.
l Treatment is mainly oriented towards achieving
ii. Goltz’s syndrome is an X-linked dominant disorder that
functional or cosmetic reults.
is lethal in males.

SHORT NOTES
Q. 1. LE cells. Q. 3. Tzank test.
Ans. Ans.
l LE cells (lupus erythematsous cells) are characteristi- l Tzank test is considered as a rapid supplemental test for
cally found in patients suffering from acute systemic pemphigus.
form of lupus erythematosus. l It involves taking of cytological smears from freshly
l The cells consist of rosette of neutrophils surrounding a opened vesicles, these smears characteristically display
pale nuclear mass derived from lymphocytes. ‘Tzank cells’ that are diagnostic of pemphigus.
l Tzank cells are clumps of large hyperchromatic epithe-
Q. 2. Tzanck cells.
lial cells lying free within the vesicular fluid.
Ans. l These cells are also characterised by swollen nucleus
and hyperchromatic staining.
l “Tzank cells” are acantholytic cells which are diagnos-
tic feature of pemphigus. Q. 4. Nickolsky’s sign.
l In cases of pemphigus bullae are filled with acantholytic
Ans.
multinucleated epithelial cells known as “Tzank cells”.
l As result of acantholysis clumps of epithelial cells are l Nikolsky’s sign is a common test for pemphigus.
found lying free within the vesicular space, these cells l Gentle traction or oblique pressure on an affected area
are called “Tzanck cells”. around the lesion causes stripping of the normal skin
l “Tzank cells” have large nuclei and condensation of or mucous membrane, this is known as “Nikolsky’s
chromatin along cell wall. sign.”
Section | III Oral Pathology 611

l Caused due to the presence of perivascular oedema that Q. 9. Stevens-Johnson syndrome.


disrupts the dermal-epidermal junction.
Ans.
Q. 5. Histopathology of oral lichen planus. i. Stevens-Johnson syndrome is simply a severe bullous
form of erythema multiforme with widespread involve-
Ans.
ment of skin, oral cavity, eyes and genitalia.
The following are the histopathological features of lichen ii. It commences with abrupt occurrence of following features:
planus: l Fever

l The overlying surface epithelium exhibits hyperortho- l Malaise

keratosis or parakeratosis. l Photophobia

l Increase in granular layer and acanthosis of spinal cell l Erythematous eruptions of oral mucosa, genitelia

layer and ‘saw-tooth’ appearance of epithelial ridges is and skin.


present. l Purulent conjunctivitis (eye)

l There is presence of necrosis or liquefaction degenera- l Constitutional disturbances

tion of basal cell layer of epithelium and subepithelial l Cutaneous lesions are similar to those of erythema

oedema. multiforme, they are commonly haemorrhagic and


l Chronic inflammatory cell infiltration is present in are often vesicular or bullous.
juxta-epithelial lesion. l Oral mucous membrane lesions may be extremely

l Few rounded or ovoid, amorphous eosinophilic bodies severe and so painful that mastication is impossible.
are present and are known as “Civatte bodies. iii. Treatment: Systemic use of corticosteroids.

Q. 6. Auspitz’s sign. Q. 10. Corps, ronds and grains.


Ans.
Ans.
l Corps, ronds and grains are characteristic histological
l Auspitz’s sign is a characteristic sign of psoriasis
features of keratosis follicularis or Darier disease.
l Psoriasis is characterised by the appearance of small dry
l Corps, ronds are slightly larger than normal squamous
papules covered by silvery scales.
cells and are present in granular layer and superficial
l Removal of deep scales reveals one or more, tiny bleed-
spinous layer.
ing points, this is known as Auspitz’s sign.
l The nucleus of corps, ronds cell is round, homogenous
Q. 7. Monro’s abscess. and basophilic and has a distinct cell membrane.
l Grains refer to the small, elongated parakeratotic cells
Ans. noted in keratin layer.

l Monro’s abscesses are a characteristic histological fea- Q. 11. Clinical types of lichen planus.
ture of psoriasis.
Ans.
l Intraepithelial microabscesses in the upper part of the
malpighian layer are known as Monro’s abscesses. Various clinical types of lichen planus in the oral cavity are
l These are nothing but the collections of neutrophils as follows:
within the parakeratin layer of epithelium. i. Linear
ii. Papular
Q. 8. White sponge nevus. iii. Confluent
iv. Reticular
Ans.
v. Annular or circular
l White sponge nevus is also called Cannon’s disease or vi. Pigmented
oral epithelial nevus or congenital leucokeratosis. vii. Vesicular or bullous
l A congenital mucosal abnormality, in some cases, may viii. Erosive or atrophic
not appear till adolescence. ix. Hypertrophic
l Oral lesions involve palate, cheeks, gingiva, floor of the
Q. 12. Oral manifestations in hereditary ectodermal
mouth, tongue.
dysplasia.
l Mucosa appears thickened, folded or corrugated
and has a soft or spongy texture with a white opales- Ans. Several oral manifestations of hereditary ectodermal
cent hue. dysplasia are as follows:
l The condition is benign and there is no treatment and is l Oligodontia or hypodontia, i.e. markedly reduced num-
not associated with any clinical complications. ber of teeth.
612 Quick Review Series for BDS 3rd Year

l Abnormal morphogenesis of teeth resulting in conical l They represent degenerating basal keratinocytes seen in
or pointed incisor teeth. the area of epithelium and connective tissue interphase.
l Anodontial Congenital absence of teeth
Q. 17. Ehlers-Danlos syndrome.
l Dry mouth, dry and cracked lips are due to the hypoplasia
of salivary glands. Ans.
Q. 13. Target’ lesions. l Ehlers-Danlos syndrome is also known as cutis hyer-
elastica. It is a group of more than 10 different inherited
Ans.
disorders.
l The ‘Target’lesions are characteristic in patients suffer- l Involves a genetic defect in synthesis of collagen and con-
ing from erythema multiforme. nective tissues and affects skin, joints and blood vessels.
l The classical dermal lesions of erythema multiforme l Characterised by hyperelasticity of skin, hyperextensive
which often appear on extremities are concentric joints and excessive bruising. Patient is known as circus
ring-like resulting from varying shades of erythema rubber man.
giving rise to terms ‘target’ ‘iris’ or “bull’s eye” l The facial features are distinctive with hypertelorism,
lesions. wide nasal bridge, epicanthic folds, protruding ears and
l The concentric erythematous lesions may be purpuric or frontal bossing.
paler in the centre, and has variety of appearances hence l Gingival tissues may be fragile and tooth structure may
the name multiforme. be altered.
l No known treatment exists.
Q. 14. Wickham’s striae.
Q. 18. Darier’s disease.
Ans.
Ans.
l Wickham’s striae are characteristic clinical feature of
lichen planus. l Darier’s disease is also known as keratosis follicularis
l The surface of the skin papules is covered by character- or Darier-White disease.
istic very fine, lace-like network of greyish white lines l It is characterised by abnormal cell-cell adhesion and
known as Wickham’s striae. aberrant epidermal keratinisation. Recently mutations in
l Reticular lichen planus is so named because of this ATP2A2 gene were found.
characteristic pattern of interlacing white lines. l Cutaneous lesions appear as small firm red papules
l They may occur any where on the skin surface but more initially, which later become greyish brown or even
often on the flexor surfaces of wrist and forearms, the purple, ulcerate and crust over.
inner aspect of knees and thighs. l Minute, whitish papules that are rough on palpation are
the common oral findings.
Q. 15. Raynaud’s phenomenon.
l It is effectively treated with oral retinoids.
Ans.
Q. 19. CREST syndrome.
l Raynaud’s phenomenon is one of the symptoms seen in
Ans.
CREST syndrome.
l It is observed when the persons hands or feet are ex- l CREST syndrome is an uncommon condition which
posed to cold temperatures. may be a mild variant of systemic sclerosis.
l Initial clinical sign is dramatic blanching of digits l The term CREST is an acronym for calcinosis cutis,
which appear dead-white in colour as a result of severe Raynaud’s phenomenon, esophageal dysfunction,
vasospasm. Few minutes later affected extremity takes sclerodactyly and telangiectasia.
bluish colour due to venous stasis. Finally, after warm- l Sclerodactyly with characteristic ‘claw’ deformity is at-
ing with the return of hyperaemic blood flow, the digits tributed to abnormal deposition of collagen.
turn to dusky red hue. l No specific treatment exists, symptomatic treatment is
l Varying degrees of throbbing pain may accompany this advised.
phenomenon.
Q. 20. Grinspan’s syndrome.
Q. 16. Civatte bodies.
Ans.
Ans.
l Grinspan’s syndrome refers to the triad of lichen planus,
l Civatte bodies are characteristic histological feature diabetes mellitus and vascular hypertension.
seen in lichen planus and are also known as colloid, l Described by Grinspan
hyaline or cytoid bodies. l This association of oral lichen planus (OLP) and sys-
l Civatte bodies are rounded or ovoid, amorphous eosino- temic diseases may be coincidental as the lichen planus
philic globules seen under the microscope. commonly occurs in older adults.
Section | III Oral Pathology 613

Topic 20

Diseases of the Nerves and Muscles

LONG ESSAYS
Q. 1. Mention the different types of neuralgias; explain l The term tic-douloureux is applied only when the pa-
in detail about trigeminal neuralgia. tient suffers from spasmodic contractions of the facial
muscles.
Ans.
l ‘Trigger zone’ is characteristic feature of the trigeminal
Various types of facial and cervical neuralgias are as neuralgia. The ‘trigger zones’ which precipitate an attack
follows: when touched are usually common on the vermillion
i. Trigeminal neuralgia border of the lips, ala of nose and around eyes.
ii. Raeder’s paratrigeminal neuralgia l Stimulation of trigger zone due to touching, laughing or
iii. Atypical pain/neuralgia eating precipitate an attack of pain. In some cases, even
iv. Geniculate neuralgia exposure to strong breeze or simply the act of eating or
v. Glossopharyngeal neuralgia smiling has been known to precipitate the pain.
vi. Migrainous neuralgia
vii. Occipital neuralgia
Differential Diagnosis
viii. Post-herpetic facial neuralgia
ix. Sphenopalatine ganglion neuralgia l Migraine or migrainous neuralgia (Horton’s syndrome,
x. Superior laryngeal neuralgia histamine headache, histamine cephalgia )
xi. Tympanic plexus neuralgia l Post-herpetic neuralgia
l Costen’s syndrome
Trotter’s syndrome
TRIGEMINAL NEURALGIA
l

Trigeminal neuralgia is a disease of trigeminal nerve or


l
Treatment
fifth cranial nerve. It is otherwise known as tic-douloureux
or Fothergill’s disease or trifacial neuralgia. The treatment of trigeminal neuralgia is extremely varied
over the years. The various treatment modalities are as
follows:
Aetiology i. Medical management: Commonly used drugs are:
l Most cases are idiopathic. l Carbamazepine (up to a dose of 600 to 1200 mg/

l Sometimes it occurs due to pressure over trigeminal day): This drug is frequently used as therapeutic
nerve, e.g. by tumours or vascular anomalies, etc. challenge to the diagnosis of trigeminal neuralgia.
l Circulatory insufficiency or reflex vasoconstriction of l Phenytoin (dilantin 100 mg TDS): Use of this drug

Gasserian ganglion. has been found be efficacious in some cases.


l An area of demyelination as found in patients with l Anticonvulsants

multiple sclerosis may be the precipitant. l Baclofen (50 to 60 mg/day)

ii. Injection of alcohol or boiling water into peripheral nerve


area or centrally into the gasserian ganglion has been re-
Clinical Features ported to be beneficial in causing respite from pain.
l More common in older adults compared to young iii. Surgical treatment:
persons. l Peripheral neurectomy: It is one of the earliest forms

l Pain is confined to area of distribution of trigeminal of the treatment for trigeminal neuralgia, which
nerve. includes sectioning of nerve at mental foramen, or at
l Right side of the face is affected in more patients com- supraorbital or infraorbital foramen.
pared to left side. l Microsurgical decompression of trigeminal nerve

l The pain is usually searing, stabbing or lancinating root is one of the newest procedures for the manage-
type, which lasts only for few seconds or minutes and is ment of trigeminal neuralgia. It has been reported to
unilateral, seldom crosses the midline. produce good results.
614 Quick Review Series for BDS 3rd Year

SHORT ESSAYS
Q. 1. Bell’s palsy. l Administration of flushing doses of nicotinic acid has
produced excellent results in a series of cases.
Ans.
l Surgical decompression of the intratemporal facial
l Bell’s palsy is one of the most common neurological nerve is used in selected cases. Topical ocular antibiot-
disorders affecting the cranial nerves. ics and artificial tears may be required to prevent cor-
l It is an abrupt, isolated, unilateral peripheral facial neal ulcerations and the eyelid may have to be taped
nerve paralysis. shut.
Q. 2. Mention criteria for diagnosis of trigeminal
Aetiology neuralgia.
l Usually considered as idiopathic. Ans.
l Viral infection (HSV) is the most likely the infectious
Trigeminal neuralgia is a disease of trigeminal nerve or fifth
agent.
cranial nerve. It is otherwise known as tic-douloureux or
l Ischaemia of the nerve near the stylomastoid foramen,
Fothergill’s disease or trifacial neuralgia.
resulting in oedema and compression of nerve while it
Necessary criteria for diagnosis of trigeminal neuralgia
passes through the temporal bone.
are as follows:
l Various inflammatory, demyelinating, ischaemic or
i. The pain is usually searing, stabbing or lancinating
compressive processes may impair neural conduction at
type, which lasts only for few seconds or minutes and
this unique anatomical site.
is unilateral, seldom crosses the midline.
ii. The pain must be limited to the known distribution of
Clinical Features one or more branches of the trigeminal nerve with no
motor deficit in the affected area.
l It begins abruptly as a paralysis of the facial muscula- iii. The onset of a pain “attack” is abrupt, often initiated
ture, usually unilaterally causing mask like expression- by a light touch to a specific and constant trigger
less face. point.
l Women are more commonly affected. iv. The duration of single pain spasm is less than 2 minutes.
l Middle-aged is most susceptible. v. For several minutes after an attack, i.e. during refrac-
l Drooping of corners of mouth, epiphora, drooling of tory period, touching the trigger point usually cannot
saliva, inability to close or wink the eye, loss of wrin- induce additional attacks.
kling of forehead, “mask-like expressionless face”. vi. The pain is dramatically diminished at least initially,
l Taste sensation is lost or altered in anterior 3rd of the with the use of carbamazepine.
tongue. vii. Spontaneous remissions occur, often lasting for more
l Melkersson-Rosenthal syndrome: Recurrent attacks of than 6 months, especially during the early phase of the
facial plasy 1 multiple episodes of non-pitting, non- disease.
inflammatory painless oedema of face, cheilitis granulo-
matosa and fissured tongue. Q. 3. Glossopharyngeal neuralgia.
l Ramsay-Hunt syndrome: Facial palsy 1 herpes zoster
Ans.
of geniculate ganglion.
l Bogorad syndrome: Crocodile tears generally follows l Glossopharyngeal neuralgia is the disease of ninth
Bell’s palsy, result of herpes zoster or head injury, lead- cranial nerve, i.e. glossopharyngeal nerve.
ing to a salivary lacrimal reflex arc. Manifested as lacri- l It is similar to trigeminal neuralgia in every way except
mation on eating. in anatomical location of the pain.
l The age of onset varies from 15-85 years but the average
age is 50 years.
Treatment l No sex predilection and rarely there is bilateral involve-
l There is no universally preferred treatment for Bell’s ment.
palsy, patients may recover within several weeks to a l Sharp shooting pain in ear, pharynx, nasopharynx, ton-
month. sil and posterior portion of tongue, i.e. at base of tongue
l Histamine and other vasodilator drugs may shorten the and fauces on one side.
duration as will systemic corticosteroids and hyperbaric l Trigger zone is present in posterior oropharynx or ton-
oxygen therapy and are proved to be beneficial in some sillar fossa. It is stimulated during swallowing, talking,
cases. coughing or yawning.
Section | III Oral Pathology 615

l May be associated with vagal symptoms such as syn- Q. 4. Frey’s syndrome.


cope, hypotension, and arrhythmias. Or cardiac arrest
Ans.
may accompany the paroxysmal pain as may coughing
or excessive salivation. l Auriculotemporal syndrome or Frey’s syndrome or gus-
tatory sweating arises from sectioning of auriculotem-
poral nerve followed by re-innervation of sweat glands
Treatment by parasympathetic salivary fibres.
l Approximately 80% of patients experience immediate l The sectioning or damage of auriculotemporal nerve may
pain relief when topical anaesthetic agent is applied to be due to injuries associated with removal of parotid
tonsil and pharynx on the side of pain. It is used as tumour or the ramus of the mandible or a parotitis of
diagnostic tool and emergency measure than a long- some type that has damaged the auriculotemporal nerve.
term treatment.
No therapy is considered to be uniformly effective or
l
Clinical Features
even adequate.
l Glossopharyngeal neuralgia is considerably less respon- The patient typically exhibits flushing and sweating of
sive than trigeminal neuralgia to treatment with anticon- involved area of the face, chiefly temporal area during
vulsant medications. eating which is increased by taking hot food.
l If the patient fails drug therapy, then surgical options
should be considered.
Treatment
l The preferred neurosurgical treatments are microvascu-
lar decompression or surgical sectioning of the glosso- The treatment of auriculotemporal syndrome by intracra-
pharyngeal nerve and the upper two rootlets of the nial division of the auriculotemporal nerve has been
vagus nerve. reported to be successful.

SHORT NOTES
Q. 1. Bell’s palsy. to auriculotemporal nerve and subsequent innervation
of sweat glands by parasympathetic salivary fibres.
Ans.
ii. The patient typically exhibits flushing and sweating of
i. Bell’s palsy is an abrupt, isolated, unilateral peripheral involved area of the face, chiefly temporal area during
facial nerve paralysis. eating which is increased by taking hot food.
ii. Aetiology: May be idiopathic or due to viral infections iii. The treatment of auriculotemporal syndrome by intra-
(HSV) or ischaemia of the nerve near the stylomastoid cranial division of the auriculotemporal nerve has been
foramen. reported to be successful.
iii. Clinical features:
l Paralysis of the facial musculature, usually unilaterally Q. 3. Fothergill’s disease.
causing mask like expressionless face. Ans.
l Middle-aged women are more commonly affected.

l Drooping of corners of mouth, epiphora, drooling of i. John Fothergill in 1773 published detailed description
saliva, inability to close or wink the eye, loss of of the trigeminal neuralgia, since then it has referred to
wrinkling of forehead, “mask-like expressionless as ‘Fothergill’s’ disease.
face”. ii. It is a disease of trigeminal nerve or fifth cranial nerve.
l Taste sensation is lost. It is otherwise known as tic douloureux or trifacial neu-
iv. Treatment: There is no universally preferred treatment ralgia.
for Bell’s palsy. iii. More common in older adults compared to young persons.
l Histamine and other vasodilator drugs iv. The treatment of trigeminal neuralgia is extremely var-
l Surgical decompression of the intratemporal facial ied over the years including both medical and
nerve. surgical modalities.

Q. 2. Frey’s syndrome. Q. 4. Trigger zones.


Ans. Ans.
i. Auriculotemporal syndrome is also known as Frey’s i. ‘Trigger zone’ is characteristic feature of the trigeminal
syndrome or gustatory sweating. Occurs due to damage neuralgia.
616 Quick Review Series for BDS 3rd Year

ii. The ‘trigger zones’ are the regions which precipitate an ii. Clinical features:
attack when touched are usually common on the ver- l Unilateral paroxysm of severe pain at or near eyes,

million border of the lips, ala of nose and around eyes. maxilla, ear, mastoid and nose base.
iii. Stimulation of trigger zone due to touching, laughing or l No trigger zone is present.

eating precipitate an attack of pain. l Usually pain occurs once in a day (alarm clock

headache).
Q. 5. Eagle’s syndrome. l Sneezing and watering from eyes are other com-

Ans. plaints.
iii. Treatment:
i. Eagle’s syndrome consists of either elongation of styloid l Alcohol injection of sphenopalatine ganglion.
process or ossification of stylohyoid ligament. l Ergotamine or methysergide often provides com-
ii. Causes dysphagia, sore throat, otalgia, glossodynia, plete relief of symptoms.
headache, vague orofacial pain or pain along the distri-
bution of the internal and external carotid arteries. Q. 9. Facial causalgia.

Q. 6. Glossopharyngeal neuralgia. Ans.

Ans. i. Atypical facial pain or facial causalgia constitutes a


group of conditions in which there is a vague deep pain
i. Glossopharyngeal neuralgia is the disease of ninth in the regions supplied by 5th and 9th cranial nerves
cranial nerve. and 2nd and 3rd cervical nerves.
ii. Sharp shooting pain in ear, pharynx, nasopharynx, ton- ii. No specific cause, however, injury of any peripheral or
sil and posterior portion of tongue, i.e. at base of tongue proximal branch of trigeminal nerve due to facial
and fauces on one side. trauma or basal skull fractures can produce the
iii. Trigger zone is present in posterior oropharynx or ton- disorder.
sillar fossa. It is stimulated during swallowing, talking, iii. It usually follows extraction of multirooted tooth.
coughing or yawning. iv. The medical treatment of atypical facial pain or facial
iv. No therapy is considered to be uniformly effective or causalgia is less satisfactory than that of trigeminal
even adequate. neuralgia. Among the non-narcotic drugs, tricyclic an-
If the patient fails to drug therapy, then surgical options tidepressants give best results.
should be considered. The preferred neurosurgical treat- Q. 10. Myasthenia gravis
ment is microvascular decompression.
Ans.
Q. 7. Tic douloureux.
i. Myasthenia gravis is an acquired autoimmune disorder
Ans. characterized by progressive weakness of skeletal
muscles, particularly those innervated by cranial
i. Nicholaus Andre in 1756 coined the term ‘Tic Doulou-
nerves.
reux’
ii. There is a defect in neuromuscular transmission.
ii. The term tic douloureux is applied to the trigeminal
The antibodies are directed towards acetylcholine
neuralgia only when the patient suffers from spasmodic
receptors (AChR) at the neuromuscular junction of
contractions of the facial muscles.
the skeletal muscles.
Q. 8. Sphenopalatine neuralgia (Horton’s syndrome) iii. It occurs chiefly in middle-aged women.
iv. Muscles of mastication and facial expression are involved.
Ans.
“Sorrowful appearance” of face.
i. It is a pain syndrome referable to the nasal ganglion. v. The drug of choice is physostigmine, an anticholines-
It may be caused either due to irritation of nasal (sphe- terase administered intramusculary for diagnosis as
nopalatine) ganglion or irritation to vidian nerve. well as to improve the strength of muscles. The remis-
sion is only temporary.
Section | III Oral Pathology 617

Topic 21

Forensic Odontology

SHORT NOTES
Q. 1. Two diagnostic uses of saliva. Q. 5. Write briefly about MacDonald’s classification of
bite marks.
Ans.
Ans.
Saliva has following diagnostic uses:
i. Saliva deposited on skin will have WBCs and sloughed MacDonald’s classification of bite marks is most cited.
epithelial cells which are a potential source of DNA thus He suggested an aetiologic classification of bite marks
saliva swab enables a direct link to the source as a foren- which is as follows:
sic evidence. i. Tooth pressure marks: Marks produced on tissue as a
ii. Saliva could profoundly influence the dental caries pro- result of “direct application of pressure by teeth”. They
cess, hence it helps in assessing the prevalence of dental are generally produced by the incisal or occlusal sur-
caries in an individual through various caries activity tests. faces of teeth.
ii. Tongue pressure marks: When sufficient amount of
Q. 2. Name the various methods of age estimation in tissue is taken into the mouth, the tongue presses
children and adults. it against rigid areas such as the lingual surfaces of
Ans. teeth and palatal rugae leaves the marks referred to as
‘suckling’.
l Dental age estimation in children and adolescents using iii. Tooth scrape marks: These are marks produced due to
the dentition are: scraping of teeth across the bitten material. They are
i. Schour and Massler’s method generally produced by anterior teeth and present as
ii. Demirjan’s method scratches or superficial abrasions.
iii. Third molars in age estimation.
l Various methods of dental age estimation in adults using Q. 6. Autoclave.
the dentition are:
i. Gustafson’s method: Based on morphological and Ans.
histological changes of the teeth. i. Autoclave is the apparatus which works on pressure
ii. Dentin translucency cooker principle for the sterilization by using moist heat,
iii. Age estimation from incremental lines of cementum. which ensures moist conditions for killing bacteria.
iv. Radiographic method of Kvaal and associates ii. Sterilisation time: Usual sterilisation time for an auto-
v. Amino acid racemisation, etc. clave is as follows:
Q. 3. Define ‘cheiloscopy’. a. 1210C at 15 lb per square inch for 15-20 minutes.
b. 1260C at 20 lb per square inch for 10 minutes.
Ans. c. 1340C at 30 lb per square inch for 3 minutes.
The imprint produced by wrinkles and grooves on the lips iii. Uses of autoclave:
l Common articles sterilised in autoclave are culture
is termed as lip print, the examination of it is referred to as
‘cheiloscopy’. media, rubber articles like tubes, gloves, etc.
l Syringes and surgical instruments, OT gowns, dress-
Q. 4. Define bite marks. ing materials, endodontic instruments and hand
Ans. instruments, etc.

Bite marks have been defined by MacDonald as “marks Q. 7. Gram staining.


caused by the teeth either alone or in combination with
Ans.
other mouth parts”.
Bite marks may be caused by humans or animals and i. The Gram stain is the most widely used differential
they may be on tissue, food items or other objects. staining procedure in bacteriology.
618 Quick Review Series for BDS 3rd Year

ii. It is called a differential stain since it differentiates Q. 10. Fixatives.


between gram-positive and gram-negative bacteria.
Ans.
iii. Bacteria that stain purple with the Gram staining proce-
dure are termed gram-positive; those that stain pink are l The fixatives preserve the tissues permanently in a lie
said to be gram-negative. like state as possible.
iv. Gram-positive and gram-negative bacteria stain differ- l Fixation should be carried out as soon as possible after
ently because of fundamental differences in the structure removal of tissues to prevent autolysis.
of their cell walls. l According to mechanism of action, there are five major
v. Procedure: The Gram staining procedure involves four groups of fixatives as follows:
basic steps. i. Aldehydes, e.g. formaldehyde, glutaraldehyde
l The primary staining with a pararosaniline dye ii. Mercurials, e.g. B-5, Zenker’s
such as crystal violet, methyl violet or gentian iii. Alcohols, e.g. methanol, ethanol, etc.
violet. iv. Oxidizing agents, e.g. potassium permanganate,
l Application of a dilute solution of iodine. potassium dichromate and osmium tetroxide
l Decolourisation with an organic solvent such as v. Pictrates, e.g. Bouin’s solution.
ethanol, acetone or aniline.
Q. 11. Exfoliative cytology.
l Counterstaining with a dye of contrasting colour

such as carbol fuschin, safranine or neutral red. Ans.


The Gram staining is an essential procedure used in the i. Exfoliative cytology has been used as a diagnostic test
identification of bacteria and is frequently the only method for precancerous and cancerous lesions present in the
required for studying their morphology. oral cavity.
ii. George N Papanicolaou introduced cytology as a tool to
Q. 8. Haematoxylin and eosin staining. detect cancer and precancer in 1928. It is now a widely
Ans. accepted method for mass screening in asymptomatic
population.
l The haematoxylin and eosin is a routine staining iii. At present, the best method for obtaining oral cytologic
method. specimens is mechanical scraping of lesions discovered
l Haematoxylin is a basic dye, and has an affinity for the through careful examination.
nucleic acids of the cell nucleus. iv. The prime application of this technique in dental practice
l Haematoxylin stains either ‘regressive’ or ‘progressive’. should be in the investigation of the innocuous-appearing
With a regressive stain, the slides are left in the solution lesions for which biopsy is not planned or indicated.
for a set period of time. This method works better for v. The major value of cytology is the non-invasive nature
large batches of slides to be stained and is more predict- of simple and pain-free procedure which can provide
able. With progressive stain, slide is dipped in haema- intact cells from different layers within the epithelium.
toxylin until desired intensity of staining is achieved,
this is simple for a single slide but not suitable for batch Q. 12. FNAC
processing. Ans.
l Eosin is an acidic dye with an affinity for cytoplasmic
components of the cell. l Fine needle aspiration cytology (FNAC), also known as
needle aspiration biopsy (NAB), fine needle aspiration
Q. 9. Culture media for Candida albicans. biopsy (FNAB) and fine needle aspiration (FNA), is a
diagnostic procedure used to investigate superficial
Ans.
lumps or masses.
Culture media for Candida albicans: l In this technique, a thin, hollow needle is inserted into
i. Specimens are inoculated on the Sabouraud’s dex- the mass to extract cells that, after being stained, will be
trose chloramphenicol agar medium at 25-37°C for examined under a microscope.
24 hours. l Fine needle aspiration biopsies are very safe, minor
ii. Candida produces creamy white, smooth colonies with surgical procedures. A needle aspiration biopsy is safer
a yeasty odour. and less traumatic than an open surgical biopsy, and
iii. The Candida albicans is identified by: significant complications are usually rare, depending on
l Germtubes: When Candida is grown in human serum the body site.
at 37°C for 3 hours. A wet KOH film shows filamen- l Common complications include bruising and soreness.
tous outgrowths. (Reynolds Braude phenomenon). l The risks involved are:
l Chlamydospores: These develop in a nutritionally False negative result.
poor medium such as cornmeal agar at 28°C. It will not enable a definitive diagnosis.
Section | III Oral Pathology 619

Key Points to Remember


ORAL PATHOLOGY
Some interesting points to be remembered about developmental disturbances of oral and paraoral
structures:

Condition Most commonly seen in


Branchial cleft cyst Lateral aspect of neck
Thyroglossal cyst Midline of neck region
Median anterior maxillary cyst or nasopalatine duct cyst or Between central incisors
incisive canal cyst
Globulomaxillary cyst Between the roots of lateral incisor and canine
Basal cell carcinoma Middle third of face
Regional odontodysplasia (ghost teeth) Permanent anteriors
Smallest tooth in human dentition Primary mandibular central incisor
Adenomatoid odontogenic tumour Maxillary or mandibular canine
Tuner hypoplasia Mandibular premolars followed by permanent maxillary
incisors
Teeth most commonly blocked out due to lack of space Permanent maxillary canine
Taurodontism Permanent or primary molars
Corps, ronds and grains Keratosis follicularis
Bilateral contagious parotitis, elevated serum amylase levels Mumps
Disease occurring due to malabsorption of zinc Acrodermatitis enteropathica
Togavirus German measles (rubella); Koplik spots are not seen
Coxsackie group A virus Herpangina
Coxsackie virus A10 Acute lymphonodular pharyngitis
Coxsackie virus A16 Hand, foot and mouth disease
PPLO (pleuropneumonia-like organisms) Behcet syndrome
Burkitt lymphoma B cell neoplasm caused by EBV; starry sky histologic
appearance
Multiple myeloma Bence Jones protein; plasma cell with cart wheel or
checkerboard pattern of chromatin; Russell bodies; mul-
tiple sharply punched-out radiolucent areas, hyperglob-
ulinaemia, anaemia
Most common intraoral nevus Blue nevus

‘Abtropfung’ or ‘dropping off’ effect Junctional nevus (premalignant condition)

Condition Most commonly seen in


Ankylosis/submerged tooth Deciduous mandibular second molar
Dens in dente (dens invaginatus) Permanent maxillary lateral incisor
Dens evaginatus Premolars
Concrescence Maxillary molars
Natal teeth Primary mandibular central incisor
Microdontia Maxillary lateral incisor followed by third molars
Continued
620 Quick Review Series for BDS 3rd Year

Condition Most commonly seen in


Supernumerary roots (bifurcated roots) Mandibular canine and premolars
Supernumerary teeth Mesiodens followed by maxillary and mandibular 4th mo-
lars
Most commonly missing permanent teeth Third molars followed by maxillary laterals and mandibu-
lar second molars
Most commonly missing deciduous teeth Deciduous maxillary and mandibular lateral incisors
Teeth that are least commonly extracted for orthodontic Maxillary anteriors
purpose
Teeth that are commonly extracted for orthodontic pur- First premolars
poses
Most commonly impacted teeth Third molars followed by maxillary cuspids
Teeth most commonly involved in nursing bottle caries Maxillary incisors

Teeth least involved in nursing bottle caries Mandibular incisors

l Epstein pearls are found along the median raphe of hard palate, while Bohn’s nodules are found along
the junction of hard and soft palate.
l Static bone cyst/Stafne’s bone cyst/developmental lingual mandibular salivary gland depression is found
below the mandibular canal.
l Traumatic/haemorrhagic/idiopathic bone cyst is found above the mandibular canal.
Important characteristic signs of various diseases:

Characteristic sign Disease


Nikolsky sign l Pemphigus
l Familial benign chronic pemphigus
l Recessive form of epidermolysis bullosa
Auspitz sign l Psoriasis
Higownenaki sign l Congenital syphilis
Crowe sign (axillary freckling) l Von Recklinghausen neurofibromatosis
Koplik spots l Measles (rubella)
Roth spots l Subacute bacterial endocarditis
Herald spot l Primary lesions of skin seen in pityriasis
rosea

Cafe-au-lait spots l Neurofibromatosis


l Fibrous dysplasia
l Peutz–Jegher syndrome

Common diagnostic tests for various diseases:

Weil–Felix test Rickettsial infection


Kahn test, VDRL, TPI, FTA-ABS Syphilis
Mantoux test Tuberculosis
Schick test, Elek’s test Diphtheria
Dick test, ScuItz–CharIton test Scarlet fever
Schirmer’s test, slit lamp test, rose Bengal test Eye tests (lacrimal gland function) for Sjogren
syndrome
Section | III Oral Pathology 621

Schilling’s test Vitamin B12


ELISA, Western blot AIDS
Rumpel–Leede phenomenon or tourniquet Test for capillary fragility
test
Rose–Waaler test Rheumatoid arthritis
Kveim–Siltzbach test Sarcoidosis
Monospot test, Paul–Bunnell test Infectious mononucleosis
Tzanck test Pemphigus, herpes simplex

Fries test Lymphogranuloma venereum

Different types of cells and the conditions in which they are found:

LE cell l Consists of a rosette of neutrophils surrounding a pale


nuclear mass apparently derived from a lymphocyte
l Systemic lupus erythematosus
Nevus cells l Pigmented mole
Glycogen-free clear cells l Acinic cell carcinoma
l Mucoepidermoid carcinoma
Glycogen-rich clear cells l Lateral periodontal cyst
l Gingival cysts of adults
Pericyte of Zimmerman l Glomus tumour
Foam cells l Lipid-laden cells seen in periapical granuloma
Acantholytic cells l Pemphigus
Safety pin cells (normoblasts), target l Thalassaemia
cells
Downey cells l Kissing disease or infectious mononucleosis
Racquet cells or Ribbon cells l Rhabdomyosarcoma
Hyaline cells l Pleomorphic adenoma
Tzanck cells l Herpes
l Pemphigus
Anitschkow cells l Aphthous ulcers
l Sickle cell disease
l Iron-deficiency anaemia
l Rheumatic heart disease
Lepra cells l Vacuolated macrophages of histiocytes seen in lepromatous
leprosy
Ghost cells l Craniopharyngioma (Rathke pouch tumour)
l Odontomas
l Ameloblastic fibrodontoma
l Calcifying odontogenic cyst (Gorlin cyst)
Hurler cells or clear cells or gargoyle l Hurler syndrome
cells l Mucopolysaccharidosis
Reed–Sternberg cell/Dorothy Reed cell l Modified B lymphocytes or macrophage—monocyte deriva-
tive seen in Hodgkin’s lymphoma

Pale cells and dark cells l Odontogenic myxoma


622 Quick Review Series for BDS 3rd Year

Different types of bodies and the conditions in which they are seen:

Verocay bodies l Neurilemmoma/Schwannoma


Civatte/hyaline/colloid/cytoid bodies l Lichen planus
Rushton bodies l Apical periodontal cyst
l Infected dentigerous cyst
l Gingival cysts of neonate
Russell bodies l Multiple myeloma
l Periapical granuloma
Dohle bodies l Chediak–Higashi syndrome
Reilly bodies or metachromatic l Hurler syndrome
granules in lymphocytes
Asteroid bodies l Sporotrichosis
Heinz bodies l In RBC of persons suffering from G6PD deficiency
Howell–Jolly bodies and Cabot’s ring l Pernicious anaemia
Pappenheimer bodies l Sideroblastic anaemia
Negri bodies l Rabies
Cowdry Type A (intranuclear) l Herpes
Cowdry Type B (intranuclear) l Poliomyelitis
Lipschutz bodies l Primary herpetic stomatitis

Guarnieri bodies (intracytoplasmic) l Vaccinia virus


l Smallpox virus

Some common infectious diseases and their incubation periods are as follows:

Disease/infection Incubation period


Diphtheria, cholera, typhoid 1–5 days
Influenza 1–3 days
Chickenpox, rubella, mumps 2–3 weeks
Measles (rubeola) 10 days
Tetanus 3–21 days
Hepatitis A 2–7 weeks

Hepatitis B 7–23 weeks

Different types of rete pegs and the conditions in which they are seen:

Test tube rete pegs Dilantin hyperplasia


Saw-tooth rete pegs Lichen planus
Absence of rete pegs Oral submucous fibrosis
Rhagades Congenital syphilis

Pseudorhagades Ectodermal dysplasia


Section | III Oral Pathology 623

Characteristic presentation of common conditions, alternate names of certain diseases and other charac-
teristic features are as follows:

Butterfly distribution of lesions Systemic lupus erythematosus


Spidery fingers Marfan syndrome
Coup-de-sabre appearance Scleroderma
Angelic look (eyes looking towards Cherubism
heaven)
Swan neck Dystrophic myotonia
Rubbery man Ehlers–Danlos syndrome
Petrified man Generalized myositis ossificans
Fish-face appearance Treacher-Collins syndrome
Bird face Bilateral TMJ ankylosis
Treacher-Collins syndrome
Pierre Robin syndrome
Mask-like face Bell’s palsy
Scleroderma
Long face or adenoid face syndrome
Hunter/Moeller tongue Pernicious anaemia
Bald tongue of Sandwith Niacin deficiency (pellagra)
Magenta or beefy-red tongue Riboflavin deficiency
Strawberry and raspberry tongue Scarlet fever
Pink disease Mercury poisoning (acrodynia or Swift
disease)
Lockjaw Tetanus
Lumpy jaw Actinomycosis
African jaw lymphoma Burkitt’s lymphoma
Migratory glossitis Geographic tongue
Atrophic glossitis Iron-deficiency anaemia and pernicious
anaemia
Luetic glossitis Syphilis
Von Recklinghausen disease of skin Neurofibromatosis
Von Recklinghausen disease of bone Hyperparathyroidism
Bull’s eye lesion Erythema multiforme
Bull teeth Taurodontism
Bence Jones proteins Multiple myeloma
Polycythemia vera
Leukaemia
Solitary myeloma
Abnormal DEJ Dentinogenesis imperfecta
Ehlers–Danlos syndrome
Onion-peel appearance seen in radiograph Ewing sarcoma
Garre osteomyelitis
Caffey disease
Multinucleated giant cells Cherubism
Hyperparathyroidism
Continued
624 Quick Review Series for BDS 3rd Year

Hypercementosis Aneurysmal bone cyst


Giant cell granulomas
Osteoclastoma
Osteosarcoma
Herpes
Leprosy
Eosinophilic granuloma
Paget’s disease
Hyperpituitarism
Blue sclera Osteogenesis imperfecta
Marfan syndrome
Cherubism
Ehlers–Danlos syndrome
Osteopetrosis
Fetal rickets
Pseudoepitheliomatous hyperplasia Granular cell myoblastoma
Blastomycosis
Papillary hyperplasia
DLE
Squamous cell carcinoma
Ground glass appearance Primary hyperparathyroidism
Monostotic fibrous dysplasia
Widened periodontal ligament Osteosarcoma
Scleroderma
Trauma from occlusion
Cotton wool appearance seen in Paget’s disease
radiograph Chronic sclerosing diffuse osteomyelitis
Hypermobility of joints Ehlers–Danlos syndrome
Marfan syndrome
Osteogenesis imperfecta
Down syndrome
Hypomobility of joints Achondroplasia
Palmar plantar keratosis Godin–Goltz syndrome
Papillon–Lefevre syndrome
Follicularis keratosis
Tumours common in anterior maxillary Compound odontoma
region Adenomatoid odontogenic tumour
Squamous odontogenic tumour

Absence of lamina dura Hyperparathyroidism


Paget’s disease

Various theories of dental caries are as follows:

Acidogenic/chemicoparasitic Miller
theory Robertson
Chemical/acidic theory
Parasitic theory Dubos
Proteolytic theory Gottlieb
Proteolysis chelation theory Schwartz and Martin
Sulphatase theory Pincus

Sucrose chelation theory Burch


Section | III Oral Pathology 625

Some interesting facts about dental caries:

Initiation of dental caries Streptococcus mutans


Propagation of dental caries Lactobacilli
Root surface caries Actinomyces species
Critical pH of dental caries 5.2
Highest incidence of dental caries in permanent Mandibular molars
teeth
High-ammonia concentration Retards formation of plaque
Element causing increase in dental caries Selenium
Element causing decrease in dental caries Vanadium
Pit and fissure caries Apex at periphery and base towards DEJ
Smooth surface caries Apex towards DEJ and base towards the outer
surface
Arch more frequently involved by caries Maxillary arch

Lowest incidence of dental caries Mandibular incisors

Zones of dental caries in the enamel (enamel caries) are as follows:

Zone I l Translucent zone; not always present


l Lies at the advancing front of the lesion
Zone II l Dark zone; positive zone
l Lies adjacent and superficial to zone I
Zone III l Body of the lesion
l Area of greatest demineralization
l Lies between the unaffected surface layer and the dark zone

Zone IV l Surface zone; relatively unaffected

Zones of dental caries in the dentin (dentinal caries) are as follows:

Zone I Zone of fatty degeneration of Tomes’ fibres


Zone II Zone of dental sclerosis (characterized by deposition of calcium
salts in dentinal tubules)
Zone III Zone of decalcification of dentin
Zone IV Zone of bacterial invasion of decalcified but intact dentin

Zone V Zone of decomposed dentin


626 Quick Review Series for BDS 3rd Year

Pioneer bacteria—these are the microorganisms that are found penetrating the dentinal tubules before there
is any clinical evidence of the carious process.

Diphtheria Grey-white pseudomembrane of oropharynx


Lichen planus Wickham’s striae
CREST syndrome Calcinosis cutis, Raynaud’s phenomenon, oesophageal motility defect,
sclerodactyly, telangiectasia
Blue nevus Most common on hard palate
Black or brown hairy tongue Discolouration and elongation of filiform papillae
Fordyce’s granules Sebaceous glands; usually multiple submucosal papules on buccal mucosa
or upper lip vermilion
Multiple myeloma Radiographically punched-out lesions
Hyperparathyroidism Multiple brown tumours
Enamel pearls Seen commonly in furcation areas of molar teeth
CEOT/Pindborg tumour Driven snow opacities

Phlebolith May occur in varicosities or haemangiomas


Section IV

Multiple Choice Questions

Section I General Medicine 629


Section II General Surgery 644
Section III Oral Pathology 659
This page intentionally left blank
Section IV

Multiple Choice Questions

Section I

General Medicine
1. The process underlying differences in expression of a 5. Which one of the following is an emerging viral disease?
gene, according to which parent has transmitted, is called a. SARS
a. Anticipation b. Chickenpox
b. Mosaicism c. Measles
c. Nonpenetrance d. Rabies
d. Genomic imprinting
6. Common threat for aspiration pneumonia is
2. Megaloblastic anaemia due to folic acid deficiency is a. Blood
commonly due to b. Pus
a. Inadequate dietary intake c. Oral flora
b. Defective intestinal absorption d. Foreign Body
c. Absence of folic acid binding protein in serum
d. Absence of glutamic acid in the intestine 7. Long-term assessment of glucose control can be
made by
3. Klinefelter’s syndrome is characterized by the karyotype a. Estimation of postprandial blood sugar
a. 47, XYY b. Estimation of fasting blood sugar
b. 47, XXY c. Estimation of urine sugar level
c. 45, YO d. Estimation of blood level of glycosylated haemoglobin
d. 45, XO
8. Hungry diarrhoea is seen in
4. Glycated haemoglobin level of 7–8% suggests a. Marasmus
a. Good diabetes control b. Kwashiorkor
b. Poor diabetes control c. Obesity
c. Moderate diabetes control d. Dehydration
d. Normal blood glucose levels

1. d 2. a 3. b 4. c 5. a 6. c 7. d 8. a

629
630 Quick Review Series for BDS 3rd Year

9. Pulsus paradoxus is seen in 16. Pastia’s lines are


a. Shock a. Seen in staphylococcal infection
b. Elderly b. Transverse red streaks in skin folds seen in scarlet
c. Emphysema asthma fever
d. High output state c. White striae seen in lichen planus
d. Erythematous rash in syphilis
10. Acromegaly is due to excess secretion of
a. Growth hormone 17. Which one of the following clinical findings excludes
b. Thyroxine the diagnosis of polymyositis?
c. ACTH a. Neck muscle involvement
d. FSH b. Extraocular muscle involvement
c. Dysphagia
11. If a normal individual receives an insulin injection that d. Abdominal muscle involvement
lowers plasma glucose to 35 mg/dL, which of the
following hormones will not be released? 18. Unilateral supranuclear lesion of facial nerve involves
a. ACTH a. Whole of upper face
b. Epinephrine b. Only ipsilateral upper part of the face
c. Growth hormone c. Only contralateral lower part of the face
d. Aldosterone d. Whole of lower face

12. The Diabetes Control and Complication Trial (DCCT) 19. Conjugated and unconjugated bilirubin levels are
provided definitive proof that reduction in chronic increased in
hyperglycaemia helps to improve a. Prehepatic jaundice
a. Microvascular complications of type-I diabetes mellitus b. Hepatocellular jaundice
b. Macrovascular complications of type-I diabetes c. Posthepatic jaundice
mellitus d. Haemolytic jaundice
c. Microvascular complications of type-II diabetes
mellitus 20. Berylliosis is caused by
d. Macrovascular complications of type-II diabetes a. Beryllium
mellitus b. Boric acid
c. Niobium
13. The commonest cause of hypercalcaemia in a patient d. BIS-GMA resin
with known cancer is
a. Ectopic parathormone production 21. All of the following are predisposing conditions for
b. Direct destruction of the bone by tumour cells thrombosis and embolism, except
c. High levels of 1,25-dihydroxyvitamin D a. Atrial fibrillation
d. Production of parathormone-like substance b. Prolonged immobilization
c. Huntington’s disease
14. Gestational diabetes develops only during d. Prosthetic cardiac valve
a. Old age
b. Young age 22. Acquired symmetric hyperpigmentation of the sun-
c. Pregnancy exposed skin of the face and neck, which is associated
d. Infancy with pregnancy and use of oral contraceptives is
known as
15. Osmotic fragility is increased in a. Melanoma
a. Sickle cell anaemia b. Café au lait spots
b. Hereditary spherocytosis c. Freckle
c. Thalassaemia d. Melasma
d. Chronic lead poisoning

9. d 10. a 11. d 12. a 13. d 14. c 15. b 16. b 17. b 18. c 19. b 20. a 21. c 22. d
Section | IV Multiple Choice Questions 631

23. One of the causes of finger clubbing is 30. The drug of choice for prompt relief of an ongoing attack
a. Carcinoma of lung of angina precipitated by exercise or emotional stress is
b. Cardiac failure a. Propranolol
c. Bronchial asthma b. Atenolol
d. Ischaemic heart disease c. Sublingual nitroglycerin
d. Sublingual nifedipine
24. Patients with atrial fibrillation have
a. Regularly irregular pulse 31. Which of the following is associated with cyanotic
b. Irregularly irregular pulse congenital heart disease?
c. Anacrotic pulse a. Cor triatriatum
d. Pulsus alternans b. Tetralogy of Fallot
c. Mitral stenosis
25. The most common cause of chronic bronchitis is d. Primary pulmonary hypertension
a. Air pollution
b. Septic fibrosis 32. Clubbing of the fingers
c. Smoking a. Always indicates heart disease
d. Recurrent aspiration pneumonia b. Is common in cirrhosis of the liver
c. Warrants a search for sickle cell disease
26. Gallbladder enlargement is seen in d. Is seen in pulmonary AV fistula
a. Carcinoma of pancreas
b. Hepatic cirrhosis 33. Platelet count is raised in
c. Chronic hepatitis a. Osteopetrosis
d. Chronic cholelithiasis b. Disseminated intravascular coagulation
c. Haemolytic anaemia
27. Black water fever is caused by d. Thiazide therapy
a. Plasmodium vivax
b. Plasmodium falciparum 34. A patient with a history of ankle oedema, shortness of
c. Plasmodium malariae breath and orthopnoea is probably suffering from
d. Plasmodium ovale a. Asthma
b. Emphysema
28. Dysphagia due to neurogenic cause includes all of the c. Cardiac decompensation
following, except d. Constrictive pericarditis
a. Multiple sclerosis
b. Tetanus 35. Features of hypothyroidism do not include
c. Parkinson’s disease a. Obesity
d. Tetany b. Hypertension
c. High TSH levels
29. Occupational disease of lungs caused by inhalation of d. Increased risk of infections
sugar cane dust is called as
a. Byssinosis 36. Nephrotic syndrome is characterized by the following
b. Bagassosis manifestations, except
c. Anthracosis a. Proteinuria
d. Silicosis b. Hyperlipidaemia
c. Oedema
d. Hypertension

23. d 24. b 25. c 26. a 27. b 28. d 29. b 30. c 31. b 32. b 33. c 34. c 35. d 36. d
632 Quick Review Series for BDS 3rd Year

37. The most common cause of massive upper GIT 44. All of the following are risk factors for atherosclerosis,
bleeding is except
a. Gastric ulcer a. Increased waist–hip ratio
b. Erosive gastritis b. Hyperhomocysteinaemia
c. Gastric carcinoma c. Decreased fibrinogen levels
d. Duodenal ulcer d. Decreased HDL levels

38. Which of the following results in a prolonged bleeding 45. A young girl has consumed barium carbonate with
time? suicidal intent. She complains of generalized muscle
a. Haemophilia C weakness. The most likely electrolyte abnormality is
b. Haemophilia B a. Hyponatraemia
c. Thrombocytopenia b. Hypocalcaemia
d. Haemophilia A c. Hypokalaemia
d. Hypomagnesaemia
39. Calcitonin is secreted by
a. Thyroid glands 46. Joint erosions are not a feature of
b. Adrenal glands a. Rheumatoid arthritis
c. Parathyroid glands b. Psoriasis
d. Ovaries c. Multicentric reticulohistiocytosis
d. Systemic lupus erythematosus
40. A child is below the third percentile for height. Growth
velocity is normal, but chronological age is more than 47. A child with a small head minor anomalies of the face
skeletal age. The most likely diagnosis is including a thin upper lip, growth delay and develop-
a. Constitutional delay in growth mental disability can have all of the following, except
b. Hypopituitarism a. A chromosomal syndrome
c. Primordial dwarfism b. A teratogenic syndrome
d. Genetic short stature c. A mendelian syndrome
d. A polygenic syndrome
41. Which one of the following is not a major criterion for
diagnosis of multiple myeloma? 48. Which of the following is not a sign of stellate ganglion
a. Lytic bone lesions block?
b. Plasmacytoma on tissue biopsy a. Meiosis
c. Bone marrow plasmacytosis .30% b. Exophthalmus
d. M-spike .3 g% for IgG, .2 g% for IgA c. Nasal congestion
d. Conjunctival redness
42. Bisphosphonates act by
a. Increasing the osteoid formation 49. Which of the following elements is known to influence
b. Increasing the mineralization of osteoid the body’s ability to handle oxidative stress?
c. Decreasing the osteoclast-mediated resorption of a. Calcium
bone b. Iron
d. Decreasing the parathyroid hormone secretion c. Potassium
d. Selenium
43. All of the following are the known causes of osteopo-
rosis, except 50. Which of the following is the earliest manifestation of
a. Fluorosis Cushing’s syndrome?
b. Hypogonadism a. Loss of diurnal variation
c. Hyperthyroidism b. Increased ACTH
d. Hyperparathyroidism c. Increased plasma cortisol
d. Increased urinary metabolites of cortisol

37. d 38. c 39. a 40. a 41. a 42. c 43. a 44. c 45. c 46. d 47. b 48. b 49. d 50. a
Section | IV Multiple Choice Questions 633

51. Persistent vomiting most likely causes 58. A patient with increased metabolic rate, hyperthyroid-
a. Hyperkalaemia ism, goitre and have deposits of calcium in capsule of
b. Hypochloraemia thyroid, it can be due to
c. Acidic urine excretion a. Follicular carcinoma of thyroid
d. Hyperventilation b. Medullary carcinoma of thyroid
c. De Quervain’s thyroiditis
52. Which of the following brain tumours does not spread d. Riedel’s thyroiditis
via CSF?
a. Germ cell tumour 59. Which of the following is seen in osteoporosis?
b. Medulloblastoma a. Increased serum calcium
c. CNS lymphoma b. Decreased serum phosphorus
d. Craniopharyngioma c. Normal serum calcium and phosphorus
d. Increased serum alkaline phosphatase
53. Which of the following is not an important cause of
hyponatraemia? 60. Delusions are seen in all of the following, except
a. Gastric fistula a. Obsessive-compulsive disorder
b. Excessive vomiting b. Depression
c. Prolonged Ryle’s tube intubations c. Mania
d. Excessive sweating d. Delirium

54. Which of the following best represents Bell’s palsy? 61. Investigation of choice for invasive amoebiasis is
a. Hemiparesis and contralateral facial nerve paralysis a. Indirect haemagglutination
b. Combined paralysis of facial, trigeminal and abdu- b. ELISA
cens nerves c. Counter-immune electrophoresis
c. Idiopathic ipsilateral paralysis of facial nerve d. Microscopy
d. Facial nerve paralysis with dry eye
62. Stimulation with TSH is useful for the diagnosis of
55. Which of the following is the most penetrative beam? a. Prolactin
a. Electron beam b. ACTH
b. 8 MY photons c. Growth hormone
c. 18 MY photons d. PTH
d. Proton beam
63. Renal osteodystrophy differs from nutritional and
56. The occurrence of hyperthyroidism following adminis- metabolic rickets due to the presence of
tration of supplemental iodine to subjects with endemic a. Hyperphosphataemia
iodine deficiency goitre is known as b. Hypercalcaemia
a. Jod–Basedow effect c. Hypophosphataemia
b. Wolff–Chaikoff effect d. Hypocalcaemia
c. Thyrotoxicosis factitia
d. De Quervain’s thyroiditis 64. Which of the following cardiovascular conditions can
result in a medical emergency in the dental orifice caus-
57. A young boy developed respiratory distress. On exami- ing substernal pain, facial pallor and cold perspiration?
nation, the chest radiograph showed hyperinflation of a. Angina pectoris
one lung. Which of the following is the most possible b. Congestive heart failure
diagnosis? c. Hypertension
a. Congenital lobar emphysema d. Patent ductus arteriosus
b. Foreign body aspiration
c. Bronchiectasis
d. Atelectasis

51. b 52. d 53. c 54. c 55. c 56. a 57. b 58. b 59. c 60. a 61. b 62. c 63. a 64. b
634 Quick Review Series for BDS 3rd Year

65. All of the following are features of scurvy, except 71. What would be the first line of treatment, if patient
a. Perifollicular hyperkeratotic papules develops ventricular fibrillation after intravenous
b. Purpura injection of potassium chloride?
c. Haemarthrosis a. Cardiac massage
d. Scorbutic rosary b. IV adrenaline
c. Defibrillation
66. All of the following are features of tetanus, except d. IPPV
a. Vertebral fractures
b. Survivors recover completely in four weeks 72. A patient with acute inferior wall myocardial infarction
c. Diagnosis is clinical and no microbiological tests has developed shock. Which of the following is the
are required most likely cause of shock?
d. Loss of consciousness occurs in moderately a. Cardiac rupture
advanced cases b. Interventricular septal perforation
c. Papillary muscle rupture
67. A 45-year-old patient presents with odontogenic infec- d. Right ventricular infarction
tion. He had recurrent infections and weight loss in the
recent past, has a good appetite with polydipsia. He 73. Hypocalcaemia is characterized by all of the following
should be investigated for features, except
a. Diabetes mellitus a. Numbness and tingling of circumoral region
b. Chronic glomerulonephritis b. Hyperactivity tendon reflexes
c. Malignancy c. Shortening of QT interval in ECG
d. Infectious mononucleosis d. Carpopedal spasm

68. Hypercalcaemia is seen in all, except 74. In a patient with wide complex tachycardia, the pres-
a. Chronic renal failure ence of all of the following in the ECG indicates ven-
b. Milk alkali syndrome tricular tachycardia, except
c. Lithium therapy a. Atrioventricular dissociation
d. Primary hyperparathyroidism b. Fusion beats
c. Typical right bundle branch block
69. Impaired glucose tolerance can result in d. Capture beats
a. Nephrotoxicity
b. Retinopathy 75. Crepitus will be seen characteristically in the tissues
c. Neurotoxicity surrounding an infection caused by
d. Ischaemic heart disease a. Staphylococcus
b. Clostridium tetani
70. A middle-aged female patient experiences severe c. Streptococcus
throbbing headache of left craniofacial region. She also d. Clostridium perfringens
has nausea with paraesthesia of right upper and lower
lip. She may be suffering from 76. Mode of spread of hepatitis E infection is
a. Cluster headache a. Feco-oral route
b. Glossopharyngeal nerve neuralgia b. Parenteral
c. Trigeminal neuralgia c. Blood transfusion
d. Migraine d. Rectal

65. b 66. b 67. a 68. c 69. d 70. d 71. c 72. d 73. c 74. c 75. d 76. a
Section | IV Multiple Choice Questions 635

77. Epstein–Barr virus (EBV) is associated with 84. After a minor head injury, a young patient was unable
a. Carcinoma of larynx to close his left eye and had drooling of saliva from left
b. Carcinoma of bladder angle of mouth. He is suffering from
c. Nasopharyngeal carcinoma a. Seventh nerve injury
d. Chronic lymphocytic leukaemia b. Fifth nerve injury
c. Third nerve injury
78. Transaminase is increased in d. Ninth nerve injury
a. Hepatitis A
b. Subacute bacterial endocarditis 85. Tic douloureux is synonym of
c. Myocardial infarction a. Trigeminal neuralgia
d. Jaundice b. Bell’s palsy
c. Glossopharyngeal neuralgia
79. In AIDS, there is a deficiency of which cell? d. Todd’s paralysis
a. CD2
b. CD3 86. Glasgow Coma Scale is based on all, except
c. CD4 a. Sensory loss
d. CD8 b. Motor response
c. Vocal response
80. Koplik’s spots are seen in d. Eye opening
a. Rubella
b. Rubeola 87. Hutchinson’s pupil is
c. Smallpox a. Seen in syphilis
d. Chickenpox b. Unilateral constricted pupil
c. Irregular pupil
81. Delusions are seen in all of the following, except d. Argyll Robertson pupil
a. Obsessive-compulsive disorder
b. Depression 88. ‘Brain death’ means loss of
c. Mania a. Cortical function
d. Schizophrenia b. Brainstem function
c. Spinal reflex
82. Most important diagnostic sign of upper motor neuron d. Corneal reflex
syndrome:
a. Exaggerated tendon reflexes 89. Herpes zoster involves
b. Babinski’s sign 2ve a. Otic ganglion
c. Absent superficial reflexes b. Geniculate ganglion
d. Lead pipe rigidity c. Gasserian ganglion
d. Celiac ganglion
83. Tongue deviates to the right side on protrusion. This is
because of paralysis of 90. All of the following conditions are likely to be associ-
a. Right hypoglossal ated with melena, except
b. Left hypoglossal a. Carcinoma of the stomach
c. Lingual nerve b. Portal hypertension
d. Right facial c. Chronic duodenal ulcer
d. Carcinoma of the rectum

77. c 78. c 79. c 80. b 81. a 82. a 83. a 84. a 85. a 86. a 87. b 88. b 89. b 90. d
636 Quick Review Series for BDS 3rd Year

91. Nephrotic syndrome is characterized by following 98. The deficiency of coagulation factor IX is associated with
manifestations, except a. Haemophilia A
a. Proteinuria b. Haemophilia B
b. Hyperlipidaemia c. Henoch–Schonlein purpura
c. Oedema d. Sickle cell anaemia
d. Hypertension
99. Presence of glucose in urine indicates
92. Icterus is most marked in sclera because of a. Patient suffers from diabetes insipidus
a. High content of collagen b. Patient has consumed excess sugar over long years
b. High content of elastin c. Renal threshold for glucose is exceeded
c. It is whine d. None of the above
d. It is exposed to sunlight
100. Addison’s disease is due to
93. Levels of uric acid are taken as diagnostic value in a. Haemorrhage in adrenal cortex
following diseases b. Basophilic adenoma of pituitary gland
a. Liver diseases c. Secondary syphilis
b. TMJ diseases d. Haemorrhage in adrenal medulla
c. Heart diseases
d. Renal diseases 101. Acromegaly occurs
a. Before closure of epiphyses of long bones
94. The most common type of hepatitis associated with b. After closure of epiphyses of long bones
transfusion is c. Not related to closure of epiphyses of long bones
a. Hepatitis C d. Depending on functioning on posterior pituitary gland
b. Hepatitis B
c. Hepatitis A 102. Barrel chest is usually associated with
d. Hepatitis D a. Chronic bronchitis
b. Left heart failure
95. A patient who has spider telangiectatic spots on the c. Anasarca
skin should have an examination to determine the d. Perforated lung
condition of his/her
a. Lungs 103. Cardiopulmonary resuscitation (CPR) should have a
b. Liver ratio of chest compression to mouth breathing of
c. Kidneys a. 1:4
d. Pancreas b. 2:3
c. 4:1
96. C-reactive protein is synthesized exclusively in d. 3:2
a. Kidney
b. Pancreas 104. Haemoptysis may occur in the following situations, except
c. Liver a. Bronchiectasis
d. Thymus b. Pulmonary tuberculosis
c. Lung abscess
97. Most important triad of symptoms in portal hyperten- d. Bronchial asthma
sion is
a. Splenomegaly, hepatomegaly and ascites 105. In which condition, breath sounds are absent?
b. Splenomegaly, oesophageal varices and ascites a. Bronchial asthma
c. Piles, splenomegaly and general anasarca b. Pneumothorax
d. Oesophageal varices, piles and general anasarca c. Emphysema
d. Pneumonia

91. d 92. b 93. d 94. a 95. b 96. c 97. b 98. b 99. c 100. a 101. b 102. a 103. c 104. d 105. b
Section | IV Multiple Choice Questions 637

106. Most common type of lung carcinoma in non-smokers 113. Tracheal shift to opposite side is seen in
a. Squamous cell carcinoma a. Pleural effusion
b. Adenocarcinoma b. Consolidation
c. Alveolar cell carcinoma c. Carcinoma of lung
d. Small cell carcinoma d. Cavity of lung

107. Which of the following auscultatory findings does the 114. Stony dull note on percussion is characteristic of
term ‘rhonchi’ refer to? a. Pleural effusion
a. Coarse, ‘bubbling’ sounds b. Consolidation
b. Fine, crackling sounds heard in late inspiration c. Pleurisy
c. Wheezing sounds during both inspiration and d. Tuberculous cavity
expiration
d. All of the above 115. The recent terminology for ‘status asthmaticus’ is
a. Episodic asthma
108. Which of the following may result from chronic b. Chronic asthma
bronchitis? c. Acute asthma
a. Cor pulmonale d. Severe acute asthma
b. Increased airway resistance
c. Metaplasia and dysplasia of respiratory epithelium 116. Steroid preparation, which can be given by inhalation is
d. All of the above a. Dexamethasone
b. Triamcinolone
109. Cotton dust is associated with c. Beclomethasone
a. Byssinosis d. Hydrocortisone
b. Bagassosis
c. Asbestosis 117. An increased level of urinary (vanillyl mandelic acid)
d. Silicosis is seen in
a. Pheochromocytoma
110. Farmer’s lung is caused by b. Retinoblastoma
a. Candida c. Neuroblastoma
b. Actinomycetes d. All of the above
c. Grey hay
d. Mouldy hay 118. Bronze discolouration of oral mucosa is seen in
a. Addison’s disease
111. Kussmaul’s breathing is due to presence of b. Amalgam tattoo
a. Bicarbonate c. Graphite
b. H ions d. Cushing’s disease
c. Cl ions
d. K1 ions 119. Which of the following tissues does not contain
creatinine phosphokinase (CPK)?
112. Pulmonary surfactant is a. Brain
a. Lecithin b. Liver
b. Dipalmitoyl phosphatidyl choline c. Myocardium
c. Dipalmitoyl phosphatidyl serine d. Skeletal muscle
d. Sphingosine

106. b 107. c 108. d 109. a 110. d 111. b 112. b 113. a 114. a 115. d 116. c 117. d 118. a 119. b
638 Quick Review Series for BDS 3rd Year

120. A patient having acute viral hepatitis shows increased 127. Trigeminal neuralgia after ear infection is called as
levels of a. Ramsay-Hunt syndrome
a. HBsAg b. Geredo-Nego syndrome
b. Bilirubin c. Auriculotemporal syndrome
c. Transaminase d. Sjögren’s syndrome
d. All of the above
128. Measles is transmitted from one person to another via
121. Clotting time is increased in a. Respiratory droplets
a. Idiopathic thrombocytopenic purpura b. Insect bites
b. Scurvy c. Fomites
c. Pernicious anaemia d. Faeces
d. Haemophilia B
129. The first heart sound begins during the
122. Which of the following is not included in complete a. QRS complex
blood examination? b. PR segment
a. Haematocrit c. PR interval
b. WBC count d. ST segment
c. Hb estimation
d. ESR 130. Hypovolaemic shock develops after loss of
a. 10% blood
123. The antidote for heparin toxicity is b. 20% blood
a. Protamine sulphate c. 30% blood
b. Pyrosulphate d. 40% blood
c. Vitamin K
d. Warfarin 131. Infectious diseases causing cirrhosis of the liver are
a. Viral hepatitis
124. Which of the following diseases is characterized by b. Toxoplasmosis
normal platelet count but increased bleeding time? c. Yellow fever
a. Haemophilia A d. All of the above
b. Haemophilia B
c. Aplastic anaemia 132. WBC count of more than 1,00,000 cells/cc of blood
d. Glanzmann disease indicates
a. Leukopenia
125. Which of the following cardiac conditions is associ- b. Leukaemia
ated with high risk for infective endocarditis? c. Leukoplakia
a. Mitral valve prolapsed without regurgitation d. Leucocytosis
b. Ventricular septal defect
c. Previous history of endocarditis 133. Diabetes mellitus is characterized by
d. Atrial septal defect a. Polyuria
b. Polydipsia
126. Which of the following is the feature of iron c. Hyperglycaemia
deficiency anaemia? d. All of the above
a. Increased MCV
b. Decreased MCV 134. Addison’s disease is associated with
c. Increased MCH a. Low blood pressure and oral pigmentation
d. Increased MCHC b. Oral ulcers
c. Thyrotoxicosis
d. Keratosis

120. d 121. d 122. d 123. a 124. d 125. c 126. b 127. b 128. a 129. a 130. b 131. d 132. b 133. d 134. a
Section | IV Multiple Choice Questions 639

135. The gold standard for the diagnosis of osteoporosis is 143. Serum glutamic oxaloacetic transaminase is increased in
a. Dual energy radiograph absorptiometry a. Renal failure
b. Single energy radiograph absorptiometry b. Prostatic carcinoma
c. Ultrasound c. Myocardial infarction
d. Computed tomography d. Pulmonary oedema

136. Rotten apple smell of breath is associated with 144. Osler’s nodes are seen at
a. Liver insufficiency a. Heart
b. Kidney insufficiency b. Knee joint
c. Unbalanced insulin-dependent diabetes c. Tip of palm and sole
d. Respiratory problems d. Anterior abdominal wall

137. Broca’s area is located in 145. The most important investigation done in infective
a. Superior frontal gyrus endocarditis is
b. Inferior frontal gyrus a. ECG
c. Superior temporal gyrus b. 2-D echo
d. Inferior temporal gyrus c. ASO titres
d. Serial blood cultures
138. Which of the following infections causes neuropathy?
a. AIDS 146. All the following are true of duodenal ulcer pain,
b. Lyme disease except
c. Leprosy a. Relieved by food
d. All of the above b. Occurs 2–3 hours after going to bed
c. Periodic pain
139. In syncope, which of the following is not seen? d. Relieved by alcohol
a. Cold extremities
b. Vomiting 147. Increased level of SGOT is present primarily in
c. Pupil constriction a. Liver disease
d. Faint b. Epilepsy
c. Addison’s disease
140. Carbamazepine is the drug of choice in d. Uncontrolled diabetes
a. Absence attacks
b. Partial complex seizures 148. Dark colour urine and clay stool is seen in
c. Myoclonus a. Obstructive jaundice
d. Infantile spasm b. Haemolytic jaundice
c. Hepatocellular jaundice
141. Cyanosis of lip is present in d. Severe dehydration
a. Polycythaemia
b. Myocardial infarction (MI) 149. Dane particle pertains to which of the following
c. Angina viruses?
d. All of the above a. Hepatitis A
b. Hepatitis B
142. Cor pulmonale is heart disease due to c. Hepatitis C
a. COPD affecting right ventricle d. Hepatitis D
b. Left ventricular failure
c. Pericardial effusion
d. None of the above

135. a 136. b 137. b 138. d 139. c 140. b 141. a 142. a 143. c 144. c 145. d 146. d 147. a 148. a 149. b
640 Quick Review Series for BDS 3rd Year

150. A diagnosis of chronic hepatitis is made, if liver 157. Best test to evaluate syphilis after treatment is
disease is present for a minimum of a. TPA
a. 3 weeks b. FTA-ABS
b. 6 weeks c. TPHA
c. 6 months d. VDRL
d. 3 months
158. Haematuria in SABE is which type of hypersensitivity
151. The insulin indicated in diabetic ketoacidosis is reaction?
a. Lente insulin SC a. Type I
b. Soluble insulin SC b. Type II
c. Protamine zinc insulin IM c. Type III
d. Soluble insulin IV infusion d. Type IV

152. In rheumatoid arthritis, there is inflammation of 159. Schwannomas are the most common tumours in
a. Cartilage a. Eighth cranial nerve
b. Synovial membrane b. Fifth cranial nerve
c. Sclerosis of joints c. Peripheral nerve
d. Articular bone d. Sympathetic nerve

153. Hyperparathyroidism causes 160. Osmotic fragility is increased in


a. Osteitis fibrosa cystica a. Sickle cell anaemia
b. Osteitis deformans b. Hereditary spherocytosis
c. Osteogenesis imperfecta c. Thalassaemia
d. Osteoradionecrosis d. Chronic lead poisoning

154. Phaeochromocytoma is a disorder primarily occurs 161. Leucocyte alkaline phosphates is elevated in
due to a defect in a. Pernicious anaemia
a. Adrenal medulla b. Sickle cell anaemia
b. Kidney c. Polycythaemia vera
c. Thyroid d. Plummer–Vinson syndrome
d. Parathyroid
162. Raised serum alkaline phosphatase is seen in
155. Extrahepatic cholestasis is seen in a. Hypothyroidism
a. Haemolytic anaemia b. Hyperthyroidism
b. Obstructive jaundice c. Hyperparathyroidism
c. Pancreatic carcinoma d. Osteoporosis
d. Viral hepatitis
163. A patient is suffering from dehydration secondary to
156. Condylomata lata is seen in diarrhoea. Which of the following intravenous agents
a. Lupus vulgaris is used?
b. Wegener’s granulomatosis a. 10% glucose
c. Syphilis b. Distilled water
d. All c. 0.9% saline
d. 5% glucose solution

150. c 151. d 152. b 153. a 154. a 155. c 156. c 157. d 158. c 159. a 160. b 161. c 162. c 163. c
Section | IV Multiple Choice Questions 641

164. Pernicious anaemia is due to deficiency of 171. Kaposi’s sarcoma is seen characteristically in
a. Iron a. Non-Hodgkin’s lymphoma
b. Cobalamin b. AIDS
c. Folic acid c. Lichen planus
d. Niacin d. Hodgkin’s lymphoma

165. Which of the following is the hallmark of chronic 172. All the following are characteristics, if CD4 cell count
renal failure? drops below 100 cells in AIDS, except
a. Metabolic acidosis a. NHL
b. Hypercalcaemia b. PGL
c. Hypokalaemia c. Cryptococcal meningitis
d. Increased proneness to hypoxia d. CNS toxoplasmosis

166. Reticulocytosis is seen in all, except 173. Iron deficiency anaemia is characterized by
a. Blood transfusion a. Increase in serum ferritin
b. Acute blood loss b. Increased percent saturation of transferrin
c. Polycythaemia vera c. Increased total iron binding capacity
d. Thrombocytopenic purpura d. All of the above

167. Antibodies are produced by 174. The biochemical marker for primary hyperparathy-
a. Plasma cells roidism is
b. T cells a. Hypercalcaemia
c. Mast cells b. Hypophosphataemia
d. Langerhans cells c. Increased alkaline phosphatase levels
d. Increased ACTH
168. All of the following are known causes of osteoporosis,
except 175. Dullness on percussion in the Traube’s semilunar area
a. Fluorosis denotes
b. Hypogonadism a. Cardiomegaly
c. Hyperthyroidism b. Splenomegaly
d. Hyperparathyroidism c. Hepatomegaly
d. All of the above
169. A young girl has consumed barium carbonate with sui-
cidal intention. She complains of generalized muscle 176. Pulses paradoxus is seen in
weakness. The most likely electrolyte abnormality is a. Shock
a. Hyponatraemia b. Elderly
b. Hypocalcaemia c. Emphysema
c. Hypokalaemia d. High output states
d. Hypomagnesaemia
177. Serum creatinine kinase-3 (CK-3) is elevated in
170. Wegener’s granulomatosis has antibodies against a. Muscular dystrophy
a. p-ANCA b. Myocardial infarction
b. c-ANCA c. Alcoholic cirrhosis
c. Antifibrillary antigens d. Brain tumours
d. Antinuclear antibodies

164. b 165. a 166. b 167. a 168. a 169. c 170. b 171. b 172. b 173. c 174. a 175. b 176. c 177. a
642 Quick Review Series for BDS 3rd Year

178. During routine check-up, a 70-year-old male is found 184. Each of the following can cause hyperirritability of
to have serum alkaline phosphatase three times the muscles, except
upper limit of the normal. Serum calcium, serum a. Hypocalcaemia
phosphorus and liver function tests are normal. The b. Hypomagnesaemia
most likely diagnosis is c. Hypokalaemia
a. Primary hyperparathyroidism d. None of the above
b. Paget’s disease of bone
c. Osteomalacia 185. Which of the following vitamins is essential for normal
d. Metastatic bone disease differentiation of epithelial cells?
a. Vitamin A
179. Glucose tolerance test is usually done to assess b. Vitamin B complex
a. Acute pancreatitis c. Vitamin C
b. Carcinoma of head of pancreas d. Vitamin E
c. Acinar function of the pancreas
d. Endocrine dysfunction of pancreas 186. Antirachitic vitamin is
a. Vitamin A
180. Region of myocardium most prone for infarction: b. Vitamin B6
a. Right ventricular epicardial c. Vitamin C
b. Left ventricular epicardial d. Vitamin D
c. Right ventricular subendocardial
d. Right ventricular subendocardial 187. Osteomalacia is most commonly seen in
a. Young adult males
181. The explosive and widespread form of secondary b. Postmenopausal females
syphilis in immunocompromised patients is known as c. Elderly males
a. Condylomata lata d. Young adult females
b. Mucous patches
c. Lues maligna 188. Which of the following vitamins is an antioxidant?
d. Lupus vulgaris a. Vitamin A
b. Vitamin D
182. Urine analysis of patient associated with haematuria c. Vitamin E
and glomerulonephritis shows d. Vitamin K
a. Isomorphic red cells
b. Red cell casts 189. Most common oral manifestation of vitamin K
c. WBC casts deficiency is
d. Hyaline cast a. Gingival bleeding
b. Discoloured teeth
183. Flow cytometry analysis is done for c. Gingivitis
a. Blood glucose estimation d. Hypocalcified teeth
b. LDL fraction estimation
c. CD4/CD8 counts 190. Deficiency of thiamine causes
d. None a. Pellagra
b. Beriberi
c. Anaemia
d. Scurvy

178. b 179. d 180. c 181. c 182. b 183. c 184. c 185. a 186. d 187. b 188. c 189. a 190. b
Section | IV Multiple Choice Questions 643

191. Pellagra occurs due to deficiency of 196. Hypocalcaemia, multiple giant cell lesions of bones
a. Thiamine and a generalized radiolucency of bones as compared
b. Pantothenic acid to normal people are features of
c. Niacin a. Hyperthyroidism
d. Biotin b. Hyperpituitarism
c. Cushing’s syndrome
192. Features of dermatitis, diarrhoea and dementia are d. Hyperparathyroidism
characteristics of
a. Pellagra 197. Acute adrenal insufficiency is also known as
b. Beriberi a. Waterhouse–Friderichsen syndrome
c. Scurvy b. Addison’s disease
d. Megaloblastic anaemia c. Cushing’s syndrome
d. Adrenogenital syndrome
193. Calcitonin is secreted by
a. Thyroid gland 198. All of the following are seen in Cushing’s disease, except
b. Parathyroid gland a. Buffalo hump
c. Pituitary gland b. Vascular hypertension
d. Adrenal medulla c. Muscular weakness
d. Hypoglycaemia
194. Hypothyroidism in adults causes
a. Cretinism 199. ‘Bald tongue of Sandwith’ occurs due to deficiency of
b. Myxoedema a. Niacin
c. Gigantism b. Folic acid
d. Acromegaly c. Riboflavin
d. Iron
195. Hypersecretion of which of the following endocrine
glands causes an increase in basal metabolic rate? 200. Chronic fluoride intoxication is characterized by
a. Pituitary d. Osteosclerosis
b. Thyroid b. Arthritic changes in spine causing rigidity
c. Parathyroid c. Calcification of muscle and tendon sheaths
d. Adrenal cortex d. All of the above

191. c 192. a 193. a 194. b 195. b 196. d 197. a 198. d 199. a 200. d
644 Quick Review Series for BDS 3rd Year

Section II

General Surgery
1. Treatment of choice for pleomorphic adenoma 7. All of the following can be used in the treatment of
a. Superficial parotidectomy thyroid storm, except
b. Radical parotidectomy a. Propranolol
c. Enucleation b. Radioactive iodine
d. Radiotherapy c. Hydrocortisone
d. Lugol’s iodine
2. Which of the following is characteristic histopathologi-
cal feature of medullary carcinoma of thyroid? 8. Which investigation is most informative and most com-
a. Anaplasia monly used in case of hypothyroidism?
b. Amyloid stroma a. Serum TSH level
c. Mitotic figures b. Serum T3, T4 level
d. Metaplasia c. Serum calcitonin assay
d. Serum TRH assay
3. Which carcinoma is most likely to occur when a patient
has pituitary tumour, pheochromocytoma and a thyroid 9. MEN-II is seen in association with the following type
nodule? of thyroid carcinoma
a. Follicular carcinoma a. Papillary
b. Medullary carcinoma b. Medullary
c. Papillary carcinoma c. Anaplastic
d. Anaplastic carcinoma d. Follicular

4. Which of the following is used in the treatment of 10. Most common metastasis in thyroid gland comes from
thyroid malignancy? carcinoma of
a. 131I a. Testes
b. 125I b. Prostate
c. 99TC c. Breast
d. 32P d. Lungs

5. A patient with long-standing multinodular goitre devel- 11. Best diagnostic modality for parotid swelling is
ops hoarseness of voice; also, the swelling undergoes a. Enucleation
sudden increase in size. Likely diagnosis is b. FNAC
a. Follicular carcinoma c. Superficial parotidectomy
b. Papillary carcinoma d. Excisional biopsy
c. Medullary carcinoma
d. Anaplastic carcinoma 12. Which of the following is the symptom of hyperthy-
roidism?
6. The most common presentation of endemic goitre is a. Palpitations
a. Hypothyroid b. Intolerance to cold
b. Diffuse goitre c. Decreased appetite
c. Hyperthyroid d. Weight gain
d. Solitary nodule

1. a 2. b 3. b 4. a 5. a 6. b 7. b 8. a 9. b 10. c 11. b 12. a


Section | IV Multiple Choice Questions 645

13. A 40-year-old female patient presented with a progres- 20. The parotid duct is known as
sively increasing lump in the parotid region. On oral a. Wharton’s duct
examination, the tonsil was pushed medially. Biopsy b. Stensen’s duct
showed it to be pleomorphic adenoma. The appropriate c. Duct of Santorini
treatment is d. Duct of Wirsung
a. Superficial parotidectomy
b. Lumpectomy 21. Most common malignant tumour in children
c. Conservative total parotidectomy a. Ewing’s sarcoma
d. Enucleation b. Metastatic carcinoma
c. Liposarcoma
14. In case of benign mixed parotid tumours, treatment of d. Osteosarcoma
choice is
a. Superficial parotidectomy 22. Treatment for primary malignant melanoma is
b. Total parotidectomy a. Excision
c. Leave facial nerve and remove all glands b. Chemotherapy
d. Radical parotidectomy c. Radiotherapy
d. Enucleation
15. All of the following are true for pleomorphic adenoma
except 23. Codman’s triangle is seen in
a. Arises from parotid a. Osteoradionecrosis
b. May turn into malignant b. Fibrous dysplasia
c. Minor salivary glands are also involved c. Osteosarcoma
d. Very rare tumour d. Squamous cell carcinoma of lip

16. Which of the following is not an indication of radio- 24. Most common mode of metastasis in osteogenic
therapy in pleomorphic adenoma of parotid? sarcoma is
a. Involvement of deep lobe a. Subperiosteal spread
b. Recurrence b. Haematogenous
c. Microscopically positive margins c. Lymphatic
d. Malignant transformation d. Transcortical

17. Treatment of choice for Warthin’s tumour 25. True about giant cell tumour is
a. Superficial parotidectomy a. Never recurs after excision
b. Enucleation b. Periarticular and epiphyseal occurrence
c. Radiotherapy c. Malignant potential is not predictable
d. Injection of a sclerosing agent d. None of the above

18. Mucoepidermoid carcinoma arises from 26. Most common site of osteoblastoma is
a. Myothelium a. Epiphysis
b. Epithelium b. Metaphysis
c. Acinus c. Diaphysis
d. Mucin secreting and epidermal cells d. All of the above

19. What should be done first for a collapsed or uncon- 27. Aneurysmal bone cysts
scious victim of illness or accident? a. Are true aneurysms of nutrient arteries
a. Maintain the airway b. Occur only in flat bones
b. Establish unresponsiveness c. Are similar to osseous haemangiomas
c. Administer abdominal thrust d. Manifest as osteolytic lesions in long bones
d. Examine the victim for bleeding and fractures

13. c 14. a 15. d 16. a 17. a 18. d 19. b 20. b 21. d 22. a 23. c 24. b 25. b 26. b 27. b
646 Quick Review Series for BDS 3rd Year

28. A boy presenting with swelling at lower end of femur 35. Structure preserved in radical neck dissection is
and calcified nodular findings in lung radiograph has a. Vagus nerve
a. Osteosarcoma b. Submandibular gland
b. Osteochondroma c. Sternocleidomastoid
c. Tuberculosis femur lower end d. Internal jugular vein
d. Osteomyelitis
36. Which of the following is true regarding cystic hygroma?
29. Densely calcified metastatic lesions are found in a. Non-transilluminant
a. Synovial cell carcinoma b. Lined by columnar epithelium
b. Osteosarcoma c. Lined by stratified squamous epithelium
c. Chondrosarcoma d. Developed from jugular lymphatic sequestration
d. Chondroblastoma
37. The most common and earliest manifestation of carci-
30. True statement regarding osteogenic sarcoma is noma of the glottis is
a. Affects middle-aged people a. Hoarseness of voice
b. Radiographs show honeycombing b. Haemoptysis
c. Can be a complication of Paget’s disease of bone c. Cervical lymph nodes
d. All of the above d. Stridor

31. Which one is not true regarding carotid body tumour? 38. Which of the following syndromes is usually associated
a. Neonates are commonly affected with cystic hygroma?
b. Surgical resection is the treatment a. Marfan’s syndrome
c. Nonchromaffin paraganglioma b. Turner’s syndrome
d. Middle age group is affected c. Down’s syndrome
d. Noonan’s syndrome
32. A 40-year-old patient is suffering from carotid body
tumour. Which of the following is the best choice of 39. The most lethal complication due to blood transfusion is
treatment? a. Antigen’s–antibody reaction
a. Excision of tumour b. Ischaemia of liver
b. Radiotherapy c. Hepatitis B
c. Chemotherapy d. HIV transmission
d. Carotid artery ligation both proximal and distal to
the tumour 40. Which of the following muscles will tend to pull the
mandible back in case of mandibular fracture in the
33. Which of the following is true of carotid body tumour? canine region?
a. Nonchromaffin paraganglioma a. Genioglossus and anterior belly of digastric
b. Good prognosis b. Genioglossus and mylohyoid
c. Rarely metastasises c. Genioglossus and thyrohyoid
d. All d. Genioglossus and masseter

34. Which of the following structures are preserved in 41. The best treatment for cystic hygroma is
functional neck dissection? a. Surgical excision
a. Carotid artery, vagus nerve b. Radiotherapy
b. Sternomastoid muscle, internal jugular vein c. Sclerotherapy
c. Spinal accessory nerve, submandibular gland d. Chemotherapy
d. Neck nodes

28. a 29. b 30. c 31. a 32. a 33. d 34. b 35. a 36. d 37. a 38. b 39. a 40. a 41. a
Section | IV Multiple Choice Questions 647

42. A graft that has been derived from another species of a 49. Which one of the following preservatives is used while
different genetic disposition is also known as packing catgut suture?
a. Allograft a. Colloidal iodine
b. Isograft b. Isopropyl alcohol
c. Homograft c. Glutaraldehyde
d. Heterograft d. Hydrogen peroxide

43. All of the following are true about cystic hygroma 50. The most common malignant tumour of adult males in
except India is
a. Aspiration is diagnostic a. Oropharyngeal carcinoma
b. 50% present at birth b. Gastric carcinoma
c. Present as posterior cervical swelling c. Colorectal carcinoma
d. Sequestration of lymphatic tissue d. Lung cancer

44. The PNS view of the skull exhibiting hanging-drop 51. In some old fractures, cartilaginous tissue forms over-
appearance in maxillary sinus indicates fractured bone ends with a cavity in between contain-
a. A nasal polyp ing clear fluid. This condition is known as
b. A blow-out fracture of the orbit a. Delayed union
c. A nasal fracture b. Pseudoarthrosis
d. An antrolith c. Nonunion
d. Slow union
45. Wound contraction can be most effectively minimized by
a. Split skin graft 52. In which of the following location, there is collection
b. Full thickness grafting of pus in quinsy?
c. Allowing secondary granulation a. Sublingual space
d. Dressing with placenta b. Parapharyngeal space
c. Retropharyngeal space
46. For reimplantation surgery, the detached limb best d. Peritonsillar space
preserved in cold is
a. Glycerol 53. Cleft lip repair is commonly done during which of the
b. Distilled water following age groups?
c. Hypertonic saline a. 3–6 months
d. Isotonic saline b. 3–6 years
c. 10–15 years
47. Virchow’s triad includes all of the following, except d. At the time of birth
a. Venous stasis
b. Injury to veins 54. In extradural haematoma, the patient presents with
c. Blood hypercoagulability a. Slow, bounding pulse and falling BP
d. Venous thrombosis b. Rapid, thready pulse and falling BP
c. Slow, bounding pulse and rising BP
48. The commonest site of oral cancer among Indian d. Rapid pulse and normal BP
population is
a. Tongue 55. A knitted Dacron artery graft is
b. Floor of mouth a. Nonporous
c. Lip b. Absorbable
d. Alveobuccal complex c. Noninfective
d. Can be easily incised and the opening resutured

42. d 43. a 44. b 45. b 46. d 47. d 48. d 49. b 50. a 51. b 52. d 53. a 54. c 55. d
648 Quick Review Series for BDS 3rd Year

56. A male patient, who is 45-year-old with history of 63. Which of the following antibiotics is most frequently
chronic smoking and pain in lower limb due to block- associated with pseudomembranous colitis?
age of femoral artery is diagnosed for a. Ampicillin
a. Thromboangiitis obliterans b. Penicillin G
b. Atherosclerosis c. Clindamycin
c. Embolism d. Cephalosporin
d. Arteritis
64. Collar-stud abscess is associated with
57. Which of the following is not used as graft material in a. Mycobacterium tuberculosis
peripheral vascular disease? b. Mycobacterium leprae
a. Dacron c. Clostridium tetani
b. Vein d. Clostridium perfringens
c. PTEE
d. PVC 65. One unit of fresh blood raises the Hb% concentration by
a. 0.1 g%
58. In the healing of an ulcer, the epithelium that eventu- b. 1 g%
ally will cover the defect is derived from c. 2 g%
a. Intact epithelium at the ulcer margin d. 2.5 g%
b. Metaplasia of fibroblasts to epithelial cells
c. Transformation of fibrinous exudates to epithelium 66. A patient of burns should be given
d. Transformation of endothelial cells to epithelial a. 5% Dextrose
cells b. Dextrose saline
c. Hypertonic saline 0.9%
59. Lumbar sympathectomy is not indicated in d. Human albumin 4.5%
a. Healing of ulcer over great toe
b. Claudication 67. Tumour which does not show spontaneous regression
c. Rest pain a. Neuroblastoma
d. Buerger’s disease b. Retinoblastoma
c. Malignant melanoma
60. In a 40-year-old male patient, thrombus in the common d. Osteosarcoma
femoral artery is because of
a. Atheroma 68. Which of the following is least useful in excessive burns?
b. Thromboangiitis obliterans a. Blood
c. Raynaud’s disease b. Dextran
d. Abdominal mass c. Ringer lactate
d. Nasogastric intubation
61. Most common form of actinomycosis is
a. Cervicofacial 69. Erysipelas is caused by
b. Thoracic a. Staphylococcus aureus
c. Lumbar b. Staphylococcus albus
d. Liver c. Streptococcus pyogenes
d. Haemophilus influenzae
62. Most common cause of aneurysm of abdominal artery is
a. Trauma 70. Treatment of contaminated wound in gas gangrene is
b. Atherosclerosis a. Debridement of wound
c. Syphilis b. Systemic penicillin
d. Cystic necrosis c. Metronidazole administration
d. Peroxide dressings

56. b 57. d 58. a 59. b 60. a 61. a 62. b 63. c 64. a 65. b 66. d 67. c 68. b 69. c 70. a
Section | IV Multiple Choice Questions 649

71. Most common benign tumour of bone 78. Delayed wound healing is seen in all of the following
a. Osteochondroma except
b. Enchondroma a. Malignancy
c. Osteoid osteoma b. Hypertension
d. Aneurysmal bone cyst c. Diabetes
d. Infection
72. Salmonella may cause
a. Enteric fever 79. Degloving injury is
b. Osteitis a. Abrasive wound
c. Bacteraemia b. Lacerated wound
d. All of the above c. Blunt injury
d. Avulsion injury
73. The treatment of choice of a glottic cancer with stage
T1NoMo is 80. Most common cause of cellulitis is
a. Brachytherapy a. Staphylococcus
b. External beam radiotherapy b. Streptococcus
c. Laryngectomy c. Escherichia coli
d. Chemotherapy d. Haemophilus

74. The best scars are seen in 81. Which of the following is true regarding superficial
a. Infants burns?
b. Children a. Always requires skin grafting
c. Adults b. Dry and inelastic
d. Very old people c. Blister formation
d. Painless
75. If suture marks are to be avoided, skin sutures should
be removed by 82. The most common retrobulbar orbital mass in adults is
a. 12 weeks a. Neurofibroma
b. 1 week b. Meningioma
c. 2 weeks c. Cavernous haemangioma
d. 3 weeks d. Schwannoma

76. The treatment of choice for nasopharyngeal carcinoma is 83. Sentinel lymph node is
a. Chemotherapy alone a. Group of lymph nodes
b. Radiotherapy alone b. First stop along the route of lymphatics
c. Radiotherapy and surgery c. A node that is skipped during distant metastasis
d. Surgery alone d. Distant metastatic node

77. Undue restlessness in a patient during the immediate 84. In haemorrhagic shock, hypotension occurs when
post-burns is often a manifestation of blood loss is more than
a. Hypoxia a. 10–15%
b. Hypovolaemia b. 15–30%
c. Hyperkalaemia c. 30–40%
d. Anxiety d. More than 40%

71. c 72. d 73. b 74. d 75. b 76. b 77. d 78. b 79. d 80. b 81. c 82. c 83. b 84. c
650 Quick Review Series for BDS 3rd Year

85. The maximum collagen content of wound tissue 92. The following statements about thyroglossal cyst are
attained from true, except
a. 3rd to 5th day a. Frequent cause of anterior midline neck masses in
b. 6th to 15th day the first decade of life
c. 17th to 21st day b. The cyst is located within 2 cm of the midline
d. 30th to 90th day c. The swelling moves upwards on protrusion of
tongue
86. The explosive and widespread form of secondary syph- d. Incision and drainage is the treatment of choice
ilis in an immunocompromised individual is known as
a. Condylomata lata 93. The most common cause of hypoparathyroidism after
b. Mucous patches thyroidectomy is
c. Lues maligna a. Removal
d. Lupus vulgaris b. Vascular injury
c. Hypertrophy
87. Mixed tumours of the salivary glands are d. Malignancy
a. Most common in submandibular gland
b. Usually malignant 94. Neointimal hyperplasia causes vascular graft failure as
c. Most common in parotid gland a result of hypertrophy of
d. Associated with calculi a. Endothelial cells
b. Collagen fibres
88. A malignant tumour of childhood that metastasizes to c. Smooth muscle cells
bones most often is d. Elastic fibres
a. Neuroblastoma
b. Wilms’ tumour 95. Dacron vascular graft is
c. Adrenal gland tumours a. Nontextile synthetic
d. Granulosa cell tumour of ovary b. Textile synthetic
c. Textile biologic
89. Laryngocoele arises as a herniation of laryngeal d. Nontextile biologic
mucosa through which of the following membranes?
a. Thyrohyoid 96. ‘Rugger jersey spine’ is seen in
b. Cricothyroid a. Fluorosis
c. Cricotracheal b. Renal osteodystrophy
d. Cricosternal c. Achondroplasia
d. Down’s syndrome
90. Nerve injury leading to contusion of the nerve, in
which continuity of both epineural sheath and the 97. Brown tumours are seen in
axons are maintained is known as a. Hyperparathyroidism
a. Neurotmesis b. Pigmented villonodular synovitis
b. Axonotmesis c. Osteomalacia
c. Neuropraxia d. Neurofibromatosis
d. Traumatic neuroma
98. Which of the following malignant tumours is
91. Teletherapy is the treatment by radioresistant?
a. Chemotherapy a. Ewing’s sarcoma
b. Laser beam therapy b. Retinoblastoma
c. Radiotherapy c. Osteosarcoma
d. Electrotherapy d. Neuroblastoma

85. c 86. c 87. c 88. a 89. a 90. c 91. c 92. d 93. b 94. c 95. b 96. b 97. a 98. c
Section | IV Multiple Choice Questions 651

99. The most preferred approach for pituitary surgery at 106. Thyroglossal fistula is
the present time is a. Congenital
a. Transcranial b. Acquired
b. Transethmoidal c. Hereditary
c. Transsphenoidal d. Discharges blood
d. Transantral
107. Most frequently encountered clostridial species in gas
100. Wide scars occur due to gangrene is
a. Early suture removal a. Clostridium botulinum
b. Use of subcutaneous absorbable sutures b. Clostridium welchii
c. Excessive skin tension c. Clostridium tetani
d. Excessive undermining of skin edges d. Clostridium difficile

101. Tachycardia, nervousness, intolerance to heat and ex- 108. Chronic lymphocytic thyroiditis is also known as
ophthalmos are seen in a. Tuberculous thyroiditis
a. Hyperthyroidism b. Hashimoto’s thyroiditis
b. Hypothyroidism c. De Quervain’s thyroiditis
c. Hyperparathyroidism d. Riedel’s thyroiditis
d. Hypoparathyroidism
109. Rigidity of facial muscles, ‘risus sardonicus’, is
102. Factors influencing wound healing is/are the following: associated with
a. Nutrition a. Tetanus
b. Adequate blood supply b. Syphilis
c. Size and location of wound c. Leprosy
d. All of the above d. Actinomycosis

103. A man is rushed to casualty, after a massive blood loss 110. Important components of collagen synthesis, wound
in an accident. There is not much time to match blood strength and contraction include all of the following,
groups, so the physician decides to order for one of except
the following blood groups a. Fibroblasts
a. O negative b. Myofibroblasts
b. O positive c. Vitamin C
c. AB positive d. Vitamin D
d. AB negative
111. Keloid scar is made up of
104. Prophylactic antibiotics in surgical practice a. Dense collagen
a. Are given orally b. Fibrous tissue
b. Depend on individual preference c. Loose areolar tissue
c. Are continued for a minimum of 10 days d. Granulation tissue
d. First dose is given before induction of anaesthesia
112. The most common site of metastasis from the man-
105. Tuberculous lymphadenitis of the cervical lymph dibular sarcoma is
nodes is also called as a. Lung
a. Pott’s disease b. Liver
b. Lupus vulgaris c. Spleen
c. Scrofula d. Heart
d. Ghon’s focus

99. c 100. c 101. a 102. d 103. a 104. d 105. c 106. b 107. b 108. b 109. a 110. d 111. a 112. a
652 Quick Review Series for BDS 3rd Year

113. Horner’s syndrome does not include 120. The most common retrobulbar orbital mass in adults is
a. Ptosis a. Neurofibroma
b. Anhidrosis b. Meningioma
c. Flushing c. Cavernous haemangioma
d. Mydriasis d. Schwannoma

114. A cold nodule in thyroiditis scan means 121. Mixed tumours of the salivary glands are
a. Nodule, which is cold a. Most common in submandibular gland
b. Hyperactive nodule b. Usually malignant
c. Nodule, which is nonfunctioning and malignant c. Most common in parotid gland
d. Nodule, which has no signs of inflammation d. Associated with calculi

115. The most important aspect in treating disseminated 122. A malignant tumour of childhood that metastasizes to
intravascular coagulation (DIC) is to bones most often is
a. Administer heparin a. Wilms’ tumour
b. Administer platelets b. Neuroblastoma
c. Treat the underlying disease process c. Adrenal gland tumours
d. Achieve normal levels of fibrinogen d. Granulosa cell tumour of ovary

116. All of the following components or qualities of stored 123. Expansile lytic osseous metastases are characteristic
whole blood tend to decrease over time except of primary malignancy of
a. Red blood cell viability a. Kidney
b. Potassium concentration b. Bronchus
c. pH c. Breast
d. Platelet activity d. Prostate

117. Hepatitis B occurs after an incubation period of 124. The most preferred approach for pituitary surgery is
a. 4–20 weeks a. Transcranial
b. 3 weeks b. Transethmoidal
c. 12 weeks c. Transsphenoidal
d. 15–30 weeks d. Transcallosal

118. Rugger jersey spine is seen in 125. Golden period for treatment of open wounds is
a. Fluorosis a. 2 hours
b. Achondroplasia b. 4 hours
c. Renal osteodystrophy c. 6 hours
d. Madan’s syndrome d. 8 hours

119. Brown tumours are seen in 126. All of the following statements are true about exosto-
a. Hyperparathyroidism sis, except
b. Pigmented villonodular synovitis a. It occurs at the growing end of bone
c. Osteomalacia b. Growth continues after skeletal maturity
d. Neurofibromatosis c. It is covered by cartilaginous cap
d. Malignant transformation may occur

113. d 114. c 115. c 116. b 117. a 118. c 119. a 120. c 121. c 122. b 123. d 124. c 125. c 126. b
Section | IV Multiple Choice Questions 653

127. A patient of burns should be given 134. The nerve sacrificed in parotid surgery
a. Human albumin a. Auriculotemporal
b. Saline b. Facial
c. Hypertonic saline 0.9% c. Buccal
d. 5% Dextrose d. Cervicofacial

128. Serum calcitonin is a marker for 135. Ptosis may occur due to damage of
a. Anaplastic carcinoma a. Oculomotor nerve
b. Papillary carcinoma b. Trochlear nerve
c. Medullary carcinoma c. Trigeminal nerve
d. Follicular carcinoma d. Facial nerve

129. Best treatment for cervical lymph node involvement 136. Alkaline phosphatase level is increased in
of papillary carcinoma of thyroid is a. Osteitis deformans
a. Radioactive iodine b. Down’s syndrome
b. Chemotherapy c. Familial fibrous dysplasia of jaws
c. Radical neck dissection d. Osteopetrosis
d. Steroid
137. Which of the following antibiotics is effective in oral
130. Most common solitary thyroid nodules is candidiasis?
a. Follicular adenoma a. Nystatin
b. Hurler cell carcinoma b. Bacitracin
c. Papillary carcinoma c. Penicillin
d. Solitary idiopathic goitre d. Tetracycline

131. Treatment of submandibular salivary gland duct 138. The most common bone cancer is
calculi is a. Osteosarcoma
a. Excision of submandibular gland b. Metastatic bone cancer
b. Excision of gland and duct c. Multiple myeloma
c. Opening the duct at the frenulum d. Squamous cell carcinoma
d. Opening the duct and removal of calculus
139. The commonest site for basal cell carcinoma is
132. Most common tumour of minor salivary glands is a. Tongue
a. Mucoepidermoid carcinoma b. Gingiva
b. Mixed tumour c. Lower lip
c. Squamous cell carcinoma d. Upper face
d. Epithelioma
140. Commonest benign tumour under 21 years of age
133. Which of the following groups of symptom constitute a. Aneurysmal bone cyst
Frey’s syndrome? b. Osteochondroma
a. Hyperhydrosis, enophthalmos and miosis c. Giant cell tumour
b. Anhidrosis, enophthalmos and miosis d. Osteoid talus
c. Redness and sweating over the auriculotemporal
region during meal
d. Pain over the distribution of the auriculotemporal
nerve during meal

127. d 128. c 129. c 130. a 131. d 132. a 133. c 134. b 135. a 136. a 137. a 138. b 139. d 140. b
654 Quick Review Series for BDS 3rd Year

141. Sunray appearance on radiograph is seen in 148. Lesions of oral tuberculosis are associated with all the
a. Ewing’s sarcoma following, except
b. Osteogenic sarcoma a. Central caseation
c. Osteomyelitis b. Hyaline degeneration
d. Chondrosarcoma c. Giant cells in the centre
d. Presence of epithelioid cells
142. Stored plasma is deficient in which of the following
factors? 149. Diabetic gangrene is due to all, except
a. VII and VIII a. Vasospasm
b. VIII and XII b. Atherosclerosis
c. VI and X c. Peripheral neuritis
d. II and V d. Increased sugar in blood

143. Which of the following is the best indicator of need 150. Most commonest cause of AV fistulae is
for transfusion? a. Congenital
a. Haematocrit b. Traumatic
b. Skin colour c. Surgical creation
c. Urine output d. Tumour erosion
d. Clinical examination
151. Most common cause of gangrene of foot in a 30-year-
144. Multiple punched-out lesions in skull is characteristic old person who is a chronic smoker is
of a. Raynaud’s disease
a. Acute leukaemia b. Myocardial infarction
b. Thalassaemia c. Atherosclerosis
c. Multiple myeloma d. Thromboangiitis obliterans
d. Carcinoma prostate
152. Most commonly varicose veins are seen with
145. Cryoprecipitate is a rich source of a. Long saphenous vein
a. Factor X b. Short saphenous vein
b. Factor VII c. Both
c. Factor VIII d. Popliteal and femoral vein
d. Factor I
153. Most common benign neoplasm of oral cavity is
146. Which tumour does not arise from cartilage? a. Myxoma
a. Osteoblastoma b. Lipoma
b. Osteochondroma c. Fibroma
c. Chondrosarcoma d. Papilloma
d. Endochondroma
154. Fracture of infraorbital margin will lead to
147. Which is a late feature of multiple myeloma? a. Paralysis of muscle of cheek
a. Osteolytic lesion b. Paralysis of muscle of lower eyelid
b. Mental spikes c. Anaesthesia of upper lip
c. Renal failure d. No effects
d. Raised serum alkaline phosphate

141. b 142. b 143. a 144. c 145. c 146. a 147. c 148. b 149. a 150. a 151. c 152. a 153. c 154. c
Section | IV Multiple Choice Questions 655

155. Single most reliable diagnostic criteria for squamous 162. Cases of haemangioma are present most commonly in
cell carcinoma: which region?
a. Invasion a. Bones
b. Degeneration b. Head and neck
c. Pleomorphism c. Gastrointestinal tract
d. Encapsulation d. Liver

156. Battle’s sign is seen in fracture of 163. Which of the following is a complication of tracheos-
a. Zygoma tomy?
b. Middle cranial fossa a. Surgical emphysema
c. Occipital bone b. Pneumothorax
d. Posterior cranial fossa c. Carotid artery injury
d. Subglottic stenosis
157. Drug of choice for penicillin-resistant infective mi-
croorganisms is 164. Which one is not true regarding Buerger’s disease?
a. Ampicillin a. Men are usually involved
b. Amoxicillin b. Occurs below 50 years of age
c. Erythromycin c. Smoking is predisposing factor
d. Cloxacillin d. Veins and nerves are never involved

158. Tracheostomy is done through 165. Nerve tumour having greatest chances of malignant
a. First tracheal ring transformation is
b. Second and fourth tracheal ring a. Neurolemmoma
c. Cricothyroid membrane b. Neurofibroma
d. Thyrohyoid ligament c. Traumatic neuroma
d. Neurogenic fibroma
159. Ludwig’s angina is
a. Unilateral involvement of submandibular, sublin- 166. Untrue about the incidence of cleft lip and palate is
gual and submental space infection a. Cleft lip and palate is most common in male
b. Unilateral involvement of sublingual and submen- b. Cleft lip is more common on right side
tal space infection c. Cleft palate alone is most common in female
c. Bilateral involvement of submandibular, sublin- d. Unilateral cleft is more common than bilateral
gual and submental space infection cleft
d. Bilateral involvement of submandibular and
submental space infection 167. Most common cause of death in patients with Buerger’s
disease is
160. Osteomyelitis is caused by a. Gangrene
a. Streptococcus pyogenes b. Pulmonary embolism
b. Staphylococcus aureus c. Myocardial infarction
c. Prevotella d. Carcinoma of lung
d. Bacteroides
168. Mixed tumours of the salivary glands are
161. Steroids cause a. Most common in submandibular gland
a. Delayed wound healing b. Usually malignant
b. No effect on wound healing c. Most common in parotid gland
c. Rapid wound healing d. Associated with calculi
d. None

155. a 156. b 157. d 158. b 159. c 160. b 161. a 162. b 163. a 164. d 165. b 166. b 167. c 168. c
656 Quick Review Series for BDS 3rd Year

169. Anterior open bite occurs in fracture of 176. The manufacturing unit of surgical needle and sutures
a. Symphysis are sterilized by
b. Bilateral angles a. Gamma rays
c. Bilateral condyles b. Radiographs
d. Unilateral condyle c. Autoclaving
d. Boiling
170. Suture material most commonly used intraorally for
wound closure is 177. The type of carcinoma develops in Marjolin’s ulcer is
a. Black silk a. Basal cell carcinoma
b. Catgut b. Fibrogenic sarcoma
c. Nylon c. Adenocarcinoma
d. Stainless steel d. None

171. Oral cancer is most commonly spread by 178. Trismus can result from the infection of which space?
a. Blood vessels a. Submandibular space
b. Lymphatics b. Sublingual space
c. Direct invasion c. Submasseteric space
d. Local infiltration d. Buccal space

172. Which of the following has no tendency to metastasis? 179. Bedsore is an example of
a. Malignant melanoma a. Typical ulcer
b. Squamous cell carcinoma b. Trophic ulcer
c. Leukoplakia c. Venous ulcer
d. Basal cell carcinoma d. Post-thrombotic ulcer

173. Odontogenic keratocyst 180. The most common histological type of thyroid cancer
a. Is developed from acute infection is
b. Is a developmental cyst a. Medullary type
c. Has low recurrence rate b. Papillary type
d. Has high protein content c. Follicular type
d. Anaplastic type
174. The repair of cleft lip is done at
a. 2–12 weeks 181. Most common primary bone tumour is
b. Just after birth a. Osteosarcoma
c. After 12 months b. Multiple myeloma
d. 2–12 months c. Ewing’s sarcoma
d. Enchondroma
175. The microorganism used in testing the efficiency of
moist heat sterilizing agents is 182. The first manifestation of shock is
a. Bacillus stearothermophilus a. Tachycardia
b. Bacillus anthracis b. Bradycardia
c. Clostridium tetani c. Constriction of pupils
d. Mycobacterium tuberculosis d. None of the above

169. c 170. a 171. b 172. d 173. b 174. a 175. a 176. a 177. a 178. c 179. c 180. b 181. b 182. a
Section | IV Multiple Choice Questions 657

183. Universal tumour is 190. Increased sodium loss leads to


a. Lipoma a. Vomiting
b. Fibroma b. Dehydration
c. Haemangioma c. Cortical deficiency
d. Hamartoma d. All of the above

184. Earliest sign of deep vein thrombosis is 191. Glasgow Coma Scale is used in
a. Calf tenderness a. Head injury
b. Rise in temperature b. Burns
c. Swelling of calf muscle c. Septicaemia
d. Homans’ sign d. Acute pancreatitis

185. Which of the following is not a compressible swelling? 192. In haemorrhagic shock, hypotension occurs when
a. Meningocoele blood loss is more than
b. Lymphangioma a. 10–15%
c. Mucocoele b. 15–30%
d. Cystic hygroma c. 30–40%
d. More than 40%
186. The following is the commonest site for venous ulcer
a. Instep of foot 193. Dacryocystitis is interruption in drainage of
b. Lower one-third of leg and ankle a. Submandibular gland
c. Lower two-thirds of leg b. Parotid gland
d. Middle one-third of leg c. Lacrimal gland
d. Sublingual gland
187. The most common retrobulbar orbital mass in adults
is 194. Tuberculous lymphadenitis of the cervical lymph
a. Neurofibroma nodes is also called as
b. Meningioma a. Pott’s disease
c. Cavernous haemangioma b. Lupus vulgaris
d. Schwannoma c. Scrofula
d. Ghon focus
188. The most common site of enlargement of the lymph
nodes in Hodgkin’s lymphoma is 195. Thyroglossal fistula is
a. Mediastinal a. Congenital
b. Axillary b. Acquired
c. Cervical c. Hereditary
d. Abdominal d. Discharges blood

189. High-risk groups for transmission of HIV include the 196. Erysipelas is caused by
following, except a. Group B staphylococci
a. Homosexuals b. Group A staphylococci
b. Haemophilics c. Gonococci
c. Children of HIV mothers d. Pneumococci
d. Health care workers

183. a 184. a 185. b 186. b 187. c 188. a 189. d 190. b 191. a 192. c 193. c 194. c 195. b 196. b
658 Quick Review Series for BDS 3rd Year

197. The most common cause of hypoparathyroidism after c. Radiotherapy and surgery
thyroidectomy is d. Surgery alone
a. Removal
b. Vascular injury 200. Teletherapy is the treatment by
c. Hypertrophy a. Chemotherapy
d. Malignancy b. Laser beam therapy
c. Radiotherapy
198. Skin incision of the face is sutured with the edges d. Electrotherapy
a. Inverted
b. Everted 201. Sentinel lymph node is
c. Same level a. Group of lymph nodes
d. Dipping b. First stop along route of lymphatic drainage
c. Lymph node that is skipped during distant metas-
199. The treatment of choice for nasopharyngeal carcinoma is tasis
a. Chemotherapy alone d. Distant metastatic node
b. Radiotherapy alone

197. b 198. b 199. b 200. c 201. b


Section | IV Multiple Choice Questions 659

Section III

Oral Pathology
1. Which of the following vitamin’s deficiency is the 7. The most common malformation of head and neck
commonest cause of craniofacial anomalies? region is
a. Riboflavin a. Cleft lip and palate
b. Folic acid b. Haemangioma
c. Ascorbic acid c. Preauricular cyst
d. Retinoic acid d. Malformation of ear

2. Premature closure of cranial sutures in an infant results in 8. The most common type of malocclusion according to
a. Cleft lip and palate Angle is
b. Hemifacial macrosomia a. Class I
c. Craniosynostosis b. Class II
d. Hemifacial microsomia c. Class III
d. None of the above
3. The genetic constitution of an individual is known as
a. Phenotype 9. Most common aetiology attributed to cleft lip and palate is
b. Heritability a. Hereditary
c. Genotype b. Environmental
d. None of the above c. Circulating substances
d. Infection
4. The frequency of occurrence of cleft lip with or without
cleft palate is 10. The surgical closure of cleft palate is done only after
a. 1 in 800 the age of
b. 1 in 500 a. 1 year
c. 1 in 1000 b. 1.5 years
d. 1 in 2500 c. 2 years
d. 2.5 years
5. Which of the following pathologies is seen in facial
hemihypertrophy? 11. Peutz–Jeghers syndrome is also known as
a. Hypoplasia a. Hereditary intestinal polyposis syndrome
b. Hypotrophy b. Cheilitis granulomatosa
c. Hyperplasia c. Facial hemiatrophy
d. Hypertrophy d. None of the above

6. All of the following defects in dentition shows dominant 12. Which of the following virus has not been known to
pattern of inheritance, except cause malignant diseases in men?
a. Dentin dysplasia a. EBV
b. Dentinogenesis imperfecta b. CMV
c. Peg lateral c. HSV type II
d. Hypomaturation of enamel d. VZV

1. b 2. c 3. c 4. a 5. c 6. d 7. a 8. a 9. a 10. b 11. a 12. d


660 Quick Review Series for BDS 3rd Year

13. An angulation or a sharp bend or curve in the root or 21. Extrapulmonary form of tuberculosis is seen com-
crown of the formed tooth is known as monly among
a. Talon cusp a. HIV patients
b. Taurodontism b. Children
c. Dilaceration c. Aged patients
d. None of the above d. Malnourished individuals

14. ‘Hereditary brown opalescent teeth’ is the term used for 22. Scarlet fever is caused by
a. Amelogenesis imperfecta a. Staphylococcus aureus
b. Dentinogenesis imperfecta b. b-haemolytic Streptococcus
c. Hypoplastic enamel c. a-haemolytic Streptococcus
d. None of the above d. g-haemolytic Streptococcus

15. The optimum level of fluoride in drinking water is 23. Most frequent intraoral site of occurrence of tubercu-
a. 0.4–0.6 ppm lous lesion is
b. 0.9–1.0 ppm a. Gingiva
c. 0.8–1.2 ppm b. Buccal mucosa
d. None of the above c. Palate
d. Tongue
16. Scarlet fever predominantly affects
a. Infants 24. Diagnostic test for sarcoidosis is
b. Children a. Kveim–Siltzbach test
c. Adults b. Dick’s test
d. Immunocompromised individuals c. Schick’s test
d. Gordon’s biological test
17. Diphtheria commonly spreads by
a. Saliva 25. Most commonly occurring cranial nerve palsy in cephalic
b. Respiratory droplets tetanus is
c. Sexual contact a. VII
d. Rodents b. V
c. IX
18. The Corynebacterium diphtheria toxins cause d. XI
a. Oedema and hyperaemia
b. Epithelial necrosis and inflammation 26. Most common cause of pulpitis is
c. Pseudomembrane formation a. Trauma
d. All the above b. Chemical irritation
c. Dental caries
19. Diphtheria causes which of the following? d. Thermal changes
a. Bull neck
b. Temporary paralysis of soft palate 27. As compared to nonimmune granulomas, immune
c. Diphtheria croup granulomas contain greater number of
d. All the above a. Giant cells
b. Macrophages
20. Mycobacterium tuberculosis is a c. Fibroblasts
a. Facultative intracellular bacteria d. Plasma cells
b. Facultative intercellular bacteria
c. Virus
d. Fungus

13. c 14. a 15. b 16. b 17. b 18. d 19. d 20. a 21. a 22. b 23. d 24. a 25. a 26. c 27. d
Section | IV Multiple Choice Questions 661

28. Majority of cells present in periapical granuloma are 35. Microorganism most commonly causing urinary
a. Macrophages infection is
b. Fibroblasts a. Staphylococcus albus
c. Lymphocytes b. Streptococcus viridans
d. Plasma cells c. Staphylococcus aureus
d. Escherichia coli
29. ‘Cholesterol slits’ may be found in histological exami-
nation of 36. The most cariogenic carbohydrate is
a. Primordial cyst a. Starch
b. Dentigerous cyst b. Sucrose
c. Periapical cyst c. Glucose
d. Gingival cyst of newborn d. Glycogen

30. Which of the following is the most common site of 37. A debilitated patient on oral penicillin has white le-
occurrence of Garre’s osteomyelitis? sions that can be stripped away from a tongue, leaving
a. Mandible a raw surface. The patient probably has
b. Tibia a. Candidiasis
c. Femur b. Lichen planus
d. Sternum c. Histoplasmosis
d. Mucosal dysplasia
31. In cases of cellulitis, the organism most commonly
implicated as causative agent is 38. Chronic hyperplastic pulpitis is essentially
a. Staphylococcus sp. a. Necrotizing lesion
b. Streptococcus sp. b. Suppurative lesion
c. Neisseria sp. c. Proliferation of inflamed pulp tissue
d. Pneumococcus sp. d. Very sensitive, painful lesion

32. Prevertebral fascia extends inferiorly to 39. Which of the following lesions is the most frequent
a. Posterior mediastinum cause of resorption at the root apex?
b. Anterior mediastinum a. Cementoma
c. Superior mediastinum b. Fibrous dysplasia
d. Inferior mediastinum c. Primordial cyst
d. Periapical granuloma
33. Ludwig’s angina is characterized by all of the following,
except 40. All of the following statements about thrush are true,
a. Dysphagia and dyspnoea except
b. Origin in mandibular molars a. It is caused by a gram-negative bacteria
c. Subcutaneous emphysema b. It is characterized by a scrapable white plaque
d. Bilateral involvement of submaxillary, sublingual c. It is a complication of immunosuppression or
and submental spaces systemic disease
d. It can affect neonates in an epidemic fashion
34. Which of the following dental procedures may cause
bacteraemia? 41. Luetic glossitis is an intraoral manifestation caused by
a. Cavity preparation a. Mycobacterium tuberculosis
b. Extraction b. Actinomycosis
c. Amalgam condensation c. Treponema pallidum
d. Bleaching d. Streptococcus

28. b 29. c 30. b 31. b 32. a 33. c 34. b 35. d 36. b 37. a 38. c 39. d 40. a 41. c
662 Quick Review Series for BDS 3rd Year

42. Id reaction is usually associated with 49. Which of the following is the suggested method for
a. Aphthous ulcer control of dental caries?
b. Herpetic stomatitis a. Chemical measures
c. Candidiasis b. Nutritional measures
d. Syphilis c. Mechanical measures
d. All of the above
43. Which of the following is the complication of Vincent’s
angina? 50. Physiologic wearing away of tooth as a result of tooth–
a. Actinomycosis tooth contact is defined as
b. Noma a. Attrition
c. Systemic candidiasis b. Abrasion
d. Cellulites c. Erosion
d. Any of the above
44. Synonym of Hansen’s disease is
a. Leprosy 51. ‘Dead tracts’ in ground section of teeth are manifested as
b. Tuberculosis a. White zone in transmitted light and black zone in
c. Diabetes reflected light
d. Lichen planus b. Black zone in transmitted light and white zone in
reflected light
45. Strawberry tongue and stomatitis scarlatina with dif- c. White zone in both transmitted and reflected lights
fuse, bright scarlet skin rashes is seen in d. Black zone in both transmitted and reflected lights
a. Diphtheria
b. Tularaemia 52. Early plaque has a preponderance of
c. Scarlet fever a. Cocci forms
d. None of the above b. Anaerobes
c. Fusobacterium
46. Which of the following is not a DNA virus? d. Spirochetes
a. Herpesvirus
b. Poxvirus 53. Foci of calcified tissues that lie freely in periodontal
c. Picornavirus ligament space are called
d. Parvovirus a. Denticles
b. Diffuse calcifications
47. Which of the following features frequently distinguishes c. Cementicles
herpes zoster from other vesiculobullous eruptions? d. Osteocementum
a. Unilateral occurrence
b. Severe burning pain 54. The ‘acidogenic theory of dental caries’ was proposed by
c. Seasonal pattern of development a. Willoughby D Miller
d. None of the above b. Gottlieb and Gottlieb
c. Schultz
48. On a radiograph, which of the following regions in the d. GV Black
crown of a tooth is often mistaken for caries?
a. Pulp horn 55. The most cariogenic carbohydrate is
b. Marginal ridge a. Sucrose
c. Cementoenamel junction b. Glucose
d. Secondary dentin c. Fructose
d. Starch

42. c 43. b 44. a 45. c 46. c 47. a 48. c 49. d 50. a 51. b 52. a 53. c 54. a 55. a
Section | IV Multiple Choice Questions 663

56. Examination of a 3.5-year-old boy reveals a fracture of 63. Which of the following diseases may produce radio­
his right leg, blue sclera and peculiarly shaped head. paque images?
Opalescent dentin is found in majority of his primary a. Osteomalacia
teeth. Most probable diagnosis is b. Cherubism
a. Osteopetrosis c. Osteopetrosis
b. Osteitis deformans d. Ameloblastoma
c. Osteogenesis imperfecta
d. Infantile cortical hyperostosis 64. Monostotic fibrous dysplasia is histologically most
similar to
57. In Down’s syndrome patients, the most common type a. Giant cell granuloma
of leukaemia to occur is b. Aneurysmal bone cyst
a. Chronic lymphocytic leukaemia c. Paget’s disease of bone
b. Chronic monocytic leukaemia d. Ossifying fibroma
c. Acute lymphocytic leukaemia
d. Acute monocytic leukaemia 65. Most common cause of TMJ ankylosis is
a. Rheumatoid arthritis
58. Type of malocclusion seen in achondroplasia is mainly b. Trauma
due to c. Osteoarthritis
a. Retruded maxilla d. Infection
b. Prognathic mandible
c. Retruded maxilla with prognathic mandible 66. Pernicious anaemia occurs due to the deficiency of
d. Maxillary and mandibular prognathism a. Intrinsic factor
b. Vitamin B12 (extrinsic factor)
59. Increased incidence of osteosarcoma is expected in c. Folic acid
bone affected by d. Iron
a. Cherubism
b. Paget’s disease 67. In microcytic hypochromic anaemia, there is
c. Osteopetrosis a. Reduced MCV and MCH, normal MCHC
d. Osteogenesis imperfecta b. Reduced MCV, MCH and MCHC
c. Normal MCV and MCH, reduced MCHC
60. Elevated serum alkaline phosphatase levels are seen in d. Increased MCI, decreased MCH and normal MCHC
a. Paget’s disease
b. Caffey’s disease 68. Howell–Jolly bodies are seen in
c. Severe polyostotic fibrous dysplasia a. Malaria
d. Any of the above b. Pernicious anaemia
c. Iron deficiency anaemia
61. Treatment of choice for cherubism is d. Leukaemia
a. Surgical recontouring
b. Radiation therapy 69. Which of the following blood dyscrasias has racial
c. Block excision predilection?
d. No treatment a. Aplastic anaemia
b. Thalassaemia
62. The most common cause of referred pain to TMJ c. Sickle cell anaemia
region is d. Cooley’s anaemia
a. Impacted third molars
b. Maxillary sinusitis
c. Otitis media
d. Infratemporal cellulitis

56. c 57. c 58. a 59. b 60. d 61. d 62. a 63. c 64. d 65. b 66. a 67. b 68. b 69. c
664 Quick Review Series for BDS 3rd Year

70. ‘Crew cut’ or ‘hair-on-end’ appearance of bone in skull 77. ‘Glossopyrosis’ refers to
radiographs is seen in a. Painful tongue
a. Sickle cell anaemia b. Burning tongue
b. Thalassaemia c. Paralysis of tongue
c. Congenital haemolytic jaundice d. Atrophy of tongue
d. Any of the above
78. A middle-aged female patient complains of an apparent
71. Enamel hypoplasia producing characteristic ‘Rh hump’ facial paralysis involving left side of face, which mani-
is seen in fested only a day before. Most likely diagnosis is
a. Polycythaemia vera a. Tic douloureux
b. Thalassaemia b. Bell’s palsy
c. Erythroblastosis fetalis c. Myasthenia gravis
d. Plummer–Vinson syndrome d. Any of the above

72. Which of the following groups of teeth shows least 79. Which of the following neuralgias is characterized by
caries incidence? ‘trigger zone’ in the posterior oropharynx or tonsillar
a. Maxillary and mandibular first molars fossa?
b. Maxillary second premolars a. Glossopharyngeal neuralgia
c. Mandibular central and lateral incisors b. Sphenopalatine neuralgia
d. Maxillary central and lateral incisors c. Trigeminal neuralgia
d. Post-herpetic neuralgia
73. In infectious mononucleosis, an early oral manifes-
tation is 80. Dysphagia, sore throat and pharyngeal pain, which oc-
a. Palatal petechiae curs due to elongated styloid process exerting pressure
b. Oral ulceration on pharyngeal wall is known as
c. Inflammation of mucous membrane a. Trotter’s syndrome
d. Oedema of soft palate and uvula b. Eagle syndrome
c. Glossopharyngeal neuralgia
74. Most common cyst in oral region is d. Post-herpetic neuralgia
a. Medial cyst
b. Radicular cyst 81. Numerous large foamy, wrinkled cytoplasm, slightly
c. Follicular cyst granular with small round pyknotic nuclei in bone mar-
d. Nasolabial cyst row is
a. Gaucher cell
75. Which of the following neuralgias is characterized b. Gargoyle cell
by paroxysmal disturbance of fifth cranial nerve and c. Niemann–Pick cell
presence of ‘trigger zone’? d. None of the above
a. Tic douloureux
b. Glossopharyngeal neuralgia 82. Drug of choice for treatment of myasthenia gravis is
c. Sphenopalatine neuralgia a. Acetylcholine
d. Trigeminal neuritis b. Edrophonium
c. Physostigmine
76. In post-herpetic neuralgia, which of the following d. Noradrenaline
divisions of trigeminal nerve is usually involved?
a. Ophthalmic 83. Which of the following may lead to anhydrosis of
b. Maxillary homolateral side of face?
c. Mandibular a. Facial nerve trauma
d. All of the above b. Frey’s syndrome
c. Cervical sympathetic paralysis
d. Trigeminal neuralgia

70. d 71. c 72. c 73. a 74. b 75. a 76. a 77. d 78. b 79. a 80. b 81. a 82. c 83. c
Section | IV Multiple Choice Questions 665

84. Dry socket is a form of 91. Which of the following syndromes is characterized by
a. Osteomyelitis early synostosis of sutures, hypoplasia of maxilla and
b. Osteitis patient nose resembling ‘parrot beak’?
c. Osteoma a. Treacher-Collins syndrome
d. Periostitis b. Crouzon’s syndrome
c. Pierre Robin syndrome
85. The most common complication of rheumatoid arthritis d. Marie–Sainton syndrome
involving the temporomandibular joint is
a. Subluxation 92. Which of the following diseases of the bone character-
b. Fibrous ankylosis istically exhibits a single lesion in a single bone in
c. Osteoma of the condyle contrast to the others?
d. Resorption of the condyle a. Paget’s disease of bone
b. Central giant cell granuloma
86. The usual radiographic appearance of an osteosarcoma is c. Hyperparathyroidism
a. Discrete radiolucency with regular borders d. Osteopetrosis
b. Multicystic radiolucency with a ‘soap bubble’
appearance 93. In which disease state, do the cartilage columns lack
c. ‘Cotton wool’ appearance orderly arrangement, fail to calcify properly and are not
d. Sunburst pattern resorbed and replaced by bone in the usual fashion?
a. Osteopetrosis
87. Osteogenesis imperfecta is manifested clinically by b. Cherubism
a. Punched-out radiolucencies in the jaws c. Pierre Robin syndrome
b. Numerous unerupted supernumerary teeth d. Achondroplasia
c. Osteoporosis and a history of anaemia
d. A history of multiple fractures and a blue tint to the 94. Which of the following diseases exhibit high arched
sclera palate?
a. Marfan’s syndrome and Down’s syndrome
88. Blue sclera may present in b. Pierre Robin syndrome and Treacher-Collins syn-
a. Marfan’s syndrome drome
b. Cherubism c. Craniofacial dysostosis and cleidocranial dysplasia
c. Osteogenesis imperfecta d. All of the above
d. All of the above
95. Which of the following blood dyscrasias has a racial
89. Tumour osteoid is seen in predilection?
a. Osteosarcoma a. Purpura
b. Ewing’s sarcoma b. Leukaemia
c. Multiple myeloma c. Haemophilia
d. Paget’s disease d. Sickle cell anaemia

90. Which of the following is a psychophysiologic disor- 96. Haemophilia occurs due to
der? a. Deficiency of factor VIII
a. Myofacial pain dysfunction syndrome b. Deficiency of prothrombin
b. Marie-Strumpell disease c. Deficiency of vitamin K
c. Marfan’s syndrome d. Deficiency of vitamin C
d. Treacher-Collins syndrome

84. b 85. b 86. d 87. d 88. d 89. a 90. a 91. b 92. b 93. d 94. b 95. d 96. a
666 Quick Review Series for BDS 3rd Year

97. Monospot test is used in the diagnosis of 104. WD Miller put forth ‘the acidogenic theory of dental
a. Leukaemia caries’ in the year
b. Infectious mononucleosis a. 1890
c. Candidiasis b. 1920
d. Herpes zoster c. 1924
d. 1980
98. Leukaemia should be considered as top on a list of
differential diagnosis when the following is noted: 105. Each of the following neuralgias is related to involve-
a. Spontaneous bleeding ment of a specific nerve or ganglia. The nerve in-
b. Persistent headache volved in each disease is listed against the disease,
c. Epigastric pain which combination is incorrect?
d. Chronic or acute joint pain a. Bell’s palsy 7th nerve
b. Herpes zoster 5th nerve only
99. Which of the following disorder of blood results in c. Tic douloureux 5th nerve
intrinsic staining of teeth? d. Glossopharyngeal neuralgia 9th nerve
a. Polycythaemia vera
b. Megaloblastic anaemia 106. Which of the following conditions may cause anhi-
c. Erythroblastosis fetalis drosis of the homolateral side of the face?
d. Haemophilia a. Cervical sympathetic paralysis
b. Facial nerve trauma
100. Which of the following conditions is least likely to c. Trigeminal nerve trauma
show oral manifestations? d. Cerebral embolus
a. Psoriasis
b. Pemphigoid 107. A ‘petrified man’ is seen in which disease?
c. Behçet’s syndrome a. Dystrophic myotonia
d. Stevens–Johnson syndrome b. Myositis ossificans generalized
c. Dermatomyositis
101. The reticuloendothelial system is primarily involved in d. Proliferative myositis
a. Histoplasmosis
b. Cryptococcosis 108. In which of the following diseases, the pain is of mus-
c. Geotrichosis culoskeletal origin?
d. Sporotrichosis a. Eagle syndrome
b. Phantom pain
102. Which of the following diseases of the skin is the c. Cluster headache
most likely to exhibit partial anodontia? d. Tic douloureux
a. Pemphigus
b. Lichen planus 109. Oral diagnostic features of scleroderma include all of
c. Keratosis follicularis the following, except
d. Hereditary ectodermal dysplasia a. A hard and rigid tongue
b. Pseudoankylosis of the TM joint
103. Pseudorhagades are found in c. Large oral aperture
a. Congenital syphilis d. Difficult in swallowing
b. Acquired syphilis
c. Psoriasis 110. One of the characteristic features of osteogenesis
d. Ectodermal dysplasia imperfecta:
a. It is a sex-linked disorder of bones
b. Blue sclera
c. Deafness
d. It is associated with amelogenesis imperfecta

97. b 98. a 99. c 100. a 101. a 102. d 103. d 104. a 105. b 106. c 107. b 108. a 109. c 110. b
Section | IV Multiple Choice Questions 667

111. Which of the following is not a characteristic of con- 118. A 10-year-old child presents with anaemia and recur-
genital syphilis? rent fractures. The radiograph shows diffuse hyper-
a. Ghon complex density of bone. The diagnosis is most likely to be
b. Interstitial keratitis a. Osteogenesis imperfecta
c. Mulberry molars b. Osteopetrosis
d. Notched incisors c. Osteochondroma
d. Hyperparathyroidism
112. Luetic glossitis is an intraoral manifestation caused by
a. Mycobacterium tuberculosis 119. False about cherubism
b. Actinomycosis a. Unilocular lesion
c. Treponema pallidum b. Bilateral
d. Streptococcus c. Presence of giant cell
d. Delayed eruption of permanent teeth
113. The primary causative factor for myofacial pain dys-
function syndrome of the TMJ is 120. Trigeminal neuralgia (tic douloureux) is characterized by
a. Infratemporal space infection a. Paralysis of one side of the face
b. Auriculotemporal neuritis b. Uncontrollable twitching of muscles
c. Muscular overextension and overcontraction c. Sharp, excruciating pain of short duration
d. Otitis media d. Prolonged episodes of pain on one side of the face

114. ‘Risus sardonicus’ is associated with 121. A neuralgia with trigger zones in the oropharynx and
a. Tetany pain in the ear, pharynx, nasopharynx, tonsils and
b. Tetanus posterior portion of tongue is most likely
c. Epilepsy a. Trigeminal neuralgia
d. Polio b. Bell’s palsy
c. Glossopharyngeal neuralgia
115. Complication of rheumatoid arthritis of condyle is d. Sphenopalatine neuralgia
a. Fibrous ankylosis
b. Subluxation 122. The latest drug of the choice with minimal side effects
c. Dislocation in the management of trigeminal neuralgia is
d. None of above a. Valproic acid
b. Carbamazepine
116. Large anterior fontanels, open sutures, slanting eyes, c. Gabapentin
decreased sexual development, macroglossia and d. None of the above
enamel hypoplasia are seen in
a. Craniofacial dysostosis 123. Carbamazepine has been utilized to successfully
b. Down’s syndrome diminish attacks in trigeminal neuralgia. During this
c. Treacher-Collins syndrome therapy, which of the following is indicated?
d. Marfan’s syndrome a. Clinical observation only
b. Clinical observation and complete blood and
117. A patient with multiple impacted supernumerary platelet counts at frequent intervals during therapy
teeth, and can bring his shoulders together is said to c. Complete blood investigation only
be suffering from d. No monitoring
a. Klinefelter’s syndrome
b. Trisomy 21 124. The following site is the common involvement in
c. Down’s syndrome cases of myositis ossificans
d. Cleidocranial dysostosis a. Masseter
b. Hyoglossus
c. Stylohyoid
d. Lateral pterygoid

111. a 112. c 113. c 114. b 115. a 116. b 117. d 118. b 119. a 120. c 121. c 122. c 123. 124. c
668 Quick Review Series for BDS 3rd Year

125. A flat, circumscribed discolouration of skin or mu- 132. Pernicious anaemia is


cosa that may vary in size and shape is termed as a. Insufficient production of red cells
a. Epulis b. Improper maturation of red cells
b. Macule c. Complete absence of RBCs
c. Nodule d. Complete absence of WBCs
d. Papule
133. Bleeding time is prolonged in
126. Grinspan syndrome is associated with a. Haemophilia A
a. Hypertension, diabetes and lichen planus b. Von Willebrand’s disease
b. Oral, ocular and genital lesions c. Haemophilia B
c. Hypertension with oral lesions d. Telangiectasia
d. Pemphigus, CHF and diabetes
134. Patient giving history of thrombocytopenic purpura
127. Darier’s disease is associated with reports for extraction. What could be the most com-
a. Pernicious anaemia mon postoperative complication?
b. Rickets a. Oedema
c. Vitamin A deficiency and involvement of oral b. Haemorrhage
epithelium and skin c. Infection spreading through tissue spaces
d. Diffuse tender ulceration on the palate d. Dry socket

128. Histopathologic section of lichen planus shows 135. Megaloblastic anaemia occurs due to
a. Rushton bodies a. Iron deficiency
b. Presence of subepithelial mononuclear infiltrate b. Folate deficiency
consisting of T cells and histiocytes c. Vitamin C deficiency
c. Antiepithelial antibodies d. Protein deficiency
d. Scattered infiltrate with ill-defined lower border
136. A patient presents with history of proneness to frac-
129. Intraepithelial bullae are found in ture of bones since childhood with pale blue sclera
a. Pemphigus may be a case of
b. Bullous pemphigoid a. Osteogenesis imperfecta
c. Bullous lichen planus b. Osteoporosis
d. Pemphigoid c. Calcium deficiency
d. None of the above
130. All of the following statements about idiopathic
thrombocytopenic purpura are true, except 137. A patient with congenital cleft palate, micrognathia
a. It is associated with platelet-specific autoantibodies and glossoptosis and respiratory difficulty with bird
b. It causes a prolonged bleeding time face could be a case of
c. It is often controllable by immunosuppressive a. Pierre Robin syndrome
treatment b. Hemifacial microsomia
d. It causes more prolonged haemorrhage than c. Treacher-Collins syndrome
haemophilia d. Crouzon’s disease

131. To prevent excessive bleeding during surgery, a patient 138. In addition to four positive findings in myofacial pain
with haemophilia A may be given dysfunction syndrome, one of the typical negative
a. Whole blood feature is an absence of clinical roentgenographic or
b. Fresh frozen plasma biochemical evidence of organic changes in the joint
c. Factor VIII concentrate the other is
d. Factor IX concentrate a. Pain
b. Muscle tenderness
c. Lack of tenderness in the joint
d. Limitation of jaw movements

125. b 126. a 127. c 128. b 129. b 130. d 131. c 132. b 133. b 134. d 135. c 136. a 137. a 138. c
Section | IV Multiple Choice Questions 669

139. The cells, which form tubular appearance of AOT 146. Presence of Heberden’s node is most characteristic of
have a. Albright’s syndrome
a. Nuclei present at the cell body farther from central b. Fibrous dysplasia
space c. Rheumatoid arthritis
b. Nuclei present in cell towards the central space d. Osteoarthritis
c. Nuclei occupying the entire cell
d. Any of the above 147. The colour of normal gingiva is
a. Red
140. Canalicular adenoma histopathologically can be mis- b. Pink
diagnosed for c. Blue
a. Pleomorphic adenoma d. None of the above
b. Mucoepidermoid carcinoma
c. Adenoid cystic carcinoma 148. Odontolithiasis is
d. Plexiform ameloblastoma a. Stone in salivary gland
b. Stone in maxillary antrum
141. A necrotizing ragged mucosal ulceration with no ap- c. Calculus deposits on teeth
parent inflammatory response is indication of d. There is no such term
a. Leucocytosis
b. Agranulocytosis 149. Tobacco smoking is thought to be a predisposing
c. Sickle cell anaemia or contributing factor in which of the following
d. Infectious mononucleosis conditions?
a. Cyclic neutropenia
142. Positive Paul–Bunnell test is characteristic and b. Juvenile periodontitis
pathognomonic of c. Chediak–Higashi syndrome
a. Infectious mononucleosis d. Necrotizing ulcerative gingivitis
b. Leukaemia
c. Pernicious anaemia 150. The bacterial flora in periodontal pockets associated
d. Neutrophilia with juvenile periodontitis is predominantly
a. Gram-positive aerobic cocci
143. In von Willebrand’s disease, the laboratory findings b. Gram-negative anaerobic rods
show c. Gram-negative aerobic rods
a. Prolonged bleeding time d. Gram-positive anaerobic cocci
b. Normal platelet count
c. Normal clotting time 151. ‘Asteroid bodies’ are seen in
d. All of the above features a. Phycomycosis
b. Sporotrichosis
144. Tumours metastatic to the jaw bones are most likely c. Rhinosporidiosis
to be found in the d. Geotrichosis
a. Mandibular condyle
b. Posterior molar region 152. A bilateral grey-white lesion of the buccal mucosa
c. Anterior maxilla disappears when the mucosa is stretched. The most
d. Maxillary tuberosity likely condition is
a. Leukoedema
145. Paget’s disease, fibrous dysplasia and hyperparathy- b. Leukoplakia
roidism are most likely to provide blood chemistry c. Lichen planus
findings of an elevated d. White sponge nevus
a. Serum calcium
b. Serum alkaline phosphatase
c. Serum glutamic oxalacetic transaminase
d. Uric acid

139. a 140. c 141. b 142. a 143. d 144. b 145. b 146. d 147. b 148. c 149. d 150. b 151. b 152. a
670 Quick Review Series for BDS 3rd Year

153. Oral mucosal pigmentation is characteristic of 160. The disadvantage of acetone in tissue processing is
a. Papillon–Lefèvre syndrome a. Slow action
b. Mikulicz’s disease b. Poor penetrance
c. Sjögren’s syndrome c. Colouring
d. Addison’s disease d. Immiscible with water

154. Most common, naturally occurring chelating agent is 161. Lipids are well preserved in after fixation with
a. Amino acid a. Osmium tetroxide
b. Hydroxy and keto-esters b. Formalin
c. Citrates c. Alcohol
d. EDTA d. All of the above

155. A patient with hyperelasticity of skin, hyperextensi- 162. Salivary secretions from which of the following
bility of the joints, termed ‘rubber man’ is a case of glands have the highest amylase content?
a. Ehlers-Danlos syndrome a. Parotid gland
b. Kawasaki disease b. Submandibular gland
c. Scleroderma c. Sublingual gland
d. None of the above d. Minor salivary glands

156. Necrotizing sialometaplasia can be mistakenly diag- 163. An odontogenic abscess due to pulpal infection,
nosed as epidermoid carcinoma because of which crosses the confines of alveolar bone and
a. Pseudoepitheliomatous hyperplasia comes to lie in neighbouring tissue is
b. Necrosis a. Periapical abscess
c. Ductal squamous metaplasia b. Dentoalveolar abscess
d. All of the above c. Periodontal abscess
d. Pericoronal abscess
157. Fixation of tissues brings about a change in the
a. Physical state of RNA 164. Primary spaces of maxilla are
b. Chemical state of RNA a. Canine, buccal and infratemporal spaces
c. Physical and chemical state of RNA and DNA b. Submental, submandibular, sublingual and buccal
d. Chemical state of RNA and DNA spaces
c. Pterygomandibular, masseteric spaces
158. Baker’s formal-calcium solution is used to preserve d. None of the above
a. Carbohydrates
b. Proteins 165. Injection of the maxillary tuberosity with infected
c. Lipids needle or solution can cause
d. Nucleic acids a. Infratemporal space infection
b. Lateral pharyngeal space infection
159. Recommended ethanol percentage for delicate tissues c. Retropharyngeal space infection
in the processing stage is d. Parotid space infection
a. 30%
b. 40% 166. Most common cause of external resorption is
c. 70% a. Periapical inflammation
d. 95% b. Reimplantation of the tooth
c. Tumour and cyst
d. All of the above

153. d 154. c 155. a 156. d 157. c 158. c 159. a 160. b 161. a 162. a 163. b 164. a 165. a 166. d
Section | IV Multiple Choice Questions 671

167. In AIDS, hairy leukoplakia is found most commonly 174. The commonest site for occurrence of mucoceles is
in which location? a. Lower lip
a. Hard palate b. Upper lip
b. Buccal mucosa c. Buccal mucosa
c. Lateral border of tongue d. Tongue
d. Soft palate
175. Histologic examination of adenoid cystic carcinoma
168. Site most frequently affected by keratoacanthoma is of parotid gland reveals
a. Buccal mucosa a. Starry sky pattern
b. Gingiva b. Honeycomb pattern
c. Lips c. Swiss cheese pattern
d. Cheeks d. Either B or C

169. Pink disease is due to toxicity reaction of 176. Which of the following salivary gland lesions is usually
a. Silver compounds associated with sicca syndrome?
b. Mercury a. Adenoid cystic carcinoma
c. Lead b. Pleomorphic adenoma
d. Tetracycline c. Benign lymphoepithelial lesion
d. Acinic cell carcinoma
170. Taurodontism frequently involves
a. Incisors 177. Which of the following cysts develops in place of a
b. Canine tooth?
c. Premolars a. Primordial cyst
d. Molars b. Dentigerous cyst
c. Keratocyst
171. Most specific factor in sunlight responsible for skin d. Radicular cyst
carcinogenesis is
a. Ultraviolet radiation 178. Ameloblastoma is most likely to develop in the lining
b. Infrared radiation of
c. Visible light a. Dentigerous cyst
d. Cosmic rays b. Primordial cyst
c. Nasopalatine cyst
172. Most common tumour of salivary gland is d. Lateral periodontal cyst
a. Papillary cystadenoma lymphomatosum
b. Oxyphilic cell adenoma 179. Most frequent site of occurrence of odontogenic kera-
c. Pleomorphic adenoma tocyst is
d. Adenoid cystic carcinoma a. Mandibular third molar and ramus areas
b. Maxillary third molar area
173. Darling’s disease is c. Maxillary cuspid area
a. Histoplasmosis d. Mandible second premolar area
b. Blastomycosis
c. Candidiasis 180. ‘Ghost cells’ may also be seen in all of the following,
d. Coccidioidomycosis except
a. Ameloblastic fibro-odontoma
b. Craniopharyngioma
c. Ameloblastic adenomatoid tumour
d. Complex and compound odontomas

167. c 168. d 169. b 170. d 171. a 172. c 173. a 174. a 175. d 176. d 177. a 178. a 179. a 180. c
672 Quick Review Series for BDS 3rd Year

181. Source of epithelial cells of Pindborg tumour is 188. Botryoid odontogenic cyst is a variant of
a. Inner enamel epithelium a. Lateral periodontal cyst
b. Stellate reticulum b. Apical periodontal cyst
c. Stratum intermedium c. Gingival cysts of newborn
d. Outer enamel epithelium d. Gingival cysts of adult

182. Which of the following lesions are seen in von 189. Dentigerous cyst is likely to cause which neoplasia?
Recklinghausen’s disease of skin? a. Ameloblastoma
a. Haemangiomas b. Adenocarcinoma
b. Ameloblastomas c. Fibrosarcoma
c. Neurofibromas d. All of the above
d. Giant cell fibromas
190. Which of the following is the cause of recurrence of
183. Reed–Sternberg cells are characteristically seen in odontogenic keratocyst?
a. Alpha-thalassaemia a. Increased mitotic activity of the epithelial lining
b. Glandular fever b. Friability of the epithelial lining
c. Hansen’s disease c. Presence of satellite cysts or daughter cysts
d. Hodgkin’s disease d. All of the above

184. Frequent bouts of epistaxis is a conspicuous feature of 191. Which of the following vitamin is essential for colla-
a. Encephalotrigeminal angiomatosis gen formation?
b. Hereditary haemorrhagic telangiectasia a. Vitamin A
c. Nasopharyngeal angiofibroma b. Vitamin C
d. Vascular nevus c. Vitamin D
d. Vitamin E
185. Lead poisoning causes
a. Wickham’s striae 192. Sialoliths occur most commonly in
b. Burtonian’s line a. Submandibular glands
c. Linea alba b. Parotid glands
d. Erythroplakia c. Sublingual glands
d. Lingual glands
186. Which of the following viruses is not implicated in
human malignancies? 193. Mikulicz’s disease is
a. Epstein-Barr virus a. An inflammatory disease
b. Herpes simplex virus type 1 b. A neoplastic disease
c. HTLV-1 c. An autoimmune disease
d. Papilloma virus d. A viral infection

187. All of the following lesions may be classified as 194. A condition of the mouth, which increases the caries
odontogenic tumours, except activity in the oral cavity is
a. Acanthomatous ameloblastoma a. Xerostomia
b. Branchial cleft cyst b. Malignancy
c. Myxoma c. Hair tongue
d. Simple ameloblastoma d. Water saliva

181. c 182. c 183. d 184. b 185. b 186. b 187. b 188. a 189. a 190. d 191. b 192. a 193. c 194. a
Section | IV Multiple Choice Questions 673

195. Which of the following hormones has most pro- 198. The recurrence of pleomorphic adenoma is attributed to
nounced effect on healing of oral wounds? a. Presence of an incomplete capsule
a. Thyroxin b. Mixed origin
b. Cortisone c. Absence of capsule
c. Parathormone d. Perineural spread
d. Testosterone
199. Odontogenic keratocyst
196. Asymptomatic facial swelling is seen in a. Occurs due to periapical infection
a. Ludwig’s angina b. Is developmental in origin
b. Angioneurotic oedema c. Can be treated by aspiration
c. Bell’s palsy d. Has low recurrence rate
d. All of the above
200. Green stains on teeth which occur frequently in
197. The most common salivary gland tumour is children are due to
a. Basal cell adenoma a. Enamel deficiency
b. Mixed cell carcinoma b. Dentin deficiency
c. Pleomorphic carcinoma c. Materia alba
d. Pleomorphic adenoma d. Chromogenic bacteria

195. b 196. b 197. a 198. a 199. b 200. d


This page intentionally left blank
Section V

Previous Years’ Question Bank

Section I General Medicine 677


Section II General Surgery  694
Section III Oral Pathology 706
This page intentionally left blank
Section V

Previous Years’ Question Bank

Section I

General Medicine
1. AIMS OF MEDICINE AND CLINICAL Short Essays
METHODS 1. Glossitis. [RGUHS Apr 2001]
Short Notes 2. Peptic ulcer. [RGUHS RS Aug 2005]
3. Acute gastritis. [BUHS Sep 1992; TN Apr 1995]
1. Jugular venous pulse. [TN Feb 2005] 4. Acid peptic disease treatment. [MUHS Jul 1998]
2. Clubbing. [TN Sep 2002] 5. Causes of malabsorption syndrome. [RGUHS Apr
3. Finger clubbing. [TN Apr 2001] 2000]
6. Diarrhoea. [NTR-OR Oct 2004]
7. Dysphagia. [NTR-NR Oct 2005]
2. DISEASES OF THE GASTROINTESTINAL
8. Constipation. [NTR-NR Apr 2005]
SYSTEM 9. Oesophageal varices. [NTR-OR Oct 2004; NTR-NR
Long Essays Apr 2007]
10. Reflux oesophagitis. [TN Sep 2002, NTR-NR Apr
1. Describe pathophysiology, clinical picture, diagnosis 2004; TN Feb 2005]
and management of reflux oesophagitis. [NTR-NR Oct 11. Plummer-Vinson syndrome. [NTR-NR Feb 2002]
2005] 12. Differential diagnosis of gastric ulcers and duodenal
2. Write the clinical features of hepatic ulcers, gastric- ulcers. [NTR-NR Oct 2002]
duodenal ulcers. What are the complications? [NTR-OR 13. Hypersplenism. [TN Nov 2001]
Oct 1996] 14. Malabsorption syndrome. [TN Apr 2000, Oct 2000]
3. Describe the aetiopathogenesis, clinical features and 15. Complications of peptic ulcer. [TN Apr 2000]
treatment of peptic ulcers. [NTR-OR Feb 2002]
4. Describe clinical features, diagnosis and management
of peptic ulcer. Enumerate its complications. [NTR-OR
Short Notes
July 1990] 1. Constipation. [NTR-NR Apr 2005]
5. Write about acid-peptic disease. [RGUHS OS Aug 2. Melaena. [RGUHS Apr 2002]
2006] 3. High fibre diet. [NTR-OR Apr 2000]
6. Discuss aetiology, clinical features and management of 4. Ulcerative colitis. [NTR-OR Apr 1992]
malabsorption syndrome. [BUHS Sep 1999] 5. Intestinal nematodes. [NTR-OR Oct 1995]
7. Describe the aetiology, clinical features, diagnosis and 6. Glossitis. [RGUHS Sep 2001; TN Oct 1996]
medical management of peptic ulcers. [BUHS Apr 1997] 7. Diarrhoea. [MUHS Jul 1991]
8. How would you investigate a case of dysphagia? Name 8. Dysphagia. [MUHS Jan 1996; TN Oct 1991]
some of the conditions causing dysphagia. [BUHS Aug 9. Acute gastritis. [BUHS Sep 1992; TN Oct 1996, Apr
1995] 2000]

677
678 Quick Review Series for BDS 3rd Year

10. Barium swallow. [BUHS Mar 1992] Short Essays


11. Lactose intolerance. [BUHS Feb 1993]
1. Classify jaundice. Write about infective hepatitis.
12. Malabsorption syndrome. [RGUHS Apr 2000, Sep
[BUHS Aug 1995]
2000; TN Oct 1997]
2. Discuss the aetiology, clinical features and manage-
13. Causes of chronic diarrhoea. [RGUHS Apr 2002]
ment of jaundice. [BUHS Jul 1991]
14. Oral ulcers—enumerate four causes. [RUOS Aug
3. How will you proceed to investigate a case of jaundice
2005]
in a patient about 50 years of age? [BUHS Aug 1988;
15. H2 antagonists. [BUHS Feb 1996; MUHS Jan 1991]
MUHS Jul 1991]
16. Stomatitis. [TN Aug 2009]
4. Discuss diagnosis and management of hepatitis B.
17. Upper gastrointestinal bleeding. [TN Oct 1991, Aug 2009]
[RGUHS Sep 2000]
18. Gingival hyperplasia. [TN Feb 2009]
5. Discuss aetiology, clinical features, complications and
19. Leukoplakia. [TN Feb 2006]
prevention of hepatitis B. [RGUHS-OS Aug 2005]
20. Gastritis. [TN Aug 2007]
6. Discuss aetiology, clinical features, investigations and
21. Three medical causes of dysphagia. [RURS Feb 2007]
management of viral hepatitis E. [RGUHS-RS Feb
2007]
3. DISEASES OF THE LIVER AND BILIARY 7. Describe the causes, clinical features, laboratory inves-
SYSTEM tigations and treatment of acute viral hepatitis. [BUHS
Jul 1990, Feb 1993]
Long Essays 8. Discuss aetiology, clinical features and management of
ascites. [BUHS Aug 1988, Mar 1992]
1. Describe aetiopathogenesis, investigations and man-
9. Hepatitis A, its clinical features and brief management.
agement of hepatic coma. [NTR-NR Apr 2007]
[RGUHS Apr 2001]
2. What causes liver abscess? Describe the clinical fea-
10. Complications of cirrhosis of liver. [RGUHS Apr 2000,
tures of liver abscess and complications. [NTR-OR Apr
Apr 2002]
1996]
11. Cirrhosis of liver—complications and management.
3. Describe clinical features, complications, diagnosis
[RGUHS Apr 2003]
and management of serum hepatitis. Add a note on its
12. Viral hepatitis—investigations. [RURS Aug 2006]
prevention. [NTR-OR Oct 1997]
13. Causes and clinical features of viral hepatitis. [RGUHS
4. Describe the aetiology, clinical features and manage-
Sep 2002, Sep 2003]
ment of viral hepatitis. [NTR-OR Mar 1994]
14. Hepatitis-B—prophylaxis. [RGUHS-OS Sep 2004,
5. Describe aetiology, clinical features, complications,
Mar 2005]
diagnosis and management of serum hepatitis B. Add a
15. Chronic active hepatitis. [RUOS Mar 2006]
note on its prevention. [NTR-OR Apr 1999]
6. Classify cirrhosis of liver and discuss the clinical fea-
tures, diagnosis and complications of cirrhosis of liver.
Short Notes
[TN Aug 2009; RGUHS Mar 1997; RGUHS-OS Sep
2004, Mar 2005; RURS Mar 2006] 1. Hepatitis C. [NTR-NR Oct 2006]
7. Discuss aetiology, clinical features, complications and 2. Liver abscess. [NTR-NR Oct 2005]
treatment of cirrhosis of the liver. [TN Apr 1993, Oct 3. Prevention of viral hepatitis B. [NTR-OR Feb 2002]
2000, Feb 2009] 4. Investigations in obstructive jaundice. [NTR-NR Apr
8. Discuss the aetiopathogenesis, clinical features and com- 1998]
plications of cirrhosis of liver. [TN Aug 2004, Aug 2007] 5. Helicobacter pylori. [TN Aug 2004]
9. Classify jaundice. Describe the aetiology, clinical fea- 6. Ascites. [RGUHS-RS Aug 2005; NTR-OR Oct 1991;
tures, diagnosis, complications and its management. TN Aug 2009]
[TN Feb 2007] 7. Jaundice. [MUHS Jan 1997, Jan 1999; NTR-OR Oct
10. Discuss the clinical features of cirrhosis of liver. Men- 1999, Oct 2003]
tion the dental implications of the same. [TN Feb 2006] 8. Gamma globulin. [NTR-OR Oct 2003]
11. Define and describe the clinical features of ascites. 9. Hepatitis vaccine. [NTR-OR Apr 2000]
Describe the pathogenesis and treatment of cirrhotic 10. Prevention of hepatitis B. [NTR-NR Oct 2005; TN Apr
ascites. [TN Apr 2001] 2001]
12. Enumerate the viruses causing acute hepatitis. De- 11. Prevention of viral hepatitis. [NTR-OR Oct 2001]
scribe the clinical features and complications of viral 12. Spider naevi. [MUHS Feb 1993
hepatitis B. Add notes on the dental significance. [TN 13. Jaundice—three causes. [RURS Aug 2007]
Apr 1993, Apr 1995, Oct 1996, Oct 1997, Nov 2001] 14. Jaundice—name four causes. [RGUHS-OS Aug 2005]
Section | V Previous Years’ Question Bank 679

15. Obstructive jaundice. [BUHS Mar 1985; TN Apr 1995] 7. After dental extraction patient has profuse bleeding
16. Importance of jaundice to dental surgeon. [BUHS Jul how you do manage immediately? How do you further
1999] investigate and manage? [NTR-OR Apr 2001]
17. Causes of ascites. [RGUHS Apr 2001] 8. Classify the coagulation disorders and discuss the
18. Ascites—name four causes. [RUOS Sep 2004, Mar aetiology, clinical features, diagnosis and treatment of
2005, Aug 2005] haemophilia. [TN Aug 2008]
19. Complications of cirrhosis of liver. [RGUHS Apr 2000] 9. Describe the mechanism of coagulation. Give an ac-
20. Three complications of cirrhosis of liver. [RGUHS-RS count of haemophilia. Explain the dental considerations.
Feb 2007] [TN Feb 2007]
21. Serum hepatitis. [BUHS Aug 1993, Feb 1996] 10. Describe the aetiological factors, clinical features and
22. Prevention of hepatitis B. [RGUHS-RS Aug 2005] management of megaloblastic anaemia. [BUHS Jul
23. Prevention of hepatitis A infection. [BUHS Jan 1990] 1991]
24. Hepatomegaly. [BUHS Sep 1992, Mar 1995; RGUHS 11. Describe the differential diagnosis of megaloblastic
Apr 2000; TN Oct 2000] anaemia. Add a note on treatment of pernicious anae-
25. Hepatomegaly—any three causes. [RURS Aug 2007] mia. [MUHS Sep 1991]
26. Tender hepatomegaly. [BUHS Sep 1992, Mar 1995; 12. Mention causes of aplastic anaemia. Describe its clinical
TN Apr 1995, Oct 1996] features, diagnosis, complications and management.
27. Tender hepatomegaly—two common causes. [RUOS [BUHS Jan 1988]
Sep 2004; RGUHS-OS Mar 2005] 13. Describe the aetiological factors, clinical features
28. Enumerate causes of tender hepatomegaly. [RGUHS and management of polycythaemia vera. [BUHS
Apr 2000, Apr 2002] Sep 1992]
29. Four causes of splenomegaly. [RGUHS-OS Aug 2006] 14. Classify leukaemias. Discuss the clinical features and
30. Hepatic encephalopathy. [MUHS Jul 1995] management of one of them. [BUHS Aug 1996]
31. Portal hypertension. [MUHS Jul 2001] 15. Classify leukaemias. Outline the clinical features and
32. Portal hypertension—clinical features. [RGUHS Sep diagnostic tests of chronic myeloid leukaemia. [MUHS
2003] Jan 1990]
33. Three causes of congenital hyperbilirubinaemia. 16. Describe the aetiology, clinical features and manage-
[RURS Feb 2007] ment of chronic myeloid leukaemia. [MUHS Jan 1990,
34. Tender liver. [TN Aug 2004] Jan 1999; TN Apr 1995]
35. Hepatitis B. [TN Aug 2008] 17. Mention various types of diagnostic criteria, complica-
36. Splenomegaly. [TN Oct 1997, Feb 2005, Aug 2007] tions of leukaemia. Outline the significance of the
system disorder in dental practice. [BUHS Jan 1996]
18. Describe the aetiological factors, clinical features and
4. DISEASES OF BLOOD management of eosinophilia. [BUHS Jul 1991]
19. Describe oral manifestations of haematological disor-
Long Essays ders. How would you treat a case of agranulocytosis?
1. Describe the aetiological factors, clinical features and [BUHS Jan 1990; MUHS Jul 1990]
management of iron deficiency anaemia. [MUHS Jul 20. How will you investigate a case of bleeding diatheses.
1998, Jul 1999; RUOS Sep 2004, Mar 2005] Mention some of the bleeding diseases and dental
2. Classify anaemia and their diagnostic approach. [NTR- considerations. [BUHS Sep 1994]
OR Oct 2004] 21. Classify bleeding and coagulation disorders. Write
3. Outline the causes of iron deficiency anaemia and how clinical features, diagnosis, complications and manage-
do you manage such a case. [NTR-OR Oct 1991] ment of haemophilia. [RURS Aug 2005; TN Apr 2001,
4. Classify anaemia. Describe clinical features, diagnosis Apr 2005]
and management of iron deficiency anaemia. [BUHS
Feb 1996; RGUHS Sep 1999, Apr 2001; NTR-OR Apr
Short Essays
1999; TN Apr 2000]
5. Enumerate causes of iron deficiency anaemia. Describe 1. Eosinophilia—causes and treatment. [RGUHS-OS Sep
its clinical features and findings in the peripheral blood 2004, Mar 2005]
smear of this condition and treatment. [NTR-OR July 2. Tropical eosinophilia. [RUOS Mar 2006; TN Apr 1996]
1990] 3. Haemophilia. [RGUHS Apr 2002; TN Feb 2009]
6. Describe the aetiopathogenesis, clinical features, inves- 4. Christmas disease. [RGUHS Sep 1999]
tigation and management of haemophilia B. [NTR-NR 5. Iron deficiency anaemia—investigations and treatment.
Apr 2003] [RGUHS-RS Aug 2006]
680 Quick Review Series for BDS 3rd Year

6. Iron deficiency anaemia. [RGUHS-RS Feb 2007; TN 20. Treatment of haemophilia. [RGUHS Apr 2003]
Aug 2009] 21. Importance of haemophilia to dental surgeon. [MUHS
7. Three signs of iron deficiency anaemia. [RGUHS-OS Jan 1999]
Aug 2006] 22. Eosinophilia. [RGUHS Sep 1999]
8. Plummer–Vinson syndrome. [RURS Mar 2006] 23. Two drugs used in eosinophilia. [RGUHS Apr 2003]
9. Aplastic anaemia—clinical features. [RURS Aug 2006] 24. Three causes of neutropenia. [RURS Mar 2006]
10. Megaloblastic anaemia. [RURS Mar 2006] 25. Aplastic anaemia. [MUHS Nov 1986; BUHS Sep
11. Discuss an approach to bleeding disorders. [NTR-NR 1994; TN Apr 1996]
Oct 2007] 26. Four causes of anaemia. [RUOS Mar 2006]
12. Leukaemia—clinical features and investigations. 27. Causes of iron deficiency anaemia. [MUHS Jul 2001]
[RGUHS-OS Aug 2005] 28. Iron deficiency anaemia—four common causes.
13. Clinical features and management of acute myeloid [RGUHS Apr 2002; RUOS Aug 2005]
leukaemia. [RGUHS Sep 2002, Apr 2003] 29. Plummer-Vinson syndrome. [BUHS Jan 1991]
14. Clinical features and management of chronic myeloid 30. Megaloblastic anaemia. [BUHS Mar 1995; MUHS Jan
leukaemia. [RGUHS Apr 2002] 1999; RGUHS Sep 2000]
15. Complications of blood transfusion. [RGUHS Apr 31. Folic acid deficiency. [RGUHS Sep 2001]
2000] 32. Classification of leukemias. [MUHS Jul 2000]
16. Oral manifestations of haematological disorders. 33. Chronic myeloid leukaemia. [BUHS Sep 1994; TN Apr
[RGUHS Sep 2000, TN Apr 2001] 1998, Oct 1999]
17. Hodgkin’s disease and non-Hodgkin’s lymphoma— 34. Hodgkin’s lymphoma. [RGUHS-OS Aug 2006]
differences. [RURS Aug 2007] 35. Agranulocytosis. [MUHS Jan 1995, Jan 1997, Jul
18. Idiopathic thrombocytopenic purpura (ITP)—investi- 1999; TN Oct 1996, Oct 1997, Oct 2000, Apr 2001]
gations and treatment. [RURS Mar 2006, Aug 2007; 36. Agranulocytosis—four causes. [RGUHS Sep 2001; TN
TN Oct 1996] Apr 1993]
37. Splenomegaly. [BUHS Mar 1992, Aug 1995; MUHS
Jan 1996; RGUHS Sep 1999]
Short Notes 38. Causes of massive splenomegaly. [MUHS Jul 2000;
1. Purpura. [NTR-NR Apr 2006; TN Aug 2009] RGUHS Apr 2001]
2. Haemophilia. [NTR-OR Jul 1990; TN Apr 1999, Apr 39. Massive splenomegaly—any four causes. [RGUHS
1996; NTR-NR Oct 2005] Sep 2002, Sep 2003; RURS Aug 2005]
3. Hypersplenism. [NTR-NR Apr 2006] 40. Haematological disorders. [RGUHS Apr 2000]
4. Acute leukaemia. [NTR-OR Oct 2003] 41. Bleeding diathesis. [MUHS Jan 1996]
5. Thrombocytopenic purpura. [NTR-OR Oct 1996] 42. Coagulation disorders—name two. [RGUHS-OS Aug
6. Idiopathic thrombocytopenic purpura. [NTR-OR Apr 2005]
2002; TN Oct 2000, Aug 2006] 43. Haematological disorders—oral manifestations.
7. Thrombotic thrombocytopenic purpura. [NTR-NR Apr [MUHS Jul 1997; RGUHS Sep 2000; NTR-OR Oct
2004] 2004]
8. Megaloblastic anaemia. [NTR-OR Apr 1995] 44. Oral manifestations of haematological disorders—
9. Peripheral smear in iron deficiency anaemia. [NTR-OR name any four. [RGUHS Apr 2003]
Apr 2001] 45. ESR. [MUHS Jul 1990]
10. Investigations of a case of anaemia. [NTR-NR Apr 46. Blood groups. [BUHS Apr 1987; TN Oct 1991]
2004] 47. Peripheral blood study. [MUHS Jan 1997]
11. Treatment of iron deficiency anaemia. [NTR-OR Apr 48. Thrombocytopenia. [TN Feb 2009; TN Oct 1999]
2001] 49. Salient haematological and bone findings in multiple
12. Iron therapy. [NTR-OR Oct 1995] myeloma. [TN Feb 2006]
13. Bone marrow aspiration. [NTR-OR Oct 1996] 50. Clotting time. [NTR-NR Apr 2007]
14. Erythrocyte sedimentation. [NTR-OR Oct 1991] 51. DIC. [NTR-NR Oct 2007]
15. Purpura. [MUHS Jan 1991, Mar 1995, Jul 1997] 52. Folic acid. [NTR-OR Oct 1998]
16. Idiopathic thrombocytic purpura. [RGUHS Sep 2001] 53. Eosinophilia. [NTR-NR Oct 2007]
17. Importance of purpura to dental surgeon. [MUHS Jul 1999] 54. Haemophilia, aetiopathogenesis. [NTR-NR Oct 2006]
18. Chronic bronchitis—complications. [RURS Aug 2007; 55. Clinical features of haemophilia. [NTR-OR Apr 2002]
TN Aug 2004] 56. Investigation in haemophilia. [NTR-NR Oct 2002]
19. Haemophilia. [MUHS Jan 1986, Jan 1991; BUHS Aug 57. Burkitt’s lymphoma. [NTR-OR Feb 1992; NTR-NR
1993; RGUHS Sep 1999, Apr 2000; TN Aug 2009] Apr 2000]
Section | V Previous Years’ Question Bank 681

58. Drugs used in Hodgkin’s lymphoma. [NTR-NR Oct 17. Outline the causes, clinical features drug treatment and
2006] management of congestive cardiac failure. [MUHS Jan
59. Prothrombin time. [NTR-NR Oct 2006; TN Apr 1996] 1999]
60. Management of acute leukaemia. [TN Oct 2003] 18. Describe signs and symptoms of angina and how
61. Haemolytic anaemia. [TN Apr 2000] would you manage the case. [MUHS Jul 1996]
19. Describe the aetiology, clinical features, investigations
and management of mitral stenosis. [MUHS Jan 1999]
5. DISEASES OF THE CARDIOVASCULAR 20. How will you evaluate a patient of ventricular septal
SYSTEM defect before a dental procedure? [MUHS Jul 1999]
21. Describe the clinical spectrum of coronary artery dis-
Long Essays ease. Mention the investigations available and add a
1. Describe diagnosis, investigations and management of note on treatment? [MUHS Sep 1991, TN Apr 1996]
hypertension. [NTR-NR Apr 2005; TN Aug 2005] 22. Discuss in detail the clinical features, investigations,
2. Describe clinical features, complications and manage- management and prevention of angina pectoris?
ment of systemic hypertension. [NTR-OR Apr 1998] [RGUHS -OS Mar 2006]
3. Enumerate the pathogenesis, complications and man- 23. Define angina pectoris. Discuss the clinical features
agement of hypertension. [NTR-NR Apr 2003] and investigations of acute myocardial infarction.
4. Describe the aetiological factors of systemic hyperten- [RGUHS-RS Aug 2007,TN Oct 1997]
sion and its treatment in a man of 50 years of age with 24. Describe the aetiology, clinical features, diagnosis and
complication in dental extraction. [NTR-OR Apr 1993] treatment of rheumatic fever. [BUHS Aug 1995, Mar
5. Describe clinical features, complications, diagnosis 1997, Oct 1997]
and management of chronic rheumatic mitral stenosis. 25. Rheumatic fever. Write about aetiology, pathology,
[NTR-OR Apr 1998] clinical features, complications and dental consider-
6. Describe clinical profile, investigation and management ations. [BUHS Aug 1995]
of acute myocardial infarction. [NTR-NR Oct 2006] 26. Discuss in detail, clinical features, diagnosis, manage-
7. What are the symptoms and signs of cardiac failure? ment and prophylaxis of acute rheumatic fever.
How will you treat this condition? [NTR-OR Mar [RGUHS-RS Feb 2007]
1991] 27. Describe the aetiology, clinical features, diagnosis and
8. Describe the clinical features, investigations and treat- treatment of infective endocarditis. [MUHS Apr 1987;
ment of rheumatic fever? [NTR-OR Feb 2002; TN Apr BUHS Jan 1991, Sep 1994, Mar 1995, Feb 1996;
2001] RGUHS Apr 2000, Sep 2000, Mar 2004; TN Apr 2000,
9. Describe aetiology, clinical features, complications, Apr 2001]
diagnosis and management of acute myocardial infec- 28. Describe the aetiology, pathogenesis and management
tion. [NTR-OR Oct 1999] of infective endocarditis. Add a note on prophylaxis.
10. A 60-year-old patient had myocardial infarction (isch- [BUHS Jan 1991]
aemic heart disease) one year ago, he has to undergo 29. Write the aetiopathogenesis, clinical features diagnosis
total extraction of teeth. Describe the line of manage- and management of subacute bacterial endocarditis.
ment and precautions to be taken. [NTR-OR Jul 1990] [RGUHS Apr 1999; TN Apr 1993]
11. Describe the aetiopathogenesis, clinical profile and 30. Describe the causes, clinical features and management
treatment of chronic cor pulmonale. [NTR-NR Apr of subacute bacterial endocarditis. [MUHS Jul 1995;
2006; TN Aug 2004] RGUHS Sep 2001]
12. Define congestive cardiac failure, its pathophysiology, 31. Mention the causative factors in bacterial endocarditis,
aetiology, clinical assessment and management. [TN clinical features and management. Add a note on dental
Apr 1998, Sep 2002, Aug 2009] considerations. [BUHS Sep 1994]
13. Discuss the aetiology, clinical features and complica- 32. What are the causes of hypertension? Classify antihy-
tions of hypertension. [TN Aug 2007] pertensive drugs. Describe the complications of hyper-
14. Describe the aetiopathogenesis, clinical features com- tension. [BUHS Jan 1986; MUHS Jan 1990]
plications and management of systemic hypertension. 33. Discuss the clinical features, complications and man-
[TN Aug 2006] agement of essential hypertension. [RGUHS Apr 2002]
15. Discuss the aetiology and clinical features of subacute 34. Describe the manifestations, diagnostic aspects, com-
bacterial endocarditis. Add a note on the principles of plications and management of systemic hypertension.
management of the same. [TN Feb 2006] [BUHS Jan 1986]
16. Describe the aetiology, clinical features and treatment 35. Enumerate the causes, investigations and drugs in sys-
of acute left ventricular failure. [BUHS Feb 1993] temic hypertension. [MUHS Jul 1999]
682 Quick Review Series for BDS 3rd Year

36. Discuss the management of essential hypertension. 28. Complications of hypertension. [RGUHS Apr 2001]
[MUHS Jan 1999] 29. Complications and management of hypotension.
37. What are the risk factors for coronary artery disease? [RGUHS Sep 2003]
Describe the clinical features, investigations and man- 30. Hypertensive encephalopathy. [MUHS Jan 2000; TN
agement of acute myocardial infarction. [TN Aug Apr 2000]
2004] 31. Beta blockers and its indications. [RGUHS Apr 2001]
32. Digoxin. [NTR-OR Apr 1998]
33. Cyanosis. [NTR-NR Oct 2002; NTR-OR Apr 1999]
Short Essays
34. Anticoagulants. [NTR-OR Apr 2003; TN Oct 1997]
1. Pulmonary oedema. [RGUHS Sep 1999] 35. Antiplatelet drugs. [NTR-OR Oct 2004]
2. Pulmonary embolism. [BUHS Oct 1987] 36. Prophylactic treatment of rheumatic fever. [NTR-OR
3. Classify causes of cyanosis. [RGUHS Sep 2001] Oct 2004]
4. Syncope. [RURS Mar 2006; TN Oct 1997, Feb 2005] 37. Treatment of hypertensive encephalopathy. [NTR-NR
5. Syncope—definition and causes. [RUOS Aug 2005] Oct 2006]
6. Classify congenital heart disease. [RGUHS Sep 1999] 38. Treatment of primary pulmonary hypertensions. [NTR-
7. Congestive cardiac failure. [BUHS Aug 1993; MUHS NR Apr 2003]
Jan 1999]
8. Congestive cardiac failure—signs and symptoms.
[RUOS Aug 2005]
Short Notes
9. Four components of Fallot’s teratology. [RUOS Aug 1. ESR. [NTR-OR Oct 1997]
2006] 2. Digoxin. [NTR-NR 2002]
10. Infective endocarditis. [RURS Aug 2005; TN Oct 3. Aspirin. [NTR-OR Oct 1997; NTR-NR Oct 2002]
1996] 4. Syncope. [NTR-NR Oct 2007; TN Apr 1993, Apr
11. Subacute bacterial endocarditis. [RUOS Mar 2006] 1999, Feb 2005]
12. Clinical signs of subacute bacterial endocarditis. 5. Cyanosis. [NTR-OR Oct 1997, Apr 2000]
[RGUHS Apr 2001] 6. Causes for central cyanosis. [NTR-NR Oct 2007]
13. Prophylaxis against infective endocarditis. [BUHS Jan 7. Pleurisy. [NTR-OR Apr 1996; TN Apr 1996]
1990] 8. Heart failure. [NTR-OR Apr 1996]
14. Jones criteria for diagnosis of rheumatic fever. [MUHS 9. Tachycardia. [NTR-OR Oct 1996]
Jul 2000] 10. Eosinophilia. [NTR-OR Apr 1990]
15. Diagnostic criteria and management of acute rheumatic 11. Hypotension. [NTR-OR Apr 1999, Apr 2001]
fever. [RGUHS Apr 2002] 12. Cardiac arrest. [NTR-OR Jan 1992]
16. Rheumatic fever—prophylaxis. [RURS Aug 2007; TN 13. Oral anticoagulants. [NTR-OR Mar 1991]
Feb 2005] 14. Beta blockers and their uses. [NTR-NR Oct 2006]
17. Prophylaxis of acute rheumatic fever. [RGUHS Apr 15. Prevention of infective endocarditis. [NTR-NR Apr
2000; TN Oct 1997] 2005]
18. Left ventricular failure. [MUHS Jul 1999] 16. Constrictive pericarditis. [NTR-NR Apr 2004]
19. Acute left ventricular failure—clinical features and 17. Treatment of rheumatic fever. [NTR-NR Apr 2004]
treatment. [RUOS Sep 2004; RUGS Mar 2005] 18. Angina pectoris. [NTR-OR Oct 1997, Oct 1998, Oct
20. Right ventricular failure—clinical features. [RGUHS 2001; TN Oct 1991, Apr 1993, Oct 1996, Aug 2007]
Sep 2001] 19. Infective endocarditis prophylaxis. [NTR-NR Apr
21. Aortic incompetence—causes and cardinal signs. 2003; TN Apr 2000, Feb 2009]
[RGUHS Apr 2003] 20. Supraventricular bradycardia. [NTR-NR Apr 2003]
22. Angina pectoris. [RGUHS Sep 2000] 21. Treatment of deep vein thrombosis. [NTR-NR Apr
23. Angina pectoris—clinical features and treatment. 2003]
[RUOS Apr 2001, Aug 2005] 22. Rheumatic fever. [NTR-OR Apr 2003; TN Feb 2009,
24. Clinical features and complications of mitral stenosis. Apr 1999]
[RGUHS Apr 2001] 23. Infective endocarditis. [NTR-OR Oct 2003]
25. Acute myocardial infarction—treatment. [RGUHS-RS 24. Acute pulmonary oedema. [NTR-OR Apr 2000]
Apr 2006] 25. Prophylaxis for endocarditis. [NTR-OR Apr 2000]
26. Myocardial infraction—clinical features and manage- 26. Complication of infective endocarditis. [NTR-OR Oct
ment. [RGUHS Sep 2003] 2001]
27. Clinical features and management of myocardial in- 27. Treatment of cardiac failure. [NTR-OR Oct 2001]
fraction. [RGUHS Sep 2002] 28. Hypertensive retinopathy. [NTR-NR Apr 2007]
Section | V Previous Years’ Question Bank 683

29. Give list of complications of hypertension. [NTR-NR 63. Mitral stenosis—name three signs. [RGUHS-RS Aug
Oct 2007] 2006; TN Feb 2005]
30. Pericarditis. [BUHS Mar 1992] 64. Three differential diagnosis of mitral stenosis. [RURS
31. Cardiac arrest. [BUHS Nov 1986; MUHS Oct 1987, Mar 2006]
Jul 1990, Jan 1998] 65. Nitrates. [BUHS Jan 1990]
32. Jones criteria. [RGUHS Sep 2001; TN Oct 2000] 66. Antianginal medication. [BUHS Jul 1990]
33. Angina pectoris. [BUHS Oct 1987, Mar 1992; RGUHS 67. Calcium channel blockers. [RGUHS Sep 1999, Apr
Mar 1997; MUHS Jul 1997, Jul 1999] 2003; TN Nov 2001]
34. Atrial fibrillation. [BUHS Sep 1994] 68. Cardiopulmonary resuscitation. [TN Oct 2003]
35. Atrial fibrillation—three causes. [RURS Aug 2006] 69. Tetralogy of Fallot. [TN Apr 1998, Oct 1999, Nov
36. Atrial septal defect. [MUHS Jan 1990; TN Oct 1996] 2001, Aug 2004, Feb 2007]
37. Three causes of sudden death in mitral stenosis. [RURS 70. Heart sounds. [TN Feb 2005]
Feb 2007] 71. Diagnosis of rheumatic fever. [TN Oct 1996, Sep 2002]
38. Rheumatic fever. [MUHS Jul 1991, Jan 1998; RGUHS 72. Beta blockers. [TN Apr 1993, Oct 1997, Sep 2002]
Apr 2000] 73. Corrigan’s pulse. [NTR-NR Apr 2007]
39. Rheumatic carditis. [MUHS Sep 1999; TN Apr 1996] 74. Intermittent claudication. [NTR-NR Apr 2007]
40. Infective endocarditis. [BUHS Sep 1992; MUHS Jan
2000; RGUHS Sep 1999]
41. Subacute bacterial endocarditis. [RGUHS Sep 2000] 6. DISEASES OF THE RESPIRATORY SYSTEM
42. Three features of infective endocarditis. [RURS Feb
2007]
Long Essays
43. Prophylaxis against infective endocarditis. [BUHS Jan 1. Describe the causes of bronchitis and its management.
1990] [NTR-OR Apr 1990]
44. Left ventricular failure. [RGUHS Aug 1996, Sep 1999, 2. Describe clinical features and management of bron-
Apr 2000] chial asthma. [NTR-OR Oct 1991]
45. Left ventricular failure—four cardinal causes. [RGUHS 3. Describe aetiology, clinical features and management
Sep 2003] of ARDS. [NTR-OR Apr 2004]
46. Treatment of left ventricular failure. [RGUHS Apr 4. What is pneumothorax? What are the causes of pneu-
2000; TN Apr 1993] mothorax? How do you manage tension pneumotho-
47. Coronary heart disease—name four risk factors. rax? [NTR-OR Apr 1996]
[RUOS Sep 2004, Mar 2005] 5. Describe the clinical features, complication and treat-
48. Congenital heart disease. [RGUHS Sep 1999] ment of community-acquired pneumococcal pneumo-
49. Classification of congenital heart disease. [RGUHS nia. [NTR-OR Oct 1996]
Apr 2000] 6. Describe aetiopathology, clinical features, complica-
50. Components of Fallot’s tetralogy. [RURS Aug 2005] tions, diagnosis, and treatment of lobar pneumonia.
51. Complications of hypertension—name any four. [NTR-OR Oct 1997]
[RGUHS Apr 2003; TN Feb 2009] 7. Describe the clinical features of pulmonary tuberculo-
52. Portal hypertension—investigation. [RURS Aug 2007; sis; write the short-term chemotherapy. [NTR-OR May
TN Aug 2005, Aug 2008] 1994]
53. Syncope. [BUHS Mar 1988, Jul 1990; MUHS Jun 8. A 40-year-old man brought to the causality with his-
1989; RGUHS Sep 2001; TN Apr 1991] tory of hematemesis. Mention its differential diagnosis,
54. Vasovagal syncope. [BUHS Jun 1989; MUHS Jan investigation and management. [NTR-NR Oct 2002]
1990; TN Oct 1996] 9. Describe clinical features of pulmonary tuberculosis.
55. Cyanosis. [MUHS Jan 1999; RGUHS Sep 2000] Flow will you confirm this diagnosis. Write down the
56. Cyanosis—name three causes. [RURS Aug 2006] treatment. [NTR-OR Jul 1990; TN Oct 1996, Apr
57. Four causes of cyanosis. [RGUHS-OS Mar 2006] 1996]
58. Central cyanosis—name any four causes. [RGUHS 10. Define and classify pneumonias. Describe the investi-
Apr 2003] gations, complications and treatment of community-
59. Bradycardia—four common causes. [RGUHS Mar acquired pneumonias. [TN Apr 1998, Feb 2009]
2004] 11. Describe the clinical features, diagnosis complications
60. Collapsing pulse. [BUHS Jul 1991] and management of post-primary tuberculosis. [TN
61. Aortic regurgitation—name three signs. [RURS Aug Oct 2000, Aug 2006]
2007; TN Apr 1995] 12. Discuss the aetiology, clinical features and treatment of
62. Mid-diastolic murmur. [MUHS Jan 1991] bronchial obstruction. [BUHS Sep 1991]
684 Quick Review Series for BDS 3rd Year

13. Describe the aetiology, clinical features and treatment 8. Management of bronchial asthma. [RURS Aug 2005]
of chronic bronchitis. [BUHS Nov 1996; MUHS Feb 9. Haematemesis—causes and investigations. [RGUHS
1993] Sep 2003, Mar 2004]
14. Discuss the aetiology, clinical features, complications 10. Chronic bronchitis—causes and clinical features.
and management of chronic bronchitis. [RGUHS-OS [RGUHS Sep 2000]
Mar 2006] 11. Chronic bronchitis—complications. [RGUHS-RS Aug
15. Describe the aetiology, clinical features, investigations 2007]
and management of bronchial asthma. [MUHS Jul 12. Pneumonia—clinical features and management.
1995, Jul 1996] [RGUHS Sep 2004]
16. Discuss the clinical features, investigations and outline 13. Four causes of clubbing. [RGUHS-OS Aug 2006]
the treatment of bronchial asthma. [RGUHS-RS Aug 14. Clubbing. [NTR-NR Oct 2005, Apr 2007; TN Sep
2006; TN Apr 1995] 2002]
17. Describe the pathogenesis, clinical features and diag- 15. Dyspnoea. [NTR-NR Oct 2007]
nosis of lung abscess. [BUHS Aug 1993] 16. Tonsillitis. [NTR-NR Apr 2003]
18. Write aetiopathogenesis, clinical features and treat- 17. Mantoux test. [NTR-NR Oct 2002, Apr 2004]
ment of lung abscess. [RGUHS Apr 2000, Apr 2002, 18. Tuberculin test. [NTR-OR Feb 2002]
Mar 2004] 19. Haematemesis. [NTR-OR Oct 1998]
19. Describe aetiology, clinical features, investigations and 20. Aspiration pneumonia. [NTR-NR Apr 2005]
management of pulmonary eosinophilia. [MUHS Jan 21. Organism causing pneumonia. [NTR-NR Apr 2006]
1997] 22. Treatment of haemoptysis. [NTR-NR Apr 2006]
20. Describe clinical features, complications and manage- 23. CSF finding in tuberculosis of meningitis. [NTR-NR
ment of adult pulmonary tuberculosis. [BUHS Jul 1990] Apr 2003]
21. Write about pleural effusion. Clinical features and 24. Pulmonary function test. [NTR-NR Apr 2003]
treatment of tubercular pleural effusion. [RGUHS Sep 25. BCG vaccination. [NTR-NR Apr 2003]
2001] 26. Bronchial breathing. [NTR-OR Apr 2001]
22. Define pneumonias. Classification of pneumonias. De- 27. Emphysema. [TN Oct 1997]
scribe the aetiology, clinical features, investigations, 28. Widal test. [TN Apr 1995, Apr 1996, Oct 1996]
complications and management of bacterial pneumo-
nia. [MUHS Jul 2000; TN Oct 2000]
23. Classify pneumonia. Describe the clinical features,
Short Notes
complications and treatment of pneumococcal pneu- 1. Clubbing. [NTR-OR Oct 1997, Oct 1998; TN Apr
monia. [BUHS Aug 1988, Sep 1994; RGUHS Sep 1993]
1999; TN Apr 1996, Feb 2005] 2. Bronchiectasis. [NTR-OR Oct 1996; NTR-NR Apr
24. Discuss the aetiology, clinical features, diagnosis and 2004; TN Aug 2004, Aug 2008]
treatment of pneumococcal pneumonia. [MUHS Jan 3. Lobar pneumonia. [NTR-OR Apr 1999]
1991, Jul 1997] 4. Aspiration pneumonia. [NTR-OR Apr 1996; TN Aug
25. Discuss aetiology, clinical features, complications and 2005, Aug 2008]
management of pneumonia. [RUGS Aug 2005] 5. Peripheral smear in diagnosis of disease. [NTR-OR
26. Discuss the aetiology, clinical features and manage- Apr 1996]
ment of pneumonia. [BUHS Mar 1997] 6. Chronic obstructive lung disease. [NTR-OR Jan 1992]
27. Describe aetiology, clinical features, investigations, 7. Drug resistance in tuberculosis. [NTR-NR Apr 2006]
complications and management of acute lobar pneu- 8. Complication of pulmonary tuberculosis. [NTR-OR
monia. [MUHS Jan 1998] Oct 1991; TN Apr 1993]
9. Status asthmaticus. [NTR-OR Apr 1999; TN Oct 1996,
Apr 2000, Feb 2005]
Short Essays
10. Status asthmaticus—management. [NTR-OR Feb 2002]
1. Dyspnoea. [RGUHS Sep 2000] 11. Bronchial breathing. [NTR-OR Apr 1999; TN Feb
2. Lung abscess. [MUHS Sep 1991; TN Apr 2000, Apr 2006]
2001, Feb 2005] 12. Hoarseness of voice. [NTR-OR Oct 1999]
3. Rheumatic fever—prophylaxis. [RURS Aug 2007] 13. Tracheal foreign body. [NTR-OR Oct 1996]
4. Bronchodilators. [RGUHS Sep 1999] 14. Viruses improving respiration system. [NTR-OR Oct
5. Pneumococcal pneumonia. [BUHS Aug 1995] 1995]
6. Clinical signs of emphysema. [MUHS Jul 2000] 15. Directly observed therapy (DOT) on tuberculosis.
7. Acute severe asthma. [RURS Feb 2007; TN Aug 2004] [NTR-OR Apr 2000; NTR-NR Apr 2005]
Section | V Previous Years’ Question Bank 685

16. Sputum of acid-fast bacillus (AFB) stain. [NTR-OR 52. Lung cancer—three extrapulmonary manifestations.
Apr 2000] [RURS Aug 2006]
17. CT scan. [BUHS Jul 1991] 53. Complications of pneumonia. [TN Oct 1996, Oct 1997,
18. Haemoptysis. [MUHS Jul 1995; RGUHS Mar 1997] Aug 2004, Aug 2009]
19. Enumerate causes of haemoptysis. [RGUHS Apr 2001] 54. Treatment of pulmonary tuberculosis. [TN Feb 2009]
20. Haemoptysis—four common causes. [RGUHS Mar 55. Investigations for pulmonary tuberculosis. [TN Feb
2005] 2006]
21. Haematemesis. [MUHS Jul 1996] 56. Lobar pneumonia—signs. [RURS Aug 2006]
22. Clubbing. [RGUHS-RS Aug 2005] 57. Tender spleen. [RURS Feb 2007]
23. Koilonychia. [BUHS Sep 1992]
24. Finger clubbing. [BUHS Jun 1989; RGUHS Apr 2000]
25. Respiratory causes of clubbing. [RGUHS Apr 2000] 7. DISEASES OF THE NERVOUS SYSTEM
26. Clubbing—four common causes. [RGUHS Sep 2003;
RGUHS-OS Sep 2004, Mar 2005]
Long Essays
27. Two causes of clubbing. [RGUHS-OS Mar 2006] 1. Describe aetiology, clinical features and localization of
28. Dyspnoea—name four causes. [RGUHS-OS Aug infranuclear facial nerve palsy. [NTR-NR Apr 2005]
2005] 2. Classify epilepsy. Describe clinical features, diagnosis
29. Paroxysmal nocturnal dyspnoea. [BUHS Aug 1995] and management of grand mal epilepsy. [NTR-OR Apr
30. ARDS management. [RGUHS-OS Aug 2006] 1998, Oct 1999]
31. Emphysema. [MUHS Jan 1998; RGUHS-OS Mar 3. What is cerebrovascular accident? How will you man-
2006; TN Nov 2001] age a patient of hemiplegia? [NTR-OR Apr 1990]
32. Bronchial asthma. [BUHS Mar 1995, Aug 1995; 4. Discuss the aetiopathogenesis, and clinical features of
RGUHS Sep 2000; TN Feb 2007] parkinsonian disease. Outline the drugs used in its
33. Bronchiectasis—three complications. [RGUHS-RS treatment. [NTR-NR Oct 2006]
Aug 2007] 5. Discuss the various factors you consider in evaluating
34. Status asthmaticus. [BUHS Aug 1993; RGUHS Sep a patient for general anaesthesia. [NTR-OR Jan 1992]
1999; TN Oct 1997, Feb 2005] 6. Mention the causes of cerebral embolism. What are its
35. Treatment of acute severe asthma. [RGUHS-OS Aug manifestations? Describe the principles of manage-
2006] ment. [NTR-OR Mar 1991; TN Oct 1991, Oct 1996]
36. Treatment of bronchial asthma. [BUHS Feb 1996; TN 7. Describe the pathogenesis, differential diagnosis and
Oct 1996] management of coma. [NTR-NR Apr 2006]
37. Bronchial asthma—three side effects. [RGUHS-RS 8. Describe the clinical features of intracerebral haemor-
Aug 2007] rhage and the general management of a comatic pa-
38. Respiratory failure. [RGUHS Apr 2000; TN Oct 1996] tient. [NTR-OR Oct 1995]
39. Enumerate causes of respiratory failure. [RGUHS Apr 9. Enumerate the causes of headache. Describe the clini-
2000] cal features and management and prevention of mi-
40. Pneumococcal pneumonia. [BUHS Aug 1995; RGUHS graine. [NTR-OR Oct 2001]
Apr 2000] 10. Describe the aetiology, clinical features and manage-
41. Enumerate complications of pneumococcal pneumo- ment of facial palsy. [RGUHS Apr 2003]
nia. [RGUHS Apr 2000] 11. Classify epilepsy. Write about its clinical features, in-
42. Pleural effusion. [BUHS Ju1 1993; TN Apr 1993] vestigations and treatment carried out for grand mal
43. Pulmonary effusion. [BUHS Jul 1991] epilepsy. [RGUHS Sep 1999]
44. Pulmonary embolism. [BUHS Oct 1987; RGUHS Mar 12. Mention the types of epilepsy. Describe the clinical
1997] features and treatment of anyone of them. [BUHS Mar
45. Pulmonary eosinophilia. [BUHS Aug 19961; TN Oct 1988]
1998] 13. Describe the aetiology, and clinical features of meningitis.
46. Mantoux test. [BUHS Jan 1991, Sep 1992; RUOS Mar How would you proceed to establish the diagnosis?
2006; TN Oct 1997] [MUHS Jan 1996]
47. Tuberculin test. [RUOS Aug 2006] 14. What are the causes of meningitis? Describe the clini-
48. Anti-tuberculosis drugs. [BUHS Aug 1993] cal features, complications and treatment of pyogenic
49. Short-term anti-tubercular therapy. [MUHS Jan 1997] meningitis. [MUHS Sep 1999; TN Apr 1993, Nov
50. Indications of the oxygen therapy. [RURS Aug 2005] 2001, Feb 2005]
51. Two causes of dull note on percussion of the chest. 15. Describe the pathogenesis, clinical features and diagno-
[RGUHS Sep 2002] sis of pyogenic meningitis. [BUHS Jan 1990, Aug 1993]
686 Quick Review Series for BDS 3rd Year

16. Discuss the aetiology, clinical features, and investiga- 6. Sudden loss of consciousness. [NTR-OR Oct 1996]
tions of pyogenic meningitis. [MUHS Jul 2001; RURS 7. Hazards of general anaesthesia. [NTR-OR Oct 1991]
Aug 2006; TN Feb 2005] 8. Contraindications of general anaesthesia. [NTR-OR
17. Discuss the clinical features, complications and man- Oct 2003]
agement of tubercular meningitis. [RGUHS Sep 2003; 9. Tonic–clonic seizure. [NTR-OR Oct 1995]
TN Apr 1993] 10. Anaesthetic agents. [NTR-OR Oct 1995]
18. Describe the clinical features, diagnosis and treatment 11. Syncope. [NTR-OR Oct 1995, Apr 2000]
of tetanus. [BUHS Nov 1996; TN Apr 1995] 12. Causes of syncope. [NTR-OR Apr 2001]
13. Status epilepticus. [NTR-OR Apr 2001; TN Oct 1998]
14. Cerebral thrombosis. [NTR-OR Apr 2001]
Short Essays
15. Prevention of poliomyelitis. [NTR-NR Oct 2006]
1. Migraine. [RGUHS Sep 2000; RURS Aug 2005; TN 16. Bell’s palsy. [NTR-OR Apr 1990, Apr 1993, Oct 2006;
Apr 1996] TN Apr 2001, Aug 2006, Aug-2007, Feb 2009]
2. Bell’s palsy. [RGUHS Jan 1991; BUHS Feb 1993, Feb 17. Bell’s palsy management. [NTR-NR Apr 2003; TN
1996, Sep 1999; TN Oct 1999, Oct 2000, Aug 2004] Apr 1996]
3. Bell’s palsy—clinical features and management. 18. Trigeminal neuralgia. [NTR-OR Apr 2000; NTR-NR
[RGUHS Mar 2004] Oct 2002; TN Apr 1999, Apr 2001, Aug 2008, Feb
4. Facial palsy. [RGUHS Apr 2000, Sep 2001, Apr 2002] 2009]
5. Generalized tonic-clonic seizures—clinical features. 19. Clinical features of trigeminal neuralgia. [NTR-OR
[RURS Aug 2006] Apr 2001]
6. Status epilepticus—treatment. [RURS Aug 2007] 20. Three causes of vertigo. [RURS Feb 2007]
7. Anti-epileptic drugs. [RGUHS Sep 2000] 21. Hysteria. [NTR-NR Oct 1991; NTR Apr 2006]
8. Meningitis—clinical features. [RUOS Sep 2004, Mar 22. Schizophrenia. [NTR-NR Oct 2005]
2005] 23. Babinski’s sign. [BUHS Sep 1992]
9. Tuberculous meningitis. [RURS Aug 2005; RURS-1 24. Four signs of Horner’s syndrome. [RGUHS-OS Aug
Feb 2007; RURS-2 Feb 2007] 2006]
10. Clinical features and management of pyogenic menin- 25. Migraine—name two drugs for treatment. [RUOS Sep
gitis. [RGUHS Apr 2002] 2004, Mar 2005]
11. Trigeminal neuralgia—clinical features. [RURS Aug 26. Headache. [MUHS Apr 1987; RGUHS Apr 2000]
2007] 27. Tension headache. [MUHS Jan 1990; RGUHS Sep
12. Trigeminal neuralgia—clinical features and treatment. 1999; TN Oct 2000]
[RUOS Aug 2005] 28. Trigeminal neuralgia. [BUHS Nov 1986; MUHS Jan
13. Aetiology and clinical manifestations of depression. 1990, Jul 1990; RGUHS Sep 2000; RGUHS-RS Feb
[RURS Feb 2007] 2007; TN Oct 1996]
14. Wrist drop. [NTR-NR Apr 2006] 29. Facial paralysis. [BUHS Aug 1988, Feb 1993; MUHS
15. Bell’s palsy. [NTR-OR Feb 2002] Jan 1991; RGUHS Apr 2000]
16. Petit mal epilepsy. [NTR-NR Apr 2007] 30. Three features of LMN facial palsy. [RGUHS-RS Mar
17. Facial nerve palsy. [NTR-NR Oct 2007] 2006]
18. Trigeminal neuralgia. [NTR-NR Apr 2006; TN Aug 31. Bell’s palsy. [BUHS Feb 1996; TN Apr 1993, Apr
2005, Apr 1996] 1995, Feb 2009]
19. Peripheral neuropathy. [NTR-NR Oct 2005] 32. Facial pain. [BUHS Mar 1997; RGUHS Apr 2002]
20. Causes of epilepsy. [TN Feb 2005] 33. Facial pain—four causes. [RGUHS Sep 2002]
21. Hypertensive encephalopathy. [NTR-NR Apr 2007] 34. Anxiety disorder—symptoms any three. [RGUHS-RS
22. Discuss the differential diagnosis of headache. [BUHS Aug 2007]
Jun 1989] 35. Petit mal epilepsy. [BUHS Sep 1994]
36. Status epilepticus. [BUHS Aug 1993; TN Oct 2000]
37. Epilepsy—importance to dental surgeon. [MUHS Jul
Short Notes 1997]
1. Lock jaw. [NTR-OR Jul 1990; NTR-NR Apr 2005] 38. Meningitis. [BUHS Mar 1988; TN Aug 2009]
2. Migraine. [NTR-NR Oct 2005; TN Oct 1996, Oct 39. Signs of meningeal irritation. [BUHS Mar 1988]
1998, Apr 2000; BUHS Feb 1996; MUHS Jan 1998] 40. Drugs used in epilepsy. [RGUHS Apr 2002]
3. Clinical features of migraine. [NTR-NR Oct 2002] 41. Four drugs used in epilepsy. [RGUHS Sep 2002]
4. Evaluation of coma. [NTR-NR Apr 2003] 42. Dilantin sodium. [BUHS Jul 1990]
5. Transient ischaemic attack. [NTR-NR Apr 2005, Apr 43. Antiepileptics. [TN Aug 2009]
2006] 44. TB meningitis. [TN Feb 2007]
Section | V Previous Years’ Question Bank 687

45. Migraine neuralgia (cluster headache). [TN Feb 2005] 8. Nephrotic syndrome. [RGUHS-RS Feb 2007; TN Apr
46. Phenytoin sodium. [TN Oct 1996] 1996]
9. Nephrotic syndrome—causes and clinical features.
[RGUHS Sep 2003, Apr 2005; RUGS Sep 2004]
8. DISEASES OF THE KIDNEYS
10. Uraemia. [NTR-NR Oct 2007]
AND GENITOURINARY SYSTEM 11. Proteinuria. [NTR-OR Apr 1999]
Long Essays 12. Diuretics. [TN Apr 1995]
13. Management of renal colic. [NTR-NR Apr 2004]
1. Write about pathogenesis and treatment of acute glo-
merulonephritis. [NTR-NR Apr 2004]
2. Mention aetiology, clinical features, complications, Short Notes
diagnosis and management of acute glomerulonephri- 1. Frusemide. [NTR-OR Oct 2003; TN Feb 2009]
tis. [NTR-OR Oct 1998] 2. Renal colic. [NTR-OR Apr 2000]
3. Young person develops sudden facial puffiness. How 3. Haematuria. [NTR-OR Oct 1997; TN Apr 1998, Aug 2007]
do you investigate a case of acute glomerulonephritis 4. Trace elements. [NTR-OR Apr 1995]
outline complications and treatment. [NTR-OR Apr 5. Nephrotic syndrome. [NTR-OR Feb 2002; TN Apr
2000] 2000, Aug 2009; BUHS Mar 1995; RGUHS Apr 2000]
4. Describe the clinical features, diagnosis and manage- 6. Acute glomerulonephritis. [NTR-OR Mar 1994; TN
ment of nephrotic syndrome. [TN Feb 2007] Feb 2005, Aug 2004]
5. Define nephrotic syndrome. Describe the clinical fea- 7. Uraemia. [MUHS Jul 1996; TN Apr 1996]
tures, investigations and treatment of nephrotic syn- 8. Acute nephritis. [MUHS Jun 1989; BUHS Aug 1993;
drome. [MUHS Jun 1989; RGUH Sep 1999; TN Oct TN Apr 2001]
1997] 9. Acute nephritis—three causes. [RGUHS-RS Aug 2007]
6. Define acute nephrotic syndrome. Discuss the clinical 10. Haematuria—causes. [RGUHS Sep 2001]
features and treatment of post-streptococcal acute glo- 11. Four causes of haematuria. [RUGS Mar 2006]
merulonephritis. [BUHS Jul 1990; TN Oct 1997, Oct 12. Complications of acute nephritis. [MUHS Jun 1989]
1999] 13. Acute renal failure. [BUHS Sep 1994]
7. Describe the aetiology, clinical features, investigations 14. Chronic renal failure—three laboratory abnormalities.
and management of nephrotic syndrome. [MUHS Jul [RGUHS-RS Aug 2007]
1995, Jan 1997; TN Oct 1996] 15. Treatment of CRF. [RGUHS-OS Aug 2006]
8. Mention the causes of nephrotic syndrome. Describe 16. Diabetic nephropathy. [BUHS Aug 1996; RGUHS Sep
its clinical features, diagnosis and treatment. [BUHS 1999]
Nov 1986] 17. Acute tubular necrosis. [BUHS Mar 1988; RGUHS
9. What are the causes and management of glomerulone- Sep 1999]
phritis. [RGUHS Sep 2001] 18. Define nephrotic syndrome. [RGUHS Apr 2000]
10. Describe the aetiology, clinical features, complica- 19. Clinical pictures and diagnosis of nephrotic syndrome.
tions, investigations and management of acute nephri- [BUHS Feb 1996]
tis. [MUHS Jan 1999] 20. Clinical features of acute glomerulonephritis. [RGUHS
Apr 2000; TN Aug 2007]
21. Complications and management of acute glomerulone-
Short Essays phritis. [TN Oct 2000]
1. Acute glomerulonephritis. [RGUHS-RS Aug 2005; TN 22. Importance of urine analysis. [MUHS Jul 1999]
Aug 2004] 23. Drugs causing kidney damage. [NTR-OR Oct 1996]
2. Causes and clinical features of acute nephritis. [RGUHS
Sep 2002]
3. Acute nephritis—clinical features and investigations. 9. DISTURBANCES IN WATER,
[RGUHS-RS Aug 2006] ELECTROLYTE AND ACID–BASE BALANCE
4. Acute nephritis—clinical features and management.
Short Essays
[RGUHS Mar 2004]
5. Acute nephritis—investigations and management. 1. Dehydration. [NTR-NR Oct 2007]
[RGUHS Apr 2003] 2. Hypokalaemia. [NTR-NR Oct 2007]
6. Clinical manifestations and treatment of acute glo- 3. Respiratory alkalosis. [NTR-NR Oct 2007]
merulonephritis. [RGUHS Apr 2000] 4. Hyperkalaemia—causes. [NTR-NR Oct 2006]
7. Clinical features and treatment of acute post-streptococcal 5. Diuretics. [BUHS Aug 1995]
glomerulonephritis. [RGUHS Feb 2000] 6. Frusemide—two indications. [RGUHS Apr 2003]
688 Quick Review Series for BDS 3rd Year

Short Notes 16. Describe the aetiology, clinical features, investigations


and management of hypothyroidism. [MUHS Jan
1. Hyponatraemia. [NTR-NR Apr 2001]
1990, Jul 1996; RGUHS Apr 2001]
2. Metabolic acidosis. [NTR-NR Apr 2003; TN Oct 1991]
17. What are the causes of hypocalcaemia? Describe the
3. Oral dehydration. [MUHS Jul 1997]
clinical features and outline the treatment of tetany.
[BUHS Jun 1999]
18. Define and classify diabetes mellitus. Mention the
10. ENDOCRINE AND METABOLIC common complications. [BUHS Sep 1994]
DISEASES 19. Discuss the aetiology, clinical feature and management
of diabetes mellitus. [BUHS Jan 1991, Sep 1992, Sep
Long Essays
1994; RGUHS Sep 2000]
1. Describe aetiology, clinical features and management 20. Classify diabetes mellitus. Mention acute complica-
of tetanus. [NTR-NR Oct 2006; TN Oct 1998] tions and management of the same in a diabetic patient.
2. Describe the aetiology and clinical features of acro- [RGUHS Sep 2002]
megaly with special experience to dental problems. 21. Classify diabetes mellitus. Discuss the treatment of a
[NTR-OR Apr 1993] middle-aged obese diabetic patient. [MUHS Jan 1991,
3. Enumerate the proliferating factors, pathogenesis, in- Feb 2000]
vestigations and management of diabetic ketoacidosis. 22. What are the two types of diabetes mellitus? Describe
[NTR-NR Apr 2004; TN Oct 1997] the clinical features, complications and treatment of
4. Classify diabetes mellitus. Discuss the complications any one. [BUHS Oct 1987]
and management of diabetes mellitus. [NTR-OR Oct 23. Distinguish between the insulin-dependent and non-
2001; TN Apr 2000] insulin-dependent diabetes mellitus. Enumerate the
5. What are the different types of diabetes mellitus? complications of diabetes mellitus. Add a note on pre-
Mention the common symptoms of each type. What cautions during surgery. [BUHS Feb 1993]
are the precautions you take while carrying out a 24. Discuss the management of diabetes mellitus. [MUHS
tooth extraction in a diabetic patient? [NTR-OR Mar Jul 1997]
1991] 25. What are the diagnostic criteria for type 2 diabetes
6. Discuss the complications of systemic corticosteroid mellitus? Mention the acute and chronic complica-
therapy with special reference to dental practice. [NTR- tions. Discuss the management of type 2 diabetes mel-
OR Oct 2003; TN Aug 1996] litus. [RURS Aug 2005; TN Oct 1999]
7. Outline the conditions which produce pigmentation of 26. Mention the complications of diabetes mellitus. Out-
oral mucosa. Describe the clinical features of Addi- line the principles of management of type 1 diabetes
son’s disease. [NTR-OR Oct 2003] mellitus. [MUHS Jan 1999]
8. Define hypo- and hyperthyroidism. Discuss clinical 27. Describe the clinical features and management of dia-
features, investigations and treatment of hypothyroid- betic ketoacidosis. [MUHS Jul 1998]
ism. [TN Aug 2009] 28. Discuss the aetiology, clinical features, diagnosis and
9. Discuss the aetiopathogenesis, clinical features and treatment of diabetes mellitus. [TN Oct 1991, Aug
complications of diabetes mellitus. Outline the man- 2008, Feb 2009]
agement. [TN Aug 2006]
10. Discuss the aetiology, clinical features and manage-
ment of thyrotoxicosis. [BUHS Jul 1991]
Short Essays
11. Describe the clinical features and treatment of thyro- 1. Tetany. [MUHS Jan 2000; RGUHS Sep 2001]
toxicosis. Mention a note on thyroid function tests. 2. Calcium homeostasis. [MUHS Jan 1997]
[BUHS Aug 1988] 3. Hypoglycaemia—clinical features and management.
12. Mention the clinical features of thyrotoxicosis. Discuss [BUHS Aug 1988, Jan 1996; RGUHS Mar 2004]
the modes of treatment available. [MUHS Jul 1999] 4. Management of type 2 diabetes. [TN Aug 2004]
13. Discuss the aetiology, clinical features and manage- 5. Hypoglycaemic coma. [BUHS Jan 1991]
ment of myxoedema. [BUHS Mar 1992, Aug 1995] 6. Frequent hypoglycaemia in a nondiabetic individual.
14. Myxoedema—describe the clinical features, compli- [RURS Mar 2006]
cations, treatment and dental considerations. [BUHS 7. Diabetic ketoacidosis. [BUHS Mar 1995; RGUHS Apr
Aug 1995] 2002; TN Apr 2000, Feb 2005]
15. Describe the aetiology, clinical features, diagnosis and 8. Diabetic ketoacidotic coma. [MUHS Sep 1990]
treatment of hyperthyroidism. [BUHS Apr 1987, Aug 9. Neurological complications of diabetes mellitus.
1988, Jul 1991] [RURS Feb 2007]
Section | V Previous Years’ Question Bank 689

10. Graves’ disease. [MUHS Jan 1999] 26. Albuminuria. [NTR-NR Apr 2005]
11. Diagnosis and management of hypoglycaemia. [NTR- 27. Myxoedema. [NTR-NR Apr 2005; TN Oct 2000, Aug
NR Oct 2007] 2004; BUHS Jan 1986, Mar 1995]
12. Clinical features and management of hyperthyroidism. 28. Gynaecomastia. [NTR-NR Apr 2007]
[RGUHS Sep 2002] 29. Thyroxin. [MUHS Jan 1999]
13. Acromegaly, its clinical features and complications. 30. Hyperthyroidism—any four cardinal signs. [RGUHS
[RGUHS Apr 2001] Sep 2003]
14. Clinical features of hypothyroidism. [RGUHS Apr 31. Goitre. [RGUHS Sep 1999; TN Oct 1996]
2002] 32. Acromegaly—any three signs. [MUHS Jul 1990;
15. Clinical features and investigations of hypothyroidism. BUHS Feb 1996; RURS Mar 2005, Aug 2005, Aug
[RGUHS Apr 2003] 2006]
16. Addison’s disease—clinical features. [RURS Aug 33. Clinical features of acromegaly. [BUHS Feb 1996]
2007] 34. Hypothyroidism. [MUHS Jan 1998; RGUHS Mar
17. Diabetes and surgery. [NTR-NR Oct 2005] 2004; TN Aug 2006, Aug 2008]
18. Gestational diabetes mellitus. [NTR-NR Apr 2006] 35. Hypothyroidism—four signs. [RUOS Sep 2004, Mar
19. Cushing’s syndrome. [NTR-OR Apr 1998] 2005]
20. Eye signs on thyrotoxicosis. [NTR-OR Oct 2004] 36. Thyroid function tests. [MUHS Jan 1998]
21. Hypothyroidism—clinical features. [NTR-NR Apr 37. Hyperparathyroidism. [MUHS Jan 1986, Jan 1999]
2003] 38. Causes of tetany. [RGUHS Apr 2001]
39. Risus sardonicus. [BUHS Oct 1987; MUHS Jan 1991]
40. Tetany cardinal signs. [RGUHS Sep 2003, Mar 2004]
Short Notes 41. Treatment of tetany. [RGUHS Sep 2002; TN Oct 2000]
1. Tetany. [NTR-OR Apr 1993, Apr 1998; TN Apr 1993, 42. Diabetes mellitus—four complications. [RUOS Sep
Apr 1999, Apr 2000, Sep 2002, Feb 2009, Feb 2010; 2004, Mar 2005]
MUHS Jul 1990; BUHS Aug 1993, Mar 1997; RGUHS 43. Diabetes mellitus—six complications. [RURS Aug
Sep 1999] 2006]
2. Prevention of tetanus. [NTR-OR Oct 2001] 44. Diagnosis of diabetes mellitus. [RGUHS Sep 2001; TN
3. Myxoedema. [NTR-OR May 1994; TN Oct 2000, Aug Oct 2000, Oct 2006]
2004, Feb 2007] 45. Oral complications of diabetes mellitus. [MUHS Jan
4. Acromegaly. [NTR-OR May 1994; TN Oct 1999, Nov 1998]
2001] 46. Diabetic ketoacidosis. [BUHS Mar 1995; RGUHS Sep
5. Addison’s disease. [NTR-OR Oct 1998; BUHS Jan 2000; TN Oct 2000, Feb 2009]
1986] 47. Treatment of diabetic ketoacidosis. [RGUHS Apr 2001]
6. Glucosuria. [NTR-OR Apr 1999] 48. Glycosuria. [BUHS Feb 1993]
7. Hypoglycaemia. [NTR-OR Apr 1998, Oct 1999, Feb 49. Albuminuria.[BUHS Aug 1996, Mar 1997]
2002; TN Apr 1995, Apr 1996, Apr 1998] 50. Hypoglycaemia. [BUHS Aug 1988; MUHS Jan 1990;
8. Insulin. [NTR-OR Oct 2003] TN Oct 2003]
9. Human insulin. [NTR-OR Apr 1993; TN Feb 2005] 51. Oral hypoglycaemic drugs. [BUHS Feb 1996; TN Feb
10. Glibenclamide. [NTR-OR Jan 1992] 2006]
11. Metronidazole. [NTR-OR Oct 1991, Apr 1998] 52. Oral antidiabetic drugs. [BUHS Apr 1987, Feb 1996]
12. Cretinism. [NTR-OR Apr 2006; TN Apr 1993, Oct 2003] 53. Antithyroid drugs. [BUHS Apr 1987]
13. Facial features of cretinism. [NTR-OR Mar 1991] 54. Diabetes mellitus. [TN Aug 2009]
14. Diagnosis of hypothyroidism. [NTR-OR Apr 2000] 55. Complications of diabetes mellitus. [TN Apr 2000,
15. Phenylketonuria. [NTR-OR Apr 1993] Nov 2001, Aug 2005]
16. Three signs of Graves’ disease. [RUOS Aug 2006]
17. Hyperthyroidism. [TN Aug 2005]
18. Hyperpituitarism. [TN Oct 2003] 11. NUTRITIONAL FACTORS IN DISEASES
19. Acromegaly. [TN Apr 2001]
20. Exophthalmos. [TN Apr 2001]
Long Essays
21. Sulphonyl urea. [RGUHS Sep 1999; TN Oct 1996] 1. Describe the various manifestations of avitaminosis
22. Oral hypoglycaemic drugs. [RGUHS Sep 2000] with particular reference to oral cavity. How will you
23. Metformin. [NTR-OR Apr 1999] treat them? [NTR-OR Mar 1991]
24. Hypoglycaemia. [NTR-NR Oct 2005, Apr 2006] 2. Describe the aetiology, clinical features, diagnosis and
25. Human insulin. [NTR-NR Apr 2005] treatment of scurvy. [BUHS Mar 1995]
690 Quick Review Series for BDS 3rd Year

3. Mention the various B complex factors and mention the 23. Vitamin A deficiency. [BUHS Aug 1996; MUHS Jan
clinical manifestations due to their deficiency. [BUHS 1990]
Jan 1986] 24. Fat-soluble vitamins—name four. [RURS Aug 2005]
4. Describe the aetiology, clinical features, diagnosis and 25. Beriberi—name any two causes. [RUGS Aug 2005;
treatment of thiamine deficiency. [TN Oct 1996] RGUHS Apr 2003
26. Vitamin C. [NTR-OR Apr 1999]
27. Vitamin E. [NTR-OR Oct 1998]
Short Essays
28. Bitotic spots. [NTR-OR Oct 1999]
1. Scurvy. [BUHS Jan 1986, Sep 1994, Feb 1996; MUHS 29. Vitamin B12. [NTR-OR Apr 1998]
Sep 1994, Jan 2000; RGUHS Sep 2000; TN Apr 1996; 30. Vitamin A deficiency. [TN Feb 2009]
RGUHS Mar 2004; NTR-NR Oct 2002]
2. Splenomegaly. [TN Oct 1997]
3. Malnutrition. [MUHS Jul 2000] 12. INFECTIOUS DISEASES
4. Vitamins—A and E. [MUHS Jul 1991] Long Essays
5. Beriberi—types and clinical features. [RURS Aug
2007] 1. What is the differential diagnosis of ulcers over the
6. Vitamin A deficiency. [RURS Mar 2006] penis? How do you treat syphilis? [NTR-NR Apr
7. Night blindness. [TN Apr 1993, Sep 2002] 2000]
8. Pellagra. [NTR-NR Oct 2002, Oct 2006; TN Apr 1993, 2. Describe the clinical features, investigations and treat-
Oct 1998] ment of typhoid fever. [NTR-NR Oct 2002; TN Oct
9. Fluorosis. [NTR-OR Apr 1998, Feb 2002] 1998]
10. Vitamin B12. [TN Oct 1996] 3. What are the clinical features, diagnosis, complications
11. Riboflavin deficiency. [TN Apr 1996] and management of typhoid fever? [NTR-OR Apr 2001]
4. Describe the pathogenesis, diagnostic investigations,
complications and management of tubercular meningi-
Short Notes tis. [NTR-NR Apr 2007; TN Apr 1996]
1. Scurvy. [NTR-OR Jan 1992, Apr 2001, Oct 2004; TN 5. Aetiology, pathology, clinical features and manage-
Apr 2000, Apr 2001, Aug 2007] ment of cervical TB lymphadenitis. [NTR-NR Apr
2. Fluorosis. [NTR-NR Apr 2004; NTR-OR Oct 2004] 2007]
3. Rickets. [NTR-OR Apr 1998, Apr 1999; TN Aug 6. Describe the aetiology, clinical features and manage-
2009] ment of intestinal amoebiasis. [BUHS Jan 1991]
4. Treatment of rickets. [NTR-NR Oct 2006] 7. Discuss the aetiology, clinical features, complications
5. Rickets and radiographic features. [NTR-NR Oct 2002, and management of enteric fever. [MUHS Jan 1986;
Apr 2003] RGUHS Sep 1999, Apr 2003; TN Apr 1993, Apr
6. Niacin. [NTR-OR Oct 1999] 1996]
7. Vitamin D. [NTR-OR Mar 1994; TN Nov 2001] 8. Describe the aetiology, clinical features, diagnosis,
8. Vitamin K. [NTR-OR Oct 2003; TN Apr 1998] complications and treatment of typhoid fever. [BUHS
9. Vitamin B6. [NTR-OR Oct 1997] Apr 1988; MUHS Feb 2000]
10. Avitaminosis A. [NTR-OR Jan 1992] 9. Diphtheria—aetiology, clinical features and manage-
11. Hyperavitaminosis A. [NTR-OR Oct 1995] ment. [RUOS Aug 2006]
12. Pyridoxine deficiency. [NTR-NR Apr 2007] 10. Describe aetiology, clinical features and management
13. Cyanocobalamine deficiency. [NTR-OR Oct 1999] of diphtheria. [MUHS Jul 1995; RGUHS Sep 2003;
14. Angular stomatitis. [NTR-OR Apr 1990, Apr 1993] TN Oct 1997]
15. Tongue in nutritional deficiency. [NTR-NR Oct 2006] 11. Describe the aetiology, clinical features, complications
16. Manifestations of nutritional deficiency in oral cavity. and treatment of diphtheria. [BUHS Apr 1987, Jun
[NTR-NR Oct 2000] 1989]
17. Scurvy. [BUHS Jan 1986, Sep 1994]
18. Vitamin D. [BUHS Aug 1995; TN Aug 2006]
Short Essays
19. Vitamin E.[MUHS Jan 1990; TN Oct 1997]
20. Pellagra. [MUHS Jul 1997; TN Aug 2005, Aug 2008] 1. Oral manifestations of AIDS. [MUHS Jul 1991; BUHS
21. Three features of pellagra. [RURS Mar 2006] Sep 1994; TN Nov 2001, Aug 2004]
22. Avitaminosis. [MUHS Jul 1995, Jul 1997; RGUHS 2. Acquired immune deficiency syndrome (AIDS)—six
Sep 2001] opportunistic infections. [RGUHS-RS Aug 2007]
Section | V Previous Years’ Question Bank 691

3. Human immunodeficiency virus (HIV)—post-expo- 39. Aminoglycosides. [BUHS Jul 1991]


sure prophylaxis. [RGUHS-RS Aug 2006] 40. Chloramphenicol. [BUHS Jul 1991]
4. Scabies. [BUHS Aug 1988] 41. Tetanus. [TN Aug 2009]
5. Gonorrhoea. [MUHS Jan 1997; RGUHS Sep 2000] 42. Diagnosis of HIV. [TN Feb 2005, Feb 2006, Aug
6. Chickenpox. [BUHS Aug 1988] 2008]
7. Herpes simplex. [RURS Mar 2006; TN Oct 1997, Oct 43. Malaria. [TN Feb 2007]
2000] 44. Metronidazole. [TN Feb 2006]
8. Herpes labialis. [BUHS Jan 1990] 45. Ludwig’s angina. [TN Feb 2005, TN Feb 2006]
9. Mumps. [MUHS Jul 1990; BUHS Feb 1993, Aug 46. Herpes zoster. [TN Oct 1996, Apr 2001, Oct 2003]
1996; RGUHS Mar 1997, Sep 2000] 47. Herpes labialis. [TN Apr 1993, Oct 2000]
10. Complications of mumps. [RGUHS Apr 2001]
11. Diphtheria. [RGUHS Sep 1999]
Short Notes
12. Three malarial parasites. [RGUHS-RS Feb 2007]
13. Diphtheria—two fatal complications. [RGUHS Apr 1. Rubella. [NTR-OR Feb 2002]
2003] 2. Mumps. [NTR-OR Mar 1994; NTR-NR Oct 2002; TN
14. Amoebiasis. [MUHS Jul 1995, Jan 1998; TN Aug Oct 1999, Aug 2007, Aug 2008]
2009] 3. Measles. [NTR-OR Oct 1991; TN Apr 2001]
15. Amoebic dysentery. [BUHS Jul 1995, Feb 1996] 4. German measles. [NTR-NR Apr 2007]
16. Amoebiasis—clinical features and treatment. [RGUHS- 5. Prevention of measles. [NTR-OR Mar 1991; TN Oct
OS Sep 2004, Mar 2005] 2003]
17. Treatment of amoebiasis. [RGUHS Apr 2001] 6. Diphtheria. [NTR-OR Mar 1994; TN Apr 2000]
18. Types of fever. [RGUHS Apr 2000] 7. Ascariasis. [NTR-OR Mar 1994]
19. Enteric fever. [RGUHS Apr 2000; TN Aug 2006] 8. Hookworm. [NTR-OR Apr 1998]
20. Types of enteric fever. [RGUHS Apr 2000] 9. Ankylostomiasis. [NTR-OR Mar 1991]
21. Enumerate complications of enteric fever. [RGUHS 10. Amoebic dysentery. [NTR-OR Apr 1990]
Apr 2000] 11. Cerebral malaria. [NTR-OR Apr 1993, Oct 2004]
22. Diagnostic tests to diagnose enteric fever. [RGUHS 12. Treatment of falciparum malaria. [NTR-NR Oct 2006]
Apr 2002] 13. Treatment of malaria. [NTR-NR Apr 2005]
23. Widal test. [MUHS Jun 1989, Jan 1990; RGUHS Sep 14. Complications of malaria. [NTR-OR Feb 2002; TN
1999] Feb 2006]
24. Widal reaction. [MUHS Jun 1989; RGUHS Sep 1999] 15. Complications of mumps. [NTR-OR Apr 1996; NTR-
25. Two spirochaetal diseases. [RGUHS-OS Aug 2006] NR Apr 2006]
26. Chancre. [BUHS Feb 1993; RGUHS Sep 1999] 16. Typhoid fever. [NTR-OR Apr 1990; TN Feb 2005]
27. Three stages of syphilis. [RGUHS-RS Feb 2007] 17. Diagnosis of typhoid fever. [NTR-OR Feb 2002; TN
28. Oral manifestations of syphilis. [RGUHS Apr 2002; Aug 2007]
TN Oct 1996] 18. Widal test. [NTR-OR Jan 1992, Oct 1997, Oct 1999]
29. Congenital syphilis. [BUHS Oct 1987; RGUHS Sep 19. Complications of typhoid fever. [NTR-OR Apr 2004]
2000; TN Oct 2003] 20. Clinical features of typhoid fever. [NTR-OR Oct 2001;
30. Abnormal teeth in syphilis. [BUHS Nov 1986] TN Oct 2003]
31. Two abnormalities in congenital syphilis. [RGUHS 21. Rabies prevention. [NTR-OR Oct 1999, July 2000, Apr
Sep 2002] 2001]
32. VDRL test. [BUHS Jan 1991] 22. Chromogenic bacteria. [NTR-OR Oct 1997, Apr 1999]
33. Two oral manifestations of viral diseases. [RGUHS 23. Teeth in congenital syphilis. [NTR-OR July 1990, Apr
Sep 2002] 2000]
34. Lymphadenopathy. [BUHS Sep 1992, Mar 1995; 24. Lepromatous leprosy. [NTR-OR Apr 1990]
MUHS Jan 1997, Jul 1997, Jul 1998] 25. Lepra reaction and its management. [NTR-NR Oct 2006]
35. Generalized lymphadenopathy. [RGUHS Apr 2000, 26. AIDS. [NTR-OR Oct 1991]
Sep 2000] 27. Diagnosis of AIDS. [NTR-OR Oct 2001; TN Apr 1998,
36. Causes of generalized lymphadenopathy. [RGUHS Apr Aug 2007]
2000] 28. Oral manifestations of HIV and AIDS. [NTR-OR Feb
37. Prednisolone. [RGUHS Sep 2000] 2002; NTR-NR Apr 2004, Oct 2005; TN Feb 2007]
38. Metronidazole. [MUHS Jun 1989; BUHS Sep 1992, 29. Post-exposure prophylaxis in HIV. [NTR-NR Oct
Sep 1994; RGUHS Sep 1999, Sep 2001] 2007]
692 Quick Review Series for BDS 3rd Year

30. Quinolones. [NTR-NR Apr 2006] 77. Four causes of lymphadenopathy. [RGUHS-OS Mar
31. Cephalosporins. [NTR-OR Oct 1995; TN Apr 1999] 2006]
32. Triple vaccine (DPT). [NTR-OR Jul 1990]
33. Anti-amoebic drugs. [NTR-OR Apr 1993] 13. IMMUNOLOGICAL FACTORS IN
34. AIDS. [NTR-OR Apr 1998]
35. Quinsy. [NTR-OR Oct 1996]
DISEASE (ANAPHYLAXIS AND DRUG
36. Rabies prevention. [NTR-OR Oct 1999] ALLERGY)
37. Rabies prophylaxis. [NTR-OR Apr 1998] Long Essays
38. Prevention of measles. [NTR-OR Mar 1991]
39. Cellulitis. [RGUHS Apr 1998] 1. Classify allergic reactions. Describe the clinical features
40. Carbuncle. [RGUHS Apr 2000] and management of generalized anaphylaxis. [TN Oct
41. Cold abscess. [RGUHS Apr 2000] 2003]
42. Erysipelas. [NTR-OR Oct 1996]
43. Antibioma. [NTR-OR Apr 1993] Short Essays
44. Cervicofacial actinomycosis. [NTR-NR Apr 2003]
45. Prevention of rabies. [NTR-OR Oct 2002] 1. Anaphylaxis. [BUHS Nov 1986, Aug 1993, Aug 1995;
46. Scarlet fever. [NTR-NR Apr 2004] MUHS Jan 1995, Jan 1996; RGUHS Sep 2000]
47. Antiviral drugs. [NTR-OR Feb 2002] 2. Anaphylactic shock. [RGUHS Apr 2001, Sep 2001; TN
48. Gonorrhoea. [RUOS Mar 2006] Oct 1998; RURS Feb 2007]
49. Chickenpox. [RGUHS Sep 2000] 3. Anaphylaxis—three drugs used for treatment. [RURS
50. Viral exanthema. [MUHS Jan 2000] Aug 2006]
51. Typhoid fever—clinical features and management. 4. Oedema—four common causes. [RGUHS Mar 2004]
[RGUHS Mar 2004] 5. Diagnostic procedures for hypersensitive reactions.
52. Widal test. [RGUHS Sep 2001] [NTR-NR Oct 2007]
53. Enteric fever. [RURS Aug 2005]
54. Complications of enteric fever. [RGUHS Sep 1999] Short Notes
55. Complications and drugs used in enteric fever. [RGUHS
1. T cells. [NTR-OR Oct 2004]
Sep 2002]
2. T lymphocytes. [NTR-OR Oct 2004]
56. Treatment of enteric fever. [RGUHS Apr 2001]
3. Anaphylaxis. [NTR-OR Apr 2000; NTR-NR Apr 2005;
57. Complications of diphtheria. [MUHS Jan 1999;
BUHS Nov 1986, Aug 1995; TN Oct 1997, Apr 2000]
RGUHS Apr 2001]
4. Anaphylactic shock. [NTR-OR Feb 2002, Oct 2003;
58. Clinical features and complications of mumps.
TN Oct 1996, Apr 1999, Aug 2004, Aug 2006]
[RGUHS Sep 2003]
5. Penicillin anaphylaxis. [NTR-OR Apr 1996]
59. Syphilis. [MUHS Jul 1991, Jul 1997]
6. Serum sickness. [NTR-NR Oct 2005]
60. Primary chancre. [RGUHS Apr 2001]
7. Immunosuppressed host. [NTR-OR Oct 2004]
61. Oral manifestations of HIV infection. [RURS Aug
8. Pemphigus vulgaris. [NTR-OR Oct 2004]
2005; TN Feb 2009]
9. Allergy. [MUHS Sep 1991]
62. CD4 cell count. [RURS Feb 2007]
10. Urticaria. [BUHS Mar 1988]
63. Describe signs, symptoms and treatment of malaria
11. Urticaria—definition. [RGUHS Apr 2001]
fever. [MUHS Jan 1991, Jan 1997, Jan 1998]
12. Anaphylaxis—clinical features. [RGUHS Mar 2004]
64. Amoebiasis—treatment and prevention. [RURS Aug
13. Treatment of anaphylactic shock. [RGUHS Sep 2002]
2006]
14. Angioedema. [NTR-NR Apr 2004]
65. BCG. [NTR-NR Oct 2002]
15. Stevensw–Johnson syndrome. [MUHS Jan 1996; NTR-
66. AIDS. [NTR-OR Apr 1998, Apr 1999]
NR Apr 2003]
67. Scabies. [NTR-NR Apr 2007]
16. Generalized anaphylaxis. [TN Feb 2005]
68. Falciparum malaria. [NTR-OR Apr 1998]
17. Immunization. [NTR-NR Oct 2005]
69. Reactions of leprosy. [NTR-NR Oct 2005]
70. Amoebic abscess of liver. [NTR-NR Oct 2006]
71. Complications of typhoid fever. [NTR-NR Apr 2007] 14. DISEASES OF CONNECTIVE TISSUES,
72. Complications of hepatitis B. [NTR-NR Apr 2007] BONES AND JOINTS
73. HIV infection. [TN Oct 1996, Feb-2005]
Long Essays
74. Chancre. [TN Apr 1993, Oct 1997, Apr 2001]
75. Chloroquine. [TN Oct 1996] 1. Describe the aetiology, clinical features and treatment
76. Human insulins. [TN Oct 1996] of rickets? [BUHS Mar 1988]
Section | V Previous Years’ Question Bank 693

Short Essays 2. Arsenic poisoning. [NTR-NR Apr 2004]


3. Barbiturate poisoning. [NTR-OR Oct 1997]
1. Osteoporosis—clinical features and treatment. [RURS
4. Scorpion bite. [NTR-OR Apr 1996]
Aug 2006]
5. Lead poisoning. [NTR-OR Jul 1990, Jan 1992]
2. Describe the aetiology, clinical features and treatment
6. Adverse drug reactions. [NTR-OR Oct 1991]
of rickets. [BUHS Mar 1988]
7. Organophosphorous poisoning. [NTR-OR Oct 1995]
3. Rickets. [BUHS Jan 1986, Sep 1994, Feb 1996]
8. Lead poisoning. [BUHS Jan 1986, Aug 1988]
4. Osteomalacia. [RGUHS Aug 1988, Sep 1999]
9. Wrist drop. [NTR-NR Apr 2006]
5. Rheumatoid arthritis—name three complications.
10. Gram stain. [NTR-OR Oct 1991]
[RURS Aug 2006]
11. Intractable pain. [NTR-OR Apr 2001]
6. Gout. [NTR-OR Oct 2004]
12. Pigmented lesions. [NTR-NR Apr 2005]
7. Scleroderma. [NTR-NR Apr 2006, Oct 2006]
13. Thrush. [NTR-OR Jan 1992]
8. Rheumatoid factors. [NTR-NR Apr 2005]
14. Halitosis. [NTR-OR Apr 1990, Oct 2002; NTR-NR
9. Deformities in rheumatoid arthritis. [NTR-NR Oct
Oct 1995]
2002; TN Oct 1991]
15. Oral ulcers. [NTR-OR Oct 1998, Apr 1999, Oct
1999]
Short Notes 16. Macroglossia. [NTR-NR Oct 2002]
17. Dental fluorosis. [NTR-OR Mar 1991, May 1994;
1. COX-2 inhibitors. [TN Aug 2005, Aug 2008]
NTR-NR Apr 2006]
18. Oral candidiasis. [NTR-OR 1994]
15. ACUTE POISONING AND 19. Aphthous stomatitis. [NTR-OR Oct 2003; NTR-NR
ENVIRONMENTAL EMERGENCIES Apr 2006]
20. Recurrent oral ulcers. [NTR-OR Oct 1995]
Long Essays 21. Aphthous reactions. [NTR-NR Oct 2005]
22. Follicular tonsillitis. [NTR-OR Apr 1990]
1. Describe the management of acute poisoning. [BUHS
23. Leukoplakia of tongue. [NTR-OR Oct 2004]
Mar 1988]
24. Colour changes in tongue. [NTR-OR Oct 2004]
2. Describe the signs, symptoms and management of fluorosis.
25. Excessive salivation. [NTR-OR Apr 1996]
[MUHS Jan 1996]
26. Dilantin. [NTR-OR July 1990, Oct 2002]
27. Ciprofloxacin. [NTR-OR May 1994]
Short Essays 28. Phenobarbitone. [NTR-OR Oct 1998]
29. Gum hypertrophy. [NTR-NR Oct 2007]
1. Atropine.[MUHS Jan 1981]
30. Gum hypertrophy medical diseases. [NTR-OR Mar
2. Arsenic poisoning. [NTR-NR Oct 2002]
1991]
3. Symptoms of lead poisoning. [NTR-OR Feb 2002]
31. Aphthous ulcers. [BUHS Oct 1987]
4. Fluorides in health and disease. [MUHS Jan 1998]
32. Dental fluorosis. [MUHS Jun 1989; BUHS Aug
5. General management of poisoning case. [NTR-NR Oct
1995]
2007]
33. Three features of fluorosis. [RURS Mar 2006]
6. Discuss the oral complication of cytotoxic therapy.
34. Hypertrophy of gums. [RUOS Sep 2004, Mar 2005;
[NTR-OR Jan 1992]
RURS Aug 2005]
7. Stomatitis. [MUHS Jul 1991, Jul 1995; BUHS Mar
35. Four causes of gum hypertrophy. [RUOS Mar 2006]
1995]
36. Gum bleeding. [RGUHS Mar 2004]
8. Gingivitis. [BUHS Mar 1992]
37. Causes of bleeding gums. [RGUHS Apr 2001]
9. Gingival hyperplasia. [MUHS Jan 1998, Jul 1999]
38. Two drugs used in treatment of hyperlipidaemia.
10. Hypertrophy of gums. [BUHS Mar 1995]
[RUOS Mar 2006]
11. Dental care in mental retardation. [RURS Mar 2006]
39. Taste disorders. [NTR-OR Mar 1990]
40. Prednisolone. [NTR-OR Oct 1999]
Short Notes
1. Food poisoning. [NTR-OR Apr 2001; NTR-NR Oct 2005]
694 Quick Review Series for BDS 3rd Year

Section II

General Surgery
1. GENERAL PRINCIPLES OF OPERATIVE 5. Discuss the factors, which influence the wound healing.
SURGERY, STERILIZATION/ASEPSIS, Add a note on healing in soft tissues. [MUHS Jul 2000]
DIATHERMY, CRYOSURGERY AND LASERS 6. What are the different types of wounds? Discuss in
detail about wound healing.
Long Essays 7. What are the factors which delay the wound of healing?
[RURS Aug 2007]
1. What is sterilization? What are the methods of steriliz-
8. Classify wound infections. Write about hospital-acquired
ing surgical materials? Mention the merits and demerits.
infections and their prevention. [TN Feb 2009]
[MUHS Jan 1990, Jul 1991, Jan 1997]
9. Discuss wound healing. [RUHS Jul 1994]
10. Classify wounds. Mention the features and management
Short Essays of each type of wound. [BUHS Sep 1994, Mar 1995]
1. Autoclave sterilization. [NTR-OR Apr 1999]
2. Absorbable suture materials. [RGUHS-RS Aug 2007] Short Essays
3. Cryosurgery in oral cavity. [RGUHS-OS Aug 2006] 1. Write about factors affecting wound healing. [RGUHS
4. Chemical sterilization. [NTR-NR Apr 2007] Mar 2004; RURS Aug 2005]
2. Delayed wound healing. [NTR-OR Oct 1999]
Short Notes 3. Healing by primary intention. [NTR-OR Jan 1992]
4. Healing by secondary intention. [NTR-OR Oct 1993;
1. Antisepsis. [RGUHS-OS Mar 2006] TN Aug 2006; NTR-RS Mar 2009]
2. Asepsis in surgery. [BUHS Mar 1994, MUHS Jan 1999, 5. What is sinus and fistula? What are their causes?
RUOS Mar 2005] [RGUHS Apr 2003]
3. Disinfection. [TN Apr 2003] 6. Keloid scar. [NTR-NR 2006 Apr; TN Oct 1998, Apr
4. Cidex (glutaraldehyde). [TN Aug 2005] 2003, Aug 2008]
5. Autoclave sterilization. [NTR-NR Apr 1999]
6. Sterilization of hot air. [NTR-NR Apr 2006]
7. Incineration. [RURS Aug 2006]
Short Notes
8. Sterilization. [MUHS Jan 1992] 1. Delayed healing. [NTR-NR Oct 2005]
9. Diathermy. [TN Oct 1997, MUHS Jan 2000] 2. Secondary healing. [NTR-NR Apr 2005; TN Apr 1996,
Apr 2001]
3. Hypertrophic scar. [NTR-NR Oct 2006]
2. WOUNDS, SINUS, FISTULAE 4. Healing by delayed primary intention. [NTR-NR Apr
AND ULCERS 2004; TN Aug 2004]
5. Healing by primary intention. [NTR-NR Apr 2003]
Long Essays
6. Keloid. [TN Apr 2001]
1. Describe the types of wounds and their management. 7. Name three factors affecting wound healing. [TN Oct
[NTR-OR Oct 1995] 1996, Apr 1998, Oct 2000, Aug 2006]
2. Discuss the pathology and healing of wounds. [NTR- 8. Sinus. [MUHS Jan 1992; RGUHS-RS Aug 2006]
OR Apr 1993, Oct 2002; NTR-NR Oct 2003, TN Apr 9. Fistula. [MUHS Jul 1999; RGUHS-OS Aug 2005]
1998] 10. Diabetic ulcer. [RGUHS-RS Aug 2005]
3. Discuss about haemorrhage and management of bleed- 11. Trophic ulcer. [BUHS Aug 1995; RGUHS Mar 2005;
ing wounds of the face. [TN Apr 2003, Aug 2006, Apr RGUHS-RS Aug 2006]
1998] 12. Snail track ulcer. [RGUHS-RS Aug 2006]
4. Discuss the stages of wound healing for closed and 13. Rodent ulcer. [NTR-OR Mar 2009]
open wounds. What are the factors affecting wound 14. Types of healing. [NTR-OR Jul 2009]
healing? [TN Oct 2003] 15. Define fistula. Give two examples. [NTR-OR Jul 2009]
Section | V Previous Years’ Question Bank 695

3. BURNS, SKIN GRAFTING AND FLAPS 5. Classify and describe the signs, symptoms and man-
agement of shock. [NTR-OR Oct 1990]
Long Essays 6. Describe the pathophysiology, clinical features and
1. Classification of burns. [NTR-OR Jun 1982] management of shock. [NTR-OR Oct 1996]
2. Pathology and treatment of burns. [NTR-OR Apr 1993; 7. Describe the pathophysiology, clinical features and
Oct 2002] treatment of septic shock. [NTR-OR Oct 1997]
3. Describe the pathology and management of 50% burns 8. Describe the pathophysiology, clinical features and man-
in a person aged 40 years. [NTR-OR Apr 1995] agement of haemorrhagic shock. [NTR-OR Apr 1996]
4. Discuss the management of thermal injuries of face and 9. What is haemorrhagic shock? Describe the signs,
oropharynx. [NTR-OR Jul 1981] symptoms and its management. [NTR-OR Jan 1982]
5. Define burns and scalds. Discuss pathophysiology and 10. Classification and clinical features of haemorrhage.
management of 20% burns. [RURS Aug 2006] [NTR-NR Oct 2006, Mar 2009]
11. Classify haemorrhage and discuss its management.
[NTR-NR Apr 2003]
Short Essays 12. Describe indications and complication of blood trans-
1. Burn shock. [NTR-NR Apr 2006] fusion. [NTR-OR Oct 1998]
2. Superficial burns. [NTR-NR Oct 2005] 13. Discuss the different types of haemorrhage and the
3. Rule of nine burns. [NTR-NR Oct 2006] management. [NTR-OR Oct 2001]
4. Rule 9 in evaluation of burns. [NTR-NR Oct 2007] 14. Describe the causes, clinical features and management
5. Burns. [RURS Feb 2007] of haemorrhage. [NTR-NR Oct 2005]
6. Skin graft. [BUHS Aug 1993; MUHS Jul 1995, Jul 15. Discuss the aetiology, types and management of shock.
1997; RURS Aug 2007] [BUHS Aug 1995]
7. Types of skin grafting. [RURS Aug 2006; NTR-OR Jul 16. Describe the haemorrhagic shock and its management.
2009] [RGUHS Sep 2000]
8. Split skin grafting. [RURS Aug 2005] 17. Describe the pathophysiology of shock and its manage-
9. Arterial thickness skin graft. [RGUHS Sep 2001] ment. [MUHS Jul 1998]
10. Composite skin grafts. [TN Apr 2003] 18. Define and classify shock. Discuss the management of
11. Skin graft. [TN Apr 1995, Apr 1996, Sep 2002, Oct 2003] hypovolaemic shock. [MUHS Jan 1996; NTR-OR Mar
2009]
19. Pathophysiology, clinical features and management of
Short Notes haemorrhagic shock. [RGUHS Apr 2001]
1. Plasma expanders. [NTR-NR Oct 2003] 20. What is shock? Classify and write the management of
2. Burns of face. [NTR-OR Apr 1996] septic shock. [MUHS Jul 1989]
3. Classification of burns. [NTR-OR Oct 1993] 21. What is shock? Describe the types, pathophysiology,
4. Skin grafting. [NTR-OR Apr 1999, Oct 2005] clinical features and treatment of shock? [BUHS Aug
5. Burn wound grafting. [RURS Feb 2005] 1993; MUHS Jul 1997; RGUHS Sep 1999]
6. Skin grafting. [RURS Mar 2006] 22. What is shock and what are the varieties and specifi-
7. Split skin graft. [RGUHS Sep 2004] cally describe the neurogenic shock and its manage-
8. Skin grafting indications. [RURS Feb 2007] ment? [MUHS Jan 1991]
9. Boil. [NTR-NR Oct 2005] 23. What are the types of haemorrhage? Describe the
10. Scalds. [NTR-NR Oct 2004] clinical features and treatment of haemorrhage.
11. Keloid scar. [NTR-NR Apr 2006; TN Oct 1998, Apr [RGUHSI Sep 2003]
2003, Aug 2008] 24. What is haemorrhage and types of haemorrhage? How
will you manage a case of primary haemorrhage after a
dental extraction? [RGUHS Apr 2000]
4. HAEMORRHAGE AND SHOCK
Long Essays Short Essays
1. Describe in detail about shock. [NTR-NR Oct 2003] 1. Vasovagal shock. [NTR-NR Apr 2006]
2. Describe pathophysiology of shock. [NTR-OR Feb 1990] 2. Hypovolaemic shock. [NTR-NR Oct 2006]
3. Describe the clinical features and management of 3. Haemophilia. [MUHS Jan 1992, Jan 1993, Jan 1996, Jul
shock. [NTR-OR Apr 2000] 2000; RURS Feb 2007]
4. Classify and describe the symptoms and treatment of 4. Complications of blood transfusion. [RGUHS Sep
shock. [NTR-OR Oct 1999] 2004; RURS Aug 2006]
696 Quick Review Series for BDS 3rd Year

5. Indications and complications of blood transfusion. 6. Describe the clinical features, diagnosis and treatment
[RGUHS Apr 2001] of tetanus. [NTR-OR Aug 1991]
6. Pathophysiology of haemorrhagic shock. [RGUHS Mar 7. Describe the aetiopathology, clinical features of teta-
2004] nus and its management. [BUHS Jul 1990, Mar 1991,
7. Reactionary bleeding. [RUOS Aug 2005] Feb 1993, Sep 1999]
8. Classification of haemorrhage and management. [RURS 8. What are the types of tetanus and its causes? Describe
Aug 2007] the clinical features and management of tetanus.
[RGUHS Sep 2002]
9. Discuss aetiopathogenesis, clinical features and man-
Short Notes
agement of tetanus. Add a note on prevention of teta-
1. Types of shock. [NTR-OR Nov 1992] nus. [RGUHS Mar 2004]
2. Septic shock. [NTR-NR Apr 2007] 10. Discuss the pathophysiology and treatment of diabetic
3. Neurogenic shock. [NTR-OR Apr 1995] carbuncle. [MUHS Jul 1998]
4. Hypovolaemic shock. [NTR-OR Aug 1991, Apr 1999] 11. Define carbuncle. Mention aetiology, clinical features
5. Crush syndrome. [NTR-OR Apr 1996] and treatment. [BUHS Mar 1992]
6. Acidosis. [NTR-OR Oct 1995] 12. Discuss the clinical features and the management of
7. Rh factor. [NTR-OR Apr 2003] gas gangrene. [BUHS Mar 1995, Aug 1995, Feb 1996]
8. Haemophilia. [NTR-OR Apr 1997, Oct 1998, Apr 1999] 13. Discuss pathogenesis, clinical features and treatment
9. Haemorrhage. [NTR-NR Apr 2005; TN Feb 2009] of gas gangrene. [RGUHS-OS Aug 2005]
10. Blood transfusion. [NTR-NR Oct 2003; NTR-OR Jul 14. Describe the aetiology, clinical features, diagnosis and
2009] treatment of chronic osteomyelitis of the jaw. [MUHS
11. Secondary haemorrhage. [NTR-OR Apr 1995, Oct 1999] Jan 1990, Jul 1997, Jul 1999; BUHS Feb 1993]
12. Reactionary haemorrhage. [NTR-OR Apr 2000]
13. Complications of blood transfusion. [NTR-NR Oct 2007]
14. Disseminated intravascular coagulation disorders.
Short Essays
[NTR-NR Apr 2003] 1. Osteomyelitis of the jaw. [MUHS Jan 1989]
15. Haemophilia. [RGUHS Apr 2000] 2. Actinomycosis. [RGUHS-OS Aug 2006]
16. Haemophilia A. [RGUHS Mar 2004] 3. Ludwig’s angina. [TN Apr 2001, Sep 2002, Apr 2003]
17. Blood groups. [BUHS Mar 1994; RGUHS Sep 2000] 4. Trismus. [TN Apr 2001]
18. Haemorrhage. [MUHS Jan 1992, Jul 1999] 5. Erysipelas. [TN Nov 2001, Feb 2005]
19. Secondary haemorrhage. [BUHS Apr 1987, Jan 1990, 6. Rodent ulcers. [NTR-NR Apr 2004]
Jul 1991, Aug 1993, Aug 1996] 7. Boils. [NTR-NR Oct 2005]
20. Reactionary haemorrhage. [RUOS Mar 2006] 8. Carbuncle. [NTR-NR Oct 2007; NTR-OR Jul 2009;
21. Blood transfusion reactions. [RURS Aug 2007] TN Sep 2002, Oct 2003, Aug 2005]
22. Complications of blood transfusion. [RGUHS Apr 9. HIV. [NTR-NR 2004 Apr; TN Oct 1996, Oct 1997, Apr
2000, Sep 2001] 1998, Oct 1998, Apr 2000]
23. Hypokalaemia. [NTR-OR Jul 2009] 10. Syphilis. [NTR-NR Oct 2003]
11. Sinus. [NTR-NR Apr 2005]
12. Branchial fistula. [NTR-NR Oct 2006]
5. BACTERIAL INFECTIONS AND 13. Nonhealing ulcer. [NTR-NR Apr 2005]
TRANSMISSIBLE VIRAL INFECTIONS 14. Active immunity. [NTR-NR Apr 2005; TN Oct 1996]
15. Passive immunity. [NTR-NR Oct 2003; NTR-OR Oct
Long Essays
2004]
1. Describe the aetiology, pathology, clinical features and 16. Dangerous area of the face. [RUGS Aug 2006]
management of actinomycosis. [NTR-OR Oct 2004] 17. Pre-auricular sinus. [RGUHS-OS Mar 2006]
2. Causative organism, signs, symptoms and treatment of 18. Median mental sinus. [MUHS Jan 1991; RGUHS Mar
tetanus. [NTR-NR Apr 2006] 2004; RGUHS-OS Aug 2006]
3. Discuss the aetiology, pathology, clinical features and 19. Acute glottic oedema. [RGUHS-RS Feb 2007]
management of tetanus. [NTR-OR 2004 Oct; TN Oct 20. Septicaemia. [NTR-OR Oct 1993; NTR-NR Mar 2009]
1991, Aug 2005] 21. Ludwig’s angina. [NTR-NR Oct 2007]
4. Describe the aetiology, pathogenesis and management 22. Preauricular sinus. [NTR-OR Oct 1997]
of tetanus. [NTR-OR 1998 Apr] 23. Median mental sinus. [NTR-OR Oct 1994, Oct 1996]
5. Describe clinical features and management of a case of 24. Boundaries of back of neck. [NTR-OR Apr 1996]
tetanus. [NTR-OR Apr 1999; TN Apr 1993] 25. TB cervical lymphadenitis. [NTR-OR Oct 1996]
Section | V Previous Years’ Question Bank 697

26. Infection of temporomandibular joint. [NTR-NR Apr 18. Ulcers. [NTR-OR Apr 2000]
2007] 19. Sinus and fistula. [NTR-NR Apr 2003]
27. Western blot test. [NTR-NR Apr 2004] 20. Venous ulcers. [NTR-NR Apr 2007; TN Feb 2010]
28. Tetanus bacilli. [NTR-OR Apr 2003] 21. Tetanus. [NTR-OR 2001 Oct; TN Apr 1996]
29. Anaerobic infections. [NTR-OR Jan 1992] 22. Tetanus bacilli. [NTR-NR Apr 1990]
30. Different levels of cervical lymph nodes. [NTR-NR 23. Tetanus prophylaxis. [NTR-OR Oct 1998; TN Oct
Apr 2006] 2000, Aug 2006]
31. Endotoxin. [NTR-NR Oct 2007] 24. Active immunity. [NTR-NR Apr 2005; TN Aug 2004]
32. Tetanus. [MUHS Jul 1997, Jul 1999] 25. Varieties of gangrene. [NTR-OR Apr 1999]
33. Clinical features and management of tetanus. [RURS 26. Gas gangrene. [NTR-NR Apr 2007; TN Apr 1996, Oct
Aug 2007] 1996, Oct 1998, Oct 2000, Feb 2005, Aug 2009, Feb
34. Keloid. [RURS Mar 2006] 2010]
35. Carbuncle. [RGUHS Sep 2004; RUOS Aug 2006; 27. Cervicofacial actinomycosis. [NTR-OR Apr 1996; TN
RURS Feb 2007] Feb 2008]
36. Septicaemia. [RGUHS Sep 1999] 28. Tuberculous cervical lymphadenitis. [TN Feb 2009]
37. Erysipelas. [RGUHS-RS Aug 2006] 29. Chemical burns. [TN Apr 2001]
38. Ludwig’s angina. [BUHS Apr 1987, Feb 1993, Mar 30. Congenital syphilis. [TN Apr 2001]
1994; MUHS Jul 1997, Jan 1998, Jan 1999] 31. Hepatitis. [TN Nov 2001]
39. Discuss about dry gangrene. [RGUHS Mar 1997, Sep 32. Pressure sores. [TN Aug 2004]
1999] 33. Cold abscess. [TN Oct 1996, Oct 1997, Oct 2000, Aug
40. Human immunodeficiency virus and importance of 2004]
dental surgery. [RGUHS Sep 2001] 34. Basal cell carcinoma. [NTR-OR Jun 1982, Apr 1996;
41. HIV. [NTR-NR Apr 2004] TN Aug 2006, Aug 2009]
42. AIDS. [RGUHS Sep 2000] 35. Trophic ulcer. [TN Feb 2005]
43. Diagnostic and confirmatory tests in AIDS. [RURS 36. Syphilis. [RUGS Mar 2006]
Aug 2007] 37. Epistaxis. [NTR-OR Apr 1991, Oct 2003]
38. Root abscess. [NTR-OR Apr 2000]
39. Koch’s bacillus. [NTR-NR Apr 2004]
Short Notes 40. Cross-infection. [NTR-NR Oct 2004]
1. Boils. [NTR-OR Nov 1992; TN Feb 2007] 41. Sequestrum. [MUHS Jul 1997, Jan 1998]
2. Alveolar abscess. [NTR-NR Apr 2005, Mar 2009] 42. Apical abscess. [NTR-OR Oct 1997]
3. Carbuncle. [NTR-OR Oct 2003, Oct 2004; NTR-NR 43. Actinomycosis. [BUHS Nov 1986, Jan 1989, Feb
Oct 2005; TN Aug 2005, Aug 2009] 1996; RGUHS Mar 2005]
4. Carbuncle of back. [NTR-OR Apr 1998; TN Nov 44. Actinomycosis of face. [BUHS Jan 1989]
2001] 45. Tetanus. [MUHS Jul 1991; RURS Mar 2006; RGUHS-
5. Carbuncle of neck. [NTR-OR Oct 1996] RS Aug 2006; NTR-OR Jul 2009]
6. Erysipelas. [NTR-OR Oct 1993, Oct 1994] 46. Immunoprophylaxis against tetanus. [RURS Aug 2005]
7. Cellulitis. [NTR-OR Oct 1994, Oct 1998, Oct 2004; 47. Lock jaw. [NTR-OR Apr 1998]
NTR-NR Oct 2006; TN Apr 1995] 48. Dry gangrene. [MUHS Jan 1990; RGUHS-OS Mar
8. Septicaemia. [NTR-OR Apr 1999; TN Apr 1998, Oct 2006; NTR-OR Jul 2009]
1999] 49. Gangrene. [NTR-NR Apr 2004; MUHS Jul 1991, Jan
9. Ludwig’s angina. [NTR-OR Apr 1997, Oct 1999; 1999]
NTR-NR Apr 2004] 50. Gas gangrene. [RGUHS-RS Aug 2005, Feb 2007, Aug
10. Rodent ulcer. [NTR-OR Apr 1997, Oct 1998, Apr 2007]
2000] 51. What is gas gangrene? What are the types? [RGUHS
11. Marjolin’s ulcer. [NTR-OR Oct 1994] Apr 2003]
12. AIDS. [NTR-OR Apr 1996] 52. Pyaemia. [MUHS Feb 2000]
13. Actinomycosis. [NTR-OR Apr 1998, Oct 2001; NTR- 53. Cellulitis. [BUHS Sep 1994; RGUHS Apr 2000, Apr
NR Oct 2003; TN Oct 1991, Oct 1997, Oct 1999] 2003]
14. Hutchinson’s teeth. [NTR-OR Apr 2000] 54. Carbuncle. [BUHS Nov 1986, Jan 1990, Aug 1993,
15. Precautions for surgeon working on HIV-infected pa- Feb 1999; RGUHS Apr 2000, Sep 2000, Sep 2002,
tient. [NTR-NR Apr 2006] Mar 2005]
16. Sinus. [NTR-OR Apr 1995] 55. Erysipelas. [RGUHS-OS Aug 2006; TN Non 2001,
17. Fistula. [NTR-NR Apr 2005] Feb 2005]
698 Quick Review Series for BDS 3rd Year

56. Septicaemia. [MUHS Jan 1990] 13. Describe the signs, symptoms and treatment of alveolar
57. Hilton’s method of abscess drainage. [RGUHS Sep abscess. [NTR-OR Apr 1990]
1999] 14. Describe signs and symptoms of bone fracture. Write
58. Ludwig’s angina. [RUOS Aug 2005] about osteomyelitis of mandible. [NTR-NR Oct 2007;
59. HIV. [NTR-NR Apr 2004, Mar 2009] TN Oct 1991]
60. AIDS. [RGUHS Sep 2000] 15. Classify and discuss the clinical features, management
61. Alveolar abscess. [MUHS Jan 1990; NTR-OR Apr of cysts of the jaw. [MUHS Jan 1992]
1995, Apr 1996; TN Apr 1993, Apr 1998, Oct 1998, 16. Describe the clinical features and management of am-
Oct 2000] eloblastoma. [BUHS Mar 1992]
62. Actinomycosis of neck. [NTR-OR Apr 1993, Oct 2002; 17. Describe the pathology, clinical features, diagnosis and
TN Oct 2000] management of adamantinoma. [MUHS Nov 1986, Jul
63. Osteomyelitis. [RGUHS Sep 2003; NTR-NR Apr 2000; BUHS Mar 1992]
2003] 18. What are the cystic lesions that occur in oral cavity and
64. Osteomyelitis of mandible. [NTR-OR Apr 1993, Oct describe clinical features of each of them? [MUHS Feb
1995, Oct 1999, Oct 2002] 2000]
65. Orbital cellulites. [TN Aug 2005] 19. Classify the tumours of the jaw. Describe the clinical
features, diagnosis and treatment of the dentigerous
cyst. [BUHS Apr 1987]
6. TUMOURS, CYSTS AND NECK 20. Name the swellings arising from the jaw. Describe the
SWELLINGS clinical features, diagnosis and treatment of the dentig-
Long Essays erous cyst. [BUHS Feb 1993; MUHS Jul 1997, Jan
1998; RGUHS Sep 2000]
1. Describe the various types of cysts of mandible and de- 21. Describe the midline swellings of neck. Discuss the
scribe in detail the management of each type. [NTR-OR aetiology, clinical features and management of thyro-
Apr 1997; TN Oct 1996, Apr 1998, Apr 2003, Aug 2008] glossal cyst.
2. Describe the clinical features, diagnosis and manage-
ment of tumour of maxilla. [NTR-OR Oct 1997]
3. What are the different types of haemangiomas. De- Short Essays
scribe the treatment of each. [BUHS Mar 1997] 1. Haemangioma. [NTR-OR Apr 2004]
4. Describe the methods of spread of carcinoma, grading 2. Dermoid cyst. [RGUHS Sep 2001; RUOS Aug 2005;
and staging of carcinoma in general. [RGUHS Sep 2001] RURS Aug 2007]
5. What are the epidermal malignant lesions of skin? De- 3. Sublingual dermoid. [RGUHS-RS Feb 2007]
scribe the clinical features and outline its management. 4. Preauricular dermoid. [BUHS Jan 1991; MUHS Jan
[BUHS Oct 1987] 1997]
6. Discuss in detail about aetiology, pathology, clinical 5. Postauricular dermoid. [BUHS Aug 1992; MUHS Jan
features, investigations and management of basal cell 1998, Jul 1999]
carcinoma. [RURS Feb 2007] 6. External angular dermoid. [BUHS Jan 1989, Mar
7. What are the premalignant conditions of skin? De- 1992]
scribe the clinical features and management of basal 7. Dentigerous cyst. [RGUHS-RS Mar 2006]
cell carcinoma. [RGUHS Apr 2003, Mar 2005] 8. Lipoma. [RGUHS-RS Feb 2007]
8. What are the premalignant conditions of oral cavity? 9. Haemangioma. [RGUHS Apr 2001, Sep 2004; TN Feb
Discuss the clinical features and management of squa- 2008]
mous cell carcinoma. [RUOS Mar 2006] 10. Marjolin’s ulcer. [RGUHS Sep 2003; RURS Mar 2006]
9. Discuss the clinical features and management of jaw 11. Basal cell carcinoma. [RGUHS-OS Aug 2005]
tumours. [NTR-NR Apr 2003] 12. Premalignant lesions of skin. [RGUHS Sep 2004]
10. Classify the tumours of the alveolar bone and describe 13. Epulis. [RGUHS Sep 2000]
clinical features and management of the bone. [NTR- 14. Burkitt’s tumour. [MUHS Jul 1999]
OR Apr 2000; TN Apr 1996] 15. Adamantinoma. [RGUHS Sep 2002, Sep 2003]
11. Describe the classification, clinical features and treat- 16. Dentigerous cyst. [BUHS Mar 1994; RGUHS-OS Aug
ment of odontoma. [NTR-OR Apr 1995; TN Apr 1996, 2005]
Oct 1997, Apr 2001, Feb 2009] 17. Epidermoid cyst. [NTR-OR Oct 2004; NTR-NR Oct
12. Describe the clinical features, diagnosis and treatment 2005; TN Nov 2001, Aug 2004]
of ameloblastoma. [NTR-OR Oct 1994, Oct 1999; TN 18. Ranula. [TN Feb 2009]
Apr 1995, Aug 2008] 19. Adenoma. [TN Oct 1991]
Section | V Previous Years’ Question Bank 699

20. Ludwig’s angina. [TN Apr 2001, Sep 2002, Apr 2003] 29. Sequestration dermoid. [BUHS Mar 19881, Aug 1996]
21. Lipoma. [TN Apr 1998, Apr 2001, Oct 2001, Sep 2002, 30. Keloid. [MUHS Apr 1987, Oct 1988, Jan 1990, Sep
Feb 2005, Feb 2006] 1999, Apr 2001, Apr 2003]
22. Sequestration dermoid. [TN Oct 2001, Apr 2003, Aug 31. Lipoma. [BUHS Feb 1993, Mar 1995, Aug 1995;
2006] MUHS Jan 1995, Jul 1997, Jul 1998]
23. Implantation dermoid. [TN Apr 1993, Feb 2005] 32. Lymphangioma. [MUHS Oct 1988]
24. Sternomastoid tumour. [TN Oct 1998, Aug 2005] 33. Haemangioma. [BUHS Mar 1992; MUHS Jan 1990,
25. Cystic hygroma. [NTR-NR Mar 2009] Jan 1992, Jan 1997, Jan 1998, Jan 1999; RGUHS Apr
2000, Sep 2000]
34. Rodent ulcers. [BUHS Aug 1995; MUHS Jun 1989,
Short Notes
Jan 1998]
1. Sebaceous cyst. [NTR-NR Apr 2005; TN Apr 1993, 35. Marjolin’s ulcer. [RGUHS Sep 2001]
Apr 1995, Apr 2003, Aug 2005, Aug 2008, Feb 2009] 36. Malignant melanoma. [BUHS Sep 1994, Aug 1996]
2. Dermoid cyst. [NTR-OR Apr 1990, Oct 1995, Oct 37. Basal cell carcinomas. [BUHS Mar 1995; RGUHS Sep
1996, Oct 1999, Apr 2000; TN Nov 2001, Feb 2008] 1999]
3. Nasopalatine cyst. [NTR-OR Oct 2001] 38. Cavernous haemangioma. [MUHS Nov 1986, Jun
4. Lipoma. [NTR-OR Aug 1991, Apr 1996, Oct 2004; 1989; BUHS Oct 1988, Feb 1996]
NTR-NR Oct 2006; TN Apr 1996, Feb 2009] 39. Complications of lipoma. [RGUHS Apr 2001]
5. Haemangioma. [NTR-OR Oct 1996, Apr 1999] 40. Clark’s level of invasion of malignant melanoma.
6. Neurofibroma. [NTR-OR Oct 1994] [RGUHS Mar 2004]
7. Epithelioma (squamous cell carcinoma). [NTR-OR 41. Microscopic picture of squamous cell carcinoma.
Aug 1991, Oct 1997, Oct 2001; TN Apr 1996] [RURS Aug 2007]
8. Spread of malignant tumours. [NTR-OR Feb 1999; TN 42. Haematoma. [RGUHS Apr 2000]
Feb 2005] 43. Osteoid osteoma. [TN Apr 1996]
9. Dermoid cyst. [TN Oct 1996, Aug 2009] 44. Epulis. [NTR-OR Feb 1990, Apr 1998, Apr 1999; TN
10. Glomus tumour. [TN Feb 2009] Oct 1997, Oct 1998, Oct 2000, Nov 2001, Feb 2006,
11. Sebaceous cyst. [TN Feb 2009] Feb 2009]
12. Ameloblastoma. [MUHS Jan 1989, Jul 2000; NTR-OR 45. Fibrous epulis. [NTR-OR Oct 1994; TN Sep 2002]
Apr 1998; NTR-NR Apr 2006; TN Oct 2001, Aug 46. Odontomes. [NTR-OR Oct 2003]
2009] 47. Dental cyst. [NTR-OR Apr 2000, Oct 2007; TN Apr
13. Radiant green. [TN Oct 1991] 1993, Oct 1993, Apr 1996, Oct 1997, Apr 1998, Oct
14. Basal cell carcinoma. [TN Oct 1997, Apr 1998, Oct 1998 Feb 2005, Aug 2005, Feb 2006, Aug 2006, Aug
1998, Oct 1999, Oct 2003, Feb 2008] 2008, Feb 2009]
15. Rodent ulcer. [TN Apr 1993, Apr 1995, Oct 1997, Apr 48. Adamantinoma. [NTR-OR Jul 1989, Apr 1993, Apr
2001, Nov 2001, Aug 2005, Feb 2010] 2002, Apr 2007; TN Oct 1996, Aug 2006, Aug 2009;
16. Malignant melanoma. [TN Nov 2001, Apr 2003] MUHS Jan 1999]
17. Premalignant lesions of the skin. [TN Nov 2001] 49. Dentigerous cyst. [NTR-OR Jul 1990, Jan 1992, Oct 1997;
18. Neurilemmoma. [NTR-NR Oct 2006] NTR-NR Oct 2005, Oct 2006; TN Apr 1996, Apr 2001]
19. Cystic hygroma. [TN Apr 1998, Feb 2005] 50. Epulis. [MUHS Oct 1988, Jul 1998; RGUHS Apr
20. Potato tumour. [TN Aug 2005] 2001; RURS Aug 2005; RGUHS-OS Aug 2005]
21. Causes for nonhealing of a fistula. [TN Apr 2000, Oct 51. Branchial cyst. [NTR-NR Mar 2009]
2000] 52. Carcinoid tumour. [NTR-NR Mar 2009]
22. Osteoclastoma. [NTR-OR Aug 1991; TN Apr 1993]
23. Spread of malignant tumours. [NTR-OR Feb 1990, Oct
2002]
7. DISEASES OF THE ORAL CAVITY
24. Dermoid. [BUHS Mar 1988, Jan 1989, Aug 1991, Mar (MOUTH, TONGUE AND LIPS)
1992, Aug 1992; RGUHS Sep 1999, Apr 2000] Long Essays
25. Sebaceous cyst. [BUHS Oct 1988, Mar 1992, Jan
1998; MUHS Jan 1993, Jan 1997; RGUHS Sep 2000; 1. Enumerate and write tumours of cheeks and floor of
RGUHS-OS Aug 2005, Aug 2006; RURS Aug 2007] mouth. [NTR-NR Oct 2006; TN Apr 1993]
26. Epidermoid cyst. [RGUHS Apr 2003] 2. Describe the lymphatic drainage of tongue. [NTR-OR
27. Sublingual dermoid. [RGUHS Sep 2000, Sep 2003] Feb 1990]
28. Implantation dermoid. [MUHS Jan 1990; RGUHS Sep 3. Discuss the aetiology, clinical features and manage-
2002] ment of carcinoma of tongue. [NTR-NR Apr 2005]
700 Quick Review Series for BDS 3rd Year

4. Describe the clinical features, diagnosis and treatment 24. Discuss the lymphatic drainage and differential diag-
of carcinoma of tongue. [NTR-OR Apr 1996] nosis of ulcers of tongue and how will you manage the
5. Discuss the differential diagnosis of ulcer tongue and case of a dental ulcer. [MUHS Jul 1990, Jan 1991;
management of carcinoma of tongue. [NTR-OR Oct RGUHS Sep 1999]
2002] 25. What are the different stages of tuberculosis lymph-
6. Describe the signs, symptoms and management of car- adenitis? How do you investigate and manage a case
cinoma of posterior third of tongue. [NTR-OR Apr of tuberculosis lymphadenitis? [RGUHS-OS Aug
1993] 2006]
7. Describe aetiology, clinical features, diagnosis and
management of carcinoma cheek. [TN Oct 1991; NTR-
Short Essays
OR Oct 1998]
8. Discuss the aetiology, pathology and clinical features 1. Stomatitis. [NTR-NR Oct 2007; TN Oct 1998, Aug 2004]
of oral carcinoma and management of gingival carci- 2. Erythroplakia. [NTR-NR Oct 2006]
noma. [NTR-NR Apr 2004] 3. Epulis. [RURS Aug 2007]
9. Describe the pathology of maxillofacial actinomycosis 4. Leukoplakia. [RGUHS Sep 1999, Apr 2000; RURS
and its treatment. [NTR-OR Jul 1990; TN Apr 1996, Aug 2007; NTR-OR Jul 2009]
Apr 1998] 5. Ameloblastoma. [RGUHS-RS Feb 2007]
10. Discuss the differential diagnosis, clinical features and 6. Carcinoma of lip. [MUHS Jan 1993, Jan 1994; RGUHS
management of gum swelling. [NTR-OR Jul 1989; TN Sep 1999, Apr 2001, Mar 2005]
Apr 1995, Oct 1996, Oct 1997, Apr 2003, Aug 2006] 7. Carcinoma of cheek. [MUHS Jul 2000]
11. Discuss the aetiology of oral cancer and how to detect 8. Treatment of carcinoma of cheek. [RGUHS Mar 2004]
a nearly oral cancer and describe the steps to prevent it. 9. Ulcer of tongue. [RGUHS-RS Aug 2007]
[NTR-OR Jul 1989] 10. Faciocervical actinomycosis. [RGUHS-RS Feb 2007]
12. Define oral ulcer. Classify ulcers. Discuss the differen- 11. Discuss the premalignant lesions of oral cavity.
tial diagnosis of ulcers of oral cavity and management [RGUHS-RS Mar 2006]
of ulcer of the palate. [NTR-OR Sep 1983] 12. Describe the anatomy of maxillary antrum. [NTR-OR
13. Surgical anatomy of maxillary sinus, clinical features Sep 1983]
and treatment of maxillary sinus. [NTR-NR Apr 2007] 13. Describe the lymphatic drainage of tongue. [NTR-OR
14. Discuss the differential diagnosis of ulcers over tongue. Feb 1990]
[BUHS Apr 1987; MUHS Jun 1989, Jun 1995] 14. Suprahyoid block dissection. [NTR-NR Mar 2009]
15. Discuss clinical features and management of ulcers of
tongue. [MUHS Jul 1991]
16. Discuss clinical features and management of cancer of
Short Notes
cheek. [MUHS Jul 1998] 1. Angular stomatitis. [NTR-OR Nov 1992; TN Feb
17. Describe the clinical features and management of car- 2006; NTR-NR Mar 2009]
cinoma of tongue. [RGUHS Apr 2001] 2. Leukoplakia. [NTR-OR Feb 1990, Apr 1993, Oct 1999,
18. Write in detail about the causes, management of carci- Oct 2002, Oct 2004; NTR-NR Oct 2003, Apr 2006; TN
noma of tongue. [RGUHS Sep 2000] Apr 1995, Oct 1996, Oct 1998, Oct 2000, Feb 2005,
19. Discuss the clinical features, diagnosis and manage- Aug 2006]
ment of cancer of the tongue. [MUHS Jan 1989; BUHS 3. Tongue tie. [NTR-OR Oct 1999; TN Apr 1996; NTR-
Mar 1994, Feb 1996; RGUHS Apr 2000] NR Mar 2009]
20. Describe the aetiology, clinical features and manage- 4. Cancrum oris. [NTR-OR Feb 1990; TN Oct 1998, Oct
ment of carcinoma of the tongue. [MUHS Nov 1986, Jan 2000, Nov 2001, Apr 2003, Feb 2005]
1989; BUHS Mar 1994, Feb 1996; RHUHS Sep 1999] 5. Lingual thyroid. [NTR-OR Apr 1997, Oct 1998; NTR-
21. Discuss aetiopathogensis, clinical features and treat- NR Apr 2004; TN Apr 1993, Oct 2000]
ment of carcinoma of anterior two-thirds of tongue. 6. Ulcers of tongue. [NTR-OR Apr 1995; TN Feb 2007]
[RGUHS Sep 2004] 7. Carcinoma of lip. [NTR-OR Apr 1996, Apr 1999]
22. Describe the aetiology, pathology of carcinoma of 8. Carcinoma of tongue. [NTR-OR Oct 1996]
cheek and how will you manage if it is involved the 9. Premalignant lesion of the tongue. [NTR-OR Apr
mandible? [BUHS Aug 1992, Mar 1995; MUHS Jul 1997]
1997, Jul 1999] 10. Signs and symptoms of carcinoma of tongue. [NTR-
23. How do you classify ulcer? Describe the aetiopathol- OR Apr 2000; TN Apr 1998, Nov 2001]
ogy, clinical features and management of carcinoma of 11. Haemangioma of the tongue. [NTR-OR Oct 1999]
the tongue. [MUHS Aug 1993, Jul 1997, Jul 1999] 12. Aseptic necrosis of the mandible. [NTR-OR Apr 1997]
Section | V Previous Years’ Question Bank 701

13. Predisposing factors—carcinoma of tongue. [NTR-NR Short Essays


Oct 2007; TN Aug 2005, Feb 2006] 1. Acute parotitis. [RGUHS-RS Aug 2006]
14. Adamantinoma. [NTR-OR Oct 2002] 2. Pleomorphic adenoma. [RGUHS-RS Apr 2005]
15. Leukoplakia. [BUHS Apr 1987, Oct 1988; MUHS Jan 3. Mixed parotid tumour. [RGUHS Apr 2001, Sep 2003]
1990, Jul 1990; RGUHS Apr 2000, Sep 2001] 4. Salivary fistula. [RURS Aug 2007]
16. Ludwig’s angina. [RGUHS-OS Mar 2006] 5. Parotid fistula. [RURS Feb 2007]
17. Cancrum oris. [MUHS Nov 1986, Oct 1987; BUHS 6. Salivary gland calculus. [RGUHS Sep 2000]
Aug 1992; RGUHS Sep 2000, Mar 2005; RGUHS-RS 7. Submandibular gland calculi. [RGUHS Apr 2001, Sep
Aug 2005] 2004]
18. Ankyloglossia. [RGUHS Sep 1999] 8. Adenolymphoma of parotid gland. [RGUHS Apr 2000;
19. Leukoplakia of tongue. [RGUHS Sep 2001] NTR-NR Mar 2009]
20. Lymphatic drainage of tongue. [RGUHS Sep 2000] 9. Surgical anatomy of parotid gland. [RGUHS Mar
21. Premalignant lesions of tongue. [RGUHS Mar 2004] 2004]
22. Glossitis. [NTR-NR Apr 2005] 10. Mixed salivary gland tumour. [TN Apr 1998, Apr 2000,
23. Sinusitis. [NTR-OR Apr 2003] Apr 2001]
24. Anaplasia. [NTR-NR Oct 2006] 11. Mucus cyst. [NTR-NR Apr 2006, Oct 2006]
25. Metaplasia. [NTR-NR Apr 2006] 12. Warthin’s tumour. [NTR-NR Apr 2004; NTR-OR Oct
26. Stomatitis. [NTR-NR Oct 2001] 1997, Oct 1999; TN Aug 2009]
27. Painful epulis. [NTR-NR Apr 2006] 13. Ranula. [TN Apr 1993, Apr 1996, Oct 1996, Apr 1998,
28. Tongue tie. [NTR-OR Oct 1999] Oct 1998, Apr 2001]
29. Oral thrush. [NTR-NR Apr 2007] 14. Xerostomia. [NTR-NR Oct 2003]
30. Impacted teeth. [NTR-OR Apr 1998]
31. Micrognathia or micrognathism. [NTR-OR Apr 1998;
NTR-NR Apr 2006]
Short Notes
32. Macroglossia. [NTR-OR Apr 1997]
1. Ranula. [NTR-OR Jul 1989, Aug 1991, Oct 1995;
RUOS Mar 2006]
8. DISEASES OF THE SALIVARY GLANDS 2. Sialogram. [MUHS Jan 1990, Oct 1988]
3. Sialography. [RUOS Mar 2006]
Long Essays 4. Parotid fistula. [NTR-OR Apr 1995, Apr 2000; RGUHS
1. Describe the aetiology, clinical features, diagnosis and Apr 2003]
management of carcinoma parotid gland. [NTR-OR Oct 5. Acute parotitis. [MUHS Jan 1990, Mar l992; BUHS
1995] Aug 1992, Aug 1993; RGUHS Sep 1999, Mar 2004]
2. Classify salivary gland tumours and describe clinical 6. Parotid abscess. [BUHS Mar 1992, Aug 1992; RGUHS
features and management of parotid gland tumours. Sep 2000]
[NTR-OR Oct 1999] 7. Warthin’s tumour. [MUHS Apr 1987, Jan 1990, Jan
3. What is mixed parotid tumour? Give differential diag- 1997, Jul 1999; BUHS Aug 1995]
nosis and management. [BUHS Jul 1990] 8. Frey’s syndrome. [RGUHS Mar 2005]
4. Describe the clinical features and management of carci- 9. Adenolymphoma. [BUHS Aug 1995; RGUHS Sep
noma of parotid gland. [RGUHS Sep 1999] 1999, Apr 2000]
5. What are the causes of acute parotitis? Describe the 10. Salivary calculus. [BUHS Nov 1986, Feb 1993; NTR-
clinical features and treatment. [BUHS Mar 1997] OR Oct 1998; MUHS Mar 1998]
6. Describe the pathology, clinical features and management 11. Mikulicz’s disease. [MUHS Jan 1991; BUHS Feb
of submandibular salivary calculus. [BUHS Sep 1994] 1996]
7. Classify salivary gland tumours and discuss the clinical 12. Mixed parotid tumour. [MUHS Mar 1988, Jan 1991,
features and management of the pleomorphic adenoma. Jan 1996, Jan 1998]
[BUHS Mar 1995, Feb 1996] 13. Pleomorphic adenoma. [MUHS Jun 1989, Jul 1996, Jul
8. Classify salivary gland tumours. Describe signs and 1999; NTR-OR Oct 1994]
symptoms, management of pleomorphic adenoma of 14. Submandibular salivary gland calculi. [RURS Feb 2007]
parotid gland. [RGUHS-OS Aug 2005] 15. Xerostomia. [NTR-OR Oct 2003]
9. Classify salivary gland tumours. Discuss in detail about 16. Parasitosis. [NTR-OR Mar 1991]
the clinical features, investigations, pathology, manage- 17. Plunging ranula. [NTR-OR Nov 1992]
ment and complications of pleomorphic adenoma of 18. Mixed parotid tumour. [NTR-OR Oct 1996, Apr 1999,
parotid gland. [RURS Mar 2006] Oct 1999]
702 Quick Review Series for BDS 3rd Year

19. Warthin’s tumour. [NTR-OR Oct 1997, Oct 1999; TN 8. Pharyngocoele. [NTR-OR Oct 1993]
Aug 2009] 9. Tracheostomy. [NTR-OR Oct 1993; NTR-NR Oct
20. Sjögren’s syndrome. [NTR-NR Oct 2007] 2003, Oct 2007]
21. Calculi of salivary glands. [NTR-OR Oct 1997] 10. Tracheal injury. [NTR-OR Oct 2001]
22. Minor salivary gland tumours. [NTR-OR Apr 1997] 11. Endotracheal intubation. [NTR-OR Oct 2002]
12. Indications for the tracheostomy. [TN Apr 2000, Oct
2000, Apr 2001, Apr 2003, Feb 2010; NTR-NR Apr
9. INFECTIONS AND DISEASES
2006]
OF THE LARYNX AND NASOPHARYNX 13. Nasogastric intubation. [NTR-OR Oct 1993]
Long Essays 14. Dysphagia. [RGUHS Sep 1999, Sep 2003; NTR-OR
Apr 2003]
1. Describe the signs, symptoms and treatment of carci- 15. Peritonsillar abscess. [NTR-OR Oct 1995]
noma of larynx. [NTR-OR Oct 1990] 16. Retropharyngeal abscess. [NTR-NR Apr 2003]
2. Discuss the differential diagnosis of obstructive lesions 17. Tracheostomy. [TN Oct 1991, Apr 1993, Oct 1999, Apr
in the oesophagus. [NTR-OR Feb 1990] 2001, Feb 2006, Aug 2006; NTR-OR Apr 2000]
3. Mention the indications for tracheostomy. Describe the 18. Ryle’s tube. [NTR-OR Oct 1993]
steps of operation. [BUHS Jan 1990, Mar 1992] 19. Acute glottic oedema. [TN Apr 1993, Aug 2008]
4. Discuss the indications, types, postoperative manage-
ment and complications of tracheostomy. [RGUHS-RS
Aug 2006] 10. DISEASES OF THE ARTERIES, VEINS
AND LYMPHATIC SYSTEM
Short Essays Long Essays
1. Indications for tracheostomy. Mention its complica- 1. Describe the clinical features, diagnosis and treatment
tions. [RURS Mar 2006] of thromboangiitis obliterans. [NTR-OR Aug 1991]
2. Tracheostomy. [RGUHS Mar 2004] 2. Define gangrene. Describe the types, clinical manifes-
3. Retropharyngeal abscess. [MUHS Jan 1999] tation and management of wet gangrene. [NTR-NR
4. Chronic retropharyngeal abscess. [RGUHS-OS Aug Apr 2005]
2005] 3. Discuss the clinical features and treatment of gangrene.
5. Peritonsillar abscess. [RURS Aug 2006] [BUHS Feb 1996]
6. Acute follicular tonsillitis. [RGUHS Sep 2000] 4. Classify gangrene. Discuss the clinical features and
7. Indications of tracheostomy. [RGUHS Sep 2001; management of diabetic gangrene. [MUHS Jul 1995]
RGUHS-OS Mar 2006] 5. Describe the symptoms, signs and treatment of vari-
8. Postoperative care in patient following tracheostomy. cose veins of leg. [NTR-OR Nov 1992]
[RURS Aug 2005] 6. Describe the signs, symptoms and treatment of lymph-
9. Give in brief the indications, operative steps and post- adenitis in neck. [NTR-OR Oct 1993; TN Oct 1997,
operative complications of tracheostomy. [TN Aug Apr 1998]
2005] 7. Describe the aetiology, stages, clinical features and
10. What are the indications for tracheostomy? How do management of tuberculous lymphadenitis. [NTR-NR
you manage tracheostomy? [TN Oct 1998] Oct 2005; TN Apr 1996]
11. Quinsy. [NTR-NR Apr 2005] 8. Describe the clinical features, diagnosis and treatment
of tuberculous cervical lymphadenitis. [NTR-OR Oct
1994, Oct 2004; TN Apr 1998, Oct 1998]
Short Notes
9. Aetiology, clinical features and management of cervi-
1. Quinsy. [MUHS Jul 1990, Jan 1993, Jan 1997, Jan cal TB lymphadenitis. [NTR-NR Oct 2007]
1998; TN Apr 1993, Aug 2004; RGUHS Sep 2001; 10. Discuss the differential diagnosis of cervical lymph-
RURS Mar 2006; NTR-NR Apr 2004] adenopathy. [BUHS Mar 1994]
2. Tonsillitis. [MUHS Jul 1991, Jul 1995; NTR-OR Apr 11. What are the methods of spread of carcinoma? De-
1993, Oct 2002, Jul 2009] scribe the block dissection of neck. [BUHS Aug 1996]
3. Acute tonsillitis. [NTR-OR Apr 1997] 12. What are the causes of chronic lymphadenopathy of
4. Tracheitis. [NTR-NR 2005 Apr] neck and outline its management? [BUHS Oct 1987]
5. Pharyngitis. [NTR-NR 2004 Oct] 13. What are specific infections? Describe the aetiopathol-
6. Arterial supply of tonsils. [NTR-NR Apr 2007] ogy, clinical features and management of TB lymphad-
7. Collar-stud abscess. [NTR-NR Apr 2007] enitis. [BUHS Aug 1993]
Section | V Previous Years’ Question Bank 703

14. Describe the pathology, clinical features and manage- 20. Signs of aneurysm. [RGUHS Sep 2004]
ment of tubercular lymphadenitis of neck. [RGUHS 21. Hamartomas. [NTR-NR Apr 2007]
Sep 2001] 22. Haemangioma of tongue. [NTR-OR Oct 1997]
15. What are the different stages of tuberculous lymphad- 23. Leukopenia. [RURS Aug 2006]
enitis? How do you investigate and manage a case of 24. Lymphadenitis. [NTR-OR Oct 2004; TN Apr 2003]
tuberculous lymphadenitis? [RGUHS-OS Aug 2006] 25. Lymphosarcoma. [NTR-OR Oct 1996]
16. Describe the stages, investigations and management of 26. Tuberculous cervical lymphadenitis. [NTR-OR Apr
tuberculous lymphadenitis. [RGUHS-RS Aug 2005] 1997, Jul 2009, NTR-NR Oct 2003]
27. Clinical classification of Hodgkin’s lymphoma. [TN
Apr 1995, TN Feb 2006, TN Aug 2008]
Short Essays
28. Use of MRI in head and neck lesions. [TN Sep 2002,
1. Aneurysm. [RGUHS-RS Aug 2006; NTR-NR Mar Aug 2004]
2009] 29. Sternomastoid tumour. [TN Apr 1996, Apr 2000, Aug
2. Bed sores. [RGUHS Sep 2001] 2005]
3. Cervical rib. [RGUHS Sep 2003] 30. Ultrasonography. [TN Aug 2005]
4. Arteriovenous fistula. [RGUHS Sep 2001] 31. Commando operation. [TN Apr 1993]
5. Aetiology and clinical features of tuberculous cervical 32. Malignant secondary lymph node. [TN Oct 1998]
lymphadenitis. [RGUHS Apr 2003] 33. Waldeyer’s ring. [RGUHS Sep 1999]
6. Aneurysm. [NTR-NR Apr 2005] 34. Cystic hygroma. [BUHS Oct 1987, Feb 1993, Feb
7. Epistaxis. [NTR-NR Apr 2000] 1996, Feb 1997; MUHS Jan 1991]
8. Venous ulcer. [NTR-NR Apr 2005] 35. Lymphatic drainage of tongue. [RGUHS Sep 2000]
9. Thrombophlebitis. [NTR-NR Apr 2004] 36. Classification of Hodgkin’s lymphoma. [RGUHS Sep
10. Venous haemorrhage. [NTR-NR Apr 2007] 2001]
11. Von Recklinghausen’s disease. [NTR-NR Apr 2007] 37. Histological classification of Hodgkin’s lymphoma.
12. Lymphadenitis. [MUHS Jan 1992] [RGUHS Sep 2002]
13. Cystic hygroma. [RGUHS Sep 1999; RGUHS-OS Mar 38. Microscopic appearance in tuberculous lymphadenitis.
2006] [RURS Mar 2006]
14. Non-Hodgkin’s lymphoma. [RGUHS-OS Aug 2006] 39. Mycotic aneurysm. [NTR-OR Jul 2009]
15. Lymphoma staging. [RGUHS-OS Mar 2006] 40. Trendelenburg’s test for varicose veins. [NTR-OR Jul
16. Hodgkin’s lymphoma of neck. [RGUHS Apr 2001] 2009]
17. Staging of Hodgkin’s lymphoma. [RGUHS-RS Aug
2005]
11. DISEASES OF THE NERVOUS SYSTEM
Short Notes Short Essays
1. Embolism. [NTR-NR Apr 2005] 1. Bell’s palsy. [MUHS Jul 1991, Jul 1992, Jan 2000;
2. Arterial ulcer. [NTR-NR Oct 2005] RGUHS Sep 2003]
3. Haemangioma. [NTR-OR Jun 1982; TN Feb 2009] 2. Trigeminal neuralgia. [RGUHS Sep 2001; RUOS Aug
4. Aneurysm of aorta. [NTR-OR Apr 1993, Oct 2002] 2006]
5. Pulmonary embolism. [NTR-OR Oct 1993] 3. Electrocoagulation of trigeminal ganglion. [NTR-NR
6. Carotid body tumour. [NTR-OR Feb 1990, Nov 1992] Apr 2007]
7. Thromboangiitis obliterans. [NTR-OR Oct 1993] 4. Nerve grafting. [RGUHS-RS Aug 2005]
8. Aneurysm. [RGUHS-RS Aug 2005] 5. Neuropraxia. [NTR-OR Oct 1993]
9. Cervical rib. [MUHS Jan 1989] 6. Neurotmesis. [NTR-OR Aug 1991]
10. Arteriography. [RGUHS Sep 2003] 7. Axonotmesis. [NTR-OR Jun 1989, Oct 1995]
11. Hodgkin’s disease—neck. [NTR-OR Oct 1995]
12. Aneurysm. [RGUHS-RS Aug 2001]
Short Notes
13. Raynaud’s disease. [RGUHS Mar 2004]
14. Cricoid aneurysm. [RGUHS Apr 2003] 1. Bell’s palsy. [NTR-OR Apr 1998, Oct 2001]
15. Treatment of cervical rib. [RGUHS Mar 2004] 2. Facial nerve palsy. [NTR-OR Oct 1994; TN Apr 1993,
16. Subclavian steal syndrome. [RGUHS Sep 2003] Aug 2008]
17. Carotid body tumour. [RGUHS-RS Feb 2007] 3. Trigeminal neuralgia. [NTR-OR Apr 1995, Oct 1997;
18. Varicose vein. [NTR-OR Apr 1995, Oct 1999] NTR-NR Oct 2006; TN Oct 1998, Sep 2002, Aug 2004,
19. Pulmonary embolism. [NTR-OR Oct 1993] Aug 2005]
704 Quick Review Series for BDS 3rd Year

4. Facial palsy. [TN Aug 2008] 7. Immediate complications of fracture. [NTR-NR Apr 2007]
5. Neuropraxia. [TN Aug 2006; TN R Apr 2000; NTR- 8. Head injury management. [RURS Mar 2006]
OR Jul 2009] 9. Artificial respiration. [NTR-OR Oct 1998]
6. Auriculotemporal (Frey’s) syndrome. [TN Feb 2007] 10. Extradural haematoma. [RGUHS Apr 2000]
7. Local anaesthesia in dentistry. [TN Oct 1997] 11. Depressed fracture skull. [BUHS Aug 1991]
8. Frey’s syndrome. [TN Oct 1999] 12. Cardiopulmonary resuscitation. [NTR-NR Apr 2003]
9. Facial nerve palsy. [RGUHS Sep 1999; MUHS Jun 1989] 13. Temporomandibular dislocations. [NTR-OR Apr 1993]
10. Trigeminal neuralgia. [RGUHS Sep 2000; NTR-NR 14. Dislocation of TMJ. [MUHS Jul 1991]
Mar 2009] 15. Mandibular dislocation. [MUHS Jul 2000]
11. Horner’s syndrome. [RGUHS-OS Aug 2006] 16. Extradural haematoma. [RGUHS Apr 2001]
12. Frey’s syndrome. [RUOS Mar 2006] 17. Subdural haematoma. [MUHS Jul 1998]
13. Neuropraxia. [RUOS Aug 2005, Aug 2006; RURS Feb 18. Black eye. [MUHS Jul 1991, Jan 1999]
2007] 19. Pulmonary resuscitation. [NTR-NR Apr 2003]
14. Axonotmesis. [NTR-OR Jun 1989] 20. Epistaxis. [NTR-OR Apr 1999]
21. Le Fort I fracture. [NTR-NR Jul 2009]
22. Compound fracture. [NTR-OR Jul 2009]
12. FRACTURES: GENERAL PRINCIPLES
Long Essays 13. ANOMALIES OF DEVELOPMENT
1. Le Fort’s classification of fractures of maxilla. [NTR- OF FACE (CLEFT LIP AND CLEFT PALATE)
NR Apr 2006]
Long Essays
2. Discuss the management of maxillofacial injuries.
[NTR-OR Oct 2003] 1. Classification of cleft lip and palate and their effect
3. Classify fractures of face and describe management of upon teeth and eating. [NTR-NR Oct 2007]
each type of fracture. [NTR-OR Apr 1997; TN Aug 2006] 2. Describe the congenital anomalies of faciomaxillary
4. Classify the fractures. Describe the treatment of frac- region and describe the management of cleft palate.
tured mandible. [MUHS Jan 1991] [NTR-OR Jul 1989; TN Aug 2005, Aug 2006]
5. Clinical signs, symptoms and general principles of 3. Describe the aetiology, pathophysiology, clinical fea-
treatment of fracture. [MUHS Jan 1998] tures and management of cleft lip and palate. [NTR-NR
Apr 2004]
4. How do you classify cleft lip and principles of manage-
Short Essays ment? [MUHS Jul 1999]
1. Fracture mandible. [MUHS Oct 1987] 5. Discuss the classification, clinical features and treatment
2. Compound fractures. [RGUHS Sep 2001] of cleft lip and cleft palate. [BUHS Aug 1988, Aug 1995]
3. Pathological fractures. [RGUHS Sep 2003] 6. Describe the pathogenesis, types and management of
4. Fracture healing. [RGUHS-RS Aug 2006] cleft palate. [MUHS Feb 2000]
5. Malunion. [NTR-NR Apr 2007] 7. Describe clinical features and management of cleft lip.
6. Definition of dislocation and subluxation. [NTR-NR [NTR-NR Mar 2009]
Apr 2007]
7. Nonunion of fractures. [TN Aug 2004]
Short Essays
8. General management of a patient with head injury.
[RGUHS-RS Aug 2005] 1. Cleft lip. [RGUHS Sep 2001; RGUHS-RS Aug 2006]
9. Unilateral proptosis. [NTR-NR Mar 2009] 2. Cleft palate. [RGUHS Sep 1999, Mar 2005; RGUHS-
OS Mar 2006]
3. Development of face. [NTR-OR Oct 1998]
Short Notes
1. Clicking jaw. [NTR-OR Jul 1989; TN Apr 2008]
Short Notes
2. Black eye. [NTR-OR Mar 1991]
3. Compound fracture. [NTR-OR Oct 1993; TN Oct 1. Cleft lip. [NTR-OR Oct 1996, Apr 1997; TN Apr 1998,
2007] Apr 2001, Oct 2007; TN R Oct 2003]
4. Fracture of long bones. [NTR-OR Aug 1991] 2. Cleft palate. [NTR-OR Oct 1994, Oct 1995, Oct 1997,
5. Signs of fracture of long bones. [NTR-OR Nov 1992] Apr 1998, Apr 2000, Oct 2001; NTR-NR Oct 2005; TN
6. Temporomandibular joint dislocation. [NTR-OR Apr Apr 2001, Feb 2005, Oct 2006, Feb 2007, Feb 2010]
1986, Apr 1991, Apr 1993, Apr 1995; NTR-R Apr 3. Cleft lip. [MUHS Jul 1995, Jul 1997, Oct 1997, NTR-
1996, Oct 2002; NTR-NR Apr 2004, Mar 2009] OR Jul 2009, BUHS Aug 1993]
Section | V Previous Years’ Question Bank 705

4. Cleft palate. [MUHS Jan 1992, Jan 1996, Jan 1997; 19. Classify goitres. Discuss the signs, symptoms and
RUOS Aug 2005] management of a solitary nodule of thyroid. [MUHS
5. Harelip. [NTR-NR 2005 Oct] Jan 2000]
20. Discuss the clinical features and management of toxic
goitre. [MUHS Jan 1992]
14. DISEASES OF THYROID 21. What is toxic goitre? What are the differences between
AND PARATHYROID GLANDS primary and secondary goitre? Outline the treatment of
primary toxic goitre of thyroid gland. [BUHS Mar
Long Essays
1988]
1. Describe clinical features, diagnosis and management 22. Classify neoplasm of thyroid gland. Outline the clini-
of carcinoma of thyroid. [NTR-OR Apr 1999; TN Apr cal manifestations, diagnosis and management of pap-
2001, Aug 2006] illary carcinoma of thyroid gland. [BUHS Aug 1996;
2. Describe clinical features, diagnosis and management MUHS Jun 1999]
of primary thyrotoxicosis. [NTR-OR 1996 Oct; TN 23. Classify goitres. How do you investigate and man-
Apr 1996, Oct 1998] age a case of hyperthyroidism? [RGUHS-RS Aug
3. Describe the signs, symptoms and treatment of com- 2005]
mon primary malignant lesions in thyroid. [NTR-OR 24. Describe the clinical features, diagnosis and manage-
Jan 1992] ment of hyperthyroidism. [RGUHS Sep 2003]
4. Discuss Graves’ disease in detail. [TN Feb 2010] 25. Describe the aetiopathology and clinical features of
5. Classify thyroid tumours. Discuss the aetiopathology primary hyperthyroidism. [MUHS Feb 2000]
and the management of differentiated thyroid malig-
nancies. [TN Oct 1998, Aug 2004]
6. Describe the aetiology, clinical features and manage-
ment of thyrotoxicosis. [RGUHS Apr 2000]
Short Essays
7. Discuss clinical features and management of carci- 1. Tetany. [MUHS Jan 1992, Jul 1998, RGUHS Mar
noma of thyroid. [BUHS Aug 1995] 2004, RUOS Mar 2006]
8. Describe clinical features, diagnosis and management 2. Hypocalcaemia (tetany). [RGUHS-RS Mar 2006]
of carcinoma of thyroid. [RGUHS Sep 2001] 3. Trismus. [BUHS Apr 1987, Mar 1988, Sep 1994, Mar
9. Describe the clinical features, diagnosis and treatment 1995; MUHS Jan 1991, Jan 1999, Jul 1999]
of primary thyrotoxicosis. [MUHS Jun 1989, Jan 1999; 4. Thyroid crisis. [RGUHS-RS Aug 2006]
BUHS Sep 1994, Feb 1996] 5. Radiotherapy. [RGUHS Sep 2000]
10. Describe the clinical features and management of pri- 6. Adenoma thyroid. [MUHS Jan 1993]
mary thyrotoxicosis. [MUHS Jul 1990, Jan 1996] 7. Multinodular goitre. [MUHS Jan 1991]
11. Discuss aetiopathogenesis, signs, symptoms and treat- 8. Hashimoto’s thyroiditis. [RGUHS Sep 2000]
ment of primary thyrotoxicosis. [RGUHS Mar 2004] 9. Solitary nodule of thyroid. [RGUHS Sep 2002]
12. Discuss the management of solitary nodule thyroid. 10. Primary thyrotoxicosis. [RGUHS-RS Feb 2007]
[MUHS Jul 1991] 11. Follicular carcinoma of thyroid. [RGUHS-RS Mar
13. Describe the aetiopathology, clinical features and treat- 2006]
ment of solitary nodule in thyroid gland. [MUHS Aug 12. Management of papillary carcinoma of thyroid.
1991] [RGUHS Mar 2004]
14. Discuss clinical features, treatment of multinodular 13. Thyroid nodule. [NTR-NR Oct 2005]
goitre. Mention four complications of multinodular 14. Thyroid storm. [TN Feb 2006, Feb 2008]
goitre. [RGUHSI Sep 2004] 15. Autoimmune thyroiditis. [TN Oct 1996, Apr 1998, Aug
15. How do you classify goitre? Discuss the clinical fea- 2008]
tures, investigations and management of papillary car- 16. Retrosternal goitre. [TN Apr 1995, Nov 2001, Aug 2005]
cinoma. [RGUHS-OS Mar 2006] 17. Classification of goitres. [NTR-NR Mar 2009]
16. Classify goitre. Discuss the clinical features and man-
agement of multinodular goitre. [MUHS Jan 1990]
Short Notes
17. Classify goitres. Discuss the aetiology, clinical fea-
tures, investigations and management of primary thy- 1. TSH. [RGUHS-OS Aug 2006]
rotoxicosis. [RGUHS-RS Aug 2007] 2. Goitre. [MUHS Jul 2000]
18. Classify neoplastic goitres. Describe the pathology, 3. Quinsy. [RGUHS Sep 2001]
clinical features and management of follicular carci- 4. Thyroid scan T131. [RUOS Aug 2006]
noma of thyroid gland. [RGUHS Apr 2003, Mar 5. Cobalt. [MUHS Jul 1990]
2005] 6. Thyrotoxicosis. [RURS Mar 2006]
706 Quick Review Series for BDS 3rd Year

7. Thyroid crisis. [BUHS Oct 1987] 5. Anaphylaxis. [RURS Aug 2006]


8. Exophthalmos. [MUHS Jul 1991; BUHS Feb 1993] 6. Radiation therapy. [RGUHS Sep 2000]
9. Tetany. [BUHS Aug 1993, Mar 1997; RGUHS Sep 7. Oral submucous fibrosis. [RGUHS Sep 2000, Sep
2001; RURS Feb 2007] 2003; RGUHS-OS Mar 2006]
10. Trismus. [RGUHS Sep 1999; RGUHS-OS Aug 2005] 8. Horner’s syndrome. [RURS Feb 2007]
11. Retrosternal goitre. [RGUHS-RS Mar 2006] 9. Principles of antimicrobial treatment. [RGUHS Sep
12. Parathyroid tetany. [RGUHS Sep 2002] 2001]
13. Hoarseness of voice. [RGUHS Sep 1999] 10. Mechanism of action LA. [NTR-OR Aug 1991]
14. Causes for dyspnoea in goitre. [RURS Feb 2007] 11. Mandibular nerve block. [NTR-OR Apr 1997]
15. Medullary carcinoma of thyroid. [RUOS Aug 2005]
16. Metastasis in papillary carcinoma of thyroid. [RURS
Aug 2007]
Short Notes
17. Primary thyrotoxicosis—medical management. [RGUHS
Sep 2004] 1. Pain. [NTR-NR Apr 2004, Mar 2009]
18. Complications of thyroid surgery. [RGUHS Sep 2003] 2. Adenoid. [NTR-OR Apr 1998]
19. Postoperative complications in thyroid surgery. 3. Otitis media. [NTR-NR Oct 2004]
[RGUHS Apr 2001] 4. Chemotherapy. [NTR-NR Apr 2005]
20. TSH. [NTR-OR Oct 2004] 5. Insulin. [NTR-NR Apr 2006]
21. Quinsy. [NTR-OR Feb 1990; NTR-OR Oct 1999; 6. Penicillin. [NTR-NR Oct 2004]
NTR-NR Oct] 7. Antioxidant. [NTR-NR Apr 2003]
22. Trismus. [NTR-NR Apr 2003] 8. Local anaesthesia. [NTR-OR Aug 1991, Oct 1998]
23. Thyroglossal cyst. [NTR-OR Oct 1995, TN Oct 1993, 9. Spinal anaesthesia. [NTR-OR Apr 1993, Feb 2002]
Oct 1996, Oct 1997] 10. Topical anaesthesia. [NTR-OR Oct 2004]
24. Solitary nodule thyroid. [NTR-OR Oct 1999; TN Oct 11. Surface acting anaesthesia. [NTR-NR Oct 2006]
1997] 12. Prophylactic antibiotics. [NTR-NR Apr 2006]
25. Glossitis. [TN Apr 1993, Aug 2005] 13. Immunity. [NTR-OR Apr 1993, Oct 2002]
26. Plunging goitre. [TN Oct 1998] 14. Cytokines. [NTR-NR Apr 2003]
15. Cell-mediated immunity. [NTR-OR Apr 2002; NTR-
NR Apr 2003]
15. LOCAL ANAESTHESIA AND BIOPSY 16. Oral hygiene. [NTR-OR Oct 1993]
17. Raynaud’s disease. [RGUHS Mar 2004]
Short Essays 18. T lymphocyte. [RGUHS Sep 2002]
1. Local anaesthesia. [MUHS Jan 1991] 19. Space of burns. [BUHS Aug 1989]
2. Complications of local anaesthesia. [BUHS Mar 1995] 20. Transillumination. [BUHS Mar 1992]
3. Antibioma. [RGUHS Sep 1999] 21. Cock’s peculiar tumour. [RURS Aug 2007]
4. FNAC. [RGUHS Apr 2000; RUGS Mar 2006; RURS 22. Antibioma. [RGUHS Sep 2002]
Aug 2007] 23. Torticollis. [BUHS Aug 1992]

Section III

Oral Pathology
1. DEVELOPMENTAL DISTURBANCES 2. Enumerate and describe developmental disturbances
OF ORAL AND PARAORAL STRUCTURES affecting the shape of teeth. [NTR-OR Oct 1999,
Apr 2001]
Long Essays
3. Enumerate the various causes of enamel hypoplasia
1. Write an essay on developmental abnormalities of the and describe hypoplasia of teeth associated with fluo-
teeth. [NTR-OR Apr 2000; TN Apr 1996, Nov 2001] rosis. [NTR-OR Apr 1992]
Section | V Previous Years’ Question Bank 707

4. Enumerate the developmental disturbances in the for- 10. Ghost teeth. [RUOS Mar 2006; RURS Mar 2006]
mation of dentin. Write in detail the clinical types, 11. Fusion, gemination and concrescence. [MUHS Jan
clinical features, radiographic features and histopatho- 1999]
logical features of dentin dysplasia. [NTR-NR Apr 12. Supernumerary teeth. [RURS Mar 2006]
2005] 13. Write on supernumerary teeth. [BUHS Feb 1993]
5. Describe in brief developmental disturbances of teeth. 14. Dense evaginatus. [RURS Aug 2005]
[MUHS Jul 1991, Jan 1997; TN Aug 2009] 15. Dentinogenesis imperfecta. [MUHS Mar 1988, Jul
6. Discuss the developmental disturbances affecting the 1990; BUHS Mar 1992; RGUHS Apr 2000]
morphology of tooth. [MUHS Nov 1986, Jan 1996] 16. Write about dentinogenesis imperfecta. [BUHS Mar
7. Enumerate the developmental anomalies affecting the 1981; MUHS Jul 1990; RGUHS Apr 2000]
shape of the tooth. Write in detail on the dens invagina- 17. Developmental anomalies affecting the shape of teeth.
tus and dens evaginatus. [RGUHS Apr 2003] Write about taurodontism. [RGUHS Sep 2001]
8. Enumerate the developmental anomalies affecting the 18. Write on enamel hypoplasia. [BUHS Mar 1997]
shape of the tooth. Write in detail on the amelogenesis 19. Fusion. [NTR-NR Apr 2004]
imperfecta and enamel hypoplasia. [RUGHS Aug 2005] 20. Enameloma. [NTR-NR Apr 2007]
9. Write in detail about the diseases of the tongue. [BUHS 21. Dens in dente. [NTR-OR Oct 2004; TN Feb 2010]
Sep 19941; MUHS Jan 2000] 22. Dens evaginatus. [NTR-NR Apr 2007]
10. Enumerate developmental disturbances affecting 23. Concrescence. [NTR-OR Oct 2004]
tongue. Describe any three of them. [MUHS Jul 1987; 24. Fordyce granules. [NTR-NR Oct 2002; TN Apr 1995]
TN Aug 2006] 25. Torus mandibularis. [NTR-NR Oct 2005, Apr 2007]
11. Describe enamel hypoplasia in detail. [MUHS Jul 26. Hunter’s glossitis. [NTR-NR Apr 2007]
1996; TN Apr 1995] 27. Moeller’s glossitis. [NTR-NR Oct 2007]
12. Classify enamel hypoplasia. Describe the pathogenesis 28. Turner’s hypoplasia. [NTR-NR Oct 2002, Oct 2005,
and clinical features of enamel hypoplasia due to con- Oct 2006]
genital syphilis. [MUHS Jul 1989] 29. Mottled enamel. [NTR-NR Oct 2007]
13. Classify enamel hypoplasia. Describe the aetiopatho- 30. Regional odontodysplasia. [TN Sep 2002]
genesis and clinical features of mottled enamel. 31. Benign lymphoepithelial cyst. [TN Oct 2000, Sep
[MUHS Jan 1990] 2002]
14. Enumerate the environmental causes for enamel hypo-
plasia. Write in detail about them. [RGUHS Sep 2002;
RUOS Mar 2005]
Short Notes
15. Define and classify hypoplasia of teeth. Describe in 1. Anodontia. [NTR-OR Oct 1997; TN Apr 1996, Feb
detail dentinogenesis imperfecta. [MUHS Jul 1997] 2005, Aug 2006, Aug 2008]
16. List the developmental abnormalities of teeth with re- 2. Xerostomia. [NTR-NR Oct 2002]
spect to its structure. Give a detailed account on den- 3. Gemination. [NTR-NR Apr 2004, Apr 2007; TN Nov
tinogenesis imperfecta. [RURS Aug 2007] 2001]
17. Clinical features of geographic tongue. [TN Apr 1993] 4. Fused tooth. [NTR-OR Oct 1997]
18. Discuss the morphological disturbances of tooth due to 5. Dens in dente. [NTR-OR Apr 1993; NTR-NR Apr
developmental disorders. [TN Oct 1997] 2007; TN Apr 2000]
6. Dens invaginatus. [NTR-OR 1990; TN Apr 1996]
7. Taurodontism. [NTR-OR Apr 1992, Apr 2006; TN Oct
Short Essays 1999, Sep 2002, Oct 2003, Feb 2009]
1. Herpangina. [RGUHS Apr 2001] 8. Unerupted teeth. [NTR-OR Apr 2000]
2. Cleft palate. [RGUHS Mar 1992; BUHS Mar 1992, 9. Tooth ankylosis. [NTR-OR Apr 1997]
Mar 1994; MUHS Jan 1992] 10. Supernumerary teeth. [NTR-OR Oct 1996, Oct 1998;
3. Write on xerostomia. [RGUHS Apr 2000] TN Oct 1998, Oct 1999, Aug 2006]
4. Median rhomboid glossitis. [MUHS Jul 1989, Jan 11. Dentin dysplasia. [NTR-OR Oct 1998]
1998; TN Oct 1996, Feb 2006] 12. Dentinogenesis imperfecta. [NTR-OR Apr 1998]
5. Micrognathia and macrognathia. [MUHS Jul 1999] 13. Amelogenesis imperfecta. [NTR-OR Oct 1995, Oct
6. Anodontia. [BUHS Apr 1987; MUHS Jun 1989, Jul 1998, Oct 2004]
1998; RURS Aug 2005; TN Apr 1996, Feb 2005] 14. Fluorosis. [NTR-NR Oct 2004]
7. Write on anodontia. [BUHS Apr 1987; MUHS Jun 15. Mottled enamel. [NTR-OR Apr 1998, Apr 1999; NTR-
1989] NR Oct 2002; TN Apr 1993, TN Oct 2000]
8. Taurodontism. [RURS Feb 2007] 16. Hypoplasia of teeth. [NTR-OR Apr 1992, Oct 1996]
9. Shell teeth. [RURS Feb 2007] 17. Chronologic hypoplasia. [NTR-NR Apr 2007]
708 Quick Review Series for BDS 3rd Year

18. Epidermoid cyst. [NTR-OR Apr 1999] 55. Describe endemic fluorosis. [MUHS Jul 1997]
19. Globulomaxillary cyst. [NTR-OR Oct 1992; NTR-NR 56. Peutz-Jeghers syndrome. [RURS Aug 2007]
Apr 2004]
20. Bohn’s nodules. [NTR-OR 1998 Oct, Oct 1999]
21. Fordyce’s granules. [NTR-OR Apr 1997; RUOS Aug 2. BENIGN AND MALIGNANT TUMOURS
2005] OF THE ORAL CAVITY
22. Gardner’s syndrome. [NTR-NR Nov 2005]
Long Essays
23. Median rhomboid glossitis. [NTR-OR Apr 1997; NTR-
NR Apr 2007; TN Oct 2000] 1. Mention nonodontogenic malignant tumours of epithe-
24. Macroglossia. [NTR-OR Apr 1989] lial tissue of oral mucosa. Describe clinical and histo-
25. Fusion. [BUHS Feb 19931; MUHS Jan 1998; RGUHS logical features of verrucous carcinoma. [RGUHS Jan
Sep 2004] 2009]
26. Gemination. [MUHS Jan 1990; RGUHS Sep 1999, Apr 2. Describe the pathogenesis, clinical radiograph appear-
2000; TN Feb 2005] ance of central ossifying fibroma with histological
27. Fusion and germination. [BUHS Aug 1931] features. [NTR-NR Nov 2005]
28. Shell teeth. [MUHS Jul 1998; RGUHS Mar 2004] 3. Describe and enumerate the pre-cancerous lesions and
29. Ghost teeth. [MUHS Jul 1989; RGUHS Sep 2002, Mar conditions of oral cavity. Describe the aetiology, clini-
2005; RGUHS RS 2009] cal features and histopathology of leukoplakia. [NTR-
30. Dilaceration. [MUHS Jul 1990, Jul 1991; RUGS Aug OR Apr 1999]
2006; TN Nov 2001; BUHS Mar 1994] 4. Enumerate the precancerous lesions. Write in detail
31. Talons cusp. [MUHS Jan 1995; RGUHS Sep 2002; about aetiology, clinical features and histopathology of
RUGS Mar 2005; TN Aug 2005] erythroplakia. [NTR-OR Oct 2000]
32. Concrescence. [MUHS Jun 1989; RGUHS Apr 2001; 5. What is erythroplakia? Give clinical and histological
RGUHS RS Jan 2009] features of speckled and homogenous types of erythro-
33. Taurodontism. [BUHS May 1986, Mar 1988, Feb plakia. [NTR-NR Apr 2007]
1991, Aug 1993; MUHS Jan 1998] 6. Mention the various malignant tumours of salivary
34. Submerged teeth. [MUHS Jan 2000; RGUHS Sep glands. Write in detail clinical and histological features
1999; RUGS Aug 2006] of adenoid cystic carcinoma. [NTR-NR Apr 2007]
35. Supernumerary teeth. [BUHS Feb 19931; MUHS Feb 7. Describe the aetiology, clinical features and histopa-
1993; RGUHS Apr 2002; TN Oct 1997] thology of verrucous carcinoma. [TN Oct 1996, Apr
36. Ramsay-Hunt syndrome. [RUGS Mar 2005, Mar 2006] 2003, Aug 2006, Aug 2007]
37. Melkersson–Rosenthal syndrome. [RGUHS Apr 2003; 8. Define premalignant lesion and premalignant condi-
TN Aug 2007] tion. Give examples of each. Discuss in detail leuko-
38. Syphilitic teeth. [MUHS Jan 1997] plakia. [TN Aug 2006]
39. Hutchinson’s triad. [RGUHS Sep 2004] 9. Pathogenesis of oral submucous fibrosis. [TN Oct
40. Cementicles. [RGUHS Apr 2001] 1996]
41. Dens in dente. [BUHS Mar 1941; MUHS Jul 1996] 10. Write about giant cell lesions of oral cavity. [BUHS
42. Dentin dysplasia. [MUHS Jan 1995; TN Feb 2007] Sep 1994]
43. Dens invaginatus. [BUHS Aug 1995; MUHS Aug 11. Define leukoplakia and describe in detail the aetiology,
1995; TN Apr 1999] clinical features and histopathology of leukoplakia.
44. Turner’s hypoplasia. [BUHS Apr 1987, Oct 1987, Mar [RURS Aug 2006; TN Sep 2002, Feb 2005]
1992; MUHS Jan 1998; RGUHS Sep 1999, Sep 2001] 12. Enumerate white lesions of oral cavity and describe in
45. Enamel hypoplasia. [BUHS Sep 1994; TN Oct 1998] detail about leukoplakia. [TN Apr 1998; BUHS Feb
46. Mottled enamel. [BUHS Aug 1995; RGUHS Sep 2004] 1991, Sep 1992, Mar 1994]
47. Fordyce’s spot. [BUHS Mar 1988, Mar 1992, Aug 13. Enumerate premalignant lesions of the oral cavity. De-
1995, Sep 1996; RURS Aug 2006] scribe in detail about leukoplakia. [BUHS Sep 1992;
48. Causes of xerostomia. [RGUHS Sep 2000] MUHS Jan 1999]
49. Dermoid cyst. [MUHS Jan 1990] 14. Describe the aetiology, clinical features and histopa-
50. Developmental cysts. [MUHS Jan 2000] thology of oral leukoplakia. [MUHS 1988]
51. Globulomaxillary cyst. [BUHS Oct 1987, Feb 1993; 15. Enumerate premalignant lesions. Describe aetiology,
MUHS Sep 1991, Jul 1997] clinical features and histopathology of erythroplakia.
52. Hairy tongue. [TN Aug 2007] [MUHS Jan 1988; TN Oct 1999]
53. Odontodysplasia. [TN Feb 2005] 16. Compare and contrast between leukoplakia and lichen
54. Black hairy tongue. [NTR-OR Oct 2005] planus. [BUHS Aug 1993]
Section | V Previous Years’ Question Bank 709

17. Enumerate the dermatological disorders affecting the 13. Write about verrucous carcinoma. [MUHS Aug 1991]
oral mucous membrane and describe in detail the aeti- 14. Histology of papilloma. [RGUHS RS Mar 2006]
ology, clinical features, histopathology and manage- 15. Histology of Kaposi’s sarcoma. [RGUHS RS Aug 2007]
ment of oral lichen planus. [RUGHS RS Aug 2006] 16. Histology of Burkitt’s lymphoma. [RGUHS-RS Feb
18. Enumerate premalignant conditions of the oral cavity. 2007]
Give an account on oral submucous fibrosis with refer- 17. Squamous cell carcinoma. [BUHS Sep 1994]
ence to aetiology, clinical features and histology. 18. Histopathology of well-differentiated squamous cell
[RUGHS RS Feb 2007] carcinoma. [BUHS Sep 1994]
19. Define precancerous lesions and conditions, enumerate 19. Giant cell granuloma. [RGUHS Sep 2001]
the same. Write on the clinical features, aetiopathogen- 20. Peripheral giant cell granuloma. [BUHS Apr 1987;
esis and histopathological features of oral submucous MUHS Jan 1991, Feb 1991]
fibrosis. [RGUHS Apr 2003; TN Apr 1993, Oct 1997, 21. Histopathology of peripheral giant cell granuloma.
Aug 2009] [BUHS Mar 1988, Aug 1993]
20. Define premalignant lesions and conditions with ex- 22. Osteosarcoma. [RGUHS RS Jan 2009]
amples. Write in detail the clinical and histopathologi- 23. TNM classification. [RURS Aug 2007]
cal features of oral submucous fibrosis. Add a note on 24. Fibrous epulis. [NTR-OR Oct 1992]
its pathogenesis. [RUGHS OS Aug 2005] 25. Torus mandibularis. [NTR-NR Apr 2007]
21. Enumerate the premalignant lesions of the oral cavity. 26. Reed-Sternberg cells. [NTR-NR Apr 2007]
Describe the aetiopathogensis, clinical features and 27. Types of leukoplakia. [NTR-NR Oct 2007]
histopathologic features of oral submucous fibrosis. 28. Nevus. [TN Aug 2006]
[MUHS Jun 1989, Jan 1996, Jul 1997, Jan 1998] 29. Leukoplakia. [TN Apr 1998, Aug 2006]
22. Describe the clinical features and histopathology of 30. Osteogenic sarcoma. [TN Sep 2002]
epidermoid carcinoma of oral cavity. [MUHS Jun 31. Epithelial dysplasia. [TN Oct 1998]
1989, Jan 1998; BUHS Mar 1992] 32. Haemangioma. [TN Apr 1998]
23. Enumerate the malignant nonodontogenic tumours of 33. Blue nevus. [RGUHS Jan 2009]
epithelial origin. Write in detail on most common ma-
lignant neoplasm of oral cavity. [RGUHS Apr 2002]
24. Describe the aetiology, clinical features and histopa-
Short Notes
thology of oral squamous cell carcinoma. [BUHS Mar 1. Noma. [NTR-NR Oct 2007]
1992; TN Oct 1996, Oct 1998, Oct 2000, Feb 2005] 2. Fibroma. [NTR-OR Apr 2006]
25. Classify white lesions of the oral cavity. Describe in 3. Papilloma. [NTR-OR Apr 1997, Apr 2001; NTR-NR
detail oral submucous fibrosis. [TN Oct 1991] Oct 2007; TN Oct 1996, Apr 2000, Apr 2001, Feb
26. Discuss oral submucous fibrosis in detail. [TN Apr 2005, Feb 2006]
1996] 4. Leukoplakia. [NTR-OR Oct 1995]
27. Enumerate precancerous diseases of oral mucosa. De- 5. Histopathology of leukoplakia. [NTR-OR Oct 1999]
scribe in detail about leukoplakia. [NTR-OR Apr 1993] 6. Histopathology of lichen planus. [NTR-NR Apr 2007]
7. Hairy leukoplakia. [NTR-OR Apr 1998]
8. Erythroplakia speckled. [NTR-NR Oct 2006]
Short Essays 9. Oral submucous fibrosis. [NTR-OR Oct 1998, Apr
1. Tori. [RURS Feb 2007] 1999, Apr 2002; TN Oct 2003, Feb 2006; NTR-NR
2. Leukoedema. [RURS Feb 2007] Apr 2005]
3. Erythroplakia. [RGUHS Sep 1999; TN Oct 1999, Apr 10. Traumatic neuroma. [NTR-OR Apr 1993]
2000, Apr 2003, Aug 2005] 11. Giant cell granuloma. [NTR-OR Apr 1997, Apr 1999]
4. Aetiology of leukoplakia. [MUHS Nov 1986] 12. Basal cell carcinoma. [NTR-OR Oct 1996, Oct 1999]
5. Histology of leukoplakia. [RURS Aug 2007] 13. Histopathology of squamous cell carcinoma. [NTR-NR
6. Write about leukoplakia. [MUHS Nov 1986] Oct 2002, Apr 2004; TN Oct 1999, Aug 2004, Feb 2006]
7. Oral submucous fibrosis. [MUHS Jul 1996, Jul 1997] 14. Epulis. [MUHS Oct 1987]
8. Histopathology of oral submucous fibrosis. [RURS 15. Congenital epulis. [MUHS Jun 1989]
Aug 2006] 16. Fibroma. [BUHS Mar 1997]
9. Carcinoma in situ. [BUHS Mar 1992, Mar 1999, Sep 17. Neurofibroma. [BUHS Mar 1995]
2002] 18. Haemangioma. [MUHS Sep 1991, Jan 2000]
10. Explain epithelial dysplasia. [MUHS Jan 1990] 19. Papilloma. [BUHS Aug 1995; MUHS Jul 1999, Jul
11. Features of epithelial dysplasia. [RUGHS OS Aug 2006] 2000]
12. Verrucous carcinoma. [RGUHS RS Aug 2005] 20. Exostoses. [BUHS Feb 1993; MUHS Feb 1993]
710 Quick Review Series for BDS 3rd Year

21. Leukoplakia. [BUHS Mar 1997; TN Aug 2004] 6. Classify salivary gland neoplasms. Discuss in detail
22. Dyskeratosis. [BUHS Oct 1987] mucoepidermoid carcinoma. [TN Apr 1998, Aug
23. Torus palatinus. [RGUHS Sep 2000] 2006]
24. Keratoacanthoma. [MUHS Jun 1989; TN Apr 1993] 7. Classify salivary gland tumours. Discuss in detail
25. Carcinoma-in-situ. [RGUHS Apr 2001; TN Feb 2005] about histopathology and clinical features of mucoepi-
26. Features of epithelial dysplasia. [BUHS Jan 1990; dermoid carcinoma. [RGUHS Apr 2000, Mar 2005]
MUHS Jan 1996, Jul 1998] 8. Classify salivary gland tumours. Write in detail about
27. Reed-Sternberg cells. [RGUHS Apr 2001, Mar 2004] clinical features and histopathology of mucoepider-
28. Bence Jones protein. [BUHS Feb 1993] moid carcinoma. [RGUHS Sep 2002]
29. Café au lait spots. [MUHS Jan 1998] 9. Classify salivary gland neoplasms and describe in de-
30. Kaposi’s sarcoma. [BUHS Aug 1988] tail pleomorphic adenoma. [RUOS Nov 1986; TN Apr
31. Verrucous carcinoma. [MUHS Jul 1990, Jul 1991, Jul 1993]
1996] 10. Classify the salivary gland tumours and describe briefly
32. Rodent ulcers. [RGUHS Mar 2004] the pathogenesis, clinical features and histology of the
33. Basal cell carcinoma. [MUHS Jul 1987] pleomorphic adenoma. [MUHS Nov 1986; BUHS Feb
34. Stages and grading in squamous cell carcinoma. 1993, Mar 1994; TN Apr 1996, Oct 1996]
[RGUHS Sep 2004] 11. Classify tumours of salivary glands. Write in detail
35. Squamous cell carcinoma of buccal mucosa. [MUHS about adenoid cystic carcinoma. [BUHS Sep 1994;
Jul 1997] MUHS Jul 1998]
36. Peripheral giant cell granuloma. [BUHS Apr 1987; TN 12. Enumerate the malignant tumours of the salivary
Feb 2005] glands. Describe in detail adenoid cystic carcinoma.
37. Radiological and biological findings of multiple my- [RGUHS Apr 2001]
eloma. [RGUHS Sep 2000] 13. Classify the salivary gland neoplasms. Discuss in detail
38. Submucous fibrosis. [TN Apr 2001, Aug 2007] the clinical features and histopathology of Sjögren’s
39. Lupus erythematosus. [TN Aug 2007] syndrome. [TN Apr 2001]
40. Leukocytosis. [TN Aug 2004]
41. Squamous cell carcinoma of cheek. [TN Oct 1991]
42. Junctional nevus. [TN Apr 1993]
Short Essays
43. Torus palatinus. [TN Apr 1996] 1. Mumps. [RGUHS Sep 2001]
44. Tori. [RGUHS Jan 2009] 2. Mucocoele. [RGUHS Sep 2004]
45. Hamartomas. [RUGS Aug 2006] 3. Mikulicz’s disease. [RGUHS Sep 2000]
46. Lymphangiomas. [RUGS Mar 2006] 4. Sjögren’s syndrome. [BUHS Feb 1996; MUHS Jan
47. Verocay bodies. [RUGS Aug 2005] 1993; RGUHS Apr 2002]
48. CREST syndrome. [RUGHS Aug 2005; RURS Mar 5. Necrotizing sialometaplasia. [RURS Aug 2006; TN
2006] Apr 2001]
49. Bence Jones protein. [RUGS Aug 2006] 6. Pleomorphic adenoma. [MUHS Jul 1990, Jul 1997;
BUHS Aug 1991, Feb 1993, Mar 1994; RGUHS Sep
1999]
3. TUMOURS OF SALIVARY GLANDS 7. Histopathology of pleomorphic adenoma. [RURS Aug
2005]
Long Essays 8. Histogenesis of pleomorphic adenoma. [RURS Aug
1. Describe clinical and histological features of mucoepi- 2007]
dermoid carcinoma. [NTR-NR Oct 2005] 9. Warthin’s tumour. [BUHS Aug 1995; MUHS Jul 1997;
2. Classify salivary gland tumours. Write about histogen- RGUHS Sep 2001]
esis and clinical features of pleomorphic adenoma. 10. Aetiopathogenesis and histopathology of Warthin’s tu-
[NTR-OR Oct 1999, Oct 2004] mour. [RGUHS Apr 2003, Mar 2005]
3. Enumerate benign tumours of salivary glands. De- 11. Histology and laboratory investigations in Sjögren’s
scribe clinical and histopathological features of pleo- syndrome. [RURS Feb 2007]
morphic adenoma. [NTR-NR Oct 2006; TN Sep 2002] 12. Histology of mucoepidermoid carcinoma. [RURS Mar
4. Classify salivary gland tumours. Write about the patho- 2006]
genesis and histopathology of pleomorphic adenoma. 13. Warthin’s tumour. [NTR-OR Oct 2004; TN Apr 2000,
[NTR-OR Apr 1999; TN Oct 2003] Sep 2002]
5. Classify salivary gland tumours. Describe the patho- 14. Pathogenesis of pleomorphic adenoma. [NTR-NR Apr
genesis, clinical features and histopathology of pleo- 2006; RGUHS Apr 2003; TN Nov 2001]
morphic adenoma. [NTR-NR Apr 2007; TN Feb 2009] 15. Xerostomia. [TN OCT 2000]
Section | V Previous Years’ Question Bank 711

Short Notes 4. Classify cysts. Write in detail the aetiology, clinical


features and radiographic features of dentigerous cyst.
1. Sialolith. [NTR-OR Apr 1993]
[NTR-OR Oct 1995]
2. Mikulicz’s disease. [NTR-OR Oct 1995; TN Aug 2007;
5. Define cyst. Classify cysts of odontogenic origin. Write
NTR-NR Apr 2007]
in detail about dentigerous cyst. [NTR-OR Apr 1999;
3. Pleomorphic adenoma. [NTR-NR Apr 2004]
TN Apr 1995, Nov 2001, Sep 2002]
4. Mucocoele and ranula. [NTR-OR Apr 1998]
6. Classify cysts. Write in detail aetiology, clinical fea-
5. Sialography. [RGUHS Sep 2001, Apr 2003]
tures and radiographic features of dentigerous cyst.
6. Pleomorphic adenoma. [BUHS Aug 1981; TN Oct
[NTR-OR Oct 1990]
1998]
7. Classify cysts of odontogenic origin. Describe patho-
7. Histopathology of Warthin’s tumour. [BUHS Aug
genesis, radiographic appearances and histological fea-
1993, Aug 1995; MUHS Jul 1997; TN Apr 1993, Oct
tures of dentigerous cyst. [NTR-NR Oct 2002, Oct
1996]
2006; TN Aug 2006]
8. Histopathology of adenoid cystic carcinoma. [MUHS
8. Classify cysts of the oral cavity. Describe the patho-
Jan 1990, Jul 1997; TN Oct 1996]
genesis, clinical features, radiographic features and
9. Pleomorphic adenoma. [MUHS Jul 1990; BUHS Aug
histopathology of dentigerous cyst. [NTR-NR Oct
1991, Feb 1993, Mar 1994; RGUHS Sep 1999]
2007]
10. Histopathology of pleomorphic adenoma. [RURS Aug
9. Classify cysts of the orofacial region. Describe patho-
2005]
genesis and histopathology of odontogenic keratocyst.
11. Histogenesis of pleomorphic adenoma. [RURS Aug
[NTR-OR Apr 1997]
2007; TN Oct 1996, Apr 2000]
10. Classify odontogenic and nonodontogenic cysts of the
12. Warthin’s tumour. [BUHS Aug 1995; MUHS Jul 1997;
oral cavity write the aetiology and histological features
RGUHS Sep 2001; TN Feb 2006]
of Gorlin cyst. [NTR-OR Oct 1998]
13. Aetiopathogenesis and histopathology of Warthin’s tu-
11. Classify the cysts of orofacial region. Give clinical
mour. [RGUHS Apr 2003, Mar 2005]
signs, symptoms and histopathological features of
14. Histology and laboratory investigations in Sjögren’s
primordial cyst. [NTR-OR Apr 1993]
syndrome. [RURS Feb 2007]
12. Classify odontogenic cyst of the jaws and describe the
15. Histology of mucoepidermoid carcinoma. [RURS Mar
clinical features, radiographic features and histopathol-
2006]
ogy of odontogenic keratocyst. [TN Oct 1996, Aug 2008]
17. Sialography. [RGUHS Sep 2001, Apr 2003]
13. Define ameloblastoma. Write in detail about types,
18. Pleomorphic adenoma. [BUHS Aug 1988]
clinical features, radiologic findings and histopathol-
19. Histopathology of Warthin’s tumour. [BUHS Aug
ogy of ameloblastoma. [TN Apr 1996, Feb 2007]
1993, Aug 1995, Jul 1997]
14. Define cyst. Classify odontogenic cysts. Discuss in
20. Histopathology of adenoid cystic carcinoma. [MUHS
detail odontogenic keratocyst. Add a note on the syn-
Jan 1990]
drome associated. [TN Apr 2001, TN Aug 2006]
21. Adenoid cystic carcinoma. [TN Aug 2008; RGUHS
15. Enumerate benign epithelial odontogenic tumours.
Jan 2009]
[BUHS Aug 1992]
22. Salivary analysis. [NTR-OR Oct 1991]
16. Classify odontogenic tumours. Describe in detail about
23. Autoimmune sialosis. [MUHS Aug 1988, Jan 1989]
odontomes. [MUHS Jan 1998]
17. Classify odontogenic neoplasms. Describe the patho-
genesis, clinical features, histopathology and differen-
4. CYSTS AND TUMOURS tial diagnosis calcifying epithelial odontogenic tumour.
OF ODONTOGENIC ORIGIN [BUHS Aug 1993; MUHS Jul 1997]
18. Write in detail about periapical cyst. [MUHS Jul 1998]
Long Essays
19. Classify tumours of odontogenic origin. Describe the
1. Describe the pathogenesis and clinical features of am- pathogenesis, clinical and histopathological features of
eloblastoma. [NTR-OR Oct 1990] primordial cysts. [MUHS Jan 1988, Jun 1989, Jul
2. Classify the benign and malignant tumours of the oral 1989, Jan 1998]
cavity. Write in detail about clinical features, and histo- 20. Classify odontogenic tumours. Describe in detail
logical features of ameloblastoma. [NTR-OR Apr 1998] about odontogenic keratocyst. [MUHS Jul 1996, Jul
3. Classify odontogenic tumours of the oral tissues. De- 1997]
scribe histogenesis, clinical features and histopatho- 21. Classify odontogenic cysts. Describe in detail the
logical features of ameloblastoma. [NTR-OR Oct clinical features, roentgenographic features and histo-
1996, Apr 2006; NTR-NR May 2004; TN Oct 1991, logical features of odontogenic keratocyst. [RURS Sep
Apr 1996, Oct 1996, Oct 1997, Feb 2006] 1999]
712 Quick Review Series for BDS 3rd Year

22. Classify cysts of odontogenic in origin. Write in detail Short Notes


about dentigerous cyst. [RGUHS Mar 1997]
1. Odontomes. [NTR-OR Apr 2001; TN Apr 1998, Oct
23. Classify tumours of odontogenic origin. Write in detail
2003]
about dentigerous cyst. [BUHS Apr 1987]
2. Radicular cyst. [NTR-OR Apr 1993]
24. Classify odontogenic cysts and write about histology
3. Primordial cyst. [NTR-OR Oct 1997]
of ameloblastoma. [BUHS Aug 1988, Aug 1991, Mar
4. Ameloblastoma. [NTR-OR Apr 1993; TN Apr 1996]
1995, Feb 1996, Sep 1996]
5. Dentigerous cyst. [NTR-NR Apr 2005; TN Oct 1991]
25. Classify odontogenic tumours of the oral cavity and
6. Pindborg tumour. [NTR-OR Apr 1999; TN Oct 1998]
describe in detail ameloblastoma. [RGUHS OS Aug
7. Epidermoid cyst. [NTR-OR Apr 1991]
2006; TN Apr 1993, Oct 1997]
8. Odontogenic keratocyst. [NTR-OR Apr 2001, Oct
26. Classify odontogenic tumours. Describe histological
2004; TN Feb 2005]
variants of ameloblastoma. [RGUHS Sep 2001]
9. Gingival cyst of an adult. [NTR-NR Apr 2007]
27. Describe the pathogenesis and histopathology of ame-
10. Malignant potential of dentigerous cyst. [NTR-NR Oct
loblastoma. [MUHS May 1986]
2005]
28. Classify odontogenic tumours. Define ameloblastoma.
11. Histopathology of odontogenic keratocyst. [NTR-OR
Write in detail on clinical features and histopathologi-
Apr 2006; TN Apr 2000, Oct 2000, Feb 2005]
cal variants of ameloblastoma. [RGUHS Jul 1990]
12. Haemorrhagic cyst. [TN Aug 2007]
29. Define neoplasm, classify odontogenic neoplasms. De-
13. Pyogenic granuloma. [BUHS Aug 1988, Mar 1995,
scribe in detail the clinical features, radiographic fea-
Sep 1996]
tures of ameloblastoma. [MUHS Mar 2004; BUHS Jul
14. Primary syphilis. [RGUHS Apr 2000]
1990]
15. Congenital syphilis. [RGUHS Feb 1991; BUHS Feb
30. Classify odontogenic cysts. Describe in detail the
1996]
clinical features, roentgenographic features and histo-
16. Hutchinson’s triad. [RGUHS Jul 2000]
logical features of ameloblastoma. [RGUHS Mar 2006]
17. Treponema pallidum. [RGUHS Jul 1990, Sep 1999]
31. Describe clinical and histopathological features of ad-
18. Adenomatoid odontogenic tumour. [TN Aug 2008]
enoameloblastoma. [MUHS Mar 1988]
19. Histopathology of dentigerous cyst. [TN Feb 2007]
32. Classify odontogenic tumours. Describe in detail
20. Histopathology of pleomorphic adenoma. [TN Feb
about adenomatoid odontogenic tumour. [RGUHS Jan
2005]
1996]
21. Potential complications of dentigerous cyst. [TN Feb
33. Classify odontogenic cysts. Describe in detail the
2005]
clinical features, roentgenographic features and histo-
22. Warthin’s tumour. [TN Oct 1998]
logical features of CEOT (Pindborg tumour). [RURS
23. Cholesterol crystals. [TN Apr 1995]
Mar 2006]
24. Rushton bodies. [RURS Feb 2007; RGUHS RS Jan
2009]
Short Essays 25. Enamel pearl. [MUHS May 1986, Mar 1988, Aug
1988]
1. Gorlin’s cyst. [RURS Aug 2007; TN Apr 1993]
26. Ghost cells. [RURS Feb 2007; TN Sep 2002]
2. Pindborg tumour. [RGUHS Apr 2002; TN Feb 2010]
3. Lateral periodontal cyst. [MUHS Jan 1995; TN Apr
2003]
4. Pathogenesis of radicular cyst. [RGUHS Apr 2003] 5. BACTERIAL INFECTIONS OF THE ORAL
5. Odontogenic keratocyst. [MUHS Sep 1991] CAVITY
6. Draw and write label histopathology of keratocyst.
Long Essays
[RGUHS Apr 2001]
7. Write about dentigerous cyst. [MUHS Nov 1986] 1. Write about the microorganisms producing infections of
8. Explain about odontomes. [MUHS Jan 1990] the oral cavity. [NTR-OR Apr 2000]
9. Adenomatoid odontogenic tumour. [MUHS Jul 1990; 2. Enumerate the bacterial infections. Describe the oral
RGUHS Apr 2000; TN Oct 2003] manifestations of congenital and acquired syphilis.
10. Rushton bodies. [NTR-NR Apr 2004] [MUHS Jul 1998; TN Apr 2003]
11. Liesegang’s ring calcification. [NTR-NR Apr 2005] 3. Write about oral thrush. [MUHS Nov 1986; BUHS May
12. Aneurysmal bone cyst. [TN Oct 2003] 1997]
13. Ameloblastic fibroma. [RGUHS Jan 2009] 4. Classify aphthous ulcers and describe its aetiology,
14. Calcifying epithelial odontogenic tumour. [RGUHS clinical features and differential diagnosis. [TN Oct
Jan 2009] 1996]
Section | V Previous Years’ Question Bank 713

Short Essays 15. Actinomycosis. [RGUHS Mar 1994; BUHS Feb 1996;
TN Oct 2000, Feb 2007]
1. Pregnancy epulis. [BUHS Mar 1995, Feb 1996; MUHS
16. Ghon’s lesion. [RUOS Aug 2006; TN Aug 2006]
Aug 1988, Jan 1998, Jul 2000]
17. Aphthous ulcers. [RGUHS Feb 1991; BUHS Sep 1992,
2. Pyogenic granuloma. [BUHS Feb 1996]
Mar 1994; TN Apr 1998]
3. Tuberculous ulcer of oral cavity. [RUOS Aug 2006]
18. Diagnosis of tuberculosis. [TN Aug 2006]
4. Lab investigations of tuberculosis. [RGUHS Sep 2001]
19. Actinomycosis. [RGUHS Mar 1994; BUHS Feb 1996;
5. Histopathology of tuberculous ulcers. [RGUHS Sep
TN Feb 2009]
1996, Sep 2000]
21. Ghon’s lesion. [RGUHS OS Aug 2006; TN Feb 2005]
6. Oral manifestations of syphilis. [MUHS Jun 1989;
20. Aphthous ulcers. [RGUHS Feb 1991; BUHS Sep 1992,
BUHS Sep 1992]
Mar 1994]
7. Secondary syphilis. [MUHS Sep 1991, Jan 1995; TN
22. Cancrum oris. [RGUHS Sep 2000; TN Aug 2009]
Oct 1999]
23. Sarcoidosis. [NTR-OR Oct 2005]
8. Oral manifestations of secondary syphilis. [MUHS Jun
24. Mantoux test. [RUGS Mar 2006]
19891; BUHS Sep 1992; RGUHS Apr 2002]
9. Oral manifestations of congenital syphilis. [RGUHS
Apr 2003; RGUHS OS Mar 2005] 6. VIRAL INFECTIONS OF THE ORAL
10. Hutchinson’s triad in congenital syphilis. [RUGHS RS CAVITY
Mar 2006]
11. Aphthous ulcers. [BUHS Apr 1987; MUHS Jan 1991] Long Essays
12. Differences between major and minor aphthous ulcers.
1. Enumerate viral infections of the oral cavity. Describe
[BUHS Feb 19931; RGUHS Sep 1999]
general and oral manifestations caused by herpes virus.
13. Gumma. [NTR-NR Apr 2004]
[NTR-OR Apr 2002]
14. Tuberculin test. [NTR-OR Oct 2004; TN Apr 2001]
2. Enumerate the various viral infections affecting the oral
15. Hutchinson’s triad. [NTR-OR Apr 2000]
cavity; write in detail the pathogenesis, clinical features
16. Condylomata lata. [NTR-NR Oct 2007]
and oral manifestations of herpes simplex virus. [NTR-
17. Treponema pallidum. [NTR-NR Oct 2006]
OR Oct 2004; TN Aug 2004]
18. Gumma-tertiary syphilis. [NTR-NR Oct 2006]
3. Describe recurrent aphthous ulcers in detail. [MUHS
19. Lumpy jaw. [TN Aug 2006]
Jan 1996]
20. Lipschutz bodies. [TN Aug 2006; RGUHS Jan 2009]
4. Write about primary herpetic stomatitis. [MUHS Apr
21. Oral manifestations in tuberculosis. [TN Feb 2005]
1987]
5. Write about herpes simplex infection. [BUHS May
1986; MUHS Jun 1989, Jul 2000]
Short Notes
6. Classify viruses and viral infections. Describe primary
1. Mumps. [NTR-OR Oct 1998] herpetic stomatitis. [BUHS Mar 1994; MUHS Jul 1999]
2. Noma. [NTR-OR Oct 1999] 7. Classify vesiculobullous lesions of the oral cavity. Write
3. Cancrum oris. [NTR-NR Oct 2005] about primary herpetic gingivostomatitis. [RGUHS Sep
4. Oral manifestation of syphilis. [NTR-OR Apr 1999, 1994; MUHS Jan 1996]
Apr 2002] 8. Enumerate the viral lesions of the oral cavity. Describe
5. Cervicofacial actinomycosis. [NTR-OR Oct 2005; in detail the clinical features, histological features and
NTR-NR Apr 2007] lab investigations of primary herpetic gingivostomatitis.
6. Histopathology of tuberculous ulcer. [NTR-OR Oct [RURS Aug 2007; TN Oct 1996, Apr 1998, Apr 2001]
2006]
7. Infection of developing tooth. [NTR-OR Oct 1997]
8. Hutchinson’s triad of congenital syphilis. [NTR-NR
Short Essays
Apr 2007] 1. Mumps. [RGUHS Sep 2001; TN Aug 2004]
9. Cancrum oris. [RGUHS Sep 2001] 2. Herpangina. [RGUHS Apr 2001]
10. Pyogenic granuloma. [BUHS Aug 1988, Mar 1995, 3. Serum hepatitis. [BUHS Aug 1988; MUHS Jan 1989]
Sep 1996] 4. Herpetic infection. [RGUHS Apr 2000]
11. Primary syphilis. [RGUHS Apr 2000] 5. Primary herpetic stomatitis. [BUHS Apr 1987; MUHS
12. Congenital syphilis. [RGUHS Feb 1991; BUHS Feb Jul 1991, Jan 1998, Jul 2000]
1996; TN Feb 2009] 6. Tzanck cells. [NTR-OR Apr 2006; NTR-NR Apr 2004]
13. Hutchinson’s triad. [RGUHS Jul 2000] 7. Koplik’s spots. [NTR-NR Oct 2006, Apr 2005]
14. Treponema pallidum. [RGUHS Jul 1990, Sep 1999] 8. Herpes simplex virus. [NTR-OR Oct 1992]
714 Quick Review Series for BDS 3rd Year

9. Acute herpetic gingivostomatitis. [NTR-NR Oct 2002; 5. Cervicofacial actinomycosis. [RGUHS RS Mar
RGUHS Mar 2005] 2006]
10. Congenital syphilis. [TN Aug 2009] 6. Acute pseudomembranous candidiasis. [RGUHS Sep
2004; TN Apr 1993]
7. Classification of oral candidiasis. [MUHS Jan 2000;
Short Notes
RGUHS RS Feb 2007]
1. AIDS. [NTR-OR Oct 1999; TN Feb 2005] 8. Write about the special stains and culture media for
2. Herpangina. [RGUHS Apr 2001] investigation of candidiasis. [RGUHS Sep 2002]
3. Herpes zoster. [NTR-OR Apr 1999; TN Oct 1996, Oct 9. Denture sore mouth. [RGUHS Sep 2000]
1998, Apr 2000] 10. Oral thrush. [BUHS Sep 1999; TN Oct 1997, Feb
4. Herpes simplex virus. [NTR-OR Apr 1997, Apr 1998, 2006]
Apr 2000; RGUHS RS Jan 2009] 11. Candida albicans. [BUHS Oct 1987, Aug 1995; RURS
5. Aphthous ulcers. [NTR-OR Apr 1998] Aug 2007; TN Oct 1991, Aug 2009]
6. Aphthous ulcer major. [NTR-NR Oct 2006] 12. Identification of Candida organisms. [RGUHS Sep
7. Oral manifestation of AIDS. [NTR-NR Apr 2004; TN 2001]
Apr 1996, Apr 2001] 13. Oral candidiasis. [TN Oct 2000]
8. Herpes labialis. [MUHS Jan 1995; TN Feb 2009]
9. Herpes zoster. [MUHS Jan 1989]
10. Herpes simplex virus. [BUHS Mar 1988; MUHS Jan 8. DISEASES OF THE PERIODONTIUM
1989]
11. Herpes simplex infections. [BUHS May 19861]
Long Essays
12. Lipschutz bodies. [RGUHS Apr 2001, Mar 2005] 1. Describe aetiology, clinical features and pathology of
13. Behçet’s syndrome. [RGUHS Apr 2002, Apr 2003; TN acute necrotizing gingivitis. [NTR-OR Oct 1990]
Aug 2009] 2. Classify periodontal diseases. Describe in detail acute
14. Koplik’s spots. [MUHS Nov 1986; RGUHS Sep 1999] necrotizing ulcerative gingivitis. [BUHS Mar 1995; TN
15. Oral manifestations of AIDS. [RURS Aug 2005] Apr 1996]
16. Clinical features and histopathology of herpes simplex 3. Describe aetiology, histopathology and clinical features
infection. [TN Aug 2006] of acute necrotizing gingivitis. [BUHS Aug 1988]
17. Lipschutz bodies. [RURS Feb 2007] 4. Enumerate causes of gingival enlargement. Discuss
pathogenesis, clinical features and histopathology of
leukaemic enlargement. [MUHS Jul 1987]
7. MYCOTIC INFECTIONS OF THE ORAL
CAVITY
Short Essays
Long Essays
1. Noma. [RUR 2007]
1. Enumerate fungal infections of the oral cavity. Write 2. Gingival enlargements. [MUHS Jan 19991; RGUHS
in detail about the clinical features and lab investiga- Apr 2002; TN Apr 1998]
tions of oral candidiasis. [RGUHS Mar 2004; TN Sep 3. Juvenile periodontitis. [MUHS Jan 1990; BUHS Feb
2002] 1996; RGUHS Sep 2000, Sep 2002; TN Apr 2001]
2. Describe the aetiology, histopathology and clinical fea- 4. Acute necrotizing ulcerative gingivitis. [MUHS Jan
tures of oral candidiasis. [MUHS Jan 1989; TN Apr 1981, Jan 1999; RGUHS Sep 1999]
1995] 5. Microbiology of organisms associated with acute necro-
tizing ulcerative gingivitis. [BUHS Mar 1988]
6. Differences between supragingival and subgingival cal-
Short Essays
culus. [BUHS Feb 1993]
1. Thrush. [MUHS Nov 1996, Jul 1997; BUHS Sep 1992]
2. Angular cheilitis. [RURS Mar 2000; TN Feb 2005]
Short Notes
1. Vincent infection. [BUHS Mar 1992; TN Apr 1996]
Short Notes 2. Morphology of ANUG. [RGUHS Sep 2004]
1. Thrush. [NTR-OR Oct 1995; NTR-NR Oct 2002] 3. Microorganisms causing ANUG. [RGUHS OS Mar
2. Oral moniliasis. [NTR-OR Oct 1998] 2006]
3. Candidiasis. [NTR-OR Apr 1993; NTR-QR Apr 1998; 4. Acute necrotizing ulcerative gingivitis. [BUHS Feb
TN Apr 1998, Apr 2003] 1991]
4. Actinomycosis. [MUHS Jan 1989, Jan 1998, Jan 1999] 5. Papillon–Lefèvre syndrome. [RGUHS OS Aug 2005]
Section | V Previous Years’ Question Bank 715

6. Dilantin gingival hyperplasia. [RGUHS RS Aug 2005] 13. Define dental caries. Write in detail the theories of
7. Juvenile periodontitis. [BUHS Feb 1996, Sep 2001; TN dental caries and zones of smooth surface caries.
Apr 2000] [RGUHS Sep 2004]
8. Pregnancy tumour. [RURS Mar 2006] 14. Define and describe dental caries. Describe the clinical
9. Epulis. [TN Oct 1991] and histopathological features of caries in enamel.
10. ANUG. [TN Apr 1995] [MUHS Jul 1998; TN Apr 1998]
11. Marginal gingivitis. [TN Oct 1996] 15. Describe the histopathology of caries in dentin and
12. Scorbutic gum. [TN Oct 1997] discuss the pioneer bacteria. [MUHS Jul 1989]
13. ANUG. [NTR-OR Apr 2006; NTR-NR Oct 2002, Apr 16. Define and classify dental caries. Describe the clinical
2005; TN Oct 2003] and histopathological features of dentinal caries.
14. Borrelia vincentii. [NTR-OR Oct 1997] [RURS Aug 2005; TN Apr 1995]
15. Papillon–Lefèvre syndrome. [NTR-OR Apr 1996] 17. Discuss about various theories of dental caries. [BUHS
16. Granuloma of periodontium. [NTR-OR Oct 1997] Sep 1992, Mar 1995; TN Oct 1996]
17. Infectious acute periodontitis. [NTR-OR Oct 1999] 18. Define dental caries. Describe the Miller’s acidogenic
18. Periapical periodontitis. [NTR-NR Oct 2007] theory. [BUHS Aug 1995; TN Apr 1996]
19. Gum boil or parulis. [RURS Aug 2007] 19. Describe the aetiopathogenesis, clinical features and
20. Intrinsic staining in teeth. [RUGS Mar 2006] histopathology of dental caries. [TN Aug 2008]

9. DENTAL CARIES Short Essays


1. Saliva. [NTR-NR Apr 2004]
Long Essays
2. Rampart caries. [NTR-NR Oct 2002; TN Oct 2003]
1. Describe the causes of dental caries. Discuss the role of 3. Streptococcus mutans. [NTR-NR Apr 2007; TN Apr
bacteria in dental caries. [NTR-OR Apr 1998] 1996]
2. Define dental caries. Write in detail different theories 4. Liquefaction foci of Miller. [NTR-NR Oct 2006; TN
of dental caries and also add a note on aetiology of Feb 2005]
dental caries. [NTR-OR Oct 1995] 5. Acidogenic theory of dental caries. [NTR-OR Oct
3. Define dental caries; mention various theories of dental 2004; TN Nov 2001]
caries. Write in detail the clinical types and factors 6. Sequelae of dental caries. [TN Oct 1996, Aug 2004]
contributing the dental caries, and write briefly the 7. Describe aetiology of dental caries. [MUHS Jan 1997;
histopathological features of enamel dental caries. TN Apr 2000]
[NTR-NR Apr 2004] 8. Describe the pathogenesis of dental caries. [BUHS Mar
4. Discuss the acidogenic theory of dental caries. [NTR- 1994]
NR Oct 2002] 9. Describe the microbiological aspects of dental caries.
5. Define dental caries. Classify dental caries. Write in [MUHS Jul 1996]
detail about acidogenic theory. [NTR-OR Oct 1999; 10. Discuss in detail the role of carbohydrates in dental
TN Feb 2005] caries. Write about histopathology of enamel caries.
6. Describe the mechanisms for bacterial decomposition [BUHS Aug 1991]
of the inorganic and organic structure of enamel and 11. Describe the role of plaque in the aetiology of dental
dentin. [NTR-OR Oct 1997] caries. [BUHS May 1986]
7. Describe the aetiology and histopathology of enamel 12. Describe the histopathology of dental caries. [BUHS
and dentinal caries. [NTR-OR Apr 1997] Sep 1996]
8. Write on histopathology of caries of the enamel, dentin 13. Define caries. Discuss in detail about acidogenic the-
and cementum. Describe the role of lactobacillus group ory. [RGUHS Apr 2000]
in caries activity with a note on their control. [NTR-OR
Oct 1997]
9. Define dental caries. Describe histopathological fea-
Short Notes
tures of caries in enamel. [NTR-OR Apr 1997] 1. Calculus. [NTR-NR Oct 2002]
10. Define and describe about dental caries. [RGUHS Aug 2. Dental plaque. [NTR-OR Apr 1993]
1931] 3. Contributing factors in dental caries. [NTR-NR Apr
11. What is plaque? Describe its role in the pathogenesis of 2005]
dental caries. [MUHS Jan 1990] 4. Role of Lactobacillus in dentinal caries. [NTR-OR Apr
12. Describe the structure of dental plaque and its role in 1998]
the aetiology of dental caries. [BUHS Mar 1988, May 5. Proteolysis—chelation theory of dental caries. [NTR-
1996; MUHS Jan 1990] NR Oct 2005, Oct 2006]
716 Quick Review Series for BDS 3rd Year

6. Zone of enamel caries. [NTR-OR Apr 1990] 5. What is osteomyelitis? Describe the aetiology, patho-
7. Zones of incipient enamel caries. [NTR-OR Apr 2006] genesis and clinical features of chronic osteomyelitis of
8. Histopathology of caries of the enamel. [NTR-OR Oct mandible. [BUHS Oct 1988; MUHS Jul 1990, Jan
2004] 1998]
9. Histopathology of dentin caries. [NTR-OR Apr 1998, 6. Describe the aetiology, histopathological, clinical and
Apr 2000] radiographic features of periapical granuloma and men-
10. Histopathology of caries of dentin. [NTR-OR Oct tion its consequences. [MUHS Jul 1990]
1992; TN Oct 2003] 7. Enumerate the periapical lesion. Describe the pathogen-
11. Histopathologic study of dental caries. [NTR-OR Apr esis, histopathological and clinical features of the ra-
2000; TN Apr 1999, Apr 2003] dicular cyst. [BUHS Sep 1995]
12. Lactobacillus count test for dental caries. [NTR-NR
Apr 2005]
13. Radiation caries. [NTR-NR Apr 2007] Short Essays
14. Zones of dentin caries with cavitation of enamel.
1. Acute pulpitis. [RGUHS Jan 2009]
[NTR-NR Apr 2007]
2. Sequestrum. [RURS Aug 2005]
15. Clinical types of caries. [MUHS Jul 2000; RGUHS
3. Acute pulpitis. [RUOS Aug 2005; TN Oct 2000, Apr
Apr 2002]
2001, Aug 2006]
16. Rampant caries. [RGUHS OS Mar 2006; RGUHS RS
4. Pulp polyp. [RURS Aug 2005; TN Sep 2002, Feb
Mar 2006]
2005]
17. Classify dental caries. Write on nursing bottle syn-
5. Chronic hyperplastic pulp. [BUHS Feb 1996]
drome. [RGUHS Sep 2001]
6. Histopathology of pulp polyp. [BUHS Aug 1995; TN
18. Histopathology of dental caries. [RGUHS Sep 1999;
Oct 1996, Oct 1999, Feb 2010]
RGUHS OS Aug 2006]
7. Periapical granuloma. [BUHS Sep 1994; MUHS Jan
19. Histopathology of caries in enamel. [BUHS Oct 1987]
1995; TN Apr 1996, Oct 1996, Oct 1997, Feb 2009]
20. Dentinal caries. [BUHS Feb 1991, Sep 1992; RGUHS
8. Histopathology of periapical granuloma. [MUHS May
RS Aug 2006]
1988; BUHS Jan 1989; TN Aug 2005]
21. Histopathology of caries in dentin. [BUHS Feb 1996;
9. Periapical cyst. [NTR-NR Apr 2004; TN Oct 2003]
MUHS Oct 1987, Jul 1999; RGUHS Apr 2003;
10. Garre’s osteomyelitis. [NTR-NR Oct 2005]
RGUHS OS Mar 2005; TN Aug 2006]
11. Chronic sclerosing osteomyelitis. [NTR-NR Oct
22. Miller’s theory of dental caries. [RURS Aug 2007]
2006]
23. Erythema multiforme. [TN Apr 2010]
12. Classification of osteomyelitis. [NTR-NR Oct 2007]
24. Microscopy of enamel caries. [TN Apr 1993, Oct
1997]
25. Dead tracts. [RGUHS RS 2009]
26. Streptococcus mutans. [RGUHS RS 2009]
Short Notes
1. Pulp polyp. [NTR-NR Apr 2006]
2. Radicular cyst. [NTR-OR Oct 1990, Oct 1996; TN Apr
10. DISEASES OF THE PULP 1998]
AND PERIAPICAL TISSUES 3. Periodontal cyst. [NTR-OR Apr 2000]
4. Alveolar abscess. [NTR-OR Apr 2000]
Long Essays
5. Periapical granuloma. [NTR-OR Oct 1992, Apr 2001;
1. Define and classify pulpitis. Write in detail about the TN Oct 1999, Aug 2004]
pathogenesis, clinical features and histopathological 6. Garre’s osteomyelitis. [NTR-OR Oct 1995, Apr 2006;
features of acute pulpitis. [NTR-NR Apr 2005; TN Oct TN Feb 2009]
1997] 7. Chronic hyperplastic pulpitis. [NTR-NR Nov 2005;
2. Describe the pathogenesis, histopathology, radiographic NTR-OR Oct 2004; TN Aug 2008]
appearance and clinical features of chronic osteomyeli- 8. Acute suppurative osteomyelitis. [NTR-OR Apr 1989]
tis of the jaw. [NTR-OR Oct 1996] 9. Pyogenic granuloma. [TN Oct 1996, Aug 2008]
3. Define and classify osteomyelitis. Write about osteomy- 10. Pyogenic granuloma. [TN Aug 2006]
elitis. [NTR-OR Sep 1987] 11. Histopathology of periapical granuloma. [TN Apr
4. Define osteomyelitis. Describe the aetiology, pathology 2001]
and clinical features of acute suppurative osteomyelitis 12. Periapical abscess. [TN Apr 2000]
of mandible in adults. [MUHS Jul 1997] 13. Fibroepithelial polyp. [MUHS May 1986]
Section | V Previous Years’ Question Bank 717

11. SPREAD OF ORAL INFECTIONS 7. Ranula. [TN Apr 1993; RGUHS Jan 2009]
8. Describe the pathogenesis and histopathology of mu-
Long Essays cocoele. [RGUHS Sep 2002]
1. Describe in detail chronic osteomyelitis affecting the 9. Mucocoele. [NTR-OR Oct 1992; TN Oct 1997, Apr 1999]
jaws. [TN Apr 2000] 10. Internal resorption. [TN Apr 1999, Feb 2005, Feb
2006, Aug 2006]
11. Clinical and histologic features of nicotine stomatitis.
Short Essays [TN Apr 2003]
1. Focal infection. [MUHS Nov 1986, Jan 1988; RGUHS
Sep 1999, Aug 2006] Short Notes
2. Ludwig’s angina. [RGUHS-RS Aug 2006; TN Sep
2002, Aug 2009] 1. Mucocoele. [NTR-OR Oct 1996; NTR-NR Apr 2005;
3. Capillary and cavernous haemangiomas. [RGUHS RS TN Apr 2000]
Mar 2006] 2. Ranula. [NTR-OR 1998; TN Apr 2000]
3. Tooth ankylosis. [NTR-OR Apr 1997, Oct 1997; NTR-
NR Apr 2004]
Short Notes 4. Mucocoele. [BUHS Aug 1993, Mar 1994; RGUHS OS
1. Cellulitis. [NTR-NR Apr 2004] Aug 2006; TN Apr 1999]
2. Focal infection. [NTR-NR Oct 2006; RGUHS Sep 5. Mucous cyst. [BUHS Aug 1992; MUHS Jul 1998]
1998; TN Oct 1991] 6. Mucous retention cyst. [MUHS Jul 1990; BUHS Aug
3. Ludwig’s angina. [NTR-OR Apr 1996, Apr 1997, Oct 1993, Mar 1999]
1998, Apr 1999; NTR-NR Apr 2005, Oct 2006; TN Oct 7. Sialolithiasis. [RGUHS Apr 2000; TN Oct 2003]
1998] 8. Plumbism. [RGUHS Sep 2004; TN Apr 1999]
4. Spaces involved in Ludwig’s angina. [RGUHS Apr 9. Amalgam tattoo. [RGUHS-OS Aug 2006]
2000, Sep 2002] 10. Pink disease. [RGUHS Apr 2002]
5. Definition of focus of infection and focal infection. 11. Ankylosis of teeth. [MUHS Jul 1997]
[RURS Aug 2007] 12. Hypersensitive dentin. [MUHS Jan 1996, Jul 1997]
6. Sequestrum. [TN Feb 2005, Aug 2008] 13. Osteoradionecrosis. [RURS Aug 2006; TN Apr 1996]
7. Garre’s osteomyelitis. [TN Apr 2001, Aug 2006] 14. Mucous extravasation cyst. [TN Feb 2009]
8. Osteoradionecrosis. [TN Feb 2006] 15. Bruxism. [TN Oct 2000; RURS 2007]
9. Focal sclerosing osteomyelitis. [TN Sep 2002] 16. Xerostomia. [TN Apr 1999]
10. Pioneer bacteria. [RGUHS-RS Jan 2009]
13. REGRESSIVE ALTERATIONS
12. PHYSICAL AND CHEMICAL INJURIES OF THE TEETH
OF THE ORAL CAVITY
Long Essays
Long Essays 1. Describe in detail the regressive alterations of teeth.
1. Describe the predisposing factors and clinical features [BUHS May 1995; RGUHS Apr 2001]
of osteoradionecrosis. [RGUHS Sep 2002] 2. Describe the regressive changes occurring in the dental
pulp. [MUHS Aug 1988]
3. Describe the reparative and regressive changes associ-
Short Essays ated with dentin and pulp. [MUHS Oct 1987]
1. Sialolithiasis. [MUHS Jul 1999; TN Oct 2000] 4. Classify resorption of teeth. Describe external resorp-
2. Traumatic bone cyst. [RGUHS Apr 2000] tion. [RGUHS Apr 2000]
3. Effects of radiation on the tissues. [BUHS Sep 1992;
MUHS Jul 1999]
Short Essays
4. Effects of radiation on oral tissues. [RGUHS RS Aug
2005] 1. Hypercementosis. [MUHS May 1986; RGUHS RS Feb
5. Effects of radiation on oral mucosa. [RGUHS Sep 2007; TN Apr 1996, Oct 2000, Nov 2001]
2001] 2. Age changes in the pulp. [MUHS May 1986]
6. Osteoradionecrosis. [MUHS Jan 1990, Jul 1996; 3. Pink tooth of Mummery. [RGUHS Sep 1999]
RGUHS OS Mar 2006] 4. Abrasion, attrition and erosion. [MUHS Jul 1997]
718 Quick Review Series for BDS 3rd Year

5. Internal resorption of the tooth. [BUHS May 1992; 3. Healing of extraction wound. [MUHS Jan 1995, Jul
MUHS Jan 1995, Jan 1996, Jan 1997] 2000; RGUHS-RS Aug 2006; RGUHS-OS Aug 2005]
6. External resorption of teeth. [RURS Aug 2007; TN 4. Healing of extraction socket. [RGUHS-OS Aug 2005]
Nov 2001] 5. Factors affecting healing of oral wound. [RURS Aug
7. External and internal resorption of teeth. [MUHS Jul 2005; TN Apr 1998]
1999] 6. Dry socket. [NTR-NR Oct 2002, Apr 2007]

Short Notes Short Notes


1. Erosion. [MUHS Mar 1998; RGUHS Sep 2000] 1. Biopsy. [NTR-OR Oct 1995, Oct 1998]
2. Attrition. [MUHS Jul 1997; RGUHS Sep 1999] 2. Dry socket. [NTR-OR Apr 1999; TN Apr 2000, Aug
3. Abrasion. [MUHS Apr 1987, Jul 1987, Jan 1997; 2008]
RGUHS Apr 2001; TN Aug 2005] 3. Exfoliate cytology. [NTR-OR Apr 2005]
4. Abrasion of teeth. [RURS Feb 2007; TN Oct 1996] 4. Oral biopsy and exfoliative cytology. [NTR-OR Apr
5. Attrition and abrasion. [RGUHS Apr 2002] 2000]
6. Pink spot. [MUHS Jan 1998; TN Apr 1995] 5. Tissue changes in root fracture and repair. [NTR-OR
7. Pink tooth. [RGUHS Apr 2003; TN Feb 2007] Apr 1994]
8. Cementicles. [RGUHS Apr 2001] 6. Tissue changes in root canal treatment. [NTR-QR Apr
9. Hypercementosis. [MUHS Nov 1986, Oct 1987, Jan 2000]
1997; BUHS Aug 1992, Sep 1992; RGUHS Sep 2002; 7. Healing of the socket following extraction. [NTR-OR
RGUHS OS Mar 2005; RGUHS RS Aug 2005] Apr 1998, Oct 2004]
10. Secondary dentin. [BUHS Sep 1996] 8. Biopsy. [MUHS Jan 1991]
11. Pink tooth of Mummery. [MUHS Nov 1986; RURS 9. Dry socket. [MUHS Nov 1986; RGUHS Sep 2000;
Aug 2006] RURS Aug 2006; TN Oct 2003, Aug 2005]
12. Factors causing external resorption of teeth. [RGUHS 10. Exfoliative cytology. [BUHS Feb 1993]
Mar 2004] 11. Factors affecting wound healing. [RGUHS Sep 2001;
13. Attrition. [NTR-NR Oct 2002] TN Aug 2008]
14. Abrasion. [NTR-NR Nov 2005] 12. Healing of wound. [TN Oct 1998, Aug 2007]
15. Erosion. [NTR-NR Oct 2006] 13. Types of biopsy. [TN Apr 1999]
16. Pink spot. [NTR-OR Oct 1992]
17. Cementicles. [NTR-NR Oct 2005]
18. Dead tracts. [NTR-NR Apr 2006] 15. ORAL ASPECTS OF METABOLIC
19. Dead tracts of dentin. [NTR-NR Oct 2004] DISEASE
20. Pink tooth of Mummery. [NTR-OR Apr 1988; NTR-
Short Essays
NR Oct 2007; TN Feb 2010]
21. Hypercementosis. [NTR-OR Oct 1997; NTR-NR Oct 1. Hypopituitarism. [MUHS Jul 2000]
2002; TN Apr 1998, Apr 2000] 2. Hyperpituitarism. [MUHS Jul 1999]
22. Pulp stones. [MUHS Jan 1997; NTR-OR Apr 2007] 3. Addison’s disease. [MUHS Jan 1998]
23. Idiopathic resorption of pulp. [NTR-OR Apr 1990, Oct 4. Diabetes mellitus. [MUHS Sep 1991]
1996] 5. Eosinophilic granuloma. [BUHS Jun 1989]
24. Tissue changes after root canal therapy. [NTR-OR Oct 6. Oral manifestations of hyperparathyroidism. [BUHS
1997] Mar 1988, Jun 1989; RURS Aug 2005]
25. Attrition, abrasion and erosion. [NTR-OR Sep 1988] 7. Scurvy. [NTR-OR Oct 2004]
8. Pellagra. [NTR-NR Apr 2007]
9. Dental fluorosis. [TN Oct 2003]
14. HEALING OF ORAL WOUNDS
Long Essays Short Notes
1. Write about healing of extraction wound and mention 1. Scurvy. [NTR-NR Apr 2005; NTR-OR Apr 1997; TN
about its complications. [RGUHS Sep 1999] Oct 1998, Oct 2003, Aug 2004]
2. Rickets. [NTR-OR Apr 1999]
3. Stomatitis. [NTR-NR Apr 2000]
Short Essays 4. Acromegaly. [NTR-OR Oct 1996]
1. Dry socket. [MUHS Jan 1996, Jan 1998] 5. Amyloidosis. [NTR-OR Apr 1999; TN Feb 2007]
2. Exfoliative cytology. [RGUHS Apr 2000, Apr 2002] 6. Hyperthyroidism. [NTR-OR Apr 1993; TN Oct 1998]
Section | V Previous Years’ Question Bank 719

7. Hyperparathyroidism. [NTR-NR Oct 2002] 3. Histopathology of Paget’s disease. [RGUHS Sep 2001;
8. Vitamin C deficiency. [NTR-OR Apr 1998] TN Apr 1996, Apr 2001, Sep 2002, Feb 2005]
9. Oral manifestation of vitamin C deficiency. [NTR-OR 4. Draw and write about histopathology of Paget’s dis-
Apr 2001] ease of bone. [RGUHS Mar 2004]
10. Scurvy. [MUHS Jan 1991, Jul 1999] 5. Fibrous dysplasia. [MUHS Oct 1987; BUHS Sep 1991,
11. Rickets. [MUHS Jan 2001] Aug 1994; RURS Aug 2006; RUOS Aug 2006]
12. Amyloidosis. [MUHS Aug 1988] 6. Monostotic fibrous dysplasia. [MUHS May 1986;
13. Gigantism. [MUHS Jan 1999] RGUHS Sep 1999, Sep 2000]
14. Acromegaly. [RGUHS Sep 2002; RUOS Mar 2005] 7. Clinical and histopathological features of monostotic
15. Hyperparathyroidism. [BUHS Aug 1991] fibrous dysplasia. [RGUHS Sep 2004]
16. Eosinophilic granuloma. [RUHS Sep 1996; TN Apr 8. Cleidocranial dysplasia. [BUHS Feb 1991; TN Oct
1995] 1997, Feb 2006]
17. Riboflavin deficiency. [BUHS Aug 1991] 9. Myofacial pain dysfunction syndrome. [RGUHS Sep
18. Vitamin B deficiency (oral manifestations). [TN Oct 2000; RGUHS OS Aug 2005]
1996; RGUHS Sep 2001] 10. Aetiopathogenesis and histopathology of cherubism.
19. Fluorosis of teeth. [TN Aug 2008] [RGUHS Apr 2003; RUOS Mar 2005]
20. Pituitary dwarfism. [TN Aug 2007] 11. Fibrous dysplasia. [NTR-NR Oct 2006]
12. Café au lait spots. [NTR-OR Oct 2004]

16. ALLERGIC AND IMMUNOLOGIC


DISEASES OF THE ORAL CAVITY Short Notes
1. Cherubism. [NTR-OR Oct 2004; NTR-NR Oct 2002;
Short Notes
TN Sep 2002, Aug 2004, Aug 2008]
1. Contact stomatitis. [TN Oct 1996] 2. Paget’s disease. [NTR-OR Apr 1997; NTR-NR Apr
2. Histamines. [RGUHS Mar 2004] 2005]
3. Immunoglobulins. [RGUHS Apr 2003] 3. Fibrous dysplasia. [TN Oct 1991; NTR-OR Oct 1995,
4. Foreign body reaction. [MUHS Aug 1989] Apr 2001; NTR-NR Apr 2004]
5. Anitschkow cells. [NTR-OR Apr 2005; RGUHS Aug 4. Monostotic fibrous dysplasia. [NTR-OR Oct 1992; TN
2005; RGUHS-RS Aug 2007] Nov 2001]
6. Delayed hypersensitivity. [NTR-OR Oct 2007] 5. Cleidocranial dysplasia. [NTR-OR Oct 1996; TN Aug
2008]
6. Osteogenesis imperfecta. [NTR-OR Oct 1998, Oct
17. DISEASES OF BONES AND JOINTS 2004]
7. Osteoporosis (marble bone disease). [NTR-NR Oct
Long Essays 2005; TN Feb 2010]
1. Classify the diseases of TMJ. Write aetiology and clini- 8. Histopathology of Paget’s disease of bone. [NTR-NR
cal features of ankylosis. [NTR-OR Apr 1993] Oct 2007; TN Oct 1996, Feb 2005]
2. Enumerate bone disorders affecting the jaws. Describe 9. Cherubism. [BUHS Mar 1941; MUHS Jan 1988;
the pathogenesis, clinical features, radiographic appear- RURS Aug 2005]
ances and histopathology of fibrous dysplasia. [NTR- 10. Blue sclera. [RUOS Mar 2005]
NR Apr 2006; TN Oct 1991, Apr 1996, Oct 1996, Oct 11. Leontiasis ossea. [MUHS Jan 1999]
2000] 12. Albright’s syndrome. [MUHS Jan 1998]
3. Enumerate bone disorders. Describe fibrous dysplasia 13. Marfan’s syndrome. [RGUHS Sep 2001]
of jaw bones. [BUHS Aug 1995, Mar 1997; MUHS Jan 14. Paget’s disease of bone. [BUHS Mar 1995]
1999; TN Oct 1998] 15. Histopathology of Paget’s disease. [BUHS Feb 1993,
4. Describe in detail about fibrous dysplasia. [TN Oct Sep 1994; MUHS Jul 1989]
1997, Aug 2006, Aug 2007] 16. Mandibulofacial dysostosis. [RGUHS Mar 1997]
5. Enumerate the osteodystrophies. Write in detail about 17. Serum alkaline phosphatase. [MUHS May 1986;
Paget’s disease of bone. [TN Feb 2007] RGUHS Apr 2000, Sep 2000]
18. Radiograph picture of fibrous dysplasia. [RGUHS Apr
2000]
Short Essays
19. Histopathology of fibrous dysplasia. [TN Oct 1999,
1. Paget’s disease. [BUHS Feb 1995; MUHS Jul 1999; Aug 2004]
RGUHS Sep 2000] 20. Pierre Robin syndrome. [TN Apr 2000, Feb 2005]
2. Pathophysiology of Paget’s disease. [RGUHS Sep 2000] 21. Cotton wool bone. [RURS Aug 2007]
720 Quick Review Series for BDS 3rd Year

22. ‘Peau d’orange’ radiographic appearance. [RGUHS-RS 15. Pernicious anaemia. [NTR-OR Apr 1998]
Mar 2006] 16. Agranulocytosis. [NTR-OR Oct 1998; NTR-NR Oct
23. Down syndrome. [NTR-OR Oct 2005, Oct 2006] 2006]
24. Brown tumour. [RGUHS Sep 2001] 17. Sickle cell anaemia. [NTR-NR Apr 2004]
25. Trisomy 21. [RUGS Mar 2006] 18. Eosinophilic granuloma. [NTR-OR Apr 1993; TN Apr
26. Philadelphia chromosome. [NTR-OR Apr 1991] 1998]
19. Haemophilia. [TN Aug 2007]
20. Plummer-Vinson syndrome. [TN Oct 2003, Feb 2007]
18. DISEASES OF BLOOD AND 21. Polycythaemia. [RGUHS RS Jan 2009]
BLOOD-FORMING ORGANS
Long Essays 19. DISEASES OF THE SKIN
1. Describe leukaemia. [MUHS Jul 2000; TN Oct 1999]
Long Essays
2. Classify anaemia. Describe pernicious anaemia in de-
tail. [MUHS Jan 1999] 1. Classify vesiculobullous lesions. Write in detail about
3. What is anaemia? Classify anaemia. Write about clini- types, clinical features, pathogenesis and histopathol-
cal features and treatment of pernicious anaemia. ogy of pemphigoid. [TN Feb 2007]
[RGUHS Sep 2000; TN Feb 2005] 2. Enumerate the vesiculobullous lesions and describe the
4. Haemophilia. [NTR-NR Apr 2005; TN Oct 1998] pemphigus vulgaris. [BUHS Aug 1993; TN Oct 1996,
5. Cyclic neutropenia. [NTR-NR Oct 2002] Apr 1999, Aug 2006]
3. Describe the aetiology, histopathology and clinical fea-
tures of lichen planus. [BUHS Sep 1996; TN Aug 2006]
Short Essays 4. List out the precancerous lesions you know and write in
1. Agranulocytosis. [RGUHS Sep 1999; TN Apr 1998] detail about lichen planus. [RGUHS Sep 1999; TN Apr
2. Pernicious anaemia. [MUHS Jul 1997] 1993]
3. Iron deficiency anaemia. [MUHS Jul 1999] 5. Hereditary ectodermal dysplasia. [TN Oct 1998, Oct
4. Plummer-Vinson syndrome. [RGUHS Apr 2001; TN 2000, Aug 2009]
Feb 2006]
5. Describe the pathogenesis and blood picture of perni-
Short Essays
cious anaemia. [RGUHS Sep 2002]
6. Clinical features and lab findings of cyclic neutropenia. 1. Erythema multiforme. [RUHS Jan 1987; MUHS Jul
[RGUHS Aug 2005] 1990; RGUHS Apr 2002]
2. Hereditary ectodermal dysplasia. [MUHS Aug 1988,
Jul 1997; BUHS Aug 1995, Aug 1996; RUOS Sep
Short Notes 2004; RGUHS Jan 2009]
1. Purpura. [MUHS Jan 1989; RURS Aug 2005] 3. Etiopathogensis of hereditary ectodermal dysplasia and
2. Rh hump. [RGUHS Sep 2004] oral manifestations. [RGUHS-OS Mar 2005]
3. Blue sclera. [RURS Mar 2006] 4. Oral manifestations of hereditary ectodermal dyspla-
4. Agranulocytosis. [MUHS Jan 1990; RURS Aug 2006] sia. [RGUHS Sep 2001]
5. Strawberry tongue. [RGUHS Sep 1999] 5. Pemphigus. [BUHS Feb 1991, Sep 1992, Feb 1996]
6. Cyclic neutropenia. [BUHS Mar 1995, Sep 1996; 6. Microscopic features of pemphigus vulgaris. [RGUHS-
MUHS Jan 1998] OS Aug 2005]
7. Plummer-Vinson syndrome. [MUHS Jul 1989] 7. Lichen planus. [BUHS Aug 1995; TN Feb 2009]
8. Oral manifestations of anaemia. [RGUHS Apr 2003] 8. Clinical features of lichen planus. [MUHS Apr 1987,
9. Oral manifestations of pernicious anaemia. [RGUHS- Jan 1999; RGUHS-RS Aug 2005]
RS Aug 2007; TN Oct 1996] 9. Histopathology of lichen planus. [MUHS Jun 1989, Jul
10. Leukaemia. [MUHS Jul 1999] 1998; BUHS Feb 1996; RUOS Mar 2004; TN Apr
11. Oral manifestations of leukaemia. [BUHS Apr 1987; 2000, Oct 2003]
TN Sep 2002] 10. Draw and write histopathological features of lichen
12. Haemophilia. [BUHS Sep 1996; RGUHS Apr 2000; planus. [RGUHS Apr 2001]
MUHS Jul 1999] 11. Auspitz’s sign. [NTR-OR Apr 2006]
13. Mention the factors involved and associated clinical 12. Nikolsky’s sign. [NTR-NR Apr 2004, Oct 2007; TN
features of haemophilia. [RGUHS Apr 1999] Apr 1995]
14. Thalassaemia. [NTR-OR Oct 1997] 13. Munro’s abscess. [NTR-NR Apr 2006]
Section | V Previous Years’ Question Bank 721

14. Grinspan syndrome. [NTR-OR Oct 2004] 3. Differences between trigeminal neuralgia and glosso-
15. Erythema multiforme. [NTR-OR Oct 2010; TN Aug pharyngeal neuralgia. [RGUHS Sep 2001; TN Oct
2009] 1991]
16. Neurilemmoma. [TN Aug 2006] 4. Neuralgic pain. [NTR-NR Oct 2002]
17. Benign mucous membrane pemphigoid. [TN Apr 5. Eagle’s syndrome. [NTR-NR Apr 2004]
2001] 6. Frey’s syndrome. [NTR-NR Apr 2005, Oct 2007]
7. Myofacial pain dysfunction syndrome. [TN Oct 1998,
Oct 2000]
Short Notes
1. Pemphigus. [NTR-OR Apr 2002]
Short Notes
2. Oral lichen planus. [NTR-OR Oct 1992, Apr 1997]
3. Pemphigus vulgaris. [NTR-OR Apr 1983, Oct 1993, 1. Bell’s palsy. [NTR-OR Oct 1992, Oct 1998, Apr 2000;
Oct 1995; TN Oct 2003] NTR-NR Apr 2007; TN Apr 1998, Nov 2001]
4. Ectodermal dysplasia. [NTR-OR Apr 1993; TN Feb 2. Frey’s syndrome. [NTR-OR Apr 1999; TN Feb 2010]
2005] 3. Eagle syndrome. [NTR-OR Oct 1995]
5. Erythema multiforme. [NTR-OR Oct 1999] 4. Trigeminal neuralgia. [NTR-OR Oct 1996; NTR-NR
6. Stevens-Johnson syndrome. [TN Apr 1996, Aug 2006] Apr 2006; TN Apr 1998, Aug 2005, Aug 2009]
7. Lupus erythematosus (LE) cells. [RGUHS Apr 2003; 5. Fothergill’s disease. [NTR-OR Oct 2003]
RUOS Aug 2005] 6. Bell’s palsy. [RUOS Mar 2006]
8. Tzanck cells. [MUHS Oct 1987; RUOS Aug 2005] 7. Trigger zones. [RURS Feb 2007]
9. Tzank test. [MUHS Nov 1986, Oct 1987, Jun 1989; 8. Eagle syndrome. [RGUHS Apr 2000]
BUHS Mar 1992; RGUHS Sep 2000, Apr 2002, Apr 9. Glossopharyngeal neuralgia. [RGUHS Apr 2003]
2003; RURS Feb 2007; TN Sep 2002] 10. Tic douloureux. [TN Aug 2007]
10. Paul-Bunnell test. [RURS Mar 2006]
11. Nikolsky’s sign. [MUHS Jan 1990, Jul 1991; RUHS
Mar 2004; TN Oct 1996] 21. FORENSIC ODONTOLOGY
12. Auspitz’s sign. [RGUHS Sep 1991, Sep 2002; RUOS
Short Notes
Mar 2005]
13. Munro’s abscess. [RUOS Mar 2006; RURS Mar 2006] 1. Two diagnostic uses of saliva. [NTR-NR Oct 2007]
14. White sponge nevus. [MUHS Jan 1951] 2. Cusp of Carabelli. [NTR-OR Apr 2006]
15. Stevens-Johnson syndrome. [MUHS Jul 1997; TN Feb
2009]
16. Corps, rounds and grains. [RUOS Sep 2004] 22. MISCELLANEOUS TOPICS
17. Clinical types of lichen planus. [RGUHS Apr 1998]
18. Oral manifestations in hereditary ectodermal dysplasia.
Short Notes
[TN Apr 2001] 1. Autoclave. [NTR-OR Apr 2000]
19. Grinspan syndrome. [RGUHS-RS Jan 2009] 2. Odontalgia. [NTR-OR Oct 1997]
20. Suprabasilar split/cleft. [RGUHS Apr 2001] 3. Gram staining. [NTR-OR Oct 1997]
4. Sterilization of the dentin. [NTR-OR Apr 2000]
5. Histopathology of dentin. [NTR-OR Apr 1996]
20. DISEASES OF THE NERVES 6. Method of taking swab in root canal infection. [NTR-OR
AND MUSCLES Apr 2000]
7. Common and special stains used or histopathological
Long Essays
diagnosis of the lesion. [NTR-OR Apr 2000]
1. Mention the different types of neuralgia; explain in 8. Haematoxylin and eosin staining. [NTR-OR Oct 1996]
detail about trigeminal neuralgia. [NTR-OR Apr 1998] 9. Barodontalgia. [NTR-OR Apr 2000]
10. Dead tracts of Fish. [NTR-OR Apr 2004]
11. Isomorphic phenomenon of Köbner. [NTR-OR Apr
Short Essays
2006]
1. Bell’s palsy. [MUHS Jan 1987, Apr 1987, Jan 1998, Jul 12. Culture media for Candida albicans and Tubercle
2000; RGUHS Apr 1999; NTR-NR Apr 2005] bacilli. [NTR-OR Apr 2006]
2. Trigeminal neuralgia. [BUHS Nov 1986; RGUHS Feb 13. Fixatives. [TN Sep 2002]
1996, Mar 1997; MUHS Jul 1996, Jan 1997; RURS Aug 14. Exfoliative cytology. [TN Apr 1996, Apr 1998, Oct
2006] 2000]
This page intentionally left blank
Index

A Acute bacterial parotitis, 319 Adenoma, 457, 458, 459 Amelogenesis imperfecta,
Abdominal actinomycosis, Acute bronchitis, 83–84 Adenomatoid odontogenic 427, 436
270, 276 Acute candidiasis, 503–504 tumour, 476, 483 Amoebic ulcerations, 10
Abfraction, 552 Acute caries, 528 Adjunctive therapy, 125 Aminoglycosides, 212
ABO incompatibility, 265 Acute circulatory failure, 186 Adrenergic discharge, 255 Amoebiasis, 21
Abortive therapy, 354 Acute gastritis, 13, 19 Adrenocorticotropic hormone, Amoebic dysentery, 187
Abrams needle, 100 Acute hepatitis, 24 259 Amoebic serology, 188
Abrasion, 240, 551–552, 556 Acute inflammation, 241 Adventitious dentin, 553 Amorphous eosinophilic
Abrasions (grazes), 246 Acute ischaemic syndrome, 57 AFB smear, 110–111, 115–116 bodies, 611
Abscess, 283 Acute left ventricular failure, 59 African jaw lymphoma, 448 Amoxicillin, 409
Absence seizure, 362 Acute leukemia, 592 Afterload, 59 Amputation neuroma, 453
Absorbable suture Acute lymphoblastic leukemia, Age changes, 555 Amputations, 338
materials, 235 593 Agglutination, 110 Amyloidosis, 572
Acanthomatous type, 470 Acute myeloid leukemia, 46, Agglutination test, 507 Anaemia, 5, 49
Acanthosis nigricans, 609 593 Aggressive lymphomas, 347 Anaerobic infections, 280
Accessory cusps, 426, 433 Acute myocardial infarction, Aggressive periodontitis, 514 Anaesthesia, 248
Accessory salivary glands, 467 56, 57–58 Aggrevating factors, 77 Analgesics, 123
Acetyl salicylic acid, 79 Acute myocardial ischaemia, Agranulocytosis, 40, 50, 596 Anaphylactic mediators, 196
Achondroplasia, 591 132 AIDS, 183, 192, 282, 286, Anaphylactic or atopic
Acid fast bacilli (AFB), 485, Acute necrotizing ulcerative 455, 500, 505, 506, 507 reaction, 195, 200
494 gingivitis (ANUG), AIDS, HIV, 261, 266 Anaphylactic shock, 199, 262
Acid peptic disease, 14 508–509, 511, 513 Airway, 247 Anaphylaxis, 196, 198, 407
Acid-fast (AFB), 479 Acute nephritis, 142, 144 Airway maintenance, 251 Anaplastic carcinoma, 384
Acidogenic theory, 517–518, Acute parotitis, 319, 320, 325 Airway management, 248 Anaplastic carcinoma of
522–524, 533 Acute pericarditis, 77 Airway potency, 263 thyroid, 383
Acids, 523, 527 Acute poisoning, 204 Albers-Schonberg disease, 587 Androgens, 43
Ackerman’s tumour, 448 Acute pseudomembranous oral Albert’stain, 491 Anemia, 36, 44, 170, 172, 594,
Acquired autoimmune candidiasis, 505 Albright’s syndrome, 590 598
disorder, 616 Acute pseudomembranous Albuminuria, 169 Aneurysm, 289, 342, 353
Acquired enamel hypoplasia, type, 503–504 Alkaline dieresis, 206, 209 Aneurysm of aorta, 351
434 Acute pulmonary oedema, 61, Alkaline phosphatase, 446 Aneurysmal bone cyst, 289,
Acquired immune deficiency 77 Allergen, 199 451, 480
syndrome, 183 Acute pulpitis, 534–535, 538 Allergic reaction, 261, 266 Angina, 66
Acquired syphilis, 487–489 Acute rheumatic fever, 64, Allergy, 195, 199, 406 Angina decubitus, 66
Acrodynia, 548 81–82 Alport syndrome, 139–140 Angina ludovici, 284
Acromegaly, 153, 163, 568 Acute suppurative Alveolar abscess, 275 Angina pectoris, 66, 77, 82
Actinic or solar keratosis, 300 osteomyelitis, 535–537 Alveolar osteitis, 560–561, 566 Angioedema, 199
Actinomyces, 531 Acute suppurative pulpitis, Amalgam tattoo, 549 Angioneurotic oedema, 576
Actinomycosis, 193, 269, 276, 534 Ambulatory Holter Angiotensin converting
285, 486–487, 492, 494 Adamantinoma, 295, 300, 302 monitoring, 62 enzyme, 54
Actinomycosis bovis, 495 Adamantoblastoma, 481 Ambulatory phlebectomy, 339 Angiotensin-converting
Activated charcoal, 208 Addison’s disease, 155, 160, Ameloblastic adenomatoid enzyme inhibitors, 58
Active immunity, 279 164, 568 tumour, 476, 483 Angular stomatitis, 179, 313
Active immunization, 33 Adenoameloblastoma, 483 Ameloblastic fibroma, 478 Anhydrotic ED, 607
Active lobules, 391 Adenoid cystic carcinoma Ameloblastic odontoma, 291 Ankyloglossia, 315
Acute alveolar abscess, 541 (cylindroma), 459, Ameloblastoma, 291, 292, Ankylosis, 579–580
Acute asthma, 104 466–67 300, 470–471, 481 Ankylosis of TMJ, 579
Acute atrophic type, 504 Adenolymphoma, 325 Ameloblasts, 295 Ankylostoma duodenale, 189

723
724 Quick Review Series for BDS 3rd Year

Ankylostomiasis, 189 ARDS, 107 Bacteremia, 281 Biopsy technique, 564–565


Anodontia, 429, 435 Arrested caries, 528, 532, 533 Bacterial adherence, 520 Biopsy wound, 564
Anorexia, 179 Arrhythmias, 62 Bacterial contamination, 246 Biotin, 171
Antacids and alginates, 7, 12 Arsenic poisoning, 207, 210 Bacterial infections, 484 Birth injuries, 428
Anterior cervical Arterial blood gas analysis, 108 Bacterial invasion, 530 Bite marks, 617
lymphadenopathy, 340 Arterial supply of tonsils, 334 Bacterial meningitis, 125 Bitot’s spots, 174, 178, 180
Anti neoplastic agents, 240, Arterial ulcer, 350 Bacterial plaque, 518, 523, Biventricular heart failure, 60
248, 251–252 Arteriography, 352 524–526, 527 B-lymphocytes, 198
Antithyroid drugs, 388 Arterio-venous fistula, 344 Bacterial pneumonia, 100 Black eye, 373
Anti-anginal medication, 80 Artificial bovine skin, 248 Bag valve mask (BVM), 375 Black hairy tongue, 438
Antibiotic regimen, 70 Artificial respiration, 375 Baillarger’s syndrome, 363 Blastomyces dermatitidis, 505
Antibiotic therapy, 22, 101, Ascariasis, 189 Balanced diet, 177 Bleeder’s disease, 595
125, 505 Ascites, 27, 28, 30, 33, 55, 62 Barbiturate poisoning, 210 Bleeding disorders 40, 41
Antibiotics, 96, 106, 127, 140, Ascites formation, 28 Barium meal, 5, 17 Bleeding gums, 571
141, 142, 143, 144, 183, Ascites in cirrhosis, 28 Barium meal contrast Bleeding manifestations, 42
186, 212, 283, 575 Ascitic fluid, 28 radiography, 9 Bleeding time, 47
Anti-cholinergics, 123, 207 Ascorbic acid (Scurvy), 171 Barium swallow, 10, 16 Bleeding wounds, 252
Anticonvulsants, 121, 361 Asepsis in surgery, 237–238 Basal cell carcinoma, 284, Bleeding/hemorrhage, 241
Anti-D sera, 268 Aspiration pneumonia, 107, 295, 300, 453, 456 Block neck dissection,
Anti-epileptic drugs, 132, 137 112 Basal cell nevus, 473 341–342
Antihaemophilic globulin Aspirin, 67, 79 Basal cell type, 471 Blood culture, 69, 125
(AHG), 595 Associated illnesses, 241 Basal crackles, 55 Blood dyscrasias, 18, 266
Anti-HAV, 31 Asthma, 84 Basedow’s disease, 165 Blood examination, 26, 138
Anti-HIV antibody, 283 Atherosclerosis, 150 BCC, 194–195, 300 Blood film, 47
Antihypertensive drugs, 54 ATLS principals, 366 BCG vaccination, 108 Blood loss, 105, 260
Antihypertensive drug therapy, Atopy, 196 Bechet’s syndrome, 577 Blood picture, 599
53 Atrial fibrillation, 55, 77 Bed rest, 64 Blood pressure, 61
Anti-microbial/Anti-diarrheal Atrial septal defect, 78 Bedsores, 343–344 Blood test, 84, 384, 386, 390
treatment, 16 Atrial tachycardia, 74 Behcet’s syndrome, 575–576 Blood transfusion, 44, 46, 258,
Antimicrobial treatment, 125 Atrophic hypothyroidism, 156 Bell’s palsy, 134, 361, 354, 260, 265, 269
Anti-motility agent, 16 Atropine, 207 355, 614, 615 Blue domed cyst, 294
Anti-oxidants, 410 Attrition, 551, 556, 558 Bell phenomenon, 120, 128 Blue nevus, 446
Antiplatelet agents, 57, 67 Atypical facial pain, 616 Bence-jones protein, 451 Blue sclera, 590
Antiplatelet drugs, 80 Aura, 121, 129 Benign cystic teratoma, 298 Blue warning card, 568
Anti-RhD globulin, 43 Aura phase, 354 Benign epithelial odontogenic Body of lesion, 519, 521, 529
Antisepsis, 237 Auriculotemporal syndrome, tumours, 480 Body surface area (BSA)
Antiseptic principle of practice 363, 615 Benign tumours, 457, 469, 475 burned, 253
of surgery, 241 Auspitz’s sign, 611 Beriberi, 173, 176, 180 Body’s defense system, 409
Antispasmodic, 207 Autoclave, 235, 617 Bernard-Horner syndrome, Bogorad syndrome, 614
Antistchkow cells, 577 Autoclave sterilization, 234, 363 Bohn’s nodules, 437
Antistreptococcal therapy, 64 238 Bernstein test, 5, 17 Boils, 277, 283
Antithrombin therapy, 58 Autoimmune disorder, 42 Beta adrenoreceptor Bone marrow, 595, 597
Antithyroid drugs, 158, 162 Autoimmune theory, 518–522 antagonists, 73 Bone marrow picture, 46
Antitoxin, 127 Autosomal dominant fibro- Beta blockers, 54, 57–58, 62, Bone marrow transplantation,
Antitubercular drugs, 90, 98, osseous lesion, 588 67, 73, 80, 256, 385 38, 593
119, 486 Autosomal recessive, 513 Beta-hemolytic group A Bone marrow study, 38
Antiviral agents, 194 Autotransplantation, 567 streptococci, 277 Borelia vincenti, 514
Anxiety disorder, 136, 361 Avitaminosis - A, 179 Betel nut, 310 Bottle mouth syndrome, 529,
Aortic regurgitation, 78 Avulsion, 240, 248 Betel quid, 408 532
Aortic valve, 63 Axillary freckles, 346 Bicarbonate administration, Boundaries of back of neck, 280
Aphthous stomatitis, 18 Axonotemesis, 359 151 Bowen’s disease, 290, 301
Aphthous ulcers, 305, 312 Bicycle ergometry, 66 Bradycardia, 78
Apical periodontal cyst, 476, B Bier’s block, 410 Brain natriuretic peptide, 62
480, 537, 541 b hemolytic streptococci, 192 Bifid rib syndrome, 473 Branchial cleft cyst, 340
Aplastic anemia, 37, 48 B lymphocyte, 199 Bile acids, 8 Brandy wine type
Appearance of epithelial Babinski’s sign, 135 Biliary tract, 274 dentenogenesis
ridges, 611 Baby bottle syndrome, 529, 532 Bilirubin metabolism, 26 imperfecta, 429
Appearance, 602 Bacille Calmette–Guerin Biopsy, 392, 386, 441, Breath tests, 9
Arachnodactyly, 590 (BCG), 108, 194 563–564, 566 Brittle bones, 586
Index 725

Bronchi, 281 Carbuncle, 273, 277, 278, 283 Cavernous hemangioma, 289 Chlamydospores, 618
Bronchial asthma, asthmatic Carcinoma cheek, 306 Cavernous sinus, 279 Chocolate colored eosinophilic
bronchitis, COPD, 207 Carcinoma in situ, 290, 308 Cavity formation, 526–527 coagulum, 463
Bronchial breathing, 109 Carcinoma of cheek, 311 CEA level, 386 Cholestatic jaundice
Bronchial obstruction, 96, 102 Carcinoma of lip, 310, 314 Cell mediated immunity, 287 (Obstructive jaundice), 27
Bronchiectasis, 94, 111, 117 Carcinoma of maxillary sinus, Cellular immunity, 287 Cholestrol crystals, 483
Bronchodilators, 84, 96, 106 307–308 Cellular ischemia, 254 Chorea (Sydenham’s chorea;
Bronchopneumonia, 100 Carcinoma of parotid gland, Cellular layer, 446 Chorea minor; Saint
Bronchoscopy, 97, 102 317 Cellulitis, 284, 545 Vitus’ dance), 64
Brudzinski’s sign, 124, 137, Carcinoma of the cheek, 311 Cemental caries, 530 Christian disease, 588
357, 362 Carcinoma of thyroid, 382 Cementicles, 554, 558 Christmas disease, 44, 595
Bruxism, 550 Carcinoma of tongue, 302, Central cyanosis, 70, 74 Chronic apical periodontitis,
Budd-Chiari syndrome, 28, 33 304–305, 314, 315 Central giant cell granuloma, 536
Buerger’s disease, 335–336 Carcinoma staging, 290 443–444, 449–450 Chronic atrophic type (denture
Buffer capacity test, 524 Carcinoma-in-situ, 448, 454 Central ossifying fibroma, stomatitis or denture sore
Bulbous rete ridges, 440 Carcinomas, 302 440 mouth), 504
Bull like teeth, 426–427, 435 Carcinomatous epulis, Central venous pressure, 260 Chronic bacterial infection, 269
Bull’s eye lesions, 612 309–310 Cephalic form, 281 Chronic bronchial asthma, 104
Burkitt’s lymphoma, 296, 449 Carcinomatous ulcer, 312 Cephalic tetanus, 272, 273 Chronic bronchitis, 84, 95, 106
Burn area, 247 Cardiac arrest, 75 Cephalosporins, 212 Chronic caries, 528
Burn depth, 245, 252 Cardiac asthma, 61 Cerebral malaria, 190 Chronic cor pulmonale, 65
Burn management, 248 Cardiac cachexia, 62 Cerebral symptoms, 61 Chronic diarrhea, 20
Burn severity, 246 Cardiac catheterisation, 62, 78 Cerebrovascular accident, 122 Chronic focal sclerosing
Burn wound grafting, 253 Cardiac failure, 59, 82 Cerebrovascular disease, 71 osteomyelitis, 541
Burning sensation, 21, 316 Cardiac injury enzymes, 56–57 Cervical (unilateral) Chronic gastritis, 13–18, 21
Burning tongue, 438 Cardiac oedema, 61 lymphadenopathy, 340 Chronic hypertrophic type, 504
Burns, 245, 246, 247, 251, 252 Cardiac syncope, 71 Cervical caries, 528, 531 Chronic inflammatory
Burns of face, 252 Cardiac vagolytic, 207 Cervical enamel extensions, reaction, 535
Buschke - Lowenstein’s Cardinal signs, 163 426, 433 Chronic leukemia, 592–593
tumour, 290, 301 Cardiogenic shock, 56, 255, Cervical lymphadenopathy, Chronic myeloid leukemia, 39,
262, 267 340 46, 50
C Cardiogenic/septic shock, 141 Cervical ranula, 297, 323 Chronic myeloproliferative
Cafe-au-lait spots, 455, 591 Cardiomegaly, 63 Cervical rib, 340, 352 disorders, 38
Caffey’s disease, 592 Cardiomyotomy and Cervical lymphadenopathy, Chronic nonspecific ulcer, 305
Calcification, 554 vagotomy, 4 184 Chronic obstructive lung
Calcifying epithelial Cardiopulmonary resuscitation Cervicofacial actinomycosis, disease, 113
odontogenic tumour, (CPR), 375 193, 270, 276, 493 Chronic obstructive pulmonary
475–477, 478, 481 Cardiorespiratory, 156, 160 Cessation of bleeding, 48 disease, 65
Calcifying odontogenic cyst, Cardiovascular changes, 247 Chancre, 278, 487, 488, 490 Chronic osteomyelitis, 535
478 Cardiovascular disorders, 76 Charcot foot, 242 Chronic periodontal abscess,
Calcinosis cutis, 612 Carditis, 63 Chediak-Higashi syndrome, 515
Calcium antagonists, 67 Carey Coombs’ murmur, 63 601 Chronic periodontitis, 515
Calcium blockers, 67 Caries activity test, 524 Cheiloschisis, 376, 377, 378 Chronic rheumatic mitral
Calcium channel blockers, Caries etiology, 518–519 Cheiloscopy, 617 stenosis, 55
54, 80 Caries susceptibility, 533 Chelating agent, 518 Chronic specific
Calcium homeostasis, 161 Caries, 516, 520–521, 522, Chelation, 518 granulomatous
Calcium metabolism, 161 527, 531 Chemical burns, 245, 252 inflammation, 339
Callus formation, 562 Cariogenic bacteria, 530 Chemical measures, 527 Chronotropic agents, 256
Cancrum oris, 313, 487, 494 Cariogenic microorganisms, Chemical sterilization, 233 Churg-Strausss syndrome, 44
Candida albicans, 438, 452, 531 Chemosis, 388 Chylous ascites, 33
503, 505, 506, 507, 618 Carotid body tumor, 351 Chemotherapy, 296, 302, 304, Circulatory failure, 183
Candida, 437 Carpopedal spasm, 168 306, 307, 311, 340, 393 Cirrhosis of liver, 25, 31, 33
Candidiasis, 503, 504, 506 Caseating tubercule, 339 Cherubism, 580, 584, 586, 588 Civatte, 605, 608
Cannon’s disease, 611 Caseation necrosis, 339 Chest radiograph, 84 Civatte bodies, 611, 612
Capillary hemangioma, 289, Cathartics, 205, 208 Chest X-ray, 55, 69, 78, 485 Clark’s level of invasion, 302
292, 447, 448 Catheter arteriography, 352 Cheyne-Stokes respiration, 61 Classic hemophilia, 603
Capnocytophaga, 531 Catheterise, 247 Chicken pox, 185, 192, 502 Classical absence, 136
Carbohydrates, 9, 517–518, Cavernous haemangioma, 301, China-blue eyes, 590 Classical migraine, 127
523, 526 454, 544 CHIVA, 339 Classification of wounds, 245
726 Quick Review Series for BDS 3rd Year

Claude Bernard-Horner Congenital syphilis, 181, 191, CSF examination, 125 Dementia, 179
syndrome, 363 193, 427, 488, 489, 490 CSOM, 409 Demirjian’s method, 617
Cleft lip, 376, 377, 378, 379 Congenital ventricular septal CT and MRI, 390 Demonstration of AFB, 111
Cleft palate, 376, 377, 378 defect, 68 CT scan, 115, 122, 306, 308 Dens evaginatus (occlusal
Cleft lip and palate, 430 Congestive cardiac failure, Culture, 506 tuberculated premolar,
Cleidocranial dysplasia, 433, 140, 141 Culture media, 618 Leong’s premolar,
585, 589 Congestive hepatomegaly, 62 Culture of bone marrow, 182 evaginated odontome),
Clonic phase, 355–356 Conjugated hyperbilirubinemia, Culture of organism, 270 426, 433
Clopidogrel, 67, 80 27, 29 Cushing’s syndrome, 52, 162 Dens in dente, 426, 433
Closed wound, 248 Conjunctivae, 174 Cusp of Carabelli, 426, 433 Dens invaginatus, 435
Clubbing, 3, 111, 116–117 Connective tissue neoplasms, Cutaneous changes, 203–204 Dens-in-dente, 435
Cluster headache, 137 457, 458, 459 Cutaneous horn, 290, 301 Dental age estimation, 617
Clusters of boils, 277 Constipation, 16, 18 Cutis hyerelastica, 612 Dental calculus, 512
Coagulation defects, 40–41 Constrictive pericarditis, 75 CVP, 256, 262 Dental caries, 516–17, 519–20,
Coagulopathy, 138 Contact stomatitis, 577 Cyanocobalamine deficiency, 522, 523, 524, 525
Cocaine drop test, 363–364 Continuous positive airway 179 Dental fluorosis, 211
Coccidioidomycosis/valley pressure (CPAP), 329 Cyanosis, 61, 70 Dental granuloma, 536
fever, 507 Control of hemorrhage, 262 Cyanotic convulsion, 281 Dental management, 570
Codman’s triangle, 446 Contusion, 240, 249 Cyclic neutropenia, 596, 599 Dental plaque, 520, 524, 531
Cold abscess, 286 Conventional NSAIDs, 203 Cylindroma, 467 Dental root end cyst, 480
Collapsing pulse, 78 Conventional radiographs, 99 Cyst, 297, 468, 471 Dental sclerosis, 552
Collar-stud abscess, 334 Cooley’s anemia, 49, 602 Cystic hygroma, 301, 340, 349 Dental stains, 515
Colloidal bismuth compounds, Coombs’ test, 47, 50 Cysts of mandible, 287 Dental ulcer, 305, 312
7, 12 COPD, 96 Cysts of jaws, 468, 471 Dentigerous cyst, 288, 292,
Colloids, 252 Coronal dentin dysplasia, Cytoid bodies, 605, 608 294, 299, 468–469, 481
Colonic stasis, 16–17 428–429 Cytokines, 287 Dentin dysplasia, 428, 436
Common lysosomal storage Coronary arteries bypass Cytologic reactions, 195–196 Dentinal sclerosis, 530, 559
disease, 574 grafting, 59 Cytotoxic drugs, 240, 248 Dentinogenesis imperfecta,
Common migraine, 127 Coronary arteriography, 66 Cytotoxic T cells, 407–408 429, 436
Community-acquired Coronary artery bypass Cytotoxin production, 15 Dentoalveolar abscess, 275
pneumonia (CAP), 88–90 grafting (CAB G), 67 Depressed fracture skull, 375
Compensatory emphysema, Coronary heart disease, 82 D Depression, 130, 132, 357
104 Coronoid fractures, 367 Dalrymple sign, 388 Depressive disorders, 130
Complete anodontia, 430 Corps, ronds and grains, 611 Dangerous area of face, 279 Depressive episode, 358
Complex odontoma, 291, 300, Corticosteroid therapy, 43, Darier’s disease, 612 Dermatitis, 175, 180
474–475, 480 204 Darier-White disease, 612 Dermoid cyst, 293, 297, 298
Complicated migraine, 127 Corticosteroids, 11–12, 240, Dark zone, 519, 521, 529, Desensitization, 105
Complication of lipoma, 301 43, 64, 87, 197, 199, 532 Desquamative gingivitis, 515
Complications 247–48 Darkfield microscopy, 181, Destructive endogenous oral
of blood transfusion, 269 Corynebacterium diphtheriae, 487, 488 infection, 508
of fracture, 374 485, 491 Darling’s disease, 506 Developmental anomalies,
of tracheostomy, 332 Cotton wool appearance of Dead tracts, 551, 552 433
Composite graft, 253 bone, 591 Debridement, 338 Developmental cysts, 438,
Compound fracture, 369, 373 Country man’s lip, 314 Decalcification, 530 473, 477
Compound odontoma, 291, COX-2 inhibitors, 203 Decomposed dentin, 530 Developmental disturbances,
480 Craniofacial dysjunctions, 376 Decubitus ulcers, 343 425, 428
Computer tomography (CT), Craniofacial form of fibrous Deep internal bleeding, 265 Diabetes mellitus, 240, 149,
321 dysplasia, 580, 584 Deep plunging ranula, 297 152, 168, 249–250, 252
Concrescence, 426, 433 Craniotabes, 175, 178, 201 Deep vein thrombosis, 81 Diabetic coma, 573
Condyloma latum, 278 Crest syndrome, 452, 612 Deficiency of vitamin D, 204 Diabetic gangrene, 337–342
Condylomata acuminatum, Cretinism, 165 Defluoridation, 428 Diabetic ketoacidosis, 149,
290, 301 Crew-cut, 49, 602 Degenerative disorders, 122 150–158, 161
Confirmatory tests in AIDS, Criggler-Najjar syndrome, 27 Dehydration, 145, 147 Diabetic ketoacidotic coma,
282 Crush syndrome, 268 Delay wound healing, 250 161
Congenital anomalies, 377 Crush wound, 249 Delayed hypersensitivity, 577 Diabetic microangiopathy, 150
Congenital epulis, 453 Cryosurgery, 21, 236, 339 Delayed primary closure, 247 Diabetic nephropathy, 150
Congenital heart disease, 71, Cryotherapy, 236 Delayed primary intention, 244 Diabetic neuropathy, 150
83 Crystalloids, 252 Delayed wound healing, 241 Diabetic ulcer, 242, 244
Congenital leukokeratosis, 611 CSF analysis, 124 Delta hepatitis, 32 Diagnosis criteria of Olin, 336
Index 727

Diagnosis of TB, 485 Dry heat sterilization, 233 Enamel-hypoplasia, 427 Exercise stress testing, 62
Diagnostic imaging Dry mouth, 464 Endocrinal disorders, 52 Exercise test, 108
techniques, 318, 321 Dry socket, 560–561, 562 Endometritis, 281 Exertional syncope, 71
Diagnostic paracentesis, 28 Dry sterilization process Endoscopy, 5, 9, 13, 16, 17, 26 Exfoliative cytology, 563,
Diagnostic uses, 617 (DSP), 234, 237 Endotracheal intubation, 262, 618
Dialysis, 206, 209 Dubin-Johnson and Rotor 334 Exophthalmos, 167, 387–388
Diarrhoea and dementia, 180 syndromes, 27, 29 Endovenous thermal ablation, Exostoses, 454, 456
Diathermy, 238 Duke’s criteria, 69 339 Expansion of mandible, 294
Diazepam, 129, 356 Dull percussion, 62 Entamoeba histolytica, 10, 21 Expectorating cough, 95
Diffuse calcifications, 553, 556 Duodenal ulcer, 8, 17 Enteric fever, 186, 193 Expired air resuscitation
Digoxin, 79 Dupuy’s syndrome, 363 Enucleation, 469, 481 (EAR), 375
Dilaceration, 427, 434, 438 Dupuytren’s contracture, 26, Environmental enamel Expired air ventilation (EAV),
Dilantin, 211 31 hypoplasia, 427–429 375
Dilantin gingival hyperplasia, D-Xylose test, 9 Eosin staining, 618 External resorption, 553–554
513 Dysphagia, 10, 16, 19, 331 Eosinophilia, 40, 43, 48 External resorption
Dilantin sodium, 133, 363 Dysphagia lusoria, 328 Eosinophilic granuloma, 572 of teeth, 559
Dilation lag test, 364 Dysplasia, 315, 448 Epidermal inclusion cyst, 437 Extra-articular origin, 579
Dip stick tests, 195 Dyspnoea, 61, 74, 101, 106 Epidermoid carcinoma, 449 Extraction, 564
Diphenylhydantoin sodium, Epidermoid carcinoma of oral Extradural hematoma, 375
211 E cavity, 444 Extrinsic disorders, 579
Diphtheria, 182, 186, 189, 190 Eagles syndrome, 616 Epidermoid cells, 467 Exudates, 28, 33
Diplopia, 388 Early onset periodontitis, 510 Epidermoid cyst, 296, 298 Exudative effusion, 99
Direct examination, 506 Eburnated dentin, 528 Epidural anesthesia, 405
Direct thrombin inhibitors, 58 Echocardiography (ECG), 55, Epigastric pain, 6 F
Directly observed treatment, 57, 62 Epilepsy, 120–122, 131, 137 Facial causalgia, 616
short course (DOTS), 99 Ectodermal dysplasia, 607 Epimyoepithelial islands, 465 Facial nerve paralysis, 406
Disease of kings, 595 Ectopic enamel, 426, 433–434 Epinephrine, 259 Facial pain, 136, 361
Disinfection, 238 Effective circulating volume, Episodic pain, 6 Facial palsy, 120, 128–129
Dislocation, 371 254 Epithelial cysts, 468, 471 Facial paralysis, 136
Disorders affecting jaws, Effective thyroxine ratio, Epithelial dysplasia, 441, 448 Falciparum malaria, 190
579–580 158–159 Epithelial odontoma, 481 Fallots tertralogy, 83
Disseminated intravascular Effects of radiation on Epithelial tissue neoplasm, False denticles, 553, 556
coagulation (DIC), 266 tissues, 547 457, 458, 459 Familial fibrous dysplasia,
Distress syndrome (ARDS), 86 Ehlers-Danlos Syndrome, 612 Epithelioma, 299 586
Distributive shock, 255–256 Electrical burns, 245, 252 Epsilon amino-caproic acid, 42 Familial fibrous dysplasia of
Diuretics, 28, 53, 62, 140, 142, Electrocardiographic Epstein-Barr Virus (EBV), jaws, 588
143, 144–145, 148, 256 manifestations, 146 452, 485, 502 Fat, 9
Dizziness, 361 Electrocardiography, 56, 62, 66 Epulis, 309, 316 Fat soluble vitamins, 180
DNA viruses, 496, 497–498 Electrocoagulation, 358 Ergotamine, 123 Fatty degeneration, 530
Doppler ultrasonography, 344 Electroencephalographic Erosion, 552, 557–558 Fecal fat estimation, 9
DOTS, 115, 340 changes, 35 Erythema marginatum, 64 Feeding, 377
Double reverse z-plasty, 378 Electromyography, 128 Erythema multiforme, 605, 607 Felty’s syndrome, 40
Down syndrome, 590 Elevation of bleeding area Erythroblastic anemia, 49 Ferrokinetic studies, 38
DPT, 194 against the gravity, 257 Erythroblastosis fetalis, 50 Fibroepithelial polyp, 542
Dressings, 251 ELISA, 192, 195, 282, 486, Erythrocyte sedimentation rate Fibroma, 451
Dressler’s syndrome, 56 489, 494, 506 (ESR), 97, 102 Fibroma molluscum, 454
Drug allergy, 199 Ellis classification, 547 Erythroedema polyneuritis, 548 Fibrous dyaplasia, 579,
Drug hypersensitivity, 607 EM major, 607 Erythroplakia, 291, 309–313, 580–582, 583, 589
Drug induced aplasia, 37–38 Embolism, 350 442, 445, 452 Fibrous healing of extraction
Drug or chemical-induced EMG, 128 Esophageal dysfunction, 612 wound, 561–562
DM, 152 Emperical antibiotics, 125 Esophagus, 328 Fibrous epulis, 309
Drug reactions, 211 Emperical therapy, 70 ESR, 50 Field block, 405
Drug resistance in tuberculosis, Emphysema, 104, 109 Ethylene oxide, 233 Fifth cranial nerve, 614
113 En bloc resection, 296, 302 Ethylene oxide gas, 236 Fine needle aspiration (FNA),
Drug therapy, 62–63, 121, 123 Enamel caries, 519, 521, Euthyroid, 388 321, 341, 406, 618
Drumstick appearance, 3 529, 532 Excessive thyroid hormones, Fine needle aspiration cytology
Dry gangrene, 286, 336–337, Enamel hypoplasia, 428 157 (FNAC), 406, 618
342, 352 Enamel pearl, 426, 433, 484 Excisional or incisional Finger clubbing, 3
Dry gangrene of pulp, 538 Enamel solubility test, 524 biopsies, 341 Fistula, 244, 285
728 Quick Review Series for BDS 3rd Year

Flapping tremor, 35 G Glossitis, 11, 19, 308 Haemorrhage, 260, 263, 267
Floating intestines, 28 GABA agonist, 133 Glossitis causing ulcers, 306 Haemorrhagic cyst, 483
Floating tooth syndrome, 586 GABA receptors, 212 Glossopharyngeal nerve, 614 Haemorrhagic strokes, 122
Flow volume curves, 108 GABA receptor agonist, Glossopharyngeal neuralgia, Hagedorn-LeMesurier repair,
Fluctuant mass, 275 362–363 614 382
Fluid balance, 89 Gaint cell granulomas, 449 Glucose tolerance test, 162 Hailey-Hailey disease, 609
Fluid replacement, 150–151 Gangrene, 336–337, 342–343 Glucosuria, 164 Hairy leukoplakia, 452
Fluid resuscitation, 247, 249, Gangrene of pulp, 534 Glutaraldehyde, 233–236, 237 Hairy tongue, 438
262, 263 Gangrenous stomatitis, 487 Glutarldehyde, 238 Halitosis, 18, 21, 306
Fluid thrill, 28 Gardener’s syndrome, 433 Glyburide, 165 Hamartomas, 352, 456
Fluorescent treponemal Garre’s osteomyelitis, 540, 542 Goiter, 167, 387, 391, 392 Hansen’s disease, 486
antibody absorption test Gas gangrene, 274, 286, 337, Goltz’s syndrome, 610 Harrison groove, 201
(FTA-Abs), 279 342–343 Gonorrhoea, 192, 485 Hartnup’s disease, 175
Fluorides, 208 Gas transfer factor, 108 Good Pasture’ssyndrome, Hazards of blood
FNAC lobectomy, 384, 387 Gastric aspiration, 93 139–140 transfusion, 50
Focal dermal hypoplasia Gastric decontamination, Gorlin and Goltz sydrome, 473 Head injuries, 372
syndrome, 610 208–209 Gorlin cyst, 473–474 Head injury management, 374
Focal enamel hypoplasia, 434 Gastric lavage, 205, 208 Gorlin’s cyst, 478 Headache, 123, 132, 135, 360
Focal infection, 543, 545 Gastric ulcers, 17 Gout, 202, 203 Heaf test, 106, 119
Focus of infection, 545 Gastritis, 13, 21 Grafts, 253 Healing of extraction socket,
Folates, 49 Gastroesophageal reflux Gram’s stain, 183, 186, 211 564
Folic acid, 49, 171 disease (GERD), 552 Gram stained smear, 270, 506 Healing of oral wound, 565
Follicular adenoma, 388 Gastro-oesophagial reflux Grand mal epilepsy, 121, 129 Healing of the fracture, 561
Follicular carcinoma, 384, 387 disease, 4, 17 Grand mal seizures, 355 Healing of wounds, 246
Follicular carcinoma of Gaucher’s disease, 574 Granular erythropakia, 445, Healing, 242, 253, 560, 564
thyroid, 383, 386, 389 Gemination, 425, 433, 435 452 Heart burn, 4
Follicular cyst, 481 Generalised juvenile Granuloma gravidarum, 542 Heart failure, 59, 60, 61–63,
Follicular odontoma, 299 periodontitis, 511 Granuloma pyogenicum 74, 82
Food impaction, 513 Generalised oedema, 200 originates, 495 Heat sterilization, 233
Food poisoning, 22 Generalised seizures, 133 Granulomatous diseases, 514 Heavy legs, 338
Foot-drop phenomenon, 211 Genetic syndromes, 152 Granulomatous epulis, 309 Helicobacter pylori infection,
Fordyce’s granules, 437, 439 Geniculate ganglion, 120 Graves’ disease, 161, 167, 387, 6
Formaldehyde solutions, German measles, 189, 502 388, 391 Hemangioma, 289, 292, 298,
233–236 Gestational diabetes mellitus, Grinspan’s syndrome, 612 340, 351, 454
Formative stage of tooth, 427 169 Grossly comminuted fractures, Hematogenous spread, 299
Fothergill’s disease, 614, 615 Gestational onset DM, 152 367 Hematological disorders, 50
Fourth heart sound, 61–62 Ghon’s complex, 494 Ground glass appearance, 570 Hematoma, 370, 372, 406
Fracture, 366–367, 368, 369, Ghost cells, 484 Gum boil, 275, 514 Hematoxylin-eosin, 479
370, 371, 374 Ghost teeth, 432, 438 Gumma, 305, 493 Hemiglossectomy, 304
Fracture healing, 370, 562 Giant cell epulis, 309 Gummatous ulcer, 305, 312 Hemiplegia, 122
Fracture mandible, 368 Giant cell lesions, 443 Gunshot wounds, 248 Hemithyroidectomy, 388
Fracture of jaws, 561 Giemsa stain, 494 Gustatory sweating, 615 Hemolytic streptococci, 485
Fracture of mandible, Gigantism, 568, 573 Gynaecomastia, 167 Hemophilia, 48, 595, 596, 602,
367–368 Gilbert’s syndrome, 27 Gynecomastia, 28 603
Fracture pattern, 366–367 Gilchrist’s disease, 505 Hemorrhagic diasthesis, 170
Fractures of maxilla, 376 Gingival bleeding, 275 H Hemorrhagic shock, 259–260,
Fractures of teeth, 546 Gingival cyst, 288, 482 H2 antagonists, 20 266–267
Fragilitas ossium, 586, 589 Gingival cysts of the adult, 288 H2-receptor antagonists, 7 Hepatic artery angiography, 23
Free radical injury, 255 Gingival diseases, 508 HAART, 124 Hepatic carcinoma, 22
Free thyroxine index, 158–159 Gingival enlargements, HACEK group, 69 Hepatic encephalopathy, 35
Frey’s syndrome, 363, 325, 209–210, 509, 511 Haemagioma of tongue, 316 Hepatitis A, 31, 32
Frey-Baillarger syndrome, Gingival hyperplasia, 209, 509 Haemangiomas, 447 Hepatitis A infection, 34
363 Gingivitis, 512 Haemarthroses, 42, 603 Hepatitis B, 23–28, 32
Frey’s syndrome, 615 Glandular disease, 502 Haematemesis, 92, 105, 116 Hepatitis C, 32
Frusemide, 145, 148 Glandular fever, 601 Haematomas, 603 Hepatitis E virus, 29
Fungal infections, 503 Glasgow coma scale, 372, 374 Haematuria, 144 Hepatocellular diseases, 27, 29
Furlow, 378, 382 Glass bead sterilizer, 233 Haemolytic anemia, 47 Hepatocellular jaundice, 34
Furuncles, 273, 277 Globulomaxillary cyst, 437 Haemophilia, 265, 268 Hepatoma, 22
Fusion, 425–426, 433, 435 Glomerulonephritis, 139, 143 Haemoptysis, 107, 116, 119 Hepatomegaly, 22, 34
Index 729

Hereditary brown enamel, 436 Hypercapnia, 117 Immune system, 42, 46 Intestinal mucosal cell, 8–9
Hereditary brown opalescent Hypercementosis, 554, 558 Immunofluorescent testing, Intra epithelial carcinoma, 448
teeth, 436 Hyperglycemia, 150, 161 604, 606 Intra-articular origin, 578–579
Hereditary ectodermal Hyperimmune reaction, 607 Immunoglobulin, 32, 577 Intracranial haemorrhage, 265
dysplasia, 607, 611 Hyperkalaemia, 146 Immunological changes, 247 Intraluminal digestion, 14
Hereditary enamel dysplasia, Hyperkeratosis, 308 Immunologically mediated Intravenous nitroglycerin, 57
436 Hyperorthokeratosis, 608 reactions, 46 Intravenous regional anesthesia
Herpangina, 499, 501 Hyperosmolar nonketotic Immunology, 523 (Bier’s block), 405
Herpes labialis, 190, 502 coma, 149 Immunosuppression, 575 Intravenous transfusions, 266
Herpes simplex, 185 Hyperostosis, 454 Immunosuppressive drugs, 296 Intrinsic disorders, 578–579
Herpes virus infections, 502 Hyperparathyroidism, 570, 572 Immunosuppressive therapy Intrinsic staining, 515
Herpes zoster, 191, 498, 501 Hyperpituitarism, 166, 568 38, 43, 198 Intussusceptions, 294
Herpetic stomatitis, 496–497 Hypersplenism, 172 Impacted teeth, 314 Invasion, 299
Herpetic ulcer, 305–306 Hypertension, 51, 53, 81, Impaired hepatic metabolism, Invasive cervical resorption,
High fiber diet, 18 140–44, 27, 29 557
High resolution computed Hypertensive encephalopathy, Incineration, 238 Invasive follicular cancers, 384
tomography (HRCT), 84 131 Incisions, 239 Invasive tumors, 327
Hilton’s method of abscess Hypertensive retinopathy, 81 Increased antidiuretic hormone Ionotopic agents, 62
drainage, 286 Hyperthyroidism, 155, 157, (ADH) secretion, 259 Iris, 612
Histamine H2-receptor 161, 387, 572 Incubation period, 183, 271 Iron deficiency anemia, 36–43,
antagonists, 12 Hypertrophic gingivitis, 509 Indirect fluorescent test, 507 49, 51, 598, 600
Histamines, 577 Hypertrophic scar, 244, 565 Indirect tests, 192–194 Irreversible loss of dental hard
Histiocytosis-X-disease, 588 Hypertrophy of gums, 209 Indolent lymphomas, 347 tissue, 552
Histopathological tests, 111 Hyperuricaemia, 39 Induction of emesis, 208 Irritable bowel syndrome, 17
Histoplasmosis, 506 Hyperventilation, 255, 258 Infantile cortical hyperostosis, Irritation dentin, 553
HIV, 282, 283, 286, 452 Hypocalcemia, 428 592 Ischaemic strokes, 122
HIV and AIDS, 192, 501 Hypoglycemia, 149–150, 159 Infantile hemangioma, 292 Ischemia, 342
Hoarseness of voice, 114 Hypokalemia, 146 Infantile osteopetrosis, 587
Hodgkin’s disease, 349, 455 Hyponatremia, 147 Infarction, 58 J
Hodgkin’s lymphoma, 350 Hypoperfusion, 258 Infectious acute periodontitis, James Ramsay Hunt
Homogenous erythroplakia, Hypopituitarism, 573 514 syndrome, 498, 501, 502
445 Hypoplasia of teeth, 436 Infectious agents, 607 Janeway’s lesions, 69
Homotransplantation, 567 Hypothyroidism, 155–156 Infectious mononucleosis, 502 Jaundice, 26, 29, 30
Honey comb appearance, 446 Hypovolemic shock, 255, Infective endarteritis, 68 Jaw clicking, 373
Hookworm diosease, 189 262–263, 264, 267, 268 Infective endocarditis, 68, 72, Joint damage, 265
Horizontal transmission, 24 Hypoxaemia, 57 75, 76, 79, 81 Jones criteria, 81
Hormones, 124 Hypoxia, 240, 247, 252–253 Infective stomatitis, 18 Jugular foramen, 299
Horner’s syndrome, 135, 363 Hypoxia/anoxia of tissue, Inflammatory cysts, 468, 471, Jugular venous pulse, 3
Horton’s syndrome, 616 254–255 473, 477 Junctional nevus, 456
Hospitalization, 260, 267, 276 Hysteria, 135 Infranuclear facial nerve palsy, Juvenile fibrous dysplasia, 586
Humoral immunity, 198, 287 Hysterical hyperventilation, 120, 128 Juvenile periodontitis, 510
Hunger pain, 6 168 Ingestion of fluoride, 428
Hunter’s glossitis, 601 Inhalation (direct), 86
Hurtle cell carcinoma, 384, I Initial stabilization, 204–205 K
389 Icteric phase, 24 Injury, 245 Kala azar, 172
Hutchinson’s incisor, 285 Idiopathic thrombocytopenic Inoculation, 506 Kaposi’s sarcoma, 452, 455
Hutchinson’s teeth, 193, 285 purpura, 42 Inotropic agents, 256 Keloid, 244, 565
Hutchinson’s triad, 438 Imaging studies, 388 Inotropic state, 59–60 Keratin cyst, 437
Hutchinson-Boeck teeth, 285 Imaging tests, 386, 392–393 Insulin, 151, 164 Keratinization pattern, 441,
Hutchinson’s triad, 493 Immature hemangioma, 289 Insulin shock, 574 445–446
Hydrocephalus, 408 Immediate effects of a fracture, Insulin-dependent DM, 152 Keratoacanthoma, 290, 301
Hydrogen peroxide, 234, 237 561–562 Interferon alpha, 40 Keratoconjunctivitis sicca,
Hyoid bone, 297 Immobilization, 369, 565, 566 Intermediate cells, 467 459–460
Hyper parathyroidism, 168 Immune (idiopathic) Internal resorption, 554 Keratocyst, 482
Hyperpigmentation, 210 thrombocytopenic Internal resorption (pink Keratomalacia, 174
Hyperthyroidism, 157, 162 purpura, 46 tooth), 557–558 Keratoses, 301
Hyperacusis, 120 Immune complex reaction, 196 Interstitial pneumonia, 100 Keratosis follicularis, 612
Hyperavitaminosis A, 179 Immune response, 109 Interstitial radiation, 304 Kernig’s sign, 124, 137, 357,
Hyperbaric oxygen, 275 Immunity, 287 Intestinal amoebiasis, 10 362
730 Quick Review Series for BDS 3rd Year

Kinetic classification of Liesegang’s ring calcification, Malignant osteopetrosis, 587 Medullary carcinoma, 384
anemia, 594 479 Malignant pleomorphic Medullary carcinoma of
Kissing disease, 502, 601 Lingua nigra, 438 adenoma, 318 thyroid, 383
Klebs- Loefller bacillus, 491 Lingua villosa, 438 Malignant potential of Megaloblastic anemia, 179
Klinefelter’s syndrome, 427, Lingual thyroid, 314 dentigerous cyst, 482 Megaloblastic macrocytic
432 Lip print, 617 Malignant transformation, 455 anemia, 36
KOH film, 618 Lipoma, 294, 298 Malignant tumors, 299, 388, Meig’s syndrome, 28
KOH preparation, 507 Lipshutz bodies, 500 439, 458, 459, 469, 475 Melaena, 18
Koilonychia, 116 Liquefaction foci, 533 Malnutrition, 240, 172, Melanocytes, 456
Koplik’s spots, 501 Lisch nodules, 346 248–249 Melkersson - Rosenthal
Korsakoff’s psychosis, 170, Liver abscess, 23, 33 Malunion, 371 syndrome, 614
176 Liver biopsy, 26 Management of haemor- Memory T cells, 408
Krebs cycle, 176 Liver function tests, 47 rhage, 257–258, 268 Meningeal irritation, 136
Kussmaul’s sign, 3 Liver transplantation, 31 Management of pain, 409 Meningial irritation, 362
Kveim-Siltzbatch test, 493 Lobar pneumonia, 90–101 Management of shock, Meningitis, 124, 129, 136,
Kyphoscoliosis, 65 Lobectomy, 384, 388 256–257, 260 356, 362
Lobstein’s disease, 586, 589 Management of tetanus, Mesiodens, 433
L Local anesthesia, 363, 405 281 Metabolic acidosis, 147
Lab findings, 49, 463–464, Local anesthetic Mandibular dislocation, 376 Metabolic changes, 247
467 administration, 406 Mandibular fractures, 368–69 Metastasis, 289
Laboratory studies, 9, 15 Local factors, 434, 513 Mandibular torus, 444 Metronidazol, 127
Laceration, 239, 247 Local inflammatory mediators, Mandibulofacial dysotosis, 591 Metronidazole, 275
Lactate dehydrogenase, 37 254 Mantoux test, 106, 118, 494 Micrognathia, 315, 431
Lactated Ringer’s solution, 262 Local reaction, 246 Marble bone disease, 587, 590 Mid-diastolic murmur, 79
Lactobacillus, 526–527 Local vasoconstriction, 70–71 Marfans syndrome, 590 Migraine, 123–124, 127, 133,
Lactobacillus colony test, 524 Localised aggressive Marginal gingivitis, 514 353, 354, 360
Lactose intolerance, 20 periodontitis, 510 Marginal resection, 296, 302 Migrainous neuralgia, 137
Lactose restricted diet, 20 Localised juvenile Marie and Santon’s disease, Migratory polyarthritis, 63
LAHSHAL, 377 periodontitis, 510–511 585, 589 Mikulicz’s disease, 465
Lamina propria, 446 Lockjaw, 133, 194, 359 Marjolin’s ulcer, 284 Miliary pneumonia, 100
Langerhan’s cell histiocytosis Loeffler’s syndrome, 43, 48 Marsupialisation, 297 Millard-Gubler syndrome, 120
(LCH), 588 Looser’s zones, 201 Mask like expressionless face, Miller’s acidogenic theory, 522
Langhans cells, 339 Lower esophageal rings, 328 615 Minerals and electrolytes, 9
Laparoscopy, 28 Low-molecular weight Massive blood transfusion, Minor aphthous ulcers, 305
Laryngeal spasm, 168 heparin, 58 261–262 Minor burns, 246
Laryngeal swabs, 93 Ludwig’s angina, 276, 284 Massive haemoptysis, 107–108 Minor criteria, 69, 76
Laryngocele, 340 Lumbar puncture, 125, 362 Massive splenomegaly, 50 Minor manifestations, 63,
Late syphilis, 488–489, 490 Lumpy jaw, 495 Masson’s trichrome, 479 82–83
Latent jaundice, 29 Lung abscess, 96–97, 102 Maxillary antrum, 312, Mitral stenosis, 55, 78
Lateral periodontal cyst, 288 Lung volumes, 108 471–472 Mitral valve replacement, 55–56
L-DOPA, 123 Lymphadenitis, 348 Maxillary sinus, 307, 313 Mitral valvotomy, 55
LE cells, 610 Lymphadenopathy, 277 Maxillo-facial injuries, 366 Mixed parotid tumor, 318, 320
Le Fort fractures, 365–366 Lymphatic drainage of tongue, Maxillomandibular fixation Mixed tumors, 465
Lead poisoning, 207, 210 309, 315, 350 (MMF), 365 Mobilization, 252
Leg sign, 125 Lymphoma staging, 347 Max-Joseph spaces, 605, 608 Mode of transmission, 282
Leontiasis ossea, 589 Lymphosarcoma, 349 McCune-Albright syndrome, Moderate burns, 246
Lepromas, 486 455, 588, 591 Modified Duke’s criteria, 69
Lepromatous leprosy, 191 M Measles, 188, 325 Moebius sign, 388
Leprosy, 486 Macroglossia, 315, 437 Measuring blood loss, 260 Moeller’s glossitis, 601
Leukemia, 39, 45, 49, 592, 601 Macrognathia, 316 Mechanical resuscitator, 375 Moh’s micrographic surgery,
Leukemic gingival Magnetic resonance imaging Mechanical ventilation, 87 295
enlargement, 510 (MRI), 321, 343, 384 Mechanism of injury, 252 Mohs surgery, 295
Leukoedema, 445 Major aphthous ulcers, 305 Meckel’s diverticulum, 5 Moist, or wet gangrene, 286
Leukoplakia, 21, 310, 312, Major burns, 246 Median rhomboid glossitis, Mongolism, 590
313, 440–442, 445, 446 Malabsorpion syndrome, 8, 9 430, 437 Mongoloid face, 49
Levine test, 66 Maladie de Roger, 68 Mediastinal emphysema, 104 Mononeuropathy, 131
Lewy body’s, 122 Malaria, 187, 190 Medical care, 336 Monospot test, 502
Lichen planus, 291, 605, 608 Malignant lesions, 386 Medical emergency, 362 Monostotic fibrous dysplasia,
Lichenoid dermatitis, 605 Malignant melanoma, 300, 302 Medical intervention, 339 584–585
Index 731

Monro’s abscess, 611 Neurofibromatosis, 454 Odontogenic tumors, 469, 475 Pain, 406, 409
Morphology, 432 Neurogenic shock, 262, 263 Odontoma, 291, 300, 474–475 Painful ulcers, 311
Moth eaten appearance, 535 Neurological deficiencies, 124 Oedema, 145, 200 Palatoplasties, 378
Mother of pearl appearance, 445 Neurotemesis, 359 Oesophageal stricture, 5 Palatoschisis, 376, 377, 378
Motor neuropathy, 242 Neutropenia, 40, 45, 48, 212 Oesophagoscopy, 10 Pallor, 179
Mottled enamel, 428, 434, 436 Nevoid basl cell carcinoma OLEP staging system, 441 Pantothenic acid, 171
Mucocele, 322, 460, 465, 547 syndrome, 473 Oliguria, 144 Papillary carcinoma, 384, 387
Mucoepidermoid carcinoma, Nevus, 446 Onion peel appearance, 446 Papillary carcinoma of thyroid,
456, 463, 467 Niacin (pellagra), 170–171, 178 Open valvotomy, 55 383, 389, 392
Mucosal ‘raft’, 7 Nickolsky’s sign, 610 Operculum, 275 Papillary cystadenoma
Mucosal invasion, 15 Nicotinic acid, 178 Ophthalmopathy, 157 lymphomatosum, 325
Mucosal lesions, 406–7 Niemann - Pick disease, 573 Opportunistic fungal infection, Papillion-Lefevre syndrome, 513
Mucous patches, 487, 488 Night blindness, 174 507 Papilloma, 451
Mucus retention cyst, 322 Night pain, 6 Oppurtunistic mycosis, 507 Paracentesis, 30
Mulberry molars, 427 Night grinding, 550 Optic atrophy, 179 Paraganglioma, 351
Multinodular goiter, 391–392 Nikolsky’s sign, 197, 604, 609 Oral anticoagulants, 79 Parakeratin plugging, 439–440
Multiple exostoses, 454 Nitrates, 57, 67, 80 Oral candidiasis, 507 Parakeratosis, 608
Multiple mononeuropathy, 131 Nocturia, 61 Oral dehydration, 147 Parenteral iron therapy, 598
Multiple myeloma, 456 Nodular growth, 455 Oral epithelial nevus, 611 Paresthesia, 406
Multiple-dose activated Noma, 487, 493 Oral hypoglycemic drugs, 166 Parkinson’s disease, 122
charcoal (MDAC), 206 Non immune transfusion Oral iron therapy, 44, 45, Parkinsonian disease, 122
Mumps, 185, 188, 191, 325 reactions, 46 598–599 Parotid fistula, 324
Muscle enzymes, 126 Non insulin-dependent DM, 152 Oral lesions, 501 Parotid gland, 318, 322, 325
Muscle spasms, 127 Non-epithelial cysts, 468, 471 Oral lichen planus OLP, 611, Parotitis, 318
Myasthenia gravis, 616 Non-epithelial tumors, 317 612 Paroxysmal headache disorder,
Mycelia, 504 Non-Hodgkin’s lymphoma, Oral manifestations of AIDS, 353
Mycobacterium leprae, 486 346, 347, 349 283 Paroxysmal nocturnal
Myeloid epulis, 309 Non-neoplastic inflammatory Oral moniliasis, 506 dyspnoea, 61
Myocardial contractility, 59 condition, 461 Oral mucosa, 439, 456, 548 Paroxysmal tachycardia, 71
Myocardial infarction, 56, 151 Non-odontogenic cyst of Oral ranula, 296–297, 323 Partial anodontia, 430
Myocardial ischaemia, 66 mandible, 288–289 Oral submucous fibrosis Partial dislocation, 371
Myocardial perfusion Non-thrombocytopenic (OSMF), 291, 408 Partial glossectomy, 304
scanning, 66 purpura, 47 Oral thrush, 316 Parulis, 275, 514
Myocardial remodelling, 60 Nonunions, 372 Organophosphates, 210 Passive immunization, 33
Myxoedema, 155, 163 Norepinephrine, 259 Ortho-phthalaldehyde, Paul-Bunnell test, 601
Myxoedema coma, 163 North American 233–234, 237 Pellagra, 174, 175, 178, 180
blastomycosis, 505 Orthopnoea, 61 Pelvic actinomycosis, 270, 276
N Nuchal rigidity, 362 Orthostatic syncope, 71 Pemberton sign, 391
Nasogastric intubation, 334 Nuclear scan, 384, 389 Osteitis deformans, 554, 582 Pemhigus vulgaris, 608
Nasopalatine duct cyst, 298 Nucleic acid amplification Osteoblasts, 446 Pemphigus vegetans, 604
National guidelines, 98–99 tests (NAAT), 110 Osteocementum, 554 Pemphigus vulgaris, 197, 199
Natural killer T cells, 408 Nursing bottle caries, 532, 533 Osteogenesis imperfecta Pemphigus, 197, 605–606
Nebulisation of b-agonists, 147 Nursing bottle syndrome, 529 (Brittle bones), 589 Penetration wounds, 248
Neck rigidity, 137 Nursing caries, 529, 532 Osteogenic sarcoma, 446 Penicillin allergy, 79, 198
Necrosis, 342 Osteomalacia, 176–177, 201 Penicillin anaphylaxis, 198
Necrosis of gingival tissue, 508 O Osteomyelitis, 535, 538 Penicillin G, 270, 275, 276
Necrotizing sialometaplasia, Obstructive jaundice, 29, 33, 34 Osteopetrosis, 587, 590 Peptic ulcer, 5, 12, 20, 207
461 Obstructive shock, 256, 262 Osteoporosis, 202 Percussion of chest, 119
Nectar americanus, 189 Obstructive signs, 390 Osteoporosis circumscripta, 583 Percutaneous coronary
Needle biopsy, 318, 390 Oculosympathetic palsy, 363 Osteoradionecrosis, 545, 550 interventions, 59
Needle breakage, 406 Odontoclastoma, 559 Osteosclerosis fragilis Percutaneous transluminal
Neisseria gonorrhoeae, 485 Odontogenic carcinomas, 474 generalisata, 587 coronary angioplasty
Neonatal period, 434 Odontogenic cyst, 288, 297, Otitis media, 409 (PTCA), 68
Neonatal tetanus, 271, 273 468, 471, 473, 477, 481 Outgrowth of bone, 456 Periapical /radicular cyst,
Neoplasm, 440, 443 Odontogenic keratocyst Over transfusion, 262, 266 287–288
Nephritis, 140 (OKC), 288, 472–473 Periapical abscess, 275, 541
Nephrotic syndrome, 138, 140 Odontogenic mandibular cyst, P Periapical cyst, 476, 480, 537
Nerve grafting, 358 287–288 Paget’s disease, 369, 582, 583, Periapical granuloma, 536,
Neurofibroma, 298, 454 Odontogenic sarcoma, 474, 475 588, 590 539, 541
732 Quick Review Series for BDS 3rd Year

Periapical lesions, 537 Platelet disorders, 41 Preformed toxin, 15 Pulmonary tuberculosis, 97,
Periapical periodontitis, 514 Platynychia, kolionychia, 45 Pregnancy tumor, 542 113, 119
Pericardial effusion, 62 Pleomorphic adenoma, 318, Prehepatic, 27, 29 Pulp calcifications, 553,
Pericoronal cyst, 481 320, 326, 458–459, 462 Preload, 59 555–556
Pericoronitis, 275 Pleural aspiration, 99–100 Premalignant lesions, 291, Pulp polyp, 539, 541
Periodic acid Schiff stain, 480 Pleural effusion, 55, 62, 99 300, 312, 440, 442 Pulp stones, 553, 555–556
Periodic agranulocytosis, 600 Pleural pain, 89 Prepubertal periodontitis, 596 Pulpitis, 534
Periodic neutropenia, 600 Plexiform neurofibromas, 346 Pressure ulcers, 343 Puncture wounds, 247
Periodontal (lateral) abscess, Plexus anesthesia, 405, 410 Prevention of dry socket, 561 Purpura, 47
515 Plumbism, 207, 211, 549 Prevention of tetanus, 273, Pushing margin, 440
Periodontal cyst, 541 Plummer Vinson or Patterson- 285–287 Putrid halitosis, 276
Periodontal diseases, 508 Kelly syndrome, 598 Prevention, 283 Pyogenic granuloma, 495, 542
Periodontal ligament PDL, 537 Plunging ranula, 326 Preventive and controlling Pyogenic meningitis, 124–127
Periodontium, 514 PMMC flap, 311 methods, 527 Pyorrhoea alveolaris, 515
Periorbital hematoma, 373 Pneumococcal pneumonia, Primary caries, 528 Pyrexial reactions, 261,
Peripheral blood film, 36, 46 100, 103, 118 Primary complex, 494 265–266
Peripheral blood picture, 51 Pneumonia, 88, 112 Primary focus, 494 Pyridoxine, 171, 179
Peripheral blood smear, 179 Pneumothorax, 87, 408 Primary intention, 241 Pyuria, 145
Peripheral cyanosis, 70–71 Poisoning, 208 Primary prevention, 65
Peripheral giant cell Polyarthritis, 63 Primary stage of syphilis, 493
granuloma, 450–451 Polycythemia, 49, 602 Primary thyrotoxicosis, 384
Q
Quadruple rhythm, 61
Peripheral nerve block, 405 Polycythemia vera, 38 Primary tuberculosis, 494
Quincke’s edema, 199, 576
Peripheral neuropathy, 131, 179 Polydioxanone, 236 Primary herpes simplex
Quinolones, 212
Peripheral signs, 78 Polyglactin 910 (Vicryl), 236 infection, 502
Quinsy, 332, 334
Peritoneal biopsy, 28 Polyglandular autoimmune Primordial cyst, 288, 472, 481
Peritoneum, 27–28, 33 syndrome, 155 Prinzmetal’s angina, 66
Peritonsillar space, 332 Polymerase chain reaction Prodromal phase, 24, 71, 121 R
Pernicious anemia, 594–596 (PCR), 486 Progressive decompensated R protein, 175
Petit mal epilepsy, 130, 136, Polyneuropathy, 131 shock, 255 Raccoon eyes, 373
362 Polyostotic fibrous dysplasia, Proliferative verrucous Radiation, 304, 548
Pharyngeal dysphagia, 331 591 leukoplakia, 291 Radiation burns, 245, 252
Pharyngitis, 333, 485 Polyostotic form of fibrous Proptosis, 387 Radiation induced pathologic
Pharyngocele, 334 dysplasia, 580, 584 Prostaglandin analogues, process, 550
Phenobarbital poisoning, 210 Polyostotic form, 581, 584 7, 12 Radiation sterilization,
Phenobarbitone, 212 Porokeratosis, 290, 301 Proteinuria, 145 234–235
Phenylketonuria, 165 Portacaval shunt surgery, 28 Proteolytic chelation theory, Radiation therapy, 250, 253
Phenytion sodium, 133, 211 Portal hypertension, 30, 35 518 Radical neck dissection, 342
Philadelphia chromosome, 590 Portwine stain, 289, 448 Proteolytic theory, 518 Radical parotidectomy, 318
Phosphate supplementation, Positive tourniquet test (Hess Prothrombin time, 32 Radicular cyst, 480, 537, 541
151 test), 43 Proton pump inhibitors, 7, 12 Radicular dentin dysplasia, 428
Photodynamic therapy, 21 Postdrome phase, 354 Protostylid, 433 Radio immune assay, 494
Physiological secondary Postextraction bleeding, 264 Pseudofractures, 201 Radioactive iodine, 158, 162
dentine, 553 Postictal phase, 121, 129 Pseudohemophilia, 50 Radiofrequency (RF) ablation,
Physiotherapy, 95 Postnatal period, 434 Psychogenic shock, 264 339
Pierre-Robin syndrome, 591 Postoperative radiotherapy, Psychological aspects, 121–122 Radioiodine, 385
Pindborg’s tumor, 475, 478 318 Psychological stress, 605 Radioiodine treatment, 392
Pink disease, 548 Post-pertussis ulcer (whooping Psychological/psychogenic Radionuclide blood-pool
Pink tooth of mummery, 556 cough), 306, 312 headache, 135 scanning, 66
Pink tooth, 554 Post-primary pulmonary Psychosis, 163 Radionuclide scan, 159
Pioneer bacteria, 522, 531, 545 tuberculosis, 92 Psychosomatic factors, 512 Radiotherapy, 306, 307, 308,
Pit and fissure caries, 528 Post-streptococcal Psychotic depression, 130 311, 593
Pitting, 142 glomerulonephritis, 143 Pulmonary complications, 266 Rain drop pigmentation, 207,
Pituitary dwarfism, 573 Postural hypotension, 75 Pulmonary embolism, 76, 118, 210
Plaque, 479, 520, 524 Postural syncope, 71 207, 351 RAI-U scan, 384, 389
Plasma biochemical markers, Potassium channel openers, 80 Pulmonary emphysema, 104 Rampant caries, 528, 533
56–57 Potassium replacement, 151 Pulmonary eosinophilia, 110 Ramsay Hunt syndrome, 614
Plasma enzymes, 56–57 Preanaesthetic medication, 207 Pulmonary flow, 83 Rana species, 296
Plasma expanders, 252 Preauricular sinus, 242 Pulmonary function test (PFT) Ranula, 296, 322, 324, 465,
Plasminogen, 561 Precipitations test, 507 or spirometry, 84, 96 466, 549
Index 733

Rapid HIV testing, 282 Rh hump, 600 Schizophrenia, 135 Solitary bone cyst, 483
Rapid tests, 195 Rheumatic fever prophylaxis, Scleral pigmentation, 29 Solitary thyroid nodules, 390
Rapidly progressive 65 Sclerodactyly, 612 Sore throat, 63
periodontitis, 511 Rheumatic fever, 62, 64, 65, Scleroderma, 203 South American
Raspberry tongue, 485 75, 81, 543 Sclerotherapy, 339 blastomycosis, 507
Raynaud’s phenomenon, 612 Rheumatoid arthritis, 203 Sclerotic dentin, 559 Spasms, 126
Reactionary haemorrhage, 244 Rheumatoid factor, 204 Scorbic child, 170 Spatula test, 272
Rectal stasis, 16, 18 Rhinosporidiosis, 506 Scorbutic gingivitis, 170 Specific hepatitis viruses, 23
Recurent aphthous minor, 305 RIA (Radioimmune assay), Scorbutic gum, 514 Speckled erythroplakia, 445
Recurrent aphthous ulcers, 19 486, 506 Scoring system of risk, 370 Sphenopalatine neuralgia, 616
Recurrent caries, 528 Riboflavin deficiency, 176, Scorpion bite, 210 Spider navei, 34
Recurrent herpes labialis, 177, 573 Screening tests, 41 Spider telangiectasia, 34
497–498 Rickets, 178, 200, 204, 571 Scrofula, 349 Spinal rigidity, 362
Recurrent herpetiform Risus sardonicus, 126, 271 Scurvy, 169, 172, 177, 570 Spirometry, 106
ulcerations, 575 RNA virus, 32, 495–496, 497 Sebaceous cyst, 298, 437 Splenectomy, 40, 43
Red blood cell transfusion, 598 Rodent ulcer, 453, 456 Secondary dentin, 553, 559 Splenomegaly, 35, 39, 43, 47,
Reduction of the fracture, 368 Role of bacteria, 512 Secondary healing, 243, 565 172, 502
Reed-Sternberg cells, 455 Role of carbohydrates, 519 Secondary intention, 242 Split skin graft, 253
Reflux oesophagitis, 17 Root caries, 528 Sedatives, 256 sponge nevus, 611
Regional lymphadenopathy, Root end cyst, 476, 537, 541 Seizures, 121, 131 Spontaneous bacterial
599 Rootless teeth, 428 Self-healing carcinoma, 454 peritonitis (SBP), 26, 31
Regional odontodysplasia, 435 RTAs, 366 Septic shock, 258, 262, 267 Sporadic type, 296
Regressive alterations of teeth, Rubella, 189, 502 Septicemia, 280, 284 Spore state, 232–233
551 Rubella (MMR) vaccine, 325 Serological tests for viral Spread of carcinoma, 289, 341
Regressive changes, 551 Rubeola, 188, 503 hepatitis, 32 Sputum of AFB, 115
Regressive changes of pulp, Rule of nines, 247 Serology, 24 Sputum studies, 97
555 Rushton bodies, 483, 541 Serotonin inhibitors, 124 Squamous cell carcinoma, 292,
Regulatory T cells, 408 Russell bodies, 542 Serum alkaline phosphatase, 299, 302, 306, 444, 449
Regurgitation, 63 27, 591 Stafne bone cyst, 288
Rehabilitation, 252, 59 S Serum bilirubin level, 32 Staging malignancy, 450–451
Reimplantation, 566 S mutans dip slide method, Serum sickness, 198 Staging of carcinoma, 289
Reiter’s syndrome, 501 524 Serum-ascites albumin Staging system for oral
Relaxed skin-tension lines, 250 Sabouraud’s dextrose gradient (SAAG), 30 leukoplakia, 441
Relief of stress, 53 chloramphenicol agar Shape of teeth, 425, 433 Staphylococcus aureus, 484
Remodelling, 60 medium, 618 Shell teeth, 432, 438 Stellwag sign, 388
Renal complications, 186 Safety-pin cells, 602 Shick test, Elek’s test, 491 Stenosis, 63
Renal disorders, 52 Salbutamol, 103 Shock, 254, 255, 256, Stereotactic radiation therapy,
Renal failure, 141 Salicylates, 64 262–263, 267 361
Renal osteodystrophy, 201 Saliva, 533, 617 Sialolithiasis, 321, 325, 546 Sterilization, 232, 238
Renal rickets, 201–202 Salivary analysis, 468 Sick sinus syndrome, 71 Sterilizing agents, 237
Replacement therapy, 42 Salivary calculus, 319, 325 Sickle cell anemia, 602 Steroids, 258
Residual cyst, 288 Salivary duct stone, 546, 549 Sinus, 243, 244, 283 Stevens-Johnson syndrome
Resorption of teeth, 553–54 Salivary gland calculus, 321 Sinus bradycardia, 77 (SJS), 200, 607, 608, 611
Respiratory alkalosis, 148 Salivary gland tumors, 457–59 Sistrunk operation, 297 Stokes-Adams-Morgagni
Respiratory changes, 247 Salivary glands, 457, 462, 548 Sjogren’s syndrome, 326, syndrome, 71
Respiratory system, 62 Salivary reductase test, 524 459–460, 461, 463, 464 Stomatitis medicamentosa, 576
Restoration of blood volume Salivary S mutans level test, Skin boils, 273 Stomatitis scarlatina, 491
after haemorrhage, 524 Skin cancers, 299 Stomatitis, 18, 20
257–258 Salmonella antigen, 110 Skin grafting, 252, 253, Stool examination, 11, 188
Resuscitation, 204–205, 256, Salmon’s patch, 448 Skin test, 111, 506 Strawberry angioma, 448
257–258, 260, 262 Saphenous stripping, 339 Sliding hiatus hernia, 4 Strawberry hemangioma, 293
Retching, 21 Sarcoidosis, 493, 495 Smooth surface caries, 528, Strawberry mark, 289
Reticular atrophy of pulp, 555 Saw-tooth, 611 531, 532 Strawberry tongue, 485, 600
Retropharyngeal abscess, 332, Scalds, 251, 253 Snail track ulcer, 245, 305 Streptococcal tonsillitis, 485
335 Scarlet fever, 485, 543 Snyder’s tests, 524, 533 Streptococcus mutans, 526,
Retro-viruses, 39 Scarlet fever, 491 Soap bubble appearance, 292 530
Reynolds-Braude Schatzki’s ring, 328 Sodium channel blocker, 362 Streptococcus pyogenes, 484
phenomenon, 618 Schilling test, 9, 37 Soft tissue injury, 406 Stroke, 132
Rh factor, 268 Schistosoma mansoni, 105 Solar keratosis, 290 Stylomastoid foramen, 120
734 Quick Review Series for BDS 3rd Year

Subacute bacterial Taurodontism (bull like teeth), T-Iymphocyte, 407 Trifacial neuralgia, 614
endocarditis, 72 431–435 TMJ disorders, 578–579 Trigeminal ganglion, 358
Subdural hematoma, 372 TB Meningitis, 137 TNM classification, 450 Trigeminal nerve, 614
Sublingual dermoid, 301 T-cells, 197 Tomes fibers, 530 Trigeminal neuralgia, 130,
Subacute bacterial Tele therapy, 304 Tongue, 308, 311, 314 134, 357, 361, 613, 614
endocarditis, 68 Temporomandibular joint Tongue pressure marks, 617 Trigger avoidance, 354
Subcutaneous emphysema, (TMJ), 578 Tongue tie, 313, 315 Trigger zones, 615
104, 117 Tender hepatomegalay, 34 Tonic clonic seizures, 121, Triiodothyronine, 158
Subcutaneous nodules, 63–64 Tennison-Randall repair, 382 129, 355 Triple rhythm, 61
Subdural hemorrhage (SDH), Tension headache, 135, 360 Tonic phase, 355 Triple vaccine, 194
372 Tetanospasmin, 126 Tonic-clonic seizure, 133 Trismus, 276, 406
Subgingival calculus, 512 Tetanus prophylaxis, 285 Tonsillitis, 114, 333 Trisomy 21 syndrome, 590
Substantia nigra, 122 Tetanus, 126, 194, 271, 272 Tooth ankylosis, 436, 550 Trophic ulcer, 245
Subtotal thyroidectomy, 158, Tetralogy of Fallot, 71 Tooth pressure marks, 617 Tropical eosinophilia, 43, 48
162, 388 Thalassemia, 49, 602 Tooth scrape marks, 617 Trousseau’s sign, 168
Sucralfate, 7 Thermal burns, 245, 252 Topical steroid therapy, 607 True anodontia, 429
Sucrose chelation theory, 518 Thermal injuries, 249 Tori, 444, 456 True denticles, 553, 555–556
Sulfonamides, 270 Thermal sensitivity, 275 Total body irradiation (TBI), True micrognathia, 315–316
Sulphonyl ureas, 166 Thiamine, 170, 171 408 Trummerfeld zone, 571
Sumatriptan, 123 Thiamine deficiency, 176 Total glossectomy, 304 TSH, 391
Summation gallop, 62 Third heart sound, 61–62 Tourniquet, 257, 260, 267 Tubercular meningitis, 125,
Sun ray appearance, 446 Thoracic actinomycosis, 270, Toxic multinodular goiter, 391 130
Supernumerary roots, 427, 434 276 Toxic neurological damage, Tubercular ulcer of tongue,
Supernumerary teeth, 314, 432 Thromboangiitis obliterans, 183, 186 305, 312
Supranuclear paralysis, 120 335 Toxin, 205 Tuberculin test, 94, 106, 118
Surface anesthesia, 405 Thrombocytopenia, 42, 45, Trace metals deficency, 241, Tuberculosis, 485–486, 489,
Surface zone, 519–520, 521, 47, 212 249–250 490, 492, 494
529–530 Thrombocytopenic purpura, Tracheal foreign body, 114 Tuberculous cervical
Surgeon-HIV infected patient, 47–48, 599 Tracheitis, 333 lymphadenitis, 348
279 Thrombolysis, 58 Tracheostomy, 329, 330, 331 Tuberculous lymphadenitis
Surgical anatomy, 321 Thrombolytic therapy, 58 Tranquillizers, 62 (scrofula), 339
Surgical biopsy, 386 Thrombophlebitis, 345 Transcatheter arterial Tularemia/rabbit fever, 493
Surgical care, 275, 336 Thrombosis, 58 chemoembolization, 23 Turner’s hypoplasia, 439
Surgical enucleation, 480 Thrush, 505, 605 Transcervical approach, 326 Turner’s teeth, 439
Surgical excision, 463 Thyroglossal cyst, 297, 340 Transfusion, 260 Type IV hypersensitivity, 577
Surgical gut, 234 Thyroid acropachy, 157, 162 Transfusion reactions, 261, Type IV osteogenesis
Surgical reconstruction, 252 Thyroid carcinoma, 389 265–266 imperfecta, 587
Surgical therapy, 270, 276, 285 Thyroid disease, 159 Transient myocardial Typhoid fever, 181, 191
Surgical, 339 Thyroid function tests, 158 ischaemia, 66, 82 Typical absence seizures, 362
Sutures, 235–236 Thyroid gland tumors, 388 Translucent zone, 519, 521, Tzanck cells, 501, 604, 606
Swab test, 524 Thyroid hormones, 385–386 529, 532 Tzanck test, 610
Swallowing, 10 Thyroid lobules, 391 Transmission of diseases, 261
Swift’s disease, 548 Thyroid scan, 390 Transoesophageal U
Swollen abdomen of frog, 466 Thyroid stimulating hormone echocardiography, 69 Ulcer, 304, 311, 312, 605
Syncope, 71–72, 76, 134 (TSH), 158, 390 Transoral approach, 326 Ulcerative colitis, 19
Syphilis, 181–183, 193, 278, Thyroid surgery, 158, 162 Transparent dentin, 552 Ulcerative stomatitis, 18
286, 305, 440, 487–489 Thyroid ultrasound, 386, 393 Transplantation, 299, 567 Ulcers of tongue, 304, 311, 314
Systemic autoimmune disease, Thyroid-binding Transudates, 27–28, 33 Ultrasonography, 28, 30, 99
387 immunoglobulins, 388 Transudative effusion, 99 Unconjugated
Systemic corticosteroid Thyroidectomy, 384, 392 Transverse clefts, 517, 522 hyperbilirubinemia, 27, 29
therapy, 154 Thyroid-stimulating Traumatic bone cyst, 288–289 Unerupted teeth, 436
Systemic diseases, 305, 312 immunoglobulins, 159 Traumatic brain injury (TBI), Unsealed source radiotherapy,
Systemic toxicity, 405–406 Thyrotoxic symptoms, 391 375 408
Thyrotoxicosis, 155, 157, 159 Traumatic neuroma, 453 Upper gastrointestinal
T Thyroxin, 167 Traumatic shock, 262–263 bleeding, 21
T helper cell, 407 Tic douloureux, 361, 614 Treadmill testing, 66 Upper motor neuron palsy, 136
Talon’s cusp, 426, 433 TIPSS, 28 Trench mouth, 508 Uptake of radioactive iodine,
Target lesions, 612 Tissue sampling procedures, Treponema pallidum, 278, 487, 158–159
Target cells, 602 318, 321 490 Uraemia, 144
Index 735

Urine analysis, 138, 140 Verocay bodies, 453 Vitamin K, 172 Wickham’s striae, 605, 608,
Utricaria, 199 Verrucous carcinoma, Volvulization, 294 612
439–440, 448, 455 Von Willebrand’s disease, 601 Widal test, 109, 193
V Vertical transmission, 24 Von Graefe’s sign, 387 Wide excision, 303–304
Vaccination, 24 Vesiculobullous lesions, 603 Von Kossa’s stain, 202 Wilson’s disease, 22, 25, 31
Variant angina, 66 Vessel wall abnormalities, 41 Von Recklinghausen’s disease, Wintrobe’s and Landsberg’s
Varicella zoster virus, 498 Vincent infection, 513 346, 570 method, 51
Varicose veins of leg, 338 Vincent’s angina, 514 Wound contamination, 253
Varieties of gangrene, 286 Vincent’s disease, 314, 514 W Wound healing, 244, 248, 249
Vasoactive drugs, 258 Viral exanthema, 193 Waldayer’s ring, 349 Wound infections, 253
Vasoconstrictive hormones, Viral hepatitis, 25, 32 Wallerian degeneration, 359 Wrist drop, 207, 211
255 Viral infections, 495, 497 Warthin’s tumor, 325, 462, Wrong tooth brushing, 551
Vasoconstrictors, 256, 263 Viruses, 23, 495, 497 463, 464, 466, 467
Vasodilators, 62, 256 Vit A deficiency, 178 Waterhouse-Friderichsen X
Vasopressin, 259 Vit. D, 204 syndrome, 125 Xerophthalmia, 174
Vasopressor agents, 259 Vitamin A and C deficiency, Wegener’s granulomatosis, Xerostomia, 323, 326, 435
Vasopressor supportive 241 209
therapy, 259 Vitamin A deficiency, 174, Werlhoff’s disease, 47 Z
Vasovagal shock, 264 178, 180 Wernicke’s encephalopathy, Z-plasty, 382
Vasovagal syncope, 71, 82 Vitamin A, 171, 173 170, 176 Zollinger-Ellison syndrome,
Vein ligation, 339 Vitamin B1, 173, 180 Westergren’s method, 51 6, 7
Venereal Disease Research Vitamin C deficiency, 170, Western blot test, 282, 283 Zone of coagulation, 246
Laboratory test, 194 172 Wet gangrene, 337, 342 Zone of hyperaemia, 247
Venous insufficiency, 338 Vitamin C, 570 Wet leathery, 443, 447 Zone of stasis, 247
Venous pulsations, 3 Vitamin D deficient rickets, Wharton’s duct, 321, 325 Zones of dentinal caries,
Venous ulcer, 285, 344 200–201 Whole bowel irrigation, 516–517
Ventilatory capacity, 108 Vitamin E, 172, 173, 248 208–209 Zoster infection, 498, 502
This page intentionally left blank

You might also like